You are on page 1of 967

Useful relations Series expansions

x2 x3
At 298.15 K ex = 1+ x + + +
2! 3!
RT 2.4790 kJ mol−1 RT/F 25.693 mV x2 x3
(RT/F) ln 10 59.160 mV kT/hc ln(1 + x ) = x − + −
207.225 cm−1 2 3
kT 25.693 meV Vm 2.4790 × 10−2 1 1
m3 mol−1 = 1− x + x 2 − = 1+ x + x 2 +
1+ x 1− x
24.790 dm3 mol−1
x3 x5 x2 x4
sin x = x − + − cos x = 1− + −
Selected units* 3! 5! 2! 4!

1N 1 kg m s−2 1J 1 kg m2 s−2
1 Pa 1 kg m−1 s−2 1W 1 J s−1 Derivatives; for Integrals, see the Resource
section
1V 1 J C−1 1A 1 C s−1
1T 1 kg s−2 A−1 1P 10−1 kg m−1 s−1 d(f + g) = df + dg d(fg) = f dg + g df
1S 1 Ω−1 = 1 A V−1 f 1 f df df dg
d = df − 2 dg = for f = f ( g (t ))
g g g dt dg dt
* For multiples (milli, mega, etc), see the Resource section
 ∂y   ∂x   ∂ y   ∂x   ∂z 
Conversion factors  ∂ x  = 1/  ∂ y   ∂ x   ∂ z   ∂ y  = −1
z z z y x
θ/°C = T/K − 273.15*
dx n deax d ln(ax ) 1
= nx n−1 = aeax =
1 eV 1.602 177 × 10−19 J 1 cal 4.184* J dx dx dx x
96.485 kJ mol−1
 ∂ g   ∂h 
8065.5 cm−1 df = g ( x , y )dx + h( x , y )dy is exact if   =  
 ∂ y  x  ∂x  y
1 atm 101.325* kPa 1 cm−1 1.9864 × 10−23 J
760* Torr Greek alphabet*
1D 3.335 64 × 10−30 C m 1Å 10−10 m*
Α, α alpha Ι, ι iota Ρ, ρ rho
* Exact value
Β, β beta Κ, κ kappa Σ, σ sigma
Mathematical relations Γ, γ gamma Λ, λ lambda Τ, τ tau
π = 3.141 592 653 59 … e = 2.718 281 828 46 … Δ, δ delta Μ, μ mu ϒ, υ upsilon
Ε, ε epsilon Ν, ν nu Φ, ϕ phi
Logarithms and exponentials
Ζ, ζ zeta Ξ, ξ xi Χ, χ chi
ln x + ln y + … = ln xy… ln x − ln y = ln(x/y) Η, η eta Ο, ο omicron Ψ, ψ psi
a ln x = ln xa ln x = (ln 10) log x Θ, θ theta Π, π pi Ω, ω omega
= (2.302 585 …) log x * Oblique versions (α, β, …) are used to denote physical
exeyez…. = ex+y+z+… ex/ey = ex−y observables.
(ex)a = eax e±ix = cos x ± i sin x
18
PERIODIC TABLE OF THE ELEMENTS VIII
VIIA

Group 1 2 1 H 13 14 15 16 17 2 He
helium
I II
Period 1 hydrogen
III IV V VI VII 4.00
1.0079
1s1 1s2
IA IIA IIIA IVA VA VIA VIIA

3 Li 4 Be 5 B 6 C 7 N 8 O 9 F 10 Ne
lithium beryllium boron carbon nitrogen oxygen fluorine neon
2 6.94 9.01 10.81 12.01 14.01 16.00 19.00 20.18
2s1 2s2 2s22p1 2s22p2 2s22p3 2s22p4 2s22p5 2s22p6

11 Na 12 Mg 13 Al 14 Si 15 P 16 S 17 Cl 18 Ar
sodium magnesium aluminium silicon phosphorus sulf ur chlorine argon
3 22.99 24.31 3 4 5 6 7 8 9 10 11 12 26.98 28.09 30.97 32.06 35.45 39.95
3s1 3s2 3s23p1 3s23p2 3s23p3 3s23p4 3s23p5 3s23p6
IIIB IVB VB VIB VIIB VIIIB IB IIB

19 K 20 Ca 21 Sc 22 Ti 23 V 24 Cr 25 Mn 26 Fe 27 Co 28 Ni 29 Cu 30 Zn 31 Ga 32 Ge 33 As 34 Se 35 Br 36 Kr
potassium calcium scandium titanium vanadium chromium manganese iron cobalt nickel copper zinc gallium germa nium arsenic selenium bromine krypton
4 39.10 40.08 44.96 47.87 50.94 52.00 54.94 55.84 58.93 58.69 63.55 65.41 69.72 72.64 74.92 78.96 79.90 83.80
Period

4s1 4s2 3d14s2 3d24s2 3d34s2 3d54s1 3d54s2 3d64s2 3d74s2 3d84s2 3d104s1 3d104s2 4s24p1 4s24p2 4s24p3 4s24p4 4s24p5 4s24p6

37 Rb 38 Sr 39 Y 40 Zr 41 Nb 42 Mo 43 Tc 44 Ru 45 Rh 46 Pd 47 Ag 48 Cd 49 In 50 Sn 51 Sb 52 Te 53 I 54 Xe
5 rubidium st rontium yttrium zirconium niobium molybdenum technetium ruthenium rhodium palladium silver cadmium indium tin antimony tellurium iodine xenon
85.47 87.62 88.91 91.22 92.91 95.94 (98) 101.07 102.90 106.42 107.87 112.41 114.82 118.71 121.76 127.60 126.90 131.29
5s1 5s2 4d15s2 4d25s2 4d45s1 4d55s1 4d55s2 4d75s1 4d85s1 4d10 4d105s1 4d105s2 5s25p1 5s25p2 5s25p3 5s25p4 5s25p5 5s25p6

55 Cs 56 Ba 57 La 72 Hf 73 Ta 74 W 75 Re 76 Os 77 Ir 78 Pt 79 Au 80 Hg 81 Tl 82 Pb 83 Bi 84 Po 85 At 86 Rn
caesium barium lanthanum hafnium tantalum tungsten rhenium osmium iridium platinum gold mercury thallium lead bismuth polonium astatine radon
6 138.91
132.91 137.33 178.49 180.95 183.84 186.21 190.23 192.22 195.08 196.97 200.59 204.38 207.2 208.98 (209) (210) (222)
6s1 6s2 5d16s2 5d26s2 5d36s2 5d46s2 5d56s2 5d66s2 5d76s2 5d96s1 5d106s1 5d106s2 6s26p1 6s26p2 6s26p3 6s26p4 6s26p5 6s26p6

87 Fr 88 Ra 89 Ac 104 Rf 105Db 106 Sg 107 Bh 108 Hs 109 Mt 110 Ds 112Cn 113 Nh 114 Fl 115 Mc 116 Lv
111 Rg 112 117 Ts 118 Og
francium radium act inium rutherfordium dubnium seaborgium bohrium hassium meitnerium darmstadtium roentgenium copernicium nihonium flerovium moscovium livermorium tennessine oganesson
7 (223) (226) (261) (262) (263) (262) (265) (266)
(227) (271) (272) ? ? ? ? ? ? ?
7s1 7s2 6d17s2 6d27s2 6d37s2 6d47s2 6d57s2 6d67s2 6d77s2 6d87s2 6d97s2 6d107s2 7s27p1 7s27p2 7s27p3 7s27p4 7s27p5 7s27p6

58 Ce 59 Pr 60 Nd 61 Pm 62 Sm 63 Eu 64 Gd 65 Tb 66 Dy 67 Ho 68 Er 69 Tm 70 Yb 71 Lu
cerium praseodymium neodymium promethium samarium europium gadiolinium terbium dysprosium holmium erbium thulium ytterbium Lanthanoids
lutetium
6 140.12 140.91 144.24 (145) 150.36 151.96 157.25 158.93 162.50 164.93 167.26 168.93 173.04 174.97 (lanthanides)
Numerical values of molar
masses in grams per mole (atomic 4f15d16s2 4f36s2 4f46s2 4f56s2 4f66s2 4f76s2 4f75d16s2 4f96s2 4f106s2 4f116s2 4f126s2 4f136s2 4f146s2 5d16s2
weights) are quoted to the number
of significant figures typical of 90 Th 91 Pa 92 U 93 Np 94 Pu 95 Am 96 Cm 97 Bk 98 Cf 99 Es 100Fm 101Md 102 No 103 Lr
most naturally occurring samples. thorium protactinium uranium neptunium plutonium americium curium berkelium californium einsteinium fermium mendelevium nobelium lawrencium Actinoids
7 232.04 231.04 238.03 (237) (244) (243) (247) (247) (251) (252) (257) (258) (259) (262) (actinides)
6d27s2 5f26d17s2 5f36d17s2 5f46d17s2 5f67s2 5f77s2 5f76d17s2 5f97s2 5f107s2 5f117s2 5f127s2 5f137s2 5f147s2 6d17s2
FUNDAMENTAL CONSTANTS

Constant Symbol Value


Power of 10 Units
Speed of light c 2.997 924 58* 108 m s−1
Elementary charge e 1.602 176 634* 10 −19
C
Planck’s constant h 6.626 070 15 10−34 Js
ħ = h/2π 1.054 571 817 10 −34
Js
Boltzmann’s constant k 1.380 649* 10−23 J K−1
Avogadro’s constant NA 6.022 140 76 1023 mol−1
Gas constant R = NAk 8.314 462 J K−1 mol−1
Faraday’s constant F = NAe 9.648 533 21 104 C mol−1
Mass
Electron me 9.109 383 70 10−31 kg
Proton mp 1.672 621 924 10 −27
kg
Neutron mn 1.674 927 498 10−27 kg
Atomic mass constant mu 1.660 539 067 10 −27
kg
Magnetic constant μ0 1.256 637 062 10−6 J s2 C−2 m−1
(vacuum permeability)
Electric constant  0  1/0c 2 8.854 187 813 10−12 J−1 C2 m−1
(vacuum permittivity)
4πε0 1.112 650 056 10−10 J−1 C2 m−1
Bohr magneton μB = eħ/2me 9.274 010 08 10 −24
J T−1
Nuclear magneton μN = eħ/2mp 5.050 783 75 10−27 J T−1
Proton magnetic moment μp 1.410 606 797 10 −26
J T−1
g-Value of electron ge 2.002 319 304
Magnetogyric ratio
Electron γe = gee/2me 1.760 859 630 1011 T−1 s−1
Proton γp = 2μp/ħ 2.675 221 674 10 8
T−1 s−1
Bohr radius a0 = 4πε0ħ2/e2me 5.291 772 109 10−11 m
Rydberg constant R  m e 4 /8h3c 2
 e 0 1.097 373 157 10 5
cm−1
hcR ∞ /e 13.605 693 12 eV
Fine-structure constant α = μ0e2c/2h 7.297 352 5693 10−3
α−1 1.370 359 999 08 102
Stefan–Boltzmann constant σ = 2π5k4/15h3c2 5.670 374 10−8 W m−2 K−4
Standard acceleration of free fall g 9.806 65* m s−2
Gravitational constant G 6.674 30 10 −11
N m2 kg−2

* Exact value. For current values of the constants, see the National Institute of Standards and Technology (NIST) website.
Atkins’
PHYSICAL CHEMISTRY
Twelfth edition

Peter Atkins
Fellow of Lincoln College,
University of Oxford,
Oxford, UK

Julio de Paula
Professor of Chemistry,
Lewis & Clark College,
Portland, Oregon, USA

James Keeler
Associate Professor of Chemistry,
University of Cambridge, and
Walters Fellow in Chemistry at Selwyn College,
Cambridge, UK
Great Clarendon Street, Oxford, OX2 6DP,
United Kingdom
Oxford University Press is a department of the University of Oxford.
It furthers the University’s objective of excellence in research, scholarship,
and education by publishing worldwide. Oxford is a registered trade mark of
Oxford University Press in the UK and in certain other countries
© Oxford University Press 2023
The moral rights of the author have been asserted
Eighth edition 2006
Ninth edition 2009
Tenth edition 2014
Eleventh edition 2018
Impression: 1
All rights reserved. No part of this publication may be reproduced, stored in
a retrieval system, or transmitted, in any form or by any means, without the
prior permission in writing of Oxford University Press, or as expressly permitted
by law, by licence or under terms agreed with the appropriate reprographics
rights organization. Enquiries concerning reproduction outside the scope of the
above should be sent to the Rights Department, Oxford University Press, at the
address above
You must not circulate this work in any other form
and you must impose this same condition on any acquirer
Published in the United States of America by Oxford University Press
198 Madison Avenue, New York, NY 10016, United States of America
British Library Cataloguing in Publication Data
Data available
Library of Congress Control Number: 2022935397
ISBN 978–0–19–884781–6
Printed in the UK by Bell & Bain Ltd., Glasgow
Links to third party websites are provided by Oxford in good faith and
for information only. Oxford disclaims any responsibility for the materials
contained in any third party website referenced in this work.
PREFACE

Our Physical Chemistry is continuously evolving in response Behind all that are The chemist’s toolkits, which provide brief
to users’ comments, our own imagination, and technical in- reminders of the underlying mathematical techniques. There
novation. The text is mature, but it has been given a new vi- is more behind them, for the collections of Toolkits available
brancy: it has become dynamic by the creation of an e-book via the e-book take their content further and provide illustra-
version with the pedagogical features that you would expect. tions of how the material is used.
They include the ability to summon up living graphs, get The text covers a very wide area and we have sought to add
mathematical assistance in an awkward derivation, find solu- another dimension: depth. Material that we judge too detailed
tions to exercises, get feedback on a multiple-choice quiz, and for the text itself but which provides this depth of treatment,
have easy access to data and more detailed information about or simply adds material of interest springing form the intro-
a variety of subjects. These innovations are not there simply ductory material in the text, can now be found in enhanced
because it is now possible to implement them: they are there to A deeper look sections available via the e-book. These sections
help students at every stage of their course. are there for students and instructors who wish to extend their
The flexible, popular, and less daunting arrangement of the knowledge and see the details of more advanced calculations.
text into readily selectable and digestible Topics grouped to- The main text retains Examples (where we guide the reader
gether into conceptually related Focuses has been retained. through the process of answering a question) and Brief illus-
There have been various modifications of emphasis to match the trations (which simply indicate the result of using an equation,
evolving subject and to clarify arguments either in the light of giving a sense of how it and its units are used). In this edition a
readers’ comments or as a result of discussion among ourselves. few Exercises are provided at the end of each major section in a
We learn as we revise, and pass on that learning to our readers. Topic along with, in the e-book, a selection of multiple-choice
Our own teaching experience ceaselessly reminds us that questions. These questions give the student the opportunity to
mathematics is the most fearsome part of physical chemis- check their understanding, and, in the case of the e-book, re-
try, and we likewise ceaselessly wrestle with finding ways to ceive immediate feedback on their answers. Straightforward
overcome that fear. First, there is encouragement to use math- Exercises and more demanding Problems appear at the end of
ematics, for it is the language of much of physical chemistry. each Focus, as in previous editions.
The How is that done? sections are designed to show that if The text is living and evolving. As such, it depends very
you want to make progress with a concept, typically making much on input from users throughout the world. We welcome
it precise and quantitative, then you have to deploy mathemat- your advice and comments.
ics. Mathematics opens doors to progress. Then there is the PWA
fine-grained help with the manipulation of equations, with JdeP
their detailed annotations to indicate the steps being taken. JK
viii 12 The properties of gases

USING THE BOOK

TO THE STUDENT

The twelfth edition of Atkins’ Physical Chemistry has been


developed in collaboration with current students of physical
chemistry in order to meet your needs better than ever before.
Our student reviewers have helped us to revise our writing
style to retain clarity but match the way you read. We have also
introduced a new opening section, Energy: A first look, which
summarizes some key concepts that are used throughout the
text and are best kept in mind right from the beginning. They
are all revisited in greater detail later. The new edition also
brings with it a hugely expanded range of digital resources, AVAIL ABLE IN THE E-BOOK
including living graphs, where you can explore the conse-
quences of changing parameters, video interviews with prac- ‘Impact on…’ sections Group theory tables
tising scientists, video tutorials that help to bring key equations ‘Impact on’ sections show how physical chemistry is applied in A link to comprehensive group theory tables can be found at
a variety of modern contexts. They showcase physical chemis- the end of the accompanying e-book.
to life in each Focus, and a suite of self-check questions. These try as an evolving subject.
Go to this location in the accompanying e-book to view a
features are provided as part of an enhanced e-book, which is list of Impacts.
The chemist’s toolkits
accessible by using the access code included in the book. The chemist’s toolkits are reminders of the key mathematical,

You will find that the e-book offers a rich, dynamic learn- ‘A deeper look’ sections physical, and chemical concepts that you need to understand in
order to follow the text.

ing experience. The digital enhancements have been crafted to These sections take some of the material in the text further and
are there if you want to extend your knowledge and see the de-
For a consolidated and enhanced collection of the toolkits
found throughout the text, go to this location in the accompa-
tails of some of the more advanced derivations. nying e-book.
help your study and assess how well you have understood the Go to this location in the accompanying e-book to view a
list of Deeper Looks.
material. For instance, it provides assessment materials that
give you regular opportunities to test your understanding.

Innovative structure TOPIC 2A Internal energy


Short, selectable Topics are grouped into overarching Focus
sections. The former make the subject accessible; the latter
provides its intellectual integrity. Each Topic opens with
RESOURCE SEC TION ➤ Why do you need to know this material?
The First Law of thermodynamics is the foundation of the
discussion of the role of energy in chemistry. Wherever the
A closed system has a boundary through which matter
cannot be transferred.
Both open and closed systems can exchange energy with their
surroundings.
the questions that are commonly asked: why is this material generation or use of energy in physical transformations or
chemical reactions is of interest, lying in the background
An isolated system can exchange neither energy nor
matter with its surroundings.
important?, what should you look out for as a key idea?, and are the concepts introduced by the First Law.

➤ What is the key idea?


what do you need to know already? The total energy of an isolated system is constant.
2A.1 Work, heat, and energy
➤ What do you need to know already?
Although thermodynamics deals with the properties of bulk
This Topic makes use of the discussion of the properties of
systems, it is enriched by understanding the molecular origins
gases (Topic 1A), particularly the perfect gas law. It builds
of these properties. What follows are descriptions of work, heat,

Resource section
on the definition of work given in Energy: A first look.
and energy from both points of view.

2A.1(a) Definitions

The Resource section at the end of the book includes a brief In thermodynamics, the universe is divided into two parts:
the system and its surroundings. The system is the part of the
The fundamental physical property in thermodynamics is
work: work is done in order to achieve motion against an op-
world of interest. It may be a reaction vessel, an engine, an elec- posing force (Energy: A first look 2a). A simple example is the
review of two mathematical tools that are used throughout the trochemical cell, a biological cell, and so on. The surroundings
comprise the region outside the system. Measurements are
process of raising a weight against the pull of gravity. A process
does work if in principle it can be harnessed to raise a weight

text: differentiation and integration, including a table of the Contents


made in the surroundings. The type of system depends on the
characteristics of the boundary that divides it from the sur-
somewhere in the surroundings. An example is the expansion
of a gas that pushes out a piston: the motion of the piston can in
roundings (Fig. 2A.1): principle be used to raise a weight. Another example is a chemi-
integrals that are encountered in the text. There is a review of An open system has a boundary through which matter
cal reaction in a battery: the reaction generates an electric cur-

units, and how to use them, an extensive compilation of tables PART 1 Mathematical resources
can be transferred.
878
rent that can drive a motor and be used to raise a weight.
The energy of a system is its capacity to do work (Energy: A
first look 2b). When work is done on an otherwise isolated sys-

of physical and chemical data, and a set of character tables. 1.1 Integration 878
tem (for instance, by compressing a gas with a piston or wind-
ing a spring), the capacity of the system to do work is increased.
Energy

Energy
Matter

That is, the energy of the system is increased. When the system

Short extracts of most of these tables appear in the Topics 1.2 Differentiation 878
does work (when the piston moves out or the spring unwinds),
it can do less work than before. That is, its energy is decreased.
When a gas is compressed by a piston, work is done on the sys-
themselves: they are there to give you an idea of the typical 1.3Open Series expansions
Closed Isolated 881
tem and its energy is increased. When that gas is allowed to ex-
pand again the piston moves out, work is done by the system,

values of the physical quantities mentioned in the text. (a) (b) (c)
and the energy of the system is decreased.
It is very important to note that when the energy of the sys-
PART 2 Quantities and units
Figure 2A.1 (a) An open system can exchange matter and energy
with its surroundings. (b) A closed system can exchange energy
882
tem increases that of the surroundings decreases by exactly the
same amount, and vice versa. Thus, the weight raised when the
with its surroundings, but it cannot exchange matter. (c) An system does work has more energy than before the expansion,
isolated system can exchange neither energy nor matter with its because a raised weight can do more work than a lowered one.
PART 3 Data
surroundings.
884
The weight lowered when work is done on the system has less

PART 4 Character tables 910


qV  CV T (2A.16)

TOPIC 2B Enthalpy
E2A.4 A sample co
This relation provides a simple way of measuring the heat ca- CV,m = 23 R , initially
pacity of a sample. A measured quantity of energy is transferred constant volume.
Using the book ix

Checklist of concepts
A checklist of key concepts is provided at the end of each Checklist of concepts
Topic, so that you can tick off the ones you have mastered.
2B.1 The definition of enthalpy
➤ Why do you need to know this material? ☐ 1. Work is the process of achieving motion against an ☐ 8. The in
The enthalpy, H, is
opposing defined as
force. raised.
The concept of enthalpy is central to many thermodynamic
discussions about processes, such as physical transforma- ☐ 2. Energy is the capacity to do work. Enthalpy
☐ 9. The Fir
H  U  pV
☐ 3. Heat is the process of transferring energy
(2B.1)
as a result of lated sy
tions and chemical reactions taking place under conditions [definition]
of constant pressure. a temperature difference. ☐ 10. Free ex
where p is the pressure of the system and V is its volume.
Physical chemistry: people and perspectives ☐ 4. An exothermic process is a process that releases energy
Because U, p, and V are all state functions, the enthalpy is a
no wor
➤ What is the key idea? as heat. ☐ 11. A rever
LeadingA figures state function too. As is true of any state function, the change
change in a varity of
in enthalpy is fields
equal share
to the their
energyunique and var-
transferred as ☐ 5. An endothermic process is a process in which energy is an infin
ied experiences and careers, and talk about the challenges they in enthalpy, ΔH, between any pair of initial and final states is
heat at constant pressure. acquired as heat. ☐ 12. To ach
faced and their achievements to give you a sense of where the independent of the path between them.
☐ 6. In molecular terms, work is the transfer of energy that sure is
study of➤physical
What dochemistry
you need cantolead.
know already? system
makes use of organized motion of atoms in the sur-
This Topic makes use of the discussion of internal energy Enthalpy
2B.1(a) roundings andchange and heat
heat is the transfer transfer
of energy that makes ☐ 13. The en
(Topic 2A) and draws on some aspects of perfect gases use of their disorderly motion. equal to
An important consequence of the definition of enthalpy in
(Topic 1A). ☐ 2B.1
7. Internal ☐ 14. Calorim
PRESENTING THE MATHEMATICS eqn is that itenergy,
can bethe total that
shown energy
theof=change
Probability
a system, is a state
|ψ|2dx in enthalpy is
function.
equal to the energy supplied as heat under conditions of con-
|ψ|2
stant pressure.

How
Theischange
thatindone?
internal energy is not equal to the energy trans- dx
How is that done? 2B.1 Deriving the relation between
ferred as heat when the system is free to change its volume,
You need to understand how an equation is derived from enthalpy change and heat transfer at constant pressure
such as when it is able to expand or contract under conditions
reasonable assumptions and the details of the steps involved.
of constant pressure. Under these circumstances some of the In a typical thermodynamic derivation, as here, a common way
This is one role for the How is that done? sections. Each one x x + dx
energy supplied as heat to the system is returned to the sur- to proceed is to introduce successive definitions of the quanti-
leadsroundings
from an asissue that arises
expansion work in the2B.1),
(Fig. text, so
develops thethan
dU is less nec-
dq. ties of 7B.1
Figure interest
Theand then apply ψ
wavefunction the
is appropriate
a probabilityconstraints.
amplitude in Figure 7B.2
essary equations, and arrives at a conclusion. These sections
In this case the energy supplied as heat at constant pressure is dimensional
the sense that its square modulus (ψ⋆ψ or |ψ|2) is a probability
maintain thethe
separation Step 1 Write an expression for H + dH in terms of the definition
equal to change in of the equation
another and its derivation
thermodynamic property ofso
the density. The probability of finding a particle in the region particle in th
thatsystem,
you canthe
find them easily for review, but at the same time of H proportiona
‘enthalpy’. between x and x + dx is proportional to |ψ|2dx. Here, the
emphasize that mathematics is an essential feature of physical For a general
probability infinitesimal
density change
is represented by in
thethe state of
density ofshading
the system,
in position.
chemistry. U changes
the to U +band.
superimposed dU, p changes to p + dp, and V changes to
V + dV, so from the definition in eqn 2B.1, H changes by dH to
13A The Boltzmann distribution 545
H + dH = (U + dU) + (p + dp)(V + dV) Wav
The chemist’s toolkits
Energy as
The chemist’s toolkit 7B.1 Complex
work = U + dU+ pV + pdV + Vdpnumbers
+ dpdV
The chemist’s
If, astoolkits areofreminders
a result collisions, ofthethe key mathematical,
system were to fluctuate Brief illustration 13A.1
physical,between
and chemical concepts that
the configurations you need
{N,0,0,…} and to
{Nunderstand
− 2,2,0,…}, it A complex
The last termnumber
is the productz has the z = x + iy, where
forminfinitesimally
of two i quan-
small 1.
The complex
titiesthe conjugate
andconfiguration
can be neglected. of a complex
Now recognize number z is z*
thatNU=+20, =
pV =x − iy.
theH on
in orderwould
to follow
almostthe text.beMany
always foundof these
inΔU Toolkits
the< second,
q more are rele-
likely con- For {1,0,3,5,10,1}, which has
figuration, especially if N were large. In other words, a system Complex
the right
weight numbers
(boxed), so
is calculated combine together
as (recall that 0! ≡ 1) according to the follow-
vant to more than oneEnergyTopic, and you can view a compilation
ing rules:
of them,free to enhancements
with switch between
as heat the two form
in the configurations
of more would
informa- show H + dH =20H! + dU + pdV +8 Vdp
properties characteristic almost exclusively of the second W 
Addition  9.31  10
and subtraction:
tion and brief illustrations, in this section of the accompany- 1!0 !3!5!10 !1!
configuration. and hence
ing e-book. (a  ib)  (c  id )  (a  c)  i(b  d )
The next step is to develop an expression for the number of dH  dU  pdV  Vdp
ways that a general configuration {N0,N1,…} can be achieved. Multiplication:
It will turn out to be more convenient to deal with the natu- Figure 7B.3
Figure 2B.1 When is a system
called is subjected to constant pressure and physical sign
Annotated This numberequations and
the equation
weight of labels
the configuration and ralStep 2 Introduceidthe
logarithm
(a  ib) (cofthe
definition
) weight,
 (ac  bd ln)W of dU
 i,(rather
bc  adthan ) with the weight
is free to change its volume, some of the energy supplied as heat this wavefun
denoted W . itself:
Because dU = dq + dw this expression becomes
may escape backmany
into the surroundings as work. In suchhow
a case, Two special relations are: distribution
We have annotated equations to help you follow they
the change in internal energy is smaller than the energy supplied dU  2 (x/y) =2 ln x −2ln1/y2
1/ln
the density
are developed. An annotation can help you travel across the dH  dq  d| w
Modulus: z | p(zdVz ) Vd(px  y )
as heat. N ! i
equals sign:Howit isis athat
reminder of the substitution used, an approxi-
done? 13A.1 = ln relation: e =cos lnN! i−sin ln(N
Evaluating the weight of a ln WEuler’s 0 ! N1 ! N
, which 2!
implies ) that ei  1, The Born
mation made, the terms that have been assumed constant, an N0 i!N1 ! N
 i2
!
configuration cos   2 (e  e ), and sin    2 i(e  e ).
1 1 i  i
significance
integral used, and so on. An annotation can also be a reminder because |ψ
Consider the N balls into We ln xy = ln x + ln y
of the significance of number of ways of
an individual distributing
term in an expression. bins
direct signifi
labelled 0, 1, 2 … such that there are N0 balls in bin 0, N1 in bin
sometimes collect into a small box a collection of numbers or = ln N ! − ln N 0 ! − ln N1 ! − ln N 2 ! − = ln N ! − ∑ ln N i ! function: o
1, and so on. The first ball can be selected in N different ways,
symbols totheshow
next how they
ball in N −carry fromways
1 different one from
line to
thethe next.
balls Many
remaining, Because |ψ| dx is a (dimensionless) probability, |ψ|2 is the
2 i
cant, and b
of the equations are labelled to highlight their significance.
and so on. Therefore, there are N(N − 1) … 1 = N! ways of probability
One reason fordensity, withlnW
introducing the isdimensions of 1/length
that it is easier to make ap-(for a may corres
selecting the balls. one-dimensional
proximations. system). the
In particular, Thefactorials can be ψ
wavefunction itself is called
simplified by region (Fig
There are N0! different ways in which the balls could have the probability
using amplitude.1For a particle free to move in three
Stirling’s approximation tive regions
been chosen to fill bin 0 (Fig. 13A.1). Similarly, there are N1! dimensions (for example, an electron near a nucleus in an because it g
ways in which the balls in bin 1 could have been chosen, and ln x !  x ln x  x Stirling’s approximation [ x >> 1] (13A.2)
atom), the wavefunction depends on the coordinates x, y, and structive in
so on. Therefore, the total number of distinguishable ways of Then
z andtheisapproximate
denoted ψ(r). expression
In this forcasethethe
weight
Bornisinterpretation is A wavefu
Checklist of concepts
☐ 1. Energy transferred as heat at constant pressure is equal ☐ 3. The heat capa
to the change in enthalpy of a system. heat capacity)
☐ 2. Enthalpy changes can be measured in an isobaric temperature.
x Using the book
calorimeter.

Checklists of equations
A handy checklist at the end of each topic summarizes the Checklist of equations
most important equations and the conditions under which
Property Equation Comment
they apply. Don’t think, however, that you have to memorize
every equation in these checklists: they are collected there for Enthalpy H = U + pV Definition

ready reference. Heat transfer at constant pressure dH = dqp, ΔH = qp No additional work


Relation between ΔH and ΔU ΔH = ΔU + ΔngRT Isothermal process, per
Heat capacity at constant pressure Cp = (∂H/∂T)p Definition
Relation between heat capacities Cp − CV = nR Perfect gas
Video tutorials on key equations
Video tutorials to accompany each Focus dig deeper into some
of the key equations used throughout that Focus, emphasizing
the significance of an equation, and highlighting connections
with material elsewhere in the book.
48 2 The First Law
Living graphs
The educational value of many graphs can be heightened by Table 2B.1 Temperature variation of molar heat capacities,
seeing—in a very direct way—how relevant parameters, such C p ,m /(JK −1 mol−1 ) = a + bT + c /T 2 *
as temperature or pressure, affect the plot. You Bcan now in- a b/(10−3 K−1) c/(105 K2)
teract with key graphs throughout the text in order to ex-
Enthalpy, H

C(s, graphite) 16.86 4.77 −8.54


plore how they respond as the parameters are changed. These
CO2(g) 44.22 8.79 −8.62
graphs are clearly flagged throughout theInternal
book, and you can
H2O(l) 75.29 0 0
find links to the dynamic
A versions in the corresponding
energy, U loca-
N2(g) 28.58 3.77 −0.50
tion in the e-book.
* More values are given in the Resource section.

Temperature, T
Atkins-Chap02_033-074.indd 49
The empirical parameters a, b, and c are independent of tempera-
Figure 2B.3 The constant-pressure heat capacity at a particular ture. Their values are found by fitting this expression to experi-
SET TING UP AND SOLVING PROBLEMS
temperature is the slope of the tangent to a curve of the enthalpy mental data on many substances, as shown in Table 2B.1.
of a system plotted against temperature (at constant pressure). If eqn 2B.8 is used to calculate the change 2BinEnthalpy 47
enthalpy be-
For gases, at a given temperature the slope of enthalpy versus
tween two temperatures T1 and T2, integration by using Integral
temperature is steeper than that of internal energy versus
A.1 in the Resource section gives
Brief illustrations
temperature, and Cp,m is larger than CV,m.
mass densities of the polymorphs are 2.71 g cm −3
(calcite) and Brief illustration
T2 2B.2
T2  c 
H   C p dT    a  bT  2  dT
−3
2.93 g cm (aragonite).
A Brief illustration shows you how to use an equation or con- T1 T1
 T
In the reaction 3 H2(g) + N2(g) → 2 NH3(g), 4 mol of gas-
cept thatishas
thejust
Collect been
your
heat introduced
perThe
thoughts
capacity mole in of
thesubstance;
starting text.
pointItfor
shows
itthe youintensive
how
calculation
is an T2
phase molecules
 c  by 2 mol of gas-phase molecules,
is replaced
to use data is and
the manipulate
property. units
relation between thecorrectly.
enthalpy ofIt also helps you
a substance andtoits   aT  12 bT 2   (2B.9)
−1
internal energy (eqn so Δng =  −2 mol. Therefore,T  T at 298 K, when RT = 2.5 kJ mol ,
become familiar
The heat with the
capacity at2B.1).
magnitudesYou need
constant to express
relatesthe
of quantities.
pressure thedifference
change in the molar enthalpy and molar internal energy changes taking
1

betweentothe
enthalpy two quantities
a change in termsFor
in temperature. of the pressure and
infinitesimal the
changes 2 by  1 1
difference of their molar volumes.
of temperature, eqn 2B.5 implies thatThe latter can be calculated place inathe
(T2 system
T1 )  12are
b Trelated
2
2  T1 
 c  
T T
Examples
from their molar masses, M, and their mass densities, ρ, by H  U  (2)  RT  5.0 kJmo  2 11 1 
m m
dH =ρC=p dM/V
using T (at
m.
constant pressure) (2B.6a)
Worked Examples are more detailed illustrations of the appli- Note that the difference is expressed in kilojoules, not joules
If The
the heat capacity
solution The ischange
constant inover the range
enthalpy when of temperatures
the transitionof as in Example
Example 2B.2 2B.1. The enthalpy change is more negative than
cation ofinterest,
the material,
occurs then
and typically require you to assemble
is for a measurable increase in temperature
Evaluating an increase in enthalpy with
the change in internal energy because, although energy escapes
and deploy several relevant concepts and equations. temperature
 T
  from the system as heat when the reaction occurs, the system
Everyone has
H madifferent
T
H
C pmd(T  Cway
aragonite T to approach solving a problem,
)dTHm (Ccalcite )
p T
H
2 2
What is the
contracts change
as the in molar
product enthalpy
is formed, of N2 is
so energy when it is heated
restored to it as
p (T 2  T1 )
and it changes with  {U mexperience.
T 1
(a )  pVm (a )} To
1
{U mhelp
(c)  in
pVmthis
(c)}} process, we from 25 °Cthe
work from 100 °C? Use the heat capacity information in
to surroundings.
suggest how
whichyou canshould
be collect
summarized your
as
U m  p{Vm (a )  Vm (c)} thoughts and then pro- Table 2B.1.
ceed to a solution. All the worked
H  C p T (at constant pressure) Examples are accompanied
(2B.6b) Collect your thoughts The heat capacity of N2 changes with
by closely related self-tests to enable you to test yourbygrasp
where a denotes aragonite and c calcite. It follows of
substitut- temperature significantly in this range, so use eqn 2B.9.
Because
the material Vm =a M/ρ
ing after change
that in through
working enthalpy can be equated to the energy
our solution as set out in Exercises
supplied as heat at constant pressure, the practical form of this The solution Using a = 28.58 J K−1 mol−1, b = 3.77 × 10−3 J K−2 mol−1,
the Example. 5 of ΔHm−1− ΔUm for the reaction N2(g) + 3 H2(g) →
E2B.1 Calculate the value
c =at−0.50
equation is  1 1  2 and
NH3(g) 473 K.× 10 J K mol , T1 = 298 K, and T2 = 373 K, eqn 2B.9
H m  U m  pM    is written as
q p  C p T   (a )  (c)  (2B.7) E2B.2 When 0.100 mol of H2(g) is burnt in a flame calorimeter it is
H that
observed H mthe water
(373  H min(298
K )bath which
K )the apparatus is immersed increases
This expression
Substitution of shows
the data, how to measure
using M = 100.09 the constant-pressure
g mol −1
, gives in temperature by 13.64 1 K. When
1 0.100 mol C4H10(g), butane, is burnt in
heat capacity of a sample: a measured quantity of energy is the same  (28.58 JK
apparatus mol )  (373
the temperature riseKis6.03
298K.KThe
) molar enthalpy of
combustionof1 (H32.(g) −285
is 10 2 −1. Calculate
3 kJ mol the
supplied
H m as
 heat
U m under
(1.0  conditions
105 Pa )  (100of.09
constant
g mo11pressure
) (as in a 2
77  JK mol 1
)  {(373 K )molar
2 enthalpy
 (298 K )2 }
of combustion of butane.
sample exposed to the  atmosphere and free to expand), and the  1 1 
1 1 (0.50  105 JKmol 1 )   
temperature rise ismonitored.
  3 
 373 K 298 K 
 2.93capacity
g cm 3
2.71 g cm   
The variation of heat
5
with temperature
3 1
can some-
Pa m 3this 1
2B.2 The variation of enthalpy with
times be ignored if 2the.8  10 Pa cm molrange
temperature  0is.28
small; molis an The final result is
Using the book xi

Self-check questions
This edition introduces self-check questions throughout the
text, which can be found at the end of most sections in the
e-book. They test your comprehension of the concepts dis-
cussed in each section, and provide instant feedback to help
you monitor your progress and reinforce your learning. Some
of the questions are multiple choice; for them the ‘wrong’ an-
swers are not simply random numbers but the result of errors
that, in our experience, students often make. The feedback
from the multiple choice questions not only explains the cor-
rect method, but also points out the mistakes that led to the
incorrect answer. By working through the multiple-choice Exercises and problems 27

questions you will be well prepared to tackle more challenging


FOCUS 1 The properties of gases
exercises and problems.
To test your understanding of this material, work through Selected solutions can be found at the end of this Focus in
the Exercises, Additional exercises, Discussion questions, and the e-book. Solutions to even-numbered questions are available

Discussion questions Problems found throughout this Focus. online only to lecturers.

Discussion questions appear at the end of each Focus, and are TOPIC 1A The perfect gas
organized by Topic. They are designed to encourage you to Discussion questions
D1A.1 Explain how the perfect gas equation of state arises by combination of D1A.2 Explain the term ‘partial pressure’ and explain why Dalton’s law is a

reflect on the material you have just read, to review the key Boyle’s law, Charles’s law, and Avogadro’s principle. limiting law.

concepts, and sometimes to think about its implications and Additional exercises

limitations.
E1A.8 Express (i) 22.5 kPa in atmospheres and (ii) 770 Torr in pascals. 60 per cent. Hint: Relative humidity is the prevailing partial pressure of water
3 vapour expressed as a percentage of the vapour pressure of water vapour at
E1A.9 Could 25 g of argon gas in a vessel of volume 1.5 dm exert a pressure
the same temperature (in this case, 35.6 mbar).
of 2.0 bar at 30 °C if it behaved as a perfect gas? If not, what pressure would
it exert? E1A.18 Calculate the mass of water vapour present in a room of volume
250 m3 that contains air at 23 °C on a day when the relative humidity is
E1A.10 A perfect gas undergoes isothermal expansion, which increases its
53 per cent (in this case, 28.1 mbar).
volume by 2.20 dm3. The final pressure and volume of the gas are 5.04 bar
and 4.65 dm3, respectively. Calculate the original pressure of the gas in E1A.19 Given that the mass density of air at 0.987 bar and 27 °C is
(i) bar, (ii) atm. 1.146 kg m−3, calculate the mole fraction and partial pressure of nitrogen
and oxygen assuming that (i) air consists only of these two gases, (ii) air also

Exercises and problems


E1A.11 A perfect gas undergoes isothermal compression, which reduces its
3 contains 1.0 mole per cent Ar.
volume by 1.80 dm . The final pressure and volume of the gas are 1.97 bar
and 2.14 dm3, respectively. Calculate the original pressure of the gas in E1A.20 A gas mixture consists of 320 mg of methane, 175 mg of argon, and
(i) bar, (ii) torr. 225 mg of neon. The partial pressure of neon at 300 K is 8.87 kPa. Calculate
−2 (i) the volume and (ii) the total pressure of the mixture.
E1A.12 A car tyre (an automobile tire) was inflated to a pressure of 24 lb in

Exercises are provided throughout the main text and, along −2


(1.00 atm = 14.7 lb in ) on a winter’s day when the temperature was −5 °C.
What pressure will be found, assuming no leaks have occurred and that the
E1A.21 The mass density of a gaseous compound was found to be 1.23 kg m
at 330 K and 20 kPa. What is the molar mass of the compound?
−3

volume is constant, on a subsequent summer’s day when the temperature is


with Problems, at the end of every Focus. They are all organ-
3
E1A.22 In an experiment to measure the molar mass of a gas, 250 cm of the
35 °C? What complications should be taken into account in practice?
gas was confined in a glass vessel. The pressure was 152 Torr at 298 K, and
E1A.13 A sample of hydrogen gas was found to have a pressure of 125 kPa after correcting for buoyancy effects, the mass of the gas was 33.5 mg. What is

ised by Topic. Exercises are designed as relatively straightfor- when the temperature was 23 °C. What can its pressure be expected to be
when the temperature is 11 °C?
the molar mass of the gas?
−3
E1A.23 The densities of air at −85 °C, 0 °C, and 100 °C are 1.877 g dm ,

ward numerical tests; the Problems are more challenging and 1.294 g dm−3, and 0.946 g dm−3, respectively. From these data, and assuming
3
E1A.14 A sample of 255 mg of neon occupies 3.00 dm at 122 K. Use the
perfect gas law to calculate the pressure of the gas. that air obeys Charles’s law, determine a value for the absolute zero of
temperature in degrees Celsius.

typically involve constructing a more detailed answer. For this


3 3
E1A.15 A homeowner uses 4.00 × 10 m of natural gas in a year to heat a
3
home. Assume that natural gas is all methane, CH4, and that methane is a E1A.24 A certain sample of a gas has a volume of 20.00 dm at 0 °C and
perfect gas for the conditions of this problem, which are 1.00 atm and 20 °C. 1.000 atm. A plot of the experimental data of its volume against the Celsius

new edition, detailed solutions are provided in the e-book in What is the mass of gas used?
E1A.16 At 100 °C and 16.0 kPa, the mass density of phosphorus vapour is
temperature, θ, at constant p, gives a straight line of slope 0.0741 dm3 °C−1.
From these data alone (without making use of the perfect gas law), determine
the absolute zero of temperature in degrees Celsius.

the same location as they appear in print.


0.6388 kg m−3. What is the molecular formula of phosphorus under these
3
conditions? E1A.25 A vessel of volume 22.4 dm contains 1.5 mol H2(g) and 2.5 mol N2(g)
at 273.15 K. Calculate (i) the mole fractions of each component, (ii) their
E1A.17 Calculate the mass of water vapour present in a room of volume

For the Examples and Problems at the end of each Focus de- 3
400 m that contains air at 27 °C on a day when the relative humidity is
partial pressures, and (iii) their total pressure.

tailed solutions to the odd-numbered questions are provided Problems


P1A.1 A manometer consists of a U-shaped tube containing a liquid. One side pressure, ρ is the mass density of the liquid in the tube, g = 9.806 m s−2 is the
in the e-book; solutions to the even-numbered questions are is connected to the apparatus and the other is open to the atmosphere. The
pressure p inside the apparatus is given p = pex + ρgh, where pex is the external
acceleration of free fall, and h is the difference in heights of the liquid in the
two sides of the tube. (The quantity ρgh is the hydrostatic pressure exerted by

available only to lecturers. Exercises and problems 139

P4B.16 Figure 4B.1 gives a schematic representation of how the chemical temperature, of these lines. Is there any restriction on the value this curvature
potentials of the solid, liquid, and gaseous phases of a substance vary with can take? For water, compare the curvature of the liquid line with that for the
Atkins-Chap01_003-032.indd 27 01-10-2022 12:40:10
temperature. All have a negative slope, but it is unlikely that they are straight gas in the region of the normal boiling point. The molar heat capacities at
lines as indicated in the illustration. Derive an expression for the curvatures, constant pressure of the liquid and gas are 75.3 J K−1 mol−1 and 33.6 J K−1 mol−1,
that is, the second derivative of the chemical potential with respect to respectively.

Integrated activities
FOCUS 4 Physical transformations of pure substances
At the end of every Focus you will find questions that span Integrated activities

several Topics. They are designed to help you use your I4.1 Construct the phase diagram for benzene near its triple point at
36 Torr and 5.50 °C from the following data: ∆fusH = 10.6 kJ mol−1,
∆vapH = 30.8 kJ mol−1, ρ(s) = 0.891 g cm−3, ρ(l) = 0.879 g cm−3.
(c) Plot Tm/(ΔhbHm/ΔhbSm) for 5 ≤ N ≤ 20. At what value of N does Tm change
by less than 1 per cent when N increases by 1?

knowledge creatively in a variety of ways.



I4.4 A substance as well-known as methane still receives research attention

I4.2 In an investigation of thermophysical properties of methylbenzene because it is an important component of natural gas, a commonly used fossil
R.D. Goodwin (J. Phys. Chem. Ref. Data 18, 1565 (1989)) presented fuel. Friend et al. have published a review of thermophysical properties of
expressions for two coexistence curves. The solid–liquid curve is given by methane (D.G. Friend, J.F. Ely, and H. Ingham, J. Phys. Chem. Ref. Data 18,
583 (1989)), which included the following vapour pressure data describing the
p/bar  p3 /bar  1000(5.60  11.727 x )x liquid–vapour coexistence curve.

where x = T/T3 − 1 and the triple point pressure and temperature are T/K 100 108 110 112 114 120 130 140 150 160 170 190
p/MPa 0.034 0.074 0.088 0.104 0.122 0.192 0.368 0.642 1.041 1.593 2.329 4.521
p3 = 0.4362 μbar and T3 = 178.15 K. The liquid–vapour curve is given by
(a) Plot the liquid–vapour coexistence curve. (b) Estimate the standard
ln( p/bar )  10.418/y  21.157  15.996 y  14.015 y 2 boiling point of methane. (c) Compute the standard enthalpy of vaporization
 5.0120 y 3  4.7334(1  y )1.70 of methane (at the standard boiling point), given that the molar volumes of
the liquid and vapour at the standard boiling point are 3.80 × 10−2 dm3 mol−1
and 8.89 dm3 mol−1, respectively.
where y = T/Tc = T/(593.95 K). (a) Plot the solid–liquid and liquid–vapour
coexistence curves. (b) Estimate the standard melting point of methylbenzene. ‡
I4.5 Diamond is the hardest substance and the best conductor of heat yet
(c) Estimate the standard boiling point of methylbenzene. (The equation you characterized. For these reasons, it is used widely in industrial applications
will need to solve to find this quantity cannot be solved by hand, so you should that require a strong abrasive. Unfortunately, it is difficult to synthesize
use a numerical approach, e.g. by using mathematical software.) (d) Calculate diamond from the more readily available allotropes of carbon, such as
the standard enthalpy of vaporization of methylbenzene at the standard boiling graphite. To illustrate this point, the following approach can be used to
point, given that the molar volumes of the liquid and vapour at the standard estimate the pressure required to convert graphite into diamond at 25 °C (i.e.
boiling point are 0.12 dm3 mol−1 and 30.3 dm3 mol−1, respectively. the pressure at which the conversion becomes spontaneous). The aim is to
I4.3 Proteins are polymers of amino acids that can exist in ordered structures
find an expression for ∆rG for the process graphite → diamond as a function
of the applied pressure, and then to determine the pressure at which the Gibbs
stabilized by a variety of molecular interactions. However, when certain
energy change becomes negative. (a) Derive the following expression for the
conditions are changed, the compact structure of a polypeptide chain may
pressure variation of ∆rG at constant temperature
collapse into a random coil. This structural change may be regarded as a phase
transition occurring at a characteristic transition temperature, the melting
  r G 
temperature, Tm, which increases with the strength and number of intermolecular    Vm,d  Vm,gr
interactions in the chain. A thermodynamic treatment allows predictions to  p T
be made of the temperature Tm for the unfolding of a helical polypeptide held
together by hydrogen bonds into a random coil. If a polypeptide has N amino where Vm,gr is the molar volume of graphite and Vm,d that of diamond. (b) The
acid residues, N − 4 hydrogen bonds are formed to form an α-helix, the most difficulty with dealing with the previous expression is that the Vm depends
common type of helix in naturally occurring proteins (see Topic 14D). Because on the pressure. This dependence is handled as follows. Consider ∆rG to be a
the first and last residues in the chain are free to move, N − 2 residues form the function of pressure and form a Taylor expansion about p = p⦵:
compact helix and have restricted motion. Based on these ideas, the molar Gibbs A 
    B
energy of unfolding of a polypeptide with N ≥ 5 may be written as ⦵   G  ⦵   2 rG  ⦵
 r G( p)   r G( p )   r  ( p  p )  12  2 
( p  p  )2
 p  p p⦵  p  p p⦵
 unfoldG  (N  4) hb H  (N  2)T  hb S where the derivatives are evaluated at p = p⦵ and the series is truncated after
the second-order term. Term A can be found from the expression in part (a)
xii Using the book

TAKING YOUR LEARNING FURTHER

‘Impact’ sections details of some of the more advanced derivations. They are
listed at the beginning of the text and are referred to where
‘Impact’ sections show you how physical chemistry is ap- they are relevant. You can find a compilation of Deeper Looks
plied in a variety of modern contexts. They showcase physical at the end of the e-book.
chemistry as an evolving subject. These sections are listed at
the beginning of the text, and are referred to at appropriate
places elsewhere. You can find a compilation of ‘Impact’ sec- Group theory tables
tions at the end of the e-book. If you need character tables, you can find them at the end of
the Resource section.
A deeper look
These sections take some of the material in the text further.
Read them if you want to extend your knowledge and see the

TO THE INSTRUC TOR


We have designed the text to give you maximum flexibility in Key equations
the selection and sequence of Topics, while the grouping of
Topics into Focuses helps to maintain the unity of the subject. Supplied in Word format so you can download and edit them.
Additional resources are:
Solutions to exercises, problems, and
Figures and tables from the book integrated activities
Lecturers can find the artwork and tables from the book in For the discussion questions, examples, problems, and in-
ready-to-download format. They may be used for lectures tegrated activities detailed solutions to the even-numbered
without charge (but not for commercial purposes without spe- questions are available to lecturers online, so they can be set as
cific permission). homework or used as discussion points in class.
Lecturer resources are available only to registered adopters
of the textbook. To register, simply visit www.oup.com/he/
pchem12e and follow the appropriate links.
ABOUT THE AUTHORS
Peter Atkins is a fellow of Lincoln College, Oxford, and emeritus professor of physical chemistry in
the University of Oxford. He is the author of over seventy books for students and a general audience.
His texts are market leaders around the globe. A frequent lecturer throughout the world, he has held
visiting professorships in France, Israel, Japan, China, Russia, and New Zealand. He was the founding
chairman of the Committee on Chemistry Education of the International Union of Pure and Applied
Chemistry and was a member of IUPAC’s Physical and Biophysical Chemistry Division.

Photograph by Natasha
Ellis-Knight.

Julio de Paula is Professor of Chemistry at Lewis & Clark College. A native of Brazil, he received a
B.A. degree in chemistry from Rutgers, The State University of New Jersey, and a Ph.D. in biophysical
chemistry from Yale University. His research activities encompass the areas of molecular spectroscopy,
photochemistry, and nanoscience. He has taught courses in general chemistry, physical chemistry, bio-
chemistry, inorganic chemistry, instrumental analysis, environmental chemistry, and writing. Among
his professional honours are a Christian and Mary Lindback Award for Distinguished Teaching, a
Henry Dreyfus Teacher-Scholar Award, and a STAR Award from the Research Corporation for Science
Advancement.

James Keeler is Associate Professor of Chemistry, University of Cambridge, and Walters Fellow in
Chemistry at Selwyn College. He received his first degree and doctorate from the University of Oxford,
specializing in nuclear magnetic resonance spectroscopy. He is presently Head of Department, and be-
fore that was Director of Teaching in the department and also Senior Tutor at Selwyn College.

Photograph by Nathan Pitt,


© University of Cambridge.
ACKNOWLEDGEMENTS

A book as extensive as this could not have been written with- Rosalind Baverstock, Durham University
out significant input from many individuals. We would like to Grace Butler, Trinity College Dublin
thank the hundreds of instructors and students who contrib- Kaylyn Cater, Cardiff University
uted to this and the previous eleven editions: Ruth Comerford, University College Dublin
Orlagh Fraser, University of Aberdeen
Scott Anderson, University of Utah
Dexin Gao, University College London
Milan Antonijevic, University of Greenwich
Suruthi Gnanenthiran, University of Bath
Elena Besley, University of Greenwich
Milena Gonakova, University of the West of England Bristol
Merete Bilde, Aarhus University
Joseph Ingle, University of Lincoln
Matthew Blunt, University College London
Jeremy Lee, University of Durham
Simon Bott, Swansea University
Luize Luse, Heriot-Watt University
Klaus Braagaard Møller, Technical University of Denmark
Zoe Macpherson, University of Strathclyde
Wesley Browne, University of Groningen
Sukhbir Mann, University College London
Sean Decatur, Kenyon College
Declan Meehan, Trinity College Dublin
Anthony Harriman, Newcastle University
Eva Pogacar, Heriot-Watt University
Rigoberto Hernandez, Johns Hopkins University
Pawel Pokorski, Heriot-Watt University
J. Grant Hill, University of Sheffield
Fintan Reid, University of Strathclyde
Kayla Keller, Kentucky Wesleyan College
Gabrielle Rennie, University of Strathclyde
Kathleen Knierim, University of Louisiana Lafayette
Annabel Savage, Manchester Metropolitan University
Tim Kowalczyk, Western Washington University
Sophie Shearlaw, University of Strathclyde
Kristin Dawn Krantzman, College of Charleston
Yutong Shen, University College London
Hai Lin, University of Colorado Denver
Saleh Soomro, University College London
Mikko Linnolahti, University of Eastern Finland
Matthew Tully, Bangor University
Mike Lyons, Trinity College Dublin
Richard Vesely, University of Cambridge
Jason McAfee, University of North Texas
Phoebe Williams, Nottingham Trent University
Joseph McDouall, University of Manchester
Hugo Meekes, Radboud University We would also like to thank Michael Clugston for proofread-
Gareth Morris, University of Manchester ing the entire book, and Peter Bolgar, Haydn Lloyd, Aimee
David Rowley, University College London North, Vladimiras Oleinikovas, and Stephanie Smith who all
Nessima Salhi, Uppsala University worked alongside James Keeler in the writing of the solutions
Andy S. Sardjan, University of Groningen to the exercises and problems. The multiple-choice questions
Trevor Sears, Stony Brook University were developed in large part by Dr Stephanie Smith (Yusuf
Gemma Shearman, Kingston University Hamied Department of Chemistry and Pembroke College,
John Slattery, University of York University of Cambridge). These questions and further exer-
Catherine Southern, DePaul University cises were integrated into the text by Chloe Balhatchet (Yusuf
Michael Staniforth, University of Warwick Hamied Department of Chemistry and Selwyn College,
Stefan Stoll, University of Washington University of Cambridge), who also worked on the living
Mahamud Subir, Ball State University graphs. The solutions to the exercises and problems are taken
Enrico Tapavicza, CSU Long Beach from the solutions manual for the eleventh edition prepared
Jeroen van Duifneveldt, University of Bristol by Peter Bolgar, Haydn Lloyd, Aimee North, Vladimiras
Darren Walsh, University of Nottingham Oleinikovas, Stephanie Smith, and James Keeler, with addi-
Graeme Watson, Trinity College Dublin tional contributions from Chloe Balhatchet.
Darren L. Williams, Sam Houston State University Last, but by no means least, we acknowledge our two com-
Elisabeth R. Young, Lehigh University missioning editors, Jonathan Crowe of Oxford University
Press and Jason Noe of OUP USA, and their teams for their
Our special thanks also go to the many student reviewers who
assistance, advice, encouragement, and patience. We owe
helped to shape this twelfth edition:
special thanks to Katy Underhill, Maria Bajo Gutiérrez, and
Katherine Ailles, University of York Keith Faivre from OUP, who skillfully shepherded this com-
Mohammad Usman Ali, University of Manchester plex project to completion.
BRIEF CONTENTS

ENERGY A First Look xxxiii FOCUS 12 Magnetic resonance 499

FOCUS 1 The properties of gases 3 FOCUS 13 Statistical thermodynamics 543

FOCUS 2 The First Law 33 FOCUS 14 Molecular interactions 597

FOCUS 3 The Second and Third Laws 75 FOCUS 15 Solids 655

FOCUS 4  hysical transformations of pure


P FOCUS 16 Molecules in motion 707
substances 119
FOCUS 17 Chemical kinetics 737
FOCUS 5 Simple mixtures 141
FOCUS 18 Reaction dynamics 793
FOCUS 6 Chemical equilibrium 205
FOCUS 19 Processes at solid surfaces 835
FOCUS 7 Quantum theory 237
Resource section
FOCUS 8 Atomic structure and spectra 305 1 Mathematical resources 878
2 Quantities and units 882
FOCUS 9 Molecular structure 343 3 Data 884
4 Character tables 910
FOCUS 10 Molecular symmetry 397
Index 915
FOCUS 11 Molecular spectroscopy 427
FULL CONTENTS

Conventions xxvii FOCUS 2 The First Law 33


Physical chemistry: people and perspectives xxvii TOPIC 2A Internal energy 34
List of tables xxviii 2A.1 Work, heat, and energy 34
List of The chemist’s toolkits xxx (a) Definitions 34
List of material provided as A deeper look xxxi (b) The molecular interpretation of heat and work 35

List of Impacts xxxii 2A.2 The definition of internal energy 36


(a) Molecular interpretation of internal energy 36
(b) The formulation of the First Law 37
ENERGY A First Look xxxiii 2A.3 Expansion work 37
(a) The general expression for work 37

FOCUS 1 The properties of gases 3 (b) Expansion against constant pressure 38


(c) Reversible expansion 39
TOPIC 1A The perfect gas 4 (d) Isothermal reversible expansion of a perfect gas 39
1A.1 Variables of state 4 2A.4 Heat transactions 40
(a) Pressure and volume 4 (a) Calorimetry 40
(b) Temperature 5 (b) Heat capacity 41
(c) Amount 5 Checklist of concepts 43
(d) Intensive and extensive properties 5
Checklist of equations 44
1A.2 Equations of state 6
(a) The empirical basis of the perfect gas law 6 TOPIC 2B Enthalpy 45
(b) The value of the gas constant 8 2B.1 The definition of enthalpy 45
(c) Mixtures of gases 9 (a) Enthalpy change and heat transfer 45
Checklist of concepts 10 (b) Calorimetry 46
Checklist of equations 10 2B.2 The variation of enthalpy with temperature 47
(a) Heat capacity at constant pressure 47
TOPIC 1B The kinetic model 11 (b) The relation between heat capacities 49
1B.1 The model 11 Checklist of concepts 49
(a) Pressure and molecular speeds 11
Checklist of equations 49
(b) The Maxwell–Boltzmann distribution of speeds 12
(c) Mean values 14 TOPIC 2C Thermochemistry 50
1B.2 Collisions 16 2C.1 Standard enthalpy changes 50
(a) The collision frequency 16 (a) Enthalpies of physical change 50
(b) The mean free path 16 (b) Enthalpies of chemical change 51
Checklist of concepts 17 (c) Hess’s law 52
Checklist of equations 17 2C.2 Standard enthalpies of formation 53
2C.3 The temperature dependence of reaction enthalpies 54
TOPIC 1C Real gases 18 2C.4 Experimental techniques 55
1C.1 Deviations from perfect behaviour 18 (a) Differential scanning calorimetry 55
(a) The compression factor 19
(b) Isothermal titration calorimetry 56
(b) Virial coefficients 20
Checklist of concepts 56
(c) Critical constants 21
Checklist of equations 57
1C.2 The van der Waals equation 22
(a) Formulation of the equation 22 TOPIC 2D State functions and exact differentials 58
(b) The features of the equation 23
2D.1 Exact and inexact differentials 58
(c) The principle of corresponding states 24
2D.2 Changes in internal energy 59
Checklist of concepts 26
(a) General considerations 59
Checklist of equations 26 (b) Changes in internal energy at constant pressure 60
xviii Full Contents

2D.3 Changes in enthalpy 62 TOPIC 3E Combining the First and Second Laws 104
2D.4 The Joule–Thomson effect 63 3E.1 Properties of the internal energy 104
Checklist of concepts 64 (a) The Maxwell relations 105
Checklist of equations 65 (b) The variation of internal energy with volume 106
3E.2 Properties of the Gibbs energy 107
TOPIC 2E Adiabatic changes 66 (a) General considerations 107
2E.1 The change in temperature 66 (b) The variation of the Gibbs energy with temperature 108
2E.2 The change in pressure 67 (c) The variation of the Gibbs energy of condensed phases
Checklist of concepts 68 with pressure 109
(d) The variation of the Gibbs energy of gases with pressure 109
Checklist of equations 68
Checklist of concepts 110
Checklist of equations 111
FOCUS 3 The Second and Third Laws 75
TOPIC 3A Entropy 76 FOCUS 4 Physical transformations
3A.1 The Second Law 76 of pure substances 119
3A.2 The definition of entropy 78
(a) The thermodynamic definition of entropy 78
TOPIC 4A Phase diagrams of pure substances 120
(b) The statistical definition of entropy 79
4A.1 The stabilities of phases 120
(a) The number of phases 120
3A.3 The entropy as a state function 80
(b) Phase transitions 121
(a) The Carnot cycle 81
(c) Thermodynamic criteria of phase stability 121
(b) The thermodynamic temperature 83
(c) The Clausius inequality 84
4A.2 Coexistence curves 122
(a) Characteristic properties related to phase transitions 122
Checklist of concepts 85
(b) The phase rule 123
Checklist of equations 85
4A.3 Three representative phase diagrams 125
TOPIC 3B Entropy changes accompanying (a) Carbon dioxide 125
(b) Water 125
specific processes 86
(c) Helium 126
3B.1 Expansion 86
Checklist of concepts 127
3B.2 Phase transitions 87
Checklist of equations 127
3B.3 Heating 88
3B.4 Composite processes 89 TOPIC 4B Thermodynamic aspects of phase
Checklist of concepts 90 transitions 128
Checklist of equations 90 4B.1 The dependence of stability on the conditions 128
(a) The temperature dependence of phase stability 128
TOPIC 3C The measurement of entropy 91 (b) The response of melting to applied pressure 129
3C.1 The calorimetric measurement of entropy 91 (c) The vapour pressure of a liquid subjected to pressure 130
3C.2 The Third Law 92 4B.2 The location of coexistence curves 131
(a) The Nernst heat theorem 92 (a) The slopes of the coexistence curves 131
(b) Third-Law entropies 93 (b) The solid–liquid coexistence curve 132
(c) The temperature dependence of reaction entropy 94 (c) The liquid–vapour coexistence curve 132
Checklist of concepts 95 (d) The solid–vapour coexistence curve 134
Checklist of equations 95 Checklist of concepts 134
Checklist of equations 135
TOPIC 3D Concentrating on the system 96
3D.1 The Helmholtz and Gibbs energies 96
(a) Criteria of spontaneity 96
FOCUS 5 Simple mixtures 141
(b) Some remarks on the Helmholtz energy 97 TOPIC 5A The thermodynamic description
(c) Maximum work 97 of mixtures 143
(d) Some remarks on the Gibbs energy 99 5A.1 Partial molar quantities 143
(e) Maximum non-expansion work 99 (a) Partial molar volume 143
3D.2 Standard molar Gibbs energies 100 (b) Partial molar Gibbs energies 145
(a) Gibbs energies of formation 100 (c) The Gibbs–Duhem equation 146
(b) The Born equation 101 5A.2 The thermodynamics of mixing 147
Checklist of concepts 102 (a) The Gibbs energy of mixing of perfect gases 148
Checklist of equations 103 (b) Other thermodynamic mixing functions 149
Full Contents xix

5A.3 The chemical potentials of liquids 150 (c) Activities in terms of molalities 188
(a) Ideal solutions 150 5F.3 The activities of regular solutions 189
(b) Ideal–dilute solutions 151 5F.4 The activities of ions 190
Checklist of concepts 153 (a) Mean activity coefficients 190
Checklist of equations 154 (b) The Debye–Hückel limiting law 191
(c) Extensions of the limiting law 192
TOPIC 5B The properties of solutions 155 Checklist of concepts 193
5B.1 Liquid mixtures 155 Checklist of equations 193
(a) Ideal solutions 155
(b) Excess functions and regular solutions 156
5B.2 Colligative properties 158 FOCUS 6 Chemical equilibrium 205
(a) The common features of colligative properties 158
(b) The elevation of boiling point 159
TOPIC 6A The equilibrium constant 206
(c) The depression of freezing point 161 6A.1 The Gibbs energy minimum 206
(d) Solubility 161 (a) The reaction Gibbs energy 206

(e) Osmosis 162 (b) Exergonic and endergonic reactions 207

Checklist of concepts 165 6A.2 The description of equilibrium 207


(a) Perfect gas equilibria 208
Checklist of equations 165
(b) The general case of a reaction 209

TOPIC 5C Phase diagrams of binary (c) The relation between equilibrium constants 211
(d) Molecular interpretation of the equilibrium constant 212
systems: liquids 166
5C.1 Vapour pressure diagrams 166 Checklist of concepts 213

5C.2 Temperature–composition diagrams 168 Checklist of equations 213


(a) The construction of the diagrams 168
(b) The interpretation of the diagrams 169 TOPIC 6B The response of equilibria to
5C.3 Distillation 170 the conditions 214
(a) Fractional distillation 170 6B.1 The response to pressure 214
(b) Azeotropes 171 6B.2 The response to temperature 216
(c) Immiscible liquids 172 (a) The van ’t Hoff equation 216
5C.4 Liquid–liquid phase diagrams 172 (b) The value of K at different temperatures 217
(a) Phase separation 173 Checklist of concepts 218
(b) Critical solution temperatures 174 Checklist of equations 218
(c) The distillation of partially miscible liquids 175
Checklist of concepts 177 TOPIC 6C Electrochemical cells 219
Checklist of equations 177 6C.1 Half-reactions and electrodes 219
6C.2 Varieties of cell 220
TOPIC 5D Phase diagrams of binary (a) Liquid junction potentials 220
systems: solids 178 (b) Notation 221
5D.1 Eutectics 178 6C.3 The cell potential 221
5D.2 Reacting systems 180 (a) The Nernst equation 222
5D.3 Incongruent melting 180 (b) Cells at equilibrium 223

Checklist of concepts 181 6C.4 The determination of thermodynamic functions 224


Checklist of concepts 225
TOPIC 5E Phase diagrams of ternary systems 182 Checklist of equations 225
5E.1 Triangular phase diagrams 182
5E.2 Ternary systems 183 TOPIC 6D Electrode potentials 226
(a) Partially miscible liquids 183 6D.1 Standard potentials 226
(b) Ternary solids 184 (a) The measurement procedure 227
Checklist of concepts 185 (b) Combining measured values 228
6D.2 Applications of standard electrode potentials 228
TOPIC 5F Activities 186 (a) The electrochemical series 228
5F.1 The solvent activity 186 (b) The determination of activity coefficients 229

5F.2 The solute activity 187 (c) The determination of equilibrium constants 229

(a) Ideal–dilute solutions 187 Checklist of concepts 230


(b) Real solutes 187 Checklist of equations 230
xx Full Contents

FOCUS 7 Quantum theory 237 Checklist of concepts 282


Checklist of equations 282
TOPIC 7A The origins of quantum mechanics 239
7A.1 Energy quantization 239 TOPIC 7F Rotational motion 283
(a) Black-body radiation 239 7F.1 Rotation in two dimensions 283
(b) Heat capacity 242 (a) The solutions of the Schrödinger equation 284
(c) Atomic and molecular spectra 243 (b) Quantization of angular momentum 285
7A.2 Wave–particle duality 244 7F.2 Rotation in three dimensions 286
(a) The particle character of electromagnetic radiation 244 (a) The wavefunctions and energy levels 286
(b) The wave character of particles 246 (b) Angular momentum 289
Checklist of concepts 247 (c) The vector model 290
Checklist of equations 247 Checklist of concepts 291
Checklist of equations 292
TOPIC 7B Wavefunctions 248
7B.1 The Schrödinger equation 248
7B.2 The Born interpretation 248 FOCUS 8 Atomic structure and spectra 305
(a) Normalization 250 TOPIC 8A Hydrogenic atoms 306
(b) Constraints on the wavefunction 251
8A.1 The structure of hydrogenic atoms 306
(c) Quantization 251
(a) The separation of variables 306
Checklist of concepts 252 (b) The radial solutions 307
Checklist of equations 252 8A.2 Atomic orbitals and their energies 309
(a) The specification of orbitals 310
TOPIC 7C Operators and observables 253
(b) The energy levels 310
7C.1 Operators 253
(c) Ionization energies 311
(a) Eigenvalue equations 253
(d) Shells and subshells 311
(b) The construction of operators 254
(e) s Orbitals 312
(c) Hermitian operators 255
(f) Radial distribution functions 313
(d) Orthogonality 256
(g) p Orbitals 315
7C.2 Superpositions and expectation values 257 (h) d Orbitals 316
7C.3 The uncertainty principle 259 Checklist of concepts 316
7C.4 The postulates of quantum mechanics 261 Checklist of equations 317
Checklist of concepts 261
Checklist of equations 262 TOPIC 8B Many-electron atoms 318
8B.1 The orbital approximation 318
TOPIC 7D Translational motion 263 8B.2 The Pauli exclusion principle 319
7D.1 Free motion in one dimension 263 (a) Spin 319
7D.2 Confined motion in one dimension 264 (b) The Pauli principle 320
(a) The acceptable solutions 264 8B.3 The building-up principle 322
(b) The properties of the wavefunctions 265 (a) Penetration and shielding 322
(c) The properties of the energy 266 (b) Hund’s rules 323
7D.3 Confined motion in two and more dimensions 268 (c) Atomic and ionic radii 325
(a) Energy levels and wavefunctions 268 (d) Ionization energies and electron affinities 326
(b) Degeneracy 270 8B.4 Self-consistent field orbitals 327
7D.4 Tunnelling 271 Checklist of concepts 328
Checklist of concepts 273 Checklist of equations 328
Checklist of equations 274
TOPIC 8C Atomic spectra 329
TOPIC 7E Vibrational motion 275 8C.1 The spectra of hydrogenic atoms 329
7E.1 The harmonic oscillator 275 8C.2 The spectra of many-electron atoms 330
(a) The energy levels 276 (a) Singlet and triplet terms 330
(b) The wavefunctions 276 (b) Spin–orbit coupling 331
7E.2 Properties of the harmonic oscillator 279 (c) Term symbols 334
(a) Mean values 279 (d) Hund’s rules and term symbols 337
(b) Tunnelling 280 (e) Selection rules 337
Full Contents xxi

Checklist of concepts 338 (c) Density functional theory 381


Checklist of equations 338 (d) Practical calculations 381
(e) Graphical representations 381
Checklist of concepts 382
FOCUS 9 Molecular structure 343 Checklist of equations 382
PROLOGUE The Born–Oppenheimer approximation 345
TOPIC 9F Computational chemistry 383
TOPIC 9A Valence-bond theory 346 9F.1 The central challenge 383
9A.1 Diatomic molecules 346 9F.2 The Hartree−Fock formalism 384
9A.2 Resonance 348 9F.3 The Roothaan equations 385
9A.3 Polyatomic molecules 348 9F.4 Evaluation and approximation of the integrals 386
(a) Promotion 349
9F.5 Density functional theory 388
(b) Hybridization 350
Checklist of concepts 389
Checklist of concepts 352
Checklist of equations 390
Checklist of equations 352

TOPIC 9B Molecular orbital theory: the hydrogen FOCUS 10 Molecular symmetry 397
molecule-ion 353
9B.1 Linear combinations of atomic orbitals 353
TOPIC 10A Shape and symmetry 398
(a) The construction of linear combinations 353 10A.1 Symmetry operations and symmetry elements 398
(b) Bonding orbitals 354 10A.2 The symmetry classification of molecules 400
(c) Antibonding orbitals 356 (a) The groups C1, Ci, and Cs 402

9B.2 Orbital notation 358 (b) The groups Cn, Cnv, and Cnh 402
(c) The groups Dn, Dnh, and Dnd 403
Checklist of concepts 358
(d) The groups Sn 403
Checklist of equations 358
(e) The cubic groups 403

TOPIC 9C Molecular orbital theory: homonuclear (f) The full rotation group 404

diatomic molecules 359 10A.3 Some immediate consequences of symmetry 404


(a) Polarity 404
9C.1 Electron configurations 359
(b) Chirality 405
(a) MO energy level diagrams 359
(b) σ Orbitals and π orbitals 360 Checklist of concepts 406
(c) The overlap integral 361 Checklist of symmetry operations and elements 406
(d) Period 2 diatomic molecules 362
9C.2 Photoelectron spectroscopy 364
TOPIC 10B Group theory 407
10B.1 The elements of group theory 407
Checklist of concepts 365
10B.2 Matrix representations 409
Checklist of equations 365
(a) Representatives of operations 409
TOPIC 9D Molecular orbital theory: heteronuclear (b) The representation of a group 409
diatomic molecules 366 (c) Irreducible representations 410

9D.1 Polar bonds and electronegativity 366 (d) Characters 411

9D.2 The variation principle 367 10B.3 Character tables 412


(a) The procedure 368 (a) The symmetry species of atomic orbitals 413

(b) The features of the solutions 370 (b) The symmetry species of linear combinations of orbitals 413
(c) Character tables and degeneracy 414
Checklist of concepts 372
Checklist of concepts 415
Checklist of equations 372
Checklist of equations 416
TOPIC 9E Molecular orbital theory: polyatomic
molecules 373 TOPIC 10C Applications of symmetry 417
9E.1 The Hückel approximation 373 10C.1 Vanishing integrals 417
(a) An introduction to the method 373 (a) Integrals of the product of functions 418

(b) The matrix formulation of the method 374 (b) Decomposition of a representation 419

9E.2 Applications 376 10C.2 Applications to molecular orbital theory 420


(a) π-Electron binding energy 376 (a) Orbital overlap 420

(b) Aromatic stability 378 (b) Symmetry-adapted linear combinations 421

9E.3 Computational chemistry 379 10C.3 Selection rules 422


(a) Basis functions and basis sets 379 Checklist of concepts 423
(b) Electron correlation 380 Checklist of equations 423
xxii Full Contents

FOCUS 11 Molecular spectroscopy 427 TOPIC 11E Symmetry analysis of vibrational spectra 466
11E.1 Classification of normal modes according to symmetry 466
TOPIC 11A General features of molecular
11E.2 Symmetry of vibrational wavefunctions 468
spectroscopy 429
(a) Infrared activity of normal modes 468
11A.1 The absorption and emission of radiation 430
(b) Raman activity of normal modes 469
(a) Stimulated and spontaneous radiative processes 430
(c) The symmetry basis of the exclusion rule 469
(b) Selection rules and transition moments 431
Checklist of concepts 469
(c) The Beer–Lambert law 431
11A.2 Spectral linewidths 433 TOPIC 11F Electronic spectra 470
(a) Doppler broadening 433
11F.1 Diatomic molecules 470
(b) Lifetime broadening 435
(a) Term symbols 470
11A.3 Experimental techniques 435 (b) Selection rules 473
(a) Sources of radiation 436 (c) Vibrational fine structure 473
(b) Spectral analysis 436 (d) Rotational fine structure 476
(c) Detectors 438
11F.2 Polyatomic molecules 477
(d) Examples of spectrometers 438
(a) d-Metal complexes 478
Checklist of concepts 439 (b) π* ← π and π* ← n transitions 479
Checklist of equations 439 Checklist of concepts 480
Checklist of equations 480
TOPIC 11B Rotational spectroscopy 440
11B.1 Rotational energy levels 440 TOPIC 11G Decay of excited states 481
(a) Spherical rotors 441 11G.1 Fluorescence and phosphorescence 481
(b) Symmetric rotors 442 11G.2 Dissociation and predissociation 483
(c) Linear rotors 444
11G.3 Lasers 484
(d) Centrifugal distortion 444
Checklist of concepts 485
11B.2 Microwave spectroscopy 444
(a) Selection rules 445
(b) The appearance of microwave spectra 446 FOCUS 12 Magnetic resonance 499
11B.3 Rotational Raman spectroscopy 447
TOPIC 12A General principles 500
11B.4 Nuclear statistics and rotational states 449
12A.1 Nuclear magnetic resonance 500
Checklist of concepts 451 (a) The energies of nuclei in magnetic fields 500
Checklist of equations 451 (b) The NMR spectrometer 502
12A.2 Electron paramagnetic resonance 503
TOPIC 11C Vibrational spectroscopy of (a) The energies of electrons in magnetic fields 503
diatomic molecules 452 (b) The EPR spectrometer 504
11C.1 Vibrational motion 452
Checklist of concepts 505
11C.2 Infrared spectroscopy 453
Checklist of equations 505
11C.3 Anharmonicity 454
(a) The convergence of energy levels 454 TOPIC 12B Features of NMR spectra 506
(b) The Birge–Sponer plot 455 12B.1 The chemical shift 506
11C.4 Vibration–rotation spectra 456 12B.2 The origin of shielding constants 508
(a) Spectral branches 457 (a) The local contribution 508
(b) Combination differences 458 (b) Neighbouring group contributions 509
11C.5 Vibrational Raman spectra 458 (c) The solvent contribution 510
Checklist of concepts 460 12B.3 The fine structure 511
Checklist of equations 460 (a) The appearance of the spectrum 511
(b) The magnitudes of coupling constants 513
TOPIC 11D Vibrational spectroscopy of 12B.4 The origin of spin-spin coupling 514
polyatomic molecules 461 (a) Equivalent nuclei 516
11D.1 Normal modes 461 (b) Strongly coupled nuclei 517
11D.2 Infrared absorption spectra 462 12B.5 Exchange processes 517
11D.3 Vibrational Raman spectra 464 12B.6 Solid-state NMR 518
Checklist of concepts 464 Checklist of concepts 519
Checklist of equations 465 Checklist of equations 520
Full Contents xxiii

TOPIC 12C Pulse techniques in NMR 521 TOPIC 13D The canonical ensemble 567
12C.1 The magnetization vector 521 13D.1 The concept of ensemble 567
(a) The effect of the radiofrequency field 522 (a) Dominating configurations 568
(b) Time- and frequency-domain signals 523 (b) Fluctuations from the most probable distribution 568
12C.2 Spin relaxation 525 13D.2 The mean energy of a system 569
(a) The mechanism of relaxation 525 13D.3 Independent molecules revisited 569
(b) The measurement of T1 and T2 526 13D.4 The variation of the energy with volume 570
12C.3 Spin decoupling 527 Checklist of concepts 572
12C.4 The nuclear Overhauser effect 528 Checklist of equations 572
Checklist of concepts 530
Checklist of equations 530 TOPIC 13E The internal energy and the entropy 573
13E.1 The internal energy 573
TOPIC 12D Electron paramagnetic resonance 531 (a) The calculation of internal energy 573
(b) Heat capacity 574
12D.1 The g-value 531
13E.2 The entropy 575
12D.2 Hyperfine structure 532
(a) Entropy and the partition function 576
(a) The effects of nuclear spin 532
(b) The translational contribution 577
12D.3 The McConnell equation 533
(c) The rotational contribution 578
(a) The origin of the hyperfine interaction 534
(d) The vibrational contribution 579
Checklist of concepts 535 (e) Residual entropies 579
Checklist of equations 535 Checklist of concepts 581
Checklist of equations 581

FOCUS 13 Statistical thermodynamics 543 TOPIC 13F Derived functions 582


13F.1 The derivations 582
TOPIC 13A The Boltzmann distribution 544 13F.2 Equilibrium constants 585
13A.1 Configurations and weights 544 (a) The relation between K and the partition function 585
(a) Instantaneous configurations 544
(b) A dissociation equilibrium 586
(b) The most probable distribution 545
(c) Contributions to the equilibrium constant 586
13A.2 The relative populations of states 548 Checklist of concepts 588
Checklist of concepts 549 Checklist of equations 588
Checklist of equations 549

FOCUS 14 Molecular interactions 597


TOPIC 13B Molecular partition functions 550
13B.1 The significance of the partition function 550 TOPIC 14A The electric properties of molecules 599
13B.2 Contributions to the partition function 552 14A.1 Electric dipole moments 599
(a) The translational contribution 552 14A.2 Polarizabilities 601
(b) The rotational contribution 554 14A.3 Polarization 603
(c) The vibrational contribution 558 (a) The mean dipole moment 603
(d) The electronic contribution 559 (b) The frequency dependence of the polarization 604
Checklist of concepts 560 (c) Molar polarization 604
Checklist of equations 560 Checklist of concepts 606
Checklist of equations 607
TOPIC 13C Molecular energies 561
13C.1 The basic equations 561
TOPIC 14B Interactions between molecules 608
14B.1 The interactions of dipoles 608
13C.2 Contributions of the fundamental modes
of motion 562 (a) Charge–dipole interactions 608

(a) The translational contribution 562 (b) Dipole–dipole interactions 609

(b) The rotational contribution 562 (c) Dipole–induced dipole interactions 612

(c) The vibrational contribution 563 (d) Induced dipole–induced dipole interactions 612

(d) The electronic contribution 564 14B.2 Hydrogen bonding 613


(e) The spin contribution 565 14B.3 The total interaction 614
Checklist of concepts 565 Checklist of concepts 616
Checklist of equations 566 Checklist of equations 617
xxiv Full Contents

TOPIC 14C Liquids 618 (c) Scattering factors 666


(d) The electron density 666
14C.1 Molecular interactions in liquids 618
(e) The determination of structure 669
(a) The radial distribution function 618
(b) The calculation of g(r) 619 15B.2 Neutron and electron diffraction 671
(c) The thermodynamic properties of liquids 620 Checklist of concepts 672
14C.2 The liquid–vapour interface 621 Checklist of equations 672
(a) Surface tension 621
(b) Curved surfaces 622 TOPIC 15C Bonding in solids 673
(c) Capillary action 623 15C.1 Metals 673
14C.3 Surface films 624 (a) Close packing 673

(a) Surface pressure 624 (b) Electronic structure of metals 675

(b) The thermodynamics of surface layers 626 15C.2 Ionic solids 677
14C.4 Condensation 627 (a) Structure 677
(b) Energetics 678
Checklist of concepts 628
15C.3 Covalent and molecular solids 681
Checklist of equations 628
Checklist of concepts 682
TOPIC 14D Macromolecules 629 Checklist of equations 682
14D.1 Average molar masses 629
14D.2 The different levels of structure 630 TOPIC 15D The mechanical properties of solids 683
14D.3 Random coils 631 Checklist of concepts 685
(a) Measures of size 631 Checklist of equations 685
(b) Constrained chains 634
(c) Partly rigid coils 634 TOPIC 15E The electrical properties of solids 686
14D.4 Mechanical properties 635 15E.1 Metallic conductors 686
(a) Conformational entropy 635 15E.2 Insulators and semiconductors 687
(b) Elastomers 636 15E.3 Superconductors 689
14D.5 Thermal properties 637 Checklist of concepts 690
Checklist of concepts 638 Checklist of equations 690
Checklist of equations 639
TOPIC 15F The magnetic properties of solids 691
TOPIC 14E Self-assembly 640 15F.1 Magnetic susceptibility 691
14E.1 Colloids 640 15F.2 Permanent and induced magnetic moments 692
(a) Classification and preparation 640
15F.3 Magnetic properties of superconductors 693
(b) Structure and stability 641
Checklist of concepts 694
(c) The electrical double layer 641
Checklist of equations 694
14E.2 Micelles and biological membranes 643
(a) The hydrophobic interaction 643
TOPIC 15G The optical properties of solids 695
(b) Micelle formation 644
15G.1 Excitons 695
(c) Bilayers, vesicles, and membranes 646
15G.2 Metals and semiconductors 696
Checklist of concepts 647
(a) Light absorption 696
Checklist of equations 647
(b) Light-emitting diodes and diode lasers 697
15G.3 Nonlinear optical phenomena 697
FOCUS 15 Solids 655 Checklist of concepts 698
TOPIC 15A Crystal structure 657
15A.1 Periodic crystal lattices 657 FOCUS 16 Molecules in motion 707
15A.2 The identification of lattice planes 659
(a) The Miller indices 659
TOPIC 16A Transport properties of a perfect gas 708
(b) The separation of neighbouring planes 660
16A.1 The phenomenological equations 708

Checklist of concepts 661 16A.2 The transport parameters 710


(a) The diffusion coefficient 711
Checklist of equations 662
(b) Thermal conductivity 712
TOPIC 15B Diffraction techniques 663 (c) Viscosity 714
(d) Effusion 715
15B.1 X-ray crystallography 663
(a) X-ray diffraction 663 Checklist of concepts 716
(b) Bragg’s law 665 Checklist of equations 716
Full Contents xxv

TOPIC 16B Motion in liquids 717 TOPIC 17E Reaction mechanisms 762
16B.1 Experimental results 717 17E.1 Elementary reactions 762
(a) Liquid viscosity 717 17E.2 Consecutive elementary reactions 763
(b) Electrolyte solutions 718 17E.3 The steady-state approximation 764
16B.2 The mobilities of ions 719 17E.4 The rate-determining step 766
(a) The drift speed 719
17E.5 Pre-equilibria 767
(b) Mobility and conductivity 721
17E.6 Kinetic and thermodynamic control of reactions 768
(c) The Einstein relations 722
Checklist of concepts 768
Checklist of concepts 723
Checklist of equations 768
Checklist of equations 723
TOPIC 17F Examples of reaction mechanisms 769
TOPIC 16C Diffusion 724
17F.1 Unimolecular reactions 769
16C.1 The thermodynamic view 724
17F.2 Polymerization kinetics 771
16C.2 The diffusion equation 726
(a) Stepwise polymerization 771
(a) Simple diffusion 726
(b) Chain polymerization 772
(b) Diffusion with convection 728
17F.3 Enzyme-catalysed reactions 774
(c) Solutions of the diffusion equation 728
Checklist of concepts 777
16C.3 The statistical view 730
Checklist of equations 777
Checklist of concepts 732
Checklist of equations 732 TOPIC 17G Photochemistry 778
17G.1 Photochemical processes 778
FOCUS 17 Chemical kinetics 737 17G.2 The primary quantum yield 779
17G.3 Mechanism of decay of excited singlet states 780
TOPIC 17A The rates of chemical reactions 739
17G.4 Quenching 781
17A.1 Monitoring the progress of a reaction 739
17G.5 Resonance energy transfer 783
(a) General considerations 739
Checklist of concepts 784
(b) Special techniques 740
Checklist of equations 784
17A.2 The rates of reactions 741
(a) The definition of rate 741
(b) Rate laws and rate constants 742 FOCUS 18 Reaction dynamics 793
(c) Reaction order 743
(d) The determination of the rate law 744 TOPIC 18A Collision theory 794
Checklist of concepts 746 18A.1 Reactive encounters 794
(a) Collision rates in gases 795
Checklist of equations 746
(b) The energy requirement 795
TOPIC 17B Integrated rate laws 747 (c) The steric requirement 798

17B.1 Zeroth-order reactions 747 18A.2 The RRK model 799


17B.2 First-order reactions 747 Checklist of concepts 800
17B.3 Second-order reactions 749 Checklist of equations 800
Checklist of concepts 752
TOPIC 18B Diffusion-controlled reactions 801
Checklist of equations 752
18B.1 Reactions in solution 801
(a) Classes of reaction 801
TOPIC 17C Reactions approaching equilibrium 753
(b) Diffusion and reaction 802
17C.1 First-order reactions approaching equilibrium 753
18B.2 The material-balance equation 803
17C.2 Relaxation methods 754
(a) The formulation of the equation 803
Checklist of concepts 756 (b) Solutions of the equation 804
Checklist of equations 756 Checklist of concepts 804
Checklist of equations 805
TOPIC 17D The Arrhenius equation 757
17D.1 The temperature dependence of rate constants 757 TOPIC 18C Transition-state theory 806
17D.2 The interpretation of the Arrhenius parameters 759 18C.1 The Eyring equation 806
(a) A first look at the energy requirements of reactions 759 (a) The formulation of the equation 806
(b) The effect of a catalyst on the activation energy 760 (b) The rate of decay of the activated complex 807
Checklist of concepts 761 (c) The concentration of the activated complex 807
Checklist of equations 761 (d) The rate constant 808
xxvi Full Contents

18C.2 Thermodynamic aspects 809 TOPIC 19B Adsorption and desorption 844
(a) Activation parameters 809
19B.1 Adsorption isotherms 844
(b) Reactions between ions 811
(a) The Langmuir isotherm 844
18C.3 The kinetic isotope effect 812 (b) The isosteric enthalpy of adsorption 845
Checklist of concepts 814 (c) The BET isotherm 847
Checklist of equations 814 (d) The Temkin and Freundlich isotherms 849
19B.2 The rates of adsorption and desorption 850
TOPIC 18D The dynamics of molecular collisions 815 (a) The precursor state 850
18D.1 Molecular beams 815 (b) Adsorption and desorption at the molecular level 850
(a) Techniques 815 (c) Mobility on surfaces 852
(b) Experimental results 816 Checklist of concepts 852
18D.2 Reactive collisions 818 Checklist of equations 852
(a) Probes of reactive collisions 818
(b) State-to-state reaction dynamics 818 TOPIC 19C Heterogeneous catalysis 853
18D.3 Potential energy surfaces 819 19C.1 Mechanisms of heterogeneous catalysis 853
18D.4 Some results from experiments and calculations 820 (a) Unimolecular reactions 853
(a) The direction of attack and separation 821 (b) The Langmuir–Hinshelwood mechanism 854
(b) Attractive and repulsive surfaces 821 (c) The Eley-Rideal mechanism 855
(c) Quantum mechanical scattering theory 822 19C.2 Catalytic activity at surfaces 855
Checklist of concepts 823 Checklist of concepts 856
Checklist of equations 823 Checklist of equations 856

TOPIC 18E Electron transfer in homogeneous TOPIC 19D Processes at electrodes 857
systems 824 19D.1 The electrode–solution interface 857
18E.1 The rate law 824 19D.2 The current density at an electrode 858
18E.2 The role of electron tunnelling 825 (a) The Butler–Volmer equation 858
18E.3 The rate constant 826 (b) Tafel plots 862

18E.4 Experimental tests of the theory 828 19D.3 Voltammetry 862


Checklist of concepts 829 19D.4 Electrolysis 865
Checklist of equations 829 19D.5 Working galvanic cells 865
Checklist of concepts 866
Checklist of equations 866
FOCUS 19 Processes at solid surfaces 835
TOPIC 19A An introduction to solid surfaces 836
Resource section 877
19A.1 Surface growth 836
1 Mathematical resources 878
19A.2 Physisorption and chemisorption 837
19A.3 Experimental techniques 838
1.1 Integration 878
(a) Microscopy 839 1.2 Differentiation 878
(b) Ionization techniques 840 1.3 Series expansions 881
(c) Diffraction techniques 841 2 Quantities and units 882
(d) Determination of the extent and rates of adsorption
and desorption 842
3 Data 884
Checklist of concepts 843 4 Character tables 910
Checklist of equations 843
Index 915
CONVENTIONS

To avoid intermediate rounding errors, but to keep track of e­ rrors, we display intermediate results as n.nnn. . . and round
values in order to be aware of values and to spot numerical the calculation only at the final step.

PHYSIC AL CHEMISTRY: PEOPLE AND PERSPEC TIVES


To watch these interviews, go to this section of the e-book.
LIST OF TABLES

Table 1A.1 Pressure units 4 Table 5F.1 Ionic strength and molality, I = kb/b⦵ 191
Table 1A.2 The (molar) gas constant 9 Table 5F.2 Mean activity coefficients in water at 298 K 192
Table 1B.1 Collision cross-sections 16 Table 5F.3 Activities and standard states: a summary 193
Table 1C.1 Second virial coefficients, B/(cm mol )
3 −1
20 Table 6C.1 Varieties of electrode 219
Table 1C.2 Critical constants of gases 21 Table 6D.1 Standard potentials at 298 K 226
Table 1C.3 van der Waals coefficients 22 Table 6D.2 The electrochemical series 229
Table 1C.4 Selected equations of state 23 Table 7E.1 The Hermite polynomials 277
Table 2A.1 Varieties of work 38 Table 7F.1 The spherical harmonics 288
Table 2B.1 Temperature variation of molar heat capacities, 48 Table 8A.1 Hydrogenic radial wavefunctions 308
Cp,m/(J K−1 mol−1) = a + bT + c/T2
Table 8B.1 Effective nuclear charge 322
Table 2C.1 Standard enthalpies of fusion and 51
Table 8B.2 Atomic radii of main-group elements, r/pm 325
vaporization at the transition temperature,

ΔtrsH /(kJ mol−1) Table 8B.3 Ionic radii, r/pm 326
Table 2C.2 Enthalpies of reaction and transition 51 Table 8B.4 First and second ionization energies 326
Table 2C.3 Standard enthalpies of formation and 52 Table 8B.5 Electron affinities, Ea/(kJ mol−1) 327
combustion of organic compounds at 298 K
Table 9A.1 Some hybridization schemes 352
Table 2C.4 Standard enthalpies of formation of inorganic 53
Table 9C.1 Overlap integrals between hydrogenic orbitals 361
compounds at 298 K
Standard enthalpies of formation of organic 53 Table 9C.2 Bond lengths 364
Table 2C.5
compounds at 298 K Table 9C.3 Bond dissociation energies, N AhcD 0 364
Table 2D.1 Expansion coefficients (α) and isothermal 61 Table 9D.1 Pauling electronegativities 367
compressibilities (κT) at 298 K
Table 10A.1 The notations for point groups 400
Table 2D.2 Inversion temperatures (TI), normal freezing 63
Table 10B.1 The C2v character table 412
(Tf ) and boiling (T b) points, and Joule–
Thomson coefficients (μ) at 1 bar and 298 K Table 10B.2 The C3v character table 412
Table 3B.1 Entropies of phase transitions, ΔtrsS/(J K−1 mol−1), 87 Table 10B.3 The C 4 character table 415
at the corresponding normal transition
Table 11B.1 Moments of inertia 440
temperatures (at 1 atm)
Table 11C.1 Properties of diatomic molecules 458
Table 3B.2 The standard enthalpies and entropies of 87
vaporization of liquids at their boiling Table 11F.1 Colour, frequency, and energy of light 470
temperatures
Table 11F.2 Absorption characteristics of some groups and 477
Table 3C.1 Standard Third-Law entropies at 298 K 93 molecules
Table 3D.1 Standard Gibbs energies of formation at 298 K 100 Table 11G.1 Characteristics of laser radiation and their 484
chemical applications
Table 3E.1 The Maxwell relations 105
Table 12A.1 Nuclear constitution and the nuclear spin 500
Table 5A.1 Henry’s law constants for gases in water at 153
quantum number
298 K
Table 12A.2 Nuclear spin properties 501
Table 5B.1 Freezing-point (K f ) and boiling-point (K b) 160
constants Table 12D.1 Hyperfine coupling constants for atoms, a/mT 534
List of Tables xxix

Table 13B.1 Rotational temperatures of diatomic molecules 556 Table 17D.1 Arrhenius parameters 757
Table 13B.2 Symmetry numbers of molecules 557 Table 17G.1 Examples of photochemical processes 778
Table 13B.3 Vibrational temperatures of diatomic molecules 559 Table 17G.2 Common photophysical processes 779
Table 14A.1 Dipole moments and polarizability volumes 599 Table 17G.3 Values of R0 for some donor–acceptor pairs 783
Table 14B.1 Interaction potential energies 612 Table 18A.1 Arrhenius parameters for gas-phase reactions 798
Table 14B.2 Lennard-Jones-(12,6) potential energy 616 Table 18B.1 Arrhenius parameters for solvolysis reactions 802
parameters in solution
Table 14C.1 Surface tensions of liquids at 293 K 621 Table 19A.1 Maximum observed standard enthalpies of 837
physisorption at 298 K
Table 14E.1 Micelle shape and the surfactant parameter 645
Table 19A.2 Standard enthalpies of chemisorption, 837
Table 15A.1 The seven crystal systems 658 ⦵
Δad H /(kJ mol−1), at 298 K
Table 15C.1 The crystal structures of some elements 674
Table 19C.1 Chemisorption abilities 856
Table 15C.2 Ionic radii, r/pm 678
Table 19D.1 Exchange-current densities and transfer 861
Table 15C.3 Madelung constants 679 coefficients at 298 K
Table 15C.4 Lattice enthalpies at 298 K, ΔHL/(kJ mol−1) 681 RESOURCE SECTION TABLES
Table 15F.1 Magnetic susceptibilities at 298 K 692 Table 1.1 Common integrals 879

Table 16A.1 Transport properties of gases at 1 atm 709 Table 2.1 Some common units 882

Table 16B.1 Viscosities of liquids at 298 K 717 Table 2.2 Common SI prefixes 882

Table 16B.2 Ionic mobilities in water at 298 K 720 Table 2.3 The SI base units 882

Table 16B.3 Diffusion coefficients at 298 K, D/(10 m s )


−9 2 −1
722 Table 2.4 A selection of derived units 883

Table 17B.1 Kinetic data for first-order reactions 748 Table 0.1 Physical properties of selected materials 885

Table 17B.2 Kinetic data for second-order reactions 749 Table 0.2 Masses and natural abundances of selected 886
nuclides
Table 17B.3 Integrated rate laws 751
LIST OF THE CHEMIST’S TOOLKITS

Number Title
2A.1 Electrical charge, current, power, and energy 41
2A.2 Partial derivatives 42
3E.1 Exact differentials 105
5B.1 Molality and mole fraction 160
7A.1 Electromagnetic radiation 239
7A.2 Diffraction of waves 246
7B.1 Complex numbers 249
7F.1 Cylindrical coordinates 283
7F.2 Spherical polar coordinates 287
8C.1 Combining vectors 332
9D.1 Determinants 369
9E.1 Matrices 375
11A.1 Exponential and Gaussian functions 434
12B.1 Dipolar magnetic fields 509
12C.1 The Fourier transform 524
16B.1 Electrostatics 720
17B.1 Integration by the method 751
LIST OF MATERIAL PROVIDED
AS A DEEPER LOOK

The list of A deeper look material that can be found via the e-book. You will also find references to this material where ­relevant
throughout the book.

Number Title
2D.1 The Joule–Thomson effect and isenthalpic change
3D.1 The Born equation
5F.1 The Debye–Hückel theory
5F.2 The fugacity
7D.1 Particle in a triangle
7F.1 Separation of variables
9B.1 The energies of the molecular orbitals of H2+
9F.1 The equations of computational chemistry
9F.2 The Roothaan equations
11A.1 Origins of spectroscopic transitions
11B.1 Rotational selection rules
11C.1 Vibrational selection rules
13D.1 The van der Waals equation of state
14B.1 The electric dipole–dipole interaction
14C.1 The virial and the virial equation of state
15D.1 Establishing the relation between bulk and molecular properties
16C.1 Diffusion in three dimensions
16C.2 The random walk
18A.1 The RRK model
19B.1 The BET isotherm
LIST OF IMPAC TS

The list of Impacts that can be found via the e-book. You will also find references to this material where relevant throughout the
book.

Number Focus Title


1 1 . . .on environmental science: The gas laws and the weather
2 1 . . .on astrophysics: The Sun as a ball of perfect gas
3 2 . . .on technology: Thermochemical aspects of fuels and foods
4 3 . . .on engineering: Refrigeration
5 3 . . .on materials science: Crystal defects
6 4 . . .on technology: Supercritical fluids
7 5 . . .on biology: Osmosis in physiology and biochemistry
8 5 . . .on materials science: Liquid crystals
9 6 . . .on biochemistry: Energy conversion in biological cells
10 6 . . .on chemical analysis: Species-selective electrodes
11 7 . . .on technology: Quantum computing
12 7 . . .on nanoscience: Quantum dots
13 8 . . .on astrophysics: The spectroscopy of stars
14 9 . . .on biochemistry: The reactivity of O2, N2, and NO
15 9 . . .on biochemistry: Computational studies of biomolecules
16 11 . . .on astrophysics: Rotational and vibrational spectroscopy of interstellar species
17 11 . . .on environmental science: Climate change
18 12 . . .on medicine: Magnetic resonance imaging
19 12 . . .on biochemistry and nanoscience: Spin probes
20 13 . . .on biochemistry: The helix–coil transition in polypeptides
21 14 . . .on biology: Biological macromolecules
22 14 . . .on medicine: Molecular recognition and drug design
23 15 . . .on biochemistry: Analysis of X-ray diffraction by DNA
24 15 . . .on nanoscience: Nanowires
25 16 . . .on biochemistry: Ion channels
26 17 . . .on biochemistry: Harvesting of light during plant photosynthesis
27 19 . . .on technology: Catalysis in the chemical industry
28 19 . . .on technology: Fuel cells
ENERGY A First Look

Much of chemistry is concerned with the transfer and trans- For ­example, the x-component, vx, is the particle’s rate of
formation of energy, so right from the outset it is important to change of position along the x-axis:
become familiar with this concept. The first ideas about energy dx Component of velocity
emerged from classical mechanics, the theory of motion for- vx = [definition]
(1a)
dt
mulated by Isaac Newton in the seventeenth century. In the
twentieth century classical mechanics gave way to quantum Similar expressions may be written for the y- and z-­components.
mechanics, the theory of motion formulated for the descrip- The magnitude of the velocity, as represented by the length of
tion of small particles, such as electrons, atoms, and molecules. the velocity vector, is the speed, v. Speed is related to the com-
In quantum mechanics the concept of energy not only survived ponents of velocity by
but was greatly enriched, and has come to underlie the whole of Speed
physical chemistry. v  (vx2  v 2y  vz2 )1/ 2 [definition]
(1b)

The linear momentum, p, of a particle, like the velocity, is


a vector quantity, but takes into account the mass of the parti-
1 Force cle as well as its speed and direction. Its components are px, py,
and pz along each axis (Fig. 1b) and its magnitude is p. A heavy
Classical mechanics is formulated in terms of the forces acting particle travelling at a certain speed has a greater linear mo-
on particles, and shows how the paths of particles respond to mentum than a light particle travelling at the same speed. For a
them by accelerating or changing direction. Much of the dis- particle of mass m, the x-component of the linear momentum
cussion focuses on a quantity called the ‘momentum’ of the is given by
particle. Component of linear momentum
px = mvx [definition]
(2)

(a) Linear momentum and similarly for the y- and z-components.


‘Translation’ is the motion of a particle through space. The
velocity, v, of a particle is the rate of change of its position. Brief illustration 1
Velocity is a ‘vector quantity’, meaning that it has both a direc-
tion and a magnitude, and is expressed in terms of how fast Imagine a particle of mass m attached to a spring. When the
the particle travels with respect to x-, y-, and z-axes (Fig. 1). particle is displaced from its equilibrium position and then
released, it oscillates back and forth about this equilibrium
position. This model can be used to describe many features
of a chemical bond. In an idealized case, known as the simple
pz harmonic oscillator, the displacement from equilibrium x(t)
vz p varies with time as
Heavy
v particle
p x(t )  A sin 2 t
th
v Light
eng In this expression, ν (nu) is the frequency of the oscillation
th L
ng
particle
e and A is its amplitude, the maximum value of the displace-
vx L px ment along the x-axis. The x-component of the velocity of the
vy
py particle is therefore
(a) (b)
dx d( A sin 2 t )
vx    2 A cos 2 t
Figure 1 (a) The velocity v is denoted by a vector of magnitude dt dt
v (the speed) and an orientation that indicates the direction The x-component of the linear momentum of the particle is
of translational motion. (b) Similarly, the linear momentum p
is denoted by a vector of magnitude p and an orientation that px  mvx  2 Am cos 2 t
corresponds to the direction of motion.
xxxiv ENERGY A First Look

(b) Angular momentum z


Jz
‘Rotation’ is the change of orientation in space around a cen- J
tral point (the ‘centre of mass’). Its description is very similar to
that of translation but with ‘angular velocity’ taking the place of
velocity and ‘moment of inertia’ taking the place of mass. The
angular velocity, ω (omega) is the rate of change of orientation
(for example, in radians per second); it is a vector with magni- Jx Jy
tude ω. The moment of inertia, I, is a measure of the mass that y

is being swung round by the rotational motion. For a particle x


of mass m moving in a circular path of radius r, the moment of
inertia is Figure 2 The angular momentum J of a particle is represented by
a vector along the axis of rotation and perpendicular to the plane
I = mr 2
Moment of inertia
(3a) of rotation. The length of the vector denotes the magnitude J of
[definition] the angular momentum. The direction of motion is clockwise to
an observer looking in the direction of the vector.
For a molecule composed of several atoms, each atom i gives a
contribution of this kind, and the moment of inertia around a
given axis is
the velocity and momentum, the force, F, is a vector quantity
I  mi ri (3b)
2 with a direction and a magnitude (the ‘strength’ of the force).
i Force is reported in newtons, with 1 N = 1 kg m s−2. For motion
along the x-axis Newton’s second law states that
where ri is the perpendicular distance from the mass mi to
the axis. The rotation of a particle is described by its angular dp x Newton’s second law
momentum, J, a vector with a length that indicates the rate at = Fx [in one dimension]
(5a)
dt
which the particle circulates and a direction that indicates the
axis of rotation (Fig. 2). The components of angular momen- where Fx is the component of the force acting along the x-axis.
tum, Jx, Jy, and Jz, on three perpendicular axes show how much Each component of linear momentum obeys the same kind of
angular momentum is associated with rotation around each equation, so the vector p changes with time as
axis. The magnitude J of the angular momentum is
dp Newton’s second law
Magnitude of angular momentum
=F [vector form]
(5b)
J  I (4) dt
[definition]
Equation 5 is the equation of motion of the particle, the equa-
tion that has to be solved to calculate its trajectory.
Brief illustration 2

A CO2 molecule is linear, and the length of each CO bond


is 116 pm. The mass of each 16O atom is 16.00mu, where Brief illustration 3
mu = 1.661 × 10−27 kg. It follows that the moment of inertia of
the molecule around an axis perpendicular to the axis of the According to ‘Hooke’s law’, the force acting on a particle under-
molecule and passing through the C atom is going harmonic motion (like that in Brief illustration 2) is
proportional to the displacement and directed opposite to the
I  mO R2  0  mO R2  2mO R2 direction of motion, so in one dimension
 2  (16.00  1.661  1027 kg )  (1.16  1010 m)2 Fx  kf x
46 2
 7.15  10 kg m where x is the displacement from equilibrium and kf is the
‘force constant’, a measure of the stiffness of the spring (or
chemical bond). It then follows that the equation of motion
of a particle undergoing harmonic motion is dpx/dt = −kfx.
(c) Newton’s second law of motion Then, because px = mvx and vx = dx/dt, it follows that dpx/dt =
mdvx/dt = md2x/dt2. With this substitution, the equation of
The central concept of classical mechanics is Newton’s second motion becomes
law of motion, which states that the rate of change of momen-
tum is equal to the force acting on the particle. This law underlies d2 x
m  kx
the calculation of the trajectory of a particle, a statement about dt 2
where it is and where it is moving at each moment of time. Like
ENERGY A First Look xxxv

Equations of this kind, which are called ‘differential equations’, Brief illustration 4
are solved by special techniques. In most cases in this text, the
solutions are simply stated without going into the details of Suppose that when a bond is stretched from its equilibrium
how they are found. value Re to some arbitrary value R there is a restoring force
Similar considerations apply to rotation. The change in proportional to the displacement x = R – Re from the equilib-
angular momentum of a particle is expressed in terms of the rium length. Then
torque, T, a twisting force. The analogue of eqn 5b is then
Fx  kf (R  Re )  kf x
dJ The constant of proportionality, kf, is the force constant intro-
= T (6)
dt duced in Brief illustration 3. The total work needed to move
an atom so that the bond stretches from zero displacement
Quantities that describe translation and rotation are analogous, (xinitial = 0), when the bond has its equilibrium length, to a dis-
as shown below: placement xfinal = Rfinal – Re is
Integral A.1


x final x final
Property Translation Rotation won an atom    (  k f x ) dx  k f  x dx
0 0
Rate linear velocity, v angular velocity, ω 2
 12 kf x final  21 kf (Rfinal  Re )2
Resistance to change mass, m moment of inertia, I
Momentum linear momentum, p angular momentum, J (All the integrals required in this book are listed in the Resource
Influence on motion force, F torque, T section.) The work required increases as the square of the dis-
placement: it takes four times as much work to stretch a bond
through 20 pm as it does to stretch the same bond through
10 pm.
2 Energy
Energy is a powerful and essential concept in science; never- (b) The definition of energy
theless, its actual nature is obscure and it is difficult to say what Now we get to the core of this discussion. Energy is the capacity
it ‘is’. However, it can be related to processes that can be meas- to do work. An object with a lot of energy can do a lot of work;
ured and can be defined in terms of the measurable process one with little energy can do only little work. Thus, a spring that
called work. is compressed can do a lot of work as it expands, so it is said
to have a lot of energy. Once the spring is expanded it can do
(a) Work only a little work, perhaps none, so it is said to have only a little
energy. The SI unit of energy is the same as that of work, namely
Work, w, is done in order to achieve motion against an op- the joule, with 1 J = 1 kg m2 s−2.
posing force. The work needed to be done to move a particle A particle may possess two kinds of energy, kinetic energy
through the infinitesimal distance dx against an opposing and potential energy. The kinetic energy, Ek, of a particle is the
force Fx is energy it possesses as a result of its motion. For a particle of
mass m travelling at a speed v,
Work
dwon the particle  Fx dx [definition]
(7a)
Kinetic energy
Ek = 12 mv 2 [definition]
(8a)

When the force is directed to the left (to negative x), Fx is nega-
A particle with a lot of kinetic energy can do a lot of work, in
tive, so for motion to the right (dx positive), the work that must
the sense that if it collides with another particle it can cause it to
be done to move the particle is positive. With force in newtons
move against an opposing force. Because the magnitude of the
and distance in metres, the units of work are joules (J), with
linear momentum and speed are related by p = mv, so v = p/m,
1 J = 1 N m = 1 kg m2 s–2.
an alternative version of this relation is
The total work that has to be done to move a particle from
xinitial to xfinal is found by integrating eqn 7a, allowing for the p2
possibility that the force may change at each point along Ek = (8b)
2m
the path:
It follows from Newton’s second law that if a particle is initially
x final stationary and is subjected to a constant force then its linear
won the particle    Fx dx Work (7b)
xinitial momentum increases from zero. Because the magnitude of the
xxxvi ENERGY A First Look

applied force may be varied at will, the momentum and there- e­ nergy can be transformed from one form to another, its total
fore the kinetic energy of the particle may be increased to any is constant.
value. An alternative way of thinking about the potential en-
The potential energy, Ep or V, of a particle is the energy it ergy arising from the interaction of charges is in terms of the
possesses as a result of its position. For instance, a stationary ­potential, which is a measure of the ‘potential’ of one charge to
weight high above the surface of the Earth can do a lot of work affect the potential energy of another charge when the second
as it falls to a lower level, so is said to have more energy, in this charge is brought into its vicinity. A charge Q 1 gives rise to a
case potential energy, than when it is resting on the surface of Coulomb potential ϕ1 (phi) such that the potential energy of
the Earth. the interaction with a second charge Q 2 is Q 2ϕ1(r). Comparison
This definition can be turned around. Suppose the weight is of this expression with eqn 11 shows that
returned from the surface of the Earth to its original height. The
work needed to raise it is equal to the potential energy that it Q1 Coulomb potential
1 (r )  [in a vacuum]
(13)
once again possesses. For an infinitesimal change in height, dx, 4  0 r
that work is −Fxdx. Therefore, the infinitesimal change in po-
tential energy is dEp = −Fxdx. This equation can be rearranged The units of potential are joules per coulomb, J C–1, so when the
into a relation between the force and the potential energy: potential is multiplied by a charge in coulombs, the result is the
potential energy in joules. The combination joules per coulomb
dE p dV Relation of force occurs widely and is called a volt (V): 1 V = 1 J C–1.
Fx   or Fx   to potential energgy
(9) The language developed here inspires an important alterna-
dx dx
tive energy unit, the electronvolt (eV): 1 eV is defined as the
No universal expression for the dependence of the potential potential energy acquired when an electron is moved through
energy on position can be given because it depends on the type a potential difference of 1 V. The relation between electronvolts
of force the particle experiences. However, there are two very and joules is
important specific cases where an expression can be given. For
a particle of mass m at an altitude h close to the surface of the 1 eV  1.602  1019 J
Earth, the gravitational potential energy is
Many processes in chemistry involve energies of a few electron-
Gravitational potential energy volts. For example, to remove an electron from a sodium atom
Ep (h)  Ep (0)  mgh [close to surrface of the Earth]
(10)
requires about 5 eV.

where g is the acceleration of free fall (g depends on location,


but its ‘standard value’ is close to 9.81 m s–2). The zero of poten-
tial energy is arbitrary. For a particle close to the surface of the 3 Temperature
Earth, it is common to set Ep(0) = 0.
The other very important case (which occurs whenever A key idea of quantum mechanics is that the translational en-
the structures of atoms and molecules are discussed), is the ergy of a molecule, atom, or electron that is confined to a re-
electrostatic potential energy between two electric charges Q 1 gion of space, and any rotational or vibrational energy that a
and Q 2 at a separation r in a vacuum. This Coulomb ­potential molecule possesses, is quantized, meaning that it is restricted
energy is to certain discrete values. These permitted energies are called
Q1Q 2 energy levels. The values of the permitted energies depend on
Coulomb potential energy
Ep (r )  [in a vacuum]
(11) the characteristics of the particle (for instance, its mass) and for
4 0 r
translation the extent of the region to which it is confined. The
Charge is expressed in coulombs (C). The constant ε0 (epsilon allowed energies are widest apart for particles of small mass
zero) is the electric constant (or vacuum permittivity), a fun- confined to small regions of space. Consequently, quantization
damental constant with the value 8.854 × 10–12 C2 J–1 m–1. It is must be taken into account for electrons bound to nuclei in
conventional (as in eqn 11) to set the potential energy equal to atoms and molecules. It can be ignored for macroscopic bodies,
zero at infinite separation of charges. for which the separation of all kinds of energy levels is so small
The total energy of a particle is the sum of its kinetic and that for all practical purposes their energy can be varied virtu-
potential energies: ally continuously.
Figure 3 depicts the typical energy level separations associ-
E  Ek  Ep , or E  Ek  V Total energy (12) ated with rotational, vibrational, and electronic motion. The
separation of rotational energy levels (in small molecules,
A fundamental feature of nature is that energy is conserved; about 10−21 J, corresponding to about 0.6 kJ mol−1) is smaller
that is, energy can neither be created nor destroyed. Although than that of vibrational energy levels (about 10−20–10−19 J, or
ENERGY A First Look xxxvii

Translation Rotation Vibration Electronic T=0 T=

1 aJ (1000 zJ)
10–100 zJ
Continuum

1 zJ

Energy
Figure 4 The Boltzmann distribution of populations (represented
Figure 3 The energy level separations typical of four types of by the horizontal bars) for a system of five states with different
system. (1 zJ = 10–21 J; in molar terms, 1 zJ is equivalent to about energies as the temperature is raised from zero to infinity. Interact
0.6 kJ mol–1.) with the dynamic version of this graph in the e-book.

6–60 kJ mol−1), which itself is smaller than that of electronic en- ­ olecule are zero. As the value of T is increased (the ‘tempera-
m
ergy levels (about 10−18 J, corresponding to about 600 kJ mol −1). ture is raised’), the populations of higher energy states increase,
and the distribution becomes more uniform. This behaviour is
illustrated in Fig. 4 for a system with five states of different en-
(a) The Boltzmann distribution ergy. As predicted by eqn 14a, as the temperature approaches
The continuous thermal agitation that molecules experience in infinity (T → ∞), the states become equally populated.
a sample ensures that they are distributed over the available en- In chemical applications it is common to use molar energies,
ergy levels. This distribution is best expressed in terms of the Em,i, with Em,i = NAεi, where NA is Avogadro’s constant. Then
occupation of states. The distinction between a state and a level eqn 14a becomes
is that a given level may be comprised of several states all of
which have the same energy. For instance, a molecule might be Ni  ( E /N  E /N ) /kT  ( E  E ) /N kT  ( E  E )/RT
 e m,i A m,j A  e m,i m,j A  e m,i m,j (14b)
rotating clockwise with a certain energy, or rotating counter- Nj
clockwise with the same energy. One particular molecule may
be in a state belonging to a low energy level at one instant, and where R = NAk. The constant R is known as the ‘gas constant’;
then be excited into a state belonging to a high energy level a it appears in expressions of this kind when molar, rather than
moment later. Although it is not possible to keep track of which molecular, energies are specified. Moreover, because it is simply
state each molecule is in, it is possible to talk about the average the molar version of the more fundamental Boltzmann con-
number of molecules in each state. A remarkable feature of na- stant, it occurs in contexts other than gases.
ture is that, for a given array of energy levels, how the molecules
are distributed over the states depends on a single parameter,
the ‘temperature’, T.
The population of a state is the average number of mol- Brief illustration 5
ecules that occupy it. The populations, whatever the nature of
Methylcyclohexane molecules may exist in one of two confor-
the states (translational, rotational, and so on), are given by
mations, with the methyl group in either an equatorial or axial
a formula derived by Ludwig Boltzmann and known as the
position. The equatorial form lies 6.0 kJ mol–1 lower in energy
Boltzmann distribution. According to Boltzmann, the ratio of
than the axial form. The relative populations of molecules in
the populations of states with energies εi and εj is the axial and equatorial states at 300 K are therefore

Ni  (   ) /kT N axial ( E E ) /RT


e i j Boltzmann distribution(14a)  e m,axial m,equatorial
Nj N equatorial
 ( 6.0103 J mol 1 ) / ( 8.3145 J K 1 mol 1 )( 300 K )
e
where k is Boltzmann’s constant, a fundamental constant with
 0.090
the value k = 1.381 × 10–23 J K–1 and T is the temperature, the
parameter that specifies the relative populations of states, re- The number of molecules in an axial conformation is therefore
gardless of their type. Thus, when T = 0, the populations of just 9 per cent of those in the equatorial conformation.
all states other than the lowest state (the ‘ground state’) of the
xxxviii ENERGY A First Look

The important features of the Boltzmann distribution to bear of motion by using the equipartition theorem. This theorem
in mind are: arises from a consideration of how the energy levels associated
with different kinds of motion are populated according to the
• The distribution of populations is an exponential func-
Boltzmann distribution. The theorem states that
tion of energy and the temperature. As the temperature is
increased, states with higher energy become progressively At thermal equilibrium, the average value of each
more populated. quadratic contribution to the energy is 12 kT .
• States closely spaced in energy compared to kT are more A ‘quadratic contribution’ is one that is proportional to the
populated than states that are widely spaced compared square of the momentum or the square of the displacement
to kT. from an equilibrium position. For example, the kinetic energy
The energy spacings of translational and rotational states are of a particle travelling in the x-direction is Ek = px2 /2m. This
typically much less than kT at room temperature. As a result, motion therefore makes a contribution of 12 kT to the energy.
many translational and rotational states are populated. In con- The energy of vibration of atoms in a chemical bond has two
trast, electronic states are typically separated by much more quadratic contributions. One is the kinetic energy arising from
than kT. As a result, only the ground electronic state of a mol- the back and forth motion of the atoms. Another is the poten-
ecule is occupied at normal temperatures. Vibrational states are tial energy which, for the harmonic oscillator, is Ep = 12 kf x 2 and
widely separated in small, stiff molecules and only the ground is a second quadratic contribution. Therefore, the total average
vibrational state is populated. Large and flexible molecules are energy is 12 kT  12 kT  kT .
also found principally in their ground vibrational state, but The equipartition theorem applies only if many of the states
might have a few higher energy vibrational states populated at associated with a type of motion are populated. At tempera-
normal temperatures. tures of interest to chemists this condition is always met for
translational motion, and is usually met for rotational motion.
Typically, the separation between vibrational and electronic
(b) The equipartition theorem states is greater than for rotation or translation, and as only a
For gases consisting of non-interacting particles it is often pos- few states are occupied (often only one, the ground state), the
sible to calculate the average energy associated with each type equipartition theorem is unreliable for these types of motion.

Checklist of concepts
☐ 1. Newton’s second law of motion states that the rate of ☐ 6. The Coulomb potential energy between two charges
change of momentum is equal to the force acting on the separated by a distance r varies as 1/r.
particle. ☐ 7. The energy levels of confined particles are quantized, as
☐ 2. Work is done in order to achieve motion against an are those of rotating or vibrating molecules.
opposing force. Energy is the capacity to do work. ☐ 8. The Boltzmann distribution is a formula for calculating
☐ 3. The kinetic energy of a particle is the energy it possesses the relative populations of states of various energies.
as a result of its motion. ☐ 9. The equipartition theorem states that for a sample at
☐ 4. The potential energy of a particle is the energy it pos- thermal equilibrium the average value of each quadratic
sesses as a result of its position. contribution to the energy is 12 kT .
☐ 5. The total energy of a particle is the sum of its kinetic and
potential energies.
ENERGY A First Look xxxix

Checklist of equations
Property Equation Comment Equation number
Component of velocity in x direction vx = dx /dt Definition; likewise for y and z 1a
Component of linear momentum in x direction px = mvx Definition; likewise for y and z 2
2
Moment of inertia I = mr Point particle 3a
I   mi ri2 Molecule 3b
i

Angular momentum J = Iω 4
Equation of motion Fx = dpx /dt Motion along x-direction 5a
F = dp/dt Newton’s second law of motion 5b
T = dJ/dt Rotational motion 6
Work opposing a force in the x direction dw = –Fxdx Definition 7a
Kinetic energy Ek = mv 1
2
2
Definition; v is the speed 8a

Potential energy and force Fx  dV /dx One dimension 9


Coulomb potential energy Ep (r )  Q 1Q 2 /4 0r In a vacuum 11
Coulomb potential 1 (r )  Q 1 /4  0r In a vacuum 13
Boltzmann distribution N i /N j  e
 ( i  j ) /kT
14a
Atkins’
PHYSICAL CHEMISTRY
FOCUS 1
The properties of gases

A gas is a form of matter that fills whatever container it occu- 1C Real gases
pies. This Focus establishes the properties of gases that are used
throughout the text. The perfect gas is a starting point for the discussion of prop-
erties of all gases, and its properties are invoked throughout
thermodynamics. However, actual gases, ‘real gases’, have prop-
erties that differ from those of perfect gases, and it is necessary
1A The perfect gas to be able to interpret these deviations and build the effects of
molecular attractions and repulsions into the model. The dis-
This Topic is an account of an idealized version of a gas, a ‘per- cussion of real gases is another example of how initially primi-
fect gas’, and shows how its equation of state may be assembled tive models in physical chemistry are elaborated to take into
from the experimental observations summarized by Boyle’s account more detailed observations.
law, Charles’s law, and Avogadro’s principle. 1C.1 Deviations from perfect behaviour; 1C.2 The van der Waals
1A.1 Variables of state; 1A.2 Equations of state equation

1B The kinetic model What is an application of this material?


A central feature of physical chemistry is its role in building The perfect gas law and the kinetic theory can be applied to the
models of molecular behaviour that seek to explain observed study of phenomena confined to a reaction vessel or encom-
phenomena. A prime example of this procedure is the develop- passing an entire planet or star. In Impact 1, accessed via the
ment of a molecular model of a perfect gas in terms of a col- e-book, the gas laws are used in the discussion of meteoro-
lection of molecules (or atoms) in ceaseless, essentially random logical phenomena—the weather. Impact 2, accessed via the
motion. As well as accounting for the gas laws, this model can e-book, examines how the kinetic model of gases has a surpris-
be used to predict the average speed at which molecules move in ing application: to the discussion of dense stellar media, such as
a gas, and its dependence on temperature. In combination with the interior of the Sun.
the Boltzmann distribution (see Energy: A first look), the model
can also be used to predict the spread of molecular speeds and
its dependence on molecular mass and temperature.
1B.1 The model; 1B.2 Collisions

➤ Go to the e-book for videos that feature the derivation and interpretation of equations, and applications of this material.
TOPIC 1A The perfect gas

these collisions are so numerous that the force, and hence the
➤ Why do you need to know this material? pressure, is steady.
The SI unit of pressure is the pascal, Pa, defined as 1 Pa =
The relation between the pressure, volume, and tempera-
1 N m−2 = 1 kg m−1 s−2. Several other units are still widely used,
ture of a perfect gas is used extensively in the develop-
and the relations between them are given in Table 1A.1. Because
ment of quantitative theories about the physical and
many physical properties depend on the pressure acting on a
chemical behaviour of real gases. It is also used extensively
sample, it is appropriate to select a certain value of the pressure
throughout thermodynamics.
to report their values. The standard pressure, p , for reporting

physical quantities is currently defined as p = 1 bar (that is,


➤ What is the key idea?


105 Pa) exactly. This pressure is close to, but not the same as,
The perfect gas law, which describes the relation between
1 atm, which is typical for everyday conditions.
the pressure, volume, temperature, and amount of sub-
Consider the arrangement shown in Fig. 1A.1 where two
stance, is a limiting law that is obeyed increasingly well as
gases in separate containers share a common movable wall.
the pressure of a gas tends to zero.
In Fig. 1A.1a the gas on the left is at higher pressure than that
➤ What do you need to know already? on the right, and so the force exerted on the wall by the gas on
the left is greater than that exerted by the gas on the right. As
You need to know how to handle quantities and units in
a result, the wall moves to the right, the pressure on the left
calculations, as reviewed in the Resource section.

Table 1A.1 Pressure units*

Name Symbol Value


The properties of gases were among the first to be established pascal Pa 1 Pa = 1 N m−2, 1 kg m−1 s−2
quantitatively (largely during the seventeenth and eighteenth bar bar 1 bar = 105 Pa
centuries) when the technological requirements of travel in atmosphere atm 1 atm = 101.325 kPa
balloons stimulated their investigation. This Topic reviews how torr Torr 1 Torr = (101 325/760) Pa = 133.32 … Pa
the physical state of a gas is described using variables such as
millimetres of mercury mmHg 1 mmHg = 133.322 … Pa
pressure and temperature, and then discusses how these vari-
pounds per square inch psi 1 psi = 6.894 757 … kPa
ables are related.
* Values in bold are exact.

1A.1 Variables of state Movable


wall
The physical state of a sample of a substance, its physical condi-
High Low
tion, is defined by its physical properties. Two samples of the
pressure pressure
same substance that have the same physical properties are said
to be ‘in the same state’. The variables of state, the variables (a)
needed to specify the state of a system, are the amount of sub-
stance it contains, n; the volume it occupies, V; the pressure, p;
Equal Equal
and the temperature, T. pressures pressures

(b)
1A.1(a) Pressure and volume
Figure 1A.1 (a) When a region of high pressure is separated from
The pressure, p, that an object experiences is defined as the a region of low pressure by a movable wall, the wall will be pushed
force, F, applied divided by the area, A, to which that force is into the low pressure region until the pressures are equal. (b) When
applied. A gas exerts a pressure on the walls of its container as the two pressures are identical, the wall will stop moving. At this
a result of the collisions between the molecules and the walls: point there is mechanical equilibrium between the two regions.
1A The perfect gas 5

­ ecreases, and that on the right increases. Eventually (as in


d The Celsius scale of temperature is commonly used to ex-
Fig. 1A.1b) the two pressures become equal and the wall no press temperatures. In this text, temperatures on the Celsius
longer moves. This condition of equality of pressure on either scale are denoted θ (theta) and expressed in degrees Celsius (°C).
side of a movable wall is a state of mechanical equilibrium be- The thermodynamic and Celsius temperatures are related by
tween the two gases. the exact expression
The pressure exerted by the atmosphere is measured with
Celsius scale
a barometer. The original version of a barometer (which was T /K   /C  273.15 [definition]
(1A.1)
invented by Torricelli, a student of Galileo) involved taking a
glass tube, sealed at one end, filling it with mercury and then This relation is the definition of the Celsius scale in terms of the
up-ending it (without letting in any air) into a bath of mercury. more fundamental Kelvin scale. It implies that a difference in
The pressure of the atmosphere acting on the surface of the temperature of 1 °C is equivalent to a difference of 1 K.
mercury in the bath supports a column of mercury of a certain The lowest temperature on the thermodynamic tempera-
height in the tube: the pressure at the base of the column, due ture scale is written T = 0, not T = 0 K. This scale is absolute,
to the mercury in the tube, is equal to the atmospheric pressure. and the lowest temperature is 0 regardless of the size of the
As the atmospheric pressure changes, so does the height of the divisions on the scale (just as zero pressure is denoted p = 0,
column. regardless of the size of the units, such as bar or pascal).
The pressure of gas in a container, and also now the atmos- However, it is appropriate to write 0 °C because the Celsius
phere, is measured by using a pressure gauge, which is a de- scale is not absolute.
vice with properties that respond to pressure. For instance,
in a Bayard–Alpert pressure gauge the molecules present in
1A.1(c) Amount
the gas are ionized and the resulting current of ions is inter-
preted in terms of the pressure. In a capacitance manometer, In day-to-day conversation ‘amount’ has many meanings but
two electrodes form a capacitor. One electrode is fixed and the in physical science it has a very precise definition. The amount
other is a diaphragm which deflects as the pressure changes. of substance, n, is a measure of the number of specified enti-
This deflection causes a change in the capacitance, which is ties present in the sample; these entities may be atoms, or mol-
measured and interpreted as a pressure. Certain semiconduc- ecules, or formula units. The SI unit of amount of substance is
tors also respond to pressure and are used as transducers in the mole (mol). The amount of substance is commonly referred
solid-state pressure gauges, including those in mobile phones to as the ‘chemical amount’ or simply ‘amount’.
(cell phones). Until 2019 the mole was defined as the number of car-
The volume, V, of a gas is a measure of the extent of the re- bon atoms in exactly 12 g of carbon-12. However, it has been
gion of space it occupies. The SI unit of volume is m3. redefined such that 1 mol of a substance contains exactly
6.022140 76 × 1023 entities. The number of entities per mole is
called Avogadro’s constant, NA. It follows from the definition
1A.1(b) Temperature of the mole that N A  6.022140 76  1023 mol 1 . Note that NA is
The temperature is formally a property that determines in a constant with units, not a pure number. Also, it is not correct
which direction energy will flow as heat when two samples to specify amount as the ‘number of moles’: the correct phrase
are placed in contact through thermally conducting walls: is ‘amount in moles’.
­energy flows from the sample with the higher temperature to The amount of substance is related to the mass, m, of the sub-
the sample with the lower temperature. The symbol T denotes stance through the molar mass, M, which is the mass per mole
the ­thermodynamic temperature, which is an absolute scale of its atoms, its molecules, or its formula units. The SI unit of
with T = 0 as the lowest point. Temperatures above T = 0 are molar mass is kg mol−1 but it is more common to use g mol−1.
expressed by using the Kelvin scale, in which the gradations The amount of substance of specified entities in a sample can
of temperature are expressed in kelvins (K; not °K). Until 2019, readily be calculated from its mass by using
the Kelvin scale was defined by setting the triple point of water m
(the temperature at which ice, liquid water, and water vapour n= Amount of substance (1A.2)
M
are in mutual equilibrium) at exactly 273.16 K. The scale has
now been redefined by referring it to the more precisely known
1A.1(d) Intensive and extensive properties
value of the Boltzmann constant.
There are many devices used to measure temperature. They Suppose a sample is divided into smaller samples. If a prop-
vary from simple devices that measure the expansion of a liquid erty of the original sample has a value that is equal to the sum
along a tube, as commonly found in laboratories, to electronic of its values in all the smaller samples, then it is said to be an
devices where the resistance of a material or the potential differ- ­extensive property. Amount, mass, and volume are examples of
ence developed at a junction is related to the temperature. extensive properties. If a property retains the same value as in
6 1 The properties of gases

the original sample for all the smaller samples, then it is said to
be intensive. Temperature and pressure are examples of inten-
1A.2 Equations of state
sive properties. Although in principle the state of a pure substance is specified
The value of a property X divided by the amount n gives the by giving the values of n, V, p, and T, it has been established
molar value of that property Xm: that is, Xm = X/n. All molar experimentally that it is sufficient to specify only three of these
properties are intensive, whereas X and n are both extensive. variables because doing so fixes the value of the fourth vari-
The mass density, ρ = m/V, is also intensive. able. That is, it is an experimental fact that each substance is de-
scribed by an equation of state, an equation that interrelates
these four variables.
Example 1A.1 Specifying the variables of state The general form of an equation of state is
When released into a certain vessel, 0.560 mg of nitrogen gas p = f (T ,V ,n) General form of an equation of state (1A.3)
is observed to exert a pressure of 10.4 Torr at a temperature of
This equation means that if the values of n, T, and V are known
25.2 °C. Express the pressure in pascals (Pa) and the thermo-
for a particular substance, then the pressure has a fixed value.
dynamic temperature in kelvins (K). Also calculate the amount
Each substance is described by its own equation of state, but
of N2, and the number of N2 molecules present. Take the molar
mass of N2 as 14.01 g mol−1.
the explicit form of the equation is known in only a few special
cases. One very important example is the equation of state of a
Collect your thoughts The SI unit of pressure is Pa, and the ‘perfect gas’, which has the form p = nRT/V, where R is a con-
conversion from Torr to Pa is given in Table 1A.1; the conver- stant independent of the identity of the gas.
sion of °C to K is given by eqn 1A.1. The amount is computed
using eqn 1A.2.
The solution From the table 1 Torr = 133.32 … Pa, so a pressure
The empirical basis of the perfect
1A.2(a)
of 10.4 Torr is converted to Pa through
gas law
p  (10.4 Torr )  (133.32  Pa Torr 1 )  1.39  103 Pa The equation of state of a perfect gas was established by com-
bining a series of empirical laws that arose from experimental
Note the inclusion of units for each quantity, and the way in observations. These laws can be summarized as
which the units cancel to give the required result. The tempera-
ture in °C is converted to K using eqn 1A.1.
Boyle ’s law : pV  constant, at constant n,T
T /K   /C  273.15  (25.2 C)/C  273.15  298 Charles ’s law : V  constant  T , at constant n, p
Thus T = 298 K. The amount is calculated by using eqn 1A.2: p  constant  T , at constant n,V
note the conversion of the mass from mg to g so as to match
Avo
ogadro ’s principle: V  constant  n, at constant p,T
the units of the molar mass.
3
m 0.560  10 g Boyle’s and Charles’s laws are strictly true only in the limit
n   3.99 105 mol  4.00  105 mol
M 14.01 g mol 1 that the pressure goes to zero (p → 0): they are examples of a
Here the intermediate result is truncated at (not rounded to) limiting law, a law that is strictly true only in a certain limit.
three figures, but the final result is rounded to three figures. However, these laws are found to be reasonably reliable at nor-
The number of molecules is found by multiplying the mal pressures (p ≈ 1 bar) and are used throughout chemistry.
amount by Avogadro’s constant. Avogadro’s principle is so-called because it supposes that the
system consists of molecules whereas a law is strictly a sum-
N  nN A  (3.99 10 5 mol)  (6.0221  1023 mol 1 )
19 mary of observations and independent of any assumed model.
 2.41  10
Figure 1A.2 depicts the variation of the pressure of a sam-
The result, being a pure number, is dimensionless. ple of gas as the volume is changed. Each of the curves in the
Self-test 1A.1 Express the pressure in bar and in atm. graph corresponds to a single temperature and hence is called
an isotherm. According to Boyle’s law, the isotherms of gases
Answer: 1.39  102 bar or 1.37  102 atm
are hyperbolas (curves obtained by plotting y against x with
xy = constant, or y = constant/x). An alternative depiction, a
plot of pressure against 1/volume, is shown in Fig. 1A.3; in
such a plot the isotherms are straight lines because p is pro-
Exercises portional to 1/V. Note that all the lines extrapolate to the point
E1A.1 Express (i) 108 kPa in torr and (ii) 0.975 bar in atmospheres. =p 0= , 1/V 0 but have slopes that depend on the temperature.
−1
E1A.2 What mass of methanol (molar mass 32.04 g mol ) contains the same The linear variation of volume with temperature summa-
number of molecules as 1.00 g of ethanol (molar mass 46.07 g mol−1)? rized by Charles’s law is illustrated in Fig. 1A.4. The lines in this
1A The perfect gas 7

Increasing Decreasing
pressure, p
Pressure, p

temperature, T

Volume, V

Extrapolation
0 0
0 0 Temperature, T
Volume, V

Figure 1A.2 The pressure-volume dependence of a fixed Figure 1A.4 The volume-temperature dependence of a fixed
amount of gas that obeys Boyle’s law. Each curve is for a different amount of gas that obeys Charles’s law. Each line is for a different
temperature and is called an isotherm; each isotherm is a pressure and is called an isobar. Each isobar is a straight line
hyperbola (pV = constant). and extrapolates to zero volume at T = 0, corresponding to
θ = −273.15 °C.

Decreasing
Increasing volume, V
Pressure, p
Pressure, p

temperature, T

Extrapolation
Extrapolation

0 0
0 0
1/Volume, 1/V Temperature, T

Figure 1A.3 Straight lines are obtained when the pressure of Figure 1A.5 The pressure-temperature dependence of a fixed
a gas obeying Boyle’s law is plotted against 1/V at constant amount of gas that obeys Charles’s law. Each line is for a different
temperature. These lines extrapolate to zero pressure at 1/V = 0. volume and is called an isochore. Each isochore is a straight line
and extrapolates to zero pressure at T = 0.

illustration are examples of isobars, or lines showing the varia- e­ xperimentally to be the same for all gases, is denoted R and
tion of properties at constant pressure. All these isobars extrap- called the (molar) gas constant. The resulting expression
olate to the point=V 0= , T 0 and have slopes that depend on
pV = nRT Perfect gas law (1A.4)
the pressure. Figure 1A.5 illustrates the linear variation of pres-
sure with temperature. The lines in this diagram are isochores, is the perfect gas law (or perfect gas equation of state). A gas
or lines showing the variation of properties at constant volume, that obeys this law exactly under all conditions is called a
and they all extrapolate to=p 0= , T 0. ­perfect gas (or ideal gas). Although the term ‘ideal gas’ is used
The empirical observations summarized by Boyle’s and widely, in this text we prefer to use ‘perfect gas’ because there is
Charles’s laws and Avogadro’s principle can be combined into an important and useful distinction between ideal and perfect.
a single expression: The distinction is that in an ‘ideal system’ all the interactions
between molecules are the same; in a ‘perfect system’, not only
pV  constant  nT
are they the same but they are also zero.
This expression is consistent with Boyle’s law, pV = constant For a real gas, any actual gas, the perfect gas law is approxi-
when n and T are constant. It is also consistent with both mate, but the approximation becomes better as the pressure
forms of Charles’s law: p ∝ T when n and V are held constant, of the gas approaches zero. In the limit that the pressure goes
and V ∝ T when n and p are held constant. The expression to zero, p → 0, the equation is exact. The value of the gas con-
also agrees with Avogadro’s principle, V ∝ n when p and T stant R can be determined by evaluating R = pV/nT for a gas
are constant. The constant of proportionality, which is found in the limit of zero pressure (to guarantee that it is behaving
8 1 The properties of gases

from one set of values to another, then because pV/nT is equal


to a constant, the two sets of values are related by the ‘combined
gas law’
Surface
Pressure, p

p1V1 p2V2
of possible = Combined gas law  (1A.5)
states n1T1 n2T2
In this case the volume is the same before and after heating, so
V1 = V2 and these terms cancel. Likewise the amount does not
change upon heating, so n1 = n2 and these terms also cancel.
,T
re
tu
Volume, V ra The solution Cancellation of the volumes and amounts on each
pe
m side of the combined gas law results in
Te

Figure 1A.6 A region of the p,V,T surface of a fixed amount of p1 p2


=
perfect gas molecules. The points forming the surface represent T1 T2
the only states of the gas that can exist. which can be rearranged into
T2
p2   p1
Isotherm T1
Isobar Substitution of the data then gives
pV = constant 500 K
p2   (100 atm)  167 atm
Pressure, p

Isochore 300 K
V∝T
Self-test 1A.2 What temperature would be needed for the same
sample to exert a pressure of 300 atm?
V
Vo
olum
mee,
e, V
p∝T Answer: 900 K

,T
re
a tu
Volume, V per
Te
m The molecular explanation of Boyle’s law is that if a sample of
gas is compressed to half its volume, then twice as many mol-
Figure 1A.7 Sections through the surface shown in Fig. 1A.6 at
ecules strike the walls in a given period of time than before it
constant temperature give the isotherms shown in Fig. 1A.2.
was compressed. As a result, the average force exerted on the
Sections at constant pressure give the isobars shown in Fig. 1A.4.
Sections at constant volume give the isochores shown in Fig. 1A.5. walls is doubled. Hence, when the volume is halved the pres-
sure of the gas is doubled, and pV is a constant. Boyle’s law ap-
plies to all gases regardless of their chemical identity (provided
­ erfectly). As remarked in Energy: A first look, the modern pro-
p the pressure is low) because at low pressures the average sepa-
cedure is to note that R = NAk, where k is Boltzmann’s constant ration of molecules is so great that they exert no influence on
and NA has its newly defined value, as indicated earlier. one another and hence travel independently.
The surface in Fig. 1A.6 is a plot of the pressure of a fixed The molecular explanation of Charles’s law lies in the fact
amount of perfect gas molecules against its volume and ther- that raising the temperature of a gas increases the average speed
modynamic temperature as given by eqn 1A.4. The surface of its molecules. The molecules collide with the walls more fre-
depicts the only possible states of a perfect gas: the gas cannot quently and with greater impact. Therefore they exert a greater
exist in states that do not correspond to points on the surface. pressure on the walls of the container. For a quantitative ac-
Figure 1A.7 shows how the graphs in Figs. 1A.2, 1A.4, and 1A.5 count of these relations, see Topic 1B.
correspond to sections through the surface.
1A.2(b) The value of the gas constant
Example 1A.2 Using the perfect gas law If the pressure, volume, amount, and temperature are expressed
in their SI units the gas constant R has units N m K−1 mol−1 which,
Nitrogen gas is introduced into a vessel of constant volume at a because 1 J = 1 N m, can be expressed in terms of J K−1 mol−1. The
pressure of 100 atm and a temperature of 300 K. The tempera- currently accepted value of R is 8.3145 J K−1 mol−1. Other com-
ture is then raised to 500 K. What pressure would the gas then binations of units for pressure and volume result in different
exert, assuming that it behaved as a perfect gas?
values and units for the gas constant. Some commonly encoun-
Collect your thoughts The pressure is expected to be greater on tered combinations are given in Table 1A.2.
account of the increase in temperature. The perfect gas law in The perfect gas law is of the greatest importance in physical
the form pV/nT = R implies that if the conditions are changed chemistry because it is used to derive a wide range of relations
1A The perfect gas 9

Table 1A.2 The (molar) gas constant* This law is valid only for mixtures of perfect gases, so it is not
used to define partial pressure. Partial pressure is defined by
R
eqn 1A.6, which is valid for all gases.
8.314 47 J K−1 mol−1
−2
8.205 74 × 10 dm3 atm K−1 mol−1
Example 1A.3 Calculating partial pressures
8.314 47 × 10−2 dm3 bar K−1 mol−1
3 −1 −1
8.314 47 Pa m K mol The mass percentage composition of dry air at sea level is
62.364 dm3 Torr K−1 mol−1 approximately N2: 75.5; O2: 23.2; Ar: 1.3. What is the partial
1.987 21 cal K−1 mol−1 pressure of each component when the total pressure is 1.20 atm?

* The gas constant is now defined as R = NAk, where NA is Avogadro’s constant and k is Collect your thoughts Partial pressures are defined by eqn 1A.6.
Boltzmann’s constant. To use the equation, first calculate the mole fractions of the
components by using eqn 1A.7 and the fact that the amount of
atoms or molecules J of molar mass MJ in a sample of mass mJ is
nJ = mJ/MJ. The mole fractions are independent of the total mass
found throughout thermodynamics. It is also of considerable of the sample, so choose the latter to be exactly 100 g (which
practical utility for calculating the properties of a perfect gas makes the conversion from mass percentages very straightfor-
under a variety of conditions. For instance, the molar v­ olume, ward). Thus, the mass of N2 present is 75.5 per cent of 100 g,
Vm = V/n, of a perfect gas under the conditions called ­standard which is 75.5 g.
ambient temperature and pressure (SATP), defined as
298.15 K and 1 bar, is calculated as 24.789 dm3 mol−1. An earlier The solution The amounts of each type of atom or molecule
definition, standard temperature and pressure (STP), was 0 °C present in 100 g of air, in which the masses of N2, O2, and Ar
are 75.5 g, 23.2 g, and 1.3 g, respectively, are
and 1 atm; at STP, the molar volume of a perfect gas under these
conditions is 22.414 dm3 mol−1. 75.5 g 75.5
n(N2 )   mol  2.69 mol
28.02 g mol 1 28.02
1A.2(c) Mixtures of gases 23.2 g 23.2
n(O2 )  1
 mol  0.725 mol
When dealing with gaseous mixtures, it is often necessary to 32.00 g mol 32.00
know the contribution that each component makes to the total 1. 3 g 1. 3
n(Ar )  1
 mol  0.033 mol
pressure of the sample. The partial pressure, pJ, of a gas J in a 39.95 g mol 39.95
mixture (any gas, not just a perfect gas), is defined as
The total is 3.45 mol. The mole fractions are obtained by divid-
pJ = x J p
Partial pressure
 (1A.6) ing each of the above amounts by 3.45 mol and the partial pres-
[definition] sures are then obtained by multiplying the mole fraction by the
where xJ is the mole fraction of the component J, the amount of total pressure (1.20 atm):
J expressed as a fraction of the total amount of molecules, n, in
N2 O2 Ar
the sample:
Mole fraction: 0.780 0.210 0.0096
n Mole fraction
xJ  J n  nA  nB  [definition]
 (1A.7) Partial pressure/atm: 0.936 0.252 0.012
n
When no J molecules are present, xJ = 0; when only J molecules are Self-test 1A.3 When carbon dioxide is taken into account, the
present, xJ = 1. It follows from the definition of xJ that, whatever the mass percentages are 75.52 (N2), 23.15 (O2), 1.28 (Ar), and
composition of the mixture, x A  x B    1 and therefore that the 0.046 (CO2). What are the partial pressures when the total
sum of the partial pressures is equal to the total pressure: pressure is 0.900 atm?
Answer: 0.703, 0.189, 0.0084, and 0.00027 atm
pA  pB    (x A  x B  ) p  p (1A.8)
This relation is true for both real and perfect gases.
When all the gases are perfect, the partial pressure as de- Exercises
fined in eqn 1A.6 is also the pressure that each gas would exert 3
E1A.3 Could 131 g of xenon gas in a vessel of volume 1.0 dm exert a pressure
if it occupied the same container alone at the same tempera- of 20 atm at 25 °C if it behaved as a perfect gas? If not, what pressure would
ture. The latter is the original meaning of ‘partial pressure’. it exert?

That identification was the basis of the original formulation E1A.4 A perfect gas undergoes isothermal compression, which reduces its
of Dalton’s law: volume by 2.20 dm3. The final pressure and volume of the gas are 5.04 bar
and 4.65 dm3, respectively. Calculate the original pressure of the gas in (i) bar,
The pressure exerted by a mixture of gases is the sum of (ii) atm.
the pressures that each one would exert if it occupied the −3
E1A.5 At 500 °C and 93.2 kPa, the mass density of sulfur vapour is 3.710 kg m .
container alone. What is the molecular formula of sulfur under these conditions?
10 1 The properties of gases

−3 3
E1A.6 Given that the mass density of air at 0.987 bar and 27 °C is 1.146 kg m , E1A.7 A vessel of volume 22.4 dm contains 2.0 mol H2(g) and 1.0 mol N2(g) at
calculate the mole fraction and partial pressure of nitrogen and oxygen 273.15 K. Calculate (i) the mole fractions of each component, (ii) their partial
assuming that air consists only of these two gases. pressures, and (iii) their total pressure.

Checklist of concepts
☐ 1. The physical state of a sample of a substance, is its form ☐ 5. An isotherm is a line in a graph that corresponds to a
(solid, liquid, or gas) under the current conditions of single temperature.
pressure, volume, and temperature. ☐ 6. An isobar is a line in a graph that corresponds to a sin-
☐ 2. Mechanical equilibrium is the condition of equality of gle pressure.
pressure on either side of a shared movable wall. ☐ 7. An isochore is a line in a graph that corresponds to a
☐ 3. An equation of state is an equation that interrelates the single volume.
variables that define the state of a substance. ☐ 8. A perfect gas is a gas that obeys the perfect gas law
☐ 4. Boyle’s and Charles’s laws are examples of a limiting under all conditions.
law, a law that is strictly true only in a certain limit, in ☐ 9. Dalton’s law states that the pressure exerted by a mix-
this case p → 0. ture of (perfect) gases is the sum of the pressures that
each one would exert if it occupied the container alone.

Checklist of equations
Property Equation Comment Equation number
Relation between temperature scales T/K = θ/°C + 273.15 273.15 is exact 1A.1
Perfect gas law pV = nRT Valid for real gases in the limit p → 0 1A.4
Partial pressure p J = x Jp Definition; valid for all gases 1A.6
Mole fraction xJ = nJ/n Definition 1A.7
n  nA  nB  
TOPIC 1B The kinetic model

1B.1(a) Pressure and molecular speeds


➤ Why do you need to know this material?
From the very economical assumptions of the kinetic model, it
This topic illustrates how a simple model for a gas can
is possible to derive an expression that relates the pressure and
lead to useful quantitative predictions of its properties.
volume of a gas to the mean speed with which the molecules
The same model is used in the discussion of the transport
are moving.
properties of gases (Topic 16A), reaction rates in gases
(Topic 18A), and catalysis (Topic 19C).

➤ What is the key idea?


How is that done? 1B.1 Using the kinetic model to derive
The model assumes that a gas consists of molecules of
negligible size in ceaseless random motion and obeying an expression for the pressure of a gas
the laws of classical mechanics in their collisions. Consider the arrangement in Fig. 1B.1, and then follow these
steps.
➤ What do you need to know already?
Step 1 Set up the calculation of the change in momentum
You need to be aware of Newton’s second law of motion
and the Boltzmann distribution (both are introduced in Consider a particle of mass m that is travelling with a com-
Energy: A first look). ponent of velocity vx parallel to the x-axis. The particle col-
lides with the wall on the right and bounces off in an elastic
collision. The linear momentum is mvx before the collision
and −mvx after the collision because the particle is then travel-
In the kinetic theory of gases (which is sometimes called the ling in the opposite direction. The x-component of momen-
kinetic-molecular theory, KMT) it is assumed that the only tum therefore changes by 2mvx on each collision; the y- and
contribution to the energy of the gas is from the kinetic en- z-components are unchanged. Many molecules collide with
ergies of the molecules. The kinetic model is one of the most the wall in an interval Δt, and the total change of momentum
­remarkable—and arguably most beautiful—models in physical is the product of the change in momentum of each molecule
chemistry, because powerful quantitative conclusions can be multiplied by the number of molecules that hit the wall ­during
reached from very slender assumptions. the interval.

1B.1 The model Before


mvx
collision
The kinetic model is based on three assumptions:

1. The gas consists of molecules of mass m in ceaseless ran-


dom motion obeying the laws of classical mechanics. –mvx
After
2. The size of the molecules is negligible, in the sense that collision
their diameters are much smaller than the average dis-
x
tance travelled between collisions; they are ‘point-like’.
3. The molecules interact only through brief elastic collisions. Figure 1B.1 The pressure of a gas arises from the impact of its
molecules on the walls. In an elastic collision of a molecule with
An elastic collision is a collision in which the total translational a wall perpendicular to the x-axis, the x-component of velocity is
kinetic energy of the molecules is unchanged. reversed but the y- and z-components are unchanged.
12 1 The properties of gases

|vx Δt | Not all the molecules travel with the same velocity, so the meas-
ured pressure, p, is the average (denoted ) of the quantity
Area, A just calculated:
Will nM  vx2 
Won’t p
V
The average values of vx2 , v 2y , and vz2 are all the same, and
x because v 2  vx2  v 2y  vz2 , it follows that  vx2   13  v 2 .
At this stage it is useful to define the root-mean-square
Volume = |vx Δt |A
speed, v rms, as the square root of the mean of the squares of
the speeds, v, of the molecules. Therefore
Figure 1B.2 A molecule will reach the wall on the right within
an interval of time ∆t if it is within a distance vx∆t of the wall and vrms   v 2 1/ 2
Root-mean-square speed
 (1B.1)
travelling to the right. [definition]

The mean square speed in the expression for the pressure can
therefore be written  vx2   13  v 2   13 vrms
2
to give
Step 2 Calculate the total change in momentum
Because a molecule with velocity component vx travels a dis- 2 (1B.2)
pV = 13 nM vrms
Relation between pressure and volume
tance vx ∆t along the x-axis in an interval Δt, all the molecules [KMT]
within a distance vx ∆t of the wall strike it if they are travelling
towards it (Fig. 1B.2). It follows that if the wall has area A, then
all the particles in a volume A  vx t reach the wall (if they are This equation is one of the key results of the kinetic model. In
travelling towards it). If the number density of particles (the the next section it will be shown that the right-hand side of
number divided by the volume) is N then it follows that the eqn 1B.2 is equal to nRT, thus giving the perfect gas equation.
number of molecules in this volume is NAvx ∆t . The number This simple model of a gas therefore leads to the same equation
density can be written as N = nN A /V , where n is the total of state as that of a perfect gas.
amount of molecules in the container of volume V and NA is
Avogadro’s constant. It follows that the number of molecules in
the volume Avx ∆t is (nN A /V )  Av x t . The Maxwell–Boltzmann distribution
1B.1(b)
At any instant, half the particles are moving to the right of speeds
and half are moving to the left. Therefore, the average In a gas the speeds of individual molecules span a wide range,
number of collisions with the wall during the interval Δt is and the collisions in the gas ensure that their speeds are cease-
1
2
nN A Avx ∆t /V . The total momentum change in that interval lessly changing. Before a collision, a molecule may be travelling
is the product of this number and the change 2mvx: rapidly, but after a collision it may be accelerated to a higher
nN A Avx t speed, only to be slowed again by the next collision. To evaluate
total momentum change   2mvx the root-mean-square speed it is necessary to know the frac-
2V
 M tion of molecules that have a given speed at any instant. The
nmN A Avx2 t nMAvx2 t fraction of molecules that have speeds in the range v to v + dv
 
V V is given by f (v)dv, where f (v) is called the distribution of
In the second line, mN A has been replaced by the molar mass M. speeds. Note that this fraction is proportional to the width
of the range, dv. An expression for this distribution can be
Step 3 Calculate the force found by recognizing that the energy of the molecules is en-
The rate of change of momentum is the change of momentum tirely kinetic, and then using the Boltzmann distribution to
divided by the interval Δt during which the change occurs: ­describe how this energy is distributed over the molecules.
nMAvx2 t 1 nMAvx2
rate of change of momentum   
V t V How is that done? 1B.2 Deriving the distribution of speeds
According to Newton’s second law of motion, this rate of
change of momentum is equal to the force. The starting point for this derivation is the Boltzmann distribu-
tion (see Energy: A first look).
Step 4 Calculate the pressure
Step 1 Write an expression for the distribution of the kinetic energy
The pressure is this force (nMAvx2 /V ) divided by the area (A) on
which the impacts occur. The areas cancel, leaving The Boltzmann distribution implies that the fraction of mol-
ecules with velocity components vx , v y , and vz is propor-
nM vx2 tional to an exponential function of their kinetic energy
pressure =
V ε : f (v)  Ke  /kT , where K is a constant of proportionality. The
1B The kinetic model 13

kinetic energy is simply the sum on the contributions from the Surface area, 4πv 2 vz
Thickness, dv
motion along x, y, and z:
  12 mvx2  12 mv 2y  12 mvz2
v
The relation a x  y  z  a x a y a z is then used to write
 ( mv2  mv2  mv2 ) / 2 kT
f (v)  Ke   /kT  Ke x y z
2  mv2 / 2 kT 2 vy
 Ke  mvx / 2 kT e y e mvz / 2 kT vx
The form of this equation implies that the distribution of
speeds can be factorized into a product of three distributions,
one for each direction
f (v ) Figure 1B.3 To evaluate the probability that a molecule has a
f    f
( vx ) (vz )
y

2  mv 2
/ 2 kT 2 speed in the range v to v + dv, evaluate the total probability that
f (v)  K x e  mvx / 2 kT K y e y K z e  mvz / 2 kT  f (vx ) f (v y ) f (vz ) the molecule will have a speed that is anywhere in a thin shell of
with K = K x K y K z . The distribution of speeds along the x direc- radius v  (v x2  v 2y  vz2 )1/ 2 and thickness dv.
tion, f (vx ), is therefore
2
f (v x )  K x e  mvx / 2 kT velocity components as defining three coordinates in ‘velocity
and likewise for the other two coordinates. space’, with the same properties as ordinary space except that
the axes are labelled (v x , v y , vz ) instead of (x , y , z ). Just as the
Step 2 Evaluate the constants Kx, Ky, and Kz volume element in ordinary space is dxdydz, so the volume
To evaluate the constant Kx, note that a molecule must have a element in velocity space is dvx dv y dvz .
velocity component somewhere in the range   vx   , so In ordinary space points that lie at a distance between
integration of the distribution over the full range of possible r and r + dr from the centre are contained in a spherical shell
values of vx must give a total probability of 1: of thickness dr and radius r. The volume of this shell is the
 product of its surface area, 4πr2, and its thickness dr, and is

f (v x )dvx  1 therefore 4 πr 2 dr . Similarly, the analogous volume in velocity
Substitution of the expression for f (vx ) then gives space is the volume of a shell of radius v and thickness dv,
namely 4 πv 2 dv (Fig. 1B.3).
Integral G.1
   1/ 2 Because f (vx ) f (v y ) f (vz ), the boxed term in eqn 1B.4,
  2kT 
1  K x  e  mvx / 2 kT dvx  K x 
2

 depends only on v 2 , and has the same value everywhere in a



 m  shell of radius v, the total probability of the molecules possess-
Therefore, K x  (m/2kT )1/ 2 and ing a speed in the range v to v + dv is the product of the boxed
1/ 2 term and the volume of the shell of radius v and thickness dv.
 m   mvx2 / 2 kT
f (v x )    e  (1B.3) If this probability is written f (v)dv , it follows that
 2kT  3/2
The expressions for f (v y ) and f (vz ) are analogous.  m   mv2 / 2 kT
f (v)dv  4 v 2dv   e
 2kT 
Step 3 Write a preliminary expression for
and f (v) itself, after minor rearrangement, is
f (vx ) f (v y ) f (vz )dvx dv y dvz
3/2
The probability that a molecule has a velocity in the range  m  2
f (v)  4    v 2e  mv / 2 kT
vx to vx + dvx , v y to v y + dv y , vz to vz + dvz is  2kT 
2 2 2
e − m ( vx +vy +vz )/2 kT The molar gas constant R is related to Boltzmann’s constant
3/2
m − mvx2 /2 kT − mv 2y /2 kT 2 k by R = NAk, where NA is Avogadro’s constant. It follows that
f (vx)f (vy)f (vz) dvxdvydvz = e e e− mvz /2 kT
2πkT m/k = mNA/R = M/R, where M is the molar mass. The term
× dvx dvydvz f (v) is then written as

m
3/2
2 M
3/2
2
(1B.5)
= e − mv /2 kT dvxdvydvz (1B.4) f (v ) = 4 π v 2e − Mv /2 RT Maxwell–Boltzmann
2πkT 2πRT
distribution [KMT]
where v 2  vx2  v 2y  vz2 .
Step 4 Calculate the probability that a molecule has a speed in The function f (v) is called the Maxwell–Boltzmann ­distribution
the range v to v + dv of speeds. Note that, in common with other distribution func-
To evaluate the probability that a molecule has a speed in the tions, f (v) acquires physical significance only after it is multi-
range v to v + dv regardless of direction, think of the three plied by the range of speeds of interest.
14 1 The properties of gases

Low temperature
or high molecular mass
Distribution function, f (v)

Distribution function, f(v)


Intermediate temperature or
molecular mass

High temperature or
low molecular mass

0 Speed, v
0 v1 v2
Speed, v

Figure 1B.4 The distribution of molecular speeds with Figure 1B.5 To calculate the probability that a molecule will have
temperature and molar mass. Note that the most probable speed a speed in the range v1 to v2, integrate the distribution between
(corresponding to the peak of the distribution) increases with those two limits; the integral is equal to the area under the curve
temperature and with decreasing molar mass, and simultaneously between the limits, as shown shaded here.
the distribution becomes broader. Interact with the dynamic
version of this graph in the e-book.

The important features of the Maxwell–Boltzmann distribu- with speeds in the range v1 to v2 is found by evaluating the
tion are as follows (and are shown pictorially in Fig. 1B.4): integral
• Equation 1B.5 includes a decaying exponential function v2
F (v1, v2 )   f (v)dv
(more specifically, a Gaussian function). Its presence v1

implies that the fraction of molecules with very high This integral is the area under the graph of f as a function of v
2
speeds is very small because e −x becomes very small when and, except in special cases, has to be evaluated numerically by
x is large. using mathematical software (Fig. 1B.5). The average value of a
• The factor M/2RT multiplying v2 in the exponent is large power of the speed, vn, is calculated as
when the molar mass, M, is large, so the exponential fac- 
tor goes most rapidly towards zero when M is large. That  vn    vn f (v)dv (1B.6)
0
is, heavy molecules are unlikely to be found with very high
speeds. In particular, integration with n = 2 results in the mean square
speed, 〈v2〉, of the molecules at a temperature T:
• The opposite is true when the temperature, T, is high:
then the factor M/2RT in the exponent is small, so the 3RT Mean square speed
v2   [KMT]
 (1B.7)
exponential factor falls towards zero relatively slowly M
as v increases. In other words, a greater fraction of the It follows that the root-mean-square speed of the molecules of
molecules can be expected to have high speeds at high the gas is
temperatures than at low temperatures. 1/ 2
• A factor v2 (the term before the e) multiplies the expo-  3RT  Root-mean-square
vrms   v 2 1/ 2    speed [KMT]
 (1B.8)
nential. This factor goes to zero as v goes to zero, so the  M 
fraction of molecules with very low speeds will also be which is proportional to the square root of the temperature
very small whatever their mass. and inversely proportional to the square root of the molar
• The remaining factors (the term in parentheses in eqn mass. That is, the higher the temperature, the higher the root-
1B.5 and the 4π) simply ensure that, when the fractions mean-square speed of the molecules, and, at a given tempera-
are summed over the entire range of speeds from zero to ture, heavy molecules travel more slowly than light molecules.
infinity, the result is 1. In the previous section, a kinetic argument was used
to derive eqn 1B.2 pV  13 nM vrms2

, a relation between the 
pressure and volume. When the expression for vrms from
1B.1(c) Mean values eqn 1B.8 is substituted into eqn 1B.2 the result is pV = nRT.
With the Maxwell–Boltzmann distribution in hand, it is This is the equation of state of a perfect gas, so this result
possible to calculate various quantities relating to the speed confirms that the kinetic model can be regarded as a model
of the molecules. For instance, the fraction, F, of molecules of a perfect gas.
1B The kinetic model 15

Example 1B.1 Calculating the mean speed of molecules


vmp = (2RT/M)1/2
in a gas
vmean = (8RT/πM)1/2

f(v)/4π(M/2πRT)1/2
Calculate vrms and the mean speed, vmean, of N2 molecules at vrms = (3RT/M)1/2
25 °C.
Collect your thoughts The root-mean-square speed is calculated
from eqn 1B.8, with M = 28.02 g mol−1 (that is, 0.028 02 kg mol−1)
and T = 298 K. The mean speed is obtained by evaluating the
integral

vmean   v f (v)dv 1 (3/2)1/2 v/(2RT/M)1/2
0 (4/π)1/2
with f (v) given in eqn 1B.5. Use either mathematical software
Figure 1B.6 A summary of the conclusions that can be deduced
or the integrals listed in the Resource section and note that from the Maxwell–Boltzmann distribution for molecules of molar
1 J = 1 kg m2 s−2. mass M at a temperature T: vmp is the most probable speed, vmean is
The solution The root-mean-square speed is the mean speed, and vrms is the root-mean-square speed.
1/ 2
 3  (8.3145 JK mol )  (298 K ) 
1 1

vrms     515 ms 1 21/2v


 0.02802 kg mol 1 
v
v
The integral required for the calculation of vmean is
Integral G.4
3/2   v
 M   v v

2
vmean  4   v 3e  M v / 2 RT
dv
 2RT  0
0 2 v 1/2
2v
3/2 2 1/ 2
 M   2RT   8RT  v v v
 4    12    
 2RT   M   M 
Substitution of the data then gives
1/ 2 Figure 1B.7 A simplified version of the argument to show that the
 8  (8.3145 JK 1 mol 1 )  (298 K ) 
vmean     475 ms 1 mean relative speed of molecules in a gas is related to the mean
   (0.02802 kg mol 1 )
  speed. When the molecules are moving in the same direction,
the mean relative speed is zero; it is 2v when the molecules are
Self-test 1B.1 Derive the expression for 〈v2〉 in eqn 1B.7 by
approaching each other. A typical mean direction of approach is
evaluating the integral in eqn 1B.6 with n = 2.
from the side, and the mean speed of approach is then 21/2v. The
last direction of approach is the most characteristic, so the mean
speed of approach can be expected to be about 21/2v. This value is
As shown in Example 1B.1, the Maxwell–Boltzmann dis-
confirmed by more detailed calculation.
tribution can be used to evaluate the mean speed, vmean, of the
molecules in a gas:
1/ 2 1/ 2
This result is much harder to derive, but the diagram in
 8RT   8  Mean speed Fig. 1B.7 should help to show that it is plausible. For the relative
vmean      vrms [KMT]
 (1B.9)
 M   3  mean speed of two dissimilar molecules of masses mA and mB
The most probable speed, vmp , can be identified from the loca- (note that this expression is given in terms of the masses of the
tion of the peak of the distribution by differentiating f (v) with molecules, rather than their molar masses):
respect to v and looking for the value of v at which the deriva-  8kT 
1/ 2
mA mB Mean relative
tive is zero (other than at v = 0 and v = ∞; see Problem 1B.11): vrel     speed [perrfect gas]
 (1B.11b)
   mA  mB
1/ 2
 2RT 
  23  vrms
1/ 2 Most probable
vmp    speed [KMT]
 (1B.10)
 M 
Exercises
Figure 1B.6 summarizes these results. −1
E1B.1 Calculate the root-mean-square speeds of H2 (molar mass 2.016 g mol )
The mean relative speed, vrel, the mean speed with which and O2 molecules (molar mass 32.00 g mol ) at 20 °C. −1

one molecule approaches another of the same kind, can also be


E1B.2 What is the relative mean speed of N2 molecules (molar mass
calculated from the distribution: 28.02 g mol−1) and H2 molecules (molar mass 2.016 g mol−1) in a gas at 25 °C?
Mean relative speed
vrel = 21/ 2 vmean [KMT, identical moleculess]
 (1B.11a) E1B.3 Calculate the most probable speed, the mean speed, and the mean
relative speed of CO2 molecules (molar mass 44.01 g mol−1) at 20 °C.
16 1 The properties of gases

1B.2 Collisions Table 1B.1 Collision cross-sections*

σ/nm2
The kinetic model can be used to calculate the average fre-
C6H6 0.88
quency with which molecular collisions occur and the average
CO2 0.52
distance a molecule travels between collisions. These results are
He 0.21
used elsewhere in the text to develop theories about the trans-
port properties of gases and the reaction rates in gases. N2 0.43

* More values are given in the Resource section.


1B.2(a) The collision frequency
The parameter σ is called the collision cross-section of the
Although the kinetic model assumes that the molecules are molecules. Some typical values are given in Table 1B.1.
point-like, a ‘hit’ can be counted as occurring whenever the cen- The number density can be expressed in terms of the pres-
tres of two molecules come within a characteristic distance d, the sure by using the perfect gas equation and R = NAk:
­collision diameter, of each other. If the molecules are modelled as
impenetrable hard spheres, then d is the diameter of the sphere. N nN A nN A pN A p
N= == = =
The kinetic model can be used to deduce the collision frequency, V V nRT /p RT kT
z, the number of collisions made by one molecule divided by the The collision frequency can then be expressed in terms of the
time interval during which the collisions are counted. pressure:
 vrel p Collision frequency
How is that done? 1B.3 Using the kinetic model to derive z [KMT]
 (1B.12b)
kT
an expression for the collision frequency
Equation 1B.12a shows that, at constant volume, the collision
One way to approach this problem is to imagine that all but frequency increases with increasing temperature, because the
one of the molecules are stationary and that this one molecule average speed of the molecules is greater. Equation 1B.12b
then moves through a stationary array of molecules, colliding
shows that, at constant temperature, the collision frequency
with those that it approaches sufficiently closely. This model
is proportional to the pressure. The greater the pressure, the
focuses on the relative motion of the molecules, so rather than
greater is the number density of molecules in the sample, and
using the mean speed of the molecules it is appropriate to use
the rate at which they encounter one another is greater.
the mean relative speed vrel .
In a period of time ∆t the molecule moves a distance vrel ∆t.
As it moves forward it will collide with any molecule with Brief illustration 1B.1
a centre that comes within a distance d of the centre of the
moving molecule. As shown in Fig. 1B.8, it follows that there In Example 1B.1 it is calculated that at 25 °C the mean speed
will be collisions with all the molecules in a cylinder with of N2 molecules is 475 m s−1. The mean relative speed is there-
cross-sectional area   d 2 and length vrel ∆t. The volume of fore given by eqn 1B.11a as vrel  21/ 2 vmean  671 ms 1. From
this cylinder is  vrel t and hence the number of molecules in Table 1B.1 the collision cross section of N2 is σ = 0.43 nm2
the cylinder is N vrel t , where N is the number density of the (­corresponding to 0.43 × 10−18 m2). The collision frequency at
molecules, the number N in a given volume V divided by that 25 °C and 1.00 atm (101 kPa) is therefore given by eqn 1B.12b as
volume. The number of collisions is therefore N vrel t and (0.43  1018 m2 )  (671 ms 1 )  (1.01  105 Pa )
hence the collision frequency z is this number divided by ∆t: z
(1.381  1023 JK 1 )  (298 K )

Miss
 7.1  109 s 1
vrelt
A given molecule therefore collides about 7 × 109 times each
second.
d
d
Hit
1B.2(b) The mean free path
Area, σ The mean free path, λ (lambda), is the average distance a mol-
ecule travels between collisions. If a molecule collides with a
Figure 1B.8 The basis of the calculation of the collision
frequency z, it spends a time 1/z in free flight between colli-
frequency in the kinetic theory of gases.
sions, and therefore travels a distance (1/z ) × vrel. It follows that
(1B.12a) the mean free path is
z = σ vrel N
Collision frequency v
[KMT]   rel Mean free path [KMT] (1B.13)
z
1B The kinetic model 17

Substitution of the expression for z from eqn 1B.12b gives s­ ample of gas provided the volume is constant. In a container
kT of fixed volume the distance between collisions depends on the
 Mean free path [perfect gas] (1B.14) number of molecules present in the given volume, not on the
p
speed at which they travel.
Doubling the pressure shortens the mean free path by a factor In summary, a typical gas (N2 or O2) at 1 atm and 25 °C can
of 2. be thought of as a collection of molecules travelling with a
mean speed of about 500 m s−1. Each molecule makes a collision
Brief illustration 1B.2 within about 1 ns, and between collisions it travels about 103
molecular diameters.
From Brief illustration 1B.1 vrel  671 ms 1 for N2 molecules at
25 °C and z = 7.1 × 109 s−1 when the pressure is 1.00 atm. Under
these circumstances, the mean free path of N2 molecules is Exercises
1
671 ms E1B.4 Assume that air consists of N2 molecules with a collision diameter of
 9 1
 9.5  108 m 395 pm and a molar mass of 28.02 g mol−1. Calculate (i) the mean speed of the
7.1  10 s
molecules, (ii) the mean free path, (iii) the collision frequency in air at 1.0 atm
or 95 nm, about 1000 molecular diameters. and 25 °C.
E1B.5 At what pressure does the mean free path of argon at 25 °C become
comparable to the diameter of a spherical 100 cm3 vessel that contains it? Take
Although the temperature appears in eqn 1B.14, in a sample σ = 0.36 nm2.
of constant volume, the pressure is proportional to T, so T/p E1B.6 At an altitude of 20 km the temperature is 217 K and the pressure is
remains constant when the temperature is increased. Therefore, 0.050 atm. What is the mean free path of N2 molecules? (σ = 0.43 nm2, molar
the mean free path is independent of the temperature in a mass 28.02 g mol−1).

Checklist of concepts
☐ 1. The kinetic model of a gas considers only the contri- ☐ 3. The root-mean-square speed of molecules can be cal-
bution to the energy from the kinetic energies of the culated from the Maxwell–Boltzmann distribution.
molecules; this model accounts for the equation of state ☐ 4. The collision frequency is the average number of colli-
of a perfect gas. sions made by a molecule in a period of time divided by
☐ 2. The Maxwell–Boltzmann distribution of speeds gives the length of that period.
the fraction of molecules that have speeds in a specified ☐ 5. The mean free path is the average distance a molecule
range. travels between collisions.

Checklist of equations
Property Equation Comment Equation number
2
Pressure–volume relationship from the kinetic model pV = 13 nM v rms Leads to perfect gas equation 1B.2
3/2 2  M v2 / 2 RT
Maxwell–Boltzmann distribution of speeds f (v)  4 ( M /2RT ) v e 1B.4
1/ 2
Root-mean-square speed vrms = (3RT /M ) 1B.8
1/ 2
Mean speed vmean  (8RT /M ) 1B.9
1/ 2
Most probable speed vmp = (2RT /M ) 1B.10

Mean relative speed vrel  (8kT / ) ,   mA mB /(mA  mB )


1/ 2
1B.11b

Collision frequency z   vrel p/kT 1B.12b

Mean free path   kT / p 1B.14


TOPIC 1C Real gases

Repulsive forces are significant only when molecules are al-


most in contact; they are short-range interactions (Fig. 1C.1).
➤ Why do you need to know this material? Therefore, repulsions can be expected to be important only
The properties of actual gases, so-called ‘real gases’, are dif- when the average separation of the molecules is small. This is
ferent from those of a perfect gas. Moreover, the deviations the case at high pressure, when many molecules occupy a small
from perfect behaviour give insight into the nature of the volume. Attractive intermolecular forces are effective over a
interactions between molecules. longer range than repulsive forces: they are important when
the molecules are moderately close together but not neces-
➤ What is the key idea? sarily touching (at the intermediate separations in Fig. 1C.1).
Attractions and repulsions between gas molecules account Attractive forces are ineffective when the distance between the
for modifications to the equation of state of a gas and molecules is large (well to the right in Fig. 1C.1). At sufficiently
account for critical behaviour. low temperatures the kinetic energy of the molecules no longer
exceeds the energy of interaction between them. When this is
➤ What do you need to know already? the case, the molecules can be captured by one another, eventu-
This Topic builds on and extends the discussion of perfect ally leading to condensation.
gases in Topic 1A. These interactions result in deviations from perfect gas
behaviour, as is illustrated in Fig. 1C.2 which shows experi-
mentally determined isotherms for CO2 at several different
temperatures. The illustration also shows, for comparison, the
perfect gas isotherm at 50 °C (in blue). At low pressures, when
Real gases do not obey the perfect gas law exactly except in the
the sample occupies a large volume, the molecules are so far
limit of p → 0. Deviations from the law are particularly signifi-
apart for most of the time that the intermolecular forces play
cant at high pressures and low temperatures, especially when a
no significant role, and the gas behaves virtually perfectly: the
gas is on the point of condensing to liquid.
perfect gas isotherm approaches the experimental isotherm on
the right of the plot.

Deviations from perfect


1C.1
behaviour
Real gases show deviations from the perfect gas law because
Potential energy and force

molecules interact with one another. A point to keep in mind is


Repulsion dominant
that repulsive forces between molecules assist expansion and at-
tractive forces assist compression. Put another way, the greater
Force
the repulsive forces the less compressible the gas becomes, in
the sense that at a given temperature a greater pressure (than Attraction dominant
0
for a perfect gas) is needed to achieve a certain reduction of vol-
ume. Attractive forces make the gas more compressible, in the Potential energy
sense that a smaller pressure (than for a perfect gas) is needed
0
to achieve a given reduction of volume. Internuclear separation
When discussing the interactions between molecules the Figure 1C.1 The dependence of the potential energy and of the
appropriate length scale is set by the molecular diameter. force between two molecules on their internuclear separation.
A ‘short’ or ‘small’ distance means a length comparable to a The region on the left of the vertical dotted line indicates where
molecular diameter, an ‘intermediate’ distance means a few the net outcome of the intermolecular forces is repulsive. At large
diameters, and a ‘long’ or ‘large’ distance means many mo- separations (far to the right) the potential energy is zero and there
lecular diameters. is no interaction between the molecules.
1C Real gases 19

140 to the line CDE, when both liquid and vapour are present in
50 °C Perfect
equilibrium, is called the vapour pressure of the liquid at the
temperature of the experiment.
100
At E, the sample is entirely liquid and the piston rests on its
F 40 °C
surface. Any further reduction of volume requires the exertion
p/atm

of considerable pressure, as is indicated by the sharply rising


60
E D C line to the left of E. Even a small reduction of volume from E to
B
F requires a great increase in pressure. In other words the liquid
20 °C A is very much more incompressible than the gas.
20

0 0.2
Vm/(dm3 mol–1)
0.4 0.6
1C.1(a) The compression factor
Figure 1C.2 Experimental isotherms for carbon dioxide at several
As a first step in understanding these observations it is useful to
temperatures; for comparison the isotherm for a perfect gas at introduce the compression factor, Z, the ratio of the measured
50 °C is shown. As explained in the text, at 20 °C the gas starts to molar volume of a gas, Vm = V/n, to the molar volume of a per-
condense to a liquid at point C, and the condensation is complete fect gas, Vm , at the same pressure and temperature:
at point E. Vm Compression factor
Z= [definition]  (1C.1)
Vm
At moderate pressures, when the average separation of the Because the molar volume of a perfect gas is equal to RT/p, an
molecules is only a few molecular diameters, the potential en- equivalent expression is Z = pVm /RT , which can be written as
ergy arising from the attractive force outweighs the potential
energy due to the repulsive force, and the net effect is a lower- pVm = RTZ  (1C.2)
ing of potential energy. When the attractive force (not its con-
tribution to the potential energy) is dominant, to the right of Because for a perfect gas Z = 1 under all conditions, deviation
the dotted vertical line in Fig. 1C.1, the gas can be expected to of Z from 1 is a measure of departure from perfect behaviour.
be more compressible than a perfect gas because the forces help Some experimental values of Z are plotted in Fig. 1C.3. At
to draw the molecules together. At high pressures, when the av- very low pressures, all the gases shown have Z ≈ 1 and behave
erage separation of the molecules is small and lies to the right nearly perfectly. At high pressures, all the gases have Z > 1,
of the dotted vertical line, the repulsive forces dominate and signifying that they have a larger molar volume than a per-
the gas can be expected to be less compressible because now fect gas. Repulsive forces are now dominant. At intermediate
the forces help to drive the molecules apart. In Fig. 1C.2 it can pressures, most gases have Z < 1, indicating that the attrac-
be seen that the experimental isotherm deviates more from the tive forces are reducing the molar volume relative to that of a
perfect gas isotherm at small molar volumes; that is, to the left ­perfect gas.
of the plot.
At lower temperatures, such as 20 °C, CO2 behaves in a
dramatically different way from a perfect gas. Consider what C2H4
happens when a sample of gas initially in the state marked A CH4
in Fig. 1C.2 is compressed (that is, its volume is reduced) at
Compression factor, Z

H2
constant temperature by pushing in a piston. Near A, the pres-
sure of the gas rises in approximate agreement with Boyle’s law. Perfect
Serious deviations from that law begin to appear when the vol- H2
1
ume has been reduced to B. Z p/atm
10
At C (which corresponds to about 60 atm for carbon diox- 0.98 CH4
ide) the piston suddenly slides in without any further rise in
NH3 0.96 C2H4
pressure: this stage is represented by the horizontal line CDE. NH3
Examination of the contents of the vessel shows that just to the 0 200 400 600 800
left of C a liquid appears, and that there is a sharply defined sur- p/atm

face separating the gas and liquid phases. As the volume is de- Figure 1C.3 The variation of the compression factor, Z, with
creased from C through D to E, the amount of liquid increases. pressure for several gases at 0 °C. A perfect gas has Z = 1 at all
There is no additional resistance to the piston because the gas pressures. Notice that, although the curves approach 1 as p → 0,
responds by condensing further. The pressure corresponding they do so with different slopes.
20 1 The properties of gases

Brief illustration 1C.1 coefficients of a gas are determined from measurements of its
compression factor.
The molar volume of a perfect gas at 500 K and 100 bar is
Vm  0.416 dm3 mol 1 . The molar volume of carbon dioxide
under the same conditions is Vm  0.366 dm3 mol 1. It follows Brief illustration 1C.2
that at 500 K
To use eqn 1C.3b (up to the B term) in a calculation of the pres-
3 1
0.366 dm mol sure exerted at 100 K by 0.104 mol O2(g) in a vessel of volume
Z  0.880
0.416 dm3 mol 1 0.225 dm3, begin by calculating the molar volume:

The fact that Z < 1 indicates that attractive forces dominate V 0.225 dm3
Vm    2.16 dm3 mol 1  2.16  103 m 3 mol 1
repulsive forces under these conditions. nO2 0.104 mol

Then, by using the value of B found in Table 1C.1 of the


Resource section,
1C.1(b) Virial coefficients
RT  B 
At large molar volumes and high temperatures the real-gas iso- p 1  
Vm  Vm 
therms do not differ greatly from perfect-gas isotherms. The
(8.3145 JK 1 mol 1 )  (100 K)  1.975  104 m3 mol 1 
small differences suggest that the perfect gas law pVm = RT is in   1  
fact the first term in an expression of the form 2.16  103 m3 mol 1  2.16  103 m3 mol 1 
 3.50  105 Pa, orr 350 kPa
pVm  RT (1  B p  C p2  ) (1C.3a)
To manipulate units the relation 1 Pa = 1 J m−3 is used. The
This expression is an example of a common procedure in physi- perfect gas equation of state would give the calculated pressure
cal chemistry, in which a simple law that is known to be a good as 385 kPa, or 10 per cent higher than the value calculated by
first approximation (in this case pVm = RT) is treated as the first using the virial equation of state. The difference is significant
term in a series in powers of a variable (in this case p). A more because under these conditions B/Vm ≈ 0.1, which is not negli-
convenient expansion for many applications is gible relative to 1.
 B C  Virial equation
pVm  RT  1   2   of state
 (1C.3b)
 V V 
m m An important point is that although the equation of state of a
These two expressions are two versions of the virial equation real gas may coincide with the perfect gas law as p → 0, not all
of state.1 By comparing the expression with eqn 1C.2 it is seen its properties necessarily coincide with those of a perfect gas in
that the terms in parentheses in eqns 1C.3a and 1C.3b are both that limit. Consider, for example, the value of dZ/dp, the slope
equal to the compression factor, Z. of the graph of compression factor against pressure. For a per-
The coefficients B, C, …, the values of which depend on the tem- fect gas dZ/dp = 0 (because Z = 1 at all pressures), but for a real
perature, are the second, third, … virial ­coefficients (Table 1C.1); gas from eqn 1C.3a
the first virial coefficient is 1. The third virial ­coefficient, C, is usu-
dZ
ally less important than the second ­coefficient, B, in the sense that  B  2 pC     B as p  0  (1C.4a)
dp
at typical molar volumes C /Vm2 << B /Vm . The values of the virial
However, B′ is not necessarily zero, so the slope of Z with re-
spect to p does not necessarily approach 0 (the perfect gas
Table 1C.1 Second virial coefficients, B/(cm3 mol−1)* value), as can be seen in Fig. 1C.4. By a similar argument
Temperature dZ
 B as Vm   (1C.4b)
273 K 600 K d(1/Vm )
Ar −21.7 11.9 Because the virial coefficients depend on the temperature,
CO2 −14.2 −12.4 there may be a temperature at which Z → 1 with zero slope
N2 −10.5 21.7 at low pressure or high molar volume (as in Fig. 1C.4). At
Xe −153.7 −19.6 this temperature, which is called the Boyle temperature, TB,
the properties of the real gas do coincide with those of a per-
* More values are given in the Resource section.
fect gas as p → 0. According to eqn 1C.4a, Z has zero slope as
1
The name comes from the Latin word for force. The coefficients are some- p → 0 if B′ = 0, so at the Boyle temperature B′ = 0. It then fol-
times denoted B2, B3, …. lows from eqn 1C.3a that pVm ≈ RTB over a more extended
1C Real gases 21

Higher 140
temperature
Compression factor, Z

50 °C
Boyle 100
temperature 40 °C
* 31.04 °C (Tc)

p/atm
Perfect gas 20 °C
60
E C

Lower
temperature 20

Pressure, p 0 0.2 0.4 0.6


Vm/(dm3 mol–1)
Figure 1C.4 The compression factor, Z, approaches 1 at low
pressures, but does so with different slopes. For a perfect gas, the Figure 1C.5 Experimental isotherms of carbon dioxide at several
slope is zero, but real gases may have either positive or negative temperatures. The ‘critical isotherm’, the isotherm at the critical
slopes, and the slope may vary with temperature. At the Boyle temperature, is at 31.04 °C. The critical point is marked with a
temperature, the slope is zero at p = 0 and the gas behaves perfectly star. As explained in the text, the gas can condense only
over a wider range of conditions than at other temperatures. at and below the critical temperature as it is compressed along
a horizontal line. The dotted curve consists of points (like
C and E for the 20 °C isotherm) for all isotherms below the critical
range of pressures than at other temperatures because the first temperature.
term after 1 (i.e. B′p) in the virial equation is zero and C′p2 and
higher terms are negligibly small. For helium TB = 22.64 K; for
air TB = 346.8 K; more values are given in Table 1C.2. as points C and E in Fig. 1C.5) get closer together; the locus of
such points for all isotherms below the critical temperature is
shown by the dotted line.
1C.1(c) Critical constants At Tc itself a surface separating two phases does not appear
Figure 1C.5 shows a set of experimental isotherms for CO2. As and the two volumes have merged to a single point, the critical
has already been discussed, at a sufficiently low temperature point of the gas. The pressure and molar volume at the criti-
(such as 20 °C) increasing the pressure will eventually lead to cal point are called the critical pressure, pc, and critical molar
condensation, as indicated by the horizontal line between C and volume, Vc, of the substance. Collectively, pc, Vc, and Tc are the
E. At higher temperatures (such as 50 °C), condensation does critical constants of a substance (Table 1C.2).
not take place, regardless of the pressure. There is a temperature, At and above Tc, the sample has a single phase which occu-
called the critical temperature, Tc, which is the highest tempera- pies the entire volume of the container. Such a phase is, by defi-
ture at which condensation can take place. For CO2 this tempera- nition, a gas. Hence, the liquid phase of a substance does not
ture is 31.04 °C, for which the isotherm is shown as a thicker line. form above the critical temperature. For example, because Tc
The critical temperature, which separates two regions of be- of oxygen is 155 K it follows that above this temperature it is
haviour, plays a special role in the theory of the states of mat- impossible to produce liquid oxygen by compression alone. To
ter. For an isotherm below Tc as the pressure increases there liquefy oxygen the temperature must first be lowered to below
comes a point at which a liquid condenses from the gas and is 155 K; then isothermal compression will cause liquefaction.
distinguishable from it by the presence of a visible surface. As The single phase that fills the entire volume when T > Tc
the temperature is increased the volumes corresponding to the may be much denser than considered typical for gases, and the
beginning and end of the horizontal part of the isotherm (such name supercritical fluid is preferred.

Table 1C.2 Critical constants of gases* Exercises


3 −1
pc /atm Vc /(cm mol ) Tc /K Zc TB/K E1C.1 A gas at 250 K and 15 atm has a molar volume 12 per cent smaller
than that calculated from the perfect gas law. Calculate (i) the compression
Ar 48.0 75.3 150.7 0.292 411.5
factor under these conditions and (ii) the molar volume of the gas. Which are
CO2 72.9 94.0 304.2 0.274 714.8 dominating in the sample, the attractive or the repulsive forces?
He 2.26 57.8 5.2 0.305 22.64 E1C.2 Use the virial equation of state (up to the B term) to calculate the
O2 50.14 78.0 154.8 0.308 405.9 pressure exerted at 273 K by 0.200 mol CO2(g) in a vessel of volume 250 cm3.
Take the value of B at this temperature as −149.7 cm3 mol−1. Compare this
* More values are given in the Resource section. pressure with that expected for a perfect gas.
22 1 The properties of gases

1C.2 The van der Waals equation Table 1C.3 van der Waals coefficients*

a/(atm dm6 mol−2) b/(10−2 dm3 mol−1)


Conclusions may be drawn from the virial equations of state
Ar 1.337 3.20
only by inserting specific values of the coefficients. It is often
CO2 3.610 4.29
useful to have a broader, if less precise, view of all gases, such
He 0.0341 2.38
as that provided by an approximate equation of state, and one
such equation is that due to van der Waals. Xe 4.137 5.16

* More values are given in the Resource section.

1C.2(a) Formulation of the equation The constants a and b are called the van der Waals coefficients,
The equation introduced by J.D. van der Waals in 1873 is an ex- with a representing the strength of attractive interactions and
cellent example of ‘model building’ in which an understanding b that of the repulsive interactions between the molecules.
of the molecular interactions and their consequences for the They are characteristic of each gas and taken to be independ-
pressure exerted by a gas is used to guide the formulation, in ent of the temperature (Table 1C.3). Although a and b are not
mathematical terms, of an equation of state. precisely defined molecular properties, they correlate with
physical properties that reflect the strength of intermolecular
interactions, such as critical temperature, vapour pressure, and
enthalpy of vaporization (Topic 2C).
How is that done? 1C.1 Deriving the van der Waals Equation 1C.5a is often written in terms of the molar volume
equation of state Vm = V/n as

The repulsive interaction between molecules is taken into RT a


p   (1C.5b)
account by supposing that it causes the molecules to behave Vm  b Vm2
as small but impenetrable spheres, so instead of moving in a
volume V they are restricted to a smaller volume V − nb, where
Brief illustration 1C.3
nb is approximately the total volume taken up by the molecules
themselves. This argument suggests that the perfect gas law For benzene a = 18.57 atm dm6 mol−2 (1.882 Pa m6 mol−2) and
p = nRT/V should be replaced by b = 0.1193 dm3 mol−1 (1.193 × 10−4 m3 mol−1); its normal boil-
nRT ing point is 353 K. Treated as a perfect gas at T = 400 K and
p
V  nb p = 1.0 atm, benzene vapour has a molar volume of Vm = RT/p =
33 dm3 mol−1. This molar volume is much greater than the
To calculate the excluded volume, note that the closest
van der Waals coefficient b, implying that the repulsive forces
distance of approach of two hard-sphere molecules of radius
have negligible effect. With this value of the molar volume
r (and volume Vmolecule  34 r 3) is 2r, so the volume excluded
the second term in the van der Waals equation is evaluated as
is 34 π(2r )3 , or 8Vmolecule . The volume excluded per molecule is
a/Vm2 ≈ 0.017 atm , which is 1.7 per cent of 1.0 atm. Therefore,
one-half this volume, or 4Vmolecule. In the proposed expression
the attractive forces only cause slight deviation from perfect gas
nb is the total excluded volume, so if there are N molecules
behaviour at this temperature and pressure.
in the sample each excluding a volume 4Vmolecule it follows that
nb  N  4Vmolecule . Because N = nN A this relation can be rear-
ranged to b ≈ 4Vmolecule N A. Example 1C.1
The pressure depends on both the frequency of collisions Using the van der Waals equation to
with the walls and the force of each collision. Both the fre- estimate a molar volume
quency of the collisions and their force are reduced by the Estimate the molar volume of CO2 at 500 K and 100 atm by
attractive interaction, which acts with a strength proportional treating it as a van der Waals gas.
to the number of interacting molecules and therefore to the
molar concentration, n/V, of molecules in the sample. Because Collect your thoughts You need to find an expression for the
both the frequency and the force of the collisions are reduced molar volume by solving the van der Waals equation, eqn 1C.5b.
by the attractive interactions, the pressure is reduced in pro- To rearrange the equation into a suitable form, multiply both
portion to the square of this concentration. If the reduction of sides by (Vm − b)Vm2 to obtain
pressure is written as a(n/V)2, where a is a positive constant (Vm  b)Vm2 p  RTVm2  (Vm  b)a
characteristic of each gas, the combined effect of the repulsive
and attractive forces gives the van der Waals equation: Then, after division by p, collect powers of Vm to obtain

nRT n2 (1C.5a)  RT  2  a  ab
p= −a 2 Vm3   b  Vm   Vm  0
V − nb V van der Waals equation of state
 p  p
  p
1C Real gases 23

6 Self-test 1C.1 Calculate the molar volume of argon at 100 °C


and 100 atm on the assumption that it is a van der Waals gas.
4 Answer: 0.298 dm3 mol−1

2
1000f(x)

0
1C.2(b) The features of the equation
–2 To what extent does the van der Waals equation predict the
behaviour of real gases? It is too optimistic to expect a single,
–4
simple expression to be the true equation of state of all sub-
–6 stances, and accurate work on gases must resort to the virial
0 0.1 0.2 x 0.3 0.4
equation, use tabulated values of the coefficients at various
Figure 1C.6 The graphical solution of the cubic equation for V in temperatures, and analyse the system numerically. The advan-
Example 1C.1. tage of the van der Waals equation, however, is that it is analyt-
ical (that is, expressed symbolically) and allows some general
conclusions about real gases to be drawn. When the equation
Although closed expressions for the roots of a cubic equation fails another equation of state must be used (some are listed in
can be given, they are very complicated. Unless analytical solu- Table 1C.4), yet another must be invented, or the virial equa-
tions are essential, it is usually best to solve such equations with tion is used.
mathematical software; graphing calculators can also be used The reliability of the equation can be judged by comparing
to help identify the acceptable root. the isotherms it predicts with the experimental isotherms in
Fig. 1C.2. Some calculated isotherms are shown in Figs. 1C.7
The solution According to Table 1C.3, a = 3.610 dm6 atm mol−2
and 1C.8. Apart from the oscillations below the critical tem-
and b = 4.29 × 10−2 dm3 mol−1. Under the stated conditions,
RT/p = 0.410 dm3 mol−1. The coefficients in the equation for Vm
perature, they do resemble experimental isotherms quite well.
are therefore The oscillations, the van der Waals loops, are unrealistic be-
cause they suggest that under some conditions an increase of
b  RT /p  0.453 dm3 mo11 pressure results in an increase of volume. Therefore they are
a/p  3.61  102 (dm 3 mo11 )2 replaced by horizontal lines drawn so the loops define equal
areas above and below the
ab/p  1.55  103 (dm3 mo11 )3
lines: this procedure is called
Equal areas
Therefore, on writing x = Vm/(dm3 mol−1), the equation to solve is the Maxwell construction
(1). The van der Waals coeffi-
x 3  0.453x 2  (3.61  102 )x  (1.55  103 )  0 cients, such as those in Table
The acceptable root is x = 0.366 (Fig. 1C.6), which implies that 1C.3, are found by fitting the
Vm = 0.366 dm3 mol−1. The molar volume of a perfect gas under calculated curves to the ex- 1
these conditions is 0.410 dm3 mol−1. perimental curves.

Table 1C.4 Selected equations of state*

Critical constants
Equation Reduced form*
pc Vc Tc
nRT
Perfect gas p=
V
nRT n2a 8Tr 3 a 8a
van der Waals p  pr   3b
V  nb V 2 3Vr  1 Vr2 27b2 27bR
1/ 2 1/ 2
nRT n2 a 8T 3 1  2aR  2  2a 
Berthelot p  pr   3b
V  nb TV 2 3Vr  1 TrVr2 12  3b3  3  3bR 

nRTe na /RTV Tr e2(11/TrVr ) a a


Dieterici p pr  2b
V  nb 2Vr  1 4 e 2b 2 4bR
nRT  nB(T ) n2C(T ) 
Virial p 1    
V  V V2 

* Reduced variables are defined as Xr = X/Xc with X = p, Vm, and T. Equations of state are sometimes expressed in terms of the molar volume, Vm = V/n.
24 1 The properties of gases

3. The critical constants are related to the van der Waals


1.5 coefficients.
For T < Tc, the calculated isotherms oscillate, and each one
Pressure, p

1.0 passes through a minimum followed by a maximum. These ex-


trema converge as T → Tc and coincide at T = Tc; at the critical
point the curve has a flat inflexion (2). From the properties of
curves, an inflexion of this
type occurs when both the
0.8 first and second derivatives
e, T
tur
Volume, V era are zero. Hence, the critical
mp
Te constants can be found by
calculating these derivatives
Figure 1C.7 The surface of possible states allowed by the van der
Waals equation. The curves drawn on the surface are isotherms, and setting them equal to 2
labelled with the value of T/Tc, and correspond to the isotherms in zero at the critical point:
Fig. 1C.8.
dp RT 2a
  0
dVm (Vm  b)2 Vm3
1.5 d2 p 2RT 6a
1.5 2
 3
 4 0
dVm (Vm  b) Vm
Reduced pressure, p/pc

1 1 The solutions of these two equations (and using eqn 1C.5b to


calculate pc from Vc and Tc; see Problem 1C.12) are

a 8a
0.5 =
Vc 3=
b pc Tc =  (1C.6)
27b2 27bR
0.8
These relations provide an alternative route to the determina-
0 tion of a and b from the values of the critical constants. They
0.1 1 10 can be tested by noting that the critical compression factor, Zc,
Reduced volume, Vm/Vc
is predicted to be
Figure 1C.8 Van der Waals isotherms at several values of T/Tc. The
van der Waals loops are normally replaced by horizontal straight pcVc
=
Zc = 3
 (1C.7)
lines. The critical isotherm is the isotherm for T/Tc = 1, and is RTc 8

shown as a thicker line.


for all gases that are described by the van der Waals equation
near the critical point. Table 1C.2 shows that although Zc is typ-
The principal features of the van der Waals equation can be ically slightly less than the value 83 = 0.375 predicted by the van
summarized as follows. der Waals equation, it is approximately constant (at 0.3) and the
discrepancy is reasonably small.
1. Perfect gas isotherms are obtained at high temperatures
and large molar volumes.
When the temperature is high, RT may be so large that the first
1C.2(c) The principle of corresponding states
term in eqn 1C.5b greatly exceeds the second. Furthermore, if An important general technique in science for comparing the
the molar volume is large in the sense Vm >> b, then the de- properties of objects is to choose a related fundamental prop-
nominator Vm − b ≈ Vm. Under these conditions, the equation erty of the same kind and to set up a relative scale on that basis.
reduces to p = RT/Vm, the perfect gas equation. The critical constants are characteristic properties of gases, so
it may be that a scale can be set up by using them as yardsticks
2. Liquids and gases coexist when the attractive and repul-
and to introduce the dimensionless reduced variables of a gas
sive effects are in balance.
by dividing the actual variable by the corresponding critical
The van der Waals loops occur when both terms in eqn 1C.5b constant:
have similar magnitudes. The first term arises from the kinetic
Vm p T Reduced variables
energy of the molecules and their repulsive interactions; the = Vr = pr Tr =  (1C.8)
Vc pc Tc [definition]
second represents the effect of the attractive interactions.
1C Real gases 25

1 The van der Waals equation sheds some light on the p


­ rinciple.
2.0 When eqn 1C.5b is expressed in terms of the reduced variables it
Compression factor, Z

0.8 becomes
RTrTc a
0.6 pr pc   2 2
1.2 Nitrogen VrVc  b Vr Vc
0.4 Methane Now express the critical constants in terms of a and b by using
1.0
Propane eqn 1C.6:
0.2
Ethene apr 8aTr /27b a
 
0
27b2 3bVr  b 9b2Vr2
0 1 2 3 4 5 6 7
Reduced pressure, p/pc and, after multiplying both sides by 27b2/a, reorganize it into
Figure 1C.9 The compression factors of four gases plotted using 8Tr 3
pr   2 (1C.9)
reduced variables. The curves are labelled with the reduced 3Vr  1 Vr
temperature Tr = T/Tc. The use of reduced variables organizes the
data on to single curves. This equation has the same form as the original, but the coeffi-
cients a and b, which differ from gas to gas, have disappeared.
It follows that if the isotherms are plotted in terms of the re-
If the reduced pressure of a gas is given, its actual pressure is duced variables (as done in fact in Fig. 1C.8 without draw-
calculated by using p = prpc, and likewise for the volume and ing attention to the fact), then the same curves are obtained
temperature. Van der Waals, who first tried this procedure, whatever the gas. This is precisely the content of the principle
hoped that gases confined to the same reduced volume, Vr, at of corresponding states, so the van der Waals equation is com-
the same reduced temperature, Tr, would exert the same re- patible with it.
duced pressure, pr. The hope was largely fulfilled (Fig. 1C.9). Looking for too much significance in this apparent triumph
The illustration shows the dependence of the compression fac- is mistaken, because other equations of state also accommo-
tor on the reduced pressure for a variety of gases at various re- date the principle. In fact, any equation of state (such as those
duced temperatures. The success of the procedure is strikingly in Table 1C.4) with two parameters playing the roles of a and b
clear: compare this graph with Fig. 1C.3, where similar data are can be manipulated into a reduced form. The observation that
plotted without using reduced variables. real gases obey the principle approximately amounts to saying
The observation that real gases at the same reduced volume that the effects of the attractive and repulsive interactions can
and reduced temperature exert the same reduced pressure is each be approximated in terms of a single parameter. The im-
called the principle of corresponding states. The principle is portance of the principle is then not so much its theoretical in-
only an approximation. It works best for gases composed of terpretation but the way that it enables the properties of a range
spherical molecules; it fails, sometimes badly, when the mol- of gases to be coordinated on to a single diagram (e.g. Fig. 1C.9
ecules are non-spherical or polar. instead of Fig. 1C.3).

Brief illustration 1C.4


Exercises
The critical constants of argon and carbon dioxide are given in E1C.3 Calculate the pressure exerted by 1.0 mol C2H6 behaving as a
Table 1C.2. Suppose argon is at 23 atm and 200 K, its reduced van der Waals gas when it is confined under the following conditions:
(i) at 273.15 K in 22.414 dm3, (ii) at 1000 K in 100 cm3. For C2H6 take
pressure and temperature are then
a = 5.507 atm dm6 mol−2 and b = 6.51 × 10−2 dm3 mol−1; it is convenient to use
23 atm 200 K R = 8.2057 × 10−2 dm3 atm K−1 mol−1.
=pr = 0.48= Tr = 1.33 3
48.0 atm 150.7 K E1C.4 Suppose that 10.0 mol C2H6(g) is confined to 4.860 dm at 27 °C.
Predict the pressure exerted by the ethane from (i) the perfect gas and (ii) the
For carbon dioxide to be in a corresponding state, its pressure van der Waals equations of state. Calculate the compression factor based on
and temperature would need to be these calculations. For ethane, a = 5.507 dm6 atm mol−2, b = 0.0651 dm3 mol−1;
it is convenient to use R = 8.2057 × 10−2 dm3 atm K−1 mol−1.
p  0.48  (72.9 atm)  35 atm 3 −1
E1C.5 The critical constants of methane are pc = 45.6 atm, Vc = 98.7 cm mol ,
T  1.33  304.2 K  405 K and Tc = 190.6 K. Calculate the van der Waals parameters of the gas. (Use
R = 8.2057 × 10−2 dm3 atm K−1 mol−1.)
26 1 The properties of gases

Checklist of concepts
☐ 1. The extent of deviations from perfect behaviour is sum- ☐ 5. The van der Waals equation is a model equation of state
marized by introducing the compression factor. for a real gas expressed in terms of two parameters,
☐ 2. The virial equation is an empirical extension of the per- one (a) representing molecular attractions and the other
fect gas equation that summarizes the behaviour of real (b) representing molecular repulsions.
gases over a range of conditions. ☐ 6. The van der Waals equation captures the general fea-
☐ 3. The isotherms of a real gas introduce the concept of tures of the behaviour of real gases, including their criti-
critical behaviour. cal behaviour.
☐ 4. A gas can be liquefied by pressure alone only if its tem- ☐ 7. The properties of real gases are coordinated by expressing
perature is at or below its critical temperature. their equations of state in terms of reduced variables.

Checklist of equations
Property Equation Comment Equation number
o
Compression factor Z = Vm /V m Definition 1C.1
2
Virial equation of state pVm  RT (1  B /Vm  C /V  )
m B, C depend on temperature 1C.3b
2
van der Waals equation of state p = nRT/(V − nb) − a(n/V) a parameterizes attractions, b parameterizes repulsions 1C.5a
Reduced variables Xr = X/Xc X = p, Vm, or T 1C.8
Exercises and problems 27

FOCUS 1 The properties of gases

To test your understanding of this material, work through Selected solutions can be found at the end of this Focus in
the Exercises, Additional exercises, Discussion questions, and the e-book. Solutions to even-numbered questions are available
Problems found throughout this Focus. online only to lecturers.

TOPIC 1A The perfect gas


Discussion questions
D1A.1 Explain how the perfect gas equation of state arises by combination of D1A.2 Explain the term ‘partial pressure’ and explain why Dalton’s law is a
Boyle’s law, Charles’s law, and Avogadro’s principle. limiting law.

Additional exercises
E1A.8 Express (i) 22.5 kPa in atmospheres and (ii) 770 Torr in pascals. 60 per cent. Hint: Relative humidity is the prevailing partial pressure of water
3 vapour expressed as a percentage of the vapour pressure of water vapour at
E1A.9 Could 25 g of argon gas in a vessel of volume 1.5 dm exert a pressure
the same temperature (in this case, 35.6 mbar).
of 2.0 bar at 30 °C if it behaved as a perfect gas? If not, what pressure would
it exert? E1A.18 Calculate the mass of water vapour present in a room of volume
250 m3 that contains air at 23 °C on a day when the relative humidity is
E1A.10 A perfect gas undergoes isothermal expansion, which increases its
53 per cent (in this case, 28.1 mbar).
volume by 2.20 dm3. The final pressure and volume of the gas are 5.04 bar
and 4.65 dm3, respectively. Calculate the original pressure of the gas in E1A.19 Given that the mass density of air at 0.987 bar and 27 °C is
(i) bar, (ii) atm. 1.146 kg m−3, calculate the mole fraction and partial pressure of nitrogen
and oxygen assuming that (i) air consists only of these two gases, (ii) air also
E1A.11 A perfect gas undergoes isothermal compression, which reduces its
contains 1.0 mole per cent Ar.
volume by 1.80 dm3. The final pressure and volume of the gas are 1.97 bar
and 2.14 dm3, respectively. Calculate the original pressure of the gas in E1A.20 A gas mixture consists of 320 mg of methane, 175 mg of argon, and
(i) bar, (ii) torr. 225 mg of neon. The partial pressure of neon at 300 K is 8.87 kPa. Calculate
−2 (i) the volume and (ii) the total pressure of the mixture.
E1A.12 A car tyre (an automobile tire) was inflated to a pressure of 24 lb in
(1.00 atm = 14.7 lb in−2) on a winter’s day when the temperature was −5 °C. E1A.21 The mass density of a gaseous compound was found to be 1.23 kg m
−3

What pressure will be found, assuming no leaks have occurred and that the at 330 K and 20 kPa. What is the molar mass of the compound?
volume is constant, on a subsequent summer’s day when the temperature is 3
E1A.22 In an experiment to measure the molar mass of a gas, 250 cm of the
35 °C? What complications should be taken into account in practice?
gas was confined in a glass vessel. The pressure was 152 Torr at 298 K, and
E1A.13 A sample of hydrogen gas was found to have a pressure of 125 kPa after correcting for buoyancy effects, the mass of the gas was 33.5 mg. What is
when the temperature was 23 °C. What can its pressure be expected to be the molar mass of the gas?
when the temperature is 11 °C? −3
E1A.23 The densities of air at −85 °C, 0 °C, and 100 °C are 1.877 g dm ,
3
E1A.14 A sample of 255 mg of neon occupies 3.00 dm at 122 K. Use the 1.294 g dm−3, and 0.946 g dm−3, respectively. From these data, and assuming
perfect gas law to calculate the pressure of the gas. that air obeys Charles’s law, determine a value for the absolute zero of
3 3 temperature in degrees Celsius.
E1A.15 A homeowner uses 4.00 × 10 m of natural gas in a year to heat a
3
home. Assume that natural gas is all methane, CH4, and that methane is a E1A.24 A certain sample of a gas has a volume of 20.00 dm at 0 °C and
perfect gas for the conditions of this problem, which are 1.00 atm and 20 °C. 1.000 atm. A plot of the experimental data of its volume against the Celsius
What is the mass of gas used? temperature, θ, at constant p, gives a straight line of slope 0.0741 dm3 °C−1.
From these data alone (without making use of the perfect gas law), determine
E1A.16 At 100 °C and 16.0 kPa, the mass density of phosphorus vapour is
the absolute zero of temperature in degrees Celsius.
0.6388 kg m−3. What is the molecular formula of phosphorus under these
3
conditions? E1A.25 A vessel of volume 22.4 dm contains 1.5 mol H2(g) and 2.5 mol N2(g)
at 273.15 K. Calculate (i) the mole fractions of each component, (ii) their
E1A.17 Calculate the mass of water vapour present in a room of volume
partial pressures, and (iii) their total pressure.
400 m3 that contains air at 27 °C on a day when the relative humidity is

Problems
P1A.1 A manometer consists of a U-shaped tube containing a liquid. One side pressure, ρ is the mass density of the liquid in the tube, g = 9.806 m s−2 is the
is connected to the apparatus and the other is open to the atmosphere. The acceleration of free fall, and h is the difference in heights of the liquid in the
pressure p inside the apparatus is given p = pex + ρgh, where pex is the external two sides of the tube. (The quantity ρgh is the hydrostatic pressure exerted by
28 1 The properties of gases

a column of liquid.) (i) Suppose the liquid in a manometer is mercury, the P1A.10 Ozone is a trace atmospheric gas which plays an important role
external pressure is 760 Torr, and the open side is 10.0 cm higher than the side in screening the Earth from harmful ultraviolet radiation, and the
connected to the apparatus. What is the pressure in the apparatus? The mass abundance of ozone is commonly reported in Dobson units. Imagine a
density of mercury at 25 °C is 13.55 g cm−3. (ii) In an attempt to determine column passing up through the atmosphere. The total amount of O3 in the
an accurate value of the gas constant, R, a student heated a container of column divided by its cross-sectional area is reported in Dobson units with
volume 20.000 dm3 filled with 0.251 32 g of helium gas to 500 °C and 1 Du = 0.4462 mmol m−2. What amount of O3 (in moles) is found in a column
measured the pressure as 206.402 cm in a manometer filled with water at of atmosphere with a cross-sectional area of 1.00 dm2 if the abundance is
25 °C. Calculate the value of R from these data. The mass density of water 250 Dobson units (a typical midlatitude value)? In the seasonal Antarctic
at 25 °C is 0.997 07 g cm−3. ozone hole, the column abundance drops below 100 Dobson units; how
many moles of O3 are found in such a column of air above a 1.00 dm2 area?
P1A.2 Recent communication with the inhabitants of Neptune has revealed
Most atmospheric ozone is found between 10 and 50 km above the surface
that they have a Celsius-type temperature scale, but based on the melting
of the Earth. If that ozone is spread uniformly through this portion of the
point (0 °N) and boiling point (100 °N) of their most common substance,
atmosphere, what is the average molar concentration corresponding to
hydrogen. Further communications have revealed that the Neptunians know
(a) 250 Dobson units, (b) 100 Dobson units?
about perfect gas behaviour and they find that in the limit of zero pressure,
the value of pV is 28 dm3 atm at 0 °N and 40 dm3 atm at 100 °N. What is the P1A.11‡ In a commonly used model of the atmosphere, the atmospheric
value of the absolute zero of temperature on their temperature scale? pressure varies with altitude, h, according to the barometric formula:
P1A.3 The following data have been obtained for oxygen gas at 273.15 K. From p  p0 e  h /H
the data, calculate the best value of the gas constant R.
where p0 is the pressure at sea level and H is a constant approximately equal to
p/atm 0.750 000 0.500 000 0.250 000
8 km. More specifically, H = RT/Mg, where M is the average molar mass of air
Vm/(dm3 mol−1) 29.8649 44.8090 89.6384 and T is the temperature at the altitude h. This formula represents the outcome
of the competition between the potential energy of the molecules in the gravi-
P1A.4 Charles’s law is sometimes expressed in the form V = V0(1 + αθ), where θ
tational field of the Earth and the stirring effects of thermal motion. Derive
is the Celsius temperature, α is a constant, and V0 is the volume of the sample this relation by showing that the change in pressure dp for an infinitesimal
at 0 °C. The following values for have been reported for nitrogen at 0 °C: change in altitude dh where the mass density is ρ is dp = −ρgdh. Remember
p/Torr 749.7 599.6 333.1 98.6 that ρ depends on the pressure. Evaluate (a) the pressure difference between
3 −1
the top and bottom of a laboratory vessel of height 15 cm, and (b) the external
10 α/°C 3.6717 3.6697 3.6665 3.6643 atmospheric pressure at a typical cruising altitude of an aircraft (11 km) when
For these data estimate the absolute zero of temperature on the Celsius scale. the pressure at ground level is 1.0 atm.

P1A.5 Deduce the relation between the pressure and mass density, ρ, of a P1A.12‡ Balloons are still used to deploy sensors that monitor meteorological
perfect gas of molar mass M. Confirm graphically, using the following data on phenomena and the chemistry of the atmosphere. It is possible to investigate
methoxymethane (dimethyl ether) at 25 °C, that perfect behaviour is reached some of the technicalities of ballooning by using the perfect gas law. Suppose
at low pressures and find the molar mass of the gas. your balloon has a radius of 3.0 m and that it is spherical. (a) What amount of
H2 (in moles) is needed to inflate it to 1.0 atm in an ambient temperature of
p/kPa 12.223 25.20 36.97 60.37 85.23 101.3 25 °C at sea level? (b) What mass can the balloon lift (the payload) at sea level,
ρ/(kg m−3) 0.225 0.456 0.664 1.062 1.468 1.734 where the mass density of air is 1.22 kg m−3? (c) What would be the payload if
He were used instead of H2?
P1A.6 The molar mass of a newly synthesized fluorocarbon was measured
in a gas microbalance. This device consists of a glass bulb forming one P1A.13‡ Chlorofluorocarbons such as CCl3F and CCl2F2 have been linked to
end of a beam, the whole surrounded by a closed container. The beam is ozone depletion in Antarctica. In 1994, these gases were found in quantities
pivoted, and the balance point is attained by raising the pressure of gas of 261 and 509 parts per trillion by volume (World Resources Institute, World
in the container, so increasing the buoyancy of the enclosed bulb. In one resources 1996–97). Compute the molar concentration of these gases under
experiment, the balance point was reached when the fluorocarbon pressure conditions typical of (a) the mid-latitude troposphere (10 °C and 1.0 atm) and
was 327.10 Torr; for the same setting of the pivot, a balance was reached (b) the Antarctic stratosphere (200 K and 0.050 atm). Hint: The composition of
when CHF3 (M = 70.014 g mol−1) was introduced at 423.22 Torr. A repeat a mixture of gases can be described by imagining that the gases are separated
of the experiment with a different setting of the pivot required a pressure of from one another in such a way that each exerts the same pressure. If one gas
293.22 Torr of the fluorocarbon and 427.22 Torr of the CHF3. What is the is present at very low levels it is common to express its concentration as, for
molar mass of the fluorocarbon? Suggest a molecular formula. example, ‘x parts per trillion by volume’. Then the volume of the separated
gas at a certain pressure is x × 10−12 of the original volume of the gas mixture
P1A.7 A constant-volume perfect gas thermometer indicates a pressure
at the same pressure. For a mixture of perfect gases, the volume of each
of 6.69 kPa at the triple point temperature of water (273.16 K). (a) What
separated gas is proportional to its partial pressure in the mixture and hence
change of pressure indicates a change of 1.00 K at this temperature? (b) What
to the amount in moles of the gas molecules present in the mixture.
pressure indicates a temperature of 100.00 °C? (c) What change of pressure
indicates a change of 1.00 K at the latter temperature? P1A.14 At sea level the composition of the atmosphere is approximately 80
3 per cent nitrogen and 20 per cent oxygen by mass. At what height above the
P1A.8 A vessel of volume 22.4 dm contains 2.0 mol H2(g) and 1.0 mol N2(g)
surface of the Earth would the atmosphere become 90 per cent nitrogen and
at 273.15 K initially. All the H2 then reacts with sufficient N2 to form NH3.
10 per cent oxygen by mass? Assume that the temperature of the atmosphere
Calculate the partial pressures of the gases in the final mixture and the total
is constant at 25 °C. What is the pressure of the atmosphere at that height?
pressure.
Hint: The atmospheric pressure varies with altitude, h, according to the
P1A.9 Atmospheric pollution is a problem that has received much attention. barometric formula, p  p0 e  h /H , where p0 is the pressure at sea level and H is
Not all pollution, however, is from industrial sources. Volcanic eruptions can a constant approximately equal to 8 km. Use a barometric formula for each
be a significant source of air pollution. The Kilauea volcano in Hawaii emits partial pressure.
200−300 t (1 t = 103 kg) of SO2 each day. If this gas is emitted at 800 °C and

1.0 atm, what volume of gas is emitted? These problems were supplied by Charles Trapp and Carmen Giunta.
Exercises and problems 29

TOPIC 1B The kinetic model


Discussion questions
D1B.1 Specify and analyse critically the assumptions that underlie the kinetic D1B.3 Use the kinetic model of gases to explain why light gases, such as He,
model of gases. are rare in the Earth’s atmosphere but heavier gases, such as O2, CO2, and N2,
once formed remain abundant.
D1B.2 Provide molecular interpretations for the dependencies of the mean
free path on the temperature, pressure, and size of gas molecules.

Additional exercises
E1B.7 Determine the ratios of (i) the mean speeds, (ii) the mean translational E1B.13 Calculate the most probable speed, the mean speed, and the mean
kinetic energies of H2 molecules and Hg atoms at 20 °C. relative speed of H2 molecules at 20 °C.
E1B.8 Determine the ratios of (i) the mean speeds, (ii) the mean translational E1B.14 Evaluate the collision frequency of H2 molecules in a gas at 1.00 atm
kinetic energies of He atoms and Hg atoms at 25 °C. and 25 °C.
E1B.9 Calculate the root-mean-square speeds of CO2 molecules and He atoms E1B.15 Evaluate the collision frequency of O2 molecules in a gas at 1.00 atm
at 20 °C. and 25 °C.
E1B.10 Use the Maxwell–Boltzmann distribution of speeds to estimate the E1B.16 The best laboratory vacuum pump can generate a vacuum of about
fraction of N2 molecules at 400 K that have speeds in the range 200–210 m s−1. 1 nTorr. At 25 °C and assuming that air consists of N2 molecules with a
Hint: The fraction of molecules with speeds in the range v to v + dv is equal to collision diameter of 395 pm, calculate at this pressure (i) the mean speed of
f(v)dv, where f(v) is given by eqn 1B.4. the molecules, (ii) the mean free path, (iii) the collision frequency in the gas.
E1B.11 Use the Maxwell–Boltzmann distribution of speeds to estimate the E1B.17 At what pressure does the mean free path of argon at 20 °C become
fraction of CO2 molecules at 400 K that have speeds in the range 400–405 m s−1. comparable to 10 times the diameters of the atoms themselves? Take
See the hint in Exercise E1B.10. σ = 0.36 nm2.
E1B.12 What is the relative mean speed of O2 and N2 molecules in a gas at E1B.18 At an altitude of 15 km the temperature is 217 K and the pressure
25 °C? is 12.1 kPa. What is the mean free path of N2 molecules? (σ = 0.43 nm2).

Problems
P1B.1 A rotating slotted-disc apparatus consists of five coaxial 5.0 cm diameter This calculation can be used to estimate the fraction of very energetic
discs separated by 1.0 cm, the radial slots being displaced by 2.0° between molecules (which is important for reactions). Evaluate the ratio for n = 3
neighbours. The relative intensities, I, of the detected beam of Kr atoms for and n = 4.
two different temperatures and at a series of rotation rates were as follows: n 1/n
P1B.6 Derive an expression for  v  from the Maxwell–Boltzmann
n/Hz 20 40 80 100 120 distribution of speeds. Hint: You will need the integrals given in the Resource
section, or use mathematical software.
I (40 K) 0.846 0.513 0.069 0.015 0.002
P1B.7 Calculate the escape velocity (the minimum initial velocity that will
I (100 K) 0.592 0.485 0.217 0.119 0.057
take an object to infinity) from the surface of a planet of radius R. What
Find the distributions of molecular velocities, f(vx), at these temperatures, and is the value for (i) the Earth, R = 6.37 × 106 m, g = 9.81 m s−2, (ii) Mars,
check that they conform to the theoretical prediction for a one-­dimensional R = 3.38 × 106 m, mMars/mEarth = 0.108. At what temperatures do H2, He,
system for this low-pressure, collision-free system. and O2 molecules have mean speeds equal to their escape speeds? What
proportion of the molecules have enough speed to escape when the
P1B.2 Consider molecules that are confined to move in a plane (a two-
temperature is (i) 240 K, (ii) 1500 K? Calculations of this kind are very
dimensional gas). Calculate the distribution of speeds and determine the important in considering the composition of planetary atmospheres.
mean speed of the molecules at a temperature T.
P1B.8 Plot different Maxwell–Boltzmann speed distributions by keeping the
P1B.3 A specially constructed velocity selector accepts a beam of molecules
molar mass constant at 100 g mol−1 and varying the temperature of the sample
from an oven at a temperature T but blocks the passage of molecules with a
between 200 K and 2000 K.
speed greater than the mean. What is the mean speed of the emerging beam,
relative to the initial value? Treat the system as one-dimensional. P1B.9 Evaluate numerically the fraction of O2 molecules with speeds in the
range 100 m s−1 to 200 m s−1 in a gas at 300 K and 1000 K.
P1B.4 What, according to the Maxwell–Boltzmann distribution, is the
proportion of gas molecules having (i) more than, (ii) less than the root- P1B.10 The maximum in the Maxwell–Boltzmann distribution occurs when
mean-square speed? (iii) What are the proportions having speeds greater df(v)/dv = 0. Find, by differentiation, an expression for the most probable
and smaller than the mean speed? Hint: Use mathematical software to speed of molecules of molar mass M at a temperature T.
evaluate the integrals.
P1B.11 A methane, CH4, molecule may be considered as spherical, with a
P1B.5 Calculate the fractions of molecules in a gas that have a speed in a radius of 0.38 nm. How many collisions does a single methane molecule
range Δv at the speed nvmp relative to those in the same range at vmp itself. make if 0.10 mol CH4(g) is held at 25 °C in a vessel of volume 1.0 dm3?
30 1 The properties of gases

TOPIC 1C Real gases


Discussion questions
D1C.1 Explain how the compression factor varies with pressure and D1C.3 Describe the formulation of the van der Waals equation and suggest a
temperature and describe how it reveals information about intermolecular rationale for one other equation of state in Table 1C.4.
interactions in real gases.
D1C.4 Explain how the van der Waals equation accounts for critical behaviour.
D1C.2 What is the significance of the critical constants?

Additional exercises
3 −1
E1C.6 Calculate the pressure exerted by 1.0 mol H2S behaving as a van der E1C.14 The critical constants of ethane are pc = 48.20 atm, Vc = 148 cm mol ,
Waals gas when it is confined under the following conditions: (i) at 273.15 K and Tc = 305.4 K. Calculate the van der Waals parameters of the gas and
in 22.414 dm3, (ii) at 500 K in 150 cm3. Use the data in Table 1C.3 of the estimate the radius of the molecules.
Resource section.
E1C.15 Use the van der Waals parameters for chlorine in Table 1C.3 of the
6 −2
E1C.7 Express the van der Waals parameters a = 0.751 atm dm mol and Resource section to calculate approximate values of (i) the Boyle temperature
b = 0.0226 dm3 mol−1 in SI base units (kg, m, s, and mol). of chlorine from TB = a/Rb and (ii) the radius of a Cl2 molecule regarded as a
6 −2 sphere.
E1C.8 Express the van der Waals parameters a = 1.32 atm dm mol and
3 −1
b = 0.0436 dm mol in SI base units (kg, m, s, and mol). E1C.16 Use the van der Waals parameters for hydrogen sulfide in Table 1C.3
of the Resource section to calculate approximate values of (i) the Boyle
E1C.9 A gas at 350 K and 12 atm has a molar volume 12 per cent larger than
temperature of the gas from TB = a/Rb and (ii) the radius of an H2S molecule
that calculated from the perfect gas law. Calculate (i) the compression factor
regarded as a sphere.
under these conditions and (ii) the molar volume of the gas. Which are
dominating in the sample, the attractive or the repulsive forces? E1C.17 Suggest the pressure and temperature at which 1.0 mol of (i) NH3,
(ii) Xe, (iii) He will be in states that correspond to 1.0 mol H2 at 1.0 atm
E1C.10 In an industrial process, nitrogen is heated to 500 K at a constant volume
and 25 °C.
of 1.000 m3. The mass of the gas is 92.4 kg. Use the van der Waals equation to
determine the approximate pressure of the gas at its working temperature of E1C.18 Suggest the pressure and temperature at which 1.0 mol of (i) H2O
500 K. For nitrogen, a = 1.352 dm6 atm mol−2, b = 0.0387 dm3 mol−1. (ii) CO2, (iii) Ar will be in states that correspond to 1.0 mol N2 at 1.0 atm
and 25 °C.
E1C.11 Cylinders of compressed gas are typically filled to a pressure of 200 bar.
6 −2
For oxygen, what would be the molar volume at this pressure and 25 °C based E1C.19 A gas obeys the van der Waals equation with a = 0.50 m Pa mol .
on (i) the perfect gas equation, (ii) the van der Waals equation? For oxygen, Its molar volume is found to be 5.00 × 10−4 m3 mol−1 at 273 K and 3.0 MPa.
a = 1.364 dm6 atm mol−2, b = 3.19 × 10−2 dm3 mol−1. From this information calculate the van der Waals constant b. What is the
compression factor for this gas at the prevailing temperature and pressure?
E1C.12 At 300 K and 20 atm, the compression factor of a gas is 0.86. Calculate (i)
6 −2
the volume occupied by 8.2 mmol of the gas molecules under these conditions E1C.20 A gas obeys the van der Waals equation with a = 0.76 m Pa mol .
and (ii) an approximate value of the second virial coefficient B at 300 K. Its molar volume is found to be 4.00 × 10−4 m3 mol−1 at 288 K and 4.0 MPa.
3 −1 From this information calculate the van der Waals constant b. What is the
E1C.13 The critical constants of methane are pc = 45.6 atm, Vc = 98.7 cm mol ,
compression factor for this gas at the prevailing temperature and pressure?
and Tc = 190.6 K. Calculate the van der Waals parameters of the gas and
estimate the radius of the molecules.

Problems
P1C.1 What pressure would 4.56 g of nitrogen gas in a vessel of volume critical temperature, (b) its Boyle temperature. Assume that the pressure is
2.25 dm3 exert at 273 K if it obeyed the virial equation of state up to and 10 atm throughout. At what temperature is the behaviour of the gas closest
including the first two terms? to that of a perfect gas? Use the following data: Tc = 126.3 K, TB = 327.2 K,
a = 1.390 dm6 atm mol−2, b = 0.0391 dm3 mol−1.
P1C.2 Calculate the molar volume of chlorine gas at 350 K and 2.30 atm
using (a) the perfect gas law and (b) the van der Waals equation. Use the P1C.5‡ The second virial coefficient of methane can be approximated by the
2
answer to (a) to calculate a first approximation to the correction term for empirical equation B(T )  a  b  c /T , where a = −0.1993 bar−1, b = 0.2002 bar−1,
attraction and then use successive approximations to obtain a numerical and c = 1131 K2 with 300 K < T < 600 K. What is the Boyle temperature of
answer for part (b). methane?
3 −1
P1C.3 At 273 K measurements on argon gave B = −21.7 cm mol and P1C.6 How well does argon gas at 400 K and 3 atm approximate a perfect
6 −2
C = 1200 cm mol , where B and C are the second and third virial coefficients gas? Assess the approximation by reporting the difference between the molar
in the expansion of Z in powers of 1/Vm. Assuming that the perfect gas law volumes as a percentage of the perfect gas molar volume.
holds sufficiently well for the estimation of the molar volume, calculate the −3
P1C.7 The mass density of water vapour at 327.6 atm and 776.4 K is 133.2 kg m .
compression factor of argon at 100 atm and 273 K. From your result, estimate
Given that for water a = 5.464 dm6 atm mol−2, b = 0.03049 dm3 mol−1, and
the molar volume of argon under these conditions.
M = 18.02 g mol−1, calculate (a) the molar volume. Then calculate the
P1C.4 Calculate the volume occupied by 1.00 mol N2 using the van der compression factor (b) from the data, and (c) from the virial expansion of
Waals equation expanded into the form of a virial expansion at (a) its the van der Waals equation.
Exercises and problems 31

P1C.8 The critical volume and critical pressure of a certain gas are P1C.16 The equation of state of a certain gas is given by
160 cm3 mol−1 and 40 atm, respectively. Estimate the critical temperature p  RT /Vm  (a  bT )/Vm2 , where a and b are constants. Find (∂Vm/∂T)p.
by assuming that the gas obeys the Berthelot equation of state. Estimate the
P1C.17 Under what conditions can liquid nitrogen be formed by the
radii of the gas molecules on the assumption that they are spheres.
application of pressure alone?
P1C.9 Estimate the coefficients a and b in the Dieterici equation of state from
P1C.18 The following equations of state are occasionally used for approximate
the critical constants of xenon. Calculate the pressure exerted by 1.0 mol Xe
calculations on gases: (gas A) pVm = RT(1 + b/Vm), (gas B) p(Vm − b) = RT.
when it is confined to 1.0 dm3 at 25 °C.
Assuming that there were gases that actually obeyed these equations of state,
P1C.10 For a van der Waals gas with given values of a and b, identify the would it be possible to liquefy either gas A or B? Would they have a critical
conditions for which Z < 1 and Z > 1. temperature? Explain your answer.
P1C.11 Express the van der Waals equation of state as a virial expansion P1C.19 Derive an expression for the compression factor of a gas that obeys the
in powers of 1/Vm and obtain expressions for B and C in terms of the equation of state p(V − nb) = nRT, where b and R are constants. If the pressure
parameters a and b. The expansion you will need is (1  x )1  1  x  x 2   . and temperature are such that Vm = 10b, what is the numerical value of the
Measurements on argon gave B = −21.7 cm3 mol−1 and C = 1200 cm6 mol−2 compression factor?
for the virial coefficients at 273 K. What are the values of a and b in the
P1C.20 What would be the corresponding state of ammonia, for the conditions
corresponding van der Waals equation of state?
described for argon in Brief illustration 1C.5?
P1C.12 The critical constants of a van der Waals gas can be found by setting
P1C.21‡ Stewart and Jacobsen have published a review of thermodynamic
the following derivatives equal to zero at the critical point:
properties of argon (R.B. Stewart and R.T. Jacobsen, J. Phys. Chem. Ref. Data
dp RT 2a 18, 639 (1989)) which included the following 300 K isotherm.
  0
dVm (Vm  b)2 Vm3
d2 p 2RT 6a p/MPa 0.4000 0.5000 0.6000 0.8000 1.000
  0
dVm2 (Vm  b)3 Vm4 Vm/(dm3 mol−1) 6.2208 4.9736 4.1423 3.1031 2.4795
Solve this system of equations and then use eqn 1C.5b to show that pc, Vc, and p/MPa 1.500 2.000 2.500 3.000 4.000
Tc are given by eqn 1C.6. Vm/(dm3 mol−1) 1.6483 1.2328 0.983 57 0.817 46 0.609 98
P1C.13 A scientist proposed the following equation of state: (a) Compute the second virial coefficient, B, at this temperature. (b) Use non-
RT B C linear curve-fitting software to compute the third virial coefficient, C, at this
p  
Vm Vm2 Vm3 temperature.
Show that the equation leads to critical behaviour. Find the critical constants P1C.22 Use the van der Waals equation of state and mathematical software
of the gas in terms of B and C and an expression for the critical compression or a spreadsheet to plot the pressure of 1.5 mol CO2(g) against volume as it
factor. is compressed from 30 dm3 to 15 dm3 at (a) 273 K, (b) 373 K. (c) Redraw the
graphs as plots of p against 1/V.
P1C.14 Equations 1C.3a and 1C.3b are expansions in p and 1/Vm, respectively.
Find the relation between B, C and B′, C′. P1C.23 Calculate the molar volume of chlorine on the basis of the van der
Waals equation of state at 250 K and 150 kPa and calculate the percentage
P1C.15 The second virial coefficient B′ can be obtained from measurements
difference from the value predicted by the perfect gas equation.
of the mass density ρ of a gas at a series of pressures. Show that the graph of
p/ρ against p should be a straight line with slope proportional to B′. Use the P1C.24 Is there a set of conditions at which the compression factor of a van
data on methoxymethane in Problem P1A.5 to find the values of B′ and B der Waals gas passes through a minimum? If so, how does the location and
at 25 °C. value of the minimum value of Z depend on the coefficients a and b?

FOCUS 1 The properties of gases


Integrated activities
I1.1 Start from the Maxwell–Boltzmann distribution and derive an kinetic energy density of the atmosphere is 0.15 J cm−3, what is the total
expression for the most probable speed of a gas of molecules at a temperature kinetic energy density, including rotation?
T. Go on to demonstrate the validity of the equipartition conclusion that
I1.3 Methane molecules, CH4, may be considered as spherical, with a
the average translational kinetic energy of molecules free to move in three
collision cross-section of σ = 0.46 nm2. Estimate the value of the van der
dimensions is 23 kT .
Waals parameter b by calculating the molar volume excluded by methane
I1.2 The principal components of the atmosphere of the Earth are diatomic molecules.
molecules, which can rotate as well as translate. Given that the translational
FOCUS 2
The First Law

The release of energy can be used to provide heat when a fuel conditions. This Topic describes methods for determining and
burns in a furnace, to do mechanical work when a fuel burns in reporting the enthalpy changes associated with both physical
an engine, and to generate electrical work when a chemical re- and chemical changes.
action pumps electrons through a circuit. The energy released 2C.1 Standard enthalpy changes; 2C.2 Standard enthalpies of
in chemical reactions can be harnessed to create new sub- formation; 2C.3 The temperature dependence of reaction enthalpies;
2C.4 Experimental techniques
stances in the laboratory and to drive the processes of industry
and life. Thermodynamics is the study of these applications of
energy and is used to make quantitative predictions about the
oucomes of physical, chemical, and biological processes.
2D State functions and exact
differentials
2A Internal energy One very useful aspect of thermodynamics is that a property
can be measured indirectly by measuring others and then com-
This Topic examines how heat and work contribute to the ex- bining their values. The relations derived in this Topic also
change of energy between the system and its surroundings. That apply to the discussion of the liquefaction of gases and to the
discussion leads to the definition of the ‘internal energy’ of a sys- relation between the heat capacities of a substance under differ-
tem and to the formulation of the ‘First Law’ of thermodynamics. ent conditions.
2A.1 Work, heat, and energy; 2A.2 The definition of internal energy; 2D.1 Exact and inexact differentials; 2D.2 Changes in internal energy;
2A.3 Expansion work; 2A.4 Heat transactions 2D.3 Changes in enthalpy; 2D.4 The Joule–Thomson effect

2B Enthalpy 2E Adiabatic changes


The second major concept of this Focus is ‘enthalpy’. This prop- ‘Adiabatic’ processes are processes that occur without the trans-
erty is used to track the heat output (or requirements) of physi- fer of energy as heat. This Topic describes reversible adiabatic
cal processes and chemical reactions taking place at constant changes involving perfect gases. They figure prominently in the
pressure. Changes in internal energy or enthalpy are measured presentation of many aspects of thermodynamics.
by techniques known collectively as ‘calorimetry’. 2E.1 The change in temperature; 2E.2 The change in pressure
2B.1 The definition of enthalpy; 2B.2 The variation of enthalpy with
temperature
What is an application of this material?
2C Thermochemistry A major application of thermodynamics is to the assessment of
fuels and their equivalent for organisms, food. Some thermo-
‘Thermochemistry’ is the study of energy released or absorbed chemical aspects of fuels and foods are described in Impact 3,
as heat during chemical reactions and how it depends upon the accessed via the e-book.

➤ Go to the e-book for videos that feature the derivation and interpretation of equations, and applications of this material.
TOPIC 2A Internal energy

A closed system has a boundary through which matter


➤ Why do you need to know this material? cannot be transferred.
The First Law of thermodynamics is the foundation of the Both open and closed systems can exchange energy with their
discussion of the role of energy in chemistry. Wherever the surroundings.
generation or use of energy in physical transformations or
An isolated system can exchange neither energy nor
chemical reactions is of interest, lying in the background
matter with its surroundings.
are the concepts introduced by the First Law.

➤ What is the key idea?


The total energy of an isolated system is constant.
2A.1 Work, heat, and energy
➤ What do you need to know already?
Although thermodynamics deals with the properties of bulk
This Topic makes use of the discussion of the properties of
systems, it is enriched by understanding the molecular origins
gases (Topic 1A), particularly the perfect gas law. It builds
of these properties. What follows are descriptions of work, heat,
on the definition of work given in Energy: A first look.
and energy from both points of view.

2A.1(a) Definitions
In thermodynamics, the universe is divided into two parts: The fundamental physical property in thermodynamics is
the system and its surroundings. The system is the part of the work: work is done in order to achieve motion against an op-
world of interest. It may be a reaction vessel, an engine, an elec- posing force (Energy: A first look 2a). A simple example is the
trochemical cell, a biological cell, and so on. The ­surroundings process of raising a weight against the pull of gravity. A process
comprise the region outside the system. Measurements are does work if in principle it can be harnessed to raise a weight
made in the surroundings. The type of system depends on the somewhere in the surroundings. An example is the expansion
characteristics of the boundary that divides it from the sur- of a gas that pushes out a piston: the motion of the piston can in
roundings (Fig. 2A.1): principle be used to raise a weight. Another example is a chemi-
cal reaction in a battery: the reaction generates an electric cur-
An open system has a boundary through which matter
rent that can drive a motor and be used to raise a weight.
can be transferred.
The energy of a system is its capacity to do work (Energy: A
first look 2b). When work is done on an otherwise isolated sys-
tem (for instance, by compressing a gas with a piston or wind-
ing a spring), the capacity of the system to do work is increased.
Energy

Energy
Matter

That is, the energy of the system is increased. When the system
does work (when the piston moves out or the spring unwinds),
it can do less work than before. That is, its energy is decreased.
When a gas is compressed by a piston, work is done on the sys-
tem and its energy is increased. When that gas is allowed to ex-
Open Closed Isolated
pand again the piston moves out, work is done by the system,
and the energy of the system is decreased.
(a) (b) (c) It is very important to note that when the energy of the sys-
Figure 2A.1 (a) An open system can exchange matter and energy tem increases that of the surroundings decreases by exactly the
with its surroundings. (b) A closed system can exchange energy same amount, and vice versa. Thus, the weight raised when the
with its surroundings, but it cannot exchange matter. (c) An system does work has more energy than before the expansion,
isolated system can exchange neither energy nor matter with its because a raised weight can do more work than a lowered one.
surroundings. The weight lowered when work is done on the system has less
2A Internal energy 35

energy than before the compression. In other words, energy is


transferred between the system and the surroundings when work
is done. Work, in other words, is not a form of energy but a way
of transferring energy.
Experiments have shown that the energy of a system may be
changed by means other than work alone. When the energy of a

Heat
system changes as a result of a temperature difference between

Heat
the system and its surroundings the energy is said to be trans-
ferred as heat. Note that, like work, heat is a way of transferring
energy between the system and its surroundings. Endothermic Exothermic Endothermic Exothermic
Not all boundaries permit the transfer of energy even though process process process process
there is a temperature difference between the system and its
(a) (b) (c) (d)
surroundings. Boundaries that do permit the transfer of en-
ergy as heat are called diathermic; those that do not are called Figure 2A.2 (a) When an endothermic process occurs in an
­adiabatic. adiabatic system, the temperature falls; (b) if the process is
An exothermic process is a process that releases energy as exothermic, then the temperature rises; (c) When an endothermic
heat. An example is the combustion of methane gas, CH4(g), process occurs in a diathermic container, energy enters as heat
written as: from the surroundings; (d) if the process is exothermic, then
energy leaves as heat.
CH 4 (g )  2 O2 (g )  CO2 (g )  2 H2 O(l)
All combustions are exothermic. An endothermic process is a
process in which energy is absorbed as heat. An example is the
vaporization of water.

Surroundings
The final temperature of a system releasing or acquiring
energy as heat depends on the type of container in which the
process occurs. Consider an exothermic process, such as a
combustion. Although the temperature of the system rises in
the course of the combustion, given enough time, a system in
a diathermic vessel returns to the temperature of its surround-
Energy

Energy

Energy
ings. If the combustion takes place in an adiabatic container,
System

the energy released remains inside the container and results in


a permanent rise in temperature. Now consider an endother-
mic process. In a diathermic container the result is a flow of
energy into the system as heat to restore the temperature to that Figure 2A.3 When energy is transferred to the surroundings as
of the surroundings. When an endothermic process takes place heat, the transfer stimulates random motion of the atoms in the
in an adiabatic container, it results in a lowering of temperature surroundings.
of the system. These features are summarized in Fig. 2A.2. If the
surroundings are much larger than the system, then energy will
flow as heat into or out of a diathermic container so as to keep s­urroundings, molecules of the system stimulate the thermal
it at the same temperature as the surroundings. As a result any motion of the molecules in the surroundings (Fig. 2A.3).
process in such a container is isothermal, meaning that it oc- In contrast, work is the transfer of energy that makes use
curs at a fixed temperature. of organized motion in the surroundings (Fig. 2A.4). When
a weight is raised or lowered, its atoms move in an organized
The molecular interpretation of heat
2A.1(b) way (up or down). When a system does work it causes atoms in
its surroundings to move in an organized way. Likewise, when
and work work is done on a system, atoms in the surroundings are used
In molecular terms, heating is the transfer of energy that to transfer energy to it in an organized way, as the atoms in a
makes use of disorderly, apparently random, molecular mo- weight are lowered.
tion in the surroundings. The disorderly motion of molecules The distinction between work and heat is made in the sur-
is called thermal motion. The thermal motion of the mole- roundings. The fact that a falling object may stimulate thermal
cules in the hot surroundings stimulates the molecules in the motion in the system is irrelevant to the distinction between
cooler system to move more vigorously and, as a result, the en- heat and work: work is identified as energy transfer making use
ergy of the cooler system is increased. When a system heats its of the organized motion of atoms in the surroundings, and heat
36 2 The First Law

The internal energy is an extensive property of a system,


which means that its value depends on the amount of substance
present (see Topic 1A). Internal energy is measured in joules
Surroundings
(1 J  1 kgm 2 s 2). The molar internal energy, U m , is the inter-
nal energy divided by the amount of substance in a system,
U m = U /n. It is an intensive property, meaning that it is inde-
pendent of the amount of substance present in the system. The
molar internal energy is commonly reported in kilojoules per
Energy

Energy

Energy
mole (kJmol −1).
System

2A.2(a)Molecular interpretation
of internal energy
Figure 2A.4 When a system does work, it stimulates orderly
A molecule has a certain number of motional degrees of free-
motion in the surroundings. For instance, the atoms shown here
dom, such as the ability to move through space (this motion
may be part of a weight that is being raised.
is called ‘translation’), to rotate, and to vibrate. The internal
energy of a system has contributions from each of these kinds
is identified as energy transfer making use of thermal motion of motion. It also has contributions from the potential energy
in the surroundings. In the compression of a gas in an adiaba- that arise from the interactions between the molecules of the
tic enclosure, for instance, work is done on the system as the system. As the temperature is raised, the internal energy of the
atoms of an incoming piston descend in an orderly way, but its sample increases and the molecules populate higher energy
effect is to accelerate the gas molecules to higher average speeds. states associated with these contributions.
Because collisions between molecules quickly randomize their For a perfect gas, in which the molecules do not interact, the
directions, the orderly motion of the atoms of the piston is in equipartition theorem (described in Energy: A first look 3b) can
effect stimulating thermal motion in the gas. The piston is ob- be used to predict the contributions of each mode of motion to
served to fall, leading to the orderly descent of its atoms, and the internal energy.
work is done even though it is stimulating thermal motion.
Brief illustration 2A.1

An atom in a gas can move in three dimensions, so its transla-


2A.2 The definition of internal energy tional kinetic energy is the sum of three quadratic contributions:
Etrans  12 mvx2  12 mv 2y  12 mvz2
In thermodynamics, the total energy of a system is called its
internal energy, U. The internal energy is the total kinetic and The equipartition theorem predicts that the average energy for
potential energy of the constituents (the atoms, ions, or mole- each of these quadratic contributions is 12 kT . Thus, the average
cules) of the system. It does not include the kinetic energy aris- kinetic energy is Etrans  3  12 kT  23 kT . The molar transla-
ing from the motion of the system as a whole, such as its kinetic tional energy is therefore Etrans,m  23 kT  N A  23 RT . At 25 °C,
energy as it accompanies the Earth on its orbit round the Sun. RT  2.48 kJmol 1, so the contribution of translation to the
That is, the internal energy is the energy ‘internal’ to the sys- molar internal energy of a perfect gas is 3.72 kJmol −1 .
tem. The change in internal energy is denoted by ∆U when a
system changes from an initial state i with internal energy U i to
a final state f with internal energy U f : There are no intermolecular interactions in a perfect gas, so
the distance between the molecules has no effect on the energy.
U  U f  U i  (2A.1)
Therefore,
A convention used throughout thermodynamics is that
The internal energy of a perfect gas is independent of the
X  X f  X i , where X is a property of the system.
volume it occupies.
The internal energy is a state function, a property with a
value that depends only on the current state of the system and The internal energy of interacting molecules in condensed
is independent of how that state has been prepared. In other phases also has a contribution from the potential energy of their
words, internal energy is a function of the variables that specify interaction, but no simple expressions can be written down in
the current state of the system, such as the temperature, pres- general. Nevertheless, it remains true that as the temperature of
sure, and amount of substance. Changing any one of these vari- a system is raised, the internal energy increases as the various
ables may result in a change in internal energy. modes of motion become more highly excited.
2A Internal energy 37

2A.2(b) The formulation of the First Law spring is U  100 J  15 J  85 J. To be clear about the direc-
tion of flow of energy into or out of a system, include the sign
It has been found experimentally that the internal energy of a of ∆U (and of ∆X in general), even if it is positive. So this result
closed system may be changed either by doing work on the sys- is written as U  85 J, not U  85 J.
tem or by heating it. It might be known that the energy transfer
has occurred as a result of doing work or through heating, but
the system is blind to the mode employed. That is,
Heat and work are equivalent ways of changing the 2A.3 Expansion work
­internal energy of a closed system.
Equation 2A.2 expresses the First Law in terms of a measurable
A system is like a bank: it accepts deposits in either currency change in the internal energy. To take the discussion further
(work or heat), but stores its reserves as internal energy. It is also and open up the full power of thermodynamics it is necessary
found experimentally that if a system is isolated from its sur- to rewrite that equation in terms of an infinitesimal change in
roundings, meaning that it can exchange neither matter nor en- the internal energy, dU. Thus, if the work done on a system is
ergy with its surroundings, then no change in internal energy dw and the energy supplied to it as heat is dq, then
takes place. This summary of observations is now known as the
First Law of thermodynamics and is expressed as follows: dU  dq  dw  (2A.3)

The internal energy of an isolated system is constant. The practical applications of this expression depend on being
able to relate dq and dw to observable events taking place in the
It is not possible to use a system to do work, then to leave it iso- surroundings.
lated, and then expect to find it restored to its original state with A good starting point is a discussion of expansion work, the
the same capacity for doing work. The experimental evidence work arising from a change in volume. An example is the work
for this observation is that no ‘perpetual motion’ machine, a done by a gas as it expands and drives back the atmosphere. The
machine that does work without consuming fuel or using some term ‘expansion work’ includes the work associated with com-
other source of energy, has ever been built. pression, a reduction in volume. In compression, the change in
These remarks may be expressed symbolically as follows. If w is the volume, ∆V, is a negative quantity.
the work done on a system, q is the energy transferred as heat to
a system, and ∆U is the resulting change in internal ­energy, then
2A.3(a) The general expression for work
U  q  w Mathematical statement of the First Law (2A.2)
The calculation of expansion work starts from the definition in
Equation 2A.2 summarizes the equivalence of heat and work for Energy: A first look 2a, dw  Fx dx, but with the sign of the op-
bringing about changes in the internal energy of a closed system. posing force written explicitly:
It also embodies the fact that in an isolated system, for which q = 0
and w = 0, the internal energy is constant. The equation states that dw   | F | dz
Work done
 (2A.4)
the change in internal energy of a closed system is equal to the [definition]
energy that passes through its boundary as heat or work. The negative sign implies that the internal energy of the sys-
Equation 2A.2 employs the ‘acquisitive convention’, in which tem doing the work decreases when it moves an object against
w and q are positive if energy is transferred to the system as an opposing force of magnitude | F |, and there are no other
work or heat. These quantities are negative if energy is lost from changes (such as an influx of energy as heat). If the motion is in
the system.1 In other words, the flow of energy as work or heat the positive z direction it follows that dz is positive and hence
is viewed from the system’s perspective. dw is negative. Because dw < 0, the internal energy decreases as
energy is transferred to the surroundings as work.
Brief illustration 2A.2 Now consider the arrangement shown in Fig. 2A.5, in which
one wall of a system is a massless, frictionless, rigid, perfectly
If an electric motor produces 15 kJ of energy each second as fitting piston of area A. If the external pressure is pex , the mag-
mechanical work and loses 2 kJ as heat to the surroundings, nitude of the force acting on the outer face of the piston is
then the change in the internal energy of the motor each | F | = pex A . The work done when the system expands through
second is U  2 kJ  15 kJ  17 kJ. Suppose that, when a a distance dz against an external pressure pex is therefore
spring is wound, 100 J of work is done on it but 15 J escapes to dw   pex Adz . The quantity Adz is the change in volume, dV,
the surroundings as heat. The change in internal energy of the in the course of the expansion. Therefore, the work done when
the system expands by dV against a pressure pex is
1
Many engineering texts adopt a different convention for work: w > 0 if en- dw   pex dV Expansion work  (2A.5a)
ergy is used to do work in the surroundings.
38 2 The First Law

Table 2A.1 Varieties of work*


External
Type of work dw Comments Units†
pressure,
pex Expansion − pex dV pex is the external pressure Pa

dV = Adz dV is the change in volume m3


Area, A dz
Surface expansion  d γ is the surface tension N m −1

Pressure, p dσ is the change in area m2

Extension f dl f is the tension N


dl is the change in length m
Electrical φ dQ φ is the electric potential V
dQ is the change in charge C
Figure 2A.5 When a piston of area A moves out through a
distance dz, it sweeps out a volume dV = Adz. The external Q dφ dφ is the potential difference V
pressure pex is equivalent to a weight pressing on the piston, Q is the charge transferred C
and the magnitude of the force opposing expansion is pexA.
* In general, the work done on a system can be expressed in the form dw   | F | dz ,
where | F | is the magnitude of a ‘generalized force’ and dz is a ‘generalized displacement’.

For work in joules (J). Note that 1 N m = 1 J and 1 V C = 1 J.
To obtain the total work done when the volume changes
from an initial value Vi to a final value Vf it is necessary to inte-
grate this expression between the initial and final volumes: throughout the expansion. For example, the piston might be
Vf
pressed on by the atmosphere, which exerts the same pressure
w    pex dV  (2A.5b) throughout the expansion. A chemical example of this condi-
Vi
tion is the expansion of a gas formed in a chemical reaction in a
The force acting on the piston, pex A , is equivalent to the force aris- container that can expand. Equation 2A.5b is then evaluated by
ing from a weight that is raised as the system expands. Because taking the constant pex outside the integral:
that newly raised weight can do more work, its energy has been Vf
increased, and that energy has come from within the system. w   pex  dV   pex (Vf  Vi )
Vi
Equation 2A.5b can still be used to calculate the work done
when the system is compressed, but now Vf < Vi. Now the same Therefore, if the change in volume is written as V  Vf  Vi,
weight is lowered in the surroundings. That newly lowered
Expansion work
weight can do less work, so its energy has decreased and that w   pex V [constant external pressure]
 (2A.6)
energy is transferred into the system.
Note that in compression or expansion it is still the external This result is illustrated graphically in Fig. 2A.6, which
pressure that determines the magnitude of the energy trans- makes use of the fact that the magnitude of an integral can
ferred as work. To understand this point think of the external be interpreted as an area. The magnitude of w, denoted |w|,
pressure as arising from a weight resting on the piston. In a com- is equal to the area beneath the horizontal line at p = pex lying
pression the weight is lowered, implying that the energy of the between the initial and final volumes. A p,V-graph used to
surroundings has decreased, as indicated by the change in height
of the weight. This energy goes into the system. In an expansion
pex
the weight is raised, implying that the energy of the surround-
ings has increased. Again, this energy comes from the system.
Pressure, p

As seen throughout the text, other types of work (for ex- Area = pexΔV
ample, electrical work), called either non-expansion work or
additional work, have analogous expressions. Each one is the
product of an extensive factor (such as the change in volume)
Volume, V
and an intensive factor (such as the external pressure). Some Vi Vf

are collected in Table 2A.1.


pex pex

2A.3(b) Expansion against constant pressure


The next task is to see how the work associated with chang- Figure 2A.6 The magnitude of the work done by a gas when it
ing the volume, the expansion work, can be calculated from expands against a constant external pressure, pex, is equal to the
eqn 2A.5b. First, suppose that the external pressure is constant shaded area in this example of an indicator diagram.
2A Internal energy 39

illustrate ­expansion work is called an indicator diagram; behaviour because an infinitesimal increase or decrease in
James Watt first used one to indicate aspects of the operation the external pressure causes changes in volume in opposite
of his steam engine. ­directions. If the external pressure is reduced infinitesimally,
Free expansion is expansion against zero opposing force. It the gas expands slightly. If the external pressure is increased
occurs when pex = 0. According to eqn 2A.6, in this case infinitesimally, the gas contracts slightly. In either case the
change is reversible in the thermodynamic sense. If, on the
w =0 Work of free expansion (2A.7) other hand, the external pressure is measurably greater than
the internal pressure, then decreasing pex infinitesimally will
That is, no work is done when a system expands freely. Expansion not decrease it below the pressure of the gas, so will not change
of this kind occurs when a gas expands into a ­vacuum. the direction of the process. Such a system is not in mechani-
cal equilibrium with its surroundings and the compression is
thermodynamically irreversible.
Brief illustration 2A.3 To achieve reversible expansion pex has to be set equal to p
at each stage of the expansion. In practice, this equalization
When a certain amount of iron reacts with hydrochloric acid could be achieved by gradually removing weights from the
in a closed vessel equipped with a movable piston it produces piston so that the downward force due to the weights always
2.00 dm3 of hydrogen gas. The work done when the reaction matches the changing upward force due to the pressure of the
takes place in a laboratory where the external pressure is gas or by gradually adjusting the external pressure to match
100 kPa is the pressure of the expanding gas. When pex = p , eqn 2A.5a
w   pex V  (1.00  105 Pa )  (0.002 00 m 3 ) becomes
 2.00  102 Pam3 Reversible expansion work
dw   pdV [ p is the pressure of the system]
 (2A.8a)
3
Because 1 Pa m = 1 J, w = −200 J. That is, the system (the reac-
tion mixture) does 200 J of work driving back the atmosphere. Although the pressure inside the system appears in this expres-
sion for the work, it does so only because pex has been arranged
to be equal to p to ensure reversibility. The total work of revers-
ible expansion from an initial volume Vi to a final volume Vf is
2A.3(c) Reversible expansion therefore
A reversible change in thermodynamics is a change that can Vf
be reversed by an infinitesimal modification of a variable. w    pdV(2A.8b)
Vi
The key word ‘infinitesimal’ sharpens the everyday meaning
of the word ‘reversible’ as something that can change direc- The integral can be evaluated once it is known how the pres-
tion. To understand the concept of reversibility, consider two sure of the confined gas depends on its volume. Equation 2A.8b
systems, A and B, at the same temperature and in contact is the link with the material covered in Focus 1 because, if the
through diathermic walls. If the temperature of A is raised equation of state of the gas is known, p can be expressed in
infinitesimally a small quantity of energy flows from A to B terms of V and the integral can be evaluated.
because A is at a higher temperature than B. If the tempera-
ture of A is lowered infinitesimally a small quantity of energy Isothermal reversible expansion of a
2A.3(d)
flows from B to A. The direction of the flow changes as a re-
perfect gas
sult of an infinitesimal change in a variable, and the process
is reversible. Two systems that are in thermal contact and at Consider the isothermal reversible expansion of a perfect gas.
the same temperature are said to be in thermal equilibrium. The expansion is made isothermal by keeping the system in
There is a very close relationship between reversibility and thermal contact with a constant-temperature bath. Because the
equilibrium: systems at equilibrium are poised to undergo equation of state is pV = nRT , at each stage p = nRT /V , with V
reversible change. the volume at that stage of the expansion. The temperature T
Suppose a gas is confined by a piston and that the exter- is constant in an isothermal expansion, so it, along with n and
nal pressure, pex , is set equal to the pressure, p, of the con- R, may be taken outside the integral. It follows that the work of
fined gas. Such a system is in mechanical equilibrium with isothermal reversible expansion of a perfect gas from Vi to Vf at
its surroundings in the sense that although the piston is free a temperature T is
to move, it is subjected to equal and opposite forces on either
Integral A.2
side and therefore does not move. In other words, there is no Work of isothermal
Vf dV V
flow of energy as work between the system and its surround- w = − nRT ∫Vi V
= − nRT ln f
Vi
reversible expansion
[perfect gas]
(2A.9)
ings. Mechanical equilibrium is another instance of ­reversible
40 2 The First Law

Brief Illustration 2A.4 is wasted when pex < p, the maximum work available from a
system operating between specified initial and final states is
When a sample of 1.00 mol Ar(g), regarded here as a perfect obtained when the change takes place reversibly. This conclu-
gas, undergoes an isothermal reversible expansion at 20.0 °C sion is general: maximum work of any kind is obtained when a
from 10.0 dm 3 to 30.0 dm 3 the work done is process occurs reversibly.
30.0 dm3
w  (1.00 mo1)  (8.3145 JK 1 mo11 )  (293.2 K )  ln
10.0 dm3 Exercises
  2.68 kJ E2A.1 A sample consisting of 1.00 mol Ar is expanded isothermally at 20 °C
from 10.0 dm3 to 30.0 dm3 (i) reversibly, (ii) against a constant external
pressure equal to the final pressure of the gas, and (iii) freely (against zero
external pressure). For the three processes calculate w.
When the final volume is greater than the initial volume, as 3
E2A.2 A sample of 4.50 g of methane occupies 12.7 dm at 310 K. (i) Calculate
in an expansion, the logarithm in eqn 2A.9 is positive and w < 0. the work done when the gas expands isothermally against a constant external
That is, energy is lost from the system as work. The equations pressure of 200 Torr until its volume has increased by 3.3 dm3. (ii) Calculate
also show that more work is done for a given change of volume the work that would be done if the same expansion occurred reversibly.
when the temperature is increased. At a higher temperature the
pressure of the confined gas is greater and so needs a higher
opposing pressure to ensure reversibility. As a result, the work 2A.4 Heat transactions
done is correspondingly greater.
The result expressed by eqn 2A.9 can be illustrated by an in- In general, the change in internal energy of a system is
dicator diagram in which the magnitude of the work done is
dU  dq  dwexp  dwadd (2A.10)
equal to the area under the isotherm p = nRT /V (Fig. 2A.7).
Superimposed on the diagram is the rectangular area obtained where dwadd is work in addition (‘add’ for additional) to the ex-
for irreversible expansion against constant external pressure pansion work, dwexp. For instance, dwadd might be the electrical
fixed at the same final value as that reached in the revers- work of driving a current of electrons through a circuit. A system
ible expansion. More work is obtained when the expansion kept at constant volume can do no expansion work, so in that case
is reversible (the area is greater) because matching the exter- dwexp = 0. If the system is also incapable of doing any other kind
nal pressure to the internal pressure at each stage of the pro- of work then dwadd = 0 too. Under these circumstances:
cess ensures that none of the pushing power of the system is
Energy transferred as heat
wasted. It is not possible to obtain more work than that for the dU = dq at constant volume
(2A.11a)
reversible process because increasing the external pressure
even infinitesimally at any stage results in compression. It fol- This relation is commonly expressed as dU = dqV , where the
lows from this discussion that, because some pushing power subscript implies the constraint of constant volume. For a
measurable change between states i and f along a path at con-
stant volume,
pi
p = nRT/V U f U i q
  V

f f
 dU   dqV
Pressure, p

pf
i i

which is summarized as
U  qV (2A.11b)
Note that the integral of dq is not written as ∆q because q,
Vi Volume, V Vf ­unlike U, is not a state function so cannot be reported as ‘final
heat − initial heat’. It follows from eqn 2A.11b that a measure-
pi pf
ment of the energy supplied as heat to a system at constant vol-
ume is equivalent to a measurement of the change in internal
energy of the system.
Figure 2A.7 The magnitude of the work done by a perfect gas
when it expands reversibly and isothermally is equal to the area 2A.4(a) Calorimetry
under the isotherm p = nRT/V. The magnitude of the work done
during the irreversible expansion against the same final pressure is Calorimetry is the study of the transfer of energy as heat dur-
equal to the rectangular area shown slightly darker. Note that the ing a physical or chemical process. A calorimeter is a device
reversible work done is greater than the irreversible work done. for measuring energy transferred as heat. The most common
2A Internal energy 41

Firing ­constant current, I, from a source of known potential difference,


leads Thermometer  , passes through a heater for a known period of time, t, the
Oxygen input
energy supplied as heat is (The chemist’s toolkit 2A.1)
Bomb
Sample q  It  (2A.13)
Oxygen
under pressure
Brief illustration 2A.5

Water If a current of 10.0 A from a 12 V supply is passed for 300 s,


then from eqn 2A.13 the energy supplied as heat is
q  (10.0 A)  (300 s)  (12 V)  3.6  104 A Vs  36 kJ
Figure 2A.8 A constant-volume bomb calorimeter. The ‘bomb’
The units are treated by using 1 A Vs  1 (C s 1 ) Vs  1 C V  1 J .
is the central vessel, which is strong enough to withstand high
If the observed rise in temperature is 5.5 K, then the calorim-
pressures. The calorimeter is the entire assembly shown here.
To ensure adiabaticity, the calorimeter is immersed in a water eter constant is C  (36 kJ)/(5.5 K )  6.5 kJK 1.
bath with a temperature continuously readjusted to that of the
calorimeter at each stage of the combustion.
Alternatively, C may be determined by burning a known
mass of substance (benzoic acid is often used) that has a known
­device for measuring qV (and therefore ∆U) is an adiabatic heat output. With C known, it is simple to interpret an observed
bomb calorimeter (Fig. 2A.8). The process to be studied— temperature rise as a release of energy as heat.
which may be a chemical reaction—is initiated inside a con-
stant-volume container, the ‘bomb’. The bomb is immersed in
a stirred water bath, and the whole device is the calorimeter.
2A.4(b) Heat capacity
The calorimeter is also immersed in an outer water bath. The The internal energy of a system increases when its temperature
temperature of the water in the calorimeter and of the outer is raised, and this increase depends on the conditions under
bath are both monitored and that of the outer bath is adjusted which the heating takes place. Suppose the system has a con-
to keep it the same as the water in the calorimeter. This ar- stant volume. If the internal energy is plotted against tempera-
rangement ensures that there is no net loss of heat from the ture, then a curve like that in Fig. 2A.9 is typically obtained.
calorimeter to the surroundings (the bath) and hence that the The slope of the tangent to the curve at any temperature is
calorimeter is adiabatic. called the heat capacity of the system at that temperature. The
The change in temperature, ∆T , of the calorimeter is com- heat capacity at constant volume is denoted CV and is defined
monly found to be proportional to the energy that the reaction ­formally as
releases or absorbs as heat:
 U  Heat capacity at constant volume
CV    (2A.14)
q  CT (2A.12)  T V [definition]

where C is the calorimeter constant. The value of this empirical (For a note on partial derivatives, see The chemist’s toolkit
constant can be found by measuring ΔT for a process of known 2A.2.) The internal energy varies with the temperature and
heat output. One possibility is to use electrical heating. If a the volume of the sample, but here only its variation with the

The chemist’s toolkit 2A.1 Electrical charge, current, power, and energy
Electrical charge, Q, is measured in coulombs, C. The motion E  Pt  It 
of charge gives rise to an electric current, I, measured in cou-
Because 1 A Vs  1 (Cs 1 ) Vs  1 C V  1 J, the energy is obtained
lombs per second, or amperes, A, where 1 A  1 Cs 1.
in joules with the current in amperes, the potential difference
When a current I flows through a potential difference Δϕ
in volts, and the time in seconds. That energy may be supplied
(measured in volts, V, with 1 V  1 JC 1), the power P, is
as either work (to drive a motor) or as heat (through a ‘heater’).
P  I  In the latter case
It follows that if a constant current flows for a period t the
q  It 
energy supplied is
42 2 The First Law

Temperature variation

Internal energy, U
of U
Internal energy, U

B Slope of U versus T
at constant V

Temperature, T

Temperature, T Volume, V

Figure 2A.9 The internal energy of a system increases as the


Figure 2A.10 The internal energy of a system varies with volume
temperature is raised; this graph shows its variation as the system
and temperature, perhaps as shown here by the surface. The
is heated at constant volume. The slope of the tangent to the
variation of the internal energy with temperature at one particular
curve at any temperature is the heat capacity at constant volume
constant volume is illustrated by the curve drawn parallel to the
at that temperature. Note that, for the system illustrated, the heat
temperature axis. The slope of this curve at any point is the partial
capacity is greater at B than at A.
derivative (∂U ∂T )V .

t­emperature is relevant, because the volume is held constant Heat capacities are extensive properties: 100 g of water, for
(Fig. 2A.10), as signified by the subscript V. instance, has 100 times the heat capacity of 1 g of water (and
therefore requires 100 times the energy as heat to bring about
the same rise in temperature). The molar heat ­capacity at
Brief illustration 2A.6
constant volume, CV,m = CV /n, is the heat capacity per mole
In Brief illustration 2A.1 it is shown that the translational of substance, and is an intensive property (all molar quan-
contribution to the molar internal energy of a perfect tities are intensive). For certain applications it is useful to
monatomic gas is 23 RT. Because this is the only contribu- know the specific heat capacity (more informally, the ‘spe-
tion to the internal energy, U m (T ) = 23 RT . It follows from cific heat’) of a substance, which is the heat capacity of the
eqn 2A.14 that sample divided by its mass, usually in grams: CV,s = CV /m .
The specific heat capacity of water at room temperature is
 3 close to 4.2 J K−1 g−1. In general, heat capacities depend on
CV, m   RT   23 R
T 2 the temperature and decrease at low temperatures. However,
The numerical value is 12.47 J K−1 mol−1. over small ranges of temperature at and above room tem-
perature, the variation is quite small and for approximate

The chemist’s toolkit 2A.2 Partial derivatives


A partial derivative of a function of more than one variable,
such as the derivative (∂f /∂x ) y of f(x,y) with respect to x with y
(f/x)y
treated as a constant, is the slope of the function with respect to (f/y)x
one of the variables, all the other variables being held constant
(Sketch 1). The ∂, ‘curly d’ is used to denote partial derivatives.
f (x,y)

Although a partial derivative shows how a function changes


when one variable changes, it may be used to evaluate how the
function changes when more than one variable changes by an
infinitesimal amount. Thus, if f is a function of x and y, then x
when x and y change by dx and dy, respectively, f changes by

 f   f 
d f    d x    dy y
 x  y  y  x
For further information, see Part 1 of the Resource section. Sketch 1
2A Internal energy 43

c­ alculations heat capacities can be treated as almost inde- as heat to the sample (by electrical heating, for example) under
pendent of ­temperature. constant volume conditions and the resulting increase in tem-
The heat capacity is used to relate a change in internal energy perature is monitored. The ratio of the energy transferred as
to a change in temperature of a constant-volume system. It fol- heat to the temperature rise it causes (qV /∆T ) is the constant-
lows from eqn 2A.14 that volume heat capacity of the sample. A large heat capacity im-
plies that, for a given quantity of energy transferred as heat,
Internal energy change on heating
dU = CV dT [constant volume]
(2A.15a) there will be only a small increase in temperature.

That is, at constant volume, an infinitesimal change in temper-


Brief illustration 2A.7
ature brings about an infinitesimal change in internal energy,
and the constant of proportionality is CV . If the heat capacity is
Suppose a 55 W electric heater immersed in a gas in a constant-
independent of temperature over the range of temperatures of volume adiabatic container was on for 120 s and it was found
interest, then that the temperature of the gas rose by 5.0 °C (an increase equiv-
T2 T2
T
  alent to 5.0 K). The heat supplied is (55 W) × (120 s) = 6.6 kJ
U   CV dT  CV  dT  CV (T2  T1 ) (with 1 J = 1 W s), so the heat capacity of the sample is
T1 T1

A measurable change of temperature, ∆T , brings about a meas- 6.6 kJ


CV   1.3 kJK 1
urable change in internal energy, ∆U, with 50 K

Internal energy change on heating


U  CV T [constant volume]
(2A.15b)

Because a change in internal energy can be identified with the


Exercises
heat supplied at constant volume (eqn 2A.11b), the last equa- E2A.3 A sample consisting of 2.00 mol He is expanded isothermally at 0 °C
from 5.0 dm3 to 20.0 dm3 (i) reversibly, (ii) against a constant external
tion can also be written as pressure equal to the final pressure of the gas, and (iii) freely (against zero
qV  CV T (2A.16) external pressure). For the three processes calculate q, w, and ΔU.
E2A.4 A sample consisting of 1.00 mol of perfect gas atoms, for which
This relation provides a simple way of measuring the heat ca- CV,m = 23 R , initially at p1 = 1.00 atm and T1 = 300 K, is heated to 400 K at
pacity of a sample. A measured quantity of energy is ­transferred constant volume. Calculate the final pressure, ΔU, q, and w.

Checklist of concepts
☐ 1. Work is the process of achieving motion against an ☐ 8. The internal energy increases as the temperature is
opposing force. raised.
☐ 2. Energy is the capacity to do work. ☐ 9. The First Law states that the internal energy of an iso-
☐ 3. Heat is the process of transferring energy as a result of lated system is constant.
a temperature difference. ☐ 10. Free expansion (expansion against zero pressure) does
☐ 4. An exothermic process is a process that releases energy no work.
as heat. ☐ 11. A reversible change is a change that can be reversed by
☐ 5. An endothermic process is a process in which energy is an infinitesimal change in a variable.
acquired as heat. ☐ 12. To achieve reversible expansion, the external pres-
☐ 6. In molecular terms, work is the transfer of energy that sure is matched at every stage to the pressure of the
makes use of organized motion of atoms in the sur- system.
roundings and heat is the transfer of energy that makes ☐ 13. The energy transferred as heat at constant volume is
use of their disorderly motion. equal to the change in internal energy of the system.
☐ 7. Internal energy, the total energy of a system, is a state ☐ 14. Calorimetry is the measurement of heat transactions.
function.
44 2 The First Law

Checklist of equations
Property Equation Comment Equation number
First Law of thermodynamics U  q  w 2A.2
Work of expansion dw   pex dV Includes compression 2A.5a
Work of expansion against a constant external pressure w   pex V pex = 0 for free expansion 2A.6
Work of isothermal, reversible expansion w  nRT ln(Vf /Vi ) Perfect gas 2A.9
Internal energy change at constant volume U  qV No other forms of work 2A.11b
Electrical heating q  It  2A.13
Heat capacity at constant volume CV  (U /T )V 2A.14
TOPIC 2B Enthalpy

2B.1 The definition of enthalpy


➤ Why do you need to know this material?
The enthalpy, H, is defined as
The concept of enthalpy is central to many thermodynamic
discussions about processes, such as physical transforma- Enthalpy
H  U  pV (2B.1)
tions and chemical reactions taking place under conditions [definition]
of constant pressure.
where p is the pressure of the system and V is its volume.
➤ What is the key idea? Because U, p, and V are all state functions, the enthalpy is a
state function too. As is true of any state function, the change
A change in enthalpy is equal to the energy transferred as
in enthalpy, ΔH, between any pair of initial and final states is
heat at constant pressure.
independent of the path between them.
➤ What do you need to know already?
This Topic makes use of the discussion of internal energy 2B.1(a) Enthalpy change and heat transfer
(Topic 2A) and draws on some aspects of perfect gases
An important consequence of the definition of enthalpy in
(Topic 1A).
eqn 2B.1 is that it can be shown that the change in enthalpy is
equal to the energy supplied as heat under conditions of con-
stant pressure.

The change in internal energy is not equal to the energy trans- How is that done? 2B.1 Deriving the relation between
ferred as heat when the system is free to change its volume,
such as when it is able to expand or contract under conditions enthalpy change and heat transfer at constant pressure
of constant pressure. Under these circumstances some of the In a typical thermodynamic derivation, as here, a common way
energy supplied as heat to the system is returned to the sur- to proceed is to introduce successive definitions of the quanti-
roundings as expansion work (Fig. 2B.1), so dU is less than dq. ties of interest and then apply the appropriate constraints.
In this case the energy supplied as heat at constant pressure is
Step 1 Write an expression for H + dH in terms of the definition
equal to the change in another thermodynamic property of the
of H
system, the ‘enthalpy’.
For a general infinitesimal change in the state of the system,
U changes to U + dU, p changes to p + dp, and V changes to
V + dV, so from the definition in eqn 2B.1, H changes by dH to
H + dH = (U + dU) + (p + dp)(V + dV)
Energy as
work = U + dU+ pV + pdV + Vdp + dpdV
The last term is the product of two infinitesimally small quan-
ΔU < q tities and can be neglected. Now recognize that U + pV = H on
the right (boxed), so
Energy
as heat H + dH = H + dU + pdV + Vdp
and hence
dH  dU  pdV  Vdp
Figure 2B.1 When a system is subjected to constant pressure and Step 2 Introduce the definition of dU
is free to change its volume, some of the energy supplied as heat
Because dU = dq + dw this expression becomes
may escape back into the surroundings as work. In such a case,
the change in internal energy is smaller than the energy supplied dU
 
as heat. dH  dq  dw  pdV  Vdp
46 2 The First Law

Step 3 Apply the appropriate constraints Gas, vapour


If the system does only expansion work, then dw = −pexdV, where
pex is the external pressure. Provided the system is in mechanical Oxygen
equilibrium with its surroundings, the external pressure is equal Products
to the pressure of the system p and so dw = −pdV
dw
dH = dq – pdV + pdV +Vdp

The two boxed terms cancel, leaving


dH  dq  Vdp
Figure 2B.2 A constant-pressure flame calorimeter consists of this
At constant pressure, dp = 0, so
component immersed in a stirred water bath. Combustion occurs
dH = dq (at constant pressure, no additional work ) as a known amount of reactant is passed through to fuel the
The constraint of constant pressure is denoted by a subscript p, flame, and the rise of temperature is monitored.
so this equation can be written
Heat transferred at constant pressure
dH = dq p  (2B.2a)
[infinitesimal change] 2B.1(b) Calorimetry
This equation states that, provided there is no additional (non- An enthalpy change can be measured calorimetrically by
expansion) work done, and the system and surroundings are in monitoring the temperature change that accompanies a phys-
mechanical equilibrium, the change in enthalpy is equal to the ical or chemical change at constant pressure. A calorimeter
energy supplied as heat at constant pressure. for studying processes at constant pressure is called an iso-
Step 4 Evaluate ΔH by integration baric calorimeter. A simple example is a thermally insulated
vessel open to the atmosphere: the energy released as heat in
For a measurable change between states i and f along a path
at constant pressure, the preceding expression is integrated
the reaction is monitored by measuring the change in temper-
as follows ature of the contents. For a combustion reaction an a­ diabatic
H f  Hi q flame calorimeter may be used to measure ΔT when a given
  p

f f amount of substance burns in a supply of oxygen (Fig. 2B.2).


 dH   dq
i i The most sophisticated way to measure enthalpy changes,
The integral over dq is not written as qf − qi = Δq because q is however, is to use a differential scanning calorimeter (DSC),
not a property of the system, and there is no meaning to the as explained in Topic 2C. Changes in enthalpy and internal
quantities qf and qi. The final result is energy may also be measured by non-calorimetric methods
(Topic 6C).
(2B.2b)
ΔH = q p One route to ΔH is to measure the internal energy change
Heat transferred at
constant pressure by using a bomb calorimeter (Topic 2A), and then to convert
[measurable change] ΔU to ΔH. Because solids and liquids have small molar vol-
umes, for them pVm is so small that the molar enthalpy and
molar internal energy are almost identical (Hm = Um + pVm ≈
Brief illustration 2B.1 Um). Consequently, if a process involves only solids or liquids,
the values of ΔH and ΔU are almost identical. Physically, such
Water is heated to boiling under a pressure of 1.0 atm. When an
processes are accompanied by a very small change in volume;
electric current of 0.50 A from a 12 V supply is passed for 300 s
the system does negligible work on the surroundings when the
through a resistance in thermal contact with the water, it is
process occurs, so the energy supplied as heat stays entirely
found that 0.798 g of water is vaporized. The enthalpy change is
within the system. However, care must be taken when consid-
H  q p  It   (0.50 A)  (300 s)  (12 V) ering processes at very high pressures because pΔVm may not
 0.50  300  12 J be negligible.
where 1 A V s = 1 J. Because 0.798 g of water is (0.798 g)/(18.02
g mol−1) = (0.798/18.02) mol H2O, the enthalpy of vaporization per Example 2B.1 Relating ΔH and ΔU
mole of H2O is
The change in molar internal energy when CaCO3(s) as cal-
0.50  12  300 J 1 cite converts to its polymorph aragonite, is +0.21 kJ mol−1.
H m   41 kJmo1
(0.798/18.02) mol Calculate the difference between the molar enthalpy and
internal energy changes when the pressure is 1.0 bar. The
2B Enthalpy 47

mass densities of the polymorphs are 2.71 g cm−3 (calcite) and Brief illustration 2B.2
2.93 g cm−3 (aragonite).
Collect your thoughts The starting point for the calculation
In the reaction 3 H2(g) + N2(g) → 2 NH3(g), 4 mol of gas-
is the relation between the enthalpy of a substance and its phase molecules is replaced by 2 mol of gas-phase molecules,
internal energy (eqn 2B.1). You need to express the difference so Δng = −2 mol. Therefore, at 298 K, when RT = 2.5 kJ mol−1,
between the two quantities in terms of the pressure and the the molar enthalpy and molar internal energy changes taking
difference of their molar volumes. The latter can be calculated place in the system are related by
from their molar masses, M, and their mass densities, ρ, by H m  U m  (2)  RT  5.0 kJmo11
using ρ = M/Vm.
Note that the difference is expressed in kilojoules, not joules
The solution The change in enthalpy when the transition as in Example 2B.1. The enthalpy change is more negative than
occurs is the change in internal energy because, although energy escapes
from the system as heat when the reaction occurs, the system
H m  H m (aragonite)  H m (calcite) contracts as the product is formed, so energy is restored to it as
 {U m (a )  pVm (a )}  {U m (c)  pVm (c)}} work from the surroundings.
 U m  p{Vm (a )  Vm (c)}

where a denotes aragonite and c calcite. It follows by substitut-


ing Vm = M/ρ that Exercises
E2B.1 Calculate the value of ΔHm − ΔUm for the reaction N2(g) + 3 H2(g) →
 1 1  2 NH3(g) at 473 K.
H m  U m  pM   
  (a )  (c)  E2B.2 When 0.100 mol of H2(g) is burnt in a flame calorimeter it is
observed that the water bath in which the apparatus is immersed increases
Substitution of the data, using M = 100.09 g mol−1, gives in temperature by 13.64 K. When 0.100 mol C4H10(g), butane, is burnt in
the same apparatus the temperature rise is 6.03 K. The molar enthalpy of
combustion of H2(g) is −285 kJ mol−1. Calculate the molar enthalpy
H m  U m  (1.0  105 Pa )  (100.09 g mo11 ) of combustion of butane.
 1 1 
 
 2.93 g cm 3 2.71 g cm 3 
 
 2.8  105 Pa cm3 mol 1  0.28 Pa m 3 mol 1
The variation of enthalpy with
2B.2
temperature
Hence (because 1 Pa m3 = 1 J), ΔHm − ΔUm = −0.28 J mol−1, which
is only 0.1 per cent of the value of ΔUm. The enthalpy of a substance increases as its temperature is
raised. The reason is the same as for the internal energy: mol-
Self-test 2B.1 Calculate the difference between ΔH and ΔU
ecules are excited to states of higher energy so their total en-
when 1.0 mol Sn(s, grey) of density 5.75 g cm−3 changes to
ergy increases. The relation between the increase in enthalpy
Sn(s, white) of density 7.31 g cm−3 at 10.0 bar.
Answer: ΔH – ΔU = −4.4 J and the increase in temperature depends on the conditions (e.g.
whether the pressure or the volume is constant).

In contrast to processes involving condensed phases, the val- 2B.2(a) Heat capacity at constant pressure
ues of the changes in internal energy and enthalpy might dif-
fer significantly for processes involving gases. The enthalpy of a The most frequently encountered condition in chemistry is
perfect gas is related to its internal energy by using pV = nRT in constant pressure. The slope of the tangent to a plot of enthalpy
the definition of H: against temperature at constant pressure is called the heat
­capacity at constant pressure (or isobaric heat capacity), Cp, at a

nRT
given temperature (Fig. 2B.3). More formally, using the partial
H  U  pV  U  nRT (2B.3)
derivative notation introduced in The chemists’ toolkit 2A.1 in
This relation implies that the change of enthalpy in a reaction Topic 2A:
that produces or consumes gas under isothermal conditions is  H  Heat capacity at constant pressure
Cp    [definition] (2B.5)
Relation between H and U  T  p
H  U  ng RT [isothermal process, perfect gas]
(2B.4)
The heat capacity at constant pressure is the analogue of the
where Δng is the change in the amount of gas molecules in the heat capacity at constant volume (Topic 2A) and is an extensive
reaction. property. The molar heat capacity at constant pressure, Cp,m,
48 2 The First Law

Table 2B.1 Temperature variation of molar heat capacities,


C p ,m /(JK −1 mol−1 ) = a + bT + c /T 2 *
B a b/(10−3 K−1) c/(105 K2)
Enthalpy, H

C(s, graphite) 16.86 4.77 −8.54


CO2(g) 44.22 8.79 −8.62
Internal H2O(l) 75.29 0 0
A energy, U
N2(g) 28.58 3.77 −0.50

* More values are given in the Resource section.

Temperature, T
The empirical parameters a, b, and c are independent of tempera-
Figure 2B.3 The constant-pressure heat capacity at a particular ture. Their values are found by fitting this expression to experi-
temperature is the slope of the tangent to a curve of the enthalpy mental data on many substances, as shown in Table 2B.1.
of a system plotted against temperature (at constant pressure). If eqn 2B.8 is used to calculate the change in enthalpy be-
For gases, at a given temperature the slope of enthalpy versus
tween two temperatures T1 and T2, integration by using Integral
temperature is steeper than that of internal energy versus
A.1 in the Resource section gives
temperature, and Cp,m is larger than CV,m.
T2 T2  c 
H   C p dT    a  bT  2  dT
T1 T1
 T 
is the heat capacity per mole of substance; it is an intensive T2
 c
property.   aT  12 bT 2   (2B.9)
The heat capacity at constant pressure relates the change in  T T
1

enthalpy to a change in temperature. For infinitesimal changes 1 1


of temperature, eqn 2B.5 implies that  
 a(T2  T1 )  12 b T22  T12  c   
 T2 T1 
dH = C p dT (at constant pressure) (2B.6a)
If the heat capacity is constant over the range of temperatures of Example 2B.2
interest, then for a measurable increase in temperature Evaluating an increase in enthalpy with
temperature
T2 T2
T
 
H   C p dT  C p  dT  C p (T2  T1 ) What is the change in molar enthalpy of N2 when it is heated
T1 T1
from 25 °C to 100 °C? Use the heat capacity information in
which can be summarized as Table 2B.1.
H  C p T (at constant pressure)(2B.6b) Collect your thoughts The heat capacity of N2 changes with
temperature significantly in this range, so use eqn 2B.9.
Because a change in enthalpy can be equated to the energy
supplied as heat at constant pressure, the practical form of this The solution Using a = 28.58 J K−1 mol−1, b = 3.77 × 10−3 J K−2 mol−1,
equation is and c = −0.50 × 105 J K mol−1, T1 = 298 K, and T2 = 373 K, eqn 2B.9
is written as
q p  C p T (2B.7)
H  H m (373 K )  H m (298 K )
This expression shows how to measure the constant-pressure
 (28.58 JK 1 mol 1 )  (373 K  298 K)
heat capacity of a sample: a measured quantity of energy is
supplied as heat under conditions of constant pressure (as in a  12 (3.77  103 JK 2 mol 1 )  {(373 K )2  (298 K )2 }
sample exposed to the atmosphere and free to expand), and the  1 1 
(0.50  105 JKmol 1 )   
temperature rise is monitored.  373 K 298 K 
 
The variation of heat capacity with temperature can some-
times be ignored if the temperature range is small; this is an The final result is
excellent approximation for a monatomic perfect gas (for H m (373 K )  H m (298 K )  2.20 kJmo11
instance, one of the noble gases at low pressure). However,
when it is necessary to take the variation into account Self-test 2B.2 At very low temperatures the heat capacity of a
for other substances, a convenient approximate empirical nonmetallic solid is proportional to T 3, and C p,m = aT 3. What
­expression is is the change in enthalpy of such a substance when it is heated
from 0 to a temperature T (with T close to 0)?
c
C p  a  bT 
Answer: H m  14 aT 4
(2B.8)
T2
2B Enthalpy 49

2B.2(b) The relation between heat capacities v­ olume. Because the molar constant-volume heat capacity of
a monatomic gas is about 23 R  12 JK 1 mol 1 (Topic 2A), the
Most systems expand when heated at constant pressure. Such difference is highly significant and must be taken into ac-
systems do work on the surroundings and therefore some of count. The two heat capacities are typically very similar for
the energy supplied to them as heat escapes back to the sur- condensed phases, and for them the difference can normally
roundings as work. As a result, the temperature of the system be ignored.
rises less than when the heating occurs at constant volume. A
smaller increase in temperature implies a larger heat capac-
ity, so in most cases the heat capacity at constant pressure of Exercises
a system is larger than its heat capacity at constant volume. As E2B.3 The constant-pressure heat capacity of a sample of a perfect gas
shown in Topic 2D, there is a simple relation between the two was found to vary with temperature according to the expression Cp/(J K−1) =
heat capacities of a perfect gas: 20.17 + 0.3665(T/K). Calculate q and ΔH when the temperature is raised from
25 °C to 100 °C at constant pressure.
Relation between heat capacities E2B.4 When 3.0 mol O2 is heated at a constant pressure of 3.25 atm, its
C p  CV  nR [perfect gas]
(2B.10)
temperature increases from 260 K to 285 K. Given that the molar heat capacity
of O2 at constant pressure is 29.4 J K−1 mol−1, calculate q, ΔH, and ΔU. You may
It follows that the molar heat capacity of a perfect gas is assume that O2 behaves as a perfect gas.
about 8 J K−1 mol−1 larger at constant pressure than at ­constant

Checklist of concepts
☐ 1. Energy transferred as heat at constant pressure is equal ☐ 3. The heat capacity at constant pressure (the isobaric
to the change in enthalpy of a system. heat capacity) is equal to the slope of enthalpy with
☐ 2. Enthalpy changes can be measured in an isobaric temperature.
­calorimeter.

Checklist of equations
Property Equation Comment Equation number
Enthalpy H = U + pV Definition 2B.1
Heat transfer at constant pressure dH = dqp, ΔH = qp No additional work 2B.2
Relation between ΔH and ΔU ΔH = ΔU + ΔngRT Isothermal process, perfect gas 2B.4
Heat capacity at constant pressure Cp = (∂H/∂T)p Definition 2B.5
Relation between heat capacities Cp − CV = nR Perfect gas 2B.10
TOPIC 2C Thermochemistry

2C.1 Standard enthalpy changes


➤ Why do you need to know this material?
Changes in enthalpy are normally reported for processes t­ aking
Thermochemistry is one of the principal branches of
place under a set of standard conditions. The standard enthalpy
­thermodynamics as its data are widely used in chemical
change, ΔH , is the change in enthalpy for a process in which

applications of thermodynamics.
the initial and final substances are in their standard states:
➤ What is the key idea? The standard state of a substance at a specified
Reaction enthalpies can be combined to provide data on temperature is its pure form at 1 bar.
other reactions of interest.
For example, the standard state of liquid ethanol at 298 K is
➤ What do you need to know already? pure liquid ethanol at 298 K and 1 bar; the standard state of
solid iron at 500 K is pure iron at 500 K and 1 bar.1 The stand-
You need to be aware of the definition of enthalpy and its
ard enthalpy change for a reaction or a physical process is the
status as a state function (Topic 2B). The material on the
difference in enthalpy between the products in their standard
temperature dependence of reaction enthalpies makes
states and the reactants in their standard states, all at the same
use of information about heat capacities (Topic 2B).
specified temperature.
An example of a standard enthalpy change is the standard
enthalpy of vaporization, ΔvapH , which is the enthalpy change

per mole of molecules when a pure liquid at 1 bar vaporizes to


a gas at 1 bar, as in2
The study of the energy transferred as heat during the
H2 O(l)  H2 O(g )  vap H (373 K)  40.66 kJmo11

course of chemical reactions is called thermochemistry.


Thermochemistry is a branch of thermodynamics because As implied by the examples, standard enthalpies may be re-
a reaction vessel and its contents form a system, and chemi- ported for any temperature. However, the conventional temper-
cal reactions result in the exchange of energy between the ature for reporting thermodynamic data is 298.15 K (commonly
system and the surroundings. Thus calorimetry can be used abbreviated to ‘298 K’). Unless otherwise mentioned or indi-
to measure the energy supplied or discarded as heat by a re- cated by attaching the temperature to ΔH , all thermodynamic

action, with q identified with a change in internal energy if data in this text are for this conventional temperature unless
the reaction occurs at constant volume (Topic 2A) or with a specified otherwise.
change in enthalpy if the reaction occurs at constant pres-
sure (Topic 2B). Conversely, if ΔU or ΔH for a reaction is
2C.1(a) Enthalpies of physical change
known, it is possible to predict the energy the reaction can
produce as heat. The standard molar enthalpy change that accompanies a change
As pointed out in Topic 2A, a process that releases energy as of physical state is called the standard enthalpy of transition
heat is classified as exothermic, and one that absorbs energy as and is denoted ΔtrsH (Table 2C.1). The standard enthalpy of

heat is classified as endothermic. Because the release of energy vaporization, ΔvapH , is one example. Another is the standard

as heat into the surroundings at constant pressure signifies a enthalpy of fusion, ΔfusH , the standard molar enthalpy change

decrease in the enthalpy of a system, it follows that an exother- accompanying the conversion of a solid to a liquid, as in
mic process at constant pressure is one for which ΔH < 0; such
H2 O(s)  H2 O(l)  fus H (273 K)  6.01 kJmo11

a process is exenthalpic. Conversely, because the absorption of


heat from the surroundings at constant pressure results in an
increase in enthalpy, an endothermic process at constant pres- 1
The definition of standard state is more sophisticated for solutions
sure has ΔH > 0; such a process is endenthalpic: (Topic 5E).
2
The attachment of the name of the transition to the symbol Δ, as in ΔvapH,
is the current convention. However, the older convention, ΔHvap, is still widely
Exenthalpic process: H  0
used. The current convention is more logical because the subscript identifies
Endenthalpic process: H  0 the type of change, not the physical observable related to the change.
2C Thermochemistry 51

Table 2C.1 Standard enthalpies of fusion and vaporization at the Because the overall result of the indirect path is the same as that
transition temperature, ΔtrsH /(kJ mol−1)*

of the direct path, the overall enthalpy change is the same in each
case (1), and (for processes occurring at the same temperature)
Tf/K Fusion Tb/K Vaporization
 sub H   fus H   vap H (2C.1)
  
Ar 83.81 1.188 87.29 6.506
C 6 H6 278.61 10.59 353.2 30.8
H2O 273.15 6.008 373.15 40.656 (44.016 at 298 K) g
He 3.5 0.021 4.22 0.084

Enthalpy, H
ΔvapH ⦵
* More values are given in the Resource section. 1 ΔsubH ⦵

ΔfusH ⦵
Table 2C.2 Enthalpies of reaction and transition s

Transition Process Symbol*


It follows that, because all enthalpies of fusion are positive, the
Transition Phase α → phase β ΔtrsH enthalpy of sublimation of a substance is greater than its en-
Fusion s→l ΔfusH thalpy of vaporization (at a given temperature).
Vaporization l→g ΔvapH Another consequence of H being a state function is that the
Sublimation s→g ΔsubH standard enthalpy change of a forward process is the negative of
Mixing Pure → mixture ΔmixH its reverse (2):
Solution Solute → solution ΔsolH H (A  B)  H (A  B)(2C.2)

⦵ 

Hydration X±(g) → X±(aq) ΔhydH


Atomization Species(s, l, g) → atoms(g) ΔatH
B

Ionization X(g) → X+(g) + e (g) ΔionH

Enthalpy, H
Electron gain X(g) + e−(g) → X−(g) ΔegH
2 ΔH ⦵(A→B) ΔH ⦵(A←B)
Reaction Reactants → products ΔrH
Combustion Compound(s, l, g) + O2(g) → CO2(g) + H2O(l, g) Δ cH
A
Formation Elements → compound ΔfH

Activation Reactants → activated complex ΔH
For instance, because the enthalpy of vaporization of water is
* IUPAC recommendations. In common usage, the process subscript is often attached +44 kJ mol−1 at 298 K, the enthalpy of condensation of water
to ΔH, as in ΔHtrs and ΔHf. All are molar quantities.
­vapour at that temperature is −44 kJ mol−1.

As in this case, it is sometimes convenient to know the standard


molar enthalpy change at the transition temperature as well as
2C.1(b) Enthalpies of chemical change
at the conventional temperature of 298 K. The different types There are two ways of reporting the change in enthalpy that
of enthalpy changes encountered in thermochemistry are sum- accompanies a chemical reaction. One is to write the thermo-
marized in Table 2C.2. chemical equation, a combination of a chemical equation and
Because enthalpy is a state function, a change in enthalpy is the corresponding change in standard enthalpy:
independent of the path between the two states. This feature is of
great importance in thermochemistry, because it implies that the CH 4 (g )  2 O2 (g )  CO2 (g )  2 H2 O(l) H  890 kJ

same value of ΔH will be obtained however the change is brought


about between specified initial and final states. For example, the where ΔH is the change in enthalpy when reactants in their

conversion of a solid to a vapour can be pictured either as occur- standard states change to products in their standard states:
ring by sublimation (the direct conversion from solid to vapour),
Pure, separate reactants in their standard states
H2 O(s)  H2 O(g )  sub H → pure, separate products in their standard states

or as occurring in two steps, first fusion (melting) and then va- Except in the case of ionic reactions in solution, the enthalpy
porization of the resulting liquid: changes accompanying mixing and separation are insignificant
H2 O(s)  H2 O(1)  fus H
 in comparison with the contribution from the reaction itself.
For the combustion of methane, the standard value refers to
H2 O(l)  H2 O(g )  vap H

the reaction in which 1 mol CH4 in the form of pure methane


Overall: H2 O(s)  H2 O(g )  fus H   vap H gas at 1 bar reacts completely with 2 mol O2 in the form of pure
 
52 2 The First Law

­oxygen gas to produce 1 mol CO2 as pure carbon dioxide at 2C.1(c) Hess’s law
1 bar and 2 mol H2O as pure liquid water at 1 bar; the numerical
value quoted is for the reaction at 298.15 K. Standard reaction enthalpies can be combined to obtain the
Alternatively, the chemical equation is written and the value for another reaction. This application of the First Law is
standard reaction enthalpy, ΔrH (or ‘standard enthalpy of
⦵ called Hess’s law:
reaction’) reported. Thus, for the combustion of methane at The standard reaction enthalpy is the sum of the values
298 K, write for the individual reactions into which the overall
CH 4 (g )  2 O2 (g )  CO2 (g )  2 H2O(l )  r H  890 kJmo11

⦵ reaction may be divided.

For a reaction of the form 2 A + B → 3 C + D the standard reac- The individual steps need not be realizable in practice: they
tion enthalpy would be may be ‘hypothetical’ reactions, the only requirement being
that their chemical equations should balance. The thermody-
 r H  {3H m (C)  H m (D)}  {2H m (A)  H m (B)}

 ⦵
 ⦵
 ⦵
 ⦵

namic basis of the law is the path-independence of the value
of ΔrH . The importance of Hess’s law is that information

where H m (J) is the standard molar enthalpy of species J at the



temperature of interest. Note how the ‘per mole’ of ΔrH comes


⦵ about a reaction of interest, which may be difficult to deter-
directly from the fact that molar enthalpies appear in this ex- mine directly, can be assembled from information on other
pression. The ‘per mole’ is interpreted by noting the stoichio- reactions.
metric coefficients in the chemical equation. In this case, ‘per
mole’ in ΔrH means ‘per 2 mol A’, ‘per mol B’, ‘per 3 mol C’, or

Example 2C.1 Using Hess’s law


‘per mol D’. In general,

Standard reaction The standard reaction enthalpy for the hydrogenation of propene,
r H   H   H

 ⦵
 ⦵

m m enthalpy  (2C.3) CH2=CHCH3 (g )  H2 (g )  CH3CH2CH3 (g )


Products Reactants [definition]
is −124 kJ mol−1. The standard reaction enthalpy for the com-
where in each case the molar enthalpies of the species are mul- bustion of propane,
tiplied by their (dimensionless and positive) stoichiometric CH3CH2CH3 (g )  5 O2 (g )  3 CO2 (g )  4 H2O(l )
coefficients, n. This formal definition is of little practical value,
is −2220 kJ mol−1. The standard reaction enthalpy for the for-
however, because the absolute values of the standard molar en-
mation of water,
thalpies are unknown; this problem is overcome by following
the techniques of Section 2C.2a. H2 (g )  12 O2 (g )  H2O(l)
Some standard reaction enthalpies have special names and is −286 kJ mol−1. Calculate the standard enthalpy of combus-
significance. For instance, the standard enthalpy of c­ ombustion, tion of propene.
ΔcH , is the standard reaction enthalpy for the complete oxida-

Collect your thoughts The skill you need to develop is the abil-
tion of an organic compound to CO2 gas and liquid H2O if the ity to assemble a given thermochemical equation from others.
compound contains C, H, and O, and to N2 gas if N is also pre- Add or subtract the reactions given, together with any others
sent. For example, the combustion of glucose is needed, so as to reproduce the reaction required. Then add or
C 6 H12 O6 (s)  6 O2 (g )  6 CO2 (g )  6 H2 O(1) subtract the reaction enthalpies in the same way.
 c H  2808 kJmol 1 The solution The combustion reaction is
⦵

The value quoted shows that 2808 kJ of heat is released when C 3H6 (g )  92 O2 (g )  3 CO2 (g )  3 H2O(l )
1 mol C6H12O6 burns under standard conditions (at 298 K). This reaction can be recreated from the following sum:
More values are given in Table 2C.3.
Δr H /(kJ mol−1)

Table 2C.3 Standard enthalpies of formation and combustion of C3H6(g) + H2(g) → C3H8(g) −124
organic compounds at 298 K* C3H8(g) + 5 O2(g) → 3 CO2(g) + 4 H2O(l) −2220
H2O(l)  H2 (g )  12 O2 (g ) +286
Δf H /(kJ mol−1) Δc H /(kJ mol−1)
⦵ ⦵

C 3H6 (g )  O2 (g )  3 CO2 (g )  3 H2O(l)


9
2 −2058
Benzene, C6H6(l) +49.0 −3268
Ethane, C2H6(g) −84.7 −1560 Self-test 2C.1 Calculate the standard enthalpy of hydrogena-
Glucose, C6H12O6(s) −1274 −2808 tion of liquid benzene from its standard enthalpy of combus-
Methane, CH4(g) −74.8 −890 tion (−3268 kJ mol−1) and the standard enthalpy of combustion
Methanol, CH3OH(l) −238.7 −726 of liquid cyclohexane (−3920 kJ mol−1).
Answer: −206 kJ mol−1
* More values are given in the Resource section.
2C Thermochemistry 53

Table 2C.4 Standard enthalpies of formation of inorganic


Exercises compounds at 298 K*
⦵ −1
E2C.1 For tetrachloromethane, ΔvapH = 30.0 kJ mol . Calculate q and ΔH
Δf H /(kJ mol−1)

when 0.75 mol CCl4(l) is vaporized at 250 K and 1 bar.


E2C.2 When 120 mg of naphthalene, C10H8(s), was burned in a bomb H2O(l) −285.83
calorimeter the temperature rose by 3.05 K. Calculate the calorimeter H2O(g) −241.82
constant. By how much will the temperature rise when 150 mg of phenol, NH3(g) −46.11
C6H5OH(s), is burned in the calorimeter under the same conditions?
N2H4(l) +50.63
(ΔcH ⦵(C10H8,s) = −5157 kJ mol−1, ΔcH ⦵(C6H5OH,s) = −3058 kJ mol−1)

NO2(g) +33.18
E2C.3 Given the reactions (1) and (2) below, determine ΔrH for reaction (3)
at 298 K. N2O4(g) +9.16
NaCl(s) −411.15
(1) H2(g) + Cl2(g) → 2 HCl(g) ΔrH ⦵ = −184.62 kJ mol−1 KCl(s) −436.75
⦵ −1
(2) 2 H2(g) + O2(g) → 2 H2O(g) ΔrH = −483.64 kJ mol * More values are given in the Resource section.
(3) 4 HCl(g) + O2(g) → 2 Cl2(g) + 2 H2O(g)
Table 2C.5 Standard enthalpies of formation of organic
compounds at 298 K*

Standard enthalpies of
2C.2 Δf H /(kJ mol−1)

formation CH4(g) −74.81


C6H6(l) +49.0
The standard enthalpy of formation, ΔfH , of a substance is the

C6H12(l) −156
standard reaction enthalpy for the formation of the compound CH3OH(l) −238.66
from its elements in their reference states: CH3CH2OH(l) −277.69

The reference state of an element is its most stable state at * More values are given in the Resource section.
the specified temperature and 1 bar.
For example, at 298 K the reference state of nitrogen is a gas of For example, if the enthalpy of formation of HBr(aq) is found to
N2 molecules, that of mercury is liquid mercury, that of carbon be −122 kJ mol−1, then the whole of that value is ascribed to the
formation of Br−(aq), and Δf H (Br−, aq) = −122 kJ mol−1. That

is graphite, and that of tin is the white (metallic) form. There is


one exception to this general prescription of reference states: value may then be combined with, for instance, the enthalpy of
formation of AgBr(aq) to determine the value of Δf H (Ag+, aq),

the reference state of phosphorus is taken to be white phospho-


rus despite this allotrope not being the most stable form but and so on. In essence, this definition adjusts the actual values of
simply the most reproducible form of the element. Standard the enthalpies of formation of ions by a fixed value, which is cho-
enthalpies of formation are expressed as enthalpies per mole of sen so that the standard value for one of them, H+(aq), is zero.
molecules or (for ionic substances) formula units of the com- Conceptually, a reaction can be regarded as proceeding by
pound. The standard enthalpy of formation of liquid benzene at decomposing the reactants into their elements in their refer-
298 K, for example, refers to the reaction ence states and then combining those elements into the prod-
ucts. The value of Δr H for the overall reaction is the sum of

6 C(s,graphite)  3 H2 (g )  C 6 H6 (l) these ‘unforming’ and forming enthalpies. Because ‘unforming’


and is +49.0 kJ mol−1. The standard enthalpies of formation is the reverse of forming, the enthalpy of an unforming step is
of elements in their reference states are zero at all tempera- the negative of the enthalpy of formation (3). Hence, in the en-
tures because they are the enthalpies of such ‘null’ reactions thalpies of formation of substances, there is enough informa-
as N2(g) → N2(g). Some enthalpies of formation are listed in tion to calculate the enthalpy of any reaction by using
Tables 2C.4 and 2C.5 and a much longer list will be found in r H    H

⦵ 

f
the Resource section. Products Standard reaction enthallpy
[practical implementation] 
(2C.5a)
The standard enthalpy of formation of ions in solution    f H

poses a special problem because it is not possible to prepare Reactants

a solution of either cations or anions alone. This problem


is overcome by defining one ion, conventionally the hydro- Elements
gen ion, to have zero standard enthalpy of formation at all
Enthalpy, H

­temperatures: 3
Reactants
 Ions in solution ΔrH ⦵
 f H (H , aq )  0

[convention] (2C.4) Products


54 2 The First Law

where in each case the enthalpies of formation of the species


that occur are multiplied by their stoichiometric coefficients.
This procedure is the practical implementation of the formal
definition in eqn 2C.3. A more sophisticated way of expressing Products rH °(T2)

Enthalpy, H
the same result is to introduce the stoichiometric numbers n J
(as distinct from the stoichiometric coefficients) which are pos-
itive for products and negative for reactants. Then rH °(T1)
Reactants

 r H   J  f H (J)(2C.5b)

 ⦵

Stoichiometric numbers, which have a sign, are denoted n J or T1


Temperature, T
T2
n (J). Stoichiometric coefficients, which are all positive, are de-
noted simply n (with no subscript). Figure 2C.1 When the temperature is increased, the enthalpy of
the products and the reactants both increase, but may do so to
different extents. In each case, the change in enthalpy depends
Brief illustration 2C.1 on the heat capacities of the substances. The change in reaction
enthalpy reflects the difference in the changes of the enthalpies
According to eqn 2C.5a, the standard enthalpy of the reaction of the products and reactants.
2 HN3(l) + 2 NO(g) → H2O2(l) + 4 N2(g) is calculated as follows:

 r H   { f H  (H2O2 ,l)  4  f H  (N2 ,g )}


⦵ ⦵ ⦵

{2 f H  (HN3 ,l)  2 f H  (NO,g)} It follows from eqn 2B.6a (dH = CpdT) that, when a substance
⦵ ⦵

 {187.78  4  (0)} kJmol 1 is heated from T1 to T2, its enthalpy changes from H(T1) to
 {2  (264.0)  2  (90.25)} kJmol 1 T2
1
H (T2 )  H (T1 )   C p dT (2C.6)
 896.3 kJmol T1

To use eqn 2C.5b, identify n(HN3) = −2, n(NO) = −2, n(H2O2) = (It has been assumed that no phase transition takes place in the
+1, and n(N2) = +4, and then write temperature range of interest.) Because this equation applies to
each substance in the reaction, the standard reaction enthalpy
 r H   f H (H2O2 ,l)  4  f H (N2 ,g )

⦵ 
⦵ 

changes from Δr H (T1) to


2 f H (HN3 ,l)  2 f H (NO,g)



⦵ 

T2
 r H (T2 )   r H (T1 )    r C p dT

 ⦵
 ⦵

which gives the same result. Kirchhoff’s law(2C.7a)
T1

Where ∆ r C p is the difference of the molar heat capacities of prod-



Exercises ucts and reactants under standard conditions weighted by the


−1
stoichiometric coefficients that appear in the chemical equation:
E2C.4 The standard enthalpy of formation of ethylbenzene is −12.5 kJ mol .
Calculate the standard enthalpy of combustion of ethylbenzene. The standard
r C p   C   C

 ⦵
 ⦵

p ,m p ,m (2C.7b)
enthalpies of formation of CO2(g) and H2O(l) are −393.5 kJ mol−1 and
Products Reactants
−285.3 kJ mol−1 respectively. All data are at 298 K.
E2C.5 Given that the standard enthalpy of formation of HCl(aq) is or, in the notation of eqn 2C.5b,
−1 ⦵ −
−167 kJ mol , what is the value of Δf H (Cl , aq)?
 r C p   JC p ,m (J)(2C.7c)

⦵ 

The temperature dependence of


2C.3 Equation 2C.7a is known as Kirchhoff ’s law. It is normally a
reaction enthalpies good approximation to assume that ∆ r C p is independent of the

temperature, at least over reasonably limited ranges. Although


Many standard reaction enthalpies have been measured at dif- the individual heat capacities might vary, their difference varies
ferent temperatures. However, in the absence of this informa- less significantly. In some cases the temperature dependence of
tion, standard reaction enthalpies at different temperatures can heat capacities is taken into account by using eqn 2C.7a. If ∆ r C p
⦵

be calculated from heat capacities and the reaction enthalpy at is largely independent of temperature in the range T1 to T2, the
some other temperature (Fig. 2C.1). In many cases heat capac- integral in eqn 2C.7a evaluates to (T2  T1 ) r C p and that equa-
⦵

ity data are more accurate than reaction enthalpies. Therefore, tion becomes
providing the information is available, the procedure about to Integrated
be described is more accurate than the direct measurement of a  r H (T2 )   r H (T1 )   r C p (T2  T1 )

 ⦵
 ⦵

form of (2C.7d)
reaction enthalpy at an elevated temperature. Kirchhoff’’s law
2C Thermochemistry 55

Example 2C.2 ‘­differential’ refers to the fact that measurements on a sample


Using Kirchhoff’s law
are compared to those on a reference material that does not
The standard enthalpy of formation of H2O(g) at 298 K is undergo a physical or chemical change during the analysis.
−241.82 kJ mol−1. Estimate its value at 100 °C (373 K) given The term ‘scanning’ refers to the fact that the temperatures of
the following values of the molar heat capacities at constant the sample and reference material are increased, or scanned,
pressure: H2O(g): 33.58 J K−1 mol−1; H2(g): 28.84 J K−1 mol−1; during the analysis.
O2(g): 29.37 J K−1 mol−1. Assume that the heat capacities are A DSC consists of two small compartments that are heated
independent of temperature. electrically at a constant rate. The temperature, T, at time t dur-
Collect your thoughts When ∆ rC p is independent of tempera-

⦵ ing a linear scan is T = T0 + αt, where T0 is the initial temperature
ture in the range T1 to T2, you can use the integrated form and α is the scan rate. A computer controls the electrical power
of the Kirchhoff equation, eqn 2C.7d. To proceed, write the supply that maintains the same temperature in the sample and
chemical equation, identify the stoichiometric coefficients, reference compartments throughout the analysis (Fig. 2C.2).
and calculate ∆ rC p from the data.
⦵
If no physical or chemical change occurs in the sample at
temperature T, the heat transferred to the sample is written
The solution The reaction is H2 (g )  12 O2 (g )  H2O(g ), so
as qp = CpΔT, where ΔT = T − T0 and Cp is assumed to be in-
dependent of temperature. Because T = T0 + αt, it follows that

⦵ 


 rC p  C p ,m (H2O,g )  C p ,m (H2 ,g )  12 C p ,m (O2 ,g )

⦵ 

 ΔT = αt. If a chemical or physical process takes place, the en-
1 1
 9.94 JK mol ergy required to be transferred as heat to attain the same change
It then follows that in temperature of the sample as the control is qp + qp,ex.
The quantity qp,ex is interpreted in terms of an apparent
 r H (373 K )  241.82 kJmo11  (75 K )  (9.94 JK 1 mol 1 )


change in the heat capacity at constant pressure, from Cp to
 242.6 kJmol 1 Cp + Cp,ex of the sample during the temperature scan:

Self-test 2C.2 Estimate the standard enthalpy of formation of q p ,ex q p ,ex Pex
C p ,ex    (2C.8)
cyclohexane, C6H12(l), at 400 K from the data in Table 2C.5 T t 
and heat capacity data given in the Resource section. where Pex = qp,ex/t is the excess electrical power necessary to
Answer: −163 kJ mol−1
equalize the temperature of the sample and reference compart-
ments. A DSC trace, also called a thermogram, consists of a
plot of Cp,ex against T (Fig. 2C.3). The enthalpy change associ-
ated with the process is
Exercises
T2
H   C p ,ex dT (2C.9)

E2C.6 Calculate Δr H at (i) 298 K, (ii) 478 K for the reaction
C(graphite) + H2O(g) → CO(g) + H2(g). The standard enthalpies T1

of formation at 298 K of CO(g) and H2O(g) are −110.53 kJ mol−1


and −241.82 kJ mol−1 respectively; the heat capactities at 298 K are where T1 and T2 are, respectively, the temperatures at which the
 
C⦵p ,m(CO,g) = 29.14 J K −1 mol −1 , C⦵p ,m(H2 ,g ) = 28.824 J K −1 mol −1 , process begins and ends. This relation shows that the enthalpy
C⦵p ,m(H2O,g )  33.58 J K 1 mol 1 , C⦵p ,m(graphite,s)  8.527 J K 1 mol 1. Assume change is equal to the area under the plot of Cp,ex against T.
all heat capacities to be constant over the temperature range of interest.
−1
E2C.7 The standard enthalpy of formation of PCl3(g) is −287 kJ mol and that of
PCl5(g) is −375 kJ mol−1. The heat capacities of PCl3(g), PCl5(g), and Cl2(g) are
71.8 J K−1 mol−1, 113 J K−1 mol−1, and 33.9 J K−1 mol−1 respectively. All these data are Thermocouples
at 298 K. Calculate Δr H ⦵ for PCl3(g) + Cl2(g) → PCl5(g) at 298 K and at 473 K.
Sample Reference

2C.4 Experimental techniques


The classic tool of thermochemistry is the calorimeter (Topics 2A
and 2B). However, technological advances have been made that
allow measurements to be made on samples with mass as little as
a few milligrams.
Heaters

2C.4(a) Differential scanning calorimetry Figure 2C.2 A differential scanning calorimeter. The sample
and a reference material are heated in separate but identical
A differential scanning calorimeter (DSC) measures the metal heat sinks. The output is the difference in power needed
energy transferred as heat to or from a sample at constant to maintain the heat sinks at equal temperatures as the
pressure during a physical or chemical change. The term temperature rises.
56 2 The First Law

9 Reference Injector Sample


cell cell

6
Cp,ex /(mJ K–1)

Heater Heater

0
30 45 60 75 90
Temperature, θ/°C Temperature comparison

Figure 2C.3 A thermogram for the protein ubiquitin at pH = 2.45. Figure 2C.4 A schematic diagram of the apparatus used for
The protein retains its native structure up to about 45 °C and then isothermal titration calorimetry.
undergoes an endothermic conformational change. (Adapted
from B. Chowdhry and S. LeHarne, J. Chem. Educ. 74, 236 (1997).)

(a)

(a) Power, (b) H


2C.4(b) Isothermal titration calorimetry (b)

Isothermal titration calorimetry (ITC) is also a ‘differen-


tial’ technique in which the thermal behaviour of a sample is
compared with that of a reference. The apparatus is shown in
Fig. 2C.4. One of the thermally conducting vessels, which have a
volume of a few cubic centimetres, contains the reference (water
for instance) and a heater rated at a few milliwatts. The second Time
vessel contains one of the reagents, such as a solution of a macro-
molecule with binding sites; it also contains a heater. At the start Figure 2C.5 (a) The record of the power applied as each injection
is made, and (b) the sum of successive enthalpy changes in the
of the experiment, both heaters are activated, and then precisely
course of the titration.
determined amounts (of volume of about a cubic millimetre) of
the second reagent are added to the sample cell. The power re-
quired to maintain the same temperature differential with the ref- s­ olution is V and the molar concentration of unreacted reagent
erence cell is monitored. If the reaction is exothermic, less power A is ci at the time of the ith injection, then the change in its
is needed; if it is endothermic, then more power must be supplied. concentration at that injection is Δci and the heat generated (or
Figure 2C.5 shows an example of the power needed to main- absorbed) by the reaction is VΔrHΔci = qi. The sum of all such
tain the temperature differential: from the power and the length quantities, given that the sum of Δci is the known initial con-
of time, Δt, for which it is supplied, the heat supplied, qi, for centration of the reactant, can then be interpreted as the value
the injection i can be calculated from qi = PiΔt. If the volume of of ΔrH for the reaction.

Checklist of concepts
☐ 1. The standard enthalpy of transition is equal to the ☐ 5. Standard enthalpies of formation are defined in terms
energy transferred as heat at constant pressure in the of the reference states of elements.
transition under standard conditions. ☐ 6. The reference state of an element is its most stable state
☐ 2. The standard state of a substance at a specified tem- at the specified temperature and 1 bar.
perature is its pure form at 1 bar. ☐ 7. The standard reaction enthalpy is expressed as the
☐ 3. A thermochemical equation is a chemical equation difference of the standard enthalpies of formation of
and its associated change in enthalpy. products and reactants.
☐ 4. Hess’s law states that the standard reaction enthalpy is ☐ 8. The temperature dependence of a reaction enthalpy is
the sum of the values for the individual reactions into expressed by Kirchhoff’s law.
which the overall reaction may be divided.
2C Thermochemistry 57

Checklist of equations
Property Equation Comment Equation number

  H   H
  
The standard reaction enthalpy ⦵
r H  f

 f

n are stoichiometric coefficients 2C.5a
Products Reactants

 r H ⦵   J  f H ⦵ (J)
 
nJ (signed) are stoichiometric numbers 2C.5b
J
T2
 r H ⦵ (T2 )   r H ⦵ (T1 )    r C⦵p dT
  
Kirchhoff ’s law General form 2C.7a
T1

 r C p⦵   JC ⦵p ,m (J)
 
2C.7c
J


⦵ 

 r H (T2 )   r H (T1 )  (T2  T1 ) r C p

⦵ 
∆ r C p⦵ independent of temperature 2C.7d
TOPIC 2D 
State functions and exact
­differentials
to a state f. In this state the system has an internal energy Uf and
➤ Why do you need to know this material? the work done on the system as it changes along Path 1 from i
to f is w1. Notice the use of language: U is a property of the state;
Thermodynamics has the power to provide relations
w is a property of the path. Now consider another process, Path 2,
between a variety of properties. This Topic introduces its
in which the initial and final states are the same as those in
principal procedure, the manipulation of equations involv-
Path 1 but in which the expansion is not adiabatic. The internal
ing state functions.
energy of both the initial and the final states are the same as be-
➤ What is the key idea? fore (because U is a state function). However, in the second path
an energy q2 enters the system as heat and the work w2 is not the
The fact that internal energy and enthalpy are state
same as w1. The work and the heat are path functions.
functions leads to relations between thermodynamic
If a system is taken along a path, U changes from Ui to Uf , and
properties.
the overall change is the sum (integral) of all the infinitesimal
➤ What do you need to know already? changes along the path:
You need to be aware that the internal energy and enthal- f

py are state functions (Topics 2B and 2C) and be familiar U   dU(2D.1)


i
with the concept of heat capacity. You need to be aware
The value of ΔU depends on the initial and final states of the
of how to write an overall change in a property when two
system but is independent of the path between them. This path-
variables change (The chemist’s toolkit 2A.2).
independence of the integral is expressed by saying that dU is
an ‘exact differential’. In general, an exact differential is an in-
finitesimal quantity that, when integrated, gives a result that is
independent of the path between the initial and final states.
A state function is a property that depends only on the current When a system is heated, the total energy transferred as heat is
state of a system and is independent of its history. The internal the sum of all individual contributions at each point of the path:
energy and enthalpy are two examples. Physical quantities with f
values that do depend on the path between two states are called q dq(2D.2)
i,path
path functions. Examples of path functions are the work and the
heating that are done when preparing a state. It is not appropriate
to speak of a system in a particular state as possessing work or
energy, U

heat. In each case, the energy transferred as work or heat relates


Internal

to the path being taken between states, not the current state itself. i Path 1
A part of the richness of thermodynamics is that it uses the (w ≠ 0, q = 0)
mathematical properties of state functions to draw far-reaching Path 2
conclusions about the relations between physical properties. (w ≠ 0, q ≠ 0)
In this way it establishes connections that may be completely
unexpected. The practical importance of this ability is the pos-
sibility of combining measurements of different properties to f
obtain the value of a desired property. Temperature, T
Volume, V

2D.1 Exact and inexact differentials Figure 2D.1 As the volume and temperature of a system are
changed, the internal energy changes. An adiabatic and a non-
Consider a system undergoing the changes depicted in Fig. 2D.1. adiabatic path are shown as Path 1 and Path 2, respectively:
The initial state of the system is i and in this state the internal en- they correspond to different values of q and w but to the same
ergy is Ui. Work is done by the system as it expands adiabatically value of ΔU.
2D State functions and exact d
­ ifferentials 59

Notice the differences between this equation and eqn 2D.1.


First, the result of integration is q and not Δq, because q is not
2D.2 Changes in internal energy
a state function and the energy supplied as heat cannot be ex- Consider a closed system of constant composition3 (the only
pressed as qf − qi. Secondly, the path of integration must be type of system considered in the rest of this Topic). The inter-
specified because q depends on the path selected (e.g. an adi- nal energy U can be regarded as a function of V, T, and p, but,
abatic path has q = 0, whereas a non-adiabatic path between the because there is an equation of state that relates these quantities
same two states would have q ≠ 0). This path dependence is ex- (Topic 1A), choosing the values of two of the variables fixes the
pressed by saying that dq is an ‘inexact differential’. In general, value of the third. Therefore, it is possible to write U in terms of
an inexact differential is an infinitesimal quantity that, when just two independent variables: V and T, p and T, or p and V.
integrated, gives a result that depends on the path between the Expressing U as a function of volume and temperature turns
initial and final states. Often dq is written đq to emphasize that out to result in the simplest expressions.
it is inexact and requires the specification of a path.
The work done on a system to change it from one state to
another depends on the path taken between the two speci- 2D.2(a) General considerations
fied states. For example, in general the work is different if the According to The chemist’s toolkit 2A.2, because the internal
change takes place adiabatically and non-adiabatically. It fol- energy is a function of the volume and the temperature, when
lows that dw is an inexact differential. It is often written đw. these two quantities change infinitesimally, the internal energy
changes by
General expression
Example 2D.1  U   U 
Calculating work, heat, and change in dU    dV    dT nge in U (2D.3)
for a chan
internal energy  V T  T V with T and V
The interpretation of this equation is that, in a closed system of
Consider a perfect gas inside a cylinder fitted with a piston.
Let the initial state be T,Vi and the final state be T,Vf. The
constant composition, any infinitesimal change in the internal
change of state can be brought about in many ways, of which energy is proportional to the infinitesimal changes of volume
two possibilities are the following: and temperature, the coefficients of proportionality being the
two partial derivatives (Fig. 2D.2).
• Path A, in which there is free expansion against zero exter-
In many cases partial derivatives have a straightforward
nal pressure;
physical interpretation, and thermodynamics gets difficult only
• Path B, in which there is reversible, isothermal expansion. when that interpretation is not kept in sight. The term (∂U/∂T)V
Calculate w, q, and ΔU for each process. should be recognized as the constant-volume heat capacity
(Topic 2A), CV . The other coefficient, (∂U/∂V)T , denoted πT ,
Collect your thoughts To find a starting point for a calculation
plays a major role in thermodynamics because it is a measure of
in thermodynamics, it is often a good idea to go back to first
the variation of the internal energy of a substance as its volume
principles and to look for a way of expressing the quantity to
is changed at constant temperature (Fig. 2D.3). Because πT has
be calculated in terms of other quantities that are easier to
the same dimensions as pressure but arises from the interac-
calculate. In this example the gas is perfect, meaning that (as
explained in Topic 2A) the internal energy depends only on
tions between the molecules within the sample, it is called the
the temperature and is independent of the volume occupied internal pressure:
by the molecules. Therefore, ΔU = 0 for Path B, which is an  U  Internal pressure
isothermal process. Path A is a free expansion (pex = 0), so no T    [definition]
(2D.4)
 V T
work is done and w = 0 (see Topic 2A). The internal energy is
independent of the volume, so ΔU = 0 for Path A. Once ΔU In terms of CV and πT , eqn 2D.3 can now be written
and q are known, use the First Law, ΔU = q + w, to find q. dU   T dV  CV dT (2D.5)
The solution Because ΔU = 0 for both paths and ΔU = q + w,
Insight into the physical significance of the internal pressure
in each case q = −w. The work of free expansion is zero
comes from consideration of the behaviour of gases. Consider a
(eqn 2A.7 of Topic 2A, w = 0); so in Path A, w = 0 and therefore
perfect gas, in which there are no interactions between the mol-
q = 0 too. For Path B, the work is given by eqn 2A.9 of Topic 2A
ecules. Then the internal energy is independent of their sepa-
(w = −nRT ln(Vf/Vi)) and consequently q = nRT ln(Vf/Vi).
ration and hence independent of the volume occupied by the
Self-test 2D.1 Calculate the values of q, w, and ΔU for an gas, implying that πT = 0. In a real gas, molecular i­nteractions
irreversible isothermal expansion of a perfect gas against a
constant non-zero external pressure.
3
Answer: q = pexΔV, w = −pexΔV, ΔU = 0 Constant composition means that the amount of each chemical species
present is fixed and does not change.
60 2 The First Law

energy, U
Repulsions

Internal
dominant, πT < 0

Internal energy, U
U
Perfect gas

U+ ( U
V )
T
( T )
dV + U dT
V
Attractions
dominant, πT > 0
dT Temperature, T
Volume, V Volume, V
dV
Figure 2D.4 For a perfect gas, the internal energy is independent
of the volume (at constant temperature). If attractions are
Figure 2D.2 An overall change in U, which is denoted dU, arises dominant in a real gas, the internal energy increases with volume
when both V and T are allowed to change. because the molecules become farther apart on average. If
repulsions are dominant, the internal energy decreases as the
gas expands.
energy, U
Internal

πT
U
High pressure
gas

Vacuum
Temperature, T
Volume, V
dV

Figure 2D.5 A schematic diagram of the apparatus used by Joule


Figure 2D.3 The internal pressure, πT , is the slope of U with in an attempt to measure the change in internal energy when a
respect to V with the temperature T held constant. gas expands into a vacuum.

become more important as the volume occupied by the gas heat because the temperature of the bath did not change, so
decreases. If attractive interactions dominate, then because an q = 0. Consequently, within the accuracy of the experiment,
increase in volume increases the average separation of the mol- ΔU = 0. Joule concluded that U does not change when a gas
ecules, the favourable interactions are weakened and the inter- expands isothermally and therefore that πT = 0. His experi-
nal energy rises, implying that πT > 0 (Fig. 2D.4). If repulsions ment, however, was crude. The heat capacity of the apparatus
are dominant, an increase in volume weakens them and the the was so large that the temperature change, which would in fact
internal energy decreases, implying that πT < 0. occur for a real gas, is simply too small to measure. Joule had
James Joule thought that he could measure πT by observing extracted an essential limiting property of a gas, a property of
the change in temperature of a gas when it is allowed to expand a perfect gas, without detecting the small deviations charac-
into a vacuum. He used two metal vessels immersed in a water teristic of real gases.
bath (Fig. 2D.5). One was filled with air at about 22 atm and
the other was evacuated. He then tried to measure the change Changes in internal energy at
2D.2(b)
in temperature of the water of the bath when a stopcock was
opened and the air expanded into a vacuum. He observed no
constant pressure
change in temperature. Partial derivatives have many useful properties which are re-
The thermodynamic implications of the experiment are as viewed in more in more detail in Part 1 of the Resource section.
follows. No work was done in the expansion into a vacuum, Skilful use of them can often turn some unfamiliar quantity into
so w = 0. No energy entered or left the system (the gas) as a quantity that can be recognized, interpreted, or measured.
2D State functions and exact d
­ ifferentials 61

As an example, to find how the internal energy varies with Example 2D.2 Calculating the expansion coefficient
temperature when the pressure rather than the volume of
of a gas
the system is kept constant, begin by dividing both sides of
eqn 2D.5 (dU   T dV  CV dT ) by dT to give Derive an expression for the expansion coefficient of a perfect gas.
dU dV Collect your thoughts The expansion coefficient is defined in
 T  CV
dT dT eqn 2D.6. To use this expression, you need to substitute the
expression for V in terms of T obtained from the equation of
Now impose the condition of constant pressure on the resulting state for the gas. As implied by the subscript in eqn 2D.6, the
differentials, so that dU/dT on the left becomes (∂U/∂T)p. At pressure, p, is treated as a constant.
this stage the equation becomes
The solution Because pV = nRT, write
U V
= πT + CV
T p
T p
pV = nRT pV= nRT

1 V 1 (nRT / p) 1 nR nR nR 1
As already emphasized, it is usually sensible in thermodynam- α= = = × = = =
V T p
V T p
V p pV nRT T
ics to inspect the output of a manipulation to see if it contains
any recognizable physical quantity. The partial derivative in the
The physical interpretation of this result is that the higher the
box on the right in this expression is the slope of the plot of
temperature, the less responsive is the volume of a perfect gas
volume against temperature (at constant pressure). This prop- to a change in temperature.
erty is normally tabulated as the expansion coefficient, α, of a
substance, which is defined as Self-test 2D.2 Derive an expression for the isothermal com-
pressibility of a perfect gas.
1  V  Expansion coefficient

Answer: κT = 1/p
(2D.6)
V  T  p [definition]

and physically is the fractional change in volume that accom-


panies a rise in temperature. A large value of α means that the Introduction of the definition of α into the equation for
volume of the sample responds strongly to changes in tem- (∂U /∂T ) p gives
perature. Table 2D.1 lists some experimental values of α. For
 U 
future reference, it also lists the isothermal compressibility, κT  T    T V  CV (2D.8)
(kappa), which is defined as  p
This equation is entirely general (provided the system is closed
1  V  Isothermal compressibility
T     [definition]
(2D.7) and its composition is constant). It expresses the dependence
V  p T of the internal energy on the temperature at constant pressure
The isothermal compressibility is a measure of the fractional in terms of CV , which can be measured in one experiment, in
change in volume when the pressure is increased; the nega- terms of α, which can be measured in another, and in terms of
tive sign in the definition ensures that the compressibility the internal pressure πT . For a perfect gas, πT = 0, so then
is a positive quantity, because an increase of pressure, im-
 U 
plying a positive dp, brings about a reduction of volume, a  T   CV [perfect gas](2D.9)
negative dV.  p
That is, although the constant-volume heat capacity is defined
Table 2D.1 Expansion coefficients (α) and isothermal as the slope of a plot of internal energy against temperature at
compressibilities (κT) at 298 K* constant volume, for a perfect gas CV is also the slope of a plot
of internal energy against temperature at constant pressure.
α/(10−4 K−1) κT/(10−6 bar−1) Equation 2D.9 provides an easy way to derive the relation be-
Liquids: tween Cp and CV for a perfect gas. They differ, as explained in
  Benzene 12.4 92.1 Topic 2B, because some of the energy supplied as heat escapes
  Water 2.1 49.6 back into the surroundings as work of expansion when the vol-
Solids:
ume is not constant. First, write
Definition
  Diamond 0.030 0.187 of C p eqn 2 D.9
 
  Lead 0.861 2.21  H   U 
C p  CV     
* More values are given in the Resource section.  T  p  T  p
62 2 The First Law

The definition of H is H = U + pV, and for a perfect gas pV = nRT,


so H = U + nRT. This result is used in the first term to obtain
2D.3 Changes in enthalpy
 (U  nRT )   U  A similar set of operations can be carried out on the enthalpy,
C p  CV      T   nR(2D.10) H = U + pV. The quantities U, p, and V are all state functions;
 T p  p
therefore H is also a state function and dH is an exact differ-
The general result for any substance (the proof makes use of the ential. It turns out that H is a useful thermodynamic function
Second Law, which is introduced in Focus 3) is when the pressure can be controlled: a sign of that is the re-
 2TV lation ΔH = qp (eqn 2B.2b). Therefore, H can be regarded as a
C p  CV  (2D.11)
function of p and T, and the argument in Section 2D.2 for the
T
variation of U can be adapted to find an expression for the vari-
This relation reduces to eqn 2D.10 for a perfect gas when ation of H with temperature at constant volume.
α = 1/T and κT = 1/p
1/(1/p )

/T 2
1 
nRT
1 pTV pV How is that done? 2D.1 Deriving an expression for the
C p  CV   2   TV  2   nR
T T T variation of enthalpy with pressure and temperature
Because expansion coefficients α of liquids and solids are small, Consider a closed system of constant composition. Because
it is tempting to deduce from eqn 2D.11 that for them Cp ≈ CV . H is a function of p and T, when these two quantities change
But this is not always so, because the compressibility κT might infinitesimally, the enthalpy changes by
also be small, so α2/κT might be large. That is, although pushing
back the atmosphere requires only a little work, a great deal of H H
dH = dp + dT
p T
work may have to be done to pull atoms apart from one another T p

as the solid expands. The second (boxed) partial derivative is Cp, so


H
dH = dp + C p dT
Brief illustration 2D.1 p T

The remaining task is to express (∂H/∂p)T in terms of recog-


The expansion coefficient and isothermal compressibility of
nizable quantities. When the enthalpy is constant, dH = 0 and
water at 25 °C are given in Table 2D.1 as 2.1 × 10−4 K−1 and
the last equation becomes
49.0 × 10 −6 bar−1 (4.90 × 10 −10 Pa−1), respectively. The molar
volume of water at that temperature, Vm = M/ρ (where ρ  H 
  dp  C pdT at constant H
is the mass density), is 18.1 cm3 mol−1 (1.81 × 10−5 m3 mol−1).  p T
Therefore, from eqn 2D.11, the difference in molar heat capac- Division of both sides by dp then gives
ities (which is given by using Vm in place of V) is
H dT
= −Cp at constant H
4 1 2 5
(2.1  10 K )  (298 K )  (1.81  10 m mol ) 3 1 p T
dp
C p,m  CV , m 
4.90  1010 Pa 1 The constraint of constant enthalpy is incorporated by writing
 0.485 Pam3 K 1 mol 1  0.485 JK 1 mol 1 the (boxed) derivative on the right as a partial derivative with
H held constant:
For water the experimental values are Cp,m = 75.3 J K−1 mol−1, so
CV,m = 74.8 J K−1 mol−1. In some cases, the two heat capacities H T
= –Cp
p p
differ by as much as 30 per cent. T H

The (boxed) partial derivative on the right is now identified as


the Joule–Thomson coefficient, μ (mu):
Exercises μ =
 ∂T 

Joule–Thomson coefficient
 (2D.12)
[definition]
E2D.1 Estimate the internal pressure of water vapour at 1.00 bar and 400 K,  ∂p  H
treating it as a van der Waals gas, for which  T  a/Vm2 . You may simplify the It follows that
problem by assuming that the molar volume can be predicted from the perfect
gas equation; take the value of a to be 5.464 dm6 atm mol−2. H
= − Cp μ
p
E2D.2 For a van der Waals gas,  T  a/V . Assuming that this relation
2 T
m
applies, calculate ΔUm for the isothermal expansion of nitrogen gas from an This relation can now be substituted into the expression for
initial volume of 1.00 dm3 to 20.00 dm3 at 298 K ; take the value of a to be
1.352 dm6 atm mol−2.
dH, to give
(2D.13)
−5
E2D.3 The isothermal compressibility, κT , of water at 293 K is 4.96 × 10 atm .
−1
dH = −μCpdp + CpdT
The variation of enthalpy with
Calculate the pressure that must be applied in order to increase its density by
temperature and pressure
0.10 per cent.
2D State functions and exact ­differentials 63

How the Joule–Thomson coefficient is measured and a process


is made isenthalpic is explained in A deeper look 2D.1, available
Gas at
in the e-book accompanying this text. Thermocouples low
pressure
Brief illustration 2D.2
Porous
barrier
The Joule–Thomson coefficient for nitrogen at 298 K and 1 bar
(Table 2D.2) is +0.27 K bar−1. (Note that μ is an intensive prop-
erty.) It follows that the change in temperature the gas under-
goes when its pressure changes by −10 bar under isenthalpic
Gas at
(constant enthalpy) conditions is
high pressure
T  p  (0.27 K bar 1 )  (10 bar )  2.7 K
Figure 2D.6 The apparatus used for measuring the Joule–
Thomson effect. The gas expands through the porous barrier,
which acts as a throttle, and the whole apparatus is thermally
2D.4 The Joule–Thomson effect insulated. As explained in A deeper look 2D.1, this arrangement
corresponds to an isenthalpic expansion (expansion at constant
The analysis of the Joule–Thomson coefficient is central to enthalpy). Whether the expansion results in a heating or a cooling
the technological problems associated with the liquefaction of the gas depends on the conditions.
of gases. To determine the coefficient, it is necessary to meas-
ure the ratio of the temperature change to the change of pres-
sure, ΔT/Δp, in a process at constant enthalpy. The strategy
Cold gas
for imposing the constraint of constant enthalpy, so that the
expansion is isenthalpic, was developed by James Joule and
William Thomson (later Lord Kelvin). They let a gas expand Heat
exchanger
through a porous barrier from one constant pressure to an-
other and monitored the difference of temperature that arose
from the expansion (Fig. 2D.6). The change of temperature that
they ­observed as a result of isenthalpic expansion is called the
Throttle
Joule–Thomson effect.
The ‘Linde refrigerator’ makes use of the Joule–Thomson ef- Liquid
fect to liquefy gases (Fig. 2D.7). The gas at high pressure is al-
Compressor
lowed to expand through a throttle; it cools and is circulated
past the incoming gas. That gas is cooled, and its subsequent Figure 2D.7 The principle of the Linde refrigerator is shown in
expansion cools it still further. There comes a stage when the this diagram. The gas is recirculated, and so long as it is beneath
circulating gas becomes so cold that it condenses to a liquid. its inversion temperature it cools on expansion through the
For a perfect gas, μ = 0; hence, the temperature of a perfect throttle. The cooled gas cools the high-pressure gas, which cools
gas is unchanged by Joule–Thomson expansion. This charac- still further as it expands. Eventually liquefied gas drips from the
teristic points clearly to the involvement of intermolecular throttle.
forces in determining the size of the effect.
Real gases have non-zero Joule–Thomson coefficients.
Depending on the identity of the gas, the pressure, the ­relative magnitudes of the attractive and repulsive intermolecular
forces, and the temperature, the sign of the coefficient may be
either positive or negative (Fig. 2D.8). A positive sign implies
Table 2D.2 Inversion temperatures (TI), normal freezing (Tf )
that dT is negative when dp is negative, in which case the gas
and boiling (Tb) points, and Joule–Thomson coefficients (μ) at
1 bar and 298 K*
cools on expansion. However, the Joule–Thomson coefficient
of a real gas does not necessarily approach zero as the pres-
TI/K Tf/K Tb/K μ /(K bar−1) sure is reduced even though the equation of state of the gas ap-
Ar 723 83.8 87.3 proaches that of a perfect gas. The coefficient behaves like the
CO2 1500 194.7 +1.11 at 300 K
properties discussed in Topic 1C in the sense that it depends on
derivatives and not on p, V, and T themselves.
He 40 4.22 −0.062
Gases that show a heating effect (μ < 0) at one temperature
N2 621 63.3 77.4 +0.27
show a cooling effect (μ > 0) when the temperature is below
* More values are given in the Resource section. their upper inversion temperature, TI (Table 2D.2, Fig. 2D.9).
64 2 The First Law

600
Upper
inversion
temperature
Heating

Temperature, T/K
Temperature, T

μ>0 400
Cooling Nitrogen
Cooling
μ>0
Heating
200
μ<0 Hydrogen Lower μ<0
inversion
temperature
Helium
0
Pressure, p 0 200 400
Pressure, p
Figure 2D.8 The sign of the Joule–Thomson coefficient, μ,
depends on the conditions. Inside the boundary, the blue Figure 2D.9 The inversion temperatures for three real gases:
area, it is positive and outside it is negative. The temperature nitrogen, hydrogen, and helium.
corresponding to the boundary at a given pressure is the
‘inversion temperature’ of the gas at that pressure. Reduction Topic 1C, molecules in a real gas attract each other (the attrac-
of pressure under adiabatic conditions moves the system
tion is not gravitational, but the effect is the same). It follows
along one of the isenthalps, or curves of constant enthalpy
that, if the molecules move apart from each other, like a ball ris-
(the blue lines). The inversion temperature curve runs through
the points of the isenthalps where their slope changes from
ing from a planet, then they should slow. It is very easy to move
negative to positive. molecules apart from each other by simply allowing the gas
to expand, which increases the average separation of the mol-
ecules. To cool a gas, therefore, expansion must occur without
As indicated in Fig. 2D.9, a gas typically has two inversion allowing any energy to enter from outside as heat. As the gas
temperatures. expands, the molecules move apart to fill the available volume,
The kinetic model of gases (Topic 1B) and the equipartition struggling as they do so against the attraction of their neigh-
theorem (Energy: A first look) jointly imply that the mean ki- bours. Because some kinetic energy must be converted into po-
netic energy of molecules in a gas is proportional to the tem- tential energy to reach greater separations, the molecules travel
perature. It follows that reducing the average speed of the more slowly as their separation increases, and the temperature
molecules is equivalent to cooling the gas. If the speed of the drops. The cooling effect, which corresponds to μ > 0, is ob-
molecules can be reduced to the point that neighbours can cap- served in real gases under conditions when attractive interac-
ture each other by their intermolecular attractions, then the tions are dominant (Z < 1, where Z is the compression factor
cooled gas will condense to a liquid. defined in eqn 1C.1, Z = Vm /Vmo ), because the molecules have
Slowing gas molecules makes use of an effect similar to that to climb apart against the attractive force in order for them to
seen when a ball is thrown up into the air: as it rises it slows travel more slowly. For molecules under conditions when re-
in response to the gravitational attraction of the Earth and its pulsions are dominant (Z > 1), the Joule–Thomson effect re-
kinetic energy is converted into potential energy. As seen in sults in the gas becoming warmer, or μ < 0.

Checklist of concepts
☐ 1. The quantity dU is an exact differential; dw and dq are not. ☐ 4. The change in internal energy with pressure and tem-
☐ 2. The change in internal energy may be expressed in perature is expressed in terms of the internal pressure
terms of changes in temperature and volume. and the heat capacity and leads to a general expression
for the relation between heat capacities.
☐ 3. The internal pressure is the variation of internal energy
with volume at constant temperature; for a perfect gas, ☐ 5. The Joule–Thomson effect is the change in temperature
the internal pressure is zero. of a gas when it undergoes isenthalpic expansion.
2D State functions and exact d
­ ifferentials 65

Checklist of equations
Property Equation Comment Equation number
Change in U(V,T) dU = (∂U/∂V)T dV + (∂U/∂T)V dT Constant composition 2D.3
Internal pressure πT = (∂U/∂V)T 2D.4
Change in U(V,T) dU = πT dV + CV dT Constant composition 2D.5
Expansion coefficient α = (1/V)(∂V/∂T)p 2D.6
Isothermal compressibility κT = −(1/V)(∂V/∂p)T 2D.7
Relation between heat capacities Cp − CV = nR Perfect gas 2D.10
Cp − CV = α2TV/κT General case 2D.11
Joule–Thomson coefficient μ = (∂T/∂p)H 2D.12
Change in H(p,T) dH = −μCpdp + CpdT Constant composition 2D.13
TOPIC 2E Adiabatic changes

U constant
➤ Why do you need to know this material? Ti 1 Ti,Vf
Ti,Vi
Adiabatic processes complement isothermal processes, 2

Temperature, T
and are used in the discussion of the Second Law of ther-

U = CV T
modynamics.

➤ What is the key idea?


Tf
The temperature of a perfect gas falls when it does work in Tf,Vf
an adiabatic expansion.

➤ What do you need to know already? Vi


Volume, V
Vf

This Topic makes use of the discussion of the properties


of gases (Topic 1A), particularly the perfect gas law. It also Figure 2E.1 To achieve a change of state from one temperature and
uses the definition of heat capacity at constant volume volume to another temperature and volume, treat the overall change
(Topic 2A) and constant pressure (Topic 2B) and the rela- as composed of two steps. In the first step, the system expands at
constant temperature; there is no change in internal energy if the
tion between them (Topic 2D).
system consists of a perfect gas. In the second step, the temperature
of the system is reduced at constant volume. The overall change in
internal energy is the sum of the changes for the two steps.

The temperature of a gas falls when it expands adiabatically in an adiabatic process. Therefore, by equating the two expres-
a thermally insulated container. Work is done, but as no heat sions for ΔU,
enters the system, the internal energy falls, and therefore the
Work of adiabatic change
temperature of the gas also falls. In molecular terms, the kinetic wad  CV T [perfect gas]
(2E.1)
energy of the molecules falls as work is done, so their average
speed decreases, and hence the temperature falls too. That is, the work done during an adiabatic expansion of a per-
fect gas is proportional to the temperature difference between
the initial and final states. That is exactly what is expected on
2E.1 The change in temperature molecular grounds, because the mean kinetic energy is propor-
tional to T, so a change in internal energy arising from tempera-
The change in internal energy of a perfect gas when the tem- ture alone is also expected to be proportional to ΔT. From these
perature is changed from Ti to Tf and the volume is changed considerations it is possible to calculate the temperature change
from Vi to Vf can be expressed as the sum of two steps of a perfect gas that undergoes reversible adiabatic expansion
(Fig. 2E.1). In the first step, only the volume changes and the (reversible expansion in a thermally insulated container).
temperature is held constant at its initial value. However, be-
cause the internal energy of a perfect gas is independent of the
volume it occupies (Topic 2A), there is no change in internal How is that done? 2E.1 Deriving an expression for the
energy in step 1. Step 2 is a change in temperature at constant
volume, for which there is a change in internal energy and, temperature change in a reversible adiabatic expansion
provided the heat capacity is independent of temperature, the of a perfect gas
change is For an expansion to be reversible the external pressure must
be equal to the pressure of the gas p at all times. As the gas
U  (Tf  Ti )CV  CV T
expands adiabatically, both the pressure and the temperature
Because the expansion is adiabatic, q = 0. From the First law, fall, so at first consider an infinitesimal change in these vari-
ΔU = q + w, it follows that ΔU = wad. The subscript ‘ad’ denotes ables only, and follow these steps.
2E Adiabatic changes 67

Step 1 Write an expression relating temperature and volume Brief illustration 2E.1
changes
The work done when the gas expands reversibly by dV is Consider the adiabatic, reversible expansion of 0.020 mol Ar,
dw = −pdV. This expression applies to any reversible change, initially at 25 °C, from 0.50 dm3 to 1.00 dm3. The molar heat
including an adiabatic change, so specifically dwad = −pdV. capacity of argon at constant volume is 12.47 J K−1 mol−1, so
Therefore, because dq = 0 for an adiabatic change, dU = dwad c = 1.501. Therefore, from eqn 2E.2a,
(the infinitesimal version of ΔU = wad). 1/1.501
 0.50 dm3 
For a perfect gas, dU = CVdT (the infinitesimal version of Tf  (298 K )    188 K
 1.00 dm3 
ΔU = CVΔT). Equating these expressions for dU gives  
It follows that ΔT = −110 K, and therefore, from eqn 2E.1, that
CV dT   pdV
wad  CV T  nCV,m T
Because the gas is perfect, p can be replaced by nRT/V to give
 (0.020 mol)  (12.47 JK 1 mol 1 )  (110 K )  27 J
CVdT = −(nRT/V)dV and therefore
Note that temperature change is independent of the amount of
CV dT nRdV gas but the work is not.

T V
Step 2 Integrate the expression to find the overall change
To integrate this expression, ensure that the limits of inte-
Exercise
gration match on each side of the equation. Note that T is E2E.1 A sample of carbon dioxide of mass 2.45 g at 27.0 °C is allowed to
equal to Ti when V is equal to Vi, and is equal to Tf when V expand reversibly and adiabatically from 500 cm3 to 3.00 dm3. What is the
work done by the gas, assuming it to be pefect?
is equal to Vf at the end of the expansion. Therefore,

Tf dT Vf dV
CV   nR 
Ti T Vi V 2E.2 The change in pressure
where CV is taken to be independent of temperature. Use
Equation 2E.2a may be used to calculate the pressure of a per-
Integral A.2 of the Resource section in each case, and obtain
fect gas that undergoes reversible adiabatic expansion.
Tf V
CV ln  nR ln f
Ti Vi How is that done? 2E.2 Deriving the relation between

Step 3 Simplify the expression pressure and volume for a reversible adiabatic expansion
of a perfect gas
Because ln(x/y) = −ln(y/x), the preceding expression rearranges to
The initial and final states of a perfect gas satisfy the perfect
CV Tf V gas law regardless of how the change of state takes place, so
ln = ln i
nR Ti Vf pV = nRT can be used to write
Next, note that CV/nR = CV,m/R = c and use ln xa = a ln x to obtain piVi Ti
=
c
pf Vf Tf
T  V However, Ti/Tf = (Vf/Vi)1/c (eqn 2E.2a). Therefore,
ln  f   ln i
T
 i Vf 1/c 11/c
piVi  Vf  pi  Vi 
This relation implies that (Tf/Ti)c = Vi /Vf and, upon    , so   1
pf Vf  Vi  pf  Vf 
­rearrangement,
For a perfect gas Cp,m − CV,m = R (Topic 2B). It follows that
1 1  c 1  CV,m /R R  CV,m C p ,m
1/c
Vi
Tf = Ti c = CV,m / R (2E.2a) 1     
Vf c c CV,m /R CV,m CV,m
and therefore that
By raising each side of this expression to the power c and reor- 
ganizing it slightly, an equivalent expression is pi  Vi 
  1
pf  Vf 
c c (2E.2b)
i i = Vf Tf
VT c = CV,m / R Temperature change [reversible which rearranges to
adiabatic expansion, perfect gas] (2E.3)
pf Vf γ = piViγ
Pressure change
[reversible adiabatic expansion, perfect gas]
This result is often summarized in the form VT c = constant.
68 2 The First Law

for a reversible path is illustrated in Fig. 2E.2. Because γ > 1, the


Isotherm, pressure in an adiabat falls more steeply (p ∝ 1/V γ) than in the
p  1/V corresponding isotherm (p ∝ 1/V). The physical reason for the
Adiabat, difference is that, in an isothermal expansion, energy flows into
Pressure, p

p 1/V γ
the system as heat and maintains the temperature; as a result,
the pressure does not fall as much as in an adiabatic expansion.
Pressure, p

Brief illustration 2E.2


T
e,
Volume, V ur When a sample of argon (for which   53 ) at 100 kPa expands
rat
Volume, pe reversibly and adiabatically to twice its initial volume the final
V m
Te pressure will be
Figure 2E.2 An adiabat depicts the variation of pressure with  5/3
V  1
volume when a gas expands adiabatically and, in this case, pf   i  pi     100 kPa   31 kPa
reversibly. Note that the pressure declines more steeply for an V
 f 2
adiabat than it does for an isotherm because in an adiabatic
For an isothermal expansion in which the volume doubles the
change the temperature falls.
final pressure would be 50 kPa.

This result is commonly summarized in the form pV γ =


­constant.
Exercises
For a monatomic perfect gas, CV, m = 23 R (Topic 2A), and E2E.2 Calculate the final pressure of a sample of carbon dioxide that expands
reversibly and adiabatically from 67.4 kPa and 0.50 dm3 to a final volume of
C p,m = 25 R (from Cp,m − CV,m = R), so   53 . For a gas of nonlin- 2.00 dm3. Take γ = 1.4.
ear polyatomic molecules (which can rotate as well as translate; −1
E2E.3 A sample consisting of 1.0 mol of perfect gas molecules with CV = 20.8 J K
­vibrations make little contribution at normal temperatures), is initially at 4.25 atm and 300 K. It undergoes reversible adiabatic expansion
CV,m = 3R and Cp,m = 4R, so   34 . The curves of pressure versus until its pressure reaches 2.50 atm. Calculate the final volume and temperature,
volume for adiabatic change are known as adiabats, and one and the work done.

Checklist of concepts
☐ 1. The temperature of a gas falls when it undergoes an ☐ 2. An adiabat is a curve showing how pressure varies with
adiabatic expansion in which work is done. volume in an adiabatic process.

Checklist of equations
Property Equation Comment Equation number
Work of reversible adiabatic expansion wad = CVΔT Perfect gas 2E.1
1/c
Temperature change, reversible adiabatic expansion Tf = Ti(Vi/Vf ) Perfect gas c = CV,m/R 2E.2a
c c
i i = Vf Tf
VT 2E.2b
 
Pressure change, reversible adiabatic expansion pf V  piVi
f Perfect gas   C p,m /CV, m 2E.3
Exercises and problems 69

FOCUS 2 The First Law

To test your understanding of this material, work through Assume all gases are perfect unless stated otherwise. Unless other-
the Exercises, Additional exercises, Discussion questions, and wise stated, thermochemical data are for 298.15 K.
Problems found throughout this Focus.
Selected solutions can be found at the end of this Focus in
the e-book. Solutions to even-numbered questions are available
online only to lecturers.

TOPIC 2A Internal energy


Discussion questions
D2A.1 Describe and distinguish the various uses of the words ‘system’ and D2A.3 Identify varieties of additional work.
‘state’ in physical chemistry.
D2A.4 Distinguish between reversible and irreversible expansion.
D2A.2 Describe the distinction between heat and work in thermodynamic
D2A.5 How may the isothermal expansion of a gas be achieved?
terms and, by referring to populations and energy levels, in molecular terms.

Additional exercises
E2A.5 Use the equipartition theorem to estimate the molar internal energy of E2A.10 A chemical reaction takes place in a container fitted with a piston of
(i) I2, (ii) CH4, (iii) C6H6 in the gas phase at 25 °C. cross-sectional area 75.0 cm2. As a result of the reaction, the piston is pushed
out through 25.0 cm against an external pressure of 150 kPa. Calculate the
E2A.6 Use the equipartition theorem to estimate the molar internal energy of
work done by the system.
(i) O3, (ii) C2H6, (iii) SO2 in the gas phase at 25 °C.
E2A.11 A sample consisting of 2.00 mol of perfect gas molecules, for which
E2A.7 Which of (i) pressure, (ii) temperature, (iii) work, (iv) enthalpy are state
CV, m = 25 R , initially at p1 = 111 kPa and T1 = 277 K, is heated reversibly to
functions?
356 K at constant volume. Calculate the final pressure, ΔU, q, and w.
E2A.8 Which of (i) volume, (ii) heat, (iii) internal energy, (iv) density are state 3
E2A.12 A sample of argon of mass 6.56 g occupies 18.5 dm at 305 K.
functions?
(i) Calculate the work done when the gas expands isothermally against a
E2A.9 A chemical reaction takes place in a container fitted with a piston of constant external pressure of 7.7 kPa until its volume has increased by 2.5 dm3.
cross-sectional area 50 cm2. As a result of the reaction, the piston is pushed (ii) Calculate the work that would be done if the same expansion occurred
out through 15 cm against an external pressure of 1.0 atm. Calculate the work reversibly.
done by the system.

Problems
P2A.1 Calculate the molar internal energy of carbon dioxide at 25 °C, taking 0° or 180° with an adjacent unit. In this case, the restoring force of a chain
into account its translational and rotational degrees of freedom. extended by x = nl is given by
P2A.2 A generator does work on an electric heater by forcing an electric kT  1    n
F ln 
current through it. Suppose 1 kJ of work is done on the heater and in turn 2l  1   N
1 kJ of energy as heat is transferred to its surroundings. What is the change in
internal energy of the heater? where k is Boltzmann’s constant, N is the total number of units, and
P2A.3 An elastomer is a polymer that can stretch and contract. In a perfect l = 45 nm for DNA. (a) What is the magnitude of the force that must be
elastomer the force opposing extension is proportional to the displacement x applied to extend a DNA molecule with N = 200 by 90 nm? (b) Plot the
from the resting state of the elastomer, so |F| = kfx, where kf is a constant. But restoring force against n, noting that n can be either positive or negative.
suppose that the restoring force weakens as the elastomer is stretched, and How is the variation of the restoring force with end-to-end distance different
kf(x) = a − bx1/2. Evaluate the work done on extending the polymer from x = 0 from that predicted by Hooke’s law? (c) Keeping in mind that the difference
to a final displacement x = l. in end-to-end distance from an equilibrium value is x = nl and, consequently,
dx = ldn = Nldn, write an expression for the work of extending a DNA
P2A.4 An approximate model of a DNA molecule is the ‘one-dimensional molecule. Hint: You must integrate the expression for w. The task can be
freely jointed chain’, in which a rigid unit of length l can make an angle of only accomplished best with mathematical software.
70 2 The First Law

P2A.5 An approximate model of a DNA molecule is the ‘one-dimensional (graphs of pressure against volume) for the isothermal reversible expansion of
freely jointed chain’, in which a rigid unit of length l can make an angle of only (a) a perfect gas, (b) a van der Waals gas in which a = 0 and b = 5.11 ×
0° or 180° with an adjacent unit. In this case, the restoring force of a chain 10−2 dm3 mol−1, and (c) a = 4.2 dm6 atm mol−2 and b = 0. The values selected
extended by x = nl is given by exaggerate the imperfections but give rise to significant effects on the indicator
diagrams. Take Vi = 1.0 dm3, Vf = 2.0 dm3, n = 1.0 mol, and T = 298 K.
kT  1    n
F ln 
2l  1   N P2A.8 A sample consisting of 1.0 mol CaCO3(s) was heated to 800 °C, at which
temperature the solid decomposed to CaO and CO2. The heating was carried
where k is Boltzmann’s constant, N is the total number of units, and l = 45 nm out in a container fitted with a piston that was initially resting on the solid.
for DNA. (a) Show that for small extensions of the chain, when n << 1, the Calculate the work done during complete decomposition at 1.0 atm. What
restoring force is given by work would be done if instead of having a piston the container was open to
the atmosphere?
 kT nkT
F 
l Nl P2A.9 Calculate the work done during the isothermal reversible expansion
of a gas that satisfies the virial equation of state (eqn 1C.3b) written with
(b) Is the variation of the restoring force with extension of the chain given in the first three terms. Evaluate (a) the work for 1.0 mol Ar at 273 K (for data,
part (a) different from that predicted by Hooke’s law? Explain your answer. see Table 1C.3) and (b) the same amount of a perfect gas. Let the expansion be
P2A.6 Suppose that attractions are the dominant interactions between gas from 500 cm3 to 1000 cm3 in each case.
molecules, and the equation of state is p = nRT/V − n2a/V2. Derive an expression P2A.10 Express the work of an isothermal reversible expansion of a van der
for the work of reversible, isothermal expansion of such a gas. Compared with a Waals gas in reduced variables (Topic 1C) and find a definition of reduced
perfect gas, is more or less work done on the surroundings when it expands? work that makes the overall expression independent of the identity of the
P2A.7 Calculate the work done during the isothermal reversible expansion of a gas. Calculate the work of isothermal reversible expansion along the critical
van der Waals gas (Topic 1C). Plot on the same graph the indicator diagrams isotherm from Vc to xVc.

TOPIC 2B Enthalpy
Discussion questions
D2B.1 Explain the difference between the change in internal energy and the D2B.2 Why is the heat capacity at constant pressure of a substance normally
change in enthalpy accompanying a process. greater than its heat capacity at constant volume?

Additional Exercises
E2B.5 When 229 J of energy is supplied as heat at constant pressure to 3.0 mol E2B.8 The constant-pressure heat capacity of a sample of a perfect gas was
Ar(g) the temperature of the sample increases by 2.55 K. Calculate the molar found to vary with temperature according to the expression Cp/(J K−1) =
heat capacities at constant volume and constant pressure of the gas. 20.17 + 0.4001(T/K). Calculate q, w, ΔU, and ΔH when the temperature is
raised from 25 °C to 100 °C (i) at constant pressure, (ii) at constant volume.
E2B.6 When 178 J of energy is supplied as heat at constant pressure to 1.9 mol
of gas molecules, the temperature of the sample increases by 1.78 K. Calculate E2B.9 When 2.0 mol CO2 is heated at a constant pressure of 1.25 atm, its
the molar heat capacities at constant volume and constant pressure of the gas. temperature increases from 250 K to 277 K. Given that the molar heat
capacity of CO2 at constant pressure is 37.11 J K−1 mol−1, calculate q, ΔH,
E2B.7 Calculate the value of ΔHm − ΔUm for the reaction C6H12O6(s) +
and ΔU.
6 O2(g) → 6 CO2(g) + 6 H2O(l) at 298 K.

Problems
P2B.1 Benzene is heated to boiling under a pressure of 1.0 atm with a 12 V By how much does the standard molar enthalpy of SO2(g) increase when
source operating at an electric current of 0.50 A. For how long would a current the temperature is raised from 298.15 K to 1500 K? Hint: Fit the data to an


need to be supplied in order to vaporize 10 g of benzene? The molar enthalpy of expression of the form of C p,m (T )  a  bT  c /T 2 , note the values of the
vaporization of benzene at its boiling point (353.25 K) is 30.8 kJ mol−1. coefficients, then use the approach in Example 2B.2 to calculate the change in
standard molar enthalpy.
P2B.2 The heat capacity of air is much smaller than that of liquid water,
and relatively modest amounts of heat are therefore needed to change the P2B.4 The following data show how the standard molar constant-pressure
temperature of air. This is one of the reasons why desert regions, though very heat capacity of ammonia depends on the temperature. Use mathematical
hot during the day, are bitterly cold at night. The molar heat capacity of air software to fit an expression of the form of eqn 2B.8 to the data and determine
at 298 K and 1.00 atm is approximately 21 J K−1 mol−1. Estimate how much the values of a, b, and c. Explore whether it would be better to express the data
energy is required to raise the temperature of the air in a room of dimensions as Cp,m = α + βT + γT 2, and determine the values of these coefficients.
5.5 m × 6.5 m × 3.0 m by 10 °C. If losses are neglected, how long will it take a
heater rated at 1.5 kW to achieve that increase, given that 1 W = 1 J s−1? T/K 300 400 500 600 700 800 900 1000


C p,m /(JK −1 mol −1 ) 35.678 38.674 41.994 45.229 48.269 51.112 53.769 56.244
P2B.3 The following data show how the standard molar constant-pressure heat
capacity of sulfur dioxide varies with temperature: P2B.5 A sample consisting of 2.0 mol CO2 occupies a fixed volume of 15.0 dm
3

at 300 K. When it is supplied with 2.35 kJ of energy as heat its temperature


T/K 300 500 700 900 1100 1300 1500 increases to 341 K. Assuming that CO2 is described by the van der Waals


C p,m /(JK −1 mol −1 ) 39.909 46.490 50.829 53.407 54.993 56.033 56.759 equation of state (Topic 1C), calculate w, ΔU, and ΔH.
Exercises and problems 71

TOPIC 2C Thermochemistry
Discussion questions
D2C.1 A simple air-conditioning unit for use in places where electrical power D2C.3 Distinguish between ‘standard state’ and ‘reference state’, and indicate
is not available can be made by hanging up strips of fabric soaked in water. their applications.
Explain why this strategy is effective. D2C.4 The expressions ‘heat of combustion’ and ‘heat of vaporization’ are
D2C.2 Describe two calorimetric methods for the determination of enthalpy used commonly, especially in the earlier literature. Why are the expressions
changes that accompany chemical processes. ‘enthalpy of combustion’ and ‘enthalpy of vaporization’ more appropriate?

Additional exercises
⦵ −1
E2C.8 For ethanol, ΔvapH = 43.5 kJ mol . Calculate q, w, ΔH, and ΔU when E2C.13 For the reaction C2H5OH(l) + 3 O2(g) → 2 CO2(g) + 3 H2O(g),
1.75 mol C2H5OH(l) is vaporized at 260 K and 1 bar. ΔrU ⦵ = −1373 kJ mol−1 at 298 K. Calculate ΔrH ⦵.
−1
E2C.9 The standard enthalpy of formation of phenol is −165.0 kJ mol . E2C.14 For the reaction 2 C6H5COOH(s) + 15 O2(g) → 14 CO2(g) + 6 H2O(g),
Calculate its standard enthalpy of combustion. ΔrU ⦵ = −772.7 kJ mol−1 at 298 K. Calculate ΔrH ⦵.
⦵ ⦵ ⦵
E2C.10 Given that the standard enthalpy of formation of HI(aq) is E2C.15 Calculate ΔrH and ΔrU at 298 K and ΔrH at 427 K for the
−55 kJ mol−1, what is the value of ΔfH (I−, aq)?

hydrogenation of ethyne (acetylene) to ethene (ethylene) from the enthalpy
of combustion and heat capacity data in Tables 2C.3 and 2C.4 of the
E2C.11 When 2.25 mg of anthracene, C14H10(s), was burned in a bomb calorimeter Resource section. Assume the heat capacities to be constant over the
the temperature rose by 1.75 K. Calculate the calorimeter constant. By how much temperature range involved.
will the temperature rise when 125 mg of phenol, C6H5OH(s), is burned in the ⦵
calorimeter under the same conditions? (ΔcH ⦵(C14H10,s) = −7061 kJ mol−1.) E2C.16 Estimate ΔrH (500 K) for the reaction C(graphite) + O2(g) → CO2(g)
⦵ ⦵
from the listed value of the standard enthalpy of formation of CO2(g) at
E2C.12 Given the reactions (1) and (2) below, determine (i) ΔrH and ΔrU 298 K in conjunction with the data on the temperature-dependence of heat

for reaction (3), (ii) ΔfH for both HI(g) and H2O(g), all at 298 K. capacities given in Table 2B.1.

(1) H2(g) + I2(s) → 2 HI(g) ΔrH ⦵ = +52.96 kJ mol−1 E2C.17 Estimate ΔrH (750 K) for the reaction N2(g) + H2(g) → NH3(g) from
the listed value of the standard enthalpy of formation of NH3(g) at 298 K in
(2) 2 H2(g) + O2(g) → 2 H2O(g) ΔrH ⦵ = −483.64 kJ mol−1 conjunction with the data on the temperature-dependence of heat capacities
(3) 4 HI(g) + O2(g) → 2 I2(s) + 2 H2O(g) given in Table 2B.1.

Problems

P2C.1 An average human produces about 10 MJ of heat each day through P2C.7 Kolesov et al. reported the standard enthalpy of combustion and
metabolic activity. If a human body were an isolated system of mass 65 kg with of formation of crystalline C60 based on calorimetric measurements
the heat capacity of water, what temperature rise would the body experience? (V.P. Kolesov et al., J. Chem. Thermodynamics 28, 1121 (1996)). In one of
Human bodies are actually open systems, and the main mechanism of heat their runs, they found the standard specific internal energy of combustion to
loss is through the evaporation of water. What mass of water should be be −36.0334 kJ g−1 at 298.15 K. Compute ΔcH ⦵ and ΔfH ⦵ of C60.
evaporated each day to maintain constant temperature? ‡
P2C.8 Silylene (SiH2) is a key intermediate in the thermal decomposition of
P2C.2 Predict the output of energy as heat from the combustion of 1.0 dm of
3
silicon hydrides such as silane (SiH4) and disilane (Si2H6). H.K. Moffat et al.
octane at 298 K and 1 bar. Its mass density is 0.703 g cm−3. (J. Phys. Chem. 95, 145 (1991)) report ΔfH ⦵(SiH2) = +274 kJ mol−1. Given that
−1
ΔfH ⦵(SiH4) = +34.3 kJ mol−1 and ΔfH ⦵(Si2H6) = +80.3 kJ mol−1, calculate the
P2C.3 The standard enthalpy of combustion of cyclopropane is −2091 kJ mol
standard enthalpy changes of the following reactions:
at 25 °C. (a) From this information and enthalpy of formation data for CO2(g)
and H2O(l), calculate the enthalpy of formation of cyclopropane. (b) The (a) SiH4(g) → SiH2(g) + H2(g)
enthalpy of formation of propene is +20.42 kJ mol−1. Calculate the enthalpy (b) Si2H6(g) → SiH2(g) + SiH4(g)
of isomerization of cyclopropane to propene. P2C.9 It is sometimes appropriate to express the temperature dependence

P2C.4 From the following data, determine Δf H for diborane, B2H6(g), at 298 K: of the heat capacity by the empirical expression Cp,m = α + βT + γT2. Use this
expression to estimate the standard enthalpy of combustion of methane to
(1) B2H6(g) + 3 O2(g) → B2O3(s) + 3 H2O(g) ΔrH ⦵ = −1941 kJ mol−1 carbon dioxide and water vapour at 500 K. Use the following data:
(2) 2B(s)  23 O2 (g )  B2O3 (s) ΔrH ⦵ = −2368 kJ mol−1 α/(J K−1 mol−1) β/(mJ K−2 mol−1) γ/(μJ K−3 mol−1)
(3) H2 (g )  12 O2 (g )  H2O(g ) ΔrH ⦵ = −241.8 kJ mol−1 CH4(g) 14.16 75.5 −17.99

P2C.5 A sample of the sugar d-ribose (C5H10O5) of mass 0.727 g was


CO2(g) 26.86 6.97 −0.82
placed in a calorimeter and then ignited in the presence of excess oxygen. O2(g) 25.72 12.98 −3.862
The temperature rose by 0.910 K. In a separate experiment in the same H2O(g) 30.36 9.61 1.184
calorimeter, the combustion of 0.825 g of benzoic acid, for which the internal
energy of combustion is −3251 kJ mol−1, gave a temperature rise of 1.940 K. P2C.10 The figure below shows the experimental DSC scan of hen white lysozyme
Calculate the enthalpy of formation of d-ribose. (G. Privalov et al., Anal. Biochem. 79, 232 (1995)) converted to joules (from
⦵ calories). Determine the enthalpy of unfolding of this protein by integration of
P2C.6 For the reaction Cr(C6H6)2(s) → Cr(s) + 2 C6H6(g), ΔrU (583 K) =
the curve and the change in heat capacity accompanying the transition.
+8.0 kJ mol−1. Find the corresponding reaction enthalpy and estimate the

standard enthalpy of formation of Cr(C6H6)2(s) at 583 K. These problems were supplied by Charles Trapp and Carmen Giunta.
72 2 The First Law

9 P2C.11 In biological cells that have a plentiful supply of oxygen, glucose is


Excess heat capacity, Cp,ex/(mJ ºC–1)
oxidized completely to CO2 and H2O by a process called aerobic oxidation.
Muscle cells may be deprived of O2 during vigorous exercise and, in
that case, one molecule of glucose is converted to two molecules of lactic
6 acid (CH3CH(OH)COOH) by a process called anaerobic glycolysis.
(a) When 0.3212 g of glucose was burned at 298 K in a bomb calorimeter
of calorimeter constant 641 J K−1 the temperature rose by 7.793 K. Calculate
(i) the standard molar enthalpy of combustion, (ii) the standard internal
3 energy of combustion, and (iii) the standard enthalpy of formation of
glucose. (b) What is the biological advantage (in kilojoules per mole of
energy released as heat) of complete aerobic oxidation compared with
anaerobic glycolysis to lactic acid?
0
30 45 60 75 90
Temperature, θ/°C

TOPIC 2D State functions and exact differentials


Discussion questions
D2D.1 Suggest (with explanation) how the internal energy of a van der Waals D2D.2 Explain why a perfect gas does not have an inversion temperature.
gas should vary with volume at constant temperature.

Additional exercises
E2D.4 Estimate the internal pressure of sulfur dioxide at 1.00 bar and 298 K, E2D.7 The volume of a certain liquid varies with temperature as
treating it as a van der Waals gas, when  T  a/Vm2 . You may simplify the
V  V {0.77  3.7  104 (T /K )  1.52  106 (T /K )2 }
problem by assuming that the molar volume can be predicted from the perfect
gas equation. where V′ is its volume at 298 K. Calculate its expansion coefficient, α, at 310 K.
−6 −1
E2D.5 For a van der Waals gas,  T  a/V . Assuming that this relation applies,
2 E2D.8 The isothermal compressibility, κT , of lead at 293 K is 2.21 × 10 atm .
m
calculate ΔUm for the isothermal expansion of argon gas from an initial Calculate the pressure that must be applied in order to increase its density by
volume of 1.00 dm3 to 30.00 dm3 at 298 K. What are the values of q and w? 0.10 per cent.
E2D.9 Use data from the Resource section to evaluate the difference Cp,m − CV,m
E2D.6 The volume of a certain liquid varies with temperature as
in molar heat capacities for liquid benzene at 298 K.
V  V {0.75  3.9  104 (T /K )  1.48  106 (T /K )2 }
E2D.10 Use data from the Resource section to evaluate the difference Cp,m − CV,m
where V′ is its volume at 300 K. Calculate its expansion coefficient, α, at 320 K. in molar heat capacities for liquid ethanol at 298 K.

Problems

P2D.1 According to the Intergovernmental Panel on Climate Change (IPCC) P2D.4 Rearrange the van der Waals equation of state, p = nRT/(V − nb) −
the global average temperature may rise by as much as 2.0 °C by 2100. Predict n2a/V2 (Topic 1C) to give an expression for T as a function of p and
the average rise in sea level due to thermal expansion of sea water based on V (with n constant). Calculate (∂T/∂p)V and confirm that (∂T/∂p)V =
temperature rises of 1.0 °C, 2.0 °C, and 3.5 °C, given that the volume of the 1/(∂p/∂T)V .
Earth’s oceans is 1.37 × 109 km3 and their surface area is 361 × 106 km2; state
P2D.5 Calculate the isothermal compressibility and the expansion coefficient of
the approximations which go into your estimates. Hint: Recall that the volume
a gas that obeys the van der Waals equation of state, p = nRT/(V − nb) − n2a/V2
V of a sphere of radius r is V  43 r 3 . If the radius changes only slightly by
(Topic 1C). Show, using Euler’s chain relation (see Part 1 of the Resource
δr, with δr << r, then the change in the volume is δV ≈ 4πr2δr. Because the
section), that κTR = α(Vm − b).
surface area of a sphere is A = 4πr2, it follows that δV ≈ Aδr.
P2D.6 The speed of sound, cs, in a perfect gas of molar mass M is related
P2D.2 Starting from the expression Cp − CV = T(∂p/∂T)V(∂V/∂T)p, use the
to the ratio of heat capacities γ by cs = (γRT/M)1/2. Show that cs = (γp/ρ)1/2,
appropriate relations between partial derivatives (see Part 1 of the Resource where ρ is the mass density of the gas. Calculate the speed of sound in
section) to show that argon at 25 °C.
T (V /T )2p ‡
C p  CV  P2D.7 A gas obeying the equation of state p(V − nb) = nRT is subjected to a Joule–
(V /p)T Thomson expansion. Will the temperature increase, decrease, or remain the same?
Use this expression to evaluate Cp − CV for a perfect gas. 2
P2D.8 Use the fact that (U /V )T  a/Vm for a van der Waals gas (Topic 1C)
P2D.3 (a) Write expressions for dV and dp given that V is a function of p and T to show that μCp,m ≈ (2a/RT) − b by using the definition of μ and appropriate
and p is a function of V and T. (b) Deduce expressions for d ln V and d ln p in relations between partial derivatives. Hint: Use the approximation pVm ≈ RT
terms of the expansion coefficient and the isothermal compressibility. when it is justifiable to do so.
Exercises and problems 73


P2D.9 Concerns over the harmful effects of chlorofluorocarbons on stratospheric (The specific constant-pressure heat capacity is 0.7649 kJ K−1 kg−1.)
ozone have motivated a search for new refrigerants. One such alternative (b) Compute μ at 1.00 MPa and 350 K from the following data (all referring
is 1,1,1,2-tetrafluoroethane (refrigerant HFC-134a). A compendium of to 350 K):
thermophysical properties of this substance has been published (R. Tillner-Roth
and H.D. Baehr, J. Phys. Chem. Ref. Data 23, 657 (1994)) from which properties p/MPa 0.80 1.00 1.2
such as the Joule–Thomson coefficient μ can be computed. (a) Compute μ at Specific enthalpy/(kJ kg−1) 461.93 459.12 456.15
0.100 MPa and 300 K from the following data (all referring to 300 K):
(The specific constant-pressure heat capacity is 1.0392 kJ K−1 kg−1.)
p/MPa 0.080 0.100 0.12
Specific enthalpy/(kJ kg−1) 426.48 426.12 425.76

TOPIC 2E Adiabatic changes


Discussion questions
D2E.1 On a p against V plot, why are adiabats steeper than isotherms? D2E.2 Why do heat capacities play a role in the expressions for adiabatic
expansion?

Additional exercises
E2E.4 Use the equipartition principle to estimate the values of γ = Cp/CV for E2E.8 A sample consisting of 2.5 mol of perfect gas molecules with
gaseous ammonia and methane. Do this calculation with and without the Cp,m = 20.8 J K−1 mol−1 is initially at 240 kPa and 325 K. It undergoes reversible
vibrational contribution to the energy. Which is closer to the experimental adiabatic expansion until its pressure reaches 150 kPa. Calculate the final
value at 25 °C? volume and temperature, and the work done.
E2E.5 Use the equipartition principle to estimate the value of γ = Cp/CV for E2E.9 A sample of nitrogen of mass 3.12 g at 23.0 °C is allowed to expand
carbon dioxide. Do this calculation with and without the vibrational reversibly and adiabatically from 400 cm3 to 2.00 dm3. What is the work done
contribution to the energy. Which is closer to the experimental value at 25 °C? by the gas?
E2E.6 Calculate the final temperature of a sample of argon of mass 12.0 g that E2E.10 Calculate the final pressure of a sample of water vapour that expands
is expanded reversibly and adiabatically from 1.0 dm3 at 273.15 K to 3.0 dm3. reversibly and adiabatically from 97.3 Torr and 400 cm3 to a final volume of
5.0 dm3. Take γ = 1.3.
E2E.7 Calculate the final temperature of a sample of carbon dioxide of mass
16.0 g that is expanded reversibly and adiabatically from 500 cm3 at 298.15 K
to 2.00 dm3.

Problems
P2E.1 Calculate the final temperature, the work done, and the change of value of γ = Cp/CV can be inferred if the decrease in pressure is also measured
internal energy when 1.00 mol NH3(g) at 298 K is used in a reversible and the constant-pressure heat capacity deduced by combining the two values.
adiabatic expansion from 0.50 dm3 to 2.00 dm3. A fluorocarbon gas was allowed to expand reversibly and adiabatically to twice
its volume; as a result, the temperature fell from 298.15 K to 248.44 K and its
P2E.2 The constant-volume heat capacity of a gas can be measured by observing
pressure fell from 202.94 kPa to 81.840 kPa. Evaluate Cp,m.
the decrease in temperature when it expands adiabatically and reversibly. The

FOCUS 2 The First Law


Integrated activities
I2.1 Give examples of state functions and discuss why they play a critical role modelling method. (c) Test the extent to which the relation
in thermodynamics. ΔcH ⦵ = constant × {M/(g mol−1)}n holds and determine the numerical
values of the constant and n.
I2.2 The thermochemical properties of hydrocarbons are commonly
investigated by using molecular modelling methods. (a) Use software to I2.3 It is often useful to be able to anticipate, without doing a detailed
predict ΔcH ⦵ values for the alkanes methane through pentane. To calculate calculation, whether an increase in temperature will result in a raising or a
ΔcH ⦵ values, estimate the standard enthalpy of formation of CnH2n+2(g) by lowering of a reaction enthalpy. The constant-pressure molar heat capacity
using molecular modelling software of your choice, and use experimental of a gas of linear molecules is approximately 72 R whereas that of a gas of
standard enthalpy of formation values for CO2(g) and H2O(l). (b) Compare nonlinear molecules is approximately 4R. Decide whether the standard

your estimated values with the experimental values of ΔcH (Table 2C.3 enthalpies of the following reactions will increase or decrease with increasing
of the Resource section) and comment on the reliability of the molecular temperature:
74 2 The First Law

(a) 2 H2(g) + O2(g) → 2 H2O(g) I2.6 The heat capacity ratio of a gas determines the speed of sound in it
(b) CH4(g) + 2 O2(g) → CO2(g) + 2 H2O(g) through the formula cs = (γRT/M)1/2, where γ = Cp/CV and M is the molar mass
of the gas. Deduce an expression for the speed of sound in a perfect gas of
(c) N2(g) + 3 H2(g) → 2 NH3(g) (a) diatomic, (b) linear triatomic, (c) nonlinear triatomic molecules at high
I2.4 The molar heat capacity of liquid water is approximately 9R. Decide
temperatures (with translation and rotation active). Estimate the speed of
whether the standard enthalpy of the first two reactions in the preceding sound in air at 25 °C.
exercise will increase or decrease with a rise in temperature if the water is I2.7 Use mathematical software or a spreadsheet to: (a) calculate the work
produced as a liquid. of isothermal reversible expansion of 1.0 mol CO2(g) at 298 K from 1.0 dm3
I2.5 As shown in Part 1 of the Resource section it is a property of partial
to 3.0 dm3 on the basis that it obeys the van der Waals equation of state;
derivatives that (b) explore how the parameter γ affects the dependence of the pressure on the
volume when the expansion is reversible and adiabatic and the gas is perfect.
   f      f   Does the pressure–volume dependence become stronger or weaker with
        
 y  x  y 
  x  x  y  x  y increasing volume?
Use this property and eqn 2A.14 to write an expression for (∂CV/∂V)T as
a second derivative of U and find its relation to (∂U/∂V)T . Then show that
(∂CV/∂V)T = 0 for a perfect gas.
FOCUS 3
The Second and Third Laws
Some things happen naturally, some things don’t. Some aspect ‘Third Law of thermodynamics’, which relates to the properties
of the world determines the spontaneous direction of change, of matter at very low temperatures and is used to set up an ab-
the direction of change that does not require work to bring it solute measure of the entropy of a substance.
about. An important point, though, is that throughout this text 3C.1 The calorimetric measurement of entropy; 3C.2 The Third Law
‘spontaneous’ must be interpreted as a natural tendency which
might or might not be realized in practice. Thermodynamics
says nothing about the rate at which a spontaneous change in 3D Concentrating on the system
fact occurs, and some spontaneous processes (such as the con-
version of diamond to graphite) may be so slow that the ten- One problem with dealing with entropy is that it requires sepa-
dency is never realized in practice whereas others (such as the rate calculations of the changes taking place in the system and
expansion of a gas into a vacuum) are almost instantaneous. the surroundings. Providing certain restrictions on the system
can be accepted, that problem is overcome by introducing the
‘Gibbs energy’. Indeed, most thermodynamic calculations in
3A Entropy chemistry focus on the change in Gibbs energy rather than the
entropy change itself.
The direction of change is related to the distribution of energy 3D.1 The Helmholtz and Gibbs energies;
and matter, and spontaneous changes are always accompanied 3D.2 Standard molar Gibbs energies
by a dispersal of energy or matter. To quantify this concept the
property called ‘entropy’ is introduced, which is central to the
formulation of the ‘Second Law of thermodynamics’. That law 3E Combining the First and Second Laws
governs all spontaneous change.
3A.1 The Second Law; 3A.2 The definition of entropy; In this Topic the First and Second Laws are combined, which
3A.3 The entropy as a state function leads to a very powerful way of applying thermodynamics to
the properties of matter.
3E.1 Properties of the internal energy;
3B Entropy changes accompanying 3E.2 Properties of the Gibbs energy

specific processes
What are the applications of this
This Topic shows how to use the definition of entropy change to
calculate its value for a number of common physical processes, material?
such as the expansion of a gas, a phase transition, and heating
a substance. The Second Law is at the heart of the operation of engines
3B.1 Expansion; 3B.2 Phase transitions; 3B.3 Heating;
of all types, including devices resembling engines that are
3B.4 Composite processes used to cool objects. See Impact 4, accessed via the e-book,
for an application to the technology of refrigeration. Entropy
considerations are also important in modern electronic
3C The measurement of entropy materials for they permit a quantitative discussion of the
concentration of impurities. See Impact 5, accessed via the
To make the Second Law quantitative, it is necessary to meas- e-book, for a note about how measurement of the entropy at
ure the entropy of a substance. That is achieved by measuring low temperatures gives insight into the purity of materials
heat capacities. The discussion in this Topic also leads to the used as superconductors.

➤ Go to the e-book for videos that feature the derivation and interpretation of equations, and applications of this material.
TOPIC 3A Entropy

➤ Why do you need to know this material?


Entropy is the concept on which almost all applications of
thermodynamics in chemistry are based: it explains why
some physical transformations and chemical reactions are
spontaneous and others are not.

➤ What is the key idea?


The change in entropy of a system can be calculated from
the heat transferred to it reversibly; a spontaneous process
in an isolated system is accompanied by an increase in
entropy. Figure 3A.1 The direction of spontaneous change for a ball
bouncing on a floor. On each bounce some of its energy is
➤ What do you need to know already? degraded into the thermal motion of the atoms of the floor, and
You need to be familiar with the First-Law concepts of that energy then disperses. The reverse process, a ball rising from
work, heat, and internal energy (Topic 2A). The Topic draws the floor as a result of acquiring energy from the thermal motion
on the expression for work of expansion of a perfect gas of the atoms in the floor, has never been observed to take place.
(Topic 2A) and on the changes in volume and temperature
that accompany the reversible adiabatic expansion of a
perfect gas (Topic 2E). A ball resting on a warm floor has never been observed to
start bouncing as a result of energy being transferred to the
ball from the floor. For bouncing to begin, something rather
special would need to happen. In the first place, some of the
thermal motion of the atoms in the floor (the surroundings)
What determines the direction of spontaneous change? It is not would have to accumulate in a single, small object, the ball (the
a tendency to achieve a lower energy, because the First Law as- system). This accumulation requires a spontaneous localization
serts that the total energy of the universe does not change in of energy from the myriad vibrations of the atoms of the floor
any process. It turns out that the direction is determined by into the much smaller number of atoms that constitute the ball
the manner in which energy and matter are distributed. This (Fig. 3A.2). Furthermore, whereas the thermal motion is ran-
concept is made precise by the Second Law of thermodynamics dom, for the ball to move upwards its atoms must all move in
and made quantitative by introducing the property known as the same direction. The localization of random, disorderly mo-
‘entropy’. tion as directed, orderly motion is so unlikely that it can be dis-
missed as virtually impossible.1
This discussion identifies the signpost of spontaneous
change: look for the direction of change that leads to the dispersal
3A.1 The Second Law of energy. This principle accounts for the direction of change
of the bouncing ball, because its energy is spread out as ther-
The role of the distribution of energy and matter can be ap- mal motion of the atoms of the floor. The reverse process is
preciated by thinking about a ball bouncing on a floor. The ball not spontaneous because it is highly improbable that energy
does not rise as high after each bounce because some of the will become localized, leading to uniform motion of the ball’s
energy associated with its motion spreads out—is dispersed— atoms.
into the thermal motion of the particles in the ball and the
floor. The direction of spontaneous change is towards a state
in which the ball is at rest with all its energy dispersed into 1
Orderly motion, but on a much smaller scale and continued only very
disorderly thermal motion of the particles in the surroundings briefly, is observed as Brownian motion, the jittering motion of small particles
(Fig. 3A.1). suspended in a liquid or gas.
3A Entropy 77

Hot source Hot source

Flow of
energy Work Work
as heat

(b)

(a) (b) Cold sink


Figure 3A.2 (a) A ball resting on a warm surface; the atoms (a)
are undergoing thermal motion (vibration, in this instance), as
indicated by the arrows. (b) For the ball to fly upwards, some Figure 3A.3 (a) A heat engine is a device in which energy is
of the random vibrational motion would have to change into extracted from a hot reservoir (the hot source) as heat and then
coordinated, directed motion. Such a conversion is highly some of that energy is converted into work and the rest discarded
improbable. into a cold reservoir (the cold sink) as heat. (b) The Kelvin
statement of the Second Law says that the process illustrated
here, in which heat is changed completely into work, is not
Matter also has a tendency to disperse. A gas does not con- possible.
tract spontaneously because to do so the random motion of its
molecules would have to take them all into the same region of
the container. The opposite change, expansion into a larger vol-
ume, takes place spontaneously as the random motion of the
molecules results in them filling the container. Hot sink
The Second Law of thermodynamics expresses these con-
clusions more precisely in terms of the classical concepts of
heat and work. One statement was formulated by Kelvin:
No process is possible in which the sole result is the
absorption of heat from a reservoir and its complete
conversion into work.
Statements like this are commonly explored by thinking about Cold source
an idealized device called a heat engine (Fig. 3A.3(a)). A heat
engine consists of two reservoirs, one hot (the ‘hot source’)
and one cold (the ‘cold sink’), connected through a device (a Figure 3A.4 According to the Clausius statement of the
‘converter’) that can convert some of the energy flowing as heat Second Law, the process shown here, in which energy as heat
between the two reservoirs into work. The Kelvin statement im- migrates from a cool source to a hot sink, does not take place
plies that it is not possible to construct a heat engine in which spontaneously. The process is not in conflict with the First Law
all the heat drawn from the hot source is completely converted because energy is conserved.
into work (Fig. 3A.3(b)). The Kelvin statement is a generaliza-
tion of the everyday observation that a ball at rest on a surface
has never been observed to leap spontaneously upwards. An cooling fluid. Although they appear somewhat different, it can
upward leap of the ball would be equivalent to the spontane- be shown that the Clausius statement is logically equivalent to
ous conversion of heat from the surface into the work of raising the Kelvin statement. One way to do so is to show that the two
the ball. observations can be summarized by a single statement.
Another statement of the Second Law is due to Rudolf First, the system and its surroundings are regarded as a sin-
Clausius (Fig. 3A.4): gle (and possibly huge) isolated system sometimes referred
to as ‘the universe’. Energy can be transferred within this
Heat does not flow spontaneously from a cool body to a
isolated system between the actual system and its surround-
hotter body.
ings, but none can enter or leave the isolated system. Then the
To transfer heat to a hotter body, it is necessary to do work on Second Law is expressed in terms of a new state function, the
the system, as in a refrigerator where a compressor circulates a entropy, S:
78 3 The Second and Third Laws

The entropy of an isolated system increases in the course f dqrev


of a spontaneous change: ΔStot > 0 S    (3A.1b)
i T

where Stot is the total entropy of the overall isolated system. The implication of this expression is that in order to calculate
That is, if S is the entropy of the system of interest, and Ssur the the difference in entropy between any two states a reversible
entropy of its surroundings, then Stot = S + Ssur. It is vitally im- path between them is first identified. Then, at each infinitesi-
portant when considering applications of the Second Law to mal stage along this path the energy supplied as heat is divided
remember that it is a statement about the total entropy of the by the current temperature. All those individual infinitesimal
overall isolated system (the ‘universe’), not just about the en- changes are then added together; that is, they are integrated
tropy of the system of interest. The following section defines from the initial state to the final state.
entropy and interprets it as a measure of the dispersal of en- According to the definition of an entropy change given in
ergy and matter, and relates it to the empirical observations dis- eqn 3A.1a, when the energy transferred as heat is expressed in
cussed so far. joules and the temperature is in kelvins, the units of entropy
In summary, the First Law uses the internal energy to iden- are joules per kelvin (J K−1). Entropy is an extensive property.
tify permissible changes; the Second Law uses the entropy to Molar entropy, the entropy divided by the amount of substance,
identify which of these permissible changes are spontaneous. Sm = S/n, is expressed in joules per kelvin per mole (J K−1 mol−1);
molar entropy is an intensive property.
Exercise
E3A.1 Consider a process in which the entropy of a system increases by Example 3A.1 Calculating the entropy change for the
125 J K−1 and the entropy of the surroundings decreases by 125 J K−1. Is the
process spontaneous? isothermal expansion of a perfect gas
Calculate the entropy change of a sample of perfect gas when it
expands isothermally from a volume Vi to a volume Vf.
3A.2 The definition of entropy Collect your thoughts The definition of entropy change in
eqn 3A.1b instructs you to find the energy supplied as heat for a
To make progress, and to turn the Second Law into a quanti- reversible path between the stated initial and final states regard-
tatively useful expression, the entropy change accompanying less of the actual manner in which the process takes place. The
various processes needs to be defined and calculated. There are process is isothermal, so T is constant and can be taken out-
two approaches, one classical and one molecular. They turn out side the integral in eqn 3A.1b. Moreover, because the internal
to be equivalent, but an understanding of entropy as a concept energy of a perfect gas is independent of its volume (Topic 2A),
is enriched by using both approaches. ΔU = 0 for the expansion. Then, because ΔU = q + w, it
follows that q = −w. The work of reversible isothermal expan-
sion is calculated in Topic 2A. Finally, calculate the change in
3A.2(a)The thermodynamic definition molar entropy from ΔSm = ΔS/n.
of entropy
The solution The temperature is constant, so eqn 3A.1b
The thermodynamic definition of entropy concentrates on the becomes
change in entropy, dS, that occurs as a result of a physical or
1 f q
T i
chemical change (in general, as a result of a ‘process’). The defi- S  dqrev  rev
T
nition is motivated by the idea that a change in the extent to
which energy is dispersed in a disorderly way depends on how From Topic 2A the reversible work in an isothermal expansion
much energy is transferred as heat, not as work. As explained is wrev = −nRT ln(Vf /Vi), hence because for this isothermal pro-
in Topic 2A, heat stimulates random motion of atoms whereas cess of a perfect gas q = −w, it follows that qrev = nRT ln(Vf /Vi).
work stimulates their uniform motion and so does not change This expression for qrev is used in S  qrev /T to give
the extent of their disorder.
The thermodynamic definition of entropy is based on the Vf S V
S  nR ln and Sm   R ln f
­expression Vi n Vi

dqrev Entropy change Self-test 3A.1 Calculate the change in entropy when the pres-
dS =  (3A.1a)
T [definition] sure of a fixed amount of perfect gas is changed isothermally
from pi to pf . What is the origin of this change?
where qrev is the energy transferred as heat reversibly to the sys-
tem at the absolute temperature T. For a measurable change be-
is compressed or expands.
Answer: ΔS = nR ln(pi/pf ); the change in volume when the gas
tween two states i and f,
3A Entropy 79

To see how the definition in eqn 3A.1a is used to formulate 2.86  105 J
an expression for the change in entropy of the surroundings, Ssur   960 JK 1
298 K
ΔSsur, consider an infinitesimal transfer of heat dqsur from the
system to the surroundings. Suppose that the surroundings This strongly exothermic reaction results in an increase in
consist of a large reservoir so that its volume does not change the entropy of the surroundings as energy is released as heat
no matter what happens in the much smaller system. Then the into them.
energy supplied to them by heating can be identified with
the change in the internal energy of the surroundings, dUsur.2
The internal energy is a state function, and dUsur is an exact You are now in a position to see how the definition of en-
differential. These properties imply that dUsur is independent tropy is consistent with Kelvin’s and Clausius’s statements of the
of how the change is brought about and in particular it is in- Second Law and unifies them. In Fig. 3A.3(b) the entropy of the
dependent of whether the process is reversible or irreversible hot source is reduced as energy leaves it as heat. The transfer
for the system. The same remarks therefore apply to dqsur, to of energy as work does not result in the production of entropy,
which dUsur is equal. Therefore, the definition in eqn 3A.1a can so the overall result is that the entropy of the (overall isolated)
be adapted to calculate the entropy change of the surroundings system decreases. The Second Law asserts that such a process
simply by deleting the constraint ‘reversible’ and writing is not spontaneous, so the arrangement shown in Fig. 3A.3(b)
cannot produce work. In the Clausius version, the entropy of
dqsur Entropy change of
dSsur =  (3A.2a) the cold source in Fig 3A.4 decreases when energy leaves it as
Tsur the surroundings
heat, but when that heat enters the hot sink the rise in entropy
is not as great (because the temperature is higher). Overall
Again, the surroundings are supposed to be much larger than
there is a decrease in entropy and so the transfer of heat from a
the system, so the temperature of the surroundings is constant
cold source to a hot sink is not spontaneous.
whatever the change. Therefore, for a measurable change
qsur
Ssur 
Tsur
 (3A.2b) 3A.2(b) The statistical definition of entropy
The molecular interpretation of the Second Law and the ‘sta-
That is, regardless of how the change is brought about in the
tistical’ definition of entropy start from the idea, introduced
system, reversibly or irreversibly, the change of entropy of the
in Energy: A first look 3, that atoms and molecules are distrib-
surroundings is calculated simply by dividing the heat trans-
uted over the energy states available to them in accord with
ferred by the temperature at which the transfer takes place.
the Boltzmann distribution. It follows that as the tempera-
Equation 3A.2b makes it very simple to calculate the changes
ture is increased, the molecules populate higher energy states.
in entropy of the surroundings that accompany any process.
Boltzmann proposed that there is a link between the spread
For instance, for any adiabatic change, qsur = 0, so
of molecules over the available energy states and the entropy,
Ssur  0 Adiabatic change  (3A.3) which he expressed as3

This expression is true however the change takes place, reversi- S = k lnW Boltzmann formula for the entropy (3A.4)
bly or irreversibly, provided no local hot spots are formed in the
surroundings. That is, it is true (as always assumed) provided where k is Boltzmann’s constant (k = 1.381 × 10−23 J K−1) and W
the surroundings remain in internal equilibrium. If hot spots is the number of microstates, the number of ways in which the
do form, then the localized energy may subsequently disperse molecules of a system can be distributed over the energy states
spontaneously and hence generate more entropy. for a specified total energy. When the properties of a system
are measured, the outcome is an average taken over the many
microstates the system can occupy under the prevailing condi-
Brief illustration 3A.1 tions. The concept of the number of microstates makes quanti-
tative the qualitative concepts of ‘disorder’ and ‘the dispersal of
To calculate the entropy change in the surroundings when
matter and energy’ used to introduce the concept of entropy: a
1.00 mol H2O(l) is formed from its elements under standard
more disorderly distribution of matter and a greater dispersal
conditions at 298 K, use Δf H = −286 kJ mol−1 from Table 2C.4.

of energy corresponds to a greater number of microstates as-


The energy released as heat from the system is supplied to the
sociated with the same total energy. This point is discussed in
surroundings, so qsur = +286 kJ. Therefore,
detail in Topic 13E.

2
Alternatively, the surroundings can be regarded as being at constant pres-
sure, in which case dqsur = dHsur. 3
He actually wrote S = k log W, and it is carved on his tombstone in Vienna.
80 3 The Second and Third Laws

Equation 3A.4 is known as the Boltzmann formula and the


entropy calculated from it is called the statistical entropy. If all
the molecules are in one energy state there is only one way of
achieving this distribution, so W = 1 and, because ln 1 = 0, it
Heat Heat
follows that S = 0. According to the Boltzmann distribution,
only the lowest energy state is occupied at T = 0, so it follows
that S = 0 at T = 0. At higher temperatures the molecules spread
out over the available energy states, W increases and therefore
so too does the entropy.
For a given temperature, the value of W increases if the sepa- (a) High temperature (b) Low temperature
ration of energy states decreases, because more states become
Figure 3A.6 The supply of energy as heat to the system results in
accessible. An example of this is the expansion of a gas confined
the molecules moving to higher energy states, so increasing the
to a container. A result of quantum theory (Topic 7D) is that
number of microstates and hence the entropy. The increase in
the translational energy states of molecules get closer together the entropy is smaller for (a) a system at a high temperature than
as the container they occupy expands (Fig. 3A.5). The value of (b) one at a low temperature because initially the number
W , and hence the entropy, is therefore expected to increase as of occupied states is greater.
the gas expands isothermally, which is in accord with the con-
clusion drawn from the thermodynamic definition of entropy
(Example 3A.1). value of the entropy of a system, whereas the thermodynamic
The molecular interpretation of entropy helps to explain definition leads only to values for a change in entropy. This
why, in the thermodynamic definition given by eqn 3A.1a, the point is developed in Focus 13 where it is shown how to relate
entropy change depends inversely on the temperature. In a sys- values of S to the structural features of atoms and molecules.
tem at high temperature the molecules are spread out over a The second point is that the Boltzmann formula cannot readily
large number of energy states. Increasing the energy of the sys- be applied to the surroundings, which are typically far too com-
tem by the transfer of heat makes more states accessible, but, plex for W to be a meaningful quantity.
given that very many states are already occupied, W changes
only slightly (Fig. 3A.6). In contrast, for a system at a low tem-
Exercises
perature fewer states are occupied, and so the transfer of the
same energy results in a more significant increase in the num- E3A.2 Consider a process in which 100 kJ of energy is transferred reversibly
and isothermally as heat to a large block of copper. Calculate the change in
ber of accessible states, and hence a larger increase in W . This entropy of the block if the process takes place at (i) 0 °C, (ii) 50 °C.
argument suggests that the change in entropy for a given trans-
E3A.3 Calculate the change in entropy of the gas when 15 g of carbon dioxide
fer of energy as heat should be greater at low temperatures than gas are allowed to expand isothermally from 1.0 dm3 to 3.0 dm3 at 300 K.
at high, as in eqn 3A.1a.
There are several final points. One is that the Boltzmann
definition of entropy makes it possible to calculate the absolute
3A.3 The entropy as a state function
Energy Allowed states Entropy is a state function. To prove this assertion, it is neces-
sary to show that the integral of dS between any two states is
independent of the path between them. To do so, it is sufficient
to prove that the integral of eqn 3A.1a round an arbitrary cycle
(a) is zero, for that guarantees that the entropy is the same at the
Energy initial and final states of the system regardless of the path taken
between them (Fig. 3A.7). That is, it is necessary to show that
Allowed states
dqrev
(b)  dS   T
 0 (3A.5)

Figure 3A.5 When a container expands from (b) to (a), the where the symbol ∫ denotes integration around a closed path.
translational energy states of gas molecules in it come closer There are three steps in the argument:
together and, for the same temperature, more become accessible
to the molecules. As a result the number of ways of achieving 1. First, to show that eqn 3A.5 is true for a special cycle
the same energy (the value of W ) increases, and so therefore (a ‘Carnot cycle’) involving a perfect gas as the ‘working
does the entropy. substance’.
3A Entropy 81

curve for stages 1 and 2; the work done compressing the gas
is the area under the curve for stages 3 and 4. It is evident
Final state from the diagram that more work is involved in the expan-
sion stages, so overall the cycle does work. This arrangement
Pressure, p

is an example of a heat engine in which energy as heat is


extracted from the hot reservoir, some of the energy is con-
verted to work, and some energy as heat is discarded into
Initial state
the cold reservoir.
Figure 3A.9 shows how the pressure and volume of the
working substance (the gas) change in each stage. The entropy
Volume, V change of the gas at each stage is calculated as follows.

Figure 3A.7 For a thermodynamic cycle that starts and ends


in the same state the overall change in a state function is zero,
1. The gas is placed in thermal contact with the hot source
regardless of the path. The cycle shown here goes from the initial (which is at temperature Th) and undergoes reversible
state to the final state by one path, and then returns to the initial isothermal expansion from A to B. The energy as heat that
state by another. enters the gas is qh; this is an endothermic process from
the point of view of the gas so qh is positive.
2. Contact with the hot source is broken and the gas then
2. Then to show that the result is true whatever the working undergoes reversible adiabatic expansion from B to C.
substance. No energy leaves or enters the gas as heat, so the change
3. Finally, to show that the result is true for any cycle. in entropy is zero. The expansion is carried on until the
temperature of the gas falls from Th to Tc, the temperature
of the cold sink.
3A.3(a) The Carnot cycle 3. The gas is placed in contact with the cold sink and then
A Carnot cycle, which is named after the French engineer Sadi undergoes a reversible isothermal compression from C to
Carnot, consists of four reversible stages in which a gas (the D at temperature Tc. The energy as heat that leaves the gas
working substance) is either expanded or compressed in a par- is qc; this is an exothermic process from the point of view
ticular sequence of ways (Fig. 3A.8). In stage 1 heat is trans- of the gas so qc is negative.
ferred from a hot source to the gas, and in stage 3 energy as heat 4. Finally, contact with the cold sink is broken and the gas
is transferred from the gas to a cold sink. then undergoes reversible adiabatic compression from
Figure 3A.8 is an indicator diagram (Topic 2A), which D to A such that the final temperature is Th. No energy
means that the area under the curve is the work done. The enters or leaves the gas as heat, so the change in entropy is
work done by the gas as it expands is the area under the zero.

A
Hot
4
Pressure, p

Stage 4 1 Isotherm Adiabat


Pressure, p

Stage 1
D
B
Cold Adiabat
Stage 3 2
Isotherm 3
Stage 2
C

Volume, V
Volume, V
Figure 3A.9 The basic structure of a Carnot cycle. Stage 1 is the
Figure 3A.8 The four stages which make up the Carnot cycle. In isothermal reversible expansion at the temperature Th. Stage 2 is
stage 1 the gas (the working substance) is in thermal contact with a reversible adiabatic expansion in which the temperature falls
the hot reservoir, and in stage 3 contact is with the cold reservoir; from Th to Tc. Stage 3 is an isothermal reversible compression at Tc.
both stages are isothermal. Stages 2 and 4 are adiabatic, with the Stage 4 is an adiabatic reversible compression, which restores the
gas isolated from both reservoirs. system to its initial state.
82 3 The Second and Third Laws

The total change in entropy of the gas for a complete cycle is the This expression can be rearranged into
sum of the changes in each of these four stages: qh qc
  0 (3A.6)
stage 1, stage 3,
Th Tc
qh 0 stage 2, qc 0 stage 4 ,
 q 0  q 0 Because the total change in entropy around the cycle is qh/Th +
qh  qc  q q qc/Tc, it follows immediately from eqn 3A.6 that, for a perfect
 d S 
Th
 0 
Tc
 0  h c
Th Tc gas, this entropy change is zero.

The next task is to show that the sum of the two terms on the
right of this expression is zero for a perfect gas and so confirm- Brief illustration 3A.2
ing, for that working substance at least, that entropy is a state
function. Consider an engine running in accord with the Carnot cycle,
and in which 100 J of energy is withdrawn from the hot source
and enters the gas (qh = +100 J) at 500 K. Some of this energy
How is that done? 3A.1 Showing that the entropy is a
is used to do work and the remainder is deposited in the cold
state function for a perfect gas sink at 300 K. According to eqn 3A.6, the heat leaving the gas is
First, you need to note that a reversible adiabatic expansion Tc 300 K
(stage 2 in Fig. 3A.9) takes the gas from Th to Tc. You can qc  qh   (100 J)   60 J
Th 500 K
then use the properties of such an expansion, specifically
VT c = constant (Topic 2E), to relate the two volumes at the start This value implies that, of the 100 J withdrawn from the hot
and end of the expansion. You also need to note that energy as source, 40 J was used to do work.
heat is transferred by reversible isothermal processes (stages 1
and 3) and, as derived in Example 3A.1, for a perfect gas It is now necessary to show that eqn 3A.5 applies to any ma-
 stage 1
 stage 3 terial, not just a perfect gas. To do so, it is helpful to introduce
VB V the efficiency, η (eta), of a heat engine:
= =
qh nRTh ln qc nRTc ln D
VA VC w
work performed Efficiencyy
  [definition]  (3A.7)
Step 1 Relate the volumes in the adiabatic expansions heat absorbed from hot source qh
For a reversible adiabatic process the temperature and volume
are related by VT c = constant (Topic 2E). Therefore Modulus signs (|…|) have been used to avoid complications
with signs: all efficiencies are positive numbers. The defini-
for the path D to A (stage 4): VAThc = VDTcc tion implies that the greater the work output for a given sup-
for the path B to C (stage 2): VCTcc = VBThc ply of heat from the hot source, the greater is the efficiency of
the engine. The definition can be expressed in terms of the heat
Multiplication of the first of these expressions by the second
transactions alone, because (as shown in Fig. 3A.10) the energy
gives
VAVCThcTcc = VDVBThcTcc
which, on cancellation of the temperatures, simplifies to
Th
VD VA Hot source
=
VC VB
20
Step 2 Establish the relation between the two heat transfers qh
You can now use this relation to write an expression for the w
5
energy that leaves the gas in stage 3 in terms of VA and VB
15
VD V V
qc = nRTc ln = nRTc ln A = −nRTc ln B qc
VC VB VA
Tc
The heat entering the gas from the hot source is Cold sink

VB
qh = nRTh ln
VA Figure 3A.10 In a heat engine, suppose that energy qh (for
It follows that example, |qh| = 20 kJ) is extracted as heat from the hot source
and energy |qc| is discarded into the cold sink (for example,
qh nRTh ln(VB /VA ) T |qc| = 15 kJ). The work done by the engine is equal to |qh| − |qc|
  h
qc nRTc ln(VB /VA ) Tc (e.g. 20 kJ − 15 kJ = 5 kJ).
3A Entropy 83

supplied as work by the engine is the difference between the production of work in the effective absence of a cold sink, is
energy supplied as heat by the hot source and that returned to contrary to the Kelvin statement of the Second Law, because
the cold sink: some heat has been converted directly into work.
Because the conclusion is contrary to experience, the initial
qh  qc qc
 1  (3A.8) assumption that engines A and B can have different efficien-
qh qh cies must be false. It follows that the relation between the heat
transfers and the temperatures must also be independent of the
It then follows from eqn 3A.6, written as |qc|/|qh| = Tc/Th that
working substance, and therefore that eqn 3A.9 is true for any
Tc substance involved in a Carnot cycle.
  1 Carnot efficiency  (3A.9)
Th For the final step of the argument note that any reversible
The next step is to show that eqn 3A.9, though derived for cycle can be approximated as a collection of Carnot cycles. This
a perfect gas as the working substance, is in fact true for any approximation is illustrated in Fig. 3A.12, which shows three
working substance. The argument hinges on the fact that the Carnot cycles A, B, and C fitted together in such a way that
Second Law of thermodynamics implies that all reversible en- their perimeter approximates the cycle indicated by the outer
gines have the same efficiency regardless of their construction. line. The entropy change around each individual cycle is zero
The first task is to show that this statement is true. (as already demonstrated). The entropy changes along the adi-
Consider the arrangement shown in Fig. 3A.11(a). Two heat abatic segments (such as a4–a1 and c2–c3) are zero, so it follows
engines work between the same hot source and cold sink but that the entropy changes along the isothermal segments of any
the engine on the left, A, is presumed to be more efficient than one cycle (such as a1–a2 and a3–a4) cancel. Therefore, the sum
that on the right, B. In engine A heat qh is extracted from the qrev/T around the perimeter is zero.
hot reservoir and heat qc is discarded to the cold sink; in the The path shown by the outer line can be approximated
process some work is done. Engine B is driven in the opposite more closely by using more Carnot cycles, each of which
direction with qc extracted from the cold sink and qh′ discarded is much smaller, and in the limit that they are infinitesi-
into the hot source. Because engine A is more efficient than mally small their perimeter matches the purple path exactly.
­engine B, not all the work it produces need be used to drive Equation 3A.5 (that the integral of dqrev/T round a general
engine B, so the joint arrangement produces work. cycle is zero) then follows immediately. This result implies
Now consider the overall effect. The cold sink is left unaf- that dS is an exact differential and therefore that S is a state
fected (energy discarded as heat by engine A and taken up by function.
engine B are the same). There is a net loss of heat from the hot
source, and there is a production of work from the joint op- 3A.3(b) The thermodynamic temperature
eration of the two engines (Fig. 3.11(b)). This outcome, the
Heat engines, have a further use: they put the measurement
of temperature on to an unambiguous foundation. Thus,
­suppose an engine works reversibly between a hot source at

Hot source Th Hot source Th a1

qh qh’ qh – qh’ b1
Pressure, p

a2 c1
A
A B A a4 b2
w
qc qc w
b4 B c2
a3 C
c4

Tc b3
Cold sink Cold sink
c3
(a) (b) Volume, V

Figure 3A.11 (a) The demonstration of the equivalence of the Figure 3A.12 The path indicated by the outer line can be
efficiencies of all reversible engines working between the same approximated by traversing the overall perimeter of the area
thermal reservoirs is based on the flow of energy represented in created by the three Carnot cycles A, B, and C; for each individual
this diagram. (b) The net effect of the processes is the conversion cycle the overall entropy change is zero. The entropy change
of heat into work without there being a need for a cold sink. This resulting from traversing the overall perimeter of the three cycles
is contrary to the Kelvin statement of the Second Law. is therefore zero.
84 3 The Second and Third Laws

a ­temperature Th and a cold sink at a temperature T, then it entropy change of the system dS and that of the surroundings
­follows from eqn 3A.9 that dSsurr: dStot  dS  dSsur . Because dSsur  dq /T and the Clausius
inequality states that dS ≥ dq /T , it follows that
T  (1   )Th  (3A.10)
dq /T  dq /T
 
This expression enabled Kelvin to define the thermodynamic dStot  dS  dSsur  0
temperature scale in terms of the efficiency of a heat engine:
construct an engine in which the hot source is at a known tem- The inequality corresponds to an irreversible process and the
perature and the cold sink is the object of interest. The tem- equality to a reversible process. That is, a spontaneous process
perature of the latter can then be inferred from the measured (dStot > 0) is an irreversible process. A reversible process, for
efficiency of the engine. The Kelvin scale (which is a special which dStot = 0, is spontaneous in neither direction: it is at equi-
case of the thermodynamic temperature scale) used to be de- librium.
fined by using water at its triple point as the notional hot source Apart from its fundamental importance in linking the defi-
and defining that temperature as 273.16 K exactly. However, as nition of entropy to the Second Law, the Clausius inequality
discussed in Topic 1A, the scale is now defined by referring it to can also be used to show that a familiar process, the cooling
the defined value of the Boltzmann constant. of an object to the temperature of its surroundings, is indeed
spontaneous. Consider the transfer of energy as heat from one
system—the hot source—at a temperature Th to another sys-
3A.3(c) The Clausius inequality tem—the cold sink—at a temperature Tc (Fig. 3A.13). When
The concept of entropy has been introduced as a way of relat- |dq| leaves the hot source (so dqh < 0), the Clausius inequal-
ing the Kelvin and Clausius statements, but can it stand alone ity implies that dS ≥ dqh/Th. When |dq| enters the cold sink
as the signpost of spontaneous change as expressed by the the Clausius inequality implies that dS ≥ dqc/Tc (with dqc > 0).
Second Law? That is, is it possible to prove that the entropy in- Overall, therefore,
creases in any spontaneous change? dq h dq c
First, note that more work is done when a change is revers- dS  
Th Tc
ible than when it is irreversible. That is, |dwrev| ≥ |dw|. Because
dw and dwrev are negative when energy leaves the system as However, dqh = −dqc, so
work, this expression is the same as −dwrev ≥ −dw, and hence
dw − dwrev ≥ 0. The internal energy is a state function, so its d q h d q c  dq c dq c  1 1 
dS         dq c
change is the same for irreversible and reversible paths between Th Tc Th Tc  Tc Th 
the same two states, and therefore
which is positive (because dqc > 0 and Th ≥ Tc). Hence, cooling
dU  dq  dw  dqrev  dwrev (the transfer of heat from hot to cold) is spontaneous, in accord
with experience.
and hence dqrev − dq = dw − dwrev . Then, because dw − dwrev ≥ 0,
it follows that dqrev − dq ≥ 0 and therefore dqrev ≥ dq. Division by
T then results in dqrev/T ≥ dq/T. From the thermodynamic defi- Th
nition of the entropy (dS = dqrev/T) it then follows that Hot source S
dq dS = –|dq|/Th
dS ≥ Clausius inequality  (3A.11)
T
This expression is the Clausius inequality. It proves to be of dq
great importance for the discussion of the spontaneity of chem-
ical reactions (Topic 3D).
Suppose a system is isolated from its surroundings, so that S
dq = 0. The Clausius inequality implies that Tc
Cold sink dS = +|dq|/Tc
dS ≥ 0 (3A.12)

That is, in an isolated system the entropy cannot decrease when a Figure 3A.13 When energy leaves a hot source as heat, the
spontaneous change occurs. This statement captures the content entropy of the source decreases. When the same quantity of
of the Second Law. energy enters a cooler sink, the increase in entropy is greater.
The Clausius inequality also implies that spontaneous pro- Hence, overall there is an increase in entropy and the process is
cesses are necessarily irreversible processes. To confirm this spontaneous. Relative changes in entropy are indicated by the
conclusion, separate the total entropy change dStot into the sizes of the arrows.
3A Entropy 85

Exercises
E3A.4 In a certain ideal heat engine, 10.00 kJ of heat is withdrawn from the hot E3A.5 What is the efficiency of an ideal heat engine in which the hot source is
source at 273 K and 3.00 kJ of work is generated. What is the temperature of at 100 °C and the cold sink is at 10 °C?
the cold sink?

Checklist of concepts
☐ 1. The entropy is a signpost of spontaneous change: the ☐ 5. The efficiency of a heat engine is the basis of the defini-
entropy of the universe increases in a spontaneous process. tion of the thermodynamic temperature scale and one
☐ 2. A change in entropy is defined in terms of reversible realization of such a scale, the Kelvin scale.
heat transactions. ☐ 6. The Clausius inequality is used to show that the entropy
☐ 3. The Boltzmann formula defines entropy in terms of of an isolated system increases in a spontaneous change
the number of ways that the molecules can be arranged and therefore that the Clausius definition is consistent
amongst the energy states, subject to the arrangements with the Second Law.
having the same overall energy. ☐ 7. Spontaneous processes are irreversible processes; pro-
☐ 4. The Carnot cycle is used to prove that entropy is a state cesses accompanied by no change in entropy are at
function. equilibrium.

Checklist of equations
Property Equation Comment Equation number
Thermodynamic entropy dS = dqrev/T Definition 3A.1a
Entropy change of surroundings ΔSsur = qsur/Tsur No hot spots 3A.2b
Boltzmann formula S = k ln W Definition 3A.4
Carnot efficiency η = 1 − Tc/Th Reversible process 3A.9
Thermodynamic temperature T = (1 − η)Th Definition 3A.10
Clausius inequality dS ≥ dq/T 3A.11
TOPIC 3B Entropy changes
­accompanying specific processes

3B.1 Expansion
➤ Why do you need to know this material?
An entropy change is associated with all physical processes In Topic 3A (specifically Example 3A.1) it is established that the
and to be able to apply the Second Law it is necessary to be change in entropy of a perfect gas when it expands isothermally
able to calculate its value. from Vi to Vf is

Vf Entropy change for the


➤ What is the key idea? S  nR ln isothermal expansion of  (3B.2)
The change in entropy accompanying a process is calcu- Vi a perfect gas
lated by identifying a reversible path between the initial
Because S is a state function, the value of ΔS of the system is
and final states.
independent of the path between the initial and final states, so
➤ What do you need to know already? this expression applies whether the change of state occurs re-
versibly or irreversibly. The logarithmic dependence of entropy
You need to be familiar with the thermodynamic definition
on volume is illustrated in Fig. 3B.1.
of entropy (Topic 3A), the First-Law concepts of work, heat,
The change in entropy of the surroundings, however, does
and internal energy (Topic 2A), and heat capacity (Topic
depend on how the expansion takes place. This entropy change
2B). The Topic makes use of the expressions for the work
is computed by using eqn 3B.1b, Ssur  qsur /Tsur , so all that is
and heat involved during the reversible, isothermal expan-
required is a knowledge of the energy supplied as heat to the
sion of a perfect gas (Topic 2A).
surroundings. Because for any process the energy lost as heat
from the system is acquired by the surroundings it follows that
qsur = −q, and hence Ssur  q /Tsur . In the reversible isother-
mal expansion of a perfect gas q = nRT ln(Vf/Vi), so qsur = −nRT
The thermodynamic definition of entropy change is (eqn 3A.1) ln(Vf/Vi), and consequently
dqrev f dqrev Entropy change q V
dS  S    (3B.1a) Ssur    nR ln f
T i T [definition] T Vi

where qrev is the energy supplied reversibly as heat to the system It follows that Stot  S  Ssur  0, as expected for a reversible
at a temperature T. This equation is the basis of all calculations process.
relating to entropy in thermodynamics. When applied to the
surroundings, it implies eqn 3A.2b, which is repeated here as 4
qsur Entropy change
Ssur  of surroundings  (3B.1b)
Tsur 3

where qsur is the energy supplied as heat to the surroundings


S/nR

and Tsur is their temperature. The entropy change of the sur- 2


roundings is the same whether or not the process it accompa-
nies is reversible or irreversible. The total change in entropy of
1
an (overall) isolated system (the ‘universe’) is

Stot  S  Ssur Total entropy change (3B.1c) 0


1 10 20 30
Vf /Vi
As explained in Topic 3A, a spontaneous process has ΔStot > 0
and is also irreversible (in the thermodynamic sense). A pro- Figure 3B.1 The logarithmic increase in entropy of a perfect gas
cess for which ΔStot = 0 is at equilibrium. as it expands isothermally.
3B Entropy changes ­accompanying specific processes 87

If the expansion is isothermal and occurs freely (pex = 0) no two phases in the system are in equilibrium. Because at constant
work is done, w = 0; then, because the process is isothermal, pressure q = ΔtrsH, the change in molar entropy of the system is1
ΔU = 0, it follows from the First Law, ΔU = q + w, that q = 0 too.
As a result, qsur = 0 and hence ΔSsur = 0. For free expansion the  trs H Entropy of phase
total entropy change is therefore equal to the entropy change of  trs S  transition at T
 (3B.3)
Ttrs trs
the system (eqn 3B.2), and Stot  nR ln(Vf /Vi ). As expected for
this irreversible process, ΔStot > 0. If the phase transition is exenthalpic (ΔtrsH < 0, as in freezing or
condensing), then the entropy change of the system is negative.
Brief illustration 3B.1 This decrease in entropy is consistent with the increased order
of the solid phase of a substance compared with that of its liquid
When the volume of a perfect gas is doubled at constant tem- phase, and with the increased order of its liquid phase compared
perature, Vf /Vi = 2, and hence the change in molar entropy of with its gas phase. The change in entropy of the surroundings,
the system is however, is positive because energy is released as heat into them.
Sm  R ln(Vf /Vi )  (8.3145 JK 1 mol 1 )  ln 2 At the transition temperature the total change in entropy is zero
 5.76 JK 1 mol 1 because the two phases are in equilibrium. If the transition is
endenthalpic (ΔtrsH > 0, as in melting and vaporization), then
If the change is carried out reversibly, the change in entropy
the entropy change of the system is positive, which is consistent
of the surroundings is −5.76 J K−1 mol−1 (the ‘per mole’ mean-
ing per mole of gas molecules in the sample). The total change with greater disorder of matter in the system. At the transition
in entropy is 0. If the expansion is free, the change in molar temperature the entropy of the surroundings decreases by the
entropy of the gas is still +5.76 J K−1 mol−1, but that of the sur- same amount, and overall the total change in entropy is zero.
roundings is 0, and the total change is +5.76 J K−1 mol−1. Table 3B.1 lists some experimental entropies of phase transi-
tions. Table 3B.2 lists in more detail the standard entropies of
vaporization of several liquids at their normal boiling points.
As always, the ‘standard’ value refers to the pure substance at
Exercises 1 bar. An interesting feature of the data is that a wide range of
E3B.1 Calculate the change in the entropies of the system and the liquids give approximately the same standard entropy of va-
surroundings, and the total change in entropy, when a sample of nitrogen porization (about 85 J K−1 mol−1): this empirical observation is
gas of mass 14 g at 298 K doubles its volume in (i) an isothermal reversible
expansion, (ii) an isothermal irreversible expansion against pex = 0, and
called Trouton’s rule. The explanation of Trouton’s rule is that
(iii) an adiabatic reversible expansion.
E3B.2 For the isothermal expansion of a perfect gas from volume Vi to Vf the Table 3B.1 Entropies of phase transitions, ΔtrsS/(J K−1 mol−1), at the
change in molar entropy is Sm  R ln(Vf /Vi ). Explain why an isothermal corresponding normal transition temperatures (at 1 atm)
expansion from pressure pi to pf (with pf < pi) has an entropy change
Sm  R ln( pi /pf ). A perfect gas undergoes an isothermal expansion such that Fusion (at Tf ) Vaporization (at Tb)
the initial pressure is halved. What is the change in the molar entropy? Argon, Ar 14.17 (at 83.8 K) 74.53 (at 87.3 K)
Benzene, C6H6 38.00 (at 279 K) 87.19 (at 353 K)
Water, H2O 22.00 (at 273.15 K) 109.1 (at 373.15 K)
3B.2 Phase transitions Helium, He 4.8 (at 1.8 K and 30 bar) 19.9 (at 4.22 K)

* More values are given in the Resource section.


When a substance freezes or boils the degree of dispersal of
matter and the associated energy changes reflect the order with Table 3B.2 The standard enthalpies and entropies of
which the molecules pack together and the extent to which the vaporization of liquids at their boiling temperatures*
energy is localized. Therefore, a phase transition is expected to
be accompanied by a change in entropy. For example, when a ΔvapS /

ΔvapH ⦵/(kJ mol−1) θb/°C


(J K−1 mol−1)
substance vaporizes, a compact condensed phase changes into
a widely dispersed gas, and the entropy of the substance can be Benzene 30.8 80.1 87.2

expected to increase considerably. The entropy of a solid also Carbon tetrachloride 30 76.7 85.8
increases when it melts to a liquid. Cyclohexane 30.1 80.7 85.1
Consider a system and its surroundings at the normal Hydrogen sulfide 18.7 −60.4 87.9
­transition temperature, Ttrs, the temperature at which two Methane 8.18 −161.5 73.2
phases are in equilibrium at 1 atm. This temperature is 0 °C Water 40.7 100.0 109.1
(273 K) for ice in equilibrium with liquid water at 1 atm, and
* More values are given in the Resource section.
100 °C (373 K) for water in equilibrium with its vapour at 1 atm.
At the transition temperature, any transfer of energy as heat be- 1
According to Topic 2C, ΔtrsH is an enthalpy change per mole of substance,
tween the system and its surroundings is reversible because the so ΔtrsS is also a molar quantity.
88 3 The Second and Third Laws

a similar change in volume occurs when any liquid evaporates the heat supplied to change its temperature from one value to
and becomes a gas. Hence, all liquids can be expected to have the other:
similar standard entropies of vaporization.
Liquids that show significant deviations from Trouton’s rule Tf dqrev 
do so on account of strong molecular interactions that result in a S(Tf )  S(Ti )   (3B.4)
Ti T
partial ordering of their molecules in the liquid. As a result, there
is a greater change in disorder when the liquid turns into a va- The most common version of this expression is for a system
pour than for when a fully disordered liquid vaporizes. An exam- subjected to constant pressure (such as from the atmos-
ple is water, where the high entropy of vaporization reflects the phere) during the heating. Under these circumstances the
presence of structure arising from hydrogen bonding in the liq- energy supplied as heat is equal to the enthalpy change, and
uid. Hydrogen bonds tend to organize the molecules in the liquid because enthalpy is a state function the enthalpy change is
so that they are less random than, for example, the molecules in unaffected by the path taken. It follows that dqrev = dH for
liquid hydrogen sulfide (in which there is no hydrogen bonding). a given change at constant pressure. From the definition of
Methane has an unusually low entropy of vaporization. A part constant-­pressure heat capacity (eqn 2B.5, Cp = (∂H/∂T)p) it
of the reason is that the entropy of the gas itself is slightly low follows that dH = CpdT, and hence dqrev = CpdT. Substitution
(186 J K−1 mol−1 at 298 K; the entropy of N2 under the same condi- into eqn 3B.4 gives
tions is 192 J K−1 mol−1). As explained in Topic 13B, fewer transla-
tional and rotational states are accessible at room temperature for
molecules with low mass and moments of inertia (like CH4) than Tf C p dT Entropy variation
for molecules with relatively high mass and moments of inertia S(Tf )  S(Ti )   with temperature  (3B.5)
Ti T [cconstant p ]
(like N2), so their molar entropy is slightly lower.

The same expression applies at constant volume, but with Cp


Brief illustration 3B.2
replaced by CV . When Cp is independent of temperature over
There is no hydrogen bonding in liquid bromine and Br2 is a the temperature range of interest, it can be taken outside the
heavy molecule which is unlikely to display unusual behaviour integral to give
in the gas phase, so it is safe to use Trouton’s rule. To predict the
standard molar enthalpy of vaporization of bromine given that Tf dT T
it boils at 59.2 °C, use Trouton’s rule in the form S(Tf )  S(Ti )  C p   S(Ti )  C p ln f  (3B.6)
Ti T Ti
 vap H   Tb  vapS   Tb  (85 JK 1 mol 1 )
⦵ ⦵

with a similar expression for heating at constant volume. The


Substitution of the data then gives
logarithmic dependence of entropy on temperature is illus-
 vap H   (332.4 K )  (85 JK 1 mol 1 )

trated in Fig. 3B.2.
 2.8  104 Jmol 1  28 kJmol 1

The experimental value is +29.45 kJ mol−1. 15

Exercises 10
3
ΔS/nR

E3B.3 Use Trouton’s rule to predict the enthalpy of vaporization of benzene


from its normal boiling point, 80.1 °C. 2
E3B.4 The enthalpy of vaporization of trichloromethane (chloroform, CHCl3) 5
is 29.4 kJ mol−1 at its normal boiling point of 334.88 K. Calculate (i) the 1
entropy of vaporization of trichloromethane at this temperature and (ii) the
entropy change of the surroundings.
0
1 10 20 30
Tf /Ti

3B.3 Heating Figure 3B.2 The logarithmic increase in entropy of a substance


as it is heated at either constant volume or constant pressure.
The thermodynamic definition of entropy change in eqn 3B.1a Different curves are labelled with the corresponding value of Cm/R,
is used to calculate the entropy of a system at a temperature Tf taken to be constant over the temperature range. For constant
from a knowledge of its entropy at another temperature Ti and volume conditions Cm = CV,m, and at constant pressure Cm = Cp,m.
3B Entropy changes ­accompanying specific processes 89

Brief illustration 3B.3 to the final volume, followed by heating at constant volume to
the final temperature. The entropy change in the first step is
The molar constant-volume heat capacity of water at 298 K is given by eqn 3B.2 and that of the second step, provided CV is
75.3 J K−1 mol−1. The change in molar entropy when it is heated independent of temperature, by eqn 3B.6 (with CV in place of
from 20 °C (293 K) to 50 °C (323 K), supposing the heat capac- Cp). In each case you need to know n, the amount of gas mol-
ity to be constant in that range, is therefore ecules, which can be calculated from the perfect gas equation
and the data for the initial state by using n = piVi/RTi.
Tf
Sm  Sm (323 K )  Sm (293 K )  CV ln
Ti The solution The amount of gas molecules is
323 K
 (75.3 J K 1 mol 1 )  ln 5 3 3
293 K piVi (1.00  10 Pa )  (0.500  10 m )
n   0.0201  mol
 7.34 J K 1 mol 1 RTi (8.3145 JK 1 mol 1 )  (298 K )

From eqn 3B.2 the entropy change in the isothermal expansion


When the heat capacity cannot be taken to be constant in the from Vi to Vf is
range of interest, the best procedure is to express Cp as a poly-
nomial in T, as described in Topic 2B, and then to integrate the Vf
S(Step 1)  nR ln
resulting expression (see Problems P3B.3 and P3B.5). Vi
1.000 dm3
 (0.0201  mol)  (8.3145 JK 1 mol 1 )ln
0.500 dm3
Exercises  0.116  JK 1

E3B.5 Estimate the increase in the molar entropy of O2(g) when the
temperature is increased at constant pressure from 298 K to 348 K, given that From eqn 3B.6, the entropy change in the second step, heating
the molar constant-pressure heat capacity of O2 is 29.355 J K−1 mol−1 at 298 K.
from Ti to Tf at constant volume, is
−1 −1
E3B.6 The molar entropy of a sample of neon at 298 K is 146.22 J K mol .
The sample is heated at constant volume to 500 K; assuming that the molar Tf T
constant-volume heat capacity of neon is 23 R, calculate the molar entropy of S(Step 2)  nCV,m ln  n  23 R ln f
the sample at 500 K. Ti Ti
373 K
 (0.0201  mol)  23  (8.3145 J K 1 mol 1 )ln
298 K
 0.0564  JK 1
3B.4 Composite processes The overall entropy change of the system, the sum of these two
changes, is
In many processes, more than one parameter changes. For in-
stance, it might be the case that both the volume and the tem-
perature of a gas are different in the initial and final states. S  0.116  JK 1  0.0564  JK 1  0.173 JK 1
Because S is a state function, the change in its value can be cal-
Self-test 3B.1 Calculate the entropy change when the same
culated by considering any reversible path between the initial
initial sample is compressed to 0.0500 dm3 and cooled to
and final states. For example, it might be convenient to split the
−25 °C.
path into two steps: an isothermal expansion to the final vol- Answer: −0.43 J K−1
ume, followed by heating at constant volume to the final tem-
perature. Then the total entropy change when both variables
change is the sum of the two contributions.

Exercises
Example 3B.1 Calculating the entropy change for a E3B.7 Calculate ΔS (for the system) when the state of 3.00 mol of gas molecules,
composite process for which C p,m = 25 R, is changed from 25 °C and 1.00 atm to 125 °C and
5.00 atm.
Calculate the entropy change when argon at 25 °C and 1.00 bar
E3B.8 Calculate the change in entropy of the system when 10.0 g of ice at
in a container of volume 0.500 dm3 is allowed to expand to −10.0 °C is converted into water vapour at 115.0 °C and at a constant pressure
1.000 dm3 and is simultaneously heated to 100 °C. (Take the of 1 bar. The molar constant-pressure heat capacities are: Cp,m(H2O(s)) =
molar heat capacity at constant volume to be 23 R.) 37.6 J K−1 mol−1; Cp,m(H2O(l)) = 75.3 J K−1 mol−1; and Cp,m(H2O(g)) =
33.6 J K−1 mol−1. The standard enthalpy of vaporization of H2O(l) is
Collect your thoughts As remarked in the text, you can break 40.7 kJ mol−1, and the standard enthalpy of fusion of H2O(l) is 6.01 kJ mol−1,
the overall process down into two steps: isothermal expansion both at the relevant transition temperatures.
90 3 The Second and Third Laws

Checklist of concepts
☐ 1. The entropy of a perfect gas increases when it expands ☐ 3. The increase in entropy when a substance is heated is
isothermally. calculated from its heat capacity.
☐ 2. The change in entropy of a substance accompanying a
change of state at its transition temperature is calculated
from its enthalpy of transition.

Checklist of equations
Property Equation Comment Equation number
Entropy of isothermal expansion ΔS = nR ln(Vf/Vi) Perfect gas 3B.2
Entropy of transition ΔtrsS = ΔtrsH/Ttrs At the transition temperature 3B.3
Variation of entropy with temperature S(Tf ) = S(Ti) + C ln(Tf /Ti) C independent of temperature and no phase 3B.6
transitions occur; C = Cp or CV
TOPIC 3C The measurement of entropy

3C.1The calorimetric measurement


➤ Why do you need to know this material?
For entropy to be a quantitatively useful concept it is
of entropy
important to be able to measure it: the calorimetric proce-
According to eqn 3C.1a, the entropy of a system at a tempera-
dure for determining entropy is described here.
ture T is related to its entropy at T = 0 by measuring its heat ca-
➤ What is the key idea? pacity Cp at different temperatures and evaluating the integral
up to temperature T. The entropy of transition for each phase
The entropies of substances can be found from measure-
transition that occurs between T = 0 and the temperature of
ments of heat capacities and the Third Law of thermody-
interest must then be included in the overall sum. For exam-
namics establishes an absolute scale for entropy.
ple, if a substance melts at Tf and boils at Tb, then its molar
➤ What do you need to know already? entropy at a particular temperature T above its boiling tem-
perature is given by
You need to be familiar with the expression for the tem-
perature dependence of entropy and how entropies of Heat solid
to its Entropy of
phase changes are calculated (Topic 3B). The discussion of melting point fusion
   
residual entropy draws on the Boltzmann formula for the 
Tf C p ,m (s,T )  fus H (Tf )
entropy (Topic 3A). Sm (T )  Sm (0)   dT  
0 T Tf
Heat liquid
to its Entropy of
boiling point vaporization
  
Tb C p ,m (l,T )  vap H (Tb )  (3C.2)
 dT  
The entropy of a substance can be determined in two ways. Tf T Tb
One, which is the subject of this Topic, is to make calorimetric Heat vapour
measurements of the heat required to raise the temperature of to the
final temperature
a sample from T = 0 to the temperature of interest. There are 
T C p ,m (g,T )
then two equations to use. One is the dependence of entropy on  dT 
temperature, which is eqn 3B.5 reproduced here as Tb T
The variable of integration has been changed to T ′ to avoid
T2 C p(T )
S(T2 )  S(T1 )   dT
Entropy and
 (3C.1a) confusion with the temperature of interest, T. All the properties
T temperature
T1
required, except Sm(0), can be measured calorimetrically, and
the integrals can be evaluated either graphically or, as is now
The second is the contribution of a phase change to the entropy,
more usual, by fitting a polynomial to the data and integrating
which according to eqn 3B.3 is
the ­polynomial analytically. The former procedure is illustrated
 trs H (Ttrs ) Entropy of phase
in Fig. 3C.1: the area under the curve of Cp,m(T)/T against T is
S(Ttrs )   (3C.1b) the ­integral required. Provided all measurements are made at
Ttrs transition
1 bar on a pure material, the final value is the standard ­entropy,
where ΔtrsH(Ttrs) is the enthalpy of transition at the transition S (T); division by the amount of substance, n, gives the

temperature Ttrs. The other method, which is described in Topic ­standard molar entropy, Sm⦵ (T ) = S ⦵ (T )/n. Because dT/T = d ln T,
13E, is to use computed parameters or spectroscopic data to an alternative procedure is to evaluate the area under a plot of
calculate the entropy by using Boltzmann’s statistical definition. Cp,m(T) against ln T.
92 3 The Second and Third Laws

Brief illustration 3C.1 Example 3C.1 Calculating the entropy at low


temperatures
The standard molar entropy of nitrogen gas at 25 °C has been
calculated from the following data: The molar constant-pressure heat capacity of a certain non-
metallic solid at 4.2 K is 0.43 J K−1 mol−1. What is its molar
Contribution to entropy at that temperature?
Sm /(JK −1 mol −1 )

Debye extrapolation 1.92 Collect your thoughts Because the temperature is so low,
Integration, from 10 K to 35.61 K 25.25 you can assume that the heat capacity varies with tempera-
ture according to Cp,m(T) = aT 3, in which case you can use
Phase transition at 35.61 K 6.43
eqn 3C.1a to calculate the entropy at a temperature T in terms
Integration, from 35.61 K to 63.14 K 23.38
of the entropy at T = 0 and the constant a. When the integra-
Fusion at 63.14 K 11.42 tion is carried out, it turns out that the result can be expressed
Integration, from 63.14 K to 77.32 K 11.41 in terms of the heat capacity at the temperature T, so the data
Vaporization at 77.32 K 72.13 can be used directly to calculate the entropy.
Integration, from 77.32 K to 298.15 K 39.20 The solution The integration required is
Correction for gas imperfection 0.92 Integral A.1
  
Total 192.06 T aT 3 T
Sm(T )  Sm(0)   dT  Sm(0)  a  T dT 
2
 
0 T 0
1 1
Therefore, Sm(298.15 K )  Sm(0)  192.1 JK mol . The Debye


C p,m(T )

extrapolation is explained in the next paragraph.  Sm(0)  13 aT 3  Sm(0)  13 C p ,m(T )
from which it follows that
One problem with the determination of entropy is the difficulty
Sm(4.2 K)  Sm(0)  0.14 JK 1 mol 1
of measuring heat capacities near T = 0. There are good theoreti-
cal grounds for assuming that the heat capacity of a non-metallic Self-test 3C.1 For metals, there is also a contribution to the
solid is proportional to T 3 when T is close to zero (see Topic 7A), heat capacity from the electrons which is linearly propor-
and this dependence is the basis of the Debye extrapolation (or tional to T when the temperature is low; that is, Cp,m(T) = bT.
the Debye T 3-law). In this method, Cp is measured down to as Evaluate its contribution to the entropy at low temperatures.
low a temperature as possible and a curve of the form aT 3 is fitted Answer: Sm(T) = Sm(0) + Cp,m(T)
to the data. The fit determines the value of a, and the expression
Cp,m(T) = aT 3 is then assumed to be valid down to T = 0.
Exercise
Melts

Boils
approximation

−1
E3C.1 At 4.2 K the heat capacity of a certain non-metallic solid is 0.0145 J K
mol−1. Assuming that the Debye law applies, determine Sm(4.2 K) − Sm(0) for
the solid.
Debye
Cp/T

Solid Liquid Gas 3C.2 The Third Law


At T = 0 thermal motion ceases, and in a perfect crystal all the
vapH/Tb
atoms or ions are in a regular, uniform array. The localization
fusH/Tf of matter and the absence of thermal motion suggest that such
S

materials also have zero entropy. This conclusion is consistent


Solid Liquid Gas
with the molecular interpretation of entropy (Topic 3A) be-
cause there is only one way of arranging the molecules when
Tf T Tb
they are all in the ground state, which is the case at T = 0. Thus,
Figure 3C.1 The variation of Cp/T with the temperature for a at T = 0, W = 1 and from S = k lnW it follows that S = 0.
sample is used to evaluate the entropy, which is equal to the area
beneath the upper curve up to the corresponding temperature, 3C.2(a) The Nernst heat theorem
plus the entropy of each phase transition encountered between
T = 0 and the temperature of interest. For instance, the entropy The Nernst heat theorem summarizes a series of experimental
denoted by the yellow dot on the lower curve is given by the dark observations that turn out to be consistent with the view that
shaded area in the upper graph. the entropy of a regular array of molecules is zero at T = 0:
3C The measurement of entropy 93

The entropy change accompanying any physical or molecules: a given O atom has two short O−H bonds and two
chemical transformation approaches zero as the tem­ long O  H bonds to its neighbours, but there is a degree of
perature approaches zero: ΔS → 0 as T → 0 provided randomness in which two bonds are short and which two are
all the substances involved are perfectly ordered. long. Topic 13E explains how the value of the residual entropy
of water may be estimated.
Brief illustration 3C.2
3C.2(b) Third-Law entropies
The entropy of the transition between orthorhombic sulfur, α,
and monoclinic sulfur, β, can be calculated from the transition Entropies reported on the basis that S(0) = 0 are called Third-
enthalpy (402 J mol−1) at the transition temperature (369 K): Law entropies (and commonly just ‘entropies’). When the sub-
stance is in its standard state at the temperature T, the standard
 trs S(369 K )  Sm(,369 K )  Sm(,369 K ) (Third-Law) entropy is denoted S (T). A list of values at 298 K

 trs H (Ttrs ) 402 Jmol


1
is given in Table 3C.1.
   1.09 JK 1 mol 1
The standard reaction entropy, ΔrS , is defined, like the

Ttrs 369 K
standard reaction enthalpy in Topic 2C, as the difference be-
The entropies of the α and β allotropes can also be determined tween the molar entropies of the pure, separated products
by measuring their heat capacities from T = 0 up to T = 369 K. and the pure, separated reactants, all substances being in their
It is found that Sm(α,369 K) = Sm(α,0) + 37 J K−1 mol−1 and Sm(β, standard states at the specified temperature:
369 K) = Sm(β,0) + 38 J K−1 mol−1. These two values imply that at
the transition temperature Standard reaction
r S    S  S
⦵ ⦵
 ⦵

m  m entropy  (3C.3a)
 trs S(369 K )  {Sm(,0)  38 JK 1 mol 1 } Products Reactants [definition]
 {Sm(,0)  37 JK 1 mol 1 }
In this expression, each term is weighted by the appropriate
 Sm(,0)  Sm(,0)  1 JK 1 mol 1
stoichiometric coefficient. A more sophisticated approach is to
The two values of ∆ trs S(369 K ) are consistent with one another adopt the notation introduced in Topic 2C and to write
provided that Sm(β,0) − Sm(α,0) ≈ 0, in accord with the theorem.
 r S    J Sm (J)
⦵ ⦵

(3C.3b)
J

It follows from the Nernst theorem that, if the value zero is where the nJ are signed (+ for products, − for reactants) stoichi-
ascribed to the entropies of elements in their perfect crystal- ometric numbers. Standard reaction entropies are likely to be
line form at T = 0, then all perfect crystalline compounds also
have zero entropy at T = 0 (because the change in entropy that Table 3C.1 Standard Third-Law entropies at 298 K*
accompanies the formation of the compounds, like the entropy
of all transformations at that temperature, is zero). This conclu- Sm⦵ /(JK −1 mol −1 )
sion is summarized by the Third Law of thermodynamics: Solids
Graphite, C(s) 5.7
The entropy of every perfect crystalline substance is zero Diamond, C(s) 2.4
at T = 0.
Sucrose, C12H22O11(s) 360.2
Iodine, I2(s) 116.1
As far as thermodynamics is concerned, choosing this common
value as zero is a matter of convenience. As noted above, the Liquids
molecular interpretation of entropy justifies the value S = 0 at Benzene, C6H6(l) 173.3
T = 0 because at this temperature W = 1.
Water, H2O(l) 69.9
In certain cases W > 1 at T = 0 and therefore S(0) > 0. This is
Mercury, Hg(l) 76.0
the case if one arrangement of the molecules has essentially the
same energy as another, in which case even as the temperature Gases
goes to zero neither arrangement is preferred. For instance, for Methane, CH4(g) 186.3
a diatomic molecule AB there may be almost no energy differ- Carbon dioxide, CO2(g) 213.7
ence between the arrangements …AB AB AB… and …BA AB
Hydrogen, H2(g) 130.7
BA… in a solid, so W > 1 even at T = 0.
Helium, He(g) 126.2
If S(0) > 0 the substance is said to have a residual entropy.
Ammonia, NH3(g) 192.4
Ice has a residual entropy of 3.4 J K−1 mol−1. It stems from the ar-
rangement of the hydrogen bonds between neighbouring water * More values are given in the Resource section.
94 3 The Second and Third Laws

positive if there is a net formation of gas in a reaction, and are water), whereas that of Mg2+(aq) is 138 J K−1 mol−1 lower (pre-
likely to be negative if there is a net consumption of gas. Note sumably because its higher charge induces more local structure
that whereas the standard enthalpies of formation of elements in the surrounding water).
in their reference states are zero, the standard molar entropies
of elements have non-zero values (provided T > 0).
The temperature dependence of
3C.2(c)
Brief illustration 3C.3 reaction entropy
The temperature dependence of entropy is given by eqn 3C.1a,
To calculate the standard reaction entropy of H2 (g )  12 O2 (g ) 
which for the molar entropy becomes
H2O(l ) at 298 K, use the data in Table 2C.7 of the Resource sec-
tion to write T2 C p ,m (T )
Sm (T2 )  Sm (T1 )   dT 
⦵ ⦵

 r S  Sm(H2O, l )  Sm(H2 , g )  Sm(O2 , g )
  
⦵ 1
2


 T1 T
 69.9 JK 1 mol 1  1330.7  12 (205.1) JK 1 mol 1 This equation applies to each substance in the reaction, so from
1 1 eqn 3C.3 the temperature dependence of the standard reaction
 163.4 JK mol
entropy, ΔrS , is

The negative value is consistent with the conversion of two


gases to a compact liquid.  r C p (T )

T2
 r S (T2 )   r S (T1 )  
 
dT  (3C.5a)
⦵ ⦵

T1 T
Just as in the discussion of enthalpies in Topic 2C, where it where ∆ r C p (T ) is the difference of the molar heat capacities of

is acknowledged that solutions of cations cannot be prepared products and reactants under standard conditions at the tem-
in the absence of anions, the standard molar entropies of ions perature T weighted by the stoichiometric numbers that appear
in solution are reported on a scale in which by convention the in the chemical equation:
standard entropy of the H+ ions in water is taken as zero at all
temperatures:  r C p (T )   JC p,m (J,T )
⦵ ⦵
(3C.5b)
J
Ions in solution
S  (H , aq )  0 

(3C.4) Equation 3C.5a is analogous to Kirchhoff ’s law for the tem-
[convention]
perature dependence of ΔrH (eqn 2C.7a). If ∆ r C p (T ) is inde-
⦵ ⦵

Table 2C.7 in the Resource section lists some values of standard pendent of temperature in the range T1 to T2, the integral in eqn
entropies of ions in solution using this convention.1 Because 3C.5a evaluates to ∆ r C p ln(T2 /T1 ) and

the entropies of ions in water are values relative to the hy-


drogen ion in water, they may be either positive or negative. T2
 r S  (T2 )   r S  (T1 )   r C p ln 
⦵ ⦵ ⦵
(3C.5c)
A positive entropy means that an ion has a higher molar en- T1
tropy than H+ in water and a negative entropy means that the
ion has a lower molar entropy than H+ in water. The varia- Brief illustration 3C.4
tion in the entropies of ions can be explained by relating the
entropy to the extent to which the ions order the water mol- The standard reaction entropy for H2 (g )  12 O2 (g )  H2O(g )
ecules around them in the solution. Small, highly charged ions at 298 K is −44.42 J K−1 mol−1, and the molar heat capacities
induce local structure in the surrounding water, and the disor- at constant pressure of the species involved are H2O(g):
der of the solution is decreased more than in the case of large, 33.58 J K−1 mol−1; H2(g): 28.82 J K−1 mol−1; O2(g): 29.55 J K−1 mol−1.
singly charged ions. The absolute, Third-Law standard molar It follows that
entropy of the proton in water can be estimated by proposing  rC p  C p ,m(H2O, g )  C p ,m(H2 , g )  12 C p ,m(O2 , g )
⦵ ⦵ ⦵ ⦵

a model of the structure it induces, and there is some agree-


 9.94 JK 1 mol 1
ment on the value −21 J K−1 mol−1. The negative value indicates
that the proton induces order in the solvent. The standard This value of ∆ rC p is used in eqn 3C.5c to find ΔrS at another
⦵ ⦵

molar entropy of Cl−(aq) is +57 J K−1 mol−1 and that of Mg2+(aq) temperature, for example at 373 K
is −138 J K−1 mol−1. That is, the molar entropy of Cl−(aq) is T2
 r S (373 K )   r S (T1 )   rC p ln
⦵ ⦵ ⦵

57 J K−1 mol−1 higher than that of the proton in water (presum- T1


ably because Cl− induces less local structure in the surrounding  44.42 JK 1 mol 1
373 K
1
In terms of the language introduced in Topic 5A, the entropies of ions in solu-
(9.94 JK 1 mol 1 )  ln
298 K
tion are actually partial molar entropies, for their values include the consequences 1 1
of their presence on the organization of the solvent molecules around them.  46.65 JK mol
3C The measurement of entropy 95

Exercises
E3C.2 Using the data at 298 K given below, calculate the standard reaction E3C.3 Using the data given below at 298 K, calculate the standard reaction
entropy at 298 K of the reaction below. entropy at 298 K when 1 mol NH3(g) is formed from its elements in their
reference states.
2 AgCl(s) + Br2(l) → 2 AgBr(s) + Cl2(g)

Sm⦵ /(J K−1 mol−1) Sm⦵ /(J K−1 mol−1)


AgCl(s) 96.2 N2(g) 191.61
Br2(l) 152.23 H2(g) 130.684
AgBr(s) 107.1 NH3(g) 192.45
Cl2(g) 223.07

Checklist of concepts
☐ 1. Entropies are determined calorimetrically by measur- ☐ 4. The Third Law of thermodynamics states that the
ing the heat capacity of a substance from low tempera- entropy of every perfect crystalline substance is zero
tures up to the temperature of interest and taking into at T = 0.
account any phase transitions in that range. ☐ 5. The residual entropy of a solid is the entropy arising
3
☐ 2. The Debye extrapolation (or the Debye T -law) is used from disorder that persists at T = 0.
to estimate heat capacities of non-metallic solids close ☐ 6. Third-Law entropies are entropies based on S(0) = 0.
to T = 0.
☐ 7. The standard entropies of ions in solution are based on
☐ 3. The Nernst heat theorem states that the entropy change setting S (H+, aq) = 0 at all temperatures.

accompanying any physical or chemical transforma-


☐ 8. The standard reaction entropy, ΔrS , is the difference

tion approaches zero as the temperature approaches


between the molar entropies of the pure, separated
zero provided all the substances involved are perfectly
products and the pure, separated reactants, all sub-
ordered.
stances being in their standard states.

Checklist of equations
Property Equation Comment Equation number

C p ,m(s,T )
Tf
Standard molar entropy from Sm(T )  Sm(0)   dT  3C.2
calorimetry
0T
 H (Tf ) Tb C p ,m(l,T )
 fus  dT 
Tf Tf T
 H (Tb ) T C p ,m(g,T )
 vap  dT 
Tb Tb T
Standard reaction entropy:  r S ⦵   S
Products


m   S
Reactants


m
n: (positive) stoichiometric coefficients 3C.3a

 r S ⦵   J Sm⦵(J) nJ: (signed) stoichiometric numbers 3C.3b


J

T2
Temperature dependence of the  r S ⦵ (T2 )   r S ⦵ (T1 )   { r C p⦵ (T )/T } dT  3C.5a
T1
standard reaction entropy
 r S⦵ (T2 )   r S ⦵ (T1 )   r C p⦵ ln(T2 /T1 ) ∆ r C p⦵ independent of temperature 3C.5c
TOPIC 3D Concentrating on the system

The inequality dS  dq /T  0 applies when the process is


➤ Why do you need to know this material? spontaneous; at equilibrium the equality applies, and then
Most processes of interest in chemistry occur at constant dS  dq /T  0. Equation 3D.1 can be developed in two ways:
temperature and pressure. Under these conditions, the first for a process taking place at constant volume and second
Gibbs energy provides a convenient way of understanding for a process taking place at constant pressure.
physical and chemical equilibria.
3D.1(a) Criteria of spontaneity
➤ What is the key idea?
A process is spontaneous under conditions of constant At constant volume and in the absence of additional (non-
temperature and pressure if it is accompanied by a expansion) work the energy transferred as heat is equal to the
decrease in the Gibbs energy. change in the internal energy: dqV = dU. Equation 3D.1 then
becomes
➤ What do you need to know already?
dU
This Topic develops the Clausius inequality (Topic 3A) and dS  0 (constant V, no additional work )
T
draws on information about standard states and reaction
enthalpy introduced in Topic 2C.
The importance of this relation is that it expresses the criteria
for spontaneous change and equilibrium solely in terms of state
functions of the system (U and S). The inequality is easily rear-
ranged into

T dS ≥ dU (constant V, no additional work ) (3D.2)


Entropy is the basic concept for discussing the direction of nat-
ural change, but to use it the changes in both the system and its If dU = 0, then it follows that TdS ≥ 0, but as T > 0, this relation
surroundings must be assessed. In Topic 3A it is shown that it can be written dSU,V ≥ 0, where the subscripts indicate which
is straightforward to calculate the entropy change in the sur- properties are held constant. This expression states that in a sys-
roundings (from ΔSsur = qsur/Tsur) and this Topic shows that it is tem at constant volume and constant internal energy (such as an
possible to devise a simple method for taking this contribution isolated system), the entropy increases in a spontaneous change.
into account automatically. This approach focuses attention on That statement is essentially the content of the Second Law.
the system and simplifies discussions. Moreover, it is the foun- When energy is transferred as heat at constant pressure and
dation of all the applications of chemical thermodynamics that there is no work other than expansion work, the energy trans-
follow. ferred as heat is equal to the enthalpy change: dqp = dH. Now
eqn 3D.1 becomes
T dS ≥ dH (constant p, no additional work ) (3D.3)
The Helmholtz and Gibbs
3D.1
If the enthalpy is constant as well as the pressure, this relation be-
energies comes TdS ≥ 0 and therefore dS ≥ 0. This conclusion and its con-
straints may be written dSH,p ≥ 0. That is, in a spontaneous process
Consider a system in thermal equilibrium with its surround- the entropy of the system at constant pressure must increase if its
ings at a temperature T. The Clausius inequality (eqn 3A.11), enthalpy remains constant (under these circumstances there can
dS ≥ dq/T, relates the entropy change of the system dS and the then be no change in entropy of the surroundings).
accompanying transfer of energy as heat between the system The criteria of spontaneity at constant volume and pressure
and the surroundings dq. For this discussion it is convenient to can be expressed more simply by introducing two more ther-
rewrite the inequality as modynamic quantities. One is the Helmholtz energy, A, which
is defined as
dq
dS   0 (3D.1)
A  U  TS
Helmholtz energy
 (3D.4a)
T [definition]
3D Concentrating on the system 97

The other is the Gibbs energy, G, defined as will be reached at which the Helmholtz energy is a minimum.
Any change from this minimum would result in an increase in
Gibbs energy
G  H  TS  (3D.4b) A, and hence not be spontaneous. Once at the minimum the
[definition]
system has reached equilibrium, and dAT,V = 0.
All the symbols in these two definitions refer to the system. The expressions dA = dU − TdS and dAT,V < 0 are sometimes
Suppose that the state of the system changes at constant tem- interpreted as follows. A negative value of dA is favoured by a
perature, meaning that U changes by dU, S by dS, and H by dH. negative value of dU and a positive value of TdS. This observation
It follows from the definitions in eqns 3D.4 that the Helmholtz suggests that the tendency of a system to move to lower A is due
and Gibbs energies change as follows: to its tendency to move towards states of lower internal energy
and higher entropy. However, this interpretation is false because
(a ) dA  dU  TdS (b) dG  dH  TdS (3D.5) the tendency to lower A is solely a tendency towards states of
greater overall entropy. Systems change spontaneously if in doing
At constant volume, TdS ≥ dU (eqn 3D.2) which, by using (a), so the total entropy of the system and its surroundings increases,
implies dA ≤ 0. At constant pressure, TdS ≥ dH (eqn 3D.3) not because they tend to lower internal energy. The form of dA
which, by using (b), implies dG ≤ 0. Using the subscript notation may give the impression that systems favour lower energy, but
to indicate which properties are held constant, the criteria of that is misleading: dS is the entropy change of the system, −dU/T
spontaneous change and equilibrium in terms of dA and dG are is the entropy change of the surroundings (when the volume of
the system is constant), and their total tends to a maximum.
Criteria of spontaneous
(a ) dAT, V ≤ 0 (b) dGT, p ≤ 0  (3D.6)
change and eq quilibrium
3D.1(c) Maximum work
These criteria, especially the second, are central to chemical ther-
modynamics. For instance, in an endenthalpic process (less pre- A short argument can be used to show that the change in the
cisely but more colloquially, an ‘endothermic’ process), such as Helmholtz energy is equal to the maximum work obtainable from
the melting of a solid, H increases, dH > 0, but if such a process is a system at constant temperature.
to be spontaneous at constant temperature and pressure, G must
decrease. It is possible for dG = dH − TdS to be negative provided How is that done? 3D.1 Relating the change in the
that the process involves an increase in the entropy of the system
Helmholtz energy to the maximum work
and that the temperature is high enough for TdS to outweigh dH.
In the case of melting there is indeed an increase in the entropy To demonstrate that maximum work can be expressed in terms
of the system and therefore if the temperature is high enough of the change in Helmholtz energy, you need to combine the
the process is spontaneous. Endenthalpic processes are therefore Clausius inequality dS ≥ dq/T in the form TdS ≥ dq with the
driven by the increase of the entropy of the system. That increase First Law, dU = dq + dw, and obtain
in entropy overcomes the reduction of entropy in the surround- dU  T dS  dw
ings brought about by their loss of heat to the system.
For processes in which there is a decrease in the entropy of The term dU is smaller than the sum of the two terms on the
the system, such as the condensation of a vapour to a liquid, right because dq has been replaced by TdS, which in general is
the term −TdS is positive, and becomes more so as the tem- larger than dq. This expression rearranges to
perature increases. Such a process is spontaneous only if dH dw  dU  T dS
is sufficiently negative for it to outweigh −TdS. In other words
a process in which the entropy of the system decreases can It follows that the most negative value of dw is obtained when
be spontaneous provided that it is exenthalpic (and the tem- the equality applies, which is for a reversible process. Thus
a reversible process produces the maximum work, and this
perature is low enough). This is the case for condensation at
maximum is given by
temperatures below the boiling point. In such a process the de-
crease in the entropy of the system is overcome by an increase dwmax  dU  TdS
in the entropy of the surroundings that results from the flow of
heat from the system in an exenthalpic process. Because at constant temperature dA = dU − TdS (eqn 3D.5), it
follows that

Some remarks on the Helmholtz


3D.1(b) dwmax = dA
(3D.7)
Maximum work
energy [constant T ]

At constant temperature and volume, a change is spontaneous In recognition of this relation, A is sometimes called the ‘work
if it corresponds to a decrease in the Helmholtz energy of the function’.1
system: dAT,V < 0. A system tends to move spontaneously to-
wards states of lower A if a path is available. Eventually a point 1
Arbeit is the German word for work; hence the symbol A.
98 3 The Second and Third Laws

When a measurable isothermal change takes place in the sys-


tem, eqn 3D.7 becomes wmax = ΔA with ΔA = ΔU − TΔS. These
two relations show that, depending on the sign of TΔS, not all |q|
the change in internal energy may be available for doing work.
If the change occurs with a decrease in entropy (of the system), U < 0
in which case TΔS < 0, then ΔU − TΔS is not as negative as ΔU S > 0
itself, and consequently the maximum work is less than ΔU. For |w| >U|
the change to be spontaneous, some of the energy must escape
as heat in order to generate enough entropy in the surroundings Ssur < 0
to overcome the reduction in entropy in the system (Fig. 3D.1).
In this case, Nature is demanding a tax on the internal energy Figure 3D.2 In this process, the entropy of the system increases;
as it is converted into work. This interpretation is the origin of hence some reduction in the entropy of the surroundings can be
the alternative name ‘Helmholtz free energy’ for A, because ΔA tolerated. That is, some of their energy may be lost as heat to the
is that part of the change in internal energy free to do work. system. This energy can be returned to them as work, and hence
Further insight into the relation between the work that a the work done can exceed |ΔU|.
system can do and the Helmholtz energy is to recall that work
is energy transferred to the surroundings as the uniform mo-
tion of atoms. The expression A = U − TS can be interpreted as Example 3D.1 Calculating the maximum available work
showing that A is the total internal energy of the system, U, less
a contribution that is stored as energy of thermal motion (the When 1.000 mol C6H12O6 (glucose) is oxidized completely to
quantity TS). Because energy stored in random thermal motion carbon dioxide and water at 25 °C according to the equation C6
cannot be used to achieve uniform motion in the surroundings, H12O6(s) + 6 O2(g) → 6 CO2(g) + 6 H2O(l), calorimetric meas-
only the part of U that is not stored in that way, the quantity urements give ΔrU = −2808 kJ mol−1 and ΔrS = +182.4 J K−1 mol−1
U − TS, is available for conversion into work. at 25 °C and 1 bar. How much of this change in internal energy
If the change occurs with an increase of entropy of the sys- can be extracted as (a) heat at constant pressure, (b) work?
tem (in which case TΔS > 0), ΔU − TΔS is more negative than Collect your thoughts You know that the heat released at
ΔU. In this case, the maximum work that can be obtained from constant pressure is equal to the value of ΔH, so you need to
the system is greater than |ΔU|. The explanation of this appar- relate ΔrH to the given value of ΔrU. To do so, suppose that all
ent paradox is that the system is not isolated and energy may the gases involved are perfect, and use eqn 2B.4 (ΔH = ΔU +
flow in as heat as work is done. Because the entropy of the sys- ΔngRT) in the form ΔrH = ΔrU + ΔngRT. For the maximum
tem increases, it is possible for the entropy of the surroundings work available from the process use wmax = ΔA in the form
to decrease but still for the overall entropy to increase, as re- wmax = ΔrA.
quired for a spontaneous process. Therefore, some energy (no The solution (a) Because Δng = 0, ΔrH = ΔrU = −2808 kJ mol−1.
more than the value of TΔS) may leave the surroundings as heat Therefore, at constant pressure, the energy available as heat is
and contribute to the work the change is generating (Fig. 3D.2). 2808 kJ mol−1. (b) Because T = 298 K, the value of Δr A is
Nature is now providing a tax refund.
 r A   rU  T  r S  2862 kJmol 1
Therefore, the complete oxidation of 1.000 mol C6H12O6 at
constant temperature can be used to produce up to 2862 kJ
of work.
|q|
U < 0 Comment. The maximum work available is greater than the
S < 0
change in internal energy on account of the positive entropy
|w| <U| of reaction (which is partly due to there being a significant
increase in the number of molecules as the reaction proceeds).
The system can therefore draw in energy from the surround-
Ssur > 0
ings (so reducing their entropy) and make it available for
doing work.
Figure 3D.1 In a system not isolated from its surroundings, the
work done may be different from the change in internal energy. In Self-test 3D.1 Repeat the calculation for the combustion of
the process depicted here, the entropy of the system decreases, so 1.000 mol CH4(g) under the same conditions, using data from
for the process to be spontaneous the entropy of the surroundings Table 2C.3 and ΔrS = −243 J K−1 mol−1 at 298 K.
must increase, so energy must pass from the system to the Answer: |qp| = 890 kJ, |wmax| = 813 kJ
surroundings as heat. Therefore, less work than ΔU can be obtained.
3D Concentrating on the system 99

3D.1(d) Some remarks on the Gibbs energy How is that done? 3D.2 Relating the change in Gibbs

The Gibbs energy (the ‘free energy’) is more common in energy to maximum non-expansion work
chemistry than the Helmholtz energy because, at least in Because H = U + pV and dU = dq + dw, the change in enthalpy
laboratory chemistry, changes occurring at constant pres- for a general change in conditions is
sure are more common than at constant volume. The crite-
rion dGT,p ≤ 0 carries over into chemistry as the observation dH  dU  d( pV )  dq  dw  d( pV )
that, at constant temperature and pressure, chemical reactions The corresponding change in Gibbs energy (G = H − TS) is
are spontaneous in the direction of decreasing Gibbs energy.
Therefore, to decide whether a reaction is spontaneous, the dG  dH  TdS  SdT  dq  dw  d( pV )  TdS  SdT
pressure and temperature being constant, it is necessary to
assess the change in the Gibbs energy. If G decreases as the Step 1 Confine the discussion to constant temperature
reaction proceeds, then the reaction has a spontaneous ten- When the change is isothermal dT = 0; then
dency to convert the reactants into products. If G increases,
dG  dq  dw  d( pV )  TdS
the reverse reaction is spontaneous. The criterion for equi-
librium, when neither the forward nor reverse process is Step 2 Confine the change to a reversible process
spontaneous, under conditions of constant temperature and If the change is reversible write the work as dwrev and the heat
pressure, is dGT,p = 0. as dqrev. From the definition of the entropy change, dS = dqrev/T,
The existence of spontaneous endenthalpic processes, in- it follows that dqrev = TdS, and so for a reversible, isothermal
cluding endenthalpic reactions (more colloquially, but less process
precisely, ‘endothermic’ reactions) provides an illustration of dqrev

the role of G, the analogue of A but with H in place of U. In dG  TdS  dwrev  d( pV )  TdS  dwrev  d( pV )
such reactions, H increases, the system rises spontaneously to
states of higher enthalpy, and dH > 0. Because the reaction is Step 3 Divide the work into different types
spontaneous, dG < 0 despite dH > 0; it follows that the entropy
The work consists of expansion work and possibly some other
of the system increases so much that TdS outweighs dH in
kind of work (for instance, the electrical work of pushing
dG = dH − TdS. Endenthalpic reactions are therefore driven by electrons through a circuit or of raising a column of liquid);
the increase of entropy of the system, and this entropy change this additional reversible work is denoted dwadd,rev . The work of
overcomes the reduction of entropy brought about in the sur- expansion is − pext dV , but for the expansion to be reversible to
roundings by the inflow of heat into the system (dSsur = −dH/T external pressure pext must be equal to the pressure of the gas, p.
at constant pressure). Exenthalpic reactions (colloquially Overall the reversible work is therefore dwrev   pdV  dwadd,rev .
‘exothermic reactions’) are commonly spontaneous because Finally, the change in pV, d(pV), is expanded as d(pV) = pdV +
dH < 0 and then dG < 0 provided TdS is not so negative that it Vdp, to give
outweighs the decrease in enthalpy.
The molar Gibbs energy is so important in chemistry that  dwrev
   d ( pV )

it is given a special name, the chemical potential and defined dG  ( pdV  dwadd,rev )  pdV  Vdp  dwadd,rev  Vdp
for a pure substance as μ = G/n. The name reflects its role in
assessing the potential of a substance to bring about chemi- Step 4 Confine the process to constant pressure
cal change. Most of chemistry concerns mixtures, and the If the change occurs at constant pressure (as well as constant
definition of chemical potential then needs to be adapted to temperature), dp = 0 and hence dG = dwadd,rev. Therefore, at
apply to systems of variable composition (Topic 5A), but the constant temperature and pressure, dwadd,rev = dG. However,
sense of μ being a measure of potential to bring about change because the process is reversible, the work done must now have
­remains. its maximum value, so it follows that

(3D.8)
3D.1(e) Maximum non-expansion work dwadd,max = dG
Maximum non-expansion work
[constant T, p]
The analogue of the maximum work interpretation of ΔA, and
the origin of the name ‘free energy’, can be found for ΔG. By an
argument like that relating the Helmholtz energy to maximum For a measurable change, the corresponding expression is
work, it can be shown that, at constant temperature and pres- wadd,max = ΔG. This relation is used for assessing the maximum
sure, the change in Gibbs energy is equal to the maximum ad- electrical work that can be produced by fuel cells and electro-
ditional (non-expansion) work. chemical cells (Topic 6C).
100 3 The Second and Third Laws

Therefore, wadd,max = −2865 kJ for the oxidation of 1.00 mol


Exercise glucose molecules, and the reaction can be used to do up to
E3D.1 Given the data below (all at 298 K), calculate the maximum non-expansion 2865 kJ of non-expansion work.
work per mole of CH4 that may be obtained from a fuel cell in which the chemical
reaction is the combustion of methane under standard conditions at 298 K. Comment. To place this result in perspective, consider that a
person of mass 70 kg needs to do 2.1 kJ of work to climb verti-
ΔfG⦵/kJ mol-1 cally through 3.0 m on the surface of the Earth; therefore, at
CO2(g) −394.36 least 0.13 g of glucose is needed to complete the task (and in
H2O(l) −237.13 practice significantly more).
CH4(g) −50.72 Self-test 3D.2 How much non-expansion work can be obtained
from the combustion of 1.00 mol CH4(g) under standard con-
ditions at 298 K? Use ∆ r S  = −243 J K−1 mol−1.

3D.2 Standard molar Gibbs energies Answer: 818 kJ

Standard enthalpies and entropies of reaction (which are in-


troduced in Topics 2C and 3C) can be combined to obtain
the standard Gibbs energy of reaction (or ‘standard reaction
3D.2(a) Gibbs energies of formation
Gibbs energy’), ΔrG :

As in the case of standard reaction enthalpies (Topic 2C), it is
convenient to define the standard Gibbs energies of forma-
Standard Gibbs energy tion, ∆ f G , the standard reaction Gibbs energy for the forma-

r G  r H   T r S  (3D.9)
⦵ ⦵ ⦵

of reaction [definitio
on]
tion of a compound from its elements in their reference states,
The standard Gibbs energy of reaction is the difference in as specified in Topic 2C. Standard Gibbs energies of formation
standard molar Gibbs energies of the products and reactants in of the elements in their reference states are zero, because their
their standard states for the reaction as written and at the speci- formation is a ‘null’ reaction. A selection of values for com-
fied temperature. pounds is given in Table 3D.1. The standard Gibbs energy of a
Calorimetry (for ΔH directly, and for S from heat capacities) reaction is then found by taking the appropriate combination:
is only one of the ways of determining Gibbs energies. They
may also be obtained from equilibrium constants (Topic 6A) Standard Gibbs
and electrochemical measurements (Topic 6D); for gases they energy off
r G 
  G 
   G 
⦵ ⦵ ⦵
f f reaction [practical  (3D.10a)
may be calculated from spectroscopic data (Topic 13E). Products Reactants
implementation]

In the notation introduced in Topic 2C,


Example 3D.2 Calculating the maximum non-expansion
work of a reaction  r G    J  f G  (J)
⦵ ⦵
(3D.10b)
J
How much energy is available for sustaining muscular and
nervous activity from the oxidation of 1.00 mol of glucose where the nJ are the (signed) stoichiometric numbers in the
molecules under standard conditions at 37 °C (blood tempera- chemical equation.
ture)? The standard entropy of reaction is +182.4 J K−1 mol−1.
Collect your thoughts The non-expansion work available from
the reaction at constant temperature and pressure is equal to
Table 3D.1 Standard Gibbs energies of formation at 298 K*
the change in standard Gibbs energy for the reaction, ∆ rG  .

To calculate this quantity, you can (at least approximately)


 f G⦵ /(kJ mol 1 )
ignore the temperature dependence of the reaction enthalpy,
Diamond, C(s) +2.9
and obtain ∆ r H  from Table 2C.4 (where the data are for 25 °C,

not 37 °C, but there is little difference), and substitute the data Benzene, C6H6(l) +124.3
into  rG    r H   T  r S . Methane, CH4(g) −50.7
⦵ ⦵ ⦵

Carbon dioxide, CO2(g) −394.4


The solution Because the standard reaction enthalpy is
−1
−2808 kJ mol , it follows that the standard reaction Gibbs Water, H2O(l) −237.1
energy is Ammonia, NH3(g) −16.5
1 1
 rG  2808 kJ mol  (310 K )  (182.4 J K mol )

 1 Sodium chloride, NaCl(s) −384.1

 2865 kJ mol 1 * More values are given in the Resource section.


3D Concentrating on the system 101

Brief illustration 3D.1 H+(g) + Cl(g) + e–


+106
–349
To calculate the standard Gibbs energy of the reaction H+(g) + ½ Cl2(g) + e– H+(g) + Cl–(g)
CO(g )  12 O2 (g )  CO2 (g ), write
ΔsolvG°(Cl–)

Gibbs energy, G

⦵ 


 rG   f G (CO2 , g )   f G (CO, g )   f G (O2 , g )

⦵ 1
2


 +1312
H+(g) + Cl–(aq)
 394.4 kJmol  (137.2)  (0) kJmol
1 1
2
1

 257.2 kJmol 1
H(g) + ½ Cl2(g)
ΔsolvG°(H+)
+218
½ H2(g) + ½ Cl2(g)
As explained in Topic 2C, the standard enthalpy of formation
of H+ in water is by convention taken to be zero; in Topic 3C, H+(aq) + Cl–(aq)
the absolute entropy of H+(aq) is also by convention set equal –{ΔfG °(H+,aq) +ΔfG °(Cl–,aq)}
to zero (at all temperatures in both cases). These conventions
are needed because it is not possible to prepare cations without Figure 3D.3 A thermodynamic cycle for discussion of the Gibbs
their accompanying anions. For the same reason, the standard energies of hydration and formation of chloride ions in aqueous
Gibbs energy of formation of H+(aq) is set equal to zero at all solution. The changes in Gibbs energies around the cycle sum to
zero because G is a state function.
temperatures:

Ions in solution
 f G  (H , aq )  0  (3D.11)

[convention]
 f G (Cl , aq )  1287 kJmol 1   solv G (H  )   solv G (Cl  )

 ⦵
 ⦵

This definition effectively adjusts the actual values of the Gibbs


The standard Gibbs energies of formation of the gas-phase
energies of formation of ions by a fixed amount, which is cho-
ions are unknown and have been replaced by ionization en-
sen so that the standard value for one of them, H+(aq), has the
ergies and electron affinities (Topic 8C). It has therefore been
value zero.
assumed that any differences from the Gibbs energies arising
from conversion to enthalpy and the inclusion of entropies to
Brief illustration 3D.2 obtain Gibbs energies in the formation of H+ are cancelled by
the corresponding terms in the electron gain of Cl. The con-
For the reaction clusions from the cycles are therefore only approximate. An
important point to note is that the value of ∆ f G  of Cl− is not

1
2
H2 (g )  12 Cl 2 (g )  H  (aq )  Cl  (aq )
 rG   131.23 kJmol 1
⦵ determined by the properties of Cl alone but includes contribu-
tions from the dissociation and ionization of hydrogen and the
the value of ∆ rG⦵ can be written in terms of standard Gibbs hydration of hydrogen ions.
energies of formation as

 rG    f G  (H , aq )   f G  (Cl , aq ) The Born equation


⦵ ⦵ ⦵
3D.2(b)
where the ∆ f G of the elements on the left of the chemical


Gibbs energies of solvation of individual ions may be estimated

equation are zero. Because by convention  f G  (H , aq )  0, on the basis of a model in which solvation is expressed as an
⦵ ⦵ 
it follows that  rG   f G (Cl , aq ) and therefore that
 
electrostatic property. The model treats the interaction between
 f G  (Cl , aq )  131.23 kJmol 1. the ion and the solvent in terms of elementary electrostatics:

the ion is regarded as a charged sphere and the solvent is treated


as a continuous medium (a continuous dielectric). The key step
The factors responsible for the Gibbs energy of formation of is then to identify the Gibbs energy of solvation with the work
an ion in solution can be identified by analysing its formation of transferring an ion from a vacuum into the solvent.
in terms of a thermodynamic cycle. As an illustration, con- That work is calculated by taking the difference of the work of
sider the standard Gibbs energy of formation of Cl− in water. charging an ion when it is in the solution and the work of charg-
The formation reaction 12 H2 (g )  12 C12 (g )  H  (aq )  Cl  (aq ) ing the same ion when it is in a vacuum. Electrostatic theory
is treated as the outcome of the sequence of steps shown in gives the work of charging a sphere of radius r immersed in a
Fig. 3D.3 (with values taken from the Resource section). The medium of permittivity ε to a final charge ze as w = z2e2/8πεr (the
sum of the Gibbs energies for all the steps around a closed derivation of this expression is given in A deeper look 3D.1, avail-
cycle is zero, so able to read in the e-book accompanying this text). Therefore, the
102 3 The Second and Third Laws

work of transferring it from a vacuum (where ε = ε0, the electric


constant, or ‘vacuum permittivity’) to a medium where ε = εrε0 Exercises
(with εr the relative permittivity, or ‘dielectric constant’) is E3D.2 Given the data below (all at 298 K), calculate the standard Gibbs energy
of reaction for 4 HI(g) + O2(g) → 2 I2(s) + 2 H2O(l) at 298 K.
z 2e2 z 2e2
w 
8 r  0 r 8 0 r
Sm⦵ /(J K−1 mol−1) Δf H ⦵/(kJ mol−1)
The molar energy as work is obtained by multiplying by NA, HI(g) 206.59 HI(g) +26.48
Avogadro’s constant, and the entire expression equated to the O2(g) 205.138 H2O(l) −285.83
­standard Gibbs energy of solvation. The result is the Born
I2(s) 116.135
equation:
H2O(l) 69.91
2 2
z e NA 1 (3D.12a)
ΔsolvG ⦵ = − 1−
8 π 0r r
Born equation
E3D.3 Calculate the standard Gibbs energy for the reaction 2 CH3CHO(g) +
O2(g) → 2 CH3COOH(l) at 298 K using the data given below (all at 298 K).

Note that  solv G   0, and that ∆ solv G  is strongly negative for


⦵ ⦵

small, highly charged ions in media of high relative permittiv- ΔfG⦵/(kJ mol−1)
ity. For water, for which εr = 78.54 at 25 °C, the Born equation CH3CHO(g) −128.86
becomes
CH3COOH(l) −389.9
z2

 solv G    6.86  104 kJmol 1

(3D.12b)
r /pm
E3D.4 The standard enthalpy of combustion of liquid ethyl ethanoate
(ethyl acetate, CH3COOC2H5) is −2231 kJ mol−1 at 298 K and its standard
Brief illustration 3D.3 molar entropy is 259.4 J K−1 mol−1. Calculate the standard Gibbs energy of
formation of the compound at 298 K using the data given in the tables below
(all at 298 K).
To estimate the difference in the values of ∆ solv G  for Cl− and

I− in water at 25 °C, given their radii as 181 pm and 220 pm,


respectively, write
Sm⦵ /(J K−1 mol−1) Δf H ⦵/(kJ mol−1)
 1 1  O2(g) 205.138 CO2(g) −393.51
 solv G  (Cl  )   solv G  (I )     4
  6.86  10 kJmol
1
⦵ ⦵

 181 220  H2(g) 130.684 H2O(l) −285.83


 67 kJmol 1 C(s) 5.740

Checklist of concepts
☐ 1. The Clausius inequality implies a number of criteria ☐ 5. The change in the Gibbs energy is equal to the ­maximum
for spontaneous change under a variety of conditions non-expansion work obtainable from a system at con-
which may be expressed in terms of the properties of the stant temperature and pressure.
system alone; they are summarized by introducing the ☐ 6. Standard Gibbs energies of formation are used to calcu-
Helmholtz and Gibbs energies. late the standard Gibbs energies of reactions.
☐ 2. A spontaneous process at constant temperature and ☐ 7. The standard Gibbs energies of formation of ions may
volume is accompanied by a decrease in the Helmholtz be estimated from a thermodynamic cycle and the Born
energy. equation.
☐ 3. The change in the Helmholtz energy is equal to the ☐ 8. The chemical potential of a system that consists of a
maximum work obtainable from a system at constant single substance is equal to the molar Gibbs energy of
temperature. that substance.
☐ 4. A spontaneous process at constant temperature and pres-
sure is accompanied by a decrease in the Gibbs energy.
3D Concentrating on the system 103

Checklist of equations
Property Equation Comment Equation number
Criteria of spontaneity dSU,V ≥ 0
dSH,p ≥ 0
Helmholtz energy A = U − TS Definition 3D.4a
Gibbs energy G = H − TS Definition 3D.4b
Criteria of spontaneous change (a) dAT,V ≤ 0 (b) dGT,p ≤ 0 Equality refers to equilibrium 3D.6
Maximum work dwmax = dA, wmax = ΔA Constant temperature 3D.7
Maximum non-expansion work dwadd,max = dG, wadd,max = ΔG Constant temperature and pressure 3D.8
Standard Gibbs energy of reaction: ⦵

rG  r H  T r S ⦵
 ⦵

Definition 3D.9
 r G   J  f G (J)

 ⦵

Practical implementation 3D.10b
J
⦵ 

 solv G (H , aq )  0 Ions in solution 3D.11

Born equation  solv G  (z e N A /8 0r )(1  1/ r )
 2 2
i
(3D.12a)
TOPIC 3E Combining the First and
Second Laws

The fact that the fundamental equation applies to both re-


➤ Why do you need to know this material? versible and irreversible changes may be puzzling at first sight.
A formulation in which the First and Second Laws are com- The reason is that only in the case of a reversible change may
bined greatly adds to the power of thermodynamics and TdS be identified with dq and −pdV with dw. When the change
increases its scope. is irreversible, TdS > dq (the Clausius inequality) and −pdV >
dw. The sum of dw and dq remains equal to the sum of TdS and
➤ What is the key idea? −pdV, provided the composition is constant.
The fact that infinitesimal changes in thermodynamic func-
tions are exact differentials leads to relations between a
variety of properties.
3E.1 Properties of the internal energy
➤ What do you need to know already?
You need to be aware of the definitions of the state func- Equation 3E.1 shows that the internal energy of a closed system
tions U (Topic 2A), H (Topic 2B), S (Topic 3A), and A and G changes in a simple way when either S or V is changed (dU ∝ dS
(Topic 3D). The notation for partial differentials is intro- and dU ∝ dV). These simple proportionalities suggest that U is
duced in The chemist’s toolkit 2A.2 and it is developed in best regarded as a function of S and V. It could be regarded as a
The chemist’s toolkit 3E.1. function of other variables, such as S and p, or T and V, because
they are all interrelated. However the simplicity of the funda-
mental equation suggests that regarding U as a function of S
and V is the best choice.
The mathematical consequence of U being a function of S
The First Law of thermodynamics may be written dU = dq + dw. and V is that an infinitesimal change dU can be expressed in
Consider the special case of a closed system with constant com- terms of changes dS and dV by
position: ‘closed’ means that matter neither enters nor leaves
the system, and ‘constant composition’ means that the amounts  U   U 
of the (chemical) species present are fixed. Now suppose that a
dU    dS    dV  (3E.2)
 S V  V S
reversible change occurs in this system and that the only work
permitted is that due to expansion (no additional work is done). The two partial derivatives (see The chemist’s toolkit 2A.2) are
This work of expansion is − pext dV , but for a reversible expan- the slopes of the plots of U against S at constant V, and U against
sion the external pressure pext must be equal to the pressure V at constant S. When this expression is compared term-by-
of the gas, p; it follows that dwrev = −pdV. From the definition term to the thermodynamic relation, eqn 3E.1, it follows that for
of the entropy change, dS = dqrev /T , it follows that dqrev = TdS systems of constant composition,
and hence the First Law can be written
 U   U 
 S   T  V    p (3E.3)
dU  TdS  pdV The fundamental equation (3E.1)  V  S

However, because dU is an exact differential, its value is inde- The first of these two equations is a purely thermodynamic
pendent of path. Therefore, the same value of dU is obtained definition of temperature as the ratio of the changes in the in-
whether the change is brought about irreversibly or reversibly. ternal energy (a First-Law concept) and entropy (a Second-Law
Consequently, this equation applies to any change—reversible or concept) of a constant-volume, closed, constant-composition
irreversible—of a closed system that does no non-expansion (ad- system. This is the first example of how the combination of the
ditional) work. This combination of the First and Second Laws First and Second Laws leads to perhaps surprising relations be-
is called the fundamental equation. tween the properties of a system.
3E Combining the First and Second Laws 105

3E.1(a) The Maxwell relations Table 3E.1 The Maxwell relations

An infinitesimal change in a function f(x,y) can be written State function Exact differential Maxwell relation
df = gdx + hdy where g and h may be functions of x and y. The  T   p 
U dU = TdS − pdV  V     S 
mathematical criterion for df being an exact differential (in the  S  V
sense that its integral is independent of path) is that  T   V 
H dH = TdS + Vdp    
 g   h   p S  S  p
     (3E.4)  p   S 
 y  x  x  y A dA = −pdV − SdT  T    V 
 V  T
This criterion is derived in The chemist’s toolkit 3E.1. Because  V   S 
the fundamental equation, eqn 3E.1, is an expression for an G dG = Vdp − SdT  T     p 
 p  T
exact differential (the internal energy), the functions multiply-
ing dS and dV (namely T and −p) must satisfy the criterion of
eqn 3E.4. Therefore, it must be the case that
Example 3E.1 Using the Maxwell relations
 T   p 
 V    S  A Maxwell relation (3E.5)
 S  V Use the Maxwell relations in Table 3E.1 to show that, at
constant temperature, the entropy of a perfect gas is linearly
A relation has been generated between quantities which, at first dependent on ln V, that is, S = a + b ln V.
sight, would not seem to be related.
Equation 3E.5 is an example of a Maxwell relation. However, Collect your thoughts The natural place to start, given that you
apart from being unexpected, it does not look particularly in- are invited to use the Maxwell relations, is to consider the rela-
tion for (∂S/∂V)T , as that differential coefficient shows how the
teresting. Nevertheless, it does suggest that there might be
entropy varies with volume at constant temperature. Be alert
other similar relations that are more useful. Indeed, the fact
for an opportunity to use the perfect gas equation of state.
that H, G, and A are all state functions can be used to derive
three more Maxwell relations. The argument to obtain them The solution From Table 3E.1,
runs in the same way in each case: because H, G, and A are state
functions, the expressions for dH, dG, and dA satisfy relations  S   p 
 V    T 
like eqn 3E.4. All four relations are listed in Table 3E.1.  T  V

The chemist’s toolkit 3E.1 Exact differentials


Suppose that df can be expressed in the following way: Taking the partial derivative with respect to x of the first equa-
tion, and with respect to y of the second gives
df  g (x ,y )dx  h(x ,y )dy
   f    g (x ,y ) 
Is df is an exact differential? If it is exact, then it can be expressed     
 y  x  y   y 
in the form  x x

 f     f    h(x ,y ) 
 f       
d f    d x    dy 
 x  y  y  x  x  y  x  y  x  y
Comparing these two expressions gives By the property of partial derivatives these two successive
derivatives of f with respect to x and y must be the same, hence
 f   f 
 x   g (x ,y )    h(x ,y )
 g (x ,y )   h(x ,y ) 
 y  y  x
   
It is a property of partial derivatives that successive derivatives  y  x  x  y
may be taken in any order: If this equality is satisfied, then df = g(x,y)dx + h(x,y)dy is an
   f      f   exact differential. Conversely, if it is known from other argu-
        ments that df is exact, then this relation between the partial
 y  x  y   x  y  
 x  x y derivatives follows.
106 3 The Second and Third Laws

Now use the perfect gas equation of state, pV = nRT, to write p (3E.6a)
p = nRT/V: πT =T −p
T V
A thermodynamic equation of state

 p   (nRT /V )  nR
 T      V Equation 3E.6a is called a thermodynamic equation of state
 V  T V because, when written in the form
At this point, write
 p 
p T   T  (3E.6b)
 S  nR  T V
 V   V
 T it is an expression for pressure in terms of a variety of thermo-
and therefore, at constant temperature, dynamic properties of the system.

dV
 dS  nR  V  nR ln V  constant
Example 3E.2 Deriving a thermodynamic relation
The integral on the left is S + constant, which completes the
Show thermodynamically that πT = 0 for a perfect gas, and
demonstration.
compute its value for a van der Waals gas.
Self-test 3E.1 How does the entropy depend on the volume of
Collect your thoughts Proving a result ‘thermodynamically’
a van der Waals gas? Suggest a rationalization of your answer.
means basing it entirely on general thermodynamic relations
and equations of state, without drawing on molecular argu-
Answer: S varies as nR ln(V − nb); molecules in a smaller available volume.

ments (such as the existence of intermolecular forces). You


know that for a perfect gas, p = nRT/V, so this relation should
be used in eqn 3E.6a. Similarly, the van der Waals equation is
The variation of internal energy
3E.1(b)
given in Table 1C.4, and for the second part of the question it
with volume should be used in eqn 3E.6a.
The internal pressure, πT (introduced in Topic 2D), is defined as The solution For a perfect gas write
πT = (∂U/∂V)T and represents how the internal energy changes
as the volume of a system is changed isothermally; it plays a  p   nRT /V  nR
central role in the manipulation of the First Law. By using a  T    T   V
 V  V
Maxwell relation, πT can be expressed as a function of pressure
Then, eqn 3E.6a becomes
and temperature.
(p(∂p/∂T)
/T )V  nR/V
= nR/V
v

 p
 
How is that done? 3E.1 Deriving a thermodynamic p
   nRT 
T  T    p V  p0
equation of state  T V  
To construct the partial differential (∂U/∂V)T you need to start The equation of state of a van der Waals gas is
from eqn 3E.2, divide both sides by dV, to obtain
nRT n2
p a 2
dU  U  dS  U  V  nb V
 
dV  S V dV  V S Because a and b are independent of temperature,

and then impose the constraint of constant temperature:  p   nRT /(V  nb)  nR
 T    T   V  nb
 V  V
T
   T  p

 U   U   S   U  Therefore, from eqn 3E.6a,


 V    S   V    V 
 T  V  T  S p
 
Next, introduce the two relations in eqn 3E.3 (as indicated by nRT nRT  nRT n2  n2 a
T  p  a 2  a 2  2
the annotations) and the definition of πT to obtain V  nb V  nb  V  nb V  V Vm

 S  Comment. This result for πT implies that the internal energy


T  T   p of a van der Waals gas increases when it expands isother-
 V T
mally, that is, (∂U/∂V)T > 0. The expression also shows that the
The third Maxwell relation in Table 3E.1 turns (∂S/∂V)T into increase is related to the parameter a, which models the attrac-
(∂p/∂T)V , to give tive interactions between the particles.
3E Combining the First and Second Laws 107

Self-test 3E.2 Calculate πT for a gas that obeys the virial equa-
Gibbs
tion of state (Table 1C.4), retaining only the term in B. energy, G
Answer:  T  RT 2 (B /T )V /Vm2 (a)
Slope = –S

(b)
Slope = +V
3E.2 Properties of the Gibbs energy
,T
re Pres
The same arguments that were used for U can also be used for rat
u sure,
p
the Gibbs energy, G = H − TS. They lead to expressions showing pe
m
Te
how G varies with pressure and temperature and which are im-
portant for discussing phase transitions and chemical reactions.
Figure 3E.1 The variation of the Gibbs energy of a system with
3E.2(a) General considerations (a) temperature at constant pressure and (b) pressure at constant
temperature. The slope of the former is equal to the negative of
When the system undergoes a change of state, G may change the entropy of the system and that of the latter is equal to the
because H, T, and S all change: volume.

dG  dH  d(TS)  dH  TdS  SdT


 Gm   Gm 
Because H = U + pV,    Sm    Vm  (3E.8b)
 T  p  p T
dH  dU  d( pV )  dU  pdV  Vdp
The first implies that:
and therefore
• Because S > 0 for all substances, G always decreases when
dG  dU  pdV  Vdp  TdS  SdT the temperature is raised (at constant pressure and com-
For a closed system doing no non-expansion work, dU can be position).
replaced by the fundamental equation dU = TdS − pdV to give • Because (∂G/∂T)p becomes more negative as S increases, G
decreases more sharply with increasing temperature as the
dG  TdS  pdV  pdV  Vdp  TdS  SdT
entropy of the system increases.
Four terms now cancel on the right, and so for a closed system
The molar entropy of a gas is greater than that of the corre-
doing no non-expansion work and at constant composition
sponding liquid or solid phases. It therefore follows that the
The fundamental equation of Gibbs energy of the gas is more sensitive to temperature than is
dG  Vdp  SdT  (3E.7)
chemical thermodynamiics the case for a liquid or solid (Fig. 3E.2).
This expression, which shows that a change in G is proportional
to a change in p or T, suggests that G may be best regarded as Gas
a function of p and T. It may be regarded as the ­fundamental
equation of chemical thermodynamics as it is so central to
Gibbs energy, G

the application of thermodynamics to chemistry. It also sug-


gests that G is an important quantity in chemistry because the
pressure and temperature are usually the variables that can be Liquid
controlled. In other words, G carries around the combined con-
sequences of the First and Second Laws in a way that makes it Solid

particularly suitable for chemical applications.


The same argument that led to eqn 3E.3, when applied to the
exact differential dG = Vdp − SdT, now gives Temperature, T

 G   G  The variation of Figure 3E.2 The variation of the Gibbs energy with the
 T   S  p   V G with T and p
 (3E.8a)
temperature is determined by the entropy. Because the entropy of
 p  T
the gaseous phase of a substance is greater than that of the liquid
and shows how the Gibbs energy varies with temperature and phase, and the entropy of the solid phase is smallest, the Gibbs
pressure (Fig. 3E.1). Alternatively, these two relations can be energy changes most steeply for the gas phase, followed by the
expressed in terms of the molar quantities liquid phase, and then the solid phase of the substance.
108 3 The Second and Third Laws

Similarly, the second relation implies that: The variation of the Gibbs energy
3E.2(b)
• Because V > 0 for all substances, G always increases when
with temperature
the pressure of the system is increased (at constant tem- The first relation in eqn 3E.8a, (∂G/∂T)p = −S, is the starting
perature and composition). point for the discussion of how the Gibbs energy varies with
• Because (∂G/∂p)T increases with V, G becomes more sen- temperature. Although it expresses the variation of G in terms
sitive to pressure as the volume of the system is increased. of the entropy, it can be expressed in terms of the enthalpy by
using the definition G = H − TS, to write S = (H − G)/T. Then
Because the molar volume of the gaseous phase of a substance
 G  G  H
is greater than that of its condensed phases, the molar Gibbs  T   T  (3E.9)
energy of a gas is more sensitive to pressure than its liquid and  p
solid phases (Fig. 3E.3). In Topic 6A it is shown that the equilibrium constant of a reac-
tion is related to G/T rather than to G itself. With this appli-
cation in mind, eqn 3E.9 can be developed to show how G/T
Brief illustration 3E.1 varies with temperature.
The mass density of liquid water is 0.9970 g cm−3 at 298 K. It fol-
lows that when the pressure is increased by 0.1 bar (at constant
How is that done? 3E.2 Deriving an expression for the
temperature), the molar Gibbs energy changes by
temperature variation of G/T
Gm
ΔGm Δp = Vm Δp First, note that
p T
Vm d( fg)/dx =f(dg/dx) + g(df/dx)
−1
18.0 g mol
= × (0.1 × 05 Nm −2 ) G / T 1 G d (1/ T ) 1 G G
0.9970 ×106 g m −3 = +G = −
T p
T T p dT T T p
T2
= + 0.18 Jmol −1 −1/T 2
1 G G
In this calculation an infinitesimal change in pressure is = −
T T p T
approximated by the measureable change ∆p and the molar
volume is computed from the mass density ρ using the molar
mass M, with Vm = M/ρ. Now replace the term (∂G/∂T)p on the right by eqn 3E.9

(G/T )p = (G−H)/T

Gas
G / T
T p
=
1
T
G
T p

G
T
=
T T
−{
1 G−H G
T
=
1 −H
T T } { }
Gibbs energy, G

from which follows the Gibbs–Helmholtz equation

G / T H (3E.10)
=−
T p
T2 Gibbs–Helmholtz equation
Liquid

Solid
The Gibbs–Helmholtz equation is most useful when it is ap-
Pressure, p
plied to changes, including changes of physical state, and chem-
Figure 3E.3 The variation of the Gibbs energy with the pressure ical reactions at constant pressure. For a change between initial
is determined by the volume of the sample. Because the volume and final states, the change in Gibbs energy is written ΔG = Gf − Gi,
of the gaseous phase of a substance is greater than that of the and likewise the change in the enthalpy is ΔH = Hf − Hi. Because
same amount of liquid phase, and the volume of the solid phase eqn 3E.10 applies to both Gf and Gi it follows that
is smallest (for most substances), the Gibbs energy changes most
steeply for the gas phase, followed by the liquid phase, and then  G /T  H
the solid phase of the substance. Because the molar volumes  T    2  (3E.11)
 p T
of the solid and liquid phases of a substance are similar, their
molar Gibbs energies vary by similar amounts as the pressure is This equation shows that if the change in enthalpy of a sys-
changed. tem that is undergoing some kind of transformation (such as
3E Combining the First and Second Laws 109

v­ aporization or reaction) is known, then how the corresponding For a difference in Gibbs energy between two states
change in Gibbs energy varies with temperature is also known.
This turns out to be a crucial piece of information in chemistry. Gm ( pf )  Gm ( pi )  ( pf  pi )Vm  (3E.13b)

The contribution of the term (pf − pi)Vm is illustrated graphi-


The variation of the Gibbs energy of
3E.2(c) cally in Fig. 3E.4. For condensed phases under normal labora-
tory conditions (pf − pi)Vm is very small and may be neglected.
condensed phases with pressure
Hence, the Gibbs energies of solids and liquids are largely in-
To find the Gibbs energy at one pressure in terms of its value at dependent of pressure. However, in geophysical applications,
another pressure, the temperature being constant, set dT = 0 in because pressures in the Earth’s interior are huge, their effect
eqn 3E.7, which leaves dG = Vdp, and integrate: on the Gibbs energy cannot be ignored. If the pressures are so
pf
great that there are substantial volume changes over the range
G( pf )  G( pi )   Vdp (3E.12a) of integration, then the complete expression, eqn 3E.12, must
pi
be used.
For molar quantities,
pf
Gm ( pf )  Gm ( pi )   Vm dp (3E.12b) Brief illustration 3E.2
pi

This expression is applicable to any phase of matter, but it is For a certain phase transition of a solid ΔtrsV = +1.0 cm3 mol−1
independent of pressure. When the pressure is increased from
necessary to know how the molar volume, Vm, depends on the
1.0 bar (1.0 × 105 Pa) to 3.0 Mbar (3.0 × 1011 Pa) the Gibbs
pressure before the integral can be evaluated. This relation also
energy of transition changes according to
applies to differences between initial and final states of a sys-
tem, such as condensation or vaporization:  trsG( pf )   trsG( pi )  ( pf  pi ) trsV
pf   trsG(1 bar )  (3.0  1011 Pa  1.0  105 Pa )
Gm ( pf )  Gm ( pi )   Vm dp (3E.12c)
pi  (l.0  106 m3 mol 1 )
The molar volume of a condensed phase (a solid or liquid)   trsG(1 bar )  3.0  102 kJmol 1
changes only slightly as the pressure changes, so in this case Vm
can be treated as constant and taken outside the integral:

The variation of the Gibbs energy of


pf
Gm ( pf )  Gm ( pi )  Vm  dp 3E.2(d)
pi
gases with pressure
Integration then gives
The molar volumes of gases are large, so the Gibbs energy of
Molar Gibbs energy a gas depends strongly on the pressure. Furthermore, because
Gm ( pf )  Gm ( pi )  ( pf  pi )Vm [incompressible  (3E.13a) the volume also varies markedly with the pressure, the volume
substance]
cannot be treated as a constant in the integral in eqn 3E.12b
(Fig. 3E.5).

Volume assumed Actual volume


constant
V = nRT/p
Volume, V

Volume, V

p
V dp
pi pf
Pressure, p
pi Pressure, p pf
Figure 3E.4 At constant temperature, the difference in Gibbs
energy of a solid or liquid between two pressures is equal to the Figure 3E.5 At constant temperature, the change in Gibbs energy
rectangular area shown. The variation of volume with pressure has for a perfect gas between two pressures is equal to the area
been assumed to be negligible. shown below the perfect-gas isotherm.
110 3 The Second and Third Laws

For a perfect gas, substitute Vm = RT/p into the integral, note


that T is constant, and find

Molar Gibbs energy, Gm


RT /p
pf
 Gm°
Gm ( pf )  Gm ( pi )   Vm dp
pi

Integral A.2  (3E.14)



  
pf 1 p
 Gm ( pi )  RT  dp  Gm ( pi )  RT ln f
pi p pi
It is usual to rewrite this equation by taking pi as the standard p°
pressure p (1 bar) in which case G = m ( pi ) G= 
m(p ) Gm, where
⦵ ⦵ ⦵
Pressure, p

Gm is the standard molar Gibbs energy. If the final pressure is

written as p it therefore follows that Figure 3E.6 At constant temperature, the molar Gibbs energy of
a perfect gas varies as ln p. At the standard pressure, p⦵, the molar
p Molar Gibbs energy Gibbs energy has its standard value. Note that, as p → 0, the molar
Gm ( p)  Gm  RT ln  (3E.15a)

p
⦵ [perfect gas, constant T ] Gibbs energy becomes negatively infinite.

This expression relates the molar Gibbs energy at pressure p to Compare this result with that obtained in Brief illustration 3E.1
the standard molar Gibbs energy. The chemical potential, μ, is where for a condensed phase the same increase in pressure
introduced in Topic 3D as another name for the molar Gibbs resulted in a considerably smaller change in the molar Gibbs
energy of a pure substance. Expressed in terms of μ, this rela- energy.
tion becomes

p Chemical potential The logarithmic dependence of the molar Gibbs energy on


 ( p)  ⦵  RT ln [perfect gas, constant T ]  (3E.15b)
p⦵ the pressure predicted by eqn 3E.15 is illustrated in Fig. 3E.6.
This very important expression applies to perfect gases (which
That the chemical potential, the potential of a substance to take is usually a good approximation).
part in physical or chemical change, increases with pressure, is
consistent with its name.
Exercises
Brief illustration 3E.3 E3E.1 The change in the Gibbs energy of a certain constant-pressure process
is found to fit the expression ΔG/J = −85.40 + 36.5(T/K). Calculate the value
of ΔS for the process, and use the Gibbs–Helmholtz equation to calculate the
When the pressure is increased isothermally on a perfect gas
value of ΔH for the process.
from 1 bar (the standard pressure) to 1.1 bar at 298 K, then
3
according to eqn 3E.15a E3E.2 Estimate the change in the Gibbs energy of 1.0 dm of liquid octane when
the pressure acting on it is increased from 1.0 atm to 100 atm. Given that the mass
Gm (1.1 bar )  Gm  (8.3145 JK 1 mol 1 )

density of octane is 0.703 g cm−3, determine the change in the molar Gibbs energy.
3
 1.1 bar  E3E.3 Suppose that 2.5 mmol of perfect gas molecules initially occupies 42 cm at
 (298 K )  ln  300 K and then expands isothermally to 600 cm3. Calculate ΔG for the process.
 1 bar 
  E3E.4 Calculate the change in the molar Gibbs energy of a perfect gas when its
 Gm  240 Jmol 1

pressure is increased isothermally from 1.0 atm to 100.0 atm at 298 K.

Checklist of concepts
☐ 1. The fundamental equation, a combination of the First ☐ 3. The Maxwell relations are a series of relations between
and Second Laws, is an expression for the change in partial derivatives of thermodynamic properties based
internal energy that accompanies changes in the volume on criteria for changes in the properties being exact dif-
and entropy of a system. ferentials.
☐ 2. Relations between thermodynamic properties are gen- ☐ 4. The Maxwell relations are used to derive the thermo-
erated by combining thermodynamic and mathematical dynamic equation of state and to determine how the
expressions for changes in their values. internal energy of a substance varies with volume.
3E Combining the First and Second Laws 111

☐ 5. The variation of the Gibbs energy of a system suggests ☐ 7. The variation of Gibbs energy with temperature is relat-
that it is best regarded as a function of pressure and ed to the enthalpy by the Gibbs–Helmholtz equation.
temperature. ☐ 8. The Gibbs energies of solids and liquids are almost inde-
☐ 6. The Gibbs energy of a substance decreases with tem- pendent of pressure; those of gases vary linearly with the
perature and increases with pressure. logarithm of the pressure.

Checklist of equations
Property Equation Comment Equation number
Fundamental equation dU = TdS − pdV No additional work 3E.1
Fundamental equation of chemical thermodynamics dG = Vdp − SdT No additional work 3E.7
Variation of G (∂G/∂p)T = V and (∂G/∂T)p = −S Constant composition 3E.8
Gibbs–Helmholtz equation (∂(G/T)/∂T)p = −H/T2 Constant composition 3E.10
Pressure dependence of Gm Gm(pf ) = Gm(pi) + Vm(pf − pi) Incompressible substance 3E.13
Gm(pf ) = Gm(pi) + RT ln(pf/pi) Perfect gas, isothermal process 3E.14 3E.15
⦵ ⦵
 
Gm ( p)  Gm  RT ln( p/p )
112 3 The Second and Third Laws

FOCUS 3 The Second and Third Laws

To test your understanding of this material, work through Selected solutions can be found at the end of this Focus in
the Exercises, Additional exercises, Discussion questions, and the e-book. Solutions to even-numbered questions are available
Problems found throughout this Focus. online only to lecturers.
Assume that all gases are perfect and that data refer to 298.15 K
unless otherwise stated.

TOPIC 3A Entropy
Discussion questions
D3A.1 The evolution of life requires the organization of a very large number D3A.2 Discuss the significance of the terms ‘dispersal’ and ‘disorder’ in the
of molecules into biological cells. Does the formation of living organisms context of the Second Law.
violate the Second Law of thermodynamics? State your conclusion clearly and
D3A.3 Discuss the relationships between the various formulations of the
present detailed arguments to support it.
Second Law of thermodynamics.

Additional exercises
E3A.6 Consider a process in which the entropy of a system increases by of argon gas of mass 2.9 g at 298 K increases from 1.20 dm3 to 4.60 dm3 in
−1 −1
105 J K and the entropy of the surroundings decreases by 95 J K . Is the (i) an isothermal reversible expansion, (ii) an isothermal irreversible
process spontaneous? expansion against pex = 0, and (iii) an adiabatic reversible expansion.
E3A.7 Consider a process in which 250 kJ of energy is transferred reversibly E3A.10 In an ideal heat engine the cold sink is at 0 °C. If 10.00 kJ of heat is
and isothermally as heat to a large block of lead. Calculate the change in withdrawn from the hot source and 3.00 kJ of work is generated, at what
entropy of the block if the process takes place at (i) 20 °C, (ii) 100 °C. temperature is the hot source?
E3A.8 Calculate the change in entropy of the gas when 4.00 g of nitrogen is E3A.11 An ideal heat engine has a hot source at 40 °C. At what temperature
allowed to expand isothermally from 500 cm3 to 750 cm3 at 300 K. must the cold sink be if the efficiency is to be 10 per cent?
E3A.9 Calculate the change in the entropies of the system and the
surroundings, and the total change in entropy, when the volume of a sample

Problems
P3A.1 A sample consisting of 1.00 mol of perfect gas molecules at 27 °C is efficiency η. (f) Confirm that your answer agrees with the efficiency given by
expanded isothermally from an initial pressure of 3.00 atm to a final pressure eqn 3A.9 and that your values for the heat involved in the isothermal stages
of 1.00 atm in two ways: (a) reversibly, and (b) against a constant external are in accord with eqn 3A.6.
pressure of 1.00 atm. Evaluate q, w, ΔU, ΔH, ΔS, ΔSsurr, and ΔStot in each case.
P3A.4 The Carnot cycle is usually represented on a pressure–volume
P3A.2 A sample consisting of 0.10 mol of perfect gas molecules is held by a diagram (Fig. 3A.8), but the four stages can equally well be represented on
piston inside a cylinder such that the volume is 1.25 dm3; the external pressure a temperature–entropy diagram, in which the horizontal axis is entropy
is constant at 1.00 bar and the temperature is maintained at 300 K by a and the vertical axis is temperature; draw such a diagram. Assume that the
thermostat. The piston is released so that the gas can expand. Calculate (a) the temperature of the hot source is Th and that of the cold sink is Tc, and that the
volume of the gas when the expansion is complete; (b) the work done when volume of the working substance (the gas) expands from VA to VB in the first
the gas expands; (c) the heat absorbed by the system. Hence calculate ΔStot. isothermal stage. (a) By considering the entropy change of each stage, derive
an expression for the area enclosed by the cycle in the temperature–entropy
P3A.3 Consider a Carnot cycle in which the working substance is 0.10 mol
diagram. (b) Derive an expression for the work done over the cycle. (Hint: The
of perfect gas molecules, the temperature of the hot source is 373 K, and that
work done is the difference between the heat extracted from the hot source
of the cold sink is 273 K; the initial volume of gas is 1.00 dm3, which doubles
and that deposited in the cold sink; or use eqns 3A.7 and 3A.9.) (c) Comment
over the course of the first isothermal stage. For the reversible adiabatic stages
on the relation between your answers to (a) and (b).
it may be assumed that VT 3/2 = constant. (a) Calculate the volume of the gas
after Stage 1 and after Stage 2 (Fig. 3A.8). (b) Calculate the volume of gas after P3A.5 A heat engine does work as a result of extracting energy as heat from
Stage 3 by considering the reversible adiabatic compression from the starting the hot source and discarding some of it into the cold sink. Such an engine
point. (c) Hence, for each of the four stages of the cycle, calculate the heat can also be used as a heat pump in which heat is extracted from a cold source;
transferred to or from the gas. (d) Explain why the work done is equal to the some work is done on the engine and thereby converted to heat which is
difference between the heat extracted from the hot source and that deposited added to that from the cold source before being discarded into the hot
in the cold sink. (e) Calculate the work done over the cycle and hence the sink. (a) Assuming that the engine is perfect and that the heat transfers are
Exercises and problems 113

reversible, use the Second Law to explain why it is not possible for heat to be found that to keep a certain room at 18 °C a heater rated at 5 kW is required.
extracted from the cold source and discarded into the hot sink without some Assuming that an ideal heat pump is used, and that all heat transfers are
work being done on the engine. (b) Assume that the hot sink is at temperature reversible, calculate the power needed to maintain the room temperature.
Th and the cold source at Tc, and that heat of magnitude |q| is extracted from Recall that 1 W = 1 J s−1. Hint: See the results from Problem P3A.5.
the cold source. Use the Second Law to find the magnitude of the work |w|
P3A.7 Prove that two reversible adiabatic paths can never cross. Assume that
needed to make it possible for heat of magnitude |q|+|w| to be discarded into
the energy of the system under consideration is a function of temperature
the hot sink.
only. Hint: Suppose that two such paths can intersect, and complete a
P3A.6 Heat pumps can be used as a practical way of heating buildings. The cycle with the two paths plus one isothermal path. Consider the changes
ground itself can be used as the cold source because at a depth of a few metres accompanying each stage of the cycle and show that they conflict with the
the temperature is independent of the air temperature; in temperate latitudes Kelvin statement of the Second Law.
the ground temperature is around 13 °C at a depth of 10 m. On a cold day it is

TOPIC 3B Entropy changes accompanying specific processes


Discussion question
D3B.1 Account for deviations from Trouton’s rule for liquids such as water,
mercury, and ethanol. Is their entropy of vaporization larger or smaller than
85 J K−1 mol−1? Why?

Additional exercises
E3B.9 Use Trouton’s rule to predict the enthalpy of vaporization of cyclohexane two blocks, the entropy change of each, and ΔStot. The specific heat capacity
from its normal boiling point, 80.7 °C. of copper is 0.385 J K−1 g−1 and may be assumed constant over the temperature
−1 range involved. Comment on the sign of ΔStot.
E3B.10 The enthalpy of vaporization of methanol is 35.27 kJ mol at its normal
boiling point of 64.1 °C. Calculate (i) the entropy of vaporization of methanol E3B.14 Calculate ΔStot when two iron blocks, each of mass 10.0 kg, one at
at this temperature and (ii) the entropy change of the surroundings. 100 °C and the other at 25 °C, are placed in contact in an isolated container
and allowed to come to equilibrium. The specific heat capacity of iron is
E3B.11 Estimate the change in the molar entropy of N2(g) when the
0.449 J K−1 g−1 and may be assumed constant over the temperature range
temperature is lowered from 298 K to 273 K, given that Cp,m(N2) =
involved. Comment on the sign of ΔStot.
29.125 J K−1 mol−1 at 298 K.
E3B.15 Calculate ΔS (for the system) when the state of 2.00 mol of gas
E3B.12 Calculate the molar entropy of a constant-volume sample of argon at
−1 −1 molecules, for which C p,m = 72 R, is changed from 25 °C and 1.50 atm to 135 °C
250 K given that it is 154.84 J K mol at 298 K; the molar constant-volume
and 7.00 atm.
heat capacity of argon is 23 R.
E3B.16 Calculate the change in entropy of the system when 15.0 g of ice at
E3B.13 Two copper blocks, each of mass 1.00 kg, one at 50 °C and the other
−12.0 °C is converted to water vapour at 105.0 °C at a constant pressure of
at 0 °C, are placed in contact in an isolated container (so no heat can escape)
1 bar. For data, see Exercise 3B.8.
and allowed to come to equilibrium. Calculate the final temperature of the

Problems
P3B.1 Consider a process in which 1.00 mol H2O(l) at −5.0 °C solidifies P3B.4 The molar heat capacity of N2(g) in the range 200 K to 400 K is given by
to ice at the same temperature. Calculate the change in the entropy of the Cp,m/(J K−1 mol−1) = 28.58 + 3.77 × 10−3(T/K). Given that the standard molar
sample, of the surroundings and the total change in the entropy. Is the entropy of N2(g) at 298 K is 191.6 J K−1 mol−1, calculate the value at 373 K.
process spontaneous? Repeat the calculation for a process in which 1.00 mol Repeat the calculation but this time assuming that Cp,m is independent of
H2O(l) vaporizes at 95.0 °C and 1.00 atm. The molar constant-pressure heat temperature and takes the value 29.13 J K−1 mol−1. Comment on the difference
capacities are: Cp,m(H2O(s)) = 37.6 J K−1 mol−1; Cp,m(H2O(l)) = 75.3 J K−1 mol−1; between the results of the two calculations.
and Cp,m(H2O(g)) = 33.6 J K−1 mol−1. The standard enthalpy of vaporization
P3B.5 Find an expression for the change in entropy when two blocks of the
of H2O(l) is 40.7 kJ mol−1, and the standard enthalpy of fusion of H2O(l) is
same substance and of equal mass, one at the temperature Th and the other
6.01 kJ mol−1, both at the relevant transition temperatures.
at Tc, are brought into thermal contact and allowed to reach equilibrium.
P3B.2 Show that a process in which liquid water at 5.0 °C solidifies to ice at the Evaluate the change in entropy for two blocks of copper, each of mass 500 g,
same temperature is not spontaneous (Hint: calculate the total change in the with Cp,m = 24.4 J K−1 mol−1, taking Th = 500 K and Tc = 250 K.
entropy). The data required are given in Exercise E3B.8.
P3B.6 According to Newton’s law of cooling, the rate of change of temperature
P3B.3 The molar heat capacity of trichloromethane (chloroform, CHCl3) is proportional to the temperature difference between the system and its
in the range 240 K to 330 K is given by Cp,m/(J K−1 mol−1) = 91.47 + 7.5 × 10−2(T/K). surroundings:
Calculate the change in molar entropy when CHCl3 is heated from 273 K dT
to 300 K.   (T  Tsur )
dt
114 3 The Second and Third Laws

where Tsur is the temperature of the surroundings and α is a constant. (b) The efficiency η is defined as the modulus of the work over the whole
(a) Integrate this equation with the initial condition that T = Ti at t = 0. cycle divided by the modulus of the heat supplied in step 2. Derive an
(b) Given that the entropy varies with temperature according to S(T) − S(Ti) = expression for η in terms of the temperatures TA−TD.
C ln(T/Ti), where Ti is the initial temperature and C the heat capacity, deduce
(c) Use the relation between V and T for the reversible adiabatic processes to
an expression entropy of the system at time t.
show that your expression for the efficiency can be written η = 1 − (VB/VA)1/c
P3B.7 A block of copper of mass 500 g and initially at 293 K is in thermal (Hint: recall that VC = VB and VD = VA.)
contact with an electric heater of resistance 1.00 kΩ and negligible mass. A
(d) Derive expressions, in terms of CV,m and the temperatures, for the change
current of 1.00 A is passed for 15.0 s. Calculate the change in entropy of the
in entropy (of the system and of the surroundings) for each step of the cycle.
copper, taking Cp,m = 24.4 J K−1 mol−1. The experiment is then repeated with
the copper immersed in a stream of water that maintains the temperature (e) Assuming that VA = 4.00 dm3, pA = 1.00 atm, TA = 300 K, and that
of the copper block at 293 K. Calculate the change in entropy of the copper VA = 10VB and pC/pB = 5, evaluate the efficiency of the cycle and the entropy
and the water in this case. changes for each step. (Hint: for the last part you will need to find TB and TD,
−1 −1 which can be done by using the relation between V and T for the reversible
P3B.8 A block of copper (Cp,m = 24.44 J K mol ) of mass 2.00 kg and at
adiabatic process; you will also need to find TC which can be done by
0 °C is introduced into an insulated container in which there is 1.00 mol
considering the temperature rise in the constant volume process.)
H2O(g) at 100 °C and 1.00 atm. Assuming that all the vapour is condensed to
liquid water, determine: (a) the final temperature of the system; (b) the heat P3B.11 When a heat engine is used as a refrigerator to lower the temperature
transferred to the copper block; and (c) the entropy change of the water, the of an object, the colder the object the more work that is needed to cool it
copper block, and the total system. The data needed are given in Exercise further to the same extent.
E3B.8.
(a) Suppose that the refrigerator is an ideal heat engine and that it extracts
P3B.9 The protein lysozyme unfolds at a transition temperature of 75.5 °C a quantity of heat |dq| from the cold source (the object being cooled) at
and the standard enthalpy of transition is 509 kJ mol−1. Calculate the entropy temperature Tc. The work done on the engine is |dw| and as a result heat
of unfolding of lysozyme at 25.0 °C, given that the difference in the molar (|dq| + |dw|) is discarded into the hot sink at temperature Th. Explain how the
constant-pressure heat capacities upon unfolding is 6.28 kJ K−1 mol−1 and Second Law requires that, for the process to be allowed, the following relation
can be assumed to be independent of temperature. (Hint: Imagine that the must apply:
transition at 25.0 °C occurs in three steps: (i) heating of the folded protein
dq dq  dw
from 25.0 °C to the transition temperature, (ii) unfolding at the transition 
temperature, and (iii) cooling of the unfolded protein to 25.0 °C. Because the Tc Th
entropy is a state function, the entropy change at 25.0 °C is equal to the sum (b) Suppose that the heat capacity of the object being cooled is C (which
of the entropy changes of the steps.) can be assumed to be independent of temperature) so that the heat transfer
P3B.10 The cycle involved in the operation of an internal combustion engine for a change in temperature dTc is dq = CdTc. Substitute this relation into the
is called the Otto cycle (see figure below). The cycle consists of the following expression derived in (a) and then integrate between Tc = Ti and Tc = Tf to give
steps: (1) Reversible adiabatic compression from A to B; (2) reversible the following expression for the work needed to cool the object from Ti to
constant-volume pressure increase from B to C due to the combustion of a Tf as
small amount of fuel; (3) reversible adiabatic expansion from C to D; and Tf
w  CTh ln  C(Tf  Ti )
(4) reversible constant-volume pressure decrease back to state A. Assume Ti
that the pressure, temperature, and volume at point A are pA, TA, and VA,
and likewise for B–D; further assume that the working substance is 1 mol of (c) Use this result to calculate the work needed to lower the temperature of
perfect gas diatomic molecules with CV, m = 25 R. Recall that for a reversible 250 g of water from 293 K to 273 K, assuming that the hot reservoir is at 293 K
adiabatic expansion VATAc = VBTBc , where c = CV,m/R, and that for a perfect gas (Cp,m(H2O(l)) = 75.3 J K−1 mol−1). (d) When the temperature of liquid water
the internal energy is only a function of the temperature. reaches 273 K it will freeze to ice, an exothermic process. Calculate the work
needed to transfer the associated heat to the hot sink, assuming that the water
(a) Evaluate the work and the heat involved in each of the four steps, remains at 273 K (the standard enthalpy of fusion of H2O is 6.01 kJ mol−1 at the
expressing your results in terms of CV,m and the temperatures TA−TD. normal freezing point). (e) Hence calculate the total work needed to freeze the
250 g of liquid water to ice at 273 K. How long will this take if the refrigerator
operates at 100 W?
C −1 −1
P3B.12 The standard molar entropy of NH3(g) is 192.45 J K mol at 298 K,
2 and its heat capacity is given by eqn 2B.8 with the coefficients given in
Adiabat
Table 2B.1. Calculate the standard molar entropy at (a) 100 °C and (b) 500 °C.
B
Pressure, p

1 D

Adiabat 4
A

Volume, V
Exercises and problems 115

TOPIC 3C The measurement of entropy


Discussion question
D3C.1 Explain why the standard entropies of ions in solution may be positive,
negative, or zero.

Additional exercises
E3C.4 At low temperatures the heat capacity of Ag(s) is found to obey the E3C.6 Use data from Tables 2C.3 and 2C.4 to calculate the standard reaction
Debye law Cp,m = aT 3, with a = 1.956 × 10−4 J K−4 mol−1. Determine entropy at 298 K of
Sm(10 K) − Sm(0) for silver. (i) Zn(s) + Cu2+(aq) → Zn2+(aq) + Cu(s)
E3C.5 Use data from Tables 2C.3 and 2C.4 to calculate the standard reaction (ii) sucrose [C12H22O11(s)] + 12 O2(g) → 12 CO2(g) + 11 H2O(l)
entropy at 298 K of
E3C.7 Calculate the standard reaction entropy at 298 K when 1 mol N2O(g) is
(i) 2 CH3CHO(g) + O2(g) → 2 CH3COOH(l) formed from its elements in their reference states.
(ii) Hg(l) + Cl2(g) → HgCl2(s)

Problems
−1 −1
P3C.1 At 10 K Cp,m(Hg(s)) = 4.64 J K mol . Between 10 K and the melting P3C.5 Use values of standard enthalpies of formation, standard entropies,
point of Hg(s), 234.3 K, heat capacity measurements indicate that the and standard heat capacities available from tables in the Resource section
entropy increases by 57.74 J K−1 mol−1. The standard enthalpy of fusion of to calculate the standard enthalpy and entropy changes at 298 K and 398 K
Hg(s) is 2322 J mol−1 at 234.3 K. Between the melting point and 298.0 K, heat for the reaction CO2(g) + H2(g) → CO(g) + H2O(g). Assume that the heat
capacity measurements indicate that the entropy increases by 6.85 J K−1 mol−1. capacities are constant over the temperature range involved.
Determine the Third-Law standard molar entropy of Hg(l) at 298 K.
P3C.6 Use values of enthalpies of formation, standard entropies, and standard
P3C.2 The measurements described in Problem P3C.1 were extended to heat capacities available from tables in the Resource section to calculate
343.9 K, the normal boiling point of Hg(l). Between the melting point the standard enthalpy and entropy of reaction at 298 K and 500 K for
and the boiling point, heat capacity measurements indicate that the entropy 2 N 2 (g )  2 H 2 (g )  NH 3 (g ). Assume that the heat capacities are constant over
1 3

increases by 10.83 J K−1 mol−1. The standard enthalpy of vaporization of the temperature range involved.
Hg(l) is 60.50 kJ mol−1 at 343.9 K. Determine the Third-Law standard
P3C.7 The compound 1,3,5-trichloro-2,4,6-trifluorobenzene is an intermediate
molar entropy of Hg(g) at 343.9 K (you will need some of the data from
in the conversion of hexachlorobenzene to hexafluorobenzene, and its
Problem P3C.1).
thermodynamic properties have been examined by measuring its heat
P3C.3 The molar heat capacity of lead varies with temperature as follows: capacity over a wide temperature range (R.L. Andon and J.F. Martin, J. Chem.
Soc. Faraday Trans. I 871 (1973)). Some of the data are as follows:
T/K 10 15 20 25 30 50
Cp,m/(J K−1 mol−1) 2.8 7.0 10.8 14.1 16.5 21.4 T/K 14.14 16.33 20.03 31.15 44.08 64.81
−1 −1
T/K 70 100 150 200 250 298 Cp,m/(J K mol ) 9.492 12.70 18.18 32.54 46.86 66.36
Cp,m/(J K−1 mol−1) 23.3 24.5 25.3 25.8 26.2 26.6 T/K 100.90 140.86 183.59 225.10 262.99 298.06
Cp,m/(J K−1 mol−1) 95.05 121.3 144.4 163.7 180.2 196.4
(a) Use the Debye T 3-law and the value of the heat capacity at 10 K to
determine the change in entropy between 0 and 10 K. (b) To determine the
Determine the Third-Law molar entropy of the compound at 100 K, 200 K,
change in entropy between 10 K and 298 K you will need to measure the area
and 300 K.
under a plot of Cp,m/T against T. This measurement can either be done by
counting squares or by using mathematical software to fit the data to a simple ⦵
 1 −1
P3C.8‡ Given that Sm  29.79 J K mol for bismuth at 100 K and using the
function (for example, a polynomial) and then integrating that function over following tabulated heat capacity data (D.G. Archer, J. Chem. Eng. Data 40,
the range 10 K to 298 K. Use either of these methods to determine the change 1015 (1995)), determine the standard molar entropy of bismuth at 200 K.
in entropy between 10 K and 298 K. (c) Hence determine the standard Third-
Law entropy of lead at 298 K, and also at 273 K. T/K 100 120 140 150 160 180 200

P3C.4 The molar heat capacity of anhydrous potassium hexacyanoferrate(II) Cp,m/(J K−1 mol−1) 23.00 23.74 24.25 24.44 24.61 24.89 25.11
varies with temperature as follows: Compare the value to the value that would be obtained by taking the heat
T/K 10 20 30 40 50 60 capacity to be constant at 24.44 J K−1 mol−1 over this range.

Cp,m/(J K−1 mol−1) 2.09 14.43 36.44 62.55 87.03 111.0 P3C.9 At low temperatures there are two contributions to the heat capacity of
a metal, one associated with lattice vibrations, which is well-approximated by
T/K 70 80 90 100 110 150
the Debye T3-law, and one due to the valence electrons. The latter is linear in
Cp,m/(J K−1 mol−1) 131.4 149.4 165.3 179.6 192.8 237.6 the temperature. Overall, the heat capacity can be written
T/K 160 170 180 190 200 Debye electronic
 
−1
Cp,m/(J K mol )−1
247.3 256.5 265.1 273.0 280.3 C p ,m(T )  aT 3  bT


Determine the Third-Law molar entropy of the compound at 200 K, 100 K. These problems were provided by Charles Trapp and Carmen Giunta.
116 3 The Second and Third Laws

The molar heat capacity of potassium metal has been measured at very low at temperature T. (Hint: you will need to integrate Cp,m(T)/T.) (d) Hence
temperatures to give the following data determine the molar entropy of potassium at 2.0 K.
P3C.10 At low temperatures the heat capacity of a metal is the sum of a
T/K 0.20 0.25 0.30 0.35 0.40 0.45 0.50 0.55
contribution due to lattice vibrations (the Debye term) and a term due to
Cp,m/(J K−1 mol−1) 0.437 0.560 0.693 0.838 0.996 1.170 1.361 1.572 the valence electrons, as given in the preceding problem. For sodium metal
a = 0.507 × 10−3 J K−4 mol−1 and b = 1.38 × 10−3 J K−2 mol−1. Determine the
(a) Assuming that the expression given above for the heat capacity applies, temperature at which the Debye contribution and the electronic contribution
explain why a plot of Cp,m(T)/T against T 2 is expected to be a straight line to the entropy of sodium are equal. At higher temperatures, which
with slope a and intercept b. (b) Use such a plot to determine the values contribution becomes dominant?
of the constants a and b. (c) Derive an expression for the molar entropy

TOPIC 3D Concentrating on the system


Discussion questions
D3D.1 The following expressions establish criteria for spontaneous change: D3D.2 Under what circumstances, and why, can the spontaneity of a process
dAT,V < 0 and dGT,p < 0. Discuss the origin, significance, and applicability of be discussed in terms of the properties of the system alone?
each criterion.

Additional exercises
E3D.5 Calculate values for the standard reaction enthalpies at 298 K for the E3D.9 Use values of the relevant standard Gibbs energies of formation from
reactions in Exercise E3C.5 by using values of the standard enthalpies of the Resource section to calculate the standard Gibbs energies of reaction at
formation from the tables in the Resource section. Combine your results 298 K of:
with the standard reaction entropies already calculated in that Exercise to (i) 2 AgCl(s) + Br2(l) → 2 AgBr(s) + Cl2(g)
determine the standard reaction Gibbs energy at 298 K for each.
(ii) Hg(l) + Cl2(g) → HgCl2(s)
E3D.6 Calculate values for the standard reaction enthalpies at 298 K for the
E3D.10 Use values of the relevant standard Gibbs energies of formation from
reactions in Exercise E3C.6 by using values of the standard enthalpies of
the Resource section to calculate the standard Gibbs energies of reaction at
formation from the tables in the Resource section. Combine your results
298 K of
with the standard reaction entropies already calculated in that Exercise to
determine the standard reaction Gibbs energy at 298 K for each. (i) Zn(s) + Cu2+(aq) → Zn2+(aq) + Cu(s)

E3D.7 Calculate the standard Gibbs energy of the reaction CO(g) + (ii) sucrose [C12H22O11(s)] + 12 O2(g) → 12 CO2(g) + 11 H2O(l)
CH3CH2OH(l) → CH3CH2COOH(l) at 298 K, using the values of standard E3D.11 The standard enthalpy of combustion of the solid glycine (the amino
entropies and enthalpies of formation given in the Resource section. The data acid NH2CH2COOH) is −969 kJ mol−1 at 298 K and its standard molar entropy
for CH3CH2COOH(l) are Δf H ⦵= −510 kJ mol−1, Sm⦵  191 J K 1 mol 1 at 298 K. is 103.5 J K−1 mol−1. Calculate the standard Gibbs energy of formation of
E3D.8 Calculate the maximum non-expansion work per mole of C3H8 that glycine at 298 K. Note that the nitrogen-containing species produced on
may be obtained from a fuel cell in which the chemical reaction is the combustion is taken to be N2(g).
combustion of propane under standard conditions at 298 K.

Problems
P3D.1 A perfect gas is contained in a cylinder of fixed volume and which is explain why it is not possible to do the same for section A. (f) Given that the
separated into two sections A and B by a frictionless piston; no heat can pass process is reversible, what does this imply about the total ΔA for the process
through the piston. Section B is maintained at a constant temperature of (the sum of ΔA for section A and B)?
300 K; that is, all changes in section B are isothermal. There is 2.00 mol of gas
P3D.2 In biological cells, the energy released by the oxidation of foods is
molecules in each section and the constant-volume heat capacity of the gas is
stored in adenosine triphosphate (ATP or ATP4−).The essence of ATP’s action
CV,m = 20 J K−1 mol−1, which can be assumed to be constant. Initially
is its ability to lose its terminal phosphate group by hydrolysis and to form
TA = TB = 300 K, VA = VB = 2.00 dm3. Energy is then supplied as heat to
adenosine diphosphate (ADP or ADP3−):
Section A so that the gas in A expands, pushing the piston out and thereby
compressing the gas in section B. The expansion takes place reversibly and the ATP 4 (aq )  H2O(1)  ADP3 (aq )  HPO24 (aq )  H3O (aq )
final volume in section B is 1.00 dm3. Because the piston is free to move, the
pressures in sections A and B are always equal; recall, too, that for a perfect At pH = 7.0 and 37 °C (310 K, blood temperature) the enthalpy and
gas the internal energy is a function of only the temperature. (a) Calculate the Gibbs energy of hydrolysis are ΔrH = −20 kJ mol−1 and ΔrG = −31 kJ mol−1,
final pressure of the gas and hence the temperature of the gas in section A. respectively. Under these conditions, the hydrolysis of 1 mol ATP4−(aq)
(b) Calculate the change in entropy of the gas in section A. (Hint: you can think results in the extraction of up to 31 kJ of energy that can be used to do non-
of the process as occurring in a constant volume step and then a constant expansion work, such as the synthesis of proteins from amino acids, muscular
temperature step.) (c) Calculate the entropy change of the gas in section B. contraction, and the activation of neuronal circuits in our brains. (a) Calculate
(d) Calculate the change in internal energy for each section. (e) Use the values and account for the sign of the entropy of hydrolysis of ATP at pH = 7.0 and
of ΔS and ΔU that you have already calculated to calculate ΔA for section B; 310 K. (b) Suppose that the radius of a typical biological cell is 10 μm and that
Exercises and problems 117

inside it 1 × 106 ATP molecules are hydrolysed each second. What is the power absolute entropy: Sm⦵ (Na  (g ))  148 J K 1 mol 1 , Sm⦵ (Cl  (g ))  154 J K 1 mol 1,
density of the cell in watts per cubic metre (1 W = 1 J s−1)? A computer battery Sm⦵ (NaCl(s))  72.1 J K 1 mol 1 (all data at 298 K). The value of ΔrG ⦵ for the
delivers about 15 W and has a volume of 100 cm3. Which has the greater power second step can be found by using the Born equation to estimate the standard
density, the cell or the battery? (c) The formation of glutamine from glutamate Gibbs energies of solvation. For these estimates, use r(Na+) = 170 pm and
and ammonium ions requires 14.2 kJ mol−1 of energy input. It is driven by the r(Cl−) = 211 pm. Hence find ΔrG ⦵ for the overall process and comment on the
hydrolysis of ATP to ADP mediated by the enzyme glutamine synthetase. How value you find.
many moles of ATP must be hydrolysed to form 1 mol glutamine?
P3D.5 The solubility of an ionic solid such as LiF can be explored by
P3D.3 Construct a cycle similar to that in Fig. 3D.3 to analyse the reaction calculating the standard Gibbs energy change for the process LiF(s) → Li+(aq) +
 
2 H 2 (g )  2 I2 (s)  H (aq )  I (aq ) and use it to find the value of the standard
1 1
F−(aq). Consider this process in two steps: (1) LiF(s) → Li+(g) + F−(g) and
Gibbs energy of formation of I−(aq). You should refer to the tables in the then (2) Li+(g) + F−(g) → Li+(aq) + F−(aq). Estimate ΔrG⦵ for the first step
Resource section for relevant values of the Gibbs energies of formation. As given that ΔrH ⦵ = 1037 kJ mol−1 in step 1 and with the following values of the
in the text, the standard Gibbs energy for the process H(g) → H+(g) + e−(g) absolute entropy: Sm⦵ (Li  )  133 J K 1 mol 1, Sm⦵ (F )  145 J K 1 mol 1,
should be approximated by the ionization energy, and that for I(g) + e−(g) → Sm⦵ (LiF, s)  35.6 J K 1 mol 1 (all data at 298 K). The value of ΔrG ⦵ for the
I−(g) by the electron affinity. The standard Gibbs energy of solvation of H+ can second step can be found by using the Born equation to estimate the standard
be taken as −1090 kJ mol−1 and of I− as −247 kJ mol−1. Gibbs energies of solvation. For these estimates, use r(Li+) = 127 pm and
P3D.4 The solubility of an ionic solid such as NaCl can be explored by r(F−) = 163 pm. Hence find ΔrG ⦵ for the overall process and comment on the
calculating the standard Gibbs energy change for the process NaCl(s) → value you find.
Na+(aq) + Cl−(aq). Consider this process in two steps: (1) NaCl(s) → Na+(g) + ⦵
P3D.6 From the Born equation derive an expression for ΔsolvS and ΔsolvH

Cl−(g) and then (2) Na+(g) + Cl−(g) → Na+(aq) + Cl−(aq). Estimate ΔrG ⦵ for (Hint: (∂G/∂T)p = −S). Comment on your answer in the light of the
the first step given that Δr H ⦵ = 787 kJ mol−1 and the following values of the assumptions made in the Born model.

TOPIC 3E Combining the First and Second Laws


Discussion questions
D3E.1 Suggest a physical interpretation of the dependence of the Gibbs energy D3E.2 Suggest a physical interpretation of the dependence of the Gibbs energy
on the temperature. on the pressure.

Additional exercises
E3E.5 Suppose that 6.0 mmol of perfect gas molecules initially occupies 52 cm
3
density of water is 0.997 g cm−3, determine the change in the molar Gibbs
at 298 K and then expands isothermally to 122 cm3. Calculate ΔG for the energy.
process.
E3E.9 The change in the molar volume accompanying fusion of solid CO2 is
E3E.6 The change in the Gibbs energy of a certain constant-pressure process is −1.6 cm3 mol−1. Determine the change in the molar Gibbs energy of fusion
found to fit the expression ΔG/J = −73.1 + 42.8(T/K). Calculate the value of ΔS when the pressure is increased from 1 bar to 1000 bar.
for the process.
E3E.10 The change in the molar volume accompanying fusion of solid benzene
E3E.7 The change in the Gibbs energy of a certain constant-pressure process is 0.5 cm3 mol−1. Determine the change in Gibbs energy of fusion when the
is found to fit the expression ΔG/J = −73.1 + 42.8(T/K). Use the Gibbs– pressure is increased from 1 bar to 5000 bar.
Helmholtz equation to calculate the value of ΔH for the process.
E3E.11 Calculate the change in the molar Gibbs energy of a perfect gas when
3
E3E.8 Estimate the change in the Gibbs energy of 100 cm of water when the its pressure is increased isothermally from 50.0 kPa to 100.0 kPa at 500 K.
pressure acting on it is increased from 100 kPa to 500 kPa. Given that the mass

Problems
P3E.1 (a) By integrating the Gibbs–Helmholtz equation between temperature P3E.3 At 298 K the standard enthalpy of combustion of sucrose is
T1 and T2, and with the assumption that ΔH is independent of temperature, −5797 kJ mol−1 and the standard Gibbs energy of the reaction is
show that −6333 kJ mol−1. Estimate the additional non-expansion work that may be
obtained by raising the temperature to blood temperature, 37 °C. (Hint: use
G(T2 ) G(T1 ) 1 1
  H    the result from Problem P3E.1 to determine ΔrG ⦵ at the higher temperature.)
T2 T1  T2 T1 
P3E.4 Consider gases described by the following three equations of state:
where ΔG(T) is the change in Gibbs energy at temperature T. (b) Using values RT
of the standard Gibbs energies and enthalpies of formation from the Resource (a) perfect: p =
Vm
section, determine ΔrG ⦵ and ΔrH ⦵ at 298 K for the reaction 2 CO(g) + O2(g) →
RT a
2 CO2(g). (c) Hence estimate ΔrG ⦵ at 375 K. (b) van der Waals: p  
Vm  b Vm2
⦵ ⦵
P3E.2 Calculate ΔrG and ΔrH at 298 K for N2(g) + 3 H2(g) → 2 NH3(g).

Then, using the result from Problem P3E.1 (a), estimate ΔrG at 500 K and at RTe  a /RTVm
(c) Dieterici: p 
1000 K. Vm  b
118 3 The Second and Third Laws

Use the Maxwell relation (∂S/∂V)T = (∂p/∂T)V to derive an expression for (a) Consider first the case where only the repulsive term is significant; that is,
(∂S/∂V)T for each equation of state. For an isothermal expansion, compare the a = 0, b ≠ 0. Rearrange the equation of state into an expression for Vm,
change in entropy expected for a perfect gas and for a gas obeying the van der substitute it into (∂Gm/∂p)T = Vm, and then integrate so as to obtain an
Waals equation of state: which has the greatest change in entropy and how can expression for the pressure dependence of Gm. Compare your result with
this conclusion be rationalized? that for a perfect gas. (b) Now consider the case where only the attractive
terms are included; that is, b = 0, a ≠ 0. The equation of state then becomes a
P3E.5 Only one of the four Maxwell relations is derived in the text. Derive the
quadratic equation in Vm. Find this equation and solve it for Vm. Approximate
remaining three to give the complete set listed in Table 3E.1. Start with the
the solution by assuming that pa/R2T2 << 1 and using the expansion
definition of H (H = U + pV), form the exact differential (dH = dU + pdV +
(1  x )1/ 2  1  12 x , which is valid for x << 1. Hence find an expression for the
Vdp), and then substitute dU = TdS − pdV. The resulting expression gives
pressure dependence of Gm, and interpret the result. (c) For CO2 a = 3.610 atm
rise to a Maxwell relation in a way analogous to how eqn 3E.5 arises from eqn
dm6 mol−2, b = 4.29 × 10−2 dm3 mol−1. Use mathematical software to plot Gm
3E.1. Repeat the process, starting with the definitions of A and then G, to give
as a function of pressure at 298 K for a perfect gas and the two cases analysed
the remaining two Maxwell relations.
above. (Use R = 8.2057 × 10−2 dm3 atm K−1 mol−1.)
P3E.6 Suppose that S is regarded as a function of p and T so that ‡
P3E.8 Nitric acid hydrates have received much attention as possible catalysts
 S   S  for heterogeneous reactions that bring about the Antarctic ozone hole.
dS    dp    dT Worsnop et al. (Science 259, 71 (1993)) investigated the thermodynamic
 p T  T  p
stability of these hydrates under conditions typical of the polar winter
Use (∂S/∂T)p = Cp/T and an appropriate Maxwell relation to show that stratosphere. They report thermodynamic data for the sublimation of mono-,
TdS = CpdT − αTVdp, where the expansion coefficient, α, is defined as di-, and trihydrates to nitric acid and water vapours, HNO3·nH2O(s) →
α = (1/V)(∂V/∂T)p. Hence, show that the energy transferred as heat, q, when HNO3(g) + nH2O(g), for n = 1, 2, and 3. Given ΔrG ⦵ and ΔrH ⦵ for these
the pressure on an incompressible liquid or solid is increased by Δp in a reactions at 220 K, use the Gibbs–Helmholtz equation to compute ΔrG ⦵ for
reversible isothermal process is given by q = −αTVΔp. Evaluate q when the each at 190 K.
pressure acting on 100 cm3 of mercury at 0 °C is increased by 1.0 kbar.
(α = 1.82 × 10−4 K−1.) n 1 2 3
P3E.7 The pressure dependence of the molar Gibbs energy is given by ΔrG ⦵/(kJ mol−1) 46.2 69.4 93.2
(∂Gm/∂p)T = Vm. This problem involves exploring this dependence for a gas ΔrH /(kJ mol−1)

127 188 237
described by the van der Waals equation of state
RT a
p 
Vm  b Vm2

FOCUS 3 The Second and Third Laws


Integrated activities
I3.1 A sample consisting of 1.00 mol gas molecules is described by the (c) Allow for the temperature dependence of the heat capacity by writing
equation of state pVm = RT(1 + Bp). Initially at 373 K, it undergoes Joule– C = a + bT + c/T2, and plot the change in entropy for different values of the
Thomson expansion (Topic 2D) from 100 atm to 1.00 atm. Given that three coefficients (including negative values of c).
C p,m = 25 R, μ = 0.21 K atm−1, B = −0.525(K/T) atm−1, and that these are
(d) Show how the first derivative of G, (∂G/∂p)T , varies with pressure, and
constant over the temperature range involved, calculate ΔT and ΔS for the gas.
plot the resulting expression over a pressure range. What is the physical
I3.2 Discuss the relation between the thermodynamic and statistical significance of (∂G/∂p)T?
definitions of entropy.
(e) Evaluate the fugacity coefficient (see A deeper look 5F.2, available to read
I3.3 Use mathematical software or a spreadsheet to: in the e-book accompanying this text) as a function of the reduced volume
of a van der Waals gas and plot the outcome over the range 0.8 ≤ Vr ≤ 3 for a
(a) Evaluate the change in entropy of 1.00 mol CO2(g) on expansion from
selection of reduced temperatures.
0.001 m3 to 0.010 m3 at 298 K, treated as a van der Waals gas.
(b) Plot the change in entropy of a perfect gas of (i) atoms, (ii) linear rotors,
(iii) nonlinear rotors as the sample is heated over the same range under
conditions of constant volume and then constant pressure.
FOCUS 4
Physical transformations of
pure substances

Vaporization, melting (fusion), and the conversion of graphite


to diamond are all examples of changes of phase without change
4B Thermodynamic aspects of phase
of chemical composition. The discussion of the phase transi- transitions
tions of pure substances is among the simplest applications of
thermodynamics to chemistry, and is guided by the principle This Topic considers the factors that determine the positions
that, at constant temperature and pressure, the tendency of a and shapes of the phase boundaries. The expressions derived
system is to minimize its Gibbs energy. show how the vapour pressure of a substance varies with tem-
perature and how the melting point varies with pressure.
4B.1 The dependence of stability on the conditions; 4B.2 The location
of coexistence curves

4A Phase diagrams of pure


substances
What is an application of this material?
One type of phase diagram is a map of the pressures and tem-
peratures at which each phase of a substance is the most sta- The properties of carbon dioxide in its supercritical fluid phase
ble. The thermodynamic criterion for phase stability leads to can form the basis for novel and useful chemical separation
a very general result, the ‘phase rule’, which summarizes the methods, and have considerable promise for the synthetic pro-
constraints on the equilibria between phases. In preparation for cedures adopted in ‘green’ chemistry. Its properties and applica-
additional discussion through the text, this rule is expressed in tions are discussed in Impact 6, accessed via the e-book.
a general way that can be applied to systems of more than one
component. This Topic also develops the concept of ‘chemi-
cal potential’ introduced in Topic 3D, a property that is at the
centre of discussions of mixtures and chemical reactions. The
Topic then describes the interpretation of the phase diagrams
of a representative selection of substances.
4A.1 The stabilities of phases; 4A.2 Coexistence curves;
4A.3 Three representative phase diagrams

➤ Go to the e-book for videos that feature the derivation and interpretation of equations, and applications of this material.
TOPIC 4A Phase diagrams of
pure substances

The number of phases in a system is denoted P. A gas,


➤ Why do you need to know this material? or a gaseous mixture, is a single phase (P = 1), a crystal of a
­substance is a single phase, and two fully mixed liquids form a
Phase diagrams summarize the behaviour of substances
single phase.
under different conditions, and identify which phase or
phases are the most stable at a particular temperature and
pressure. Such diagrams are important tools for understand- Brief illustration 4A.1
ing the behaviour of both pure substances and mixtures.
A solution of sodium chloride in water is a single phase (P = 1).
➤ What is the key idea? Ice is a single phase even though it might be chipped into small
A pure substance tends to adopt the phase with the lowest fragments. A slurry of ice and water is a two-phase system (P = 2)
chemical potential. even though it is difficult to map the physical boundaries between
the phases. A system in which calcium carbonate undergoes the
➤ What do you need to know already? thermal decomposition CaCO3(s) → CaO(s) + CO2(g) consists
This Topic builds on the fact that the Gibbs energy is a sign- of two solid phases (one consisting of calcium carbonate and
post of spontaneous change under conditions of constant the other of calcium oxide) and one gaseous phase (consisting of
temperature and pressure (Topic 3D). carbon dioxide), so P = 3.

Two metals form a two-phase system (P = 2) if they are


i­mmiscible, but a single-phase system (P = 1), an alloy, if they
One of the most succinct ways of presenting the physical are miscible (and actually mixed). A solution of solid B in solid
changes of state that a substance can undergo is in terms of its A—a homogeneous mixture of the two miscible substances—
‘phase diagram’. This material is also the basis of the discussion is uniform on a molecular scale. In a solution, atoms of A are
of mixtures in Focus 5. surrounded by atoms of A and B, and any sample cut from the
sample, even microscopically small, is representative of the
composition of the whole. It is therefore a single phase.
A dispersion is uniform on a macroscopic scale but not on
4A.1 The stabilities of phases a microscopic scale, because it consists of grains or droplets of
one substance in a matrix of the other (Fig. 4A.1). A small sam-
Thermodynamics provides a powerful framework for describ-
ple could come entirely from one of the minute grains of pure
ing and understanding the stabilities and transformations of
phases, but the terminology must be used carefully. In particu-
lar, it is necessary to understand the terms ‘phase’, ‘component’,
and ‘degree of freedom’.

4A.1(a) The number of phases


A phase is a form of matter that is uniform throughout in chem-
ical composition and physical state. Thus, there are the solid,
liquid, and gas phases of a substance, as well as various solid
phases, such as the white and black allotropes of phosphorus,
(a) (b)
or the aragonite and calcite polymorphs of calcium ­carbonate.1
Figure 4A.1 The difference between (a) a single-phase solution,
1
An allotrope is a particular molecular form of an element (such as O2 and in which the composition is uniform on a molecular scale, and
O3) and may be solid, liquid, or gas. A polymorph is one of a number of solid (b) a dispersion, in which microscopic regions of one component
phases of an element or compound. are embedded in a matrix of a second component.
4A Phase diagrams of pure substances 121

A and would not be representative of the whole. A dispersion t­hermodynamics to be spontaneous might occur too slowly
therefore consists of two phases. to be significant in practice. For instance, at normal tem-
peratures and pressures the molar Gibbs energy of graphite
is lower than that of diamond, so there is a thermodynamic
4A.1(b) Phase transitions tendency for diamond to change into graphite. However, for
A phase transition, the spontaneous conversion of one phase this transition to take place, the C atoms must change their
into another phase, occurs at a characteristic transition locations, which, except at high temperatures, is an immeas-
­temperature, Ttrs, for a given pressure. At the transition temper- urably slow process in a solid. The discussion of the rate of
ature the two phases are in equilibrium, meaning that the Gibbs attainment of equilibrium is a kinetic problem and is outside
energy of the system is a minimum at the prevailing pressure. the range of thermodynamics. In gases and liquids the mo-
bilities of the molecules allow phase transitions to occur rap-
idly, but in solids thermodynamic instability may be frozen
Brief illustration 4A.2
in. Thermodynamically unstable phases that persist because
At 1 atm, ice is the stable phase of water below 0 °C, but above the transition is kinetically hindered are called metastable
0 °C liquid water is more stable. This difference indicates that phases. Diamond is a metastable but persistent phase of car-
below 0 °C the Gibbs energy decreases as liquid water changes bon under normal conditions.
into ice, but that above 0 °C the Gibbs energy decreases as ice
changes into liquid water. The numerical values of the Gibbs 4A.1(c) Thermodynamic criteria of phase
energies are considered in Brief illustration 4A.3.
stability
The discussion in this Topic is based on the definition of the
The detection of a phase transition is not always straightfor- chemical potential, which for a single substance is simply an-
ward as there may be nothing to see, especially if the two phases other name for the molar Gibbs energy: μ = G/n (Topic 3D). It
are both solids. Thermal analysis, which takes advantage of is also based on the following consequence of the Second Law
the heat that is evolved or absorbed during a transition, can be (Fig. 4A.3):
used. Thus, if the phase transition is exothermic and the tem-
At equilibrium, the chemical potential of a substance is
perature of a sample is monitored as it cools, the presence of
the same in and throughout every phase present in the
the transition can be recognized by a pause in the otherwise
system.
steady fall of the temperature (Fig. 4A.2). Similarly, if a sample
is heated steadily and the transition is endothermic, there will To see the validity of this remark, consider a system in which
be a pause in the temperature rise at the transition tempera- the chemical potential of a substance is μ1 at one location and
ture. Differential scanning calorimetry (Topic 2C) is also used μ2 at another location. The locations may be in the same or in
to detect phase transitions, and X-ray diffraction (Topic 15B) is different phases. When an infinitesimal amount dn of the sub-
useful for detecting phase transitions in a solid, because the two stance is transferred from one location to the other, the Gibbs
phases will have different structures. energy of the system changes by −μ1dn (i.e. dG = −Gm,1dn)
As always, it is important to distinguish between the ther- when material is removed from location 1. It changes by +μ2dn
modynamic description of a process and the rate at which (i.e. dG = Gm,2dn) when that material is added to location 2.
the process occurs. A phase transition that is predicted by The overall change is therefore dG = (μ2 − μ1)dn. If the chemical

Liquid
cooling
Temperature, T

Liquid
Tf freezing Same chemical
Solid potential
cooling

Time, t

Figure 4A.2 A cooling curve at constant pressure. The flat section Figure 4A.3 When two or more phases are in equilibrium,
corresponds to the pause in the fall of temperature while an the chemical potential of a substance (and, in a mixture, a
exothermic transition (freezing) occurs. This pause enables Tf to be component) is the same in each phase, and is the same at all
located even if the transition cannot be observed visually. points in each phase.
122 4 Physical transformations of pure substances

Characteristic properties related to


4A.2(a)
Critical phase transitions
point
Solid Consider a liquid sample of a pure substance in a closed vessel.
Liquid
The pressure of a vapour in equilibrium with the liquid is its va-
Pressure, p

pour pressure (the property introduced in Topic 1C; Fig. 4A.5).


Triple
point Therefore, the liquid–vapour coexistence curve in a phase dia-
gram shows how the vapour pressure of the liquid varies with
Vapour temperature. Similarly, the solid–vapour coexistence curve
shows the temperature variation of the sublimation vapour
T3 Tc
pressure, the vapour pressure of the solid phase. The vapour
Temperature, T pressure of a substance increases with temperature because at
Figure 4A.4 The general regions of pressure and temperature higher temperatures more molecules have sufficient energy to
where solid, liquid, or gas is stable are shown on this phase escape from their neighbours.
diagram. For example, the solid phase is the most stable phase at When a liquid is in an open vessel and subject to an external
low temperatures and high pressures. pressure, it is possible for the liquid to vaporize from its surface.
However, only when the temperature is such that the vapour
pressure is equal to the external pressure will it be possible for
potential at location 1 is higher than that at location 2, the vaporization to occur throughout the bulk of the liquid and for
transfer is accompanied by a decrease in G, and so has a spon- the vapour to expand freely into the surroundings. This condi-
taneous tendency to occur. Only if μ1 = μ2 is there no change in tion of free vaporization throughout the liquid is called ­boiling.
G, and only then is the system at equilibrium. The temperature at which the vapour pressure of a liquid is
equal to the external pressure is called the boiling temperature
at that pressure. For the special case of an external pressure
Brief illustration 4A.3 of 1 atm, the boiling temperature is called the normal boiling
point, Tb. With the replacement of 1 atm by 1 bar as standard
The standard molar Gibbs energy of formation of water vapour pressure, there is some advantage in using the standard ­boiling
at 298 K (25 °C) is −229 kJ mol−1, and that of liquid water at point instead: this is the temperature at which the vapour pres-
the same temperature is −237 kJ mol−1. It follows that there is sure reaches 1 bar. Because 1 bar is slightly less than 1 atm
a decrease in Gibbs energy when water vapour condenses to
(1.00 bar = 0.987 atm), the standard boiling point of a liquid is
the liquid at 298 K, so condensation is spontaneous at that
slightly lower than its normal boiling point. For example, the
temperature (and 1 bar).
normal boiling point of water is 100.0 °C, but its standard boil-
ing point is 99.6 °C.
Boiling does not occur when a liquid is heated in a rigid,
Exercise closed vessel. Instead, the vapour pressure, and hence the den-
E4A.1 The difference in chemical potential of a particular substance between sity of the vapour, rises as the temperature is raised (Fig. 4A.6).
two regions of a system is +7.1 kJ mol−1. By how much does the Gibbs energy At the same time, the density of the liquid decreases slightly as
change when 0.10 mmol of that substance is transferred from one region to
the other?

4A.2 Coexistence curves Vapour


pressure,
Vapour p
The phase diagram of a pure substance shows the regions of
pressure and temperature at which its various phases are ther-
modynamically stable (Fig. 4A.4). In fact, any two intensive
variables may be used (such as temperature and magnetic field;
Liquid
in Topic 5A mole fraction is another variable), but this Topic
or solid
focuses on pressure and temperature. The lines separating the
regions, which are called coexistence curves (or phase bounda-
ries), show the values of p and T at which two phases coexist in Figure 4A.5 The vapour pressure of a liquid or solid is the
equilibrium and their chemical potentials are equal. A single pressure exerted by the vapour in equilibrium with the condensed
phase is represented by an area on a phase diagram. phase.
4A Phase diagrams of pure substances 123

Brief illustration 4A.4

The triple point of water lies at 273.16 K and 611 Pa (6.11 mbar,
4.58 Torr), and the three phases of water (ice, liquid water, and
water vapour) coexist in equilibrium at no other combination
of pressure and temperature. This invariance of the triple point
was the basis of its use in the now superseded definition of the
Kelvin scale of temperature (Topic 3A).

(a) (b) (c) 4A.2(b) The phase rule


Figure 4A.6 (a) A liquid in equilibrium with its vapour. (b) When In one of the most elegant arguments in the whole of chemi-
a liquid is heated in a sealed container, the density of the vapour cal thermodynamics, J.W. Gibbs deduced the phase rule, which
phase increases and the density of the liquid decreases slightly. gives the number of parameters that can be varied indepen-
There comes a stage, (c), at which the two densities are equal and dently (at least to a small extent) while the number of phases
the interface between the fluids disappears. This disappearance in equilibrium is preserved. The phase rule is a general relation
occurs at the critical temperature. between the variance, F, the number of components, C, and the
number of phases at equilibrium, P, for a system of any compo-
sition. Each of these quantities has a precisely defined meaning:
a result of its expansion. There comes a stage when the density
of the vapour is equal to that of the remaining liquid and the • The variance (or number of degrees of freedom), F, of a
surface between the two phases disappears. The temperature at system is the number of intensive variables that can be
which the surface disappears is the critical temperature, Tc, of changed independently without disturbing the number of
the substance. The vapour pressure at the critical temperature phases in equilibrium.
is called the critical pressure, pc. At and above the critical tem- • A constituent of a system is any chemical species that is
perature, a single uniform phase called a supercritical fluid fills present.
the container and an interface no longer exists. That is, above • A component is a chemically independent constituent of a
the critical temperature, the liquid phase of the substance does system.
not exist.
The temperature at which, under a specified pressure, the The number of components, C, in a system is the minimum num-
liquid and solid phases of a substance coexist in equilibrium is ber of types of independent species (ions or molecules) necessary
called the melting temperature. Because a substance melts at to define the composition of all the phases present in the system.
exactly the same temperature as it freezes, the melting tempera-
ture of a substance is the same as its freezing temperature. The Brief illustration 4A.5
freezing temperature when the pressure is 1 atm is called the
normal freezing point, Tf, and its freezing point when the pres- A mixture of ethanol and water has two constituents. A solu-
sure is 1 bar is called the standard freezing point. The normal tion of sodium chloride has three constituents: water, Na+ ions,
and standard freezing points are negligibly different for most and Cl− ions, but only two components because the numbers
purposes. The normal freezing point is also called the normal of Na+ and Cl− ions are constrained to be equal by the require-
melting point. ment of charge neutrality.
There is a set of conditions under which three different
phases of a substance (typically solid, liquid, and vapour) all
simultaneously coexist in equilibrium. These conditions are The relation between these quantities, which is called the
represented by the triple point, a point at which the three phase rule, is established by considering the conditions for
coexistence curves meet. The temperature at the triple point equilibrium to exist between the phases in terms of the chemi-
is denoted T3. The triple point of a pure substance cannot be cal potentials of all the constituents.
changed: it occurs at a single definite pressure and temperature
characteristic of the substance. How is that done? 4A.1 Deducing the phase rule
As can be seen from Fig. 4A.4, the triple point marks the low-
est pressure at which a liquid phase of a substance can exist. If The argument that leads to the phase rule is most easily appre-
(as is common) the slope of the solid–liquid coexistence curve ciated by first thinking about the simpler case when only one
is as shown in the diagram, then the triple point also marks the component is present and then generalizing the result to an
lowest temperature at which the liquid can exist. arbitrary number of components.
124 4 Physical transformations of pure substances

Step 1 Consider the case where only one component is present vary one of the P(C − 1) + 2 intensive variables. It follows that
When only one phase is present (P = 1), both p and T can be the total number of degrees of freedom is
varied independently, so F = 2. Now consider the case where F  P (C  1)  2  C(P  1)
two phases α and β are in equilibrium (P = 2). If the phases
are in equilibrium at a given pressure and temperature, their The right-hand side simplifies to give the phase rule in the form
chemical potentials must be equal: derived by Gibbs:

 (; p,T )   (; p,T ) (4A.1)


F=C−P+2 The phase rule
This equation relates p and T: when the pressure changes, the
changes in the chemical potentials are different in general, so
in order to keep them equal, the temperature must change too. The implications of the phase rule for a one-component sys-
To keep the two phases in equilibrium only one variable can be tem, when
changed arbitrarily, so F = 1.
F 3P The phase rule [ C = 1] (4A.2)
If three phases of a one-component system are in mutual
equilibrium, the chemical potentials of all three phases (α, β, are summarized in Fig. 4A.7. When only one phase is present
and γ) must be equal: in a one-component system, F = 2 and both p and T can be var-
 (; p,T )   (; p,T )   ( ; p,T ) ied independently (at least over a small range) without chang-
ing the number of phases. The system is said to be bivariant,
This relation is actually two equations μ(α; p,T) = μ(β; p,T) and meaning having two degrees of freedom. In other words, a sin-
μ(β; p,T) = μ(γ; p,T), in which there are two ­variables: pres- gle phase is represented by an area on a phase diagram.
sure and temperature. With two equations for two unknowns,
When two phases are in equilibrium F = 1, which implies
there is a single solution (just as the pair of algebraic equations
that pressure is not freely variable if the temperature is set; in-
x + y = xy and 3x − y = xy have the single, fixed solutions x = 2
deed, at a given temperature, a liquid has a characteristic va-
and y = 2). There is therefore only one single, unchangeable
pour pressure. It follows that the equilibrium of two phases is
value of the pressure and temperature as a solution. The con-
represented by a line in the phase diagram. Instead of select-
clusion is that there is no freedom to choose these variables,
so F = 0. ing the temperature, the pressure could be selected, but having
Four phases cannot be in mutual equilibrium in a one- done so the two phases would be in equilibrium only at a single
component system because the three equalities definite temperature. Therefore, freezing (or any other phase
transition) occurs at a definite temperature at a given pressure.
 (; p,T )   (; p,T ),  (; p,T )   ( ; p,T ),
When three phases are in equilibrium, F = 0 and the system
and  (  ; p,T )   (; p,T ) is invariant, meaning that it has no degrees of freedom. This
are three equations with only two unknowns (p and T), which special condition can be established only at a definite tempera-
are not consistent because no values of p and T satisfy all three ture and pressure that is characteristic of the substance and
equations (just as the three equations x + y = xy, 3x − y = xy, cannot be changed. The equilibrium of three phases is therefore
and 4x − y = 2xy2 have no solution). represented by a point, the triple point, on a phase diagram.
In summary, for a one-component system (C = 1) it has been
shown that: F = 2 when P = 1; F = 1 when P = 2; and F = 0
when P = 3. The general result is that for C = 1, F = 3 − P.
Phase  Phase 
Step 2 Consider the general case of any number of components, C
Begin by counting the total number of intensive variables. The
Pressure, p

Phase 
pressure, p, and temperature, T, count as 2. The composition F = 2, Phase 
of a phase is specified by giving the mole fractions of the C P=1 P = 4,
components, but as the sum of the mole fractions must be 1, forbidden
F = 0,
only C − 1 mole fractions are independent. Because there are P P=3
F = 1,
phases, the total number of composition variables is P(C − 1). At P=2
Phase 
this stage, the total number of intensive variables is P(C − 1) + 2.
At equilibrium, the chemical potential of a component J is Temperature, T
the same in every phase:
Figure 4A.7 The typical regions of a one-component phase
J (; p,T )  J (; p,T )   for P phases diagram. The lines represent conditions under which the two
adjoining phases are in equilibrium. A point represents the unique
There are P − 1 equations of this kind to be satisfied for each set of conditions under which three phases coexist in equilibrium.
component J. As there are C components, the total number of Four phases cannot mutually coexist in equilibrium when only
equations is C(P − 1). Each equation reduces the freedom to one component is present.
4A Phase diagrams of pure substances 125

Four phases cannot be in equilibrium in a one-component sys- Critical


tem because F cannot be negative. point
72.9
Solid C
Liquid

Pressure, p/atm
Exercises 67
E4A.2 How many phases are present at each of the points a–d indicated in the B
5.11 D
phase diagram below?
Triple
point Vapour
1

298.15
A

304.2
216.8
194.7
Tb T3 Tc

c Temperature, T/K
Pressure

a
Figure 4A.8 The experimental phase diagram for carbon dioxide;
b note the break in the vertical scale. As the triple point lies at
pressures well above atmospheric, liquid carbon dioxide does not
d exist under normal conditions; a pressure of at least 5.11 atm must
be applied for liquid to be formed. The path ABCD is discussed in
Brief illustration 4A.6.
Temperature

E4A.3 What is the maximum number of phases that can be in mutual


equilibrium in a two-component system?
E4A.4 In a one-component system, is the condition P = 1 represented on a phase
Brief illustration 4A.6
diagram by an area, a line, or a point? How do you interpret this value of P?
Consider the path ABCD in Fig. 4A.8. At A the carbon dioxide
is a gas. When the temperature and pressure are adjusted to B,
the vapour condenses directly to a solid. Increasing the pres-
Three representative phase
4A.3 sure and temperature to C results in the formation of the liquid
diagrams phase, which evaporates to the vapour when the conditions are
changed to D.
Carbon dioxide, water, and helium illustrate the significance of
the various features of a phase diagram.
4A.3(b) Water
Figure 4A.9 shows the phase diagram for water. The liquid–­
4A.3(a) Carbon dioxide vapour coexistence curve in the phase diagram summa-
Figure 4A.8 shows the phase diagram for carbon dioxide. The rizes how the vapour pressure of liquid water varies with
features to notice include the positive slope (up from left to ­temperature. It also summarizes how the boiling tempera-
right) of the solid–liquid coexistence curve; the direction of ture varies with pressure: simply read off the temperature at
this line is characteristic of most substances. This slope indi- which the vapour pressure is equal to the prevailing atmos-
cates that the melting temperature of solid carbon dioxide rises pheric pressure. The solid (ice I)–liquid coexistence curve
as the pressure is increased. Notice also that, as the triple point shows how the melting temperature varies with the pres-
lies above 1 atm, the liquid cannot exist at normal atmospheric sure. Its very steep slope indicates that enormous pressures
pressures whatever the temperature. As a result, the solid are needed to bring about significant changes. Notice that
sublimes when left in the open (hence the name ‘dry ice’). To the line has a negative slope (down from left to right) up to
obtain the liquid, it is necessary to exert a pressure of at least 2 kbar, which means that the melting temperature falls as the
5.11 atm. Cylinders of carbon dioxide generally contain the pressure is raised.
liquid or compressed gas; at 25 °C that implies a vapour pres- The reason for this almost unique behaviour can be traced
sure of 67 atm if both gas and liquid are present in equilibrium. to the decrease in volume that occurs on melting: it is more
When the gas is released through a tap (which acts as a throttle) ­favourable for the solid to transform into the liquid as the
the gas cools by the Joule–Thomson effect, so when it emerges pressure is raised. The decrease in volume is a result of the very
into a region where the pressure is only 1 atm, it condenses into open structure of ice: as shown in Fig. 4A.10, the water mole-
a finely divided snow-like solid. That carbon dioxide gas cannot cules are held apart, as well as together, by the hydrogen bonds
be liquefied except by applying high pressure reflects the weak- between them. This hydrogen-bonded structure partially
ness of the intermolecular forces between the nonpolar carbon collapses on melting and the liquid is denser than the solid.
dioxide molecules (Topic 14B). Other consequences of its extensive hydrogen bonding are the
126 4 Physical transformations of pure substances

1012
X XI 100
VII X
VIII B VI C 50
10 9
V VII
II B C
A III D Liquid
Pressure, p/Pa

VIII

Pressure, p/GPa
Liquid 5
106 Vapour
I
VI
XV XIV XII Liquid
103 Solid 1

IV II D
V
1 XIII V IX A III
0
0 200 400 600 800 0 200 400 600
Temperature, T/K Temperature, T/K

Figure 4A.9 The phase diagram for water showing the different solid phases, which are indicated with Roman numerals I, II, …; solid
phase I (ice I) is ordinary ice. The path ABCD is discussed in Brief illustration 4A.7.

4A.3(c) Helium
The two isotopes of helium, 3He and 4He, behave differently
at low temperatures because 4He is a boson whereas 3He is a
fermion, and are treated differently by the Pauli principle
(Topic 8B). Figure 4A.11 shows the phase diagram of helium-4.
Helium behaves unusually at low temperatures because the
mass of its atoms is so low and there are only very weak interac-
tions between neighbours. At 1 atm, the solid and gas phases of
Figure 4A.10 A fragment of the structure of ice I. Each O atom is helium are never in equilibrium however low the temperature:
linked by two covalent bonds to H atoms and by two hydrogen the atoms are so light that they vibrate with a large-amplitude
bonds to a neighbouring O atom, in a tetrahedral array. motion even at very low temperatures and the solid simply
shakes itself apart. Solid helium can be obtained, but only by
a­ nomalously high boiling point of water for a molecule of its
holding the atoms together by applying pressure.
molar mass, and its high critical temperature and pressure.
Pure helium-4 has two liquid phases. The phase marked He-I
The diagram shows that water has one liquid phase but many
in the diagram behaves like a normal liquid; the other phase,
different solid phases other than ordinary ice (‘ice I’). Some of
these phases melt at high temperatures. Ice VII, for instance,
100
melts at 100 °C but exists only above 25 kbar. Note that several Solid hcp
bcc
more triple points occur in the diagram other than the one Liquid
where vapour, liquid, and ice I coexist. Each one occurs at a 10
He-I
definite pressure and temperature that cannot be changed. The Critical
Pressure, p/bar

-line point
solid phases of ice differ in the arrangement of the water mol- Liquid
1 He-II
ecules: under the influence of very high pressures, hydrogen
(super-
bonds buckle and the H2O molecules adopt different arrange- fluid) B
ments. These polymorphs of ice may contribute to the advance C A
0.1 Gas
of glaciers, for ice at the bottom of glaciers experiences very Triple
D point
high pressures where it rests on jagged rocks.
0.01
0 1 2 3 4 5 6
Temperature, T/K

Brief illustration 4A.7 Figure 4A.11 The phase diagram for 4He. The λ-line marks the
conditions under which the two liquid phases are in equilibrium;
Consider the path ABCD in Fig. 4A.9. Water is present at A He-II is the superfluid phase. Note that a pressure of over 20 bar
as ice V. Increasing the pressure to B at the same temperature must be exerted before solid helium can be obtained. The labels
results in the formation of ice VIII. Heating to C leads to the hcp and bcc denote different solid phases in which the atoms
formation of ice VII, and reduction in pressure to D results in pack together differently: hcp denotes hexagonal close packing
the solid melting to liquid. and bcc denotes body-centred cubic (Topic 15A). The path ABCD
is discussed in Brief illustration 4A.8.
4A Phase diagrams of pure substances 127

40 Helium-3 also has a superfluid phase. Helium-3 is unusual


in that melting is exenthalpic (∆fusH < 0) and therefore (from
∆fusS = ∆fusH/Tf ) at the melting point the entropy of the liquid is
Cp,m/(J K–1 mol–1)

30
lower than that of the solid.
He-II
He-I
20 Brief illustration 4A.8

10 Consider the path ABCD in Fig. 4A.11. At A, helium is pre-


sent as a vapour. On cooling to B it condenses to helium-I,
and further cooling to C results in the formation of helium-II.
00 1 T/K 2 3 Adjustment of the pressure and temperature to D results in a
system in which three phases, helium-I, helium-II, and vapour
Figure 4A.12 The heat capacity of superfluid He-II increases are in mutual equilibrium.
with temperature and rises steeply as the transition temperature
to He-I is approached. The appearance of the plot has led the
transition to be described as a λ-transition and the line on the
phase diagram to be called a λ-line.
Exercises
E4A.5 Refer to Fig. 4A.8. Which phase or phases would you expect to be
present for a sample of CO2 at: (i) 200 K and 2.5 atm; (ii) 300 K and 4 atm;
He-II, is a superfluid. It is so called because it flows without (iii) 310 K and 50 atm?
­viscosity.2 The liquid–liquid coexistence curve is called the
E4A.6 Consider the phase diagram for CO2 shown in Fig. 4A.8. Describe a
λ-line (lambda line) for reasons related to the shape of a plot of path by which a liquid represented by point C can be converted to vapour
the heat capacity of helium-4 against temperature at the transi- represented by point D without going through any intermediate state where
tion temperature (Fig. 4A.12). two phases are in equilibrium.

Checklist of concepts
☐ 1. A phase is a form of matter that is uniform throughout ☐ 4. A phase diagram indicates the values of the pressure
in chemical composition and physical state. and temperature at which a particular phase is most
☐ 2. A phase transition is the spontaneous conversion of one stable or is in equilibrium with other phases.
phase into another. ☐ 5. The phase rule relates the number of variables that
☐ 3. The thermodynamic analysis of phases is based on the may be changed while the phases of a system remain in
fact that at equilibrium, the chemical potential of a sub- mutual equilibrium.
stance is the same throughout a sample.

Checklist of equations
Property Equation Comment Equation number
Chemical potential μ = G/n Single substance
Phase rule F=C−P+2 4A.1

2
Water might also have a superfluid liquid phase.
TOPIC 4B Thermodynamic aspects of
phase transitions

4B.1The dependence of stability on


➤ Why do you need to know this material?
Thermodynamic arguments explain the appearance of
the conditions
phase diagrams and can be used to make predictions
At sufficiently low temperatures the solid phase of a substance
about the effect of pressure on phase transitions. They
commonly has the lowest chemical potential and is therefore
provide insight into the properties that account for the
the most stable phase. However, the chemical potentials of dif-
behaviour of matter under different conditions.
ferent phases depend on temperature to different extents (be-
cause the molar entropy of each phase is different), and above
➤ What is the key idea?
a certain temperature the chemical potential of another phase
The effect of temperature and pressure on the chemical (perhaps another solid phase, a liquid, or a gas) might turn out
potential of a substance in each phase depends on its
to be lower. Then a transition to the second phase becomes
molar entropy and molar volume, respectively.
spontaneous and occurs if it is kinetically feasible.
➤ What do you need to know already?
You need to be aware that phases are in equilibrium when The temperature dependence of
4B.1(a)
their chemical potentials are equal (Topic 4A) and that phase stability
the variation of the molar Gibbs energy of a substance Because Sm > 0 for all substances above T = 0, eqn 4B.1a shows
depends on its molar volume and entropy (Topic 3E). The that the chemical potential of a pure substance decreases as
Topic makes use of expressions for the entropy of transi- the temperature is raised. That is, a plot of chemical potential
tion (Topic 3B) and of the perfect gas law (Topic 1A). against temperature slopes down from left to right. It also im-
plies that because Sm(g) > Sm(l), the slope is steeper for gases
than for liquids. Because it is almost always the case that
Sm(l) > Sm(s), the slope is also steeper for a liquid than for the
As explained in Topic 4A, the thermodynamic criterion for phase corresponding solid. These features are illustrated in Fig. 4B.1.
equilibrium is the equality of the chemical potentials of each sub-
stance in each phase. For a one-component system, the chemical
potential is the same as the molar Gibbs energy (μ = Gm = G/n).
In Topic 3E it is shown that when the pressure and temperature
Chemical potential, μ

are changed, the change in Gibbs energy is dG  Vdp  SdT (this Solid
is eqn 3E.7). Because dG is an exact differential it follows that Liquid
(G /T ) p  S and (G /p)T  V (eqn 3E.8a). These relations Vapour
apply to molar quantities, so dGm  Vm dp  Sm dT and conse-
quently d  Vm dp  Sm dT . It follows that
Variation of chemical Solid Liquid Vapour
   Tf Tb
 T   Sm
stable stable stable
potential with T  (4B.1a)
 p [constant p ] Temperature, T

Variation of chemical Figure 4B.1 The schematic temperature dependence of the


  
   Vm potential with p  (4B.1b) chemical potential of the solid, liquid, and gas phases of a
 p T [constant T ] substance (in practice, the lines are curved). The phase with the
lowest chemical potential at a specified temperature is the most
These expressions, combined with the requirement that the stable one at that temperature. The transition temperatures,
chemical potential of each phase in mutual equilibrium must the freezing (melting) and boiling temperatures (Tf and Tb,
be the same, are used to explore how the equilibrium between respectively), are the temperatures at which the chemical
phases is affected by changes in temperature and pressure. potentials of the two phases are equal.
4B Thermodynamic aspects of phase transitions 129

The steeper slope of μ(l) compared with that of μ(s) results you need to know the molar volumes of the two phases of
in μ(l) falling below μ(s) when the temperature is high enough; water. These values are obtained from the mass density, ρ, and
then the liquid becomes the stable phase, and melting is spon- the molar mass, M, by using Vm = M/ρ. Then Δμ = MΔp/ρ. To
taneous. The chemical potential of the gas phase plunges more keep the units straight, you will need to express the mass densi-
steeply downwards than does that of the liquid as the tempera- ties in kilograms per cubic metre (kg m−3) and the molar mass
ture is raised (because the molar entropy of the vapour is higher in kilograms per mole (kg mol−1), and use 1 Pa m3 = 1 J.
than that of the liquid), and there is a temperature at which it The solution The molar mass of water is 18.02 g mol−1 (i.e. 1.802 ×
lies below that of the liquid. Then the gas is the stable phase and 10−2 kg mol−1); therefore, when the pressure is increased by
vaporization is spontaneous. 1.00 bar (1.00 × 105 Pa)

(1.802  102 kg mol 1 )  (1.00  105 Pa )


 (ice) 
Brief illustration 4B.1 917 kg m 3
 1.97 Jmol 1
The standard molar entropy of liquid water at 100 °C is (1.802  102 kg mol 1 )  (1.00  105 Pa )
86.8 J K−1 mol−1 and that of water vapour at the same tempera-  (water ) 
999 kg m 3
ture is 195.98 J K−1 mol−1. It follows that when the temperature
is raised by 1.0 K the changes in chemical potential are  1.80 Jmol 1

 (l )  Sm (l )T  87 Jmol 1 The chemical potential of ice rises by more than that of water,
 (g )  Sm (g )T  196 Jmol 1 so if they are initially in equilibrium at 1 bar, then there is a
tendency for the ice to melt at 2 bar.
At 100 °C the two phases are in equilibrium with equal chemi-
Self-test 4B.1 Calculate the effect of an increase in pressure
cal potentials. At 101 °C the chemical potential of both vapour
of 1.00 bar on the chemical potentials of the liquid and solid
and liquid are lower than at 100 °C, but the chemical potential
phases of carbon dioxide (molar mass 44.0 g mol−1) with mass
of the vapour has decreased by a greater amount. It follows
densities 2.35 g cm−3 and 2.50 g cm−3, respectively. If the two
that the vapour is the stable phase at the higher temperature,
phases are initially in equilibrium, which phase is predicted to
so vaporization will then be spontaneous.
be the more stable after the pressure has increased by 1.00 bar?
Answer: Δμ(l) = +1.87 J mol−1, Δμ(s) = +1.76 J mol−1; the solid.

The response of melting to applied


4B.1(b)
pressure
High
Equation 4B.1b shows that because Vm > 0, an increase in pres- pressure
sure raises the chemical potential of any pure substance. In
Chemical potential, μ

most cases, Vm(l) > Vm(s), so an increase in pressure increases High Liquid
Liquid
the chemical potential of the liquid phase of a substance more pressure
Solid
than that of its solid phase. As shown in Fig. 4B.2(a), the effect
of pressure in such a case is to raise the freezing temperature
Low Low
slightly. For water, however, Vm(l) < Vm(s), and an increase in pressure pressure
pressure increases the chemical potential of the solid more than Solid
that of the liquid. In this case, the freezing temperature is low-
ered slightly (Fig. 4B.2(b)). Tf Tf’ Tf’ Tf
(a) Temperature, T (b) Temperature, T

Figure 4B.2 The pressure dependence of the chemical potential


Example 4B.1 of a substance depends on the molar volume of the phase. The
Assessing the effect of pressure on the
lines show schematically the effect of increasing pressure on
chemical potential the chemical potential of the solid and liquid phases (in practice,
Calculate the effect on the chemical potentials of ice and water the lines are curved), and the corresponding effects on the
of increasing the pressure from 1.00 bar to 2.00 bar at 0 °C. The freezing temperatures. (a) In this case the molar volume of the
mass density of ice is 0.917 g cm−3 and that of liquid water is solid is smaller than that of the liquid and μ(s) increases less
0.999 g cm−3 under these conditions. than μ(l). As a result, the freezing temperature rises. (b) Here
the molar volume is greater for the solid than the liquid (as for
Collect your thoughts From dμ = Vmdp, you can infer that the water), μ(s) increases more strongly than μ(l), and the freezing
change in chemical potential of an incompressible substance temperature is lowered. Interact with the dynamic version of this
when the pressure is changed by Δp is Δμ = VmΔp. Therefore, graph in the e-book.
130 4 Physical transformations of pure substances

The vapour pressure of a liquid


4B.1(c) Step 3 Set up the integration of this expression by identifying the
appropriate limits
subjected to pressure
When there is no additional pressure acting on the liquid, P
Pressure can be exerted on the condensed phase mechani- (the pressure experienced by the liquid) is equal to the normal
cally or by subjecting it to the applied pressure of an inert gas vapour pressure p*, so when P = p*, p = p* too. When there
(Fig. 4B.3). In the latter case, the partial vapour pressure is is an additional pressure ΔP on the liquid, so P = p + ΔP, the
the partial pressure of the vapour in equilibrium with the con- vapour pressure is p (the value required). Provided the effect of
densed phase. When pressure is applied to a condensed phase, pressure on the vapour pressure is small (as will turn out to be
its vapour pressure rises: in effect, molecules are squeezed out the case) a good approximation is to replace the p in p + ΔP by
of the phase and escape as a gas. The effect can be explored p* itself, and to set the upper limit of the integral to p* + ΔP.
thermodynamically and a relation established between the ap- The integrations required are therefore as follows:
plied pressure P and the vapour pressure p. p dp p   P
RT      Vm (l )dP
p p p

How is that done? 4B.1 Deriving an expression for the


where in the first integral, the variable of integration has
vapour pressure of a pressurized liquid been changed from p to p′ to avoid confusion with the p at the
At equilibrium the chemical potentials of the liquid and its upper limit.
vapour are equal: μ(l) = μ(g). It follows that, for any change Step 4 Carry out the integrations
that preserves equilibrium, the resulting change in μ(l) must be
Divide both sides by RT and assume that the molar volume of the
equal to the change in μ(g). Therefore, dμ(g) = dμ(l).
liquid is the same throughout the range of pressures involved:
Step 1 Express changes in the chemical potentials that arise from Integral A.2 Integral A.1
changes in pressure    
p dp 1 p  P Vm (l ) p  P
When the pressure P on the liquid is increased by dP, the p p  RT p Vm (l)dP  RT p dP
chemical potential of the liquid changes by dμ(l) = Vm(l)dP. The
chemical potential of the vapour changes by dμ(g) = Vm(g)dp, Both integrations are straightforward, and lead to
where dp is the change in the vapour pressure. If the vapour is
p Vm (l )
treated as a perfect gas, the molar volume can be replaced by ln  P
Vm(g) = RT/p, to give dμ(g) = (RT/p)dp. p RT

Step 2 Equate the changes in chemical potentials of the vapour which (by using eln x = x) rearranges to
and the liquid
(4B.2)
Equate dμ(l) = Vm(l)dP and dμ(g) = (RT/p)dp: p = p*eVm (l)∆P/RT
Effect of applied pressure ∆P on vapour
RTdp pressure p
= Vm (l)dP
p One complication that has been ignored is that, if the con-
Be careful to distinguish between P, the applied pressure, and densed phase is a liquid, then the pressurizing gas (if that is the
p, the vapour pressure. source of the additional pressure) might dissolve and change
its properties. Another complication is that the gas-phase mol-
Pressure, ΔP ecules might attract molecules out of the liquid by the process of
gas solvation, the attachment of molecules to gas-phase species.
Vapour Vapour plus
inert pressurizing Brief illustration 4B.2
Piston gas
permeable to For water, which has mass density 0.997 g cm−3 at 25 °C and
vapour but therefore molar volume 18.1 cm3 mol−1, when the applied pres-
not liquid
sure is increased by 10 bar (i.e. ΔP = 1.0 × 106 Pa)
5 3 1 6
p Vm (1)P (1.81  10 m mol )  (1.0  10 Pa )
(a) (b) ln   1 1
p 
RT (8.3145 JK mol )  (298 K )
 0.0073
Figure 4B.3 Pressure may be applied to a condensed phase
either (a) by compressing it or (b) by subjecting it to an inert where 1 J = 1 Pa m3. It follows that p = 1.0073p*; the vapour
pressurizing gas. When pressure is applied, the vapour pressure pressure increases, but by only 0.73 per cent.
of the condensed phase increases.
4B Thermodynamic aspects of phase transitions 131

Exercises Phase α
E4B.1 The standard molar entropy of liquid water at 273.15 K is b
65 J K−1 mol−1, and that of ice at the same temperature is 43 J K−1 mol−1.

Pressure, p
Calculate the change in chemical potential of liquid water and of ice when
the temperature is increased by 1 K from the normal melting point. Giving dp
a
your reasons, explain which phase is thermodynamically the more stable at
Phase β
the new temperature.
E4B.2 Water is heated from 25 °C to 35 °C. By how much does its chemical dT
potential change? The standard molar entropy of liquid water at 298 K is
69.9 J K−1 mol−1. Temperature, T
E4B.3 By how much does the chemical potential of copper change when the
pressure exerted on a sample is increased from 100 kPa to 10 MPa? Take the
Figure 4B.4 Two phases are in equilibrium at point a and at point
mass density of copper to be 8960 kg m−3. b. To go from a to b there must be a relation between the change
in the pressure dp and the change in the temperature dT. That
relation leads to an expression for the coexistence curves in the
phase diagram.
The location of coexistence
4B.2
curves the change in (molar) volume, the volume of transition, is
ΔtrsV = Vm(β) − Vm(α). Therefore,
The precise locations of the coexistence curves—the pres-
sures and temperatures at which two phases can coexist—can  trs SdT   trsVdp
be found by making use once again of the fact that, when two This relation rearranges into the Clapeyron equation:
phases are in equilibrium, their chemical potentials must be
equal. Therefore, when the phases α and β are in equilibrium, dp  trs S
 Clapeyron equation  (4B.4a)
dT  trsV
 (; p,T )   (; p,T ) (4B.3)

Solution of this equation for p in terms of T gives an equation The Clapeyron equation is an exact expression for the slope of
for the coexistence curve. the tangent to the coexistence curve at any point and applies
to any phase equilibrium of any pure substance. It implies that
thermodynamic data can be used to predict the appearance of
4B.2(a) The slopes of the coexistence curves phase diagrams and to understand their form. A more practi-
Imagine that at some particular pressure and temperature the cal application is to the prediction of the response of freezing
two phases are in equilibrium: their chemical potentials are and boiling points to the application of pressure, when it can be
then equal. Now p and T are changed infinitesimally, but in used in the form obtained by inverting both sides:
such a way that the phases remain in equilibrium: after these dT  trsV
  (4B.4b)
changes, the chemical potentials of the two phases change dp  trs S
but remain equal (Fig. 4B.4). It follows that the change in the
chemical potential of phase α must be the same as the change in
chemical potential of phase β, so dμ(α) = dμ(β). Brief illustration 4B.3
The fundamental equation of chemical thermodynamics,
eqn 3E.7 (dG = Vdp − SdT) gives the variation of G with p and For water at 0 °C, the standard volume of transition of ice to
T. Therefore with μ = Gm, it follows that dμ = Vmdp − SmdT for liquid is −1.6 cm3 mol−1, and the corresponding standard entro-
each phase. Therefore the relation dμ(α) = dμ(β) can be written py of transition is +22 J K−1 mol−1. The slope of the solid–liquid
coexistence curve at that temperature is therefore
Vm ()dp  Sm ()dT  Vm ()dp  Sm ()dT
6 3 1
dT 1.6  10 m mol K
  7.3  108
where Sm(α) and Sm(β) are the molar entropies of the two dp 1
22 JK mol 1
Jm 3
phases, and Vm(α) and Vm(β) are their molar volumes. Hence 8
 7.3  10 K Pa 1

{Sm ()  Sm ()}dT  {Vm ()  Vm ()}dp which corresponds to −7.3 mK bar−1. An increase of 100 bar
therefore results in a lowering of the freezing point of water
The change in (molar) entropy accompanying the phase tran-
by 0.73 K.
sition, the entropy of transition, is ΔtrsS = Sm(β) − Sm(α);
132 4 Physical transformations of pure substances

4B.2(b) The solid–liquid coexistence curve When T is close to T*, the logarithm can be approximated by
using the expansion ln(1  x )  x  12 x 2   (see Part 1 of the
Melting (fusion) is accompanied by a molar enthalpy change Resource section) and neglecting all but the leading term:
ΔfusH, and if it occurs at a temperature T the molar entropy
of melting is ΔfusH/T (Topic 3B); all points on the coexistence T  T T   T T 
 ln  1 
ln 
curve correspond to equilibrium, so T is in fact a transition T 
 T  T
temperature, Ttrs. The Clapeyron equation for this phase transi-
Therefore
tion then becomes
 fus H
p  p  (T  T  ) (4B.7)
dp  fus H T   fusV
 Slope of solid-liquid coexistence curve (4B.5)
dT T  fusV This expression is the equation of a steep straight line when p is
plotted against T (as in Fig. 4B.5).
where ΔfusV is the change in molar volume that accompanies
melting. The enthalpy of melting is positive (the only exception
Brief illustration 4B.4
is helium-3); the change in molar volume is usually positive
and always small. Consequently, the slope dp/dT is steep and The enthalpy of fusion of ice at 0 °C (273 K) and 1 bar is
usually positive (Fig. 4B.5). 6.008 kJ mol−1 and the volume of fusion is −1.6 cm3 mol−1. It
The equation for the coexistence curve is found by inte- follows that the solid–liquid phase boundary is given by the
grating dp/dT and assuming that ΔfusH and ΔfusV change so equation
little with temperature and pressure that they can be treated
6.008  103 Jmol 1
as constant. If the melting temperature is T* when the pres- p  (1.0  105 Pa )  (T  T  )
(273 K )  (1.6  106 m 3 mol 1 )
sure is p*, and T when the pressure is p, the integration re-
quired is  (1.0  105 Pa )  (1.4  107 Pa K 1 )  (T  T  )

Integral A.2
That is,

p  fus H T dT p/bar  1  140(T  T  )/K
p dp   fusV T  T with T* = 273 K. This expression is plotted in Fig. 4B.6.

Therefore, the approximate equation of the solid–liquid coex-


istence curve is 4B.2(c) The liquid–vapour coexistence curve
The entropy of vaporization at a temperature T is equal to
 H T ΔvapH/T (as before, all points on the coexistence curve cor-
p  p  fus ln  

(4B.6)
 fusV T respond to equilibrium, so T is a transition temperature, Ttrs),

This equation was originally obtained by yet another


Thomson—James, the brother of William, Lord Kelvin.
15

Water
Pressure, p/bar

10
Solid
Pressure, p

5
Liquid Benzene

1
–0.1 –0.05 0 0.05 0.1
Temperature, T Temperature difference, (T – T *)/K

Figure 4B.5 A typical solid–liquid coexistence curve slopes Figure 4B.6 The solid–liquid coexistence curve (the melting
steeply upwards. This slope implies that, as the pressure is raised, point curve) for water as calculated in Brief illustration 4B.4. For
the melting temperature rises. Most substances behave in this comparison, the curve for benzene is included. It has the opposite
way, water being the notable exception. slope because the volume of fusion of benzene is positive.
4B Thermodynamic aspects of phase transitions 133

so the Clapeyron equation for the liquid–vapour coexistence By using dx/x = d ln x, the terms in boxes simplify to d ln p. The
curve can therefore be written resulting expression is the Clausius–Clapeyron equation for
the variation of vapour pressure with temperature:
dp  vap H Slope of liquid-vapour
  (4B.8)
dT T  vapV coexistence curve d ln p  vap H
 Clausius-Clapeyron equation (4B.9)
The enthalpy of vaporization is positive and ΔvapV is large and dT RT 2
positive, so dp/dT is positive, but much smaller than for the Like the Clapeyron equation (which is exact), the Clausius–
solid–liquid coexistence curve. Consequently dT/dp is large, Clapeyron equation (which is an approximation) is important
and the boiling temperature is more responsive to pressure for understanding the appearance of phase diagrams, particu-
than the freezing temperature. larly the location and shape of the liquid–vapour and solid–
vapour coexistence curves. It can be used to predict how the
vapour pressure varies with temperature and how the boiling
Example 4B.2 Estimating the effect of pressure on the temperature varies with pressure. For instance, if it is also as-
boiling temperature sumed that the enthalpy of vaporization is independent of tem-
perature, eqn 4B.9 can be integrated as follows:
Estimate the typical size of the effect of increasing pressure on
the boiling point of a liquid. Integral A.1,
Integral A.1
with x  ln p 


Collect your thoughts To use eqn 4B.8 you need to estimate ln p  vap H T dT

the right-hand side. At the boiling point, the term ΔvapH/T is ln p 
d ln p 
R T  T 2
Trouton’s constant (Topic 3B). Because the molar volume of a
gas is so much greater than the molar volume of a liquid, you hence
can write ΔvapV = Vm(g) − Vm(l) ≈ Vm(g) and take for Vm(g) the
p  vap H  1 1 
molar volume of a perfect gas (at low pressures, at least). You ln  
will need to use 1 J = 1 Pa m3. p 
R  T T  
The solution Trouton’s constant has the value 85 J K−1 mol−1. The where p* is the vapour pressure when the temperature is T*, and
molar volume of a perfect gas is about 25 dm3 mol−1 at 1 atm p the vapour pressure when the temperature is T. It follows that
and near but above room temperature. Therefore,
 vap H  1 1 
dp 85 JK 1 mol 1 p  p e     (4B.10)
  3.4  103 Pa K 1 R  T T  
dT 2.5  102 m3 mol 1
Equation 4B.10 is plotted as the liquid–vapour coexistence
This value corresponds to 0.034 atm K−1 and hence to dT/dp = curve in Fig. 4B.7. The line does not extend beyond the critical
29 K atm−1. Therefore, a change of pressure of +0.1 atm can be temperature, Tc, because above this temperature the liquid does
expected to change a boiling temperature by about +3 K. not exist.
Self-test 4B.2 Estimate dT/dp for water at its normal boiling
point using the information in Table 3B.2 and Vm(g) = RT/p.
Answer: 28 K atm−1 Brief illustration 4B.5

Equation 4B.10 can be used to estimate the vapour pressure of a


liquid at any temperature from knowledge of its normal boiling
Because the molar volume of a gas is so much greater than point, the temperature at which the vapour pressure is 1.00 atm
the molar volume of a liquid, ΔvapV ≈ Vm(g) (as in Example (101 kPa). The normal boiling point of benzene is 80 °C (353 K)
4B.2). Moreover, if the gas behaves perfectly, Vm(g) = RT/p. and (from Table 3B.2) ∆vapH = 30.8 kJ mol−1. Therefore, to cal-

These two approximations turn the exact Clapeyron equa- culate the vapour pressure at 20 °C (293 K), write
tion into
3.08  104 Jmol 1  1 1 
dp  vap H p vap H  1 
 
  8.3145 JK mol  293 K 353 K 
1

dT T (RT /p) RT 2  2.14 

Division of both sides by p gives Because exponential functions are so sensitive, it is best
to carry out numerical calculations like this without using
dp/p = dln p rounded values for results of intermediate steps. Then substitu-
tion of this value of χ into eqn 4B.10 with p* = 101 kPa gives
1 dp ∆ H p = 12 kPa. The experimental value is 10 kPa.
= vap 2
p dT RT
134 4 Physical transformations of pure substances

Liquid

Solid

Pressure, p
Pressure, p

Liquid

Vapour
Vapour

Temperature, T Temperature, T

Figure 4B.7 A typical liquid–vapour coexistence curve. The curve Figure 4B.8 At temperatures close to the triple point the solid–
can be interpreted as a plot of the vapour pressure against the vapour coexistence curve is steeper than the liquid–vapour
temperature. This coexistence curve terminates at the critical coexistence curve because the enthalpy of sublimation is greater
point (not shown). than the enthalpy of vaporization.

4B.2(d) The solid–vapour coexistence curve d ln p 45.0  103 Jmol 1


(a )   0.0726 K 1
The only difference between expressions for the solid−vapour dT (8.3145 JK 1 mol 1 )  (273 K)2
and the liquid−vapour coexistence curves is the replacement d ln p 51.0  103 Jmol 1
(b)   0.0823 K 1
of the enthalpy of vaporization by the enthalpy of sublima- dT (8.3145 JK 1 mol 1 )  (273 K)2
tion, ΔsubH. Because the enthalpy of sublimation is greater than
the enthalpy of vaporization (recall that ΔsubH = ΔfusH + The slope of ln p plotted against T is greater for the solid–
ΔvapH), at similar temperatures the equation predicts a steeper vapour coexistence curve than for the liquid–vapour coexist-
slope for the sublimation curve than for the vaporization ence curve at the triple point.
curve. These two coexistence curves meet at the triple point
(Fig. 4B.8).
Exercises
3 −1
E4B.4 The molar volume of a certain solid is 161.0 cm mol at 1.00 atm and
Brief illustration 4B.6
350.75 K, its melting temperature. The molar volume of the liquid at this
temperature and pressure is 163.3 cm3 mol−1. At 100 atm the melting temperature
The enthalpy of fusion of ice at the triple point of water (6.1 mbar, changes to 351.26 K. Calculate the enthalpy and entropy of fusion of the solid.
273 K) is approximately the same as its standard enthalpy of
E4B.5 The vapour pressure of benzene between 10 °C and 30 °C fits the
fusion at its freezing point, which is 6.008 kJ mol−1. The enthalpy expression log(p/Torr) = 7.960 − (1780 K)/T. Calculate (i) the enthalpy of
of vaporization at that temperature is 45.0 kJ mol−1 (once again, vaporization and (ii) the normal boiling point of benzene.
ignoring differences due to the pressure not being 1 bar). The
E4B.6 Naphthalene, C10H8, melts at 80.2 °C. If the vapour pressure of the liquid
enthalpy of sublimation is therefore 51.0 kJ mol−1. Therefore, is 1.3 kPa at 85.8 °C and 5.3 kPa at 119.3 °C, use the Clausius–Clapeyron
the equations for the slopes of (a) the liquid–vapour and (b) the equation to calculate (i) the enthalpy of vaporization, (ii) the normal boiling
solid–vapour coexistence curves at the triple point are point, and (iii) the entropy of vaporization at the boiling point.

Checklist of concepts
☐ 1. The chemical potential of a substance decreases with ☐ 4. The Clapeyron equation is an exact expression for the
increasing temperature in proportion to its molar entropy. slope of a coexistence curve.
☐ 2. The chemical potential of a substance increases with ☐ 5. The Clausius–Clapeyron equation is an approximate
increasing pressure in proportion to its molar volume. expression for the coexistence curve of a condensed
☐ 3. The vapour pressure of a condensed phase increases phase and its vapour.
when pressure is applied.
4B Thermodynamic aspects of phase transitions 135

Checklist of equations
Property Equation Comment Equation number
Variation of chemical potential with temperature (∂μ/∂T)p = −Sm 4B.1a
Variation of chemical potential with pressure (∂μ/∂p)T = Vm 4B.1b
 Vm ( l )P /RT
Vapour pressure in the presence of applied pressure p p e ∆P = P − p* 4B.2
Clapeyron equation dp/dT = ∆trsS/∆trsV 4B.4a
Clausius–Clapeyron equation d ln p/dT = ∆vapH/RT2 4B.9
136 4 Physical transformations of pure substances

FOCUS 4 Physical transformations of pure


substances

To test your understanding of this material, work through Selected solutions can be found at the end of this Focus in
the Exercises, Additional exercises, Discussion questions, and the e-book. Solutions to even-numbered questions are available
Problems found throughout this Focus. online only to lecturers.

TOPIC 4A Phase diagrams of pure substances


Discussion questions
D4A.1 Describe how the concept of chemical potential unifies the discussion D4A.3 Explain why four phases cannot be in equilibrium in a one-component
of phase equilibria. system.
D4A.2 Why does the chemical potential change with pressure even if the D4A.4 Discuss what would be observed as a sample of water is taken along a
system is incompressible (i.e. remains at the same volume when pressure is path that encircles and is close to its critical point.
applied)?

Additional exercises
E4A.7 How many phases are present at each of the points a–d indicated in the E4A.8 The difference in chemical potential of a particular substance between
following illustration? two regions of a system is −8.3 kJ mol−1. By how much does the Gibbs energy
change when 0.15 mmol of that substance is transferred from one region to
the other?

b E4A.9 What is the maximum number of phases that can be in mutual


equilibrium in a four-component system?
c E4A.10 Refer to Fig. 4A.9. Which phase or phases would you expect to be
Pressure

a present for a sample of H2O at: (i) 100 K and 1 atm; (ii) 300 K and 10 atm;
(iii) 273.16 K and 611 Pa?
d

Temperature

Problems
P4A.1 Refer to Fig. 4A.8. Describe the phase or phases present as a sample of only phase β is present; at high T and low p, only phase α is present; at high T
CO2 is heated steadily from 100 K: (a) at a constant pressure of 1 atm; (b) at a and high p, only phase δ is present; phases γ and δ are never in equilibrium.
constant pressure of 70 atm. Comment on any special features of your diagram.
P4A.2 Refer to Fig. 4A.8. Describe the phase or phases present as the pressure P4A.4 For a one-component system draw a schematic labelled phase diagram
on a sample of CO2 is steadily increased from 0.1 atm: (a) at a constant given that at low T and low p, phases α and β are in equilibrium; as the
temperature of 200 K; (b) at a constant temperature of 310 K; (c) at a constant temperature and pressure rise there comes a point at which phases α, β, and
temperature of 216.8 K. γ are all in equilibrium; at high T and high p, only phase γ is present; at low T
and high p, only phase α is present. Comment on any special features of your
P4A.3 For a one-component system draw a schematic labelled phase diagram
diagram.
given that at low T and low p, only phase γ is present; at low T and high p,
Exercises and problems 137

TOPIC 4B Thermodynamic aspects of phase transitions


Discussion questions
D4B.1 What is the physical reason for the decrease of the chemical potential of D4B.3 How may differential scanning calorimetry (DSC) be used to identify
a pure substance as the temperatures is raised? phase transitions?
D4B.2 What is the physical reason for the increase of the chemical potential of
a pure substance as the pressure is raised?

Additional exercises
−1
E4B.7 Repeat the calculation in Exercise E4B.1 but for a decrease in E4B.19 When a certain liquid (with M = 46.1 g mol ) freezes at 1 bar and
temperature by 1.5 K. Giving your reasons, explain which phase is at −3.65 °C its mass density changes from 0.789 g cm−3 to 0.801 g cm−3. Its
thermodynamically the more stable at the new temperature. enthalpy of fusion is 8.68 kJ mol−1. Estimate the freezing point of the liquid at
100 MPa.
E4B.8 Iron is heated from 100 °C to 150 °C. By how much does its chemical
potential change? Take Sm⦵  53 J K 1 mol 1 for the entire range. E4B.20 Estimate the difference between the normal and standard melting
points of ice. At the normal melting point, the enthalpy of fusion of water is
E4B.9 By how much does the chemical potential of benzene change when the
6.008 kJ mol−1, and the change in molar volume on fusion is −1.6 cm3 mol−1.
pressure exerted on a sample is increased from 100 kPa to 10 MPa? Take the
mass density of benzene to be 0.8765 g cm−3. E4B.21 Estimate the difference between the normal and standard boiling
points of water. At the normal boiling point the enthalpy of vaporization of
E4B.10 Pressure was exerted with a piston on water at 20 °C. The vapour
water is 40.7 kJ mol−1.
pressure of water when the applied pressure is 1.0 bar is 2.34 kPa. What is its
vapour pressure when the pressure on the liquid is 20 MPa? The molar volume E4B.22 In July in Los Angeles, the incident sunlight at ground level has a
of water is 18.1 cm3 mol−1 at 20 °C. power density of 1.2 kW m−2 at noon. A swimming pool of area 50 m2 is
directly exposed to the Sun. What is the maximum rate of loss of water?
E4B.11 Pressure was exerted with a piston on molten naphthalene at
Assume that all the radiant energy is absorbed; take the enthalpy of
95 °C. The vapour pressure of naphthalene when the applied pressure is
vaporization of water to be 44 kJ mol−1.
1.0 bar is 2.0 kPa. What is its vapour pressure when the pressure on the
liquid is 15 MPa? The mass density of naphthalene at this temperature is E4B.23 Suppose the incident sunlight at ground level has a power density of
1.16 g cm−3. 0.87 kW m−2 at noon. What is the maximum rate of loss of water from a lake
3 −1 of area 1.0 ha? (1 ha = 104 m2.) Assume that all the radiant energy is absorbed;
E4B.12 The molar volume of a certain solid is 142.0 cm mol at 1.00 atm
take the enthalpy of vaporization of water to be 44 kJ mol−1.
and 427.15 K, its melting temperature. The molar volume of the liquid at
this temperature and pressure is 152.6 cm3 mol−1. At 1.2 MPa the melting E4B.24 An open vessel containing water stands in a laboratory measuring
temperature changes to 429.26 K. Calculate the enthalpy and entropy of fusion 5.0 m × 5.0 m × 3.0 m at 25 °C; the vapour pressure of water at this
of the solid. temperature is 3.2 kPa. When the system has come to equilibrium, what
mass of water will be found in the air if there is no ventilation? Repeat the
E4B.13 The vapour pressure of dichloromethane at 24.1 °C is 53.3 kPa and its
calculation for open vessels containing benzene (vapour pressure 13.1 kPa)
enthalpy of vaporization is 28.7 kJ mol−1. Estimate the temperature at which its
and mercury (vapour pressure 0.23 Pa).
vapour pressure is 70.0 kPa.
E4B.25 On a cold, dry morning after a frost, the temperature was −5 °C and
E4B.14 The vapour pressure of a substance at 20.0 °C is 58.0 kPa and its
−1 the partial pressure of water in the atmosphere fell to 0.30 kPa. Will the frost
enthalpy of vaporization is 32.7 kJ mol . Estimate the temperature at which its
sublime? The enthalpy of sublimation of water is 51 kJ mol−1. (Hint: Use eqn
vapour pressure is 66.0 kPa.
4B.10 to calculate the vapour pressure expected for ice at this temperature; for
E4B.15 The vapour pressure of a liquid in the temperature range 200–260 K p* and T* use the values for the triple point of 611 Pa and 273.16 K.)
was found to fit the expression ln(p/Torr) = 16.255 − (2501.8 K)/T. What is
E4B.26 The normal boiling point of hexane is 69.0 °C. Estimate (i) its enthalpy
the enthalpy of vaporization of the liquid?
of vaporization and (ii) its vapour pressure at 25 °C and at 60 °C. (Hint: You
E4B.16 The vapour pressure of a liquid in the temperature range 200–260 K will need to use Trouton’s rule.)
was found to fit the expression ln(p/Torr) = 18.361 − (3036.8 K)/T. What is
E4B.27 Estimate the melting point of ice under a pressure of 50 bar. Assume
the enthalpy of vaporization of the liquid?
that the mass density of ice under these conditions is approximately
E4B.17 The vapour pressure of a liquid between 15 °C and 35 °C fits the 0.92 g cm−3 and that of liquid water is 1.00 g cm−3. The enthalpy of fusion of
expression log(p/Torr) = 8.750 − (1625 K)/T. Calculate (i) the enthalpy of water is 6.008 kJ mol−1 at the normal melting point.
vaporization and (ii) the normal boiling point of the liquid.
E4B.28 Estimate the melting point of ice under a pressure of 10 MPa.
E4B.18 When benzene freezes at 1 atm and at 5.5 °C its mass density changes Assume that the mass density of ice under these conditions is approximately
from 0.879 g cm−3 to 0.891 g cm−3. The enthalpy of fusion is 10.59 kJ mol−1. 0.915 g cm−3 and that of liquid water is 0.998 g cm−3. The enthalpy of fusion of
Estimate the freezing point of benzene at 1000 atm. water is 6.008 kJ mol−1 at the normal melting point.
138 4 Physical transformations of pure substances

Problems
P4B.1 Imagine the vaporization of 1 mol H2O(l) at the normal boiling point θ/ºC 57.4 100.4 133.0 157.3 203.5 227.5
and against 1 atm external pressure. Calculate the work done by the water
vapour and hence what fraction of the enthalpy of vaporization is spent on p/Torr 1.00 10.0 40.0 100 400 760
expanding the vapour. The enthalpy of vaporization of water is 40.7 kJ mol−1 at
the normal boiling point. Determine (a) the normal boiling point and (b) the enthalpy of vaporization
of carvone.
P4B.2 The temperature dependence of the vapour pressure of solid sulfur

dioxide can be approximately represented by the relation log(p/Torr) = P4B.11 (a) Starting from the Clapeyron equation, derive an expression,
10.5916 − (1871.2 K)/T and that of liquid sulfur dioxide by log(p/Torr) = analogous to the Clausius–Clapeyron equation, for the temperature variation
8.3186 − (1425.7 K)/T. Estimate the temperature and pressure of the triple of the vapour pressure of a solid. Assume that the vapour is a perfect gas and
point of sulfur dioxide. that the molar volume of the solid is negligible in comparison to that of the
gas. (b) In a study of the vapour pressure of chloromethane, A. Bah and
P4B.3 Prior to the discovery that freon-12 (CF2Cl2) is harmful to the Earth’s
N. Dupont-Pavlovsky (J. Chem. Eng. Data 40, 869 (1995)) presented data for
ozone layer it was frequently used as the dispersing agent in spray cans for
the vapour pressure over solid chloromethane at low temperatures. Some of
hair spray etc. Estimate the pressure that a can of hair spray using freon-12
that data is as follows:
has to withstand at 40 °C, the temperature of a can that has been standing in
sunlight. The enthalpy of vaporization of freon-12 at its normal boiling point T/K 145.94 147.96 149.93 151.94 153.97 154.94
of −29.2 °C is 20.25 kJ mol−1; assume that this value remains constant over the
temperature range of interest. p/Pa 13.07 18.49 25.99 36.76 50.86 59.56

P4B.4 The enthalpy of vaporization of a certain liquid is found to be


Estimate the standard enthalpy of sublimation of chloromethane at 150 K.
14.4 kJ mol−1 at 180 K, its normal boiling point. The molar volumes of
the liquid and the vapour at the boiling point are 115 cm3 mol−1 and P4B.12 The change in enthalpy dH resulting from a change in pressure dp and
14.5 dm3 mol−1, respectively. (a) Use the Clapeyron equation to estimate temperature dT is given by dH = CpdT + Vdp. The Clapeyron equation relates
dp/dT at the normal boiling point. (b) If the Clausius–Clapeyron equation is dp and dT at equilibrium, and so in combination the two equations can be used
used instead to estimate dp/dT, what is the percentage error in the resulting to find how the enthalpy changes along a coexistence curve as the temperature
value of dp/dT? changes and the two phases remain in equilibrium. (a) Show that along such a
curve dΔtrsH = ΔtrsCpdT + (ΔtrsH/T)dT, where ΔtrsH is the enthalpy of transition
P4B.5 Calculate the difference in slope of the chemical potential against
and ΔtrsCp is the difference in molar heat capacity accompanying the transition.
temperature on either side of (a) the normal freezing point of water
(b) Show that this expression can also be written d(ΔtrsH/T) = ΔtrsCpd ln T.
and (b) the normal boiling point of water. The molar entropy change
(Hint: The last part is most easily approached by starting with the second
accompanying fusion is 22.0 J K−1 mol−1 and that accompanying evaporation
expression and showing that it can be rewritten as the first.)
is 109.9 J K−1 mol−1. (c) By how much does the chemical potential of water
supercooled to −5.0 °C exceed that of ice at that temperature? P4B.13 In the ‘gas saturation method’ for the measurement of vapour pressure,
a volume V of gas at temperature T and pressure P is bubbled slowly through
P4B.6 Calculate the difference in slope of the chemical potential against
the liquid that is maintained at the same temperature T. The mass m lost
pressure on either side of (a) the normal freezing point of water and (b) the
from the liquid is measured and this can be related to the vapour pressure
normal boiling point of water. The mass densities of ice and water at 0 °C are
in the following way. (a) If the molar mass of the liquid is M, derive an
0.917 g cm−3 and 1.000 g cm−3, and those of water and water vapour at 100 °C are
expression for the mole fraction of the liquid vapour. (Hint: If it is assumed
0.958 g cm−3 and 0.598 g dm−3, respectively. (c) By how much does the chemical
to be a perfect gas, the amount in moles of the input gas can be found from
potential of water vapour exceed that of liquid water at 1.2 atm and 100 °C?
its pressure, temperature, and volume.) (b) Hence derive an expression for
−1
P4B.7 The enthalpy of fusion of mercury is 2.292 kJ mol at its normal freezing the partial pressure of the liquid vapour, assuming that the gas remains at the
point of 234.3 K; the change in molar volume on melting is +0.517 cm3 mol−1. total pressure P after it has passed through the liquid. (c) Then show that the
At what temperature will the bottom of a column of mercury (mass density vapour pressure p is given by p = AmP/(1 + Am), where A = RT/MPV. (d) The
13.6 g cm−3) of height 10.0 m be expected to freeze? The pressure at a depth gas saturation method was used to measure the vapour pressure of geraniol
d in a fluid with mass density ρ is ρgd, where g is the acceleration of free fall, (M = 154.2 g mol−1) at 110 °C. It was found that, when 5.00 dm3 of nitrogen
9.81 m s−2. at 760 Torr was passed slowly through the heated liquid, the loss of mass was
3 0.32 g. Calculate the vapour pressure of geraniol.
P4B.8 Suppose 50.0 dm of dry air at 25 °C was slowly bubbled through
a thermally insulated beaker containing 250 g of water initially at 25 °C. P4B.14 The vapour pressure of a liquid in a gravitational field varies with the
Calculate the final temperature of the liquid. The vapour pressure of water depth below the surface on account of the hydrostatic pressure exerted by
is approximately constant at 3.17 kPa throughout, and the heat capacity of the overlying liquid. The pressure at a depth d in a fluid with mass density ρ
the liquid is 75.5 J K−1 mol−1. Assume that the exit gas remains at 25 °C and is ρgd, where g is the acceleration of free fall (9.81 m s−2). Use this relation to
that water vapour is a perfect gas. The standard enthalpy of vaporization adapt eqn 4B.2 to predict how the vapour pressure of a liquid of molar mass
of water at 25 °C is 44.0 kJ mol−1. (Hint: Start by calculating the amount M varies with depth. Estimate the effect on the vapour pressure of water at
in moles of H2O in the 50.0 dm3 of air after it has bubbled through the 25 °C in a column 10 m high.
liquid.) −a/H
P4B.15 The ‘barometric formula’, p = p0e , where H = 8 km, gives the dependence
P4B.9 The vapour pressure, p, of nitric acid varies with temperature as follows: of the pressure p on the altitude, a; p0 is the pressure at sea level, assumed to be
Determine (a) the normal boiling point and (b) the enthalpy of vaporization 1 atm. Use this expression together with the Clausius–Clapeyron equation to
of nitric acid. derive an expression for how the boiling temperature of a liquid depends on
the altitude. (Hint: The boiling point is the temperature at which the vapour
θ/ºC 0 20 40 50 70 80 90 100 pressure is equal to the external pressure.) Use your result to predict the boiling
temperature of water at 3000 m. The normal boiling point of water is 373.15 K and
p/kPa 1.92 6.38 17.7 27.7 62.3 89.3 124.9 170.9
you may take that the standard enthalpy of vaporization as 40.7 kJ mol−1.
−1
P4B.10 The vapour pressure of carvone (M = 150.2 g mol ), a component of oil

of spearmint, is as follows: These problems were supplied by Charles Trapp and Carmen Giunta.
Exercises and problems 139

P4B.16 Figure 4B.1 gives a schematic representation of how the chemical temperature, of these lines. Is there any restriction on the value this curvature
potentials of the solid, liquid, and gaseous phases of a substance vary with can take? For water, compare the curvature of the liquid line with that for the
temperature. All have a negative slope, but it is unlikely that they are straight gas in the region of the normal boiling point. The molar heat capacities at
lines as indicated in the illustration. Derive an expression for the curvatures, constant pressure of the liquid and gas are 75.3 J K−1 mol−1 and 33.6 J K−1 mol−1,
that is, the second derivative of the chemical potential with respect to respectively.

FOCUS 4 Physical transformations of pure substances


Integrated activities
I4.1 Construct the phase diagram for benzene near its triple point at (c) Plot Tm/(ΔhbHm/ΔhbSm) for 5 ≤ N ≤ 20. At what value of N does Tm change
36 Torr and 5.50 °C from the following data: ∆fusH = 10.6 kJ mol−1, by less than 1 per cent when N increases by 1?
∆vapH = 30.8 kJ mol−1, ρ(s) = 0.891 g cm−3, ρ(l) = 0.879 g cm−3. ‡
I4.4 A substance as well-known as methane still receives research attention

I4.2 In an investigation of thermophysical properties of methylbenzene because it is an important component of natural gas, a commonly used fossil
R.D. Goodwin (J. Phys. Chem. Ref. Data 18, 1565 (1989)) presented fuel. Friend et al. have published a review of thermophysical properties of
expressions for two coexistence curves. The solid–liquid curve is given by methane (D.G. Friend, J.F. Ely, and H. Ingham, J. Phys. Chem. Ref. Data 18,
583 (1989)), which included the following vapour pressure data describing the
p/bar  p3 /bar  1000(5.60  11.727 x )x liquid–vapour coexistence curve.

where x = T/T3 − 1 and the triple point pressure and temperature are T/K 100 108 110 112 114 120 130 140 150 160 170 190
p/MPa 0.034 0.074 0.088 0.104 0.122 0.192 0.368 0.642 1.041 1.593 2.329 4.521
p3 = 0.4362 μbar and T3 = 178.15 K. The liquid–vapour curve is given by
(a) Plot the liquid–vapour coexistence curve. (b) Estimate the standard
ln( p/bar )  10.418/y  21.157  15.996 y  14.015 y 2 boiling point of methane. (c) Compute the standard enthalpy of vaporization
 5.0120 y 3  4.7334(1  y )1.70 of methane (at the standard boiling point), given that the molar volumes of
the liquid and vapour at the standard boiling point are 3.80 × 10−2 dm3 mol−1
and 8.89 dm3 mol−1, respectively.
where y = T/Tc = T/(593.95 K). (a) Plot the solid–liquid and liquid–vapour
coexistence curves. (b) Estimate the standard melting point of methylbenzene. ‡
I4.5 Diamond is the hardest substance and the best conductor of heat yet
(c) Estimate the standard boiling point of methylbenzene. (The equation you characterized. For these reasons, it is used widely in industrial applications
will need to solve to find this quantity cannot be solved by hand, so you should that require a strong abrasive. Unfortunately, it is difficult to synthesize
use a numerical approach, e.g. by using mathematical software.) (d) Calculate diamond from the more readily available allotropes of carbon, such as
the standard enthalpy of vaporization of methylbenzene at the standard boiling graphite. To illustrate this point, the following approach can be used to
point, given that the molar volumes of the liquid and vapour at the standard estimate the pressure required to convert graphite into diamond at 25 °C (i.e.
boiling point are 0.12 dm3 mol−1 and 30.3 dm3 mol−1, respectively. the pressure at which the conversion becomes spontaneous). The aim is to
find an expression for ∆rG for the process graphite → diamond as a function
I4.3 Proteins are polymers of amino acids that can exist in ordered structures
of the applied pressure, and then to determine the pressure at which the Gibbs
stabilized by a variety of molecular interactions. However, when certain
energy change becomes negative. (a) Derive the following expression for the
conditions are changed, the compact structure of a polypeptide chain may
collapse into a random coil. This structural change may be regarded as a phase pressure variation of ∆rG at constant temperature
transition occurring at a characteristic transition temperature, the melting
  r G 
temperature, Tm, which increases with the strength and number of intermolecular    Vm,d  Vm,gr
interactions in the chain. A thermodynamic treatment allows predictions to  p T
be made of the temperature Tm for the unfolding of a helical polypeptide held
together by hydrogen bonds into a random coil. If a polypeptide has N amino where Vm,gr is the molar volume of graphite and Vm,d that of diamond. (b) The
acid residues, N − 4 hydrogen bonds are formed to form an α-helix, the most difficulty with dealing with the previous expression is that the Vm depends
common type of helix in naturally occurring proteins (see Topic 14D). Because on the pressure. This dependence is handled as follows. Consider ∆rG to be a
the first and last residues in the chain are free to move, N − 2 residues form the function of pressure and form a Taylor expansion about p = p⦵:
compact helix and have restricted motion. Based on these ideas, the molar Gibbs A 
    B
energy of unfolding of a polypeptide with N ≥ 5 may be written as ⦵   r G  ⦵   rG 
2


 r G( p)   r G( p )   
(p  p )  2 
1
2 
( p  p  )2
 p  p p⦵  p  p p⦵
 unfoldG  (N  4) hb H  (N  2)T  hb S where the derivatives are evaluated at p = p⦵ and the series is truncated after
the second-order term. Term A can be found from the expression in part (a)
where ΔhbH and ΔhbS are, respectively, the molar enthalpy and entropy of by using the molar volumes at p⦵. Term B can be found by using a knowledge
dissociation of hydrogen bonds in the polypeptide. (a) Justify the form of the of the isothermal compressibility of the solids, κT = −(1/V)(∂V/∂p)T. Use this
equation for the Gibbs energy of unfolding. That is, why are the enthalpy and definition to show that at constant temperature
entropy terms written as (N − 4)ΔhbH and (N − 2)ΔhbS, respectively? (b) Show
that Tm may be written as  2 rG 
 (Vm,d  Vm,gr )  T ,dVm,d  T ,grVm,gr
p2 p
(N  4) hb H where κT,d and κT,gr are the isothermal compressibilities of diamond and
Tm 
(N  2) hb S graphite, respectively. (c) Substitute the results from (a) and (b) into the
140 4 Physical transformations of pure substances

expression for ΔrG(p) in (b) and hence obtain an expression for ΔrG(p) Graphite Diamond
in terms of the isothermal compressibilities and molar volumes under
3 −1
standard conditions. (d) At 1 bar and 298 K the value of ∆rG for the transition Vs/(cm g ) at 1 bar 0.444 0.284
graphite → diamond is +2.8678 kJ mol−1. Use the following data to estimate κT/kPa−1
3.04 × 10 −8
0.187 × 10−8
the pressure at which this transformation becomes spontaneous. Assume that
κT is independent of pressure.
FOCUS 5
Simple mixtures

Mixtures are an essential part of chemistry, either in their own


right or as starting materials for chemical reactions. This group of
5C Phase diagrams of binary
Topics deals with the physical properties of mixtures and shows systems: liquids
how to express them in terms of thermodynamic quantities.
One widely employed device used to summarize the equilib-
rium properties of mixtures is the phase diagram. This Topic
describes phase diagrams of systems of liquids with gradually
5A The thermodynamic description increasing complexity. In each case the phase diagram for the
of mixtures system summarizes empirical observations on the conditions
under which the liquid and vapour phases of the system are sta-
The first Topic in this Focus develops the concept of chemi- ble, and the composition of the liquid phases.
cal potential as an example of a partial molar quantity and 5C.1 Vapour pressure diagrams; 5C.2 Temperature–composition
explores how the chemical potential of a substance is used to diagrams; 5C.3 Distillation; 5C.4 Liquid–liquid phase diagrams
describe the physical properties of mixtures. The key idea is
that at equilibrium the chemical potential of a species is the
same in every phase. By making use of the experimental ob-
servations known as Raoult’s and Henry’s laws, it is possible 5D Phase diagrams of binary
to express the chemical potential of a substance in terms of its
mole fraction in a mixture. systems: solids
5A.1 Partial molar quantities; 5A.2 The thermodynamics of mixing;
5A.3 The chemical potentials of liquids As shown in this Topic, the phase diagrams of solid mixtures
summarize experimental results on the conditions under
which the liquid and solid phases of the system are stable, and
the composition of these phases.
5B The properties of solutions 5D.1 Eutectics; 5D.2 Reacting systems; 5D.3 Incongruent melting

In this Topic, the concept of chemical potential is applied to


the discussion of the effect of a solute on certain properties
of a solution. These properties include the lowering of the va- 5E Phase diagrams of ternary systems
pour pressure of the solvent, the elevation of its boiling point,
the depression of its freezing point, and the origin of osmotic This Topic shows how the behaviour of a system with three
pressure. It is possible to construct a model of a certain class of components is represented using a triangular phase diagram
non-ideal solutions called ‘regular solutions’, which have prop- and how such diagrams are interpreted.
5E.1 Triangular phase diagrams; 5E.2 Ternary systems
erties that diverge from those of ideal solutions.
5B.1 Liquid mixtures; 5B.2 Colligative properties
5F Activities What is an application of this material?
The extension of the concept of chemical potential to real so- Two applications of this material are discussed, one from biol-
lutions involves introducing an effective concentration called ogy and the other from materials science, from among the huge
an ‘activity’. In certain cases, the activity may be interpreted in number that could be chosen for this centrally important field.
terms of intermolecular interactions; an important example is a Impact 7, accessed via the e-book, shows how the phenomenon
solution containing ions. Such solutions often deviate consider- of osmosis contributes to the ability of biological cells to main-
ably from ideal behaviour on account of the strong, long-range tain their shapes. In Impact 8, accessed via the e-book, phase
interactions between the charged species. This Topic shows diagrams of the technologically important liquid crystals are
how a model can be used to estimate the deviations from ideal discussed.
behaviour when the solution is very dilute, and how to extend
the resulting expressions to more concentrated solutions.
5F.1 The solvent activity; 5F.2 The solute activity; 5F.3 The activities of
regular solutions; 5F.4 The activities of ions

➤ Go to the e-book for videos that feature the derivation and interpretation of equations, and applications of this material.
TOPIC 5A The thermodynamic
­description of mixtures

5A.1(a) Partial molar volume


➤ Why do you need to know this material?
Imagine a huge volume of pure liquid water (at 25 °C) and then
Chemical reactions and many other chemical phenomena imagine adding 1 mol H2O to this large volume. The total volume
involve mixtures of substances. Therefore, it is important is found to increase by 18 cm3. It follows that the molar volume
to generalize the concepts introduced in Focus 4 for pure of pure water is 18 cm3 mol−1. However, if 1 mol H2O is added
substances and be able to describe the thermodynamics to a huge volume of pure ethanol, the total volume is found to
of mixtures. increase by only 14 cm3. The reason for the different increase in
volume is that the volume occupied by a given number of water
➤ What is the key idea?
molecules depends on the identity of the molecules that sur-
The chemical potential of a substance in a mixture is a round them. When a small amount of water is added to a very
logarithmic function of its concentration. large amount of ethanol each H2O molecule is surrounded by
ethanol molecules. The network of hydrogen bonds that nor-
➤ What do you need to know already?
mally hold H2O molecules at certain distances from each other
This Topic builds on the concept of chemical potential in pure water does not form. As a result, the H2O molecules are
introduced in the context of pure substances in Topic 4A. packed more tightly and the increase in the volume is less than
It makes use of the relation between the temperature when water is added to pure water. The quantity 14 cm3 mol−1 is
dependence of the Gibbs energy and entropy (Topic 3E), the ‘partial molar volume’ of water in pure ethanol. In general, the
and the concept of partial pressure (Topic 1A). partial molar volume of a substance A in a mixture is the change
in volume per mole of A added to a large volume of the mixture.
The partial molar volumes of the components of a mixture
vary with composition because the environment of each type of
molecule changes as the composition changes from pure A to
pure B. This changing molecular environment, and the conse-
The consideration of mixtures of substances that do not react quential changes in the interactions between molecules, results
together is a first step towards dealing with chemical reactions in the variation of the thermodynamic properties of a mixture
(which are treated in Topic 6A). Initially, the discussion centres as its composition is changed. The partial molar volumes of
on binary mixtures, which are mixtures of two components, A water and ethanol across the full composition range at 25 °C are
and B. Topic 1A shows how the partial pressure, which is the shown in Fig. 5A.1.
contribution of one component to the total pressure, is used to The partial molar volume, VJ, of a substance J at some gen-
discuss the properties of mixtures of gases. For a more general eral composition is defined formally as follows:
description of the thermodynamics of mixtures other analo-
gous ‘partial’ properties need to be introduced.  V  Partial molar volume
VJ     (5A.1)
 n [definition]
 J  p ,T , n 

where the subscript n′ signifies that the amounts of all other


5A.1 Partial molar quantities substances present are constant. This definition is illustrated
graphically in Fig. 5A.2, which shows how the total volume var-
The easiest partial molar property to visualize is the ‘partial ies with the amount of component J. The partial molar volume
molar volume’, the contribution that a component of a mixture at any particular composition (such as a) is the slope of a tan-
makes to the total volume of a sample. gent to the curve at this composition, the pressure, temperature,
144 5 Simple mixtures

58 that the partial molar volumes VA and VB are themselves con-

V(C2H5OH)/(cm3 mol–1)
Partial molar volume of ethanol,
Partial molar volume of water,

stant and so can be taken outside the integrals:


Ethanol
18
56 nA nB nA nB
V   VA dnA   VB dnB  VA  dnA  VB  dnB
 (5A.3)
V(H2O)/(cm3 mol–1)

0 0 0 0
Water  VA nA  VBnB
16
54
The integrals have been evaluated by supposing that the com-
ponents have been added in a way that ensures the composition
14 is constant and has its final value at all stages. But because V is
0 0.2 0.4 0.6 0.8 1 a state function the final result in eqn 5A.3 is valid however the
Mole fraction of ethanol, x(C2H5OH) solution is in fact prepared.
Figure 5A.1 The partial molar volumes of water and ethanol at Partial molar volumes can be measured in several ways. One
25 °C. Note the different scales (water on the left, ethanol on the method is to measure the dependence of the volume on the
right). composition and to fit the observed volume to a polynomial
in the amount of the substance. Once the polynomial has been
found, its slope can be determined at any composition of inter-
est by differentiation.
Total volume, V

V(b) Example 5A.1 Determining a partial molar volume


V(a)
A polynomial fit to measurements of the total volume of a
water/ethanol mixture at 25 °C that contains 1.000 kg of water is

v  1002.93  54.6664 z  0.363 94 z 2  0.028 256z 3


a b
where v = V/cm3, z = nE/mol, and nE is the amount of ethanol
Amount of J, nJ
molecules present. Determine the partial molar volume of
Figure 5A.2 The partial molar volume of a substance at a ethanol.
particular composition is the slope of the tangent to the curve Collect your thoughts Apply the definition in eqn 5A.1, taking
of total volume plotted against the amount of that substance.
care to convert the derivative with respect to nJ to a derivative
In general, partial molar volumes vary with the composition, as
with respect to z and keeping the units intact.
shown by the different slopes at a and b. Note that the partial
molar volume at b is negative: the overall volume of the sample The solution The partial molar volume of ethanol, VE, is
decreases as A is added.
 V   (V /cm3 )  cm 3
VE     
 nE  p ,T, nW    nE /mol   p ,T, nW mol
 
and amounts of the other components being held constant. As
 v 
is clear from the figure, the slope depends on the composition.   cm3 mol 1
z
  p ,T, nW
The definition in eqn 5A.1 implies that when the compo-
sition of a binary mixture is changed by the addition of an
where nW is the amount of H2O. The derivative is
amount dnA of A and an amount dnB of B, then the total volume
of the mixture changes by dv
 54.6664  2(0.363 94)z  3(0.028 256)z 2
dz
 V   V 
dV    dnA    dnB
 nA  p ,T, nB  nB  p ,T, nA  (5A.2) It follows that

 VA dnA  VB dnB VE /(cm3 mol 1 )  54.6664  0.727 88z  0.0847 68z 2


This equation can be integrated with respect to nA and nB pro-
Figure 5A.3 shows a graph of this function.
vided that the amounts of A and B are both increased in such a
way as to keep their ratio constant. That is, provided the com- Self-test 5A.1 When 500 g of water is added to 500 g of ethanol
position of the mixture is kept constant. This constancy ensures the resulting mixture has a volume of 1094 cm3. Given that the
5A The thermodynamic ­description of mixtures 145

Partial molar volume, VE/(cm3 mol–1) 56

μ(b)

Gibbs energy, G
55 μ(a)

54

a b
53
0 5 10 Amount of J, nJ
z = nE/mol
Figure 5A.4 The chemical potential of substance J is the slope
Figure 5A.3 The partial molar volume of ethanol as determined
of the tangent to the curve of Gibbs energy plotted against the
in Example 5A.1.
amount of that substance. In general, the chemical potential
varies with composition, as shown for the two values at a and b.
partial molar volume of water in the solution is 17.4 cm3 mol−1,
use eqn 5A.3 to find the partial molar volume of the ethanol.
By the same argument that led to eqn 5A.3, it follows that the
Answer: 56.3 cm3 mol−1

total Gibbs energy of a binary mixture is

G  nA  A  nB B (5A.5)
Molar volumes are always positive, but partial molar quan-
tities need not be. For example, the limiting partial molar vol- where μA and μB are the chemical potentials at the composi-
ume of MgSO4 in water (its partial molar volume in the limit tion of the mixture. That is, the chemical potential of a sub-
of zero concentration) is −1.4 cm3 mol−1, which means that stance, multiplied by the amount of that substance present in
the addition of 1 mol MgSO4 to a large volume of water re- the mixture, is its contribution to the total Gibbs energy of the
sults in a decrease in volume of 1.4 cm3. The mixture contracts mixture.
because as the Mg2+ and SO2− 4 ions become hydrated the open
In Topic 3E it is shown that the Gibbs energy varies with
structure of water is broken up, resulting in a slight collapse of pressure and temperature according to eqn 3E.7 (dG = Vdp –
the structure. SdT). For a mixture, the Gibbs energy also depends on the
composition (through eqn 5A.5, and its generalization to more
than two components). Therefore, this equation must be modi-
5A.1(b) Partial molar Gibbs energies fied to include how the Gibbs energy changes with changes in
The concept of a partial molar quantity can be broadened to in- the amounts of all the components present:
clude any extensive state function. An exceptionally important dG  Vdp  SdT   A dnA  B dnB  
example is the partial molar Gibbs energy, which is introduced 
Fundamental equation of chemical th
hermodynamics (5A.6)
in Topic 3D for a pure substance and called there the ‘chemical
potential’. For a pure substance, the chemical potential is simply This expression is the fundamental equation of chemical ther-
the molar Gibbs energy μ = G/n. The generalization of the defi- modynamics. Its implications and consequences are explored
nition for a substance J in a mixture is and developed in this Focus and Focus 6. When using this
equation, remember that the chemical potentials also depend
 G  Chemical potential on the composition; that is, the chemical potential of J de-
J  
 n  [definition]  (5A.4)
pends on the amounts of all the substances present, not just the
 J  p ,T , n 
amount of J.
where, as usual, n′ is used to denote that the amounts of all At constant pressure and temperature, eqn 5A.6 simplifies to
other components of the mixture are held constant. Figure 5A.4
dG   A dnA  B dnB   (5A.7)
is a graphical visualization of this definition. It shows a plot of
the Gibbs energy against amount of substance J, and at any Under these conditions it is shown in Topic 3D that dG =
particular composition (such as a) the chemical potential is dwadd,max. Therefore, at constant temperature and pressure,
the slope of a tangent to the curve at this point, the pressure,
dwadd, max   A dnA  B dnB   (5A.8)
temperature, and amounts of the other components being held
constant. This general definition is consistent with the defini- This relation shows that additional (non-expansion) work
tion for a pure substance, because then G = nJGJ,m and μJ = GJ,m. arises from the changing composition of a system. For instance,
146 5 Simple mixtures

in an electrochemical cell the electrical work the cell per- 5A.1(c) The Gibbs–Duhem equation
forms can be traced to its changing composition as products
are formed from reactants. But it is also possible to show that The total Gibbs energy of a binary mixture is given by eqn 5A.5
the chemical potential does more than show how G varies with (G = nAμA + nBμB). Because the chemical potentials depend on
composition. the composition, when the compositions are changed infinites-
imally the Gibbs energy of a binary system changes by
dG   A dnA  B dnB  nA dA  nB dB
How is that done? 5A.1 Demonstrating the wider
However, at constant pressure and temperature the change in
significance of the chemical potential
Gibbs energy is given by eqn 5A.7 (dG   A dnA  B dnB  ).
Start by combining the definition of G, G = H – TS, with that of Because G is a state function, these two expressions for dG
H, H = U + pV, to obtain G = U + pV – TS. Then rearrange this must be equal, which implies that at constant temperature and
relation into U = −pV + TS + G and consider an infinitesimal pressure
change. The result is
nA d A  nB dB  0 (5A.12a)
dU   pdV  Vdp  SdT  TdS  dG
This equation is a special case of the Gibbs–Duhem equation:
Now substitute eqn 5A.6 (dG  Vdp  SdT   A dnA  BdnB ),
Gibbs Duhem equation
and find n d
J
J J 0 [constant T and p ]  (5A.12b)

dU   pdV  Vdp  SdT  TdS The significance of the Gibbs–Duhem equation is that it im-
(Vdp  SdT   A dnA  BdnB  ) plies that the chemical potential of one component of a mix-
  pdV  TdS   A dnA  BdnB   ture cannot change independently of the chemical potentials
of the other components. For example, in a binary mixture, if
This expression is the generalization of eqn 3E.1, dU = TdS – one chemical potential increases, then the other must decrease,
pdV, to systems in which the composition may change. It fol- with the two changes related by eqn 5A.12a and therefore
lows that at constant volume and entropy,
nA
d B   d A  (5A.13)
dU   A dnA  BdnB   (5A.9) nB

and hence that


Brief illustration 5A.1

 U  Variation of the internal If the composition of a mixture is such that nA = 2nB, and
J  
 n  energy withh composition (5A.10) a small change in composition results in μA changing by
 J  S ,V , n 
∆μA = +1 J mol−1, μB will change by
That is, the chemical potential not only expresses how G chang-
es with composition under conditions of constant pressure and B  2  (1 Jmol 1 )  2 Jmol 1
temperature but also expresses how U changes with composi-
tion, under conditions of constant volume and entropy. Similar
arguments also lead to the relations
The same argument applies to all partial molar quantities.
 H  For instance, changes in the partial molar volumes of the spe-
Variation of the enthalpy
J  
 n  with compossition  (5A.11a) cies in a mixture are related by
 J  S ,p ,n 

 A 
n dVJ J  0 (5A.14a)
Variation of the Helmholtz J
J    (5A.11b)
 n  energy witth composition
 J T ,V, n For a binary mixture,
nA
Thus, μJ shows how all the extensive thermodynamic proper- dVB   dVA  (5A.14b)
nB
ties U, H, A, and G depend on the composition under various
constraints. This universality is why the chemical potential is As seen in Fig. 5A.1, where the partial molar volume of water
so central to chemistry. increases, the partial molar volume of ethanol decreases.
Equation 5A.14b also implies that as nA/nB increases, a given
5A The thermodynamic ­description of mixtures 147

change in the partial molar volume of A leads to a larger change 40 18.079


in the partial molar volume of B. This feature is seen on the
right-hand side of the plot in Fig. 5A.1: as the mole fraction of

V(K2SO4)/(cm3 mol–1)
38 18.078
ethanol (A) approaches 1 the change in the partial molar vol-

(cm3 mol–1)
V(H2O)/
ume of water (B) becomes much greater than that of ethanol.
In practice, the Gibbs–Duhem equation is used to determine 36 K2SO4
Water
the partial molar volume of one component of a binary mixture
from measurements of the partial molar volume of the second 18.076
34
component.

32 18.075
0 0.05 0.1
Example 5A.2 Using the Gibbs–Duhem equation b/(mol kg ) –1

In this example the concentration of the solute is expressed as a Figure 5A.5 The partial molar volumes of the components
molality. The molality, bB, of a solute is the amount (in moles) of an aqueous solution of potassium sulfate, as calculated in
of solute molecules in a solution divided by the mass of the Example 5A.2.
solvent (in kilograms), mA: bB = nB /mA . It is usual to define the
‘standard’ value of the molality as b = 1 mol kg−1.

Hence
The experimental values of the partial molar volume of b /b

vA  vA  9.108 M Ab  z 1/ 2 dz


K2SO4(aq) at 298 K are found to fit the expression 0

 vA  23 (9.108 M Ab)(b/b)3/ 2


⦵ ⦵

1/ 2
vB  32.280  18.216z
It then follows, by substituting the data (including MA = 1.802 ×
where vB  VK2 SO4 /(cm3 mol 1 ) and z is the numerical value 10−2 kg mol−1, the molar mass of water), that
of the molality of K2SO4, z = b/b . Use the Gibbs–Duhem

VA /(cm3 mol 1 )  18.079  0.1094(b/b)3/ 2



equation to derive an equation for the molar volume of water
in the solution. The molar volume of pure water at 298 K is The partial molar volumes are plotted in Fig. 5A.5.
18.079 cm3 mol−1.
Self-test 5A.2 Repeat the calculation for a salt B for which
Collect your thoughts Let A denote H2O, the solvent, and B VB/(cm3 mol−1) = 6.218 + 5.146z − 7.147z2 with z = b/b .

denote K2SO4, the solute. Because the Gibbs–Duhem equation Answer: VA/(cm3 mol−1) = 18.079 − 0.0464z2 + 0.0859z3
for the partial molar volumes of two components implies that
dvA = −(nB/nA)dvB, vA can be found by integration:
vB nB
vA  vA   dv B Exercises
0 nA
E5A.1 A polynomial fit to measurements of the total volume of a binary
3 1
where v  V /(cm mol ) is the numerical value of the molar
  mixture of A and B is
A A
volume of pure A. The first step is to change the variable of v  987.93  35.6774 x  0.45923x 2  0.017325x 3
integration from vB to z = b/b ; then integrate the right-hand

where v = V/cm3, x = nB/mol, and nB is the amount of B present. Derive an


side between z = 0 (pure A) and the molality of interest. expression for the partial molar volume of B.

The solution It follows from the information in the question E5A.2 The volume of an aqueous solution of NaCl at 25 °C was measured at a
that, with B = K2SO4, dvB/dz = 9.108z−1/2. Therefore, the inte- series of molalities b, and it was found to fit the expression v = 1003 + 16.62x +
1.77x3/2 + 0.12x2 where v = V/cm3, V is the volume of a solution formed from
gration required is ⦵
1.000 kg of water, and x = b/b . Calculate the partial molar volume of the
components in a solution of molality 0.100 mol kg−1.
b /b⦵ nB 1/ 2
vA  vA  9.108  z dz E5A.3 Suppose that in a mixture of A and B nA = 0.10nB and a small change in
0 nA
composition results in the chemical potential of A, μA, changing by
δμA = +12 J mol−1. By how much will μB change?
The amount of A (H2O) is nA = (1 kg)/MA, where MA is the
molar mass of water, and nB/(1 kg), which then occurs in the
ratio nB/nA, should be recognized as the molality b of B:
5A.2 The thermodynamics of mixing
nA = (1 kg)/MA nB/(1kg) = b
The dependence of the Gibbs energy of a mixture on its com-
nB nB nM ⦵ position is given by eqn 5A.5. As established in Topic 3E, at
= = B A = bM A = zb M A
nA (1 kg)/M A 1 kg constant temperature and pressure systems tend towards lower
148 5 Simple mixtures

Gibbs energy. This dependence and relation together provide Dependence of


the link needed in order to apply thermodynamics to the dis- pJ chemical potential
J  J  RT ln


 (5A.15b)
cussion of changes of composition, such as the mixing of

p on partial pressure
[mixture of perfect gases]
substances. One simple example is the mixing of two gases in-
The quantity μJ is the standard chemical potential of J, the

troduced into the same container: their mixing is spontaneous
and so must be accompanied by a decrease in G. chemical potential of the pure gas J at 1 bar.
The total Gibbs energy of the separated gases is
The Gibbs energy of mixing
5A.2(a)  ⦵ p 
Ginitial  nA  A  nB B  nA   A  RT ln ⦵ 
of perfect gases  p 

An expression for the Gibbs energy of mixing of two per-  ⦵ p 
 nB  B  RT ln ⦵  (5A.16a)
fect gases, at constant temperature and pressure, is found by p 

analysing the arrangement shown in Fig. 5A.6. Initially, two
perfect gases consisting of an amount nA of A and an amount The total Gibbs energy after mixing is a similar expression but
nB of B at a common pressure p and temperature T are kept with partial pressures in place of the total pressure:
separate. The wall separating the two gases is removed and
 ⦵ p   ⦵ p 
the gases mix. Because the gases are perfect, the pressure and Gfinal  nA  A  RT ln ⦵A   nB  B  RT ln ⦵B  (5A.16b)
temperature remain the same in the final mixed state. In this  p   p 
state the partial pressures of A and B are pA and pB, respec- Then, because ln x – ln y = ln(x/y), the Gibbs energy of mixing,
tively, and the sum of these partial pressures is the total pres- ΔmixG = Gfinal − Ginitial, is
sure, pA + pB = p.
In the initial arrangement, each gas is pure and so its chemi- pA p
cal potential is equal to its molar Gibbs energy, μ = Gm. The  mix G  nA RT ln  nB RT ln B  (5A.16c)
p p
molar Gibbs energy depends on the pressure according to
 
eqn 3E.15, Gm ( p)  Gm  RT ln( p/p ), and so the dependence
⦵ ⦵
At this point nJ can be replaced by xJn, where n is the total
of the chemical potential on pressure is amount of A and B, and the relation between partial pressure
and mole fraction (Topic 1A, pJ = xJp) can be used to write
Dependence of pJ/p = xJ for each component. The result is
p chemical potential
    RT ln ⦵

on pressure [pure,
 (5A.15a)
p Gibbs energy of
perfect gas]  mix G  nRT (x A ln x A  x B ln x B ) mixing [perfect  (5A.17)
gass, constant T and p ]
where μ is the standard chemical potential, the chemical po-

tential of the pure gas at 1 bar. Because mole fractions are never greater than 1, the loga-
There are no interactions between the molecules of a perfect rithms in this equation are negative, and ∆mixG < 0 (Fig. 5A.7).
gas, so its chemical potential is unaffected by the presence of The conclusion that ∆mixG is negative for all compositions con-
molecules of other gases. Therefore the chemical potential of a firms that perfect gases mix spontaneously in all proportions.
perfect gas J depends similarly on its contribution to the total
pressure, its partial pressure, pJ,
0

–0.2
ΔmixG/nRT
Initial

nA, T, p
–0.4
nB, T, p

–0.6

–0.8
Final

T, pA, pB with pA + pB = p 0 0.5 1


Mole fraction of A, xA

Figure 5A.7 The Gibbs energy of mixing of two perfect gases at


constant temperature and pressure, and (as discussed later) of two
liquids that form an ideal solution. The Gibbs energy of mixing is
Figure 5A.6 The arrangement for calculating the thermodynamic negative for all compositions, so perfect gases mix spontaneously
functions of mixing of two perfect gases. in all proportions.
5A The thermodynamic ­description of mixtures 149

Example 5A.3
3
p 1
p
Calculating a Gibbs energy of mixing  mixG  (3.0 mol )RT ln 2  (1.0 mol )RT ln 2
3p p
A container is divided into two equal compartments (Fig. 5A.8).  (3.0 mol )RT ln 2  (1.0 mol )RT ln 2
One contains 3.0 mol H2(g) at 25 °C; the other contains 1.0 mol
 (4.0 mol )RT ln 2
N2(g) at 25 °C. Calculate the Gibbs energy of mixing when the
partition is removed. Assume that the gases are perfect.  (4.0 mol )  (8.3145 JK 1 mol 1 )  (298 K )  ln 2
 6.9 kJ
Collect your thoughts Equation 5A.17 cannot be used direct-
ly because the two gases are initially at different pressures, so Comment. In this example, the value of ∆mixG is the sum of two
proceed by calculating the total Gibbs energy before and after contributions: the mixing itself, and the changes in pressure of
mixing directly from the chemical potentials. Because the the two gases to their final total pressure, 2p. Even though the
two compartments have the same volume and are at the same pressure is not constant, because there is no heat transaction
temperature, it follows from the perfect gas equation that the with the surroundings, ΔG < 0 remains a criterion of spontane-
pressure of H2 is three times that of N2 because the amount ous change and this process is spontaneous. When 3.0 mol H2
of H2 is three times that of N2. Therefore, if the pressure of mixes with 1.0 mol N2 at the same pressure, with the volumes of
N2 is p, that of H2 is 3p. When the partition is removed the the vessels adjusted accordingly, the change of Gibbs energy is
volume available to each gas is doubled, therefore the partial −5.6 kJ. Because this value is for a change at constant pressure and
pressure it exerts is halved. Thus the partial pressure of H2 is temperature, the fact that it is negative also implies spontaneity.
3
p and the partial pressure of N2 is 12 p, giving a total pressure
2 Self-test 5A.3 Suppose that 2.0 mol H2 at 2.0 atm and 25 °C
of 23 p  12 p  2 p.
and 4.0 mol N2 at 3.0 atm and 25 °C are mixed by removing the
The solution Given that the initial pressure of nitrogen is p, partition between them. Calculate ∆mixG.
and that of hydrogen is 3p, the initial Gibbs energy is Answer: −9.7 kJ
 ⦵ 3p 
Ginitial  (3.0 mol)    (H2 )  RT ln ⦵ 
 p 
 ⦵ p  Other thermodynamic mixing
5A.2(b)
(1.0 mol)    (N2 )  RT ln ⦵ 
 p  functions
After mixing the partial pressure of nitrogen is 12 p and that of In Topic 3E it is shown that (∂G/∂T)p = −S. It follows imme-
hydrogen is 23 p. Therefore, the Gibbs energy changes to diately from eqn 5A.17 that, for the mixing of perfect gases at
constant pressure, the entropy of mixing, ∆mixS, is
 ⦵ 3
p
Gfinal  (3.0 mol)    (H2 )  RT ln 2 ⦵ 
 p    G 
 mix S    mix   nR(x A ln x A  x B ln x B )
 ⦵ 1
p  T  p
(1.0 mol)    (N2 )  RT ln 2 ⦵ 
 p  Entropy of mixing [perfect gases, constant T and p ]  (5A.18)
The Gibbs energy of mixing is the difference of these two
quantities: Because ln x < 0, it follows that ∆mixS > 0 for all compositions
(Fig. 5A.9).

0.8

3.0 mol H2
Initial

1.0 mol N2 0.6

3p
ΔmixS/nR

p
0.4

3.0 mol H2 0.2


Final

2p 1.0 mol N2
p(H2) = 3/2 p
p(N2) = 1/2 p 0
0 0.5 1
Mole fraction of A, xA

Figure 5A.8 The initial and final states considered in the Figure 5A.9 The entropy of mixing of two perfect gases at
calculation of the Gibbs energy of mixing of gases at different constant temperature and pressure, and (as discussed later) of two
initial pressures, as in Example 5A.3. liquids that form an ideal solution.
150 5 Simple mixtures

Brief illustration 5A.2 that, as established in Topic 4A, at equilibrium the chemical po-
tential of a substance present as a vapour must be equal to its
For equal amounts of perfect gas molecules that are mixed at chemical potential in the liquid.
the same pressure, set x=
A x=
B
1
2
and obtain

 mix S  nR  12 ln 12  12 ln 12   nR ln 2
5A.3(a) Ideal solutions
When discussing the thermodynamics of solutions, quantities
with n the total amount of gas molecules. For 1 mol of each relating to pure substances are denoted by a superscript *. Thus,
species, so n = 2 mol, the chemical potential of pure A is written μ A or as μ A (l) when
it is necessary to emphasize that A is a liquid. If the vapour pres-
 mix S  (2 mol)  R ln 2  11.5 JK 1
sure of the pure liquid is pA it follows from eqn 5A.15b that the
An increase in entropy is expected when one gas spreads into chemical potential of A in the vapour (treated as a perfect gas) is
the other and the disorder increases. A (g)  RT ln( pA /p). The chemical potential of A in the liquid
⦵ ⦵

and A in the vapour are equal at equilibrium (Fig. 5A.10), so


liquid vapour
Under conditions of constant pressure and temperature, the   
⦵  (5A.20a)
A (l)  A (g )  RT ln( pA /p)

 ⦵
enthalpy of mixing, ∆mixH, the enthalpy change accompany-
ing mixing, of two perfect gases can be calculated from ∆G =
If another substance, a solute, is also present in the liquid, the
∆H − T∆S, rearranged into ∆H = ∆G + T∆S. Equation 5A.17 for
chemical potential of A in the liquid is changed to μA(l) and its
∆mixG and eqn 5A.18 for ∆mixS gives
vapour pressure is changed to pA. The vapour and solvent are
 mix H   mixG  T  mix S still in equilibrium, so
 nRT (x A ln x A  x B ln x B ) A (l)  A (g )  RT ln( pA /p)
⦵ ⦵
(5A.20b)
T {nR(x A ln x A  x B ln x B )}
The next step is the combination of these two equations to
Hence eliminate the standard chemical potential of the gas, μ A (g ), first

by writing eqn 5A.20a as  A (g )  A (l)  RT ln( pA /p ) and then


 
⦵   ⦵

Enthalpy of mixing
 mix H  0 [perfect gases, constant T and p]]
 (5A.19) substituting this expression into eqn 5A.20b. The result is

A (l)  A (g )  RT ln( pA /p)


⦵ ⦵
The enthalpy of mixing is zero, as expected when there are no
interactions between the molecules forming the gaseous mix-  A (l)  RT ln( pA /p)  RT ln( pA /p)
⦵ ⦵

ture. It follows that, because the entropy of the surroundings  (5A.21)


p
is unchanged, the whole of the driving force for mixing comes   (l)  RT ln A

A
from the increase in entropy of the system. pA

Exercises
3
E5A.4 Consider a container of volume 5.0 dm that is divided into two
compartments of equal size. In the left compartment there is nitrogen at
1.0 atm and 25 °C; in the right compartment there is hydrogen at the same A(g) + B(g)
μA(g, p)
temperature and pressure. Calculate the entropy and Gibbs energy of mixing
when the partition is removed. Assume that the gases are perfect.
=

E5A.5 Two samples of perfect gases A and B are kept separate and each exert a μA(l)
pressure of 1.2 atm at 29 °C. The gases are then allowed to mix in such a way A(l) + B(l)
that the pressure remains the same. What is the temperature of the resulting
mixture? What is the value of ∆mixH?

Figure 5A.10 At equilibrium, the chemical potential of the


5A.3 The chemical potentials of liquids gaseous form of a substance A is equal to the chemical potential
of its condensed phase. The equality is preserved if a solute is
To discuss the equilibrium properties of liquid mixtures it is also present. Because the chemical potential of A in the vapour
necessary to know how the Gibbs energy of a liquid varies with depends on its partial vapour pressure, it follows that the chemical
composition. The calculation of this dependence uses the fact potential of liquid A can be related to its partial vapour pressure.
5A The thermodynamic ­description of mixtures 151

80
pB* Total pressure

Benzene
60
Total

Pressure, p/Torr
Partial
Pressure

pressure of B
pA* 40

Partial pressure
of A 20
Methylbenzene
0
0 Mole fraction of A, xA 1 0
Mole fraction of methylbenzene, x(C6H5CH3)
0 1
Figure 5A.11 The partial vapour pressures of the two components
of an ideal binary mixture are proportional to the mole fractions of Figure 5A.12 Two similar liquids, in this case benzene and
the components, in accord with Raoult’s law. The total pressure is methylbenzene (toluene), behave almost ideally, and the variation
also linear in the mole fraction of either component. of their vapour pressures with composition resembles that for an
ideal solution.
The final step draws on additional experimental informa-
tion about the relation between the ratio of vapour pressures
and the composition of the liquid. In a series of experiments For an ideal solution, it follows from eqns 5A.21 and 5A.22 that
on mixtures of closely related liquids (such as benzene and
methylbenzene), François Raoult found that the ratio of the A (l)  A (l)  RT ln x A
Chemical potential
 (5A.23)
[ideal solution]
partial vapour pressure of each component to its vapour pres-
sure when present as the pure liquid, pA /pA , is approximately
This important equation can be used as the definition of an ideal
equal to the mole fraction of A in the liquid mixture. That is, he
solution (so that it implies Raoult’s law rather than stemming
established what is now called Raoult’s law:
from it). It is in fact a better definition than eqn 5A.22 because
pA = x A pA Raoult’s law [ideal solution] (5A.22) it does not assume that the vapour is a perfect gas.
The molecular origin of Raoult’s law is the effect of the sol-
This law is illustrated in Fig. 5A.11. Some mixtures obey ute on the entropy of the solution. In the pure solvent, the mol-
Raoult’s law very well, especially when the components are ecules have a certain disorder and a corresponding entropy;
structurally similar (Fig. 5A.12). Mixtures that obey the law the vapour pressure then represents the tendency of the system
throughout the composition range from pure A to pure B are and its surroundings to reach a higher entropy. When a solute
called ideal solutions. is present, the solution has a greater disorder than the pure sol-
vent because a molecule chosen at random might or might not
Brief illustration 5A.3 be a solvent molecule. Because the entropy of the solution is
higher than that of the pure solvent, the solution has a lower
The vapour pressure of pure benzene at 25 °C is 95 Torr and tendency to acquire an even higher entropy by the solvent va-
that of pure methylbenzene is 28 Torr at the same temperature. porizing. In other words, the vapour pressure of the solvent in
=
In an equimolar mixture x benzene x=
methylbenzene
1
2
so the partial the solution is lower than that of the pure solvent.
vapour pressure of each one in the mixture is Some solutions depart significantly from Raoult’s law
pbenzene  12  95 Torr  47.5 Torr (Fig. 5A.13). Nevertheless, even in these cases the law is obeyed
increasingly closely for the component in excess (the solvent)
pmethylbenzene  21  28 Torr  14 Torr as it approaches purity. The law is another example of a limiting
the total vapour pressure of the mixture is 61.5 Torr. Given the law (in this case, achieving reliability as xA → 1) and is a good
two partial vapour pressures, it follows from the definition of approximation for the properties of the solvent if the solution
partial pressure (Topic 1A, pJ = xJp) that the mole fractions in is dilute.
the vapour are
=
x vap,benzene p=
benzene /ptotal (47.5 Torr )/(61.5 Torr ) = 0.77
5A.3(b) Ideal–dilute solutions
and
In ideal solutions the solute, as well as the solvent, obeys
=
x vap,methylbenzene p=
methylbenzene /ptotal (14 Torr )/(61.5 Torr ) = 0.23 Raoult’s law. However, William Henry found experimentally
The vapour is richer in the more volatile component (benzene). that, for real solutions at low concentrations, although the va-
pour pressure of the solute is proportional to its mole fraction,
152 5 Simple mixtures

500
Total
400 Solute
Pressure, p/Torr

Carbon disulfide
300

200
Acetone
100

0
0 Mole fraction of carbon disulfide, x(CS2) 1
Figure 5A.15 In a dilute solution, the solvent molecules (the
Figure 5A.13 Strong deviations from ideality are shown by blue spheres) are in an environment that differs only slightly
dissimilar liquids (in this case carbon disulfide and acetone from that of the pure solvent. The solute particles (the red
(propanone)). The dotted lines show the values expected from spheres), however, are in an environment totally unlike that
Raoult’s law. of the pure solute.

molecules are in an environment very much like the one they


KB have in the pure liquid (Fig. 5A.15). In contrast, the solute mole-
cules are surrounded by solvent molecules, which is entirely dif-
Ideal-dilute ferent from their environment when it is in its pure form. Thus,
solution
the solvent behaves like a slightly modified pure liquid, but the
Pressure, p

(Henry)
pB* solute behaves entirely differently from its pure state unless
Real the solvent and solute molecules happen to be very similar. In
solution
the latter case, the solute also obeys Raoult’s law.

Ideal solution
(Raoult)
Example 5A.4 Investigating the validity of Raoult’s and
0 Mole fraction of B, xB 1 Henry’s laws
Figure 5A.14 When a component (the solvent) is nearly pure, The vapour pressures of each component in a mixture of pro-
it has a vapour pressure that is proportional to the mole fraction panone (acetone, A) and trichloromethane (chloroform, C)

with a slope pB (Raoult’s law). When it is the minor component were measured at 35 °C with the following results:
(the solute) its vapour pressure is still proportional to the
mole fraction, but the constant of proportionality is now KB xC 0 0.20 0.40 0.60 0.80 1
(Henry’s law). pC/kPa 0 4.7 11 18.9 26.7 36.4
pA/kPa 46.3 33.3 23.3 12.3 4.9 0

the constant of proportionality is not the vapour pressure of the Confirm that the mixture conforms to Raoult’s law for the
pure substance (Fig. 5A.14). Henry’s law is: component in large excess and to Henry’s law for the minor
component. Find the Henry’s law constants.
pB  x B K B Henry’s law [ideal dilute solution] (5A.24)
Collect your thoughts Both Raoult’s and Henry’s laws are state-
In this expression xB is the mole fraction of the solute and KB ments about the form of the graph of partial vapour pressure
is an empirical constant (with the dimensions of pressure). against mole fraction. Therefore, plot the partial vapour pres-
Its value is chosen so that the plot of the vapour pressure of sures against mole fraction. Raoult’s law is tested by comparing
B against its mole fraction is the tangent of the experimen- the data with the straight line pJ = x J pJ for each component in
tal curve at xB = 0. Henry’s law is therefore also a limiting law, the region in which it is in excess (and acting as the solvent).
achieving reliability as xB → 0. Henry’s law is tested by finding a straight line pJ = xJKJ that is
Mixtures for which the solute B obeys Henry’s law and the tangent to each partial vapour pressure curve at low x, where
the component can be treated as the solute.
solvent A obeys Raoult’s law are called ideal–dilute solutions.
The difference in behaviour of the solute and solvent at low con- The solution The data are plotted in Fig. 5A.16 together with
centrations (as expressed by Henry’s and Raoult’s laws, respec- the Raoult’s law lines. Henry’s law requires KA = 24.5 kPa for
tively) arises from the fact that in a dilute solution the solvent acetone and KC = 23.5 kPa for chloroform.
5A The thermodynamic ­description of mixtures 153

50 Table 5A.1 Henry’s law constants for gases in water at 298 K*


p*A
K/(kPa kg mol−1)
40
p*C CO2 3.01 × 103
Pressure, p/kPa

30 H2 1.28 × 105
Raoult’s law
KA KC N2 1.56 × 105
20 O2 7.92 × 104

* More values are given in the Resource section.


10
Henry’s law
0 Brief illustration 5A.4
0 Mole fraction of chloroform, xC 1
To estimate the molar solubility of oxygen in water at 25 °C and
Figure 5A.16 The experimental partial vapour pressures
a partial pressure of 21 kPa, its partial pressure in the atmos-
of a mixture of chloroform (trichloromethane) and acetone
phere at sea level, write
(propanone) based on the data in Example 5A.4. The values
of K are obtained by extrapolating the dilute solution vapour pO2 21 kPa
bO2    2.7  104 mol kg 1
pressures, as explained in the Example. K O2 7.9  10 kPa kg mol 1
4

Comment. Notice how the system deviates from both Raoult’s The molality of the saturated solution is therefore 0.27 mmol kg−1.
and Henry’s laws even for quite small departures from x = 1 and To convert this quantity to a molar concentration, assume that
x = 0, respectively. These deviations are discussed in Topic 5F. the mass density of this dilute solution is essentially that of pure
water at 25 °C, or ρ = 0.997 kg dm−3. It follows that the molar
Self-test 5A.4 The vapour pressure of chloromethane at various concentration of oxygen is
mole fractions in a mixture at 25 °C was found to be as follows:
[O2 ]  bO2   (2.7  104 mol kg 1 )  (0.997 kg dm 3 )
x 0.005 0.009 0.019 0.024  0.27 mmol dm 3
p/kPa 27.3 48.4 101 126

Estimate the Henry’s law constant for chloromethane.


Answer: 5 MPa Exercises
E5A.6 The vapour pressure of benzene at 20 °C is 10 kPa and that of
methylbenzene is 2.8 kPa at the same temperature. What is the vapour
pressure of a mixture of equal masses of each component?
For practical applications, Henry’s law is expressed in terms
E5A.7 At 300 K, the partial vapour pressures of HCl (i.e. the partial pressures
of the molality, b, of the solute, pB = bBKB. Some Henry’s law data
of the HCl vapour) in liquid GeCl4 are as follows:
for this convention are listed in Table 5A.1. As well as provid-
ing a link between the mole fraction of the solute and its par- xHCl 0.005 0.012 0.019
tial pressure, the data in the table may also be used to calculate pHCl/kPa 32.0 76.9 121.8
gas solubilities. Knowledge of Henry’s law constants for gases Show that the solution obeys Henry’s law in this range of mole fractions, and
in blood and fats is important for the discussion of respiration, calculate Henry’s law constant at 300 K.
especially when the partial pressure of oxygen is abnormal, as E5A.8 Use Henry’s law to calculate the solubility (as a molality) of CO2 in water
in diving and mountaineering, and for the discussion of the ac- at 25 °C when its partial pressure is (i) 0.10 atm, (ii) 1.00 atm. The Henry’s law
tion of gaseous anaesthetics. constant for CO2 in water at this temperature is 3.01 × 103 kPa kg mol−1.

Checklist of concepts
☐ 1. The partial molar volume of a substance in a mixture is ☐ 4. The Gibbs–Duhem equation shows how the changes in
the change in volume per mole of that substance added chemical potentials (and, by extension, of other partial
to a large volume of the mixture. molar quantities) of the components of a mixture are
☐ 2. The chemical potential is the partial molar Gibbs energy. related.
☐ 3. The chemical potential also expresses how, under a vari- ☐ 5. The Gibbs energy of mixing is negative for perfect gases
ety of different conditions, the thermodynamic func- at the same pressure and temperature.
tions vary with composition.
154 5 Simple mixtures

☐ 6. The entropy of mixing of perfect gases at the same pres- is a limiting law valid as the mole fraction of the species
sure and temperature is positive and the enthalpy of approaches 1.
mixing is zero. ☐ 9. Henry’s law provides a relation between the vapour
☐ 7. Raoult’s law provides a relation between the vapour pressure of a solute and its mole fraction in a mixture; it
pressure of a substance and its mole fraction in a liquid is the basis of the definition of an ideal–dilute solution.
mixture. ☐ 10. An ideal–dilute solution is a solution that obeys Henry’s
☐ 8. An ideal solution is a solution that obeys Raoult’s law law at low concentrations of the solute, and for which
over its entire range of compositions; for real solutions it the solvent obeys Raoult’s law.

Checklist of equations
Property Equation Comment Equation number
Partial molar volume VJ = (∂V/∂nJ)p,T,n′ Definition 5A.1
Chemical potential μJ = (∂G/∂nJ)p,T,n′ Definition 5A.4
Total Gibbs energy G = nAμA + nBμB Binary mixture 5A.5
Fundamental equation of chemical dG  Vdp  SdT  A dnA  B dnB   5A.6
thermodynamics
Gibbs–Duhem equation ΣJnJdμJ = 0 5A.12b
Chemical potential of a pure gas

    RT ln( p/p )
 ⦵

Perfect gas 5A.15a

Chemical potential of a gas in a mixture



J  J  RT ln( pJ /p )
 ⦵

Perfect gases 5A.15b
Gibbs energy of mixing ΔmixG = nRT(xA ln xA + xB ln xB) Perfect gases and ideal solutions 5A.17
Entropy of mixing ΔmixS = −nR(xA ln xA + xB ln xB) Perfect gases and ideal solutions 5A.18
Enthalpy of mixing ΔmixH = 0 Perfect gases and ideal solutions 5A.19
Raoult’s law pA = x A p  True for ideal solutions; limiting law as xA → 1 5A.22
A

Chemical potential of component A (l)  A (l)  RT ln x A Ideal solution 5A.23

Henry’s law pB = xBKB True for ideal–dilute solutions; limiting law as 5A.24
xB → 0
TOPIC 5B The properties of solutions

5B.1(a) Ideal solutions


➤ Why do you need to know this material?
The Gibbs energy of mixing of two liquids to form an ideal
Mixtures and solutions play a central role in chemistry, and solution is calculated in exactly the same way as for two gases
so it is important to understand how their compositions (Topic 5A). Thus, the total Gibbs energy before the liquids are
affect their thermodynamic properties. One very impor- mixed is
tant physical property of a solution is its osmotic pressure,
which is used, for example, to determine the molar masses Ginitial  nA  A  nB B (5B.2a)
of macromolecules.
where the * denotes the pure liquid. When they are mixed, the
➤ What is the key idea? individual chemical potentials are given by eqn 5B.1 and the
The chemical potential of a substance in a mixture is the total Gibbs energy is
same in every phase in which it occurs.
Gfinal  nA ( A  RT ln x A )  nB (B  RT ln x B ) (5B.2b)
➤ What do you need to know already?
Consequently, the Gibbs energy of mixing, the difference of
This Topic is based on the expressions derived in Topic 5A
these two quantities, is
for the dependence of the chemical potential of a gas on
its partial pressure and the dependence of the chemical Gibbs energy
potential of a liquid in a mixture on its mole fraction. One  mix G  nRT (x A ln x A  x B ln x B ) of mixing  (5B.3)
derivation makes use of the Gibbs–Helmholtz equation [ideal soluttion]
(Topic 3E).
where n = nA + nB and nJ = xJn. As for gases, it follows that the
ideal entropy of mixing of two liquids is

Entropy of
 mix S  nR(x A ln x A  x B ln x B ) mixing  (5B.4)
Thermodynamics can provide insight into the properties of liq- [ideal solution]
uid mixtures and sheds light on their boiling and freezing tem-
peratures and related properties, such as their vapour pressure Then from ΔmixG = ΔmixH − TΔmixS it follows that the ideal
and osmotic pressure. The chemical potential is the key concept ­enthalpy of mixing is zero: ΔmixH = 0. The ideal volume of mixing,
used to develop a quantitative description of these properties. As the change in volume on mixing, is also zero. To see why, note
explained in Topic 3D, the chemical potential of a substance is a that (∂G/∂p)T = V (this is eqn 3E.8), so ΔmixV = (∂ΔmixG/∂p)T .
measure of its potential to bring about change, and equilibrium But ΔmixG is independent of pressure, so the derivative with re-
exists when these potentials are balanced throughout the system. spect to pressure is zero, and therefore ΔmixV = 0.
The equations derived so far are the same as those for the
mixing of two perfect gases and all the conclusions drawn there
are valid here. Specifically, because the enthalpy of mixing is
5B.1 Liquid mixtures zero, there is no change in the entropy of the surroundings,
so the driving force for mixing is the increasing entropy of the
The development in this Topic is based on the expression de- system as the molecules mingle with no change of energy. It
rived in Topic 5A for the chemical potential μJ of a component J should be noted, however, that solution ideality means some-
in an ideal mixture or solution: thing different from gas perfection. In a perfect gas there are
no interactions between the molecules. In ideal solutions there
Chemical potential
J  J  RT ln x J  (5B.1) are interactions, but the average energy of A–B interactions in
[ideal solution]
the mixture is the same as the average energy of A–A and B–B
where μJ is the chemical potential of pure J and xJ is its mole ­interactions in the pure liquids. The variation of the Gibbs en-
fraction. Note that for any mixture with xJ < 1, the chemical ergy and entropy of mixing with composition is the same as
potential is lower than when the substance is pure (xJ = 1). that for gases and shown in Figs. 5B.1 and 5B.2.
156 5 Simple mixtures

0 from the way in which the molecules of one type might cluster
together instead of mingling freely with the others.
–0.2
If the enthalpy of mixing is sufficiently positive so that
∆mixH > T∆mixS, the Gibbs energy of mixing is positive and mix-
ing is not spontaneous. In other words, the liquids are immis-
ΔmixG/nRT

–0.4 cible. It is also possible for the entropy of mixing to be negative,


for example as a result of specific interactions between the two
–0.6
species (such as hydrogen bonds) that prevent complete rand-
omization. If the enthalpy of mixing is sufficiently negative so
that ∆mixH < T∆mixS mixing will still occur, but if this is not the
–0.8
0 0.5 1 case the liquids will be immiscible. Some liquids are partially
Mole fraction of A, xA miscible, which means that they are miscible over only a lim-
Figure 5B.1 The Gibbs energy of mixing of two liquids that form ited range of compositions.
an ideal solution.
5B.1(b) Excess functions and regular solutions
0.8
The thermodynamic properties of real solutions are expressed
in terms of the excess functions, XE, the difference between the
0.6 observed thermodynamic property of mixing and the property
for an ideal solution:
ΔmixS/nR

0.4 Excess function


X E   mix X   mix X ideal [definition]
 (5B.5)

0.2 The excess entropy, SE, for example, is calculated by using the
value of ∆mixSideal given by eqn 5B.4. The excess enthalpy, HE, and
volume, VE, are both equal to the observed enthalpy and vol-
0
0 0.5 1 ume of mixing, because the ideal values are zero in each case.
Mole fraction of A, xA
Figure 5B.3 shows two examples of the composition-­
Figure 5B.2 The entropy of mixing of two liquids that form an dependence of excess functions. Figure 5B.3a shows data for a
ideal solution. benzene/cyclohexane mixture: the positive values of HE, which
implies that ∆mixH > 0, indicate that the A–B interactions in the
mixture are less attractive than the A–A and B–B interactions
Brief illustration 5B.1 in the pure liquids. The symmetrical shape of the curve reflects
the similar strengths of the A–A and B–B interactions. Figure
Consider a mixture of benzene and methylbenzene, which form
5B.3b shows the composition dependence of VE of a mixture
an approximately ideal solution, and suppose 1.0 mol C6H6(l) is
of tetrachloroethene and cyclopentane. At high mole fractions
mixed with 2.0 mol C6H5CH3(l). For the mixture, xbenzene = 0.33
and xmethylbenzene = 0.67. The Gibbs energy and entropy of mixing
at 25 °C, when RT = 2.48 kJ mol−1, are 8
800
 mixG /n  (2.48 kJmol 1 )  (0.33 ln 0.33  0.67 ln 0.67)
4
 1.6 kJmol 1
VE/(mm3 mol–1)
HE/(J mol–1)

0
 mix S /n  (8.3145 JK 1 mol 1 )  (0.33 ln 0.33  0.67 ln 0.67) 400
 5.3 JK 1 mol 1 –4

The enthalpy of mixing is zero (presuming that the solution is –8


ideal).
0 –120
0 0.5 1 0.5 1
(a) x(C6H6) (b) x(C2Cl4)

Real solutions are composed of molecules for which the Figure 5B.3 Experimental excess functions at 25 °C. (a) HE for
A–A, A–B, and B–B interactions are all different. There may benzene/cyclohexane; this graph shows that the mixing is
be enthalpy and volume changes when such liquids mix. There endothermic (because ∆mixH = 0 for an ideal solution). (b) The
may also be an additional contribution to the entropy arising excess volume, V E, for tetrachloroethene/cyclopentane.
5B The properties of solutions 157

of cyclopentane VE < 0, indicating that the solution contracts +0.1


as tetrachloroethene is added. One interpretation of this con- 3
0
traction is that the ring structure of cyclopentane results in in-
2.5
efficient packing of the molecules, but as tetrachloroethene is –0.1
added, the molecules in the mixture pack together more tightly.

ΔmixG/nRT
2
Similarly, at high mole fractions of tetrachloroethene, the solu- –0.2
tion expands as cyclopentane is added. In this case the nearly 1.5
–0.3
flat tetrachloroethene molecules pack efficiently in the pure
liquid, but become disrupted as the bulky ring cyclopentane is 1
–0.4
added.
A useful model for discussing the behaviour of non-ideal so- –0.5
0 0.5 1
lutions is the regular solution, a solution for which HE ≠ 0 but xA
SE = 0. A regular solution can be thought of as one in which the
Figure 5B.5 The Gibbs energy of mixing for different values of the
two kinds of molecules are distributed randomly (as in an ideal
parameter ξ. Interact with the dynamic version of this graph in the
solution) but have different energies of interaction with each e-book.
other. To express this concept more quantitatively, suppose that
the excess enthalpy depends on composition as
the graph shows two minima separated by a local maximum
H E  n RTx A x B  (5B.6) at x A = 12 . The implication of this observation is that, provided
ξ > 2, the system will separate spontaneously into two phases
where ξ (xi) is a dimensionless (and, in general, temperature
with compositions corresponding to the two minima, because
dependent) parameter that is a measure of the energy of A–B
such a separation corresponds to a reduction in Gibbs energy.
interactions relative to that of the A–A and B–B interactions.
This feature is developed in Topic 5C.
(For HE expressed as a molar quantity, discard the n.) This func-
tion is plotted in Fig. 5B.4; it resembles the experimental curve
in Fig. 5B.3a. If ξ < 0, then mixing is exenthalpic and the A–B
Example 5B.1 Identifying the parameter for a regular
interactions are more favourable than the A–A and B–B inter-
actions. If ξ > 0, then the mixing is endenthalpic. Because the solution
entropy of mixing has its ideal value for a regular solution, the Identify the value of the parameter ξ that would be appropriate
Gibbs energy of mixing is to model a mixture of benzene and cyclohexane at 25 °C, and
mix H mix S ideal estimate the Gibbs energy of mixing for an equimolar mixture.
   
 mix G  n RTx A x B  T {nR( x A ln x A  x B ln x B )} (5B.7)
Collect your thoughts Refer to Fig. 5B.3a and identify the
value of the maximum in the curve; then relate it to eqn 5B.6
 nRT (x A ln x A  x B ln x B   x A x B ) written as a molar quantity (HE = ξRTxAxB). For the second
part, assume that the solution is regular and that the Gibbs
Figure 5B.5 shows how ∆mixG varies with composition for energy of mixing is given by eqn 5B.7.
different values of ξ. The important feature is that for ξ > 2
The solution The experimental data plotted in Fig. 5B.3a show
a maximum in HE of 700 J mol−1 close to x=
A x=
B
1
2
. It follows
+0.5 that
2

HE 700 Jmol 1
1  
RTx A x B (8.3145 JK 1 mol 1 )  (298 K )  12  12
 1.13
HE/nRT

0
0

–1 If the solution is regarded as regular, the Gibbs energy of


mixing to form an equimolar solution, one with composition
–2 x=A x=
B
1
2
, is
–0.5
0 0.5 1
xA  mixG /n  12 RT ln 12  12 RT ln 12  (700 Jmol 1 )
  RT ln 2  (700 Jmol 1 )
Figure 5B.4 The excess enthalpy according to a model in which it
is proportional to ξxAxB, for different values of the parameter ξ.  1.72 kJmol 1  0.700 kJmol 1  1.02 kJmol 1
158 5 Simple mixtures

Self-test 5B.1 The graph in Fig. 5B.3a suggests the following


Pure
values:
liquid

Chemical potential, μ
x 0.1 0.2 0.3 0.4 0.5 0.6 0.7 0.8 0.9 Solid
E −1 Vapour
H /(J mol ) 150 350 550 680 700 690 600 500 280
Solution
Use a curve-fitting procedure to fit these data to an expression
Freezing Boiling
of the form in eqn 5B.6 written as HE/n = Ax(1 − x).
point point
Answer: The best fit is with A = 690 J mol−1 depression elevation

Tf’ Tf Tb Tb’
Temperature, T

Exercises Figure 5B.6 The chemical potential of the liquid solvent in a


solution is lower than that of the pure liquid. As a result, the
E5B.1 Calculate the Gibbs energy, entropy, and enthalpy of mixing when temperature at which the chemical potential of the solvent is
0.50 mol C6H14 (hexane) is mixed with 2.00 mol C7H16 (heptane) at 298 K.
equal to that of the solid solvent (the freezing point) is lowered,
Treat the solution as ideal.
and the temperature at which it is equal to that of the vapour
E5B.2 What proportions of hexane and heptane should be mixed (i) by mole (the boiling point) is raised. The lowering of the liquid’s chemical
fraction, (ii) by mass in order to achieve the greatest entropy of mixing? potential has a greater effect on the freezing point than on the
E5B.3 Two liquids A and B form a regular solution with ξ = 1.50. Calculate the boiling point because the slope of the solid line is shallower than
maximum value of the molar enthalpy of mixing of A and B at 298 K. that of the vapour line.

because ln xA is negative when xA < 1. Because the solute appears


5B.2 Colligative properties in neither the vapour nor the solid, the presence of the solute
has no direct influence on either the chemical potential of the
A colligative property is a physical property that depends on solvent vapour or of the solid solvent, so their chemical poten-
the relative number of solute particles present but not their tials are those of the pure solvent in its two different phases.
chemical identity (‘colligative’ denotes ‘depending on the col- How the lowering of the chemical potential of the solvent
lection’). Such properties include the lowering of vapour pres- ­affects the boiling point and the freezing point is illustrated in
sure, the elevation of boiling point, the depression of freezing Fig. 5B.6, which shows how the chemical potentials of the solid,
point, and the osmotic pressure arising from the presence of a liquid (or solution), and vapour vary with temperature. As dis-
solute. In dilute solutions these properties depend only on the cussed in Topic 4B, the slope of each line depends on the molar
number of solute particles present, not their identity. entropy of the relevant phase, and the points at which the lines
In the discussion that follows, the solvent is denoted by A intersect give the temperatures at which the two phases are in
and the solute by B. There are two assumptions. First, the solute equilibrium.
is not volatile, so it does not contribute to the vapour. Second, The addition of solute lowers the chemical potential of the
the solute does not dissolve in the solid solvent: that is, the pure liquid (from that shown by the upper line to that shown by the
solid solvent separates when the solution is frozen. The latter as- lower line in the illustration). As a result, the intersection be-
sumption is quite drastic, although it is valid for many mixtures; tween the line for the liquid in solution and for the solid moves
it can be avoided at the expense of more algebra, but that intro- to a lower temperature than is the case for the pure liquid. In
duces no new principles. In the cases of boiling and freezing, the other words, the freezing point is depressed. Similarly, the in-
pressure is taken to be 1 atm, so the boiling and freezing temper- tersection between the line for the liquid in solution and for the
atures are the normal boiling and freezing points, respectively. vapour moves to a higher temperature than is the case for the
pure liquid. That is, the boiling point is raised.
The molecular origin of the lowering of the chemical poten-
The common features of colligative
5B.2(a) tial is not the energy of interaction of the solute and solvent
properties particles, because the lowering occurs even in an ideal solu-
All the colligative properties stem from the reduction of the tion (for which the enthalpy of mixing is zero). If it is not an
chemical potential of the liquid solvent as a result of the pres- enthalpy effect, it must be an entropy effect.1 When a solute
ence of solute. For an ideal solution (one that obeys Raoult’s law, 1
More precisely, if it is not an enthalpy effect (that is, an effect arising from
Topic 5A; pA = x A pA ), the chemical potential changes from μ A changes in the entropy of the surroundings due to the transfer of energy as heat
for the pure solvent to  A   A  RT ln x A when a solute is pre- into or from them), then it must be an effect arising from the entropy of the
sent. The chemical potential is lower than for the pure solvent system.
5B The properties of solutions 159

A(g)

pA* pA μA*(g,p)

=
μA(l)

A(l) + B

Figure 5B.8 The equilibrium involved in the calculation of the


(a) (b) elevation of boiling point is between A present as pure vapour
and A in the mixture, A being the solvent and B a non-volatile
Figure 5B.7 The vapour pressure of a pure liquid represents solute.
a balance between the increase in disorder arising from
vaporization and the decrease in disorder of the surroundings.
The pressure of 1 atm is the same throughout and will not be
(a) Here the structure of the liquid is represented highly
schematically by the grid of squares. (b) When solute (the dark written explicitly. It can be shown that a consequence of this
green squares) is present, the disorder of the condensed phase relation is that the normal boiling point of the solvent is raised
is higher than that of the pure liquid, and there is a decreased and that in a dilute solution the increase is proportional to the
tendency to acquire the disorder characteristic of the vapour. mole fraction of solute.

How is that done? 5B.1 Deriving an expression for the


is present, there is an additional contribution to the entropy
of the solvent which results is a weaker tendency to form the elevation of the boiling point
vapour (Fig. 5B.7). This weakening of the tendency to form a The starting point for the calculation is the equality of the
vapour lowers the vapour pressure and hence raises the boil- chemical potentials of the solvent in the liquid and vapour
ing point. Similarly, the enhanced molecular randomness of phases, eqn 5B.8. The strategy then involves examining how the
the solution opposes the tendency to freeze. Consequently, temperature must be changed to maintain that equality when
a lower temperature must be reached before equilibrium be- solute is added. You need to follow these steps.
tween solid and solution is achieved. Hence, the freezing point
Step 1 Relate ln xA to the Gibbs energy of vaporization
is lowered.
Equation 5B.8 can be rearranged into
The strategy for the quantitative discussion of the elevation
of boiling point and the depression of freezing point is to iden-  A (g )   A (l)  vapG
tify the temperature at which, at 1 atm, one phase (the pure sol- ln x A  
RT RT
vent vapour or the pure solid solvent) has the same chemical
potential as the solvent in the solution. This temperature is the where ∆vapG is the (molar) Gibbs energy of vaporization of the
new equilibrium temperature for the phase transition at 1 atm, pure solvent (A).
and hence corresponds to the new boiling point or the new Step 2 Write an expression for the variation of ln xA with
freezing point of the solvent. ­temperature
Now differentiate both sides of the expression from Step 1 with
5B.2(b) The elevation of boiling point respect to temperature. Then use the Gibbs–Helmholtz equa-
tion (Topic 3E, (∂(G/T)/∂T)p = −H/T2) to rewrite the term on
The equilibrium of interest when considering boiling is be- the right. The result is
tween the solvent vapour and the solvent in solution, both
phases being at 1 atm (Fig. 5B.8). The equilibrium is estab- d ln x A 1 d( vapG /T )  vap H
 
lished at the temperature at which the chemical potential of dT R dT RT 2
A in the gas phase, μ A (g ), is equal to the chemical potential of
A in the liquid, μ A (l). Because the vapour is pure its chemical This expression relates the change d ln x A when solute is added
potential is μ A (g ) and in the liquid the chemical potential is to the change in temperature, dT, needed to maintain equilib-
given by eqn 5B.1,  A (l)  A (l)  RT ln x A. At equilibrium it rium. To emphasize this point it is written as
follows that
 vap H
d ln x A   dT
A (g )  A (l)  RT ln x A (5B.8) RT 2
160 5 Simple mixtures

Step 3 Find the relation between the measurable changes in ln xA Step 5 Rearrange the expression
and T by integration The calculation has shown that the presence of a solute at a
Integrate the preceding expression from xA = 1 (when ln xA = 0), mole fraction xB causes an increase in normal boiling point by
corresponding to pure solvent with boiling point T*, to some ∆Tb. After minor rearrangement of eqn 5B.9a the relation is
final mole fraction xA with boiling point T. To avoid confusing
the variables of integration with their final values, replace the RT * 2 (5B.9b)
ΔTb = Kx B K =
variable xA by x′A and T by T ′ to give ΔvapH Elevation of boiling point
[ideal solution]

ln xA 1 T  vap H
0
d ln xA  
R T T 2

dT 
Because eqn 5B.9b makes no reference to the identity of the
solute, only to its mole fraction, it follows that the elevation of
The left-hand side integrates to ln xA, which is equal to boiling point is a colligative property. The relation can be de-
ln(1 − xB). The right-hand side can be integrated if the enthalpy veloped further by noting that ΔvapH/T* is the entropy change
of vaporization is assumed to be constant over the small
of vaporization which, according to Trouton’s rule (Topic 3B),
range of temperatures involved, and so can be taken outside
is a constant for most solvents. It follows that eqn 5B.9b has the
the integral:
form ΔTb ∝ T*, implying that the higher the boiling point of
Integral A.1 the solvent the greater will be the elevation of the boiling point.
with n 2
 When xB << 1 the mole fraction of B is proportional to its
 vap H T 1
molality, b (see The chemist’s toolkit 5B.1). Equation 5B.9b can
R T T 2
ln(1  x B )   
dT 
therefore be written as
Therefore Boiling point elevation
Tb  K bb  (5B.9c)
[empirical relation]
 vap H  1 1 
ln(l  x B )   where Kb is the boiling-point constant of the solvent, which is
R  T T  
best regarded as an empirical quantity (Table 5B.1).
Step 4 Approximate the expression for dilute solutions
Suppose that the amount of solute present is so small that xB << 1; Table 5B.1 Freezing-point (Kf ) and boiling-point (Kb) constants*
the approximation ln(1 − x) ≈ −x (see Part 1 of the Resource
Substance Kf/(K kg mol−1) Kb/(K kg mol−1)
section) can then be used. It follows that
Benzene 5.12 2.53
 H 1 1 Camphor 40
x B  vap    
R T T  Phenol 7.27 3.04
Water 1.86 0.51
Finally, because the increase in the boiling point is small and
T ≈ T*, it also follows that * More values are given in the Resource section.

1 1 T  T  T  T  Tb Brief illustration 5B.2


    2
T T TT  T 2 T
The boiling-point constant of water is 0.51 K kg mol−1, so a
with ∆Tb = T − T*, the elevation of boiling point. The previous solute present at a molality of 0.10 mol kg−1 would result in an
equation then becomes elevation of boiling point of only 0.051 K. The boiling-point
constant of benzene is significantly larger, at 2.53 K kg mol−1, so
 vap H Tb
xB    (5B.9a) the elevation would be 0.25 K.
R T 2

The chemist’s toolkit 5B.1 Molality and mole fraction


Let A be the solvent and B the solute. The molality, bB, of a solute amount of solvent molecules is nA = xAn = (1 − xB)n. If the molar
is the amount (in moles) of solute molecules in a solution divid- mass of the solvent is MA then the mass of the solvent present is
ed by the mass of the solvent (in kilograms), mA: bB = nB /mA . mA = nAMA = (1 − xB)nMA. The molality of the solute is therefore
The molality is related to the mole fraction in the following way. nB x Bn xB
Consider a solution with one solute and in which the total bB   
mA (1  x B )nM A (1  x B )M A
amount of molecules of solute and solvent is n. If the mole frac-
tion of the solute is xB, the amount of solute molecules is nB = xBn. When xB << 1 it follows that bB ≈ x B /M A therefore the mole frac-
The mole fraction of solvent molecules is xA = 1 − xB, so the tion of B is proportional to its molality.
5B The properties of solutions 161

5B.2(c) The depression of freezing point


B
The equilibrium now of interest is between pure solid solvent dissolved in
A and the solution with solute present at a mole fraction xB A µB(solution)
(Fig. 5B.9). At the freezing point, the chemical potentials of A
in the two phases are equal: B(s)
µB*(s)

 (s)   (l)  RT ln x A 

A

A
(5B.10)

where μ A (s) is the chemical potential of pure solid A. The only


difference between this calculation and the last is the appearance Figure 5B.10 The equilibrium involved in the calculation of the
of the chemical potential of the solid in place of that of the va- solubility is between pure solid B and B in solution.
pour. Therefore the result can be written directly from eqn 5B.9b:

Freezing point 5B.2(d) Solubility


RT 2
Tf  K x B K   depression  (5B.11)
 fus H [ideal sollution]
Although solubility is not a colligative property (because solu-
bility varies with the identity of the solute), it may be estimated
where T* is the freezing point of the pure liquid, ∆Tf = T * − T in a similar way. When a solid solute is left in contact with a sol-
is the freezing point depression, and ∆fusH is the enthalpy of vent, it dissolves until the solution is saturated. Saturation is a
fusion of the solvent. It appears that larger depressions should state of equilibrium, with the undissolved solute in equilibrium
be observed in solvents with low enthalpies of fusion and high with the dissolved solute. Therefore, in a saturated solution
melting points, but in practice a high value of ∆fusH is typically the chemical potential of the pure solid solute, μB (s), and the
accompanied by a high Tf , so the effects tend to cancel. When chemical potential of B in solution, μB, are equal (Fig. 5B.10).
the solution is dilute, the mole fraction is proportional to the Because the latter is related to the mole fraction in the solution
molality of the solute, b, and it is common to write the last by B  B (l)  RT ln x B, it follows that
equation as
B (s)  B (l)  RT ln x B (5B.13)
Freezing point depression
Tf  K f b  (5B.12)
[empirical relation] This expression is the same as the starting equation of the last
section, except that the quantities refer to the solute B, not the
where Kf is the freezing-point constant, which, like Kb, is best solvent A. It can be used in a similar way to derive the relation
regarded as an empirical constant (Table 5B.1). between the solubility and the temperature.

Brief illustration 5B.3


How is that done? 5B.2 Deriving a relation between the
The freezing-point constant of water is 1.86 K kg mol−1, so a
solubility and the temperature
solute present at a molality of 0.10 mol kg−1 would result in a
depression of freezing point of only 0.19 K. The freezing-point In the present case, the goal is to find the mole fraction of B in
constant of camphor is significantly larger, at 40 K kg mol−1, so solution at equilibrium when the temperature is T. Therefore,
the depression would be 4.0 K. start by rearranging eqn 5B.13 into

B (s)  B (l)  G


ln x B    fus
RT RT
where ∆fusG is the (molar) Gibbs energy of fusion of pure B.
µA(l) As in the derivation of eqn 5B.9, differentiate both sides of this
A(l) + B
equation with respect to T to relate the change in composition to
=

µA*(s) the change in temperature, and then use the Gibbs–Helmholtz


A(s) equation. Finally, integrate the resulting expression from the
melting temperature of pure B (when xB = 1 and ln xB = 0) to the
temperature of interest (when xB has a value between 0 and 1):

ln x B 1 T  fus H
Figure 5B.9 The equilibrium involved in the calculation of the 0
d ln x B 
R Tf T 2
dT 
lowering of freezing point is between A present as pure solid
and A in the mixture, A being the solvent and B a solute that is where ∆fusH is the enthalpy of fusion of the solute and Tf is its
insoluble in solid A. melting point.
162 5 Simple mixtures

1 permeable to the solvent but not to the solute (Fig. 5B.12).


The osmotic pressure, Π (uppercase pi), is the pressure that
0.8 must be applied to the solution to stop the influx of solvent.
Mole fraction of B, xB

0.1 Important examples of osmosis include transport of fluids


0.6 through cell membranes, dialysis, and osmometry, the deter-
0.3 mination of molar mass by the measurement of osmotic pres-
0.4
1
sure. Osmometry is used to determine the molar masses of
macromolecules.
3
0.2 In the simple arrangement shown in Fig. 5B.13, the opposing
10 pressure arises from the column of solution that the osmosis
0
0 0.5 1
itself produces. Equilibrium is reached when the pressure due
T/Tf to that column matches the osmotic pressure. The complicating
feature of this arrangement is that the entry of solvent into the
Figure 5B.11 The variation of solubility, the mole fraction of
solution results in its dilution, and so it is more difficult to treat
solute in a saturated solution, with temperature; T f is the freezing
than the arrangement in Fig. 5B.12, in which there is no flow
temperature of the solute. Individual curves are labelled with the
value of ∆fusH/RTf .
and the concentrations remain unchanged.
The thermodynamic treatment of osmosis depends on not-
In the final step, suppose that the enthalpy of fusion of B is ing that, at equilibrium, the chemical potential of the solvent
constant over the range of temperatures of interest, and take it must be the same on each side of the membrane. The chemical
outside the integral. The result of the calculation is then potential of the solvent is lowered by the solute, but is restored
to its ‘pure’ value by the application of pressure. The challenge
ΔfusH 1 1 (5B.14)
ln x B = −
R Tf T Ideal solubility
p p+Π
This equation is plotted in Fig. 5B.11. It shows that the solubil-
ity of B decreases as the temperature is lowered from its melting
point. The illustration also shows that solutes with high melt- Pure solvent Solution
ing points and large enthalpies of fusion have low solubilities at
normal temperatures. However, the detailed content of eqn μA*(p) μA(p + Π)
5B.14 should not be treated too seriously because it is based on
highly questionable approximations, such as the ideality of the
Equal at equilibrium
solution. One aspect of its approximate character is that it fails
to predict that solutes will have different solubilities in different Figure 5B.12 The equilibrium involved in the calculation of
solvents, for no solvent properties appear in the expression. osmotic pressure, Π, is between pure solvent A at a pressure p on
one side of the semipermeable membrane and A as a component
of the mixture on the other side of the membrane, where the
Brief illustration 5B.4
pressure is p + Π.
The ideal solubility of naphthalene in benzene is calculated from
eqn 5B.14 by noting that the enthalpy of fusion of naphthalene
is 18.80 kJ mol−1 and its melting point is 354 K. Then, at 20 °C,

1.880  104 Jmol 1  1 1  Solution Height proportional


ln x naphthalene  1 
   1.32  .
8.3145 JK mol  354 K 293 K 
1  to osmotic pressure

and therefore xnaphthalene = 0.26. This mole fraction corresponds Semipermeable


to a molality of 4.5 mol kg−1 (580 g of naphthalene in 1 kg of Solvent membrane
benzene).

5B.2(e) Osmosis
Figure 5B.13 In a simple version of the osmotic pressure
The phenomenon of osmosis (from the Greek word for ‘push’) experiment, solvent is at equilibrium on each side of the
is the spontaneous passage of a pure solvent into a solution membrane when enough has passed into the solution to cause a
separated from it by a semipermeable membrane, a membrane hydrostatic pressure difference.
5B The properties of solutions 163

in this instance is to show that, provided the solution is dilute, Step 3 Evaluate the integral
the extra pressure to be exerted is proportional to the molar Suppose that the pressure range in the integration is so small
concentration of the solute in the solution. that the molar volume of the solvent is a constant. Then the
right-hand side of eqn 5B.15 simplifies to
p p
How is that done? 5B.3 Deriving a relation between the p
Vm dp  Vm 
p
dp  Vm 
osmotic pressure and the molar concentration of solute
which implies that eqn 5B.15a can be written
In Fig. 5B.12 on the left the solvent is pure and at pressure p
so its chemical potential is μ A ( p). On the right (the solution) RT ln x A  Vm   (5B.15b)
the chemical potential of the solvent is lowered by the presence
of the solute but is raised on account of the greater pressure, On the left-hand side of this expression, ln xA may be replaced
p + Π, that the solution experiences. The chemical potential on by ln(1 − xB). If it is assumed that the solution is dilute
this side is written  A (x A ,p   ), where xA is the mole fraction ln(1 − xB) ≈ −xB (see Part 1 of the Resource section), then
of the solvent in the solution. Now follow these steps, and keep
RT ln x A   RT ln(1  x B )  RTx B
in mind that because the solution is dilute (xB << 1) it may be
possible to make a number of approximations. Equation 5B.15b then becomes
Step 1 Write an expression for the chemical potential of the sol-
vent in the solution RTx B   Vm (5B.15c)

At equilibrium the chemical potential of A is the same in both Step 4 Simplify the expression for the osmotic pressure for dilute
compartments: solutions
When the solution is dilute, xB ≈ nB/nA, and therefore RTxB ≈
 A ( p)   A (x A ,p   )
RTnB/nA. Then eqn 5B.15c becomes RTnB/nA = ΠVm, which
The presence of solute is taken into account in the normal way rearranges into RTnB = nAΠVm. Next, recognize nAVm as the
by using eqn 5B.1: total volume V of the solvent, giving RTnB = ΠV. Finally, divide
through by V and note that nB/V is the molar concentration [B]
 A (x A ,p   )   A ( p   )  RT ln x A of the solute B. It follows that for ideal solutions the osmotic
pressure is given by the van ’t Hoff equation:
By combining these two expressions it follows that
(5B.16)
Π = [B]RT
van ’t Hoff equation
 A ( p)   A ( p   )  RT ln x A [Ideal solution]

and therefore
One of the most common applications of osmometry is to the
 A ( p   )   A ( p)  RT ln x A measurement of molar masses of macromolecules, such as pro-
teins and synthetic polymers. As these huge molecules dissolve
Step 2 Evaluate the effect of pressure on the chemical potential to produce solutions that are far from ideal, it is assumed that the
of the solvent van ’t Hoff equation is only the first term of a virial-like expan-
The effect of pressure is taken into account by using eqn 3E.12b, sion, much like the extension of the perfect gas equation to real
gases (Topic 1C) to take into account molecular interactions:
pf
Gm ( pf )  Gm ( pi )   Vm dp
pi
  [J]RT {1  B[J]  } Osmotic virial expansion (5B.17)

A similar relation holds for the pressure dependence of the (The solute is denoted J to avoid too many different Bs in this
chemical potential, so expression.) The additional terms take the non-ideality into
p
account; the empirical constant B is called the osmotic virial
 A ( p   )   A ( p)   Vm dp ­coefficient. When it is possible to ignore corrections beyond
p
the term depending on B, the osmotic pressure is written as
where Vm is the molar volume of the pure solvent A. On sub-
stituting  A ( p   )   A ( p)  RT ln x A into this expression and   [J]RT {1  B[J]} or  /[J]  RT  BRT[J] (5B.18)
cancelling the μ A ( p), it follows that
It follows that the osmotic virial coefficient may be obtained
from the slope, BRT, of a plot of Π/[J] against [J], as shown in
p
RT ln x A   Vm dp (5B.15a) Fig. 5B.14a.
p
164 5 Simple mixtures

Slope: BRT cmass,J/(g dm−3) 1.00 2.00 4.00 7.00 9.00


−3
(Π/Pa)/(cmass,J/g dm ) 27 35 49.2 71.4 87.2
Slope: BRT/M 2

Π/cmass,J
Π/[J]

The intercept with the vertical axis at cmass,J = 0 (which is best


found by using linear regression and mathematical software)
is at
Intercept: RT
Intercept: RT/M  /Pa
0
0 [J]
0
0 cmass,J  19.8
(a) (b) cmass,J /(g dm 3 )

Figure 5B.14 The plot and extrapolation made to analyse which rearranges into
the results of an osmometry experiment using (a) the molar
concentration and (b) the mass concentration.  /cmass,J  19.8 Pa g 1 dm3

Therefore, because this intercept is equal to RT/M,


Example 5B.2 Using osmometry to determine the molar RT RT
mass of a macromolecule M 
19.8 Pa g dm 1.98  102 Pa g 1 m 3
1 3

The osmotic pressures of solutions of a polymer, denoted J, in


water at 298 K are given below. Determine the molar mass of It follows that
the polymer.
1 J = 1 Pa m3
cmass,J/(g dm−3) 1.00 2.00 4.00 7.00 9.00
(8.3145 J K −1 mol −1 ) ×(298 K)
Π/Pa 27 70 197 500 785 M= = 1.25 ×105 g mol −1
1.98 ×10−2 Pa g −1 m3
Collect your thoughts This example is an application of The molar mass of the polymer is therefore 125 kg mol−1. Note
eqn 5B.18, but as the data are in terms of the mass concentra- that once M is known, the coefficient B can be determined
tion, that equation must first be converted. To do so, note that from the slope of the graph, which is equal to BRT/M2, as
the molar concentration [J] and the mass concentration cmass,J shown in Fig. 5B.14b.
are related by [J] = cmass,J/M, where M is the molar mass of J.
Then identify the appropriate plot and the quantity (it will turn Self-test 5B.2 The osmotic pressures of solutions of poly(vinyl
out to be the intercept on the vertical axis at cmass,J = 0) that chloride), PVC, in dioxane at 25 °C were as follows:
gives you the value of M.
cmass,J/(g dm−3) 0.50 1.00 1.50 2.00 2.50
The solution To express eqn 5B.18 in terms of the mass con- Π/Pa 33.6 35.2 36.8 38.4 40.0
centration, substitute [J] = cmass,J/M and obtain
Determine the molar mass of the polymer.
Π/[J] BRT [J] Answer: 77 kg mol−1
ΠM BRTcmass,J
= RT +
cmass,J M

Division through by M gives


Exercises
y intercept slope E5B.4 The addition of 100 g of a compound to 750 g of CCl4 lowered the
x freezing point of the solvent by 10.5 K. Calculate the molar mass of the
RT BRT compound, given that the freezing point constant of CCl4 is 30 K kg mol−1.
= + cmass,J
cmass,J M M2 −1
E5B.5 The enthalpy of fusion of anthracene is 28.8 kJ mol and its melting
point is 217 °C. Calculate its ideal solubility in benzene at 25 °C.
It follows that, by plotting Π/cmass,J against cmass,J, the results
E5B.6 At 90 °C, the vapour pressure of methylbenzene is 53.3 kPa and that of
should fall on a straight line with intercept RT/M on the verti-
1,2-dimethylbenzene is 20.0 kPa. What is the composition of a liquid mixture
cal axis at cmass,J = 0. The following values of Π/cmass,J can be that boils at 90 °C when the pressure is 0.50 atm? What is the composition of
calculated from the data: the vapour produced?
5B The properties of solutions 165

Checklist of concepts
☐ 1. The Gibbs energy of mixing of two liquids to form an ☐ 6. The elevation of boiling point is proportional to the
ideal solution is negative and the process is spontaneous molality of the solute.
at all compositions. ☐ 7. The depression of freezing point is also proportional to
☐ 2. The enthalpy of mixing for an ideal solution is zero and the molality of the solute.
the Gibbs energy of mixing is due entirely to the entropy ☐ 8. The osmotic pressure is the pressure that when applied
of mixing. to a solution prevents the influx of solvent through a
☐ 3. A regular solution is one in which the entropy of mix- semipermeable membrane.
ing is the same as for an ideal solution but the enthalpy ☐ 9. The relation of the osmotic pressure to the molar
of mixing is non-zero. concentration of the solute is given by the van ’t Hoff
☐ 4. A colligative property depends only on the number of equation and is a convenient way of determining molar
solute particles present, not their identity. mass, especially of macromolecules.
☐ 5. All the colligative properties stem from the reduction of
the chemical potential of the liquid solvent as a result of
the presence of solute.

Checklist of equations
Property Equation Comment Equation number
Gibbs energy of mixing ΔmixG = nRT(xA ln xA + xB ln xB) Ideal solutions 5B.3
Entropy of mixing ΔmixS = −nR(xA ln xA + xB ln xB) Ideal solutions 5B.4
Enthalpy of mixing ΔmixH = 0 Ideal solutions
Excess function XE = ∆mixX − ∆mixXideal Definition 5B.5
Regular solution HE = nξRTxAxB Model; SE = 0 5B.6
Elevation of boiling point ΔTb = Kbb Empirical, non-volatile solute 5B.9c
Depression of freezing point ΔTf = Kfb Empirical, solute insoluble in solid solvent 5B.12
Ideal solubility ln xB = (ΔfusH/R)(1/Tf − 1/T) Ideal solution 5B.14
van ’t Hoff equation Π = [B]RT Valid as [B] → 0 5B.16
Osmotic virial expansion Π = [J]RT{1 + B[J] + ···} Empirical 5B.17
TOPIC 5C Phase diagrams of binary
­systems: liquids

➤ Why do you need to know this material? p A*

The separation of complex mixtures by distillation is a com- Liquid


mon procedure in both the laboratory and the chemical

Pressure, p
industry. The information needed to understand the tech-
nique is contained in phase diagrams, so it is important to
be able to interpret them. pB* Vapour

Pure A
Pure B
➤ What is the key idea?
The phase diagram of a liquid mixture shows how the
0 1
composition of the liquid and vapour phases varies with Mole fraction of A, xA
temperature.
Figure 5C.1 The variation of the total vapour pressure of an ideal
binary mixture with the mole fraction of A in the liquid.
➤ What do you need to know already?
It would be helpful to review the interpretation of one-
component phase diagrams and the phase rule (Topic 4A).
This Topic also draws on Raoult’s law (Topic 5A) and the
concept of partial pressure (Topic 1A). At one point it refers
to the discussion of regular solutions (Topic 5B). xB = 1 − xA

p = pA + pB = xA pA* + x B p*B = p*B + (pA* − p*)x


B A

Total vapour pressure


One-component phase diagrams are described in Topic 4A. [ideal mixture]
(5C.2)
The phase equilibria of binary systems are more complex be-
cause the composition, the relative amounts of the two compo-
This expression shows that the total vapour pressure (at a fixed
nents, is an additional variable. However, such phase diagrams
temperature) changes linearly with the composition from pB to
provide very useful summaries of phase equilibria in both ideal
pA as xA changes from 0 to 1 (Fig. 5C.1).
and real systems. This Topic focuses on binary mixtures of
The compositions of the liquid and vapour phases that are
liquids. In this context the term ideal mixture replaces ‘ideal
in mutual equilibrium are not necessarily the same. Common
­solution’, and means a mixture of liquids that obeys Raoult’s law
sense suggests that the vapour should be richer in the more
throughout the composition range.
volatile component; this expectation can be confirmed as fol-
lows. If the mole fraction of component J in the vapour is yJ
then its partial pressure is pJ = yJp, with p the total pressure
5C.1 Vapour pressure diagrams (that relation does not suppose that the vapour is a perfect
gas). Therefore
The partial vapour pressures of the components of an ideal
mixture of two volatile liquids are related to its composition by = pA pB
yA = yB  (5C.3)
Raoult’s law (Topic 5A): p p

=pA x= pB x B pB  Provided the mixture is ideal, the partial pressures and the
A pA

(5C.1)
total pressure may be expressed in terms of the mole frac-
where pJ is the vapour pressure of pure J and xJ is the mole frac- tions in the liquid by using eqn 5C.1 for pJ and eqn 5C.2 for
tion of J in the liquid; J is either A or B. The total vapour pres- the total vapour pressure p. The result of combining these
sure p of the mixture is therefore ­relations is
5C Phase diagrams of binary s­ ystems: liquids 167

1
How is that done? 5C.1 Deriving an expression for the
Mole fraction of A in the vapour, yA
1000

0.8
50 total vapour pressure of a binary mixture in terms of the
10 composition of the vapour
0.6
4 Equation 5C.4 can be rearranged as follows to express xA in
2 terms of yA. First, multiply both sides by pB  ( pA  pB )x A to
0.4 obtain
1

0.2
pB y A  ( pA  pB )x A y A  x A pA

Then collect terms in xA:


0
0 0.2 0.4 0.6 0.8 1
Mole fraction of A in the liquid, xA pB y A  { pA  ( pB  pA ) y A }x A
Figure 5C.2 The mole fraction of A in the vapour of a binary which rearranges to
ideal mixture expressed in terms of its mole fraction in the liquid,
 
calculated using eqn 5C.4 for various values of pA /pB . pB y A
xA 
pA  ( pB  pA ) y A

This expression for xA can now be substituted into eqn 5C.2


x A pA Composition to give
yA   yB  1  yA of vapour  (5C.4)
pB  ( pA  pB )x A [ideal mixture] ( pA  pB ) pB y A
p  pB  ( pA  pB )x A  pB 
pA  ( pB  pA ) y A
Figure 5C.2 shows the composition of the vapour plotted This expression for the total pressure is then brought over a
against the composition of the liquid for various values of pA /pB . common denominator to give
Provided that pA /pB > 1 then yA > xA, meaning that the vapour
is richer than the liquid in the more volatile component. Note p⋆A pB⋆ + ( pB⋆ − pA⋆ ) pB⋆ y A + ( pA⋆ − pB⋆ ) pB⋆ y A
that if B is not volatile, so pB = 0 at the temperature of interest, p=
pA⋆ + ( pB⋆ − pA⋆ ) y A
then it makes no contribution to the vapour (yB = 0).
The boxed terms cancel to give

Brief illustration 5C.1 p⋆A pB⋆ (5C.5)


p=
pA + ( pB⋆ − pA⋆ ) y A
⋆ Total vapour pressure
[ideal mixture]
The vapour pressures of pure benzene and methylbenzene at
20 °C are 75 Torr and 21 Torr, respectively. The composition
of the vapour in equilibrium with an equimolar liquid mixture This expression is plotted in Fig. 5C.3.
 x benzene  x methylbenzene  12 is 
1
1
1
 (75 Torr )
Total vapour pressure, p/pA*

y benzene  2
 0.78 0.8
21 Torr  (75 Torr  21 Torr )  12
2
y methylbenzene  1  0.78  0.22
0.6
4
The partial vapour pressure of each component is
0.4
10
pbenzene  12  (75 Torr )  37.5 Torr
0.2 30
pmethylbenzene  21  (21 Torr )  10.5 Torr 1000
0
and the total vapour pressure is the sum of these two values, 0 0.2 0.4 0.6 0.8 1
Mole fraction of A in the vapour, yA
48 Torr.
Figure 5C.3 The dependence of the vapour pressure of the same
system as in Fig. 5C.2, but expressed in terms of the mole fraction
Equations 5C.2 and 5C.4 can be combined to express the of A in the vapour by using eqn 5C.5. Individual curves are
total vapour pressure in terms of the composition of the vapour.
 
labelled with the value of pA /pB .
168 5 Simple mixtures

the liquid. The upper line gives the composition of the vapour
Exercise in equilibrium with the liquid at each temperature. Because the
E5C.1 A mixture of two liquids A and B obeys Raoult’s law. If pA= 2 pB ,
 
vapour is richer in the more volatile component, the vapour
calculate the mole fraction of A in the vapour when the mole fraction of A in curve is displaced towards the more volatile pure component.
the liquid is 0.5. Without further calculation, would you expect this fraction
to increase or decrease if pA= 3 pB ?
  Figure 5C.4 is a phase diagram in which each composition
and temperature is represented by a point. If the point lies
below the lower line, only liquid is present. If it lies above the
upper line, only vapour is present. If the point lies between the
Temperature–composition
5C.2 two lines, both liquid and vapour are present.

diagrams
Example 5C.1 Constructing a temperature–composition
A temperature–composition diagram is a phase diagram in
diagram
which the coexistence curves (the ‘phase boundaries’) show
the composition of the phases that are in equilibrium at various The following temperature/composition data were obtained
temperatures (and a given pressure, typically 1 atm). An exam- for a mixture of octane (O) and methylbenzene (M) at 1 atm,
ple is shown in Fig. 5C.4. Note that the liquid phase lies in the where xM is the mole fraction of M in the liquid and yM the
lower part of the diagram. In the following discussion, keep in mole fraction in the vapour at equilibrium.
mind a system consisting of a liquid and its vapour confined
θ/°C 110.9 112.0 114.0 115.8 117.3 119.0 121.1 123.0
inside a cylinder fitted with a movable piston that exerts a con-
stant pressure. In this arrangement, the liquid and its vapour xM 0.908 0.795 0.615 0.527 0.408 0.300 0.203 0.097
are in equilibrium at the normal boiling point of the mixture. yM 0.923 0.836 0.698 0.624 0.527 0.410 0.297 0.164

The boiling points are 110.6 °C and 125.6 °C for M and O,


5C.2(a) The construction of the diagrams respectively. Plot the temperature–composition diagram for
the mixture.
In principle, a temperature–composition diagram could be con-
structed from vapour-pressure diagrams. That could be done by Collect your thoughts Plot the composition of each phase (on
examining the temperature dependence of the vapour pressures the horizontal axis) against the temperature (on the vertical
of the components and identifying the temperature at which the axis). The two boiling points give two further points cor-
total vapour pressure becomes equal to 1 atm. However, they are responding to xM = 1 and xM = 0, respectively. Use a spread-
normally constructed from empirical data on the composition sheet or mathematical software to fit the points of the liquid
of the phases in equilibrium at each temperature. and vapour composition curves to a polynomial of the form
a + bz + cz2 + dz3, with z = xM for the liquid line and z = yM for
Provided the ambient pressure is 1 atm, the point represent-
the vapour line.
ing liquid/vapour equilibrium for each of the pure liquid com-
ponents is its normal boiling point. The lower line shows how The solution The points are plotted in Fig. 5C.5. The two sets
the boiling point of the mixture depends on the composition of of points are fitted to the following polynomials:

130

Vapour
Vapour 125
Temperature, T

composition Vapour
Liquid
θ/°C

120

Liquid
Boiling
temperature 115
of liquid

0 Mole fraction of A, xA and yA 1 110


0 0.2 0.4 0.6 0.8 1
xM and yM
Figure 5C.4 The temperature–composition diagram
corresponding to an ideal mixture with the component A more Figure 5C.5 The plot of data and the fitted curves for a mixture of
volatile than component B. octane (O) and methylbenzene (M) in Example 5C.1.
5C Phase diagrams of binary s­ ystems: liquids 169

100 v­ apour with a composition that is the same as that of the origi-
95
nal mixture (because all of it is now vapour).
68.966 + 43.041y –14.05y2 If the point lies on the liquid curve, then the liquid and its
90 vapour are in equilibrium. Now the composition of the vapour
85 is given by the point where the tie line meets the vapour curve.
Note that the coexistence curve, representing the points at
θ/°C

80
which liquid and vapour are in equilibrium, is the liquid line:
75 the vapour line simply provides additional information.
68.707 + 17.443x +12.068x2 As a liquid mixture is heated through its boiling point and
70
beyond, the overall composition of the mixture does not change
65
0 0.2 0.4 0.6 0.8 1
as no material is gained or lost by the system. For this reason it
xheptane and yheptane can be useful to consider the horizontal axis of a temperature–
composition diagram as the overall composition. For a binary
Figure 5C.6 The plot of data and the fitted curves for a system this overall composition is given by the mole fraction of
mixture of hexane and heptane in Self-test 5C.1. A, zA, as in Fig. 5C.7.
Consider what happens when a mixture with the overall
For the liquid line:  /C  125.422  22.9494 x M  6.646 02 x M2 composition indicated by a in Fig. 5C.7 is heated. The overall
1.326 23x M3 composition does not change regardless of how much liquid
vaporizes because no material is gained or lost by the system,
For the vapour line:  /C  125.485  11.9387 y M  12.5626 y M2 so the system moves up the vertical line at a. Such a vertical line
9.365 42 y M3 is called an isopleth (from the Greek words for ‘equal abun-
dance’).
Self-test 5C.1 Repeat the analysis for the following data on hex- At a1 the liquid boils. The liquid is in equilibrium with its
ane and heptane (a polynomial with terms up to z2 is sufficient):
vapour of composition a1′, as given by the tie line. This vapour
θ/°C 68.7 72.7 77.6 83.5 90.4 94.2 98.2 is richer in the more volatile component (B), so the liquid is
depleted in B. As it becomes richer in A, the boiling point of the
xheptane 0 0.20 0.40 0.60 0.80 0.90 1
remaining liquid moves to a2. Correspondingly, the composi-
yheptane 0 0.09 0.21 0.38 0.64 0.81 1 tion of the vapour in equilibrium with that liquid changes to
Answer: Fig. 5C.6
a2′. Further heating results in the proportion of A in the liquid
increasing, the boiling point rises to a3, and the composition
of the vapour changes accordingly to a3′. The process continues
until the composition of the vapour reaches a4′, at which point
the composition of the vapour is the same as the overall com-
5C.2(b) The interpretation of the diagrams position of the mixture because all the liquid has vaporized.
In a temperature–composition diagram, such as that shown in Above that temperature, only vapour is present and it has the
Fig. 5C.4, the horizontal axis indicates two different quantities. same composition as the initial overall composition.
When interpreting the liquid line, the horizontal axis gives the
mole fraction of A in the liquid, xA; when interpreting the va-
pour line, the axis gives the mole fraction of A in the vapour,
yA. A vertical line at a particular composition xA of the liquid TA*
a4’
intersects the liquid line at the boiling temperature of the mix- a4
ture as prepared. A horizontal line drawn at this temperature a3’
Temperature, T

a2’ lV lL a3
intersects the vapour line, and from this intersection the com- a1’ a2
position yA of the vapour can be read from the horizontal axis. Vapour a1
The line connecting these two intersections is called a tie line.
Liquid
Each point on a temperature–composition diagram corre-
sponds to a mixture with a given composition and temperature. TB*
If a point lies below the liquid line the temperature is below the
boiling point of the mixture, implying that the vapour pres- zA a
0 1
sure is less than 1 atm. As a result, the entire sample is liquid.
If a point lies above the vapour line, the temperature is higher Figure 5C.7 The points of the temperature–composition diagram
than the boiling point of the mixture, implying that the vapour discussed in the text. The vertical line through a is an isopleth, a
pressure is greater than 1 atm. The entire sample is therefore a line of constant composition of the entire system.
170 5 Simple mixtures

It is also possible, by using a result known as the ‘lever rule’, Brief illustration 5C.2
to predict the abundances of liquid and vapour at any stage of
heating when the temperature and overall composition corre- The lever rule can be written nL/nV = lV/lL, and the ratio nL/nV is
spond to a point between the liquid and vapour lines. computed for each of the tie lines in Fig. 5C.7 simply by meas-
uring lV and lL from the figure. The results are

How is that done? 5C.2 Establishing the lever rule tie line a1–a1′ a2–a2′ a3–a3′ a4–a4′
lV/lL = nL/nV almost infinite 6.9 2.0 0
If the amount of A molecules in the vapour is nA,V and the
amount in the liquid phase is nA,L, the total amount of A At the temperature corresponding to tie line a1–a1′ the ratio
molecules is nA = nA,L + nA,V and likewise for B molecules. nL/nV is also almost infinite, which implies that there is only
The overall mole fraction of A is zA = (nA,L + nA,V)/(nA + nB). a trace of vapour present. As the temperature increases the
The total amount of molecules in the liquid (both A and B) is amount in the vapour increases and the amount in the liquid
nL = nA,L + nB,L, and the total amount of molecules in the vapour decreases. At the temperature corresponding to the a4–a4′ tie
is likewise nV = nA,V + nB,V. The amount of A in the liquid phase line there is no liquid present and the sample is entirely vapour.
is nLxA. Similarly, the amount of A in the vapour phase is nVyA.
The total amount of A is therefore

nA  nL x A  nV y A

The total amount of A molecules is also Exercise


E5C.2 The boiling temperature of a mixture of liquids A and B is given by the
nA  nz A  nL z A  nV z A equation

By equating these two expressions it follows that nLxA +  L /C  10.0 x A2  25.0 x A  125
nVyA = nLzA + nVzA, and therefore The vapour curve is given by

l l  V /C  10.0 y A2  5.00 y A  125


 L   V

nL (x A  z A )  nV (z A  y A ) where x A is the mole fraction of A in the liquid and y A its mole fraction in the
vapour. What is the composition of the vapour in equilibrium with liquid of
As shown in Fig. 5C.8, with xA − zAdefined as the ‘length’ composition (i) x A = 0.250 and (ii) x A = 0.750 at their respective boiling points?
lL, and zA − yA defined as the ‘length’ lV , this relation can be
expressed as the lever rule:

nLlL = nVlV
(5C.6)
Lever rule
5C.3 Distillation
In a simple distillation, the vapour is withdrawn and con-
The lever rule applies to any phase diagram, not only to liquid− densed. This technique is used to separate a volatile liquid from
vapour equilibria. a non-volatile solute or solid. The liquid simply evaporates at
its boiling point and it is not necessary to discuss the process in
terms of phase diagrams. In fractional distillation, the boiling
and condensation cycle is repeated successively. This technique
is used to separate volatile liquids and can be discussed in terms
Vapour V
Temperature, T

of phase diagrams like that in Fig. 5C.9.


nV nL

lV lL 5C.3(a) Fractional distillation


L
Consider what happens when a liquid of composition a1 in
Liquid Fig. 5C.9 is heated. It boils when the temperature reaches T2.
Then the liquid has composition a2 (the same as a1) and the va-
0 1
Overall composition, z pour (which is present only as a trace) has composition a2′. The
Figure 5C.8 The lever rule. The distances lV and lL are used to find vapour is richer in the more volatile component A. Now sup-
the proportions of the amounts of the vapour and liquid present pose that the vapour at a2′ is condensed, and then this conden-
at equilibrium. The lever rule is so called because a similar rule sate (of composition a3) is reheated. The phase diagram shows
relates the masses at two ends of a lever to their distances from a that this mixture boils at T3 and yields a vapour of composition
pivot (in that case mVlV = mLlL for balance). a3′, which is even richer in the more volatile component. That
5C Phase diagrams of binary s­ ystems: liquids 171

Vapour 5C.3(b) Azeotropes


Although many liquids have temperature–composition phase
T2 a2’ Vapour
a2 diagrams resembling the ideal version shown in Fig. 5C.9, in
Temperature, T

composition
a number of important cases there are marked deviations. A
maximum in the phase diagram (Fig. 5C.11) may occur when
T3 a3’
the favourable interactions between A and B molecules reduce
a3
Boiling the vapour pressure of the mixture below the ideal value and so
temperature raise its boiling temperature: in effect, the strong A–B interac-
Liquid
of liquid a4
a1 tions stabilize the liquid. In such cases the excess Gibbs energy,
GE (Topic 5B), is negative, corresponding to mixing being more
0 Mole fraction of A, xA and yA 1
favourable than for the components of an ideal mixture.
Figure 5C.9 The temperature–composition diagram Phase diagrams showing a minimum imply the opposite be-
corresponding to an ideal mixture with the component A more haviour (Fig. 5C.12). They indicate that the mixture is desta-
volatile than component B. The points labelled a illustrate the bilized relative to the ideal solution, the A–B interactions then
process of fractional distillation and are discussed in the text. being unfavourable. In this case, the boiling temperature is low-
ered. For such mixtures GE is positive (less favourable to mixing
than ideal), and there may be contributions from both enthalpy
vapour is drawn off, and the first drop condenses to a liquid and entropy effects.
of composition a4. The cycle can then be repeated until in due Deviations from ideality are not always so strong as to lead to
course almost pure A is obtained in the vapour and pure B re- a maximum or minimum in the phase diagram, but when they
mains in the liquid.
The efficiency of a fractionating column is expressed in
terms of ‘theoretical plates’. The number of theoretical plates a4’
a4
is the number of effective vaporization and condensation steps a3’
a3
that are required to achieve a condensate of given composition
Vapour a2’
Temperature, T

from a given distillate. a2


composition

Boiling
Brief illustration 5C.3 temperature
of liquid
To achieve the degree of separation shown in Fig. 5C.10a, the
fractionating column must correspond to three theoretical b a
plates. To achieve the same separation for the system shown in 0 Mole fraction of A, xA and yA 1
Fig. 5C.10b, in which the components have more similar nor-
mal boiling points, the fractionating column must be designed Figure 5C.11 A high-boiling azeotrope. When the liquid of
to correspond to four theoretical plates. composition a is distilled, the composition of the remaining liquid
changes towards b but no further.

Vapour
composition
1
Boiling
Temperature, T

1 temperature of liquid
Temperature, T

3
a2’ a2
2 4
a3’
a3
3 a4 a1
b a
0 Mole fraction of A, xA and yA 1
0 Composition, x 10 Composition, x 1
(a) (b)
Figure 5C.12 A low-boiling azeotrope. When the mixture at a is
Figure 5C.10 The two systems shown correspond to (a) 3, (b) 4 fractionally distilled, the vapour in equilibrium in the fractionating
theoretical plates. column moves towards b and then remains unchanged.
172 5 Simple mixtures

do there are important consequences for distillation. Consider


a liquid of composition a on the right of the maximum in
Fig. 5C.11. The vapour (at a2′) of the boiling mixture (at a2) is
richer in A. If that vapour is removed (and condensed else-
where), then the remaining liquid will move to a composition
that is richer in B, such as that represented by a3. The vapour
in equilibrium with this mixture has composition indicated by
a3′. As that vapour is removed, the composition of the boiling (a) (b)
liquid shifts to a point such as a4, and the composition of the
vapour shifts to a4′. Hence, as evaporation proceeds, the compo- Figure 5C.13 The distillation of (a) two almost completely
sition of the remaining liquid shifts towards B as A is drawn off. immiscible, but well agitated, liquids is quite different from
The boiling point of the liquid rises, and the vapour becomes (b) the joint distillation of the separated components, because in
richer in B. When so much A has been evaporated that the liq- the former, boiling occurs when the sum of the partial pressures
uid has reached the composition b, the vapour has the same equals the external pressure.
composition as the liquid. Evaporation then occurs without
change of composition. The mixture is said to form an azeo-
trope.1 When the azeotropic composition has been reached, of B dissolved in A: both liquids are saturated with the other
distillation cannot separate the two liquids because the conden- component (Fig. 5C.13a). As a result, the total vapour pres-
sate has the same composition as the azeotropic liquid. sure of the mixture is close to p  pA  pB . If the temperature is
The system shown in Fig. 5C.12 is also azeotropic, but shows raised to the value at which this total vapour pressure is equal
its azeotropy in a different way. Suppose that the initial mixture to the atmospheric pressure, boiling commences and the com-
has composition a1; then following the changes in the compo- ponents are purged from the liquid. However, this boiling re-
sition of the vapour that rises through a fractionating column sults in a vigorous agitation of the mixture, so each component
leads to the following conclusions: is kept saturated in the other component, and the purging con-
• The mixture boils at a2 to give a vapour of composition a2′. tinues as the compositions of the mixtures are restored. This
intimate contact is essential: two immiscible liquids heated
• This vapour condenses in the column to a liquid of the
in a container like that shown in Fig. 5C.13b would not boil
same composition (now marked a3).
at the same temperature. The presence of the saturated mix-
• That liquid reaches equilibrium with its vapour at a3′, tures means that the ‘mixture’ boils at a lower temperature
which condenses higher up the tube to give a liquid of the than either component would alone because boiling begins
same composition and denoted a4. when the total vapour pressure reaches 1 atm, not when either
• The fractionation therefore shifts the vapour towards the vapour pressure reaches 1 atm. This distinction is the basis of
azeotropic composition at b, but not beyond, and the steam distillation, which enables some heat-sensitive, water-­
azeotropic vapour emerges from the top of the column. insoluble organic compounds to be distilled at a lower temper-
ature than their normal boiling point. The only snag is that the
composition of the condensate is in proportion to the vapour
Brief illustration 5C.4 pressures of the components, so oils of low volatility distil in
low ­abundance.
Examples of the behaviour of the type shown in Fig. 5C.11
include (a) trichloromethane/propanone and (b) nitric acid/
water mixtures. Hydrochloric acid/water is azeotropic at 80
Exercise
per cent by mass of water and boils unchanged at 108.6 °C.
Examples of the behaviour of the type shown in Fig. 5C.12 E5C.3 Consider the temperature–composition diagram shown in Fig. 5C.11
and imagine the distillation of a mixture the composition of which is to the
include (c) dioxane/water and (d) ethanol/water mixtures.
left of the composition of the azeotrope (i.e. to the left of b). Describe how the
Ethanol/water boils unchanged when the water content is 4 per composition of the vapour and liquid will change as the mixture is boiled and
cent by mass and the temperature is 78 °C. the vapour drawn off as described in the text.

5C.3(c) Immiscible liquids 5C.4 Liquid–liquid phase diagrams


Consider the distillation of two nearly completely immiscible
liquids, such as octane and water. At equilibrium, there is a Now consider the temperature–composition diagrams for sys-
tiny amount of A dissolved in B, and similarly a tiny amount tems that consist of pairs of partially miscible liquids, which
are liquids that do not mix in all proportions at all tempera-
1
The name comes from the Greek words for ‘boiling without changing’. tures. An example is hexane and nitrobenzene.
5C Phase diagrams of binary s­ ystems: liquids 173

5C.4(a) Phase separation Example 5C.2 Interpreting a liquid–liquid phase diagram


Suppose a small amount of a liquid B is added to a sample of The phase diagram for the system nitrobenzene/hexane at 1 atm
another liquid A at a temperature T ′. Liquid B dissolves com- is shown in Fig. 5C.15. A mixture of 50 g of hexane (0.59 mol
pletely, and the binary system remains a single phase. As more C6H14) and 50 g of nitrobenzene (0.41 mol C6H5NO2) was pre-
B is added, a stage comes at which no more dissolves. The sam- pared at 290 K. What are the compositions of the phases, and in
ple now consists of two phases in equilibrium with each other. what proportions do they occur? To what temperature must the
The more abundant phase consists mostly of A with a small sample be heated in order to obtain a single phase?
proportion of B. The less abundant phase consists mostly of B
with a small proportion of A. In the temperature–composition P=1
diagram drawn in Fig. 5C.14, the composition of the former is
294
represented by the point a′ and that of the latter by the point a″. 292

Temperature, T/K
If the overall composition is represented by point a, the rela- 290
tive abundances of the two phases can be predicted by using
the lever rule. In the case shown here. the amount of phase a′ is
almost seven times that of a″. P=2
When more B is added the overall composition a moves to
the right on the diagram. However, if the temperature is the
same, the compositions of the two phases in equilibrium re- 273
main a′ and a″; what changes is the relative proportion of the 0 0.2 0.4 0.6 0.8 1
Mole fraction of nitrobenzene, xN
two phases. As yet more B is added, composition a moves fur-
ther to the right and eventually crosses the coexistence curve
Figure 5C.15 The temperature–composition diagram for
into the one-phase region. So much B is now present that it can hexane and nitrobenzene at 1 atm, with the points and lengths
dissolve all the A and the system reverts to a single phase. The discussed in the text.
addition of more B now simply dilutes the solution, and from
then on a single phase remains. Collect your thoughts The compositions of phases in equilib-
The composition of the two phases at equilibrium varies with rium are given by the points where the tie line at the relevant
the temperature. For the system shown in Fig. 5C.14, raising temperature intersects the coexistence curve. Their propor-
the temperature increases the miscibility of A and B. The two- tions are given by the lever rule (eqn 5C.6). The temperature
phase region therefore becomes narrower because each phase at which the components are completely miscible is found by
in equilibrium is richer in its minor component: the A-rich following the isopleth upwards and noting the temperature
phase is richer in B and the B-rich phase is richer in A. The at which it enters the one-phase region of the phase diagram.
entire phase diagram can be constructed by repeating the ob- Denote hexane by H and nitrobenzene by N.
servations at different temperatures and drawing the envelope The solution The mole fraction of N in the mixture is 0.41/(0.41 +
of the two-phase region. 0.59) = 0.41. From Fig. 5C.15 it is seen that the point xN = 0.41,
T = 290 K occurs in the two-phase region. The tie line at this tem-
perature cuts the coexistence curve at xN = 0.35 (call this phase α)
and xN = 0.83 (phase β), so those are the compositions of the two
P=1 Composition phases. According to the lever rule, the ratio of amount of phase
Tuc of second
α to that of phase β is given to the ratio of the distances lβ and lα:
Composition of phase
Temperature, T

first phase n l 0.83  0.41 0.42


   7
n l 0.41  0.35 0.06
P=2
That is, there is about 7 times more hexane-rich phase than
a” nitrobenzene-rich phase. Heating the sample to 292 K takes it
a’
into the single-phase region.
a Comment. Because the phase diagram has been construct-
0 1
xB ed experimentally, these conclusions are not based on any
assumptions about ideality. They would be modified if the
Figure 5C.14 The temperature–composition diagram for a
system were subjected to a different pressure.
mixture of A and B. The region below the curve corresponds
to the compositions and temperatures at which the liquids Self-test 5C.2 Repeat the problem for 50 g of hexane and 100 g
are partially miscible. The upper critical temperature, Tuc, is the of nitrobenzene at 273 K.
temperature above which the two liquids are miscible in all Answer: xN = 0.09 and 0.95 in ratio 1:1.3; 292 K
proportions.
174 5 Simple mixtures

5C.4(b) Critical solution temperatures


+0.1
The upper critical solution temperature, Tuc (or upper conso- 3
lute temperature) of a binary liquid mixture is the highest tem- 0
perature at which phase separation occurs. Above the upper 2.5
–0.1
critical solution temperature the two components are fully mis-

ΔmixG/nRT
cible. This temperature exists because the greater thermal mo- 2
–0.2
tion overcomes any potential energy advantage in molecules of
1.5
one type clustering together. An example is the nitrobenzene/ –0.3
hexane system shown in Fig. 5C.15. 1
–0.4
The thermodynamic interpretation of the upper critical solu-
tion temperature focuses on the Gibbs energy of mixing and its –0.5
variation with temperature. The simple model of a real solution 0 0.5 1
xA
(specifically, of a regular solution) discussed in Topic 5B results
in a Gibbs energy of mixing that behaves as shown in Fig. 5C.16. Figure 5C.16 The variation with composition of the Gibbs energy
If the parameter ξ introduced in eqn 5B.6 (HE = nξRTxAxB) is of mixing of a regular solution plotted for different values of
greater than 2, the Gibbs energy of mixing has a double mini- the parameter ξ. When two minima are present in one of these
curves, the system separates into two phases with compositions
mum. The system will then achieve the minimum Gibbs energy
corresponding to the position of the local minima. This illustration
by separating into two phases with compositions that are given
is the same as Fig. 5B.5.
by the locations of these minima. If   2 there is only one mini-
mum: there is then only a single phase and mixing is complete
for all compositions. 6
The compositions corresponding to the minima are obtained
Intersection
by looking for the conditions at which (∂ΔmixG/∂xA)T,p = 0. The 4
first step in computing this derivative it to rewrite eqn 5B.7 ln[x/(1 – x)]
2
(∆mixG = nRT(xA ln xA + xB ln xB + ξxAxB)), solely in terms of xA ξ=12 3 4
by using xB = 1 − xA 0
–ξ(1 – 2x)

 Δ mixG –2

xA T, p –4 Intersection
{x A ln x A + (1− x A )ln(1 − x A ) + x A( 1− x A )}
= nRT –6
xA T, p
0 0.2 0.4 x 0.6 0.8 1

= nRT {ln x A +1 − ln(1 − x A ) −1 + (1 − 2 x A )} Figure 5C.17 The graphical procedure for solving eqn 5C.7. When
ξ < 2, the only intersection occurs at x = 0. When ξ ≥ 2, there are

{ }
two solutions (those for ξ = 3 are marked).
xA
= nRT ln + (1− 2 x A )
1− x A
2
The Gibbs-energy minima therefore occur where
xA
ln   (1  2 x A ) (5C.7)
1  xA
This equation is an example of a ‘transcendental equation’, an ξ 2.5
equation that does not have a solution that can be expressed
in a closed form. The solutions (the values of xA that satisfy the
equation) can be found numerically by using mathematical
software. An alternative procedure is to plot the terms on the
left and right against xA for a choice of values of ξ and identify- 3
ing the values of xA where the plots intersect, which is where 0 0.5 1
xA
the two expressions are equal (Fig. 5C.17). The solutions found
in this way are plotted in Fig. 5C.18. As ξ decreases, the two Figure 5C.18 The location of the coexistence curve as a function
minima move together and merge when ξ = 2. of the parameter ξ.
5C Phase diagrams of binary s­ ystems: liquids 175

Brief illustration 5C.5 Tuc


210

In the system composed of benzene and cyclohexane treated

Temperature, θ/°C
in Example 5B.1 it is established that ξ = 1.13, so a two-phase P=2
system is not expected. That is, the two components are com-
pletely miscible at this temperature. The single solution of the
equation
xA Tlc
ln  1.13(1  2 x A )  0
1  xA 61 P=1

is x A = 12 , corresponding to a single minimum of the Gibbs 0 0.2 0.4 0.6 0.8 1


energy of mixing, and there is no phase separation. Mole fraction of nicotine, xN

Figure 5C.20 The temperature–composition diagram for water


and nicotine, which has both upper and lower critical solution
Some systems show a lower critical solution temperature, temperatures. Note the high temperatures for the liquid (especially
Tlc (or lower consolute temperature), below which they mix in the water): the diagram corresponds to a sample under pressure.
all proportions and above which they form two phases. An ex-
ample is water and triethylamine (Fig. 5C.19). In this case, at molecule to avoid each other. There are two possibilities: one
low temperatures the two components are more miscible be- in which the liquids become fully miscible before they boil; the
cause they form a weak complex; at higher temperatures the other in which boiling occurs before mixing is complete.
complexes break up and the two components are less miscible. Figure 5C.21 shows the phase diagram for two components
Some systems have both upper and lower critical solution that become fully miscible before they boil. Distillation of a
temperatures. They occur because, after the weak complexes mixture of composition a1 leads to a vapour of composition
have been disrupted, leading to partial miscibility, the thermal b1, which condenses to the completely miscible single-phase
motion at higher temperatures homogenizes the mixture again, solution at b2. Phase separation occurs only when this distil-
just as in the case of ordinary partially miscible liquids. The late is cooled to a point in the two-phase liquid region, such as
most famous example is nicotine and water, which are partially b3. This description applies only to the first drop of distillate.
miscible between 61 °C and 210 °C (Fig. 5C.20). If distillation continues, the composition of the remaining liq-
uid changes. In the end, when the whole sample has evaporated
The distillation of partially
5C.4(c) and condensed, the composition is back to a1.
Figure 5C.22 shows the second possibility, in which there is
miscible liquids no upper critical solution temperature. The distillate obtained
Consider a pair of liquids that are partially miscible and form a from a liquid initially of composition a1 has composition b3 and
low-boiling azeotrope. This combination is quite common be- is a two-phase mixture. One phase has composition b3′ and the
cause both properties reflect the tendency of the two kinds of other has composition b3″.

P=1 Vapour b1
Composition
of first a2
P=2
Temperature, T

phase Composition
Temperature, T

of second P=2
phase b2
P=2
Liquid
P=1

Tlc
P=1 Liquid P=2
a1
b3
0 0.2 0.4 0.6 0.8 1
Mole fraction of triethylamine, xE 0 Mole fraction of B, xB 1

Figure 5C.19 The temperature–composition diagram for water Figure 5C.21 The temperature–composition diagram for a binary
and triethylamine. This system shows a lower critical solution system in which the upper critical solution temperature is less
temperature at 292 K. The labels indicate the interpretation of the than the boiling point at all compositions. The mixture forms a
boundaries. low-boiling azeotrope.
176 5 Simple mixtures

Collect your thoughts The area in which the point lies gives the
P=1 Vapour number of phases; the compositions of the phases are given by the
points at the intersections of the horizontal tie line with the coex-
P=2
istence curves; the relative abundances are given by the lever rule.
Temperature, T

e3
b1 a2
P=2 The solution The initial point is in the one-phase region. When
Liquid e2 heated it boils at 350 K (a2) giving a vapour of composition
P=1
xB = 0.66 (b1). The liquid gets richer in B, and the last drop (of
Liquid P=2 pure B) evaporates at 390 K. The boiling range of the liquid is
b3’ e1 b3 b3” therefore 350–390 K.
a1 If the initial vapour is drawn off, it has a composition xB = 0.66.
0 Mole fraction of B, xB 1 Cooling this vapour corresponds to moving down the xB = 0.66
isopleth. At 330 K, for instance, the liquid phase has composition
Figure 5C.22 The temperature–composition diagram for a binary xB = 0.87, the vapour xB = 0.49; their relative proportions are 1:6.
system in which boiling occurs before the two liquids are fully At 320 K the sample consists of three phases: the vapour and
miscible. two liquids. One liquid phase has composition xB = 0.30; the
other has composition xB = 0.80 in approximately equal abun-
The behaviour of a system of composition represented by dance. Further cooling moves the system into the region where
the isopleth e in Fig. 5C.22 is interesting. A system at e1 forms two liquid phases are present (but no vapour), and at 298 K the
two phases, which persist (but with changing proportions) up compositions of these are xB = 0.20 and xB = 0.90 in the ratio
to the boiling point at e2. The vapour of this mixture has the 0.82:1. As further distillate boils over, the overall composition of
same composition as the liquid (the liquid is an azeotrope). the distillate becomes richer in B. When the last drop has been
Similarly, the condensation of a vapour of composition e3 gives condensed the phase composition is the same as at the beginning.
a two-phase liquid of the same overall composition. At this Self-test 5C.3 Repeat the discussion, beginning at the point
overall composition the mixture vaporizes and condenses at a xB = 0.4, T = 298 K.
single characteristic temperature, like a single substance. The
horizontal line passing through e2 is the lowest temperature at
which vapour can be present. Below this line, two liquid phases
are present; above this line, one liquid phase (either the phase Exercises
rich in A or the phase rich in B) and vapour are present. If the
E5C.4 The figure below shows the phase diagram for two partially miscible
overall composition happens to lie on the isopleth through e, liquids, which can be taken to be that for water (A) and 2-methylpropan-1-ol
then the horizontal line indicates that three phases, the two liq- (B). Describe what will be observed when a mixture of composition xB = 0.8 is
uids and the vapour, are present in equilibrium. heated, at each stage giving the number, composition, and relative amounts of
the phases present.

Example 5C.3 Interpreting a phase diagram


State the changes that occur when a mixture of composition T1
Temperature, T

xB = 0.95 (a1) in Fig. 5C.23 is boiled and the vapour condensed. T2

398
390
Temperature, T/K

b1 0 0.2 0.4 0.6 0.8 1


350 a2 Mole fraction of B, xB
330 0.87
e2 0.49 E5C.5 Phenol and water form non-ideal liquid mixtures. When 7.32 g of
320 0.30
0.80 phenol and 7.95 g of water are mixed together at 60 °C they form two
immiscible liquid phases with mole fractions of phenol of 0.042 and 0.161.
b3’ 0.20 b3
0.90 a1 (i) Calculate the overall mole fraction of phenol in the mixture. (ii) Use the
298 lever rule to determine the relative amounts of the two phases.
b3”
0 0.66 0.95 1 E5C.6 Hexane and perfluorohexane show partial miscibility below 22.70 °C
Mole fraction of B, xB (the upper critical temperature). At 22.0 °C the two solutions in equilibrium
have x = 0.24 and x = 0.48, respectively, where x is the mole fraction of C6F14.
Figure 5C.23 The points of the phase diagram in Fig. 5C.22 Describe the phase changes that occur when perfluorohexane is added to a
that are discussed in Example 5C.3. fixed amount of hexane at (i) 23 °C, (ii) 22 °C.
5C Phase diagrams of binary s­ ystems: liquids 177

Checklist of concepts
☐ 1. Raoult’s law is used to calculate the total vapour pressure ☐ 5. Separation of a liquid mixture by fractional distillation
of an ideal mixture of two volatile liquids. involves repeated cycles of boiling and condensation.
☐ 2. A temperature–composition diagram is a phase diagram ☐ 6. An azeotrope is a liquid mixture that boils without
in which the boundaries show the composition of the change of composition.
phases that are in equilibrium at various temperatures. ☐ 7. Phase separation of partially miscible liquids may occur
☐ 3. The composition of the vapour and the liquid phase in when the temperature is below the upper critical solu-
equilibrium at a particular temperature are located at tion temperature or above the lower critical solution
each end of a tie line. temperature. A simple model of a regular solution can
☐ 4. The lever rule is used to deduce the relative abundances explain the appearance of an upper critical solution
of each phase in equilibrium. temperature.

Checklist of equations
Property Equation Comment Equation number
Composition of vapour y A  x A p /{ p  ( p  p )x A }
   
Ideal solution 5C.4
A B A B

yB = 1 − yA
Total vapour pressure p  pA pB /{ pA  ( pB  pA ) y A } Ideal solution 5C.5

Lever rule nLlL = nVlV (liquid and vapour phase at equilibrium) In general, nαlα = nβlβ for phases α and β 5C.6
TOPIC 5D Phase diagrams of binary
­systems: solids

in the fabrication of modern materials, and the formation of


➤ Why do you need to know this material? compounds. As in Topic 5C, you need to be aware of the term
Phase diagrams of solid mixtures are used widely in materi- ‘isopleth’ which refers to a vertical line, a line of constant com-
als science, metallurgy, geology, and the chemical industry position, in the phase diagram.
to summarize the composition of the various phases of
mixtures.

➤ What is the key idea? 5D.1 Eutectics


A phase diagram shows which phases are stable under
particular conditions, the composition of those phases, Consider the temperature−composition phase diagram of
and their relative abundances. a two-component system shown in Fig. 5D.1. The two solids
are assumed to be immiscible. Imagine starting with a liquid
➤ What do you need to know already? of composition a1 and then allowing it gradually to cool. These
It would be helpful to review the interpretation of liquid– changes occur:
liquid phase diagrams and the lever rule (Topic 5C). • a1 → a2. As the system reaches the coexistence curve at a2
it passes from a one-phase into a two-phase region. At a2
pure solid B starts to come out of solution, and the liquid
becomes richer in A.
Alloys and other composite materials provide the infrastruc-
ture of the modern world. Steel is essential to the fabrication • a2 → a3. Along this path more solid B forms and the
of buildings, vehicles, and machinery of all kinds. Many steels solution becomes richer in A. A tie line, such as that at
are specially formulated to withstand corrosion and to offer re- a3, gives the composition of the liquid, and the relative
quired mechanical properties. Computer chips are never pure, amounts of the solid and liquid are found by using the
single substances, but consist of substrates doped with a vari- lever rule (Topic 5C). At the temperature corresponding
ety of other materials. It is vital for the reliable manufacture of to a3 there are approximately equal amounts of each phase,
these materials to control their composition and to know how and the composition of the liquid phase is given by b3.
it depends on the conditions, such as temperature and pres-
sure. This composition and its dependence on the conditions
can be expressed by phase diagrams of the kind introduced in
a1
Topic 5C. In most cases, it is sufficient to consider phase dia-
Liquid a2
grams at temperatures lower than the boiling point of the sys- P=1
tem, and that will be the focus in this Topic.
Temperature, T

The interpretation of phase diagrams for solid mixtures fol- a3


b3
lows the same general principles as in Topic 5C. Thus, areas Liquid + A Liquid + B
in the diagram correspond to the most stable phase under the
a4
prevailing conditions. Areas between coexistence curves indi- Solid
e2
cate the phases that are both present in equilibrium, and the tie a5’ a5 a5”
line in conjunction with the lever rule indicates their relative P=2
abundances. As in Topic 5C, pressure is discarded as a variable, 0 1
Mole fraction of B, xB
and the resulting temperature−composition diagrams depict
the phase behaviour for temperature and composition as varia- Figure 5D.1 The temperature–composition phase diagram
bles. A number of special issues arise in connection with solids, for two almost immiscible solids and their completely miscible
such as the existence of ‘eutectics’, which play an important role liquids. Note the similarity to Fig. 5C.22.
5D Phase diagrams of binary s­ ystems: solids 179

• a3 → a4. Along this path more solid B forms and the liq-
uid becomes richer in A, ending at the composition given e
by e2. At the temperature corresponding to a4 the liquid a1 Liquid cooling

Temperature
freezes to give a two-phase system consisting of pure B B precipitating
and pure A (the solids are immiscible). a2 Eutectic solidifying
The isopleth (constant-composition line) at e2 in Fig. 5D.1 Solid cooling
corresponds to the eutectic composition, the mixture with the a3
Composition a4
lowest melting point.1 A liquid with the eutectic composition a5
freezes at a single temperature, without previously depositing Time
solid A or B as it cools. A solid with the eutectic composition
melts, without change of composition, at the lowest tempera- Figure 5D.2 The cooling curves for the system shown in Fig. 5D.1.
ture of any mixture. Solutions of composition to the right of e2 For isopleth a, the rate of cooling slows after a2 because solid
deposit B as they cool, and solutions to the left deposit A: only B deposits from solution in an exothermic process. There is a
the eutectic mixture (apart from pure A or pure B) solidifies at a complete halt between a3 and a4 while the eutectic solidifies. This
halt is longest for the eutectic isopleth, e. Cooling curves are used
single definite temperature without gradually unloading one or
to construct the phase diagram.
other of the components from the liquid.
The horizontal line passing though e2 represents the lowest
temperature at which liquid can be present. At temperatures
e­xothermic process that therefore retards the cooling, as is
just below the line two solid phases are present; at temperatures
seen in the figure. When the remaining liquid reaches the eu-
just above the line and in the ‘Liquid + A or B’ regions one liq-
tectic composition, the temperature remains constant until the
uid and one solid phase are present. The exception to this last
whole sample has solidified: this region of constant tempera-
point is that, as noted above, at composition e2 only liquid is
ture is the eutectic halt. If the liquid has the eutectic composi-
present.
tion e initially, the liquid cools steadily down to the freezing
One eutectic that was technologically important until re-
temperature of the eutectic. After that point there is a long eu-
placed by modern materials is a formulation of solder in which
tectic halt as the entire sample solidifies (like the freezing of a
the mass composition is about 67 per cent tin and 33 per cent
pure liquid).
lead and melts at 183 °C. The eutectic formed by 23 per cent
NaCl and 77 per cent H2O by mass melts at −21.1 °C. When
salt is added to ice under isothermal conditions (for example,
when spread on an icy road) the mixture melts if the tempera- Brief illustration 5D.1
ture is above −21.1 °C (and the eutectic composition has been
Figure 5D.3 shows the phase diagram for the binary system
achieved).2 When salt is added to ice under adiabatic condi-
silver/copper. Phase α is a solid consisting of some silver dis-
tions (for example, when added to ice in a vacuum flask) the ice
solved in copper, and phase β is a solid consisting of some
melts, which is an endothermic process and so results in a cool-
copper dissolved in silver. When a liquid of composition a is
ing of the rest of the mixture. The temperature of the system
cooled, solid phase α begins to precipitate at a1 and the sample
falls and, if enough salt is added, cooling continues down to the
solidifies completely at a2.
eutectic temperature.
Eutectic formation occurs in the great majority of binary alloy
systems, and is of great importance for the microstructure of
1200
solid materials. Although a eutectic solid is a two-phase system,
it crystallizes in a nearly homogeneous mixture of microcrys- 1000 a1 Liquid
Temperature, θ/ºC

tals. The two microcrystalline phases can be distinguished by


α
microscopy and structural techniques such as X-ray diffraction. 800 β
a2
Thermal analysis is a very useful practical way of detecting
eutectics. How it is used can be understood by considering the 600
α+β
rate of cooling down the isopleth through a1 in Fig. 5D.1. As
400
shown in Fig. 5D.2, the liquid cools steadily until it reaches
a2. From this point onwards B starts to solidify, which is an 200
a
0 20 40 60 80 100
Composition (mass% Ag)
1
The name comes from the Greek words for ‘easily melted’.
2
This temperature is close to 0 °F. It served as the definition of the lower Figure 5D.3 The phase diagram for silver/copper discussed in
point of Fahrenheit’s scale, being the lowest temperature then relatively easily Brief illustration 5D.1.
reached.
180 5 Simple mixtures

Monitoring the cooling curves at different overall compo- The pure ­compound C melts congruently, that is, the com-
sitions gives a clear indication of the structure of the phase position of the liquid it forms is the same as that of the solid
diagram. The solid–liquid boundary is given by the points at compound.
which the rate of cooling changes. The longest eutectic halt
gives the location of the eutectic composition and its melting
temperature.
Exercise
E5D.1 Methyl ethyl ether (A) and diborane, B2H6 (B), form a compound
which melts congruently at 133 K. The system exhibits two eutectics, one at
25 mol per cent B and 123 K and a second at 90 mol per cent B and 104 K.
The melting points of pure A and B are 131 K and 110 K, respectively. Sketch
5D.2 Reacting systems the phase diagram for this system. Assume negligible solid–solid solubility.

Many binary mixtures react to produce compounds, and tech-


nologically important examples of this behaviour include the
Group 13/15 (III/V) semiconductors, such as the gallium ar- 5D.3 Incongruent melting
senide system, which forms the compound GaAs. Although
three constituents are present, there are only two components In some cases the compound C is not stable as a liquid. An ex-
because GaAs is formed from the reaction Ga + As → GaAs. ample from the Na/K system, the phase diagram of which is
To illustrate some of the principles involved, consider a shown in Fig. 5D.5, is the alloy Na2K, which survives only as a
­system in which A and B react to form a compound C that solid. Consider what happens as a liquid at a1 is cooled:
­contains A and B in a 1:1 ratio. Compound C forms eutectic
• a2 → a3. A solid solution rich in Na is deposited, and the
mixtures with both A and B. The phase diagram is shown in
remaining liquid is richer in K.
Fig. 5D.4.
A system prepared by mixing an excess of B with A consists • Below a3. The sample is now entirely solid and consists of a
of C and unreacted B. This is a binary (B,C) system, which in solid solution which contains significant amounts of solid
this illustration is supposed to form a eutectic. The principal Na and of solid Na2K.
change from the eutectic phase diagram in Fig. 5D.1 is that Now consider the isopleth through b1:
the whole of the phase diagram is squeezed into the range of
compositions lying between equal amounts of A and B (xB = • b1 → b2. No obvious change occurs until the coexistence
0.5, marked C in Fig. 5D.4) and pure B. The interpretation of curve is reached at b2 when a solid solution rich in Na
the information in the diagram is obtained in the same way begins to deposit.
as for Fig. 5D.1. The solid deposited on cooling along the iso- • b2 → b3. A solid solution rich in Na deposits, but at b3 a
pleth a is the compound C. At temperatures below a4 there reaction occurs to form Na2K: this compound is formed
are two solid phases, one consisting of C and the other of B. by the K atoms diffusing into the solid Na.

b1
T1 a1 1 liq
a1 P=1
2 liq + K(s) with some Na
Liquid a2
T2 3 K(s) + K(s) with some Na
P=1 a2 1
b2 8
Temperature, T

7 4 Na2K(s) + K(s) with some Na


T2’
a3 b3 a3 5 liq + Na2K(s)
T3 2 5
3 b4 6 6 Na2K(s) + Na(s) with some K
a4 e T4
P=2 P=2 7 Na(s) + Na(s) with some K
Solid 4 8 liq + Na(s) with some K
Solid
P=2
P=2 K Na2K Na
a Composition

A C B
Composition
Figure 5D.5 The phase diagram for an actual system (sodium and
potassium) like that shown in Fig. 5D.4, but with two differences.
Figure 5D.4 The phase diagram for a system in which A and B One is that the compound is Na2K, corresponding to A2B and
react to form a compound C in which A and B are in a 1:1 ratio. not AB as in that illustration. The second is that the compound
This diagram resembles two versions of Fig. 5D.1 in each half of exists only as the solid, not as the liquid. The transformation of
the diagram. The constituent C is a true compound, not just an the compound at its melting point is an example of incongruent
equimolar mixture. melting.
5D Phase diagrams of binary s­ ystems: solids 181

• At b3, three phases are in mutual equilibrium: the liquid,


the compound Na2K, and a solid solution rich in Na. The Exercises
horizontal line representing this three-phase equilibrium E5D.2 Indicate on the phase diagram below the feature that denotes
is called a peritectic line. At this stage the liquid Na/K incongruent melting. What is the composition of the eutectic mixture and at
what temperature does it melt?
mixture is in equilibrium with a little solid Na2K, but there
is still no liquid compound. 500 a b

• b3 → b4. As cooling continues, the amount of solid

Temperature, θ/°C
400
compound increases until at b4 the liquid reaches its T1
eutectic composition. It then solidifies to give a two-
300
phase solid consisting of a solid solution rich in K and
solid Na2K. 200
T2

If the solid is reheated, the sequence of events is reversed. 100


No liquid Na2K forms at any stage because it is too unstable to 0
0 0.2 0.4 0.6 0.8
exist as a liquid. This behaviour is an example of incongruent Mole fraction of B, xB
­melting, in which a compound melts into its components and
E5D.3 Sketch the cooling curves for the isopleths a and b in the phase diagram
does not itself form a liquid phase. above.

Checklist of concepts
☐ 1. At the eutectic composition the liquid phase solidifies ☐ 3. In congruent melting the composition of the liquid a com-
without change of composition. pound forms is the same as that of the solid compound.
☐ 2. A peritectic line in a phase diagram represents an equi- ☐ 4. During incongruent melting, a compound melts into its
librium between three phases. components and does not itself form a liquid phase.
TOPIC 5E Phase diagrams of ternary
­systems

5E.1 Triangular phase diagrams


➤ Why do you need to know this material?
Ternary phase diagrams have become important in mate- The mole fractions of the three components of a ternary system
rials science as more complex materials are investigated, satisfy xA + xB + xC = 1. A phase diagram drawn as an equilat-
such as the ceramics found to have superconducting eral triangle of side 1 ensures that this property is satisfied au-
properties. tomatically. Thus, the sum of the distances to a point inside the
triangle, where the distances denoting the mole fractions are
➤ What is the key idea? measured parallel to the sides, is equal to 1 (Fig. 5E.1).
At a fixed temperature and pressure the phase diagram Figure 5E.1 shows how this approach works in practice. The
of a ternary system is conveniently represented using a side AB corresponds to xC = 0, and likewise for the other two
triangular graph. sides. Hence, each of the three sides corresponds to one of the
three binary systems (A,B), (B,C), and (C,A). An interior point
➤ What do you need to know already? corresponds to a system in which all three components are pre-
It would be helpful to review the interpretation of two- sent. The point P, for instance, represents xA = 0.50, xB = 0.10,
component phase diagrams (Topics 5C and 5D) and the xC = 0.40.
phase rule (Topic 4A). Consider the dotted line a in Fig. 5E.1 that connects the apex
A to a point on the opposite side. Moving along this line from
the base to the apex represents a series of compositions that
become progressively richer in A but for which the ratio B:C
remains constant. Any ternary system formed by adding A to a
B/C mixture with xB = 0.2 lies at some point on line a.
Many modern materials have more than two components, and
their phase diagrams need to be presented in a special way. A
ternary system has three components, so C = 3, and shows some A
0 1 A Increasing A
of the techniques involved. In terms of the phase rule (Topic
4A), F = 5 − P. If the system is restricted to constant tempera- 0.2 0.8
ture and pressure, two degrees of freedom are discarded leav- Increasing B
re

Bin
xtu

ing 3 – P degrees of freedom. If two phases are present (P = 2), 0.4 0.6
ary
mi

B
then the system has just one degree of freedom: changing the xC
A)

xA
(A

P
(C,

,B

(xA,xB,xC)
amount of one component results in changes in the amounts of
)m

0.6 Increasing C
ary

0.4
ixt

= (0.5,0.1,0.4)
Bin

the other two components. This condition is represented by an


ure

a C
area in the phase diagram. If three phases are present (P = 3), 0.8 0.2
the system has no degrees of freedom and is represented by a
single point on the phase diagram. 1 0
C 0 0.2 0.4 x 0.6 0.8 1 B
As for the phase diagrams of binary systems, lines in ter- B

nary phase diagrams represent conditions under which two Binary (B,C) mixture

phases may coexist. Two phases are in equilibrium when they Figure 5E.1 The triangular coordinates used for the discussion
are connected by tie lines, as in binary phase diagrams, and of three-component systems. Each side corresponds to a binary
the lever rule (Topic 5C) can be used to estimate their relative system. All points along the dotted line a correspond to mole
amounts. fractions of C and B in the same ratio.
5E Phase diagrams of ternary s­ ystems 183

Brief illustration 5E.1


Exercise
The following points are represented on Fig. 5E.2. E5E.1 Mark the following features on triangular coordinates: (i) the point
(0.2, 0.2, 0.6), (ii) the point (0, 0.2, 0.8), (iii) the point at which all three mole
Point xA xB xC fractions are the same.
a 0.20 0.80 0
b 0.42 0.26 0.32
c 0.80 0.10 0.10
5E.2 Ternary systems
d 0.10 0.20 0.70
Ternary phase diagrams are widely used in metallurgy and ma-
e 0.20 0.40 0.40
terials science. Although they can become quite complex, they
f 0.30 0.60 0.10
can be interpreted in much the same way as binary diagrams.
Note that the points d, e, and f have xA/xB = 0.50 and lie on a However, there is an important difference when it comes to
straight line. constructing tie lines.
In a binary diagram tie lines are horizontal and so can simply
A be drawn on the diagram, but this is no longer so for a ternary
0 1
system. In such a system the tie lines in the two-phase region
0.2
(c)
0.8 are constructed experimentally by determining the composi-
tions of the two phases that are in equilibrium, marking them
0.4 0.6 on the diagram, and then joining them with a straight line. An
xC xA exception is that for the binary system that is represented by
0.6 (b) 0.4 one of the sides of the triangle the tie line simply lies along the
side. Once the tie lines have been constructed the lever rule can
(f)
0.8
(e) (a)
0.2 be used to determine the relative amounts of the two phases.
(d)
1 0
C 0 0.2 0.4
xB
0.6 0.8 1 B 5E.2(a) Partially miscible liquids
Figure 5E.4 shows the phase diagram at room temperature for
Figure 5E.2 The points referred to in Brief illustration 5E.1.
the ternary system water (W), ethanoic acid (E), and trichloro-
methane (T). The components W and E are fully miscible, as
A single triangle represents the equilibria where the pressure are E and T, but W and T are only partially miscible.
and temperature are fixed. At a different temperature the equi- On the phase diagram the (W,E) system appears along the
librium behaviour changes and therefore a different triangular left-hand side of the triangle, and the (E,T) system appears
phase diagram describes the system. The phase diagram at a along the right-hand side. These pairs form single-phase
particular temperature may be regarded as a horizontal slice ­regions: along these sides no coexistence curves are encoun-
through a three-dimensional triangular prism, such as that tered. The (W,T) system appears along the base of the triangle,
shown in Fig. 5E.3. and for some compositions it separates into two phases. That is,
coexistence curves are encountered along this side.
A single-phase system is formed when enough E is added to
the binary (W,T) mixture. This effect is illustrated by following
the line a in Fig. 5E.4:
• a1. The system consists of two phases, one rich in W and
one rich in T. The tie line is horizontal and the relative
Temperature

amounts of the two phases can be found by using the


lever rule.
• a1 → a2. The addition of E takes the system along the line
joining a1 to the E apex. At a2 the solution still has two phas-
es, but the amount of W in the T-rich phase has increased
(represented by the point a2″) and similarly there is more T
Figure 5E.3 When temperature is included as a variable, the in the W-rich phase (a2′ ). The presence of E increases the
phase diagram becomes a triangular prism. Horizontal sections solubility of T in W, and vice versa. The phase diagram also
through the prism correspond to the triangular phase diagrams shows that there is more E in the W-rich phase than in the
being discussed and illustrated in Fig. 5E.1. T-rich phase (a2′ is closer than a2″ to the E apex).
184 5 Simple mixtures

E
0 1
A rich
0.2 0.8
P=1
a b Composition
Composition of phase 2
0.4 0.6
a4 of phase 1
xW xE
P=2
0.6 0.4 f
a3 P s o um
tion bri
a2’ p osei quili
0.8 0.2 Com in
a2 ses
a2’’ C rich pha B rich
a1
1 0
W 0 0.2 0.4 0.6 0.8 1 T Figure 5E.5 The interpretation of a triangular phase diagram. The
xT
shaded region inside the curved line consists of two phases, and
Figure 5E.4 The phase diagram at room temperature and the compositions of the two phases in equilibrium are given by
ambient pressure of the three-component system ethanoic acid the points at the ends of the tie lines (the tie lines are determined
(E), trichloromethane (T), and water (W). Only some of the tie lines experimentally).
have been drawn in the two-phase region. All points along the
line a correspond to trichloromethane and water present in the
same ratio. Brief illustration 5E.3

Figure 5E.6 is a simplified version of the phase diagram for a


• a2 → a3. At a3 two phases are present, but only a trace of stainless steel consisting of iron, chromium, and nickel. The
the T-rich phase is present (lever rule). axes denote the mass percentage compositions instead of the
• a3 → a4. Further addition of E takes the system into a one- mole fractions, but as the three percentages add up to 100 per
phase region which continues through a4 and beyond. cent, the interpretation of points in the triangle is essentially
the same as for mole fractions. The point a corresponds to the
composition 74 per cent Fe, 18 per cent Cr, and 8 per cent Ni. It
Brief illustration 5E.2 corresponds to the most common form of stainless steel, ‘18-8
stainless steel’. The composition corresponding to point b lies in
Referring to Fig. 5E.4, consider an initial mixture of water and
the two-phase region, one phase consisting of Cr and the other
trichloromethane with xW = 0.40 and xT = 0.60, and then con-
of the alloy γ-FeNi.
sider what happens as ethanoic acid is added to it. The relative
proportions of W and T remain constant, so the point repre- Cr
senting the overall composition moves along the straight line b
from xT = 0.60 on the base to the E apex. The initial composi-
tion is in a two-phase region: one phase has the composition
(xW, xT, xE) = (0.05, 0.95, 0) and the other has composition (xW, Cr +
FeNi
xT, xE) = (0.88, 0.12, 0). When sufficient E has been added to b
raise the overall mole fraction of E to 0.18 (where the line b
crosses the a2′-a2″ tie line) the system consists of two phases of
composition a2″, (0.07, 0.82, 0.11), and a2′, (0.57, 0.20, 0.23), in a FeNi
the ratio 1:3. 18/8 Stainless steel
Fe c Ni

Figure 5E.6 A simplified triangular phase diagram of the ternary


The point marked P in Fig. 5E.4 is called the plait point: at system represented by a stainless steel composed of iron,
this point the compositions of the two phases in equilibrium chromium, and nickel.
become identical. It is yet another example of a critical point.
For convenience, the general interpretation of a triangular
phase diagram is summarized in Fig. 5E.5.
Exercises
5E.2(b) Ternary solids E5E.2 Mark the following points on a ternary phase diagram for the system
NaCl/Na2SO4·10H2O/H2O: (i) 25 per cent by mass NaCl, 25 per cent
The triangular phase diagram in Fig. 5E.6 is typical of that for a Na2SO4·10H2O, and the rest H2O, (ii) the line denoting the same relative
solid alloy with varying compositions of three metals, A, B, and C. composition of the two salts but with changing amounts of water.
5E Phase diagrams of ternary s­ ystems 185

E5E.3 Refer to the ternary phase diagram in Fig. 5E.4. How many phases 0 1 H2O
are present, and what are their compositions and relative abundances, in a
mixture that contains 2.3 g of water, 9.2 g of trichloromethane, and 3.1 g of 0.2 0.8
ethanoic acid? Describe what happens when (i) water, (ii) ethanoic acid is P=1
added to the mixture.
0.4 0.6
E5E.4 Shown here is the phase diagram for the ternary system NH4Cl/
(NH4)2SO4/H2O at 25 °C. Identify the number of phases present for mixtures P=2 P=2
of compositions (i) (0.2, 0.4, 0.4), (ii) (0.4, 0.4, 0.2), (iii) (0.2, 0.1, 0.7), 0.6 0.4
(iv) (0.4, 0.16, 0.44). The numbers are mole fractions of the three components
in the order (NH4Cl, (NH4)2SO4, H2O). 0.8 0.2
P=3

NH4Cl1 0 (NH4)2SO4
0 0.2 0.4 0.6 0.8 1

Checklist of concepts
☐ 1. The phase diagram of a ternary system at a given tem- ☐ 2. A phase diagram drawn as an equilateral triangle ensures
perature and pressure can be represented as an equilat- that the property xA + xB + xC = 1 is satisfied automatically.
eral triangle in which the vertices represent the three ☐ 3. At the plait point, the compositions of the two phases in
pure components. equilibrium are identical.
TOPIC 5F Activities

If the solvent A obeys Raoult’s law, its partial vapour pressure


➤ Why do you need to know this material? is given by pA = x A pA , where xA is the mole fraction of A. The
The concept of an ideal solution is a good starting point expression for the chemical potential of A can then be written
for the discussion of mixtures, but to understand real solu- A  A  RT ln x A. The form of this relation can be preserved
tions it is important to be able to describe deviations from when the solution does not obey Raoult’s law by writing
ideal behaviour and to express them in terms of molecular Activity of solvent
A  A  RT ln aA [definition]
 (5F.1)
interactions.
The quantity aA is the activity of A, a kind of ‘effective’ mole
➤ What is the key idea?
fraction.
The expressions derived for the description of ideal behav-
Because eqn 5F.1 is true for both real and ideal solutions,
iour can be adapted for the description of real solutions by
comparison with  A   A  RT ln( pA /pA ) gives
introducing the concept of activity.
pA Activity of solvent
➤ What do you need to know already? aA = [measurement]
 (5F.2)
pA
This Topic is based on the expression for chemical potential
of a species derived from Raoult’s and Henry’s laws (Topic There is nothing mysterious about the activity of a solvent: it
5A). It also uses the formulation of a model of a regular can be determined experimentally simply by measuring the va-
solution introduced in Topic 5B. pour pressure and then using this relation.

Brief illustration 5F.1


No mixture is perfectly ideal, and techniques are needed to de-
scribe real solutions. The concept of ideality is a good starting The vapour pressure of an aqueous solution of KNO3 of con-
place for the description of mixtures and gives rise to a number centration 0.500 mol dm−3, which is due solely to the solvent
of useful expressions. The approach developed here is to retain water, at 100 °C is 99.95 kPa, and the vapour pressure of pure
the form of these expressions but re-interpret them to accom- water at this temperature is 1.00 atm (101 kPa). It follows that
modate deviations from ideal behaviour. the activity of water in this solution at this temperature is
As in other Topics in this Focus, the solvent is denoted by
99.95 kPa
A, the solute by B, and a general component by J. Equilibrium = aA = 0.990
is attained when the chemical potential of a component is the 101 kPa
same in all the phases in which it is present. Much of the devel-
opment in this Topic is based on Raoult’s law (pA = pA*xA) and
Henry’s law (pB = KBxB), which are introduced in Topic 5A. Because all solvents obey Raoult’s law more closely as the
concentration of solute approaches zero, the activity of the sol-
vent approaches the mole fraction as xA → 1:

5F.1 The solvent activity aA → x A as x A → 1 (5F.3)

In Topic 5A it is shown that the chemical potential of a liquid A convenient way of expressing this convergence is to intro-
A is given by eqn 5A.21,  A  A  RT ln( pA /pA ), where pA is duce the activity coefficient, γ (gamma), by the definition
the vapour pressure of pure A and pA is the vapour pressure of
A when it is a component of a solution. The derivation of this Activity
relation assumes that the vapour behaves as a perfect gas but aA   A x A  A  1 as x A  1 coefficient of  (5F.4)
solvent [deffinition]
makes no assumptions about the solution.
5F Activities 187

at all temperatures and pressures. With this definition, eqn 5F.1 Brief illustration 5F.2
can be rewritten
In Example 5A.4 it is established that in a mixture of acetone
Chemical potential (propanone) and chloroform (trichloromethane) at 298 K
A    RT ln x A  RT ln  A

 (5F.5)
A of solvent Kacetone = 24.5 kPa, whereas pacetone

= 46.3 kPa . It follows from
eqn 5F.7 that
The standard state of the solvent is established when xA = 1 (the
pure solvent), when γA = 1 too, and the pressure is 1 bar. 24.5 kPa
acetone


 acetone

 RT ln
46.3 kPa
24.5
 acetone

 (8.3145 JK 1 mol 1 )  (298 K)  ln
46.3
Exercise
 acetone

 1.58 kJmol 1
E5F.1 The vapour pressure of water in a saturated solution of calcium nitrate
at 20 °C is 1.381 kPa. The vapour pressure of pure water at that temperature is and the standard value differs from the value for the pure liquid
2.3393 kPa. What is the activity of water in this solution?
by −1.58 kJ mol−1.

5F.2(b) Real solutes


5F.2 The solute activity
Real solutions deviate from ideal–dilute, Henry’s law behav-
The problem with defining activity coefficients and standard iour. By analogy with the procedure adopted for the solvent, the
states for solutes is that they approach ideal–dilute (Henry’s activity aB is used in place of xB in eqn 5F.8 to give
law) behaviour as xB → 0, not as xB → 1 (corresponding to pure
Chemical potential
solute). B  B  RT ln aB

of solute [definition] (5F.9)

The standard state remains unchanged, which means that all


5F.2(a) Ideal–dilute solutions
the deviations from ideality are captured in the activity aB.
A solute B that satisfies Henry’s law has a vapour pressure given The next step is to relate the activity to measurable quanti-
by pB = KBxB, where KB is an empirical constant. In this case, the ties, as is done for the solvent by eqn 5F.2, and it can perhaps
chemical potential of B is be suspected that the relation becomes aB = pB/KB. The key to
confirming this relation is to realize that, just as for the solvent,
pB = KBxB μ ⦵B the chemical potential of B is related to its vapour pressure ac-
cording to B  B  RT ln( pB /pB ). This relation assumes that
pB K x K
μB = μB⋆ + RT ln = μB⋆ + RT ln B B = μB⋆ + RT ln ⋆B + RT ln x B the vapour is a perfect gas but makes no assumptions about the
pB* pB* pB
solution. After rewriting eqn 5F.7 as B  B  RT ln(K B /pB ), it

(5F.6) may be substituted into B  B  RT ln( pB /pB ) to give

Both KB and pB are characteristics of the solute, so the two pB K p


μB = μB⋆ + RT ln = μB – RT ln ⋆B + RT ln B⋆

boxed terms may be combined to give a new standard chemical pB⋆ pB pB


potential, μB

p
= μB + RT ln B

KB
KB Comparison of this expression with eqn 5F.9 identifies the ac-
B  B  RT ln

 (5F.7)
pB tivity aB, as anticipated, to be

It then follows that the chemical potential of a solute in an pB Activity of solute


aB =  (5F.10)
ideal–dilute solution is related to its mole fraction by KB [measurement]

As for the solvent, it is sensible to introduce an activity coef-


B  B  RT ln x B 

(5F.8)
ficient through
If the solute obeys Raoult’s law, pB = x B pB , it follows that aB   B x B
Activity coefficient of solute
 (5F.11)
K B = pB and eqn 5F.7 reduces to B  B, as expected.

[definition]
188 5 Simple mixtures

Now all the deviations from ideality are captured in the activity 1 2
Activity Activity
coefficient γB. Because the solute obeys Henry’s law as its con-

Activity, a, and activity coefficient, γ


coefficient, γ coefficient, γ
centration goes to zero. It follows that 0.8 1.6

aB  x B and  B  1 as x B  0 (5F.12) 0.6 1.2

at all temperatures and pressures. Deviations of the solute from


0.4 0.8
ideality disappear as its concentration approaches zero.
Activity, a Activity, a
0.2 0.4
Example 5F.1 Measuring activities and activity
coefficients 0 0
0 0.2 0.4 0.6 0.8 1 0 0.2 0.4 0.6 0.8 1
Use the following information to calculate the activity and (a) Mole fraction, xC (b) Mole fraction, xC
activity coefficient of trichloromethane (chloroform, C) in
Figure 5F.1 The variation of activity and activity coefficient for a
a mixture with propanone (acetone, A) at 25 °C, treating
trichloromethane/propanone (chloroform/acetone) mixture with
trichloromethane first as a solvent and then as a solute. For the
composition according to (a) Raoult’s law, (b) Henry’s law.
Henry’s law constant, use KC = 23.5 kPa (the value established
in Example 5A.4).

xC 0 0.20 0.40 0.60 0.80 1


pC/kPa 0 4.7 11 18.9 26.7 36.4 5F.2(c) Activities in terms of molalities
pA/kPa 46.3 33.3 23.3 12.3 4.9 0 The selection of a standard state is entirely arbitrary and can be
chosen in a way that suits the description of the composition
Collect your thoughts For the activity of trichloromethane of the system best. For a solute it is often convenient to express
as a solvent (the Raoult’s law activity), write aC = pC /pC and the composition as a molality, b, rather than a mole fraction
 C  aC /xC . For its activity as a solute (the Henry’s law activity), (Topic 5A). The chemical potential of the solute in an ideal so-
write aC = pC/KC and γC = aC/xC with the new activity. lution is then written
The solution Because pC = 36.4 kPa and KC = 23.5 kPa, con-
bB
struct the following tables. For instance, at xC = 0.20, in the B  B  RT ln

 (5F.13)
b

Raoult’s law case aC = (4.7 kPa)/(36.4 kPa) = 0.13 and γC =


0.13/0.20 = 0.65; likewise, in the Henry’s law case, aC = (4.7 kPa)/ where b is the standard molality (1 mol kg−1). Note that be-

(23.5 kPa) = 0.20 and γC = 0.20/0.20 = 1.0. cause according to this expression μB has its standard value
when b = 1 mol kg−1, μB has a different value from the standard

From Raoult’s law (trichloromethane regarded as the solvent):


value introduced earlier. This relation implies that as bB → 0,
xC 0 0.20 0.40 0.60 0.80 1 μB → −∞; that is, as the solution becomes more dilute, the
aC 0 0.13 0.30 0.52 0.73 1.00 solute becomes increasingly thermodynamically stable. The
practical consequence of this result is that it is very difficult to
γC 0.65 0.75 0.87 0.92 1.00
remove the last traces of a solute from a solution.
From Henry’s law (trichloromethane regarded as the solute): As before, deviations from ideality are incorporated by
­introducing a dimensionless activity aB and a dimensionless
xC 0 0.20 0.40 0.60 0.80 1 ­activity coefficient γB, and writing
aC 0 0.20 0.47 0.80 1.14 1.55
bB
γC 1 1.00 1.17 1.34 1.42 1.55 aB   B , where  B  1 as bB  0  (5F.14)
b

These values are plotted in Fig. 5F.1. Notice that γC → 1 as


at all temperatures and pressures. The standard state remains
xC → 1 in the Raoult’s law case, but that γC → 1 as xC → 0 in
unchanged in this last stage and, as before, all the deviations
the Henry’s law case.
from ideality are captured in the activity coefficient γB. The final
Self-test 5F.1 Calculate the activities and activity coeffi- expression for the chemical potential of a real solute at any mo-
cients for propanone according to the two conventions (use lality is then
pA = 46.3 kPa and KA = 24.5 kPa).
bB
B  B  RT ln aB  B  RT ln  B 
⦵ ⦵
Answer: At xA = 0.60, for instance aR = 0.50; γR = 0.84; aH = 0.95, γH = 1.59 (5F.15)
b

5F Activities 189

2 2
 xB  xA
   1


Exercises x A ln  A  x B ln  B   x A x B   x B x A   (x A  x B ) x A x B
2 2

E5F.2 Substances A and B are both volatile liquids with pA = 300 Torr ,

  xA xB
pB = 250 Torr , and KB = 200 Torr (concentration expressed as mole fraction).
When xA = 0.900, pA = 250 Torr, and pB = 25 Torr. Calculate the activities of A
and B. Use the mole fraction, Raoult’s law basis system for A and the Henry’s It follows that the activity coefficients of a regular solution with
law basis system for B. Go on to calculate the activity coefficient of A. HE = nξRTxAxB are given by what are known as the Margules
E5F.3 By measuring the equilibrium between liquid and vapour phases of a equations:
propanone(P)/methanol(M) solution at 57.2 °C at 1.00 atm, it was found that
xP = 0.400 when yP = 0.516. Calculate the activities and activity coefficients
of both components in this solution on the Raoult’s law basis. The vapour (5F.16)
pressures of the pure components at this temperature are: pP = 105 kPa and ln γA = ξ x B2 ln γB = ξ xA2
Margules equations
pM = 73.5 kPa . (xP is the mole fraction in the liquid and yP the mole fraction in
the vapour.)

Note that the activity coefficients behave correctly for dilute so-
5F.3 The activities of regular solutions lutions: γA → 1 as xB → 0 and γB → 1 as xA → 0. Also note that
A and B are treated here as equal components of a mixture, not
The concept of a regular solution is introduced in Topic 5B. as solvent and solute.
Such solutions have the same entropy of mixing as an ideal so- At this point the Margules equations can be used to write the
lution, but the enthalpy of mixing is not zero. A simple model activity of A as
for a regular solution is a mixture of liquids for which the ex-
cess enthalpy of mixing is given by eqn 5B.6 (HE = nξRTxAxB). It γA=e
ξ x B2

is possible to develop expressions for the activity coefficients in 2 2


terms of the parameter ξ. aA = γ A x A = x Aeξ x = xAeξ (1−x
B A)
(5F.17)

xB = 1 − xA

How is that done? 5F.1 Developing expressions for the


with a similar expression for aB. The activity of A, though, is the
activity coefficients of a mixture treated as a regular ratio of the vapour pressure of A in the solution to the vapour
solution pressure of pure A (eqn 5F.2, aA = pA /pA ), so
The Gibbs energy of mixing to form an ideal binary mixture is
given in eqn. 5B.3: pA  aA pA  x A e (1 xA) pA 
2
(5F.18)

 mixG  nRT (x A ln x A  x B ln x B )
This function is plotted in Fig. 5F.2, and interpreted as follows:
The corresponding expression for a real mixture is
• When ξ = 0, corresponding to an ideal solution, pA = pA x A ,
 mixG  nRT (x A ln aA  x B ln aB ) in accord with Raoult’s law.
This relation follows in the same way as for an ideal mixture but • Positive values of ξ (endenthalpic mixing, unfavourable
with activities in place of mole fractions. However, in Topic 5B A–B interactions) give vapour pressures higher than for
it is established (eqn 5B.7) that for a regular solution with an ideal solution.
HE = nξRTxAxB • Negative values of ξ (exenthalpic mixing, favourable A–B
interactions) give a vapour pressure lower than for an
 mixG  nRT {x A ln x A  x B ln x B   x A x B } ideal solution.
The last two equations can be made consistent as follows. First All the lines in Fig. 5F.2 coincide with the Raoult’s law line as
replace each activity by γJxJ: xA → 1 and the exponential function in eqn 5F.18 approaches 1.
ΔmixG = nRT(xA ln γAxA + x B ln γB x B)
When xA << 1, eqn 5F.18 is well approximated as

= nRT(xA ln xA + x B ln x B + xA ln γA + x B ln γB )
pA  x A e pA  (5F.19)
For consistency, the sum of the two boxed terms must be equal
to ξxAxB, which can be achieved by writing ln  A   x B2 and This expression has the form of Henry’s law once KA is identified
ln  B   x A2 , because then with eξ pA , which is different for each solute–solvent system.
190 5 Simple mixtures

1.5
3
5F.4 The activities of ions
2.5
Interactions between ions are so strong that the approximation
1 of replacing activities by molalities is valid only in very dilute
solutions (less than 1 mmol kg−1 in total ion concentration),
pA/pA*

2
and in precise work activities themselves must be used.
1.5
0.5 If the chemical potential of the cation M+ is denoted μ+ and
1
that of the anion X− is denoted μ−, the molar Gibbs energy of
0
the ions in the electrically neutral solution is the sum of these
–1 partial molar quantities. The molar Gibbs energy of an ideal so-
0
0 0.5 1 lution of such ions is
xA

Figure 5F.2 The vapour pressure of A above a regular mixture in Gmideal   ideal  ideal  (5F.20)
which the excess enthalpy is nξRTxAxB; the lines are labelled with
the value of ξ. An ideal solution corresponds to ξ = 0 and gives a with Jideal  J  RT ln x J. However, for a real solution of M+ and

straight line, in accord with Raoult’s law. Positive values of ξ give X− of the same molality it is necessary to write J  J  RT ln aJ

vapour pressures higher than ideal. Negative values of ξ give a with aJ = γJxJ, which implies that J  J  RT ln  J. It then fol-
ideal

lower vapour pressure. lows that

Brief illustration 5F.3


Gm        ideal  ideal  RT ln    RT ln  

 Gmideal  RT ln     (5F.21)
In Example 5B.1 it is established that ξ = 1.13 for a mixture of
benzene and cyclohexane at 25 °C. Because ξ > 0 the vapour All the deviations from ideality are contained in the last term.
pressure of the mixture is expected to be greater than its ideal
value. The total vapour pressure of the mixture is therefore
2
5F.4(a) Mean activity coefficients
p  pbenzene

x benzenee1.13(1 xbenzene )
There is no experimental way of separating the product γ+γ−
1.13(1x cyclohexane )2
p 
x
cyclohexane cyclohexane e into contributions from the cations and the anions. The best
that can be done experimentally is to assign responsibility for
This expression is plotted in Fig. 5F.3, using pbenzene

= 10.0 kPa the non-ideality equally to both kinds of ion. Therefore, the
and pcyclohexane = 10.4 kPa.

‘mean activity coefficient’ is introduced as the geometric mean
of the individual coefficients, where the geometric mean of xp
14
p
and yq is (xpyq)1/(p+q). For a 1,1-electrolyte p = 1, q = 1 and the re-
12 quired geometric mean is
Vapour pressure/kPa

10
pcyclohexane    (    )1/2 (5F.22)
8

6 The individual chemical potentials of the ions are then written


pbenzene
4
  ideal  RT ln     ideal  RT ln    (5F.23)
2
The sum of these two chemical potentials is the same as before,
0
0 0.2 0.4
xbenzene
0.6 0.8 1 eqn 5F.21, but now the non-ideality is shared equally.
To generalize this approach to the case of a compound MpXq
Figure 5F.3 The computed vapour pressure curves for a that dissolves to give a solution of p cations and q anions from
mixture of benzene and cyclohexane at 25 °C as derived in each formula unit, the molar Gibbs energy of the ions is written
Brief illustration 5F.3. as the sum of their chemical potentials:
Gm  p   q   Gmideal  pRT ln    qRT ln   (5F.24)

The mean activity coefficient can now be defined in a more


Exercise
general way as
E5F.4 A binary mixture of two liquids A and B forms a regular solution
with pA = 75 Torr, pB = 100 Torr, and ξ = −1.1 at 298 K. Calculate the partial Mean activity
pressures of A and B when xA = 0.25, and compare them to the Raoult’s    ( p q )1/ s s  p  q coefficient  (5F.25)
law values. [definition
n]
5F Activities 191

and the chemical potential of each ion written as in which the stabilization is minimized, to achieve precipitation
of ions from electrolyte solutions.
i  iideal  RT ln   (5F.26) The model leads to the result that at very low concentra-
tions the activity coefficient can be calculated from the Debye–
Hückel limiting law
5F.4(b) The Debye–Hückel limiting law
The long range and strength of the Coulombic interaction be- log    A z  z  I 1/ 2 
Debye Huckel limiting law (5F.27)

tween ions means that it is likely to be primarily responsible for


the departures from ideality in ionic solutions and to dominate where A = 0.509 for an aqueous solution at 25 °C and I is the
all the other contributions to non-ideality. This domination is dimensionless ionic strength of the solution:
the basis of the Debye–Hückel theory of ionic solutions, which Ionic strength
I  12 zi2 (bi /b )  (5F.28)

was devised by Peter Debye and Erich Hückel in 1923. The fol- [definition]
i
lowing is a qualitative account of the theory and its principal
conclusions. For a quantitative treatment, see A deeper look 5F.1, In this expression zi is the charge number of an ion i (posi-
available to read in the e-book accompanying this text. tive for cations, negative for anions) and bi is its molality. The
Oppositely charged ions attract one another. As a result, in ionic strength occurs widely, and often as its square root (as
the solution anions are more likely to be found near cations, in eqn 5F.27) wherever ionic solutions are discussed. The sum
and vice versa (Fig. 5F.4). Overall, the solution is electrically extends over all the ions present in the solution. For solutions
neutral, but near any given ion there is an excess of counter ions consisting of two types of ion at molalities b+ and b−,
(ions of opposite charge). Averaged over time, counter ions are
I  12 (b z 2  b z 2 )/b 

(5F.29)
more likely to be found near any given ion. This time-averaged,
spherical haze around the central ion, in which counter ions The ionic strength emphasizes the charges of the ions because
outnumber ions of the same charge as the central ion, has a the charge numbers occur as their squares. For example, sup-
net charge equal in magnitude but opposite in sign to that on pose the salt M2X3 dissociates to give M3+ and X2− ions in solu-
the central ion, and is called its ionic atmosphere. The energy, tion. If the molality of M2X3 is b then the ionic strength is
and therefore the chemical potential, of any given central ion is
I  12 (b z 2  b z 2 )/b

lowered as a result of its electrostatic interaction with its ionic
atmosphere. This lowering of energy appears as the difference  12 (2  b  (3)2  3  b  (2)2 )/b  15b/b
⦵ ⦵

between the molar Gibbs energy Gm and the ideal value of the
molar Gibbs energy Gmideal of the solute, and hence can be identi- Table 5F.1 summarizes the relation of ionic strength and mo-
fied with the term RT ln γ± in eqn 5F.21. The stabilization of lality in an easily usable form.
ions by their interaction with their ionic atmospheres is part of
the explanation why chemists commonly use dilute solutions,
Brief illustration 5F.4

The mean activity coefficient of 5.0 mmol kg−1 KCl(aq) at 25°C


⦵ ⦵
is calculated by writing I  12 (b  b )/b  b/b , where b is the
molality of the solution (and b+ = b− = b). Then, from eqn 5F.27,

log    0.509  (5.0  103 )1/ 2  0.03 

Hence, γ± = 0.92. The experimental value is 0.927.


Table 5F.1 Ionic strength and molality, I = kb/b
Figure 5F.4 The model underlying the Debye–Hückel theory is of
a tendency for anions to be found around cations, and of cations k X− X2− X3− X4−
to be found around anions (one such local clustering region is M+ 1 3 6 10
shown by the shaded sphere). The ions are in ceaseless motion, M2+ 3 4 15 12
and the diagram represents a snapshot of their motion. The 3+
M 6 15 9 42
solutions to which the theory applies are far less concentrated
M4+ 10 12 42 16
than shown here.
192 5 Simple mixtures

Table 5F.2 Mean activity coefficients in water at 298 K* 0



b/b KCl CaCl2
0.001 0.966 0.888 –0.02
Extended law
0.01 0.902 0.732 (Davies equation)

log γ±
0.1 0.770 0.524 –0.04
1.0 0.607 0.725 Limiting
law
* More values are given in the Resource section. –0.06

The name ‘limiting law’ is applied to eqn 5F.27 because ionic –0.08
0 4 8 12 16
solutions of moderate molalities may have activity coefficients 100I1/2
that differ from the values given by this expression, but all solu-
tions are expected to conform as b → 0. Table 5F.2 lists some Figure 5F.6 The Davies equation gives agreement with
experimental values of activity coefficients for salts of various experiment over a wider range of molalities than the limiting law
valence types. Figure 5F.5 shows some of these values plotted (shown as a dotted line), but it fails at higher molalities. The data
are for a 1,1-electrolyte.
against I1/2, and compares them with the theoretical straight
lines calculated from eqn 5F.27. The agreement at very low mo-
lalities (less than about 1 mmol kg−1, depending on charge type)
is impressive and convincing evidence in support of the model. where B is a dimensionless constant. A more flexible extension
Nevertheless, the departures from the theoretical curves above is the Davies equation proposed by C.W. Davies in 1938:
these molalities are large and show that the approximations are
valid only at very low concentrations. A z  z  I 1/ 2
log      CI Davies equation (5F.30b)
1  BI 1/ 2
5F.4(c) Extensions of the limiting law where C is another dimensionless constant. Although B can be
When the ionic strength of the solution is too high for the lim- interpreted as a measure of the closest approach of the ions, it
iting law to be valid, the activity coefficient may be estimated (like C) is best regarded as an adjustable empirical parameter.
from the extended Debye–Hückel law (sometimes called the A graph drawn on the basis of the Davies equation is shown
Truesdell–Jones equation): in Fig. 5F.6. It is clear that eqn 5F.30b accounts for some ac-
tivity coefficients over a moderate range of dilute solutions (up
A z  z  I 1/ 2 Extended to about 0.1 mol kg−1); nevertheless it remains very poor near
log      (5F.30a)
1  BI 1/ 2 
Debye Huckel law 1 mol kg−1.
Current theories of activity coefficients for ionic solutes take
an indirect route. They set up a theory for the dependence of
0
the activity coefficient of the solvent on the concentration of the
(+1,–1) solute, and then use the Gibbs–Duhem equation (eqn 5A.12a,
–0.05 nAdμA + nBdμB = 0) to estimate the activity coefficient of the sol-
ute. The results are reasonably reliable for solutions with mo-
(+2,–1)
lalities greater than about 0.1 mol kg−1 and are valuable for the
log γ±

–0.1 NaCl discussion of mixed salt solutions, such as sea water.


MgCl2
–0.15
MgSO4
Exercises
(+2,–2)
−1
E5F.5 Calculate the ionic strength of a solution that is 0.10 mol kg in KCl(aq)
−1
–0.2 and 0.20 mol kg in CuSO4(aq).
0 2 4 6 8 10
100I 1/2 E5F.6 Calculate the masses of (i) Ca(NO3)2 and, separately, (ii) NaCl to add to
a 0.150 mol kg−1 solution of KNO3(aq) containing 500 g of solvent to raise its
Figure 5F.5 An experimental test of the Debye–Hückel limiting ionic strength to 0.250.
law. Although there are marked deviations for moderate ionic E5F.7 Estimate the mean ionic activity coefficient of CaCl2 in a solution that is
strengths, the limiting slopes (shown as lines) as I → 0 are 0.010 mol kg−1 CaCl2(aq) and 0.030 mol kg−1 NaF(aq) at 25 °C.
in good agreement with the theory, so the law can be used E5F.8 The mean activity coefficients of HBr in three dilute aqueous solutions
for extrapolating data to very low molalities. The numbers in at 25 °C are 0.930 (at 5.00 mmol kg−1), 0.907 (at 10.0 mmol kg−1), and 0.879 (at
parentheses are the charge numbers of the ions. 20.0 mmol kg−1). Estimate the value of B in the Davies equation.
5F Activities 193

Table 5F.3 Activities and standard states: a summary*

Component Basis Standard state Activity Limits


Solid or liquid Pure, 1 bar a=1
Solvent Raoult Pure solvent, 1 bar a = p/p*, a = γx γ → 1 as x → 1 (pure solvent)
Solute Henry (1) A hypothetical state of the pure solute a = p/K, a = γx γ → 1 as x → 0
⦵ ⦵
(2) A hypothetical state of the solute at molality b a = γb/b γ → 1 as b → 0
Gas Fugacity† Pure, a hypothetical state of 1 bar and behaving as a perfect gas f = γp γ → 1 as p → 0

* In each case, μ = μ + RT ln a.


Fugacity is discussed in A deeper look 5F.2, available to read in the e-book accompanying this text.

Checklist of concepts
☐ 1. The activity is an effective concentration that preserves ☐ 5. Mean activity coefficients apportion deviations from
the form of the expression for the chemical potential. ideality equally to the cations and anions in an ionic
See Table 5F.3. solution.
☐ 2. The chemical potential of a solute in an ideal–dilute ☐ 6. An ionic atmosphere is the time-averaged accumulation
solution is defined on the basis of Henry’s law. of counter ions that exists around an ion in s­ olution.
☐ 3. The activity of a solute takes into account departures ☐ 7. The Debye–Hückel theory ascribes deviations from
from Henry’s law behaviour. ideality to the Coulombic interaction of an ion with the
☐ 4. The Margules equations relate the activities of the com- ionic atmosphere around it.
ponents of a model regular solution to its composition. ☐ 8. The Debye–Hückel limiting law is extended by includ-
They lead to expressions for the vapour pressures of the ing two further empirical constants.
components of a regular solution.

Checklist of equations
Property Equation Comment Equation number
Chemical potential of solvent A    RT ln aA

A
Definition 5F.1

Activity of solvent aA = pA /p 
A
aA → xA as xA → 1 5F.2

Activity coefficient of solvent aA = γAxA γA → 1 as xA → 1 5F.4



Chemical potential of solute B  B  RT ln aB

Definition 5F.9

Activity of solute aB = pB/KB aB → xB as xB → 0 5F.10


Activity coefficient of solute aB = γBxB γB → 1 as xB → 0 5F.11
Margules equations ln  A   x , ln  B   x
2
B
2
A
Regular solution 5F.16
 (1 x A )2
Vapour pressure pA  p x A e 
A
Regular solution 5F.18

Mean activity coefficient    (  ) p q 1/s


  s  pq Definition 5F.25
1/2
Debye–Hückel limiting law log γ± = −A|z+z−|I Valid as I → 0 5F.27

z
2
Ionic strength I 1
2 i (bi /b ) 
Definition 5F.28
i
1/2 1/2
Davies equation log γ± = −A|z+z−|I /(1 + BI ) + CI A, B, C empirical constants 5F.30b
194 5 Simple mixtures

FOCUS 5 Simple mixtures

To test your understanding of this material, work through Selected solutions can be found at the end of this Focus in
the Exercises, Additional exercises, Discussion questions, and the e-book. Solutions to even-numbered questions are available
Problems found throughout this Focus. online only to lecturers.

TOPIC 5A The thermodynamic description of mixtures


Discussion questions
D5A.1 Explain the concept of partial molar quantity, and justify the remark D5A.3 Are there any circumstances under which two (real) gases will not mix
that the partial molar properties of a solute depend on the properties of the spontaneously?
solvent too.
D5A.4 Explain how Raoult’s law and Henry’s law are used to specify the
D5A.2 Explain how thermodynamics relates non-expansion work to a change chemical potential of a component of a mixture.
in composition of a system.
D5A.5 Explain the molecular origin of Raoult’s law and Henry’s law.

Additional exercises
E5A.9 A polynomial fit to measurements of the total volume of a binary E5A.16 At 25 °C, the mass density of a 50 per cent by mass ethanol–water
mixture of A and B is solution is 0.914 g cm−3. Given that the partial molar volume of water in the
solution is 17.4 cm3 mol−1, calculate the partial molar volume of the ethanol.
v  778.55  22.5749 x  0.568 92 x 2  0.010 23x 3  0.002 34 x 4
E5A.17 At 20 °C, the mass density of a 20 per cent by mass ethanol–water
where v = V/cm3, x = nB/mol, and nB is the amount of B present. Derive an
solution is 968.7 kg m−3. Given that the partial molar volume of ethanol in the
expression for the partial molar volume of B.
solution is 52.2 cm3 mol−1, calculate the partial molar volume of the water.
E5A.10 At 18 °C the total volume V of a solution formed from MgSO4 and
E5A.18 At 310 K, the partial vapour pressures of a substance B dissolved in a
1.000 kg of water fits the expression v = 1001.21 + 34.69(x − 0.070)2, where
⦵ liquid A are as follows:
v = V/cm3 and x = b/b . Calculate the partial molar volumes of the salt and
the solvent in a solution of molality 0.050 mol kg−1. xB 0.010 0.015 0.020
E5A.11 Suppose that nA = 0.22nB and a small change in composition results in pB/kPa 82.0 122.0 166.1
μA changing by δμA = −15 J mol−1, by how much will μB change?
3
E5A.12 Consider a container of volume 250 cm that is divided into two Show that the solution obeys Henry’s law in this range of mole fractions, and
compartments of equal size. In the left compartment there is argon at 100 kPa calculate Henry’s law constant at 310 K.
and 0 °C; in the right compartment there is neon at the same temperature E5A.19 Calculate the molar solubility of nitrogen in benzene exposed to air at
and pressure. Calculate the entropy and Gibbs energy of mixing when the 25 °C; the partial pressure of nitrogen in air is calculated in Example 1A.2 of
partition is removed. Assume that the gases are perfect. Topic 1A.
E5A.13 At 90 °C the vapour pressure of 1,2-dimethylbenzene is 20 kPa and that E5A.20 Calculate the molar solubility of methane at 1.0 bar in benzene at
of 1,3-dimethylbenzene is 18 kPa. What is the composition of the vapour of an 25 °C.
equimolar mixture of the two components?
E5A.21 The mole fractions of N2 and O2 in air at sea level are approximately
E5A.14 The partial molar volumes of propanone (acetone) and 0.78 and 0.21. Calculate the molalities of the solution formed in an open flask
trichloromethane (chloroform) in a mixture in which the mole fraction of of water at 25 °C.
CHCl3 is 0.4693 are 74.166 cm3 mol−1 and 80.235 cm3 mol−1, respectively.
What is the volume of a solution of mass 1.000 kg? E5A.22 A water carbonating plant is available for use in the home and operates
by providing carbon dioxide at 5.0 atm. Estimate the molar concentration of
E5A.15 The partial molar volumes of two liquids A and B in a mixture CO2 in the carbonated water it produces.
in which the mole fraction of A is 0.3713 are 188.2 cm3 mol−1 and
176.14 cm3 mol−1, respectively. The molar masses of the A and B are E5A.23 A water carbonating plant is available for use in the home and operates
241.1 g mol−1 and 198.2 g mol−1. What is the volume of a solution of mass by providing carbon dioxide at 2.0 atm. Estimate the molar concentration of
1.000 kg? CO2 in the carbonated water it produces.
Exercises and problems 195

Problems
P5A.1 The experimental values of the partial molar volume of a salt in water to be perfect and calculate the partial pressures of the two components. Plot
are found to fit the expression vB = 5.117 + 19.121x1/2, where them against their respective mole fractions in the liquid mixture and find the
vB = VB/(cm3 mol−1) and x is the numerical value of the molality of Henry’s law constants for the two components.

B (x = b/b ). Use the Gibbs–Duhem equation to derive an equation for the
molar volume of water in the solution. The molar volume of pure water at the xA 0 0.0898 0.2476 0.3577 0.5194 0.6036
same temperature is 18.079 cm3 mol−1. yA 0 0.0410 0.1154 0.1762 0.2772 0.3393
P5A.2 Use the Gibbs–Duhem equation to show that the partial molar volume p/kPa 36.066 34.121 30.900 28.626 25.239 23.402
(or any partial molar property) of a component B can be obtained if the
partial molar volume (or other property) of A is known for all compositions
xA 0.7188 0.8019 0.9105 1
up to the one of interest. Do this by proving that
yA 0.4450 0.5435 0.7284 1
VA xA
VB  VB   dVA p/kPa 20.6984 18.592 15.496 12.295
VA 1  xA
where the xA are functions of the VA. Use the following data (which are for P5A.5 The mass densities of aqueous solutions of copper(II) sulfate at 20 °C
298 K) to evaluate the integral graphically to find the partial molar volume of were measured as set out below. Determine and plot the partial molar volume
propanone at x(CHCl3) = 0.500. of CuSO4 in the range of the measurements.

x(CHCl3) 0 0.194 0.385 0.559 0.788 0.889 1.000 m(CuSO4)/g 5 10 15 20


3
Vm/(cm mol ) −1
73.99 75.29 76.50 77.55 79.08 79.82 80.67 ρ/(g cm−3) 1.051 1.107 1.167 1.230

where m(CuSO4) is the mass of CuSO4 dissolved in 100 g of solution.


P5A.3 Consider a gaseous mixture with mass percentage composition 75.5 (N2),
23.2 (O2), and 1.3 (Ar). (a) Calculate the entropy of mixing when the mixture P5A.6 Haemoglobin, the red blood protein responsible for oxygen transport,
is prepared from the pure (and perfect) gases. (b) Air may be taken as a binds about 1.34 cm3 of oxygen per gram. Normal blood has a haemoglobin
mixture with mass percentage composition 75.52 (N2), 23.15 (O2), 1.28 (Ar), concentration of 150 g dm−3. Haemoglobin in the lungs is about 97 per cent
and 0.046 (CO2). What is the change in entropy from the value calculated in saturated with oxygen, but in the capillary is only about 75 per cent saturated.
part (a)? What volume of oxygen is given up by 100 cm3 of blood flowing from the
lungs in the capillary?
P5A.4 For a mixture of methylbenzene (A) and butanone in equilibrium at
303.15 K, the following table gives the mole fraction of A in the liquid phase, P5A.7 Use the data from Example 5A.1 to determine the value of b at which VE
xA, and in the gas phase, yA, as well as the total pressure p. Take the vapour has a minimum value.

TOPIC 5B The properties of solutions


Discussion questions
D5B.1 Explain what is meant by a regular solution; what additional features D5B.5 Why are freezing-point constants typically larger than the
distinguish a real solution from a regular solution? corresponding boiling-point constants of a solvent?
D5B.2 Would you expect the excess volume of mixing of oranges and melons D5B.6 Explain the origin of osmosis in terms of the thermodynamic and
to be positive or negative? molecular properties of a mixture.
D5B.3 Explain the physical origin of colligative properties. D5B.7 Colligative properties are independent of the identity of the solute.
Why, then, can osmometry be used to determine the molar mass of a solute?
D5B.4 Identify the feature that accounts for the difference in boiling-point
constants of water and benzene.

Additional exercises
E5B.7 Predict the partial vapour pressure of HCl above its solution in E5B.10 The vapour pressure of 2-propanol is 50.00 kPa at 338.8 °C, but it fell
liquid germanium tetrachloride of molality 0.10 mol kg−1. For data, see to 49.62 kPa when 8.69 g of a non-volatile organic compound was dissolved in
Exercise E5A.7. 250 g of 2-propanol. Calculate the molar mass of the compound.
E5B.8 Predict the partial vapour pressure of the component B above its E5B.11 The addition of 5.00 g of a compound to 250 g of naphthalene lowered
solution in A in Exercise E5A.18 when the molality of B is 0.25 mol kg−1. the freezing point of the solvent by 0.780 K. Calculate the molar mass of the
The molar mass of A is 74.1 g mol−1. compound.
3
E5B.9 The vapour pressure of benzene is 53.3 kPa at 60.6 °C, but it fell to E5B.12 Estimate the freezing point of 200 cm of water sweetened by the
51.5 kPa when 19.0 g of a non-volatile organic compound was dissolved in addition of 2.5 g of sucrose. Treat the solution as ideal.
500 g of benzene. Calculate the molar mass of the compound.
196 5 Simple mixtures

3
E5B.13 Estimate the freezing point of 200 cm of water to which 2.5 g of phase when the mole fraction of B is 0.066 on the assumption that the
sodium chloride has been added. Treat the solution as ideal. conditions of the ideal–dilute solution are satisfied at this concentration.
E5B.14 The osmotic pressure of an aqueous solution at 300 K is 120 kPa. E5B.21 At 90 °C, the vapour pressure of 1,2-dimethylbenzene is 20 kPa
Estimate the freezing point of the solution. and that of 1,3-dimethylbenzene is 18 kPa What is the composition of a
liquid mixture that boils at 90 °C when the pressure is 19 kPa? What is the
E5B.15 The osmotic pressure of an aqueous solution at 288 K is 99.0 kPa.
composition of the vapour produced?
Estimate the freezing point of the solution.
E5B.22 The vapour pressure of pure liquid A at 300 K is 76.7 kPa and that of
E5B.16 Calculate the Gibbs energy, entropy, and enthalpy of mixing when
pure liquid B is 52.0 kPa. These two compounds form ideal liquid and gaseous
1.00 mol C6H14 (hexane) is mixed with 1.00 mol C7H16 (heptane) at 298 K.
mixtures. Consider the equilibrium composition of a mixture in which the
Treat the solution as ideal.
mole fraction of A in the vapour is 0.350. Calculate the total pressure of the
E5B.17 What proportions of benzene and ethylbenzene should be mixed (i) by vapour and the composition of the liquid mixture.
mole fraction, (ii) by mass in order to achieve the greatest entropy of mixing?
E5B.23 The vapour pressure of pure liquid A at 293 K is 68.8 kPa and that of
E5B.18 Predict the ideal solubility of lead in bismuth at 280 °C given that its pure liquid B is 82.1 kPa. These two compounds form ideal liquid and gaseous
melting point is 327 °C and its enthalpy of fusion is 5.2 kJ mol−1. mixtures. Consider the equilibrium composition of a mixture in which the
mole fraction of A in the vapour is 0.612. Calculate the total pressure of the
E5B.19 A dilute solution of bromine in carbon tetrachloride behaves as an
vapour and the composition of the liquid mixture.
ideal dilute solution. The vapour pressure of pure CCl4 is 33.85 Torr at 298 K.
The Henry’s law constant when the concentration of Br2 is expressed as a mole E5B.24 It is found that the boiling point of a binary solution of A and B with
fraction is 122.36 Torr. Calculate the vapour pressure of each component, xA = 0.6589 is 88 °C. At this temperature the vapour pressures of pure A and B
the total pressure, and the composition of the vapour phase when the mole are 127.6 kPa and 50.60 kPa, respectively. (i) Is this solution ideal? (ii) What is
fraction of Br2 is 0.050, on the assumption that the conditions of the ideal the initial composition of the vapour above the solution?
dilute solution are satisfied at this concentration.
E5B.25 It is found that the boiling point of a binary solution of A and B with
E5B.20 The vapour pressure of a pure liquid A is 23 kPa at 20 °C and the xA = 0.4217 is 96 °C. At this temperature the vapour pressures of pure A and B
Henry’s law constant of B in liquid A is 73 kPa. Calculate the vapour pressure are 110.1 kPa and 76.5 kPa, respectively. (i) Is this solution ideal? (ii) What is
of each component, the total pressure, and the composition of the vapour the initial composition of the vapour above the solution?

Problems
P5B.1 Potassium fluoride is very soluble in glacial acetic acid (ethanoic acid) Determine the extent to which the data fit the exponential S = S0eτ/T and
and the solutions have a number of unusual properties. In an attempt to obtain values for S0 and τ. Express these constants in terms of properties of
understand them, freezing-point depression data were obtained by taking a the solute.
solution of known molality and then diluting it several times (J. Emsley,
P5B.5 The excess Gibbs energy of solutions of methylcyclohexane (MCH) and
J. Chem. Soc. A, 2702 (1971)). The following data were obtained:
tetrahydrofuran (THF) at 303.15 K were found to fit the expression
−1
b/(mol kg ) 0.015 0.037 0.077 0.295 0.602 G E  RTx(1  x ){0.4857  0.1077(2 x  1)  0.0191(2 x  1)2 }
ΔT/K 0.115 0.295 0.470 1.381 2.67 where x is the mole fraction of MCH. Calculate the Gibbs energy of mixing
when a mixture of 1.00 mol MCH and 3.00 mol THF is prepared.
Calculate the apparent molar mass of the solute and suggest an interpretation.
Use ΔfusH = 11.4 kJ mol−1 and Tf = 290 K for glacial acetic acid. P5B.6 The excess Gibbs energy of a certain binary mixture is equal to
P5B.2 In a study of the properties of an aqueous solution of Th(NO3)4 by gRTx(1 − x) where g is a constant and x is the mole fraction of a solute B. Find
A. Apelblat, D. Azoulay, and A. Sahar (J. Chem. Soc. Faraday Trans., I, 1618, an expression for the chemical potential of B in the mixture and sketch its
(1973)), a freezing-point depression of 0.0703 K was observed for an aqueous dependence on the composition.
solution of molality 9.6 mmol kg−1. What is the apparent number of ions per P5B.7 The molar mass of a protein was determined by dissolving it in water,
formula unit? and measuring the height, h, of the resulting solution drawn up a capillary

P5B.3 Comelli and Francesconi examined mixtures of propionic acid with tube at 20 °C. The following data were obtained.
various other organic liquids at 313.15 K (F. Comelli and R. Francesconi,
J. Chem. Eng. Data 41, 101 (1996)). They report the excess volume of mixing c/(mg cm−3) 3.221 4.618 5.112 6.722
propionic acid with tetrahydropyran (THP, oxane) as VE = x1x2{a0 + h/cm 5.746 8.238 9.119 11.990
a1(x1 − x2)}, where x1 is the mole fraction of propionic acid, x2 that of THP,
a0 = −2.4697 cm3 mol−1, and a1 = 0.0608 cm3 mol−1. The density of propionic The osmotic pressure may be calculated from the height of the column as
acid at this temperature is 0.971 74 g cm−3; that of THP is 0.863 98 g cm−3. Π = hρg, taking the mass density of the solution as ρ = 1.000 g cm−3 and the
(a) Derive an expression for the partial molar volume of each component at acceleration of free fall as g = 9.81 m s−2. Determine the molar mass of the
this temperature. (b) Compute the partial molar volume for each component protein.
in an equimolar mixture. ‡
P5B.8 Polymer scientists often report their data in a variety of units. For

P5B.4 Equation 5B.14 indicates, after it has been converted into an expression example, in the determination of molar masses of polymers in solution
for xB, that solubility is an exponential function of temperature. The data by osmometry, osmotic pressures are often reported in grams per square
in the table below gives the solubility, S, of calcium ethanoate in water as a centimetre (g cm−2) and concentrations in grams per cubic centimetre
function of temperature. (g cm−3). (a) With these choices of units, what would be the units of R in the
van ’t Hoff equation? (b) The data in the table below on the concentration
θ/°C 0 20 40 60 80 dependence of the osmotic pressure of polyisobutene in chlorobenzene at
S/(g/100 g solvent) 36.4 34.9 33.7 32.7 31.7 25 °C have been adapted from J. Leonard and H. Daoust (J. Polymer Sci. 57,
53 (1962)). From these data, determine the molar mass of polyisobutene by

These problems were provided by Charles Trapp and Carmen Giunta. plotting Π/c against c. (c) ‘Theta solvents’ are solvents for which the second
Exercises and problems 197


osmotic coefficient is zero; for ‘poor’ solvents the plot is linear and for P5B.9 K. Sato, F.R. Eirich, and J.E. Mark (J. Polymer Sci., Polym. Phys. 14, 619
good solvents the plot is nonlinear. From your plot, how would you classify (1976)) have reported the data in the table below for the osmotic pressures
chlorobenzene as a solvent for polyisobutene? Rationalize the result in terms of polychloroprene (ρ = 1.25 g cm−3) in toluene (ρ = 0.858 g cm−3) at 30 °C.
of the molecular structure of the polymer and solvent. (d) Determine the Determine the molar mass of polychloroprene and its second osmotic virial
second and third osmotic virial coefficients by fitting the curve to the virial coefficient.
form of the osmotic pressure equation. (e) Experimentally, it is often found
that the virial expansion can be represented as c/(mg cm−3) 1.33 2.10 4.52 7.18 9.87
−2
 RT Π/(N m ) 30 51 132 246 390
 (1  Bc  gB2 c 2  )
c M P5B.10 Using mathematical software or an electronic spreadsheet, draw graphs
and in good solvents, the parameter g is often about 0.25. With terms beyond of ∆mixG against xA at different temperatures in the range 298–500 K. For what
the second power ignored, obtain an equation for (Π/c)1/2 and plot this value of xA does ∆mixG depend on temperature most strongly?
quantity against c. Determine the second and third virial coefficients from the
P5B.11 Use mathematical software or a spreadsheet to reproduce Fig. 5B.4.
plot and compare to the values from the first plot. Does this plot confirm the
Then fix ξ and vary the temperature. For what value of xA does the excess
assumed value of g?
enthalpy depend on temperature most strongly?
10−2(Π/c)/(g cm−2/g cm−3) 2.6 2.9 3.6 4.3 6.0 12.0 P5B.12 Derive an expression for the temperature coefficient of the solubility,
−3
c/(g cm ) 0.0050 0.010 0.020 0.033 0.057 0.10 dxB/dT, and plot it as a function of temperature for several values of the
enthalpy of fusion.
10−2(Π/c)/(g cm−2/g cm−3) 19.0 31.0 38.0 52 63
P5B.13 Calculate the osmotic virial coefficient B from the data in Example 5B.2.
c/(g cm−3) 0.145 0.195 0.245 0.27 0.29

TOPIC 5C Phase diagrams of binary systems: liquids


Discussion questions
D5C.1 Draw a two-component, temperature–composition, liquid–vapour D5C.2 What molecular features determine whether a mixture of two liquids
diagram featuring the formation of an azeotrope at xB = 0.333 and complete will show high- and low-boiling azeotropic behaviour?
miscibility. Label the regions of the diagrams, stating what materials are
D5C.3 What factors determine the number of theoretical plates required to
present, and whether they are liquid or gas.
achieve a desired degree of separation in fractional distillation?

Additional exercises
E5C.7 The following temperature–composition data were obtained for The boiling points are 124 °C for A and 155 °C for B. Plot the temperature–
a mixture of octane (O) and methylbenzene (M) at 1.00 atm, where x is composition diagram for the mixture. What is the composition of the vapour
the mole fraction in the liquid and y the mole fraction in the vapour at in equilibrium with the liquid of composition (i) xA = 0.50 and (ii) xB = 0.33?
equilibrium.
E5C.9 The figure below shows the phase diagram for two partially miscible
θ/°C 110.9 112.0 114.0 115.8 117.3 119.0 121.1 123.0 liquids, which can be taken to be that for water (A) and 2-methylpropan-1-ol
(B). Describe what will be observed when a mixture of composition xB = 0.3 is
xM 0.908 0.795 0.615 0.527 0.408 0.300 0.203 0.097 heated, at each stage giving the number, composition, and relative amounts of
yM 0.923 0.836 0.698 0.624 0.527 0.410 0.297 0.164 the phases present.

The boiling points are 110.6 °C and 125.6 °C for M and O, respectively.
Plot the temperature–composition diagram for the mixture. What is the
composition of the vapour in equilibrium with the liquid of composition
(i) xM = 0.250 and (ii) xO = 0.250?
Temperature, T

E5C.8 The following temperature/composition data were obtained for a T2


mixture of two liquids A and B at 1.00 atm, where x is the mole fraction in the
liquid and y the mole fraction in the vapour at equilibrium.

θ/°C 125 130 135 140 145 150


xA 0.91 0.65 0.45 0.30 0.18 0.098
yA 0.99 0.91 0.77 0.61 0.45 0.25 0 0.2 0.4 0.6 0.8 1
Mole fraction of B, xB
198 5 Simple mixtures

E5C.10 Aniline, C6H5NH2, and hexane, C6H14, form partially miscible liquid– E5C.11 Two liquids, A and B, show partial miscibility below 52.4 °C. The
liquid mixtures at temperatures below 69.1 °C. When 42.8 g of aniline and critical concentration at the upper critical temperature is x = 0.459, where x is
75.2 g of hexane are mixed together at a temperature of 67.5 °C, two separate the mole fraction of A. At 40.0 °C the two solutions in equilibrium have
liquid phases are formed, with mole fractions of aniline of 0.308 and 0.618. x = 0.22 and x = 0.60, respectively, and at 42.5 °C the mole fractions are 0.24
(i) Determine the overall mole fraction of aniline in the mixture. (ii) Use the and 0.48. Sketch the phase diagram. Describe the phase changes that occur
lever rule to determine the relative amounts of the two phases. when B is added to a fixed amount of A at (i) 48 °C, (ii) 52.4 °C.

Problems
P5C.1 The vapour pressures of benzene and methylbenzene at 20 °C are 100
75 Torr and 21 Torr, respectively. What is the composition of the vapour in
equilibrium with a mixture in which the mole fraction of benzene is 0.75? 90

Temperature, θ/°C
P5C.2 Dibromoethene (DE, pDE = 22.9 kPa at 358 K) and dibromopropene

(DP, pDP
= 17.1 kPa at 358 K) form a nearly ideal solution. If zDE = 0.60, what is 80
(a) ptotal when the system is all liquid, (b) the composition of the vapour when
the system is still almost all liquid. 70 760 Torr
P5C.3 Benzene and methylbenzene (toluene) form nearly ideal solutions.
Consider an equimolar solution of benzene and methylbenzene. At 20 °C the 60
vapour pressures of pure benzene and methylbenzene are 9.9 kPa and 2.9 kPa,
respectively. The solution is boiled by reducing the external pressure below 0 0.2 0.4 0.6 0.8 1
the vapour pressure. Calculate (a) the pressure when boiling begins, (b) the
Mole fraction of hexane, zH
composition of each component in the vapour, and (c) the vapour pressure
when only a few drops of liquid remain. Assume that the rate of vaporization
is low enough for the temperature to remain constant at 20 °C. P5C.6 Suppose that in a phase diagram, when the sample was prepared
‡ with the mole fraction of component A equal to 0.40 it was found that the
P5C.4 1-Butanol and chlorobenzene form a minimum boiling azeotropic
compositions of the two phases in equilibrium corresponded to the mole
system. The mole fraction of 1-butanol in the liquid (x) and vapour (y) phases
fractions xA,α = 0.60 and xA,β = 0.20. What is the ratio of amounts of the two
at 1.000 atm is given below for a variety of boiling temperatures (H. Artigas
phases?
et al., J. Chem. Eng. Data 42, 132 (1997)).
P5C.7 To reproduce the results of Fig. 5C.2, first rearrange eqn 5C.4 so that yA
T/K 396.57 393.94 391.60 390.15 389.03 388.66 388.57 is expressed as a function of xA and the ratio pA /pB . Then plot yA against xA for
x 0.1065 0.1700 0.2646 0.3687 0.5017 0.6091 0.7171 several values of ratio pA /pB > 1.

y 0.2859 0.3691 0.4505 0.5138 0.5840 0.6409 0.7070 P5C.8 To reproduce the results of Fig. 5C.3, first rearrange eqn 5C.5 so that the
ratio p/pA is expressed as a function of yA and the ratio pA /pB . Then plot p/pA
Pure chlorobenzene boils at 404.86 K. (a) Construct the chlorobenzene-rich against yA for several values of pA /pB > 1.
portion of the phase diagram from the data. (b) Estimate the temperature at P5C.9 In the system composed of benzene and cyclohexane treated in
which a solution whose mole fraction of 1-butanol is 0.300 begins to boil. Example 5B.1 it is established that ξ = 1.13, so the two components are
(c) State the compositions and relative proportions of the two phases present completely miscible at the temperature of the experiment. Would phase
after a solution initially 0.300 1-butanol is heated to 393.94 K. separation be expected if the excess enthalpy were modelled by the expression
P5C.5 The figure below shows the experimentally determined phase diagrams H E   RTx A2 x B2? Hint: The left-hand plot below shows the variation of H E
for the nearly ideal solution of hexane and heptane. (a) Indicate which phases with x for various values of ξ . The right-hand plot shows the solutions of the
are present in each region of the diagram. (b) For a solution containing 1 mol resulting equation for the minima of the Gibbs energy of mixing.
each of hexane and heptane molecules, estimate the vapour pressure at 70 °C
when vaporization on reduction of the external pressure just begins. (c) What 0.15
is the vapour pressure of the solution at 70 °C when just one drop of liquid ξ=2
remains? (d) Estimate from the figures the mole fraction of hexane in the 2x(1 – x)2
0.1
liquid and vapour phases for the conditions of part (b). (e) What are the mole ξ=1
fractions for the conditions of part (c)? (f) At 85 °C and 760 Torr, what are the
0.05
amounts of substance in the liquid and vapour phases when zheptane = 0.40?
HE/nRT

ξ=0
0
900
–0.05 ξ = 1 2 6 10
70°C ξ = –1
700
Pressure, p/Torr

–0.1
(a) ξ = –2 (b) –2ξ(1 – x)2x
500 –0.15
0 0.2 0.4 0.6 0.8 1 0 0.2 0.4 0.6 0.8 1
x x

300 P5C.10 Generate the plot of ξ at which ΔmixG is a minimum against xA by one
of two methods: (a) solve the transcendental equation ln{x/(1 − x)} +
0 0.2 0.4 0.6 0.8 1 ξ(1 − 2x) = 0 numerically, or (b) plot the first term of the transcendental equation
Mole fraction of hexane, zH against the second and identify the points of intersection as ξ is changed.
Exercises and problems 199

TOPIC 5D Phase diagrams of binary systems: solids


Discussion questions
D5D.1 Draw a two-component, temperature–composition, solid–liquid D5D.2 Draw a two-component, temperature–composition, solid–liquid
diagram for a system where a compound AB forms and melts congruently, diagram for a system where a compound of formula AB2 forms that melts
and there is negligible solid–solid solubility. Label the regions of the diagrams, incongruently, and there is negligible solid–solid solubility.
stating what materials are present and whether they are solid or liquid.

Additional exercises
E5D.4 Sketch the phase diagram of the system NH3/N2H4 given that the two for liquid mixtures in which x(B2H6) is (i) 0.10, (ii) 0.30, (iii) 0.50, (iv) 0.80,
substances do not form a compound with each other, that NH3 freezes at and (v) 0.95.
−78 °C and N2H4 freezes at +2 °C, and that a eutectic is formed when the mole
E5D.9 Indicate on the phase diagram in the figure below the feature that
fraction of N2H4 is 0.07 and that the eutectic melts at −80 °C.
denotes incongruent melting. What is the composition of the eutectic mixture
E5D.5 Methane (melting point 91 K) and tetrafluoromethane (melting point and at what temperature does it melt?
89 K) do not form solid solutions with each other, and as liquids they are only
partially miscible. The upper critical temperature of the liquid mixture is 94 K
at x(CF4) = 0.43 and the eutectic temperature is 84 K at x(CF4) = 0.88. At 86 K, b a
the phase in equilibrium with the tetrafluoromethane-rich solution changes
from solid methane to a methane-rich liquid. At that temperature, the two T1

Temperature, T
liquid solutions that are in mutual equilibrium have the compositions
x(CF4) = 0.10 and x(CF4) = 0.80. Sketch the phase diagram.
E5D.6 Describe the phase changes that take place when a liquid mixture of T2
4.0 mol B2H6 (melting point 131 K) and 1.0 mol CH3OCH3 (melting point T3
135 K) is cooled from 140 K to 90 K. These substances form a compound
(CH3)2OB2H6 that melts congruently at 133 K. The system exhibits
one eutectic at x(B2H6) = 0.25 and 123 K and another at x(B2H6) = 0.90
and 104 K. 0 0.2 0.4 0.6 0.8 1
Mole fraction of B, xB
E5D.7 Refer to the information in Exercise E5D.5 and sketch the cooling
curves for liquid mixtures in which x(CF4) is (i) 0.10, (ii) 0.30, (iii) 0.50,
(iv) 0.80, and (v) 0.95. E5D.10 Sketch the cooling curves for the isopleths a and b in the temperature–
composition diagram shown in Exercise E5D.9.
E5D.8 Consider a liquid mixture of B2H6 (melting point 131 K) and CH3OCH3
(melting point 135 K). These substances form a compound (CH3)2OB2H6 that E5D.11 Use the phase diagram accompanying Exercise E5D.9 to state (i) the
melts congruently at 133 K. The system exhibits one eutectic at x(B2H6) = 0.25 solubility of B in A at 500 °C, (ii) the solubility of AB2 in A at 390 °C, and
and 123 K and another at x(B2H6) = 0.90 and 104 K. Sketch the cooling curves (iii) the solubility of AB2 in B at 300 °C.

Problems
P5D.1 Uranium tetrafluoride and zirconium tetrafluoride melt at 1035 °C and P5D.3 Consider the phase diagram below, which represents a solid–liquid
912 °C respectively. They form a continuous series of solid solutions with a equilibrium. Label all regions of the diagram according to the chemical
minimum melting temperature of 765 °C and composition x(ZrF4) = 0.77. At species exist in that region and their phases. Indicate the number of species
900 °C, the liquid solution of composition x(ZrF4) = 0.28 is in equilibrium and phases present at the points labelled b, d, e, f, g, and k. Sketch cooling
with a solid solution of composition x(ZrF4) = 0.14. At 850 °C the two curves for compositions xB = 0.16, 0.23, 0.57, 0.67, and 0.84.
compositions are 0.87 and 0.90, respectively. Sketch the phase diagram for this
system and state what is observed when a liquid of composition x(ZrF4) = 0.40
is cooled slowly from 900 °C to 500 °C. b
P5D.2 Phosphorus and sulfur form a series of binary compounds. The
Temperature, T

best characterized are P4S3, P4S7, and P4S10, all of which melt congruently. f
Assuming that only these three binary compounds of the two elements
g
exist, (a) draw schematically only the P/S phase diagram plotted against
c
xS. Label each region of the diagram with the substance that exists in that k
region and indicate its phase. Label the horizontal axis as xS and give the e
d
numerical values of xS that correspond to the compounds. The melting
point of pure phosphorus is 44 °C and that of pure sulfur is 119 °C.
0.23

0.57

0.67

0.84
0.16

(b) Draw, schematically, the cooling curve for a mixture of composition


xS = 0.28. Assume that a eutectic occurs at xS = 0.2 and negligible solid– 0 0.2 0.4 0.6 0.8 1
solid solubility. Mole fraction of B, xB
200 5 Simple mixtures

P5D.4 Sketch the phase diagram for the Mg/Cu system using the following P5D.6 Iron(II) chloride (melting point 677 °C) and potassium chloride
information: θf(Mg) = 648 °C, θf(Cu) = 1085 °C; two intermetallic (melting point 776 °C) form the compounds KFeCl3 and K2FeCl4 at elevated
compounds are formed with θf(MgCu2) = 800 °C and θf(Mg2Cu) = 580 °C; temperatures. KFeCl3 melts congruently at 399 °C and K2FeCl4 melts
eutectics of mass percentage Mg composition and melting points 10 per incongruently at 380 °C. Eutectics are formed with compositions x = 0.38
cent (690 °C), 33 per cent (560 °C), and 65 per cent (380 °C). A sample (melting point 351 °C) and x = 0.54 (melting point 393 °C), where x is the
of Mg/Cu alloy containing 25 per cent Mg by mass was prepared in a mole fraction of FeCl2. The KCl solubility curve intersects the A curve at
crucible heated to 800 °C in an inert atmosphere. Describe what will be x = 0.34. Sketch the phase diagram. State the phases that are in equilibrium
observed if the melt is cooled slowly to room temperature. Specify the when a mixture of composition x = 0.36 is cooled from 400 °C to 300 °C.
composition and relative abundances of the phases and sketch the cooling ‡
P5D.7 An, Zhao, Jiang, and Shen investigated the liquid–liquid coexistence
curve.
curve of N,N-dimethylacetamide and heptane (X. An et al., J. Chem.

P5D.5 The temperature/composition diagram for the Ca/Si binary system Thermodynamics 28, 1221 (1996)). Mole fractions of N,N-dimethylacetamide
is shown in the figure below. (a) Identify eutectics, congruent melting in the upper (x1) and lower (x2) phases of a two-phase region are given below
compounds, and incongruent melting compounds. (b) A melt with as a function of temperature:
composition xSi = 0.20 at 1500 °C is cooled to 1000 °C. What phases (and
T/K 309.820 309.422 309.031 308.006 306.686
phase composition) would be at equilibrium? Estimate the relative amounts
of each phase. (c) Describe the equilibrium phases observed when a melt with x1 0.473 0.400 0.371 0.326 0.293
xSi = 0.80 is cooled to 1030 °C. What phases, and relative amounts, would be x2 0.529 0.601 0.625 0.657 0.690
at equilibrium at a temperature (i) slightly higher than 1030 °C, (ii) slightly
lower than 1030 °C? T/K 304.553 301.803 299.097 296.000 294.534

1500
x1 0.255 0.218 0.193 0.168 0.157
0.402

Liquid x2 0.724 0.758 0.783 0.804 0.814


1314 1324
0.694
Temperature, θ/°C

1300 (a) Plot the phase diagram. (b) State the proportions and compositions of the
1268 two phases that form from mixing 0.750 mol of N,N-dimethylacetamide with
1100 0.250 mol of heptane at 296.0 K. To what temperature must the mixture be
1040 1030
0.056

heated to form a single-phase mixture?


Ca2Si

CaSi2

800
CaSi

700
0 0.2 0.4 0.6 0.8 1
Mole fraction of B, xB

TOPIC 5E Phase diagrams of ternary systems


Discussion questions
D5E.1 What is the maximum number of phases that can be in equilibrium in a D5E.3 Could a regular tetrahedron be used to depict the properties of a four-
ternary system? component system?
D5E.2 Does the lever rule apply to a ternary system? D5E.4 Consider the phase diagram for a stainless steel shown in Fig. 5E.6.
Identify the composition represented by point c.

Additional exercises
E5E.5 Mark the following features on triangular coordinates: (i) the point (0.6, 0 1 H2O
0.2, 0.2), (ii) the point (0.8, 0.2, 0), (iii) the point (0.25, 0.25, 0.50).
E5E.6 Mark the following points on a ternary phase diagram for the system 0.2 0.8
NaCl/Na2SO4·10H2O/H2O: (i) 33 per cent by mass NaCl, 33 per cent P=1
Na2SO4·10H2O, and the rest H2O; (ii) the line denoting the same relative 0.4 0.6
composition of the two salts but with changing amounts of water.
P=2 P=2
E5E.7 Refer to the ternary phase diagram in Fig. 5E.4. How many phases 0.6 0.4
are present, and what are their compositions and relative abundances, in a
mixture that contains 55.0 g of water, 8.8 g of trichloromethane, and 3.7 g of
ethanoic acid? Describe what happens when (i) water, (ii) ethanoic acid is 0.8 0.2
P=3
added to the mixture.
E5E.8 Refer to the accompanying figure and identify the number of phases NH4Cl1 0 (NH4)2SO4
0 0.2 0.4 0.6 0.8 1
present for mixtures of compositions (i) (0.4, 0.1, 0.5), (ii) (0.8, 0.1, 0.1),
(iii) (0, 0.3,0.7), (iv) (0.33, 0.33, 0.34). The numbers are mole fractions
of the three components in the order (NH4Cl, (NH4)2SO4, H2O).
Exercises and problems 201

E5E.9 Referring to the ternary phase diagram associated with Exercise E5E.4 in E5E.10 Referring to the ternary phase diagram associated with Exercise E5E.4
the main text deduce the molar solubility of (i) NH4Cl, (ii) (NH4)2SO4 in water in the main text describe what happens when (i) (NH4)2SO4 is added to a
at 25 °C. saturated solution of NH4Cl in water in the presence of excess NH4Cl,
(ii) water is added to a mixture of 25 g of NH4Cl and 75 g of (NH4)2SO4.

Problems
P5E.1 At a certain temperature, the solubility of I2 in liquid CO2 is x(I2) = 0.03. The addition of liquid carbon dioxide to mixtures of nitroethane and DEC
At the same temperature its solubility in nitrobenzene is 0.04. Liquid carbon increases the range of miscibility, and the plait point is reached when the mole
dioxide and nitrobenzene are miscible in all proportions, and the solubility of fraction of CO2 is 0.18 and x = 0.53. The addition of nitroethane to mixtures
I2 in the mixture varies linearly with the proportion of nitrobenzene. Sketch a of carbon dioxide and DEC also results in another plait point at x = 0.08 and
phase diagram for the ternary system. y = 0.52. (a) Sketch the phase diagram for the ternary system. (b) For some
binary mixtures of nitroethane and liquid carbon dioxide the addition of
P5E.2 The binary system nitroethane/decahydronaphthalene (DEC) shows
arbitrary amounts of DEC will not cause phase separation. Find the range of
partial miscibility, with the two-phase region lying between x = 0.08 and
concentration for such binary mixtures.
x = 0.84, where x is the mole fraction of nitroethane. The binary system
liquid carbon dioxide/DEC is also partially miscible, with its two-phase P5E.3 Prove that a straight line from the apex A of a ternary phase diagram
region lying between y = 0.36 and y = 0.80, where y is the mole fraction of to the opposite edge BC represents mixtures of constant ratio of B and C,
DEC. Nitroethane and liquid carbon dioxide are miscible in all proportions. however much A is present.

TOPIC 5F Activities
Discussion questions
D5F.1 What are the contributions that account for the difference between D5F.4 Why do the activity coefficients of ions in solution differ from 1? Why
activity and concentration? are they less than 1 in dilute solutions?
D5F.2 How is Raoult’s law modified so as to describe the vapour pressure of D5F.5 Describe the general features of the Debye–Hückel theory of electrolyte
real solutions? solutions.
D5F.3 Summarize the ways in which activities may be measured. D5F.6 Suggest an interpretation of the additional terms in extended versions of
the Debye–Hückel limiting law.

Additional exercises
E5F.9 The vapour pressure of a salt solution at 100 °C and 1.00 atm is E5F.13 Suppose it is found that for a hypothetical regular solution that
90.00 kPa. What is the activity of water in the solution at this temperature? ξ = −1.40, pA = 15.0 kPa , and pB = 11.6 kPa . Draw plots similar to Fig. 5F.3.
−1
E5F.10 Given that p*(H2O) = 0.023 08 atm and p(H2O) = 0.022 39 atm in E5F.14 Calculate the ionic strength of a solution that is 0.040 mol kg in
a solution in which 0.122 kg of a non-volatile solute (M = 241 g mol−1) K3[Fe(CN)6](aq), 0.030 mol kg−1 in KCl(aq), and 0.050 mol kg−1 in NaBr(aq).
is dissolved in 0.920 kg water at 293 K, calculate the activity and activity
E5F.15 Calculate the masses of (i) KNO3 and, separately, (ii) Ba(NO3)2 to add
coefficient of water in the solution.
to a 0.110 mol kg−1 solution of KNO3(aq) containing 500 g of solvent to raise
E5F.11 By measuring the equilibrium between liquid and vapour phases of a its ionic strength to 1.00.
solution at 30 °C at 1.00 atm, it was found that xA = 0.220 when yA = 0.314.
E5F.16 Estimate the mean ionic activity coefficient of NaCl in a solution that is
Calculate the activities and activity coefficients of both components in
0.020 mol kg−1 NaCl(aq) and 0.035 mol kg−1 Ca(NO3)2(aq) at 25 °C.
this solution on the Raoult’s law basis. The vapour pressures of the pure
components at this temperature are: pA = 73.0 kPa and pB = 92.1 kPa (xA is E5F.17 The mean activity coefficients of KCl in three dilute aqueous solutions
the mole fraction in the liquid and yA the mole fraction in the vapour). at 25 °C are 0.927 (at 5.00 mmol kg−1), 0.902 (at 10.0 mmol kg−1), and 0.816 (at
50.0 mmol kg−1). Estimate the value of B in the Davies equation.
E5F.12 Suppose it is found that for a hypothetical regular solution that
ξ = 1.40, pA = 15.0 kPa , and pB = 11.6 kPa . Draw plots similar to Fig. 5F.3.

Problems

P5F.1 Francesconi, Lunelli, and Comelli studied the liquid–vapour equilibria Compute the activity coefficients of both components on the basis of
of trichloromethane and 1,2-epoxybutane at several temperatures (J. Chem. Raoult’s law.
Eng. Data 41, 310 (1996)). Among their data are the following measurements
P5F.2 Use mathematical software or a spreadsheet to plot pA /pA against xA

of the mole fractions of trichloromethane in the liquid phase (xT) and the
with ξ = 2.5 by using eqn 5F.18 and then eqn 5F.19. Above what value
vapour phase (yT) at 298.15 K as a function of total pressure.
of xA do the values of pA /pA given by these equations differ by more than
10 per cent?
p/kPa 23.40 21.75 20.25 18.75 18.15 20.25 22.50 26.30
xT 0 0.129 0.228 0.353 0.511 0.700 0.810 1
yT 0 0.065 0.145 0.285 0.535 0.805 0.915 1
202 5 Simple mixtures

1/2
P5F.3 The mean activity coefficients for aqueous solutions of NaCl at 25 °C are P5F.4 Consider the plot of log γ± against I with B = 1.50 and C = 0 in
given below. Confirm that they support the Debye–Hückel limiting law and the Davies equation as a representation of experimental data for a certain
that an improved fit is obtained with the Davies equation. MX electrolyte. Over what range of ionic strengths does the application
of the Debye–Hückel limiting law lead to an error in the value of the
b/(mmol kg−1) 1.0 2.0 5.0 10.0 20.0 activity coefficient of less than 10 per cent of the value predicted by the
γ± 0.9649 0.9519 0.9275 0.9024 0.8712 extended law?

FOCUS 5 Simple mixtures


Integrated activities

I5.1 The table below lists the vapour pressures of mixtures of iodoethane (I) and I5.4 The following data have been obtained for the liquid–vapour equilibrium
ethyl ethanoate (E) at 50 °C. Find the activity coefficients of both components compositions of mixtures of nitrogen and oxygen at 100 kPa.
on (a) the Raoult’s law basis, (b) the Henry’s law basis with iodoethane as solute.
T/K 77.3 78 80 82 84 86 88 90.2
xI 0 0.0579 0.1095 0.1918 0.2353 0.3718
100x(O2) 0 10 34 54 70 82 92 100
pI/kPa 0 3.73 7.03 11.7 14.05 20.72
100y(O2) 0 2 11 22 35 52 73 100
pE/kPa 37.38 35.48 33.64 30.85 29.44 25.05
p*(O2)/Torr 154 171 225 294 377 479 601 760

xI 0.5478 0.6349 0.8253 0.9093 1.0000 Plot the data on a temperature–composition diagram and determine the
pI/kPa 28.44 31.88 39.58 43.00 47.12 extent to which it fits the predictions for an ideal solution by calculating the
activity coefficients of O2 at each composition.
pE/kPa 19.23 16.39 8.88 5.09 0
I5.5 For the calculation of the solubility c of a gas in a solvent, it is often
I5.2 Plot the vapour pressure data for a mixture of benzene (B) and ethanoic convenient to use the expression c = Kp, where K is the Henry’s law constant.
acid (E) given below and plot the vapour pressure/composition curve for the Breathing air at high pressures, such as in scuba diving, results in an
mixture at 50 °C. Then confirm that Raoult’s and Henry’s laws are obeyed in increased concentration of dissolved nitrogen. The Henry’s law constant for
the appropriate regions. Deduce the activities and activity coefficients of the the solubility of nitrogen is 0.18 μg/(g H2O atm). What mass of nitrogen is
components on the Raoult’s law basis and then, taking B as the solute, its activity dissolved in 100 g of water saturated with air at 4.0 atm and 20 °C? Compare
and activity coefficient on a Henry’s law basis. Finally, evaluate the excess Gibbs your answer to that for 100 g of water saturated with air at 1.0 atm. (Air is
energy of the mixture over the composition range spanned by the data. 78.08 mol per cent N2.) If nitrogen is four times as soluble in fatty tissues as in
water, what is the increase in nitrogen concentration in fatty tissue in going
from 1 atm to 4 atm?
xE 0.0160 0.0439 0.0835 0.1138 0.1714
pE/kPa 0.484 0.967 1.535 1.89 2.45 I5.6 Dialysis may be used to study the binding of small molecules to
macromolecules, such as an inhibitor to an enzyme, an antibiotic to DNA,
pB/kPa 35.05 34.29 33.28 32.64 30.90 and any other instance of cooperation or inhibition by small molecules
attaching to large ones. To see how this is possible, suppose inside the dialysis
xE 0.2973 0.3696 0.5834 0.6604 0.8437 0.9931 bag the molar concentration of the macromolecule M is [M] and the total
pE/kPa 3.31 3.83 4.84 5.36 6.76 7.29 concentration of small molecule A is [A]in. This total concentration is the
sum of the concentrations of free A and bound A, which we write [A]free and
pB/kPa 28.16 26.08 20.42 18.01 10.0 0.47 [A]bound, respectively. At equilibrium, μA,free = μA,out, which implies that [A]free =

[A]out, provided the activity coefficient of A is the same in both solutions.
I5.3 Chen and Lee studied the liquid–vapour equilibria of cyclohexanol with Therefore, by measuring the concentration of A in the solution outside the
several gases at elevated pressures (J.-T. Chen and M.-J. Lee, J. Chem. Eng. bag, the concentration of unbound A in the macromolecule solution can be
Data 41, 339 (1996)). Among their data are the following measurements of found and, from the difference [A]in − [A]free = [A]in − [A]out, the concentration
the mole fractions of cyclohexanol in the vapour phase (y) and the liquid of bound A. Now explore the quantitative consequences of the experimental
phase (x) at 393.15 K as a function of pressure. arrangement just described. (a) The average number of A molecules bound to
M molecules, ν, is
p/bar 10.0 20.0 30.0 40.0 60.0 80.0
ycyc 0.0267 0.0149 0.0112 0.00947 0.00835 0.00921 [A]bound [A]in  [A]out
 
[M] [M]
xcyc 0.9741 0.9464 0.9204 0.892 0.836 0.773
The bound and unbound A molecules are in equilibrium, M + A ⇌ MA.
Determine the Henry’s law constant of CO2 in cyclohexanol, and compute the Recall from introductory chemistry that the equilibrium constant for binding,
activity coefficient of CO2. K, may be written as
Exercises and problems 203

[MA]c 

are a total of six sites per polymer. Four of the sites are identical and have an
K= intrinsic binding constant of 1 × 105. The binding constants for the other two
[M]free[A]free
sites are 2 × 106.
Now show that I5.8 The addition of a small amount of a salt, such as (NH4)2SO4, to a solution
containing a charged protein increases the solubility of the protein in water.
 c This observation is called the salting-in effect. However, the addition of large
K
(1  )[A]out amounts of salt can decrease the solubility of the protein to such an extent
that the protein precipitates from solution. This observation is called the
(b) If there are N identical and independent binding sites on each salting-out effect and is used widely by biochemists to isolate and purify
macromolecule, each macromolecule behaves like N separate smaller proteins. Consider the equilibrium PVν(s) ⇌ Pν+(aq) + νX−(aq), where Pν+ is a
macromolecules, with the same value of K for each site. It follows that the polycationic protein of charge ν+ and X− is its counterion. Use Le Chatelier’s
average number ⦵ of A molecules per site is ν/N. Show that, in this case, principle and the physical principles behind the Debye–Hückel theory to
the Scatchard equation provide a molecular interpretation for the salting-in and salting-out effects.

⦵ I5.9 The osmotic coefficient ϕ is defined as ϕ = −(xA/xB) ln aA. By writing


 c r = xB/xA, and using the Gibbs–Duhem equation, show that the activity of B
 KN  K
[A]out can be calculated from the activities of A over a composition range by using
the formula
is obtained. (c) To apply the Scatchard equation, consider the binding
of ethidium bromide (E−) to a short piece of DNA by a process called aB r  1

intercalation, in which the aromatic ethidium cation fits between two adjacent ln     (0)   dr
r 0 r
DNA base pairs. An equilibrium dialysis experiment was used to study the
binding of ethidium bromide (EB) to a short piece of DNA. A 1.00 μmol dm−3 I5.10 Show that the osmotic pressure of a real solution is given by ΠV =
aqueous solution of the DNA sample was dialysed against an excess of EB. −RT ln aA. Noting that the osmotic coefficient ϕ is defined as ϕ = −(xA/xB) ln aA,
The following data were obtained for the total concentration of EB, [EB]/ show that, provided the concentration of the solution is low, this expression
(μmol dm−3): takes the form ΠV = ϕRT[B] and hence that the osmotic coefficient may be
determined from osmometry.
Side without DNA 0.042 0.092 0.204 0.526 1.150
I5.11 Show that the freezing-point depression of a real solution in which the
Side with DNA 0.292 0.590 1.204 2.531 4.150 solvent of molar mass M has activity aA obeys
From these data, make a Scatchard plot and evaluate the intrinsic equilibrium
constant, K, and total number of sites per DNA molecule. Is the identical and d ln aA M

independent sites model for binding applicable? d(T ) Kf

I5.7 For the case that the binding sites, i, are non-identical but still and use the Gibbs–Duhem equation to show that
independent the Scatchard equation is modified to
d ln aB 1
⦵ 
 c Ni Ki d(T ) bB K f
[A]out 
 ⦵
i 1  K i [A]out /c

where aB is the solute activity and bB is its molality. Use the Debye–Hückel
Plot ν/[A]out for the following cases. (a) There are four independent sites on an limiting law to show that the osmotic coefficient ϕ, defined as ϕ = −(xA/xB) ln

enzyme molecule and the intrinsic binding constant is K = 1.0 × 107. (b) There aA, is given by   1  13 AI with A′ = 2.303A and I = b/b .
FOCUS 6
Chemical equilibrium

Chemical reactions tend to move towards a dynamic equilib- cell equipped with electrodes, with the spontaneous reaction
rium in which both reactants and products are present but have forcing electrons through an external circuit. The electric po-
no further tendency to undergo net change. In some cases, tential of the cell is easily measured and is related to the reac-
the concentration of products in the equilibrium mixture is so tion Gibbs energy, thus providing a convenient method for the
much greater than that of the unchanged reactants that for all determination of some thermodynamic quantities.
practical purposes the reaction is ‘complete’. However, in many 6C.1 Half-reactions and electrodes; 6C.2 Varieties of cells; 6C.3 The cell
important cases the equilibrium mixture has significant con- potential; 6C.4 The determination of thermodynamic functions
centrations of both reactants and products.

6A The equilibrium constant 6D Electrode potentials


This Topic develops the concept of chemical potential and shows As elsewhere in thermodynamics, electrochemical data can be
how it is used to account for the equilibrium composition of a reported in a compact form and applied to problems of chemi-
reaction mixture. The equilibrium composition corresponds to cal significance. Those applications include the prediction of
a minimum in the Gibbs energy. By locating this minimum it is the spontaneous direction of reactions and the calculation of
possible to establish the relation between the equilibrium con- equilibrium constants.
stant and the standard Gibbs energy of reaction. 6D.1 Standard potentials; 6D.2 Applications of standard electrode
potentials
6A.1 The Gibbs energy minimum; 6A.2 The description of equilibrium

6B The response of equilibria to the What is an application of this material?


conditions
The thermodynamic description of spontaneous reactions
The thermodynamic formulation of the equilibrium constant is has numerous practical and theoretical applications. One
used to establish the quantitative effects of changes in the con- is to the discussion of biochemical processes, where one re-
ditions. One very important aspect of equilibrium is the con- action drives another (Impact 9, accessed via the e-book).
trol that can be exercised by varying the conditions, such as the Ultimately that is why we have to eat, for the reaction that
pressure or temperature. takes place when one substance is oxidized can drive for-
6B.1 The response to pressure; 6B.2 The response to temperature ward non-spontaneous reactions, such as protein synthesis.
Another makes use of the great sensitivity of electrochemical
processes to the concentration of electroactive materials, and
leads to the design of electrodes used in chemical analysis
6C Electrochemical cells (Impact 10, accessed via the e-book).

Because many reactions involve the transfer of electrons, they


can be studied (and used) by allowing them to take place in a

➤ Go to the e-book for videos that feature the derivation and interpretation of equations, and applications of this material.
TOPIC 6A The equilibrium constant

varies with composition and then identifying the composition


➤ Why do you need to know this material? that corresponds to minimum G.
Equilibrium constants lie at the heart of chemistry and
are a key point of contact between thermodynamics and 6A.1(a) The reaction Gibbs energy
laboratory chemistry. To understand the behaviour of reac-
First, consider the simplest possible equilibrium with one re-
tions, you need to see how equilibrium constants arise and
actant and one product: A  B. Examples of such equilibria
understand how thermodynamic properties account for
include isomerization (e.g. cis/trans FN=NF), tautomeriza-
their values.
tion (ketone/enol), and conformational interchange (axial/­
➤ What is the key idea? equatorial in cyclohexane).
Suppose that an infinitesimal amount dξ of A turns into B.
At constant temperature and pressure, the composition of
The change in the amount of A present is dnA = −dξ and the
a reaction mixture tends to change until the Gibbs energy
change in the amount of B present is dnB = +dξ. The quantity
is a minimum.
ξ (xi) is called the extent of reaction; it has the dimensions of
➤ What do you need to know already? amount of substance and is reported in moles. In general, the
amount of a component J changes by  J d , where n J is the stoi-
Underlying the whole discussion is the expression of the
chiometric number of the species J (positive for products, nega-
direction of spontaneous change in terms of the Gibbs
tive for reactants). In this case  A  1 and  B  1.
energy of a system (Topic 3D). This material draws on the
The reaction Gibbs energy, ΔrG, is defined as the slope of the
concept of chemical potential and its dependence on the
graph of the Gibbs energy plotted against the extent of reaction:
concentration or pressure of the substances in a mixture
(Topic 5A). Some of the discussion draws on the Boltzmann  G 
distribution, which is introduced in Energy: A first look.
r G    Reaction Gibbs energy [definition] (6A.1)
   p ,T
Although ∆ normally signifies a difference in values, here it sig-
nifies a derivative, the slope of G with respect to ξ. However,
to see that there is a close relationship with the normal usage,
The direction of spontaneous change at constant temperature suppose the reaction advances by dξ, meaning that dnA = −dξ
and pressure is towards lower values of the Gibbs energy, G and dnB = +dξ, as described above. The corresponding change
(Topic 3D). This criterion is entirely general, and in this Topic in Gibbs energy is
is applied to the spontaneity of chemical reactions. Thus, at con-
dG   A dnA  B dnB    A d  B d  (B   A)d
stant temperature and pressure, a mixture of reactants has a ten-
dency to undergo reaction until the Gibbs energy of the mixture This equation can be reorganized into
has reached a minimum. The final composition corresponds to a
 G 
state of chemical equilibrium, a state of the reaction mixture at    B   A
which the reaction has no tendency to undergo further change.    p ,T
The equilibrium is dynamic in the sense that the forward and Comparison of this equation with eqn 6A.1 gives
reverse reactions continue, but at matching rates. As always in
 r G  B   A (6A.2)
the application of thermodynamics, spontaneity is a tendency:
there might be kinetic reasons why that tendency is not realized. In other words, ∆rG can also be interpreted as the difference
between the chemical potentials (the partial molar Gibbs ener-
gies) of the reactants and products at the current composition of
the reaction mixture.
6A.1 The Gibbs energy minimum Chemical potentials vary with composition. Therefore, the
slope of the plot of Gibbs energy against extent of reaction, and
The equilibrium composition of a reaction mixture is identified hence the reaction Gibbs energy, changes as the reaction pro-
by establishing how the Gibbs energy of the reaction mixture ceeds (Fig. 6A.1). The spontaneous direction of reaction lies
6A The equilibrium constant 207

Δr G < 0
Gibbs energy, G
Pure reactants

ΔrG > 0

products
Pure
Δ rG = 0
0 Extent of reaction, ξ Fig. 6A.2 If two weights are coupled as shown here, then the
Fig. 6A.1 As the reaction advances, represented by the extent heavier weight will move the lighter weight upwards. On its own,
of reaction ξ increasing, the total Gibbs energy of the reaction the lighter weight has a tendency to move downwards, but by
mixture changes in the way shown. Equilibrium corresponds to coupling it to the heavier weight it moves in the opposite, non-
the minimum in the Gibbs energy, which is where the slope of spontaneous, direction. The weights are the analogues of two
the curve is zero. Interact with the dynamic version of this graph in chemical reactions: a reaction with a large negative ∆rG can be
the e-book. coupled to another reaction with a less negative or more positive
∆rG to drive the latter in a non-spontaneous direction.

in the direction of decreasing G (down the slope of G plot-


ted against ξ). Thus, the reaction A → B is spontaneous when from amino acids, muscle contraction, and brain activity. A re-
ΔrG < 0, whereas the reverse reaction is spontaneous when action for which ∆rG > 0 is called endergonic (signifying ‘work
ΔrG > 0. The slope is zero, and the reaction is at equilibrium and consuming’); such a reaction can be made to occur only by
spontaneous in neither direction, when doing work on it.
r G  0 Condition of equilibrium (6A.3)
Brief illustration 6A.1
This condition occurs when μB = μA. It follows that the equilib-
rium composition can be identified by finding the composition
The reaction Gibbs energy of a certain reaction is −200 kJ mol−1,
for which this condition holds. Note that the chemical potential so the reaction is exergonic, and in a suitable device (a fuel cell,
is now fulfilling the role its name suggests: it represents the po- for instance) operating at constant temperature and pressure, it
tential for chemical change, and equilibrium is attained when could produce 200 kJ of electrical work for each mole of reaction
these potentials are in balance. events. The reverse reaction, for which ∆rG = +200 kJ mol−1, is
endergonic and at least 200 kJ of work must be done to achieve
6A.1(b) Exergonic and endergonic reactions it, perhaps through electrolysis.

The spontaneity of a reaction at constant temperature and pres-


sure can be expressed in terms of the reaction Gibbs energy at
the current composition of the reaction mixture: Exercises
E6A.1 Consider the reaction A → 2 B. Initially 1.50 mol A is present and no B.
If ∆rG < 0, the forward reaction is spontaneous. What are the amounts of A and B when the extent of reaction is 0.60 mol?
If ∆rG > 0, the reverse reaction is spontaneous. E6A.2 When the reaction A → 2 B advances by 0.10 mol (i.e. Δξ = +0.10 mol)
If ∆rG = 0, the reaction is at equilibrium. the molar Gibbs energy of the system changes by −6.4 kJ mol−1. What is the
Gibbs energy of reaction at this stage of the reaction?
A reaction for which ∆rG < 0 is called exergonic (from the ⦵ −1
E6A.3 Given that at 298 K ΔfG = −6.4 kJ mol for methane, classify the
Greek words for ‘work producing’). The name signifies that, be- formation of methane from its elements in their reference states as exergonic
cause the process is spontaneous, it can be used to drive another or endergonic under standard conditions at 298 K.
process, such as another reaction, or used to do non-expansion
work. A simple mechanical analogy is a pair of weights joined
by a string (Fig. 6A.2): the lighter of the pair of weights will be
pulled up as the heavier weight falls down. Although the lighter 6A.2 The description of equilibrium
weight has a natural tendency to move down, its coupling to the
heavier weight results in it being raised. In biological cells, the The same principles apply to more complex chemical reac-
oxidation of carbohydrates acts as the heavy weight that drives tions taking place in a variety of phases. In each case, the vari-
other reactions forward and results in the formation of p ­ roteins ation of ∆rG with composition is established by noting how the
208 6 Chemical equilibrium

c­ hemical potentials of the reactants and products depend on value; that is, K is a constant for the reaction being c­ onsidered.
their abundances in the reaction mixture. Then the composi- This equation is exceptionally important as it makes the link
tion at which ∆rG = 0 is identified and taken to be the equilib- between the value of ∆rG , which can be found from tabulated

rium composition of the reaction. thermodynamic data, and the equilibrium ­constant.

6A.2(a) Perfect gas equilibria Brief illustration 6A.2


The chemical potential of a perfect gas J in a mixture At 298 K the standard Gibbs energy of formation of cis-
depends on its partial pressure according to eqn 5A.15b,
­ 2-butene is +65.95 kJ mol−1, and that of trans-2-butene is
J  J  RT ln( pJ /p ), where p = 1 bar, the standard pres-
⦵ ⦵ ⦵

+63.06 kJ mol−1. It follows that the standard Gibbs energy of


sure. For the reaction A(g) → B(g), the cis → trans isomerization reaction is +63.06 kJ mol−1 –
(+65.95 kJ mol−1) = −2.89 kJ mol−1. Therefore, the equilibrium
 ⦵ p   ⦵ p 
 r G  B  A   B  RT ln ⦵B    A  RT ln ⦵A  constant for the reaction, with eqn 6A.8 written in the form
 p   p 

K  e  r G /RT , is

p
  r G   RT ln B  (6A.4)

 ( 2.89103 J mol 1 ) / (( 8.3145 J K 1mol 1 )( 298 K ))
K e  e1.16  3.2
pA

where  r G   B   A . The chemical potential of a pure sub-


⦵ ⦵ ⦵

stance (which is implied by the term ‘standard state’) is equal An important feature to note is that the minimum in the
to its molar Gibbs energy, therefore ∆rG can be identified with

Gibbs energy for the reaction A(g) → B(g) (and in more gen-
the difference of the standard molar Gibbs energies: eral reactions) stems from the Gibbs energy of mixing of the
two gases. If the contribution from mixing is ignored, the Gibbs
 r G   Gm (B)  Gm (A)
⦵ ⦵ ⦵
(6A.5) energy would change linearly from its value for pure reactants
As explained in Topic 3D, the difference in standard molar to its value for pure products; there would be no intermedi-
Gibbs energies of the products and reactants is equal to the ate minimum in the graph (Fig. 6A.3). When the mixing is
difference in their standard Gibbs energies of formation. taken into account, there is an additional contribution to the
Therefore, in practice ∆rG is calculated from

Gibbs energy that is given by eqn 5A.17, ∆mixG = nRT(xA ln xA +
xB ln xB). This expression makes a U-shaped contribution to the
 r G    f G  (B)   f G  (A)
⦵ ⦵ ⦵
(6A.6) total Gibbs energy. As can be seen from Fig. 6A.3, when it is
Finally, with the ratio of partial pressures denoted by Q, it fol- included there is an intermediate minimum in the total Gibbs
lows that energy, and its position corresponds to the equilibrium compo-
sition of the reaction mixture.
pB
 r G   r G   RT l n Q Q

 (6A.7)
pA

Equilibrium
The ratio Q is an example of a ‘reaction quotient’, a quantity Without
to be defined more formally shortly. It is a pure number that mixing
ranges from 0 when pB = 0 (corresponding to pure A) to infinity
Gibbs energy, G

when pA = 0 (corresponding to pure B).


At equilibrium ∆rG = 0 and, if the equilibrium pressures are Including
mixing
pA,eq and pB,eq, it follows from eqn 6A.4 that
0
pB,eq
0   r G   RT ln

Mixing
pA,eq

which rearranges to 0 Extent of reaction, ξ

pB,eq Fig. 6A.3 If the mixing of reactants and products is ignored,


 r G   RT ln K

K  (6A.8)
pA,eq the Gibbs energy changes linearly from its initial value (pure
reactants) to its final value (pure products) and the slope of the
The ratio of the equilibrium pressures is the ‘equilibrium con- ⦵
line is ΔrG . However, as products are produced, there is a further
stant’ for the reaction (this quantity is also defined formally contribution to the Gibbs energy arising from their mixing (lowest
below). For a given reaction at a particular temperature ∆rG has

curve). The sum of the two contributions has a minimum, which
a certain value, so it follows from eqn 6A.8 that K has a certain corresponds to the equilibrium composition of the system.
6A The equilibrium constant 209

It follows from eqn 6A.8 that, when ΔrG > 0, K < 1.



is written in terms of the activity aJ as J  J  RT ln aJ

Therefore, at equilibrium the partial pressure of A exceeds that (Topic 5F). When this relation is substituted into the expres-
of B, which means that the reactant A is favoured in the equi- sion for ΔrG the result, after using x ln y = ln yx, is
librium. When ΔrG < 0, K > 1, so at equilibrium the partial

J
ln a

pressure of B exceeds that of A. Now the product B is favoured r G 

   J

in the equilibrium. It is often said that ‘a reaction is spontane-  rG   J J  RT  J ln aJ


ous if ΔrG < 0’. However, whether or not a reaction is sponta-



J J

  rG   RT  ln aJ J
⦵ 
neous at a particular composition depends on the value of ΔrG
J
at that composition, not ΔrG . The correct interpretation of the

sign of ΔrG is that it indicates whether K is greater or smaller




where ∆rG is the standard reaction Gibbs energy, which is the
than 1. difference in the standard chemical potentials of products and
reactants taking into account the stoichiometry of the reaction.

As noted above in practice ∆rG is calculated by using
6A.2(b) The general case of a reaction
 rG     f G     f G 
⦵ ⦵ ⦵
To extend the argument that led to eqn 6A.8 to a general reac-
Products Reactants
tion, first note that a chemical reaction may be expressed sym-
Standard reaction Gibbs energy
bolically in terms of stoichiometric numbers as [practical implementation]  (6A.10)

0   J J
Chemical equation
[symbolic form]
 (6A.9) where the n are the (positive) stoichiometric coefficients. More
J
formally,
where J denotes the substances and the n J are the correspond-
 rG    J  f G  (J) Standard reaction Gibbs energy
⦵ ⦵

ing stoichiometric numbers in the chemical equation. Note that [forrmal implementation]
 (6A.11)
J
whereas stoichiometric coefficients are positive, stoichiometric
numbers are positive for products and negative for reactants. In where the nJ are the (signed) stoichiometric numbers.
the reaction 2 A + B → 3 C + D, for instance,  A  2,  B  1, Because ln x1  ln x2    ln x1x2 , it follows that
 C  3, and  D  1.
 
With these points in mind, it is possible to write an expres-  ln x i  ln  xi 
 i 
sion for the reaction Gibbs energy, ∆rG, at any stage during the i

reaction. The symbol Π denotes the product of what follows it (just as Σ


denotes the sum). The expression for the Gibbs energy change
then simplifies to

 rG   rG   RT ln aJ J
⦵ 
How is that done? 6A.1 Deriving an expression for
J
the dependence of the reaction Gibbs energy on the
composition Step 2 Introduce the reaction quotient
Consider a reaction with stoichiometric numbers n J. When the Now define the reaction quotient as
reaction advances by dξ, the amounts of reactants and products Q  aJ J
 Reaction quotient
 (6A.12)
change by dnJ   Jd . The resulting infinitesimal change in the [definition]
J
Gibbs energy at constant temperature and pressure is Reactants have negative stoichiometric numbers, so they
 d automatically appear as the denominator when the product is
J
 
dG   J dnJ   J J d    J J  d written out explicitly. Therefore, this expression has the form
J J  J 
activities of products Reaction quootient
Q= [general form]
It follows that activities of reactants

 G  with the activity of each species raised to the power given by its
 rG      J J stoichiometric coefficient. It follows that the expression for the
   p ,T J
reaction Gibbs energy simplifies to
Step 1 Write the chemical potential in terms of the activity
The chemical potential of species J depends on the com- (6A.13)
ΔrG = ΔrG + RT ln Q

position of the mixture. For a mixture of perfect gases Reaction Gibbs energy
J  J  RT ln( pJ /p ). More generally, the chemical ­potential
⦵ ⦵
at an arbitrary stage
210 6 Chemical equilibrium

At equilibrium ΔrG = 0 and the activities then have their Brief illustration 6A.4
equilibrium values. It follows from eqn 6A.13 that
 The equilibrium constant for the heterogeneous equilibrium
 
 r G   RT ln Qequilibrium  RT ln  aJ J 

CaCO3(s) ⇌ CaO(s) + CO2(g) is
 J equilibrium
 1
The reaction quotient evaluated for the equilibrium activities is aCaO(s ) aCO2 ( g )
1
the thermodynamic equilibrium constant, K: K  aCaCO a a
3 ( s ) CaO ( s ) CO2 ( g )
  aCO2 ( g )
aCaCO3 (s )

  
   Equilibrium constant 1
K   aJ J  [definition]
 (6A.14)
 J equilibrium Provided the carbon dioxide can be treated as a perfect gas,
go on to write
From now on, the ‘equilibrium’ subscript will not be written ex-
plicitly, but it will be clear from the context that Q is defined in

K = pCO2 /p
terms of the activities at an arbitrary stage of the reaction and K
is the value of Q at equilibrium. It follows that and conclude that in this case the equilibrium constant
depends only on the equilibrium pressure of CO2 above the
 r G   RT ln K
⦵ Thermodynamic equilibrium
 (6A.15) solid sample.
constant

This equation is an exact and highly important thermodynamic


relation. It allows the calculation of the equilibrium constant
for any reaction from tables of thermodynamic data, and hence Example 6A.1 Calculating an equilibrium constant
the prediction of the equilibrium composition of the reaction
mixture. Calculate the equilibrium constant for the ammonia synthesis
Because activities are dimensionless, the thermodynamic reaction, N2(g) + 3 H2(g) ⇌ 2 NH3(g), at 298 K, and show how
equilibrium constant is also dimensionless. In elementary ap- K is related to the equilibrium partial pressures of the species
plications, the activities that occur in eqn 6A.14 are often re- when the overall pressure is low enough for the gases to be
placed as follows: treated as perfect.
Collect your thoughts Calculate the standard reaction Gibbs
State Measure Approximation Definition energy using eqn 6A.10 and use its value in eqn 6A.15 to evalu-
for aJ ate the equilibrium constant. The expression for the equilib-
Solute molality bJ/bJ⦵ b⦵ = 1 mol kg−1 rium constant is obtained from eqn 6A.14. The gases are taken

molar concentration [J]/c⦵ c⦵ = 1 mol dm−3 to be perfect, so you can replace each activity by the ratio pJ/p ,
Gas phase partial pressure pJ/p⦵
p⦵ = 1 bar where pJ is the partial pressure of species J.
Pure solid, liquid 1 (exact) The solution The standard Gibbs energy of the reaction is

⦵ ⦵ ⦵ ⦵
 rG   2 f G  (NH3 ,g )  { f G  (N2 ,g )  3 f G  (H2 ,g )}
Note that the activity is 1 for pure solids and liquids, so such
 2 f G  (NH3 ,g )  2  (16.45 kJmol 1 )

substances make no contribution to Q or K even though they


might appear in the chemical equation. When the approxi-
mations are made, the resulting values for Q and K are only Then,
approximate. The approximation is particularly severe for elec-
trolyte solutions, for in them activity coefficients differ from 1  rG 

2  (1.645  104 Jmol 1 )
ln K     1 3. 2 
even in very dilute solutions (Topic 5F). RT (8.3145 JK 1 mol 1 )  (298 K)

Hence, K = 5.8 × 105. This result is thermodynamically exact.


Brief illustration 6A.3
The thermodynamic equilibrium constant for the reaction is
Consider the reaction 2 A + 3 B → C + 2 D, in which case 2
aNH
nA = −2, nB = −3, nC = +1, and nD = +2. The reaction quotient K= 3

is then aN2 aH3 2

aCaD2 and has the value just calculated. At low overall pressures, the
Q  aA2aB3aCaD2 
aA2 aB3

activities can be replaced by the ratios pJ/p and an approxi-
mate form of the equilibrium constant is
6A The equilibrium constant 211

( pNH3 /p )2 2
pNH p 2   12   p1/ 2
⦵ ⦵ 1/ 2
pH2 pO1/22  3/ 2 p1/ 2
=K =  3
3
K  

( pN2 /p )( pH2 /p ) pN2 pH3 2
⦵ ⦵

(1   ) 1  12  
 1/ 2 1/ 2  1/ 2
(1   )(2   )1/ 2 p
⦵ ⦵
pH2 O p p
 1/ 2

Self-test 6A.1 Evaluate the equilibrium constant for N2O4(g) ⇌ Now make the approximation that α << 1, so 1 − α ≈ 1 and
2 NO2(g) at 298 K. 2 + α ≈ 2, and hence obtain
Answer: K = 0.15
1/ 2
 3/ 2 p1/ 2   3 p 
K  1/ 2 ⦵ 1/ 2   ⦵ 
2 p  2p 
Example 6A.2 Estimating the degree of dissociation at
Under the stated conditions, p = 1.00 bar (that is, p/p⦵ = 1.00),
equilibrium so α ≈ (2K2)1/3 = 0.0205. That is, about 2 per cent of the water
The degree of dissociation (or extent of dissociation) α is defined has decomposed. The assumption that α << 1 is justified by the
as the fraction of reactant that has decomposed. If the initial final result.
amount of reactant is n and the amount at equilibrium is neq, Self-test 6A.2 For the same reaction, the standard Gibbs energy
then α = (n − neq)/n. The standard reaction Gibbs energy for the of reaction at 2000 K is +135.2 kJ mol−1. Suppose that steam is
decomposition H2O(g )  H2 (g )  12 O2 (g ) is +118.08 kJ mol−1 at passed through a furnace tube at that temperature. Calculate
2300 K. What is the degree of dissociation of H2O at 2300 K the mole fraction of O2 present in the output gas stream,
when the reaction is allowed to come to equilibrium at a total assuming that the total pressure is 200 kPa.
pressure of 1.00 bar?
Answer: 0.00221
Collect your thoughts The equilibrium constant is obtained
from the standard Gibbs energy of reaction by using eqn
6A.15, so your task is to relate the degree of dissociation, α,
to K and then to find its numerical value. Proceed by express-
ing the equilibrium compositions in terms of α, and solve for The relation between equilibrium
6A.2(c)
α in terms of K. The standard reaction Gibbs energy is large constants
and positive, so you can anticipate that K << 1 and hence that
An equilibrium constant is defined in terms of activities. For
α << 1, which opens the way to making approximations to
practical applications, however, it is often necessary to relate the
obtain its numerical value.
value of the equilibrium constant to concentrations. In Topic
The solution The equilibrium constant is obtained from 5F it is shown that the activity and concentration of a species
eqn 6A.15 in the form J are related by the activity coefficient γJ. If the concentration
is expressed as a molality bJ, the activity is aJ = γJbJ/b . If it is


 G 1.1808  105 Jmol 1 written as a molar concentration [J], the activity is aJ = γJ[J]/c .

ln K   r   6.17 
RT (8.3145 JK 1 mol 1 )  (2300 K) For example, the equilibrium constant of a reaction of the form
A(aq)  B(aq) + C(aq) between solutes is expressed in terms
It follows that K = 2.08 × 10−3. The equilibrium composition of molalities as
is expressed in terms of α by drawing up the following table:
aB aC  B C bBbC
K    K Kb  (6A.16)
aA  A bA b

H2O → H2 + 12 O2

Initial amount n 0 0 The activity coefficients must be evaluated at the equilibrium


composition of the mixture (for instance, by using one of the
Change to reach −αn +αn  12  n Debye–Hückel expressions, Topic 5F). The calculation might
equilibrium
become complicated because the activity coefficients can
Amount at (1 − α)n αn 1
αn Total: 1  12   n
equilibrium
2
be evaluated only if the equilibrium composition is already
known. In elementary applications, and to begin the iterative
Mole fraction, xJ 1   1

1  12  1  12  1  12 
2
calculation of the concentrations in a real example, the assump-
tion is often made that the activity coefficients are all so close
Partial pressure, pJ (1   ) p p p
1
2
to unity that Kγ = 1. In that case, K ≈ Kb, which allows equilibria
1  12  1  12  1  21  to be discussed in terms of the molalities (or molar concentra-
tions) themselves.
A special case arises when the equilibrium constant of a gas-
where, for the entries in the last row, pJ = xJp (eqn 1A.6) has phase reaction is to be expressed in terms of molar concentra-
been used. The equilibrium constant is therefore tions instead of the partial pressures that appear in the formal
212 6 Chemical equilibrium

expression for the thermodynamic equilibrium constant. If the 6A.2(d)Molecular interpretation of the
gas is perfect and at partial pressure pJ it follows that pJV = nJRT. equilibrium constant
It follows that pJ = (nJ/V)RT = [J]RT, where [J] is the molar con-
centration of J. The substitution of this relation for pJ in the ex- Deeper insight into the origin and significance of the equilib-
pression for K gives rium constant can be obtained by considering the Boltzmann
distribution of molecules over the available states of a system
 p 
J 
 [J]RT 
J 
composed of reactants and products (see Energy: A first look).
K  a    ⦵J     ⦵ 
J
J The reactant and product molecules each have their character-
J J  p  J  p 
J J
istic sets of energy levels, but the Boltzmann distribution does

 RT   [J] 
J
 c ⦵ RT  not distinguish between their identities, only their energies.
 [J] J    ⦵     ⦵     ⦵ 


J J  p  J c  J  p  The molecules distribute themselves over all the energy levels


in accord with the Boltzmann distribution. At a given tempera-
(To obtain the first term of the second line, note that prod- ture, there is a specific distribution of populations, and hence a
ucts can always be factorized in this way: abcdef is the same specific composition of the reaction mixture.
as abc × def.) The (dimensionless) equilibrium constant Kc is Consider the equilibrium A  B. Figure 6A.4 shows the
defined as energy levels of A and those of B, and how the molecules
are distributed over these levels. In (a), A and B have simi-
J
 [J]  lar arrays of molecular energy levels, but the ground state
K c    ⦵  K for gas-phase reactions [definition]  (6A.17)
J c 
c of B is higher in energy than that of A; that is, the reac-
tion is endothermic. In this case, the energy levels of A are
It follows that more populated than those of B; therefore, A is favoured at
J equilibrium. In (b) the energy levels of A are more widely
 c ⦵ RT  spaced than those of B. As a result, the total population of
K  K c    ⦵   (6A.18a)
J  p  the levels of A is less than that of B. Now B is favoured at
equilibrium.
This expression is simplified by defining    J J , which is The different outcome of the two cases can be interpreted in
easier to think of as  (products)  (reactants) , to give terms of entropy. First, note that the more closely spaced are the
 energy levels, the higher is the entropy (Topic 3A). Therefore,
 c ⦵ RT  Relation between K and K c the entropy of B in (b) is higher than that of A. Although the
K  K c   ⦵  for gas-phase reactions
 (6A.18b)
 p 
In this expression, ∆n is the change in the number of gase-
ous species as the reaction proceeds according to the written A B A B
stoichiometric equation. With p = 1 bar and c = 1 mol dm−3,
⦵ ⦵

p /c R evaluates to 12.03 K.
⦵ ⦵
Energy, E

Energy, E

Brief illustration 6A.5 Boltzmann Boltzmann


distribution distribution
For the reaction N2(g) + 3 H2(g) → 2 NH3(g), ∆n = 2 − 3 − 1 =
−2, so
2
Population, P Population, P
2
 T   12.03 K  (a) (b)
K  Kc    Kc  
 12.03 K  
   T  Fig. 6A.4 The Boltzmann distribution of populations over the
energy levels of two species A and B. The reaction A → B is
At 298.15 K the relation is endothermic in this example. In (a) the two species have similar
2
densities of energy levels: the bulk of the population is associated
 12.03 K  Kc with the species A, so that species is dominant at equilibrium. In
K  Kc   
 298.15 K  614.2 (b) the density of energy levels in B is much greater than that in A,
 
and as a result, even though the reaction A → B is endothermic,
so Kc = 614.2K. Note that both K and Kc are dimensionless. the population associated with B is greater than that associated
with A, so B is dominant at equilibrium.
6A The equilibrium constant 213

reaction is endothermic, the effect of entropy is to favour the Note that the exenthalpic character of the reaction encour-
formation of product. ages the formation of products (it results in a large increase in
The role of entropy can be seen most clearly by rewriting eqn entropy of the surroundings). However, the decrease in entropy
 ⦵

6A.15 as K  e  r G /RT and then using ∆rG = ∆rH − T∆rS : of the system as H atoms are pinned to N atoms opposes their
⦵ ⦵ ⦵

formation.



/RT r S 

K  e  r H e /R (6A.19)

Note that a positive standard reaction enthalpy results in a low


equilibrium constant. That is, an endenthalpic reaction can
be expected to have an equilibrium composition that favours Exercises
the reactants. However, even in an endenthalpic reaction, if the E6A.4 Write the reaction quotient for A + 2 B → 3 C.
standard reaction entropy is large and positive, K may still ex-
E6A.5 Write the equilibrium constant for the reaction P4(s) + 6 H2(g) ⇌
ceed 1 and the equilibrium composition favour products. 4 PH3(g), with the gases treated as perfect.
−1
E6A.6 One reaction has a standard Gibbs energy of −320 kJ mol and a second
reaction has a standard Gibbs energy of −55 kJ mol−1, both at 300 K. What is
Brief illustration 6A.6 the ratio of their equilibrium constants at 300 K?
E6A.7 At 2257 K and 1.00 bar total pressure, water is 1.77 per cent
From data provided in the Resource section it is found that dissociated at equilibrium by way of the reaction 2 H2O(g) ⇌ 2 H2(g) +
for the reaction N2(g) + 3 H2(g) → 2 NH3(g) at 298 K, O2(g). Calculate K.
∆rG = −32.9 kJ mol−1, ∆rH = −92.2 kJ mol−1, and ∆rS =
⦵ ⦵ ⦵
E6A.8 Establish the relation between K and Kc for the reaction H2CO(g) ⇌
−1 −1
−198.8 J K mol . The contributions to K are therefore CO(g) + H2(g).

 ( 9.22104 J mol 1 ) / ( 8.3145 J K 1 mol 1 )( 298 K ) ( 198.8 J K 1 mol 1 ) / ( 8.3145 J K 1 mol 1 )
K e e
37.2  23.9 
=e e  5.98  104

Checklist of concepts
☐ 1. The reaction Gibbs energy ∆rG is the slope of the plot of ☐ 3. The reaction quotient is a combination of activities used
Gibbs energy against extent of reaction. to express the current value of the reaction Gibbs energy.
☐ 2. Reactions that have ∆rG < 0 are classified as exergonic, ☐ 4. The equilibrium constant is the value of the reaction
and those with ∆rG > 0 are classified as endergonic. quotient at equilibrium.

Checklist of equations
Property Equation Comment Equation number
Reaction Gibbs energy ΔrG = (∂G/∂ξ)p,T Definition 6A.1

 
⦵ ⦵ ⦵
Standard reaction Gibbs energy rG    f G    f G  n are positive; nJ are signed 6A.10 and 11
Products Reactants

  J  f G  (J)

 r G   r G   RT ln Q, Q  aJ J
⦵ 
Evaluated at arbitrary stage of reaction 6A.12 and 13
J

  
Equilibrium constant K   aJ J  Definition 6A.14
 J equilibrium

Thermodynamic equilibrium constant ΔrG⦵ = −RT ln K 6A.15


⦵ ∆n
Relation between K and Kc K = Kc(c RT/p ) ⦵
Gas-phase reactions; perfect gases 6A.18b
TOPIC 6B The response of equilibria to
the conditions

ΔrG , and hence of K, is therefore independent of the pres-


➤ Why do you need to know this material? sure at which the equilibrium is actually established; that is,
Chemists, and chemical engineers designing a chemical (K /p)T  0.
plant, need to know how the position of equilibrium will Although K itself is independent of pressure, the effect of
respond to changes in the conditions, such as a change pressure on the equilibrium composition depends on how the
in pressure or temperature. In another application of this pressure is applied. The pressure within a reaction vessel can be
material, the temperature dependence of the equilibrium increased by injecting an inert gas into it. In this case, you need
constant can be used to determine the standard enthalpy to note that the partial pressure of a perfect gas is the pressure it
and entropy of a reaction. would exert if it occupied the container on its own, so the pres-
ence of another gas has no effect on its value. Therefore, pro-
➤ What is the key idea? vided the gases are perfect, the addition of a pressurizing gas
A system at equilibrium, when subjected to a disturbance, leaves all the partial pressures of the reacting gases unchanged.
tends to respond by minimizing the effect of the disturbance. It follows that pressurization by the addition of an inert gas has
no effect on the equilibrium composition of the system (pro-
➤ What do you need to know already? vided the gases are perfect).
This Topic builds on the relation between the equilibrium Alternatively, the pressure of the system may be increased by
constant and the standard Gibbs energy of reaction (Topic confining the gases to a smaller volume (that is, by compres-
6A). The discussion of the temperature dependence of K sion). Now, although the equilibrium constant is unchanged,
makes use of the Gibbs–Helmholtz equation (Topic 3E). the individual partial pressures on which it depends do change
but in such a way as to leave K unchanged. Consider, for in-
stance, the perfect gas equilibrium A(g)  2 B(g), for which
the equilibrium constant is
The equilibrium constant for a reaction is not affected by the pB2
presence of a catalyst. As explained in detail in Topics 17F and K=
pA p

19C, catalysts increase the rate at which equilibrium is attained


but do not affect its position. The equilibrium constant is also The right-hand side of this expression remains constant
independent of pressure, but as will be seen, that does not nec- when the mixture is compressed only if an increase in pA
essarily mean that the composition at equilibrium is independ- cancels an increase in the square of pB. This relatively steep
ent of pressure. The equilibrium constant does depend on the increase of pA compared to pB implies that the equilibrium
temperature, and it turns out that its dependence can be pre- composition must shift in favour of A at the expense of B.
dicted from the standard reaction enthalpy. Then the number of A molecules will increase as the volume
The central relation of this Topic is eqn 6A.15 (ΔrG = of the container is decreased and the partial pressure of A

−RT ln K). It links the chemically interesting equilibrium con- will rise more rapidly than can be ascribed to a simple change
stant K and its properties to the quantity ΔrG , which can be in volume alone (Fig. 6B.1).

explored using thermodynamic arguments. The increase in the number of A molecules and the corre-
sponding decrease in the number of B molecules in the equilib-
rium A(g)  2 B(g) is a special case of a principle proposed by
the French chemist Henri Le Chatelier, which states that:
6B.1 The response to pressure
A system at equilibrium, when subjected to a disturbance,
The equilibrium constant depends on the value of ΔrG , tends to respond in a way that minimizes the effect of the

which is defined at a single, standard pressure. The value of disturbance.


6B The response of equilibria to the conditions 215

1
100

Degree of dissociation, α
0.8

0.6
10
0.4

0.2 1
(a) (b) 0.1
0
0 4 8 12 16
Figure 6B.1 When a reaction at equilibrium is compressed
p/p
(from a to b), the reaction responds by reducing the number of
molecules in the gas phase (in this case by producing the dimers Figure 6B.2 The pressure dependence of the degree of
represented by the linked spheres). dissociation, α, at equilibrium for an A(g)  2 B(g) reaction for
different values of the equilibrium constant K (the line labels). The
value α = 0 corresponds to pure A; α = 1 corresponds to pure B.

The principle implies that, if a system at equilibrium is com-


pressed, then the reaction will tend to adjust so as to minimize that as p is increased, α decreases, in accord with Le Chatelier’s
the increase in pressure. This minimization can be achieved by principle.
reducing the number of particles in the gas phase, which im-
plies a shift A(g) ← 2 B(g).
Brief illustration 6B.1
To treat the effect of compression quantitatively, suppose
that there is an amount n of A present initially (and no B). The equilibrium constant for the reaction N2O4(g) ⇌ 2 NO2(g)
At equilibrium the amount of A is nA = (1 − α)n, where α is K = 0.148 at 298 K. If the total pressure is 0.10 bar the degree
is the degree of dissociation of A into 2B. From the reaction of dissociation is
stoichiometry A(g) → 2 B(g), the amount of B is nB = 2αn
and the total amount of A and B molecules is ntot = (1 − α)n +  1 
1/ 2
 1
1/ 2

  
2αn = (1 + α)n. It follows that the mole fractions present at  

 1  4  (0.10 bar )/(0.148)  (1 bar ) 
 1  4 p/Kp   
equilibrium are
 0.52
nA (1   )n 1   An increase in total pressure to 0.2 bar gives α = 0.40. As
xA   
ntot (1   )n 1   expected from Le Chatelier’s principle, increasing the pressure
n 2 n 2 results in less dissociation, shifting the equilibrium composi-
xB  B   tion in favour of the dimer.
ntot (1   )n 1  

The equilibrium constant for the reaction, with pA = xAp and


pB = xBp, is Exercise
E6B.1 For the equilibrium
p2 x 2 p2 {2 /(1   )}2 p
K  B ⦵  B ⦵  PCl5(g)  PCl3(g) + Cl2(g)
pA p x A pp {(1   )/(1   )} p

ΔrG is +11.25 kJ mol−1 at 450 K. If the degree of dissociation of PCl5 is α, show


4 2 p/p  4 2 p/p
⦵ ⦵

  that the equilibrium constant is given by


(1   )(1   ) 12 2 p
K ⦵
(1   )(1   ) p
where p is the total pressure. This expression rearranges to
where p is the total pressure.
1/ 2 If the reaction takes place in a container of volume V show that, if the gases
 1  can be assumed to be perfect, p can be written as n0 (1   )RT /V , where n0 is
   
 (6B.1)
the initial amount of PCl5. Hence show that
 1  4 p/Kp 

 2 n0 RT
K
This formula shows that, even though K is independent of ⦵
(1   ) pV
pressure, the degree of dissociation, and hence the amounts Calculate α when 0.010 moles of PCl5 are introduced into a container with
of A and B, does depend on pressure (Fig. 6B.2). It also shows volume 1.0 dm3 and at 450 K and allowed to come to equilibrium.
216 6 Chemical equilibrium

6B.2 The response to temperature Step 2 Use the Gibbs–Helmholtz equation


To develop the preceding equation, use the Gibbs–Helmholtz
Le Chatelier’s principle predicts that a system at equilibrium equation (eqn 3E.10, d(G/T)/dT = −H/T 2) in the form
tends to shift in the endothermic direction if the temperature ⦵
d( rG  /T )  H

is raised, because then energy is absorbed as heat and the rise  r 2


dT T
in temperature is opposed. Conversely, an equilibrium can be
expected to tend to shift in the exothermic direction if the tem- ⦵
where ΔrH is the standard reaction enthalpy at the tempera-
perature is lowered, because then energy is released and the re- ture T. The combination of this equation with the expression
duction in temperature is opposed. These conclusions can be from Step 1 gives
summarized as follows:
d ln K  r H 

R 
Exothermic reactions: increased temperature favours the dT T2
reactants.
and therefore
Endothermic reactions: increased temperature favours the
products. d ln K ΔH

(6B.2)
= r 2
dT RT van ’t Hoff equation

6B.2(a) The van ’t Hoff equation


Equation 6B.2 is known as the van ’t Hoff equation. For a
The response to temperature can be explored quantitatively by
reaction that is exenthalpic (like most exothermic reactions)
deriving an expression for the slope of a plot showing the de-
under standard conditions (ΔrH < 0), the equation implies that

pendence of the equilibrium constant (specifically, of ln K) on


d ln K/dT < 0 (and therefore that dK/dT < 0). A negative slope
the temperature.
means that ln K, and therefore K itself, decreases as the tem-
perature rises. Therefore, in line with Le Chatelier’s principle, in
the case of an exenthalpic reaction, the equilibrium shifts away
How is that done? 6B.1 Deriving an expression for the from products as the temperature increases. The opposite oc-
curs in the case of endenthalpic reactions.
variation of ln K with temperature
Insight into the thermodynamic basis of this behaviour
The starting point for this derivation is eqn 6A.15 (ΔrG =

comes from the expression ΔrG = ΔrH − TΔrS written in
⦵ ⦵ ⦵

−RT ln K), in the form the form −ΔrG /T = −ΔrH /T + ΔrS . When the reaction is ex-
⦵ ⦵ ⦵

enthalpic under standard conditions, −ΔrH /T corresponds to




 G
ln K   r a positive change of entropy of the surroundings. When the
RT temperature is raised, −ΔrH /T decreases and the increasing

Now follow these steps. entropy of the surroundings has a less important role. As a re-
sult, the equilibrium lies less to the right. When the reaction is
Step 1 Differentiate the expression for ln K with respect to
endenthalpic under standard conditions (like most endother-
temperature, T
mic reactions), the contribution of the unfavourable change
The derivative of the left-hand side is simply d ln K /dT . In of entropy of the surroundings is reduced if the temperature is
calculating the derivative of the right-hand side remember that raised (because then ΔrH /T is less negative), and the equilib-


ΔrG depends on the temperature. For reasons that become rium composition shifts towards products.
clear in the next step, it is convenient to combine the temper-
These remarks have a molecular basis that stems from the
ature-dependent terms and to take the derivative of ∆ rG  /T .

Boltzmann distribution of molecules over the available energy


The overall result is
levels (see Energy: A first look). The typical arrangement of en-
d ln K

1 d( rG  /T ) ergy levels for an endothermic reaction A → B (and therefore
 typically of endenthalpic reactions too) is shown in Fig. 6B.3a.
dT R dT
When the temperature is increased, the Boltzmann distribution
which can be rearranged into adjusts and the populations change as shown. The change cor-
⦵ responds to an increased population of the higher energy states
d( rG  /T ) d ln K
 R at the expense of the population of the lower energy states. The
dT dT
states that belong to the B molecules become more populated
The differentials are complete (that is, they are not partial at the expense of the A molecules. Therefore, the total popu-
lation of B states increases, and B becomes more abundant in

derivatives) because K and ΔrG depend only on temperature,
and not on pressure. the equilibrium mixture. Conversely, if the reaction A → B is
6B The response of equilibria to the conditions 217

A B A B
The solution Draw up the following table:

T/K 350 400 450 500


(103 K)/T 2.86 2.50 2.22 2.00
–ln K 7.83 4.26 1.68 −0.392
Energy, E

High temperature High temperature These points are plotted in Fig. 6B.4. The slope of the graph is
Endothermic

Exothermic
+9.6 × 103. It follows from slope × K = ΔrH /R that

Low temperature Low temperature

 r H   (9.6  103 K )  R  80 kJmol 1


Population, P Population, P 8
(a) (b)

Figure 6B.3 The effect of temperature on a chemical equilibrium


6
can be interpreted in terms of the change in the Boltzmann
distribution with temperature and the effect of that change in
the population of the species. (a) In an endothermic reaction

–ln K
4
A → B, the population of B increases at the expense of A as the
temperature is raised. (b) If the same reaction is exothermic, the
opposite happens. 2

0
e­xothermic (Fig. 6B.3b), and therefore typically exenthalpic 2 2.2 2.4 2.6 2.8 3
(103 K)/T
too, then an increase in temperature increases the population
of the A states (which start at higher energy) at the expense of Figure 6B.4 When −ln K is plotted against 1/T, a straight line
the B states, so the reactants become more abundant. ⦵
is expected with slope equal to ΔrH /R if the standard reaction
enthalpy does not vary appreciably with temperature over the range
of the data. The points plotted are from the data in Example 6B.1.

Example 6B.1 Measuring a standard reaction enthalpy Self-test 6B.1 The equilibrium constant of the reaction
2 SO2(g) + O2(g)  2 SO3(g) is 4.22 × 105 at 650 K, 3.00 × 104 at
The data below show the temperature variation of the equilib- 700 K, and 3.04 × 103 at 750 K. Estimate the standard reaction
rium constant of the reaction Ag2CO3(s)  Ag2O(s) + CO2(g). enthalpy at 700 K.
Calculate the standard reaction enthalpy of the decomposition. Answer: −200 kJ mol−1

T/K 350 400 450 500


−4 −2 −1
K 3.98 × 10 1.41 × 10 1.86 × 10 1.48

Collect your thoughts You need to adapt the van ’t Hoff equa- The temperature dependence of the equilibrium constant
tion into a form that when plotted corresponds to a straight provides a non-calorimetric method of determining ΔrH . A

line. Start by noting that d(1/T)/dT = −1/T 2, which rearranges drawback is that the standard reaction enthalpy is actually tem-
to dT = −T 2d(1/T). When this expression is substituted into the perature-dependent, so the plot is not expected to be perfectly
left-hand side of eqn 6B.2 the result is linear. However, the temperature dependence is weak in many
d ln K r H 
⦵ cases, so the plot is reasonably straight. In practice, the method
 is not very accurate, but it is often the only one available.
T 2d(l/T ) RT 2
Cancellation of T2 gives
6B.2(b) The value of K at different
d ln K  r H 

  temperatures
d(l/T ) R
To find the value of the equilibrium constant at a temperature
Therefore, provided the standard reaction enthalpy is inde- T2 in terms of its value K1 at another temperature T1, integrate
pendent of temperature over the range of the data, a plot of eqn 6B.2 between these two temperatures:
−ln K against 1/T should be a straight line of slope ΔrH /R.

1 T2  r H 

The actual plot of dimensionless quantities is of −ln K against
R T1 T 2

ln K 2  ln K1  dT  (6B.3)
1/(T/K), so equate ΔrH /R to slope × K.
218 6 Chemical equilibrium

If ΔrH varies only slightly with temperature over the tempera-



 9.22  104 Jmol 1   1 1 
ln K 2  ln(5.8  105 )    
ture range of interest, it may be taken outside the integral. It  8.3145 JK 1 mol 1   500 K 298 K 
   
follows that
 1.7 
Integral A.1,
n 2
  That is, K2 = 0.17, a lower value than at 298 K, as expected for
 H T2 1

this exenthalpic reaction.
ln K 2  ln K1  r
R T1 T 2 dT
and therefore that
r H   1 1 

Temperature Exercises
ln K 2  ln K1      dependence of K
 (6B.4)
R  T2 T1  E6B.2 The standard reaction enthalpy of Zn(s) + H2O(g) → ZnO(s) + H2(g)
is approximately constant at +224 kJ mol−1 from 920 K up to 1280 K. The
standard reaction Gibbs energy is +33 kJ mol−1 at 1280 K. Estimate the
temperature at which the equilibrium constant becomes greater than 1.
Brief illustration 6B.2
E6B.3 Dinitrogen tetroxide dissociates according to the equilibrium
N2O4(g) ⇌ 2 NO2(g); at 25 °C and 1.00 bar dinitrogen tetroxide is
To estimate the equilibrium constant for the synthesis of ammo-
18.46 per cent dissociated. Calculate K at (i) 25 °C, (ii) 100 °C given
nia at 500 K from its value at 298 K (5.8 × 105 for the reaction ⦵
that ΔrH = +56.2 kJ mol−1 over the temperature range.
written as N2(g) + 3 H2(g) ⇌ 2 NH3(g)), use the standard reac-
E6B.4 What is the standard enthalpy of a reaction for which the equilibrium
tion enthalpy. That value can be obtained from Table 2C.7 in the constant is (i) doubled, (ii) halved when the temperature is increased by 10 K
Resource section by using ΔrH = 2ΔfH (NH3,g). Assume that
⦵ ⦵

at 298 K?
its value is constant over the range of temperatures. Then, with
ΔrH = −92.2 kJ mol−1, from eqn 6B.4 it follows that

Checklist of concepts
☐ 1. The thermodynamic equilibrium constant is independ- ☐ 3. The dependence of the equilibrium constant on the
ent of the presence of a catalyst and independent of temperature is expressed by the van ’t Hoff equation
pressure. and can be explained in terms of the distribution of
☐ 2. The response of composition to changes in the condi- molecules over the available states.
tions is summarized by Le Chatelier’s principle.

Checklist of equations

Property Equation Comment Equation number


2
van ’t Hoff equation ⦵
d ln K/dT = ΔrH /RT 6B.2
d ln K/d(1/T) = −ΔrH⦵/R Alternative version
Temperature dependence of equilibrium constant ln K2 − ln K1 = −(ΔrH⦵/R)(1/T2−1/T1) ΔrH⦵ assumed constant 6B.4
TOPIC 6C Electrochemical cells

metal’ shown as part of the specification (e.g. platinum) is pre-


➤ Why do you need to know this material? sent to act as a source or sink of electrons. It takes no other part
One very special case of the material treated in Topic 6B, with in the reaction other than perhaps acting as a catalyst for it.
enormous fundamental, technological, and economic signifi- For the cell to function, the electrolytes in the two compart-
cance, concerns reactions that take place in electrochemical ments must be in physical contact so that ions can pass be-
cells. Moreover, the ability to make very precise measure- tween them, but the contents of the compartments must not be
ments of potential differences means that electrochemical allowed to mix. In some cases, a porous solid ceramic is used
methods can be used to determine thermodynamic proper- to separate the electrolytes: bulk mixing is prevented, but ions
ties of reactions that might be inaccessible by other methods. can move through the pores in the material. Another option
is to join the compartments by a salt bridge, which is a tube
➤ What is the key idea? containing a concentrated electrolyte solution (for instance,
The electrical work that a reaction can perform at constant potassium chloride in agar jelly). If the electrolyte in the two
pressure and temperature is equal to the reaction Gibbs compartments is the same, no special measures are needed, and
energy. the electrodes can share the common electrolyte.
A galvanic cell is an electrochemical cell that produces elec-
➤ What do you need to know already? tricity as a result of the spontaneous reaction occurring inside
This Topic develops the relation between the Gibbs energy it. An electrolytic cell is an electrochemical cell in which a
and non-expansion work (Topic 3D). You need to be aware non-spontaneous reaction is driven by an external source of
of how to calculate the work of moving a charge through current.
an electrical potential difference (Topic 2A). The equations
make use of the definition of the reaction quotient Q and
the equilibrium constant K (Topic 6A).
6C.1 Half-reactions and electrodes
It should be familiar from introductory chemistry that ­oxidation
is the removal of electrons from a species, reduction is the ad-
An electrochemical cell consists of two electrodes, or metallic
dition of electrons to a species, and a redox reaction is one in
conductors, each in contact with an electrolyte, an ionic con-
which there is a transfer of electrons from one species to another.
ductor (which may be a solution, a liquid, or a solid). An elec-
The electron transfer may be accompanied by other events, such
trode and its electrolyte comprise an electrode ­compartment;
as atom or ion transfer. The reducing agent (or reductant) is the
the two electrodes may share the same electrolyte. The various
electron donor; the oxidizing agent (or oxidant) is the electron
kinds of electrode are summarized in Table 6C.1. Any ‘inert
acceptor. Any redox reaction may be expressed as the difference
of two reduction half-reactions, which are conceptual reactions
showing the gain of electrons. Even reactions that are not redox
Table 6C.1 Varieties of electrode reactions can often be expressed as the difference of two reduc-
tion half-reactions. The reduced and oxidized species in a half-
Electrode Redox
type
Designation
couple
Half-reaction reaction form a redox couple. A couple is denoted Ox/Red and
the corresponding reduction half-reaction is written
Metal/ M(s)|M+(aq) M+/M M+(aq) + e− → M(s)
metal ion
Ox   e   Red  (6C.1)
+ +
Gas Pt(s)|X2(g)|X (aq) X /X2  
X (aq )  e  X 2 (g )
1
2

Pt(s)|X2(g)|X−(aq) X2/X− 1
2 X 2 (g )  e   X  (aq )
− −
Brief illustration 6C.1
Metal/ M(s)|MX(s)|X (aq) MX/M,X MX(s) + e− → M(s) +
insoluble X−(aq)
salt The dissolution of silver chloride in water AgCl(s) → Ag+(aq) +
Redox Pt(s)|M+(aq),M2+(aq) M2+/M+ M2+(aq) + e− → M+(aq) Cl−(aq), which is not a redox reaction, can be expressed as the
difference of the following two reduction half-reactions:
220 6 Chemical equilibrium

AgCl(s)  e   Ag(s)  Cl  (aq ) – + Electrons


 
Ag (aq )  e  Ag(s) Anode Cathode
− +
The redox couples are AgCl/Ag,Cl and Ag /Ag, respectively.

It is often useful to express the composition of an electrode


Oxidation Reduction
compartment in terms of the reaction quotient, Q, for the half-
reaction. This quotient is defined like the reaction quotient for

the overall reaction (eqn 6A.12, Q  aJ J ), but the electrons are
Figure 6C.1 When a spontaneous reaction takes place in a
ignored because they are stateless. J
galvanic cell, electrons are deposited in one electrode (the site
of oxidation, the anode) and collected from another (the site
of reduction, the cathode), and so there is a net flow of current
which can be used to do work.
Brief illustration 6C.2

The reaction quotient for the reduction of O2 to H2O in acid


solution, O2(g) + 4 H+(aq) + 4 e− → 2 H2O(l), is 6C.2 Varieties of cell
aH2 2 O ⦵
p The simplest type of cell has a single electrolyte common
Q 4
 to both electrodes (as in Fig. 6C.1). In some cases it is neces-
a aO2
H
aH4  pO2
sary to immerse the electrodes in different electrolytes, as in
The approximations used in the second step are that the activ- the ‘Daniell cell’ in which the redox couple at one electrode is
ity of water is 1 (because the solution is dilute) and the oxygen Cu2+/Cu and at the other is Zn2+/Zn (Fig. 6C.2). In an electrolyte

behaves as a perfect gas, so aO2 = pO2 /p . concentration cell, the electrode compartments are identical
except for the concentrations of the electrolytes. In an electrode
­concentration cell the electrodes themselves have different con-
centrations, either because they are gas electrodes operating at
The reduction and oxidation processes responsible for the
different pressures or because they are amalgams (solutions in
overall reaction in a cell are separated in space: oxidation takes
mercury) or analogous materials with different concentrations.
place at one electrode and reduction takes place at the other. As
the reaction proceeds, the electrons released in the oxidation
Red1 → Ox1 + n e− at one electrode travel through the external 6C.2(a) Liquid junction potentials
circuit and re-enter the cell through the other electrode. There
In a cell with two different electrolyte solutions in contact,
they bring about reduction Ox2 + n e− → Red2. The electrode
as in the Daniell cell, there is an additional source of poten-
at which oxidation occurs is called the anode; the electrode at
tial difference across the interface of the two electrolytes.
which reduction occurs is called the cathode. In a galvanic cell,
the cathode has a higher potential than the anode: the spe-
cies undergoing reduction, Ox2, withdraws electrons from its – +
electrode (the cathode, Fig. 6C.1), so leaving a relative positive
charge on it (corresponding to a high potential). At the anode,
oxidation results in the transfer of electrons to the electrode, Porous
pot
so giving it a relative negative charge (corresponding to a low
potential). Zinc Copper
Zinc sulfate Copper(II) sulfate
solution
Exercise solution

E6C.1 Write down the half equations for the following electrodes

(i) Zn(s) in contact with an aqueous solution of Zn2+ ions. Figure 6C.2 One version of the Daniell cell. The copper electrode
(ii) H+(aq) in contact with H2(g), with an inert Pt electrode. is the cathode and the zinc electrode is the anode. Electrons leave
(iii) H+(aq) in contact with O2(g), with an inert Pt electrode (solvent H2O the cell from the zinc electrode and enter it again through the
can be used to balance the equation). copper electrode.
6C Electrochemical cells 221

Salt bridge The cell in Fig. 6C.3 is denoted


Electrode Electrode
Zn Cu
Zn(s) | ZnSO4 (aq ) || CuSO4 (aq ) | Cu(s)
An example of an electrolyte concentration cell in which the
liquid junction potential is assumed to be eliminated is
Pt(s) | H2 (g ) | HCl(aq ,b1 ) || HCl(aq ,b2 ) | H2 (g ) | Pt(s)

ZnSO4(aq) CuSO4(aq)
Exercise
Electrode compartments
E6C.2 Write down the two half reactions for each of the following cells

Figure 6C.3 The salt bridge, essentially an inverted U-tube full (i) Pt(s)|Fe3+(aq), Fe2+(aq)||H+(aq)|H2(g)|Pt(s)
of concentrated salt solution in a jelly, has two opposing liquid (ii) Ag(s)|AgCl(s)|HCl(aq)|Cl2(g)|Pt(s)
junction potentials that almost cancel. (iii) Pt(s)|O2(g)|H+(aq)|H2(g)|Pt(s)

This contribution is called the liquid junction potential, Elj.


Another example of a junction potential is that at the interface
between different concentrations of an acid, HA. At the junc- 6C.3 The cell potential
tion, the mobile H+ ions diffuse into the more dilute solution.
The bulkier A− ions follow, but initially do so more slowly, The current produced by a galvanic cell arises from the
which results in a potential difference at the junction. The po- ­spontaneous chemical reaction taking place inside it. The cell
tential then settles down to a value such that, after that brief reaction is the reaction in the cell written on the assumption
initial period, the ions diffuse at the same rates. All electrolyte that the right-hand electrode is the cathode. In other words, it
concentration cells have a liquid junction; electrode concen- is written on the assumption that reduction is taking place in
tration cells do not. the right-hand compartment. If the right-hand electrode is in
The contribution of the liquid junction to the potential dif- fact the cathode, then the cell reaction is spontaneous as writ-
ference can be reduced (to about 1–2 mV) by joining the elec- ten. If the left-hand electrode turns out to be the cathode, then
trolyte compartments through a salt bridge (Fig. 6C.3). The the reverse of the corresponding cell reaction is spontaneous.
reason for the success of the salt bridge is that, provided the To write the cell reaction corresponding to a cell diagram,
ions dissolved in the jelly have similar mobilities, then the liq- first write the right-hand half-reaction as a reduction. Then
uid junction potentials at either end are largely independent subtract from it the left-hand reduction half-reaction (because,
of the concentrations of the two dilute solutions, and so nearly by implication, that electrode is the site of oxidation). If neces-
cancel. sary, adjust the numbers of electrons in the two half-reactions
to be the same.
6C.2(b) Notation
The following notation is used for electrochemical cells: Brief illustration 6C.4

The two electrodes and their reduction half-reactions for the


| An interface between components or phases
cell Zn(s)|ZnSO4(aq)||CuSO4(aq)|Cu(s) are
⋮ A liquid junction
|| An interface for which it is assumed that the junction potential has Right-hand electrode: Cu 2  (aq )  2 e   Cu(s)
been eliminated
Left-hand electrode: Zn2  (aq )  2 e   Zn(s)

The same number of electrons is involved in each half-reaction.


Brief illustration 6C.3 The overall cell reaction is the difference Right − Left:

A cell in which two electrodes share the same electrolyte is Cu 2  (aq )  2 e   Zn2  (aq )  2 e   Cu(s)  Zn(s)
Pt(s) | H2 (g ) | HCl(aq ) | AgCl(s) | Ag(s) which, after cancellation of the 2 e−, rearranges to
The cell in Fig. 6C.2 is denoted Cu 2  (aq )  Zn(s)  Cu(s)  Zn2  (aq )
Zn(s) | ZnSO4 (aq )CuSO4 (aq ) | Cu(s)
222 6 Chemical equilibrium

6C.3(a) The Nernst equation This work is infinitesimal, and the composition of the system is
virtually constant when it occurs.
A cell in which the overall cell reaction has not reached chemi- If the reaction advances by dξ, then ndξ electrons must travel
cal equilibrium can do electrical work as the reaction drives from the anode to the cathode, where n is the stoichiometric
electrons through an external circuit. The work that a given coefficient of the electrons in the half-reactions into which
transfer of electrons can accomplish depends on the potential the cell reaction can be divided. The total charge transported
difference between the two electrodes. When the potential between the electrodes when this change occurs is −neNAdξ
difference is large, a given number of electrons travelling be- (because ndξ is the amount of electrons in moles and the
tween the electrodes can do a lot of electrical work. When the charge per mole of electrons is −eNA). Hence, because eNA = F,
potential difference is small, the same number of electrons can Faraday’s constant, the total charge transported is −nFdξ. The
do only a little work. A cell in which the overall reaction is at work done when an infinitesimal charge Q moves through a
equilibrium can do no work, and then the potential difference potential difference ∆ϕ is Q∆ϕ; in this case a charge −nFdξ
is zero. travels from the anode to the cathode between which the
According to the discussion in Topic 3D, the maximum non- potential difference is Ecell so
expansion work a system can do at constant temperature and dwe   FEcell d
pressure is given by eqn 3D.8 (wadd,max = ∆G). In electrochem-
By comparing this relation to dwe = ∆rGdξ it follows that
istry, the additional (non-expansion) work is identified with
electrical work, we: the system is the cell, and ∆G is the Gibbs (6C.2)
energy of the cell reaction, ∆rG. Because maximum work is pro- −FEcell = ΔrG
The cell potential
duced when a change occurs reversibly, it follows that, to draw
thermodynamic conclusions from measurements of the work
that a cell can do, it is necessary to ensure that the cell is operat- This equation is the key connection between electrical meas-
ing reversibly. Moreover, it is established in Topic 6A that the urements on the one hand and thermodynamic properties on
reaction Gibbs energy is actually a property relating, through the other. It is the basis of all that follows.
the term RT ln Q, to a specified composition of the reaction It follows from eqn 6C.2 that, by knowing the reaction Gibbs
mixture. Therefore, the cell must be operating reversibly at a energy at a specified composition, the cell potential is known
specific, constant composition. Both conditions are achieved by at that composition. Note that a negative reaction Gibbs en-
measuring the potential difference generated by the cell when it ergy, signifying a spontaneous cell reaction, corresponds to a
is balanced by an exactly opposing source of potential differ- positive cell potential, one in which a voltmeter connected to
ence. Then the cell reaction occurs reversibly, the composition the cell shows that the right-hand electrode (as in the specifi-
is constant, and no current flows. In effect, the cell reaction is cation of the cell, not necessarily how the cell is arranged on
poised for change, but not actually changing. The resulting po- the bench) is the positive electrode. Another way of looking at
tential difference is called the cell potential, Ecell, of the cell.1 the content of eqn 6C.2 is that it shows that the driving power
As this introduction has indicated, there is a close relation be- of a cell (that is, its potential difference) is proportional to the
tween the cell potential and the reaction Gibbs energy. It can be slope of the Gibbs energy with respect to the extent of reaction
established by considering the electrical work that a cell can do. (the significance of ΔrG). It is intuitively plausible that a reac-
tion that is far from equilibrium (when the slope is steep) has a
strong tendency to drive electrons through an external circuit
(Fig. 6C.4). When the slope is close to zero (when the cell reac-
How is that done? 6C.1 Establishing the relation between
tion is close to equilibrium), the cell potential is small.
the cell potential and the reaction Gibbs energy
Consider the change in G when the cell reaction advances by an
Brief illustration 6C.5
infinitesimal amount dξ at some composition. From Topic 6A,
specifically the equation ΔrG = (∂G/∂ξ)T,p, it follows that Equation 6C.2 provides an electrical method for measuring a
dG   r G d reaction Gibbs energy at any composition of the reaction mix-
ture: simply measure the cell potential and convert it to ∆rG.
The maximum non-expansion (electrical) work, we, that the
Conversely, if the value of ∆rG is known at a particular composi-
reaction can do as it advances by dξ at constant temperature
tion, then it is possible to predict the cell potential. For example,
and pressure is therefore
if ∆rG = −1.0 × 102 kJ mol−1 and n = 1, then (using 1 J = 1 C V):
dw e   r G d
 rG (1.0  105 Jmol 1 )
Ecell     1. 0 V
1
The former name is electromotive force (emf), but the term is not approved F 1  (9.6485  104 C mol 1 )
by IUPAC because a potential difference is not a force.
6C Electrochemical cells 223

 RT
Ecell  Ecell  ln Q Nernst equation (6C.4)

E>0 F
ΔrG < 0
This equation for the cell potential in terms of the composition
Gibbs energy, G

is called the Nernst equation; the dependence that it predicts is


summarized in Fig. 6C.5.
E<0 Through eqn 6C.4, the standard cell potential can be inter-
Pure reactants

ΔrG > 0 preted as the cell potential when all the reactants and products
E=0
in the cell reaction are in their standard states, because then all

products
ΔrG = 0
activities are 1, so Q = 1 and ln Q = 0. However, the fact that the

Pure
0 Extent of reaction, ξ
standard cell potential is merely a disguised form of the stand-
ard reaction Gibbs energy (eqn 6C.3) should always be kept in
Figure 6C.4 A spontaneous reaction occurs in the direction of mind and underlies all its applications.
decreasing Gibbs energy. The spontaneous direction of change
can also be expressed in terms of the cell potential, Ecell. The cell
reaction is spontaneous as written when Ecell > 0. The reverse Brief illustration 6C.6
reaction is spontaneous when Ecell < 0. When the cell reaction is at
equilibrium, the cell potential is zero. Because RT/F = 25.7 mV at 25 °C, a practical form of the Nernst
equation at this temperature is

8 25.7 mV
Ecell  Ecell

 ln Q

6 
4 It then follows that, for a reaction in which n = 1, if Q is
increased by a factor of 10, with ln 10 = 2.303, then the cell
(E – E ⦵)/(RT/F)

2
potential decreases by 59.2 mV.
0 
–2 3

–4 2
An important feature of a standard cell potential is that it is
1
–6 unchanged if the chemical equation for the cell reaction is mul-
–8 tiplied by a numerical factor. A numerical factor increases the
–3 –2 –1 0 1 2 3
log Q value of the standard Gibbs energy for the reaction. However, it
also increases the number of electrons transferred by the same
Figure 6C.5 The variation of cell potential with the value of the 
factor, and by eqn 6C.3 the value of Ecell

remains unchanged. A
reaction quotient for the cell reaction for different values of n (the practical consequence is that a cell potential is independent of
number of electrons transferred). At 298 K, RT/F = 25.69 mV, so at this
the physical size of the cell. In other words, cell potential is an
temperature the vertical scale refers to multiples of this value. Note
intensive property.
that the horizontal axis is the common (base 10) logarithm of Q.

6C.3(b) Cells at equilibrium


The reaction Gibbs energy is related to the composition of
A special case of the Nernst equation has great importance
the reaction mixture by eqn 6A.13 (ΔrG = ΔrG + RT ln Q). It

in electrochemistry and provides a link to the discussion


follows, on division of both sides by −nF and recognizing that
of equilibrium in Topic 6A. Suppose the reaction has reached
ΔrG/(−nF) = Ecell, that
­equilibrium; then Q = K, where K is the equilibrium constant of
 r G  RT

the cell reaction. However, a chemical reaction at equilibrium
Ecell    ln Q cannot do work, and hence it generates zero potential differ-
F F
ence between the electrodes of a galvanic cell. Therefore, setting
The first term on the right is written Ecell = 0 and Q = K in the Nernst equation gives
r G 


Ecell  Standard cell potential [definition] (6C.3)

RT Equilibrium constant and
F 
Ecell

 ln K  (6C.5)
F standard cell potenttial
and called the standard cell potential. That is, the standard cell
potential is the standard reaction Gibbs energy expressed as a This very important equation (which could also have been ob-
potential difference (in volts). It follows that tained more directly by substituting eqn 6A.15, ΔrG = −RT ln K,

224 6 Chemical equilibrium

into eqn 6C.3) can be used to predict equilibrium constants and so it follows that  rG     f G  (Ag  , aq ). Therefore, with
⦵ ⦵

from measured standard cell potentials. n = 1,

 f G  (Ag  , aq )   rG   (FE  )
⦵ ⦵ ⦵

Brief illustration 6C.7


 (9.6485  104 C mol 1 )  (0.7996 V)
The standard potential of the Daniell cell is +1.10 V. Therefore,  77.15 kJmol 1
the equilibrium constant for the cell reaction Cu2+(aq) +
Zn(s) → Cu(s) + Zn2+(aq), for which n = 2, is K = 1.5 × 1037 at which is in close agreement with the value in Table 2C.7 of the
298 K. That is, the displacement of copper by zinc goes virtually Resource section.
to completion. Note that a cell potential of about 1 V is easily
measurable but corresponds to an equilibrium constant that
would be impossible to measure by direct chemical analysis. The temperature coefficient of the standard cell potential,

dEcell /dT , gives the standard entropy of the cell reaction. This

conclusion follows from the thermodynamic relation (∂G/∂T)p =


Exercises −S derived in Topic 3E and eqn 6C.3, which combine to give

E6C.3 Devise cells in which the following are the reactions and using the data dEcell  S
⦵ ⦵
Temperature coefficient of
given below calculate the standard cell potential in each case:  r standard cell po
otential
 (6C.6)
dT F
(i) 2 AgCl(s) + H2(g) → 2 HCl(aq) + 2 Ag(s) 
The derivative is complete (not partial) because Ecell , like ΔrG ,
⦵ ⦵

(ii) 2 H2(g) + O2(g) → 2 H2O(l)


is independent of the pressure. This relation provides an elec-
AgCl(s) + e− → Ag(s) + Cl− E⦵ = +0.22 V trochemical technique for obtaining standard reaction entro-
O2(g) + 4 H+(aq) + 4 e− → 2 H2O(l) E⦵ = +1.23 V pies and through them the entropies of ions in solution.
O2(g) + 2 H2O(l) + 4 e− → 4 OH−(aq) E⦵ = +0.40 V Finally, the combination of the results obtained so far leads
2 H2O(l) + 2 e− → 2 OH−(aq) + H2(g) E⦵ = −0.83 V to an expression for the standard reaction enthalpy:

E6C.4 The standard potential of a Daniell cell, with cell reaction Zn(s) +
 ⦵ 
dEcell


 r H   r G  T  r S   F  Ecell  T
  

⦵ ⦵ ⦵
(6C.7)
Cu2+(aq) → Zn2+(aq) + Cu(s), is +1.10 V at 25 °C. Calculate the corresponding  dT 
standard reaction Gibbs energy.
E6C.5 By how much does the cell potential change when Q is decreased by a
This expression provides a non-calorimetric method for meas-
uring ΔrH and, through the convention ΔfH (H+, aq) = 0, the
⦵ ⦵
factor of 10 for a reaction in which n = 2 at 298 K?
standard enthalpies of formation of ions in solution (Topic 2C).

The determination of
6C.4 Example 6C.1 Using the temperature coefficient of the
thermodynamic functions standard cell potential

The standard potential of a cell is related to the standard reac- The standard potential of the cell Pt(s)|H2(g)|HBr(aq)|AgBr(s)|
tion Gibbs energy through eqn 6C.3 written as  FEcell 

  r G .
⦵ Ag(s) was measured over a range of temperatures, and the data
Therefore, this important thermodynamic quantity can be ob- were found to fit the following polynomial:

tained by measuring Ecell . Its value can then be used to calculate


Ecell /V  0.07131  4.99  104 (T /K  298)

the Gibbs energy of formation of ions by using the convention  3.45  106 (T /K  298)2
explained in Topic 3D, that ΔfG (H+, aq) = 0.

The cell reaction is AgBr(s)  12 H2 (g )  Ag(s)  HBr(aq ), and


(as can be inferred from its half-reactions) has n = 1. Evaluate
Brief illustration 6C.8 the standard reaction Gibbs energy, enthalpy, and entropy at
298 K.
The reaction taking place in the cell
Collect your thoughts The standard Gibbs energy of reaction is
Pt(s) | H2 (g ) | H  (aq ) || Ag  (aq ) | Ag(s) Ecell

 0.7996 V


obtained by using eqn 6C.3 after evaluating Ecell

at 298 K and by
is using 1 V C = 1 J. The standard reaction entropy is obtained by
using eqn 6C.6, which involves differentiating the polynomial
Ag  (aq )  12 H2 (g )  H  (aq )  Ag(s)
with respect to T and then setting T = 298 K. The standard
With the exception of Ag+(aq), the standard Gibbs energy of reaction enthalpy is obtained by combining the values of the
formation of all the species in this cell are zero by convention, standard Gibbs energy and entropy.
6C Electrochemical cells 225

The solution At T = 298 K, Ecell



 0.07131 V, so  r H    rG   T  r S 
⦵ ⦵ ⦵ ⦵

 6.880 kJmol 1  (298 K )  (0.0481 kJK 1 mol 1 )


 rG    FEcell

 (1)  (9.6485  104 C mol 1 )  (0.07131 V)
⦵ ⦵

 21.2 kJmol 1
 6.880  103 C V mol 1  6.880 kJmol 1
One difficulty with this procedure lies in the accurate meas-
The temperature coefficient of the standard cell potential is
urement of small temperature coefficients of cell potential.

dEcell

Nevertheless, it is another example of the striking ability of
 4.99  104 V K 1  2(3.45  106 )(T /K  298) V K 1 thermodynamics to relate the apparently unrelated, in this case
dT
to relate electrical measurements to thermal properties.
At T = 298 K this expression evaluates to

Self-test 6C.1 Predict the standard potential of the Harned
dEcell

 4.99  104 V K 1 cell, Pt(s)|H2(g)|HCl(aq)|AgCl(s)|Ag(s), at 303 K from tables


dT of thermodynamic data.
Answer: +0.2222 V
So, from eqn 6C.6, the standard reaction entropy is

dEcell

r S    F  (1)  (9.6485  104 C mol 1 )


dT
 (4.99  104 V K 1 ) Exercise
1 1
 48.1 JK mol E6C.6 At 298 K a cell has a standard potential of +0.0458 V, and the


temperature coefficient of the standard potential, dEcell /dT , is +0.338 ×
The negative value stems in part from the elimination of gas in 10−3 V K−1. Calculate ΔrG⦵, ΔrS⦵, and ΔrH⦵ for the cell.
the cell reaction. It then follows that

Checklist of concepts
☐ 1. A cell reaction is expressed as the difference of two shows that the right-hand electrode (in the specification
reduction half-reactions; each half-reaction Ox + n e− → of the cell) is the positive electrode.
Red defines a redox couple denoted Ox/Red. ☐ 5. The Nernst equation relates the cell potential to the
☐ 2. Galvanic cells can have different electrodes or elec- composition of the reaction mixture.
trodes that differ in either the electrolyte or electrode ☐ 6. The standard cell potential may be used to calculate the
concentration. standard Gibbs energy of the cell reaction and hence its
☐ 3. A liquid junction potential arises at the junction of two equilibrium constant.
electrolyte solutions. ☐ 7. The temperature coefficient of the standard cell poten-
☐ 4. The cell potential is the potential difference measured tial is used to measure the standard entropy and stand-
under reversible conditions. It is positive if a voltmeter ard enthalpy of the cell reaction.

Checklist of equations
Property Equation Comment Equation number
Cell potential and reaction Gibbs energy −nFEcell = ΔrG Constant temperature and pressure 6C.2
⦵ ⦵
Standard cell potential 
Ecell   r G  / F Definition 6C.3

Nernst equation Ecell  Ecell  (RT / F ) ln Q

6C.4

Equilibrium constant of cell reaction Ecell  (RT / F ) ln K

6C.5
⦵ ⦵
Temperature coefficient of cell potential dEcell /dT   r S / F
 
6C.6
TOPIC 6D Electrode potentials

Table 6D.1 Standard potentials at 298 K⋆


➤ Why do you need to know this material? Couple E⦵/V
A very powerful, compact, and widely used way to report 4+ −
Ce (aq) + e → Ce (aq) 3+
+1.61
standard cell potentials is to ascribe a potential to each
Cu2+(aq) + 2 e− → Cu(s) +0.34
electrode. Electrode potentials are used in chemistry to − −
AgCl(s) + e → Ag(s) + Cl (aq) +0.22
assess the oxidizing and reducing power of redox couples
  1
and to infer thermodynamic properties, including equilib- H (aq )  e  H (g )
2 2
  0
rium constants. 2+
Zn (aq) + 2 e → Zn(s) −
−0.76
+ −
Na (aq) + e → Na(s) −2.71
➤ What is the key idea?

Each electrode of a cell can be supposed to make a char- More values are given in the Resource section.

acteristic contribution to the cell potential; redox couples


with low electrode potentials tend to reduce those with
higher potentials. gas must be 1 bar.1 The standard potential, E (X), of another

redox couple X is then equal to the cell potential in which it


➤ What do you need to know already?
forms the right-hand electrode and the standard hydrogen
This Topic develops the concepts in Topic 6C, so you need electrode is the left-hand electrode:
to understand the concept of cell potential and standard
Pt(s) | H2 (g ) | H  (aq ) || X E  (X )  Ecell

⦵ ⦵
cell potential; it also makes use of the Nernst equation.
The measurement of standard potentials makes use of the Standard potentials
Debye–Hückel limiting law (Topic 5F). [conveention]  (6D.2)

The standard potential of a cell of the form L||R, where L is


the left-hand electrode of the cell as written (not necessarily
as arranged on the bench) and R is the right-hand electrode,
As explained in Topic 6C, a galvanic cell is a combination of two is then given by the difference of the two standard (electrode)
electrodes. Each electrode can be considered to make a charac- ­potentials:
teristic contribution to the overall cell potential. Although it is 
L || R Ecell  E  (R )  E  (L) Standard cell potential (6D.3)
⦵ ⦵ ⦵

not possible to measure the contribution of a single electrode,


the potential of one of the electrodes can be defined as zero, so A list of standard potentials at 298 K is given in Table 6D.1,
values can be assigned to others on that basis. and longer lists in numerical and alphabetical order are in the
Resource section.

6D.1 Standard potentials Brief illustration 6D.1

The specially selected electrode is the standard hydrogen The cell Ag(s)|AgCl(s)|HCl(aq)|O2(g)|Pt(s) can be regarded as
­electrode (SHE): formed from the following two electrodes, with their standard
potentials taken from the Resource section:
Pt(s) | H2 (g ) | H  (aq ) E   0 at all temperatures

Standard potentiaals [convention] (6D.1) 1


Strictly speaking the fugacity, which is the equivalent of activity for a gas
(see A deeper look 5F.2, available to read in the e-book accompanying this text),
To achieve standard conditions, the activity of the hydrogen should be 1. This complication is ignored here, which is equivalent to assuming
ions must be 1 (that is, pH = 0) and the pressure of the ­hydrogen perfect gas behaviour.
6D Electrode potentials 227

Standard The equation for Ecell then becomes


Electrode Half-reaction
potential
R: Pt(s)|O2(g)|H+(aq) O2(g) + 4 H+(aq) + 4 e− → 2 H2O(l) +1.23 V 1/2
2 RT b 2 ART ln10 b
Ecell + ln =E + as b → 0

− − −
L: Ag(s)|AgCl(s)|Cl (aq) AgCl(s) + e → Ag(s) + Cl (aq) +0.22 V F b

F b



Ecell  1.01 V
With the boxed term denoted C, this equation becomes.
slope x
 y
 intercept 
 1/ 2
2RT b  b 
Ecell  ln ⦵  E   C   ⦵  

(6D.4)
6D.1(a) The measurement procedure F b b 
The procedure for measuring a standard potential can be illus- where C is a constant. To use this equation, which has the form
trated by considering a specific case, the silver/silver chloride y = intercept + slope × x with x = (b/b )1/2, the expression on the

electrode. The measurement is made on the ‘Harned cell’: left is evaluated at a range of molalities, plotted against (b/b )1/2, ⦵

and extrapolated to b = 0. The intercept at b1/2 = 0 is the value


Pt(s) | H2 (g ) | HCl(aq ,b) | AgCl(s) | Ag(s)
of E for the silver/silver chloride electrode. In precise work,

1
2
H2 (g )  AgCl(s)  HCl(aq )  Ag(s) the (b/b )1/2 term is brought to the left, and a higher-order cor-

 
E 
 E (AgCl/Ag,Cl )  E (SHE)  E (AgCl/Ag,Cl ),   1
  
rection term from extended versions of the Debye–Hückel law
⦵ ⦵ ⦵ ⦵
cell
(Topic 5F) is used on the right.
for which the Nernst equation is

RT aH aCl 
Ecell  E  (AgCl/Ag,Cl  )  ln 1/ 2

F aH2 Example 6D.1 Evaluating a standard potential


If the hydrogen gas is at the standard pressure of 1 bar, then
The potential of the Harned cell at 25 °C has the following
aH2 = 1. For simplicity, writing the standard potential of the
values:
AgCl/Ag,Cl− electrode as E , turns this equation into


b/(10−3b ) 3.215 5.619 9.138 25.63
RT

Ecell E  ln aH aCl 

Ecell/V 0.520 53 0.492 57 0.468 60 0.418 24
F

The activities in this expression can be written in terms of the Determine the standard potential of the silver/silver chloride
molality b of HCl(aq) through aH    b/b  and aCl     b/b, as
⦵ ⦵
electrode.
established in Topic 5F:
Collect your thoughts As explained in the text, evaluate y = Ecell +
(2RT/F) ln(b/b ) and plot it against (b/b )1/2; then extrapolate
⦵ ⦵

RT  b

2 2
Ecell E  ln ⦵2 

to b = 0.
F b
The solution To determine the standard potential of the cell,
2RT b 2RT
 E  ln ⦵  ln  

draw up the following table, using 2RT/F = 0.051 39 V at 298 K:
F b F

and therefore b/(10−3b )



3.215 5.619 9.138 25.63
−3 ⦵ 1/2
{b/(10 b )} 1.793 2.370 3.023   5.063
2RT b 2RT
Ecell  ln ⦵  E   ln  

Ecell/V 0.520 53 0.492 57 0.468 60   0.418 24
F b F
y/V 0.2256 0.2263 0.2273   0.2299
From the Debye–Hückel limiting law for a 1,1-electrolyte
(eqn 5F.27, log γ± = −A|z+z−|I1/2), it follows that as b → 0
The data are plotted in Fig. 6D.1; as can be seen, they extrapo-
late to E = +0.2232 V (the value obtained, to preserve the

log     A | z  z  | I 1/ 2
  A(b/b )  1/ 2

precision of the data, by linear regression).


Therefore, because ln x  ln10  log x , Self-test 6D.1 The following data are for the cell Pt(s)|H2(g)|
HBr(aq,b)|AgBr(s)|Ag(s) at 25 °C and with the hydrogen gas at
ln    ln10  log    ( A ln10)(b/b )1/ 2

1 bar. Determine the standard cell potential.
228 6 Chemical equilibrium

0.2300 The required reaction is


(c) Cu2+(aq) + e− → Cu+(aq)  E (c) = −ΔrG (c)/F
⦵ ⦵
0.2290
E/V + 0.051 39 ln(b/b )

0.2280 Because (c) = (a) − (b), the standard Gibbs energy of reaction (c) is
 rG  (c)   rG  (a )   rG  (b)  (0.680 V)F  (0.522 V)F
⦵ ⦵ ⦵
0.2270

0.2260  (0.158 V)F

Therefore, E (c) = −ΔrG (c)/F = +0.158 V.


⦵ ⦵
0.2250

Self-test 6D.2 Evaluate E (Fe3+,Fe2+) from E (Fe3+,Fe) and


⦵ ⦵
0.2240
E (Fe2+,Fe).

0.2230
0 1 2 3 4 5 Answer: +0.76 V
(b/10–3b )1/2

Figure 6D.1 The plot and the extrapolation used for the
experimental measurement of a standard cell potential. The

intercept at b1/2 = 0 is E cell. The generalization of the calculation in Example 6D.2 is

Combination of standard
 c E  (c)   a E  (a)  b E  (b)  (6D.5)
⦵ ⦵ ⦵
b/(10−4b⦵) 4.042 8.444 37.19
potentials
Ecell/V 0.469 42 0.436 36   0.361 73
Answer: +0.071 V with the nr the stoichiometric coefficients of the electrons in
each half-reaction.

6D.1(b) Combining measured values 6D.2Applications of standard


The standard potentials in Table 6D.1 may be combined to give electrode potentials
values for couples that are not listed there. However, to do so, it
is necessary to take into account the fact that different couples Cell potentials are a convenient source of data on equilibrium
might correspond to the transfer of different numbers of elec- constants and the Gibbs energies, enthalpies, and entropies of
trons. The procedure is illustrated in the following Example. reactions. In practice, the standard values of these quantities
are the ones normally determined and tabulated.

Example 6D.2 Evaluating a standard potential from two


6D.2(a) The electrochemical series
others Consider two redox couples, OxL/RedL and OxR/RedR, formed
Given that the standard potentials of the Cu /Cu and Cu /Cu 2+ + into the following cell
couples are +0.340 V and +0.522 V, respectively, evaluate L || R  Ox L /Red L || Ox R /Red R
E (Cu2+,Cu+).

Ox L   e   Red L Ox R   e   Red R  (6D.6a)


Collect your thoughts First, note that reaction Gibbs energies 
Ecell  E (R
⦵ 
R )  E (L)
⦵ 

may be added (as in a Hess’s law analysis of reaction enthalp-



ies). Therefore, you should convert the E values to ΔrG by

The cell reaction is
using eqn 6C.3 (−nFE = ΔrG ), add them appropriately, and
⦵ ⦵

⦵ ⦵ R  L : Red L  Ox R  Ox L  Red R  (6D.6b)


then convert the overall ΔrG to the required E by using eqn
6C.3 again. This roundabout procedure is necessary because, as This reaction has K > 1 if Ecell > 0, and therefore if E (L) < E (R).

⦵ ⦵ ⦵

seen below, although the factor F cancels (and should be kept Because in the cell reaction RedL reduces OxR, it follows that
in place until it cancels), the factor n in general does not cancel.
RedL has a thermodynamic tendency (in the sense K > 1)
The solution The electrode half-reactions are as follows: to reduce OxR if E (L) < E (R).
⦵ ⦵

(a) Cu2+(aq) + 2 e− → Cu(s)   E (a) = +0.340 V


More briefly: low reduces high.


so ΔrG (a) = −2 × (+0.340 V)F = (−0.680 V)F

Table 6D.2 shows a part of the electrochemical series, the


(b) Cu+(aq) + e− → Cu(s) metallic elements (and hydrogen) arranged in the order of
E (b) = +0.522 V, so ΔrG (b) = −(+0.522 V)F = (−0.522 V)F their reducing power as measured by their standard potentials
⦵ ⦵
6D Electrode potentials 229

Table 6D.2 The electrochemical series⋆ which can be rearranged into


E   Ecell

Least strongly reducing b
ln     ln
2RT /F b 

3+
Gold (Au /Au) (6D.7)
Platinum (Pt2+/Pt)
Silver (Ag+/Ag)
Brief illustration 6D.3
Mercury (Hg2+/Hg)
Copper (Cu2+/Cu) The data in Example 6D.1 include the fact that Ecell = 0.468 60 V
when b = 9.138 × 10−3b . Because 2RT/F = 0.051 39 V at 298 K,

Hydrogen (H+/H2)

and in the Example it is established that Ecell = 0.2232 V, the

Tin (Sn2+/Sn)
Nickel (Ni2+/Ni) mean activity coefficient at this molality is
Iron (Fe2+/Fe) 0.2232 V  0.46860 V
ln     ln(9.138  103 )  0.0799 
Zinc (Zn2+/Zn) 0.05139 V
Chromium (Cr3+/Cr)
Aluminium (Al3+/Al) Therefore, γ± = 0.9232.
Magnesium (Mg2+/Mg)
Sodium (Na+/Na)
Calcium (Ca2+/Ca) 6D.2(c) The determination of equilibrium
Potassium (K+/K) constants
Most strongly reducing
The principal use for standard potentials is to calculate the

The complete series can be inferred from Table 6D.1 in the Resource section. standard potential of a cell formed from any two electrodes and
then to use that value to evaluate the equilibrium constant of

the cell reaction. To do so, construct Ecell  E  (R )  E  (L) and
⦵ ⦵ ⦵

in aqueous solution. A metal low in the series (with a lower


then use eqn 6C.5 of Topic 6C (Ecell  (RT / F )ln K , arranged

­standard potential) can reduce the ions of metals with higher


into ln K   FEcell

/RT ).

standard potentials. This conclusion is qualitative. The quanti-


tative value of K is obtained by doing the calculations described
previously and reviewed below.
Brief illustration 6D.4

Brief illustration 6D.2 A disproportionation reaction is a reaction in which a species


is both oxidized and reduced. To study the disproportionation
Zinc lies above magnesium in the electrochemical series, so 2 Cu+(aq) → Cu(s) + Cu2+(aq) at 298 K, combine the following
zinc cannot reduce magnesium ions in aqueous solution. Zinc electrodes:
can reduce hydrogen ions, because hydrogen lies higher in the
series. However, even for reactions that are thermodynamically R : Cu(s) | Cu  (aq ) Cu  (aq )  e   Cu(s)

favourable, there may be kinetic factors that result in very slow E  (R )  0.52 V
rates of reaction. L : Pt(s) | Cu 2  (aq ),Cu  (aq ) Cu 2  (aq )  e   Cu  (aq )

E  (L)  0.16 V

6D.2(b)The determination of activity The cell reaction is therefore 2 Cu+(aq) → Cu(s) + Cu2+(aq),
and the standard cell potential is
coefficients

Ecell  (0.52 V)  (0.16 V)  0.36 V

Once the standard potential of an electrode in a cell is known,
it can be used to determine mean activity coefficients by meas-
Now calculate the equilibrium constant of the cell reaction.
uring the cell potential with the ions at the concentration of
Because n = 1, from eqn 6C.5 with RT/F = 0.025 693 V,
interest. For example, in the Harned cell, the mean activity
coefficient of the ions in hydrochloric acid of molality b is ob-  FEcell


0.36 V
tained from the relation ln K    14.0 
RT 0.025 693 V
2RT b 2RT
Ecell  ln ⦵  E   ln  

Hence, K = 1.2 × 106.
F b F
230 6 Chemical equilibrium

Sn(s) + 2 AgCl(s) ⇌ SnCl2(aq) + 2 Ag(s)


Exercises
AgCl(s) + e− → Ag(s) + Cl− E ⦵ = +0.22 V
E6D.1 Given the data below, will mercury produce zinc metal from aqueous
Sn2+(aq) + 2 e− → Sn(s) E ⦵ = −0.14 V
zinc sulfate under standard conditions?
E6D.3 Use the data given below to calculate the standard potential of the cell
Hg2SO4(s) + 2 e− → 2 Hg(l) + S O2− E ⦵ = +0.62 V
4 (aq) Ag(s)|AgNO3(aq)||Cu(NO3)2(aq)|Cu(s) and the standard Gibbs energy of the
2+ −
Zn (aq) + 2 e → Zn(s) ⦵
E = −0.76 V cell reaction at 25 °C.

Ag+(aq) + e− → Ag(s) E ⦵ = +0.80 V


E6D.2 Calculate the equilibrium constant of the following reaction at 25 °C from
2+ −
the standard potential data given below: Cu (aq) + 2 e → Cu(s) E ⦵ = +0.34 V

Checklist of concepts
☐ 1. The standard potential of a couple is the potential of a ☐ 3. The difference of the cell potential from its standard
cell in which the couple forms the right-hand electrode value is used to measure the activity coefficient of ions
and the left-hand electrode is a standard hydrogen elec- in solution.
trode, all species being present at unit activity. ☐ 4. Standard potentials are used to calculate the standard
☐ 2. The electrochemical series lists the metallic elements in cell potential and then to calculate the equilibrium con-
the order of their reducing power as measured by their stant of the cell reaction.
standard potentials in aqueous solution: low reduces high.

Checklist of equations
Property Equation Comment Equation number
⦵ ⦵ ⦵
Standard cell potential from standard potentials   
Ecell  E (R )  E (L) Cell: L||R 6D.3
Combined standard potentials ncE (c) = naE (a) − nbE (b)
⦵ ⦵ ⦵
6D.5
Exercises and problems 231

FOCUS 6 Chemical equilibrium

To test your understanding of this material, work through Selected solutions can be found at the end of this Focus in
the Exercises, Additional exercises, Discussion questions, and the e-book. Solutions to even-numbered questions are available
Problems found throughout this Focus. online only to lecturers.

TOPIC 6A The equilibrium constant


Discussion questions
D6A.1 Explain how the mixing of reactants and products affects the position D6A.2 What is the physical justification for not including a pure liquid or solid
of chemical equilibrium. in the expression for an equilibrium constant?

Additional exercises
E6A.9 Consider the reaction 2 A → B. Initially 1.75 mol A and 0.12 mol B E6A.21 In the gas-phase reaction 2 A + B ⇌ 3 C + 2 D it was found that,
are present. What are the amounts of A and B when the extent of reaction is when 1.00 mol A, 2.00 mol B, and 1.00 mol D were mixed and allowed to
0.30 mol? come to equilibrium at 25 °C, the resulting mixture contained 0.90 mol C at
a total pressure of 1.00 bar. Calculate (i) the mole fractions of each species at
E6A.10 When the reaction 2 A → B advances by 0.051 mol (i.e. Δξ = +0.051 mol)
equilibrium, (ii) K, and (iii) ΔrG⦵.
the molar Gibbs energy of the system changes by −2.41 kJ mol−1. What is the
Gibbs energy of reaction at this stage of the reaction? E6A.22 In the gas-phase reaction A + B ⇌ C + 2 D it was found that, when
2.00 mol A, 1.00 mol B, and 3.00 mol D were mixed and allowed to come
E6A.11 Classify the formation of liquid benzene from its elements in their
to equilibrium at 25 °C, the resulting mixture contained 0.79 mol C at a
reference states as exergonic or endergonic under standard conditions at 298 K.
total pressure of 1.00 bar. Calculate (i) the mole fractions of each species at
E6A.12 Write the reaction quotient for 2 A + B → 2 C + D. equilibrium, (ii) K, and (iii) ΔrG⦵.
E6A.13 Write the equilibrium constant for the reaction CH4(g) + 3 Cl2(g) ⇌ E6A.23 The standard reaction Gibbs energy of the isomerization of borneol
CHCl3(l) + 3 HCl(g), with the gases treated as perfect. (C10H17OH) to isoborneol in the gas phase at 503 K is +9.4 kJ mol−1. Calculate
the reaction Gibbs energy in a mixture consisting of 0.15 mol of borneol and
E6A.14 Use data found in the Resource section to decide which of the following
0.30 mol of isoborneol when the total pressure is 600 Torr.
reactions have K > 1 at 298 K: (i) 2 CH3CHO(g) + O2(g) ⇌ 2 CH3COOH(l),
(ii) 2 AgCl(s) + Br2(l) ⇌ 2 AgBr(s) + Cl2(g) E6A.24 The equilibrium pressure of H2 over solid uranium and uranium
hydride, UH3, at 500 K is 139 Pa. Calculate the standard Gibbs energy of
E6A.15 Use data found in the Resource section to decide which of the following
formation of UH3(s) at 500 K.
reactions have K < 1 at 298 K: (i) Hg(l) + Cl2(g) ⇌ HgCl2(s), (ii) Zn(s) +
−1
Cu2+(aq) ⇌ Zn2+(aq) + Cu(s). E6A.25 The standard Gibbs energy of formation of NH3(g) is −16.5 kJ mol
−1 at 298 K. What is the corresponding reaction Gibbs energy when the partial
E6A.16 One reaction has a standard Gibbs energy of −200 kJ mol and a
−1 pressures of the N2, H2, and NH3 (treated as perfect gases) are 3.0 bar, 1.0 bar,
second reaction has a standard Gibbs energy of +30 kJ mol , both at 300 K.
and 4.0 bar, respectively? What is the spontaneous direction of the reaction in
What is the ratio of their equilibrium constants at 300 K?
this case?
E6A.17 The standard Gibbs energy of the reaction N2(g) + 3 H2(g) → 2 NH3(g) −1
E6A.26 The standard Gibbs energy of formation of PH3(g) is +13.4 kJ mol
is −32.9 kJ mol−1 at 298 K. What is the value of ΔrG when Q = (i) 0.010, (ii) 1.0,
at 298 K. What is the corresponding reaction Gibbs energy when the
(iii) 10.0, (iv) 100 000, (v) 1 000 000? Estimate (by interpolation) the value of
partial pressures of the H2 and PH3 (treated as perfect gases) are 1.0 bar and
K from the values you calculate. What is the actual value of K?
0.60 bar, respectively? What is the spontaneous direction of the reaction in
E6A.18 The standard Gibbs energy of the reaction 2 NO2(g) → N2O4(g) is this case?
−4.73 kJ mol−1 at 298 K. What is the value of ΔrG when Q = (i) 0.10, (ii) 1.0, 2+ − −11
E6A.27 For CaF2(s) ⇌ Ca (aq) + 2 F (aq), K = 3.9 ×10 at 25 °C and the
(iii) 10, (iv) 100? Estimate (by interpolation) the value of K from the values
standard Gibbs energy of formation of CaF2(s) is −1167 kJ mol−1. Calculate the
you calculate. What is the actual value of K?
standard Gibbs energy of formation of CaF2(aq).
E6A.19 For the equilibrium, N2O4(g) ⇌ 2 NO2(g), the degree of dissociation, α, 2+ − −8
E6A.28 For PbI2(s) ⇌ Pb (aq) + 2 I (aq), K = 1.4 × 10 at 25 °C and the
at 298 K is 0.201 at 1.00 bar total pressure. Calculate K.
standard Gibbs energy of formation of PbI2(s) is −173.64 kJ mol−1. Calculate
E6A.20 Establish the relation between K and Kc for the reaction 3 N2(g) + the standard Gibbs energy of formation of PbI2(aq).
H2(g) ⇌ 2 HN3(g).
232 6 Chemical equilibrium

Problems
P6A.1 The equilibrium constant for the reaction, I2(s) + Br2(g) ⇌ 2 IBr(g) Standard reaction Gibbs energies at 190 K are as follows:
is 0.164 at 25 °C. (a) Calculate ΔrG⦵ for this reaction. (b) Bromine gas is (i) H2O(g) → H2O(s) ΔrG⦵ = −23.6 kJ mol−1
introduced into a container with excess solid iodine. The pressure and
(ii) H2O(g) + HNO3(g) → HNO3 · H2O(s) ΔrG⦵ = −57.2 kJ mol−1
temperature are held at 0.164 atm and 25 °C, respectively. Find the partial
pressure of IBr(g) at equilibrium. Assume that all the bromine is in the (iii) 2 H2O(g) + HNO3(g) → HNO3 · 2 H2O(s) ΔrG⦵ = −85.6 kJ mol−1
gaseous form and that the vapour pressure of iodine is negligible. (c) In fact, (iv) 3 H2O(g) + HNO3(g) → HNO3 · 3 H2O(s) ΔrG⦵ = −112.8 kJ mol−1
solid iodine has a measurable vapour pressure at 25 °C. In this case, how
would the calculation have to be modified? Which solid is thermodynamically most stable at 190 K if pH2O  0.13 bar
and pHNO3 = 0.41 nbar? Hint: Try computing ∆rG for each reaction under
P6A.2 Calculate the equilibrium constant of the reaction CO(g) + H2(g) ⇌ the prevailing conditions. If more than one solid forms spontaneously, then
H2CO(g) given that, for the production of liquid methanal (formaldehyde), examine ∆rG for the conversion of one solid to another.
ΔrG⦵ = +28.95 kJ mol−1 at 298 K and that the vapour pressure of methanal is
1500 Torr at that temperature. P6A.5 Express the equilibrium constant of a gas-phase reaction A + 3 B ⇌
2 C in terms of the equilibrium value of the extent of reaction, ξ, given
P6A.3 A sealed container was filled with 0.300 mol H2(g), 0.400 mol I2(g), and that initially A and B were present in stoichiometric proportions. Find an
0.200 mol HI(g) at 870 K and total pressure 1.00 bar. Calculate the amounts expression for ξ as a function of the total pressure, p, of the reaction mixture
of the components in the mixture at equilibrium given that K = 870 for the and sketch a graph of the expression obtained.
reaction H2(g) + I2(g) ⇌ 2 HI(g).
P6A.6 Consider the equilibrium N2O4(g) ⇌ 2 NO2(g). From the tables of data

P6A.4 Nitric acid hydrates have received much attention as possible catalysts in the Resource section, assess the contributions of ΔrH⦵ and ΔrS⦵ to the value
for heterogeneous reactions that bring about the Antarctic ozone hole. of K at 298 K.

TOPIC 6B The response of equilibria to the conditions


Discussion questions
D6B.1 Suggest how the thermodynamic equilibrium constant may respond D6B.3 Explain the molecular basis of the van ’t Hoff equation for the
differently to changes in pressure and temperature from the equilibrium temperature dependence of K.
constant expressed in terms of partial pressures.
D6B.2 Account for Le Chatelier’s principle in terms of thermodynamic
quantities. Could there be exceptions to Le Chatelier’s principle?

Additional exercises
E6B.5 Molecular bromine is 24 per cent dissociated at 1600 K and 1.00 bar in with A = −1.04, B = −1088 K, and C = 1.51 × 105 K2. Calculate the standard
the equilibrium Br2(g) ⇌ 2 Br(g). Calculate K at (i) 1600 K, (ii) 2000 K given reaction enthalpy and standard reaction entropy at 400 K.
that ΔrH⦵ = +112 kJ mol−1 over the temperature range.
E6B.10 The equilibrium constant of a reaction is found to fit the expression ln
E6B.6 From information in the Resource section, calculate the standard K = A + B/T + C/T3 between 400 K and 500 K with A = −2.04, B = −1176 K,
Gibbs energy and the equilibrium constant at (i) 298 K and (ii) 400 K for the and C = 2.1 × 107 K3. Calculate the standard reaction enthalpy and standard
reaction PbO(s,red) + CO(g) ⇌ Pb(s) + CO2(g). Assume that the standard reaction entropy at 450 K.
reaction enthalpy is independent of temperature.
E6B.11 Calculate the percentage change in Kx (the equilibrium constant expressed
E6B.7 From information in the Resource section, calculate the standard Gibbs in terms of mole fractions) for the reaction H2CO(g) ⇌ CO(g) + H2(g) when the
energy and the equilibrium constant at (i) 25 °C and (ii) 50 °C for the reaction total pressure is increased from 1.0 bar to 2.0 bar at constant temperature.
CH4(g) + 3 Cl2(g) ⇌ CHCl3(l) + 3 HCl(g). Assume that the standard reaction
E6B.12 Calculate the percentage change in Kx (the equilibrium constant
enthalpy is independent of temperature. At 298.15 K ΔfG⦵ (CHCl3(l)) =
expressed in terms of mole fractions) for the reaction CH3OH(g) +
−73.7 kJ mol−1 and ΔfH⦵ (CHCl3(l)) = −134.1 kJ mol−1.
NOCl(g) ⇌ HCl(g) + CH3NO2(g) when the total pressure is increased
E6B.8 The standard enthalpy of a certain reaction is approximately constant at from 1.0 bar to 2.0 bar at constant temperature.
+125 kJ mol−1 from 800 K up to 1500 K. The standard reaction Gibbs energy
E6B.13 The equilibrium constant for the gas-phase isomerization of borneol
is +22 kJ mol−1 at 1120 K. Estimate the temperature at which the equilibrium
(C10H17OH) to its isomer isoborneol at 503 K is 0.106. A mixture consisting
constant becomes greater than 1.
of 7.50 g of borneol and 14.0 g of isoborneol in a container of volume 5.0 dm3
E6B.9 The equilibrium constant of the reaction 2 C3H6(g) ⇌ C2H4(g) + C4H8(g) is heated to 503 K and allowed to come to equilibrium. Calculate the mole
is found to fit the expression ln K = A + B/T + C/T 2 between 300 K and 600 K, fractions of the two substances at equilibrium.
E6B.14 The equilibrium constant for the reaction N2(g) + O2(g) ⇌ 2 NO(g)
is 1.69 × 10−3 at 2300 K. A mixture consisting of 5.0 g of nitrogen and 2.0 g

These problems were supplied by Charles Trapp and Carmen Giunta. of oxygen in a container of volume 1.0 dm3 is heated to 2300 K and
Exercises and problems 233

allowed to come to equilibrium. Calculate the mole fraction of NO at reaction of A2B(s), calculate (i) the equilibrium constant, (ii) the standard
equilibrium. reaction Gibbs energy, (iii) the standard enthalpy, and (iv) the standard
entropy of dissociation, all at 422 °C. Assume that the vapour behaves as a
E6B.15 What is the standard enthalpy of a reaction for which the equilibrium
perfect gas and that ΔrH⦵ and ΔrS⦵ are independent of temperature in the
constant is (i) doubled, (ii) halved when the temperature is increased by 15 K
range given.
at 310 K?
E6B.19 Solid ammonium chloride dissociates according to NH4Cl(s) →
E6B.16 Estimate the temperature at which the equilibrium constant for the
NH3(g) + HCl(g). The dissociation vapour pressure of NH4Cl at 427 °C is
decomposition of CaCO3(s, calcite) to CO2(g) and CaO(s) becomes 1;
608 kPa but at 459 °C it has risen to 1115 kPa. Calculate (i) the equilibrium
assume pCO2 = 1 bar.
constant, (ii) the standard reaction Gibbs energy, (iii) the standard enthalpy,
E6B.17 Estimate the temperature at which the equilibrium constant for (iv) the standard entropy of dissociation, all at 427 °C. Assume that the
CuSO4·5 H2O(s) → CuSO4(s) + 5 H2O(g) becomes 1; assume pH2O = 1 bar . vapour behaves as a perfect gas and that ΔrH⦵ and ΔrS⦵ are independent of
temperature in the range given.
E6B.18 The dissociation vapour pressure of a salt A2B(s) ⇌ A2(g) + B(g) at
367 °C is 208 kPa but at 477 °C it has risen to 547 kPa. For the dissociation

Problems
P6B.1 The equilibrium constant for the reaction N2(g) + 3 H2(g) ⇌ 2 NH3(g) found that 0.0380 g of the acid was present. Calculate the equilibrium constant
is 2.13 × 106 at 288 K and 1.75 × 105 at 308 K. Calculate the standard reaction for the dimerization of the acid in the vapour, and the standard enthalpy of
enthalpy, assuming it to be constant over this temperature range. the dimerization reaction.

P6B.2 Consider the dissociation of methane, CH4(g), into the elements H2(g) P6B.8 The dissociation of I2(g) can be monitored by measuring the total
and C(s, graphite). (a) Given that ΔfH⦵(CH4,g) = −74.85 kJ mol−1 and that pressure, and three sets of results are as follows:
ΔfS⦵ = −80.67 J K−1 mol−1 at 298 K, calculate the value of the equilibrium
constant at 298 K. (b) Assuming that ΔrH⦵ is independent of temperature, T/K 973 1073 1173
calculate K at 50 °C. (c) Calculate the degree of dissociation, α, of methane
100p/atm 6.244 6.500 9.181
at 298 K and a total pressure of 0.010 bar. (d) Without doing any numerical
calculations, explain how the degree of dissociation for this reaction will 104 nI2 2.4709 2.4555 2.4366
change as the pressure and temperature are varied.
where nI is the amount of I2 molecules introduced into a container of volume
P6B.3 The equilibrium pressure of H2 over U(s) and UH3(s) between 450 K 2
342.68 cm3. Calculate the equilibrium constants of the dissociation and the
and 715 K fits the expression ln(p/Pa) = A + B/T + C ln(T/K), with A = 69.32, standard enthalpy of dissociation assuming it to be constant over the range of
B = −1.464 × 104 K, and C = −5.65. Find an expression for the

standard temperatures.
enthalpy of formation of UH3(s) and from it calculate ∆ f C p .
‡ ⦵ −1
P6B.9 The 1980s saw reports of ΔfH (SiH2) ranging from 243 to 289 kJ mol .
P6B.4 Use the following data on the reaction H2(g) + Cl2(g) ⇌ 2 HCl(g) to If the standard enthalpy of formation is uncertain by this amount, by what
determine the standard reaction enthalpy: factor is the equilibrium constant for the formation of SiH2 from its elements
uncertain at (a) 298 K, (b) 700 K?
T/K 300 500 1000
P6B.10 Fuel cells show promise as power sources for automobiles.
K 4.0 × 1031 4.0 × 1018 5.1 × 108
Hydrogen and carbon monoxide have been investigated for use in fuel
cells, so their solubilities, s, in molten salts are of interest. Their solubilities
P6B.5 The degree of dissociation, α, of CO2(g) into CO(g) and O2(g) at high
in a molten NaNO3/KNO3 mixture were found to fit the following
temperatures and 1 bar total pressure was found to vary with temperature as
expressions:
follows:

T/K 1395 1443 1498 768


log(sH2 /mol cm 3 bar 1 )  5.39 
−4 T /K
α/10 1.44 2.50 4.71
980
log(sco /mol cm 3 bar 1 )  5.98 

T /K
Assume ΔrH to be constant over this temperature range, and calculate K,
ΔrG⦵, ΔrH⦵, and ΔrS⦵ at 1443 K. Make any justifiable approximations. Calculate the standard molar enthalpies of solution of the two gases
P6B.6 The standard reaction enthalpy for the decomposition of CaCl2 · NH3(s) at 570 K.
into CaCl2(s) and NH3(g) is nearly constant at +78 kJ mol−1 between 350 K P6B.11 Find an expression for the standard reaction Gibbs energy at a
and 470 K. The equilibrium pressure of NH3 in the presence of CaCl2 · NH3 is temperature T′ in terms of its value at another temperature T and the
1.71 kPa at 400 K. Find an expression for the temperature dependence of ΔrG⦵ coefficients a, b, and c in the expression for the molar heat capacity listed
in the same range. in Table 2B.1 (Cp,m = a + bT + c/T2). Evaluate the standard Gibbs energy of
P6B.7 Ethanoic acid (acetic acid) was evaporated in container of volume formation of H2O(l) at 372 K from its value at 298 K.
21.45 cm3 at 437 K and at an external pressure of 101.9 kPa, and the container
P6B.12 Derive an expression for the temperature dependence of Kc for a
was then sealed. The mass of acid present in the sealed container was 0.0519 g.
general gas-phase reaction.
The experiment was repeated with the same container but at 471 K, and it was
234 6 Chemical equilibrium

TOPIC 6C Electrochemical cells


Discussion questions
D6C.1 Explain why reactions that are not redox reactions may be used to D6C.4 Why is it necessary to measure the cell potential under zero-current
generate an electric current. conditions?
D6C.2 Distinguish between galvanic and electrolytic cells. D6C.5 Identify contributions to the cell potential when a current is being
drawn from the cell.
D6C.3 Explain the role of a salt bridge.

Additional exercises
You will need to draw on information from Topic 6D to complete the answers. (ii) H2(g) + I2(g) → 2 HI(aq)
E6C.7 Write the cell reaction and electrode half-reactions and calculate the (iii) H3O+(aq) + OH−(aq) → 2 H2O(l)
standard potential of each of the following cells: E6C.10 Use the Debye–Hückel limiting law and the Nernst
(i) Zn(s)|ZnSO4(aq)||AgNO3(aq)|Ag(s) equation to estimate the potential of the cell Ag(s)|AgBr(s)|KBr(aq,
0.050 mol kg−1)||Cd(NO3)2(aq, 0.010 mol kg−1)|Cd(s) at 25 °C.
(ii) Cd(s)|CdCl2(aq)||HNO3(aq)|H2(g)|Pt(s)

(iii) Pt(s)|K3[Fe(CN)6](aq),K4[Fe(CN)6](aq)||CrCl3(aq)|Cr(s) E6C.11 Consider the cell Pt(s)|H2(g,p )|HCl(aq)|AgCl(s)|Ag(s), for which
the cell reaction is 2 AgCl(s) + H2(g) → 2 Ag(s) + 2 HCl(aq). At 25 °C and
E6C.8 Write the cell reaction and electrode half-reactions and calculate the a molality of HCl of 0.010 mol kg−1, Ecell = +0.4658 V. (i) Write the Nernst
standard potential of each the following cells: equation for the cell reaction. (ii) Calculate ΔrG for the cell reaction.
(i) Pt(s)|Cl2(g)|HCl(aq)||K2CrO4(aq)|Ag2CrO4(s)|Ag(s) (iii) Assuming that the Debye–Hückel limiting law holds at this concentration,
calculate E⦵(AgCl/Ag,Cl−).
(ii) Pt(s)|Fe3+(aq),Fe2+(aq)||Sn4+(aq),Sn2+(aq)|Pt(s)
 
E6C.12 The cell reaction for the ‘Bunsen cell’ is Zn(s)  2NO3 (aq )  4 H (aq ) 
(iii) Cu(s)|Cu2+(aq)||Mn2+(aq),H+(aq)|MnO2(s)|Pt(s) 2+
Zn (aq ) + 2 H2O(l) + 2 NO2 (g ). The standard cell potential at 25 °C is −0.040 V.
E6C.9 Devise cells in which the following are the reactions and calculate the Calculate the electrical work that can be done by the cell.
standard cell potential in each case:
E6C.13 By how much does the cell potential change when Q is increased by a
(i) 2 Na(s) + 2 H2O(l) → 2 NaOH(aq) + H2(g) factor of 5 for a reaction in which n = 3 at 298 K?

Problems
You will need to draw on information from Topic 6D to complete the answers. P6C.4 State what is expected to happen to the cell potential when the specified
changes are made to the following cells. Confirm your prediction by using the
P6C.1 A fuel cell develops an electric potential difference from the chemical
Nernst equation in each case.
reaction between reagents supplied from an outside source. What is the
standard potential of a cell fuelled by (a) hydrogen and oxygen, (b) the (a) The molar concentration of silver nitrate in the left-hand compartment
combustion of butane at 1.0 bar and 298 K? is increased in the cell Ag(s)|AgNO3(aq,mL)||AgNO3(aq,mR)|Ag(s).

P6C.2 Calculate the value of ΔfG (H2O,l) at 298 K from the standard potential (b) The pressure of hydrogen in the left-hand compartment is increased in
⦵
of the cell Pt(s)|H2(g)|HCl(aq)|O2(g)|Pt(s), Ecell  1.23 V. the cell Pt(s)|H2(g,pL)|HCl(aq)|H2(g,pL)|Pt(s).
(c) The pH of the right-hand compartment is decreased in the cell
P6C.3 Although the hydrogen electrode may be conceptually the
Pt(s)|K3[Fe(CN)6](aq),K4[Fe(CN)6](aq)||Mn2+(aq),H+(aq)|MnO2(s)|Pt(s).
simplest electrode and is the basis for the choice of reference potential
in electrochemical systems, it is cumbersome to use. Therefore, several (d) The concentration of HCl is increased in the cell Pt(s)|Cl2(g)|HCl(aq)||
substitutes for it have been devised. One of these alternatives is the HBr(aq)|Br2(l)|Pt(s).
quinhydrone electrode (quinhydrone, Q·QH2, is a complex of quinone, (e) Some iron(III) chloride is added to both compartments of the cell
C6H4O2 = Q, and hydroquinone, C6H4O2H2 = QH2), where the concentrations Pt(s)|Fe3+(aq),Fe2+(aq)||Sn4+(aq),Sn2+(aq)|Pt(s)
of Q·QH2 and QH2 are equal to each other. The electrode half-reaction
(f) Acid is added to both compartments of the cell Fe(s)|Fe2+(aq)||
is Q(aq) + 2 H+(aq) + 2 e− → QH2(aq), E⦵ = +0.6994 V. If the cell
Mn2+(aq), H+(aq)|MnO2(s)|Pt(s).
Hg(s)|Hg2Cl2(s)|HCl(aq)|Q · QH2|Au(s) is prepared, and the measured cell
potential is +0.190 V, what is the pH of the HCl solution?

TOPIC 6D Electrode potentials


Discussion questions
D6D.1 Describe a method for the determination of the standard potential of a D6D.2 Suggest reasons why a glass electrode can be used for the determination
redox couple. of the pH of an aqueous solution.
Exercises and problems 235

Additional exercises
E6D.4 Calculate the equilibrium constants of the following reactions at 25 °C E6D.7 Calculate the standard potential of the cell Pt(s)|cystine(aq),
from standard potential data: cysteine(aq)||H+(aq)|O2(g)|Pt(s) and the standard Gibbs energy of the
cell reaction at 25 °C. Use E⦵ = −0.34 V for cystine(aq) + 2 H+(aq) +
(i) Sn(s) + CuSO4(aq) ⇌ Cu(s) + SnSO4(aq)
2 e− → 2 cysteine(aq).
(ii) Cu2+(aq) + Cu(s) ⇌ 2 Cu+(aq)
E6D.8 Can chlorine gas oxidize water to oxygen gas under standard conditions
E6D.5 The standard potential of the cell Ag(s)|AgI(s)|AgI(aq)|Ag(s) is +0.9509 V in basic solution?
at 25 °C. Calculate the equilibrium constant for the dissolution of AgI(s).
E6D.6 The standard potential of the cell Bi(s)|Bi2S3(s)|Bi2S3(aq)|Bi(s) is +0.96 V
at 25 °C. Calculate the equilibrium constant for the dissolution of Bi2S3(s).

Problems

P6D.1 Tabulated thermodynamic data can be used to predict the standard P6D.4 The potential of the cell Pt(s)|H2(g,p )|HCl(aq,b)|Hg2Cl2(s)|Hg(l) has
potential of a cell even if it cannot be measured directly. The standard Gibbs been measured with high precision with the following results at 25 °C:
energy of the reaction K2CrO4(aq) + 2 Ag(s) + 2 FeCl3(aq) → Ag2CrO4(s) +
2 FeCl2(aq) + 2 KCl(aq) is −62.5 kJ mol−1 at 298 K. (a) Calculate the standard b/(mmol kg−1) 1.6077 3.0769 5.0403 7.6938 10.9474
potential of the corresponding galvanic cell and (b) the standard potential of
the Ag 2CrO4 /Ag ,CrO24− couple. E/V 0.600 80 0.568 25 0.543 66 0.522 67 0.505 32

P6D.2 A fuel cell is constructed in which both electrodes make use of the Determine the standard cell potential and the mean activity coefficient of HCl
oxidation of methane. The left-hand electrode makes use of the complete at these molalities. (Make a least-squares fit of the data to the best straight line.)
oxidation of methane to carbon dioxide and liquid water; the right-hand
P6D.5 For a hydrogen/oxygen fuel cell, with an overall four-electron cell
electrode makes use of the partial oxidation of methane to carbon monoxide
reaction 2 H2(g) + O2(g) → 2 H2O(l), the standard cell potential is +1.2335 V
and liquid water. (a) Which electrode is the cathode? (b) What is the cell
at 293 K and +1.2251 V at 303 K. Calculate the standard reaction enthalpy and
potential at 25 °C when all gases are at 1 bar?
entropy within this temperature range.
P6D.3 One ecologically important equilibrium is that between carbonate and −
P6D.6 The standard potential of the AgCl/Ag,Cl couple fits the expression
hydrogencarbonate (bicarbonate) ions in natural water. (a) The standard
− −1
Gibbs energies of formation of CO2− 3 (aq ) and HCO3 (aq ) are −527.81 kJ mol

E  /V  0.23659  4.8564  104 ( /C)  3.4205  106 ( /C)2
and −586.77 kJ mol−1, respectively. What is the standard potential of the
HCO3− /CO23− , H2 couple? (b) Calculate the standard potential of a cell in  5.869  109 ( /C)3
which the cell reaction is Na2CO3(aq) + H2O(l) → NaHCO3(aq) + NaOH(aq). Calculate the standard Gibbs energy and enthalpy of formation of Cl−(aq) and
(c) Write the Nernst equation for the cell, and (d) predict and calculate the its standard entropy at 298 K.
change in cell potential when the pH is changed to 7.0 at 298 K.

FOCUS 6 Chemical equilibrium


Integrated activities

I6.1 Thorn et al. (J. Phys. Chem. 100, 14178 (1996)) carried out a study of mean ionic activity and activity coefficient of ZnCl2 from the measured cell
Cl2O(g) by photoelectron ionization. From their measurements, they report potential, and (e) the mean ionic activity coefficient of ZnCl2 from the Debye–
ΔfH⦵(Cl2O) = +77.2 kJ mol−1. They combined this measurement with literature Hückel limiting law. (f) Given that (∂Ecell/∂T)p = −4.52 × 10−4 V K−1. Calculate
data on the reaction Cl2O (g) + H2O(g) → 2 HOCl(g), for which K = 8.2 × 10−2 ∆rS and ∆rH.
and ΔrS⦵ = +16.38 J K−1 mol−1, and with readily available thermodynamic data ⦵
I6.4 Careful measurements of the potential of the cell Pt|H2(g,p )|NaOH
on water vapour to report a value for ΔfH⦵(HOCl). Calculate that value. All −1 −1
(aq,0.0100 mol kg ), NaCl(aq, 0.011 25 mol kg )|AgCl(s)|Ag(s) have been
quantities refer to 298 K.
reported. Among the data is the following information:
⦵ −1 2+
I6.2 Given that ΔrG = −212.7 kJ mol for the reaction Zn(s) + Cu (aq) →
Zn2+(aq) + Cu(s) in the Daniell cell at 25 °C, and b(CuSO4) = 1.00 × θ/°C 20.0 25.0 30.0
10−3 mol kg−1 and b(ZnSO4) = 3.00 × 10−3 mol kg−1, calculate (a) the ionic
Ecell/V 1.04774 1.04864 1.04942
strengths of the solutions, (b) the mean ionic activity coefficients in the
compartments, (c) the reaction quotient, (d) the standard cell potential, and
Calculate pKw at these temperatures and the standard enthalpy and entropy
(e) the cell potential. (Take γ+ = γ− = γ± in the respective compartments. Use
of the autoprotolysis of water at 25.0 °C. Recall that Kw is the equilibrium
the Debye–Hückel limiting law.)
constant for the autoprotolysis of liquid water.
−1
I6.3 Consider the cell, Zn(s)|ZnCl2(0.0050 mol kg )|Hg2Cl2(s)|Hg(l), for which
I6.5 Measurements of the potential of cells of the type Ag(s)|AgX(s)|MX(b1)|Mx
the cell reaction is Hg2Cl2(s) + Zn(s) → 2 Hg(l) + 2 Cl−(aq) + Zn2+(aq). The
Hg|MX(b2)|AgX(s)|Ag(s), where MxHg denotes an amalgam and the electrolyte
cell potential is +1.2272 V, E⦵(Zn2+,Zn) = −0.7628 V, and E⦵(Hg2Cl2,Hg) =
is LiCl in ethylene glycol, are given below for M = Li and X = Cl. Estimate
+0.2676 V. (a) Write the Nernst equation for the cell. Determine (b) the
⦵ the activity coefficient at the concentration marked ⋆ and then use this value
standard cell potential, (c) ΔrG, ΔrG , and K for the cell reaction, (d) the
236 6 Chemical equilibrium

to calculate activity coefficients from the measured cell potential at the other [glucose] = 5.6 pmol dm−3, [ATP] = [ADP] = [Pi] = 0.10 mmol dm−3, pH = 7.4,
concentrations. Base your answer on the Davies equation (eqn 5F.30b) with T = 310 K. Assuming that activities can be replaced by the numerical values
A = 1.461, B = 1.70, C = 0.20, and I = b/b⦵. For b2 = 0.09141 mol kg−1: of molar concentrations, calculate the efficiency of aerobic respiration under
these physiological conditions. (c) A typical diesel engine operates between
b1/(mol kg−1) 0.0555 0.09141 0.1652 0.2171 1.040 1.350⋆ Tc = 873 K and Th = 1923 K with an efficiency that is approximately 75 per cent
E/V of the theoretical limit of 1 − Tc/Th (see Topic 3A). Compare the efficiency
−0.0220 0.0000 0.0263 0.0379 0.1156 0.1336
of a typical diesel engine with that of aerobic respiration under typical
‡ physiological conditions (see part (b)). Why is biological energy conversion
I6.6 The table below summarizes the potential of the cell Pd(s)|H2(g, 1 bar)
more or less efficient than energy conversion in a diesel engine?
|BH(aq, b), B(aq, b)|AgCl(s)|Ag(s). Each measurement is made at equimolar
concentrations of 2-aminopyridinium chloride (BH) and 2-aminopyridine (B). I6.10 In anaerobic bacteria, the source of carbon may be a molecule other than
The data are for 25 °C and it is found that E⦵ = 0.222 51 V. Use the data to glucose and the final electron acceptor is some molecule other than O2. Could
determine pKa for the acid at 25 °C and the mean activity coefficient (γ±) of BH a bacterium evolve to use the ethanol/nitrate pair instead of the glucose/O2
as a function of molality (b) and ionic strength (I). Use the Davies equation pair as a source of metabolic energy?
(eqn 5F.30b) with A = 0.5091 and B and C are parameters that depend upon
I6.11 The standard potentials of proteins are not commonly measured by the
the ions.
methods described in this chapter because proteins often lose their native
structure and function when they react on the surfaces of electrodes. In an
b/(mol kg−1) 0.01 0.02 0.03 0.04 0.05 alternative method, the oxidized protein is allowed to react with an appropriate
Ecell(25 °C)/V 0.74 452 0.72 853 0.71 928 0.71 314 0.70 809 electron donor in solution. The standard potential of the protein is then
determined from the Nernst equation, the equilibrium concentrations of all
b/(mol kg−1) 0.06 0.07 0.08 0.09 0.10 species in solution, and the known standard potential of the electron donor.
This method can be illustrated with the protein cytochrome c. The one-electron
Ecell(25 °C)/V 0.70 380 0.70 059 0.69 790 0.69 571 0.69 338 reaction between cytochrome c, cyt, and 2,6-dichloroindophenol, D, can be
followed spectrophotometrically because each of the four species in solution
Hint: Use mathematical software or a spreadsheet. has a distinct absorption spectrum. Write the reaction as cytox + Dred ⇌
cytred + Dox, where the subscripts ‘ox’ and ‘red’ refer to oxidized and reduced
I6.7 Read Impact 9 (accessed via the e-book) before attempting this problem. ⦵
 ⦵
states, respectively. (a) Consider Ecyt and ED to be the standard potentials
Here you will investigate the molecular basis for the observation that the
of cytochrome c and D, respectively. Show that, at equilibrium, a plot of
hydrolysis of adenosine triphosphate (ATP) to adenosine diphosphate (ADP)
ln([Dox]eq/[Dred]eq) versus ln([cytox]eq/[cytred]eq) is linear with slope of 1 and
is exergonic at pH = 7.0 and 310 K. (a) It is thought that the exergonicity ⦵
 ⦵
y-intercept F (Ecyt − ED )/RT , where equilibrium activities are replaced by the
of ATP hydrolysis is due in part to the fact that the standard entropies of
numerical values of equilibrium molar concentrations. (b) The following data
hydrolysis of polyphosphates are positive. Why would an increase in entropy
were obtained for the reaction between oxidized cytochrome c and reduced D
accompany the hydrolysis of a triphosphate group into a diphosphate and
in a pH 6.5 buffer at 298 K. The ratios [Dox]eq/[Dred]eq and [cytox]eq/[cytred]eq were
a phosphate group? (b) Under identical conditions, the Gibbs energies of
adjusted by titrating a solution containing oxidized cytochrome c and reduced
hydrolysis of H4ATP and MgATP2−, a complex between the Mg2+ ion and
D with a solution of sodium ascorbate, which is a strong reductant. From
ATP4−, are less negative than the Gibbs energy of hydrolysis of ATP4−. This
the data and the standard potential of D of 0.237 V, determine the standard
observation has been used to support the hypothesis that electrostatic
potential cytochrome c at pH 6.5 and 298 K.
repulsion between adjacent phosphate groups is a factor that controls the
exergonicity of ATP hydrolysis. Provide a rationale for the hypothesis and
discuss how the experimental evidence supports it. Do these electrostatic [Dox]eq/[Dred]eq 0.00279 0.00843 0.0257 0.0497 0.0748 0.238 0.534
effects contribute to the ∆rH or ∆rS terms that determine the exergonicity of [cytox]eq/[cytred]eq 0.0106 0.0230 0.0894 0.197 0.335 0.809 1.39
the reaction? Hint: In the MgATP2− complex, the Mg2+ ion and ATP4− anion
form two bonds: one that involves a negatively charged oxygen belonging to
the terminal phosphate group of ATP4− and another that involves a negatively ‡
I6.12 The dimerization of ClO in the Antarctic winter stratosphere is believed
charged oxygen belonging to the phosphate group adjacent to the terminal to play an important part in that region’s severe seasonal depletion of ozone.
phosphate group of ATP4−. The following equilibrium constants are based on measurements on the
reaction 2 ClO(g) → (ClO)2(g).
I6.8 Read Impact 9 (accessed via the e-book) before attempting this problem.
To get a sense of the effect of cellular conditions on the ability of adenosine
triphosphate (ATP) to drive biochemical processes, compare the standard T/K 233 248 258 268 273 280
Gibbs energy of hydrolysis of ATP to ADP (adenosine diphosphate) with the 8 7 7 6 6
K 4.13 × 10 5.00 × 10 1.45 × 10 5.37 × 10 3.20 × 10 9.62 × 105
reaction Gibbs energy in an environment at 37 °C in which pH = 7.0 and the
ATP, ADP, and Pi− concentrations are all 1.0 mmol dm−3. T/K 288 295 303
5 5
I6.9 Read Impact 9 (accessed via the e-book) before attempting this problem. K 4.28 × 10 1.67 × 10 6.02 × 104
Under biochemical standard conditions, aerobic respiration produces
approximately 38 molecules of ATP per molecule of glucose that is completely (a) Derive the values of ΔrH⦵ and ΔrS⦵ for this reaction. (b) Compute the
oxidized. (a) What is the percentage efficiency of aerobic respiration under standard enthalpy of formation and the standard molar entropy of (ClO)2
biochemical standard conditions? (b) The following conditions are more ⦵
given ΔfH⦵(ClO,g) = +101.8 kJ mol−1 and Sm (ClO,g )  226.6 J K 1 mol 1.
likely to be observed in a living cell: pCO2  5.3  102 atm, pO2 = 0.132 atm,
FOCUS 7
Quantum theory

It was once thought that the motion of atoms and subatomic ­ perators are constructed and used. One consequence of their
o
particles could be expressed using ‘classical mechanics’, the use is the ‘uncertainty principle’, one of the most profound de-
laws of motion introduced in the seventeenth century by Isaac partures of quantum mechanics from classical mechanics.
Newton, for these laws were very successful at explaining the 7C.1 Operators; 7C.2 Superpositions and expectation values;
motion of everyday objects and planets. However, a proper de- 7C.3 The uncertainty principle; 7C.4 The postulates of quantum
mechanics
scription of electrons, atoms, and molecules requires a different
kind of mechanics, ‘quantum mechanics’, which is introduced
in this Focus and applied widely throughout the text.
7D Translational motion
Translational motion, motion through space, is one of the
7A The origins of quantum mechanics fundamental types of motion treated by quantum mechan-
ics. According to quantum theory, a particle constrained to
Experimental evidence accumulated towards the end of the move in a finite region of space is described by only certain
nineteenth century showed that classical mechanics failed wavefunctions and can possess only certain energies. That is,
when it was applied to particles as small as electrons. More quantization emerges as a natural consequence of solving the
specifically, careful measurements led to the conclusion that Schrödinger equation and the conditions imposed on it. The
particles may not have an arbitrary energy and that the classi- solutions also expose a number of non-classical features of par-
cal concepts of a particle and wave blend together. This Topic ticles, especially their ability to tunnel into and through regions
shows how these observations set the stage for the development where classical physics would forbid them to be found.
of the concepts and equations of quantum mechanics in the 7D.1 Free motion in one dimension; 7D.2 Confined motion in one
early twentieth century. dimension; 7D.3 Confined motion in two and more dimensions;
7A.1 Energy quantization; 7A.2 Wave–particle duality 7D.4 Tunnelling

7B Wavefunctions 7E Vibrational motion


This Topic introduces the ‘harmonic oscillator’, a simple but
In quantum mechanics, all the properties of a system are ex-
very important model for the description of vibrations. It
pressed in terms of a wavefunction, which is obtained by solv-
shows that the energies of an oscillator are quantized and that
ing the equation proposed by Erwin Schrödinger. This Topic
an oscillator may be found at displacements that are forbidden
focuses on the interpretation of the wavefunction, and specifi-
by classical physics.
cally what it reveals about the location of a particle.
7E.1 The harmonic oscillator; 7E.2 Properties of the harmonic oscillator
7B.1 The Schrödinger equation; 7B.2 The Born interpretation

7F Rotational motion
7C Operators and observables
The constraints on the wavefunctions of a body rotating in two
A central feature of quantum theory is its representation of and three dimensions result in the quantization of its energy.
observables by ‘operators’, which act on the wavefunction and In addition, because the energy is related to the angular mo-
extract the information it contains. This Topic shows how mentum, it follows that angular momentum is also restricted
to ­certain values. The quantization of angular momentum is a useful technology. It is based on the expectation that a ‘quan-
very important aspect of the quantum theory of electrons in tum computer’ can carry out calculations on many states of
atoms and of rotating molecules. a system simultaneously, leading to a new generation of very
7F.1 Rotation in two dimensions; 7F.2 Rotation in three dimensions fast computers. ‘Nanoscience’ is the study of atomic and mo-
lecular assemblies with dimensions ranging from 1 nm to
about 100 nm, and ‘nanotechnology’ is concerned with the
What is an application of this material? incorporation of such assemblies into devices. Impact 12, ac-
cessed via the e-book, explores quantum mechanical effects
Impact 11, accessed via the e-book, highlights an application that show how the properties of a nanometre-sized assembly
of quantum mechanics that is beginning to gain ground as a depend on its size.

➤ Go to the e-book for videos that feature the derivation and interpretation of equations, and applications of this material.
TOPIC 7A The origins of quantum
mechanics

The classical mechanics developed by Newton in the seven-


➤ Why do you need to know this material? teenth century is an extraordinarily successful theory for de-
Quantum theory is central to almost every explanation in
scribing the motion of everyday objects and planets. However,
chemistry. It is used to understand atomic and molecular
late in the nineteenth century scientists started to make ob-
structure, chemical bonds, and most of the properties of
servations that could not be explained by classical mechanics.
matter.
They were forced to revise their entire conception of the nature
of matter and replace classical mechanics by a theory that be-
➤ What is the key idea? came known as quantum mechanics.
Experimental evidence led to the conclusion that energy
can be transferred only in discrete amounts, and that the
classical concepts of a ‘particle’ and a ‘wave’ blend together. 7A.1 Energy quantization
➤ What do you need to know already? Three experiments carried out near the end of the nineteenth
You should be familiar with the basic principles of classical century drove scientists to the view that energy can be trans-
mechanics, especially momentum, force, and energy set ferred only in discrete amounts.
out in Energy: A first look. The discussion of heat capacities
of solids makes light use of material in Topic 2A. 7A.1(a) Black-body radiation
The key features of electromagnetic radiation according to clas-
sical physics are described in The chemist’s toolkit 7A.1. It is

The chemist’s toolkit 7A.1 Electromagnetic radiation


Electromagnetic radiation consists of oscillating electric and A wave is characterized by its wavelength, λ (lambda), the
magnetic disturbances that propagate as waves. The two com- distance between consecutive peaks of the wave. The classifica-
ponents of an electromagnetic wave are mutually perpendicular tion of electromagnetic radiation according to its wavelength is
and are also perpendicular to the direction of propagation shown in Sketch 2. Light, which is electromagnetic radiation that
(Sketch 1). Electromagnetic waves travel through a vacuum at a is visible to the human eye, has a wavelength in the range 420 nm
constant speed called the speed of light, c, which has the defined (violet light) to 700 nm (red light). The properties of a wave may
value of exactly 2.997 924 58 × 108 m s−1. also be expressed in terms of its frequency, n (nu), the number
of oscillations in a time interval divided by the duration of the
interval. Frequency is reported in hertz, Hz, with 1 Hz = 1 s−1
Propagation (that is, 1 cycle per second). Light spans the frequency range
direction, at speed c
from 710 THz (violet light) to 430 THz (red light).
The wavelength and frequency of an electromagnetic wave are
related by
Magnetic
The relation between wavelength
field c   and frequency in a vacuum

Wavelength λ It is also common to describe a wave in terms of its wavenum-


Electric 
ber,  (nu tilde), which is defined as
field
 1   Wavenumber
Sketch 1
 , or equivalently   [definition]
 c
240 7 Quantum theory

Wavelength, λ/m

10–12

10–13

10–14
10–10

10–11
10–1

10–2

1 mm 10–3

10–4

10–5

10–6

10–7

10–8

10–9
1

1 nm

1 pm
1 μm
1 dm

1 cm
1m

Ultraviolet
Microwave

ultraviolet
Vacuum
infrared

Cosmic
infrared
Visible

X-ray
Radio

γ-ray
Near
Far

ray
14 000 cm–1

24 000 cm–1
430THz
700 nm

420 nm

710THz
Sketch 2

Thus, wavenumber is the reciprocal of the wavelength and can Electromagnetic radiation that consists of a single fre-
be interpreted as the number of wavelengths in a given distance. quency (and therefore single wavelength) is monochromatic,
In spectroscopy, for historical reasons, wavenumber is usually because it corresponds to a single colour. White light consists
reported in units of reciprocal centimetres (cm−1). Visible light of electromagnetic waves with a continuous, but not uni-
therefore corresponds to electromagnetic radiation with a wave- form, spread of frequencies throughout the visible region of
number of 14 000 cm−1 (red light) to 24 000 cm−1 (violet light). the spectrum.

observed that all objects emit electromagnetic radiation over a r­ adiation from a black body varies with wavelength at several
range of frequencies with an intensity that depends on the tem- temperatures. At each temperature T there is a wavelength, λmax,
perature of the object. A familiar example is a heated metal bar at which the intensity of the radiation is a maximum, with T
that first glows red and then becomes ‘white hot’ upon further and λmax related by the empirical Wien’s law:
heating. As the temperature is raised, the colour shifts from red
towards blue and results in the white glow. maxT  constant Wien’s law (7A.1)
The radiation emitted by hot objects is discussed in terms of
a black body, a body that emits and absorbs electromagnetic The constant is found to have the value 2.9 mm K. The intensity
radiation without favouring any wavelengths. A good ap- of the emitted radiation at any temperature declines sharply
proximation to a black body is a small hole in an empty con- at short wavelengths (high frequencies). The intensity is effec-
tainer (Fig. 7A.1). Figure 7A.2 shows how the intensity of the tively a window on to the energy present inside the container,

Maximum
Energy spectral density, ρ(λ,T )

Container of ρ
at a
temperature T Detected Increasing
radiation temperature

Pinhole

Wavelength, λ
Figure 7A.1 Black-body radiation can be detected by allowing
it to leave an otherwise closed container through a pinhole. Figure 7A.2 The energy spectral density of radiation from a
The radiation is reflected many times within the container and black body at several temperatures. Note that as the temperature
comes to thermal equilibrium with the wall. Radiation leaking out increases, the maximum in the energy spectral density moves
through the pinhole is characteristic of the radiation inside the to shorter wavelengths and the total energy (the area under the
container. curve) increases.
7A The origins of quantum mechanics 241

in the sense that the greater the intensity at a given wavelength,

Energy spectral density, ρ(λ,T)


the greater is the energy inside the container due to radiation at
that wavelength. Rayleigh–Jeans
The energy density, E(T ), is the total energy inside the con- law
tainer divided by its volume. The energy spectral density,
ρ(λ,T), is defined so that ρ(λ,T)dλ is the energy density at tem-
perature T due to the presence of electromagnetic radiation Experimental
with wavelengths between λ and λ + dλ. A high energy spectral
density at the wavelength λ and temperature T simply means
that there is a lot of energy associated with wavelengths lying
between λ and λ + dλ at that temperature. The energy density is Wavelength, λ
obtained by summing (integrating) the energy spectral density
over all wavelengths: Figure 7A.3 Comparison of the experimental energy spectral
density with the prediction of the Rayleigh–Jeans law (eqn 7A.4).
 The latter predicts an infinite energy spectral density at short
E(T )    ( ,T )d  (7A.2)
0 wavelengths and infinite overall energy density.
The units of E(T ) are joules per metre cubed (J m−3), so the units
of ρ(λ,T) are J m−4. Empirically, the energy density is found to vary E  nh n  0, 1, 2,  (7A.5)
as T 4, an observation expressed by the Stefan–Boltzmann law:
In this expression h is a fundamental constant now known as
=
E (T ) constant × T 4
Stefan–Boltzmann law  (7A.3) Planck’s constant. The limitation of energies to discrete values
is called energy quantization. On this basis Planck was able to
with the constant found to have the value 7.567 × 10−16 J m−3 K−4. derive an expression for the energy spectral density which is
The container in Fig. 7A.1 emits radiation that can be now called the Planck distribution:
thought of as oscillations of the electromagnetic field stimu-
8hc
lated by the oscillations of electrical charges in the material of  ( ,T )  Planck distribution (7A.6a)
the wall. According to classical physics, every oscillator is ex-  5 (ehc / kT  1)
cited to some extent, and according to the equipartition prin- This expression is plotted in Fig. 7A.4 and fits the experimental
ciple (see Energy: A first look) every oscillator, regardless of its data very well at all wavelengths. The value of h, which is an
frequency, has an average energy of kT. On this basis, the physi- undetermined parameter in the theory, can be found by vary-
cist Lord Rayleigh, with minor help from James Jeans, deduced ing its value until the best fit is obtained between the eqn 7A.6a
what is now known as the Rayleigh–Jeans law: and experimental measurements. The currently accepted value
8πkT is h = 6.626 × 10−34 J s.
ρ (λ ,T ) = Rayleigh– Jeans law  (7A.4) For short wavelengths, hc/λkT >> 1. Because ehc/λkT → ∞ faster
λ4
than λ5 → 0, it follows that ρ → 0 as λ → 0. Hence, the ­energy
where k is Boltzmann’s constant (k = 1.381 × 10−23 J K−1).
The Rayleigh–Jeans law is not supported by the experimen- 1
tal measurements. As is shown in Fig. 7A.3, although there is
agreement at long wavelengths, it predicts that the energy spec- 0.8
tral density (and hence the intensity of the radiation emitted)
ρ/{8π(kT)5/(hc)4}

increases without going through a maximum as the wavelength 0.6


decreases. That is, the Rayleigh–Jeans law is inconsistent with
Wien’s law. Equation 7A.4 also implies that the radiation is in- 0.4
tense at very short wavelengths and becomes infinitely intense
as the wavelength tends to zero. The concentration of radiation
0.2
at short wavelengths is called the ultraviolet catastrophe, and
is an unavoidable consequence of classical physics.
0
In 1900, Max Planck found that the experimentally observed 0 0.5 1 1.5 2
λkT/hc
intensity distribution of black-body radiation could be ex-
plained by proposing that the energy of each oscillator is lim- Figure 7A.4 The Planck distribution (eqn 7A.6a) accounts for the
ited to discrete values. In particular, Planck assumed that for an experimentally determined energy distribution of black-body
electromagnetic oscillator of frequency n, the permitted ener- radiation. It coincides with the Rayleigh–Jeans distribution at long
gies are integer multiples of hn: wavelengths.
242 7 Quantum theory

spectral density approaches zero at short wavelengths, and so It is sometimes convenient to express the Planck distribution
the Planck distribution avoids the ultraviolet catastrophe. For in terms of the frequency. Then ρ(n,T)dn is the energy density
long wavelengths in the sense hc/λkT << 1, the denominator at temperature T due to the presence of electromagnetic radia-
in the Planck distribution can be replaced by (see Part 1 of the tion with frequencies between n and n + dn, and
Resource section)
8h 3 Planck distribution in
 ( , T )   (7A.6b)
hc / kT  hc  hc c (eh /kT  1)
3 terms of freequency
e  1  1     1 
  kT   kT
When this approximation is substituted into eqn 7A.6a, the 7A.1(b) Heat capacity
Planck distribution reduces to the Rayleigh–Jeans law, eqn 7A.4. When energy is supplied as heat to a substance its temperature
The wavelength at the maximum can be found by differen- rises; the heat capacity (Topic 2A) is the constant of propor-
tiation, and is given by λmaxT = constant, in accord with Wien’s tionality between the energy supplied and the temperature
law. The value of the constant found in this way is equal to rise (C = dq/dT and, at constant volume, CV,m = (∂Um/∂T)V).
c2/5 (c2 = hc/k = 1.439 cm K is the second radiation constant) and Experimental measurements made during the nineteenth cen-
agrees with the experimental value. Wien’s law is then written as tury had shown that at room temperature the molar heat ca-
maxT  c2 /5 (7A.7) pacities of many monatomic solids are about 3R, where R is
the gas constant.1 However, when measurements were made at
Finally, the total energy density is much lower temperatures it was found that the heat capacity
 8hc 8 5 k 4 decreased, tending to zero as the temperature approached zero.
E(T )   d   aT 4
, with a   (7A.8) Classical physics was unable to explain this temperature depend-
0  5 (ehc / kT  1) 15h3 c 3
ence. The classical picture of a solid is of atoms oscillating about
which is finite and agrees with the Stefan–Boltzmann law fixed positions, with the expectation that each oscillating atom will
(eqn 7A.3), also predicting the value of its constant (a) correctly. have the same average energy kT. This model predicts that a solid
consisting of N atoms, each free to oscillate in three dimensions, will
Brief illustration 7A.1 have energy U = 3NkT and hence heat capacity CV = (∂U/∂T)V =
3Nk. The molar heat capacity is therefore predicted to be 3NAk
Consider eqn 7A.6a with λ1 = 450 nm (blue light) and which, recognizing that NAk = R, is equal to 3R at all temperatures.
λ2 = 700 nm (red light), and T = 298 K. It follows that In 1905, Einstein suggested applying Planck’s hypothesis and
supposing that each oscillating atom could have an energy nhn,
hc (6.626  1034 Js)  (2.998  108 ms 1 ) where n is an integer and n is the frequency of the oscillation.
  107.2 
1kT (450  109 m)  (1.381  1023 JK 1 )  (298 K ) Einstein went on to show by using the Boltzmann distribution
hc (6.626  1034 Js)  (2.998  108 m s 1 ) that each oscillator is unlikely to be excited to high energies and
  68.9  at low temperatures few oscillators can be excited at all. As a
2kT (700  109 m)  (1.381  1023 JK 1 )  (298 K)
consequence, because the oscillators cannot be excited, the heat
and capacity falls to zero. The quantitative result that Einstein ob-
5 tained (as shown in Topic 13E) is
 (450 nm, 298 K )  700  109 m  e68.9  1
   2
    eE / 2T 
2
 (700 nm, 298 K)  450  109 m  e107.2  1 CV, m (T )  3Rf E (T ), f E (T )   E    /T 
 9.11  (2.30  1017 )  2.10  1016  T   e E 1 
Einsteiin formula  (7A.9a)
At room temperature, the proportion of shorter wavelength
radiation is insignificant. In this expression θE is the Einstein temperature, θE = hn/k.
At high temperatures (in the sense T >> θE) the exponen-
Planck’s approach proved successful on account of a key tials in fE can be expanded as e x  1  x   and higher terms
assumption that Planck made about the energy of the oscilla- ignored. The result is
tors. Rayleigh allowed each oscillator to have the same aver- 2 2 2 2
age energy regardless of its frequency. Planck assumed that the     1   E /2T      E   1 
f E (T )   E        1
energy has to be a multiple of hn. As a result, the minimum  T   (1   E /T  )  1   T   E /T 
energy needed to excite oscillators of very high frequency is
 (7A.9b)
too large and they remain unexcited. This elimination of the
­contribution from very high frequency oscillators avoids the 1
The gas constant occurs in the context of solids because it is actually the
ultraviolet catastrophe. more fundamental Boltzmann’s constant in disguise: R = NAk.
7A The origins of quantum mechanics 243

Emission intensity
2
CV,m/R

415 420
0 Wavelength, λ/nm
0 0.5 1 1.5 2
T/θE Figure 7A.6 A region of the spectrum of radiation emitted by
excited iron atoms consists of radiation at a series of discrete
Figure 7A.5 Experimental low-temperature molar heat capacities wavelengths (or frequencies).
(open circles) and the temperature dependence predicted on the
basis of Einstein’s theory (solid line, eqn 7A.9). The theory accounts

for the dependence fairly well, but is everywhere too low. wavenumber (   /c, see The chemist’s toolkit 7A.1) is called its
spectrum (from the Latin word for appearance).
An atomic emission spectrum is shown in Fig. 7A.6, and a
and the classical result (CV,m = 3R) is obtained. At low tempera- molecular absorption spectrum is shown in Fig. 7A.7. The ob-
tures (in the sense T << θE), eE /T  1 and vious feature of both is that radiation is emitted or absorbed at
2 2 2
a series of discrete frequencies. This observation can be under-
    eE / 2T     stood if the energy of the atoms or molecules is also confined
f E (T )   E    /T    E  e E /T  (7A.9c)
T  eE  T  to discrete values, because then the energies that a molecule
can discard or acquire are also confined to discrete values
The strongly decaying exponential function goes to zero more (Fig. 7A.8). If the energy of an atom or molecule decreases by
rapidly than 1/T 2 goes to infinity; so fE → 0 as T → 0, and the ΔE, and this energy is carried away as radiation, the frequency
heat capacity approaches zero, as found experimentally. The of the radiation n and the change in energy are related by the
physical reason for this success is that as the temperature is Bohr frequency condition:
lowered, less energy is available to excite the atomic oscilla-
E  h Bohr frequency condition (7A.10)
tions. At high temperatures many oscillators are excited into
high energy states leading to classical behaviour. A molecule is said to undergo a spectroscopic transition, a
Figure 7A.5 shows the temperature dependence of the heat change of state, and as a result an emission ‘line’, a sharply de-
capacity predicted by the Einstein formula and some experi- fined peak, appears in the spectrum at frequency n.
mental data; the value of the Einstein temperature is adjusted
to obtain the best fit to the data. The general shape of the
curve is satisfactory, but the numerical agreement is in fact Rotational
transitions
quite poor. This discrepancy arises from Einstein’s assump-
Absorption intensity

tion that all the atoms oscillate with the same frequency. A
more sophisticated treatment, due to Peter Debye, allows the Vibrational
oscillators to have a range of frequencies from zero up to a transitions
maximum. This approach results in much better agreement
with the experimental data and there can be little doubt that Electronic
mechanical motion as well as electromagnetic radiation is Electronic transition
quantized. transition

200 240 280 320


Wavelength, λ/nm
7A.1(c) Atomic and molecular spectra
Figure 7A.7 A molecule can change its state by absorbing
The most compelling and direct evidence for the quantiza- radiation at definite frequencies. This spectrum is due to the
tion of energy comes from spectroscopy, the detection and electronic, vibrational, and rotational excitation of sulfur dioxide
analysis of the electromagnetic radiation absorbed, emitted, or (SO2) molecules. The observation of discrete spectral lines
scattered by a substance. The record of the variation of the in- suggests that molecules can possess only discrete energies, not an
tensity of this radiation with frequency (n), wavelength (λ), or arbitrary energy.
244 7 Quantum theory

Another experiment shows that electrons—which classical


E3
physics treats as particles—also display the characteristics of
 = E3 – E2
h
waves. This wave–particle duality, the blending together of the
E2 characteristics of waves and particles, lies at the heart of quan-
tum mechanics.
Energy, E

 = E2 – E1
h

h = E3 – E1
7A.2(a)The particle character of
electromagnetic radiation
E1 The Planck treatment of black-body radiation introduced the
idea that an oscillator of frequency n can have only the ener-
gies 0, hn, 2hn,…. This quantization leads to the suggestion
Figure 7A.8 Spectroscopic transitions, such as those shown in (and at this stage it is only a suggestion) that the resulting elec-
Fig. 7A.6, can be accounted for by supposing that an atom (or
tromagnetic radiation of that frequency can be thought of as
molecule) emits electromagnetic radiation as it changes from a
consisting of 0, 1, 2,… particles, each particle having an en-
discrete level of high energy to a discrete level of lower energy.
High-frequency radiation is emitted when the energy change is
ergy hn. These particles of electromagnetic radiation are now
large. Transitions like those shown in Fig. 7A.7 can be explained called photons. Thus, if an oscillator of frequency n is excited
by supposing that a molecule (or atom) absorbs radiation as it to its first excited state, then one photon of that frequency is
changes from a low-energy level to a higher-energy level. present, if it is excited to its second excited state, then two
photons are present, and so on. The observation of discrete
emission spectra from atoms and molecules can be pictured
Brief illustration 7A.2
as the atom or molecule generating a photon of energy hn
Atomic sodium produces a yellow glow (as when a sodium salt when it discards an energy of magnitude ΔE, with ΔE = hn.
is introduced into a flame or in some street lamps) resulting Note that one transition generates one photon, not a shower
from the emission of radiation of 590 nm. The spectroscopic of many photons.
transition responsible for the emission involves electronic
Example 7A.1 Calculating the number of photons
energy levels that have a separation given by eqn 7A.10:
34 8 1
hc (6.626  10 Js)  (2.998  10 ms ) Calculate the number of photons emitted by a 100 W yellow
E  h   lamp in 1.0 s. Take the wavelength of yellow light as 560 nm,
 9
590  10 m
and assume 100 per cent efficiency.
 3.37  1019 J
Collect your thoughts Each photon has an energy hn, so the
This energy difference can be expressed in a variety of ways. total number N of photons needed to produce an energy E is
For instance, multiplication by Avogadro’s constant results in N = E/hn. To use this equation, you need to know the frequency
an energy separation per mole of atoms, of 203 kJ mol−1, com- of the radiation (from n = c/λ) and the total energy emitted by
parable to the energy of a weak chemical bond. the lamp. The latter is given by the product of the power (P, in
watts) and the time interval, Δt, for which the lamp is turned
on: E = PΔt (see The chemist’s toolkit 2A.1).

Exercises The solution The number of photons is


E7A.1 Calculate the wavelength and frequency at which the intensity of the E P t  P t
radiation is a maximum for a black body at 298 K. N  
h h(c / ) hc
E7A.2 The intensity of the radiation from an object is found to be a maximum at
2000 cm−1. Assuming that the object is a black body, calculate its temperature. Substitution of the data gives (using 1 W = 1 J s−1)
E7A.3 Calculate the molar heat capacity of a monatomic non-metallic solid at (5.60  107 m)  (100 Js 1 )  (1.0 s)
298 K which is characterized by an Einstein temperature of 2000 K. Express N  2.8  1020
(6.626  1034 J s)  (2.998  108 mss 1 )
your result as a multiple of 3R.
Self-test 7A.1 How many photons does a monochromatic
(single frequency) infrared rangefinder of power 1 mW and
7A.2 Wave–particle duality wavelength 1000 nm emit in 0.1 s?
Answer: 5 × 1014

The experiments about to be described show that electro-


magnetic radiation—which classical physics treats as wave- So far, the existence of photons is only a suggestion.
like—actually also displays the characteristics of particles. Experimental evidence for their existence comes from the
7A The origins of quantum mechanics 245

Kinetic energy of photoelectrons, Ek


Rb K Na

2.30 eV Ek

2.25 eV

Energy, E
2.09 eV

h Φ h Φ
Increasing
work function

Frequency of incident radiation,  (a) (b)

Figure 7A.9 In the photoelectric effect, it is found that no Figure 7A.10 The photoelectric effect can be explained if it is
electrons are ejected when the incident radiation has a frequency supposed that the incident radiation is composed of photons
below a certain value that is characteristic of the metal. Above that have energy proportional to the frequency of the radiation.
that value, the kinetic energy of the photoelectrons varies linearly (a) The energy of the photon is insufficient to drive an electron
with the frequency of the incident radiation. out of the metal. (b) The energy of the photon is more than
enough to eject an electron, and the excess energy is carried
away as the kinetic energy of the photoelectron (the ejected
electron).
measurement of the energies of electrons produced in the
­photoelectric effect, the ejection of electrons from metals
when they are exposed to ultraviolet radiation. The experimen- A practical application of eqn 7A.10 is that it provides a tech-
tal characteristics of the photoelectric effect are as follows: nique for the determination of Planck’s constant, because the
• No electrons are ejected, regardless of the intensity of the slopes of the lines in Fig. 7A.9 are all equal to h.
radiation, unless its frequency exceeds a threshold value
characteristic of the metal.
• The kinetic energy of the ejected electrons increases lin- Example 7A.2 Calculating the longest wavelength
early with the frequency of the incident radiation but is
capable of photoejection
independent of the intensity of the radiation.
• Even at low radiation intensities, electrons are ejected A photon of radiation of wavelength 305 nm ejects an electron
immediately if the frequency is above the threshold value. with a kinetic energy of 1.77 eV from a metal. Calculate the
longest wavelength of radiation capable of ejecting an electron
Figure 7A.9 illustrates the first and second characteristics. from the metal.
These observations strongly suggest that in the photoelectric
Collect your thoughts You can use eqn 7A.11, rearranged into
effect a particle-like projectile collides with the metal and, if the
Φ = hn − Ek, to compute the work function because you know
kinetic energy of the projectile is high enough, an electron is
the frequency of the photon from n = c/λ. The threshold for
ejected. If the projectile is a photon of energy hn (n is the fre-
photoejection is the lowest frequency at which electron ejection
quency of the radiation), the kinetic energy of the electron is Ek, occurs without there being any excess energy. This threshold
and the energy needed to remove an electron from the metal, will be reached when the photon energy is equal to the work
which is called its work function, is Φ (uppercase phi), then as function. The corresponding wavelength is then found by using
illustrated in Fig. 7A.10, the conservation of energy implies that λmax = hc/Φ.

h  Ek   or Ek  h   Photoelectric effect  (7A.11) The solution The work function is

This model explains the three experimental observations: = c /λ

hc
• Photoejection cannot occur if hn < Φ because the photon Φ = h − Ek = − Ek
λ
brings insufficient energy.
(6.626 × 10−34 J s) × (2.998 × 108 m s−1)
• The kinetic energy of an ejected electron increases linearly =
305 × 10−9 m
with the frequency of the photon.
− (1.77 eV) × (1.602 × 10−19 J eV−1)
• When a photon collides with an electron, it gives up all = 3.67... × 10−19 J
its energy, so electrons should appear as soon as the colli-
sions begin, provided the photons have sufficient energy. The longest wavelength that can cause photoejection is t­ herefore
246 7 Quantum theory

34 8 1
hc (6.626  10 Js)  (2.998  10 ms ) Diffracted
max    540 nm electrons
 3.67 1019 J

Electron
Self-test 7A.2 When ultraviolet radiation of wavelength 165 nm beam
strikes a certain metal surface, electrons are ejected with a
speed of 1.24 Mm s−1. Calculate the speed of electrons ejected
by radiation of wavelength 265 nm.
Answer: 735 km s−1

Ni crystal

7A.2(b) The wave character of particles Figure 7A.11 The Davisson–Germer experiment. The scattering
of an electron beam from a nickel crystal shows a variation in
Although contrary to the long-established wave theory of ra- intensity characteristic of a diffraction experiment in which
diation, the view that radiation consists of particles had been waves interfere constructively and destructively in different
held before, but discarded. No significant scientist, however, directions.
had taken the view that matter is wave-like. Nevertheless, ex-
periments carried out in 1927 forced people to consider that
suggested that any particle, not only photons, travelling with a
possibility. The crucial experiment was performed by Clinton
linear momentum p = mv (with m the mass and v the speed of
Davisson and Lester Germer, who observed the diffraction of
the particle) should have in some sense a wavelength given by
electrons by a crystal (Fig. 7A.11). As remarked in The chem-
what is now called the de Broglie relation:
ist’s toolkit 7A.2, diffraction is the interference caused by an
object in the path of waves. Davisson and Germer’s success h
was a lucky accident, because a chance rise of temperature  de Broglie relation (7A.12)
p
caused their polycrystalline sample to anneal, and the or-
dered planes of atoms then acted as a diffraction grating. The That is, a particle with a high linear momentum has a short
Davisson–Germer experiment, which has since been repeated wavelength. Macroscopic bodies have such high momenta
with other particles (including α particles, molecular hydro- even when they are moving slowly (because their mass is so
gen, and neutrons), shows clearly that particles have wave-like great), that their wavelengths are undetectably small, and the
properties. At almost the same time, G.P. Thomson showed wave-like properties cannot be observed. This undetectability
that a beam of electrons was diffracted when passed through is why classical mechanics can be used to explain the behaviour
a thin gold foil. of macroscopic bodies. It is necessary to invoke quantum me-
Some progress towards accounting for wave–particle dual- chanics only for microscopic bodies, such as atoms and mol-
ity had already been made by Louis de Broglie who, in 1924, ecules, in which masses are small.

The chemist’s toolkit 7A.2 Diffraction of waves


A characteristic property of waves is that they interfere with one
another, which means that they result in a greater amplitude
where their displacements add and a smaller amplitude where
their displacements subtract (Sketch 1). The former is called
‘constructive interference’ and the latter ‘destructive interfer-
ence’. The regions of constructive and destructive interference
Constructive interference
show up as regions of enhanced and diminished intensity. The
phenomenon of diffraction is the interference caused by an
object in the path of waves and occurs when the dimensions of
the object are comparable to the wavelength of the radiation.
Light waves, with wavelengths of the order of 500 nm, are dif-
fracted by narrow slits.
Destructive interference
Sketch 1
7A The origins of quantum mechanics 247

Example 7A.3 6.626  1034 Js


Estimating the de Broglie wavelength 
{2  (9.109  1031 kg )  (1.602  1019 C)  (4.0  104 V))}1/ 2
Estimate the wavelength of electrons that have been accelerated
 6.1  1012 m
from rest through a potential difference of 40 kV. Electrons
accelerated in this way are used in the technique of electron or 6.1 pm.
diffraction for imaging biological systems and for the determi-
nation of the structures of solid surfaces (Topic 19A). Self-test 7A.3 Calculate the wavelength of (a) a neutron with a
translational kinetic energy equal to kT at 300 K, (b) a tennis
Collect your thoughts To use the de Broglie relation, you need ball of mass 57 g travelling at 80 km h−1.
to know the linear momentum, p, of the electrons. To calculate Answer: (a) 178 pm, (b) 5.2 × 10−34 m
the linear momentum, note that the energy acquired by an
electron accelerated through a potential difference Δϕ is eΔϕ,
where e is the magnitude of its charge. At the end of the period
of acceleration, all the acquired energy is in the form of kinetic
=
energy, Ek 12= mev 2 p2 /2me. You can therefore calculate p by
2
setting p /2me equal to eΔϕ. For the manipulation of units use Exercises
1 V C = 1 J and 1 J = 1 kg m2 s−2. E7A.4 A sodium lamp emits yellow light (550 nm). How many photons does it
2 emit each second if its power is (i) 1.0 W, (ii) 100 W?
The solution The expression p /2me = eΔϕ implies that
p = (2meeΔϕ)1/2 then, from the de Broglie relation λ = h/p, E7A.5 The work function of metallic caesium is 2.14 eV. Calculate the
kinetic energy and the speed of the electrons ejected by light of wavelength
h (i) 700 nm, (ii) 300 nm.

(2mee )1/ 2 E7A.6 To what speed must an electron be accelerated from rest for it to have
a de Broglie wavelength of 100 pm? What accelerating potential difference is
Substitution of the data and the fundamental constants gives needed?

Checklist of concepts
☐ 1. A black body is an object capable of emitting and ☐ 4. The photoelectric effect establishes the view that elec-
absorbing all wavelengths of radiation without favour- tromagnetic radiation, regarded in classical physics as
ing any wavelength. wave-like, consists of particles (photons).
☐ 2. An electromagnetic field of a given frequency can take ☐ 5. The diffraction of electrons establishes the view that elec-
up energy only in discrete amounts. trons, regarded in classical physics as particles, are wave-
☐ 3. Atomic and molecular spectra show that atoms like with a wavelength given by the de Broglie relation.
and molecules can take up energy only in discrete ☐ 6. Wave–particle duality is the recognition that the con-
amounts. cepts of particle and wave blend together.

Checklist of equations
Property Equation Comment Equation number
Wien’s law λmaxT = c2/5 c2 is the second radiation constant, c2 = hc/k 7A.1, 7A.7
4 5 4 3 3
Stefan–Boltzmann law E(T ) = aT a  8 k /15h c 7A.3, 7A.8
Planck distribution ρ(λ,T) = 8πhc/{λ5(ehc/λkT − 1)} Black-body radiation 7A.6
3 3 hν/kT
ρ(n,T) = 8πhn /{c (e − 1)}
Einstein formula for heat capacity of a solid CV,m(T) = 3RfE(T) Einstein temperature: θE = hn/k 7A.9
 E / 2T  E /T
f E (T )  ( E /T ) {e
2
/(e 2
 1)}
Bohr frequency condition ΔE = hn 7A.10
Photoelectric effect Ek = hn − Φ Φ is the work function 7A.11
de Broglie relation λ = h/p 7A.12
TOPIC 7B Wavefunctions

The Schrödinger equation can be regarded as a fundamen-


➤ Why do you need to know this material? tal postulate of quantum mechanics, but its plausibility can be
Wavefunctions provide the essential foundation for under- demonstrated by showing that, for the case of a free particle, it
standing the properties of electrons in atoms and mol- is consistent with the de Broglie relation (Topic 7A). The first
ecules, and are central to explanations in chemistry. step is to note the potential energy V(x) is zero everywhere, so
E is equal to the kinetic energy E = p2/2m and eqn 7B.1 can be
➤ What is the key idea? written as
All the dynamical properties of a system are contained
2 d 2 (x ) p2
in its wavefunction, which is obtained by solving the    (x )
Schrödinger equation. 2m dx 2 2m

A wave of wavelength λ has the form ψ(x) = sin(2πx/λ) so the


➤ What do you need to know already?
left-hand side of this equation is
You need to be aware of the shortcomings of classical
physics that drove the development of quantum theory 2 d 2 (x ) 2 d 2 sin(2x / )
(Topic 7A).  
2m dx 2
2m dx 2
 (x)
2  2  
2

 sin(2x / )
2m   
In classical mechanics an object travels along a definite path or h2
  (x )
trajectory. In quantum mechanics a particle in a particular state 2m 2
is described by a wavefunction, ψ (psi), which is spread out in
space, rather than being localized. The wavefunction contains Comparison of the two preceding equations shows that
all the dynamical information about the object in that state, p2 /2m  h2 /2m 2 and therefore that p2 = h2/λ2. Therefore
such as its position and momentum. λ2 = h2/p2, which implies the de Broglie relation, λ = h/p.

7B.1 The Schrödinger equation 7B.2 The Born interpretation


In 1926 Erwin Schrödinger proposed an equation for find- One piece of dynamical information contained in the wave-
ing the wavefunctions of any system. The time-independent function is the location of the particle. Max Born used an anal-
Schrödinger equation for a particle of mass m moving in one ogy with the wave theory of radiation, in which the square of
dimension with energy E in a system that does not change with the amplitude of an electromagnetic wave in a region is inter-
time (for instance, its volume remains constant) is preted as its intensity and therefore (in quantum terms) as a
measure of the probability of finding a photon present in the
2 d 2 Time -independent region. The Born interpretation of the wavefunction is:
  V(x )  E 
Schrodinger equation
n
 (7B.1)
2m dx 2 If the wavefunction of a particle has the value ψ at x,
then the probability of finding the particle between x
The constant ħ = h/2π (which is read h-cross or h-bar) is a con- and x + dx is proportional to |ψ|2dx (Fig. 7B.1).
venient modification of Planck’s constant used widely in quan-
tum mechanics; V(x) is the potential energy of the particle at x. The quantity |ψ|2 = ψ⋆ψ, where ψ⋆ is the complex conjugate of
Because the total energy E is the sum of potential and kinetic ψ, allows for the possibility that ψ is complex (see The chemist’s
energies, the first term on the left must be related (in a manner toolkit 7B.1). If the wavefunction is real, like sin(2πx/λ) is, then
explored later) to the kinetic energy of the particle. |ψ|2 = ψ2.
7B Wavefunctions 249

z
Probability = |ψ|2dx
|ψ|2

dz
dx
r
dx
dy

x y
x x + dx

Figure 7B.1 The wavefunction ψ is a probability amplitude in Figure 7B.2 The Born interpretation of the wavefunction in three-
the sense that its square modulus (ψ⋆ψ or |ψ|2) is a probability dimensional space implies that the probability of finding the
density. The probability of finding a particle in the region particle in the volume element dτ = dxdydz at some position r is
between x and x + dx is proportional to |ψ|2dx. Here, the proportional to the product of dτ and the value of |ψ(r)|2 at that
probability density is represented by the density of shading in position.
the superimposed band.

Wavefunction Probability density

The chemist’s toolkit 7B.1 Complex numbers


A complex number z has the form z = x + iy, where i  1.
The complex conjugate of a complex number z is z* = x − iy.
Complex numbers combine together according to the follow-
ing rules:
Addition and subtraction:
(a  ib)  (c  id )  (a  c)  i(b  d )
Multiplication: Figure 7B.3 The sign of a wavefunction has no direct
physical significance: the positive and negative regions of
(a  ib) (c  id )  (ac  bd )  i(bc  ad ) this wavefunction both correspond to the same probability
Two special relations are: distribution (as given by the square modulus of ψ and depicted by
the density of the shading).
Modulus: | z |  (z  z )1/ 2  (x 2  y 2 )1/ 2
Euler’s relation: ei  cos   i sin  , which implies that ei  1, The Born interpretation does away with any worry about the
cos   12 (ei  e  i ), and sin    12 i(ei  e  i ). significance of a negative (and, in general, complex) value of ψ
because |ψ|2 is always real and nowhere negative. There is no
direct significance in the negative (or complex) value of a wave-
function: only the square modulus is directly physically signifi-
Because |ψ|2dx is a (dimensionless) probability, |ψ|2 is the cant, and both negative and positive regions of a wavefunction
probability density, with the dimensions of 1/length (for a may correspond to a high probability of finding a particle in a
one-dimensional system). The wavefunction ψ itself is called region (Fig. 7B.3). However, the presence of positive and nega-
the probability amplitude. For a particle free to move in three tive regions of a wavefunction is of great indirect significance,
dimensions (for example, an electron near a nucleus in an because it gives rise to the possibility of constructive and de-
atom), the wavefunction depends on the coordinates x, y, and structive interference between different wavefunctions.
z and is denoted ψ(r). In this case the Born interpretation is A wavefunction may be zero at one or more points, and at
(Fig. 7B.2): these locations the probability density is also zero. It is impor-
tant to distinguish a point at which the wavefunction is zero
If the wavefunction of a particle has the value ψ(r)
(for instance, far from the nucleus of a hydrogen atom) from
at r, then the probability of finding the particle in an
the point at which it passes through zero. The latter is called
infinitesimal volume dτ = dxdydz at that position is
a node. A location where the wavefunction approaches zero
proportional to |ψ(r)|2dτ.
without actually passing through zero is not a node. Thus,
In this case, |ψ|2 has the dimensions of 1/length3 and the the wavefunction sin(2πx/λ) has nodes where the wave passes
wavefunction itself has dimensions of 1/length3/2 (and units through zero, but the wavefunction e−kx has no nodes, despite
such as m−3/2). becoming zero as x → ∞.
250 7 Quantum theory

Brief illustration 7B.1 In quantum mechanics it is common to write all such integrals
in a short-hand form as
The wavefunction of an electron in the lowest energy state of a
  d  1

(7B.4c)
hydrogen atom is proportional to e −r /a0 , where a0 is a constant
and r the distance from the nucleus. The relative probabili- where dτ is the appropriate volume element and the integration
ties of finding the electron inside a tiny region of volume δV is understood as being over all space.
located at the nucleus and a distance a0 from the nucleus are
Example 7B.1 Normalizing a wavefunction
P (0)  (0)2 V  (0)2 e0
   2  7.4
P (a0 )  (a0 ) V  (a0 ) e
2 2
Carbon nanotubes are thin hollow cylinders of carbon with
diameters between 1 nm and 2 nm, and lengths of several
That is, it is more probable (by a factor of 7) that the electron
micrometres. According to one simple model, the lowest-­
will be found in a volume element at the nucleus than in a
energy electrons of the nanotube are described by the wave-
volume element of the same size located at a distance a0 from
function sin(πx/L), where L is the length of the nanotube. Find
the nucleus.
the normalized wavefunction.
Collect your thoughts Because the wavefunction is one-dimen-
sional, you need to find the factor N that guarantees that the
7B.2(a) Normalization
integral in eqn 7B.4a is equal to 1. The wavefunction is real, so
A mathematical feature of the Schrödinger equation is that if ψ ψ⋆ = ψ. Relevant integrals are found in the Resource section.
is a solution, then so is Nψ, where N is any constant. This fea-
The solution Write the wavefunction as ψ = N sin(πx/L), where
ture is confirmed by noting that because ψ occurs in every term
N is the normalization factor. The limits of integration are
in eqn 7B.1, it can be replaced by Nψ and the constant factor x = 0 to x = L because the space available to the electron spans
N cancelled to recover the original equation. This freedom to the length of the tube. It follows that
multiply the wavefunction by a constant factor means that it is
always possible to find a normalization constant, N, such that Integral

 T.2

rather than the probability density being proportional to |ψ|2 it L  x
  d  N 0 sin L dx  12 N L
 2 2 2

becomes equal to |ψ|2.


A normalization constant is found by noting that, for a nor- For the wavefunction to be normalized, this integral must be
malized wavefunction Nψ, the probability that a particle is in equal to 1; that is, 12 N 2 L = 1, and hence
the region dx is equal to (Nψ⋆)(Nψ)dx (N is taken to be real). 1/ 2
Furthermore, the sum over all space of these individual prob- 2
N  
abilities must be 1 (the probability of the particle being some- L
where is 1). Expressed mathematically, the latter requirement is
The normalized wavefunction is therefore

   dx  1
2
N 
(7B.2) 2
1/ 2
x
     sin
L L
and therefore
Because L is a length, the dimensions of ψ are 1/length1/2, and
1 therefore those of ψ2 are 1/length, as is appropriate for a prob-
N  (7B.3)
 
 1/ 2
  dx ability density in one dimension.

Self-test 7B.1 The wavefunction for the next higher energy
Provided this integral has a finite value (that is, the wavefunc- level for the electrons in the same tube is sin(2πx/L). Normalize
tion is ‘square integrable’), the normalization factor can be this wavefunction.
Answer: N = (2/L)1/2
found and the wavefunction ‘normalized’ (and specifically
‘normalized to 1’). From now on, unless stated otherwise, all
wavefunctions are assumed to have been normalized to 1, in To calculate the probability of finding the system in a finite
which case in one dimension region of space the probability density is summed (integrated)

over the region of interest. Thus, for a one-dimensional system,

  dx  1 (7B.4a) the probability P of finding the particle between x1 and x2 is
given by
and in three dimensions
x2 Probability
  
P   | (x ) |2 dx  (7B.5)
  
  
  dxdydz  1 (7B.4b) x1 [one -dimensional region]
7B Wavefunctions 251

Example 7B.2 Evaluating a probability


As seen in Example 7B.1, the lowest-energy electrons of a car- ψ ψ
bon nanotube of length L can be described by the normalized
wavefunction (2/L)1/2 sin(πx/L). What is the probability of find- (a) (b)
ing the electron between x = L/4 and x = L/2?
Collect your thoughts Use eqn 7B.5 and the normalized wave-
function to write an expression for the probability of finding ψ ψ
the electron in the region of interest. Relevant integrals are
given in the Resource section. (c) (d)

The solution From eqn 7B.5 the probability is Figure 7B.4 The wavefunction must satisfy stringent conditions
Integral T.2 for it to be acceptable: (a) unacceptable because it is infinite over
  
2 L /2 2 a finite region; (b) unacceptable because it is not single-valued;
P   sin (x /L) dx (c) unacceptable because it is not continuous; (d) unacceptable
L L/4
because its slope is discontinuous.
It follows that
L /2
2  x sin(2x /L)  2L L 1  Overall, therefore, the constraints on the wavefunction,
P    0  0.409
L  2 4 /L  L/4

L 4 8 4 /L  which are summarized in Fig. 7B.4, are that it
It follows that there is a chance of about 41 per cent that the • must not be infinite over a finite region;
electron will be found in the region. • must be single-valued;
Self-test 7B.2 As remarked in Self-test 7B.1, the normalized • must be continuous;
wavefunction of the next higher energy level of the electron in
• must have a continuous first derivative (slope).
this model of the nanotube is (2/L)1/2 sin(2πx/L). What is the
probability of finding the electron between x = L/4 and x = L/2 The last of these constraints does not apply if the potential en-
when it is described by this wavefunction? ergy has abrupt, infinitely high steps (as in the particle-in-a-
box model treated in Topic 7D).
Answer: 0.25

7B.2(c) Quantization
7B.2(b) Constraints on the wavefunction
The constraints just noted are so severe that acceptable solu-
The Born interpretation puts severe restrictions on the accept- tions of the Schrödinger equation do not in general exist for
ability of wavefunctions. The first constraint is that ψ must not arbitrary values of the energy E. In other words, a particle may
be infinite over a finite region; if it were, the Born interpretation possess only certain energies, for otherwise its wavefunction
would fail. This requirement rules out many possible solutions of would be physically unacceptable. That is,
the Schrödinger equation, because many mathematically accept-
able solutions rise to infinity and are therefore physically unac- As a consequence of the restrictions on its wavefunction,
ceptable. The Born interpretation also rules out solutions of the the energy of a particle is quantized.
Schrödinger equation that give rise to more than one value of |ψ|2 These acceptable energies are found by solving the Schrödinger
at a single point. It would be absurd to have more than one value equation for motion of various kinds, and selecting the solu-
of the probability density for the particle at a point. This restric- tions that conform to the restrictions listed above.
tion is expressed by saying that the wavefunction must be single-
valued; that is, it must have only one value at each point of space.
The Schrödinger equation itself also implies some math- Exercises
ematical restrictions on the type of functions that can occur. E7B.1 A possible wavefunction for an electron in a region of length L (i.e. from
Because it is a second-order differential equation (in the sense x = 0 to x = L) is sin(2πx/L). (i) Normalize this wavefunction (to 1). (ii) What
that it depends on the second derivative of the wavefunction), is the probability of finding the electron in the range dx at x = L/2? (iii) At
what value or values of x is the probability density a maximum? Locate the
d2ψ/dx2 must be well-defined if the equation is to be applica-
positions of any nodes in the wavefunction. You need consider only the range
ble everywhere. The second derivative is defined only if the first x = 0 to x = L.
derivative is continuous. That is (except as specified below),
E7B.2 Imagine a particle free to move in the x direction. Which of the
there can be no kinks in the function. In turn, the first deriva- following wavefunctions would be acceptable for such a particle? In each
tive is defined only if the function is continuous: no sharp steps case, give your reasons for accepting or rejecting each function. (i) ψ(x) = x2;
2

are permitted. (ii) ψ(x) = 1/x; (iii)  (x )  e  x .


252 7 Quantum theory

Checklist of concepts
☐ 1. A wavefunction is a mathematical function that con- ☐ 5. A wavefunction is normalized if the integral over all
tains all the dynamical information about a system. space of its square modulus is equal to 1.
☐ 2. The Schrödinger equation is a second-order differential ☐ 6. A wavefunction must be single-valued, continuous, not
equation used to calculate the wavefunction of a system. infinite over a finite region of space, and (except in spe-
☐ 3. According to the Born interpretation, the probability cial cases) have a continuous slope.
density at a point is proportional to the square of the ☐ 7. The quantization of energy stems from the constraints
wavefunction at that point. that an acceptable wavefunction must satisfy.
☐ 4. A node is a point where a wavefunction passes through
zero.

Checklist of equations
Property Equation Comment Equation number
The time-independent Schrödinger equation −(ħ2/2m)(d2ψ/dx2) + V(x)ψ = Eψ One-dimensional system⋆ 7B.1

Normalization   d  1 Integration over all space 7B.4c


x2
Probability of a particle being between x1 and x2 P   | ( x ) |
2
dx One-dimensional region 7B.5
x1


Higher dimensions are treated in Topics 7D, 7F, and 8A.
TOPIC 7C Operators and observables

The quantity Ĥ (commonly read h-hat) is an operator, an


➤ Why do you need to know this material? ­expression that carries out a mathematical operation on a func-
To interpret the wavefunction fully it is necessary to be
tion. In this case, the operation is to take the second derivative
able to extract dynamical information from it. The predic-
of ψ, and (after multiplication by −ħ2/2m) to add the result to
tions of quantum mechanics are often very different from
the outcome of multiplying ψ by V(x).
those of classical mechanics, and those differences are
The operator Ĥ plays a special role in quantum mechanics,
essential for understanding the structures and properties
and is called the hamiltonian operator after the nineteenth cen-
of atoms and molecules.
tury mathematician William Hamilton. He developed a form of
classical mechanics which, it subsequently turned out, is well
➤ What is the key idea? suited to the formulation of quantum mechanics. The hamilto-
The dynamical information in the wavefunction is extract-
nian operator (and commonly simply ‘the hamiltonian’) is the
ed by calculating the expectation values of hermitian
operator corresponding to the total energy of the system, the
operators.
sum of the kinetic and potential energies. In eqn 7C.1b the sec-
ond term on the right is the potential energy, so the first term
➤ What do you need to know already? (the one involving the second derivative) must be the operator
You need to know that the state of a particle is fully
for kinetic energy.
described by a wavefunction (Topic 7B), and that the prob-
In general, an operator acts on a function to produce a new
ability density is proportional to the square modulus of the
function, as in
wavefunction. (operator )(function) = (new function)
In some cases the new function is the same as the original
function, perhaps multiplied by a constant. Combinations of
operators and functions that have this property are of great im-
A wavefunction contains all the information it is possible to
portance in quantum mechanics.
obtain about the dynamical properties of a particle (for ex-
ample, its location and momentum). The Born interpretation
(Topic 7B) provides information about location, but the wave- Brief illustration 7C.1
function contains other information, which is extracted by
using the methods described in this Topic. When the operator d/dx, which means ‘take the derivative of
the following function with respect to x’, acts on the function
sin ax, it generates the new function a cos ax. However, when
d/dx operates on e−ax it generates −ae−ax, which is the original
7C.1 Operators function multiplied by the constant −a.

The Schrödinger equation can be written in the succinct form


Operator form of
7C.1(a) Eigenvalue equations
Ĥψ = Eψ 
Schrodinger equation
 (7C.1a)
The Schrödinger equation written as in eqn 7C.1a is an eigen-
Comparison of this expression with the one-dimensional value equation, an equation of the form
Schrödinger equation (operator )(function)  (constant factor )  (same function)
 d
2 2  (7C.2a)
  V(x )  E
2m dx 2 In an eigenvalue equation, the action of the operator on the
function generates the same function, multiplied by a con-
shows that in one dimension
stant. If a general operator is denoted Ω̂ (where Ω is uppercase
2 d 2 omega) and the constant factor by ω (lowercase omega), then
Hˆ =
− + V( x ) Hamiltonian operator  (7C.1b)
2m dx 2 an eigenvalue equation has the form
254 7 Quantum theory

ˆ = ωψ
Ωψ Eigenvalue equation (7C.2b) e­ xtension the operator for x2 is multiplication by x and then by
x again, or multiplication by x2. The operator corresponding to
If this relation holds, the function ψ is said to be an eigenfunc- 1
k x 2 is therefore
2 f
tion of the operator Ω̂ , and ω is the eigenvalue associated with
that eigenfunction. With this terminology, eqn 7C.2a can be ˆ( x )
=
V 1
kf x 2 × (7C.4)
2
written
In practice, the multiplication sign is omitted and multiplica-
(operator )(eigenfunction)  (eigenvalue)  (eigenfunction) tion is understood. To construct the operator for kinetic en-
 (7C.2c) ergy, the classical relation between kinetic energy and linear
Equation 7C.1a is therefore an eigenvalue equation in which ψ momentum, Ek = px2 /2m is used. Then, by using the operator
is an eigenfunction of the hamiltonian and E is the associated for px from eqn 7C.3:
eigenvalue. It follows that ‘solving the Schrödinger equation’ pxˆ px ˆ
 
  

can be expressed as ‘finding the eigenfunctions and eigenvalues 1   d   d  2 d 2
of the hamiltonian operator for the system’. Eˆ k = = −  (7C.5)
2m  i dx 
 i dx 
 2m dx 2
Just as the hamiltonian is the operator corresponding to
the total energy, there are operators that represent other It follows that the operator for the total energy, the hamiltonian
­observables, the measurable properties of the system, such as operator, is
linear momentum or electric dipole moment. For each such op-
ˆ = ωψ ,
erator Ω̂ there is an eigenvalue equation of the form Ωψ 2 d 2
Hˆ = Eˆ k + Vˆ =− + Vˆ( x ) Hamiltonian operator  (7C.6)
with the following significance: 2m dx 2
If the wavefunction is an eigenfunction of the operator Ω̂ where Vˆ(x ) is the operator corresponding to whatever form the
corresponding to the observable Ω, then the outcome of potential energy takes, exactly as in eqn 7C.1b.
a measurement of the property Ω will be the eigenvalue
corresponding to that eigenfunction.
For instance, if the property of interest, Ω, is the energy, E, Example 7C.1 Evaluating an observable
then the corresponding operator Ω̂ is the hamiltonian, and
What is the linear momentum of a free particle described by
the outcome of a measurement of the energy will be the eigen-
the wavefunctions (a) ψ(x) = eikx and (b) ψ(x) = e−ikx?
value E corresponding to the eigenfunction ψ in the equation
Ĥψ = Eψ . Quantum mechanics is formulated by constructing Collect your thoughts You need to operate on ψ with the opera-
the operator corresponding to the observable of interest and tor corresponding to linear momentum (eqn 7C.3), and inspect
then predicting the outcome of a measurement by examining the result. If the outcome is the original wavefunction multi-
the eigenvalues of the operator. plied by a constant (that is, if the application of the operator
results in an eigenvalue equation), then you can identify the
constant with the value of the observable.
7C.1(b) The construction of operators
The solution (a) For ψ(x) = eikx,
Observables are represented by operators built from the follow- Eigenvalue
ing position and linear momentum operators: dψ deikx
pˆx ψ = = = × ik eikx = +k ψ
i dx i dx i
 d
xˆ = x × pˆ x = Specification of operators (7C.3)
i dx This is an eigenvalue equation, with eigenvalue +kħ. It follows that
That is, a measurement of the momentum will give the value px = +kħ.

• the operator for location along the x-axis is multiplication (b) For ψ(x) = e−ikx,
(of the wavefunction) by x, Eigenvalue

ˆpxψ =  dψ =  de
− ikx

• the operator for linear momentum parallel to the x-axis = × (−ik )e = −kψ
− ikx

i dx i dx i
is ħ/i times the derivative (of the wavefunction) with
respect to x. Now the eigenvalue is −kħ, so px = −kħ. In case (a) the momen-
tum is positive, meaning that the particle is travelling in the
The definitions in eqn 7C.3 are used to construct opera- positive x-direction, whereas in (b) the particle is moving in
tors for other spatial observables. For example, suppose the the opposite direction. This result illustrates a general feature
potential energy has the form V (x ) = 12 kf x 2, where kf is a con- of quantum mechanics: taking the complex conjugate of a
stant (this potential energy describes the vibrations of atoms in wavefunction reverses the direction of travel. An implication is
­molecules). Because the operator for x is multiplication by x, by that if the wavefunction is real (such as cos(2πx/λ) for a wave
7C Operators and observables 255

with wavelength λ), then taking the complex conjugate leaves Region contributes
the wavefunction unchanged: there is no net direction of travel. high kinetic energy

Wavefunction, ψ
Self-test 7C.1 What is the kinetic energy of a particle described
by the wavefunction cos kx?
Answer: Ek = ħ2k2/2m

Region contributes
The expression for the kinetic energy operator (eqn 7C.5) low kinetic energy
reveals an important point about the Schrödinger equation. In
mathematics, the second derivative of a function is a measure
of its curvature: a large second derivative indicates a sharply x

curved function (Fig. 7C.1). It follows that a sharply curved Figure 7C.2 The observed kinetic energy of a particle is an
wavefunction is associated with a high kinetic energy, and one average of contributions from the entire space covered by the
with a low curvature is associated with a low kinetic energy. wavefunction. Sharply curved regions contribute a high kinetic
The curvature of a wavefunction in general varies from energy to the average; less sharply curved regions contribute only
place to place (Fig. 7C.2). Wherever a wavefunction is sharply a small kinetic energy.
curved, its contribution to the total kinetic energy is large.

Wavefunction, ψ
Wherever the wavefunction is not sharply curved, its contribu-
tion to the overall kinetic energy is low. The observed kinetic
energy of the particle is an average of all the contributions of
the kinetic energy from each region. Hence, a particle can be
expected to have a high kinetic energy if the average curvature
Etotal
of its wavefunction is high (that is, has sharply curved regions).
Locally there can be both positive and negative contributions to Ek
Energy, E

the kinetic energy (because the curvature can be either positive,


∪, or negative, ∩), but the average is always positive.
The association of high curvature with high kinetic energy V
is a valuable guide to the interpretation of wavefunctions and
x
the prediction of their shapes. For example, suppose the wave-
function of a particle with a given total energy and a potential Figure 7C.3 The wavefunction of a particle with a potential energy
energy that decreases with increasing x is required. Because the V that decreases towards the right. As the total energy is constant,
difference E − V = Ek increases from left to right, the wavefunc- the kinetic energy Ek increases to the right, which results in a faster
tion must become more sharply curved by oscillating more oscillation and hence greater curvature of the wavefunction.
rapidly as x increases (Fig. 7C.3). It is therefore likely that the
wavefunction will look like the function sketched in the illus-
7C.1(c) Hermitian operators
tration, as more detailed calculation confirms.
All the quantum mechanical operators that correspond to ob-
servables have a very special mathematical property: they are
‘hermitian’. A hermitian operator is one for which the follow-
High curvature, ing relation is true:
high kinetic energy
Wavefunction, ψ

{ }

ˆ dτ = ψ  Ωψ
∫ j ˆ i dτ
Hermiticity
∫ψ Ωψ

i j [definition]  (7C.7)

As stated in Topic 7B, in quantum mechanics d implies in-


Low curvature,
tegration over the full range of all relevant spatial variables.
low kinetic energy

Brief illustration 7C.2


x
The position operator (x ×) is hermitian because in this case
Figure 7C.1 The average kinetic energy of a particle can be the order of the factors in the integrand can be changed:
inferred from the average curvature of the wavefunction. This
 x d 

 x j d   j xi d 
 
i j i
figure shows two wavefunctions: the sharply curved function
corresponds to a higher kinetic energy than the less sharply The final step uses (  )   .
curved function.
256 7 Quantum theory

The demonstration that the linear momentum operator is her- Next suppose that ψ is an eigenfunction of Ω̂ with eigenvalue
mitian is more involved because the order of functions being ˆ = ωψ . Now use this relation in both integrals
ω. That is, Ωψ
differentiated cannot be changed. on the left- and right-hand sides:

 

  d    d

How is that done? 7C.1 Proving that the linear


The eigenvalue is a constant that can be taken outside the
momentum operator is hermitian integrals:
The task is to show that
{ } =ω

ω ∫ψ *ψ dτ = ω ∫ψ ⋆ψ dτ ⋆
∫ψψ ⋆dτ
{∫ψ }

∫ψ pˆ xψ j dτ = pˆ xψi dτ
 
i j
Finally, the (boxed) integrals cancel, leaving ω = ω⋆. It follows
with pˆ x given in eqn 7C.3. To do so, use ‘integration by parts’ that ω is real.
(see Part 1 of the Resource section) which, when applied to the
present case, gives
dg /dx 7C.1(d) Orthogonality
f
ψj
To say that two different functions ψi and ψj are orthogonal
∫ψ ⋆ pˆ ψ dτ = i ∫
i x j

ψ i⋆
dx
dx
means that the integral (over all space) of ψ iψ j is zero:
0
df/dx
fg g
  d  0 for i  j Orthogonality [definition] (7C.8)

dψ i⋆ i j
= ψ i ψj
i



i ∫ −
ψj
dx
dx
Functions that are both normalized and mutually orthogonal
The boxed term is zero because all wavefunctions are either are called orthonormal. Hermitian operators have the impor-
zero at x = ±∞ (see Topic 7B) or the product ψ iψ j converges to tant property that
the same value at x = +∞ and x = −∞. As a result
Eigenfunctions that correspond to different eigenvalues of
 dψ ∞
 dψ i  


a hermitian operator are orthogonal.
∫ψ − ∫ ψj
pˆ xψ j dτ =  ∫−∞ψ j
dx = dx 
 i
i
i −∞ dx  i d x  The proof of this property also follows from the definition of
{ }

= ∫ψ pˆ xψi dτ

j
hermiticity (eqn 7C.7).

as was to be proved. The penultimate line uses (  )   and


i   i. How is that done? 7C.3 Proving that the eigenfunctions of
hermitian operators are orthogonal
Start by supposing that ψj is an eigenfunction of Ω̂ with eigen-
Hermitian operators are enormously important in quantum ˆ = ω ψ ) and that ψ is an eigenfunction
value ωj (that is, Ωψ j j j i
mechanics because their eigenvalues are real: that is, in eqn ˆ = ω ψ , with ω ≠ ω ).
with a different eigenvalue ωi (that is, Ωψi i i i j
7C.2b ω⋆ = ω. Any measurement must yield a real value be- Then eqn 7C.7 becomes
cause a position, momentum, or an energy cannot be complex
 

or imaginary. Because the outcome of a measurement of an ob-   j j d   j 
i i d

i
servable is one of the eigenvalues of the corresponding opera-
tor, those eigenvalues must be real. It therefore follows that an The eigenvalues are constants and can be taken outside the
operator that represents an observable must be hermitian. The integrals; moreover, they are real (being the eigenvalues of
proof that their eigenfunctions are real makes use of the defini- hermitian operators), so i  i . Then
tion of hermiticity in eqn 7C.7.
 

 j  i j d  i  ji d

  d    

Next, note that 
j i j i

d , so
How is that done? 7C.2 Proving that the eigenvalues of
hermitian operators are real  j  i j d  i  j i d , hence ( j  i ) i j d  0
Begin by setting ψi and ψj to be the same, writing them both as The two eigenvalues are different, so ωj − ωi ≠ 0; therefore it
ψ. Then eqn 7C.7 becomes must be the case that i j d  0. That is, the two eigenfunc-
tions are orthogonal, as was to be proved.
{ }

∫ψ

Ωˆψ dτ = ∫ψ  Ωψ
ˆ dτ
7C Operators and observables 257

The hamiltonian operator is hermitian (it corresponds to an E7C.3 Functions of the form sin(nπx/L), where n = 1, 2, 3…, are wavefunctions
observable, the energy, but its hermiticity can be proved spe- in a region of length L (between x = 0 and x = L). Show that the wavefunctions
with n = 2 and 3 are orthogonal; you will find the necessary integrals in the
cifically). Therefore, if two of its eigenfunctions correspond to Resource section. (Hint: Recall that sin(nπ) = 0 for integer n.)
different energies, the two functions must be orthogonal. The
property of orthogonality is of great importance in quantum
mechanics because it eliminates a large number of integrals
from calculations. Orthogonality plays a central role in the the- 7C.2Superpositions and expectation
ory of chemical bonding (Focus 9) and spectroscopy (Focus 11). values
Example 7C.2
The hamiltonian for a free particle moving in one dimension is
Verifying orthogonality
2 d 2
Two possible wavefunctions for a particle constrained to be Hˆ = −
somewhere on the x axis between x = 0 and x = L are ψ1 =
2m dx 2
sin(πx/L) and ψ2 = sin(2πx/L). Outside this region the wave- The particle is ‘free’ in the sense that there is no potential to
functions are zero. The wavefunctions correspond to differ- constrain it, hence V(x) = 0. It is easily confirmed that ψ(x) =
ent energies. Verify that the two wavefunctions are mutually cos kx is an eigenfunction of this operator, as inferred in Topic
orthogonal. 7B (with k = 2π/λ):
Collect your thoughts To verify the orthogonality of two func-
2 d 2 k 2 2
tions, you need to integrate  2 1  sin(2x /L)sin(x /L) over Hˆψ (x ) =
− cos kx = cos kx
2m dx 2
2m
all space, and show that the result is zero. In principle the inte-
gral is taken from x = −∞ to x = +∞, but the wavefunctions are The energy associated with this wavefunction, k2ħ2/2m, is
zero outside the range x = 0 to L so you need integrate only over therefore well defined, as it is the eigenvalue of an eigenvalue
this range. Relevant integrals are given in the Resource section. equation. However, the same is not necessarily true of other ob-
The solution To evaluate the integral, use Integral T.5 from the servables. For instance, cos kx is not an eigenfunction of the
Resource section with a = 2π/L and b = π/L: linear momentum operator:
 dψ  d cos kx k
 sin(πx /L) sin(3πx /L) 
L
pˆ xψ (x ) = = = − sin kx  (7C.9)
L
i dx i dx i
∫0 sin(2πx /L)sin(πx /L)d=x  2(π /L) − 2(3π /L) 
 0 This expression is not an eigenvalue equation, because the
 sin π sin 3π  function on the right (sin kx) is different from that on the left
= −  (cos kx).
 2(π /L) 2(3π /L)  When the wavefunction of a particle is not an eigenfunction
 sin 0 sin 0  of an operator, the corresponding observable does not have a
− − 
 2(π /L) 2(3π /L)  definite value. However, in the current example the momen-
=0 tum is not completely indefinite because the cosine wavefunc-
tion can be written as a linear combination, or sum,1 of eikx and
The sine functions have been evaluated by using sin nπ = 0 e−ikx by using the identity cos kx  12 (eikx  e  ikx ) (see The chem-
for n = 0, ±1, ±2,…. The two functions are therefore mutually ist’s toolkit 7B.1). As shown in Example 7C.1, these two expo-
orthogonal. nential functions are eigenfunctions of pˆ x with eigenvalues +kħ
and −kħ, respectively. Therefore each one corresponds to a state
Self-test 7C.2 The next higher energy level has ψ3 = sin(3πx/L). of definite but different momentum. The wavefunction cos kx is
Confirm that the functions ψ1 = sin(πx/L) and ψ3 = sin(3πx/L) said to be a superposition of the two individual wavefunctions
are mutually orthogonal.
0 eikx and e−ikx, and is written
Answer:  sin(3x /L) sin(x /L) dx  0
L
 ikx  ikx
 e  e
Particle with linear Particle with linear
momentum  k momentum k

Exercises The interpretation of this superposition is that if many repeated


E7C.1 Construct the potential energy operator of a particle with potential
measurements of the momentum are made, then half the
energy V (x ) = 12 kf x 2 , where kf is a constant.
E7C.2 Identify which of the following functions are eigenfunctions of the 1
A linear combination is more general than a sum, for it includes weighted
sums of the form ax + by +… where a, b,… are constants. A sum is a linear
ikx −ax 2
operator d/dx: (i) cos(kx); (ii) e , (iii) kx, (iv) e . Give the corresponding
eigenvalue where appropriate. combination with a = b =… = 1.
258 7 Quantum theory

­ easurements would give the value px = +kħ, and half would


m Now suppose the (normalized) wavefunction is the linear
give the value px = −kħ. The two values ±kħ occur equally often combination of two eigenfunctions of the operator Ω̂ , each of
since eikx and e−ikx contribute equally to the superposition. All which is individually normalized to 1. Then
that can be inferred from the wavefunction cos kx about the
〈Ω 〉
= ∫ (c ψ
1 1 + c2ψ 2 ) Ωˆ (c1ψ 1 + c2ψ 2 )dτ
linear momentum is that the particle it describes is equally
ωψ ω2ψ 2
likely to be found travelling in the positive and negative x direc-  1 1

∫ (c1ψ 1 + c2ψ 2 ) (c1 Ωˆψ 1 + c2 Ωˆψ 2 )dτ
= 
tions, with the same magnitude, kħ, of the momentum.
A similar interpretation applies to any wavefunction writ- ∫ (c1ψ 1 + c2ψ 2 ) (c1ωψ
= 1 1 + c2ω2ψ 2 )dτ

ten as a linear combination of eigenfunctions of an operator. In  1   1 


general, a wavefunction can be written as the following linear = c1 c1ω1 ∫ψ 1ψ 1dτ + c2 c2ω2 ∫ψ 2ψ 2dτ
combination  0   0 

Linear combination + c1 c2ω2 ∫ψ 1 ψ 2dτ + c2 c1ω1 ∫ψ 2ψ 1dτ


   

  c1 1  c2 2    ck k of eigenfunctions


 (7C.10)
k The first two integrals on the right are both equal to 1 because
where the ck are numerical (possibly complex) coefficients the wavefunctions ψ1 and ψ2 are individually normalized.
and the ψk are different eigenfunctions of the operator Ω̂ cor- Because ψ1 and ψ2 correspond to different eigenvalues of a her-
responding to the observable of interest. The functions ψk are mitian operator, they are orthogonal, so the third and fourth
said to form a complete set in the sense that any arbitrary func- integrals on the right are zero. Therefore
tion can be expressed as a linear combination of them. Then, Ω 〉 | c1 |2ω1 + | c2 |2ω2
〈=
according to quantum mechanics:
The interpretation of this expression is that in a series of meas-
• A single measurement of the observable corresponding urements each individual measurement yields either ω1 or ω2,
to the operator Ω̂ will give one of the eigenvalues corre- but that the probability of ω1 occurring is |c1|2, and likewise the
sponding to the ψk that contribute to the superposition. probability of ω2 occurring is |c2|2. The average is the sum of
• In a series of measurements, the probability of obtaining a spe- the two eigenvalues, but with each weighted according to the
cific eigenvalue is proportional to the square modulus (|ck|2) of probability that it will occur in a measurement:
the corresponding coefficient in the linear combination. average  (probability of 1 occurring)  1
The average value of a large number of measurements of an  (probability of 2 occurring) 2
observable Ω is called the expectation value of the operator The expectation value therefore predicts the result of taking a
Ω̂ , and is written 〈Ω〉. For a normalized wavefunction ψ, the series of measurements, each of which gives an eigenvalue, and
expectation value of Ω̂ is calculated by evaluating the i­ ntegral then taking the weighted average of these values. This conclu-
sion justifies the form of eqn 7C.11.
〈 Ω 〉 =∫ψ  Ωˆψ dτ
Expectation value [normalized
wavefunction, definition]
 (7C.11)

This definition can be justified by considering two cases, one


Example 7C.3 Calculating an expectation value
where the wavefunction is an eigenfunction of the operator Ω̂
and another where the wavefunction is a superposition of that Calculate the average value of the position of an electron in the
operator’s eigenfunctions. lowest energy state of a one-dimensional box of length L, with
the (normalized) wavefunction ψ = (2/L)1/2 sin(πx/L) inside the
How is that done? 7C.4 Justifying the expression for the box and zero outside it.
expectation value of an operator Collect your thoughts The average value of the position is the
If the wavefunction ψ is an eigenfunction of Ω̂ with eigenvalue expectation value of the operator corresponding to position,
ω (so Ωψˆ = ωψ ), which is multiplication by x. To evaluate 〈x〉, you need to evalu-
ate the integral in eqn 7C.11 with Ωˆ = x̂= x ×.
ω a constant ψ normalized
ωψ The solution The expectation value of position is
〈Ω〉 =∫ ψ Ωˆψ dτ = ∫ψ ⋆ωψ dτ = ω ∫ψ ⋆ψ dτ = ω

1/2
L 2 πx
〈 x〉 = ∫0 ψ xˆψ dx with ψ =  L  sin L and xˆ = x ×

The interpretation of this expression is that, because the wave-


function is an eigenfunction of Ω̂ , each observation of the The integral is restricted to the region x = 0 to x = L because
property Ω results in the same value ω; the average value of all outside this region the wavefunction is zero. Use Integral T.11
the observations is therefore ω. from the Resources section to obtain
7C Operators and observables 259

Integral T.11 This conclusion is an example of the consequences of the


2 L πx 2 L2 1 Heisenberg uncertainty principle, one of the most celebrated
〈x〉 = ∫ x sin 2 d x = × = L
L 0 L L 4 2 results of quantum mechanics:

This result means that if a very large number of measurements It is impossible to specify simultaneously, with arbitrary
of the position of the electron are made, then the mean value precision, both the linear momentum and the position of
will be at the centre of the box. a particle.

Self-test 7C.3 Evaluate the mean square position, 〈x2〉, of the Note that the uncertainty principle also implies that if the po-
electron; you will need Integral T.12 from the Resource section. sition is known precisely, then the momentum cannot be pre-

 21 2  0.217 L2 3
1
 Answer: L dicted. The argument runs as follows.
Suppose the particle is known to be at a definite location,
2

then its wavefunction must be large there and zero every-


The mean kinetic energy of a particle in one dimension is the where else (Fig. 7C.4). Such a wavefunction can be created by
expectation value of the operator given in eqn 7C.5. Therefore, superimposing a large number of harmonic (sine and cosine)
­functions, or, equivalently, a number of eikx functions (because
∞ 2 ∞  d 2ψ
〈 Ek 〉 = ∫ ψ  Eˆ kψ dx = −
2m ∫−∞
ψ dx  (7C.12) eikx = cos kx + i sin kx). In other words, a sharply localized wave-
−∞ dx 2 function, called a wavepacket, can be created by forming a
This conclusion confirms the previous assertion that the kinetic linear combination of wavefunctions that correspond to many
energy is related to the average of the curvature of the wave- different linear momenta.
function: a large contribution to the observed value comes from The superposition of a few harmonic functions gives a wave-
regions where the wavefunction is sharply curved (so d2ψ/dx2 function that spreads over a range of locations (Fig. 7C.5).
is large) and the wavefunction itself is large (so that ψ⋆ is large However, as the number of wavefunctions in the superposition
there too). increases, the wavepacket becomes sharper on account of the
more complete interference between the positive and nega-
tive regions of the individual waves. When an infinite number
Exercises of components are used, the wavepacket is a sharp, infinitely
narrow spike, which corresponds to perfect localization of the
E7C.4 An electron in a region of length L is described by the normalized
wavefunction ψ(x) = (2/L)1/2sin(2πx/L) in the range x = 0 to x = L; outside this particle. Now the particle is perfectly localized but all informa-
range the wavefunction is zero. Evaluate 〈x3〉. The necessary integrals will be tion about its momentum has been lost. A measurement of the
found in the Resource section. momentum will give a result corresponding to any one of the
E7C.5 An electron in a one-dimensional region of length L is described by infinite number of waves in the superposition, and which one it
the normalized wavefunction ψ(x) = (2/L)1/2sin(2πx/L) in the range x = 0 to will give is unpredictable. Hence, if the location of the particle
x = L; outside this range the wavefunction is zero. The expectation value of the
is known precisely (implying that its wavefunction is a super-
momentum of the electron is found from eqn 7C.11, which in this case is
position of an infinite number of momentum eigenfunctions),
2 L
sin(2πx /L) ˆ then its momentum is completely unpredictable.
L ∫0
=
〈 px 〉 px sin(2πx /L)dx
The quantitative version of the uncertainty principle is
Evaluate the differential and then the integral, and hence find 〈px〉. The
necessary integrals will be found in the Resource section. pq q  12  Heisenberg uncertainty principle  (7C.13a)

7C.3 The uncertainty principle


Wavefunction, ψ

The wavefunction ψ = eikx is an eigenfunction of pˆ x with ei-


genvalue +kħ: in this case the wavefunction describes a parti-
cle with a definite state of linear momentum. Where, though,
is the particle? The probability density is proportional to
ψ⋆ψ, so if the particle is described by the wavefunction eikx Location
of particle
the probability density is proportional to (eikx)⋆eikx = e−ikxeikx =
e−ikx + ikx = e0 = 1. In other words, the probability density is the
Position, x
same for all values of x: the location of the particle is com-
pletely unpredictable. In summary, if the momentum of the Figure 7C.4 The wavefunction of a particle at a well-defined
particle is known precisely, it is not possible to predict its location is a sharply spiked function that has zero amplitude
­location. everywhere except at the position of the particle.
260 7 Quantum theory

The solution The minimum uncertainty in position is



q 
2 2mv
Wavefunction, ψ

1.055  1034 Js
  5  10 26 m
5 2  (1.0  10 3 kg )  (1  10 6 ms 1 )
21 This uncertainty is completely negligible for all practical pur-
poses. However, if the mass is that of an electron, then the same
uncertainty in speed implies an uncertainty in position far
larger than the diameter of an atom (the analogous calculation
Position, x
gives Δq = 60 m).
Self-test 7C.4 Estimate the minimum uncertainty in the speed
Figure 7C.5 The wavefunction of a particle with an ill-
of an electron in a one-dimensional region of length 2a0, the
defined location can be regarded as a superposition of several
approximate diameter of a hydrogen atom, where a0 is the Bohr
wavefunctions of definite wavelength that interfere constructively
in one place but destructively elsewhere. As more waves are
radius, 52.9 pm.
Answer: 500 km s−1
used in the superposition (as given by the numbers attached to
the curves), the location becomes more precise at the expense
of uncertainty in the momentum of the particle. An infinite The Heisenberg uncertainty principle is more general than
number of waves are needed in the superposition to construct the even eqn 7C.13a suggests. It applies to any pair of observables,
wavefunction of the perfectly localized particle.
called complementary observables, for which the correspond-
ing operators Ωˆ 1 and Ωˆ 2 have the property
In this expression Δpq is the ‘uncertainty’ in the linear momen- Ωˆ 1Ωˆ 2ψ ≠ Ωˆ 2 Ωˆ 1ψ Complementarity of observables (7C.14)
tum parallel to the axis q, and Δq is the uncertainty in position
along that axis. These ‘uncertainties’ are given by the root- The term on the left implies that Ωˆ 2 acts first, then Ωˆ 1 acts on the
mean-square deviations of the observables from their mean result, and the term on the right implies that the operations are
values: performed in the opposite order. When the effect of two operators
applied in succession depends on their order (as this equation im-
pq  ( pq2    pq  2 )1/ 2 q  (q 2   q 2 )1/ 2  (7C.13b) plies), they do not commute. The different outcomes of the effect
of applying Ωˆ 1 and Ωˆ 2 in a different order are expressed by intro-
If there is complete certainty about the position of the particle
ducing the commutator of the two operators, which is defined as
(Δq = 0), then the only way that eqn 7C.13a can be satisfied
is for Δpq = ∞, which implies complete uncertainty about the
[Ωˆ 1, Ωˆ 2] = Ωˆ 1Ωˆ 2− Ωˆ 2 Ωˆ 1 Commutator [definition] (7C.15)
momentum. Conversely, if the momentum parallel to an axis is
known exactly (Δpq = 0), then the position along that axis must By using the definitions of the operators for position and
be completely uncertain (Δq = ∞). ­momentum, an explicit value of this commutator can be found.
The p and q that appear in eqn 7C.13a refer to the same di-
rection in space. Therefore, whereas simultaneous specification
How is that done? 7C.5 Evaluating the commutator of
of the position on the x-axis and momentum parallel to the
x-axis are restricted by the uncertainty relation, simultaneous position and momentum
location of position on x and motion parallel to y or z are not You need to consider the effect of xp ˆˆ x (that is, the effect of pˆ x
restricted. followed by the effect on the outcome of multiplication by x) on
an arbitrary wavefunction ψ, which need not be an eigenfunc-
tion of either operator.
Example 7C.4 Using the uncertainty principle  dψ
ˆˆ xψ = x ×
xp
Suppose the speed of a projectile of mass 1.0 g is known to i dx
within 1 μm s−1. What is the minimum uncertainty in its Then you need to consider the effect of pˆ x xˆ on the same func-
­position? tion (that is, the effect of multiplication by x followed by the
effect of pˆ x on the outcome):
Collect your thoughts You can estimate Δp from mΔv, where
Δv is the uncertainty in the speed; then use eqn 7C.13a to esti- d( fg )/d x = (d f/d x) g + f (dg/dx)
mate the minimum uncertainty in position, Δq, by using it in
the form pq  12  rearranged into q  12 /p . You will need d(xψ ) dψ
pˆ x xˆψ = = ψ +x
to use 1 J = 1 kg m2 s−2. i dx i dx
7C Operators and observables 261

The second expression is different from the first, so pˆ x xˆψ ≠ xp


ˆˆ xψ The postulates of quantum
7C.4
and therefore the two operators do not commute. You can infer
the value of the commutator from the difference of the two mechanics
expressions:
The principles of quantum theory can be summarized as a se-

[ xˆ ,pˆ x ]ψ =
ˆˆ xψ − pˆ x xˆψ =
xp iψ , so [ xˆ ,pˆ x ]ψ =
− ψ = iψ ries of postulates, which will form the basis for chemical appli-
i
cations of quantum mechanics throughout the text.
This relation is true for any wavefunction ψ, so the commuta- The wavefunction: All dynamical information is contained
tor is in the wavefunction ψ for the system, which is a mathematical
function found by solving the appropriate Schrödinger equa-
(7C.16)
[xˆ , pˆ x ] = i Commutator of position
tion for the system.
and momentum operators The Born interpretation: If the wavefunction of a particle has
the value ψ at some position r, then the probability of find-
The commutator in eqn 7C.16 is of such central significance ing the particle in an infinitesimal volume dτ = dxdydz at that
in quantum mechanics that it is taken as a fundamental distinc- position is proportional to |ψ(r)|2dτ.
tion between classical mechanics and quantum mechanics. In Acceptable wavefunctions: An acceptable wavefunction must
fact, this commutator may be taken as a postulate of quantum be single-valued, continuous, not infinite over a finite region
mechanics. Classical mechanics supposed, falsely as is now of space, and (except in special cases) have a continuous slope.
known, that the position and momentum of a particle could Observables: Observables, Ω, are represented by hermitian
be specified simultaneously with arbitrary precision. However, operators, Ω̂ , built from the position and momentum opera-
quantum mechanics shows that position and momentum are tors specified in eqn 7C.3.
complementary, and that a choice must be made: position can Observations and expectation values: A single measurement
be specified, but at the expense of momentum, or momentum of the observable represented by the operator Ω̂ gives one of
can be specified, but at the expense of position. the eigenvalues of Ω̂ . If the wavefunction is not an eigenfunc-
tion of Ω̂ , the average of many measurements is given by the
Exercises expectation value, 〈Ω〉, defined in eqn 7C.11.
The Heisenberg uncertainty principle: It is impossible to spec-
E7C.6 Calculate the minimum uncertainty in the speed of a ball of mass 500 g
that is known to be within 1.0 μm of a certain point on a bat. What is the
ify simultaneously, with arbitrary precision, both the linear
minimum uncertainty in the position of a bullet of mass 5.0 g that is known to momentum and the position of a particle and, more generally,
have a speed somewhere between 350.000 01 m s−1 and 350.000 00 m s−1? any pair of observables represented by operators that do not
−1
E7C.7 The speed of a certain proton is 0.45 Mm s . If the uncertainty in commute.
its momentum is to be reduced to 0.0100 per cent, what uncertainty in its
location must be tolerated?

Checklist of concepts
☐ 1. The Schrödinger equation is an eigenvalue equation. ☐ 7. Observables are represented by hermitian operators.
☐ 2. An operator carries out a mathematical operation ☐ 8. Orthonormal functions are sets of functions that are
on a function, in general transforming it into a new both normalized and mutually orthogonal.
­function. ☐ 9. When the system is not described by a single eigen-
☐ 3. The hamiltonian operator is the operator correspond- function of an operator, it may be expressed as a
ing to the total energy of the system, the sum of the ­superposition of such eigenfunctions.
kinetic and potential energies. ☐ 10. The mean value of a series of observations is given by
☐ 4. The wavefunction corresponding to a specific energy is the expectation value of the corresponding operator.
an eigenfunction of the hamiltonian operator. ☐ 11. The uncertainty principle restricts the precision with
☐ 5. Two different functions are orthogonal if the integral which complementary observables may be specified and
(over all space) of their product is zero. measured simultaneously.
☐ 6. Hermitian operators have real eigenvalues and orthog- ☐ 12. Complementary observables are observables for which
onal eigenfunctions. the corresponding operators do not commute.
262 7 Quantum theory

Checklist of equations
Property Equation Comment Equation number
Eigenvalue equation ˆ = ωψ
Ωψ ψ eigenfunction; ω eigenvalue 7C.2b

{ ∫ψ }

Hermiticity ∫ψ i

Ωˆψ j dτ = 
j
ˆ dτ
Ωψ i
Hermitian operators have real eigenvalues and orthogonal 7C.7
eigenfunctions
Orthogonality   d  0 for i ≠ j Integration over all space 7C.8

i j

Expectation value ˆ dτ
〈 Ω 〉 =∫ψ Ωψ 
Definition; assumes ψ normalized 7C.11

Heisenberg uncertainty principle pq q  12  For position and momentum 7C.13a

Commutator of two operators [Ωˆ1,=


Ωˆ 2] Ωˆ1 Ωˆ 2− Ωˆ 2 Ωˆ1 The observables are complementary if [Ωˆ1, Ωˆ 2] ≠ 0 7C.15

Special case: [xˆ ,pˆ x ] = i 7C.16


TOPIC 7D Translational motion

How is that done? 7D.1 Finding the solutions to the


➤ Why do you need to know this material?
Schrödinger equation for a free particle in one dimension
The application of quantum theory to translational motion
reveals the origin of quantization and non-classical fea- The general solution of a second-order differential equation of
tures, such as tunnelling and zero-point energy. This mate- the kind in eqn 7D.1 is
rial is important for the discussion of atoms and molecules
 k (x )  Aeikx  Be  ikx
that are free to move within a restricted volume, such as a
gas in a container. where k, A, and B are constants. You can verify that ψk(x) is a
solution of eqn 7D.1 by substituting it into the left-hand side of
➤ What is the key idea? the equation, evaluating the derivatives, and then confirming
The translational energy of a particle confined to a finite that you have generated the right-hand side. Because de±ax/dx =
region of space is quantized, and under certain conditions ±ae±ax, the left-hand side becomes
particles can pass into and through classically forbidden
ψk (x)
regions. 2
d2 2
− 2 ( Ae
ikx
+ Be − ikx ) = − { A(ik )2 eikx + B(−ik )2 e − ikx }
2m dx 2m
➤ What do you need to know already?
Ek
You should know that the wavefunction is the solution ψk (x)
of the Schrödinger equation (Topic 7B). In one instance k 2
2
= Aeikx + Be − ikx )
you need to be familiar with the techniques of deriving 2m
dynamical properties from the wavefunction by using the
operators corresponding to the observables (Topic 7C). The left-hand side is therefore equal to a constant × ψk(x),
which is the same as the term on the right-hand side of
eqn 7D.1 provided the constant, the boxed term, is identified
with E. The value of the energy depends on the value of k, so
Translation, motion through space, is one of the basic types of henceforth it will be written Ek. The wavefunctions and ener-
motion. Quantum mechanics, however, shows that translation gies of a free particle are therefore
can have several non-classical features, such as its confinement
to discrete energies and passage into and through classically
forbidden regions. k2 2 (7D.2)
ψ k (x ) = A ikx
+ Be − ikx Ek =
2m Wavefunctions and
energies
[one dimension]

7D.1 Free motion in one dimension The wavefunctions in eqn 7D.2 are continuous, have con-
tinuous slope everywhere, are single-valued, and do not go to
A free particle is unconstrained by any potential, which infinity: they are therefore acceptable wavefunctions for all val-
may be taken to be zero everywhere. In one dimension ues of k. Because k can take any value, the energy can take any
V(x) = 0 everywhere, so the Schrödinger equation becomes non-negative value, including zero. As a result, the translational
(Topic 7B) energy of a free particle is not quantized.
As seen in Topic 7C in general a wavefunction can be writ-
2 d 2 (x ) Free motion 
  E (x ) [one dimension]
(7D.1) ten as a superposition (a linear combination) of the eigenfunc-
2m dx 2 tions of an operator. The wavefunctions of eqn 7D.2 can be
The most straightforward way to solve this simple second-­ recognized as superpositions of the two functions e±ikx which
order differential equation is to take the known general form of are eigenfunctions of the linear momentum operator with ei-
solutions of equations of this kind, and then show that it does genvalues ±kħ (Topic 7C). These eigenfunctions correspond to
indeed satisfy eqn 7D.1. states with definite linear momentum:
264 7 Quantum theory

 k (x )  Ae  ikx  Be  ikx  
 
Particle with linear Particle with linear
momentum  k momentumk

Potential energy, V
According to the interpretation given in Topic 7C, if a system
is described by the wavefunction ψk(x), then repeated meas-
urements of the momentum will give +kħ (that is, the particle
travelling in the positive x-direction) with a probability propor-
tional to A2, and −kħ (that is, the particle travelling in the nega-
tive x-direction) with a probability proportional to B2. Only if A
or B is zero does the particle have a definite momentum of −kħ
or +kħ, respectively.
0 Location, x L
Brief illustration 7D.1 Figure 7D.1 The potential energy for a particle in a one-
dimensional box. The potential is zero between x = 0 and
Suppose an electron emerges from an accelerator moving x = L, and then rises to infinity outside this region, resulting in
towards positive x with kinetic energy 1.0 eV (1 eV = 1.602 × impenetrable walls which confine the particle.
10−19 J). The wavefunction for such a particle is given by
eqn 7D.2 with B = 0 because the momentum is definitely in the
positive x-direction. The value of k is found by rearranging the
but rises abruptly to infinity at the walls located at x = 0 and
expression for the energy in eqn 7D.2 into
x = L (Fig. 7D.1). When the particle is between the walls, the
1/ 2
 2  (9.109  1031 kg )  (1.6  1019 J) 
1/ 2 Schrödinger equation is the same as for a free particle (eqn
 2m E 
k   e2 k    7D.1), so the general solutions given in eqn 7D.2 are also the
    (1.055  1034 Js)2
  same. However, it will prove convenient to rewrite the wave-
 5.1  109 m 1 function in terms of sines and cosines by using e±ikx = cos kx ±
i sin kx (The chemist’s toolkit 7B.1)
or 5.1 nm−1 (with 1 nm = 10−9 m). Therefore, the wavefunction
is ψ(x) = Ae5.1ix/nm.  k (x )  Aeikx  Be  ikx
 A(cos kx  i sin kx )  B(cos kx  i sin kx )
So far, the motion of the particle has been confined to the  ( A  B)cos kx  i( A  B)sin kx
x-axis. In general, the linear momentum is a vector (see Energy:
A first look) directed along the line of travel of the particle. Then The constants i(A − B) and A + B can be denoted C and D, re-
p = kħ, the magnitude of the vector is p = kħ, and its component spectively, in which case
on each axis is pq = kqħ, with q = x, y, or z. The wavefunction
ik q
for each component is proportional to e q and overall equal to  k (x )  C sin kx  D cos kx (7D.3)
i( k x + k y + k z )
e x y z .1 Outside the box the wavefunctions must be zero as the particle
will not be found in a region where its potential energy would
Exercises be infinite:

E7D.1 Evaluate the linear momentum and kinetic energy of a free electron For x  0 and x  L, k (x )  0 (7D.4)
described by the wavefunction eikx with k = 3 nm−1.
E7D.2 Write the wavefunction for a particle of mass 2.0 g travelling to the left
with kinetic energy 20 J. 7D.2(a) The acceptable solutions
One of the requirements placed on a wavefunction is that it
must be continuous. It follows that since the wavefunction is
Confined motion in one
7D.2 zero when x < 0 (where the potential energy is infinite) the
wavefunction must be zero at x = 0 which is the point where
dimension the potential energy rises to infinity. Likewise, the wavefunc-
tion is zero where x > L and so must be zero at x = L where
Consider a particle in a box in which a particle of mass m is
the potential energy also rises to infinity. These two restric-
confined to a region of one-dimensional space between two
tions are the boundary conditions, or constraints on the
impenetrable walls. The potential energy is zero inside the box
function:

1
In terms of scalar products, this overall wavefunction would be written eik·r.  k (0)  0 and  k (L)  0 Boundary conditions (7D.5)
7D Translational motion 265

Now it is necessary to show that the requirement that the wave- n


function must satisfy these boundary conditions implies that 100 10
Classically allowed energies
only certain wavefunctions are acceptable, and that as a conse-

Energy, E/E1; E1 = h2/8mL2


81 9
quence only certain energies are allowed.
64 8

49 7
How is that done? 7D.2 Showing that the boundary
36 6
conditions lead to quantized levels
25 5
You need to start from the general solution and then explore 16 4
9 3
the consequences of imposing the boundary conditions. 0
41
1
2

Step 1 Apply the boundary conditions


Figure 7D.2 The energy levels for a particle in a box. Note that the
At x = 0, ψk (0) = C sin 0 + D cos 0 = D (because sin 0 = 0 and energy levels increase as n2, and that their separation increases as
cos 0 = 1). One boundary condition is ψk(0) = 0, so it follows the quantum number increases. Classically, the particle is allowed
that D = 0. to have any value of the energy shown as a tinted area.
At x = L, ψk(L) = C sin kL. The boundary condition ψk(L) = 0
therefore requires that sin kL = 0, which in turn requires that The fact that n is restricted to positive integer values im-
kL = nπ with n = 1, 2,…. Although n = 0 also satisfies the plies that the energy of the particle in a one-dimensional box is
boundary condition it is ruled out because the wavefunction quantized. This quantization arises from the boundary condi-
would be C sin 0 = 0 for all values of x, and the particle would tions that ψ must satisfy. This conclusion is general:
be found nowhere. Negative integral values of n also satisfy the
boundary condition, but simply result in a change of sign of the The need to satisfy boundary conditions implies that only
wavefunction (because sin(−θ) = −sin θ). It therefore follows certain wavefunctions are acceptable, and hence restricts
that the wavefunctions that satisfy the two boundary condi- the eigenvalues to discrete values.
tions are ψk(x) = C sin(nπx/L) with n = 1, 2,… and k = nπ/L.
The integer n that has been used to label the wavefunctions
Step 2 Normalize the wavefunctions and energies is an example of a ‘quantum number’. In general, a
To normalize the wavefunction, write it as N sin(nπx/L) and quantum number is an integer (in some cases, Topic 8B, a half-
require that the integral of the square of the wavefunction over integer) that labels the state of the system. For a particle in a
all space is equal to 1. The wavefunction is zero outside the one-dimensional box there are an infinite number of acceptable
range 0 ≤ x ≤ L, so the integration needs to be carried out only solutions, and the quantum number n specifies the one of inter-
inside this range: est (Fig. 7D.2).2 As well as acting as a label, a quantum number
can often be used to calculate the value of a property, such as
Integral T.2 the energy corresponding to the state, as in eqn 7D.6.
1/2
L L nπx L 2
∫ψ d x = N 2 ∫ sin 2
2
d x = N 2 × = 1, so N =
0 0 L 2 L 7D.2(b) The properties of the wavefunctions
Figure 7D.3 shows some of the wavefunctions of a particle in a
Step 3 Identify the allowed energies
one-dimensional box. The points to note are as follows.
According to eqn 7D.2, Ek = k2ħ2/2m, but because k is limited to
the values k = nπ/L with n = 1, 2,… the energies are restricted • The wavefunctions are all sine functions with the same
to the values maximum amplitude but different wavelengths; the wave-
length gets shorter as n increases.
k 2 2 (nπ / L )2 (h /2 π)2 n 2h 2 • Shortening the wavelength results in a sharper average
Ek = = =
2m 2m 8mL2 curvature of the wavefunction and therefore an increase
in the kinetic energy of the particle (recall that, as V = 0
At this stage it is sensible to replace the label k by the label inside the box, the energy is entirely kinetic).
n, and to label the wavefunctions and energies as ψn(x)
• The number of nodes (the points where the wavefunction
and En. The normalized wavefunctions and energies are
passes through zero) also increases as n increases; the
­therefore
wavefunction ψn has n − 1 nodes.
1/2
2 nπx (7D. 6)
ψ n (x ) = sin 2
L L Particle in a one- You might object that the wavefunctions have a discontinuous slope at the
dimensional box edges of the box, and so do not qualify as acceptable according to the criteria in
n 2h 2 , Topic 7B. This is a rare instance where the requirement does not apply, because
En = n = 1, , … the potential energy suddenly jumps to an infinite value.
8mL2
266 7 Quantum theory

54 3 2 1 Integral T.2
  L /2
L /2 2 L /2
2  x  2 x 1  2x  
P    1 dx   sin   dx   
2
sin 
0 L 0  L  L  2 4 /L  L   0
0
2L 1  1
Wavefunction, ψ

   sin    2
L  4 4 /L 

0 x L
The result should not be a surprise, because the probability
density is symmetrical around x = L/2. The probability of find-
ing the particle between x = 0 and x = L/2 must therefore be
half of the probability of finding the particle between x = 0 and
x = L, which is 1.

Figure 7D.3 The first five normalized wavefunctions of a particle


As n increases the separation between the maxima in the
in a box. As the energy increases the wavelength decreases,
probability density ψ n2 (x ) decreases (Fig. 7D.5). For a mac-
and successive functions possess one more half wave. The
wavefunctions are zero outside the box. roscopic object this separation is very much smaller than the
dimensions of the object. Therefore, when averaged over the
dimensions of the object, the probability density becomes con-
stant. This behaviour of the probability density at high quantum
n=1 numbers reflects the classical result that a particle bouncing be-
tween the walls of a container spends equal times at all points.
(a) n=2 This conclusion is an example of the correspondence principle,
n=2 which states that as high quantum numbers are reached, the
n=1 classical result emerges from quantum mechanics.

(b)
7D.2(c) The properties of the energy
n=2
The linear momentum of a particle in a box is not well de-
n=1
fined because the wavefunction sin kx is not an eigenfunction
(c) of the linear momentum operator. However, because sin kx =
Figure 7D.4 (a) The first two wavefunctions for a particle in (eikx − e−ikx)/2i,
a box, (b) the corresponding probability densities, and (c) a 1/ 2 1/ 2
2  nx  1  2  inx /L
representation of the probability density in terms of the darkness  n (x )    sin      (e  e  inx /L ) (7D.8)
of shading. Interact with the dynamic version of this graph in the L  L  2i  L 
e-book. It follows that, if repeated measurements are made of the linear
momentum, half will give the value +nπħ/L and half will give
The probability density for a particle in a one-dimensional box is the value −nπħ/L. This conclusion is the quantum mechanical

2  nx 
 n2 (x )  sin2   (7D.7)
L  L 

and varies with position. The non-uniformity in the probability


|ψ(x)|2

density is pronounced when n is small (Fig. 7D.4). The maxima


in the probability density give the locations at which the parti-
cle has the greatest probability of being found.

Brief illustration 7D.2 0


x/L
1

As explained in Topic 7B, the total probability of finding the Figure 7D.5 The probability density ψ2(x) for a large quantum
particle in a specified region is the integral of ψ(x)2dx over that number (here n = 50 compared with n = 1). For high n the
region. Therefore, the probability of finding the particle with probability density is nearly uniform when averaged over several
n = 1 in a region between x = 0 and x = L/2 is oscillations.
7D Translational motion 267

version of the classical picture in which the particle bounces ­ ne-dimensional box, with electrons having energies given
o
back and forth in the box, spending equal times travelling to by eqn 7D.6. If each CC bond length is taken to be 140 pm,
the left and to the right. the length of the molecular box in β-carotene is L = 2.94 nm.
Because n cannot be zero, the lowest energy that the particle Estimate the wavelength of the light absorbed by this molecule
may possess is not zero (as allowed by classical mechanics, cor- when it undergoes a transition from its ground state to the
responding to a stationary particle) but next higher excited state.

h2
E1 = Zero-point energy  (7D.9)
8mL2
This lowest, irremovable energy is called the zero-point ­energy.
The physical origin of the zero-point energy can be explained
1 -Carotene
in two ways:
• The Heisenberg uncertainty principle states that px x  12 . Collect your thoughts Recall that each C atom in the π sys-
For a particle confined to a box, Δx has a finite value, there- tem contributes one p electron to the π-orbitals and two
fore Δpx cannot be zero, as that would violate the uncer- electrons occupy each of the energy levels. Use eqn 7D.10
tainty principle. Therefore the kinetic energy cannot be to calculate the energy separation between the highest
zero. occupied and the lowest unoccupied levels, and convert that
• If the wavefunction is to be zero at the walls, but smooth, energy to a wavelength by using the Bohr frequency condi-
continuous, and not zero everywhere, then it must be tion (eqn 7A.9, ΔE = hn).
curved, and curvature in a wavefunction implies the pos- The solution There are 22 C atoms in the π system, so in the
session of kinetic energy. ground state each of the levels up to n = 11 is occupied by two
electrons. The first excited state is one in which an electron is
Brief illustration 7D.3 promoted from n = 11 to n = 12. The energy difference between
the ground and first excited state is therefore
The lowest energy of an electron in a region of length 100 nm
is given by eqn 7D.6 with n = 1: E  E12  E11
(6.626  1034 Js)2
12  (6.626  1034 Js)2  (2  11  1)
E1   6.02  1024 J 8  (9.109  1031 kg )  (2.94  109 m)2
8  (9.109  1031 kg )  (100  109 m)2
 1.60 1019 J
where 1 J = 1 kg m2 s−2 has been used. The energy E1 can be
expressed as 6.02 yJ (1 yJ = 10−24 J). or 0.160 aJ. It follows from the Bohr frequency condition
(ΔE = hn) that the frequency of radiation required to cause this
transition is
The separation between adjacent energy levels with quantum
19
numbers n and n + 1 is E 1.60 10 J
  34
 2.42  1014 s 1
h 6.626  10 Js
(n  1)2 h2 n2 h2 h2
En 1  En    (2n  1)  (7D.10)
8mL 2
8mL 2
8mL2 or 242 THz (1 THz = 1012 Hz), corresponding to a wavelength
  c/  1240 nm.
This separation decreases as the length of the container in-
creases, and is very small when the container has macro- Comment. The experimental value is 603 THz (λ = 497 nm),
scopic dimensions. The separation of adjacent levels becomes corresponding to radiation in the visible range of the elec-
tromagnetic spectrum. The model is too crude to expect
zero when the walls are infinitely far apart. Atoms and mol-
quantitative agreement, but the calculation at least predicts a
ecules free to move in normal laboratory-sized vessels may
wavelength in the right general range.
therefore be treated as though their translational energy is
not quantized. Self-test 7D.1 Estimate a typical nuclear excitation energy in
electronvolts (1 eV = 1.602 × 10−19 J) by calculating the first
Example 7D.1 excitation energy of a proton confined to a one-dimensional
Estimating an absorption wavelength
box with a length equal to the diameter of a nucleus (approxi-
β-Carotene (1) is a linear polyene in which 10 single and mately 1 × 10−15 m, or 1 fm).
11 double bonds alternate along a chain of 22 carbon Answer: 0.6 GeV, where 1 GeV = 109 eV
atoms. The delocalized π system can be approximated as a
268 7 Quantum theory

Exercises 7D.3(a) Energy levels and wavefunctions


E7D.3 Calculate the energy separations in joules, kilojoules per mole, The procedure for finding the allowed wavefunctions and en-
electronvolts, and reciprocal centimetres between the levels (i) n = 2 and ergies involves starting with the two-dimensional Schrödinger
n = 1, (ii) n = 6 and n = 5 of an electron in a box of length 1.0 nm. equation, and then applying the ‘separation of variables’ tech-
E7D.4 Calculate the probability that a particle will be found between 0.49L nique to turn it into two separate one-dimensional equations.
and 0.51L in a box of length L for (i) ψ1, (ii) ψ2. You may assume that the
wavefunction is constant in this range, so the probability is ψ2δx.
How is that done? 7D.3 Constructing the wavefunctions
E7D.5 For a particle in a box of length L sketch the wavefunction
corresponding to the state with the lowest energy and on the same graph for a particle in a two-dimensional box
sketch the corresponding probability density. Without evaluating any
integrals, explain why the expectation value of x is equal to L/2. The ‘separation of variables’ technique, which is explained and
used here, is used in several cases in quantum mechanics.
Step 1 Apply the separation of variables technique
Confined motion in two and
7D.3 First, recognize the presence of two operators, each of which
more dimensions acts on functions of only x or y:

2 ∂ 2 2 ∂ 2
Now consider a rectangular two-dimensional region, be- Hˆ x =
− 2
Hˆ y =

2m ∂x 2m ∂y 2
tween 0 and L1 along x, and between 0 and L2 along y. Inside
this region the potential energy is zero, but at the edges it Equation 7D.11, which is
rises to infinity (Fig. 7D.6). As in the one-dimensional case,
the wave-function can be expected to be zero at the edges of Hx ˆ Hˆ
 
  y

this region (at x = 0 and L1, and at y = 0 and L2), and to be zero
2 ∂ 2 2 ∂ 2
outside the region. Inside the region the particle has contri- − ψ− ψ=

2m ∂x 2 2m ∂y 2
butions to its kinetic energy from its motion along both the
x and y directions, and so the Schrödinger equation has two then becomes
kinetic energy terms, one for each axis. For a particle of mass
m the equation is Hˆ xψ + Hˆ yψ =

2   2  2  Now suppose that the wavefunction ψ can be expressed as the


     E  (7D.11)
product of two functions, ψ(x,y) = X(x)Y(y), one depending
2m  x 2 y 2 
only on x and the other depending only on y. This assump-
Equation 7D.11 is a partial differential equation, and the re- tion is the central step of the procedure, and does not work
sulting wavefunctions are functions of both x and y, denoted for all partial differential equations: that it works here must be
ψ(x,y). demonstrated. With this substitution the preceding equation
becomes

Hˆ x X (x )Y ( y ) + Hˆ y X (x )Y ( y ) =
EX ( x )Y ( y )
V
Then, because Ĥx operates on (takes the second derivatives
with respect to x of) X(x), and likewise for Ĥy and Y(y), this
equation is the same as

Y ( y )Hˆ x X (x ) + X (x )Hˆ yY ( y ) =
EX (x )Y ( y )
Particle 0
confined L2 Division of both sides by X(x)Y(y) then gives
to surface y Depends only on x only on y
   Depends
  
 A constant
1 ˆ 1 ˆ 
L1
H x X (x ) + H yY ( y ) =E
x X (x ) Y ( y)

If x is varied, only the first term can change; but the other two
Figure 7D.6 A two-dimensional rectangular well. The potential terms do not change, so the first term must be a constant for
goes to infinity at x = 0 and x = L1, and y = 0 and y = L2, but in the equality to remain true. The same is true of the second
between these values the potential is zero. The particle is confined term when y is varied. Therefore, denoting these constants as
to this rectangle by impenetrable walls. EX and EY ,
7D Translational motion 269

1 ˆ ˆ the most probable locations correspond to sin2(πx/L) = 1


= H x X (x ) E=
X , so H x X ( x ) E X X (x ) and sin2(2πy/L) = 1, or (x,y) = (L/2,L/4) and (L/2,3L/4).
X (x )
1 ˆ The least probable locations (the nodes, where the wave-
= H yY ( y ) E= ˆ
Y , so H yY ( y ) EY Y ( y ) function passes through zero) correspond to zeroes in the
Y ( y)
probability density within the box, which occur along the
line y = L/2.
with EX + EY = E. The procedure has successfully separated
the partial differential equation into two ordinary differential
equations, one in x and the other in y.
Step 2 Recognize the two ordinary differential equations There is one other two-dimensional case that can be solved
Each of the two equations is identical to the Schrödinger exactly (but is not separable into x- and y-solutions): a particle
equation for a particle in a one-dimensional box, one for the confined to an equilateral triangular area. The solutions are set
coordinate x and the other for the coordinate y. The boundary out in A deeper look 7D.1, available to read in the e-book ac-
conditions are also essentially the same (that the wavefunction companying this text.
must be zero at the walls). Consequently, the two solutions are A three-dimensional box can be treated in the same way as
a planar rectangular region: the wavefunctions are products of
1/ 2
2  n x  n12h2 three terms and the energy is a sum of three terms. As before,
Xn1 (x )    sin  1  E X, n1 
 L1   L1  8mL21 each term is analogous to that for the one-dimensional case.
1/ 2 Overall, therefore,
 2   n y  n22h2
Yn2 ( y )    sin  2  EY, n2 
 L2   L2  8mL22 1/ 2
 8   n1 x   n2 y   n3 z 
 n1, n2 , n3 (x ,y ,z )    sin   sin   sin  
L L L
 1 2 3  L1   L2   L3 
with each of n1 and n2 taking the values 1, 2,… independently.
Wavefunctions [three dimensions] (7D.13a)
Step 3 Assemble the complete wavefunction

Inside the box, which is when 0 ≤ x ≤ L1 and 0 ≤ y ≤ L2, the
wavefunction is the product Xn1 (x )Yn2 ( y ), and is given by eqn
7D.12a below. Inside the material of the walls, the wavefunc-
tion is zero. The energies are the sum E X, n1 + EY, n2 . The two
quantum numbers take the values n1 = 1, 2,… and n2 = 1, 2, …
independently. Overall, therefore,
+ + –
(7D.12a)
2 n1πx n πy
ψ n1, n2 ( x , y ) = sin 2 Wavefunctions
( L1 L2 )1/2 L1 L2
[two dimensions]

(7D.12b)
n12 n22 h 2
En1, n2 = + Energy levels
L21 L22 8m [two dimensions] (a) (b)

Some of the wavefunctions are plotted as contours in


Fig. 7D.7. They are the two-dimensional versions of the wave-
functions shown in Fig. 7D.3. Whereas in one dimension the – – +
wavefunctions resemble states of a vibrating string with ends
fixed, in two dimensions the wavefunctions correspond to vi-
brations of a rectangular plate with fixed edges.
+ + –

Brief illustration 7D.4


(c) (d)
Consider an electron confined to a square cavity of side L (that
is L1 = L2 = L), and in the state with quantum numbers n1 = 1, Figure 7D.7 The wavefunctions for a particle confined to a
n2 = 2. Because the probability density is rectangular surface depicted as contours of equal amplitude.
(a) n1 = 1, n2 = 1, the state of lowest energy; (b) n1 = 2, n2 = 1;
4  x   2y  (c) n1 = 1, n2 = 2; (d) n1 = 2, n2 = 2. Interact with the dynamic version
 12,2 (x ,y )  2
sin2   sin2  
L  L   L  of this graph in the e-book.
270 7 Quantum theory

for 0 ≤ x ≤ L1, 0 ≤ y ≤ L2, 0 ≤ z ≤ L3

 n2 n2 n2  h 2 –
Energy levels
En1, n2 , n3   12  22  23  [threee dimensions]
 (7D.13b)
 L1 L2 L3  8m
+ –
The quantum numbers n1, n2, and n3 are all positive integers
1, 2,… that can be chosen independently. The system has a +
zero-point energy, the value of E1,1,1.

(a) (b)
7D.3(b) Degeneracy
Figure 7D.8 Two of the wavefunctions for a particle confined to
A special feature of the solutions arises when a two-­dimensional a geometrically square well: (a) n1 = 1, n2 = 2; (b) n1 = 2, n2 = 1. The
box is not merely rectangular but square, with L1 = L2 = L. Then two functions correspond to the same energy and are said to be
the wavefunctions and their energies are degenerate. Note that one wavefunction can be converted into
the other by rotation of the plane by 90°: degeneracy is always a
2  n x   n y  consequence of symmetry.
 n1, n2 (x ,y )  sin  1  sin  2  Waavefunctions
L  L   L  [square] (7D.14a)
for 0  x  L,0  y  L
 n1, n2 (x ,y )  0 outside box Brief illustration 7D.5

h2 Energy levels The energy of a particle in a two-dimensional square box of
En1, n2  (n12  n22 ) [square]
 (7D.14b) side L in the energy level with n1 = 1, n2 = 7 is
8mL2
h2 50h2
Consider the cases n1 = 1, n2 = 2 and n1 = 2, n2 = 1: E1,7  (12  72 ) 
8mL 8mL2
2

2  x   2y  h2 5h2 The level with n1 = 7 and n2 = 1 has the same energy. Thus,
 1,2  sin   sin  E1,2  (12  22 ) 
L  L   L  8mL2 8mL2 at first sight the energy level 50h2/8mL2 is doubly degenerate.
However, in certain systems there may be levels that are not
2  2x   y  h2 5h2 apparently related by symmetry but have the same energy
 2,1  sin  sin E2,1  (22  12 ) 
L  L   L  8mL 8mL2
2 and are said to be ‘accidentally’ degenerate. Such is the case
here, for the level with n1 = 5 and n2 = 5 also has energy
50h2/8mL2. The level is therefore actually three-fold degener-
Although the wavefunctions are different, they correspond to
ate. Accidental degeneracy is also encountered in the hydrogen
the same energy. The technical term for different wavefunc-
atom (Topic 8A) and can always be traced to a ‘hidden’ sym-
tions corresponding to the same energy is degeneracy, and in
metry, one that is not immediately obvious.
this case energy level 5h2/8mL2 is ‘doubly degenerate’. In gen-
eral, if N wavefunctions correspond to the same energy, then
that level is ‘N-fold degenerate’.
The occurrence of degeneracy is related to the symmetry of
the system. Figure 7D.8 shows contour diagrams of the two
degenerate functions ψ1,2 and ψ2,1. Because the box is square, Exercises
one wavefunction can be converted into the other simply by E7D.6 An electron is confined to a square well of length L. What would be the
rotating the plane by 90°. Interconversion by rotation through length of the box such that the zero-point energy of the electron is equal to
its rest mass energy, mec2? Express your answer in terms of the parameter
90° is not possible when the plane is not square, and ψ1,2 and λC = h/mec, the ‘Compton wavelength’ of the electron.
ψ2,1 are then not degenerate. Similar arguments account for
E7D.7 For a particle in a square box of side L, at what position (or
the degeneracy of the energy levels of a particle in a cubic box. positions) is the probability density a maximum if the wavefunction has
Other examples of degeneracy occur in quantum mechanical n1 = 2, n2 = 2? Also, describe the position of any node or nodes in the
systems (for instance, in the hydrogen atom, Topic 8A), and wavefunction.
all of them can be traced to the symmetry properties of the E7D.8 Consider a particle in a cubic box. What is the degeneracy of the level
system. that has an energy three times that of the lowest level?
7D Translational motion 271

7D.4 Tunnelling as can be verified by substituting this solution into the left-hand side
of eqn 7D.16. The important feature to note is that the two exponen-
A new quantum-mechanical feature appears when the potential tials in eqn 7D.17 are now real functions, as distinct from the com-
energy does not rise abruptly to infinity at the walls (Fig. 7D.9). plex, oscillating functions for the region where V = 0. To the right of
Consider the case in which there are two regions where the po- the barrier (x > W), where V = 0 again, the wavefunctions are
tential energy is zero separated by a barrier where it rises to a Wavefunction
  Aeikx k  (2mE )1/ 2  (7D.18)
finite value, V0. Suppose the energy of the particle is less than to right of barrier
V0. A particle arriving from the left of the barrier has an oscillat-
Note that to the right of the barrier, the particle can be mov-
ing wavefunction but inside the barrier the wavefunction decays
ing only to the right and therefore only the term eikx contributes
rather than oscillates (this feature is justified below). Provided
as it corresponds to a particle with positive linear momentum
the barrier is not too wide the wavefunction emerges to the
(moving to the right).
right, but with reduced amplitude; it then continues to oscillate
The complete wavefunction for a particle incident from the
once it is back in a region where it has zero potential energy. As a
left consists of (Fig. 7D.10):
result of this behaviour the particle has a non-zero probability of
passing through the barrier. That is forbidden classically because • an incident wave (Aeikx corresponds to positive linear
a particle cannot have a potential energy that exceeds its total momentum);
energy. The ability of a particle to penetrate into, and possibly • a wave reflected from the barrier (Be−ikx corresponds to
pass through, a classically forbidden region is called tunnelling. negative linear momentum, motion to the left);
The Schrödinger equation can be used to calculate the prob-
• the exponentially changing amplitude inside the barrier
ability of tunnelling of a particle of mass m incident from the
(eqn 7D.17);
left on a rectangular potential energy barrier of width W. On
the left of the barrier (x < 0) the wavefunctions are those of a • an oscillating wave (eqn 7D.18) representing the propaga-
particle with V = 0, so from eqn 7D.2, tion of the particle to the right after tunnelling through
the barrier successfully.
Wavefunction
  Aeikx  Be  ikx k  (2mE )1/ 2 to left of barrier  (7D.15) The probability that a particle is travelling towards positive x
The Schrödinger equation for the region representing the bar- (to the right) on the left of the barrier (x < 0) is proportional
rier (0 ≤ x ≤ W), where the potential energy has the constant to |A|2, and the probability that it is travelling to the right after
value V0, is passing through the barrier (x > W) is proportional to |A′|2.
The ratio of these two probabilities, |A′|2/|A|2, which expresses
2 d 2 (x ) the probability of the particle tunnelling through the barrier, is
  V0 (x )  E (x ) (7D.16)
2m dx 2 called the transmission probability, T.
Provided E < V0 the general solutions of eqn 7D.16 are

  Ce x  De  x    {2m(V0  E )}1/2 Incident


Wavefunction inside barrier (7D.17) wave
Wavefunction, ψ

V0
Wavefunction, ψ

Transmitted
wave

Reflected
wave
E
x

0 W x Figure 7D.10 When a particle is incident on a barrier from the


left, the wavefunction consists of a wave representing linear
Figure 7D.9 The wavefunction for a particle encountering a momentum to the right, a reflected component representing
potential barrier. Provided that the barrier is neither too wide nor momentum to the left, a varying but not oscillating component
too tall, the wavefunction will be non-zero as it exits to the right. inside the barrier, and a (weak) wave representing motion to the
Interact with the dynamic version of this graph in the e-book. right on the far side of the barrier.
272 7 Quantum theory

0.5 1

Transmission probability, T
0.4 0.8
Wavefunction, ψ

0.3 0.6
2 10
0.2 0.4

0.1 0.2
4

x 10
0 0
0 0.2 0.4 0.6
0.8 1 1 2 3 4
Figure 7D.11 The wavefunction and its slope must be continuous (a) Incident energy, E/V0 (b) Incident energy, E/V0
at the edges of the barrier. The conditions for continuity enable the
wavefunctions at the junctions of the three zones to be connected Figure 7D.12 The transmission probabilities T for passage
and hence relations between the coefficients that appear in the through a rectangular potential barrier. The horizontal axis is
solutions of the Schrödinger equation to be obtained. the energy of the incident particle expressed as a multiple of
the barrier height. The curves are labelled with the value of
W(2mV0)1/2/ħ. (a) E < V0; (b) E > V0.
The values of the coefficients A, B, C, and D are found by ap-
plying the usual criteria of acceptability to the wavefunction.
Because an acceptable wavefunction must be continuous at the For high, wide barriers (in the sense that κW >> 1), eqn
edges of the barrier (at x = 0 and x = W) 7D.20a simplifies to

at x  0 : A  B  C  D Rectangular potential
 (7D.19a) T  16 (1   )e 2W barrier;  W >> 1  (7D.20b)
at x  W : Ce W
 De  W
 Ae ikW

The transmission probability decreases exponentially with the


Their slopes (their first derivatives) must also be continuous at thickness of the barrier and with m1/2 (because κ ∝ m1/2). It fol-
these positions (Fig. 7D.11): lows that particles of low mass are more able to tunnel through
barriers than heavy ones (Fig. 7D.13). Tunnelling is very im-
at x  0 : ikA  ikB   C   D portant for electrons and muons (mμ ≈ 207me), and moderately
 (7D.19b)
at x  W :  CeW   De W  ikAeikW important for protons (mp ≈ 1840me); for heavier particles it is
less important.
After lengthy algebraic manipulations of these four equations, A number of effects in chemistry depend on the ability of
the transmission probability turns out to be the proton to tunnel more readily than the deuteron. The very
1 rapid equilibration of proton transfer reactions is also a mani-
 (eW  e W )2  Transmission probability
T  1    (7D.20a) festation of the ability of protons to tunnel through barriers and
16 (1   )  [reectangular barrier]
 transfer quickly from an acid to a base. Tunnelling of protons
between acidic and basic groups is also an important feature of
where ε = E/V0. This function is plotted in Fig. 7D.12. The trans- the mechanism of some enzyme-catalysed reactions.
mission probability for E > V0 is shown there too. The transmis-
sion probability has the following properties:
• T ≈ 0 for E << V0: there is negligible tunnelling when the
energy of the particle is much lower than the height of the Light particle
Wavefunction, ψ

barrier;
• T increases as E approaches V0: the probability of tunnel-
ling increases as the energy of the particle rises to match Heavy particle
the height of the barrier;
• T approaches 1 for E > V0, but the fact that it does not
immediately reach 1 means that there is a probability of
the particle being reflected by the barrier even though x
according to classical mechanics it can pass over it;
Figure 7D.13 The wavefunction of a heavy particle decays more
• T ≈ 1 for E >> V0, as expected classically: the barrier is rapidly inside a barrier than that of a light particle. Consequently,
invisible to the particle when its energy is much higher a light particle has a greater probability of tunnelling through the
than the barrier. barrier.
7D Translational motion 273

Brief illustration 7D.6

Potential energy, V
Suppose that a proton of an acidic hydrogen atom is con-
fined to an acid that can be represented by a barrier of
height 2.000 eV and length 100 pm. The probability that a
proton with energy 1.995 eV (corresponding to 0.3195 aJ)
can escape from the acid is computed using eqn 7D.20a,
with ε = E/V0 = 1.995 eV/2.000 eV = 0.9975 and V0 − E = 0 L
Location, x
0.005 eV (corresponding to 8.0 × 10−22 J). The quantity κ is
given by eqn 7D.17: Figure 7D.14 A potential well with a finite depth.

{2  (1.67  1027 kg )  (8.0  1022 J)}1/ 2



1.055  1034 Js
 1.54  1010 m 1

It follows that

Potential energy,V
Wavefunction,ψ
n=2

W  (1.54 1010 m 1 )  (100  1012 m)  1.54

Equation 7D.20a then yields n=1


1
 (e1.54  e 1.54 )2 
T  1   0 Location, x L
 16  0.9975  (1  0.9975) 
 1.97  103 Figure 7D.15 Wavefunctions of the lowest two bound levels for a
particle in the potential well shown in Fig. 7D.14.

A problem related to tunnelling is that of a particle in a for the problem shows that the number of bound states is equal
square-well potential of finite depth (Fig. 7D.14). Inside the to N, with
well the potential energy is zero and the wavefunctions os- (8mV0 )1/ 2 L
cillate as they do for a particle in an infinitely deep box. At N 1   N (7D.21)
h
the edges, the potential energy rises to a finite value V0. If
where V0 is the depth of the well and L is its width. This re-
E < V0 the wavefunction decays as it penetrates into the walls,
lation shows that the deeper and wider the well, the greater
just as it does when it enters a barrier. The wavefunctions are
found by ensuring, as in the discussion of the potential bar- the number of bound states. As the depth becomes infinite,
so the ­number of bound states also becomes infinite, as for the
rier, that they and their slopes are continuous at the edges of
particle in a box treated earlier in this Topic.
the potential. The two lowest energy solutions are shown in
Fig. 7D.15.
For a potential well of finite depth, there are a finite number Exercise
of wavefunctions with energy less than V0: they are referred to E7D.9 Suppose that the junction between two semiconductors can be
as bound states, in the sense that the particle is mainly confined represented by a barrier of height 2.0 eV and length 100 pm. Calculate the
to the well. Detailed consideration of the Schrödinger equation transmission probability of an electron with energy 1.5 eV.

Checklist of concepts
☐ 1. The translational energy of a free particle is not quantized. ☐ 4. A particle in a box possesses a zero-point energy, an
☐ 2. The need to satisfy boundary conditions implies that irremovable minimum energy.
only certain wavefunctions are acceptable and restricts ☐ 5. The correspondence principle states that the quantum
observables, specifically the energy, to discrete values. mechanical result with high quantum numbers should
☐ 3. A quantum number is an integer (in certain cases, a agree with the predictions of classical mechanics.
half-integer) that labels the state of the system.
274 7 Quantum theory

☐ 6. The wavefunction for a particle in a two- or three- ☐ 9. The occurrence of degeneracy is a consequence of the
dimensional box is the product of wavefunctions for the symmetry of the system.
particle in a one-dimensional box. ☐ 10. Tunnelling is penetration into or through a classically
☐ 7. The energy of a particle in a two- or three-dimensional forbidden region.
box is the sum of the energies for the particle in two or ☐ 11. The probability of tunnelling decreases with an increase
three one-dimensional boxes. in the height and width of the potential barrier.
☐ 8. Energy levels are N-fold degenerate if N wavefunctions ☐ 12. Light particles are more able to tunnel through barriers
correspond to the same energy. than heavy ones.

Checklist of equations
Property Equation Comment Equation number
Free particle: All values of k allowed 7D.2
Wavefunctions ψk = Aeikx + Be−ikx
Energies Ek = k2ħ2/2m
Particle in a box
One dimension:
Wavefunctions ψn(x) = (2/L)1/2 sin(nπx/L), 0 ≤ x ≤ L n = 1, 2,… 7D.6
ψn(x) = 0, x < 0 and x > L
Energies En = n2h2/8mL2
Two dimensions*:
Wavefunctions  n1 ,n2 (x ,y )  Xn1 (x )Yn2 ( y ) n1, n2 = 1, 2,… 7D.12a

Xn1 (x )  (2/L1 )1/ 2 sin(n1x /L1 ), 0  x  L1

Yn2 ( y )  (2/L2 )1/ 2 sin(n2 y /L2 ), 0  y  L2


Energies En1, n2  (n12 /L21  n22 /L22 )h2 /8m 7D.12b
κW −κW 2 −1
Transmission probability T = {1 + (e −e ) /16ε(1 − ε)} Rectangular potential barrier 7D.20a
−2κW
T ≈ 16ε(1 − ε)e High, wide rectangular barrier 7D.20b

* Solutions for three dimensions can be written analogously.


TOPIC 7E Vibrational motion

f­ requency of the oscillator is ω = 2πn (units: radians per ­second).


➤ Why do you need to know this material? It follows that the angular frequency of a classical harmonic os-
cillator is ω = (kf/m)1/2.
Molecular vibration plays a role in the interpretation
The potential energy V is related to force by F = −dV/dx (see
of thermodynamic properties, such as heat capacities
Energy: A first look), so the potential energy corresponding to a
(Topics 2A and 13E), and of the rates of chemical reactions
Hooke’s law restoring force is
(Topic 18C). The measurement and interpretation of the
vibrational frequencies of molecules is the basis of infrared V (x ) = 12 kf x 2 Parabolic potential energy  (7E.3)
spectroscopy (Topics 11C and 11D).
This form of potential energy is called a ‘harmonic potential
➤ What is the key idea? energy’ or a ‘parabolic potential energy’ (Fig. 7E.1). As the par-
ticle moves away from the equilibrium position its potential
The energy of vibrational motion is quantized, with the
energy increases and so its kinetic energy, and hence its speed,
separation of energy levels large for strong restoring forces
decreases. At some point all the energy is potential and the par-
and light masses.
ticle comes to rest at a turning point. The particle then acceler-
➤ What do you need to know already? ates back towards and through the equilibrium position. The
greatest probability of finding the particle is where it is moving
You should know how to formulate the Schrödinger equa-
most slowly, which is close to the turning points.
tion for a given potential energy. You should also be
The turning point, xtp, of a classical oscillator occurs when its
familiar with the concepts of tunnelling (Topic 7D) and the
potential energy 12 kf x 2 is equal to its total energy, so
expectation value of an observable (Topic 7C).
1/ 2
 2E  Turning point
x tp     [classical harmonic oscillaator] (7E.4)
 kf 
Atoms in molecules and solids vibrate around their e­ quilibrium The turning point increases with the total energy: in classical
positions as bonds stretch, compress, and bend. As seen in Energy: terms, the amplitude of the swing of a pendulum or the dis-
A first look, the simplest model for this kind of motion is the ‘har- placement of a mass on a spring increases.
monic oscillator’, which is considered in detail in this Topic. The Schrödinger equation for mass m experiencing a har-
monic potential energy is

7E.1 The harmonic oscillator


 
In classical mechanics a harmonic oscillator is a particle of
mass m that experiences a ‘Hooke’s law’ restoring force propor-
tional to its displacement, x, from the equilibrium position. For
Potential energy, V

a one-dimensional system,
Fx  kf x Hooke’s law  (7E.1)

where kf is the force constant, which characterizes the strength


of the restoring force and is expressed in newtons per metre
(N m−1). As seen in Energy: A first look, the displacement varies
with time as
0
1/ 2 Displacement, x
1 k 
x(t )  A sin 2 t    f   (7E.2) Figure 7E.1 The potential energy for a harmonic oscillator is the
2  m 
parabolic function V ( x ) = 21 k f x 2 , where x is the displacement from
That is, the position of the particle oscillates harmonically (as equilibrium. The larger the force constant kf the steeper the curve
a sine function) with frequency n (units: Hz). The a­ngular and narrower the curve becomes.
276 7 Quantum theory


Schrodinger The physical reason for the existence of this zero-point energy
2 d 2 (x ) 1 equation is the same as for the particle in a box (Topic 7D). The particle is
  2 kf x 2 (x )  E (x ) [harm
monic
 (7E.5)
2m dx 2 confined, so its position is not completely uncertain. It follows
oscillator]
that its momentum, and hence its kinetic energy, cannot be
The potential energy becomes infinite at x = ±∞, and so the zero. A classical interpretation of the zero-point energy is that
wavefunction is zero at these limits. However, as the potential the quantum oscillator is never completely at rest and therefore
energy rises smoothly rather than abruptly to infinity, as it does has kinetic energy; moreover, because its motion samples the
for a particle in a box, the wavefunction decreases smoothly potential energy away from the equilibrium position, it also has
towards zero rather than becoming zero abruptly. The bound- non-zero potential energy.
ary conditions ψ(±∞) = 0 imply that only some solutions of the The model of a particle oscillating in a parabolic potential is
Schrödinger equation are acceptable, and therefore that the en- used to describe the vibrational motion of a diatomic molecule
ergy of the oscillator is quantized. A–B (and, with elaboration, Topic 11D, polyatomic molecules).
In this case both atoms move as the bond between them is
stretched and compressed and the mass m is replaced by the
7E.1(a) The energy levels effective mass, μ, given by
Equation 7E.5 is a standard form of differential equation and mA mB Effective mass
its solutions are well known to mathematicians.1 The energies  [diatomic molecule]
 (7E.9)
mA  mB
permitted by the boundary conditions are
When A is much heavier than B, mB can be neglected in the
Ev   v  12     (kf /m)1/2 v  0, 1, 2, denominator and the effective mass is μ ≈ mB, the mass of
Energy levels (7E.6) the lighter atom. In this case, only the light atom moves and the
heavy atom acts as a stationary anchor.
where v is the vibrational quantum number. Note that the
energies depend on ω, which has the same dependence on the
mass and the force constant as the angular frequency of a clas-
Brief illustration 7E.1
sical oscillator (eqn 7E.2) and is high when the force constant
is large and the mass small. The separation of adjacent levels is The effective mass of 1H35Cl is
Ev 1  Ev   (7E.7) mHmCl 1.0078mu  34.9688mu
   0.9796mu
for all v. The energy levels therefore form a uniform ladder with mH  mCl 1.0078mu  34.9688mu
spacing ħω (Fig. 7E.2). The energy separation ħω is negligibly which is close to the mass of the hydrogen atom. The force con-
small for macroscopic objects (with large mass) but significant stant of the bond is kf = 516.3 N m−1. It follows from eqn 7E.6
for objects with mass similar to that of an atom. and 1 N = 1 kg m s−2, with μ in place of m, that
The energy of the lowest level, with v = 0, is not zero: 1/ 2
1/ 2
k   516.3 N m 1 
E0   1
Zero -point energy  (7E.8)   f  
2
  0.9796  (1.66054  1027 kg ) 
 
 5.634  1014 s 1
7
Potential energy or (after division by 2π) 89.67 THz. The separation of adjacent
6
levels (eqn 7E.7) is
Allowed energies, Ev

5
4 Ev 1  Ev    (1.054 57  1034 J s)  (5.634  1014 s 1 )
hω 3  5.941  1020 J
2 or 59.41 zJ, about 0.37 eV. This energy separation corresponds
1 to 36 kJ mol−1, which is chemically significant. The zero-point
v =0 energy (eqn 7E.8) of this molecular oscillator is 29.71 zJ, which
corresponds to 0.19 eV, or 18 kJ mol−1.
0
Displacement, x

Figure 7E.2 The energy levels of a harmonic oscillator are evenly


spaced with separation ħω, where ω = (kf/m)1/2. Even in its lowest 7E.1(b) The wavefunctions
energy state, an oscillator has an energy greater than zero. The acceptable solutions of eqn 7E.5, all have the form
1
For details, see our Molecular quantum mechanics, Oxford University
 (x )  N  (polynomial in x )
Press, Oxford (2011).  (bell-shaped Gaussian function)
7E Vibrational motion 277

and probability density, ψ2


0.8

Wavefunction, ψ,
0.6
exp(–x2)

ψ
ψ2
0.4

0.2

0
–2 –1 0 1 2 –4 –2 0 2 4
x Displacement, y = x/α

Figure 7E.3 The graph of the Gaussian function, f ( x )  e  x .


2
Figure 7E.4 The normalized wavefunction and probability density
(shown also by shading) for the lowest energy state of a harmonic
oscillator.
where N is a normalization constant. A Gaussian function is
2
a bell-shaped function of the form e −x (Fig. 7E.3). The precise and the corresponding probability density is
form of the wavefunctions is
2 2
Ground- state
/ 2
probability density 
1/ 4
 y2 /2 x  2   02 (x )  N 02 e  y  N 02 e  x (7E.13b)
v (x )  N v H v ( y )e , y  ,    [harmonic oscillator]
  mkf 
Wavefunctions  (7E.10) The wavefunction and the probability density are shown in
Fig. 7E.4. The probability density has its maximum value at
The factor Hv(y) is a Hermite polynomial; the form of these x = 0, the equilibrium position, but is spread about this posi-
polynomials are listed in Table 7E.1. Note that the first few tion. The curvature is consistent with the kinetic energy being
Hermite polynomials are rather simple: for instance, H0(y) = 1 non-zero; the spread is consistent with the potential energy
and H1(y) = 2y. Hermite polynomials, which are members of a also being non-zero, so jointly resulting in a zero-point energy.
class of functions called ‘orthogonal polynomials’, have a wide The wavefunction for the first excited state, v = 1, is
range of important properties which allow a number of quan-
tum mechanical calculations to be done with relative ease. For 2 2 2 2
 1 (x )  N1 2 ye  y /2
 N1   xe  x / 2
example, a useful ‘recursion’ relation is  
H v 1  2 yH v  2vH v 1  0 (7E.11) First excited- state
wavvefunction  (7E.14)
and an important integral is [harmonic oscillator]

 0 if v   v This function has a node at zero displacement (x = 0), and the

2
H v  H v e  y d y   1/ 2 v  (7E.12)

 2 v ! if v   v probability density has maxima at x = ±α (Fig. 7E.5).
The wavefunction for the ground state, which has v = 0, is
2 2
/ 2 2 Ground- state
 0 (x )  N 0 e y /2
 N 0 e x wavefunction
 (7E.13a)
and probability density, ψ2

ψ
Wavefunction, ψ,

ψ2
Table 7E.1 The Hermite polynomials

v Hv(y)
0 1
1 2y
2 4y2 − 2
3 8y3 − 12y –4 –2 0 2 4
Displacement, y = x/α
4 2
4 16y − 48y + 12
Figure 7E.5 The normalized wavefunction and probability density
5 32y5 − 160y3 + 120y
6 4 2
(shown also by shading) for the first excited state of a harmonic
6 64y − 480y + 720y –120
oscillator.
278 7 Quantum theory

Example 7E.1 The shapes of several of the wavefunctions are shown in


Confirming that a wavefunction is a
Fig. 7E.6 and the corresponding probability densities are
solution of the Schrödinger equation
shown in Fig. 7E.7. These probability densities show that, as the
Confirm that the ground-state wavefunction (eqn 7E.11a) is a quantum number increases, the positions of highest probability
solution of the Schrödinger equation (eqn 7E.5). migrate towards the classical turning points. This behaviour is
Collect your thoughts You need to substitute the wavefunction
another example of the correspondence principle (Topic 7D)
given in eqn 7E.13a into eqn 7E.5 and see that the left-hand in which at high quantum numbers the classical behaviour
side generates the right-hand side of the equation. Use the emerges from the quantum behaviour.
definition of α in eqn 7E.10.
Ev v
The solution First, evaluate the second derivative of the ground- 13
hω 6
state wavefunction by differentiating it twice in succession,
2

starting with 11
2 hω 5
d 2 2  x  2 2
N 0 e  x / 2   N 0  2  e  x / 2
dx   9
hω 4

Wavefunction, ψ
2

and then
7
hω 3
f g
2

d2 − x 2 /2α 2 d x 2 2

2 N 0e = − N0 e − x /2α 5
hω 2
dx dx α2
2

d(fg)/dx = fdg/dx + gdf/dx


3
2hω 1
2
N 0 − x 2 /2α 2 x 2
/2α 2 1

=− e + N0 e− x 2 0
α2 α2
= −(1/ α 2 )ψ 0 + ( x 2 / α 4 )ψ 0 –1 0 1
Displacement, y = x/α
Now substitute this expression and α2 = (ħ2/mkf )1/2 into the left-
hand side of eqn 7E.5, which first becomes Figure 7E.6 The normalized wavefunctions for the first seven
states of a harmonic oscillator. Note that the number of nodes is
2  1  2  x 2  equal to v. The wavefunctions with even v are symmetric about
       0  2 kf x  0  E 0
1 2

2m   2 
0
2m   4  y = 0, and those with odd v are anti-symmetric. The wavefunctions
are shown superimposed on the potential energy function,
and then
and the horizontal axis for each wavefunction is set at the
(ħ/2)(k f /m)1/2 kf /2 corresponding energy. The classical turning points occur where
1/2
these horizontal lines cross the potential energy curve. Interact
2 2
mkf mkf with the dynamic version of this graph in the e-book.
ψ0 − x 2ψ 0 + 12 kf x 2ψ 0 = Eψ 0
2m 2
2m 2

Therefore (keeping track of the boxed terms) v

1/2
20
kf
ψ0 − 12 kf x 2ψ 0 + 12 kf x 2ψ 0 = Eψ 0
2 m
15
The boxed terms cancel, leaving
 10
1/ 2
  kf 
 0  E 0
2  m  5

It follows that ψ0 is a solution to the Schrödinger equation for


0
the harmonic oscillator with energy E  12 (kf /m)1/ 2  12  , in
–1 0 1
accord with eqn 7E.8 for the zero-point energy. Displacement, y = x/α
Self-test 7E.1 Confirm that the wavefunction in eqn 7E.13 is a
Figure 7E.7 The probability densities for the states of a harmonic
solution of eqn 7E.5 and evaluate its energy. oscillator with v = 0, 5, 10, 15, and 20. Note how the regions of
Answer: Yes, with E1  23  highest probability density move towards the turning points of
the classical motion as v increases.
7E Vibrational motion 279

The wavefunctions have the following features:


Exercises
• The Gaussian function decays quickly to zero as the dis-
E7E.1 Calculate the zero-point energy of a harmonic oscillator consisting of a
placement in either direction increases, so all the wave- particle of mass 2.33 × 10−26 kg and force constant 155 N m−1.
functions approach zero at large displacements: the particle −25
E7E.2 For a certain harmonic oscillator of effective mass 1.33 × 10 kg, the
is unlikely to be found at large displacements. difference in adjacent energy levels is 4.82 zJ. Calculate the force constant of
• The wavefunction oscillates between the classical turning the oscillator.
points but decays without oscillating outside them. E7E.3 Calculate the wavelength of the photon needed to excite a transition
2 2 1/2 between neighbouring energy levels of a harmonic oscillator of effective mass
• The exponent y is proportional to x × (mkf ) , so the equal to that of a proton (1.0078mu) and force constant 855 N m−1.
wavefunctions decay more rapidly for large masses and
strong restoring forces (stiff springs).
• As v increases, the Hermite polynomials become larger
at large displacements (as xv), so the wavefunctions grow Properties of the harmonic
7E.2
large before the Gaussian function damps them down to oscillator
zero: as a result, the wavefunctions spread over a wider
range as v increases (Fig. 7E.6). The average value of a property is calculated by evaluating the
expectation value of the corresponding operator (eqn 7C.11,
ˆ dτ for a normalized wavefunction). For a har-
〈 Ω 〉 = ∫ψ  Ωψ
monic oscillator,
Example 7E.2 Normalizing a harmonic oscillator ∞
ˆ dx 
〈 Ω 〉 v =∫ ψ v Ωψ (7E.16)
wavefunction −∞ v

Find the normalization constant for the harmonic oscillator When the explicit wavefunctions are substituted, the integrals
wavefunctions. might look fearsome, but the Hermite polynomials have many
Collect your thoughts A wavefunction is normalized (to 1) by features that simplify the calculation.
evaluating the integral of |ψ|2 over all space and then finding
the normalization factor from eqn 7B.3 (N  1/(  d )1/ 2).
7E.2(a) Mean values
The normalized wavefunction is then equal to Nψ. In this one-
dimensional problem, the volume element is dx and the inte- Equation 7E.16 can be used to calculate the mean displace-
gration is from −∞ to +∞. The wavefunctions are expressed ment, 〈x〉, and the mean square displacement, 〈x2〉, for a har-
in terms of the dimensionless variable y = x/α, so begin by monic oscillator in a state with quantum number v.
expressing the integral in terms of y by using dx = αdy. The
integral required is given by eqn 7E.12. How is that done? 7E.1 Calculating the mean values of x
The solution The unnormalized wavefunction is and x2 for a harmonic oscillator

v (x )  H v ( y )e  y
2
/2 The evaluation of the integrals needed to compute 〈x〉 and 〈x2〉
is simplified by recognizing the symmetry of the problem and
It follows from eqn 7E.12 that using the special properties of the Hermite polynomials.

  
Step 1 Use a symmetry argument to find the mean displacement
 vv dx    vv dy    H v2 ( y )e  y dy   1/ 2 2v v !
2

   The mean displacement 〈x〉 is expected to be zero because


the probability density of the oscillator is symmetrical about
where v! = v(v − 1)(v − 2)… 1 and 0! ≡ 1. Therefore, zero; that is, there is equal probability of positive and negative
displacements.
1/ 2
 1 
N v   1/ 2 v  Normalization constant  (7E.15) Step 2 Confirm the result by examining the necessary integral
   2 v!
More formally, the mean value of x, which is the expectation
Note that Nv is different for each value of v. value of x, is
〈x〉v = ∫ ψv⋆x ψv d x = Nv2 ∫ ( Hv e − y /2 )x ( Hv e − y /2 )dx
2 2

Self-test 7E.2 Confirm, by explicit evaluation of the integral, − −


that ψ0 and ψ1 are orthogonal. x = α y dx = α dy

and eqn 7E.13. An odd function


2
Answer: Show that   0 1dx  0 by using the information in Table 7E.1
y ( Hv e − y /2 )2 dy

2 2
=α N v


280 7 Quantum theory

The integrand is an odd function because when y → −y it The result for  x  v shows that the oscillator is equally likely
changes sign (the squared term does not change sign, but the to be found on either side of x = 0 (like a classical oscillator).
term y does). The integral of an odd function over a symmetri- The result for  x 2  v shows that the mean square displacement
cal range is necessarily zero, so increases with v. This increase is apparent from the probabil-
 x  v  0 for all v Mean displacement (7E.17a) ity densities in Fig. 7E.7, and corresponds to the amplitude of
a classical harmonic oscillator becoming greater as its energy
Step 3 Find the mean square displacement ­increases.
The mean square displacement, the expectation value of x2, is The mean potential energy of an oscillator, which is the ex-
pectation value of V = 12 kf x 2, can now be calculated:
x 2 v = N v2 ∫ ( Hv e − y /2 )x 2 ( Hv e − y /2 )dx
2 2

− 1/ 2
k 
x = αy dx = αdy V  v  12  kf x 2  v  12 kf  x 2  v  12  v  12    f 
m
= α 3 N 2v ∫ ( H ve − y /2 ) y 2 ( H ve − y /2 )dy
2 2


or
2
You can develop the factor y Hv by using the recursion relation V  v  12  v  12   Mean potential energy (7E.18a)
(eqn 7E.11) rearranged into yH v  vH v 1  12 H v 1. After multi-
plying this expression by y it becomes The total energy in the state with quantum number v is
2
y H v  v yH v 1  yH v 1 1
2
 v  12  , so it follows that
Now use the recursion relation (with v replaced by v − 1 or V  v  12 Ev Mean kinetic energy (7E.18b)
v + 1) again for both yHv−1 and yHv+1:
The total energy is the sum of the potential and kinetic ener-
yH v 1  (v  1)H v  2  21 H v gies, Ev  V  v   Ek  v , so the mean kinetic energy of the oscil-
yH v 1  (v  1)H v  12 H v  2 lator is
It follows that  Ek  v  Ev  V  v  Ev  12 Ev  12 Ev
Mean kinetic energy  (7E.18c)
y2Hv = vyHv−1 + 12 yHv+1 =v{(v − 1)Hv−2 + 12 Hv}
The result that the mean potential and kinetic energies of a
+ 12 {(v + 1)Hv + 12 Hv+2 } harmonic oscillator are equal (and therefore that both are
= v(v − 1)Hv−2 + ( v + 1
2 )H v + 14 Hv+2 equal to half the total energy) is a special case of the virial
theorem:
Substitution of this result into the integral gives If the potential energy of a particle has the form V = axb,
then its mean potential and kinetic energies are related by
y Hv 2

 
) v(v  1)H v  2   v  12  H v  14 H v  2 e  y / 2dy

 x 2  v   3 N v2   y2 /2 2
(H v e 2 Ek   bV  Virial theorem (7E.19)

0 

 For a harmonic oscillator b = 2, so  Ek  v  V  v. The virial
  3 N v2v(v  1)  H v H v  2e  y dy
2

 theorem is a short cut to the establishment of a number



 2 v!

1/ 2 v
0 
 of useful results, and it is used elsewhere (for example, in
  3 N v2  v  12   H v H v e  y dy  14  3 N v2  H v H v  2e  y dy
 2  2
Topic 8A).
 

  3 N v2  v  12  1/ 2 2v v !
7E.2(b) Tunnelling
Each of the three integrals is evaluated by making use of eqn
A quantum oscillator may be found at displacements with V > E,
7E.12. Therefore, after noting the expression for Nv in eqn 7E.15,
which are forbidden by classical physics because they correspond
Nv 2
  to negative kinetic energy. That is, a harmonic oscillator can tun-
1 nel into classically forbidden displacements. As shown in Example
 x  v    v  12   2 v ! 1/ 2 v   v  12  2
2 3 1 / 2 v

 2 v ! 7E.3, for the lowest energy state of the harmonic oscillator, there
is about an 8 per cent chance of finding the oscillator at classically
Finally, with,  2  (2 /mkf )1/ 2
forbidden displacements in either direction. As illustrated in the
following Example, these tunnelling probabilities are independ-
〈x 2〉v = ( v + 12 ) (7E.17b) ent of the force constant and mass of the oscillator (as explained
(mkf )1/2 Mean square displacement
in the Example).
7E Vibrational motion 281

Example 7E.3 The integral must be evaluated numerically (by using math-
Calculating the tunnelling probability for
ematical software), and is equal to 0.139…. It follows that
the harmonic oscillator
P = 0.079. That is, in 7.9 per cent of a large number of observa-
Calculate the probability that the ground-state harmonic oscil- tions of an oscillator in the state with quantum number v = 0,
lator will be found in a classically forbidden region. the particle will be found beyond the (positive) classical turn-
ing point. It will be found with the same probability at nega-
Collect your thoughts Find the expression for the classical turn-
tive forbidden displacements. The total probability of finding
ing point, xtp, where the kinetic energy goes to zero, by equating
the oscillator in a classically forbidden region when it is in its
the potential energy to the total energy of the harmonic oscilla-
ground state is about 16 per cent.
tor. You can then calculate the probability of finding the oscil-
lator at a displacement beyond xtp by integrating ψ2dx between Comment. Note that the result is independent of both kf and
xtp and infinity m. As seen in Topic 7D, tunnelling is most important for

particles of low mass, so how can this conclusion be justi-
P   v2dx fied physically? The answer lies in the fact that the turning
x tp
point comes closer to the equilibrium point as either the
By symmetry, the probability of the particle being found in the mass or the force constant increases. The amplitude of the
classically forbidden region from −xtp to −∞ is the same. wavefunction at xtp therefore becomes greater, and although
The solution The substitution of Ev into eqn 7E.4 for the classi- the tunnelling falls off more rapidly, it starts from a higher
cal turning point xtp gives value at xtp itself. The effects balance, and the net outcome
is independence of the penetration probability on mass and
 2E 
1/ 2 force constant.
x tp    v 
 kf  Self-test 7E.3 Calculate the probability that a harmonic oscil-
lator in the state with quantum number v = 1 will be found at
The variable of integration in the integral P is best expressed in a classically forbidden extension. You will need to use math-
terms of y = x/α with α = (ħ2/mkf )1/4. With these substitutions, ematical software to evaluate the integral.
and also using Ev   v  12   , the turning points are given by Answer: P = 0.056

ω = (kf /m)1/2
2( v + ) ω
1/2
x tp 1
2 The probability of finding the oscillator in classically
y tp = = = (2v +1)1/2
α α 2 kf forbidden regions decreases quickly with increasing v, and
vanishes entirely as v approaches infinity, as is expected
For the state of lowest energy (v = 0), ytp = 1 and the probability from the correspondence principle. Macroscopic oscillators
of being beyond that point is (such as pendulums) are in states with very high quantum
numbers, so the tunnelling probability is wholly negligible
dx = αdy and classical mechanics is reliable. Molecules, however, are
normally in their vibrational ground states, and for them
P = ∫ ψ 02dx = α ∫ ψ 02dy = α N 02 ∫ e − y dy
2

x tp 1 1 the probability is very significant and classical mechanics is


misleading.
with

20 = 1; 0!≡ 1 Exercises
1/2 1/2 E7E.4 Sketch the form of the wavefunctions for the harmonic oscillator with
1 1
N0 = = quantum numbers v = 0 and 1. Use a symmetry argument to explain why
α π 2 0!
1/2 0
α π1/2 these two wavefunctions are orthogonal (do not evaluate any integrals).
E7E.5 How many nodes are there in the wavefunction of a harmonic oscillator
Therefore with (i) v = 3; (ii) v = 4?
E7E.6 At what displacements is the probability density a maximum for a state
1   y2
1/ 2  1
P e dy of a harmonic oscillator with v = 1? (Express your answers in terms of the
coordinate y.)
282 7 Quantum theory

Checklist of concepts
☐ 1. The energy levels of a quantum mechanical harmonic ☐ 4. The probability of finding a quantum mechanical har-
oscillator are evenly spaced. monic oscillator at classically forbidden displacements
☐ 2. The wavefunctions of a quantum mechanical harmonic is significant for the ground vibrational state (v = 0) but
oscillator are products of a Hermite polynomial and a decreases quickly with increasing v.
Gaussian (bell-shaped) function.
☐ 3. A quantum mechanical harmonic oscillator has zero-
point energy, an irremovable minimum energy.

Checklist of equations
Property Equation Comment Equation number
Energy levels Ev   v  1
2     (kf /m) 1/ 2
v = 0, 1, 2,… 7E.6
Zero-point energy E0  1
2
7E.8
2
Wavefunctions  v (x )  N v H v ( y )e  y /2
v = 0, 1, 2,… 7E.10
y  x / ,   (2 /mkf )1/ 4
Normalization constant N v  (1/1/ 2 2v v !)1/ 2 7E.15
Mean displacement  x v  0 7E.17a
Mean square displacement  x 2  v   v  12  /(mkf )1/ 2 7E.17b
b
Virial theorem 2 Ek   bV  V = ax 7E.19
TOPIC 7F Rotational motion

The Schrödinger equation for motion around the ring is


➤ Why do you need to know this material?
Angular momentum is central to the description of the 2   2 2 
  2  2  (x ,y )  E (x ,y ) (7F.1)
electronic structure of atoms and molecules and the inter- 2m  x y 
pretation of molecular spectra.
The equation is best expressed in cylindrical coordinates r and
➤ What is the main idea? ϕ with z = 0 (The chemist’s toolkit 7F.1) because they reflect the
The energy, angular momentum, and orientation of the symmetry of the system. In cylindrical coordinates
angular momentum of a rotating body are quantized.
2 2 2 1  1 2
2 + 2 = 2 + + 2 2 (7F.2)
➤ What do you need to know already? x y r r r r ϕ
You should be aware of the postulates of quantum mechan-
ics and the role of boundary conditions (Topics 7C and 7D). However, because the radius of the path is fixed, the (boxed) de-
Background information on the description of rotation is rivatives with respect to r can be discarded. Only the last term in
found in Energy: A first look. eqn 7F.2 then survives and the Schrödinger equation becomes
2 d 2 ( )
  E ( ) (7F.3a)
2mr 2 d 2
The partial derivative has been replaced by a complete deriv-
Rotational motion is encountered in many aspects of chemis- ative because ϕ is now the only variable. The term mr2 is the
try, including the electronic structures of atoms, because elec-
trons orbit (in a quantum mechanical sense) around nuclei and The chemist’s toolkit 7F.1 Cylindrical coordinates
spin on their axis. Molecules also rotate; transitions between
their rotational states affect the appearance of spectra and their For systems with cylindrical symmetry it is best to work in
detection gives valuable information about the structures of cylindrical coordinates r, ϕ, and z (Sketch 1), with
molecules. x  r cos  y  r sin 
and where

0r  0    2    z  
7F.1 Rotation in two dimensions
The volume element is
Consider a particle of mass m constrained to move in a circular d  rdrd dz
path (a ‘ring’) of radius r in the xy-plane (Fig. 7F.1). The particle
For motion in a plane, z = 0 and the volume element is
might not be an actual body but could represent the centre of a
molecule or other object constrained to rotate in a plane, such d  rdrd
as the group of three hydrogen atoms in a methyl group freely
rotating about its axis.

z z

r
x r
m y y
x ϕ
Figure 7F.1 A particle on a ring is free to move in the xy-plane Sketch 1
around a circular path of radius r.
284 7 Quantum theory

moment of inertia, I = mr2, and so the Schrödinger equation Real


becomes Imaginary

Wavefunction, ψ
 d ψ (φ )
2 2
Schrödinger equation
− Eψ (φ )
= [particle on a ring]
 (7F.3b)
2 I dφ 2
0 Angle, ϕ

7F.1(a) The solutions of the Schrödinger


equation
(a)
The most straightforward way of finding the solutions of eqn ϕ
0
7F.3b is to take the known general solution to a second-order
differential equation of this kind and show that it does indeed

Wavefunction, ψ
satisfy the equation. Then find the allowed solutions and ener-
gies by imposing the relevant boundary conditions.
0 Angle, ϕ

How is that done? 7F.1 Finding the solutions of the


Schrödinger equation for a particle on a ring (b)
A solution of eqn 7F.3b is
Figure 7F.2 Two possible solutions of the Schrödinger equation
 ( )  eiml for a particle on a ring. The circumference has been opened out
where, as yet, ml is an arbitrary dimensionless number (the into a straight line; the points at ϕ = 0 and 2π are identical. The
notation is explained later). This solution is not the most gen- solution in (a), eiϕ = cos ϕ + i sin ϕ, is acceptable because after
a complete revolution the wavefunction has the same value.
eral, which would be  ( )  Aeiml  Be  iml , but is sufficiently
The solution in (b), e0.9iϕ = cos(0.9ϕ) + i sin(0.9ϕ) is unacceptable
general for the present purpose.
because its value, both for the real and imaginary parts, is not the
Step 1 Verify that the function satisfies the equation same at ϕ = 0 and 2π.
To verify that ψ(ϕ) is a solution note that
  Step 3 Normalize the wavefunction
d 2 iml d iml  2 iml  2 iml 
e  (iml )e  (iml ) e  ml e  ml2 A wavefunction is normalized by finding the normalization
d 2 d
 
1/ 2
constant N given by eqn 7B.3, N  1/   d . In this case,
Then the wavefunction depends only on the angle ϕ, the ‘volume
element’ is dϕ, and the range of integration is from ϕ = 0 to 2π,
d 2ψ
2 2
m2 2 so the normalization constant is
− 2 =− (−ml2ψ ) = l ψ
2 I dϕ 2 I 2I
1 1 1
N 1/ 2
 1/ 2

 2   2   iml iml  (22)1/ 2
which has the form constant × ψ, so the proposed wavefunction  0   d 

   0 e
e  d 
is indeed a solution and the corresponding energy is ml2 2 /2 I .  1 
Step 2 Impose the appropriate boundary conditions The normalized wavefunctions and corresponding energies
The requirement that a wavefunction must be single-valued are labelled with the integer ml, which is playing the role of a
implies the existence of a cyclic boundary condition, the quantum number:
requirement that the wavefunction must be the same after a
eimlϕ ,
complete revolution: ψ(ϕ + 2π) = ψ(ϕ) (Fig. 7F.2). In this case ψ ml (ϕ ) = (7F.4)
(2 π)1/2 Wavefunctions and energy
ml2 2 levels of a particle on a ring
 (  2)  eiml (  2 )  eiml e2 iml Eml = , ml = 0, ± ± 2, …
2I
2 iml i 2 ml
  ( )e   ( )(e )
Apart from the level with ml = 0, each of the energy levels is
As eiπ = −1, this relation is equivalent to doubly degenerate because the dependence of the energy on ml2
means that two values of ml (such as +1 and −1) correspond to
 (  2)  (1)2ml  ( )
the same energy.1
The cyclic boundary condition ψ(ϕ + 2π) = ψ(ϕ) requires
(1)2ml  1; this requirement is satisfied for any positive or 1
When quoting the value of ml, it is good practice always to give the sign,
negative integer value of ml, including 0. even if ml is positive. Thus, write ml = +1, not ml = 1.
7F Rotational motion 285

7F.1(b) Quantization of angular momentum The wavefunction is an eigenfunction of the angular momen-
tum, with the eigenvalue ml . In summary,
For a particle travelling on a circular path of radius r about the
z-axis, and therefore confined to the xy-plane, the angular mo- (7F.7)
mentum is represented by a vector that is perpendicular to the lˆzψ ml (ϕ ) = ml ψ ml (ϕ ) ml = 0, ±1, ± 2, …
Eigenfunctions of lˆz
plane. Its only component is
lz   pr  (7F.5)
Because ml is confined to discrete values, the z-component of
where p is the magnitude of the linear momentum in the
angular momentum is quantized. In accord with the signifi-
xy-plane at any instant. When lz > 0, the particle travels in a
cance of the orientation of lz in 1 and 2, when ml is positive,
clockwise direction as viewed from below (1); when lz < 0, the
the z-component of angular momentum is positive (clock-
motion is anticlockwise (2). However, not all values of lz are al-
wise rotation when seen from below); when ml is negative, the
lowed and it can be shown that, like rotational energy, angular
z-component of angular momentum is negative (anticlockwise
momentum is quantized.
when seen from below).
The important features of the results so far are:
r • The energies are quantized because ml is confined to inte-
lz > 0
p
ger values.
r p lz < 0
• The occurrence of ml as its square means that the energy
1 2 of rotation is independent of the sense of rotation (the
sign of ml), as expected physically.
• Apart from the state with ml = 0, all the energy levels are
How is that done? 7F.2 Showing that angular momentum
doubly degenerate; rotation can be clockwise or anticlock-
is quantized wise with the same energy.
As explained in Topic 7C, the outcome of a measurement • When ml = 0, E0 = 0. That is, there is no zero-point energy:
of a property is one of the eigenvalues of the corresponding the particle can be stationary.
operator. The first step is therefore to identify the operator • As ml increases, the wavefunctions oscillate with shorter
corresponding to angular momentum, and then to identify its wavelengths and so have greater curvature, corresponding
eigenvalues. to increasing kinetic energy (Fig. 7F.3).
Step 1 Construct the operator for angular momentum • As pointed out in Topic 7D, a wavefunction that is com-
Because the particle is confined to the xy-plane, its angular plex represents a direction of motion, and taking its
momentum is directed along the z-axis, so only this compo- complex conjugate reverses the direction. For two states
nent need be considered. As you may recall from introduc- with the same value of |ml|, the wavefunctions with ml > 0
tory physics, the z-component of the angular momentum is and ml < 0 are each other’s complex conjugate. They cor-
lz  xp y  ypx . The corresponding operator is formed by replac- respond to motion in opposite directions.
ing x, y, px, and py by their corresponding operators (Topic 7C;
q̂= q × and = pˆ q (/i)∂ /∂q, with q = x and y), which gives

 ∂ ∂ 
=lˆz x − y  Operator for l z  (7F.6a)
i  ∂y ∂x 
ml = 0
When expressed in cylindrical coordinates this operator becomes
 d
lˆz =  (7F.6b)
i dφ

Step 2 Verify that the wavefunctions are eigenfunctions of this operator


To decide whether the wavefunctions in eqn 7F.4 are eigen- |ml| = 1 |ml| = 2
functions of lˆz , allow it to act on the wavefunction (ignoring
the normalization factor):
ψ Figure 7F.3 The real parts of the wavefunctions of a particle on a
 ml

ˆ  d imlφ  iml φ ring. As the energy increases, so do the number of nodes and the
lzψ ml
= = e =iml e ml ψ ml
i dφ i curvature.
286 7 Quantum theory

The probability density predicted by the wavefunctions of Self-test 7F.1 Use the particle-on-a-ring model to calculate
eqn 7F.4 is uniform around the ring: the energy separation between the ml = +3 and ml = +4 states
of π electrons in coronene, C24H12 (3). Assume that the radius
 of the ring is three times the carbon–carbon bond length in
 eiml   eiml 
 ( ) ml ( )  

benzene and that the electrons are confined to the periphery
ml 1/ 2   1/ 2 
 (2)   (2)  of the molecule.
 e  iml   eiml  1
 1/ 2   1/ 2 

 (2)   (2)  2

Angular momentum and angular position are a pair of com-


plementary observables (in the sense defined in Topic 7C),
and the inability to specify them simultaneously with arbi-
trary precision is another example of the uncertainty princi-
ple. In this case, the z-component of angular momentum is
known exactly (as mlħ) but the location of the particle on the 3 Coronene
(model ring dotted)
ring is completely unknown, as implied by the uniform prob-
ability density. Answer: ΔE = 0.0147 zJ or 8.83 J mol−1

Example 7F.1 Using the particle-on-a-ring model


Exercises
The particle-on-a-ring is a crude but illustrative model of
E7F.1 Calculate the minimum excitation energy (i.e. the difference in energy
cyclic, conjugated molecular systems. Treat the π electrons in
between the first excited state and the ground state) of a proton constrained to
benzene as particles freely moving over a circular ring of six rotate in a circle of radius 100 pm around a fixed point.
carbon atoms and calculate the minimum energy required for
E7F.2 The wavefunction for the motion of a particle on a ring is of the form
the excitation of a π electron. The carbon–carbon bond length   Nei m l . Evaluate the normalization constant, N.
in benzene is 140 pm.
Collect your thoughts Each carbon atom contributes one π
electron, so there are six electrons to accommodate. Each state
is occupied by two electrons, so only the ml = 0, +1, and −1 7F.2 Rotation in three dimensions
states are occupied (with the last two being degenerate). The
minimum energy required for excitation corresponds to a Now consider a particle of mass m that is free to move anywhere
transition of an electron from the ml = +1 (or −1) state to the on the surface of a sphere of radius r. As for two-dimensional
ml = +2 (or −2) state. Use eqn 7F.4, and the mass of the elec- rotation the ‘particle’ may be an electron in motion around a
tron, to calculate the energies of the states. A hexagon can be nucleus or a point in a rotating molecule.
inscribed inside a circle with a radius equal to the side of the
hexagon, so take r = 140 pm.
7F.2(a) The wavefunctions and energy levels
The solution From eqn 7F.4, the energy separation between the
states with ml = +1 and ml = +2 is The potential energy of the particle is the same at all angles
and for convenience may be taken to be zero (in an atom, the
potential due to the nucleus is constant at a given radius). The
(1.055  1034 Js)2
E  E2  E1  (4  1)  Schrödinger equation is therefore
2  (9.109  1031 kg )  (1.40  10 10 m)2
 9.35  1019 J 2   2 2 2 
  2  2  2   E  (7F.8a)
2m  x y z 
Therefore the minimum energy required to excite an electron
is 0.935 aJ or 563 kJ mol−1. This energy separation corresponds The sum of the three second derivatives is the ‘laplacian’
to an absorption frequency of 1410 THz (1 THz = 1012 Hz) and and denoted ∇2 (and read ‘del-squared’). Then the equation
a wavelength of 213 nm; the experimental value for a transition becomes
of this kind is 260 nm. Such a crude model cannot be expected
to give quantitative agreement, but the value is at least of the 2 2
    E  (7F.8b)
right order of magnitude. 2m
7F Rotational motion 287

To take advantage of the symmetry of the problem it is appro-


priate to change to spherical polar coordinates (The chemist’s θ
toolkit 7F.2) when the laplacian becomes
1 2 1 ϕ
2  r  2 2
r r 2 r
The derivatives with respect to θ and ϕ have been collected in
Λ2, which is called the ‘legendrian’ and is given by
1 2 1  
2   sin 
sin  
2 2
sin   
Figure 7F.4 The wavefunction of a particle on the surface of a
In the present case, r is fixed, so the derivatives with respect to sphere must reproduce itself on passing round the equator and
r in the laplacian can be ignored and only the term Λ2/r2 sur- over the poles. This double requirement leads to wavefunctions
vives. The Schrödinger equation then becomes described by two quantum numbers.

2 1 2
   ( , )  E ( , ) ­ avefunction must join up on completing a circuit around the
w
2m r 2
equator, as specified by the angle ϕ. The second is a similar re-
The term mr2 in the denominator can be recognized as quirement that the wavefunction must join up on encircling
the moment of inertia, I, of the particle, so the Schrödinger over the poles, as specified by the angle θ. Although θ sweeps
equation is from 0 only to π, as ϕ sweeps from 0 to 2π the entire spherical
surface is covered, the wavefunctions must match everywhere,
2 2 Schrödinger equation
− Λ ψ (θ ,φ ) =
Eψ (θ ,φ ) [particle on a sphere]
 (7F.9) and the boundary condition on θ is effectively cyclic. These
2I
two conditions are illustrated in Fig. 7F.4. Once again, it can be
There are two cyclic boundary conditions to fulfil. The shown that the need to satisfy them leads to the conclusion that
first is the same as for the two-dimensional case, where the both the energy and the angular momentum are quantized.

The chemist’s toolkit 7F.2 Spherical polar coordinates


The mathematics of systems with spherical symmetry (such as 0
atoms) is often greatly simplified by using spherical polar coor- θ
dinates (Sketch 1): r, the distance from the origin (the radius),
θ, the colatitude, and ϕ, the azimuth. The ranges of these coordi- ϕ
nates are (with angles in radians, Sketch 2): 0 ≤ r ≤ +∞, 0 ≤ θ ≤ π,
2π 0
0 ≤ ϕ ≤ 2π.

Sketch 2
z
r sinθ dr dθdϕ
2
Cartesian and polar coordinates are related by
dr
θ
x  r sin cos  y  r sin sin  z  r cos
ϕ
r sinθdθ The volume element in Cartesian coordinates is dτ = dxdydz,
r r dϕ
and in spherical polar coordinates it becomes
y
d  r 2 sin drd d
x
An integral of a function f(r,θ,ϕ) over all space in polar coordi-
nates therefore has the form

Sketch 1   2
 fd   
r 0  0  0
f (r , , )r 2 sin drd d
288 7 Quantum theory

How is that done? 7F.3 Finding the solutions of the The spherical harmonics are therefore solutions of the
Schrödinger equation with energies E = l(l +1)ħ2/2I. Note that
Schrödinger equation for a particle on a sphere the energies depend only on l but not on ml.
The functions known as spherical harmonics, Yl ,ml ( , ) Step 2 Show that the wavefunctions are also eigenfunctions of the
(Table 7F.1), are well known to mathematicians and are the z-component of angular momentum
solutions of the equation2
The operator for the z-component of angular momentum is
 2Yl ,ml ( , )  l(l  1)Yl ,ml ( , ), lˆz (/i)∂ /∂φ . From Table 7F.1 note that each spherical har-
=
 (7F.10)
monic is of the form Yl ,ml ( , )  eiml f ( ). It then follows that
l  0, 1, 2,, ml  0,  1,,  l
These functions satisfy the two boundary conditions and are Yl , m (θ ,ϕ )
l
normalized in the sense that 
lˆzYl ,ml (θ ,ϕ ) = lˆz eimlϕ f (θ ) = eimlϕ f (θ ) = ml × eimlϕ f (θ )
2  i ϕ
  Yl ,ml ( , ) Yl ,ml ( , )sin  d d  1

 0  0 = ml × Yl ,ml (θ ,ϕ )
Step 1 Show that the spherical harmonics solve the Schrödinger
Therefore, the Yl ,ml ( , ) are eigenfunctions of lˆz with eigenval-
equation
ues mlħ.
The spherical harmonics are substituted for the wavefunctions In summary, the Yl ,ml ( , ) are solutions to the Schrödinger
in eqn 7F.9 to give equation for a particle on a sphere, with the corresponding
energies given by
− l( l + 1)Yl , m E
l
2 2
− Λ 2Yl, ml (θ ,ϕ ) = l(l +1) Yl, ml (θ ,ϕ ) (7F.11)
2I 2I El, ml = l(l +1) l = 0, 2 … ml = 0, ±1, … ± l
2I Energy levels
[particle on a
sphere]

Table 7F.1 The spherical harmonics


The integers l and ml are now identified as quantum numbers: l
l ml Yl, ml  ,   is the orbital angular momentum quantum number (the name
is justified in the following section) and ml is the magnetic
1/ 2

0 0  1  quantum number (for reasons related to early spectroscopic


 4 
  observations in magnetic fields). The energy is specified by l
 3 
1/ 2
alone, but for each value of l there are 2l + 1 values of ml, so each
1 0  4   cos 
  energy level is (2l + 1)-fold degenerate. Each wavefunction is
1/ 2 also an eigenfunction of ˆlz and therefore corresponds to a defi-
±1  3 
   sin  e  i nite value, mlħ, of the z-component of the angular momentum.
 8 
1/ 2
Figure 7F.5 shows a representation of the spherical harmon-
2  5  ics for l = 0 to 4 and ml = 0 in each case. The labels denoting
0  16  (3 cos   1)
2

  the different signs of the wavefunction emphasizes the location


±1
 15 
1/ 2
of the angular nodes (the positions at which the wavefunction
   cos  sin  e  i
 8  passes through zero). Note that:
1/ 2

±2  15  2 i • There are no angular nodes around the z-axis for func-


 32  sin  e
2

  tions with ml = 0. The spherical harmonic with l = 0, ml = 0


 7 
1/ 2
has no nodes: it has a constant value at all positions of the
3 0  16  (5 cos   3 cos  )
3

  surface and corresponds to a stationary particle.


 21 
1/ 2
 i
• The number of angular nodes for states with ml = 0 is
 (5 cos   1)sin  e
2
±1 
 64   equal to l. As the number of nodes increases, the wave-
1/ 2
functions become more buckled, and with this increasing
 105  curvature the kinetic energy of the particle increases.
±2  32  sin  cos  e
2 2 i

 
1/ 2 According to eqn 7F.11,
 35 
±3   sin  e
3 3 i

 64   • Because l is confined to non-negative integral values, the


energy is quantized.
2 • The energies are independent of the value of ml, because
See the first section of A deeper look 7F.1, available to read in the e-book
accompanying this text, for details of how the separation of variables procedure the energy is independent of the direction and orientation
is used to find the form of the spherical harmonics. of the rotational motion.
7F Rotational motion 289

It follows that
+ +
2 (1.055  1034 Js)2
  1.30  1022 J
2 I 2  (4.29  1047 kg m 2 )
– or 0.130 zJ. Draw up the following table:
l = 0, ml = 0 l = 1, ml = 0
l E/zJ Degeneracy
+ + + 0 0 1
– 1 0.260 3

2 0.780 5
– +
+ + – + 3 1.56 7

l = 2, ml = 0 l = 3, ml = 0 l = 4, ml = 0
The energy separation between the two lowest rotational
energy levels (l = 0 and 1) is 2.60 × 10−22 J, which corresponds
Figure 7F.5 A representation of the wavefunctions of a particle
to a photon of frequency
on the surface of a sphere that emphasizes the location of angular
nodes. Note that the number of nodes increases as the value of Hz
E 2.60  1022 J
l increases. All these wavefunctions correspond to ml = 0; the   11 1
 3.92  10 s  392 GHz
wavefunctions do not depend on the angle ϕ. h 6.626  1034 Js

Comment. Radiation of this frequency belongs to the micro-


wave region of the electromagnetic spectrum, so microwave
• There are 2l + 1 different wavefunctions (one for each spectroscopy is used to study molecular rotations (Topic 11B.2).
value of ml) that correspond to the same energy, so it fol- Because the transition frequencies depend on the moment of
lows that a level with quantum number l is (2l + 1)-fold inertia and frequencies can be measured with great precision,
degenerate. microwave spectroscopy is a very precise technique for the
• There is no zero-point energy: E0,0 = 0. determination of bond lengths.
Self-test 7F.2 What is the frequency of the transition between
the lowest two rotational levels in 2H127I? (Assume that the
bond length is the same as for 1H127I and that the iodine atom
Example 7F.2 Using the rotational energy levels is stationary.)
Answer: 196 GHz
The particle on a sphere is a good starting point for developing
a model for the rotation of a diatomic molecule. Treat the rota-
tion of 1H127I as a hydrogen atom rotating around a stationary
I atom (this is a good first approximation as the I atom is so 7F.2(b) Angular momentum
heavy it hardly moves). The bond length is 160 pm. Evaluate
the energies and degeneracies of the lowest four rotational Orbital angular momentum is the angular momentum of
energy levels of 1H127I. What is the frequency of the transition a particle around a fixed point in space. It is represented by a
between the lowest two rotational levels? vector l, a quantity with both magnitude and direction. The
components of l when it lies in a general orientation in three
Collect your thoughts The moment of inertia is I = m 1 H R 2 with
R = 160 pm; the rotational energies are given in eqn 7F.11.3 A
dimensions are
transition between two rotational levels can be brought about lx  ypz  zp y l y  zpx  xpz lz  xp y  ypx
by the emission or absorption of a photon with a frequency
Components of l  (7F.12)
given by the Bohr frequency condition (Topic 7A, hn = ΔE).
where px is the component of the linear momentum in the
The solution The moment of inertia is
x-direction at any instant, and likewise py and pz in the other
m
 1H
  R  2
directions. The square of the magnitude of the angular momen-
I  (1.675  10 kg )  (1.60  10 10 m)2  4.29  1047 kgm2
27
tum vector is given by

l 2  lx2  l 2y  lz2 Square of the magnitude of l  (7F.13)


3
As seen in Topic 11B.1, the rotational energy levels of a molecule are usu-
Classically, the kinetic energy of a particle circulating on a
ally denoted by the angular momentum quantum number J rather than l. To
keep this example in line with the current discussion, we use l to describe the ring is Ek = J2/2I (see Energy: A first look), where J, in the nota-
angular momentum of the H atom. tion used there, is the magnitude of the angular momentum.
290 7 Quantum theory

By comparing this relation with eqn 7F.11, it follows that the


square of the magnitude of the orbital angular momentum is z
l(l +1)ħ2, so the magnitude of the orbital angular momentum is ml = –1
ml = +2
1/ 2
Magnitude  {l(l  1)} , l  0, 1, 2, 
Magnitude of orbital angulaar momentum (7F.14)
ml = 0
The spherical harmonics are also eigenfunctions of ˆlz and the
corresponding eigenvalues give the z-component of the orbital
angular momentum:
ml = –2
z -Component  ml , ml  0, 1, l ml = +1
z - Component of orbital angular momentum  (7F.15)

So, both the magnitude and the z-component of orbital angular


momentum are quantized.
Figure 7F.6 The permitted orientations of angular momentum
when l = 2. This representation is too specific because the
Brief illustration 7F.1 azimuthal orientation of the vector (its angle around z) is
indeterminate.
The lowest four rotational energy levels of any object rotating
in three dimensions correspond to l = 0, 1, 2, 3. The following
table can be constructed by using eqns 7F.14 and 7F.15.
ˆl  ∂ ∂ 
Magnitude of z-Component of
= x y −z 
i  ∂z ∂y 
l orbital angular Degeneracy orbital angular
momentum/  momentum/   ∂ ∂ 
= ˆl z −x 
Orbital angular
0 0 1 0 y
i  ∂x ∂z  momentum operators (7F.16)

1 21/2 3 0, ±1
=ˆl   x ∂ − y ∂ 
2 61/2
5 0, ±1, ±2 z  
i  ∂y ∂x 
3 121/2 7 0, ±1, ±2, ±3
Each of these expressions can be derived in the same way as
eqn 7F.6a by converting the classical expressions for the com-
ponents of the angular momentum in eqn 7F.12 into their
7F.2(c) The vector model quantum mechanical operator equivalents. The commutation
The result that ml is confined to the values 0, ±1,… ±l for a given relations among the three operators (Problem P7F.9), are
value of l means that the component of angular momentum
about the z-axis—the contribution to the total angular momen- = [ˆlx ,ˆl y ] i=
ˆl
z [ˆl y ,ˆlz ] i=
ˆl
x [ˆlz ,ˆlx ] iˆl y
tum of rotation around that axis—may take only 2l + 1 values. Angular momentum commutation relations  (7F.17)
If the orbital angular momentum is represented by a vector of
length {l(l + 1)}1/2, it follows that this vector must be oriented so The three operators do not commute, so they represent com-
that its projection on the z-axis is ml and that it can have only plementary observables (Topic 7C). It is possible to have pre-
2l + 1 orientations rather than the continuous range of orien- cise knowledge of only one of the components of the angular
tations of a rotating classical body (Fig. 7F.6). The remarkable momentum, so if lz is specified exactly (as in the preceding dis-
implication is that cussion), neither lx nor ly can be specified.
The operator for the square of the magnitude of the angular
The orientation of a rotating body is quantized. momentum in eqn 7F.6b commutes with all three components
The quantum mechanical result that a rotating body may not (Problem P7F.11):
take up an arbitrary orientation with respect to some specified
[lˆ2 ,lˆq ] 0=
Commutators of angular
axis (such as an axis defined by the direction of an externally = q x , y , and z momentum operators  (7F.18)
applied electric or magnetic field) is called space quantization.
The preceding discussion has referred to the z-component of It follows that both the square magnitude and one component,
orbital angular momentum and there has been no reference to commonly the z-component, of the angular momentum can
the x- and y-components. The reason for this omission is found be specified simultaneously and precisely. The illustration in
by examining the operators for the three components, each one Fig. 7F.6, which is summarized in Fig. 7F.7(a), therefore gives
being given by a term like that in eqn 7F.6a: a false impression of the state of the system, because it suggests
7F Rotational motion 291

z
z momentum can be thought of as lying with its tip on any point
+2
+2 on the mouth of the cone. At this stage it should not be thought
ml +1 of as sweeping round the cone; that aspect of the model is
+1
added when the picture is allowed to convey more information
0 0 (Topics 8B and 8C).

–1 –1
Brief illustration 7F.2
–2
–2
(a) (b) If the wavefunction of a rotating molecule is given by the
spherical harmonic Y3,+2 then the angular momentum can be
Figure 7F.7 (a) A summary of Fig. 7F.6. However, because the represented by a cone
azimuthal angle of the vector around the z-axis is indeterminate,
a better representation is as in (b), where each vector lies at an • with a side of length 121/2 (representing the magnitude of
unspecified azimuthal angle on its cone. 121/2ħ); and
• with a projection of +2 on the z-axis (representing the
z-component of +2ħ).
definite values for the x- and y-components too. A better pic-
ture must reflect the impossibility of specifying lx and ly if lz is
known.
The vector model of angular momentum uses pictures like Exercises
that in Fig. 7F.7(b). The cones are drawn with side {l(l + 1)}1/2 −47 2
E7F.3 The moment of inertia of a CH4 molecule is 5.27 × 10 kg m . What is
units, and represent the magnitude of the angular momen- the minimum energy needed to start it rotating?
tum. Each cone has a definite projection (of ml units) on to the E7F.4 What is the magnitude of the angular momentum of a CH4 molecule
z-axis, representing the precisely known value of lz. The pro- when it is rotating with its minimum energy?
jections of the vector on to the x- and y-axes, which give the E7F.5 Draw scale vector diagrams to represent the states (i) l = 1, ml = +1,
values of lx and ly, are indefinite: the vector representing angular (ii) l = 2, ml = 0.

Checklist of concepts
☐ 1. The energy and angular momentum for a particle rotat- ☐ 6. Space quantization refers to the quantum mechanical
ing in two- or three-dimensions are quantized; quanti- result that a rotating body may not take up an arbitrary
zation results from the requirement that the wavefunc- orientation with respect to some specified axis.
tion satisfies cyclic boundary conditions. ☐ 7. The three components of orbital angular momentum are
☐ 2. All energy levels of a particle rotating in two dimen- mutually complementary observables.
sions are doubly degenerate except for the lowest level ☐ 8. Only the magnitude of the orbital angular momentum
(ml = 0). and one of its components can be specified simultane-
☐ 3. A rotating particle does not have a zero-point energy. ously with arbitrary precision.
☐ 4. It is impossible to specify simultaneously the angular ☐ 9. In the vector model of angular momentum, the orbital
momentum and location of a particle with arbitrary angular momentum is represented by a cone with a side
precision. of length {l(l + 1)}1/2 and a projection of ml on the z-axis.
☐ 5. For a particle rotating in three dimensions, the cyclic The vector can be thought of as lying with its tip on an
boundary conditions imply that the magnitude and indeterminate point on the mouth of the cone.
z-component of the angular momentum are quantized.
292 7 Quantum theory

Checklist of equations
Property Equation Comment Equation number
iml
Wavefunction of particle on a ring  ml ( )  e /(2) 1/2
ml = 0, ±1, ±2,… 7F.4
2 2 2
Energy of particle on a ring Eml = m  /2 I
l ml = 0, ±1, ±2,…, I = mr 7F.4
z-Component of angular momentum of particle mlħ ml = 0, ±1, ±2,… 7F.7
on a ring
Wavefunction of particle on a sphere  ( , )  Yl ,ml ( , ) Y is a spherical harmonic (Table 7F.1)
2
Energy of particle on a sphere E l, ml  l(l  1) /2 I l = 0, 1, 2,… 7F.11
1/2
Magnitude of angular momentum {l(l + 1)} ħ l = 0, 1, 2,… 7F.14
z-Component of angular momentum mlħ ml = 0, ±1, ±2,… ±l 7F.15
Angular momentum commutation relations [lˆx ,lˆy ] = ilˆz , [lˆy ,lˆz ] = ilˆx , [lˆz ,lˆx ] = ilˆy 7F.17

[lˆ2 =
,lˆq ] 0,=
q x ,y ,z 7F.18
Exercises and problems 293

FOCUS 7 Quantum theory

To test your understanding of this material, work through Selected solutions can be found at the end of this Focus in
the Exercises, Additional exercises, Discussion questions, and the e-book. Solutions to even-numbered questions are available
Problems found throughout this Focus. online only to lecturers.

TOPIC 7A The origins of quantum mechanics


Discussion questions
D7A.1 Summarize the evidence that led to the introduction of quantum D7A.3 Explain how Einstein’s introduction of quantization accounted for the
mechanics. properties of heat capacities at low temperatures.
D7A.2 Explain how Planck’s introduction of quantization accounted for the D7A.4 Explain the meaning and summarize the consequences of wave–particle
properties of black-body radiation. duality.

Additional exercises
E7A.7 Calculate the wavelength and frequency at which the intensity of the E7A.17 The work function of metallic rubidium is 2.09 eV. Calculate the
radiation is a maximum for a black body at 2.7 K. kinetic energy and the speed of the electrons ejected by light of wavelength
(i) 650 nm, (ii) 195 nm.
E7A.8 The intensity of the radiation from an object is found to be a maximum
at 282 GHz (1 GHz = 109 Hz). Assuming that the object is a black body, E7A.18 A glow-worm of mass 5.0 g emits red light (650 nm) with a power
calculate its temperature. of 0.10 W entirely in the backward direction. To what speed will it have
accelerated after 10 y if released into free space and assumed to live?
E7A.9 Calculate the molar heat capacity of a monatomic non-metallic solid
at 500 K which is characterized by an Einstein temperature of 300 K. Express E7A.19 A photon-powered spacecraft of mass 10.0 kg emits radiation of
your result as a multiple of 3R. wavelength 225 nm with a power of 1.50 kW entirely in the backward
direction. To what speed will it have accelerated after 10.0 y if released into
E7A.10 Calculate the energy of the quantum involved in the excitation of (i) an
free space?
electronic oscillation of period 1.0 fs, (ii) a molecular vibration of period 10 fs,
(iii) a pendulum of period 1.0 s. Express the results in joules and kilojoules E7A.20 To what speed must a proton be accelerated from rest for it to have a
per mole. de Broglie wavelength of 100 pm? What accelerating potential difference is
needed?
E7A.11 Calculate the energy of the quantum involved in the excitation of
(i) an electronic oscillation of period 2.50 fs, (ii) a molecular vibration of E7A.21 To what speed must an electron be accelerated for it to have a
period 2.21 fs, (iii) a balance wheel of period 1.0 ms. Express the results in de Broglie wavelength of 3.0 cm?
joules and kilojoules per mole.
E7A.22 To what speed must a proton be accelerated for it to have a de Broglie
E7A.12 Calculate the energy of a photon and the energy per mole of photons wavelength of 3.0 cm?
for radiation of wavelength (i) 600 nm (red), (ii) 550 nm (yellow), (iii) 400 nm
E7A.23 The ‘fine-structure constant’, α, plays a special role in the structure of
(blue).
matter; its approximate value is 1/137. What is the de Broglie wavelength of an
E7A.13 Calculate the energy of a photon and the energy per mole of photons electron travelling at αc, where c is the speed of light?
for radiation of wavelength (i) 200 nm (ultraviolet), (ii) 150 pm (X-ray),
E7A.24 Calculate the linear momentum of photons of wavelength 350 nm. At
(iii) 1.00 cm (microwave).
what speed does a hydrogen molecule need to travel for it to have the same
E7A.14 Calculate the speed to which a stationary H atom would be accelerated linear momentum?
if it absorbed a photon of wavelength (i) 600 nm (red), (ii) 550 nm (yellow),
E7A.25 Calculate the de Broglie wavelength of (i) a mass of 1.0 g travelling at
(iii) 400 nm (blue).
1.0 cm s−1; (ii) the same, travelling at 100 km s−1; (iii) a He atom travelling at
4
E7A.15 Calculate the speed to which a stationary He atom (mass 4.0026mu) 1000 m s−1 (a typical speed at room temperature).
would be accelerated if it absorbed a photon of wavelength (i) 200 nm
E7A.26 Calculate the de Broglie wavelength of an electron accelerated from
(ultraviolet), (ii) 150 pm (X-ray), (iii) 1.00 cm (microwave).
rest through a potential difference of (i) 100 V; (ii) 1.0 kV; (iii) 100 kV.
E7A.16 A laser used to read CDs emits red light of wavelength 700 nm. How
many photons does it emit each second if its power is (i) 0.10 W, (ii) 1.0 W?
294 7 Quantum theory

Problems
P7A.1 Calculate the energy density in the range 650 nm to 655 nm inside a P7A.7 The total energy density of black-body radiation is found by integrating
cavity at (a) 25 °C, (b) 3000 °C. For this relatively small range of wavelength it the energy spectral density over all wavelengths, eqn 7A.2. Evaluate this
is acceptable to approximate the integral of the energy spectral density ρ(λ, T) integral for the Planck distribution. This is most easily done by making the

between λ1 and λ2 by ρ(λ,T) × (λ2 − λ1). substitution x = hc/λkT; you will need the integral 0 {x 3 /(e x  1)}dx  4 /15.
−1 −1 Hence deduce the Stefan–Boltzmann law that the total energy density
P7A.2 Calculate the energy density in the range 1000 cm to 1010 cm inside
of black-body radiation is proportional to T 4, and find the constant of
a cavity at (a) 25 °C, (b) 4 K.
proportionality.
P7A.3 Demonstrate that the Planck distribution reduces to the Rayleigh–Jeans ‡
P7A.8 Prior to Planck’s derivation of the distribution law for black-body
law at long wavelengths.
radiation, Wien found empirically a closely related distribution function
P7A.4 The wavelength λmax at which the Planck distribution is a maximum can which is very nearly but not exactly in agreement with the experimental
be found by solving dρ(λ, T)/dT = 0. Differentiate ρ(λ,T) with respect to T and results, namely ρ(λ,T) = (a/λ5)e−b/λkT. This formula shows small deviations
show that the condition for the maximum can be expressed as xex − from Planck’s at long wavelengths. (a) Find a form of the Planck distribution
5(ex − 1) = 0, where x = hc/λkT. There are no analytical solutions to this which is appropriate for short wavelengths (Hint: consider the behaviour
equation, but a numerical approach gives x = 4.965 as a solution. Use this of the term ehc/λkT −1 in this limit). (b) Compare your expression from part
result to confirm Wien’s law, that λmaxT is a constant, deduce an expression (a) with Wien’s empirical formula and hence determine the constants a and b.
for the constant, and compare it to the value quoted in the text. (c) Integrate Wien’s empirical expression for ρ(λ,T) over all wavelengths and
show that the result is consistent with the Stefan–Boltzmann law (Hint: to
P7A.5 For a black body, the temperature and the wavelength of the emission
compute the integral use the substitution x = hc/λkT and then refer to the
maximum, λmax, are related by Wien’s law, λmaxT = hc/4.965k; see Problem
Resource section). (d) Show that Wien’s empirical expression is consistent
7A.4. Values of λmax from a small pinhole in an electrically heated container
with Wien’s law.
were determined at a series of temperatures, and the results are given below.

Deduce the value of Planck’s constant. P7A.9 The temperature of the Sun’s surface is approximately 5800 K. On the
assumption that the human eye evolved to be most sensitive at the wavelength
θ/°C 1000 1500 2000 2500 3000 3500 of light corresponding to the maximum in the Sun’s radiant energy
distribution, identify the colour of light to which the eye is the most sensitive.
λmax/nm 2181 1600 1240 1035 878 763
P7A.10 The Einstein frequency is often expressed in terms of an equivalent
‡ −2
P7A.6 Solar energy strikes the top of the Earth’s atmosphere at 1361 W m . temperature θE, where θE = hn/k. Confirm that θE has the dimensions of
About 30 per cent of this energy is reflected directly back into space. The temperature, and express the criterion for the validity of the high-temperature
Earth–atmosphere system absorbs the remaining energy and re-radiates it form of the Einstein equation in terms of θE. Evaluate θE for (a) diamond,
into space as black-body radiation at 5.672 × 10−8(T/K)4 W m−2, where T is the for which n = 46.5 THz, and (b) for copper, for which n = 7.15 THz. Use
temperature. Assuming that the arrangement has come to equilibrium, what is these values to calculate the molar heat capacity of each substance at 25 °C,
the average black-body temperature of the Earth? Calculate the wavelength at expressing your answers as multiples of 3R.
which the black-body radiation from the Earth is at a maximum.

TOPIC 7B Wavefunctions
Discussion questions
D7B.1 Describe how a wavefunction summarizes the dynamical properties of a D7B.3 Identify the constraints that the Born interpretation puts on acceptable
system and how those properties may be predicted. wavefunctions.
D7B.2 Explain the relation between probability amplitude, probability density,
and probability.

Additional exercises
2
E7B.3 A possible wavefunction for an electron in a region of length L is (i) sin(ax); (ii) cos(ax) e −x ? Which of these functions are acceptable as
sin(3πx/L). Normalize this wavefunction (to 1). wavefunctions?
2
−ax
E7B.4 Normalize (to 1) the wavefunction e in the range −∞ ≤ x ≤ ∞, with a > 0. E7B.8 A possible wavefunction for an electron confined to a region of length L is
Refer to the Resource section for the necessary integral. sin(3πx/L). What is the probability of finding the electron in the range dx at x = L/6?
−ax
E7B.5 Normalize (to 1) the wavefunction e in the range 0 ≤ x ≤ ∞, with a > 0. E7B.9 A possible wavefunction for an electron confined to a region of length L
is sin(2πx/L). What is the probability of finding the electron between x = L/4
E7B.6 Which of the following functions can be normalized (in all cases the
2 and x = L/2?
range for x is from x = −∞ to ∞, and a is a positive constant): (i) e −ax ; (ii) e−ax.
Which of these functions are acceptable as wavefunctions? E7B.10 A possible wavefunction for an electron confined to a region of length
L is sin(3πx/L). What is the probability of finding the electron between x = 0
E7B.7 Which of the following functions can be normalized (in all
and x = L/3?
cases the range for x is from x = −∞ to ∞, and a is a positive constant):
E7B.11 What are the dimensions of a wavefunction that describes a particle

These problems were supplied by Charles Trapp and Carmen Giunta. free to move in both the x and y directions?
Exercises and problems 295

E7B.12 The wavefunction for a particle free to move between x = 0 and such a particle? In each case, give your reasons for accepting or rejecting each
x = L is (2/L)1/2 sin(πx/L); confirm that this wavefunction has the expected function. (i) cos ϕ; (ii) sin ϕ; (iii) cos(0.9ϕ).
dimensions.
E7B.14 A possible wavefunction for an electron confined to a region of length
E7B.13 Imagine a particle confined to move on the circumference of a circle (‘a L is sin(3πx/L). At what value or values of x is the probability density a
particle on a ring’), such that its position can be described by an angle ϕ in the maximum? Locate the position or positions of any nodes in the wavefunction.
range 0–2π. Which of the following wavefunctions would be acceptable for You need consider only the range x = 0 to x = L.

Problems
P7B.1 Imagine a particle confined to move on the circumference of a circle (‘a P7B.8 A normalized wavefunction for a particle confined between 0 and L in
particle on a ring’), such that its position can be described by an angle ϕ in the the x direction, and between 0 and L in the y direction (that is, to a square of
range 0 to 2π. Find the normalizing factor for the wavefunctions: (a) eiϕ and side L) is ψ = (2/L) sin(πx/L) sin(πy/L). The probability of finding the particle
(b) eim l φ , where ml is an integer. between x1 and x2 along x, and between y1 and y2 along y is
y  y2 x  x2
P7B.2 Imagine a particle confined to move on the circumference of a circle (‘a P
particle on a ring’), such that its position can be described by an angle ϕ in the y  y1 
x  x1
 2 dx dy

range 0 to 2π. Find the normalizing factor for the wavefunctions: (a) cos ϕ; Calculate the probability that the particle is: (a) between x = 0 and x = L/2,
(b) sin mlϕ, where ml is an integer. y = 0 and y = L/2 (i.e. in the bottom left-hand quarter of the square);
(b) between x = L/4 and x = 3L/4, y = L/4 and y = 3L/4 (i.e. a square of
P7B.3 A particle is confined to a two-dimensional region with 0 ≤ x ≤ Lx side L/2 centred on x = y = L/2).
and 0 ≤ y ≤ Ly. Normalize (to 1) the functions (a) sin(πx/Lx)sin(πy/Ly) and
(b) sin(πx/L)sin(πy/L) for the case Lx = Ly = L. P7B.9 The normalized ground-state wavefunction of a hydrogen atom is
2 2
 (r )  (1/a03 )1/ 2 e  r /a0 where a0 = 53 pm (the Bohr radius) and r is the distance
− ax − by
P7B.4 Normalize (to 1) the wavefunction e e for a system in two from the nucleus. (a) Calculate the probability that the electron will be found
dimensions with a > 0 and b > 0, and with x and y both allowed to range from somewhere within a small sphere of radius 1.0 pm centred on the nucleus.
−∞ to ∞. Refer to the Resource section for relevant integrals. (b) Now suppose that the same sphere is located at r = a0. What is the
probability that the electron is inside it? You may approximate the probability
P7B.5 Suppose that in a certain system a particle free to move along one
of being in a small volume δV at position r by ψ(r)2δV.
dimension (with 0 ≤ x ≤ ∞) is described by the unnormalized wavefunction
ψ(x)e−ax with a = 2 m−1. What is the probability of finding the particle at a P7B.10 Atoms in a chemical bond vibrate around the equilibrium bond
distance x ≥ 1 m? (Hint: You will need to normalize the wavefunction before length. An atom undergoing vibrational motion is described by the
2 2
using it to calculate the probability.) wavefunction  (x )  Ne  x / 2a , where a is a constant and −∞ ≤ x ≤ ∞. (a) Find
the normalizing factor N. (b) Use mathematical software to calculate the
P7B.6 Suppose that in a certain system a particle free to move along x (without
2 probability of finding the particle in the range −a ≤ x ≤ a (the result will be
constraint) is described by the unnormalized wavefunction  (x )  e  ax with
expressed in terms of the ‘error function’, erf(x)).
a = 0.2 m−2. Use mathematical software to calculate the probability of finding
the particle at x ≥1 m. P7B.11 Atoms in a chemical bond vibrate around the equilibrium bond
length. Suppose that an atom undergoing vibrational motion is described by
P7B.7 A normalized wavefunction for a particle confined between 0 and L in 2 2

1/2
the wavefunction  (x )  Nxe  x / 2a . Where is the most probable location of
the x direction is ψ = (2/L) sin(πx/L). Suppose that L = 10.0 nm. Calculate the atom?
the probability that the particle is (a) between x = 4.95 nm and 5.05 nm,
(b) between x = 1.95 nm and 2.05 nm, (c) between x = 9.90 nm and 10.00 nm,
(d) between x = 5.00 nm and 10.00 nm.

TOPIC 7C Operators and observables


Discussion questions
D7C.1 How may the curvature of a wavefunction be interpreted? D7C.3 Use the properties of wavepackets to account for the uncertainty
relation between position and linear momentum.
D7C.2 Describe the relation between operators and observables in quantum
mechanics.

Additional exercises
E7C.8 Construct the potential energy operator of a particle with potential E7C.11 Functions of the form cos(nπx/L), where n = 1, 3, 5…, can be used to
energy V(x) = De(1 − e−ax)2, where De and a are constants. model the wavefunctions of particles confined to the region between x = −L/2
and x = +L/2. The integration is limited to the range −L/2 to +L/2 because
E7C.9 Identify which of the following functions are eigenfunctions of the
2 the wavefunction is zero outside this range. Show that the wavefunctions are
operator d2/dx2: (i) cos(kx); (ii) eikx, (iii) kx, (iv) e −ax. Give the corresponding
orthogonal for n = 1 and 3. You will find the necessary integral in the Resource
eigenvalue where appropriate.
section.
E7C.10 Functions of the form sin(nπx/L), where n = 1, 2, 3…, are
E7C.12 For the same system as in Exercise E7C.11 show that the wavefunctions
wavefunctions in a region of length L (between x = 0 and x = L). Show that the
with n = 3 and 5 are orthogonal.
wavefunctions with n = 2 and 4 are orthogonal.
296 7 Quantum theory

E7C.13 Imagine a particle confined to move on the circumference of a circle 2π


ˆ
Ω m = ∫ ψ m (φ )Ωψ m (φ )dφ
(‘a particle on a ring’), such that its position can be described by an angle ϕ in l 0 l l

the range 0–2π. The wavefunctions for this system are of the form  m ( )  eiml
where  ml ( ) are the normalized wavefunctions. Compute the expectation
with ml an integer. Show that the wavefunctions with ml = +1 and +2 are
value of the position, specified by the angle ϕ, for the case ml = +1, and then
orthogonal. (Hint: Note that (eix)⋆ = e−ix, and that eix = cos x + i sin x.)
for the general case of integer ml.
E7C.14 For the same system as in Exercise E7C.13 show that the wavefunctions
E7C.18 For the system described in Exercise E7C.17, evaluate the expectation
with ml = +1 and −2 are orthogonal.
value of the angular momentum represented by the operator (ħ/i)d/dϕ for the
1/2
E7C.15 Find 〈x〉 when the wavefunction is ψ(x) = (2/L) sin(πx/L) for a case ml = +1, and then for the general case of integer ml.
particle confined between x = 0 and x = L.
E7C.19 An electron is confined to a linear region with a length of the same
E7C.16 Find 〈px〉 for the case where the normalized wavefunction is order as the diameter of an atom (about 100 pm). Calculate the minimum
ψ(x) = (2/L)1/2sin(πx/L) for a particle confined between x = 0 and x = L. uncertainties in its position and speed.
−1
E7C.17 Imagine a particle confined to move on the circumference of a circle E7C.20 The speed of a certain electron is 995 km s . If the uncertainty in
(‘a particle on a ring’), such that its position can be described by an angle ϕ in its momentum is to be reduced to 0.0010 per cent, what uncertainty in its
the range 0–2π. The normalized wavefunctions for this system are of the form location must be tolerated?
 ml ( )  (1/2)1/ 2 eiml with ml an integer. The expectation value of a quantity
represented by the operator Ω̂ is given by

Problems
P7C.1 Identify which of the following functions are eigenfunctions of the P7C.6 Calculate the expectation value of the linear momentum px of a particle
inversion operator î , which has the effect of making the replacement described by the following normalized wavefunctions (in each case N is the
x → −x: (a) x3 − kx, (b) cos kx, (c) x2 + 3x − 1. Identify the eigenvalue of î appropriate normalizing factor, which you do not need to find): (a) Neikx,
2
when relevant. (b) N cos kx, (c) Ne − ax , where in each one x ranges from −∞ to +∞.
P7C.2 An electron in a one-dimensional region of length L is described by P7C.7 A particle freely moving in one dimension x with 0 ≤ x ≤ ∞ is in a
the wavefunction ψn(x) = sin(nπx/L), where n = 1, 2,…, in the range x = 0 to state described by the normalized wavefunction ψ(x) = a1/2e−ax/2, where a is a
x = L; outside this range the wavefunction is zero. The orthogonality of these constant. Evaluate the expectation value of the position operator.
wavefunctions is confirmed by considering the integral
L
P7C.8 The normalized wavefunction of an electron in a linear accelerator
I   sin(nx /L)sin(mx /L)dx is ψ = (cos χ)eikx + (sin χ)e−ikx, where χ (chi) is a parameter. (a) What is the
0
probability that the electron will be found with a linear momentum (a) +kħ,
(a) Use the identity sin A sin B  12 cos( A  B)  cos( A  B) to rewrite the
(b) −kħ? (c) What form would the wavefunction have if it were 90 per cent
integrand as a sum of two terms. (b) Consider the case n = 2, m = 1, and
certain that the electron had linear momentum +kħ? (d) Evaluate the kinetic
make separate sketch graphs of the two terms identified in part (a) in the
energy of the electron.
range x = 0 to x = L. (c) Make use of the properties of the cosine function to
argue that the area enclosed between the curves and the x axis is zero in both P7C.9 (a) Show that the expectation value of a hermitian operator is real.
cases, and hence that the integral is zero. (d) Generalize the argument for the (Hint: Start from the definition of the expectation value and then apply the
case of arbitrary n and m (n ≠ m). definition of hermiticity to it.) (b) Show that the expectation value of an
2 2 2 operator that can be written as the square of a hermitian operator is positive.
P7C.3 Confirm that the kinetic energy operator, −(ħ /2m)d /dx , is hermitian.
(Hint: Start from the definition of the expectation value for the operator
(Hint: Use the same approach as in the text, but because a second derivative ˆ ˆ ; recognize that Ωψ
ˆ is a function, and then apply the definition of
ΩΩ
is involved you will need to integrate by parts twice; you may assume that the
hermiticity.)
derivatives of the wavefunctions go to zero as x → ±∞.)
P7C.10 Suppose the wavefunction of an electron in a one-dimensional region
P7C.4 The operator corresponding to the angular momentum of a particle is
is a linear combination of cos nx functions. (a) Use mathematical software or
(ħ/i)d/dϕ, where ϕ is an angle. For such a system the criterion for an operator
a spreadsheet to construct superpositions of cosine functions as
Ω̂ to be hermitian is
ˆ (φ )dφ =  ψ  (φ )Ωψ

1 N

∫ j ˆ i (φ )dφ 
2π 2π
∫0
ψ i (φ )Ωψ j  0 
 (x ) 
N
 cos(kx)
k 1

Show that (ħ/i)d/dϕ is a hermitian operator. (Hint: Use the same where the constant 1/N (not a normalization constant) is introduced to
approach as in the text; recall that the wavefunction must be single-valued, keep the superpositions with the same overall magnitude. Set x = 0 at the
so ψi(ϕ) = ψi(ϕ + 2π).) centre of the screen and build the superposition there; consider the range
P7C.5 (a) Show that the sum of two hermitian operators  and B̂ is also a x = −1 to +1. (b) Explore how the probability density ψ2(x) changes with the
hermitian operator. (Hint: Start by separating the appropriate integral into value of N. (c) Evaluate the root-mean-square location of the packet,  x 2 1/ 2.
two terms, and then apply the definition of hermiticity.) (b) Show that the (d) Determine the probability that a given momentum will be observed.
product of a hermitian operator with itself is also a hermitian operator. Start P7C.11 A particle is in a state described by the normalized wavefunction
by considering the integral 2
 (x )  (2a/)1/ 4 e  ax , where a is a constant and −∞ ≤x ≤ ∞. (a) Calculate the
ˆ ˆ dτ
I = ∫ψ i ΩΩψ expectation values  x  ,  x 2 ,  px  , and  px2  ; the necessary integrals will be found
j
in the Resource section. (b) Use these results to calculate px  { px2    px  2 }1/ 2
Recall that Ω̂ψ j is simply another function, so the integral can be thought of as and x  { x 2    x  2 }1/ 2 . (c) Hence verify that the value of the product ΔpxΔx is
a
function consistent with the predictions from the uncertainty principle.

I = ∫ψ i Ωˆ (Ωψ
ˆ ) dτ
j
P7C.12 A particle is in a state described by the normalized wavefunction
ψ(x) = a1/2e−ax/2, where a is a constant and 0 ≤ x ≤ ∞. Evaluate the expectation
Now apply the definition of hermiticity and complete the proof. value of the commutator of the position and momentum operators.
Exercises and problems 297

P7C.13 Evaluate the commutators of the operators (a) d/dx and 1/x, (b) d/dx P7C.15 Evaluate the commutators (a) [H ˆ ,pˆ ] and (b) [Hˆ ,xˆ] where
x
and x2. (Hint: Follow the procedure in the text by considering, for case (a), =Hˆ pˆ x2 /2m + Vˆ (x ). Choose (i) V(x) = V0, a constant, (ii) V (x ) = 12 kf x 2 .
(d/dx)(1/x)ψ and (1/x)(d/dx)ψ; recall that ψ is a function of x, so it will be (Hint: See the hint for Problem P7C.13.)
necessary to use the product rule to evaluate some of the derivatives.)
+
P7C.14 Evaluate the commutators of the operators â and â where
ˆ (xˆ + ipˆ x )/21/2 and aˆ=
a= +
(xˆ − ipˆ x )/21/2 .

TOPIC 7D Translational motion


Discussion questions
D7D.1 Explain the physical origin of quantization for a particle confined to the D7D.3 Explain the physical origin of quantum mechanical tunnelling.
interior of a one-dimensional box. Why is tunnelling more likely to contribute to the mechanisms of electron
transfer and proton transfer processes than to mechanisms of group transfer
D7D.2 Describe the features of the solution of the particle in a one-
reactions, such as AB + C → A + BC (where A, B, and C are large molecular
dimensional box that appear in the solutions of the particle in two- and three-
groups)?
dimensional boxes. What feature occurs in the two- and three-dimensional
box that does not occur in the one-dimensional box?

Additional exercises
E7D.10 Evaluate the linear momentum and kinetic energy of a free proton E7D.22 For a particle in a box of length L, write the expression for the energy
described by the wavefunction e−ikx with k = 5 nm−1. levels, En, and then write a similar expression En′ for the energy levels when
the length of the box has increased to 1.1L (that is, an increase by 10 per cent).
E7D.11 Write the wavefunction for a particle of mass 1.0 g travelling to the
Calculate (En  En )/En, the fractional change in the energy that results from
right at 10 m s−1.
extending the box.
E7D.12 Calculate the energy separations in joules, kilojoules per mole,
E7D.23 For a particle in a cubical box of side L, write the expression for
electronvolts, and reciprocal centimetres between the levels (i) n = 3 and
the energy levels, En1, n2, n3 , and then write a similar expression En′1, n2 , n3 for the
n = 2, (ii) n = 7 and n = 6 of an electron in a box of length 1.50 nm.
energy levels when the side of the box has decreased to 0.9L (that is, a
E7D.13 For a particle in a one-dimensional box, show that the wavefunctions ψ1 decrease by 10 per cent). Calculate (En1, n2 , n3  En1, n2 , n3 )/En1, n2 , n3 , the fractional
and ψ2 are orthogonal. The necessary integrals will be found in the Resource section. change in the energy that results from expanding the box.
E7D.14 For a particle in a one-dimensional box, show that the wavefunctions E7D.24 Find an expression for the value of n of a particle of mass m in
ψ1 and ψ3 are orthogonal. a one-dimensional box of length L such that the separation between
neighbouring levels is equal to the mean energy of thermal motion  12 kT  .
E7D.15 Calculate the probability that a particle will be found between 0.65L
Calculate the value of n for the case of a helium atom in a box of length
and 0.67L in a box of length L for the case where the wavefunction is (i) ψ1,
1 cm at 298 K.
(ii) ψ2. You may make the same approximation as in Exercise E7D.4.
E7D.25 Find an expression for the value of n of a particle of mass m in a one-
E7D.16 Without evaluating any integrals, state the value of the expectation
dimensional box of length L such that the energy of the level is equal to the
value of x for a particle in a box of length L for the case where the
mean energy of thermal motion  12 kT . Calculate the value of n for the case of
wavefunction has n = 2. Explain how you arrived at your answer.
an argon atom in a box of length 0.1 cm at 298 K.
E7D.17 For a particle in a box of length L sketch the wavefunction
E7D.26 For a particle in a square box of side L, at what position (or
corresponding to the state with n = 1 and on the same graph sketch the
positions) is the probability density a maximum if the wavefunction has
corresponding probability density. Without evaluating any integrals, explain
n1 = 1, n2 = 3? Also, describe the position of any node or nodes in the
why for this wavefunction the expectation value of x2 is not equal to (L/2)2.
wavefunction.
E7D.18 For a particle in a box of length L sketch the wavefunction
E7D.27 For a particle in a rectangular box with sides of length L1 = L and
corresponding to the state with n = 1 and on the same graph sketch the
L2 = 2L, find a state that is degenerate with the state n1 = n2 = 2. (Hint: You will
corresponding probability density. For this wavefunction, explain whether
need to experiment with some possible values of n1 and n2.) Is this degeneracy
you would expect the expectation value of x2 to be greater than or less than
associated with symmetry?
the square of the expectation value of x.
E7D.28 For a particle in a rectangular box with sides of length L1 = L and
E7D.19 An electron is confined to a cubic box of side of L. What would be
L2 = 2L, find a state that is degenerate with the state n1 = 2, n2 = 8. Would you
the length of the box such that the zero-point energy of the electron is equal
expect there to be any degenerate states for a rectangular box with L1 = L and
to its rest mass energy, mec2? Express your answer in terms of the parameter
L2 = 2L ? Explain your reasoning.
λC = h/mec, the ‘Compton wavelength’ of the electron.
E7D.29 Consider a particle in a cubic box. What is the degeneracy of the level
E7D.20 For a particle in a box of length L and in the state with n = 3, at what
that has an energy 143 times that of the lowest level?
positions is the probability density a maximum? At what positions is the
probability density zero? E7D.30 Suppose that a proton of an acidic hydrogen atom is confined to an
acid that can be represented by a barrier of height 2.0 eV and length 100 pm.
E7D.21 For a particle in a box of length L and in the state with n = 5, at what
Calculate the probability that a proton with energy 1.5 eV can escape from
positions is the probability density a maximum? At what positions is the
the acid.
probability density a minimum?
298 7 Quantum theory

Problems
P7D.1 Calculate the separation between the two lowest levels for an O2 P7D.9 (a) Set up the Schrödinger equation for a particle of mass m in a
molecule in a one-dimensional container of length 5.0 cm. At what value three-dimensional rectangular box with sides L1, L2, and L3. Show that the
of n does the energy of the molecule reach 12 kT at 300 K, and what is the Schrödinger equation is separable. (b) Show that the wavefunction and the
separation of this level from the one immediately below? energy are defined by three quantum numbers. (c) Specialize the result from
3 part (b) to an electron moving in a cubic box of side L = 5 nm and draw an
P7D.2 A nitrogen molecule is confined in a cubic box of volume 1.00 m .
energy diagram resembling Fig. 7D.2 and showing the first 15 energy levels.
(i) Assuming that the molecule has an energy equal to 23 kT at T = 300 K,
Note that each energy level might be degenerate. (d) Compare the energy
what is the value of n  (nx2  n2y  nz2 )1/ 2 for this molecule? (ii) What is the
level diagram from part (c) with the energy level diagram for an electron in
energy separation between the levels n and n + 1? (iii) What is the de Broglie
a one-dimensional box of length L = 5 nm. Are the energy levels more or less
wavelength of the molecule?
sparsely distributed in the cubic box than in the one-dimensional box?
2
P7D.3 Calculate the expectation values of x and x for a particle in the state
P7D.10 In the text the one-dimensional particle-in-a-box problem involves
with n = 1 in a one-dimensional square-well potential.
confining the particle to the range from x = 0 to x = L. This problem explores
2
P7D.4 Calculate the expectation values of px and px for a particle in the state a similar situation in which the potential energy is zero between x = −L/2
with n = 2 in a one-dimensional square-well potential. and x = +L/2, and infinite elsewhere. (a) Identify the boundary conditions
that apply in this case. (b) Show that cos(kx) is a solution of the Schrödinger
P7D.5 When β-carotene (1) is oxidized in vivo, it breaks in half and forms two
equation for the region with zero potential energy, find the values of k for
molecules of retinal (vitamin A), which is a precursor to the pigment in the retina
which the boundary conditions are satisfied, and hence derive an expression
responsible for vision. The conjugated system of retinal consists of 11 C atoms
for the corresponding energies. Sketch the three wavefunctions with the lowest
and one O atom. In the ground state of retinal, each level up to n = 6 is occupied
energies. (c) Repeat the process, but this time with the trial wavefunction
by two electrons. Assuming an average internuclear distance of 140 pm, calculate
sin(k′x). (d) Compare the complete set of energies you have obtained in
(a) the separation in energy between the ground state and the first excited state
parts (b) and (c) with the energies for the case where the particle is confined
in which one electron occupies the state with n = 7, and (b) the frequency of the
between 0 and L: are they the same? (e) Normalize the wavefunctions (the
radiation required to produce a transition between these two states. (c) Using
necessary integrals are in the Resource section). (f) Without evaluating any
your results, choose among the words in parentheses to generate a rule for the
integrals, explain why  x  = 0 for both sets of wavefunctions.
prediction of frequency shifts in the absorption spectra of linear polyenes:
P7D.11 Many biological electron transfer reactions, such as those associated
The absorption spectrum of a linear polyene shifts to (higher/lower)
with biological energy conversion, may be visualized as arising from electron
frequency as the number of conjugated atoms (increases/decreases).
tunnelling between protein-bound co-factors, such as cytochromes, quinones,
flavins, and chlorophylls. This tunnelling occurs over distances that are
often greater than 1.0 nm, with sections of protein separating electron donor
from acceptor. For a specific combination of donor and acceptor, the rate
of electron tunnelling is proportional to the transmission probability, with
κ ≈ 7 nm−1 (eqn 7D.17). By what factor does the rate of electron tunnelling
between two co-factors increase as the distance between them changes from
1 -Carotene
2.0 nm to 1.0 nm? You may assume that the barrier is such that eqn 7D.20b is
P7D.6 Consider a particle of mass m confined to a one-dimensional box appropriate.
of length L and in a state with normalized wavefunction ψn. (a) Without
P7D.12 Derive eqn 7D.20a, the expression for the transmission probability and
evaluating any integrals, explain why  x  = L/2. (b) Without evaluating
show that when κW >> 1 it reduces to eqn 7D.20b. The derivation proceeds by
any integrals, explain why  px  = 0. (c) Derive an expression for  x 2  (the
requiring that the wavefunction and its first derivative are continuous at the
necessary integrals will be found in the Resource section). (d) For a particle
edges of the barrier, as expressed by eqns 7D.19a and 7D.19b.
in a box the energy is given by En = n2h2/8mL2 and, because the potential

energy is zero, all of this energy is kinetic. Use this observation and, without P7D.13 A particle of mass m moves in one dimension in a region divided into
evaluating any integrals, explain why  px2   n2h2 /4 L2. three zones: zone 1 has V = 0 for −∞ < x ≤ 0; zone 2 has V = V2 for
0 ≤ x ≤ W; zone 3 has V = V3 for W ≤ x < ∞. In addition, V3 < V2. In zone 1
P7D.7 Consider a particle of mass m confined to a one-dimensional box
the wavefunction is A1eik1x  B1e  ik1x ; the term eik1x represents the wave incident
of length L and in a state with normalized wavefunction ψn. According to
on the barrier V2, and the term e − ik1x represents the reflected wave. In zone
Topic 7C, the uncertainty in the position is x  { x 2    x  2 }1/ 2 and for
2 the wavefunction is A2 ek2x  B2 e  k2x . In zone 3 the wavefunction has only
the linear momentum it is px  { px2    px  2 }1/ 2 . (a) For this system find
a forward component, A3eik3x , which represents a particle that has traversed
expressions for  x  ,  x 2 ,  px  , and  px2  ; hence find expressions for Δx and
the barrier. Consider a case in which the energy of the particle E is greater
Δpx. (b) Go on to find an expression for the product ΔxΔpx. (c) Show that
than V3 but less than V2, so that zone 2 represents a barrier. The transmission
for n = 1 and n = 2 the result from (b) is in accord with the Heisenberg
probability, T, is the ratio of the square modulus of the zone 3 amplitude
uncertainty principle, and infer that this is also true for n ≥ 1.
to the square modulus of the incident amplitude, that is, T = |A3|2/|A1|2.

P7D.8 A particle is confined to move in a one-dimensional box of length L. (a) Derive an expression for T by imposing the requirement that the
If the particle is behaving classically, then it simply bounces back and forth in wavefunction and its slope must be continuous at the zone boundaries. You
the box, moving with a constant speed. (a) Explain why the probability density, can simplify the calculation by assuming from the outset that A1 = 1. (b) Show
P(x), for the classical particle is 1/L. (Hint: What is the total probability of that this equation for T reduces to eqn 7D.20b in the high, wide barrier limit
finding the particle in the box?) (b) Explain why the average value of xn when V1 = V3 = 0. (c) Draw a graph of the probability of proton tunnelling
L
is  x n   0 P (x )x ndx . (c) By evaluating such an integral, find  x  and  x 2 . when V3 = 0, W = 50 pm, and E = 10 kJ mol−1 in the barrier range E < V2 < 2E.
 
(d) For a quantum particle  x  = L/2 and  x 2   L2 13  1/2n2 2 . Compare
P7D.14 A potential barrier of height V extends from x = 0 to positive x.
these expressions with those you have obtained in part (c), recalling that the
Inside this barrier the normalized wavefunction is ψ = Ne−κx. Calculate
correspondence principle states that, for very large values of the quantum
(a) the probability that the particle is inside the barrier and (b) the average
numbers, the predictions of quantum mechanics approach those of classical
penetration depth of the particle into the barrier.
mechanics.
Exercises and problems 299

P7D.15 Use mathematical software or a spreadsheet for the following (c) Use mathematical software to generate three-dimensional plots of the
procedures: wavefunctions for a particle confined to a rectangular surface with
(i) n1 = 1, n2 = 1, the state of lowest energy, (ii) n1 = 1, n2 = 2, (iii) n1 = 2,
(a) Plot the probability density for a particle in a box with n = 1, 2,… 5, and
n2 = 1, and (iv) n1 = 2, n2 = 2. Deduce a rule for the number of nodal lines
n = 50. How do your plots illustrate the correspondence principle?
in a wavefunction as a function of the values of n1 and n2.
(b) Plot the transmission probability T against E/V for passage by (i) a hydrogen
molecule, (ii) a proton, and (iii) an electron through a barrier of height V.

TOPIC 7E Vibrational motion


Discussion questions
D7E.1 Describe the variation with the mass and force constant of the D7E.3 To what quantum mechanical principle can you attribute the existence
separation of the vibrational energy levels of a harmonic oscillator. of a zero-point vibrational energy?
D7E.2 In what ways does the quantum mechanical description of a harmonic
oscillator merge with its classical description at high quantum numbers?

Additional exercises
E7E.7 Calculate the zero-point energy of a harmonic oscillator consisting of a Ev = 12 kf x tp2 . For a particle of mass mu undergoing harmonic motion with
particle of mass 5.16 × 10−26 kg and force constant 285 N m−1. force constant kf = 1000 N m−1, calculate the energy of the state with v = 0 and
−25 hence find the separation between the classical turning points. Repeat the
E7E.8 For a certain harmonic oscillator of effective mass 2.88 × 10 kg, the
calculation for an oscillator with kf = 100 N m−1.
difference in adjacent energy levels is 3.17 zJ. Calculate the force constant of
the oscillator. E7E.14 The classical turning points of a harmonic oscillator occur at the
displacements at which all of the energy is potential energy; that is, when
E7E.9 Calculate the wavelength of the photon needed to excite a transition
Ev = 12 kf x tp2 . For a particle of mass mu undergoing harmonic motion with force
between neighbouring energy levels of a harmonic oscillator of effective mass
constant kf = 1000 N m−1, calculate the energy of the state with v = 1 and hence
equal to that of an oxygen atom (15.9949mu) and force constant 544 N m−1.
find the separation between the classical turning points. Express your answer
E7E.10 Sketch the form of the wavefunctions for the harmonic oscillator with as a percentage of a typical bond length of 110 pm.
quantum numbers v = 1 and 2. Use a symmetry argument to explain why
E7E.15 How many nodes are there in the wavefunction of a harmonic
these two wavefunctions are orthogonal (do not evaluate any integrals).
oscillator with (i) v = 5; (ii) v = 35?
35
E7E.11 Assuming that the vibrations of a Cl2 molecule are equivalent to those
E7E.16 Locate the nodes of a harmonic oscillator wavefunction with v = 2.
of a harmonic oscillator with a force constant kf = 329 N m−1, what is the zero-
(Express your answers in terms of the coordinate y.)
point energy of vibration of this molecule? Use m(35Cl) = 34.9688mu.
14 E7E.17 Locate the nodes of the harmonic oscillator wavefunction with v = 3.
E7E.12 Assuming that the vibrations of a N2 molecule are equivalent to those
−1
of a harmonic oscillator with a force constant kf = 2293.8 N m , what is the E7E.18 At what displacements is the probability density a maximum for a state
zero-point energy of vibration of this molecule? Use m(14N) = 14.0031mu. of a harmonic oscillator with v = 3?
E7E.13 The classical turning points of a harmonic oscillator occur at the
displacements at which all of the energy is potential energy; that is, when

Problems
P7E.1 If the vibration of a diatomic A–B is modelled using a harmonic frequency of 1H35Cl is 5.63 × 1014 s−1). (c) Compare your answer with the
1/2
oscillator, the vibrational frequency is given by ω = (kf/μ) , where μ is the value obtained in the Problem P7E.1. (d) In organic molecules it is commonly
effective mass, μ = mAmB/(mA + mB). If atom A is substituted by an isotope observed that the C–H stretching frequency is reduced by a factor of around
(for example 2H substituted for 1H), then to a good approximation the force 0.7 when 1H is substituted by 2H: rationalize this observation.
constant remains the same. Why? (Hint: Is there any change in the number 1
P7E.3 The vibrational frequency of H2 is 131.9 THz. What is the vibrational
of charged species?) (a) Show that when an isotopic substitution is made for
frequency of 2H2 and of 3H2? Use integer relative atomic masses for this
atom A, such that its mass changes from mA to mA′, the vibrational frequency
estimate.
of A′–B, ωA′B, can be expressed in terms of the vibrational frequency of A–B,
−1
ωAB as ωA′B = ωAB (μAB/μA′B)1/2, where μAB and μA′B are the effective masses of P7E.4 The force constant for the bond in CO is 1857 N m . Calculate the
A–B and A′–B, respectively. (b) The vibrational frequency of 1H35Cl is 5.63 × vibrational frequencies (in Hz) of C O, C O, C O, and 13C18O. Use
12 16 13 16 12 18

1014 s−1. Calculate the vibrational frequency of (i) 2H35Cl and (ii) 1H37Cl. Use integer relative atomic masses for this estimate.
integer relative atomic masses.
P7E.5 In infrared spectroscopy it is common to observe a transition from the
P7E.2 Consider the case where in the diatomic molecule A–B the mass of B v = 0 to v = 1 vibrational level. If this transition is modelled as a harmonic
is much greater than that of A. (a) Show that for an isotopic substitution of oscillator, the energy of the photon involved is ℏω, where ω is the vibrational
A, the ratio of vibrational frequencies is ωA′B ≈ ωAB(mA/mA′)1/2. (b) Use this frequency. (a) Show that the wavenumber of the radiation corresponding to
expression to calculate the vibrational frequency of 2H35Cl (the vibrational photons of this energy,  , is given by    /2c , where c is the speed of light.
300 7 Quantum theory

(b) The vibrational frequency of 1H35Cl is ω = 5.63 × 1014 s−1; calculate  . The expectation value of this operator for an harmonic oscillator
(c) Derive an expression for the force constant kf in terms of  . (d) For 12C16O wavefunction with quantum number v is
the v = 0 → 1 transition is observed at 2170 cm−1. Calculate the force constant
and estimate the wavenumber at which the corresponding absorption occurs
 d2
T  v   12  N v2  H v e  y
2 2
/2
H v e  y / 2 dy
for 14C16O. Use integer relative atomic masses for this estimate.
 dy 2

P7E.6 The following data give the wavenumbers (wavenumbers in cm ) of the


−1 where Nv is the normalization constant (eqn 7E.10) and α is defined in
v = 0 → 1 transition of a number of diatomic molecules. Calculate the force eqn 7E.7 (the term α arises from dx = αdy). (b) Evaluate the second derivative
constants of the bonds and arrange them in order of increasing stiffness. Use and then use the property H v  2 yH v  2vH v  0; where the prime indicates a
integer relative atomic masses. derivative, to rewrite the derivatives in terms of the Hv (you should be able to
eliminate all the derivatives). (c) Now proceed as in the text, in which terms
1
H35Cl 1
H81Br 1
H127I 12
C16O 14
N16O of the form yHv are rewritten by using the property Hv+1 − 2yHv + 2vHv−1 = 0;
you will need to apply this twice. (d) Finally, evaluate the integral using the
2990 2650 2310 2170 1904 properties of the integrals of the Hermite polynomials given in Table 7E.1 and
2+
so obtain the result quoted in the text.
P7E.7 Carbon monoxide binds strongly to the Fe ion of the haem (heme)
3 4
group of the protein myoglobin. Estimate the vibrational frequency of CO P7E.12 Calculate the values of  x  v and  x  v for a harmonic oscillator by
bound to myoglobin by making the following assumptions: the atom that using the properties of the Hermite polynomials given in Table 7E.1; follow
binds to the haem group is immobilized, the protein is infinitely more massive the approach used in the text.
than either the C or O atom, the C atom binds to the Fe2+ ion, and binding P7E.13 Use the same approach as in Example 7E.3 to calculate the probability
of CO to the protein does not alter the force constant of the CO bond. The that a harmonic oscillator in the first excited state will be found in the
wavenumber of the v = 0 → 1 transition of 12C16O is 2170 cm−1. classically forbidden region. You will need to use mathematical software to
2+
P7E.8 Carbon monoxide binds strongly to the Fe ion of the haem (heme) evaluate the appropriate integral. Compare the result you obtain with that for
group of the protein myoglobin. It is possible to estimate the vibrational the ground state and comment on the difference.
frequency of CO bound to myoglobin by assuming that the atom that binds P7E.14 Use the same approach as in Example 7E.3 to calculate the probability
to the haem group is immobilized, and that the protein is infinitely more that a harmonic oscillator in the states v = 0, 1,…7 will be found in the
massive than either the C or O atom. Suppose that you have at your disposal a classically forbidden region. You will need to use mathematical software to
supply of myoglobin, a suitable buffer in which to suspend the protein, 12C16O, evaluate the final integrals. Plot the probability as a function of v and interpret
13 16
C O, 12C18O, 13C18O, and an infrared spectrometer. Assuming that isotopic the result in terms of the correspondence principle.
substitution does not affect the force constant of the CO bond, describe a set
of experiments that: (a) proves which atom, C or O, binds to the haem group P7E.15 The intensities of spectroscopic transitions between the vibrational
of myoglobin, and (b) allows for the determination of the force constant of the states of a molecule are proportional to the square of the integral ∫ψv′xψvdx
CO bond for myoglobin-bound carbon monoxide. over all space. Use the relations between Hermite polynomials given in
− gx 2
eqn 7E.12 to show that the only permitted transitions are those for which
P7E.9 A function of the form e is a solution of the Schrödinger equation for v′ = v ± 1 and evaluate the integral in these cases.
the harmonic oscillator (eqn 7E.5), provided that g is chosen correctly. In this
P7E.16 The potential energy of the rotation of one CH3 group relative to its
2
problem you will find the correct form of g. (a) Start by substituting   e  gx
into the left-hand side of eqn 7E.5 and evaluating the second derivative. neighbour in ethane can be expressed as V(ϕ) = V0 cos 3ϕ. Show that for
(b) You will find that in general the resulting expression is not of the form small displacements the motion of the group is harmonic and derive an
constant × ψ, implying that ψ is not a solution to the equation. However, expression for the energy of excitation from v = 0 to v = 1. (Hint: Use a series
by choosing the value of g such that the terms in x2 cancel one another, a expansion for cos 3ϕ.) What do you expect to happen to the energy levels and
solution is obtained. Find the required form of g and hence the corresponding wavefunctions as the excitation increases to high quantum numbers?
energy. (c) Confirm that the function so obtained is indeed the ground state P7E.17 (a) Without evaluating any integrals, explain why you expect  x  v  0
of the harmonic oscillator, as quoted in eqn 7E.13a, and that it has the energy for all states of a harmonic oscillator. (b) Use a physical argument to explain
expected from eqn 7E.6. why  px  v  0. (c) Equation 7E.18c gives  Ek  v  12 Ev . Recall that the kinetic
P7E.10 Write the normalized form of the ground state wavefunction of the energy is given by p2/2m and hence find an expression for  px2  v. (d) Note
harmonic oscillator in terms of the variable y and the parameter α. (a) Write from Topic 7C that the uncertainty in the position, Δx, is given by Δx =
the integral you would need to evaluate to find the mean displacement  y , (〈x2〉 − 〈x〉2)1/2 and likewise for the momentum px  ( px2    px  2 )1/ 2 . Find
and then use a symmetry argument to explain why this integral is equal expressions for Δx and Δpx (the expression for  x 2  v is given in the text).
to 0. (b) Calculate  y 2  (the necessary integral will be found in the Resource (e) Hence find an expression for the product ΔxΔpx and show that the
section). (c) Repeat the process for the first excited state. Heisenberg uncertainty principle is satisfied. (f) For which state is the product
ΔxΔpx a minimum?
P7E.11 The expectation value of the kinetic energy of a harmonic oscillator
is most easily found by using the virial theorem, but in this Problem you P7E.18 Use mathematical software or a spreadsheet to gain some insight into
will find it directly by evaluating the expectation value of the kinetic energy the origins of the nodes in the harmonic oscillator wavefunctions by plotting
operator with the aid of the properties of the Hermite polynomials given in the Hermite polynomials Hv(y) for v = 0 through 5.
eqn 7E.12. (a) Write the kinetic energy operator T̂ in terms of x and show that
it can be rewritten in terms of the variable y (introduced in eqn 7E.10) and the
frequency ω as
d2
Tˆ = − 12 ω 2
dy
Exercises and problems 301

TOPIC 7F Rotational motion


Discussion questions
D7F.1 Discuss the physical origin of quantization of energy for a particle D7F.3 Describe the vector model of angular momentum in quantum
confined to motion on a ring. mechanics. What features does it capture?
D7F.2 Describe the features of the solution of the particle on a ring that appear
in the solution of the particle on a sphere. What concept applies to the latter
but not to the former?

Additional exercises
−45
E7F.6 The rotation of a molecule can be represented by the motion of a E7F.16 The moment of inertia of an SF6 molecule is 3.07 × 10 kg m2.
particle moving over the surface of a sphere. Calculate the magnitude of its Calculate the energy needed to excite an SF6 molecule from a state with l = 2
angular momentum when l = 1 and the possible components of the angular to a state with l = 3.
momentum along the z-axis. Express your results as multiples of ħ.
E7F.17 What is the magnitude of the angular momentum of an SF6 molecule
E7F.7 The rotation of a molecule can be represented by the motion of a particle when it is rotating with its minimum energy?
moving over the surface of a sphere with angular momentum quantum
E7F.18 Draw the vector diagram for all the permitted states of a particle with
number l = 2. Calculate the magnitude of its angular momentum and the
l = 6.
possible components of the angular momentum along the z-axis. Express your
results as multiples of ħ. E7F.19 How many angular nodes are there for the spherical harmonic Y3,0 and
at which values of θ do they occur?
E7F.8 For a particle on a ring, how many nodes are there in the real part, and
in the imaginary part, of the wavefunction for (i) ml = 0 and (ii) ml = +3? In E7F.20 Based on the pattern of nodes in Fig. 7F.5, how many angular nodes do
both cases, find the values of ϕ at which any nodes occur. you expect there to be for the spherical harmonic Y4,0? Does it have a node at
θ = 0?
E7F.9 For a particle on a ring, how many nodes are there in the real part, and
in the imaginary part of the wavefunction for (i) ml = +1 and (ii) ml = +2? In E7F.21 Consider the real part of the spherical harmonic Y1,+1. At which values
both cases, find the values of ϕ at which any nodes occur. of ϕ do angular nodes occur? These angular nodes can also be described as
planes: identify the positions of the corresponding planes (for example, the
E7F.10 The wavefunction for the motion of a particle on a ring can also be
angular node with ϕ = 0 is the xz-plane). Do the same for the imaginary part.
written ψ = N cos(mlϕ), where ml is integer. Evaluate the normalization
constant, N. E7F.22 Consider the real part of the spherical harmonic Y2,+2. At which values
2 of ϕ do angular nodes occur? Identify the positions of the corresponding
E7F.11 By considering the integral 0   ml d , where ml  ml, confirm that

ml
planes. Repeat the process for the imaginary part.
wavefunctions for a particle in a ring with different values of the quantum
number ml are mutually orthogonal. E7F.23 What is the degeneracy of a molecule rotating with J = 3?
2
E7F.12 By considering the integral 0 cos ml cos ml d , where ml  ml, E7F.24 What is the degeneracy of a molecule rotating with J = 4?
confirm that the wavefunctions cos mlϕ and cos ml for a particle on a
E7F.25 Draw diagrams to scale, and similar to Fig. 7F.7a, representing the
ring are orthogonal. (Hint: To evaluate the integral, first apply the identity
states (i) l = 1, ml = −1, 0, +1, (ii) l = 2 and all possible values of ml.
cos A cos B  12 {cos( A  B)  cos( A  B)}.)
E7F.26 Draw diagrams to scale, and similar to Fig. 7F.7a, representing the
E7F.13 Consider a proton constrained to rotate in a circle of radius 100 pm
states (i) l = 0, (ii) l = 3 and all possible values of ml.
around a fixed point. For this arrangement, calculate the value of |ml|
corresponding to a rotational energy equal to the classical average energy at E7F.27 Derive an expression for the angle between the vector representing
25 °C (which is equal to 12 kT ). angular momentum l with z-component ml = +l (that is, its maximum value)
−45 2 and the z-axis. What is this angle for l = 1 and for l = 5?
E7F.14 The moment of inertia of an SF6 molecule is 3.07 × 10 kg m . What is
the minimum energy needed to start it rotating? E7F.28 Derive an expression for the angle between the vector representing
−47 2 angular momentum l with z-component ml = +l and the z-axis. What value
E7F.15 The moment of inertia of a CH4 molecule is 5.27 × 10 kg m .
does this angle take in the limit that l becomes very large? Interpret your
Calculate the energy needed to excite a CH4 molecule from a state with l = 1
result in the light of the correspondence principle.
to a state with l = 2.

Problems
P7F.1 The particle on a ring is a useful model for the motion of electrons
around the porphyrin ring (2), the conjugated macrocycle that forms the NH N
structural basis of the haem (heme) group and the chlorophylls. The group
may be modelled as a circular ring of radius 440 pm, with 22 electrons in
the conjugated system moving along its perimeter. In the ground state of the N HN
molecule each state is occupied by two electrons. (a) Calculate the energy and
angular momentum of an electron in the highest occupied level. (b) Calculate
the frequency of radiation that can induce a transition between the highest
occupied and lowest unoccupied levels. 2 Porphyrin ring
302 7 Quantum theory

iϕ −2iϕ
P7F.2 Consider the following wavefunctions (i) e , (ii) e , (iii) cos ϕ, the operators correctly it is helpful to imagine that they are acting on some
iϕ −iϕ
and (iv) (cos χ)e + (sin χ)e each of which describes a particle on a ring. arbitrary function f: it does not matter what f is, and at the end of the proof it
(a) Decide whether or not each wavefunction is an eigenfunction of the operator is simply removed. Consider [lˆx ,lˆ=
y] lˆx lˆy − lˆy lˆx. Consider the effect of the first
lˆz for the z-component of the angular momentum (lˆz = (/i)(d/dφ )); where term on some arbitrary function f and evaluate
the function is an eigenfunction, give the eigenvalue. (b) For the functions
 A B
 C 
D
that are not eigenfunctions, calculate the expectation value of lz (you will first
 ∂ ∂   ∂f ∂f
need to normalize the wavefunction). (c) Repeat the process but this time for lˆx lˆy f = 2
−  y − z   z − x 
the kinetic energy, for which the operator is −(ℏ2/2I)(d2/dϕ2). (d) Which of  ∂z ∂y   ∂x ∂z 
these wavefunctions describe states of definite angular momentum, and which
The next step is to multiply out the parentheses, and in doing so care needs to be
describe states of definite kinetic energy?
taken over the order of operations. (b) Repeat the procedure for the other term
P7F.3 Is the Schrödinger equation for a particle on an elliptical ring of semi- in the commutator, lˆy lˆx f . (c) Combine the results from parts (a) and (b) so as to
major axes a and b separable? (Hint: Although r varies with angle ϕ, the two evaluate lˆx lˆy f − lˆy lˆx f ; you should find that many of the terms cancel. Confirm
are related by r2 = a2 sin2 ϕ + b2 cos2 ϕ.) that the final expression you have is indeed ilˆz f , where lˆz is given in eqn 7F.13.
1 127 (d) The definitions in eqn 7F.13 are related to one another by cyclic permutation
P7F.4 Calculate the energies of the first four rotational levels of H I free
2 of the x, y, and z. That is, by making the permutation x→y, y→z, and z→x, you
to rotate in three dimensions; use for its moment of inertia I = μR , with
can move from one definition to the next: confirm that this is so. (e) The same
μ = mHmI/(mH + mI) and R = 160 pm. Use integer relative atomic masses for
cyclic permutation can be applied to the commutators of these operators. Start
this estimate.
with [lˆx , lˆy ] = ilˆz and show that cyclic permutation generates the other two
P7F.5 Consider the three spherical harmonics (a) Y0,0, (b) Y2,−1, and (c) Y3,+3. commutators in eqn 7F.14.
(a) For each spherical harmonic, substitute the explicit form of the function
P7F.10 Show that lˆz and lˆ both commute with the hamiltonian for a
2
taken from Table 7F.1 into the left-hand side of eqn 7F.8 (the Schrödinger
hydrogen atom. What is the significance of this result? Begin by noting that
equation for a particle on a sphere) and confirm that the function is a solution
lˆ2 = lˆx2 + lˆy2 + lˆz2. Then show that =
[lˆz , lˆq2 ] [lˆz , lˆq ]lˆq + lˆq [lˆz , lˆq ] and then use the
of the equation; give the corresponding eigenvalue (the energy) and show that
angular momentum commutation relations in eqn 7F.14.
it agrees with eqn 7F.10. (b) Likewise, show that each spherical harmonic is an
eigenfunction of lˆz = (/i)(d/dφ ) and give the eigenvalue in each case. P7F.11 Starting from the definition of the operator lˆz given in eqn 7F.13, show
that in spherical polar coordinates it can be expressed as lˆz =−i∂ /∂φ . (Hint:
P7F.6 Confirm that Y1,+1, taken from Table 7F.1, is normalized. You will need to
You will need to express the Cartesian coordinates in terms of the spherical
integrate Y1,+1Y1,+1 over all space using the relevant volume element:
polar coordinates; refer to The chemist’s toolkit 7F.2).
volume
element

 2 P7F.12 A particle confined within a spherical cavity is a starting point for
 
 0  0
Y1,+1Y1,1 sin  d d
the discussion of the electronic properties of spherical metal nanoparticles.
Here, you are invited to show in a series of steps that the l = 0 energy levels
P7F.7 Confirm that Y1,0 and Y1,+1, taken from Table 7F.1, are orthogonal. You
of an electron in a spherical cavity of radius R are quantized and given by
will need to integrate Y1,+1Y1,+1 over all space using the relevant volume element:
En = n2h2/8meR2. (a) The hamiltonian for a particle free to move inside a
volume
element
 spherical cavity of radius a is
 2
 
 0  0
Y1,0Y1,1 sin  d d
2 2 1 ∂2 1
Hˆ =− ∇ with ∇2 = r + 2 Λ2
(Hint: A useful result for evaluating the integral is (d/dθ)sin3 θ = 3sin2 θ cos θ.) 2m r ∂r 2 r

P7F.8 (a) Show that   c1Yl ,ml  c2Yl ,ml , is an eigenfunction of Λ with
2
Show that the Schrödinger equation is separable into radial and angular
eigenvalue −l(l +1); c1 and c2 are arbitrary coefficients. (Hint: Apply Λ2 to ψ components. That is, begin by writing ψ(r,θ,ϕ) = R(r)Y(θ,ϕ), where R(r)
and use the properties given in eqn 7F.9.) (b) The spherical harmonics Y1,+1 depends only on the distance of the particle from the centre of the sphere,
and Y1,−1 are complex functions (see Table 7F.1), but as they are degenerate and Y(θ,ϕ) is a spherical harmonic. Then show that the Schrödinger equation
eigenfunctions of Λ2, any linear combination of them is also an eigenfunction, can be separated into two equations, one for R(r), the radial equation, and the
as was shown in part (a). Show that the combinations ψa = −Y1,+1 + Y1,−1 and other for Y(θ,ϕ), the angular equation. (b) Consider the case l = 0. Show by
ψb = i(Y1,+1 + Y1,−1) are real. (c) Show that ψa and ψb are orthogonal (you differentiation that the solution of the radial equation has the form
will need to integrate using the relevant volume element, sinθdθdϕ). (d)
sin(nr / a)
Normalize ψa and ψb. (e) Identify the angular nodes in these two functions R(r )  (2a)1/2
and the planes to which they correspond. (f) Is ψa an eigenfunction of lˆz ? r
Discuss the significance of your answer. (c) Now go on to show (by acknowledging the appropriate boundary
conditions) that the allowed energies are given by En = n2h2/8ma2. With
P7F.9 In this problem you will establish the commutation relations, given
substitution of me for m and of R for a, this is the equation given above for the
in eqn 7E.14, between the operators for the x-, y-, and z-components of
energy.
angular momentum, which are defined in eqn 7F.13. In order to manipulate

FOCUS 7 Quantum theory


Integrated activities

I7.1 A star too small and cold to shine has been found by S. Kulkarni et al. (a) Derive an expression for ΔrG⦵ for CH4(g) → C(graphite) + 2 H2(g) at
(Science, 1478 (1995)). The spectrum of the object shows the presence of temperature T. Proceed by using data from the tables in the Resource section
methane which, according to the authors, would not exist at temperatures to find ΔrH⦵ and ΔrS⦵ at 298 K and then convert these values to an arbitrary
much above 1000 K. The mass of the star, as determined from its gravitational temperature T by using heat capacity data, also from the tables (assume that
effect on a companion star, is roughly 20 times the mass of Jupiter. The star is the heat capacities do not vary with temperature). (b) Find the temperature
considered to be a brown dwarf, the coolest ever found. above which ΔrG⦵ becomes positive. (The solution to the relevant equation
Exercises and problems 303

cannot be found analytically, so use mathematical software to find a numerical I7.4 Suppose that 1.0 mol of perfect gas molecules all occupy the lowest
solution or plot a graph). Does your result confirm the assertion that methane energy level of a cubic box. (a) How much work must be done to change the
could not exist at temperatures much above 1000 K? (c) Assume the star volume of the box by ΔV? (b) Would the work be different if the molecules
behaves as a black body at 1000 K, and calculate the wavelength at which the all occupied a state n ≠ 1? (c) What is the relevance of this discussion to
radiation from it is maximum. (d) Estimate the fraction of the energy density the expression for the expansion work discussed in Topic 2A? (d) Can you
of the star that it emitted in the visible region of the spectrum (between identify a distinction between adiabatic and isothermal expansion?
420 nm and 700 nm). (You may assume that over this wavelength range Δλ 2 2 1/ 2
I7.5 Evaluate x  ( x    x  ) and px  ( px2    px  2 )1/ 2 for the ground
it is acceptable to approximate the integral of the Planck distribution by
state of (a) a particle in a box of length L and (b) a harmonic oscillator.
ρ(λ,T)Δλ.)
Discuss these quantities with reference to the uncertainty principle.
I7.2 Describe the features that stem from nanometre-scale dimensions that are
I7.6 Repeat Problem I7.5 for (a) a particle in a box and (b) a harmonic
not found in macroscopic objects.
oscillator in a general quantum state (n and v, respectively).
I7.3 Explain why the particle in a box and the harmonic oscillator are useful
models for quantum mechanical systems: what chemically significant systems
can they be used to represent?
FOCUS 8
Atomic structure and spectra

This Focus discusses the use of quantum mechanics to de- Topic uses hydrogenic atomic orbitals to describe the structures
scribe and investigate the ‘electronic structure’ of atoms, the of many-electron atoms. Then, in conjunction with the concept
arrangement of electrons around their nuclei. The concepts of ‘spin’ and the ‘Pauli exclusion principle’, it describes the ori-
are of central importance for understanding the properties gin of the periodicity of atomic properties and the structure of
of atoms and molecules, and hence have extensive chemical the periodic table.
­applications. 8B.1 The orbital approximation; 8B.2 The Pauli exclusion principle;
8B.3 The building-up principle; 8B.4 Self-consistent field orbitals

8A Hydrogenic atoms
8C Atomic spectra
This Topic uses the principles of quantum mechanics intro-
duced in Focus 7 to describe the electronic structure of a ‘hy- The spectra of many-electron atoms are more complicated than
drogenic atom’, a one-electron atom or ion of general atomic that of hydrogen. Similar principles apply, but Coulombic and
number Z. Hydrogenic atoms are important because their magnetic interactions between the electrons give rise to a vari-
Schrödinger equations can be solved exactly. Moreover, they ety of energy differences, which are summarized by construct-
provide a set of concepts that are used to describe the struc- ing ‘term symbols’. These symbols act as labels that display the
tures of many-electron atoms and molecules. Solving the total orbital and spin angular momentum of a many-electron
Schrödinger equation for an electron in an atom involves the atom and are used to express the selection rules that govern
separation of the wavefunction into angular and radial parts their spectroscopic transitions.
and the resulting wavefunctions are the hugely important 8C.1 The spectra of hydrogenic atoms; 8C.2 The spectra of ­
many-electron atoms
‘atomic orbitals’ of hydrogenic atoms.
8A.1 The structure of hydrogenic atoms; 8A.2 Atomic orbitals and
their energies

What is an application of this material?


8B Many-electron atoms Impact 13, accessed via the e-book, focuses on the use of atomic
spectroscopy to examine stars. By analysing their spectra it is
A ‘many-electron atom’ is an atom or ion with more than one possible to determine the composition of their outer layers and
electron. Examples include all neutral atoms other than H; so the surrounding gases and to determine features of their physi-
even He, with only two electrons, is a many-electron atom. This cal state.

➤ Go to the e-book for videos that feature the derivation and interpretation of equations, and applications of this material.
TOPIC 8A Hydrogenic atoms

The Swedish spectroscopist Johannes Rydberg noted (in 1890)


➤ Why do you need to know this material? that the wavenumbers of all the lines are given by the expression
An understanding of the structure of hydrogenic atoms is
 1 1 
central to the description of all other atoms, the periodic   R H  2  2  Spectral lines of a hydrogen atom (8A.1)
table, and bonding. All accounts of the structures of mol-  n1 n2 
ecules are based on the language and concepts introduced
here. with n1 = 1 (the Lyman series), 2 (the Balmer series), and 3 (the
Paschen series), and that in each case n2 = n1 + 1, n1 + 2,….
➤ What is the key idea? The constant R H is now called the Rydberg constant for
Atomic orbitals are one-electron wavefunctions for atoms; the hydrogen atom and is found empirically to have the value
they are labelled by three quantum numbers that specify 109 677 cm−1.
the energy and angular momentum of the electron.

➤ What do you need to know already?


You need to be aware of the concept of a wavefunction
The structure of hydrogenic
8A.1
(Topic 7B) and its interpretation. You also need to know atoms
how to set up a Schrödinger equation and how boundary
conditions result in only certain solutions being acceptable Consider a hydrogenic atom, an atom or ion of arbitrary
(Topics 7D and 7F). The solutions make use of the proper- atomic number Z but having a single electron. Hydrogen itself
ties of orbital angular momentum (Topic 7F). is an example (with Z = 1). The Coulomb potential energy of an
electron in a hydrogenic atom of atomic number Z and there-
fore nuclear charge Ze is (see Energy: A first look)

When an electric discharge is passed through gaseous hydrogen, Ze 2


the H2 molecules are dissociated and the energetically excited V (r )    (8A.2)
4 0 r
H atoms that are produced emit electromagnetic radiation at
a number of discrete frequencies (and therefore discrete wave- where r is the distance of the electron from the nucleus and ε0 is
numbers), producing a spectrum of a series of ‘lines’ (Fig. 8A.1). the electric constant (formerly the ‘vacuum permittivity’). The
hamiltonian for the entire atom, which consists of an electron
Visible and a nucleus of mass mN, is therefore
Wavelength, λ/nm
2000
1000
800
600
500

400

300

200

150

120

100

Hˆ = Eˆ k,electron + Eˆ k,nucleus + Vˆ (r )
Hamiltonian for a
2 2 2 2 Ze 2 hydrogenic atom
 (8A.3)
=− ∇e − ∇N −
2me 2mN 4 πε 0r

Balmer Lyman The subscripts e and N on ∇2 indicate differentiation with re-


spect to the electron or nuclear coordinates.
Analysis

Paschen

Brackett
8A.1(a) The separation of variables
Physical intuition suggests that the full Schrödinger equation
Figure 8A.1 The spectrum of atomic hydrogen. Both the observed ought to separate into two equations, one for the motion of
spectrum and its resolution into overlapping series are shown. the atom as a whole through space and the other for the mo-
Note that the Balmer series lies in the visible region. tion of the electron relative to the nucleus. The Schrödinger
8A Hydrogenic atoms 307

equation for the internal motion of the electron relative to the 0


nucleus is1 l≠0

Effective potential energy, Veff


2 2 Ze 2
     E 
Schrodinger
2 4  0 r
equation for a  (8A.4)
1 1 1 hydrogenic atom
  l=0
 me mN
where differentiation is now with respect to the coordinates of the
electron relative to the nucleus. The quantity μ is called the ­reduced
mass. The reduced mass is very similar to the electron mass be-
cause mN, the mass of the nucleus, is much larger than the mass of Radius, r

an electron, so 1/μ ≈ 1/me and therefore μ ≈ me. In all except the Figure 8A.2 The effective potential energy of an electron in
most precise work, the reduced mass can be replaced by me. the hydrogen atom. When the electron has zero orbital angular
Because the potential energy is centrosymmetric (independ- momentum, the effective potential energy is the Coulombic
ent of angle), the equation for the wavefunction is expected to potential energy. When the electron has non-zero orbital
be separable into radial and angular components, as in angular momentum, the centrifugal effect gives rise to a positive
contribution which is very large close to the nucleus. The l = 0 and
 (r , , )  R(r )Y ( , ) (8A.5) l ≠ 0 wavefunctions are therefore very different near the nucleus.

with R(r) the radial wavefunction and Y(θ,ϕ) the angular


wavefunction. The equation does separate, and the two contri-
butions to the wavefunction are solutions of two equations: in classical physics would be called the centrifugal force arising
from the angular momentum of the electron around the nu-
 2Y  l(l  1)Y  (8A.6a) cleus. The consequences are:

 2  d 2 R 2 dR  • When l = 0, the electron has no angular momentum,


     Veff R  ER  (8A.6b)
and the effective potential energy is purely Coulombic;
2   dr 2 r dr 
the force exerted on the electron is attractive at all radii
where (Fig. 8A.2).
Ze 2 l(l  1)2 • When l ≠ 0, the centrifugal term gives a positive contribu-
Veff (r )     (8A.6c) tion to the effective potential energy, corresponding to a
4  0 r 2 r 2
repulsive force at all radii.
Equation 8A.6a is the same as the Schrödinger equation for a
• When the electron is close to the nucleus (r ≈ 0), the cen-
particle free to move at constant radius around a central point
trifugal contribution to the potential energy, which is pro-
(Topic 7F). The allowed solutions are the spherical harmon-
portional to 1/r2, dominates the Coulombic contribution,
ics (Table 7F.1), and are specified by the quantum numbers l
which is proportional to 1/r; the net result is an effective
and ml. As explained in Topic 7F, the orbital angular quantum
repulsion of the electron from the nucleus.
number l specifies the magnitude of the orbital angular mo-
mentum and the magnetic quantum number ml specifies the It follows that the two effective potential energies, the one for
z-­component of that momentum (see below). Equation 8A.6b l = 0 and the one for l ≠ 0, are qualitatively very different close
is called the radial wave equation. The radial wave equation de- to the nucleus. However, they are similar at large distances be-
scribes the motion of a particle of mass μ in a one-dimensional cause the centrifugal contribution tends to zero more rapidly
region 0 ≤ r < ∞ where the potential energy is Veff(r). (as 1/r2) than the Coulombic contribution (as 1/r). Therefore,
the solutions with l = 0 and l ≠ 0 are expected to be quite differ-
8A.1(b) The radial solutions ent near the nucleus but similar far away from it.
Two features of the radial wavefunction are important:
Some features of the shapes of the radial wavefunctions can
be anticipated by examining the form of Veff(r). The first term • Close to the nucleus the radial wavefunction is pro-
in eqn 8A.6c is the Coulomb potential energy of the electron portional to (r/a0)l with a0 the Bohr radius, 52.9 pm
in the field of the nucleus. The second term stems from what (see below), and r/a0 < 1. The higher the orbital angular
momentum, the closer (r/a0)l remains to zero and it is less
1
likely that the electron will be found there (Fig. 8A.3).
See the first section of A deeper look 8A.1, available to read in the e-book
accompanying this text, for full details of this separation procedure and then • Far from the nucleus all radial wavefunctions approach
the second section for the calculations that lead to eqn 8A.6. zero exponentially.
308 8 Atomic structure and spectra

Note that the energies are independent of l and ml, so each level
l=0 is n2-fold degenerate.
So far, only the general form of the radial wavefunctions has
Wavefunction, ψ

been given. It is now time to show how they depend on vari-


ous fundamental constants and the atomic number of the atom.
1
They are most simply written in terms of the dimensionless
2 quantity ρ (rho), where
3
2 Zr me 4  0 2
 a a a0   (8A.9)
na  0 me e 2
Radius, r The Bohr radius, a0, has the value 52.9 pm; it is so called be-
Figure 8A.3 Close to the nucleus, orbitals with l = 1 are cause the same quantity appeared in Bohr’s early model of the
proportional to r, orbitals with l = 2 are proportional to r2, and hydrogen atom as the radius of the electron orbit of lowest en-
orbitals with l = 3 are proportional to r3. Electrons are progressively ergy. In practice, because me << mN (so me/μ ≈ 1) there is so
excluded from the neighbourhood of the nucleus as l increases. little difference between a and a0 that it is safe to use a0 in the
An orbital with l = 0 has a finite, non-zero value at the nucleus. definition of ρ for all atoms (even for 1H, a = 1.0005a0). In terms
of these quantities and with the various quantum numbers dis-
played, the radial wavefunctions for an electron with quantum
The detailed solution of the radial equation for the full range numbers n and l are the (real) functions
of radii shows how the form r l close to the nucleus blends into
the exponentially decaying form at great distances. It turns out Rn,l (r )  N n,l  l Ln,l (  )e   /2 Radial wavefunctions  (8A.10)
that the two regions are bridged by a polynomial in r and that
where Ln,l(ρ) is an associated Laguerre polynomial. These poly-
Dominant
close to the Bridges the two Dominant far
nomials have quite simple forms, such as 1, ρ, and 2 – ρ (they
nucleus ends of the function from the nucleus can be picked out in Table 8A.1). The factor Nn,l ensures that the
      
 (r ) 
R rl  (polynomial in r ) (decaying exponential in r) radial wavefunction is normalized to 1 in the sense that

(8A.7)
0
Rn,l (r )2 r 2 dr  1 (8A.11)

The radial wavefunction therefore has the form where the r2 comes from the volume element in spherical coor-
dinates; see The chemist’s toolkit 7F.2. Specifically, the compo-
R(r )  r l L(r )e  r
nents of eqn 8A.10 can be interpreted as follows:
with various constants and where L(r) is the bridging polyno- • The exponential factor ensures that the wavefunction
mial. That polynomial has the following features: approaches zero far from the nucleus.
• Close to the nucleus (r ≈ 0) the polynomial is a constant
and e−r ≈ 1, so R(r) ∝ r l. Table 8A.1 Hydrogenic radial wavefunctions
• Far from the nucleus the dominant term in the polyno-
n l Rn,l(r)
mial is proportional to r n−l−1, where n is an integer; so, 3/2
regardless of the value of l, all the wavefunctions of a given Z
1 0 2  e  /2
value of n are proportional to r n−1e−r and decay exponen- a
3/2
tially to zero in the same way.2 2 0
1 Z
(2   )e   / 2
81/ 2  a 
The detailed solution also shows that, for the wavefunction to 3/2
1 Z
be acceptable, the value of n that appears in the polynomial can 2 1  e  /2
241/ 2  a 
take only positive integral values, and specifically n = 1, 2,… 3/2
and the two other quantum numbers l and ml are confined to 1 Z
3 0 (6  6    2 )e   / 2
2431/ 2  a 
l = 0, 1, 2,…, n – 1 and ml = 0, ±1, ±2,…, ±l. The allowed ener-
gies are:
3/2
1 Z
3 1 (4   )  e   / 2
4861/ 2  a 
e 4 Z2 1 Z
3/2
En,l ,ml    2 Bound -state energies (8A.8) 3 2  2e  /2
32  0  n 24301/ 2  a 
2 2 2

ρ = (2Z/na)r with a = 4πε0ħ2/μe2. For an infinitely heavy nucleus (or one that may be
2
Note that exponential functions e−x always dominate simple powers, xn. assumed to be), μ = me and a = a0, the Bohr radius.
8A Hydrogenic atoms 309

2 0.15
0.8
n = 1, n = 2, n = 2,
l=0 l=0 l=1
1.5 0.6
0.1

R(r)/(Z/a0)3/2
R(r)/(Z/a0)3/2

R(r)/(Z/a0)3/2
0.4
1
0.2
0.05
0.5
0

0 –0.2 0
0 1 2 3 0 5 10 15 0 5 10 15
Zr/a0 Zr/a0 Zr/a0

0.4 0.1 0.05


n = 3, n = 3, n = 3,
l=0 l=1 l=2
0.3 0.04

0.05
R(r)/(Z/a0)3/2

0.2

R(r)/(Z/a0)3/2
R(r)/(Z/a0)3/2

0.03

0.1 0.02
0

0 0.01

–0.1 –0.05
0 7.5 15 22.5 0
0 7.5 15 22.5 0 7.5 15 22.5
Zr/a0 Zr/a0 Zr/a0

Figure 8A.4 The radial wavefunctions of the first few states of hydrogenic atoms of atomic number Z. Note that the orbitals with
l = 0 have a non-zero and finite value at the nucleus. The horizontal scales are different in each case: as the principal quantum number
increases, so too does the size of the orbital. Interact with the dynamic version of this graph in the e-book.

• The factor ρl ensures that (provided l > 0) the wavefunc- The probability density is therefore
tion vanishes at the nucleus.
Z3
• The associated Laguerre polynomial is a function that  12,0,0 (0, , ) 
a03
in general oscillates from positive to negative values and
accounts for the presence of radial nodes. The zero at r = 0 which evaluates to 2.15 × 10−6 pm−3 when Z = 1.
is not a radial node because the radial wavefunction does
not pass through zero at that point (because r cannot be
negative). Exercises
Expressions for some radial wavefunctions are given in E8A.1 The wavefunction for the ground state of a hydrogen atom is Ne
− r /a0
.
Table 8A.1 and illustrated in Fig. 8A.4. Finally, with the form Evaluate the normalization constant N.
of the radial wavefunction established, the total wavefunction, E8A.2 Evaluate the probability density at the nucleus of an electron with n = 2,
eqn 8A.5, in full dress becomes l = 0, ml = 0.
E8A.3 By differentiation of the 2s radial wavefunction, show that it has two
 n,l ,ml (r , , )  Rn,l (r )Yl ,ml ( , ) (8A.12) extrema in its amplitude, and locate them.

Brief illustration 8A.1 Atomic orbitals and their


8A.2
To calculate the probability density at the nucleus for an elec- energies
tron with n = 1, l = 0, and ml = 0, evaluate ψ at r = 0:
An atomic orbital is a one-electron wavefunction for an elec-
3/2 1/ 2
Z  1  tron in an atom, and for hydrogenic atoms has the form speci-
 1,0,0 (0, , )  R1,0 (0)Y0,0 ( , )  2    
fied in eqn 8A.12. Each hydrogenic atomic orbital is defined by
 a0   4  
310 8 Atomic structure and spectra

three quantum numbers, designated n, l, and ml. An electron Continuum


described by one of the wavefunctions in eqn 8A.12 is said to H+ + e– n
0
‘occupy’ that orbital. For example, an electron described by
–hcRH/9 3
the wavefunction ψ1,0,0 is said to ‘occupy’ the orbital with n = 1,
–hcRH/4 2
l = 0, and ml = 0.

Energy, E
Classically
8A.2(a) The specification of orbitals allowed
energies
Each of the three quantum numbers specifies a different attrib-
ute of the orbital: –hcRH 1
• The principal quantum number, n, specifies the energy of the
orbital (through eqn 8A.8); it takes the values n = 1, 2, 3,…. Figure 8A.5 The energy levels of a hydrogen atom. The values
are relative to an infinitely separated, stationary electron and a
• The orbital angular momentum quantum number, l,
proton.
specifies the magnitude of the angular momentum of the
electron as {l(l + 1)}1/2ћ, with l = 0, 1, 2,…, n − 1.
• The magnetic quantum number, ml, specifies the z-com-    m e4
R N   R R  2e 3 Rydberg constant  (8A.14)
ponent of the angular momentum as mlћ, with ml = 0, ±1, me 8 0 h c
±2,…, ±l.
where μ is the reduced mass of the atom and R ∞ is the Rydberg
Note how the value of the principal quantum number controls
constant; the constant R N is the value that constant takes for a
the maximum value of l, and how the value of l controls the
specified atom N (not nitrogen!), such as hydrogen, when N is
range of values of ml.
replaced by H and μ takes the appropriate value. Insertion of
the values of the fundamental constants into the expression for
8A.2(b) The energy levels R H gives almost exact agreement with the experimental value
for hydrogen. The only discrepancies arise from the neglect of
The energy levels predicted by eqn 8A.8 are depicted in
relativistic corrections (in simple terms, the increase of mass
Fig. 8A.5. Some important features are:
with speed), which the non-relativistic Schrödinger equation
• The energies, and also the separation of neighbouring ignores.
levels, are proportional to Z2, so the levels are four times
as wide apart (and the ground state four times lower in
Brief illustration 8A.2
energy) in He+ (Z = 2) than in H (Z = 1).
• All the energies given by eqn 8A.8 are negative. They refer The value of R ¥ is 109 737 cm−1. The reduced mass of a hydro-
to the bound states of the atom, in which the energy of the gen atom with mp = 1.672 62 × 10−27 kg and me = 9.109 38 ×
atom is lower than that of the infinitely separated, station- 10−31 kg is
ary electron and nucleus (which corresponds to the zero
memp (9.109 38  1031 kg )  (1.672 62  10 27 kg )
of energy).  
me  mp (9.109 38  1031 kg )  (1.672 62  1027 kg )
• There are also solutions of the Schrödinger equation with
positive energies. These solutions correspond to unbound  9.104 42  1031 kg
states of the electron, the states to which an electron is It then follows that
raised when it is ejected from the atom by a high-energy
31
collision or photon. The energies of the unbound electron 9.104 42  10 kg
R H   109 737 cm 1  109 677 cm 1
are not quantized and form the continuum states of the 9.109 38  1031 kg
atom. and that the ground state of the electron (n = 1, l = 0, ml = 0)
Equation 8A.8, which can be written as lies at

E1,0,0  hcR H  (6.626 08  1034 Js)


hcZ 2 R N  e 4
En,l ,ml  , RN  Bound -state energies  (8A.13) (2.997 945  1010 cms 1 )  (109 677 cm 1 )
n2 8 02 ch3
 2.178 70  1018 J
is consistent with the spectroscopic result summarized by
eqn 8A.1, with the Rydberg constant for the atom identified as or −2.178 70 aJ. This energy corresponds to −13.598 eV.
8A Hydrogenic atoms 311

8A.2(c) Ionization energies 110

The ionization energy, I, of an element is the minimum energy


required to remove an electron from the ground state, the state
100
of lowest energy, of one of its atoms in the gas phase. Because

~/(103 cm–1)
the ground state of hydrogen is the state with n = 1, with energy
E1  hcR H and the atom is ionized when the electron has been
excited to the level corresponding to n = ∞ (see Fig. 8A.5), the 90
energy that must be supplied is

I = hcR H (8A.15)
80
0 0.1 0.2
The value of I is 2.179 aJ (1 aJ = 10−18 J), which corresponds to 1/n2
13.60 eV.3
Figure 8A.6 The plot of the data in Example 8A.1 used to
determine the ionization energy of an atom (in this case, of H).
Example 8A.1 Measuring an ionization energy
spectroscopically
Self-test 8A.1 The emission spectrum of atomic deuterium
The emission spectrum of atomic hydrogen shows lines at shows lines at 15 238, 20 571, 23 039, and 24 380 cm−1, which
82 259, 97 492, 102 824, 105 292, 106 632, and 107 440 cm−1, correspond to transitions from successive upper states with
which correspond to transitions to the same lower state from n = 3, 4,… to the same lower state. Determine (a) the ioniza-
successive upper states with n = 2, 3,…. Determine the ioniza- tion energy of the lower state, (b) the ionization energy of the
tion energy of the lower state. ground state, (c) the mass of the deuteron (by expressing the
Rydberg constant in terms of the reduced mass of the electron
Collect your thoughts The spectroscopic determination of ioni-
and the deuteron, and solving for the mass of the deuteron).
zation energies depends on the identification of the ‘series limit’,
very sensitive to R D
the wavenumber at which the series terminates and becomes a
continuum. If the upper state lies at an energy −hcR H /n2, then
Answer: (a) 328.1 kJ mol−1, (b) 1312.4 kJ mol−1, (c) 2.8 × 10−27 kg, a result

the wavenumber of the photon emitted when the atom makes


a transition to the lower state, with energy Elower, is

I = −Elower
8A.2(d) Shells and subshells
RH Elower R I All the orbitals of a given value of n are said to form a single
ν =− − = − H2 +
n2 hc n hc
shell of the atom. In a hydrogenic atom (and only in a hydro-
A plot of the wavenumbers against 1/n2 should give a straight genic atom), all orbitals of given n, and therefore belonging to
line of slope −R H and intercept I/hc. Use software to calculate the same shell, have the same energy. It is common to refer to
a least-squares fit of the data in order to obtain a result that successive shells by letters:
reflects the precision of the data.
n  1 2 3 4
The solution The wavenumbers are plotted against 1/n2 in Specification of shells
K L M N
Fig. 8A.6. From the (least-squares) intercept, it follows that
I/hc = 109 679 cm−1, so the ionization energy is Thus, all the orbitals of the shell with n = 2 form the L shell of
the atom, and so on.
I  hc  (109 677 cm 1 )
The orbitals with the same value of n but different values of l
 (6.626 08  1034 Js)  (2.997 945  1010 cms 1 ) are said to form a subshell of a given shell. These subshells are
 (109 677 cm 1 ) also generally referred to by letters:
 2.1787  1018 J l  0 1 2 3 4 5 6
Specification of subshells
−1
or 2.1787 aJ, corresponding to 1312.1 kJ mol (the negative of s p d f g h i
the value of E calculated in Brief illustration 8A.2).
All orbitals of the same subshell have the same energy in all
3
Ionization energies are sometimes referred to as ionization potentials. That kinds of atoms, not only hydrogenic atoms. After l = 3 the
is incorrect, but not uncommon. If the term is used at all, it should denote the letters run alphabetically (j is not used because in some lan-
electrical potential difference through which an electron must be moved for the
guages i and j are not distinguished). Figure 8A.7 is a version
change in its potential energy to be equal to the ionization energy, and reported
in volts: the ionization energy of hydrogen is 13.60 eV; its ionization potential of Fig. 8A.5 which shows the subshells explicitly. Because l can
is 13.60 V. range from 0 to n − 1, giving n values in all, it follows that there
312 8 Atomic structure and spectra

n s p d f of these quantum numbers when n = 1). From Table 8A.1 and



4 4s [1]
with Y0,0 = (1/4π)1/2 (Table 7F.1) it follows that (for Z = 1):
4p [3] 4d [5] 4f [7]
3
3s[1] 3p [3] 3d [5] 1
 3 1/ 2
e  r /a0  (8A.16)
2 (a0 )
2s [1] 2p[3]
Energy

This wavefunction has the following features:


• It is independent of angle and has the same value at all
points of constant radius; that is, the 1s orbital (the s orbital
with n = 1, and in general ns) is ‘spherically symmetrical’.
1s [1] • It decays exponentially from a maximum value of 1/(πa03 )1/ 2
1 at the nucleus (at r = 0), so that the probability density of
the electron is greatest at the nucleus itself.
Figure 8A.7 The energy levels of a hydrogenic atom showing the
subshells and (in square brackets) the numbers of orbitals in each The general form of the ground-state wavefunction can be un-
subshell. All orbitals of a given shell have the same energy. derstood by considering the contributions of the potential and
kinetic energies to the total energy of the electron in the atom.
The more compact the orbital, the shorter the average distance
Subshells of the electron from the nucleus, and therefore the lower (more
s p d negative) its average potential energy. However, the more com-
pact the wavefunction, the greater its curvature and hence the
M shell, n = 3 greater its average kinetic energy. This dependence of energy on
the compactness of the wavefunction is illustrated in Fig 8A.9.
Orbitals The most compact wavefunction has the lowest potential en-
L shell, n = 2 ergy but the highest kinetic energy and a high energy overall.
The least compact wavefunction has a low kinetic energy but a
high potential energy and a high energy overall. The wavefunc-
Shells

K shell, n = 1 tion that lies between these two has a moderately high kinetic
energy, a moderately low potential energy, and a total energy
that is lowest of the three. It corresponds to the observed
Figure 8A.8 The organization of orbitals (small squares) into
ground-state wavefunction.
subshells (characterized by l) and shells (characterized by n).
One way of depicting the probability density of the elec-
tron is to represent |ψ|2 by the density of shading (Fig. 8A.10).
A s­ impler procedure is to show only the boundary surface, the
are n subshells of a shell with principal quantum number n. The
organization of orbitals in the shells is summarized in Fig. 8A.8.
The number of orbitals in a shell of principal quantum num-
ber n is n2, so in a hydrogenic atom each energy level is n2-fold Low potential energy
but
­degenerate. high kinetic energy
Wavefunction, ψ

a
Lowest total energy
Brief illustration 8A.3
c Low kinetic energy
but
When n = 1 there is only one subshell, that with l = 0, and that high potential energy
subshell contains only one orbital, with ml = 0 (the only value b
of ml permitted). When n = 2, there are four orbitals, one in the
s subshell with l = 0 and ml = 0, and three in the l = 1 subshell
Radius, r
with ml = +1, 0, −1. When n = 3 there are nine orbitals (one
with l = 0, three with l = 1, and five with l = 2). Figure 8A.9 The balance of kinetic and potential energies that
accounts for the structure of the ground state of hydrogenic
atoms. (a) The sharply curved but localized orbital has high
8A.2(e) s Orbitals mean kinetic energy, but low mean potential energy; (b) the
mean kinetic energy is low, but the potential energy is not very
The orbital occupied in the ground state is the one with n = 1 favourable; (c) the compromise of moderate kinetic energy and
(and therefore with l = 0 and ml = 0, the only possible values moderately favourable potential energy.
8A Hydrogenic atoms 313

z where the wavefunction is normalized. You need to evaluate


z
the integral by using the wavefunctions given in Table 8A.1 and
dτ = r2dr sin θ dθ dϕ (The chemist’s toolkit 7F.2). The angular
parts of the wavefunction (Table 7F.1) are normalized in the
sense that
 2 2

x y
 
 0  0
Yl ,ml sin d d  1

y The relevant integral over r is given in the Resource section.


x
The solution With the wavefunction written in the form
ψ = RY, the integration (with the integral over the angular
(a) 1s (b) 2s
­variables, which is equal to 1, in the boxes) is

π 2π 2
Figure 8A.10 Representations of cross-sections through the 〈r〉 = ∫ ∫ ∫ rRn2,l Yl ,ml r 2 d r sinθ dθ dϕ = ∫ r 3 Rn2,l d r
0 0 0 0
(a) 1s and (b) 2s hydrogenic atomic orbitals in terms of their electron
probability densities (as represented by the density of shading). For a 1s orbital
3/2
Z
R1,0  2   e  Zr /a0
z
 a0 
Hence
Integral
 E.3

4 Z 3  3 2 Zr /a0 4Z 3 3! 3a
r   3  r e dr  3   0
a0 0 a0 (2 Z /a0 )4 2 Z
x
y
Self-test 8A.2 Evaluate the mean radius of a 3s orbital. Hint:
Use mathematical software.
Answer: 27a0/2Z

Figure 8A.11 The boundary surface of a 1s orbital, within which


there is a 90 per cent probability of finding the electron. All s
orbitals have spherical boundary surfaces. All s orbitals are spherically symmetric, but differ in the
number of radial nodes. For example, the 1s, 2s, and 3s orbit-
als have 0, 1, and 2 radial nodes, respectively. In general, an ns
surface that mirrors the shape of the orbital and captures a high orbital has n − 1 radial nodes. As n increases, the radius of the
proportion (typically about 90 per cent) of the electron prob- spherical boundary surface that essentially captures a given
ability. Formally, computer software typically constructs an iso- fraction of the probability also increases.
surface, a surface of constant probability density that essentially
captures a high proportion of the probability and mirrors the
Brief illustration 8A.4
characteristic shape of the orbital. Most of the orbital depictions
in this text are notional: they will be referred to as ‘boundary The radial nodes of a 2s orbital lie at the locations where the
surfaces’ but apart from those for s orbitals they are only general associated Laguerre polynomial factor (Table 8A.1) is equal to
indications of their shapes. For the 1s orbital, the boundary sur- zero. In this case the factor is simply 2 − ρ so there is a node at
face is a sphere centred on the nucleus (Fig. 8A.11). ρ = 2. For a 2s orbital, ρ = Zr/a0, so the radial node occurs at
r = 2a0/Z (see Fig. 8A.4).

Example 8A.2 Calculating the mean radius of an orbital 8A.2(f ) Radial distribution functions
Calculate the mean radius of a hydrogenic 1s orbital. The wavefunction yields, through the value of |ψ|2, the proba-
Collect your thoughts The mean radius is the expectation bility of finding an electron in any region. As explained in Topic
value 7B, |ψ|2 is a probability density (dimensions: 1/volume) and can
be interpreted as a (dimensionless) probability when multi-
r    r d   r |  |2d plied by the (infinitesimal) volume of interest. Imagine a probe
314 8 Atomic structure and spectra

with a fixed volume dτ, and sensitive to electrons, that can this shell is the probability density at the radius r multiplied by
move around near the nucleus of a hydrogenic atom. Because the volume of the probe, or |ψ(r)|2 × 4πr2dr. This expression has
the probability density in the ground state of the atom is pro- the form P(r)dr, where
portional to e −2 Zr /a0 , the reading from the detector decreases
exponentially as the probe is moved out along any radius but Radial distribution function
P (r )  4 r 2 | (r ) |2 [s orbitals only]
 (8A.17a)
is constant if the probe is moved on a circle of constant radius
(Fig. 8A.12). The function P(r) is called the radial distribution function (in
Now consider the total probability of finding the electron this case, for an s orbital). It is also possible to devise a more
anywhere between the two walls of a spherical shell of thick- general expression which applies to orbitals that are not spheri-
ness dr at a radius r. The sensitive volume of the probe is now cally symmetrical.
the volume of the shell (Fig. 8A.13), which is 4πr2dr (the prod-
uct of its surface area, 4πr2, and its thickness, dr). Note that the
volume probed increases with distance from the nucleus and How is that done? 8A.1 Deriving the general form of the
is zero at the nucleus itself, when r = 0. The probability that the
radial distribution function
electron will be found between the inner and outer surfaces of
The probability of finding an electron in a volume element
dτ when its wavefunction is ψ = RY is |RY|2dτ with dτ =
r2dr sinθdθdϕ. The total probability of finding the electron at
any angle in a shell of radius r and thickness dr is the integral of
Probability, ψ*ψdτ

this probability over the entire surface, and is written P(r)dr; so


r
π 2π 2
P(r )dr = ∫ ∫
0 0
R(r )2 Yl ,ml r 2 d r sinθ dθ dϕ

Because the spherical harmonics are normalized to 1 (the boxed


integration, as in Example 8A.2, gives 1), the final result is

Radius, r (8A.17b)
P(r ) = r 2 R(r )2
Radial distribution function
Figure 8A.12 For a 1s orbital a constant-volume electron-sensitive
[general form]
detector (the small cube) gives its greatest reading at the nucleus,
and a smaller reading elsewhere. The same reading is obtained
anywhere on a circle of given radius at any orientation: the s
orbital is spherically symmetrical.
The radial distribution function is a probability density in
the sense that, when it is multiplied by dr, it gives the probabil-
ity of finding the electron anywhere between the two walls of a
0.6
Radial distribution function, P/(Z/a0)

spherical shell of thickness dr at the radius r. For a 1s orbital,

4 Z 3 2 2 Zr /a0
P (r )  r e  (8A.18)
0.4 r a03
This expression can be interpreted as follows:
0.2 • Because r2 = 0 at the nucleus, P(0) = 0. The volume of the
shell is zero when r = 0 so the probability of finding the
electron in the shell is zero.
0 • As r → ∞, P(r) → 0 on account of the exponential term.
0 1 2 3 4
Radius, Zr/a0 The wavefunction has fallen to zero at great distances
from the nucleus and there is little probability of finding
Figure 8A.13 The radial distribution function P(r) is the the electron even in a large shell.
probability density that the electron will be found anywhere
in a shell of radius r; the probability itself is P(r)dr, where dr is • The increase in r2 and the decrease in the exponential
the thickness of the shell. For a 1s electron in hydrogen, P(r) is a ­factor means that P passes through a maximum at an
maximum when r is equal to the Bohr radius a0. The value of P(r)dr intermediate radius (see Fig. 8A.13); it marks the most
is equivalent to the reading that a detector shaped like a spherical probable radius at which the electron will be found
shell of thickness dr would give as its radius is varied. regardless of direction.
8A Hydrogenic atoms 315

5/2
Example 8A.3 Calculating the most probable radius 1 Z
ψ 2,1,0 = R2,1 (r )Y1,0 (θ ,ϕ ) = r cos θ e − Zr /2a
0

4(2π )1/2 a0
Calculate the most probable radius, rmp, at which an electron (8A.19a)
will be found when it occupies a 1s orbital of a hydrogenic = r cosθ f (r )
atom of atomic number Z, and tabulate the values for the one-
electron species from H to Ne9+. where f(r) is a function only of r. Because in spherical polar
­coordinates z = r cos θ (The chemist’s toolkit 7F.2), this wave-
Collect your thoughts You need to find the radius at which
the radial distribution function of the hydrogenic 1s orbital
function may also be written
has a maximum value by solving dP/dr = 0. If there are sev-
 2,1,0  zf (r ) (8A.19b)
eral maxima, you should choose the one corresponding to the
greatest amplitude. All p orbitals with ml = 0 and any value of n have wavefunctions
The solution The radial distribution function is given in eqn of this form, but f(r) depends on the value of n. This way of
8A.18. It follows that writing the orbital is the origin of the name ‘pz orbital’: the gen-
eral form of its boundary surface is shown in Fig. 8A.14. The
dP 4 Z 3  2 Zr 2  2 Zr /a0 8rZ 3  Zr  2 Zr /a0 wavefunction is zero everywhere in the xy-plane, where z = 0,
 3  2r  e  3 1   e
dr a0  a0  a0  a0  so the xy-plane is a nodal plane of the orbital: the wavefunction
changes sign on going from one side of the plane to the other.
This function is zero other than at r = 0 where the term in The wavefunctions of 2p orbitals with ml = ±1 are
parentheses is zero, which is at
5/2
a 1 Z
rmp = 0  2,1, 1  R2,1 (r )Y1, 1 ( , )   1/2   r sin e  i e  Zr / 2a0
Z 8  a0 
Then, with a0 = 52.9 pm, the most probable radii are  1 (8A.20)
  1/ 2 r sin e  i f (r )
2
H He+ Li2+ Be3+ B4+ C5+ N6+ O7+ F8+ Ne9+
rmp/pm 52.9 26.5 17.6 13.2 10.6 8.82 7.56 6.61 5.88 5.29 In Topic 7D it is explained that a particle described by a com-
plex wavefunction has net motion. In the present case, the
Notice how the 1s orbital is drawn towards the nucleus as functions correspond to non-zero angular momentum about
the nuclear charge increases. At uranium the most probable the z-axis: e+iϕ corresponds to clockwise rotation when viewed
radius is only 0.58 pm, almost 100 times closer than for hydro- from below, and e−iϕ corresponds to counterclockwise rotation
gen. (On a scale where rmp = 10 cm for H, rmp = 1 mm for U.) (from the same viewpoint). They have zero amplitude where
However, extending this result to very heavy atoms neglects θ = 0 and 180° (along the z-axis) and maximum amplitude at 90°,
important relativistic effects that complicate the calculation.
Self-test 8A.3 Find the most probable distance of a 2s electron
from the nucleus in a hydrogenic atom. Hint: Use mathemati- θ = 90°
cal software.
ϕ=0
atom as its energy increases.
ϕ = 90°
Answer: (3 + 51/2)a0/Z = 5.24a0/Z; this value reflects the expansion of the

+


8A.2(g) p Orbitals +
+

All three 2p orbitals have l = 1, and therefore the same magni- –


tude of angular momentum; they are distinguished by different pz px py
values of ml, the quantum number that specifies the component z
θ
of angular momentum around a chosen axis (conventionally y
taken to be the z-axis). The orbital with ml = 0, for instance,
x ϕ
has zero angular momentum around the z-axis. Its angular
variation is given by the spherical harmonic Y1,0, which is pro- Figure 8A.14 The general shapes of the boundary surfaces of 2p
portional to cos θ (see Table 7F.1). Therefore, the probability orbitals. A nodal plane passes through the nucleus and separates
density, which is proportional to cos2 θ, has its maximum value the two lobes of each orbital. The dark and light lobes denote
on either side of the nucleus along the z-axis (at θ = 0 and 180°, regions of opposite sign of the wavefunction. The angles that
where cos2 θ = 1). Specifically, the wavefunction of a 2p orbital specify the nodal planes are shown. All p orbitals have boundary
with ml = 0 is surfaces like those shown here.
316 8 Atomic structure and spectra

which is in the xy-plane. To draw the functions it is usual to


z
represent them by forming the linear combinations +
y
+
eiϕ + e−iϕ = 2cos ϕ x – –

1 –
ψ2 p = (−ψ 2,1,+1 + ψ 2,1, −1 ) = r sinθ cosϕ f (r ) = x f (r )
x
21/2 +
eiϕ − e−iϕ = 2isin ϕ (8A.21) +

i dz2 dx2–y2
ψ2 p = (ψ + ψ 2,1, −1 ) = r sinθ sin ϕ f (r ) = y f (r ) –
y
21/2 2,1,+1 +

These linear combinations correspond to zero orbital angular + –


– +

momentum around the z-axis, as they are superpositions of –
+
+
+
states with equal and opposite values of ml. The px orbital has – +
– +
the same shape as a pz orbital, but it is directed along the x-axis +

(see Fig. 8A.14); the py orbital is similarly directed along the dxy dyz dzx
y-axis. The wavefunction of any p orbital of a given shell can be
Figure 8A.15 The general form of the boundary surfaces of 3d
written as a product of x, y, or z and the same function f (which
orbitals. The purple and yellow areas (which are mirrored in the
depends on the value of n).
insert) denote regions of opposite sign of the wavefunction. All d
orbitals have boundary surfaces like those shown here.
8A.2(h) d Orbitals
When n = 3, l can be 0, 1, or 2. As a result, this shell consists These linear combinations give rise to the notation dxy, dyz, etc.
of one 3s orbital, three 3p orbitals, and five 3d orbitals. Each for the d-orbitals. With the exception of the d z 2 orbital, each
value of the quantum number ml = 0, ±1, ±2 corresponds to combination has two angular nodes that divide the orbital into
a different value of the component of angular momentum four lobes. For the d z 2 orbital, the two angular nodes combine
about the z-axis. As for the p orbitals, d orbitals with opposite to give a conical surface that separates the main lobes from a
values of ml (and hence opposite senses of motion around smaller toroidal component encircling the nucleus.
the z-axis) may be combined in pairs to give real wavefunc-
tions, and the boundary surfaces of the resulting functions
are shown in Fig. 8A.15. The real linear combinations have
Exercises
the following forms, with the function f(r) depending on the E8A.4 What subshells and orbitals are available in the M shell?

value of n: E8A.5 What is the orbital angular momentum (as multiples of ħ) of an electron
in the orbitals (i) 1s, (ii) 3s, (iii) 3d? Give the numbers of angular and radial
nodes in each case.
 d xy  xyf (r )  d yz  yzf (r )  dzx  zxf (r )
 (8A.22) E8A.6 Write down the expression for the radial distribution function of a 2p
d 2  (x  y ) f (r )  d 2  1211/2 (3z 2  r 2 ) f (r )
1
2
2 2
electron in a hydrogenic atom of atomic number Z and identify the radius at
x  y2 z
which the electron is most likely to be found.

Checklist of concepts
☐ 1. The Schrödinger equation for a hydrogenic atom sepa- ☐ 6. The ionization energy of an element is the minimum
rates into angular and radial equations. energy required to remove an electron from the ground
☐ 2. Close to the nucleus the radial wavefunction is propor- state of one of its atoms.
tional to r l; far from the nucleus all hydrogenic wave- ☐ 7. Orbitals of a given value of n form a shell of an atom,
functions approach zero exponentially. and within that shell orbitals of the same value of l form
☐ 3. An atomic orbital is a one-electron wavefunction for an subshells.
electron in an atom. ☐ 8. Orbitals of the same shell all have the same energy in
☐ 4. An atomic orbital is specified by the values of the hydrogenic atoms; orbitals of the same subshell of a shell
­quantum numbers n, l, and ml. are degenerate in all types of atoms.
☐ 5. The energies of the bound states of hydrogenic atoms ☐ 9. s Orbitals are spherically symmetrical and have nonzero
are proportional to −Z2/n2. probability density at the nucleus.
8A Hydrogenic atoms 317

☐ 10. A radial distribution function is the probability density ☐ 11. There are three p orbitals in a given subshell; each one
for the distribution of the electron as a function of dis- has one angular node.
tance from the nucleus. ☐ 12. There are five d orbitals in a given subshell; each one has
two angular nodes.

Checklist of equations
Property Equation Comment Equation number
  R H (1/n  1/n ) R H is the Rydberg constant for hydrogen 8A.1
 2 2
Wavenumbers of the spectral lines of a 1 2
hydrogen atom (expressed as a wavenumber)
Bohr radius a0 = 4πε0ħ2/mee2 a0 = 52.9 pm 8A.9
Wavefunctions of hydrogenic atoms  n,l ,ml (r , , )  Rn,l (r )Yl ,ml ( , ) Yl ,ml are spherical harmonics 8A.12

Energies of hydrogenic atoms En,l ,ml  hcZ R N /n2 , R N   e 4 /8 02ch3


2
R N  R  , the Rydberg constant; 8A.13
μ = memN/(me + mN)
Radial distribution function P(r) = r2R(r)2 P(r) = 4πr2ψ2 for s orbitals 8A.17b
TOPIC 8B Many-electron atoms

The individual orbitals can be assumed to resemble the hydro-


➤ Why do you need to know this material? genic orbitals based on nuclei with charges modified by the
Many-electron atoms are the building blocks of all com- presence of all the other electrons in the atom. This assumption
pounds, and to understand their properties, including their can be justified if, to a first approximation, electron–electron
ability to participate in chemical bonding, it is essential to interactions are ignored.
understand their electronic structure. Moreover, a knowl-
edge of that structure explains the structure of the periodic How is that done? 8B.1 Justifying the orbital
table and all that it summarizes.
approximation
➤ What is the key idea? Consider a system in which the hamiltonian for the energy is
Electrons occupy the orbitals that result in the lowest the sum of two contributions, one for electron 1 and the other
energy of the atom, subject to the requirements of the =
for electron 2: Hˆ Hˆ + Hˆ . In an actual two-electron atom
1 2
Pauli exclusion principle. (such as a helium atom, with Z = 2), there is an additional term
(proportional to 1/r12, where r12 is the distance between the two
➤ What do you need to know already? electrons) corresponding to their interaction:
This Topic builds on the account of the structure of hydro- H1 ˆ H2 ˆ
     
genic atoms (Topic 8A), especially their shell structure. 2 2 2
ˆ  2 2e  2 2e 2 e2
H =− ∇1 − − ∇2 − +
2me 4 πε 0r1 2me 4 πε 0r2 4 πε 0r12

In the orbital approximation the final term is ignored. Then


the task is to show that if ψ(r1) is an eigenfunction of Ĥ1 with
A many-electron atom (or polyelectron atom) is an atom with energy E1, and ψ(r2) is an eigenfunction of Ĥ 2 with energy E2,
more than one electron. The Schrödinger equation for a many- then the product Ψ(r1,r2) = ψ(r1)ψ(r2) is an eigenfunction of
electron atom is complicated because all the electrons interact the combined hamiltonian Ĥ . To do so write
with one another. One very important consequence of these HˆΨ (r1 ,=
r2 ) (Hˆ 1 + Hˆ 2 )ψ (r1 )ψ (r2 )
interactions is that orbitals of the same value of n but differ-
ˆ ˆ
ent values of l are no longer degenerate. Moreover, even for a ψ (r2 )H1ψ (r1 )
    ψ (r1 )H2ψ (r2 )
 
helium atom, with just two electrons, it is not possible to find = Hˆ 1ψ (r1 )ψ (r2 ) + Hˆ 2ψ (r1 )ψ (r2 )
analytical expressions for the orbitals and energies, so it is nec-
E1ψ (r1 ) E2ψ (r2 )
essary to use various approximations. 
   
  
ˆ ˆ
= ψ (r2 ) H1ψ (r1 ) + ψ (r1 ) H 2ψ (r2 )
ψ (r2 )E1ψ (r1 ) + ψ (r1 )E2ψ (r2 ) =
= (E1 + E2 )ψ (r1 )ψ (r2 )

8B.1 The orbital approximation = EΨ (r1 ,r2 )

where E = E1 + E2, which is the desired result. Note how each


The wavefunction of a many-electron atom is a very compli-
hamiltonian operates on only its ‘own’ wavefunction. If the
cated function of the coordinates of all the electrons, written electrons interact (as they do in fact), then the term in 1/r12
as Ψ(r1,r2,…), where ri is the vector from the nucleus to elec- must be included, and the proof fails. Therefore, this descrip-
tron i (uppercase psi, Ψ, is commonly used to denote a many- tion is only approximate, but it is a useful model for discussing
electron wavefunction). The orbital approximation states that the chemical properties of atoms and is the starting point for
a reasonable first approximation to this exact wavefunction is more sophisticated descriptions of atomic structure.
obtained by thinking of each electron as occupying its ‘own’ or-
bital, and writing
The orbital approximation can be used to express the elec-
 (r1 ,r2 ,)   (r1 ) (r2 ) Orbital approximation (8B.1)
tronic structure of an atom by reporting its configuration, a
8B Many-electron atoms 319

statement of its occupied orbitals (usually, but not necessarily,


in its ground state). Thus, as the ground state of a hydrogenic
atom consists of the single electron in a 1s orbital, its configura-
tion is reported as 1s1 (read ‘one-ess-one’).
A He atom has two electrons. The first electron occupies a 1s (b)
hydrogenic orbital, but because Z = 2 that orbital is more com-
pact than in H itself. The second electron joins the first in the 1s
orbital, so the electron configuration of the ground state of He
is 1s2 (read ‘one-ess-two’).
(a) (c)
Brief illustration 8B.1
Figure 8B.1 (a) The experimental arrangement for the Stern–
According to the orbital approximation, each electron in He Gerlach experiment: the magnet provides an inhomogeneous
occupies a hydrogenic 1s orbital of the kind given in Topic 8A. field. (b) The classically expected result. (c) The observed outcome
Anticipating (see below) that the electrons experience an effec- using silver atoms.
tive nuclear charge Zeffe rather than the actual charge on the
nucleus with Z = 2 (specifically, as seen later, a charge 1.69e
rather than 2e), then the two-electron wavefunction of the i­ nhomogeneous magnetic field (Fig. 8B.1). The idea behind the
atom is experiment was that each atom possesses a certain electronic
 1s (r1 )  1s (r2 ) angular momentum and (because moving charges generate
   
3/2 3/2 a magnetic field) as a result behaves like a small bar magnet
Zeff  Zeff r1 /a0 Zeff
 (r1 ,r2 )  e  e  Zeff r2 /a0 aligned with the direction of the angular momentum vector. As
(a03 )1/ 2 (a03 )1/ 2
the atoms pass through the inhomogeneous magnetic field they
3
Zeff are deflected, with the deflection depending on the relative ori-
 e  Zeff (r1  r2 )/a0
a03 entation of the applied magnetic field and the atomic magnet.
The classical expectation is that the electronic angular mo-
There is nothing particularly mysterious about a two-electron mentum, and hence the resulting magnet, can be oriented in
wavefunction: in this case it is a simple exponential function of
any direction. Each atom would be deflected into a direction
the distances of the two electrons from the nucleus.
that depends on the orientation and the beam should spread
out into a broad band as it emerges from the magnetic field. In
contrast, the expectation from quantum mechanics is that the
Exercise angular momentum, and hence the atomic magnet, has only
discrete orientations (Topic 7F). Each of these orientations re-
E8B.1 Construct the wavefunction for an excited state of the He atom with
sults in the atoms being deflected in a specific direction, so the
configuration 1s12s1. Use Zeff = 2 for the 1s electron and Zeff = 1 for the 2s
electron. beam should split into a number of sharp bands, each corre-
sponding to a different orientation of the angular momentum
of the electrons in the atom.
In their first experiment, Stern and Gerlach appeared to con-
8B.2 The Pauli exclusion principle firm the classical prediction. However, the experiment is dif-
ficult because collisions between the atoms in the beam blur
It is tempting to suppose that the electronic configurations of the the bands. When they repeated the experiment with a beam of
atoms of successive elements with atomic numbers Z = 3, 4,…, very low intensity (so that collisions were less frequent), they
and therefore with Z electrons, are simply 1sZ. That, however, observed discrete bands, and so confirmed the quantum pre-
is not the case. The reason lies in two aspects of nature: that diction. However, Stern and Gerlach observed two bands of Ag
electrons possess ‘spin’ and that they must obey the very funda- atoms in their experiment. This observation seems to conflict
mental ‘Pauli principle’. with one of the predictions of quantum mechanics, because an
angular momentum l gives rise to 2l + 1 orientations, which is
equal to 2 only if l = 12 , contrary to the requirement that l is an
8B.2(a) Spin integer. The conflict was resolved by the suggestion that the an-
The quantum mechanical property of electron spin, the gular momentum they were observing was not due to orbital
possession of an intrinsic angular momentum, was identi- angular momentum (the motion of an electron around the
fied by an experiment performed by Otto Stern and Walther atomic nucleus) but arose instead from the rotation of the elec-
Gerlach in 1921, who shot a beam of silver atoms through an tron about its own axis, its ‘spin’.
320 8 Atomic structure and spectra

only electrons, this angular momentum is  34    0.866, or


1/ 2
ms = –½
9.13 × 10−35 J s. The component on the z-axis is msħ, which for
a spin- 12 particle is ± 12 , or ±5.27 × 10−35 J s.
ms = +½

Figure 8B.2 The vector representation of the spin of an Particles with half-integral spin are called fermions and
electron. The length of the side of the cone is 31/2/2 units and the those with integral spin (including 0) are called bosons. Thus,
projections on to the z-axis are ± 21 units. electrons and protons are fermions; photons are bosons. It is a
very deep feature of nature that all the elementary particles that
The spin of an electron does not have to satisfy the same constitute matter are fermions whereas the elementary parti-
boundary conditions as those for a particle circulating through cles that transmit the forces that bind fermions together are all
space around a central point, so the quantum number for spin bosons. Photons, for example, transmit the electromagnetic
angular momentum is subject to different restrictions. The spin force that binds together electrically charged particles. Matter,
quantum number s is used in place of the orbital angular mo- therefore, is an assembly of fermions held together by forces
mentum quantum number l (Topic 7F; like l, s is a non-negative conveyed by bosons.
number) and ms, the spin magnetic quantum number, is used
in place of ml for the projection on the z-axis. The magnitude of
the spin angular momentum is {s(s + 1)}1/2ħ and the component
8B.2(b) The Pauli principle
msħ is restricted to the 2s + 1 values ms = s, s − 1,…, −s. To ac- With the concept of spin established, it is possible to resume
count for Stern and Gerlach’s observation, s = 12 and ms   12 .1 discussion of the electronic structures of atoms. Lithium, with
The detailed analysis of the spin of a particle is sophisticated Z = 3, has three electrons. The first two occupy a 1s orbital
and shows that the property should not be taken to be an a­ ctual drawn even more closely than in He around the more highly
spinning motion. It is better to regard ‘spin’ as an intrinsic charged nucleus. The third electron, however, does not join the
property like mass and charge: every electron has exactly the first two in the 1s orbital because that configuration is forbid-
same value and the magnitude of the spin angular momentum den by the Pauli exclusion principle:
of an electron cannot be changed. However, the picture of an
No more than two electrons may occupy any given orbital,
actual spinning motion can be very useful when used with care.
and if two do occupy one orbital, then their spins must be
In the vector model of angular momentum (Topic 7F), the spin
paired.
may lie in two different orientations (Fig. 8B.2). One orienta-
tion corresponds to ms   12 (this state is often denoted α or ↑); Electrons with paired spins, denoted ↑↓, have zero net spin an-
the other orientation corresponds to ms   12 (this state is de- gular momentum because the spin of one electron is cancelled
noted β or ↓). by the spin of the other. Specifically, one electron has ms   12
Other elementary particles have characteristic spin. For ex- the other has ms   12 . In the vector model they are orientated
ample, protons and neutrons are spin- 12 particles (i.e. s = 12 ). on their respective cones so that the resultant spin is zero
Because the masses of a proton and a neutron are so much (Fig. 8B.3). The exclusion principle is the key to the structure
greater than the mass of an electron, yet they all have the same of complex atoms, to chemical periodicity, and to molecular
spin angular momentum, the classical picture would be of these structure. It was proposed by Wolfgang Pauli in 1925 when he
two particles spinning much more slowly than an electron.
Some mesons, another variety of fundamental particle, are
spin-1 particles (i.e. s = 1), as are some atomic nuclei, but for
our purposes the most important spin-1 particle is the photon.
The importance of photon spin in spectroscopy is explained in ms = +1/2
Topic 11A; nuclear spin is the basis of nuclear magnetic reso-
nance (Topic 12A).
ms = –1/2
Brief illustration 8B.2

The magnitude of the spin angular momentum, like any angu-


lar momentum, is {s(s + 1)}1/2ħ. For any spin- 12 particle, not Figure 8B.3 Electrons with paired spins have zero resultant spin
angular momentum. They can be represented by two vectors that
1
You will sometimes see the quantum number s used in place of ms, and lie at an indeterminate position on the cones shown here, but
written s   12 . That is wrong: like l, s is never negative and denotes the magni- wherever one lies on its cone, the other points in the opposite
tude of the spin angular momentum. For the z-component, use ms. direction; their resultant is zero.
8B Many-electron atoms 321

was trying to account for the absence of some lines in the spec-  (1) (2) (1) (2)  (1) (2) (1) (2)
 (8B.4)
trum of helium. Later he was able to derive a very general form  (1) (2)  (1,2)  (1) (2)  (1,2)
of the principle from theoretical considerations.
The Pauli exclusion principle is a special case of a general Step 2 Apply the Pauli principle
statement called the Pauli principle: The Pauli principle states that for a wavefunction to be accept-
able (for electrons), it must change sign when the electrons
When the labels of any two identical fermions are are exchanged. In each case, exchanging the labels 1 and 2
exchanged, the total wavefunction changes sign; when the converts ψ(1)ψ(2) into ψ(2)ψ(1), which is the same, because
labels of any two identical bosons are exchanged, the sign the order of multiplying the functions does not change the
of the total wavefunction remains the same. value of the product. The same is true of α(1)α(2) and β(1)β(2).
By ‘total wavefunction’ is meant the entire wavefunction, in- Therefore, ψ(1)ψ(2)α(1)α(2) and ψ(1)ψ(2)β(1)β(2) are not
cluding the spin of the particles. It is possible to show that the allowed, because they do not change sign. When the labels are
Pauli principle implies the Pauli exclusion principle. exchanged the combination σ+(1,2) becomes
1
  (2,1)  { (2) (1)   (2) (1)}    (1,2)
21/ 2
How is that done? 8B.2 Justifying the Pauli exclusion
because the central term is simply the original function written
principle in a different order. The product ψ(1)ψ(2)σ+(1,2) is therefore
Consider the wavefunction for two electrons, Ψ(1,2). The Pauli also disallowed. Finally, consider σ−(1,2):
principle implies that it is a fact of nature (which has its roots 1
in the theory of relativity) that the wavefunction must change   (2,1)  { (2) (1)   (2) (1)}
21/ 2
sign if the labels 1 and 2 are interchanged wherever they occur 1
in the function:   1/ 2 { (1) (2)   (1) (2)}    (1,2)
2
 (2,1)   (1,2) (8B.2) The combination ψ(1)ψ(2)σ−(1,2) therefore does change sign
Suppose the two electrons occupy the same orbital ψ, then in (it is ‘antisymmetric’) and is acceptable.
the orbital approximation the overall spatial wavefunction is Step 3 Summarize the results
ψ(r1)ψ(r2), which for simplicity will be denoted ψ(1)ψ(2). To
apply the Pauli principle, it is necessary to consider the total In summary, only one of the four possible states is allowed by
wavefunction, the wavefunction including spin. The rest of the the Pauli principle: the one that survives has paired α and β
argument uses the following steps. spins. This is the content of the Pauli exclusion principle.

Step 1 Write the total wavefunctions


For each electron the spin wavefunction can be α, corre- The Pauli exclusion principle (but not the more general Pauli
sponding to ms   12 , or β, corresponding to ms   12 . For two principle) is irrelevant when the orbitals occupied by the elec-
electrons there are four possible spin wavefunctions: both α, trons are different, and both electrons may then have, but need
denoted α(1)α(2), both β, denoted β(1)β(2), and one α and the not have, the same spin state. In each case the overall wave-
other β, denoted either α(1)β(2) or α(2)β(1). Because it is not function must still be antisymmetric and must satisfy the Pauli
possible to know which electron is α and which is β, in the last principle itself.
case it is appropriate to express the spin contribution to the Now returning to lithium, Li (Z = 3), the third electron can-
total wavefunction as the (normalized) linear combinations2 not enter the 1s orbital because that orbital is already full: the
1 K shell (the shell with n = 1, Topic 8A) is complete and the two
  (1,2)  { (1) (2)   (1) (2)} electrons form a closed shell, a shell in which all the orbitals are
21/ 2  (8B.3)
1 fully occupied. Because a similar closed shell is characteristic
  (1,2)  1/ 2 { (1) (2)   (1) (2)} of the He atom, it is commonly denoted [He]. The third elec-
2
tron cannot enter the K shell and must occupy the next avail-
These combinations allow one spin to be α and the other β with
able orbital, which is one with n = 2 and hence belonging to the
equal probability; the former corresponds to parallel spins (the
L shell (which consists of the four orbitals with n = 2). It is now
individual spins do not cancel) and the latter to paired spins
necessary to decide whether the next available orbital is the 2s
(the individual spins cancel). The total wavefunction of the
orbital or a 2p orbital, and therefore whether the lowest energy
system is therefore the product of the orbital part and one of
the four spin contributions:
configuration of the atom is [He]2s1 or [He]2p1.

Exercise
2
A stronger justification for taking these linear combinations is that they
correspond to eigenfunctions of the total spin operators S2 and Sz, with MS = 0
and, respectively, S = 1 and 0. E8B.2 How many electrons can occupy subshells with l = 3?
322 8 Atomic structure and spectra

8B.3 The building-up principle

Radial distribution function, P


Unlike in hydrogenic atoms, the 2s and 2p orbitals (and, in gen-
eral, the subshells of a given shell) do not have the same energy
in many-electron atoms.
3p

8B.3(a) Penetration and shielding


3s
An electron in a many-electron atom experiences a Coulombic
repulsion from all the other electrons present. If the electron is 0
0 4 8 12 16 20
at a distance r from the nucleus, it experiences an average re- Radius, Zr/a0
pulsion that can be represented by a point negative charge lo-
cated at the nucleus and equal in magnitude to the total charge Figure 8B.5 An electron in an s orbital (here a 3s orbital) is more
likely to be found close to the nucleus than an electron in a p
of all the other electrons within a sphere of radius r (Fig. 8B.4).
orbital of the same shell (note the closeness of the innermost
This property is a conclusion of classical electrostatics, where
peak of the 3s orbital to the nucleus at r = 0). Hence an s electron
the effect of a spherical distribution of charge can be repre- experiences less shielding and is more tightly bound than a p
sented by a point charge of the same magnitude located at its electron of the same shell.
centre. The effect of this point negative charge is to reduce the
full charge of the nucleus from Ze to Zeffe, the effective nuclear
charge. In everyday parlance, Zeff itself is commonly referred to sphere defined by the location of the electron of interest con-
as the ‘effective nuclear charge’. The electron is said to experi- tribute to shielding, an s electron experiences less shielding
ence a shielded nuclear charge, and the difference between Z than a p electron. Consequently, as a result of the combined ef-
and Zeff is called the shielding constant, σ: fects of penetration and shielding, an s electron is more tightly
bound than a p electron of the same shell. Similarly, a d electron
Zeff  Z   Nuclear shielding (8B.5) penetrates less than a p electron of the same shell (recall that a
d orbital is proportional to (r/a0)2 close to the nucleus, whereas
The electrons do not actually ‘block’ the full Coulombic attrac- a p orbital is proportional to r/a0, so the amplitude of a d orbital
tion of the nucleus: the shielding constant is simply a way of is smaller there than that of a p orbital), and therefore experi-
expressing the net outcome of the nuclear attraction and the ences more shielding.
electronic repulsions in terms of a single equivalent charge at Shielding constants for different types of electrons in atoms
the centre of the atom. have been calculated from wavefunctions obtained by numeri-
The shielding constant is different for s and p electrons be- cal solution of the Schrödinger equation, leading to the effective
cause they have different radial distribution functions and nuclear charges shown in Table 8B.1. In general, valence-shell s
therefore respond to the other electrons in the atom to differ- electrons do experience higher effective nuclear charges than p
ent extents (Fig. 8B.5). An s electron has a greater penetration electrons, although there are some discrepancies.
through inner shells than a p electron, in the sense that an s
Table 8B.1 Effective nuclear charge*
electron is more likely to be found close to the nucleus than a
p electron of the same shell. Because only electrons inside the Element Z Orbital Zeff
He 2 1s 1.6875
C 6 1s 5.6727
No net effect of Electron location 2s 3.2166
these electrons 2p 3.1358
r
* More values are given in the Resource section.

Net effect equivalent Brief illustration 8B.3


to a point charge at
the nucleus
The effective nuclear charge for 1s, 2s, and 2p electrons in a
Figure 8B.4 An electron at a distance r from the nucleus carbon atom are 5.6727, 3.2166, and 3.1358, respectively. The
experiences a Coulombic repulsion from all the electrons within radial distribution functions for these orbitals (Topic 8A) are
a sphere of radius r. This repulsion is equivalent to that from a generated by forming P(r) = r2R(r)2, where R(r) is the radial
point negative charge located on the nucleus. The negative charge wavefunction, which is given in Table 8A.1. The three radial
reduces the effective nuclear charge of the nucleus from Ze to Zeffe. distribution functions, taking into account the effective nuclear
8B Many-electron atoms 323

3.5 the bare nucleus of atomic number Z, and then feed into the
3
orbitals Z electrons in succession. The order of occupation, fol-
lowing the shells and their subshells arranged in order of in-
2.5 creasing energy, is
1s
2
P(r)a0

0
0
1s 2s 2p 3s 3p 4s 3d 4 p 5s 4d 5p 6s
1.5

1
Each orbital may accommodate up to two electrons.
2s
0.5
2p Brief illustration 8B.4
0
0 1 2 3 4 5
r/a0 Consider the carbon atom, for which Z = 6 and there are six
electrons to accommodate. Two electrons enter and fill the 1s
Figure 8B.6 The radial distribution functions for electrons in a
orbital, two enter and fill the 2s orbital, leaving two electrons to
carbon atom, as calculated in Brief illustration 8B.3.
occupy the orbitals of the 2p subshell. Hence the ground-state
configuration of C is 1s22s22p2, or more succinctly [He]2s22p2,
with [He] the helium-like 1s2 core.
charges, are plotted in Fig. 8B.6. As can be seen (especially in
the magnified view close to the nucleus), the s orbital has great-
er penetration than the p orbital. The average radii of the 2s and It is possible to be more precise about the configuration of a
2p orbitals are 99 pm and 84 pm, respectively, which shows that carbon atom than in Brief illustration 8B.4. The last two elec-
the average distance of a 2s electron from the nucleus is greater trons are expected to occupy different 2p orbitals because they
than that of a 2p orbital. To account for the lower energy of the are then farther apart on average and repel each other less
2s orbital, the extent of penetration is more important than the than if they were in the same orbital. Thus, one electron can be
average distance from the nucleus. thought of as occupying the 2px orbital and the other the 2py or-
bital (the x, y, z designation is arbitrary, and it would be equally
valid to use the complex forms of these orbitals), and the lowest
The consequence of penetration and shielding is that the energy configuration of the atom is [He]2s2 2p1x 2p1y . The same
energies of subshells of a shell in a many-electron atom (those rule applies whenever degenerate orbitals of a subshell are
with the same values of n but different values of l) in general lie available for occupation. Thus, another rule of the building-up
in the order s < p < d < f. The individual orbitals of a given sub- principle is:
shell (those with the same value of l but different values of ml)
Electrons occupy different orbitals of a given subshell
remain degenerate because they all have the same radial char-
before doubly occupying any one of them.
acteristics and so experience the same effective nuclear charge.
To complete the Li story, consider that, because the shell For instance, nitrogen (Z = 7) has the ground-state configura-
with n = 2 consists of two subshells, with the 2s subshell lower tion [He]2s2 2p1x 2p1y 2p1z , and only at oxygen (Z = 8) is a 2p or-
in energy than the 2p subshell, the third electron occupies the bital doubly occupied, giving [He]2s2 2p2x 2p1y 2p1z .
2s orbital (the only orbital in that subshell). This occupation re- When electrons occupy orbitals singly it is necessary to in-
sults in the ground-state configuration 1s22s1, with the central voke Hund’s maximum multiplicity rule:
nucleus surrounded by a complete helium-like shell of two 1s
An atom in its ground state adopts a configuration with
electrons, and around that a more diffuse 2s electron. The elec-
the greatest number of unpaired electrons.
trons in the outermost shell of an atom in its ground state are
called the valence electrons because they are largely responsi- The explanation of Hund’s rule is subtle, but it reflects the quan-
ble for the chemical bonds that the atom forms (and ‘valence’, tum mechanical property of spin correlation. In essence, the
as explained in Focus 9, refers to the ability of an atom to form effect of spin correlation is to allow the atom to shrink slightly
bonds). Thus, the valence electron in Li is a 2s electron and its when the spins are parallel, so the electron–nucleus interac-
other two electrons belong to its core. tion is improved. As a consequence, in the ground state of the
carbon atom, the two 2p electrons have parallel spins, all three
2p electrons in the N atoms have parallel spins, and the two 2p
8B.3(b) Hund’s rules electrons in different orbitals in the O atom have parallel spins
The extension of the argument used to account for the struc- (the two in the 2px orbital are necessarily paired). The effect can
tures of H, He, and Li is called the building-up principle, or the be explained by considering the Pauli principle and showing
Aufbau principle, from the German word for ‘building up’, and that electrons with parallel spins behave as if they have a ten-
should be familiar from introductory courses. In brief, ­imagine dency to stay apart, and hence repel each other less.
324 8 Atomic structure and spectra

How is that done? 8B.3 Exploring the origins of spin i­ nteraction of the electrons in the valence shell, and penetration
arguments alone are no longer reliable.
correlation Calculations of the type discussed in Topic 8B show that
Suppose electron 1 is in orbital a and described by a wavefunc- for the atoms from scandium to zinc the energies of the 3d
tion ψa(r1), and electron 2 is in orbital b with wavefunction orbitals are always lower than the energy of the 4s orbital,
ψb(r2). Then, in the orbital approximation, the joint spatial in spite of the greater penetration of a 4s electron. However,
wavefunction of the electrons is the product Ψ = ψa(r1)ψb(r2). spectroscopic results show that Sc has the configuration
However, this wavefunction is not acceptable, because it sug- [Ar]3d14s2, not [Ar]3d3 nor [Ar]3d24s1. To understand this
gests that it is possible to know which electron is in which observation, consider the nature of electron–electron repul-
orbital. According to quantum mechanics, the correct descrip- sions in 3d and 4s orbitals. Because the average distance of a
tion is either of the two following wavefunctions: 3d electron from the nucleus is less than that of a 4s electron,
1 two 3d electrons are so close together that they repel each
  { a (r1 ) b (r2 )   b (r1 ) a (r2 )} other more strongly than two 4s electrons do, and 3d2 and 3d3
21/ 2
configurations are disfavoured. As a result, Sc has the config-
According to the Pauli principle, because Ψ+ is symmetrical
uration [Ar]3d14s2 rather than the two alternatives, for then
under interchange of the electrons, it must be combined with
the strong electron–electron repulsions in the 3d orbitals
an antisymmetric spin wavefunction (the one denoted σ−). That
combination corresponds to a spin-paired state. Conversely, are minimized. The total energy of the atom is lower despite
Ψ− is antisymmetric, so it must be combined with one of the the cost of allowing electrons to populate the high energy 4s
three symmetric spin wavefunctions. These three symmetric orbital (Fig. 8B.7). The effect just described is generally true
states correspond to electrons with parallel spins (see Topic 8C for scandium to zinc, so their electron configurations are of
for an explanation of this point). the form [Ar]3dn4s2, where n = 1 for scandium and n = 10
Now consider the behaviour of the two wavefunctions Ψ± for zinc. Two notable exceptions, which are observed experi-
when one electron approaches another, and r1 = r2. As a result, mentally, are Cr, with electron configuration [Ar]3d54s1, and
Ψ− vanishes, which means that there is zero probability of find- Cu, with electron configuration [Ar]3d104s1. At gallium, these
ing the two electrons at the same point in space when they have complications disappear and the building-up principle is
parallel spins. In contrast, the wavefunction Ψ+ does not vanish used in the same way as in preceding periods. Now the 4s and
when the two electrons are at the same point in space. Because 4p subshells constitute the valence shell, and the period ter-
the two electrons have different relative spatial distributions minates with krypton. Because 18 electrons have intervened
depending on whether their spins are parallel or not, it follows since argon, this row is the first ‘long period’ of the periodic
that their Coulombic interaction is different, and hence that table.
the two states described by these wavefunctions have different At this stage it becomes apparent that sequential occupation
energies, with the spin-parallel state lower in energy than the of the orbitals in successive shells results in periodic similarities
spin-paired state. in the electronic configurations. This periodicity of structure
accounts for the formulation of the periodic table.
The vertical columns of the periodic table are called groups
Neon, with Z = 10, has the configuration [He]2s22p6, which and (in the modern convention) numbered from 1 to 18.
completes the L shell. This closed-shell configuration is de- Successive rows of the periodic table are called periods, the
noted [Ne], and acts as a core for subsequent elements. The
next electron must enter the 3s orbital and begin a new shell,
so an Na atom, with Z = 11, has the configuration [Ne]3s1. Like
lithium with the configuration [He]2s1, sodium has a single s
electron outside a complete core. This analysis hints at the ori-
Energy

gin of chemical periodicity. The L shell is completed by eight


electrons, so the element with Z = 11 (Na) should have similar
properties to the element with Z = 3 (Li). Likewise, Mg (Z = 12)
should be similar to Be (Z = 4), and so on, up to the noble gases
He (Z = 2), Ne (Z = 10), and Ar (Z = 18).
At potassium (Z = 19) the next orbital in line for occupa-
Figure 8B.7 Strong electron–electron repulsions in the 3d orbitals
tion is 4s: this orbital is brought below 3d by the effects of pen- are minimized in the ground state of Sc if the atom has the
etration and shielding, and the ground state configuration is configuration [Ar]3d14s2 (shown on the left) instead of [Ar]3d24s1
[Ar]4s1. Calcium (Z = 20) is likewise [Ar]4s2. At this stage the (shown on the right). The total energy of the atom is lower when it
five 3d orbitals are in line for occupation, but there are com- has the [Ar]3d14s2 configuration despite the cost of populating the
plications arising from the energy changes arising from the high energy 4s orbital.
8B Many-electron atoms 325

number of the period being equal to the principal quantum to the nucleus. The increase in nuclear charge is partly
number of the valence shell. cancelled by the increase in the number of electrons, but
The periodic table is divided into s, p, d, and f blocks, accord- because electrons are spread over a region of space, one
ing to the subshell that is last to be occupied in the formula- electron does not fully shield one nuclear charge, so the
tion of the electronic configuration of the atom. The members increase in nuclear charge dominates.
of the d block (specifically the members of Groups 3–11 in the • The increase in atomic radius down a group (despite the
d block) are also known as the transition metals; those of the f increase in nuclear charge) is explained by the fact that the
block (which is not divided into numbered groups) are some- valence shells of successive periods correspond to higher
times called the inner transition metals. The upper row of the principal quantum numbers. That is, successive periods
f block (Period 6) consists of the lanthanoids (still commonly correspond to the start and then completion of successive
the ‘lanthanides’) and the lower row (Period 7) consists of the (and more distant) shells of the atom that surround each
actinoids (still commonly the ‘actinides’). other like the successive layers of an onion. The need to
The configurations of cations of elements in the s, p, and d occupy a more distant shell leads to a larger atom despite
blocks of the periodic table are derived by removing electrons the increased nuclear charge.
from the ground-state configuration of the neutral atom in a
specific order. First, remove valence p electrons, then valence A modification of the increase down a group is encountered
s electrons, and then as many d electrons as are necessary to in Period 6, for the radii of the atoms in the d block and in the
achieve the specified charge. The configurations of anions of following atoms of the p block are not as large as would be ex-
the p-block elements are derived by continuing the building-up pected by simple extrapolation down the group. The reason can
procedure and adding electrons to the neutral atom until the be traced to the fact that in Period 6 the f orbitals are in the
configuration of the next noble gas has been reached. process of being occupied. An f electron is a very inefficient

Brief illustration 8B.5 Table 8B.2 Atomic radii of main-group elements, r/pm

Li Be B C N O F
Because the configuration of vanadium is [Ar]3d34s2, the V2+ 157 112 88 77 74 66 64
cation has the configuration [Ar]3d3. It is reasonable to remove
Na Mg Al Si P S Cl
the more energetic 4s electrons in order to form the cation, but 191 160 143 118 110 104 99
it is not obvious why the [Ar]3d3 configuration is preferred in
K Ca Ga Ge As Se Br
V2+ over the [Ar]3d14s2 configuration, which is found in the 235 197 153 122 121 117 114
isoelectronic Sc atom. Calculations show that the energy dif-
Rb Sr In Sn Sb Te I
ference between [Ar]3d3 and [Ar]3d14s2 depends on Zeff. As Zeff 250 215 167 158 141 137 133
increases, transfer of a 4s electron to a 3d orbital becomes more Cs Ba Tl Pb Bi Po
favourable because the electron–electron repulsions are com- 272 224 171 175 182 167
pensated by attractive interactions between the nucleus and the
electrons in the spatially compact 3d orbital. Indeed, calcula-
tions reveal that, for a sufficiently large Zeff, [Ar]3d3 is lower
in energy than [Ar]3d14s2. This conclusion explains why V2+ 300
Cs
has a [Ar]3d3 configuration and also accounts for the observed Rb
[Ar]4s03dn configurations of the M2+ cations of Sc through Zn. K
Atomic radius, r/pm

Eu Yb
200
Na Pb Ac
Lu
8B.3(c) Atomic and ionic radii Li
Zn
The atomic radius of an element is half the distance between
Lanthanoids
I Am
100
the centres of neighbouring atoms in a solid (such as Cu) or, for Cl
Br Po

non-metals, in a homonuclear molecule (such as H2 or S8). As F


seen in Table 8B.2 and Fig. 8B.8, atomic radii tend to decrease
from left to right across a period of the periodic table, and in- 0
1 20 40 60 80 100
crease down each group. These general trends are justified as Atomic number, Z
follows:
Figure 8B.8 The variation of atomic radius through the periodic
• The decrease across a period can be traced to the increase table. Note the contraction of radius following the lanthanoids in
in nuclear charge, which draws the electrons in closer Period 6 (following Lu, lutetium).
326 8 Atomic structure and spectra

shielder of nuclear charge (for reasons connected with its radial Brief illustration 8B.6
extension), and as the atomic number increases from La to Lu,
there is a considerable contraction in radius. By the time the d The Ca2+, K+, and Cl− ions have the configuration [Ar].
block resumes (at hafnium, Hf), the poorly shielded but con- However, their radii differ because they have different nuclear
siderably increased nuclear charge has drawn in the surround- charges. The Ca2+ ion has the largest nuclear charge, so it has
ing electrons, and the atoms are compact. They are so compact, the strongest attraction for the electrons and the smallest
that the metals in this region of the periodic table (iridium to radius. The Cl− ion has the lowest nuclear charge of the three
lead) are very dense. The reduction in radius below that ex- ions and, as a result, the largest radius.
pected by extrapolation from preceding periods is called the
lanthanide contraction.
The ionic radius of an element is its share of the distance 8B.3(d) Ionization energies and electron
between neighbouring ions in an ionic solid. That is, the dis-
affinities
tance between the centres of a neighbouring cation and anion
is the sum of the two ionic radii. The size of the ‘share’ leads The minimum energy necessary to remove an electron from
to some ambiguity in the definition. One common definition a many-electron atom in the gas phase is the first ionization
sets the ionic radius of O2− equal to 140 pm, but there are other ­energy, I1, of the element. The second ionization energy, I2,
scales, and care must be taken not to mix them. Ionic radii also is the minimum energy needed to remove a second electron
vary with the number of counterions (ions of opposite charge) (from the singly charged cation). The variation of the first ioni-
around a given ion; unless otherwise stated, the values in this zation energy through the periodic table is shown in Fig. 8B.9
text have been corrected to correspond to an environment of and some numerical values are given in Table 8B.4.
six counterions. The electron affinity, Eea, is the energy released when an
When an atom loses one or more valence electrons to form electron attaches to a gas-phase atom (Table 8B.5). In a com-
a cation, the remaining atomic core is smaller than the par- mon, logical (given its name), but not universal convention
ent atom. Therefore, a cation is invariably smaller than its (which is adopted here), the electron affinity is positive if en-
parent atom. For example, the atomic radius of Na, with the ergy is released when the electron attaches to the atom. That is,
configuration [Ne]3s1, is 191 pm, but the ionic radius of Na+, Eea > 0 implies that electron attachment is exothermic.
with the configuration [Ne], is only 102 pm (Table 8B.3). Like
atomic radii, cation radii increase down each group because
30
electrons are occupying shells with higher principal quantum
numbers. He
An anion is larger than its parent atom because the electrons Ne
Ionization energy, I/eV

added to the valence shell repel one another. Without a com- 20


pensating increase in the nuclear charge, which would draw the Ar
electrons closer to the nucleus and each other, the ion expands. Kr Xe Rn
The variation in anion radii shows the same trend as that for Hg
10
atoms and cations, with the smallest anions at the upper right
of the periodic table, close to fluorine (Table 8B.3).
Li Na Rb Cs Tl
0
0 20 40 60 80 100
Table 8B.3 Ionic radii, r/pm* Atomic number, Z

Li+(4) Be2+(4) B3+(4) N3− O2−(6) F−(6) Figure 8B.9 The first ionization energies of the elements plotted
59 27 12 171 140 133 against atomic number.
Na+(6) Mg2+(6) Al3+(6) P3− S2−(6) Cl−(6)
102 72 53 212 184 181
K+(6) Ca2+(6) Ga3+(6) As3−(6) Se2−(6) Br−(6) Table 8B.4 First and second ionization energies*
138 100 62 222 198 196
Rb+(6) Sr2+(6) In3+(6) Te2−(6) I−(6) Element I1/(kJ mol−1) I2/(kJ mol−1)
149 116 79 221 220 H 1312
Cs+(6) Ba2+(6) Tl3+(6) He 2372 5251
167 136 88
Mg 738 1451
* Numbers in parentheses are the coordination numbers of the ions, the numbers of species Na 496 4562
(for example, counterions, solvent molecules) around the ions. Values for ions without a
coordination number stated are estimates. More values are given in the Resource section. * More values are given in the Resource section.
8B Many-electron atoms 327

Table 8B.5 Electron affinities, Ea/(kJ mol−1)* invariably endothermic, so Eea is negative. The incoming elec-
tron is repelled by the charge already present. Electron affinities
Element Ea/(kJ mol−1) Element Ea/(kJ mol−1)
are also small, and may be negative, when an electron enters
Cl 349 an orbital that is far from the nucleus (as in the heavier alkali
F 322 metal atoms) or is forced by the Pauli principle to occupy a new
H 73 shell (as in the noble gas atoms).
O 141 O− −844

* More values are given in the Resource section. Exercises


E8B.3 Write the ground-state electron configurations of the d-metals from
As will be familiar from introductory chemistry, ionization
scandium to zinc.
energies and electron affinities show periodicities. The former 2+
E8B.4 Write the electronic configuration of the Ni ion.
is more regular and concentrated on here:
E8B.5 Consider the atoms of the Period 2 elements of the periodic table.
• Lithium has a low first ionization energy because its outer- Predict which element has the lowest first ionization energy.
most electron is well shielded from the nucleus by the core
(Zeff = 1.3, compared with Z = 3).
• The ionization energy of Be (Z = 4) is greater but that of
B is lower because in the latter the outermost electron 8B.4 Self-consistent field orbitals
occupies a 2p orbital and is less strongly bound than if it
had been a 2s electron. The preceding treatment of the electronic configuration of
many-electron species is only approximate because of the
• The ionization energy increases from B to N on account of
complications introduced by electron–electron interactions.
the increasing effective nuclear charge.
However, computational techniques are available that give reli-
• However, the ionization energy of O is less than would be able approximate solutions for the wavefunctions and energies.
expected by simple extrapolation. The explanation is that The techniques were originally introduced by D.R. Hartree (be-
at oxygen a 2p orbital must become doubly occupied, and fore computers were available) and then modified by V. Fock to
the electron–electron repulsions are increased above what take into account the Pauli principle correctly. In broad outline,
would be expected by simple extrapolation along the row. the Hartree–Fock self-consistent field (HF-SCF) procedure is
In addition, the loss of a 2p electron results in a configu- as follows.
ration with a half-filled subshell (like that of N), which is Start with an idea of the structure of the atom as suggested
an arrangement of low energy, so the energy of O+ + e− is by the building-up principle. In the Ne atom, for instance, the
lower than might be expected, and the ionization energy principle suggests the configuration 1s22s22p6 with the orbit-
is correspondingly low too. (The kink is less pronounced als approximated by hydrogenic atomic orbitals with the ap-
in the next row, between phosphorus and sulfur, because propriate effective nuclear charges. Now consider one of the
their orbitals are more diffuse.) 2p electrons. A Schrödinger equation can be written for this
• The values for O, F, and Ne fall roughly on the same line, electron by ascribing to it a potential energy due to the nuclear
the increase of their ionization energies reflecting the attraction and the average repulsion from the other electrons.
increasing attraction of the more highly charged nuclei for Although the equation is for the 2p orbital, that repulsion,
the outermost electrons. and therefore the equation, depends on the wavefunctions of
• The outermost electron in sodium (Z = 11) is 3s. It is far all the other occupied orbitals in the atom. To solve the equa-
from the nucleus, and the latter’s charge is shielded by the tion, guess an approximate form of the wavefunctions of all the
compact, complete neon-like core, with the result that other orbitals and then solve the Schrödinger equation for the
Zeff ≈ 2.5. As a result, the ionization energy of Na is sub- 2p orbital. The procedure is then repeated for the 1s and 2s
stantially lower than that of Ne (Z = 10, Zeff ≈ 5.8). orbitals. This sequence of calculations gives the form of the 2p,
2s, and 1s orbitals, and in general they will differ from the set
• The periodic cycle starts again along this row, and the
used to start the calculation. These improved orbitals can be
variation of the ionization energy can be traced to similar
used in another cycle of calculation, and a second improved
reasons.
set of orbitals and a better energy are obtained. The recycling
Electron affinities are greatest close to fluorine, for the in- continues until the orbitals and energies obtained are insig-
coming electron enters a vacancy in a compact valence shell nificantly different from those used at the start of the current
and can interact strongly with the nucleus. The attachment of cycle. The solutions are then self-consistent and accepted as
an electron to an anion (as in the formation of O2− from O−) is solutions of the problem.
328 8 Atomic structure and spectra

1s
2p
Figure 8B.10 shows plots of some of the HF-SCF radial
2s distribution functions for sodium. They show the grouping
Radial distribution function, P

of electron density into shells, as was anticipated by the early


chemists, and the differences of penetration as discussed above.
These calculations therefore support the qualitative discussions
that are used to explain chemical periodicity. They also consid-
erably extend that discussion by providing detailed wavefunc-
K L M tions and precise energies.
3s

0 2 4 6 8
Radius, r/a0

Figure 8B.10 The radial distribution functions for the orbitals of


Na based on HF-SCF calculations. Note the shell-like structure,
with the 3s orbital outside the inner concentric K and L shells.

Checklist of concepts
☐ 1. In the orbital approximation, each electron is regarded ☐ 8. The atomic radius of an element is half the distance
as being described by its own wavefunction; the overall between the centres of neighbouring atoms in a solid or
wavefunction of a many-electron atom is the product of in a homonuclear molecule.
the orbital wavefunctions. ☐ 9. The ionic radius of an element is its share of the distance
☐ 2. The configuration of an atom is the statement of its between neighbouring ions in an ionic solid.
occupied orbitals. ☐ 10. The first ionization energy is the minimum energy
☐ 3. The Pauli exclusion principle, a special case of the Pauli necessary to remove an electron from a many-electron
principle, limits to two the number of electrons that can atom in the gas phase.
occupy a given orbital. ☐ 11. The second ionization energy is the minimum energy
☐ 4. In many-electron atoms, s orbitals lie at a lower energy needed to remove an electron from a singly charged
than p orbitals of the same shell due to the combined cation.
effects of penetration and shielding. ☐ 12. The electron affinity is the energy released when an
☐ 5. The building-up principle is a procedure for predicting electron attaches to a gas-phase atom.
the ground state electron configuration of an atom. ☐ 13. The atomic radius, ionization energy, and electron affin-
☐ 6. Electrons occupy different orbitals of a given subshell ity vary periodically through the periodic table.
before doubly occupying any one of them. ☐ 14. In the Hartree–Fock self-consistent field (HF-SCF)
☐ 7. An atom in its ground state adopts a configuration with procedure the Schrödinger equation is solved numeri-
the greatest number of unpaired electrons. cally and iteratively until the solutions no longer change
(to within certain criteria).

Checklist of equations
Property Equation Comment Equation number
Orbital approximation Ψ(r1,r2,…) = ψ(r1)ψ(r2) … 8B.1
Effective nuclear charge Zeff = Z − σ The charge is this number times e 8B.5
TOPIC 8C Atomic spectra

the angular momentum carried away by the photon. Thus, an


➤ Why do you need to know this material? electron in a d orbital (l = 2) cannot make a transition into an
A knowledge of the energies of electrons in atoms is s orbital (l = 0) because the photon cannot carry away enough
essential for understanding many chemical properties and angular momentum. Similarly, an s electron cannot make a
chemical bonding. transition to another s orbital, because there would then be no
change in the angular momentum of the electron to make up
➤ What is the key idea? for the angular momentum carried away by the photon. A more
The frequency and wavenumber of radiation emitted formal treatment of selection rules requires mathematical ma-
or absorbed when atoms undergo electronic transitions nipulation of the wavefunctions for the initial and final states of
provide detailed information about their electronic energy the atom.
states.
How is that done? 8C.1 Identifying selection rules
➤ What do you need to know already?
This Topic draws on knowledge of the energy levels of The underlying classical idea behind a spectroscopic transi-
hydrogenic atoms (Topic 8A) and the configurations of tion is that, for an atom or molecule to be able to interact
many-electron atoms (Topic 8B). In places, it uses the prop- with the electromagnetic field and absorb or create a photon
erties of angular momentum (Topic 7F). of frequency n, it must possess, at least transiently, a dipole
oscillating at that frequency. The consequences of this idea are
explored in the following steps.
Step 1 Write an expression for the transition dipole moment
The general idea behind atomic spectroscopy is straightfor- The transient dipole is expressed quantum mechanically as the
ward: lines in the spectrum (in either emission or absorption) transition dipole moment, μ fi, between the initial and final
occur when the electron distribution in an atom undergoes a states i and f, where1
transition, a change of state, in which its energy changes by ΔE.
This transition leads to the emission or is accompanied by the μ fi = ∫ψ f μˆψ i dτ  (8C.1)
absorption of a photon of frequency   E /h and wavenum-
and μ̂ is the electric dipole moment operator. For a one-­
ber   E /hc. In spectroscopy, transitions are said to take
electron atom μ̂ is multiplication by −er. Because r is a vector
place between two terms. Broadly speaking, a term is simply with components x, y, and z, μ̂ is also a vector, with compo-
another name for the energy level of an atom, but as this Topic nents μx = −ex, μy = −ey, and μz = −ez. If the transition dipole
progresses its full significance will become clear. moment is zero, then the transition is forbidden; the transition
is allowed if the transition dipole moment is non-zero.
Step 2 Formulate the integrand in terms of spherical harmonics
The spectra of hydrogenic
8C.1 To evaluate a transition dipole moment, consider each compo-
atoms nent in turn. For example, for the z-component,

 z ,fi  e  f z i d
Not all transitions between the possible terms are observed.
Spectroscopic transitions are allowed, if they can occur, or for- In spherical polar coordinates (see The chemist’s toolkit 7F.2)
bidden, if they cannot occur. A selection rule is a statement z = r cos θ. It is convenient to express cos θ in terms of spherical
about which transitions are allowed. harmonics. From Table 7F.1 it is seen that Y1,0 = (3/4π)1/2 cos θ,
The origin of selection rules can be identified by considering so it follows that cos θ = (4π/3)1/2Y1,0. Therefore z can be written
transitions in hydrogenic atoms. A photon has an intrinsic spin as z = r cos θ = (4π/3)1/2rY1,0. The wavefunctions for the initial
angular momentum corresponding to s = 1 (Topic 8B). Because
total angular momentum is conserved in a transition, the angu- 1
See our Physical chemistry: Quanta, matter, and change (2014) for a
lar momentum of the electron must change to compensate for detailed development of the form of eqn 8C.1.
330 8 Atomic structure and spectra

and final states are atomic orbitals of the form Rn,l (r )Yl ,ml ( , ) s p d s p d s p d
(Topic 8A). With these substitutions the integral becomes

∫ψ ⋆zψ dτ =
f i
z 15 328 (Hα)
ψ f* ψi dτ
1/2 20 571 (Hβ)
π 2π 4π 23 039 (Hγ)
∫ ∫∫ 102 824

Rnf , lf Ylf , ml ,f r Y1, 0 Rni , li Yli , ml, i r 2dr sinθ dθ dϕ 97 492 24 380 (Hδ)
0 0 0 3
82 259
This multiple integral is the product of three factors, an integral
over r and two integrals (boxed) over the angles, so the factors
Lyman Balmer Paschen
on the right can be grouped as follows:

∫ f

z i dτ =
Figure 8C.1 A Grotrian diagram that summarizes the appearance
1/2
4π π 2π and analysis of the spectrum of atomic hydrogen. The
∫ ∫ ∫

3
Rnf , lf r Rni, li dr Ylf ,ml, f Y1,0Yl i,ml ,i sinθ dθ d ϕ
3 0 0 0 wavenumbers of some transitions (in cm−1) are indicated.

Step 3 Evaluate the angular integral


It follows from the properties of the spherical harmonics that
the integral
 2 Exercises
I
0 
0
Ylf,ml ,f Yl ,mYli ,ml ,i sin d d
E8C.1 Identify the transition responsible for the shortest and longest
wavelength lines in the Lyman series.
is zero unless lf = li ± l and ml,f = ml,i + m. Because in the pre-
sent case l = 1 and m = 0, the angular integral, and hence the E8C.2 Calculate the wavelength, frequency, and wavenumber of the n = 2 →
n = 1 transition in He+.
z-component of the transition dipole moment, is zero unless
Δl = lf − li = ±1 and Δml = ml,f − ml,i = 0, which is a part of the E8C.3 Which of the following transitions are allowed in the electronic emission
set of selection rules. The same procedure, but considering the spectrum of a hydrogenic atom: (i) 2s → 1s, (ii) 2p → 1s, (iii) 3d → 2p?
x- and y-components, results in the complete set of rules:
Selection rules for
l  1 ml  0, 1 hydrogenic atoms  (8C.2)
The spectra of many-electron
8C.2
The principal quantum number n can change by any amount atoms
consistent with the value of Δl for the transition, because it
does not relate directly to the angular momentum.
The spectra of atoms rapidly become very complicated as the
number of electrons increases, in part because their energy lev-
els, their terms, are not given solely by the energies of the orbit-
Brief illustration 8C.1 als but depend on the interactions between the electrons.
To identify the orbitals to which a 4d electron may make radia-
tive transitions, first identify the value of l and then apply the 8C.2(a) Singlet and triplet terms
selection rule for this quantum number. Because l = 2, the final
orbital must have l = 1 or 3. Thus, an electron may make a
Consider the energy levels of a He atom, with its two electrons.
transition from a 4d orbital to any np orbital (subject to Δml = The ground-state configuration is 1s2, and an excited configu-
0, ±1) and to any nf orbital (subject to the same rule). However, ration is one in which an electron has been promoted into a
it cannot undergo a transition to any other orbital, such as an different orbital to give, for instance, the configuration 1s12s1.
ns or an nd orbital. The two electrons need not be paired because they occupy dif-
ferent orbitals. According to Hund’s maximum multiplicity
rule (Topic 8B), the state of the atom with the spins parallel lies
The selection rules and the atomic energy levels jointly ac- lower in energy than the state in which they are paired. Both
count for the structure of a Grotrian diagram (Fig. 8C.1), states are permissible, correspond to different terms, and can
which summarizes the energies of the terms and the transitions contribute to the spectrum of the atom.
between them. In some versions, the thicknesses of the transi- Parallel and antiparallel (paired) spins differ in their total
tion lines in the diagram denote their relative intensities in the spin angular momentum. In the paired case, the two spin
spectrum. momenta cancel, and there is zero net spin (as depicted in
8C Atomic spectra 331

1
S P
1 1
D F
1
S
3 3
P 3
D F
3
0
667.8 587.6
1083

Energy, E/eV
51.56
MS = +1 52.22
–10
53.71
58.44

–20

MS = 0 MS = –1

(a) S = 0 (b) S = 1 Figure 8C.3 Some of the transitions responsible for the spectrum
Figure 8C.2 (a) Electrons with paired spins have zero resultant of atomic helium. The labels give the wavelengths (in nanometres)
spin angular momentum (S = 0). They can be represented by of the transitions. The labels along the top of the diagram are term
two vectors that lie at an indeterminate position on the cones symbols introduced in Section 8C.2(c).
shown here, but wherever one lies on its cone, the other points
in the opposite direction; their resultant is zero. (b) When two
electrons have parallel spins, they have a non-zero total spin The origin of the energy difference lies in the effect of spin cor-
angular momentum (S = 1). There are three ways of achieving relation on the Coulombic interactions between electrons, as in
this resultant, which are shown by these vector representations.
the case of Hund’s maximum multiplicity rule for ground-state
Note that, whereas two paired spins are precisely antiparallel,
configurations (Topic 8B): electrons with parallel spins tend to
two ‘parallel’ spins are not strictly parallel. The notation S, MS is
explained later.
avoid each other. Because the Coulombic interaction between
electrons in an atom is strong, the difference in energies be-
tween singlet and triplet terms of the same configuration can be
Fig. 8C.2a). Its state is the one denoted σ− in the discussion of large. The singlet and triplet terms of the configuration 1s12s1 of
the Pauli principle (Topic 8B): He, for instance, differ by 6421 cm−1 (corresponding to 0.80 eV).
The spectrum of atomic helium is more complicated than
1 that of atomic hydrogen, but there are two simplifying features.
  (1, 2)  { (1) (2)   (1) (2)} (8C.3a)
21/ 2 One is that the only excited configurations to consider are of
The angular momenta of two parallel spins add to give a non- the form 1s1nl1; that is, only one electron is excited. Excitation
zero total spin. As illustrated in Fig. 8C.2(b), there are three of two electrons requires an energy greater than the ioniza-
ways of achieving non-zero total spin. The wavefunctions of the tion energy of the atom, so the He+ ion is formed instead of the
three spin states are the symmetric combinations introduced in doubly excited atom. Second, and as seen later in this Topic,
Topic 8B: no radiative transitions take place between singlet and triplet
terms because the relative orientation of the two electron spins
 (1) (2) cannot change during a transition. Thus, there is a spectrum
1 arising from transitions between singlet terms (including the
  (1, 2)  { (1) (2)   (1) (2)} (8C.3b)
21/ 2 ground state) and between triplet terms, but not between the
 (1) (2) two. Spectroscopically, helium behaves like two distinct spe-
cies. The Grotrian diagram for helium in Fig. 8C.3 shows the
The state of the He atom in which the two electrons are paired
two sets of transitions.
and their spins are described by eqn 8C.3a gives rise to a singlet
term. The alternative arrangement, in which the spins are par-
allel and are described by any of the three expressions in eqn 8C.2(b) Spin–orbit coupling
8C.3b, gives rise to a triplet term. The fact that the parallel ar-
An electron has a magnetic moment that arises from its spin.
rangement of spins in the triplet term of the 1s12s1 configura-
Similarly, an electron with orbital angular momentum (that is,
tion of the He atom lies lower in energy than the antiparallel
an electron in an orbital with l > 0) is in effect a circulating cur-
arrangement, the singlet term, can now be expressed by saying
rent, and possesses a magnetic moment that arises from its or-
that the triplet term of the 1s12s1 configuration of He lies lower
bital momentum. The interaction of the spin magnetic moment
in energy than the singlet term. This is a general conclusion and
with the magnetic moment arising from the orbital angular
applies to other atoms (and molecules):
momentum is called spin–orbit coupling. The strength of the
For states arising from the same configuration, the triplet coupling, and its effect on the energy levels of the atom, depend
term generally lies lower than the singlet term. on the relative orientations of the spin and orbital ­magnetic
332 8 Atomic structure and spectra

l l momenta is to say that it depends on the total angular momen-


tum of the electron, the vector sum of its spin and orbital mo-
High j menta (see The chemist’s toolkit 8C.1 for the procedures used
Low j
s for addition and subtraction of vectors). Thus, when the spin
and orbital angular momenta are nearly parallel, the total an-
gular momentum is high; when the two angular momenta are
opposed, the total angular momentum is low (Fig. 8C.5).
The total angular momentum of an electron is described by
High Low the quantum numbers j and mj, with j  l  12 or j  l  12 . The
(a) energy (b) energy s different values of j that can arise for a given value of l label the
Figure 8C.4 Spin–orbit coupling is a magnetic interaction levels of a term. For l = 0, the only permitted value is j = 12 (the
between spin and orbital magnetic moments; the black arrows total angular momentum is the same as the spin angular mo-
show the direction of the angular momentum and the open mentum because there is no other source of angular momen-
arrows show the direction of the associated magnetic moments. tum in the atom). When l = 1, j may be either 23 (the spin and
When the angular momenta are parallel, as in (a), the magnetic orbital angular momenta are in the same sense) or 12 (the spin
moments are aligned unfavourably; when they are opposed, as and angular momenta are in opposite senses).
in (b), the interaction is favourable. This magnetic coupling is the
cause of the splitting of a term into levels.
Brief illustration 8C.2
s=½
s To identify the levels that may arise from the configurations
s=½ (a) d1 and (b) s1, identify the value of l and then the possible
s
values of j. (a) For a d electron, l = 2 and there are two levels
l j l
l=2 l=2
in the configuration, one with j  2  12  25 and the other with
j j  2  12  23 . (b) For an s electron l = 0, so only one level is pos-
j = 5/2
j = 3/2 sible, and j = 12 .

With a little work, it is possible to incorporate the effect of


Figure 8C.5 The coupling of the spin and orbital angular
spin–orbit coupling on the energies of the levels.
momenta of a d electron (l = 2) gives two possible values of j
depending on the relative orientations of the spin and orbital
angular momenta of the electron. How is that done? 8C.2 Deriving an expression for the

moments, and therefore on the relative orientations of the two energy of spin–orbit interaction
angular momenta (Fig. 8C.4). Classically, the energy of a magnetic moment μ in a magnetic
One way of expressing the dependence of the spin–orbit field B is equal to their scalar product    B. Follow these steps
­interaction on the relative orientation of the spin and orbital to arrive at an expression for the spin–orbit interaction energy.

The chemist’s toolkit 8C.1 Combining vectors


In three dimensions, the vectors u (with components ux, uy, and 2. Subtraction:
uz) and n (with components nx, ny, and nz) have the general form:
  u  ( x  ux )i  ( y  u y ) j  ( z  uz )k
u  u x i  u y j  uz k    x i   y j   z k 3. Multiplication (the ‘scalar product’):

where i, j, and k are unit vectors, vectors of magnitude 1, point-   u   x u x   y u y   z uz


ing along the positive directions on the x, y, and z axes. The oper-
ations of addition, subtraction, and multiplication are as follows: The scalar product of a vector with itself gives the square mag-
nitude of the vector:
1. Addition:
    x2  y2  z2   2
  u  ( x  ux )i  ( y  u y ) j  ( z  uz )k
8C Atomic spectra 333

Step 1 Write an expression for the energy of interaction


+½hcÃ
If the magnetic field arises from the orbital angular momentum
j = 3/2
of the electron, it is proportional to l; if the magnetic moment
μ is that of the electron spin, then it is proportional to s. It fol- 2p1

Energy
lows that the energy of interaction is proportional to the scalar
States
product s·l:

Energy of interaction     B  s  l
–hcÃ
Step 2 Express the scalar product in terms of the magnitudes of j = 1/2
the vectors
Note that the total angular momentum is the vector sum of the Figure 8C.6 The levels of a 2p1 configuration arising from spin–
spin and orbital momenta: j = l + s. The magnitude of the vector orbit coupling. Note that the low-j level lies below the high-j level
j is calculated by evaluating in energy. The number of states in a level with quantum number j
j2 2 2 is 2j + 1.
 l s
j  j  (l  s)  (l  s)  l  l  s  s  2 s  l
E1,1/ 2,3/ 2  12 hcA  23  25  1  2  12  23   12 hcA
so
E1,1/ 2,1/ 2  12 hcA  12  23  1  2  12  23   hcA
j2  l 2  s2  2s  l
The corresponding energies are shown in Fig. 8C.6. Note
That is,
that the barycentre (the ‘centre of gravity’) of the levels is
s  l  12 ( j 2  l 2  s 2 ) unchanged, because there are four states of energy 12 hcA and
two of energy −hcA .
This equation is a classical result.
Step 3 Replace the classical magnitudes by their quantum me-
chanical versions The strength of the spin–orbit coupling depends on the
To derive the quantum mechanical version of this expres- nuclear charge. To understand why this is so, imagine riding
sion, replace all the quantities on the right with their quan- on the orbiting electron and seeing a charged nucleus appar-
tum-mechanical values, which are of the form j( j + 1)2 , etc ently orbiting around you (like the Sun rising and setting). As
(Topic 7F): a result, you find yourself at the centre of a ring of current.
The greater the nuclear charge, the greater is this current,
s  l  12 { j( j  1)  l(l  1)  s(s  1)}2 and therefore the stronger is the magnetic field you detect.
Because the spin magnetic moment of the electron interacts
Then, by inserting this expression into the formula for the
with this orbital magnetic field, it follows that the greater the
energy of interaction (E  s  l) and writing the constant of
nuclear charge, the stronger is the spin–orbit interaction. It
proportionality as hcA /2, obtain an expression for the energy
turns out that the coupling increases sharply with atomic
in terms of the quantum numbers and the spin–orbit coupling
number (as Z4) because not only is the current greater but the
constant, A (a wavenumber):
electron is drawn closer to the nucleus. Whereas the coupling
(8C.4) is only weak in H (giving rise to shifts of energy levels of no
%
El,s,j = 12 hcA{j(j + 1) − l(l + 1) − s(s + 1)}
Spin–orbit more than about 0.4 cm−1), in heavy atoms like Pb it is very
interaction energy strong (giving shifts of the order of thousands of reciprocal
centimetres).
Two spectral lines are observed when the p electron of an
electronically excited alkali metal atom undergoes a transition
Brief illustration 8C.3
into a lower s orbital. One line is due to a transition starting in
The unpaired electron in the ground state of an alkali metal a j = 23 level of the upper term; the other line is due to a transi-
atom has l = 0, so j = 12 . Because the orbital angular momentum tion starting in the j = 12 level of the same term. The two lines
is zero in this state, the spin–orbit coupling energy is zero (as are jointly an example of the fine structure of a spectrum, the
is confirmed by setting j = s and l = 0 in eqn 8C.4). When the structure due to spin–orbit coupling. Fine structure can be
electron is excited to an orbital with l = 1, it has orbital angular seen in the emission spectrum from sodium vapour excited by
momentum and can give rise to a magnetic field that interacts an electric discharge or in a flame. The yellow line at 589 nm
with its spin. In this configuration the electron can have j = 23 (close to 17 000 cm−1) is actually a doublet composed of one
or j = 12 , and the energies of these levels are line at 589.76 nm (16 956.2 cm−1) and another at 589.16 nm
334 8 Atomic structure and spectra

8C.2(c) Term symbols


2
P3/2
17 cm–1 16 973
Wavenumber, /cm–1 The discussion so far has used expressions such as ‘the j = 23
2
P1/2 level of a doublet term with l = 1’. A term symbol, which is a
16 956 symbol looking like 2P3/2 or 3D2, gives the total spin, total orbital
D1 D2 angular momentum, and total overall angular momentum, in a
succinct way.

589.16 nm
589.76 nm A term symbol gives three pieces of information:
• The letter (P or D in the examples) indicates the total
orbital angular momentum quantum number, L.
2
S1/2
Wavenumber,  • The left superscript in the term symbol (the 2 in 2P3/2)
gives the multiplicity of the term.
0
• The right subscript on the term symbol (the 3/2 in 2P3/2) is
Figure 8C.7 The energy-level diagram for the formation of the the value of the total angular momentum quantum num-
sodium D lines; the j values are the subscripts of term symbols ber, J, and labels the level of the term.
such as 2P1/2. The splitting of the spectral lines (by 17 cm−1) reflects
the splitting of the levels of the 2P term. The meaning of these statements can be discussed in the light
of the contributions to the energies summarized in Fig. 8C.8.
When several electrons are present, it is necessary to judge
(16 973.4 cm−1); the components of this doublet are the ‘D lines’ how their individual orbital angular momenta add together
of the spectrum (Fig. 8C.7). Therefore, in Na, the spin–orbit to augment or oppose each other. The total orbital angular
coupling affects the energies by about 17 cm−1. momentum quantum number, L, gives the magnitude of the
angular momentum through {L(L + 1)}1/2ħ. It has 2L + 1 orien-
tations distinguished by the quantum number ML, which can
take the values 0, ±1,…, ±L. Similar remarks apply to the total
Example 8C.1 Analysing a spectrum for the spin–orbit spin quantum number, S, and the quantum number MS, and
coupling constant the total angular momentum quantum number, J, and the
The origin of the D lines in the spectrum of atomic sodium is quantum number MJ.
shown in Fig. 8C.7. Calculate the spin–orbit coupling constant The value of L (a non-negative integer) is obtained by cou-
for the upper configuration of the Na atom. pling the individual orbital angular momenta by using the
Clebsch–Gordan series:
Collect your thoughts It follows from Fig. 8C.7 that the split-
ting of the lines is equal to the energy separation of the j = 23
L  l1  l2 ,l1  l2  1,, l1  l2 Clebsch- Gordan series (8C.5)
and 12 levels of the excited configuration. You need to express
this separation in terms of A by using eqn 8C.4.
The solution The two levels are split by


  E1, 1 , 3  E1, 1 , 1 /hc  12 A
2 2 2 2
 
3
2
3
2 
 1  12  12  1  23 A Configuration p2
Spin Electrostatic
correlation
The experimental value of  is 17.2 cm−1. Therefore
Orbital Electrostatic
3
P P
1
A  23  (17.2 cm 1 )  11.5 cm 1 occupation

The same calculation repeated for the atoms of other alkali D P S Spin–orbit
metals gives Li: 0.23 cm−1, K: 38.5 cm−1, Rb: 158 cm−1, Cs: Magnetic
interaction
370 cm−1. Note the increase of A with atomic number (but
more slowly than Z4 for these many-electron atoms). 3
P2 P1
3 3
P0
6 1
Self-test 8C.1 The configuration … 4p 5d of rubidium has two
levels at 25 700.56 cm−1 and 25 703.52 cm−1 above the ground Figure 8C.8 A summary of the types of interaction that are
state. What is the spin–orbit coupling constant in this excited responsible for the various kinds of splitting of energy levels in
state? atoms. For light (low Z) atoms, magnetic interactions are small,
Answer: 1.18 cm−1 but in heavy (high Z) atoms they may dominate the electrostatic
(charge–charge) interactions.
8C Atomic spectra 335

l=1
where the series terminates). When there are more than two
l=1 electrons to couple together, you need to use two series in suc-
l=1
cession: first to couple two electrons, and then to couple the
third to each combined state, and so on.
l=2 l=2 l=2 The solution (a) The minimum value is |l1 − l2| = |2 − 2| = 0.
Therefore,

L=3 L=2 L=1 L  2  2,2  2  1,,0  4,3,2,1,0

corresponding to G, F, D, P, and S terms, respectively. (b)


Figure 8C.9 The total orbital angular momenta of a p electron Coupling two p electrons gives a minimum value of |1 − 1| = 0.
and a d electron correspond to L = 3, 2, and 1 and reflect the Therefore,
different relative orientations of the two momenta.
L  1  11
,  1  1,,0  2,1,0

The modulus signs are attached to l1 − l2 to ensure that L is non- Now couple l3 = 1 with L′ = 2, to give L = 3, 2, 1; with L′ = 1,
negative. The maximum value, L = l1 + l2, is obtained when the to give L = 2, 1, 0; and with L′ = 0, to give L = 1. The overall
two orbital angular momenta are in nearly the same direction; result is
the lowest value, |l1 − l2|, is obtained when they are nearly in
L = 3,2,2,111
, , ,0
opposite directions. Note the ‘nearly’: the magnitudes of the
component vectors and the resultant do not in general allow giving one F, two D, three P, and one S term.
the vectors to be exactly in the same or opposite directions. The
Self-test 8C.2 Repeat the question for the configurations
intermediate values represent possible intermediate relative (a) f1d1 and (b) d3.
orientations of the two momenta (Fig. 8C.9). For two p elec- Answer: (a) H, G, F, D, P; (b) I, 2H, 3G, 4F, 5D, 3P, S
trons (for which l1 = l2 = 1), L = 2, 1, 0. The code for converting
the value of L into a letter is the same as for the s, p, d, f,… des-
ignation of orbitals, but uses uppercase Roman letters:2
When there are several electrons to be taken into account,
L: 0 1 2 3 4 5 6… their total spin angular momentum quantum number, S (a
S P D F G H I… non-negative integer or half-integer), must be assessed. Once
again the Clebsch–Gordan series is used, but now in the form
Thus, a p2 configuration has L = 2, 1, 0 and gives rise to D, P, and
S terms. The terms differ in energy on account of the different S  s1  s2 ,s1  s2  1,,|s1  s2 | (8C.6)
spatial distribution of the electrons and the consequent differ-
ences in repulsion between them.3 to decide on the value of S, noting that each electron has s = 12 .
A closed shell has zero orbital angular momentum be- For two electrons the possible values of S are 1 and 0
cause all the individual orbital angular momenta sum to zero. (Fig. 8C.10). If there are three electrons, the total spin angular
Therefore, when working out term symbols, only the electrons momentum is obtained by coupling the third spin to each of the
of the unfilled shell need to be considered. In the case of a sin- values of S for the first two spins, which results in S = 23 and 12 .
gle electron outside a closed shell, the value of L is the same as
the value of l; so the configuration [Ne]3s1 has only an S term. s=½

S=1
Example 8C.2Deriving the total orbital angular
s=½ s=½
momentum of a configuration
Find the terms that can arise from the configurations (a) d2, S=0
(b) p3.
s=½
Collect your thoughts Use the Clebsch–Gordan series and (a) (b)
begin by finding the minimum value of L (so that you know Figure 8C.10 For two electrons (each of which has s = 21 ), only
two total spin states are permitted (S = 0, 1). (a) The state with
2 S = 0 can have only one value of MS (MS = 0) and gives rise to
The convention of using lowercase letters to label orbitals and uppercase
letters to label overall states applies throughout spectroscopy, not just to atoms. a singlet term; (b) the state with S = 1 can have any of three
3
Throughout this discussion of atomic spectroscopy, distinguish italic S, values of MS (+1, 0, −1) and gives rise to a triplet term. The vector
the total spin quantum number, from Roman S, the term label. representations of the S = 0 and 1 states are shown in Fig. 8C.2.
336 8 Atomic structure and spectra

The multiplicity of a term is the value of 2S + 1. When S = 0 find L and the spins to find S. Next, couple L and S to find J.
(as for a closed shell, like 1s2) the electrons are all paired and Finally, express the term as 2S+1{L}J, where {L} is the appropriate
there is no net spin: this arrangement gives a singlet term, 1S. letter. For F, for which the valence configuration is 2p5, treat
A lone electron has S= s= 12 , so a configuration such as [Ne]3s1 the single gap in the closed-shell 2p6 configuration as a single
can give rise to a doublet term, 2S. Likewise, the configuration spin- 12 particle.
[Ne]3p1 is a doublet, 2P. When there are two unpaired (parallel The solution (a) For Na, the configuration is [Ne]3s1, and con-
spin) electrons S = 1, so 2S + 1 = 3, giving a triplet term, such sider only the single 3s electron. Because L = l = 0 and S= s= 12 ,
as 3D. The relative energies of singlets and triplets are discussed the only possible value is J = 12 . Hence the term symbol is 2S1/2.
earlier in the Topic, where it is seen that their energies differ on (b) For F, the configuration is [He]2s22p5, which can be treated
account of spin correlation. as [Ne]2p−1 (where the notation 2p−1 signifies the absence of a
As already explained, the quantum number j gives the rela- 2p electron). Hence L = l = 1, and S= s= 12 . Two values of J are
tive orientation of the spin and orbital angular momenta of possible: J = 23 , 12 . Hence, the term symbols for the two levels are
a single electron. The total angular momentum quantum 2
P3/2 and 2P1/2. (c) This is a two-electron problem, and l1 = l2 = 1,
­number, J (a non-negative integer or half-integer), does the s=1 s=
2
1
2
. It follows that L = 2, 1, 0 and S = 1, 0. The terms are
same for several electrons. If there is a single electron outside therefore 3D and 1D, 3P and 1P, and 3S and 1S. For 3D, L = 2 and
a closed shell, J = j, with j either l + 12 or | l − 12 |. The [Ne]3s1 con- S = 1; hence J = 3, 2, 1 and the levels are 3D3, 3D2, and 3D1. For
1
figuration has j = 12 (because l = 0 and s = 12 ), so the 2S term has D, L = 2 and S = 0, so the single level is 1D2. The three levels
a single level, denoted 2S1/2. The [Ne]3p1 configuration has l = 1; of 3P are 3P2, 3P1, and 3P0, and there is just one level for the
therefore j = 23 and 12 ; the 2P term therefore has two levels, 2P3/2 singlet,1P1. For the 3S term there is only one level, 3S1 (because
and 2P1/2. These levels lie at different energies on account of the J = 1 only), and the singlet term is 1S0.
spin–orbit interaction. Comment. Fewer terms arise from a configuration like … 2p2
If there are several electrons outside a closed shell it is neces- or … 3p2 than from a configuration like … 2p13p1 because the
sary to consider the coupling of all the spins and all the orbital Pauli exclusion principle forbids parallel arrangements of spins
angular momenta. This complicated problem can be simplified when two electrons occupy the same orbital. The analysis of
when the spin–orbit coupling is weak (for atoms of low atomic the terms arising in such cases requires more detail than given
number), by using the Russell–Saunders coupling scheme. here.
This scheme is based on the view that, if spin–orbit coupling
Self-test 8C.3 Identify the terms arising from the configura-
is weak, then it is effective only when all the orbital momenta tions (a) 2s12p1, (b) 2p13d1.
are operating cooperatively. That is, all the orbital angular mo- (b) 3F4, 3F3, 3F2, 1F3, 3D3, 3D2, 3D1, 1D2, 3P2, 3P1, 3P0, 1P1
menta of the electrons couple to give a total L, and all the spins Answer: (a) 3P2, 3P1, 3P0, 1P1;
are similarly coupled to give a total S. Only at this stage do the
two kinds of momenta couple through the spin–orbit interac-
tion to give a total J. The permitted values of J are given by the
Clebsch–Gordan series Russell–Saunders coupling fails when the spin–orbit cou-
pling is large (in heavy atoms, those with high Z). In that case,
J  L  S,L  S  1,,|L  S| (8C.7) the individual spin and orbital momenta of the electrons are
coupled into individual j values; then these momenta are com-
For example, in the case of the 3D term of the configuration
bined into a grand total, J, given by a Clebsch–Gordan series.
[Ne]2p13p1, the permitted values of J are 3, 2, 1 (because 3D has
This scheme is called jj-coupling. For example, in a p2 configu-
L = 2 and S = 1), so the term has three levels, 3D3, 3D2, and 3D1.
ration, the individual values of j are 23 and 12 for each electron. If
When L ≥ S, the multiplicity is equal to the number of lev-
the spin and the orbital angular momentum of each electron
els. For example, a 2P term  L  1  S  12  has the two levels 2P3/2
are coupled together strongly, it is best to consider each elec-
and 2P1/2, and 3D (L = 2 > S = 1) has the three levels 3D3, 3D2,
tron as a particle with angular momentum j = 23 or 12 . These in-
and 3D1. However, this is not the case when L < S: the term 2S
dividual total momenta then couple as follows:
 L  0  S  12 , for example, has only the one level 2S1/2.
j1 j2 J
Example 8C.3 3 3
3, 2, 1, 0
Deriving term symbols 2 2

3
2
1
2 2, 1
Write the term symbols arising from the ground-state configu- 1 3
2, 1
rations of (a) Na and (b) F, and (c) the excited configuration 2 2

1s22s22p13p1 of C.
1
2
1
2 1, 0

Collect your thoughts Begin by writing the configurations, but For heavy atoms, in which jj-coupling is appropriate, it is best
ignore inner closed shells. Then couple the orbital momenta to to discuss their energies by using these quantum numbers.
8C Atomic spectra 337

This rule can be explained classically by noting that two elec-


Pure Russell–Saunders Pure jj
coupling coupling trons have a high orbital angular momentum if they circulate
in the same direction, in which case they can stay apart. If they
1
S0 circulate in opposite directions, they meet. Thus, a D term is
(3/2,3/2)
expected to lie lower in energy than an S term of the same mul-
tiplicity.
Energy

1
D2
3. For atoms with less than half-filled shells, the level with
(3/2,1/2)
the lowest value of J lies lowest in energy; for more than
3
P2
3
P1 half-filled shells, the highest value of J lies lowest.
3
P0
(1/2,1/2)
This rule arises from considerations of spin–orbit coupling.
C Si Ge Sn Pb Thus, for a state of low J, the orbital and spin angular momenta
Period lie in opposite directions, and so too do the corresponding
Figure 8C.11 The correlation diagram for some of the states of a magnetic moments. In classical terms the magnetic moments
two-electron system. All atoms lie between the two extremes, but are then antiparallel, with the N pole of one close to the S pole
the heavier the atom, the closer it lies to the pure jj-coupling case. of the other, which is a low-energy arrangement.

8C.2(e) Selection rules


Although jj-coupling should be used for assessing the ener-
Any state of the atom, and any spectral transition, can be speci-
gies of heavy atoms, the term symbols derived from Russell–
fied by using term symbols. For example, the transitions giv-
Saunders coupling can still be used as labels. To see why this
ing rise to the yellow sodium doublet (which are shown in
procedure is valid, it is useful to examine how the energies of
Fig. 8C.7) are
the atomic states change as the spin–orbit coupling increases
in strength. Such a correlation diagram is shown in Fig. 8C.11. 12 12 12 12
3p P3/ 2 → 3s S1/ 2 3p P1/ 2 → 3s S1/ 2
It shows that there is a correspondence between the low spin–
orbit coupling (Russell–Saunders coupling) and high spin– By convention, the upper term precedes the lower. The corre-
orbit coupling (jj-coupling) schemes, so the labels derived by sponding absorptions are therefore denoted 2P3/2 ← 2S1/2 and
2
using the Russell–Saunders scheme can be used to label the P1/2 ← 2S1/2. (The configurations have been omitted.)
states of the jj-coupling scheme. As seen in Section 8C.1, selection rules arise from the con-
servation of angular momentum during a transition and from
the fact that a photon has a spin of 1. They can therefore be ex-
8C.2(d) Hund’s rules and term symbols pressed in terms of the term symbols, because the latter carry
Hund’s rules for predicting the ground state configuration of information about angular momentum. A detailed analysis
a many-electron atom are introduced in Topic 8B. With the leads to the following rules:
notation introduced in this Topic, they can be extended and
S  0,
expressed by referring to term symbols. As already remarked, Selection rules
the terms arising from a given configuration differ in energy L  0, 1, l  1,  (8C.8)
for light atoms
because they represent different relative orientations of the an- J  0, 1 but J  0 | J  0
gular momenta of the electrons and therefore different spatial
where the symbol ←|→ denotes a forbidden transition. The
distributions. The terms arising from the ground-state configu-
rule about ΔS (no change of overall spin) stems from the fact
ration of an atom (and less reliably from other configurations)
that electromagnetic radiation interacts with all the spins in an
can be put into the order of increasing energy by using the fol-
atom equally and therefore cannot twist individual spins into
lowing version of the rules, which summarize the preceding
new relative orientations. The rules about ΔL and Δl express
discussion:
the fact that the orbital angular momentum of an individual
1. For a given configuration, the term of greatest multiplicity electron must change (so Δl = ±1), but whether or not this
lies lowest in energy. ­results in an overall change of orbital momentum depends on
the coupling.
As discussed in Topic 8B, this rule is a consequence of spin cor-
The selection rules given above apply when Russell–Saunders
relation, the quantum-mechanical tendency of electrons with
coupling is valid (in light atoms, those of low Z). If labelling
parallel spins to stay apart from one another.
the terms of heavy atoms with symbols like 3D, then the selec-
2. For a given multiplicity, the term with the highest value of tion rules progressively fail as the atomic number increases
L lies lowest in energy. because the quantum numbers S and L become ill defined as
338 8 Atomic structure and spectra

jj-coupling becomes more appropriate. As explained above,


Russell–Saunders term symbols are only a convenient way of Exercises
labelling the terms of heavy atoms: they do not bear any di- 1 1
E8C.4 What atomic terms are possible for the electron configuration ns nd ?
rect relation to the actual angular momenta of the electrons Which term is likely to lie lowest in energy?
in a heavy atom. For this reason, transitions between singlet 1
E8C.5 Calculate the shifts in the energies of the two terms of a d configuration
and triplet states (for which ΔS = ±1), while forbidden in light that can arise from spin–orbit coupling.
atoms, are allowed in heavy atoms. E8C.6 Which of the following transitions between terms are allowed in the
electronic emission spectrum of a many-electron atom: (i) 3D2 → 3P1,
(ii) 3P2 → 1S0, (iii) 3F4 → 3D3?

Checklist of concepts
☐ 1. Two electrons with paired spins in a configuration give ☐ 7. The multiplicity of a term is the value of 2S + 1.
rise to a singlet term; if their spins are parallel, they give ☐ 8. The total angular momentum in light atoms is obtained
rise to a triplet term. on the basis of Russell–Saunders coupling; in heavy
☐ 2. The orbital and spin angular momenta interact magneti- atoms, jj-coupling is used.
cally. ☐ 9. The term with the maximum multiplicity lies lowest in
☐ 3. Spin–orbit coupling results in the levels of a term hav- energy.
ing different energies. ☐ 10. For a given multiplicity, the term with the highest value
☐ 4. Fine structure in a spectrum is due to transitions to dif- of L lies lowest in energy.
ferent levels of a term. ☐ 11. For atoms with less than half-filled shells, the level with
☐ 5. A term symbol specifies the angular momentum states the lowest value of J lies lowest in energy; for more than
of an atom. half-filled shells, the highest value of J lies lowest.
☐ 6. Angular momenta are combined into a resultant by ☐ 12. Selection rules for light atoms include the fact that
using the Clebsch–Gordan series. changes of total spin do not occur.

Checklist of equations
Property Equation Comment Equation number

Spin–orbit interaction energy El ,s ,j  12 hcA{ j( j  1)  l(l  1)  s(s  1)} 8C.4


Clebsch–Gordan series J = j1 + j2, j1 + j2 − 1,…,|j1 − j2| J, j denote any kind of angular momentum 8C.5
Selection rules ΔS = 0, Light atoms 8C.8
ΔL = 0, ±1, Δl = ±1,
ΔJ = 0, ±1, but J = 0 ←|→ J = 0
Exercises and problems 339

FOCUS 8 Atomic structure and spectra

To test your understanding of this material, work through Selected solutions can be found at the end of this Focus in
the Exercises, Additional exercises, Discussion questions, and the e-book. Solutions to even-numbered questions are available
Problems found throughout this Focus. online only to lecturers.

TOPIC 8A Hydrogenic atoms


Discussion questions
D8A.1 Describe the separation of variables procedure as it is applied to D8A.3 Explain the significance of (a) a boundary surface and (b) the radial
simplify the description of a hydrogenic atom free to move through space. distribution function for hydrogenic orbitals.
D8A.2 List and describe the significance of the quantum numbers needed to
specify the internal state of a hydrogenic atom.

Additional exercises
2
E8A.7 State the orbital degeneracy of the levels in a hydrogen atom that have E8A.17 The wavefunction of one of the d orbitals is proportional to sin θ sin 2ϕ.
energy (i) −hcR H ; (ii) − 19 hcR H ; (iii) − 251 hcR H. At what angles does it have nodal planes?
E8A.8 State the orbital degeneracy of the levels in a hydrogenic atom (Z in E8A.18 Write down the expression for the radial distribution function of a 2s
parentheses) that have energy (i) −4hcR N , (2); (ii) − 14 hcR N (4), and electron in a hydrogenic atom of atomic number Z and identify the radius at
(iii) −hcR N (5). which it is a maximum. Hint: Use mathematical software.
E8A.9 The wavefunction for the 2s orbital of a hydrogen atom is E8A.19 Write down the expression for the radial distribution function of a 3s
N (2 − r /a0 )e − r / 2a0 . Evaluate the normalization constant N. electron in a hydrogenic atom of atomic number Z and identify the radius
at which the electron is most likely to be found. Hint: Use mathematical
E8A.10 Evaluate the probability density at the nucleus of an electron with
software.
n = 3, l = 0, ml = 0.
E8A.20 Write down the expression for the radial distribution function of a 3p
E8A.11 By differentiation of the 3s radial wavefunction, show that it has three
electron in a hydrogenic atom of atomic number Z and identify the radius
extrema in its amplitude, and locate them.
at which the electron is most likely to be found. Hint: Use mathematical
E8A.12 At what radius does the probability density of an electron in the H software.
atom fall to 50 per cent of its maximum value?
E8A.21 What subshells and orbitals are available in the N shell?
E8A.13 At what radius in the H atom does the radial distribution function of
E8A.22 What is the orbital angular momentum (as multiples of ħ) of an
the ground state have (i) 50 per cent, (ii) 75 per cent of its maximum value?
electron in the orbitals (i) 4d, (ii) 2p, (iii) 3p? Give the numbers of angular
E8A.14 Locate the radial nodes in the 3s orbital of a hydrogenic atom. and radial nodes in each case.
E8A.15 Locate the radial nodes in the 4p orbital of a hydrogenic atom. You E8A.23 Locate the radial nodes of each of the 2p orbitals of a hydrogenic atom
need to know that, in the notation of eqn 8A.10, L4,1(ρ) ∝ 20 − 10ρ + ρ2, with of atomic number Z.
  12 Zr /a0.
E8A.24 Locate the radial nodes of each of the 3d orbitals of a hydrogenic atom
E8A.16 The wavefunction of one of the d orbitals is proportional to of atomic number Z.
cos θ sin θ cos ϕ. At what angles does it have nodal planes?

Problems
P8A.1 At what point (not radius) is the probability density a maximum for the P8A.5 Explicit expressions for hydrogenic orbitals are given in Tables 7F.1 (for
2p electron? the angular component) and 8A.1 (for the radial component). (a) Verify both
that the 3px orbital is normalized (to 1) and that 3px and 3dxy are mutually
P8A.2 Show by explicit integration that (a) hydrogenic 1s and 2s orbitals,
orthogonal. Hint: It is sufficient to show that the functions eiϕ and e2iϕ are
(b) 2px and 2py orbitals are mutually orthogonal.
mutually orthogonal. (b) Identify the positions of both the radial nodes and
P8A.3 The value of R ∞ is 109 737 cm . What is the energy of the ground state
−1
nodal planes of the 3s, 3px, and 3dxy orbitals. (c) Calculate the mean radius
of a deuterium atom? Take mD = 2.013 55mu. of the 3s orbital. Hint: Use mathematical software. (d) Draw a graph of the
2+ radial distribution function for the three orbitals (of part (b)) and discuss the
P8A.4 Predict the ionization energy of Li given that the ionization energy of
significance of the graphs for interpreting the properties of many-electron
He+ is 54.36 eV.
atoms.
340 8 Atomic structure and spectra

P8A.6 Determine whether the px and py orbitals are eigenfunctions of lz. If not, balanced by the centrifugal effect of the orbital motion. Bohr proposed that
does a linear combination exist that is an eigenfunction of lz? the angular momentum is limited to integral values of ħ. When the two forces
are balanced, the atom remains in a stationary state until it makes a spectral
P8A.7 The ‘size’ of an atom is sometimes considered to be measured by the
transition. Calculate the energies of a hydrogenic atom using the Bohr model.
radius of a sphere within which there is a 90 per cent probability of finding the
electron in the outermost occupied orbital. Calculate the ‘size’ of a hydrogen P8A.10 The Bohr model of the atom is specified in Problem 8A.9. (a) What
atom in its ground state according to this definition. Go on to explore how the features of it are untenable according to quantum mechanics? (b) How does
‘size’ varies as the definition is changed to other percentages, and plot your the ground state of the Bohr atom differ from the actual ground state? (c) Is
conclusion. there an experimental distinction between the Bohr and quantum mechanical
models of the ground state?
P8A.8 Some atomic properties depend on the average value of 1/r rather than
the average value of r itself. Evaluate the expectation value of 1/r for (a) a P8A.11 Atomic units of length and energy may be based on the properties
hydrogenic 1s orbital, (b) a hydrogenic 2s orbital, (c) a hydrogenic 2p orbital. of a particular atom. The usual choice is that of a hydrogen atom, with the
(d) Does 〈1/r〉 = 1/〈r〉? unit of length being the Bohr radius, a0, and the unit of energy being the
‘hartree’, Eh, which is equal to twice the (negative of the) energy of the 1s
P8A.9 One of the most famous of the obsolete theories of the hydrogen atom
orbital (specifically, and more precisely, Eh  2hcR  ). Positronium consists of
was proposed by Niels Bohr. It has been replaced by quantum mechanics, but
an electron and a positron (same mass, opposite charge) orbiting round their
by a remarkable coincidence (not the only one where the Coulomb potential
common centre of mass. If the positronium atom (e+,e−) were used instead,
is concerned), the energies it predicts agree exactly with those obtained
with analogous definitions of units of length and energy, what would be the
from the Schrödinger equation. In the Bohr atom, an electron travels in a
relation between these two sets of atomic units?
circle around the nucleus. The Coulombic force of attraction (Ze2/4πε0r2) is

TOPIC 8B Many-electron atoms


Discussion questions
D8B.1 Describe the orbital approximation for the wavefunction of a many- D8B.3 Describe and account for the variation of first ionization energies along
electron atom. What are the limitations of the approximation? Period 2 of the periodic table. Would you expect the same variation in Period 3?
D8B.2 Outline the electron configurations of many-electron atoms in terms of D8B.4 Describe the self-consistent field procedure for calculating the form of
their location in the periodic table. the orbitals and the energies of many-electron atoms.

Additional exercises
E8B.6 Construct the wavefunction for an excited state of the He atom with E8B.8 Write the ground-state electron configurations of the d-metals from
configuration 1s13s1. Use Zeff = 2 for the 1s electron and Zeff = 1 for the 3s yttrium to cadmium.
electron. 2−
E8B.9 Write the electronic configuration of the O ion.
E8B.7 How many electrons can occupy subshells with l = 5?
E8B.10 Consider the atoms of the Period 2 elements of the periodic table.
Predict which element has the lowest second ionization energy.

Problems
P8B.1 In 1976 it was mistakenly believed that the first of the ‘superheavy’ accompanies the conversion of Fe(II) to Fe(III) when O2 attaches triggers a
elements had been discovered in a sample of mica. Its atomic number was conformational change in the protein. Which do you expect to be larger: Fe2+
believed to be 126. What is the most probable distance of the innermost or Fe3+? Why?
electrons from the nucleus of an atom of this element? (In such elements,
P8B.5 Thallium, a neurotoxin, is the heaviest member of Group 13 of the
relativistic effects are very important, but ignore them here.)
periodic table and is found most usually in the +1 oxidation state. Aluminium,
1 2
P8B.2 Why is the electronic configuration of the yttrium atom [Kr]4d 5s and which causes anaemia and dementia, is also a member of the group but its
that of the silver atom [Kr]4d105s1? chemical properties are dominated by the +3 oxidation state. Examine this
issue by plotting the first, second, and third ionization energies for the Group
P8B.3 The d-metals iron, copper, and manganese form cations with different
13 elements against atomic number. Explain the trends you observe. Hints:
oxidation states. For this reason, they are found in many oxidoreductases and
The third ionization energy, I3, is the minimum energy needed to remove an
in several proteins of oxidative phosphorylation and photosynthesis. Explain
electron from the doubly charged cation: E2+(g) → E3+(g) + e−(g), I3 = E(E3+) −
why many d-metals form cations with different oxidation states.
E(E2+).
P8B.4 One important function of atomic and ionic radius is in regulating
the uptake of oxygen by haemoglobin, for the change in ionic radius that
Exercises and problems 341

TOPIC 8C Atomic spectra


Discussion questions
D8C.1 Discuss the origin of the series of lines in the emission spectrum of D8C.3 Explain the origin of spin–orbit coupling and how it affects the
hydrogen. What region of the electromagnetic spectrum is associated with appearance of a spectrum.
each of the series shown in Fig. 8C.1?
D8C.4 Why does the spin–orbit coupling constant depend so strongly on the
D8C.2 Specify and account for the selection rules for transitions in atomic number?
(a) hydrogenic atoms, and (b) many-electron atoms.

Additional exercises
E8C.7 The Pfund series has n1 = 5. Identify the transition responsible for the E8C.19 Suppose that an atom has (i) 4, (ii) 5, electrons in different orbitals.
shortest and longest wavelength lines in the Pfund series. What are the possible values of the total spin quantum number S? What is the
multiplicity in each case?
E8C.8 Calculate the wavelength, frequency, and wavenumber of the n = 5 →
n = 4 transition in Li2+. E8C.20 What are the possible values of the total spin quantum numbers S and
MS for the Ni2+ ion?
E8C.9 Which of the following transitions are allowed in the electronic emission
spectrum of a hydrogenic atom: (i) 5d → 2s, (ii) 5p → 3s, (iii) 6p → 4f? E8C.21 What are the possible values of the total spin quantum numbers S and
1 MS for the V2+ ion?
E8C.10 Identify the levels of the configuration p .
1 1
1 E8C.22 What atomic terms are possible for the electron configuration np nd ?
E8C.11 Identify the levels of the configuration f .
Which term is likely to lie lowest in energy?
E8C.12 What are the permitted values of j for (i) a d electron, (ii) an f electron? 1 2 3
E8C.23 What values of J may occur in the terms (i) S, (ii) P, (iii) P? How
E8C.13 What are the permitted values of j for (i) a p electron, (ii) an h electron? many states (distinguished by the quantum number MJ) belong to each level?
3 4 2
E8C.14 An electron in two different states of an atom is known to have j = 3
2 E8C.24 What values of J may occur in the terms (i) D, (ii) D, (iii) G? How
and . What is its orbital angular momentum quantum number in each case?
1
2 many states (distinguished by the quantum number MJ) belong to each level?
1 1
E8C.15 What are the allowed total angular momentum quantum numbers of a E8C.25 Give the possible term symbols for (i) Li [He]2s , (ii) Na [Ne]3p .
composite system in which j1 = 5 and j2 = 3? 10 2
E8C.26 Give the possible term symbols for (i) Zn [Ar]3d 4s ,
1 10 2 5
E8C.16 What information does the term symbol D2 provide about the angular (ii) Br [Ar]3d 4s 4p .
momentum of an atom? 1
E8C.27 Calculate the shifts in the energies of the two terms in an f
3
E8C.17 What information does the term symbol F4 provide about the angular configuration that can arise from spin–orbit coupling.
momentum of an atom?
E8C.28 Which of the following transitions between terms are allowed in the
E8C.18 Suppose that an atom has (i) 2, (ii) 3 electrons in different orbitals. electronic emission spectrum of a many-electron atom: (i) 2P3/2 → 2S1/2,
What are the possible values of the total spin quantum number S? What is the (ii) 3P0 → 3S1, (iii) 3D3 → 1P1?
multiplicity in each case?

Problems
P8C.1 The Humphreys series is a group of lines in the spectrum of atomic P8C.5 A series of lines in the spectrum of neutral Li atoms rise from
hydrogen. It begins at 12 368 nm and has been traced to 3281.4 nm. What transitions between 1s22p1 2P and 1s2nd1 2D and occur at 610.36 nm,
are the transitions involved? What are the wavelengths of the intermediate 460.29 nm, and 413.23 nm. The d orbitals are hydrogenic. It is known that
transitions? the transition from the 2P to the 2S term (which arises from the ground-state
configuration 1s22s1) occurs at 670.78 nm. Calculate the ionization energy of
P8C.2 A series of lines involving a common level in the spectrum of atomic
the ground-state atom.
hydrogen lies at 656.46 nm, 486.27 nm, 434.17 nm, and 410.29 nm. What is

the wavelength of the next line in the series? What is the ionization energy of P8C.6 W.P. Wijesundera et al. (Phys. Rev. A 51, 278 (1995)) attempted to
the atom when it is in the lower state of the transitions? determine the electron configuration of the ground state of lawrencium,
element 103. The two contending configurations are [Rn]5f147s27p1 and
P8C.3 The distribution of isotopes of an element may yield clues about the
[Rn]5f146d7s2. Write down the term symbols for each of these configurations,
nuclear reactions that occur in the interior of a star. Show that it is possible to
and identify the lowest level within each configuration. Which level would be
use spectroscopy to confirm the presence of both 4He+ and 3He+ in a star by
lowest according to a simple estimate of spin–orbit coupling?
calculating the wavenumbers of the n = 3 → n = 2 and of the n = 2 → n = 1
transitions for each ionic isotope. P8C.7 An emission line from K atoms is found to have two closely spaced
2+ −1 components, one at 766.70 nm and the other at 770.11 nm. Account for this
P8C.4 The Li ion is hydrogenic and has a Lyman series at 740 747 cm ,
−1 −1 observation, and deduce what information you can.
877 924 cm , 925 933 cm , and beyond. Show that the energy levels are of
the form −hcR Li /n2 and find the value of R Li for this ion. Go on to predict the
wavenumbers of the two longest-wavelength transitions of the Balmer series

of the ion and find its ionization energy. These problems were supplied by Charles Trapp and Carmen Giunta.
342 8 Atomic structure and spectra

P8C.8 Calculate the mass of the deuteron given that the first line in the Lyman ml = +1 moves up in energy by μBB, the state with ml = 0 is unchanged, and
series of 1H lies at 82 259.098 cm−1 whereas that of 2H lies at 82 281.476 cm−1. the state with ml = −1 moves down in energy by μBB, where μB = eħ/2me =
Calculate the ratio of the ionization energies of 1H and 2H. 9.274 × 10−24 J T−1 is the ‘Bohr magneton’. Therefore, a transition between a
1
P8C.9 Positronium consists of an electron and a positron (same mass, opposite
S0 term and a 1P1 term consists of three spectral lines in the presence of a
charge) orbiting round their common centre of mass. The broad features magnetic field where, in the absence of the magnetic field, there is only one.
of the spectrum are therefore expected to be hydrogen-like, the differences (a) Calculate the splitting in reciprocal centimetres between the three spectral
arising largely from the mass differences. Predict the wavenumbers of the first lines of a transition between a 1S0 term and a 1P1 term in the presence of a
three lines of the Balmer series of positronium. What is the binding energy of magnetic field of 2 T (where 1 T = 1 kg s−2 A−1). (b) Compare the value you
the ground state of positronium? calculated in (a) with typical optical transition wavenumbers, such as those
for the Balmer series of the H atom. Is the line splitting caused by the normal
P8C.10 The Zeeman effect is the modification of an atomic spectrum by the Zeeman effect relatively small or relatively large?
application of a strong magnetic field. It arises from the interaction between
P8C.11 Some of the selection rules for hydrogenic atoms were derived in the
applied magnetic fields and the magnetic moments due to orbital and spin
angular momenta (recall the evidence provided for electron spin by the text. Complete the derivation by considering the x- and y-components of the
Stern–Gerlach experiment, Topic 8B). To gain some appreciation for the electric dipole moment operator.
so-called normal Zeeman effect, which is observed in transitions involving P8C.12 Hydrogen is the most abundant element in all stars. However, neither
singlet states, consider a p electron, with l = 1 and ml = 0, ±1. In the absence of absorption nor emission lines due to neutral hydrogen are found in the
a magnetic field, these three states are degenerate. When a field of magnitude spectra of stars with effective temperatures higher than 25 000 K. Account for
B is present, the degeneracy is removed and it is observed that the state with this observation.

FOCUS 8 Atomic structure and spectra


Integrated activities
+
I8.1 An electron in the ground-state He ion undergoes a transition to a state calculate the average radius (see the preceding activity), and the ionization
specified by the quantum numbers n = 4, l = 1, ml = +1. (a) Describe the energy. (c) Could a thermal collision with another hydrogen atom ionize this
transition using term symbols. (b) Calculate the wavelength, frequency, and Rydberg atom? (d) What minimum velocity of the second atom is required?
wavenumber of the transition. (c) By how much does the mean radius of the (e) Sketch the likely form of the radial wavefunction for a 100s orbital.
electron change due to the transition? You need to know that the mean radius ‡
I8.3 Stern–Gerlach splittings of atomic beams are small and require either
of a hydrogenic orbital is
large magnetic field gradients or long magnets for their observation. For a
beam of atoms with zero orbital angular momentum, such as H or Ag, the
n2a0  1  l(l  1)  
rn,l ,ml  1  2 1   deflection is given by x  ( B L2 /4 Ek )dB /dz , where μB = eħ/2me = 9.274 ×
Z   n2   10−24 J T−1 is the ‘Bohr magneton’, L is the length of the magnet, Ek is the
‡ average kinetic energy of the atoms in the beam, and dB/dz is the magnetic
I8.2 Highly excited atoms have electrons with large principal quantum
field gradient across the beam. Calculate the magnetic field gradient required
numbers. Such Rydberg atoms have unique properties and are of interest to
to produce a splitting of 1.00 mm in a beam of Ag atoms from an oven at
astrophysicists. (a) For hydrogen atoms with large n, derive a relation for the
1000 K with a magnet of length 50 cm.
separation of energy levels. (b) Calculate this separation for n = 100; also
FOCUS 9
Molecular structure

The concepts developed in Focus 8, particularly those of that spreads over all the atoms in a molecule. This Topic focuses
­orbitals, can be extended to a description of the electronic on the hydrogen molecule-ion, setting the scene for the applica-
structures of molecules. There are two principal quantum tion of the theory to more complicated molecules.
mechanical theories of molecular electronic structure: 9B.1 Linear combinations of atomic orbitals; 9B.2 Orbital notation
‘valence-bond theory’ is centred on the concept of the shared
electron pair; ‘molecular orbital theory’ treats electrons as
being distributed over all the nuclei in a molecule.
9C Molecular orbital theory:
Prologue The Born–Oppenheimer homonuclear diatomic molecules
approximation The principles established for the hydrogen molecule-ion are ex-
tended to other homonuclear diatomic molecules and ions. The
The starting point for the theories discussed here and the in- principal differences are that all the valence-shell atomic orbitals
terpretation of spectroscopic results (Focus 11) is the ‘Born- must be included and that they give rise to a more varied collec-
Oppenheimer approximation’, which separates the relative tion of molecular orbitals. The building-up principle for atoms
motions of nuclei and electrons in a molecule. is extended to the occupation of molecular orbitals and used to
predict the electronic configurations of molecules and ions.
9C.1 Electron configurations; 9C.2 Photoelectron spectroscopy
9A Valence-bond theory
The key concept of this Topic is the wavefunction for a shared
electron pair, which is then used to account for the structures 9D Molecular orbital theory:
of a wide variety of molecules. The theory introduces the con-
cepts of σ and π bonds, promotion, and hybridization, which
heteronuclear diatomic molecules
are used widely in chemistry.
The molecular orbital theory of heteronuclear diatomic mol-
9A.1 Diatomic molecules; 9A.2 Resonance; 9A.3 Polyatomic molecules
ecules introduces the possibility that the atomic orbitals on the
two atoms contribute unequally to the molecular orbital. As a
result, the molecule is polar. The polarity can be expressed in
9B Molecular orbital theory: the terms of the concept of electronegativity. This Topic shows how
hydrogen molecule-ion quantum mechanics is used to calculate the energy and form of
a molecular orbital arising from the overlap of different atomic
In molecular orbital theory the concept of an atomic orbital is orbitals.
extended to that of a ‘molecular orbital’, which is a wavefunction 9D.1 Polar bonds and electronegativity; 9D.2 The variation principle
9E Molecular orbital theory: physical and chemical properties of substances as simple as
diatomic molecules and as complex as biopolymers. This Topic
polyatomic molecules introduces the main ideas behind these methods, again using
the hydrogen molecule as an example. Initially the focus is
Molecular orbital theory applies equally well to larger mole- on the formulation of the ‘Hartree–Fock equations’, which
cules; however, the calculations involved can no longer be done guide the calculation of molecular orbitals. The ‘density func-
by hand but require the use of sophisticated computer pro- tional’ approach, which offers greater precision and computa-
grams. A flavour of how molecular orbitals are calculated comes tional efficiency, is also described.
from using the ‘Hückel method’ to find the orbitals of planar 9F.1 The central challenge; 9F.2 The Hartree−Fock formalism;
conjugated polyenes. Severe approximations are necessary in 9F.3 The Roothaan equations; 9F.4 Evaluation and approximation
order to make the calculations tractable by simple methods of the integrals; 9F.5 Density functional theory
but the method sets the scene for more sophisticated procedures.
The latter have given rise to the huge and vibrant field of compu-
tational theoretical chemistry in which elaborate computations What is an application of this material?
are used to predict molecular properties. This Topic describes
briefly how those calculations are formulated and displayed. The concepts introduced in this chapter pervade the whole of
9E.1 The Hückel approximation; 9E.2 Applications; 9E.3 Computational chemistry and are encountered throughout the text. Two bio-
chemistry chemical aspects are discussed here. In Impact 14, accessed via
the e-book, simple concepts are used to account for the reac-
tivity of small molecules that occur in organisms. Impact 15,
9F Computational chemistry accessed via the e-book, provides a glimpse of the contribution
of computational chemistry to the explanation of the thermo-
Molecular orbital theory is the foundation of current computa- dynamic and spectroscopic properties of several biologically
tional methods that can predict molecular structure and many significant molecules.

➤ Go to the e-book for videos that feature the derivation and interpretation of equations, and applications of this material.
PROLOGUE The Born–Oppenheimer
approximation

All theories of molecular structure make the same simplifi-


cation at the outset. Whereas the Schrödinger equation for a
hydrogen atom can be solved exactly, an exact solution is not
possible for any molecule because even the simplest mole-
cule consists of three particles (two nuclei and one electron).

Energy
Therefore, it is common to adopt the Born–Oppenheimer
approximation in which it is supposed that the nuclei, being so Re
0
much heavier than an electron, move relatively slowly and may Internuclear
be treated as stationary while the electrons move in their field. separation, R
~
That is, the nuclei are assumed to be fixed at arbitrary locations, –hcDe
and the Schrödinger equation is then solved for the wavefunc-
A molecular potential energy curve. The equilibrium bond length
tion of the electrons alone.
corresponds to the energy minimum.
To use the Born–Oppenheimer approximation for a dia-
tomic molecule, the nuclear separation is set at a chosen value,
the Schrödinger equation for the electrons is then solved Focus 11), it is possible to identify two important parameters.
and the energy calculated. Then a different separation is se- The first is the equilibrium bond length, Re, the internuclear
lected, the calculation repeated, and so on for other values of separation at the minimum of the curve. The second is the
the separation. In this way the variation of the energy of the depth, hcD e, of the minimum below the energy of the infinitely
molecule with bond length is explored, and a molecular poten- widely separated atoms (when R is very much larger than Re).
tial energy curve is obtained (see the illustration). It is called When more than one molecular parameter is changed in
a potential energy curve because the kinetic energy of the sta- a polyatomic molecule, such as its various bond lengths and
tionary nuclei is zero. angles, a potential energy surface is obtained. The overall
Once the curve has been calculated or determined experi- equilibrium shape of the molecule corresponds to the global
mentally (by using the spectroscopic techniques described in minimum of the surface.
TOPIC 9A Valence-bond theory

ψA(1) ψB(2) ψA(2) ψB(1)


➤ Why do you need to know this material?
The language introduced by valence-bond theory is used ψA(1)ψB(2) ψA(2)ψB(1)
throughout chemistry, especially in the description of the Enhanced
properties and reactions of organic compounds. electron density

➤ What is the key idea?


A bond forms when an electron in an atomic orbital on one
ψA(1)ψB(2) + ψA(2)ψB(1)
atom pairs its spin with that of an electron in an atomic
orbital on another atom.
Figure 9A.1 It is very difficult to represent valence-bond
wavefunctions because they refer to two electrons simultaneously.
➤ What do you need to know already?
However, this illustration is an attempt. The atomic orbital for
You need to know about atomic orbitals (Topic 8A) and electron 1 is represented by the full line, and that of electron 2
the concepts of normalization and orthogonality (Topics is represented by the dotted line. The left illustration represents
7B and 7C). This Topic also makes use of the Pauli principle ψA(1)ψB(2) and the right illustration represents the contribution
(Topic 8B). ψA(2)ψB(1). When the two contributions are superimposed, there
is interference between them, resulting in an enhanced (two-
electron) density in the internuclear region.

Valence-bond theory (VB theory) begins by considering the


chemical bond in molecular hydrogen, H2. The basic concepts
are first extended to other diatomic molecules and ions, and alone is one of the (unnormalized) linear combinations
then to polyatomic molecules. Ψ(1,2) = ψA(1)ψB(2) ± ψA(2)ψB(1). The combination with lower
energy turns out to be the one with a + sign, so the valence-
bond wavefunction of the electrons in an H2 molecule is

9A.1 Diatomic molecules  (1,2)   A (1) B (2)   A (2) B (1)


A valence-bond
wavefunction
(9A.2)

Consider two widely separated H atoms, each with one elec- The reason why this linear combination has a lower energy
tron. Electron 1 is in the H1s atomic orbital on atom A, ψ H1sA , than either the separate atoms or the linear combination
and electron 2 is in the H1s atomic orbital on atom B,ψ H1sB . The with a negative sign can be traced to the constructive inter-
spatial wavefunction describing this arrangement is ference between the wave patterns represented by the terms
ψA(1)ψB(2) and ψA(2)ψB(1), and the resulting enhancement
 (1,2)   H1sA(r1 ) H1sB(r2 ) (9A.1)
of the probability density of the electrons in the internuclear
For simplicity, this wavefunction will be written Ψ(1,2) = region (Fig. 9A.1).
ψA(1)ψB(2). When the atoms are close together, it is not pos-
sible to know whether it is electron 1 or electron 2 that is on A.
An equally valid description is therefore Ψ(1,2) = ψA(2)ψB(1), Brief illustration 9A.1
in which electron 2 is on A and electron 1 is on B. When two
outcomes are equally probable in quantum mechanics, the The wavefunction for an H1s orbital (Z = 1) is given in Topic 8A
true state of the system is described as a superposition of as (a03 )1/ 2 e  r /a0 . If the distance of electron 1 from nucleus A is
the wavefunctions for each possibility (Topic 7C). Therefore written rA1, and likewise for the other electron and nucleus, the
a better description of the molecule than either w ­ avefunction wavefunction in eqn 9A.2 can be written
9A Valence-bond theory 347

 A (1)  B (2 )
     
1  rA 1 /a0 1  rB 2 /a0
 (1,2)  e  e
(a03 )1/ 2 (a03 )1/ 2
 A (2 )  B (1) (b)
     
1  rA 2 /a0 1  rB 1 /a0
 e  e
(a03 )1/ 2 (a03 )1/ 2
1
 3 {e (rA1  rB 2 )/a0  e (rA 2  rB1 )/a0 }
a0
(a)

Figure 9A.2 The orbital overlap and spin pairing between


The electron distribution described by the wavefunction in
electrons in two s orbitals that results in the formation of a σ bond.
eqn 9A.2 is called a σ bond (Fig. 9A.2). A σ bond has cylin-
drical symmetry around the internuclear axis, and is so called
because, when viewed along the internuclear axis, it resembles atom, which is 2s2 2p1x 2p1y 2p1z . It is conventional to take the
a pair of electrons in an s orbital (and σ is the Greek equivalent z-axis to be the internuclear axis in a linear molecule, so each
of s). atom is imagined as having a 2pz orbital pointing towards a 2pz
A chemist’s picture of a covalent bond is one in which the orbital on the other atom, with the 2px and 2py orbitals perpen-
spins of two electrons pair as the atomic orbitals overlap. It can dicular to the axis. A σ bond is then formed by spin pairing
be shown that the origin of the role of spin is that the wave- between the two electrons in the two 2pz orbitals (Fig. 9A.3). Its
function in eqn 9A.2 can be formed only by two spin-paired spatial wavefunction is given by eqn 9A.2, but now ψA and ψB
electrons. stand for the two 2pz orbitals.
The remaining N2p orbitals (2px and 2py) cannot merge to
How is that done? 9A.1 Establishing the origin of electron give σ bonds as they do not have cylindrical symmetry around
the internuclear axis. Instead, they merge to form two ‘π
pairs in VB theory
bonds’. A π bond arises from the spin pairing of electrons in
The Pauli principle requires the overall wavefunction of two two p orbitals that approach side-by-side (Fig. 9A.3). It is so
electrons, the wavefunction including spin, to change sign called because, viewed along the internuclear axis, a π bond
when the labels of the electrons are interchanged (Topic 8B). resembles a pair of electrons in a p orbital (and π is the Greek
The overall VB wavefunction for two electrons is equivalent of p).1
 (1,2)  {A (1) B (2)  A (2) B (1)} (1,2)
where σ represents the spin component of the wavefunction.
When the labels 1 and 2 are interchanged, this wavefunction
becomes
 (2,1)  { A (2) B (1)   A (1) B (2)} (2,1)
 {A (1) B (2)   A (2) B (1)} (2,1)
The Pauli principle requires that Ψ(2,1) = −Ψ(1,2), which is
satisfied only if σ(2,1) = −σ(1,2). The combination of two spins
that has this property is
1
  (1,2)  { (1) (2)   (1) (2)}
21/ 2
which corresponds to paired electron spins (Topic 8B).
Therefore, the state of lower energy (and hence the formation
of a chemical bond) is achieved if the electron spins are paired. Figure 9A.3 (a) Orbital overlap and spin pairing between electrons
Spin pairing is not an end in itself: it is necessary in order to in two p orbitals pointing along the internuclear axis (the z-axis)
achieve the lowest energy wavefunction. results in the formation of a σ bond. (b) A π bond results from
orbital overlap and spin pairing between electrons in p orbitals
with their axes perpendicular to the internuclear axis. Each bond
The VB description of H2 can extended to other homonu- has two lobes of electron density separated by a nodal plane.
clear diatomic molecules, such as N2. The starting point for the
discussion of N2 is the valence electron configuration of each 1
π bonds can also be formed from d orbitals in the appropriate orientation.
348 9 Molecular structure

­written as  HCl  H Cl   H Cl  with λ (lambda) a numerical


coefficient.
In general, the VB wavefunction, taking into account reso-
nance, is written
  covalent   ionic (9A.3)

where Ψcovalent is the two-electron wavefunction for the purely


covalent form of the bond and Ψionic is the two-electron wave-
function for the ionic form of the bond. In this case, where one
structure is pure covalent and the other pure ionic, it is called
Figure 9A.4 The dinitrogen molecule has one σ bond and two π ionic–covalent resonance. The interpretation of the (unnormal-
bonds.
ized) wavefunction, which is called a resonance hybrid, is that if
the molecule is inspected, then the probability that it would be
There are two π bonds in N2, one formed by spin pairing in two found with an ionic structure is proportional to λ2. If λ2 << 1, the
neighbouring 2px orbitals and the other by spin pairing in two covalent description is dominant. If λ2 >> 1, the ionic descrip-
neighbouring 2py orbitals. The overall bonding pattern in N2 is tion is dominant. Resonance is not a flickering between the con-
therefore a σ bond plus two π bonds (Fig. 9A.4), which is con- tributing states: it is a blending of their characteristics. It is only
sistent with the Lewis structure :N≡N: for dinitrogen. a mathematical device for achieving a closer approximation to
the true wavefunction of the molecule than that represented by
any single contributing electronic structure alone.
Exercise A systematic way of calculating the value of λ is provided by
E9A.1 Write the valence-bond wavefunction for the single bond in HF. the variation principle:
If an arbitrary wavefunction is used to calculate the energy,
9A.2 Resonance then the value calculated is never less than the true energy.
(This principle is derived and used in Topic 9C.) The arbitrary
For a given molecule there may be more than one plausible way wavefunction is called the trial wavefunction. The principle
of describing the bonding using electron pair bonds, and in VB implies that if the energy, the expectation value of the hamilto-
theory this possibility is taken into account by introducing the nian, is calculated for various trial wavefunctions with different
concept of resonance. The idea is that the VB wavefunction can values of the parameter λ, then the best value of λ is the one that
be a superposition of the wavefunctions that correspond to dif- results in the lowest energy. The ionic contribution to the reso-
ferent descriptions of the bonding. Put slightly more formally, nance is then proportional to λ2.
resonance is the superposition of wavefunctions that represent
different electron distributions in the same nuclear framework.
Brief illustration 9A.2
To understand the physical significance of resonance, con-
sider some possible VB descriptions of the HCl molecule. One Consider a bond described by eqn 9A.3. If the lowest energy is
possibility is reached when λ = 0.1, then the best description of the bond in
 H Cl   A (1) B (2)   A (2) B (1) the molecule is a resonance structure described by the wave-
function Ψ = Ψcovalent + 0.1Ψionic. This wavefunction implies that
where ψA is a H1s orbital and ψB is a Cl3p orbital. In this wave- the probabilities of finding the molecule in its covalent and
function electron 1 is on the H atom when electron 2 is on the ionic forms are in the ratio 100:1 (because 0.12 = 0.01).
Cl atom, and vice versa. In other words, the electron pair is
shared equally between the two atoms, and the bond can be de-
scribed as purely covalent.
Two other possibilities involve both electrons being on the Exercise
Cl atom, which is the ionic form H+Cl−, or both electrons being E9A.2 Write the valence-bond wavefunction for the resonance hybrid
on the H atom, which is the ionic form H−Cl+ HF ↔ H+F− ↔ H−F+ (allow for different contributions of each structure).

 H Cl    B (1) B (2)  H Cl    A (1) A (2)


9A.3 Polyatomic molecules
Chemical intuition indicates that the latter possibility is much
less likely than the former, so it will be ignored from now on. Each σ bond in a polyatomic molecule is formed by the spin
A better description of the wavefunction for the molecule pairing of electrons in atomic orbitals with cylindrical symme-
is as a superposition of the covalent and ionic descriptions, try around the relevant internuclear axis. Likewise, π bonds are
9A Valence-bond theory 349

formed by pairing electrons that occupy atomic orbitals of the Resonance always lowers the energy, and +
appropriate symmetry. the lowering is greatest when the contributing –
2
structures have similar energies. The wavefunc-
Brief illustration 9A.3 tion of benzene is improved still further, and
the calculated energy of the molecule is lowered still further,
The VB description of H2O is as follows. The valence-electron if ionic–covalent resonance is also considered, by allowing a
configuration of an O atom is 2s2 2p2x 2p1y 2p1z . The two unpaired small admixture of ionic structures, such as (2).
electrons in the O2p orbitals can each pair with an electron in
an H1s orbital, and each combination results in the formation
of a σ bond (each bond has cylindrical symmetry about the
respective O−H internuclear axis). Because the 2py and 2pz 9A.3(a) Promotion
orbitals lie at 90° to each other, the two σ bonds also lie at 90° A deficiency of this initial formulation of VB theory is its in-
to each other (Fig. 9A.5). Therefore, H2O is predicted to be an ability to account for the fact that carbon commonly forms four
angular molecule, which it is. However, the theory predicts bonds (it is ‘tetravalent’). The ground-state configuration of
a bond angle of 90°, whereas the actual bond angle is 104.5°.
carbon is 2s2 2p1x 2p1y , which suggests that a carbon atom should
be capable of forming only two bonds, not four.
Resonance plays an important role in the VB description This deficiency is overcome by allowing for promotion, the
of polyatomic molecules. One of the most famous examples excitation of an electron to an orbital of higher energy. In car-
of resonance is in the VB description of benzene, where the bon, for example, the promotion of a 2s electron to a 2p orbital
wavefunction of the molecule is written as a superposition of can be thought of as leading to the configuration 2s1 2p1x 2p1y 2p1z ,
the many-electron wavefunctions of the two covalent Kekulé with four unpaired electrons in separate orbitals. These elec-
structures: trons may pair with four electrons in orbitals provided by four
other atoms (such as four H1s orbitals if the molecule is CH4),
Ψ =Ψ ( )+Ψ ( ) (9A.4) and hence form four σ bonds. Although energy is required to
promote the electron, it is more than recovered by the pro-
The two contributing structures have moted atom’s ability to form four bonds in place of the two
identical energies, so they contribute bonds of the unpromoted atom.
equally to the superposition. In this case Promotion, and the formation of four bonds, is a charac-
1
the effect of resonance (which is repre- teristic feature of carbon because the promotion energy is
sented by a double-headed arrow (1)) is to distribute double- quite small: the promoted electron leaves a doubly occupied
bond character around the ring and to make the lengths and 2s orbital and enters a vacant 2p orbital, hence significantly
strengths of all the carbon–carbon bonds identical. The wave- relieving the electron–electron repulsion it experiences in
function is improved by allowing resonance because it allows the ground state. However, it is important to remember that
the electrons to adjust into a distribution of lower energy. This promotion is not a ‘real’ process in which an atom somehow
lowering is called the resonance stabilization of the molecule becomes excited and then forms bonds: rather, it is a notional
and, in the context of VB theory, is largely responsible for the contribution to the overall energy change that occurs when
unusual stability of aromatic rings. bonds form.

H1s Brief illustration 9A.4

Sulfur can form six bonds (an ‘expanded octet’), as in the


O2pz molecule SF6. Because the ground-state electron configura-
tion of sulfur is [Ne]3s23p4, in one version of VB theory this
bonding pattern requires the promotion of a 3s electron and
a 3p electron to two different 3d orbitals, which are nearby in
energy, to produce the notional configuration [Ne]3s13p33d2.
H1s Now all six of the valence electrons are in different orbitals and
capable of bond formation with six electrons provided by six F
O2px
atoms. In an alternative approach, the molecule is regarded as a
resonance hybrid of structures of the form (SF4)2+(F−)2, without
Figure 9A.5 In a primitive view of the structure of an H2O the need to invoke d-orbitals. Calculations suggest the latter
molecule, each bond is formed by the overlap and spin pairing of model is closer to the truth.
an H1s electron and an O2p electron.
350 9 Molecular structure

solution, a = −b, simply corresponds to choosing different


9A.3(b) Hybridization absolute phases for the p orbitals. A similar argument but with
The description of the bonding in CH4 (and other alkanes) h3 = as + b(−px + py − pz) and h4 = as + b(px − py − pz) leads to
is still incomplete because it implies the presence of three σ two other hybrids. In summary,
bonds of one type (formed from H1s and C2p orbitals) and a
h1 = s + px + py + pz h2 = s − px − py + pz (9A.5)
fourth σ bond of a distinctly different character (formed from
h 3 = s − px + p y − p z h 4 = s + px − p y − p z sp3 hybrid
H1s and C2s). This problem is overcome by realizing that the orbitals
electron density distribution in the promoted atom can be de-
scribed in an alternative way in which each electron occupies a As a result of the interference between the component orbit-
hybrid orbital formed from a combination of the C2s and C2p als, each hybrid orbital consists of a large lobe pointing in the
orbitals of the same atom. If these atomic orbitals are combined direction of one corner of a regular tetrahedron (Fig. 9A.6).
in a suitable way the result is four hybrid orbitals in a tetrahe- The angle between the axes of the hybrid orbitals is the tetra-
dral arrangement. hedral angle, arccos( 13 )  109.47 . Because each hybrid is built
from one s orbital and three p orbitals, it is called an sp3 hybrid
orbital.
How is that done? 9A.2 Constructing tetrahedral hybrid It is now straightforward to see how the VB description of
the CH4 molecule leads to a tetrahedral molecule containing
orbitals
four equivalent C−H bonds. Each hybrid orbital of the pro-
Each tetrahedral bond can be regarded as directed to one cor- moted C atom contains a single unpaired electron; an H1s elec-
ner of a unit cube (3). Suppose that each hybrid can be written tron can pair with each one, giving rise to a σ bond pointing
in the form h = as + bxpx + bypy + bzpz, where the orbital des- to a corner of a tetrahedron. For example, the (unnormalized)
ignations represent the corresponding atomic wavefunctions. two-electron wavefunction for the bond formed by the hybrid
The hybrid h1 that points to the corner with coordinates (1,1,1) orbital h1 and the H1s orbital is
must have equal contributions from all three p orbitals, so the
three b coefficients can be set equal to each other and h1 = as +  (1,2)  h1 (1) H1s (2)  h1 (2) H1s (1) (9A.6)
b(px + py + pz). The other three hybrids have the same composi- As for H2, to achieve this wavefunction, the two electrons it
tion (they are equivalent, apart from their direction in space), describes must be paired. Because each sp3 hybrid orbital has
but are orthogonal to h1. This orthogonality is achieved by the same composition, all four σ bonds are identical apart from
choosing different signs for the p orbitals but the same overall their orientation in space (Fig. 9A.7).
composition. For instance, choosing h2 = as + b(−px − py + pz),
A hybrid orbital has enhanced amplitude in the internuclear
the orthogonality condition is
region, which arises from the constructive interference be-
tween the s orbital and the positive lobes of the p orbitals. As a
 h h d   {as  b(p
1 2 x  p y  p z )}{as  b( p x  p y  p z )}d
1  1 1 1 result, the bond strength is greater than for a bond formed from

 a  s d  b  p x d  b  p y d  b  p2z d
2 2 22 2 2 2 an s or p orbital alone. This increased bond strength is another
factor that helps to repay the promotion energy.
0
  
 0 
The hybridization of N atomic orbitals always results in the
ab  sp x d    b  p x p y d  
2
formation of N hybrid orbitals, which may either form bonds
 a 2  b2  b2  b2  a 2  b2  0 or may contain lone pairs of electrons, pairs of electrons that

(0,0,1)

(1,1,1)

(½,½,½)
3

(0,0,0)
(1,0,0) (0,1,0)
109.47º

The values of the integrals come from the fact that the atomic
orbitals are normalized and mutually orthogonal (Topic 7C). It Figure 9A.6 An sp3 hybrid orbital formed from the superposition
follows that a solution is a = b and that the two hybrid orbitals of s and p orbitals on the same atom. There are four such hybrids:
are h1 = s + px + py + pz and h2 = s − px − py + pz. The a­ lternative each one points towards the corner of a regular tetrahedron.
9A Valence-bond theory 351

H
H1s
H

C sp2

sp3 C
H

H
H (a) (b)

Figure 9A.7 Each sp hybrid orbital forms a σ bond by overlap


3
Figure 9A.8 (a) An s orbital and two p orbitals can be hybridized
with an H1s orbital located at the corner of the tetrahedron. This to form three equivalent orbitals that point towards the corners of
model is consistent with the equivalence of the four bonds in CH4. an equilateral triangle. (b) The remaining, unhybridized p orbital is
perpendicular to the plane.

do not participate directly in bond formation (but may influ- H1s orbitals. The σ framework therefore consists of C−H and
ence the shape of the molecule). C−C σ bonds at 120° to each other. When the two CH2 groups
lie in the same plane, each electron in the two unhybridized p
Brief illustration 9A.5 orbitals can pair and form a π bond (Fig. 9A.9). The formation
of this π bond locks the framework into the planar arrange-
To accommodate the observed bond angle of 104.5° in H2O ment, because any rotation of one CH2 group relative to the
in VB theory it is necessary to suppose that the oxygen 2s and other leads to a weakening of the π bond (and consequently an
three 2p orbitals hybridize. As a first approximation, suppose increase in energy of the molecule).
they hybridize to form four equivalent sp3 orbitals. Two of the A similar description applies to ethyne, HC≡CH, a linear
hybrids are occupied by electron pairs, and so become lone molecule. Now the C atoms are sp hybridized, and the σ bonds
pairs. The remaining two pairs of electrons form two O−H are formed using hybrid atomic orbitals of the form
bonds at 109.5°. The actual hybridization will be slightly differ-
ent to account for the observed bond angle not being exactly h1  s  p z h2  s  p z sp hybrid orbitals (9A.8)

the tetrahedral angle. These two hybrids lie along the internuclear axis (the z-axis).
The electrons in them pair either with an electron in the cor-
Hybridization is also used to describe the structure of an responding hybrid orbital on the other C atom or with an elec-
ethene molecule, H2C=CH2, and the torsional rigidity of dou- tron in one of the H1s orbitals. Electrons in the two remaining
ble bonds. An ethene molecule is planar, with HCH and HCC p orbitals on each atom, which are perpendicular to the molec-
bond angles close to 120°. To reproduce the σ bonding struc- ular axis, pair to form two perpendicular π bonds (Fig. 9A.10).
ture, each C atom is regarded as being promoted to a 2s12p3 Other hybridization schemes, particularly those involving
configuration. However, instead of using all four orbitals to d orbitals, are often invoked in VB descriptions of molecular
form hybrids, sp2 hybrid orbitals are formed: structure to be consistent with other molecular geometries
(Table 9A.1).
h1  s  21/ 2 p y
h2  s   23  p x   12  p y
1/ 2 1/ 2
sp hybrid orbitals
2
(9A.7)
h3  s   
3 1/ 2
2
px   
1 1/ 2
2
py

These hybrids lie in a plane and point towards the corners of an


equilateral triangle at 120° to each other (Fig. 9A.8). The third
2p orbital (2pz) is not included in the hybridization; it lies along
an axis perpendicular to the plane formed by the hybrids. The
different signs of the coefficients, as well as ensuring that the
hybrids are mutually orthogonal, also ensure that constructive
interference takes place in different regions of space, so giving
the patterns in the illustration.
The sp2-hybridized C atoms each form three σ bonds by spin Figure 9A.9 A representation of the structure of a double bond in
pairing with either a hybrid orbital on the other C atom or with ethene; only the π bond is shown explicitly.
352 9 Molecular structure

Table 9A.1 Some hybridization schemes

Coordination
Arrangement Composition
number
2 Linear sp, pd, sd
Angular sd
3 Trigonal planar sp2, p2d
Unsymmetrical planar spd
Trigonal pyramidal pd2
4 Tetrahedral sp3, sd3
Figure 9A.10 A representation of the structure of a triple bond in Irregular tetrahedral spd2, p3d, pd3
ethyne; only the π bonds are shown explicitly. Square planar p2d2, sp2d
5 Trigonal bipyramidal sp3d, spd3
Tetragonal pyramidal sp2d2, sd4, pd4, p3d2
Brief illustration 9A.6
Pentagonal planar p2d3
Consider an octahedral molecule, such as SF6. The promotion 6 Octahedral sp3d2
of sulfur’s electrons as in Brief illustration 9A.4, followed by Trigonal prismatic spd4, pd5
the model that invokes sp3d2 hybridization results in six equiva- Trigonal antiprismatic p3d3
lent hybrid orbitals pointing towards the corners of a regular
­octahedron.

Exercises
+
E9A.3 Describe the structure of a P2 molecule in valence-bond terms. Why is E9A.4 Account for the ability of nitrogen to form four bonds, as in NH 4 .
P4 a more stable form of molecular phosphorus than P2?
E9A.5 Describe the bonding in 1,3-butadiene using hybrid orbitals.

Checklist of concepts
☐ 1. A bond forms when an electron in an atomic orbital on ☐ 4. A π bond has symmetry like that of a p orbital perpen-
one atom pairs its spin with that of an electron in an dicular to the internuclear axis.
atomic orbital on another atom. ☐ 5. Promotion is the notional excitation of an electron to
☐ 2. A σ bond has cylindrical symmetry around the internu- an empty orbital to enable the formation of additional
clear axis. bonds.
☐ 3. Resonance is the superposition of structures with ☐ 6. Hybridization is the blending together of atomic orbit-
different electron distributions but the same nuclear als on the same atom to achieve the appropriate direc-
arrangement. tional properties and enhanced overlap.

Checklist of equations
Property Equation Comment Equation number
Valence-bond wavefunction Ψ = ψA(1)ψB(2) + ψA(2)ψB(1) Spins must be paired* 9A.2
Resonance Ψ = Ψcovalent + λΨionic Ionic–covalent resonance 9A.3
Hybridization h  as  bp   All atomic orbitals on the same atom; specific forms in the text 9A.5, 9A.7, and 9A.8

* The spin contribution is   (1,2)  (1/2 ){ (1) (2)   (1) (2)}
1/ 2
TOPIC 9B Molecular orbital theory: the
hydrogen molecule-ion
electron and the nuclei; the remain- e
➤ Why do you need to know this material? ing term is the repulsive interaction rB1
rA1
between the nuclei. The collection
Molecular orbital theory is the basis of almost all descrip-
of fundamental constants e2/4πε0 R
tions of chemical bonding and computational methods, for A B
occurs widely throughout this Focus
both individual molecules and solids. 1
and is denoted j0.
➤ What is the key idea? The one-electron wavefunctions

obtained by solving the Schrödinger equation H   E  are
Molecular orbitals are wavefunctions that spread over all
called molecular orbitals. A molecular orbital ψ gives, through
the atoms in a molecule and are commonly represented as
the value of |ψ|2, the distribution of the electron in the mol-
linear combinations of atomic orbitals.
ecule. A molecular orbital is like an atomic orbital, but spreads
➤ What do you need to know already? throughout the molecule.
You need to be familiar with the shapes of atomic orbitals
(Topic 8A) and how an energy is calculated from a wave- The construction of linear
9B.1(a)
function (Topic 7C). The entire discussion is within the combinations
framework of the Born–Oppenheimer approximation (see
The Schrödinger equation for H2+ can be solved analytically.
the Prologue for this Focus).
However, the resulting wavefunctions are very complicated and
the approach cannot be extended to larger molecules. Here a
simpler approach is adopted which, although it is more approx-
imate, can be extended readily to other molecules.
In molecular orbital theory (MO theory), electrons do not If the two nuclei in H2+ are far apart, then the electron is found
belong to particular bonds but spread throughout the entire either in the H1s atomic orbital of atom A, ψA, or in the H1s
molecule. This theory has been more fully developed than atomic orbital of atom B, ψB. The two atomic orbitals are iden-
valence-bond theory (Topic 9A) and provides the language that tical so there is equal probability of the electron being found in
is widely used in modern discussions of bonding. This Topic in- each. Therefore the wavefunction for the electron in the molecule
troduces the essential features of the theory by applying it to the (the molecular orbital) can be written    A   B because in this
very simplest molecular species, the hydrogen molecule-ion, wavefunction the two atomic orbitals appear with equal weight.
H2+, which has only one electron. These ideas are used in the An equally acceptable alternative would be to write the molecu-
other Topics in this Focus to describe the structures of progres- lar orbital as    A   B. The probability density depends on the
sively more complex systems. square of the wavefunction, so the minus sign does not affect the
conclusion that the electron has equal probability of being found
in either orbital. As the separation of the nuclei is reduced, it is as-
sumed that the molecular orbitals retain these forms.
Linear combinations of
9B.1 The two molecular orbitals    A   B and    A   B are
atomic orbitals just the simplest example of the outcome of a procedure known
as the linear combination of atomic orbitals (LCAO) in which
The hamiltonian for the single electron in H2+ is a molecular orbital is constructed from a superposition of
atomic orbitals. An approximate molecular orbital formed in
2 2 e2  1 1 1 this way is called an LCAO-MO. For H2+ the two LCAO-MOs
Hˆ =− ∇1 + V V =−  + −  (9B.1)
2me 4 πε 0  rA1 rB1 R  formed from the two H1s atomic orbitals are:
Linear combination of
where rA1 and rB1 are the distances of the electron from the two    N  ( A   B ) atomic orbitals  (9B.2)
nuclei A and B (1) and R is the distance between the two nu-
clei. In the expression for V, the first two terms in parentheses where N± is a normalization factor. Because they are con-
are the attractive contribution from the interaction between the structed from s atomic orbitals both molecular orbitals have
354 9 Molecular structure

cylindrical symmetry around the internuclear axis and are Example 9B.1 Normalizing a molecular orbital
therefore described as σ orbitals. More precisely, this classifi-
cation indicates that the orbital has zero orbital angular mo- Normalize the molecular orbital ψ+ in eqn 9B.2.
mentum around the internuclear axis. Figure 9B.1 shows two
Collect your thoughts You need to find the factor N+ such that
different representations of the molecular orbital ψ+. Figures
   d  1, where the integration is over the whole of space.

9B.2 and 9B.3 are three-dimensional representations of ψ+ and To proceed, you should substitute the LCAO-MO into this
ψ− in which surfaces of constant amplitude are plotted. integral and make use of the fact that the atomic orbitals are
individually normalized and (in this case) real.
The solution Substitution of the wavefunction gives

  d  N 2  ( A   B )2d

 

 
1
1
 S

 N   A d   Bd  2  A Bd 
2

 2 2

 2(1  S)N 2

where S   A Bd and has a value that depends on the nucle-


(a) (b) ar separation (this ‘overlap integral’ will play a significant role
later). For the integral to be equal to 1,
Figure 9B.1 (a) The amplitude of the molecular orbital ψ+ of the
hydrogen molecule-ion in a plane containing the two nuclei and 1
N 
(b) a contour representation of the amplitude. {2(1  S)}1/ 2

For H2+ at its equilibrium bond length S ≈ 0.59, so N+ = 0.56.


Self-test 9B.1 Normalize the orbital ψ− in eqn 9B.2 and evalu-
ate N− for S = 0.59.
Answer: N− = 1/{2(1 − S)}1/2, so N− = 1.10

9B.1(b) Bonding orbitals


According to the Born interpretation, the probability density
of the electron at each point in H2+ is proportional to the square
modulus of its wavefunction at that point. The probability den-
sity corresponding to the wavefunction ψ+ in eqn 9B.2 is
Bonding probability
Figure 9B.2 Surfaces of constant amplitude of the wavefunction  2   A2   B2  2 A B density (9B.3)
ψ+ of the hydrogen molecule-ion.
This probability density is plotted in Fig. 9B.4. An important
feature becomes apparent in the internuclear region, where


+

Figure 9B.4 The electron density resulting from the wavefunction


Figure 9B.3 Surfaces of constant amplitude of the wavefunction ψ+ of the hydrogen molecule-ion shown in Fig. 9B.2. Note the
ψ− of the hydrogen molecule-ion. accumulation of electron density in the internuclear region.
9B Molecular orbital theory: the hydrogen molecule-ion 355

both atomic orbitals have similar amplitudes. According to eqn where EH1s is the energy of a H1s orbital, j0/R is the poten-
9B.3, the total probability density is proportional to the sum of: tial energy of repulsion between the two nuclei (recall that
j0 = e2/4πε0), and
• ψ A2 , the probability density if the electron were confined to
atom A;  R  R    R /a0
2

S   A B d  1   3    e
1
 (9B.5a)
• ψ B2, the probability density if the electron were confined to a0
  a0  
atom B;
• 2ψAψB, an extra contribution to the density from both  A2 j   R 
atomic orbitals. j  j0  d  0 1   1   e 2 R /a0   (9B.5b)
rB R   a0  
The last contribution, the overlap density, is crucial, because
it represents an enhancement of the probability of finding the  A B j  R   R /a0
k  j0  d  0  1  e  (9B.5c)
electron in the internuclear region. The enhancement can be rB a0  a0 
traced to the constructive interference of the two atomic orbit-
als: each has a positive amplitude in the internuclear region, Note that
so the total amplitude is greater there than if the electron were
confined to a single atom. This observation is summarized as j0 e2 e2 me e 2 me e 4
    2 2  2hcR  (9B.5d)
a0 4  0 a0 4  0  0 h2 4 0 h
Bonds form as a result of the build-up of electron density
where atomic orbitals overlap and interfere constructively.
The numerical value of 2hcR ∞ (when expressed in electronvolts)
The conventional explanation of this observation is based on is 27.21 eV. Figure 9B.5 shows how the integrals S, j, and k vary
the notion that accumulation of electron density between the with the internuclear separation R. These integrals are inter-
nuclei puts the electron in a position where it interacts strongly preted as follows:
with both nuclei. Hence, the energy of the molecule is lower
than that of the separate atoms, where each electron can inter- • All three integrals are positive and decline towards zero
act strongly with only one nucleus. at large internuclear separations (S and k on account of
This conventional explanation, however, has been called into the exponential term, j on account of the factor 1/R). The
question, because shifting an electron away from a nucleus into integral S is discussed in more detail in Topic 9C.
the internuclear region raises its potential energy. The modern • The integral j is a measure of the interaction between one
(and still controversial) explanation does not emerge from the nucleus and the electron density centred on the other
simple LCAO treatment given here. It seems that, at the same nucleus.
time as the electron shifts into the internuclear region, the atomic
• The integral k is a measure of the interaction between one
orbitals shrink. This orbital shrinkage improves the electron–
nucleus and the excess electron density in the internuclear
nucleus attraction more than it is decreased by the migration to
region arising from overlap.
the internuclear region, so there is a net lowering of potential en-
ergy. The kinetic energy of the electron is also modified because
the curvature of the wavefunction is changed, but the change in
kinetic energy is dominated by the change in potential energy.
Throughout the following discussion the strength of chemi-
1 1
cal bonds is ascribed to the accumulation of electron density in
the internuclear region. In molecules more complicated than
0.8 0.8
H2+ the true source of energy lowering may be this accumula-
tion of electron density or some indirect but related effect.
j/j0 and ka0/j0

0.6 0.6
The σ orbital just described is an example of a bonding orbital,
S

an orbital which, if occupied, helps to bind two atoms together. An


0.4 0.4
electron that occupies a σ orbital is called a σ electron. With this
j
notation, the configuration of the ground state of H2+ is written σ1. 0.2 0.2
The energy Eσ of the σ orbital is:1 k
0 0
j0 j  k 0 2 4 6 8 10 0 2 4 6 8 10
E  EH1s   Energy of bonding orbital (9B.4) (a) R/a0 (b) R/a0
R 1 S
1
For a derivation of eqn 9B.4, see A deeper look 9B.1, available to read in the Figure 9B.5 The dependence of the integrals (a) S, (b) j and k on
e-book accompanying this text. the internuclear distance, each calculated for H+2.
356 9 Molecular structure

Brief illustration 9B.1


+
+
It turns out (see below) that the minimum value of Eσ occurs at +
+
R = 2.49a0. At this separation
ψA
ψB ψA+ψB
 2.492  2.49
S  1  2.49  e  0.461
 3 
j /a
j  0 0 (1  3.49e 4.98 )  0.392 j0 /a0
2.49 (a) (b ) Region of constructive
j interference
k  a0 (1  2.49)e 2.49  0.289 j0 /a0
0
Figure 9B.7 A representation of the constructive interference
Therefore, with j0/a0 = 27.21 eV, j = 10.7 eV, and k = 7.87 eV. that occurs when two H1s orbitals overlap and form a bonding
The energy separation between the bonding MO and the H1s σ orbital. In (a) the atomic orbitals are far apart and so do
atomic orbital (being cautious with rounding) is not interact; in (b) the orbitals are close enough to interact
significantly.
j0 j  k
E  EH1s  
R 1 S
distances, resulting in a minimum in the potential energy
27.21 eV (10.7 eV)  (7.87 eV)
  curve. The depth of this minimum is denoted hcD e. From the
2.49 1  0.461 expressions given in eqns 9B.4 and 9B.5 it is possible to compute
 1.78 eV that Re = 2.49a0 = 132 pm and hcD e  1.76 eV (171 kJmol 1 ).
The experimental values are 106 pm and 2.6 eV, so this simple
LCAO-MO description of the molecule, while inaccurate, is
Figure 9B.6 shows a plot of E  EH1s against R, that is a plot not absurdly wrong.
of the energy of the orbital relative to that of the separated
atoms. The energy of the σ orbital decreases as the internuclear
separation is decreased from large values because electron den-
9B.1(c) Antibonding orbitals
sity accumulates in the internuclear region as the constructive The linear combination ψ− in eqn 9B.2 has higher energy than
interference between the atomic orbitals increases (Fig. 9B.7). ψ+. The orbital has cylindrical symmetry about the internuclear
However, at small separations there is too little space between access so is classified as a σ orbital. For now it is labelled σ*,
the nuclei for significant accumulation of electron density with the star indicating that it is an antibonding orbital. This
there. In addition, the potential energy of nucleus–nucleus orbital has a nodal plane perpendicular to the internuclear axis
repulsion (which is proportional to 1/R) becomes large. As a and passing through the mid-point of the bond where ψA and
result, the energy of the molecular orbital rises at short ψB cancel exactly (Figs. 9B.8 and 9B.9).
The probability density is
0.2 Antibonding probability
 2   A2   B2  2 A B density (9B.6)

0.15
~
– EH1s)/hcR

0.1
+ +
* 0.05 –
* –
or
Energy, (E

0 ψA
ψB ψA– ψB
–0.05

–0.1
0 2 4 6 8 10
Internuclear distance, R/a0
(a) (b ) Region of destructive
interference
Figure 9B.6 The calculated molecular potential energy curves for a
hydrogen molecule-ion showing the variation of the energies of the Figure 9B.8 A representation of the destructive interference that
bonding and antibonding orbitals as the internuclear distance is occurs when two H1s orbitals overlap and form an antibonding σ*
changed. The energy Eσ is that of the σ orbital and Eσ* is that of σ*. orbital.
9B Molecular orbital theory: the hydrogen molecule-ion 357

(a)

(b)
(a) (b)
Figure 9B.11 A partial explanation of the origin of bonding
Figure 9B.9 (a) The amplitude of the antibonding molecular and antibonding effects. (a) In a bonding orbital, the nuclei
orbital ψ− in a hydrogen molecule-ion in a plane containing the are attracted to the accumulation of electron density in the
two nuclei and (b) a contour representation of the amplitude. internuclear region. (b) In an antibonding orbital, the nuclei are
Note the internuclear nodal plane. attracted to an accumulation of electron density outside the
internuclear region.

whereas a bonding electron pulls two nuclei together, an anti-


bonding electron pulls the nuclei apart (Fig. 9B.11).
Figure 9B.6 also shows another feature drawn on later:
|E *  EH1s |  |E  EH1s |, which indicates that the antibonding
orbital is more antibonding than the bonding orbital is bonding.
This important conclusion stems in part from the presence
of the nucleus–nucleus repulsion ( j0/R): this contribution raises
the energy of both molecular orbitals.

Brief illustration 9B.2

Figure 9B.10 The electron density calculated by forming the At the minimum of the bonding orbital energy R = 2.49a0,
square of the wavefunction used to construct Fig. 9B.9. Note the and, from Brief illustration 9B.1, S = 0.461, j = 10.7 eV, and
reduction of electron density in the internuclear region. k = 7.87 eV. It follows that at that separation, the energy of
the antibonding orbital relative to that of a hydrogen atom 1s
orbital is
There is a reduction in the probability density between the
nuclei due to the term −2ψAψB (Fig. 9B.10); in physical terms, j0 j  k
E *  EH1s  
there is destructive interference where the two atomic orbitals R 1 S
overlap. The σ* orbital is an example of an antibonding orbital. 27.21 eV (10.7 eV)  (7.87 eV)
If such an orbital is occupied it contributes to a reduction in the  
2.49 1  0.461
cohesion between two atoms and helps to raise the energy of  5.75 eV
the molecule relative to the separated atoms.
The energy Eσ of the σ* antibonding orbital is2 At this separation the antibonding orbital lies 5.75 eV above the
energy of the separated atoms, whereas the bonding orbital lies
j0 j  k 1.76 eV below this energy. As commented on above, the anti-
E  EH1s    (9B.7)
R 1 S bonding orbital is more antibonding that the bonding orbital
where the integrals S, j, and k are the same as in eqn 9B.5. The is bonding.
variation of Eσ with R is shown in Fig. 9B.6. It is evident from
this plot that if the electron is in the antibonding orbital the en-
ergy of the molecule is raised above that of the separated atoms.
In other words, the antibonding electron has a destabilizing Exercises
effect. This effect is partly due to the fact that an antibonding
E9B.1 Normalize to 1 the molecular orbital ψ = ψA + λψB in terms of the
electron is excluded from the internuclear region and hence parameter λ and the overlap integral S. Assume that ψA and ψB are normalized
is distributed largely outside the bonding region. In effect, to 1.
E9B.2 Derive an expression for the energy separation of σ and σ* orbitals in
2
This result is obtained by applying the strategy in A deeper look 9B.1, avail- H2+ in terms of j, k, and S. Simplify your expression for the case | S | << 1. How
able to read in the e-book accompanying this text. do you interpret this result?
358 9 Molecular structure

9B.2 Orbital notation + –

For homonuclear diatomic molecules (molecules consisting of


two atoms of the same element, such as N2), it proves helpful to
+ +
label a molecular orbital according to its inversion symmetry.
This label indicates the behaviour of the wavefunction when it is
inverted through the centre of the molecule.3 Thus, any point on
Figure 9B.12 The inversion symmetry of an orbital is even (g) if its
the bonding σ orbital that is projected through the centre of the
wavefunction is unchanged under inversion through the centre
molecule and out an equal distance on the other side leads to an
of inversion (the open circle) of the molecule, but odd (u) if the
identical value (and sign) of the wavefunction (Fig. 9B.12). This wavefunction changes sign. Heteronuclear diatomic molecules do
so-called gerade symmetry (from the German word for ‘even’) not have a centre of inversion, so for them the g, u classification is
is denoted by a subscript g, as in σg. The same procedure ap- irrelevant.
plied to the antibonding σ* orbital results in the same amplitude
but opposite sign of the wavefunction. This ungerade symmetry
(‘odd symmetry’) is denoted by a subscript u, as in σu.
The inversion symmetry classification is not applicable to
heteronuclear diatomic molecules (diatomic molecules formed Exercise
by atoms from two different elements, such as CO) because E9B.3 Identify the g or u character of bonding and antibonding π orbitals
these molecules do not have a centre of inversion. formed by side-by-side overlap of p atomic orbitals.

Checklist of concepts
☐ 1. A molecular orbital is constructed from a linear combi- ☐ 4. σ Orbitals have cylindrical symmetry and zero orbital
nation of atomic orbitals. angular momentum around the internuclear axis.
☐ 2. A bonding orbital arises from the constructive overlap ☐ 5. A molecular orbital in a homonuclear diatomic mol-
of neighbouring atomic orbitals. ecule is labelled ‘gerade’ (g) or ‘ungerade’ (u) according
☐ 3. An antibonding orbital arises from the destructive to its behaviour under inversion symmetry.
overlap of neighbouring atomic orbitals.

Checklist of equations
Property Equation Comment Equation number
Linear combination of atomic orbitals ψ± = N±(ψA ± ψB) Homonuclear diatomic molecule 9B.2
Energies of σ orbitals formed from two 1s atomic orbitals E  EH1s  j0 /R  ( j  k )/(1  S) 9B.4
E*  EH1s  j0 /R  ( j  k)/(1  S) 9B.7
Molecular integrals S   A B d 9B.5a
j  j0 ( /rB )d
2
A
9B.5b

k  j0 ( A B /rB )d 9B.5c

3
More formally this is an inversion through a centre of inversion, as is dis-
cussed in Topic 10A.
TOPIC 9C Molecular orbital theory:
homonuclear diatomic molecules
9C.1(a) MO energy level diagrams
➤ Why do you need to know this material?
To be useful molecular orbital theory needs to be extended
Consider H2, the simplest many-electron diatomic molecule.
to cover molecules with more than one electron.
Each H atom contributes a 1s orbital (as in H2+), which combine
to form bonding σg and antibonding σu orbitals, as explained in
➤ What is the key idea? Topic 9B. At the equilibrium nuclear separation these orbitals
Molecular orbitals are formed from all available valence-
have the energies shown in Fig. 9C.1, which is called a molecular
shell orbitals and the building-up principle is used to iden-
orbital energy level diagram. Note that from two atomic orbit-
tify the configuration of lowest energy.
als two molecular orbitals are built. In general, from N atomic
orbitals N molecular orbitals can be built.
➤ What do you need to know already? There are two electrons to accommodate, and both can enter
You need to be familiar with the discussion of the bond-
the σg orbital by pairing their spins, as required by the Pauli
ing and antibonding molecular orbitals (Topic 9B) and the
principle. The ground-state configuration is therefore σ2g and
building-up principle for atoms (Topic 8B).
the bond consists of an electron pair in a bonding σ orbital.
This approach shows that an electron pair, which was the focus
of Lewis’s account of chemical bonding, represents the maxi-
mum number of electrons that can enter a bonding molecular
The ground electronic configurations of many-electron atoms orbital.
are found using hydrogenic atomic orbitals and the building- A straightforward extension of this argument explains why
up principle (Topic 8B). A similar approach is used for many- helium does not form diatomic molecules. Each He atom
electron homonuclear diatomic molecules, but in this case contributes a 1s orbital, so σg and σu molecular orbitals can
using the molecular orbitals resembling those for the one- be constructed. Although these orbitals differ in detail from
electron hydrogen molecule-ion (Topic 9B). those in H2, their general shapes are the same and the same
qualitative energy level diagram can be used in the discussion.
There are four electrons to accommodate. Two can enter the σg
9C.1 Electron configurations orbital, but then it is full, and the next two must enter the
σu orbital (Fig. 9C.2). The ground electronic configuration of
The starting point of the molecular orbital theory (MO theory) He2 is therefore σ2g σ2u. Recall from the discussion in Topic 9B
of bonding in diatomic molecules (and ions) is the construc- that the σu orbital is higher in energy above the separate atoms
tion of molecular orbitals as linear combinations of the avail-
able atomic orbitals. Once the molecular orbitals have been
formed, a building-up principle, like that for atoms (Topic 8B),
is used to establish their ground-state electron configurations:
• The electrons are accommodated in the molecular orbitals
Energy

H1s H1s
so as to achieve the lowest overall energy subject to the
constraint of the Pauli exclusion principle that no more
than two electrons may occupy a single orbital (and then
their spins must be paired).
• If several degenerate molecular orbitals are available, elec-
trons are added singly to each individual orbital before Figure 9C.1 A molecular orbital energy level diagram for orbitals
constructed from the overlap of H1s orbitals. The energies of the
any one orbital is doubly occupied (because that mini-
atomic orbitals are indicated by the lines at the outer edges of the
mizes electron–electron repulsions).
diagram, and the energies of the molecular orbitals are shown in
• According to Hund’s maximum multiplicity rule, if two elec- the middle. The ground electronic configuration of H2 is obtained
trons do occupy different degenerate orbitals, then a lower by accommodating the two electrons in the lowest available
energy is obtained if their spins are parallel. orbital (the bonding orbital, σg).
360 9 Molecular structure

From these four atomic orbitals four molecular orbitals of σ


symmetry can be formed by an appropriate choice of the coef-
Energy ficients c.
He1s He1s
Because the 2s and 2p orbitals on each atom have such dif-
ferent energies, they may be treated separately (this approxima-
tion is removed later). That is, the four σ molecular orbitals fall
approximately into two sets, one consisting of two molecular
orbitals formed from the 2s orbitals

Figure 9C.2 The ground-state electronic configuration of   cA 2s A 2s  cB2s B2s  (9C.2a)
the hypothetical four-electron molecule He2 (at an arbitrary
and another consisting of two orbitals formed from the 2pz
internuclear separation) has two electrons in the bonding orbital
orbitals
σg and two in the antibonding orbital σu. The molecule has a
higher energy than the separated atoms, and so is unstable with   c A 2 pz  A 2 pz  c B 2 pz  B 2 pz  (9C.2b)
respect to dissociation.
In a homonuclear diatomic molecule the energies of the 2s
than the σg is lower. Therefore, the He2 molecule has a higher orbitals on atoms A and B are the same. Their coefficients
energy than the separated atoms, so it is unstable relative to are therefore the same, cA 2s = cB2s, or simply differ in sign,
them and dihelium does not form. cA 2s   cB2s . The same is true of the 2pz orbitals on each atom.
Therefore, the two sets of orbitals have the form ψA2s ± ψB2s and
 A 2 pz   B2 pz , and in each case there is a bonding combination
9C.1(b) σ Orbitals and π orbitals and an antibonding combination.
The concepts introduced so far also apply to homonuclear dia- The molecular orbitals arising from 2s and 2pz orbitals are
tomics in general. In the elementary treatment used here, only classified as σg or σu in the same way as in Topic 9B. Orbitals
the orbitals of the valence shell are used to form molecular or- with the same label are distinguished by numbering them,
bitals so, for molecules formed with atoms from Period 2 ele- starting with the lowest in energy, as 1σg, 2σg, and so on.
ments, only the 2s and 2p atomic orbitals are considered. Therefore, the σ bonding orbital formed from the 2s orbitals
A general principle of MO theory is that is denoted 1σg and the σ antibonding orbital formed from the
same atomic orbitals is denoted 1σu.
All orbitals of the appropriate symmetry contribute to a
The two 2pz orbitals directed along the internuclear axis also
molecular orbital.
overlap strongly. They may interfere either constructively to
Thus, σ orbitals are built by forming linear combinations of all give a bonding σg orbital or destructively to give an antibond-
atomic orbitals that have cylindrical symmetry about the inter- ing σu orbital (Fig. 9C.4). If it is supposed that the 2p atomic or-
nuclear axis. These orbitals include the 2s orbitals on each atom bitals lie significantly higher in energy than the 2s orbitals, then
and the 2pz orbitals on the two atoms (Fig. 9C.3; the z-axis on the bonding and antibonding orbitals arising from the overlap
each atom lies along the internuclear axis and points towards of the 2pz lie higher in energy than the 1σg and 1σu orbitals, and
the neighbouring atom). The general form of the σ orbitals that so are labelled 2σg and 2σu, respectively.
may be formed is therefore Now consider the 2px and 2py orbitals of each atom. These
orbitals are perpendicular to the internuclear axis and overlap
  cA 2sA 2s  cB2s B2s  cA 2 pz A 2 pz  cB2 pz B2 pz  (9C.1)
broadside-on when the atoms are close together. This overlap

2s 2s
2pz 2pz
– + – +

A B
– + + –

Figure 9C.3 According to molecular orbital theory, σ orbitals


are built from all orbitals that have the appropriate symmetry. In
homonuclear diatomic molecules of Period 2, that means that two Figure 9C.4 A representation of the form of the bonding and
2s and two 2pz orbitals should be used. From these four orbitals, antibonding σ orbitals built from the overlap of p orbitals. These
four molecular orbitals can be built. illustrations are schematic.
9C Molecular orbital theory: homonuclear diatomic molecules 361

+ + + + – + + –

A B A B

(a) (b)
– +
Figure 9C.6 (a) The two orbitals are far apart so in regions of
(a) u
(b) g
space where the wavefunction of one is large, the other is small.
The overlap integral S is therefore small. (b) When the orbitals
Figure 9C.5 The parity of (a) π bonding and (b) π antibonding are closer together, there are regions of space in which both
molecular orbitals. The open circle is the centre of inversion. wavefunctions are large and so the overlap integral is significant.
Note that the overlap integral will decrease as the two atoms
may be constructive, resulting in a bonding π orbital, or destruc- approach more closely than shown here, because the region of
tive, resulting in an antibonding π orbital (Fig. 9C.5). The no- negative amplitude of the p orbital starts to overlap the positive
tation π is the analogue of p in atoms: when viewed along the amplitude of the s orbital. When the centres of the atoms
axis of the molecule, a π orbital looks like a p orbital and has one coincide, S = 0.
unit of orbital angular momentum around the internuclear axis.
The two neighbouring 2px orbitals overlap to give a bonding
and antibonding πx orbital, and the two 2py orbitals overlap to Table 9C.1 Overlap integrals between hydrogenic orbitals*
give two πy orbitals. The πx and πy bonding orbitals are degener-
ate; so too are their antibonding partners. As seen in Fig. 9C.5, Orbitals Overlap integral, S
a bonding π orbital has odd parity (u) and the antibonding π 1s,1s 1      e
1
3
2 

orbital has even parity (g). The lower two doubly degenerate
orbitals are therefore labelled 1πu and their higher energy anti-
2s,2s 1        e
1
2
1
12
2 1
240
4  / 2

bonding partners are labelled 1πg. 2px,2px (π) 1        e


1
2
1
10
2 1
120
3  / 2

2pz,2pz (σ)  1       
1
2
1
20
2 1
60
3 1
240  4  e  / 2

9C.1(c) The overlap integral *   ZR/a0

As in the discussion of the hydrogen molecule-ion, the lower-


ing of energy that results from constructive interference be-
tween neighbouring atomic orbitals (and the raising of energy (Fig. 9C.8). The integral over the region where the product of
that results from destructive interference) correlates with the the wavefunctions is positive exactly cancels the integral over
extent of overlap of the orbitals. As explained in Topic 9B, the region where the product is negative, so overall S = 0 ex-
the extent to which two atomic orbitals overlap is measured actly. Therefore, there is no net overlap between the s and px
by the overlap integral, S: orbitals in this arrangement.
The extent of overlap as measured by the overlap integral
S   A B d Overlap integral [definition] (9C.3) is suggestive of the contribution that different kinds of orbital
overlap makes to bond formation, but the value of the integral
If the amplitude of atomic orbital ψA on A is small in all the must be treated with caution. Thus, the overlap integral for
regions of space where the amplitude of atomic orbital ψB on B broadside overlap of 2px or 2py orbitals is typically greater than
is large, the product ψAψB will be small everywhere. Therefore, that for the overlap of 2pz orbitals, suggesting weaker overlap in
the overlap integral—the sum of these products—is small (Fig. σ bonding than in π bonding.
9C.6a). In contrast, if the amplitudes of the two orbitals are However, the constructive overlap in the region between
both large in the same region of space, the product ψAψB will the nuclei and on the axis is greater in σ interactions, and its
be large and the overlap integral will be significant (Fig. 9C.6b). effect on bonding is more important than the overall extent
If the two atomic orbitals are identical and are normalized (for of overlap. As a result, the separation of 1πu and 1πg orbit-
instance, 1s orbitals on the same nucleus), then S = 1 when the als is likely to be smaller than the separation of 2σg and 2σu
internuclear separation is zero. In some cases, simple formulas orbitals in the same molecule. The relative energies of these
can be given for overlap integrals (Table 9C.1); they are plotted orbitals is therefore likely to be as shown in Fig. 9C.9, and
in Fig. 9C.7. electrons occupying πu orbitals are likely to be less effective
Now consider the arrangement in which an s orbital spreads at bonding than those occupying the σg orbitals derived from
into the same region of space as a px orbital of a different atom the same p orbitals.
362 9 Molecular structure

1 2 u

1
Overlap integral, S

0.8 g

Energy
0.6 2p 2p
2s,2s 1 u

0.4

1s,1s 2 g
0.2
Figure 9C.9 As explained in the text, the separation of 1πu and
0 1πg orbitals is likely to be smaller than the separation of 2σg and
0 4 8 12 16 20
2σu orbitals in the same molecule, leading to the relative energies
Internuclear separation, ZR/a0
shown here.
1
Atom Molecule Atom
2p,2p ()
Overlap integral, S

0.5 2 u
2p,2p () 1 g

2p 2p
0
1 u

–0.5 2 g

1 u

–1 2s 2s
0 4 8 12 16 20
1
Internuclear separation, ZR/a0 g

Figure 9C.7 The variation of the overlap integral, S, between two


Figure 9C.10 The molecular orbital energy level diagram for
hydrogenic orbitals with the internuclear separation. A negative
homonuclear diatomic molecules. The lines in the middle are an
value of S corresponds to separations at which the contribution
indication of the energies of the molecular orbitals that can be
to the overlap of the positive region of one 2p orbital with the
formed by overlap of atomic orbitals. Energy increases upwards.
negative lobe of the other 2p orbital outweighs that from the
As remarked in the text, this diagram is appropriate for O2 (the
regions where both have the same sign.
configuration shown) and F2.

Li2 Be2 B2 C2 N2 O2 F2
+ 2
Constructive u

interference 1 g 2 u
2 g
+ 1 g
Energy

Destructive
interference 1 u
– 1 u
2 g
1 u

1 1
Figure 9C.8 A p orbital in the orientation shown here has zero net g u

overlap (S = 0) with the s orbital at all internuclear separations. 1 g

Figure 9C.11 The variation of the orbital energies of Period 2


9C.1(d) Period 2 diatomic molecules homonuclear diatomics.
To construct the molecular orbital energy level diagram for
Period 2 homonuclear diatomic molecules, eight molecular or- the same symmetry around the internuclear axis and contrib-
bitals are formed from the eight valence shell orbitals (four from ute jointly to the four σ orbitals. Hence, there is no guarantee
each atom). The ordering suggested by the discussion above is that this order of energies will be found, and detailed calcula-
shown in Fig. 9C.10. However, remember that this scheme as- tion shows that the order varies along Period 2 (Fig. 9C.11).
sumes that the 2s and 2pz orbitals contribute to different sets The order shown in Fig. 9C.12 is appropriate as far as N2, and
of σ molecular orbitals. In fact all four atomic orbitals have Fig. 9C.10 is appropriate for O2 and F2. The relative order is
9C Molecular orbital theory: homonuclear diatomic molecules 363

Atom Molecule Atom The ground-state electron configuration of O2, with 12


2 u
valence electrons, is found using the ordering of orbitals in
1 g Fig. 9C.10, and is 12g 12u 22g 1u4 12g (or 12g 1u*2 22g 1u4 1*g2).
2p 2p The bond order is b  12 (8  4)  2. According to the building-
2 g
up principle the two 1πg electrons occupy two different orbitals:
1 u
one will enter 1πg,x and the other will enter 1πg,y. Because the
electrons are in different orbitals they can have parallel spins,
which corresponds to a lower energy than if they were paired.
1 u Therefore, an O2 molecule is predicted to have a net spin angu-
2s 2s lar momentum with S = 1 and, in the language introduced in
1 g Topic 8C, to be in a triplet state. As electron spin is the source
of a magnetic moment, oxygen is also predicted to be paramag-
Figure 9C.12 An alternative molecular orbital energy level netic, a substance that tends to be drawn into a magnetic field
diagram for homonuclear diatomic molecules. Energy increases (Topic 15C). This prediction, which VB theory does not make,
upwards. As remarked in the text, this diagram is appropriate is confirmed by experiment.
for Period 2 homonuclear diatomics up to and including N2 (the An F2 molecule has two more electrons than an O2 molecule.
configuration shown). Its configuration is therefore 12g 1u*2 22g 1u4 1*g and b = 1, so F2
is a singly bonded molecule, in agreement with its Lewis struc-
controlled by the energy separation of the 2s and 2p orbitals in ture. The hypothetical molecule dineon, Ne2, has two more
the atoms, which increases across the period. The change in the electrons than F2: its configuration is 12g 1u*2 22g 1u4 1*g 2*u2
order of the 1πu and 2σg orbitals occurs at about N2. and b = 0. The bond order of zero is consistent with the fact that
With the molecular orbital energy level diagram established, neon occurs as a monatomic gas.
the probable ground-state configurations of the molecules are The bond order is a useful parameter for discussing the
deduced by adding the appropriate number of electrons to the characteristics of bonds, because it correlates with bond length
orbitals and following the building-up rules. Anionic species and bond strength. For bonds between atoms of a given pair of
(such as the peroxide ion, O2− 2 ) need more electrons than the ­elements:
parent neutral molecules; cationic species (such as O2+ ) need
fewer. • The greater the bond order, the shorter the bond.
Consider N2, which has 10 valence electrons. Two electrons • The greater the bond order, the greater the bond strength.
fill the 1σg orbital (with paired spins), and two more fill the
1σu orbital. There are two 1πu orbitals, so four electrons can Table 9C.2 lists some typical bond lengths in diatomic and
be accommodated in them. The last two electrons enter the polyatomic molecules. The strength of a bond is measured by
2σg orbital. Therefore, the ground-state configuration of N2 is its bond dissociation energy, and Table 9C.3 lists some experi-
12g 12u 1u4 22g , shown in Fig. 9C.12. It is sometimes helpful to mental values.
include an asterisk to denote an antibonding orbital, in which
case this configuration would be denoted 12g 1u*2 1u4 22g .
A measure of the net bonding in a diatomic molecule is its
bond order, b:
Brief illustration 9C.2
b  12 (N  N  ) Bond order [definition] (9C.4)
From Fig. 9C.12, the electron configurations and bond orders
where N is the number of electrons in bonding orbitals and N⋆
of N2 and N2+ are
is the number of electrons in antibonding orbitals.

Brief illustration 9C.1


N2 12g 1*u21u4 22g b3
N2 12g 1*u21u4 21g b  2 12
Each electron pair in a bonding orbital increases the bond
order by 1 and each pair in an antibonding orbital decreases
b by 1. For H2, b = 1, corresponding to a single bond, H−H, The cation has the smaller bond order, so you should expect
between the two atoms. In He2, b = 0, and there is no bond. it to have the smaller dissociation energy. The experimental
In N2, b  12 (8  2)  3. This bond order accords with the Lewis dissociation energies are 942 kJ mol−1 for N2 and 842 kJ mol−1
structure of the molecule (:N≡N:). for N2+ .
364 9 Molecular structure

Table 9C.2 Bond lengths⋆ X+ + e–(moving, Ek)

h – Ii
Bond Order Re/pm
HH 1 74.14
X+ + e–(stationary)
NN 3 109.76
HCl 1 127.45 Ii
CH 1 114 h

CC 1 154
2 134
3 120 Orbital i X

* More values will be found in the Resource section. Numbers in italics are mean values Figure 9C.13 An incoming photon carries an energy hn; an
for polyatomic molecules. energy Ii is needed to remove an electron from an orbital i, and
the difference appears as the kinetic energy of the electron.
Table 9C.3 Bond dissociation energies, NA hcD 0⋆
and the kinetic energy of the photoelectron (moving at speed v)
Bond Order N A hcD 0 /(kJ mol −1 ) must be equal to the energy of the incident photon, hn (Fig. 9C.13):
HH 1 432.1
h  12 me v 2  I  (9C.5)
NN 3 941.7
HCl 1 427.7 This equation can be refined in two ways. First, photoelectrons
CH 1 435 may originate from one of a number of different orbitals, each
CC 1 368
one having a different ionization energy. Hence, a series of pho-
toelectrons with different kinetic energies will be obtained, each
2 720
one satisfying h  12 me v 2  I i , where Ii is the ionization energy
3 962
for ejection of an electron from an orbital i. Therefore, by meas-
* More values will be found in the Resource section. Numbers in italics are mean values uring the kinetic energies of the photoelectrons, and knowing
for polyatomic molecules. the frequency ν, these ionization energies can be determined.
Photoelectron spectra are interpreted in terms of an ap-
proximation called Koopmans’ theorem, which states that the
Exercises ionization energy Ii is equal to the orbital energy of the ejected
E9C.1 Give the ground-state electron configurations, identify the highest electron (formally: Ii = −εi). That is, the ionization energy can
occupied molecular orbital (the HOMO), and state the bond orders of (i) Li2, be identified with the energy of the orbital from which it is
(ii) Be2, and (iii) C2. ejected. The theorem is only an approximation because it ig-
E9C.2 From the ground-state electron configurations of B2 and C2, predict nores the fact that the remaining electrons adjust their distribu-
which molecule should have the greater dissociation energy. tions when ionization occurs.
E9C.3 Which has the higher dissociation energy, F2 or F2 ?
+
The ionization energies of molecules are several electronvolts,
even for valence electrons, so it is essential to work in at least the
­ultraviolet region of the spectrum and with wavelengths of less
than about 200 nm. Much work has been done with radiation gen-
9C.2 Photoelectron spectroscopy erated by a discharge through helium: the He(I) line (1s12p1 → 1s2)
lies at 58.43 nm, corresponding to a photon energy of 21.22 eV.
So far, molecular orbitals have been regarded as purely theo- Its use gives rise to the technique of ultraviolet photoelectron
retical constructs, but is there experimental evidence for their spectroscopy (UPS). When core electrons are being studied,
existence? A particularly useful technique for probing the ener- photons of even higher energy are needed to expel them: X-rays
gies of molecular orbitals is photoelectron spectroscopy (PES). are used, and the technique is denoted XPS.
In this technique the molecules are irradiated with high-energy The kinetic energies of the photoelectrons are measured
photons of known energy. The photons are sufficiently ener- using an electrostatic deflector that produces different deflec-
getic to cause electrons to be ejected from some orbitals. By tions in the paths of the photoelectrons as they pass between
measuring the energy of the photoelectron, the ejected elec- charged plates (Fig. 9C.14). As the field strength between the
tron, it is possible to infer the energy of the orbital from which plates is increased, electrons of different speeds, and therefore
it has been ejected. kinetic energies, reach the detector. The electron flux can be re-
Energy is conserved when a photon ionizes a sample. corded and plotted against kinetic energy to obtain the photo-
Therefore, the sum of the ionization energy, I, of the m ­ olecule electron spectrum (Fig. 9C.15).
9C Molecular orbital theory: homonuclear diatomic molecules 365

Sample Brief illustration 9C.3

Lamp The photoelectrons of highest kinetic energy ejected from N2


in a spectrometer using He(I) radiation have kinetic ener-
gies of 5.63 eV. Photons of helium(I) radiation have energy
21.22 eV, so it follows that 21.22 eV = 5.63 eV + Ii, and therefore
Ii = 15.59 eV. This ionization energy is the energy needed to
Detector
Electrostatic remove an electron from the occupied molecular orbital with
analyser the highest energy of the N2 molecule, the 2σg bonding orbital.
+
Photoelectrons are also detected at 4.53 eV, corresponding to
– an ionization energy of 16.7 eV. The likely origin of these elec-
trons is the 1πu orbital.
Figure 9C.14 A photoelectron spectrometer consists of a source
of ionizing radiation (such as a helium discharge lamp for UPS
and an X-ray source for XPS), an electrostatic analyser, and an
electron detector. The deflection of the electron path caused by Photoejection commonly results in cations that are excited
the analyser depends on the speed of the electrons. vibrationally. Because different energies are needed to excite
different vibrational states of the ion, the photoelectrons ap-
pear with different kinetic energies. The result is vibrational
fine structure, a progression of lines with a spacing in energy
that corresponds to the vibrational frequency of the molecular
ion. This fine structure occurs between 16.7 eV and 18 eV in the
Signal

photoelectron spectrum of N2 shown in Fig. 9C.15.

Exercises
E9C.4 What is the speed of a photoelectron ejected from an orbital with
16 17 18 19 20 ionization energy 12.0 eV by a photon of radiation of wavelength 100 nm?
Ionization energy, I/eV
E9C.5 Predict the form of the ultraviolet photoelectron spectrum of H2.
Figure 9C.15 The photoelectron spectrum of N2 recorded using Speculate on how the corresponding spectrum of H2+ might differ from that
He(I) radiation. of H2.

Checklist of concepts
☐ 1. Molecular orbitals are constructed as linear com- ☐ 5. According to the building-up principle, electrons occupy
binations of all valence orbitals of the appropriate the available molecular orbitals so as to achieve the low-
symmetry. est total energy subject to the Pauli exclusion principle.
☐ 2. As a first approximation, σ orbitals are constructed ☐ 6. If electrons occupy different orbitals, the lowest energy
separately from valence s and p orbitals. is obtained if their spins are parallel.
☐ 3. π Orbitals are constructed from the sideways overlap of ☐ 7. The greater the bond order of a molecule or ion between
p orbitals. the same two atoms, the shorter and stronger is the bond.
☐ 4. An overlap integral is a measure of the extent of orbital ☐ 8. Photoelectron spectroscopy is a technique for deter-
overlap. mining the energies of electrons in molecular orbitals.

Checklist of equations
Property Equation Comment Equation number
Overlap integral S     B d

A
Integration over all space 9C.3

Bond order b  (N  N )
1
2

N and N are the numbers of electrons in bonding and antibonding orbitals, respectively 9C.4

Photoelectron spectroscopy h  me v  I
1
2
2
Interpret I as Ii, the ionization energy from orbital i 9C.5
TOPIC 9D Molecular orbital theory:
heteronuclear diatomic molecules
➤ Why do you need to know this material?
Many diatomic molecules are built from different elements
H F
and to understand their structure and properties it is nec-
essary to take into account the difference in the atomic
orbitals used to form their molecular orbitals.

➤ What is the key idea? Figure 9D.1 The electron density of the molecule HF, computed
The molecular orbitals in heteronuclear diatomic mole- using one of the methods described in Topic 9F. Different colours
cules have unequal contributions from the atomic orbitals show the variation in the electrostatic potential and hence the
from which they are constructed, resulting in polarization net charge, with blue representing the region with largest partial
of the bond. positive charge, and red the region with largest partial negative
charge.
➤ What do you need to know already?
You need to be familiar with the molecular orbitals of homo-
nuclear diatomic molecules (Topic 9C) and the concepts of A polar bond consists of two electrons in a bonding molecu-
normalization and orthogonality (Topic 7C). This Topic makes lar orbital of the form
light use of determinants (The chemist’s toolkit 9D.1).
  cA A  cB B Wavefunction of a polar bond (9D.1)

with unequal coefficients. The interpretation of this wavefunc-


tion is that the probability that the electron will be found on
The electrons in a covalent bond in a heteronuclear diatomic atom A is proportional to |cA|2, and the probability that it will
species are not distributed equally over the atoms: it is ener- be found on atom B is proportional to |cB|2. A nonpolar bond
getically favourable for the electron pair to be found closer to has |cA|2 = |cB|2, and a pure ionic bond has one coefficient equal
one atom than to the other. This imbalance results in a polar to zero (so the species A+B− would have cA = 0 and cB = 1). The
bond, a bond in which the bonding electron density is shared atomic orbital with the lower energy makes the larger contribu-
unequally between the bonded atoms. The bond in HF, for in- tion to the bonding molecular orbital. The opposite is true of
stance, is polar, with the bonding electron density greater near the antibonding orbital, for which the dominant contribution
the F atom than the H atom. The accumulation of bonding elec- comes from the atomic orbital with higher energy.
tron density near the F atom results in that atom having a net The distribution of partial charges in bonds is commonly dis-
negative charge, which is called a partial negative charge and cussed in terms of the electronegativity, χ (chi), of the elements
denoted δ−. There is a matching partial positive charge, δ+, on involved. The electronegativity is a parameter introduced by
the H atom (Fig. 9D.1). Linus Pauling as a measure of the power of an atom in a bond to
attract electrons to itself. Pauling used valence-bond arguments
to suggest that an appropriate numerical scale of electronega-
tivities could be defined in terms of bond dissociation energies
9D.1Polar bonds and and proposed that the difference in electronegativities could be
expressed as
electronegativity
|  A   B |  hcD 0 (AB)/eV  12 [hcD 0 (AA)/eV  hcD 0 (BB)/eV]
1/ 2

The description of polar bonds is a straightforward extension


of the molecular orbital theory of homonuclear diatomic mol-  Paulling electronegativity [definition] (9D.2)
ecules (Topic 9C). The principal difference stems from the fact
that the atomic orbitals on the two atoms have different ener- where hcD 0 (XY) is the dissociation energy of an X−Y bond.
gies and spatial extents. This expression gives differences of electronegativities; to
9D Molecular orbital theory: heteronuclear diatomic molecules 367

Table 9D.1 Pauling electronegativities⋆


Exercise
Element χP −1
E9D.1 The bond dissociation energy of O–H is 467 kJ mol , that of H–H is
H 2.2 432 kJ mol−1, and that of O–O is 146 kJ mol−1. Calculate |  Pauling (O)   Pauling (H)|;
C 2.6 note that the bond energies must be converted to eV to evaluate this
difference in electronegativities.
N 3.0
O 3.4
F
Cl
4.0
3.2
9D.2 The variation principle
Cs 0.79
The systematic way of finding the coefficients in the linear com-
* More values are given in the Resource section. binations used to build molecular orbitals is provided by the
variation principle:
establish an absolute scale Pauling chose individual values If an arbitrary wavefunction is used to calculate the energy,
that gave the best match to the values obtained from eqn 9D.2. the value calculated is never less than the true ground-state
Electronegativities based on this definition are called Pauling energy.
electronegativities (Table 9D.1). The most electronegative ele-
It can be justified by setting up an arbitrary ‘trial function’ and
ments are found in Groups 15, 16, and 17, especially in Periods
showing that the corresponding energy is not less than the true
2 and 3. It is found that the greater the difference in electron-
ground-state energy (it might be the same).
egativities, the greater the polar character of the bond. The
difference for HF, for instance, is 1.8; a C−H bond, which is
commonly regarded as almost nonpolar, has an electronegativ- How is that done? 9D.1 Justifying the variation principle
ity difference of 0.4.
Any arbitrary function can be expressed as a linear combina-
Brief illustration 9D.1 tion of the eigenfunctions ψn of the exact hamiltonian for
a molecule. In the present case, consider a normalized trial
The bond dissociation energies of H2, Cl2, and HCl are 4.52 eV, wavefunction written as a linear combination  trial  n cn n and
2.51 eV, and 4.47 eV, respectively. From eqn 9D.2, suppose that the ψn are themselves normalized and mutually
orthogonal.
|  Pauling (H)   Pauling (Cl) |  4.47  12 (4.52  2.51)  0.98  1.0
1/ 2

Step 1 Write an expression for the difference between the calcu-


lated and true energy
The energy associated with the normalized trial function is the
The spectroscopist Robert Mulliken proposed an alterna-
expectation value of the hamiltonian
tive definition of electronegativity. Mulliken argued that an
element is likely to be highly electronegative if it has a high E = ∫ψ trial
* H ˆ ψ dτ
trial
ionization energy (so it will not release electrons readily) and a
The lowest energy of the system (the energy of the ground
high ­electron affinity (so it is energetically favourable to acquire
state) is E0, the eigenvalue of Ĥ corresponding to ψ0. Consider
electrons). The Mulliken electronegativity scale is therefore
the difference E − E0 , which can be expressed as follows
based on the definition
  1 
Mulliken electronegativity ˆ
  (I  Eea )/eV ∫ψ Hψ trial dτ − E0 ∫ψ trialψ trial dτ
1
2 [definition]
(9D.3) =
E − E0 *
trial

because the trial wavefunction is normalized. Bringing the


where I is the ionization energy of the element and Eea is its constant E0 into the second integral leads to
electron affinity. The greater the value of the Mulliken electron-
egativity the greater is the contribution of that atom to the elec- E − E0 = ∫ψ ⋆trial Hˆψ trial dτ −∫ψ ⋆trial E0ψ trial dτ
tron distribution in the bond. There is one word of caution: the = ∫ψ trial

( Hˆ − E0 ) ψtrial dτ
values of I and Eea in eqn 9D.3 are strictly those for a special ‘va-
lence state’ of the atom, not a true spectroscopic state, but that Replacing the trial wavefunction with the linear combination
complication is ignored here. The Mulliken and Pauling scales of eigenfunctions of the hamiltonian gives
are approximately in line with each other. A reasonably reliable
conversion between the two is E − E0 = ∫ ∑c ψ ⋆
n

n ( Hˆ − E0 ) ∑ cn ψ n dτ
n n

 Pauling  1.35  Mulliken


1/ 2
 1.37  (9D.4) = ∑cn cn ∫ψ n ( Ĥ − E0 )ψ n dτ
⋆ ⋆

n,n
368 9 Molecular structure

where the coefficients are gathered outside the integral and take arbitrary values. Because it is real, write  *   , and to
the two sums become a double sum over all possible values of normalize it multiply it by N  1/( * d )1/ 2  1/( 2 d )1/ 2 .
n and n′. From now on Nψ is used as the trial function. Then follow
Step 2 Simplify the expression these steps.
Because ∫ψ n* Hˆψ n′dτ =
En′ ∫ψ n*ψ n′dτ and  n* E0 nd  Step 1 Write an expression for the energy
E0  n nd , write
*
The energy is the expectation value of the hamiltonian. Using
the normalized real trial function Nψ this expectation value is
∫ψ *
n (Hˆ − E0 )ψ n′dτ =
(En′ − E0 )∫ψ n*ψ n′dτ
ˆ
It follows that ˆψ dτ ∫ψ Hψ dτ 
∫ψ
2
=E N= H (9D.5)
1 if n n ∫ψ 2dτ
0 otherwise
  The denominator is
E  E0  cn*cn (En  E0 )  n* nd
 d   (cA A  cB B )2 d
n ,n  2

1 1  S 


Step 3 Analyse the final expression
 cA2  A2 d  cB2  B2d  2cAcB  A Bd
The eigenfunctions ψn are orthogonal, so only terms with n′ = n
contribute to this sum. Because each eigenfunction is normal-  cA2  cB2  2cAcBS
ized, each surviving integral is 1. Consequently
because the individual atomic orbitals are normalized to 1
0 
 0 
 and the third integral is the overlap integral S (eqn 9C.3,
E  E0   cn*cn (En  E0 )  0 S   A Bd ). The numerator is
n

The quantity cn*cn is necessarily real and greater than or equal to ∫ψ Hˆψ ∫ (cAψ A + cBψ B )Hˆ (cAψ A + cBψ B ) dτ
dτ =
αA αB β
zero, and because E0 is the lowest energy, En − E0 is also greater    
ˆ ˆ ˆ
= c ∫ψ A Hψ A dτ + cB ∫ψ B Hψ Bdτ + cAcB ∫ψ A Hψ Bdτ
2 2
than or equal to zero. It follows that the product of the two A

terms on the right is greater than or equal to zero. Therefore,  β


E ≥ E0, as asserted. + cAcB ∫ψ B Hˆψ A dτ

The significance of the quantities αA, αB, and β (which are all
The variation principle is the basis of all modern molecu- energies) is discussed shortly.1 The hamiltonian is hermitian
lar structure calculations. The principle implies that, if the (Topic 7C), therefore the third and fourth integrals are equal and
coefficients in the trial wavefunction are varied until the low-
∫ψ Hˆψ dτ = c α + cB2α B + 2cAcB β
2
est energy is achieved (by evaluating the expectation value A A

of the hamiltonian for the wavefunction in each case), then


At this point the complete expression for E is
those coefficients will be the best for that particular form of
trial function. A lower energy might be obtained with a more cA2  A  cB2 B  2cAcB 
E
complicated wavefunction, such as by taking a linear combi- cA2  cB2  2cAcBS
nation of several atomic orbitals on each atom. However, for
a ­molecular orbital constructed from a given basis set, a given Step 2 Minimize the energy
set of atomic orbitals, the variation principle gives the optimum Now search for values of the coefficients in the trial function that
­molecular orbital of that kind. minimize the value of E. The procedure is a standard problem in
calculus, and is solved by finding the coefficients for which

9D.2(a) The procedure E


0
E
0
cA cB
The practical application of the variation principle can be
­illustrated by applying it to the trial wavefunction in eqn 9D.1, The derivative is most easily calculated by first multiplying
where the coefficients define the trial function. both sides of the expression for E by the denominator of the
quotient to give
How is that done? 9D.2 Applying the variation principle to E(cA2  cB2  2cAcBS)  cA2  A  cB2 B  2cAcB 
a heteronuclear diatomic molecule
The trial wavefunction in eqn 9D.1,   cA A  cB B , is real 1
Note that here α and β are nothing to do with electron spin: here they sym-
but not normalized because at this stage the coefficients can bolize two contributions to the energy.
9D Molecular orbital theory: heteronuclear diatomic molecules 369

The derivative with respect to cA is then found, being careful to The chemist’s toolkit 9D.1
note that E is a function of cA so the term on the left must be
Determinants
differentiated as a product. Therefore A 2 × 2 determinant is the entity
 (uv )/x
   a b
 (u /x )v
  ad  bc 2  2 Determinant
 u(v /x )
  c d
E 2
(cA  cB2  2cAcBS)  E(2cA  2cBS)  2cA A  2cB 
cA A 3 × 3 determinant is evaluated by expanding it as a sum of
2 × 2 determinants:
Now bring the term E(2cA + 2cBS) to the right, divide both sides
by cA2 + cB2 + 2cAcBS and obtain a b c
e f d f d e
E 2cA A  2cB   E(2cA  2cBS) d e f a b c
 h i g i g h
cA cA2  cB2  2cAcBS g h i
2{( A  E )cA  (  SE )cB }  a(ei  fh)  b(di  fg )  c(dh  eg )

cA2  cB2  2cAcBS
3  3 Determinant
The expression for ∂E /∂cB is found by exchanging the indices
A and B. The two expressions for the derivatives are therefore Note the sign change in alternate columns (b occurs with a
negative sign in the expansion). Determinants can be evaluated
most easily by using mathematical software.
 E 2{(α A − E )cA + (β − SE )cB }
=
c A cA2 + cB2 + 2cAcB S
 E 2{(α B − E )cB + (β − SE )cA }
=
cB cA2 + cB2 + 2cAcB S
value in eqn 9D.6. As for any set of simultaneous equations, the
For the derivatives to be equal to 0, the numerators, and spe- secular equations have a solution if the secular determinant,
cifically the terms in boxes, of these expressions must vanish, the determinant of the coefficients (The chemist’s toolkit 9D.1),
leading to the secular equations:2 is zero. That is, if

(α A − E )cA + (β − SE )cB = 0 (9D.6a)  A  E   SE


 ( A  E )( B  E )  (  SE )2
(α B − E )cB + (β − SE )cA = 0 Secular equations (9D.6b)   SE  B  E
 (1  S 2 )E 2  {2  S  ( A   B )}E  (9D.8)
The quantities αA, αB, β, and S in the secular equations are  ( A B   )  0
2

ψ Hˆψ dτ α ∫ψ Hˆψ B dτ
Coulomb
=αA ∫= A A B B integrals  (9D.7a) This is a quadratic equation for E. A quadratic equation of the
form ax2 + bx + c = 0 has the solutions
=β ψ Hˆψ dτ ∫ψ
∫= A B B Hˆψ A dτ Resonance
integral  (9D.7b)
b  (b2  4ac)1/ 2
x
2a
S   A B d Overlap integral (9D.7c)
In the present case, a = 1 − S2, b = 2βS − (αA + αB), and c = αAαB − β2,
The parameter α is called a Coulomb integral. It is negative so the solutions (the energies) are
and can be interpreted as the energy of the electron when it oc-
cupies ψA (for αA) or ψB (for αB). In a homonuclear diatomic E 
 A   B  2  S  {(2  S  ( A   B ))2  4(1  S 2 )( A B   2 )}1/ 2
molecule, αA = αB. The parameter β is called a resonance inte-
gral (for classical reasons). It vanishes when the orbitals do not 2(1  S 2 )
(9D.9a)
overlap, and at equilibrium bond lengths it is normally nega-
tive. The overlap integral S is discussed in Topic 9C. which, according to the variation principle, are the closest ap-
In order to solve the secular equations for the coefficients proximations to the true energy for a trial function of the form
cA and cB it is necessary to know the energy E and then use its given in eqn 9D.1. They are the energies of the bonding and
antibonding molecular orbitals formed from the two atomic
2 orbitals.
The name ‘secular’ is derived from the Latin word for age or generation.
The term comes from astronomy, where the same equations appear in connec- Equation 9D.9a can be simplified. For a homonuclear dia-
tion with slowly accumulating modifications of planetary orbits. tomic, αA = αB = α and then
370 9 Molecular structure

0 X+ + e– I/eV Eea/eV  21 {I  Eea }/eV


H 13.6 0.75 −7.2
I(X)

Energy
F 17.4 3.34 −10.4
–½{I(X) + Eea(X)}

–I(X) X Therefore set αA = αH = −7.2 eV and αB = αF = −10.4 eV. With


Eea(X) β = −1.0 eV, a typical value, and with S = 0 for simplicity,
–I(X) – Eea(X) X–
eqn 9D.9c becomes
1/ 2
Figure 9D.2 The procedure for estimating the Coulomb integral in   2.0 2 
terms of the ionization energy and electron affinity. E /eV  12 (7.2  10.4)  12 (7.2  10.4) 1    
  7.2  10.4  
(2  2 S ) 2  8.8  1.9  10.7 and  6.9
 
2  2  S  {(2  S  2 )  4(1  S 2 )( 2   2 )}1/ 2
2
These values, representing a bonding orbital at −10.7 eV and an
E 
1  S2 )
2 ( antibonding orbital at −6.9 eV, are shown in Fig. 9D.3.
(1 S )(1 S )
Ionization limit
   S  (   S) (   )(1 ∓ S)
 
(1  S)(1  S) (1  S)(1  S)

10.7 eV

10.4 eV
6.9 eV
7.2 eV
That is,
0.96ψH1s – 0.28ψF2p
   
E  E  Homonuclear diatomics  (9D.9b) H1s
1 S 1 S

For β < 0, E+ is the lower energy solution. F2p


0.28ψH1s + 0.96ψF2p
For heteronuclear diatomic molecules, making the approxi-
mation that S = 0 (simply to obtain a more transparent expres- Figure 9D.3 The estimated energies of the Coulomb integrals α
sion) turns eqn 9D.9a into for H1s and F2p in HF and the resulting energies of the bonding
and antibonding molecular orbitals.
1/ 2
  2  
2

E  ( A   B )  ( A   B ) 1  
1
2
1
2  
  B  
  A   9D.2(b) The features of the solutions
Zero oveerlap approximation (9D.9c)
An important feature of eqn 9D.9c is that as the energy differ-
ence |αA − αB| between the interacting atomic orbitals increases,
The values of the Coulomb integrals αA and αB may be esti-
the bonding and antibonding effects decrease (Fig. 9D.4).
mated as follows. The extreme case of an atom X in a molecule
is X+ if it has lost control of the electron it supplied, X if it is 6
sharing the electron pair equally with its bonded partner, and
X− if it has gained control of both electrons in the bond. If X+ is 4
taken as defining the energy 0, then X lies at −I(X) and X− lies
E/|αA + αB| + ½

2 E–
at −{I(X) + Eea(X)}, where I is the ionization energy and Eea the
electron affinity (Fig. 9D.2). The actual energy of the electron in 0
the molecule lies at an intermediate value, and in the absence of
further information, it is reasonable to estimate it as half-way –2 E+
down to the lower of these values, namely at  12 {I ( X )  Eea ( X )}.
–4
This quantity should be recognized (apart from its sign) as the
Mulliken definition of electronegativity. –6
0 2 4 6 8 10
|αA – αB|/|αA + αB|
Brief illustration 9D.2
Figure 9D.4 The variation of the energies of the molecular orbitals
Consider HF. The general form of the molecular orbital is (the thicker lines) as the energy difference of the contributing atomic
ψ = cHψH + cFψF, where ψH is an H1s orbital and ψF is an F2pz orbitals is changed; the plots are for β = −1 (in the units adopted for E
orbital (with z along the internuclear axis, the convention for and α). The thinner lines are for the energies in the absence of mixing
linear molecules). The relevant data are as follows: (i.e. β = 0) and correspond to energies αA and αB of the atomic orbitals.
9D Molecular orbital theory: heteronuclear diatomic molecules 371

When |αB − αA| >> 2|β| it is possible to use the approximation As before, this expression becomes more transparent in two
(1  x )1/ 2  1  12 x (see the Resource section) to obtain cases. First, for a homonuclear diatomic, with αA = αB = α and
E± given in eqn 9D.9b, the results are
2 2
E   A  E   B   (9D.10)
A  B A  B   1 Homonuclear
E  cA  cB  c A diatomics
 (9D.14a)
1 S {2(1  S)}1/ 2
As these expressions show, and as can be seen from the graph,
when the energy difference |αA − αB| is very large, the energies   1 Homonuclear
of the resulting molecular orbitals (the curved lines) differ only E  cA  cB  cA diatomics
 (9D.14b)
1 S {2(1  S)}1/ 2
slightly from those of the atomic orbitals (the lines), which im-
plies in turn that the bonding and antibonding effects are small.
That is: These values are the same as those asserted in Topic 9B on the
basis of symmetry considerations.
The strongest bonding and antibonding effects are For a heteronuclear diatomic with S = 0,
obtained when the two contributing orbitals have similar
energies. 1
cA  1/ 2

The large difference in energy between core and valence orbitals     E 2 
Zero overlap
1  
A
  approximation (9D.15)
    
is the justification for neglecting the contribution of core orbitals
to molecular orbitals constructed from valence atomic orbitals.
Although the core orbitals of one atom have a similar energy to with the appropriate values of E = E± taken from eqn 9D.9c. The
the core orbitals of the other atom, so might be expected to com- coefficient cB is then calculated from eqn 9D.11 (with S = 0).
bine strongly, core–core interaction is largely negligible because
the core orbitals are so contracted that the interaction between
them, as measured by the value of |β|, is negligible. It is also a
justification for treating the s and pz contributions to σ orbital Brief illustration 9D.3
formation separately, an approximation used in Topic 9C in the
discussion of homonuclear diatomic molecules. Consider HF again. In Brief illustration 9D.2, with αH =
The values of the coefficients in the linear combination in −7.2 eV, αF = −10.4 eV, β = −1.0 eV, and S = 0, the two orbital
eqn 9D.1 are obtained by solving the secular equations after energies were found to be E+ = −10.7 eV and E– = −6.9 eV.
substituting the two energies obtained from the secular de- When these values are substituted into eqn 9D.15 the following
coefficients are found:
terminant. The lower energy, E+, gives the coefficients for the
bonding molecular orbital, the upper energy, E–, the coeffi-
cients for the antibonding molecular orbital. The secular equa- E  10.7 eV    0.28 H  0.96 F
tions give expressions for the ratio of the coefficients. Thus, E  6.9 eV    0.96 H  0.28 F
the first of the two secular equations in eqn 9D.6a, (αA − E)cA +
(β − ES)cB = 0, gives Notice how the lower energy orbital (the one with energy
−10.7 eV) has a composition that is more F2p orbital than H1s,
  E  and that the opposite is true of the higher energy, antibonding
cB    A  cA  (9D.11)
orbital.
   ES 
The wavefunction should also be normalized. It has already
been shown that  2 d  cA2  cB2  2cA cB S, so normalization
requires that

cA2  cB2  2cA cB S  1 (9D.12) Exercises


E9D.2 Using the data below estimate the orbital energies to use in a calculation
When eqn 9D.11 is substituted into this expression, the result is
of the molecular orbitals of HCl.
1
cA  1/ 2

    E 2 I/(kJ mol−1) Eea/(kJ mol−1)
   E  
1    2S  A
A
  (9D.13) H 1312.0 72.8
    ES     ES  
Cl 1251.1 348.7
which, together with eqn 9D.11, gives explicit expressions for
the coefficients once the appropriate values of E = E± given in E9D.3 Use the values derived in E9D.2 to estimate the molecular orbital
eqn 9D.9a are substituted. energies in HCl using (i) S = 0 and (ii) S = 0.20. Take   1.0 eV.
372 9 Molecular structure

Checklist of concepts
☐ 1. A polar bond can be regarded as arising from a molecu- ☐ 4. The variation principle provides a criterion for optimiz-
lar orbital that is concentrated more on one atom than ing a trial wavefunction.
its partner. ☐ 5. A basis set is the set of atomic orbitals from which the
☐ 2. The electronegativity of an element is a measure of the molecular orbitals are constructed.
power of an atom to attract electrons to itself in a bond. ☐ 6. The bonding and antibonding effects are strongest when
☐ 3. The electron pair in a bonding orbital is more likely to contributing atomic orbitals have similar energies.
be found on the more electronegative atom; the opposite
is true for electrons in an antibonding orbital.

Checklist of equations
Property Equation Comment Equation number
Molecular orbital ψ = cAψA + cBψB 9.D1
Pauling electronegativity |  A   B|  {hcD 0 (AB)/eV  12 [hcD 0 (AA)/eV  hcD 0 (BB)/eV]}1/ 2 9.D2

Mulliken electronegativity   12 (I  Eea )/eV 9.D3


Coulomb integral α A = ∫ψ A Hˆψ A dτ Definition 9D.7a

Resonance integral β = ∫ψ A Hˆψ B dτ Definition 9D.7b


TOPIC 9E Molecular orbital theory:
polyatomic molecules
­ olecules, and then solving them for the energies (Topic 9D).
m
➤ Why do you need to know this material? That step involves formulating the secular determinant and
finding the values of the energy that ensure the determinant is
Molecular orbital theory is readily extendable to molecules
equal to 0. Finally, these energies are used in the secular equa-
considerably more complex than diatomic molecules. This
tions to find the coefficients of the atomic orbitals for each mo-
material gives insight into how more sophisticated calcula-
lecular orbital. Such calculations are best done using software,
tions work.
which is now widely available.
➤ What is the key idea? Symmetry considerations play a central role in the construc-
tion of molecular orbitals of polyatomic molecules, because
Molecular orbitals can be expressed as linear combinations
only atomic orbitals of matching symmetry have non-zero
of all the atomic orbitals of the appropriate symmetry.
overlap and contribute to a molecular orbital. To discuss these
➤ What do you need to know already? symmetry requirements fully requires the machinery devel-
oped in Focus 10, especially Topic 10C. There is one type of
This Topic extends the approach used for heteronuclear
symmetry, however, that is intuitive: the planarity of conju-
diatomic molecules in Topic 9D, particularly the concepts
gated hydrocarbons. That symmetry provides a distinction be-
of secular equations and secular determinants. The prin-
tween the σ and π orbitals of the molecule, and in elementary
cipal mathematical technique used is matrix algebra (The
approaches such molecules are commonly discussed in terms
chemist’s toolkit 9E.1). You should become familiar with
of the characteristics of their π orbitals, with the σ bonds pro-
the use of mathematical software to manipulate matrices
viding a rigid framework that determines the general shape of
numerically.
the molecule.

The principal difference between diatomic and polyatomic


9E.1 The Hückel approximation
molecules lies in the greater range of shapes that are possi-
The π molecular orbital energy level diagrams of conjugated
ble: a diatomic molecule is necessarily linear, but a triatomic
molecules can be constructed by using a set of approximations
molecule, for instance, may be either linear or angular (bent)
suggested by Erich Hückel in 1931, and provide a simple in-
with a characteristic bond angle. The shape of a polyatomic
troduction to this very technical subject. The method is intro-
molecule—the specification of its bond lengths and its bond
duced here by considering the π orbitals of ethene.
angles—can be predicted by calculating the total energy of the
molecule for a variety of nuclear positions, and then identifying
the conformation that corresponds to the lowest energy. 9E.1(a) An introduction to the method
The molecular orbitals of polyatomic molecules are built in
The π orbitals are expressed as linear combinations of the C2p
much the same way as in diatomic molecules (Topic 9D), the
orbitals that lie perpendicular to the molecular plane. In ethene
only difference being that more atomic orbitals are used to con-
there are just two such orbitals, and so
struct them. As for diatomic molecules, polyatomic molecular
orbitals spread over the entire molecule and the electrons that   cA A  cB B (9E.2)
occupy them spread like a web over the atoms to bind them all where ψA and ψB are C2p orbitals on atoms A and B. Next, the
together. A molecular orbital has the general form optimum coefficients and energies are found by the variation
principle as explained in Topic 9D. That is, the appropriate
  ci i General form of LCAO-MO (9E.1) secular determinant is set up, equated to 0, and the equation
i
solved for the energies. For ethene, the starting point is the pair
where ψi is an atomic orbital and the sum extends over all the of secular equations developed in Topic 9D:
valence orbitals of all the atoms in the molecule. As will be
explained in detail as this Topic unfolds, the coefficients are ( A  E )cA  (  ES)cB  0
 (9E.3a)
found by setting up the secular equations, just as for diatomic (  ES)cA  ( B  E )cB  0
374 9 Molecular structure

The corresponding secular determinant is The building-up principle results in the configuration 1π2,
because each carbon atom supplies one electron to the π system
 A  E   ES and both electrons can occupy the bonding orbital. The highest
 (9E.3b)
  ES  B  E occupied molecular orbital in ethene, its HOMO, is the 1π
orbital. The lowest unoccupied molecular orbital, its LUMO,
The determinant is much simpler to handle if the Hückel is the 2π orbital. These two orbitals jointly form the frontier
approximations are made. These approximations are orbitals of the molecule. The frontier orbitals are important be-
cause they are largely responsible for many of the chemical and
• All overlap integrals are set equal to zero.
spectroscopic properties of this and analogous molecules.
• All resonance integrals between non-neighbours are set
equal to zero.
• All Coulomb integrals for carbon atoms are set equal (to α). Brief illustration 9E.1
• All remaining resonance integrals are set equal (to β).
The energy needed to excite a π* ← π transition is equal to
the separation of the 1π and 2π orbitals, which in the Hückel
Both α and β are negative. These approximations are obviously
framework is 2|β|. This transition is known to occur at close to
very severe and so the resulting molecular orbitals are only ap-
40 000 cm−1, corresponding to 5.0 eV. It follows that a plausible
proximate. Nevertheless, the method is useful in that it gives,
value of β is about −2.5 eV (−240 kJ mol−1).
without too much computation, a general picture of the mo-
lecular orbitals and their energies.
When the Hückel approximations are used the secular deter-
minant has the following structure: 9E.1(b) The matrix formulation of the method
• All diagonal elements: α − E To make the Hückel theory readily applicable to bigger mole-
cules, it is helpful to reformulate the secular equations in terms
• Off-diagonal elements between neighbouring atoms: β
of matrices (see The chemist’s toolkit 9E.1 for a short introduc-
• All other elements: 0 tion to matrices; for more detail refer to the extended version of
In the case of ethene the secular determinant (eqn 9E.3b) becomes this toolkit available via the e-book).
To generalize the secular equations, eqn 9E.3a, the first step
 E  is to write αJ = HJJ (with J = A or B), β = HAB = HBA, and then to
  E invoke the Hückel approximation that S = 0:
The energies are found by setting the determinant equal to zero. (H AA  E )cA  H AB cB  0
The determinant is then expanded as explained in The chemist’s
H BA cA  (H BB  E )cB  0
toolkit 9D.1 to give
The resonance integral β in the second equation has been writ-
α−E β ten HBA in order to show the symmetry of the two equations on
= (α − E )2 − β 2
β α−E interchanging A and B.
= (α − E + β )(α − E − β ) = 0 (9E.4) The next step is to note that these two simultaneous equa-
tions have two solutions. That is, they are satisfied when E has
The second line follows from a 2  b2  (a  b)(a  b). The roots either of the two values E1 and E2, and for each energy there
of the equation are E = α ± β. The + sign corresponds to the is a different wavefunction with its own set of coefficients. The
bonding combination (β is negative) and the − sign corre- coefficients in the wavefunction with energy E1 are written c1,A
sponds to the antibonding combination (Fig. 9E.1). and c1,B, and those in the wavefunction with energy E2 are c2,A
and c2,B. For the first solution the equations are therefore
2 α–β
(H AA  E1 )c1,A  H AB c1,B  0
 (9E.5a)
H BA c1,A  (H BB  E1 )c1,B  0
Energy

C2p C2p
and for the second they are

(H AA  E2 )c2,A  H AB c2,B  0
1 α+β  (9E.5b)
H BA c2,A  (H BB  E2 )c2,B  0

Figure 9E.1 The Hückel molecular orbital energy levels of ethene. When the terms in E1 are taken to the right in eqn 9E.5a the
Two electrons occupy the lower π orbital. result is
9E Molecular orbital theory: polyatomic molecules 375

H AA c1,A  H AB c1,B  E1c1,A Then, for example, comparison of the top left-hand element on
H BA c1,A  H BB c1,B  E1c1,B each side gives
H AA c1,A  H AB c1,B  E1c1,A
These two equations can be written in matrix form as
which is one of the original equations in eqn 9E.5a. Similarly
 H AA H AB   c1,A   c1,A  comparison of the top right-hand element on each side gives
    E1   (9E.6a)
 H BA H BB   c1,B   c1,B  H AA c2,A  H AB c2,B  E2 c2,A
That eqn 9E.6a is true can be verified by multiplying the col- which is one of the equations in eqn 9E.5b.
umn vector by the matrix, as shown in The chemist’s toolkit The matrix equation 9E.7 can be written
9E.1. A similar matrix equation arises from eqn 9E.5b:
Hc = cE  (9E.8)
 H AA H AB   c2,A   c2,A 
    E2   (9E.6b) where
 H BA H BB   c2,B   c2,B 
 H AA H AB   c1,A c2,A   E1 0
Equations 9E.6a and 9E.6b can be expressed as a single equa- H   c   E  
tion by combining both sets of coefficients into a single matrix,  H BA H BB   c1,B c2,B  0 E2 
with each pair of coefficients forming a column in the new
matrix, and writing the energies as the elements of a diagonal The first matrix, H, is called the hamiltonian matrix. The final
­matrix step is to multiply both sides of eqn 9E.8 from the left by c−1, the
inverse of the matrix c, to give
 H AA H AB   c1,A c2,A   c1,A c2,A   E1 0 1
     (9E.7) 
 H BA H BB   c1,B c2,B   c1,B c2,B   0 E2  c Hc  c c E
1 1

That eqn 9E.7 is true can be verified by multiplying the matrices By definition c−1c = 1, and multiplication by the unit matrix 1
on the left and right, and then comparing them element by ele- has no effect, Therefore
ment on the left and right. Multiplication gives c 1 Hc  E  (9E.10)

 H AA c1,A  H AB c1,B H AA c2,A  H AB c2,B   E1c1,A E2 c2,A  The matrix E is diagonal, with diagonal elements E1 and E2. The
   left-hand side must therefore also be a diagonal matrix. The in-
 H BA c1,A  H BB c1,B H BA c2,A  H BB c2,B   E1c1,B E2 c2,B 
terpretation of this equation is that the energies are calculated

The chemist’s toolkit 9E.1 Matrices


A matrix is an array of numbers, as in the following two exam- 2 Matrix multiplication:
ples of 2 × 2 matrices:
M11 M12 N11 N 12
 M11 M12   N11 N12  MN =
M   N   M21 M22 N21 N 22
 M21 M22   N 21 N 22 
M11 N11 + M12 N21 M11 N 12 + M12 N 22
A general element is denoted Mr[ow]c[olumn]. In a diagonal matrix =
M21 N11 + M22 N21 M21 N 12 + M22 N 22
the only non-zero elements are along the diagonal, that is ele-
ments for which the indices r and c are the same. A unit matrix, Note that MN might differ from NM. If N is a matrix with 2
1, is a diagonal matrix in which the elements along the diagonal rows and 1 column, a ‘column vector’, simply ignore all but the
are all 1. For example, a 2 × 2 unit matrix is first column.
1 0 3 Matrix division
1 
0 1
Just as x/y can be expressed as y−1x, so matrix division is multi-
1 Matrix addition and subtraction. Add or subtract correspond- plication by the inverse of the matrix, the matrix M−1 such that
ing elements: M−1M = MM−1 = 1. Inverse matrices are best found by using
 M11  N11 M12  N12  mathematical software.
MN  
 M21  N 21 M22  N 22  For more detail, see the extended version of this toolkit via
the e-book.
 M11  N11 M12  N12 
MN  
 M21  N 21 M22  N 22 
376 9 Molecular structure

by finding a matrix c which, by constructing the product c−1Hc, The unit matrix is already diagonal. The matrix c that diagonal-
results in a diagonal matrix. This procedure is called matrix izes M is given by mathematical software as
diagonalization and is best carried out by using mathematical
software (see the extended version of The chemist’s toolkit 9E.1  0.372 0.602 0.602 0.372 
 
available via the e-book). Once the software finds the matrix c,  0.602 0.372 0.372 0.602 
c
the definitions following eqn 9E.8 show that its columns are the  0.602 0.372 0.3372 0.602 
coefficients of the orbitals used as the basis set. Then, as implied  
 0.372 0.602 0.602 0.372 
by eqn 9E.10, the elements along the diagonal of the diagonal-
ized matrix are the energies of the orbitals. and the diagonalized version of H is
It follows that the molecular orbitals can be found as follows:
   1.62  0 0 0 
1. Construct the matrix H  
0   0.62  0 0
2. Find the matrix c that diagonalizes H E= 
 0 0   0.62  0 
3. The columns of c are the coefficients of the molecular orbitals  
 0 0 0   1.62  
4. Their energies are given by the corresponding diagonal
elements of the matrix c−1Hc. The columns of the matrix c are the coefficients of the atomic
orbitals for the corresponding molecular orbital. It follows that
Although these equations have been developed for a basis of the energies and molecular orbitals are
just two orbitals they apply equally well to larger basis sets. The
matrix H is simply expanded accordingly, as is done in the fol- E1    1.62   1  0.372 A  0.602 B  0.602 C  0.372 D
lowing example. E2    0.62   2  0.602 A  0.372 B  0.372 C  0.602 D
E3    0.62   3  0.602 A  0.372 B  0.372 C  0.602 D
Example 9E.1 Finding molecular orbitals by matrix E4    1.62   4  0.372 A  0.602 B  0.602 C  0.372 D
diagonalization
where the C2p atomic orbitals are denoted by ψA,…, ψD. The
Set up and solve the matrix equations within the Hückel molecular orbitals are mutually orthogonal and, with overlap
approximation for the π orbitals of butadiene (1). neglected, normalized.
Comment. Note that ψ1,…, ψ4 correspond to the 1π,…, 4π
1 Butadiene molecular orbitals of butadiene.

Collect your thoughts The matrices are four-dimensional (that Self-test 9E.1 Repeat the exercise for the allyl radical,
is, 4 × 4) as each carbon atom in the π system contributes one ·CH2−CH=CH2; assume that each carbon atom is sp2 hybridized,
out-of-plane p orbital. You need to follow the four steps set and take as a basis one out-of-plane 2p orbital on each atom.
out in the text. Mathematical software operates with numerical ψ2 = 0.707ψA − 0.707ψC, ψ3 = 0.500ψA − 0.707ψB + 0.500ψC
matrix elements, so write H, which is expressed in terms of the Answer: E = α + 1.41β, α, α − 1.41β; ψ1 = 0.500ψA + 0.707ψB + 0.500ψC,
parameters α and β, as the sum of two matrices, one multi-
plied by α and the other multiplied by β. Then diagonalize the
numerical matrix by using software.
Exercise
The solution With the atoms labelled A, B, C, and D, the ham- E9E.1 Set up the secular determinants for (i) linear H3, (ii) cyclic H3 within the
iltonian matrix H is Hückel approximation.

α β 0 0
H AA H AB H AC H AD Hückel α β 0 0 9E.2 Applications
approximation
H BA H BB H BC H BD β α β 0
H= Although the Hückel method is very primitive, it can be used
H CA H CB H CC H CD 0 β α β
to account for some of the properties of conjugated polyenes.
H DA H DB H DC H DD 0 0 β α
1 M

1 0 0 0 0 1 0 0 9E.2(a) π-Electron binding energy


0 1 0 0 1 0 1 0 As seen in Example 9E.1, the energies of the four LCAO-MOs
=a +b = a 1+bM
0 0 1 0 0 1 0 1 for butadiene are
0 0 0 1 0 0 1 0 E    1.62  ,   0.62   (9E.11)
9E Molecular orbital theory: polyatomic molecules 377

4 A closely related quantity is the π-bond formation energy,


α – 1.62β + – + –
Ebf, the energy released when a π bond is formed. Because the
contribution of α is the same in the molecule as in the atoms,
3 –– +
α – 0.62β + the π-bond formation energy can be calculated from the
Energy

C2p π-electron binding energy by writing


2
α + 0.62β + +– –
-Bond formation energy
Ebf  E  N C  (9E.12)
[definition]
1 + + ++
α + 1.62β where NC is the number of carbon atoms in the molecule. The
π-bond formation energy in butadiene, for instance, is 4.48β.
Figure 9E.2 The Hückel molecular orbital energy levels of
butadiene and the top view of the corresponding π orbitals. The Example 9E.2
four p electrons (one supplied by each C) occupy the two lower π Estimating the delocalization energy
orbitals. Note that all the orbitals are delocalized. Use the Hückel approximation to find the energies of the π
orbitals of cyclobutadiene, and estimate the delocalization
These orbitals and their energies are drawn in Fig. 9E.2. Note energy.
that: Collect your thoughts Set up the hamiltonian matrix using the
• The greater the number of internuclear nodes, the higher same basis as for butadiene, but note that atoms A and D are
the energy of the orbital. also now neighbours. Then diagonalize the matrix to find the
energies. For the delocalization energy, subtract from the total
• There are four electrons to accommodate, so the ground-
π-bond energy the energy of two π-bonds.
state configuration is 1π22π2.
• The frontier orbitals of butadiene are the 2π orbital (the The solution The hamiltonian matrix is
HOMO, which is largely bonding) and the 3π orbital (the  M


LUMO, which is largely antibonding).   0  0 1 0 1
   
   0 1 0 1 0
‘Largely bonding’ means that an orbital has both bonding and H   1     1   M
antibonding interactions between various neighbours, but the 0    0 1 0 1
   
bonding effects dominate. ‘Largely antibonding’ indicates that  0   1 0 1 0
the antibonding effects dominate.
An important point emerges by calculating the total Use mathematical software to diagonalize M by forming c −1 Mc :
π-electron binding energy, Eπ, the sum of the energies of each
2 0 0 0
π electron, and comparing it with the energy of a hypothetical  
0 0 0 0
molecule in which the π bonds are localized. For example, in c 1 Mc  
butadiene π-electron binding energy is 0 0 0 0
 
0 0 0 2 
E (butadiene)  2(  1.62  )  2(  0.62  )  4  4.48 
The π-electron binding energy of a localized π bond is simply The diagonalized form of H (which is identified as E) is
that of the π bond in ethene. The two electrons are in the 1π therefore
orbital with energy α + β, so the π-electron binding energy in
   2 0 0 0 
ethene is  
0  0 0
E (ethene)  2(   )  2  2  E= 
 0 0  0 
 
The difference in π-electron binding energy between butadiene  0 0 0   2  
and a molecule with two localized π bonds is
and the energies of the orbitals are   2  ,  ,  ,   2  .
E (butadiene)  2  E (ethene)
Four electrons must be accommodated. Two occupy the
 (4  4.48  )  2(2  2  )  0.48  lowest orbital (of energy α + 2β), and two occupy the doubly
Therefore, the energy of the delocalized butadiene molecule degenerate orbitals (of energy α). The total energy is therefore
lies lower by 0.48β (about 115 kJ mol−1) than of the hypothetical 4α + 4β. Two isolated π bonds would have an energy 4α + 4β;
molecule with localized bonding (recall that β is negative). This therefore, in this case, the delocalization energy is zero.
extra stabilization of a conjugated system compared with a set Self-test 9E.2 Repeat the calculation for benzene (use software!).
of localized π bonds is called the delocalization energy of the Answer: See next subsection
molecule.
378 9 Molecular structure

9E.2(b) Aromatic stability +


– –
The most notable example of delocalization conferring extra α – 2β b2g + +
stability is benzene and the aromatic molecules based on its –
structure. In elementary accounts, the structure of benzene, + – –
and other aromatic compounds, is often expressed in a mixture α –β e2u + +
– + + +
of valence-bond and molecular orbital terms, with typically

Energy

valence-bond language (Topic 9A) used for its σ framework
and molecular orbital language used to describe its π electrons. –
– – + –
First, the valence-bond component. The six C atoms are re- α +β e1g + +
garded as sp2 hybridized, with a single unhybridized perpen- + + –
dicular 2p orbital. One H atom is bonded by (Csp2,H1s) overlap +
+ +
to each C carbon, and the remaining hybrids overlap to give a
α + 2β a2u
regular hexagon of atoms (Fig. 9E.3). The internal angle of a + +
+
regular hexagon is 120°, so sp2 hybridization is ideally suited
for forming σ bonds. The hexagonal shape of benzene permits
Figure 9E.4 The Hückel orbitals of benzene and the
strain-free σ bonding.
corresponding energy levels. The orbital labels are explained
Now consider the molecular orbital component of the de-
in Topic 10B. The bonding and antibonding character of the
scription. The six C2p orbitals overlap to give six π orbitals that delocalized orbitals reflects the numbers of nodes between
spread all round the ring. Their energies are calculated within the atoms. In the ground state, only the bonding orbitals are
the Hückel approximation by diagonalizing the hamiltonian occupied.
matrix
The MO energies are deduced from the diagonal elements of
α β 0 0 0 β this matrix and are
β α β 0 0 0 E    2 ,    ,    ,    ,    ,   2  (9E.13)
0 β α β 0 0
H= as shown in Fig. 9E.4. The orbitals there have been given sym-
0 0 β α β 0 metry labels that are explained in Topic 10B. Note that the low-
0 0 0 β α β est energy orbital is bonding between all neighbouring atoms,
β 0 0 0 β α the highest energy orbital is antibonding between each pair
of neighbours, and the intermediate orbitals are a mixture of
bonding, nonbonding, and antibonding character between ad-
0 1 0 0 0 1 2 0 0 0 0 0 jacent atoms. There are two orbitals with energy    , that is
1 0 1 0 0 0 Diagonalize 0 1 0 0 0 0 there is a degenerate pair of MOs, and likewise a second degen-
0 1 0 1 0 0 0 0 1 0 0 0 erate pair with energy    .
= α 1+ β Now apply the building-up principle to the π system. There
0 0 1 0 1 0 0 0 0 −1 0 0
0 0 0 1 0 1 0 0 0 0 −1 0 are six electrons to accommodate (one from each C atom), so
the three lowest orbitals (a2u and the doubly degenerate pair e1g)
1 0 0 0 1 0 0 0 0 0 0 −2
are fully occupied, giving the ground-state configuration a 22 u e14g.
A significant point is that the only molecular orbitals occupied
are those with net bonding character (the analogy with the
strongly bonded N2 molecule, Topic 9B, should be noted).
The π-electron binding energy of benzene is
E  2(  2  )  4(   )  6  8 
H
C If delocalization is ignored and the molecule is thought of as
having three isolated π bonds, it would be ascribed a π-electron
energy of only 3(2α + 2β) = 6α + 6β. The delocalization energy
is therefore 2β ≈ −480 kJ mol−1, which is considerably more than
for butadiene. The π-bond formation energy in benzene is 8β.
Figure 9E.3 The σ framework of benzene is formed by the This discussion suggests that aromatic stability can be traced
overlap of Csp2 hybrids, which fit without strain into a hexagonal to two main contributions. First, the shape of the regular
arrangement. hexagon is ideal for the formation of strong σ bonds: the σ
9E Molecular orbital theory: polyatomic molecules 379

framework is relaxed and without strain. Second, the π orbitals Regardless of the precise computational method, all elec-
are such as to be able to accommodate all the electrons in bond- tronic structure calculations involve finding the molecular
ing orbitals, and the delocalization energy is large. orbitals iteratively and self-consistently, just as for the self-
consistent field (SCF) approach to atoms (Topic 8B). First, the
molecular orbitals for the electrons present in the molecule are
Brief illustration 9E.2 formulated (for example, as LCAOs). One molecular orbital is
then selected and all the others are used to set up an expression
In Example 9E.2 the delocalization energy of cyclobutadiene is
for the potential energy of the electron in the chosen orbital.
found to be zero. The cyclobutadiene dication, C 4 H2+4 , has two
The form of the chosen orbital is then optimized for this poten-
π electrons to accommodate so the total π-electron binding
tial, for example by adjusting the orbital coefficients or other
energy is 2(α + 2β) = 2α + 4β. The energy of a single localized
parameters. The procedure is repeated for all the molecular
π-bond is 2(α + β), so the delocalization energy is 2β. Benzene,
with six π electrons has a delocalization energy of 8β. Cyclic
orbitals and used to calculate the total energy of the molecule.
conjugated molecules with two or six π electrons are regarded Once one cycle of optimization is completed the orbitals will all
as aromatic and benefit from extra stabilization as a result of have changed and so will the potential that each electron expe-
delocalization. A molecule with four π electrons has no such riences. The process of optimizing each orbital is repeated until
stabilization and is not regarded as aromatic. the computed orbitals and energy are constant to within some
tolerance.

Exercises 9E.3(a) Basis functions and basis sets


E9E.2 Predict the electron configurations of (i) the benzene anion, (ii) the Although it is useful to think of a molecular orbital as a lin-
benzene cation. Estimate the π-electron binding energy in each case. ear combination of atomic orbitals, the theory does not require
E9E.3 What is the delocalization energy and π-bond formation energy of that the MOs are formed from actual AOs. In fact, any conveni-
(i) the benzene anion, (ii) the benzene cation? ent set of basis functions can be used. Numerical calculations
of molecular orbitals require the evaluation of a large number
of integrals (such as those occurring in the definitions of HJJ
9E.3 Computational chemistry and HIJ) in which these basis functions appear. Therefore, it
is important to choose a set of functions that facilitates such
The power and speed of modern computers makes it possible calculations.
to calculate molecular orbitals without resorting to the severe It turns out that calculations using hydrogenic AOs are com-
assumptions of the Hückel method. With readily available putationally time-consuming, so are not convenient basis func-
software packages it is possible to make quite sophisticated tions. Gaussian-type orbitals (GTOs) are a computationally
calculations on small to medium-sized molecules, and with more convenient choice and are used widely. A GTO is centred
the aid of supercomputers it is possible to tackle much larger on each nucleus and has the form
molecules and obtain results which stand up well in compari-   Nx i y j z k e  r
2
Gaussian-type orbitals
son to experiment. These software packages go well beyond
simply calculating molecular orbitals. They also offer the pos- where x, y, and z are the Cartesian coordinates of the electron at
sibility of predicting the equilibrium geometries of molecules, a distance r from the nucleus, ζ is a positive adjustable parame-
as well as molecular properties such as vibrational frequencies ter, and i, j, and k are non-negative integers. An s-type GTO has
and NMR chemical shifts, and even thermodynamic quanti- i = j = k = 0, a p-type GTO has one of the (i, j, k) set to 1 and the
2
ties such as enthalpies of formation. It is also possible to study others are zero (for example Nze  r ), and a d-type GTO has
2 2
chemical reactivity by exploring how the energy and electronic i + j + k = 2 (for example Nyze  r or Nx 2 e  r ). There are three
structures of reacting molecules change as they encounter one p-type and six d-type GTOs.
another. The development and exploitation of methods for the Important considerations when selecting a basis are the form
computation of molecular electronic structure has received an of the GTOs and their number. There needs to be enough func-
enormous amount of attention and has become a keystone of tions to construct the MOs to the required level of precision,
modern chemical research. but larger basis sets lead to more integrals and longer compu-
The underlying theory behind these calculations and their tational times. A great deal of effort has been put into devising
practical implementation as computer programs are both very relatively small basis sets that allow rapid calculations with reli-
complex matters and cannot be explored in detail here. This able results.
section simply aims to provide a brief introduction to the key The notation used to specify a basis set can seem daunting
ideas and language used in such calculations. An introduction at first sight, but becomes clear when interpreted in stages.
to the background theory is given in Topic 9F. Carbon provides an example. Its ground-state electron
380 9 Molecular structure

configuration is 1s22s22p2, so at least five GTOs are needed, are used for all but hydrogen, the basis is denoted 6-31G*, and
one for each type of occupied orbital (one 1s, one 2s, and if polarization functions are also used for hydrogen the basis
three 2p orbitals). Such a basis is called a single zeta (SZ) is denoted 6-31G**. In an alternative notation, parentheses
basis, the name deriving from the single parameter ζ in denote the number and type of polarization functions. For ex-
each of the five GTOs. Such a small basis lacks the flexibility ample, 6-31G(2df,2p) indicates that two d-type and one f-type
needed to describe adequately the contribution the carbon GTO are used for non-hydrogen atoms, and that two p-type
atom makes to MOs, so the number of basis functions may GTOs are used for hydrogen.
be doubled (giving a double zeta, DZ, basis) or tripled (giv- If the molecule is an anion or contains many lone pairs, it
ing a triple zeta, TZ, basis). In such bases, two or three GTOs helps to add GTOs that decay more slowly (have smaller val-
with different values of ζ are used to describe a 1s orbital, and ues of ζ) than those considered so far. These additional func-
similarly for the other orbitals. tions are called diffuse functions, and their use is indicated
The core and valence orbitals are commonly treated sepa- by adding a plus sign and more characters to the name of the
rately in the basis set on the grounds that the core orbitals are basis set. For example, 6-31G+G(d) indicates the use of dif-
likely to be so compact that they contribute little to the bond- fuse functions of the same number and type as used for the
ing, and so do not need to be represented very elaborately. Such valence functions.
an approach is described as a split basis set. One choice is to Here, in summary, is the anatomy of the notation 6-31G and
use a SZ basis for the core electrons and either a DZ or TZ basis its extensions:
for the valence electrons. For carbon, a split basis set with SZ
for the core and TZ for the valence electrons would involve four
s-type GTOs (one for the core and three for the valence shell) Symbol Significance
and a total of nine p-type GTOs (three sets of three), giving a 6 Number of GTOs in the contracted GTO used for the core
orbitals
total of 13 GTOs.
- Separates core from valence shell description
A single GTO is a rather poor approximation to an orbital
wavefunction close to the nucleus. Therefore, the basis set is 3 Number of GTOs in the contracted GTO used for each valence
orbital (inner functions)
formed from linear combinations of GTOs called contracted
1 Number of additional GTOs used for each valence orbital
GTOs. Each term in a contracted GTO has a different value of ζ. (outer functions)
The coefficients of the linear combination and the ζ values are G Specifies a Gaussian basis
optimized for use in a particular basis set and are unchanged
* Polarization functions included for all atoms except H
during the calculation.
** Polarization functions included for all atoms
The next step in nomenclature involves the specification of
the basis. It is here that expressions such as 6-31G are encoun- + Diffuse functions included

tered. In this case the hyphen separates the core and valence G(d) Diffuse functions of the same type as used for the valence
functions included
orbitals. The number 6 in the ‘core group’ indicates that a con-
tracted GTO formed from six GTOs is used to describe the core
orbitals (for example, the single 1s orbital of carbon). The num-
bers after the hyphen specify the GTOs used to describe the va-
lence orbitals. The 3 indicates that each type of valence orbital
9E.3(b) Electron correlation
is represented by a contracted GTO formed from three GTOs The SCF approach treats the effects of electron–electron in-
(the inner functions) and the 1 indicates the use of one further teractions by computing each MO on the basis that the
(non-contracted) GTO (the outer function). In total 22 GTOs electron experiences a potential that is the average of the inter-
are used to describe carbon with this basis set: six for the core, actions with all the other electrons. This approximation ignores
twelve for the inner functions (three sets of four), and four for electron correlation, the details of the interaction between the
the outer functions. electrons. The failure to take into account electron correlation
The 6-31G basis is sufficient for simple organic molecules, commonly gives calculated quantities that are in poor agree-
but to cope with more unusual types of bonding, such as that ment with the experimental data.
found in small rings or so-called ‘bent bonds’, it is necessary Several methods have been developed that attempt to take
to add more GTOs. These additional functions are called into account the effects of electron correlation: they all involve
polarization functions, and correspond to d-type orbitals (for additional calculations, and the most precise results require a
second-row elements) and p-type orbitals (for hydrogen). significant increase in the amount of computation. Two widely
These functions allow for more flexibility in the form of the used procedures are the Møller–Plesset (MP) and coupled
MOs than can be achieved simply with s- and p-type functions. cluster singles and doubles (CCSD) methods. For these highly
The use of polarization functions is indicated by adding aster- technical emendations of the basic calculations you need to
isks to the specification of the basis. If polarization functions consult specialist texts.
9E Molecular orbital theory: polyatomic molecules 381

9E.3(c) Density functional theory 9E.3(e) Graphical representations


A technique that has gained considerable ground in recent One of the most significant developments in computational
years to become one of the most widely used approaches for the chemistry has been the introduction of graphical representa-
calculation of molecular structure is density functional theory tions of molecular orbitals and electron densities. The raw
(DFT). It has proved to be computationally more efficient and output of a molecular structure calculation is a list of the co-
often to give better results than the methods described so far. efficients of the atomic orbitals in each molecular orbital and
The central focus of DFT is the electron density, ρ, rather than the energies of these orbitals. The graphical representation of a
the wavefunction, ψ. The ‘functional’ part of the name comes molecular orbital uses stylized shapes to represent the basis set,
from the fact that the energy of the molecule is a function of the and then scales their size to indicate the coefficient in the linear
electron density, written E[ρ], and the electron density is itself a combination. Different signs of the wavefunctions are repre-
function of position, ρ(r): in mathematics a function of a function sented by different colours.
is called a ‘functional’. Topic 9F outlines how this procedure is Once the coefficients are known, it is possible to construct a
used to calculate molecular orbitals. The method involves choos- representation of the electron density in the molecule by noting
ing a basis set and solving a set of equations iteratively and self- which orbitals are occupied and then forming the squares of
consistently. An important feature of DFT is that, in principle, the those orbitals. The total electron density at any point is then the
effects of electron correlation are included in the calculation. sum of the squares of the wavefunctions evaluated at that point.
An enormous amount of effort has gone into developing The outcome is commonly represented by an isodensity surface,
functionals that are computationally efficient and produce re- a surface of constant total electron density (Fig. 9E.5). As
sults that are in accord with experiment. Some functionals shown in the illustration, there are several styles of represent-
have been developed to give the best results for certain kinds of ing an isodensity surface, as a solid form, as a transparent form
molecule or certain molecular properties. Artificial intelligence with a ball-and-stick representation of the molecule within,
(AI) has also been used to identify reliable functionals. It is not or as a mesh. A related representation is a solvent-accessible
uncommon for the functionals to be adjusted empirically to fit surface in which the shape represents the shape of the molecule
experimental data. by imagining a sphere representing a solvent molecule rolling
A very commonly used functional is B3LYP, developed by across the surface and plotting the locations of the centre of
Lee, Yang, and Parr (hence LYP). The ‘B’ indicates the use of that sphere.
an ‘exchange’ term in the functional (Topic 9F) and the ‘3’ in- One of the most important aspects of a molecule other than
dicates the presence of three parameters that can be adjusted its geometrical shape is the distribution of charge over its sur-
to give the best fit to experimental data. There are variants on face, which is commonly depicted as an electrostatic potential
this functional; for example, the B3LYP-D functional has addi- surface (an ‘elpot surface’). The potential energy, Ep, of an im-
tional terms included designed to deal with long-range (disper- aginary positive charge Q at a point is calculated by taking into
sion) interactions. account its interaction with the nuclei and the electron density
throughout the molecule. Then, because Ep  Q , where ϕ is
the electric potential, the potential energy can be interpreted
9E.3(d) Practical calculations as a potential and depicted as an appropriate colour (Fig. 9E.6).
How a calculation is approached depends on the computing Electron-rich regions usually have negative potentials and
power available, the type of molecule being considered, the electron-poor regions usually have positive potentials.
properties to be calculated, and the precision required. A typi- Representations such as those illustrated here are of criti-
cal starting point is to use DFT with 6-31G* basis set and one cal importance in a number of fields. For instance, they may
of the widely used functionals such as B3LYP. This approach be used to identify an electron-poor region of a molecule that
might be sufficient to obtain the general form of the MOs or the
equilibrium geometry.
If more precise results are needed it is appropriate to refer
to the literature to find an approach that has been shown to be
effective for the intended computation. The calculation of vi-
brational frequencies, for instance, needs a different approach
from the calculation of NMR chemical shifts.
The most precise calculations often involve using different
methods in sequence. For example, the geometry of a molecule (a) (b) (c)
might first be optimized by using DFT, and then the ener- Figure 9E.5 Various representations of an isodensity surface of
gies of the orbitals are refined by taking into account electron ethanol: (a) solid surface, (b) transparent surface, and (c) mesh
correlation. surface.
382 9 Molecular structure

Electron is susceptible to association with or chemical attack by an


rich electron-rich region of another molecule. Such considerations

are important for assessing the pharmacological activity of
potential drugs.

+
Electron Exercises
poor
E9E.4 Write down the form of the three p-type and six d-type GTOs.
Figure 9E.6 An elpot diagram of ethanol; the molecule has the E9E.5 How many GTOs are there in a single zeta basis set for (i) a carbon
same orientation as in Fig. 9E.5. Red denotes regions of negative atom, (ii) a nitrogen atom?
electrostatic potential and blue regions of positive potential (as in E9E.6 Which GTOs are included in the 3-21G basis set?
δ−
O−Hδ+).

Checklist of concepts
☐ 1. The Hückel method neglects overlap and interactions ☐ 6. The delocalization energy is the difference between the
between orbitals on atoms that are not neighbours. π-electron binding energy and the energy of the same
☐ 2. The highest occupied molecular orbital (HOMO) and molecule with localized π bonds.
the lowest unoccupied molecular orbital (LUMO) are ☐ 7. The stability of benzene arises from the geometry of the
the frontier orbitals of a molecule. ring and the high delocalization energy.
☐ 3. The Hückel method may be expressed in a compact ☐ 8. Density functional theories develop equations based
manner by introducing matrices. on the electron density rather than the wavefunction
☐ 4. The π-bond formation energy is the energy released itself.
when a π bond is formed. ☐ 9. Graphical techniques are used to plot a variety of sur-
☐ 5. The π-electron binding energy is the sum of the ener- faces based on electronic structure calculations.
gies of each π electron.

Checklist of equations
Property Equation Comment Equation number
LCAO-MO    ci i ψi are atomic orbitals 9E.1
i

Hückel equations Hc = cE Hückel approximations: HAB = 0 except between 9E.8


neighbours; overlap neglected
Diagonalization c−1Hc = E 9E.10
π-Electron binding energy Eπ = sum of energies of π electrons Definition
π-Bond formation energy Ebf = Eπ − NCα Definition; NC is the number of carbon atoms 9E.12
π-Delocalization energy Edeloc = Eπ − NC(α + β)
TOPIC 9F Computational chemistry

illustrated by using from the outset the simplest possible many-


electron molecule, dihydrogen (H2), with two electrons labelled
➤ Why do you need to know this material? 1 and 2 and two nuclei labelled A and B. Some of the techniques
Modern computational procedures for predicting molecu- introduced do not need to be applied to this simple molecule,
lar structure and reactivity are now widely available and but they serve to illustrate them in a simple manner. Molecules
used throughout chemistry. Like any tool, it is important to with more electrons are then relatively straightforward elabora-
know its foundations. tions of the equations for H2: these more general equations are
given in A deeper look 9F.1, available to read in the e-book ac-
➤ What is the key idea? companying this text.
Numerical procedures in computational chemistry typical- Topic 9B introduces the electronic Hamiltonian for the one-
ly proceed by solving equations until they do not change electron system H2+. The corresponding hamiltonian for H2 has
on successive iterations. additional terms to describe the kinetic energy of the second
electron, its interaction with both nuclei, and the interaction
➤ What do you need to know already? between the two electrons:
This Topic develops the approach introduced in Topic 9E
kinetic energy
and makes extensive use of matrix manipulations (see the of the electrons

extended version of The chemist’s toolkit 9E.1 available on
2
the website). It is based on the variation principle (Topic 9A). Hˆ = −
2me
(
∇12 + ∇22 )
potential energy of
attraction to the nuclei

e2 e2 e2 e2
The field of computational chemistry, the use of computers − − − − 
to predict molecular structure and reactivity, has grown in the 4 πε 0 r1A 4 πε 0 r2A 4 πε 0 r1B 4 πε 0 r2B
past few decades due to the tremendous advances in computer electron − electron
repulsion
hardware and to the development and wide availability of effi-  
cient software packages. The latter are now applied routinely to e2
+ (9F.1)
compute molecular properties in a wide variety of chemical ap- 4 πε 0 r12
plications including drug search and design, atmospheric and
environmental chemistry, nanotechnology, and materials sci- In this expression riJ is the distance from electron i to nucleus J,
ence. As mentioned in Topic 9E, most software packages have and r12 is the distance between the electrons. As in Topic 9B the
sophisticated graphical interfaces that permit the visualization term e 2 /4  0 is written j0,1 and with this substitution the hamil-
of results. tonian takes the form

Hˆ = Hˆ 1 + Hˆ 2 + Hˆ 12  (9F.2)

9F.1 The central challenge with


2 2  1 1 
The central aim of electronic structure calculations in compu- Hˆ 1 =− ∇1 − j0  + 
2me  r1A r1B 
tational chemistry is the solution of the electronic Schrödinger
equation, ĤΨ = EΨ , where E is the electronic energy and Ψ is  2
 1 1 
Hˆ 2 =− ∇22 − j0  + 
the many-electron wavefunction, a function of the coordinates 2me  r2A r2B 
of all the electrons and the nuclei. To make progress, it is neces- j
sary to invoke the Born–Oppenheimer approximation and the Hˆ 12 = 0
r12
separation of electronic and nuclear motion (see the Prologue
to this Focus).
To avoid a sequence of overly complicated equations, the 1
In some accounts j0 is replaced by 1, signalling that atomic units are being
computational procedures described in this Topic will be used.
384 9 Molecular structure

It is hopeless to expect to find analytical eigenfunctions 


1  a (1)  a (1)


(wavefunctions) and eigenvalues (energies) for a hamiltonian   Slater determinant  (9F.3a)


21/ 2  a (2)  a (2)
of this complexity. The whole thrust of computational chem-
istry is to formulate and implement numerical procedures that The factor 1/21/2 (and in general 1/Ne!1/2, where Ne is the number
give ever more reliable approximations to them. of electrons) ensures that the wavefunction is normalized if the
component spinorbitals are normalized. To save the tedium of
writing out large determinants when there are many electrons in a
Exercises molecule, the wavefunction is normally written showing only the
E9F.1 Write down the form of the hamiltonian for the three-electron spinorbitals along the principal diagonal of the determinant:
system H2−.
+
E9F.2 Write the electronic hamiltonian for HeH .   (1/21/2 )  a (1) a (2)  (9F.3b)

According to the variation principle (Topic 9A) the form of


Ψ which is the best approximation to the actual wavefunction
is the one that corresponds to the lowest achievable energy as
9F.2 The Hartree−Fock formalism the ψ are varied. That is, the wavefunctions ψ must be found
that minimize the expectation value, E = ∫Ψ * HˆΨ dτ , of the
The orbital approximation is introduced in Topic 8B in the
­hamiltonian.
context of many-electron atoms and is used here too. In this
D.R. Hartree and V. Fock showed that the optimum orbital
approximation it is supposed that each electron is described
wavefunctions each satisfy what is, at first sight, a very simple
by its ‘own’ wavefunction, called an orbital, and that the over-
set of equations:
all wavefunction can be written as a product of these orbitals.
For example, if electron 1 occupies an orbital with wavefunc- f1 a (1)   a a (1) Hartree Fock equations (9F.4)
tion ψa and electron 2 occupies an orbital orbital with wave-
function ψb, the overall wavefunction is written ψa(r1)ψb(r2). where f1 is the Fock operator for electron 1. This is the equa-
This approximation is severe and loses many of the details of tion to solve to find ψa; there are analogous equations for any
the dependence of the wavefunction on the relative locations of other orbitals that are occupied. The explicit effect of the Fock
the electrons. To simplify the appearance of the expressions the operator on ψa(1) is as follows
wavefunction is written ψa(1)ψb(2).
kinetic energy electron
Now suppose that electron 1 is in orbital a with spin α, and and attraction repulsion exchange
electron 2 occupies the same orbital with spin β. The many- to the nuclei of electrons correction
   
electron wavefunction is written as the product Ψ = ψ aα(1)ψ aβ(2). f1ψ a (1) = Hˆ 1ψ a (1) + 2 Jˆa (1)ψ a (1) − Kˆ a (1)ψ a (1) (9F.5a)
The combination of an orbital and a spin function, such as
ψ aα(1), is called a spinorbital. For example, the spinorbital ψ aα
where Ĥ1 is specified in eqn 9F.2, Jˆa is called the coulomb oper-
is interpreted as the product of the spatial wavefunction ψa and
ator, and Kˆ a is called the exchange operator. In general (when
the spin wavefunction α:
more than one orbital is occupied)
spatial spin
spinorbital 1
 wavefunction Jˆb (1)ψ a (1) = j0 ∫ψ a (1) ψ b* (2)ψ b (2) dτ 2
wavefunction
 
 a (1)   a (1)   (1) r12
Coulomb operator [definition]  (9F.5b)
and likewise for the other spinorbitals.
1
Kˆ b (1)ψ a (1) = j0 ∫ψ b (1) ψ b* (2)ψ a (2) dτ 2
Exchange operator
A simple product wavefunction such as ψ aα(1)ψ aβ(2) does [definition]
r12
not satisfy the Pauli principle, which requires the function
to change sign under the interchange of any pair of electrons For H2 both electrons are in ψa and so there is no distinction
(Topic 8B). To ensure that the wavefunction does satisfy the between the two operators and
principle, it is expressed as a sum of all possible permutations
1
of the occupation of spinorbitals by the electrons, using alter- Jˆa (1)ψ a (1) Kˆ=
= a (1)ψ a (1) j0 ∫ψ a (1) ψ a* (2)ψ a (2)dτ 2  (9F.5c)
nating plus and minus signs: r12

   a 1 a  2   a 1 a  2  Equation 9F.5a now becomes

This sum can be represented by a determinant called a Slater 2 2  1 1  1


 1 a (1)  j0    a (1)  j0  a (1)  a* (2) a (2)d2
determinant, the expansion of which (see The Chemists Toolkit 2me  r1A r1B  r12
9D.1) generates the same alternating sum of terms:     (1) (9F.5d)
a a
9F Computational chemistry 385

Before exploring how this very complicated equation can be


solved, note that it reveals a second principal approximation
9F.3 The Roothaan equations
of the Hartree–Fock formalism (the first being its dependence The difficulty with the HF-SCF procedure lies in the numerical so-
on the orbital approximation). Instead of electron 1 (or any lution of the Hartree–Fock equations, a challenging task even for
other electron) responding to the instantaneous positions of powerful computers. As a result, a modification of the technique
the other electrons in the molecule through terms of the form was needed before the procedure could be of use to chemists. In
1/r12, it responds to an averaged location of the other electrons 1951 C.C.J. Roothaan and G.G. Hall independently found a way
through integrals of the kind that appear in eqn 9F.5d: to convert the Hartree–Fock equations for the molecular orbitals
1 * into equations for the coefficients that appear in the LCAO used
j0  a (1)  a (2) a (2)d 2 Interpreted
 as

r12 to construct the molecular orbital. Their approach (see A deeper
average value look 9F.2, available to read in the e-book accompanying this text)
/r12
of
1  leads to the Roothaan equations.
probability of
electron 2 being The atomic orbitals used to construct the LCAO are referred
in d 2 at r2
 to as the basis set for the calculation, and the choice of basis is
 a* (2) a (2) an important part of the procedure (see below). Note that if the
j0 a (1) d 2 basis set consists of No atomic orbitals, then No linearly inde-
r
12  pendent linear combinations (molecular orbitals) can be con-
average repulsion energy
for an electron at r1 structed from them.
Once again, seeing how this procedure works for H2 gives a
That is, the Hartree–Fock method ignores electron correlation, hint of the general form of the Roothaan equations. As this ac-
the tendency of electrons to avoid one another to minimize re- count unfolds you will see many similarities between it and the
pulsions. This failure to take into account electron correlation is matrix formulation of Hückel theory in Topic 9E, with equa-
a principal reason for inaccuracies in the calculations and leads tions having a similar structure but interpreted in a more so-
to calculated energies that are higher than the ‘true’ (that is, ex- phisticated way.
perimental) values. First a molecular orbital is constructed by the linear combi-
Although eqn 9F.5d is the equation to solve to find ψa, it is nation of two basis orbitals χA and χB (think of these as hydro-
necessary to know the wavefunctions of all the occupied orbit- gen 1s orbitals, but they could be other types). This molecular
als in order to set up the integrals J and K and hence to find orbital is written ψm = cAm χA + cBm χB, with m = a for what will
ψa. Even when only one orbital is occupied, its form must be become the occupied orbital and b for the unoccupied orbital
known in order to evaluate the integral that is then used to find (a ‘virtual’ molecular orbital in the jargon). Now write the coef-
that orbital. To make progress with this difficulty the calcula- ficients cAm and so on as the matrix c:
tion proceeds in the following way.
1. The initial form of ψa is guessed.  cAa cAb 
c  
2. The assumed form of ψa is used to set up the integral in  c Ba c Bb 
eqn 9F.5d. The overlap matrix S of the individually normalized atomic or-
3. The Hartree–Fock equations are solved to find a better bitals is written
version of ψa.
1 S
4. Steps 2 to 3 are repeated using this better wavefunction. S  S    A  B d
S 1
This process is continued until successive cycles of the calcula-
Next, set up the Fock matrix using the Fock operator f that ap-
tion leaves the energies εa and orbital wavefunction ψa unchanged
pears in eqn 9F.4:
to within a chosen criterion. This is the origin of the term self-
consistent field (SCF) for this type of procedure in general and  FAA FAB 
of Hartree–Fock self-consistent field (HF-SCF) for the approach F   FXY    X (1) f1  Y (1)d 1
 FBA FBB 
based on the orbital approximation. When more than one orbital
is occupied, this procedure is repeated for all of them. For now, regard the FXY as variable quantities; keep in mind
that they are simply energies arising from the kinetic energy of
Exercise the electrons, their attractions to the nuclei, and repulsions be-
tween the electrons. Finally, define the matrix ε, which is just a
E9F.3 What arrangement of electrons does the following Slater determinant
represent?
diagonal matrix of the energies εa and εb:
   0
1  a (1)  a (1)   a 
  1/ 2  
2  b (2)  b (2) 0 b 
386 9 Molecular structure

The Roothaan equations are written in terms of these matrices which expands into εa2 – 2αεa + α2 – β2 = 0 and the roots are
εa = α ± β. If β < 0 (as in the Hückel treatment), the lower
 F   c   S  c     
 energy orbital is the one with εa = α + β.
 FAA FAB   cAa cAb   1 S   cAa cAb    a 0 
     
 FBA FBB   cBa cBb   S 1   cBa cBb   0  b  Despite this apparent simplicity, there is a hidden problem
Roothaan equations for H2 (9F.6) that becomes apparent as soon as the calculation becomes more
sophisticated than the Hückel procedure. In general (and even
As the annotation indicates, the general form of the Roothaan for H2) the FXY depend on the coefficients that we are trying to
equations is Fc = Scε. Note their similarity to the Hückel treat- find! For instance, the explicit form of FAA for H2 is
ment equations developed as eqn 9E.7 in Topic 9E:

 H AA H AB   c1,A c2,A   c1,A c2,A   E1 0 = 1


FAA ∫χ (1)Hˆ 1 χ A (1)dτ 1 + 2 j0 ∫ χ A (1)χ A (1) ψ a (2)ψ a (2)dτ 1dτ 2
     A
r12
 H BA H BB   c1,B c2,B   c1,B c2,B   0 E2 
1
 − j0 ∫ χ A (1)ψ a (1) ψ a (2)χ A (2)dτ 1dτ 2 (9F.7)
After multiplying out the matrices in eqn 9F.6 and matching the r12
elements corresponding to the occupied orbital a, they become
FAA cAa  FAB cBa   a cAa   a ScBa But when ψa = cAa χA + cBa χB is substituted into the second and
third integrals the result is an expression that involves the coef-
FBA cAa  FBB cBa   a cBa   a ScAa
ficients and having the form
Then, by rearranging and collecting terms,
FAA  A  BcA2 a  CcB2 a  DcAa cBa
(FAA   a )cAa  (FAB   a S)cBa  0
(FBA   a S)cAa  (FBB   a )cBa  0 Therefore, solving the equations for the coefficients and
the energies is far from straightforward. Once again, a self-
which are nothing other than a pair of simultaneous equations consistency procedure has to be adopted. First, the coefficients
for the coefficients, as in Hückel theory, where they were are estimated and used to evaluate the FXY. The resulting equa-
tions are solved for the coefficients and energies and the new
(H AA  E1 )c1,A  H AB c1,B  0
coefficients are used to calculate another round of the FXY. The
H BA c1,A  (H BB  E1 )c1,B  0 procedure is repeated until the coefficients and energies no
A general feature of simultaneous equations (The Chemist's longer change significantly in each new cycle.
Toolkit 9D.1) is that their solution requires the determinant of
the factors multiplying the two unknowns, in this case cAa and
cBa, to vanish. That is, it is necessary that 9F.4Evaluation and approximation
FAA   a FAB   a S
0
of the integrals
FBA   a S FBB   a
It should be clear that the computation of energies and coef-
As in Topic 9E, this determinant is called a secular determinant ficients depends upon knowing the values of numerous inte-
(in general the criterion is |F − εS| = 0). When expanded, the grals, all of which must be evaluated numerically. How that is
resulting expression is a quadratic equation in the energy εa, done is central to the variety of software packages currently
which can easily be solved. available. The integrals that are appearing are becoming seri-
ously cumbersome to write and so from now the following no-
Brief illustration 9F.1
tation is used:

As has been emphasized, these equations are very similar to 1


(AB | CD)  j0   A (1) B (1)  C (2) D (2)d 1d 2
those arising in the Hückel method. One of the Hückel approx- r12
Integral notation (9F.8)
imations is to set all overlap integrals equal to zero (S = 0).
Next, noting that the FXY are simply a variety of energies, the
diagonal Fock matrix elements are set equal to α (FAA = FBB = α), Integrals like this are fixed throughout the calculation (for a
and the off-diagonal elements are set equal to β (FAB = FBA = β). particular nuclear geometry) because they depend only on the
The secular determinant then becomes choice of basis, so they can be tabulated once and for all and
then used whenever required. In many cases the integrals have
  a 
0 a clear physical significance, which can be used as a guide to
   a their approximation if that is necessary.
9F Computational chemistry 387

Brief illustration 9F.2 location of the electron relative to the atomic nucleus. Several
such basis functions are typically centred on each atom, with
When the molecular orbitals ψa = cAa χA + cBa χB in eqn 9F.7 are each basis function characterized by a unique set of values of a,
expanded in terms of the basis orbitals, terms like the following b, l, and ml.
are obtained: The introduction of Gaussian-type orbitals (GTO), as men-
1 tioned in Topic 9E, largely overcame the problem of having to
j0   A (1) A (1)  a (2) a (2)d 1d 2
r12 calculate huge numbers of integrals. Cartesian Gaussian func-
 cA2 a (AA|AA)  2cAacBa (AA|BA)  cBa
2
(AA|BB) tions centred on atomic nuclei have the form

The last of the three integrals on the right is 2


  Nx i y j z k e  r Gaussian-type orbitals (9F.10b)
1
( AA|BB)  j0   A2 (1)  B2 (2)d 1d 2
r12 where (x, y, z) are the Cartesian coordinates of the electron at a
distance r from the nucleus, (i, j, k) is a set of non-negative inte-
and represents the Coulombic interaction of one electron in an gers, and ζ is a positive constant. An s-type Gaussian has i = j =
orbital on A with another electron in an orbital on B. k = 0; a p-type Gaussian has i + j + k = 1; a d-type Gaussian has
i + j + k = 2, and so on.
The advantage of GTOs is that the product of two Gaussian
One approach to the evaluation of integrals is brute force: functions on different centres is equivalent to a single Gaussian
more formally, an a priori method. Each integral is evaluated function located at a point between the two centres. Therefore,
numerically after making a particular choice of basis. In some two-electron integrals on three and four different atomic cen-
cases it is possible to evaluate an integral analytically. For in- tres can be reduced to integrals over two different centres,
stance, if the basis consists of two H1s orbitals at a distance R which are much easier to evaluate numerically.
apart, then the integral in Brief illustration 9F.2 is An alternative approach is to estimate the integrals, perhaps
by varying them until certain computed molecular properties
j0 j  2a 11 3R R 2  2 R /a0 (such as enthalpies of formation) match experimentally de-
(AA|BB)   0  0   e  (9F.9) termined values. An extreme version of these semi-­empirical
R 2a0  R 4 2a0 2a02 
procedures is the Hückel procedure where many integrals are
where a0 is the Bohr radius. All the integrals that occur for H2 simply ignored. More sophisticated procedures discard in-
with the H1s basis set can be evaluated similarly. However, as tegrals based on various criteria. For instance, in the proce-
soon as the molecules being considered become more compli- dure called complete neglect of differential overlap (CNDO),
cated, this approach fails. all (AB|CD) integrals are set to zero unless χA and χB are the
In all but the very simplest molecules some of the integrals same, and likewise for χC and χD. That is, only integrals of
that arise have atomic orbitals centred on four different atomic the form
nuclei, and (AB|CD) is in general a so-called ‘four-centre, two-
electron integral’. The number of such integrals increases as 1 2
(AA|CC)  j0   A2 (1)  C (2)d 1d 2
the fourth power of the number of basis functions, N o4, so for a r12
basis consisting of several dozen functions there will be tens of
thousands of integrals of this form to evaluate. survive and are then set equal to values that result in agreement
For example, for CH4 the simplest basis set would consist with experiment on certain classes of molecules.
of four H1s orbitals, one C2s orbital, and three C2p orbitals, The origin of the term ‘differential overlap’ is that what is nor-
giving a total of eight functions. The number of integrals to mally taken to be a measure of ‘overlap’ is the integral   A  B d .
evaluate would be 84 = 4096. The efficient calculation of such The differential of an integral of a function is the function itself,
integrals poses the greatest challenge in an HF-SCF calcula- so in this sense the ‘differential’ overlap is the product χAχB. The
tion but can be alleviated by a clever choice of basis functions. implication of the CNDO approach is to note which orbitals, χA
One of the earliest choices for basis set functions was that of and χB, occur in the integral. If they are the same, the integral is
Slater-type orbitals (STO) centred on each of the atomic nuclei evaluated; if they are different, the integral is ignored. More re-
in the molecule and of the form cent semi-empirical methods make less severe decisions about
  Nr a e br Yl ,ml ( ,  ) Slater -type orbitals (9F.10a) which integrals are to be ignored, but they are all descendants
of the early CNDO technique. For instance, in one modifica-
where N is a normalization constant, a and b are (non- tion, (AB|AB) is also allowed to survive if A and B are on the
negative) parameters, Yl ,ml is a spherical harmonic (Table 8A.1), same atom (for instance, if A is a C2s orbital and B is a C2p
and (r,θ,ϕ) are the spherical polar coordinates describing the orbital in methane).
388 9 Molecular structure

Brief illustration 9F.3 that the exact ground-state energy of a molecule is uniquely de-
termined by the electron probability density. They showed that
The integral in eqn 9F.9 is expressed in terms of the three it is possible to write
integrals (AA|AA), (AA|BA), and (AA|BB). Only the first and
E[  ]  EClassical [  ]  EXC [  ] Energy functional (9F.11)
third integrals survive under CNDO, so in this approximation
1 where EClassical[ρ] is the sum of the contributions of kinetic
j0   A (1) A (1)  a (2) a (2)d 1d 2  cA2 a (AA|AA)
r12 energy, electron–nucleus interactions, and the classical
2
 cBa (AA|BB) electron–electron potential energy, and EXC[ρ] is the exchange–
correlation energy. This term takes into account all the non-
The integral (AA|AA) is the energy of interaction between the classical electron–electron effects due to spin and applies small
two electrons both localized on A, and the integral (AA|BB) is
corrections to the kinetic energy part of EClassical that arise from
the energy of interaction when one electron is on nucleus A
electron–electron interactions. The crucial point is that these
and the other is on B.
quantum mechanical effects can be expressed in terms of the
classically significant electron density, not the quantum me-
chanical wavefunction itself. The Hohenberg–Kohn theorem
guarantees the existence of EXC[ρ] but—like so many existence
Exercises theorems in mathematics—gives no clue about how it should
be calculated.
E9F.4 Write out in full the integral represented by (AA|AA). What interaction
does this integral represent? The first step in the implementation of this approach is to
calculate the electron density. The relevant equations were de-
E9F.5 Which values of a, l, and ml in a Slater-type orbital would you choose to
best represent (i) a 1s atomic orbital, (ii) a 2p atomic orbital, (iii) a 2s atomic duced by W. Kohn and L.J. Sham in 1965, who showed that ρ
orbital. Are there any features of the atomic orbital which the Slater-type can be expressed as a contribution from each electron present
orbital does not represent? in the molecule: for H2 it is simply  (r )  2 a2 (r ).The func-
tion ψi is called a Kohn–Sham orbital and is a solution of the
Kohn–Sham equation, which closely resembles the form of the
Schrödinger equation (on which it is based). For a two-electron
9F.5 Density functional theory system the Kohn–Sham equation is
ρ (2)
As remarked in Topic 9E, a technique that has gained consider- Hˆ 1ψ a (1) + j0 ∫ dτ 2 ψ a (1) + VXC (1)ψ a (1) =
ε aψ a (1)
able ground in recent years to become one of the most widely r12
used procedures for the calculation of molecular structure is  Kohn-Sham equation (two electrons) (9F.12)
density functional theory (DFT). As mentioned there, the
where, as usual, 1 stands for the location r1 of electron 1 and
central focus of DFT is the electron probability density, ρ. The
2 for the location r2 of electron 2. The εa is the Kohn–Sham
‘functional’ part of the name comes from the fact that the en-
orbital energy. The first term on the left is familiar from the
ergy of the molecule is a function of the electron density ρ(r)
earlier discussion and accounts for the kinetic energy of elec-
and the electron density is itself a function of the location in
tron 1 and its interactions with the nuclei. The second term
the molecule. In mathematics a function of a function is called
is the classical interaction between electron 1 and electron 2.
a functional, and in this specific case the energy is written as
The third term, the one involving the exchange–correlation
the functional E[ρ]. Functionals have been used elsewhere in
potential, VXC, takes exchange effects into account. To under-
this text but without using the terminology. For example the
stand its significance, note that if the electron density is modi-
expectation value of the hamiltonian is the energy expressed as
fied everywhere (or just somewhere), the exchange correlation
a functional of the wavefunction because a single value of the
energy will also change by a certain amount. Specifically, VXC is
energy, E[ψ], is associated with each function ψ.
simply a function that enables the change in the exchange cor-
As discussed earlier in this Topic, the energy of a molecule
relation energy to be related to any change in the electron den-
is a sum of the kinetic energy, the electron–nuclear interac-
sity by evaluating the following integral:
tions, and the electron–electron interaction. The first two con-
tributions depend on the electron density distribution. The change in EXC [  ] a change in
arising from  (r ) everywhere Exchange-correlation
electron–electron interaction is likely to depend on the same 
   
definition]  (9F.13)
potential [d
quantity, but it is modified by a quantum mechanical exchange EXC [  ]   VXC (r )  (r ) d
term (the contribution which in Hartree–Fock theory is ex-
pressed by the operator K̂ ). That the exchange contribution can The greatest challenge in density functional theory is to find an
be expressed in terms of the electron density is not at all obvi- accurate expression for the exchange–correlation energy and
ous, but in 1964 P. Hohenberg and W. Kohn were able to prove through that identify the exchange–correlation potential that
9F Computational chemistry 389

relates it to changes in the electron density distribution in the self-­consistently. First, the electron density is estimated; it is
molecule. common to use a superposition of atomic electron probability
One widely used but approximate form for EXC[ρ] is based on densities. Second, the exchange–correlation potential is calcu-
the model of a uniform electron gas, a hypothetical electrically lated by assuming an approximate form, such as that for the
neutral system in which electrons move in a space of continu- free electron gas. Next, the Kohn–Sham equations are solved
ous and uniform distribution of positive charge. For a uniform to obtain an initial set of Kohn–Sham orbitals. This set of or-
electron gas bitals is used to obtain a better approximation to the electron
1/ 2 probability density and the process is repeated until the density
3 remains constant to within some specified tolerance.
EXC [  ]  A   (r )4 / 3 d A   98 j0  
 To apply such a process to H2, begin by assuming that the
Uniform electron gaas  (9F.14a) electron density is a sum of atomic electron densities arising
from the presence of electrons in the atomic orbitals χA and χB
It is now possible to identify the exchange-correlation energy. (which may be STOs or GTOs) and write ρ(i) = |χA(i)|2+ |χB(i)|2
for electron i = 1 and 2. With the exchange–correlation poten-
tial in eqn 9F.14b, the Kohn–Sham orbital for the molecule is a
How is that done? 9F.1 Identifying an exchange-
solution of
correlation energy
ρ (2)
To identify the exchange-correlation energy in eqn 9F.13 you Hˆ 1ψ a (1) + j0 ∫ dτ 2 ψ a (1) + 34 ρ (1)1/3ψ a (1) =
ε aψ a (1)
r12
need to know the definition of a ‘functional derivative’ of a
functional F[ f ] of a function f, δF[ f ]: Now insert the ρ(1) and ρ(2), and solve this equation numeri-
cally for ψa. Once that orbital has been obtained, replace the
F[ f ]
F[ f ]   f d original guess at the electron density by ρ(i) = |ψa(i)|2, i = 1 and 2.
f This density is then substituted back into the Kohn–Sham
For the functional EXC[ ρ ], substitute the expression in eqn equation to obtain an improved function ψa and exchange–
9F.14a: correlation energy, and hence the exchange–correlation
potential. The process is repeated until the density and
EXC[  ] A 4 / 3 exchange–correlation energy are unchanged to within a speci-
E XC[  ]    d    d
  fied tolerance on successive iterations.
VXC
 When convergence of the iterations has been achieved, the
  3 A 1/ 3  d
4 electronic energy is calculated from
ρ (1)ρ (2)
You can conclude that = E[ ρ ] 2 ∫ψ a (1)Hˆ 1ψ a (1)dτ 1 + j0 ∫ dτ 1 dτ 2
r12
(9F.15b) 4/3
4
VXC (r) = —
3 Ar (r)
1/3 − A ∫ ρ (1) dτ 1  (9F.16)
Uniform electron gas
where the first term is the sum of the energies of the two elec-
There is now enough equipment assembled to under- trons in the field of the two nuclei, the second term is the
stand the strategy of a DFT calculation, bearing in mind that electron–electron repulsion, and the final term includes the
the Kohn–Sham equations must be solved iteratively and correction due to nonclassical electron–electron effects.

Checklist of concepts
☐ 1. A spinorbital is the combination of a spatial orbital and ☐ 4. The Hartree–Fock method ignores electron correlation,
a spin function. the tendency of electrons to avoid one another to mini-
☐ 2. A Slater determinant ensures that a many-electron mize repulsions.
wavefunction satisfies the Pauli principle. ☐ 5. The Roothaan equations are used to express the
☐ 3. The Hartree–Fock self-consistent field (HF-SCF) pro- Hartree–Fock procedure in terms of linear combina-
cedure is a systematic solution of the Schrödinger equa- tions of a basis set of atomic orbitals.
tion for a many-electron system. ☐ 6. In the a priori method, each molecular integral is evalu-
ated numerically.
390 9 Molecular structure

☐ 7. In a semi-empirical procedure many integrals are ☐ 9. The Hohenberg–Kohn theorem guarantees that the
either ignored or estimated on the basis of experimental energy of a molecule is determined solely by the elec-
data. tron density.
☐ 8. In density functional theory (DFT), the focus is on the ☐ 10. A Kohn–Sham orbital is a solution of the Kohn–Sham
electron density itself rather than the wavefunction. equation.

Checklist of equations
Property Equation Comment Equation number

Spinorbital  (i)   a (i) (i)
a Electron i in spatial orbital ψ a with spin α
 
Slater determinant 1  (1)  (1)
a a Satisfies Pauli principle 9F.3a
 
21/ 2  (2)  (2)

a

a

Hartree–Fock equations f1 a (1)   a a (1) 9F.4

1
Integral notation (AB|CD)  j0   A (1) B (1)  C (2) D (2)d 1d 2 9F.8
r 12

Slater-type orbital   Nr a e brYl ,ml (,  ) 9F.10a


j k  r 2
Gaussian-type orbital   Nx y z e
i
9F.10b
Exercises and problems 391

FOCUS 9 Molecular structure

To test your understanding of this material, work through Selected solutions can be found at the end of this Focus in
the Exercises, Additional exercises, Discussion questions, and the e-book. Solutions to even-numbered questions are available
Problems found throughout this Focus. online only to lecturers.

TOPIC 9A Valence-bond theory


Discussion questions
D9A.1 Discuss the role of the Born–Oppenheimer approximation in the hybridization explain that in allene, CH2=C=CH2, the two CH2 groups lie in
valence-bond calculation of a molecular potential energy curve or surface. perpendicular planes?
D9A.2 Why are promotion and hybridization invoked in valence-bond theory? D9A.4 Why is spin-pairing so common a features of bond formation (in the
context of valence-bond theory)?
D9A.3 Describe the various types of hybrid orbitals and how they are
used to describe the bonding in alkanes, alkenes, and alkynes. How does D9A.5 What are the consequences of resonance?

Additional exercises
E9A.6 Write the valence-bond wavefunction for the triple bond in N2. E9A.13 Show that the linear combinations h1 = s + px + py + pz and
h2 = s − px − py + pz are mutually orthogonal.
E9A.7 Write the valence-bond wavefunction for the resonance hybrid
+ − 2− 2+
N2 ↔ N N ↔ N N ↔ structures of similar energy. E9A.14 Show that the linear combinations h1 = (sin ζ )s + (cos ζ )p and
h2 = (cos ζ )s − (sin ζ )p are mutually orthogonal for all values of the angle
E9A.8 Describe the structures of SO2 and SO3 in terms of valence-bond theory.
ζ (zeta).
E9A.9 Account for the ability of phosphorus to form five bonds, as in PF5. 2 1/2
E9A.15 Normalize to 1 the sp hybrid orbital h = s + 2 p given that
E9A.10 Describe the bonding in 1,3-pentadiene using hybrid orbitals. the s and p orbitals are each normalized to 1.
E9A.11 Describe the bonding in methylamine, CH3NH2, using hybrid orbitals. E9A.16 Normalize to 1 the linear combinations h1 = (sin ζ )s + (cos ζ )p
and h2 = (cos ζ )s − (sin ζ )p, given that the s and p orbitals are each
E9A.12 Describe the bonding in pyridine, C5H5N, using hybrid orbitals.
normalized to 1.

Problems
P9A.1 Use the wavefunction for a H1s orbital to write a valence-bond Use a graphical argument to show that this function points in the specified
wavefunction of the form Ψ(1,2) = A(1)B(2) + A(2)B(1) in terms of the direction. (Hint: Consider the px and py orbitals as being represented by unit
Cartesian coordinates of each electron, given that the internuclear separation vectors along x and y.)
(along the z-axis) is R.
P9A.3 Confirm that the hybrid orbitals in eqn 9A.7 make angles of 120° to
2
P9A.2 An sp hybrid orbital that lies in the xy-plane and makes an angle of each other. (Hint: Consider the px and py orbitals as being represented by unit
120° to the x-axis has the form vectors along x and y.)
λ
P9A.4 Show that if two equivalent hybrid orbitals of the form sp make an
1  1 31/ 2  angle θ to each other, then λ = ±(−1/cos θ)1/2. Plot a graph of λ against θ and
 1/ 2 
s  1/ 2 p x  1/ 2 p y 
3  2 2  confirm that θ = 180° when no s orbital is included and θ = 120° when λ = 2.

TOPIC 9B Molecular orbital theory: the hydrogen molecule-ion


Discussion questions
D9B.1 Discuss the role of the Born–Oppenheimer approximation in the D9B.2 What feature of molecular orbital theory is responsible for bond formation?
molecular-orbital calculation of a molecular potential energy curve
D9B.3 Why is spin-pairing so common a feature of bond formation (in the
or surface.
context of molecular orbital theory)?
392 9 Molecular structure

Additional exercises
E9B.4 Normalize to 1 the molecular orbital ψ = ψA + λψB + λ′ψB′ in terms of +
E9B.8 The energy of H2 with internuclear separation R is given by eqn 9B.4.
the parameters λ and λ′ and the appropriate overlap integrals, where ψB and The values of the contributions are given below.
ψB′ are mutually orthogonal and normalized orbitals on atom B.
E9B.5 Suppose that a molecular orbital has the (unnormalized) form R/a0 0 1 2 3 4
0.145A + 0.844B. Find a linear combination of the orbitals A and B that is j/Eh 1.000 0.729 0.472 0.330 0.250
orthogonal to this combination and determine the normalization constants of
k/Eh 1.000 0.736 0.406 0.199 0.092
both combinations using S = 0.250.
S 1.000 0.858 0.587 0.349 0.189
E9B.6 Suppose that a molecular orbital has the (unnormalized) form
0.727A + 0.144B. Find a linear combination of the orbitals A and B that is
orthogonal to this combination and determine the normalization constants of where Eh = 27.2 eV, a0 = 52.9 pm, and EH1s = −½Eh. Plot the molecular
both combinations using S = 0.117. potential energy curve for the antibonding orbital, which is given by eqn 9B.7.
+
E9B.7 The energy of H2 with internuclear separation R is given by eqn 9B.4. E9B.9 Identify the g or u character of bonding and antibonding δ orbitals
The values of the contributions are given below. Plot the molecular potential formed by face-to-face overlap of d atomic orbitals.
energy curve and find the bond dissociation energy (in electronvolts) and the
equilibrium bond length.

R/a0 0 1 2 3 4
j/j0 1.000 0.729 0.472 0.330 0.250
k/j0 1.000 0.736 0.406 0.199 0.092
S 1.000 0.858 0.587 0.349 0.189

where j0 = 27.2 eV, a0 = 52.9 pm, and EH1s   12 j0.

Problems
P9B.1 Calculate the (molar) energy of electrostatic repulsion between two of the antibonding orbital by one electron. Is your conclusion true at all
hydrogen nuclei at the separation in H2 (74.1 pm). The result is the energy that internuclear separations?
must be overcome by the attraction from the electrons that form the bond.
P9B.4 Use mathematical software or a spreadsheet to plot the amplitude of
Does the gravitational attraction between the nuclei play any significant role?
the σ and σ⋆ wavefunctions (given in eqn 9B.2) along the z-axis for different
Hint: The gravitational potential energy of two masses is equal to −Gm1m2/r;
values of the internuclear distance. Identify the features of the σ orbital that
the gravitational constant G is listed on page i.
lead to bonding, and of the σ⋆ orbital that lead to antibonding. The forms of
3
P9B.2 Imagine a small electron-sensitive probe of volume 1.00 pm inserted the two H1s orbitals are  A  (a03 )1/2 e  rA1 /a0 and  B  (a03 )1/2 e  rB1 /a0 where rA1
+
into an H molecule-ion in its ground state. Calculate the probability that
2 is the distance from the electron to nucleus A, and rB1 is the distance from the
it will register the presence of an electron at the following positions: (a) at electron to nucleus B.
nucleus A, (b) at nucleus B, (c) half way between A and B, (d) at a point
P9B.5 (a) Calculate the total amplitude of the normalized bonding and
20 pm along the bond from A and 10 pm perpendicularly. Do the same for
antibonding LCAO-MOs that may be formed from two H1s orbitals at a
the molecule-ion the instant after the electron has been excited into the
separation of 2a0 = 106 pm. Plot the two amplitudes for positions along the
antibonding LCAO-MO. Take R = 2.00a0.
molecular axis both inside and outside the internuclear region. (b) Plot
+
P9B.3 Examine whether occupation of the bonding orbital in the H2 molecule- the probability densities of the two orbitals. Then form the difference
ion by one electron has a greater or lesser bonding effect than occupation density, the difference between ψ2 and 12 ( A2   B2 ).

TOPIC 9C Molecular orbital theory: homonuclear diatomic molecules


Discussion questions
D9C.1 Draw diagrams to show the various orientations in which a p orbital D9C.3 What is the justification for treating s and p atomic orbital contributions
and a d orbital on adjacent atoms may form bonding and antibonding to molecular orbitals separately?
molecular orbitals.
D9C.4 To what extent can orbital overlap be related to bond strength? To what
D9C.2 Outline the rules of the building-up principle for homonuclear diatomic extent might that be a correlation rather than an explanation?
molecules.

Additional exercises
E9C.6 Give the ground-state electron configurations and bond orders of E9C.7 From the ground-state electron configurations of Li2 and Be2, predict
(i) F2− , (ii) N2, and (iii) O2−
2 . which molecule should have the greater dissociation energy.
Exercises and problems 393

+ − 2− + −
E9C.8 Arrange the species O2 , O2, O2 , O2 in order of increasing bond length. E9C.12 For each Period 2 homonuclear diatomic cation, X 2 , and anion, X 2 ,
specify which molecular orbital is the LUMO (the lowest energy unoccupied
E9C.9 Evaluate the bond order of each Period 2 homonuclear diatomic
orbital).
molecule.
E9C.13 What is the speed of a photoelectron ejected from a molecule with
E9C.10 Evaluate the bond order of each Period 2 homonuclear diatomic
+ − radiation of energy 21 eV and known to come from an orbital of ionization
cation, X , and anion, X .
2 2
energy 12 eV?
+ −
E9C.11 For each Period 2 homonuclear diatomic cation, X 2 , and anion, X 2 ,
specify which molecular orbital is the HOMO (the highest energy occupied
orbital).

Problems
P9C.1 Familiarity with the magnitudes of overlap integrals is useful when on their separation. The overlap integral between an H1s orbital and
considering bonding abilities of atoms, and hydrogenic orbitals give an indication an H2p orbital directed towards it on nuclei separated by a distance R
of their values. (a) The overlap integral between two hydrogenic 2s orbitals is is S  (R/a0 ){1  (R/a0 )  13 (R/a0 )2 }e  R /a0 . Plot this function, and find the
separation for which the overlap is a maximum.
 ZR 1  ZR 2 1  ZR    ZR /2a0
4

S(2s,2s)  1        e ‡
P9C.3 Use the 2px and 2pz hydrogenic atomic orbitals to construct
2a0 12  a0  240  a0  
  simple LCAO descriptions of 2pσ and 2pπ molecular orbitals. (a) Make a
Plot this expression. (b) For what internuclear distance is S(2s,2s) = 0.50? probability density plot, and both surface and contour plots of the xz-plane
(c) The side-by-side overlap of two 2p orbitals of atoms of atomic number Z is amplitudes of the 2pzσ and 2pzσ molecular orbitals. (b) Plot the amplitude
of the 2pxπ and 2pxπ molecular orbital wavefunctions in the xz-plane.
 ZR 1  ZR 2 1  ZR    ZR /2a0
3
Include plots for both an internuclear distance, R, of 10a0 and 3a0, where
S(2p,2p)  1        e a0 = 52.9 pm. Interpret the graphs, and explain why this graphical
 2a0 10  a0  120  a0  
information is useful.
Plot this expression. (d) Evaluate S(2s,2p) at the internuclear distance you
P9C.4 In a photoelectron spectrum using 21.21 eV photons, electrons were
calculated in part (b).
ejected with kinetic energies of 11.01 eV, 8.23 eV, and 15.22 eV. Sketch the
P9C.2 Before doing a calculation, sketch how the overlap between a 1s molecular orbital energy level diagram for the species, showing the ionization
orbital and a 2p orbital directed towards it can be expected to depend energies of the three identifiable orbitals.

TOPIC 9D Molecular orbital theory: heteronuclear diatomic molecules


Discussion questions
D9D.1 Describe the Pauling and Mulliken electronegativity scales. Why should D9D.3 Discuss the steps involved in the calculation of the energy of a system
they be approximately in step? by using the variation principle. Are any assumptions involved?
D9D.2 Why do both ionization energy and electron affinity play a role in D9D.4 What is the physical significance of the Coulomb and resonance
estimating the energy of an atomic orbital to use in a molecular orbital integrals?
calculation?

Additional exercises
E9D.4 Give the ground-state electron configurations of (i) CO, (ii) NO, and E9D.10 A reasonably reliable conversion between the Mulliken and Pauling
(iii) CN−. electronegativity scales is given by eqn 9D.4. Use Table 9D.1 in the Resource
section to assess how good the conversion formula is for Period 2 elements.
E9D.5 Give the ground-state electron configurations of (i) XeF, (ii) PN, and
(iii) SO−. E9D.11 A reasonably reliable conversion between the Mulliken and Pauling
electronegativity scales is given by eqn 9D.4. Use Table 9D.1 in the Resource
E9D.6 Sketch the molecular orbital energy level diagram for XeF and deduce
section to assess how good the conversion formula is for Period 3 elements.
its ground-state electron configuration. Is XeF likely to have a shorter bond
length than XeF+? E9D.12 Estimate the orbital energies to use in a calculation of the molecular
orbitals of HBr. For data, see Tables 8B.4 and 8B.5. Take β = −1.00 eV.
E9D.7 Sketch the molecular orbital energy level diagram for IF and deduce its
ground-state electron configuration. Is IF likely to have a shorter bond length E9D.13 Estimate the molecular orbital energies in HBr; use S = 0. Hint: Estimate the
than IF− or IF+? atomic orbital energies by using the data in Tables 8B.4 and 8B.5; take β = −1.00 eV.
− +
E9D.8 Use the electron configurations of NO and NO to predict which is E9D.14 Estimate the molecular orbital energies in HBr; use S = 0.20. Hint:
likely to have the shorter bond length. Estimate the atomic orbital energies by using the data in Tables 8B.4 and 8B.5;
− +
take β = −1.00 eV.
E9D.9 Use the electron configurations of SO and SO to predict which is likely

to have the shorter bond length. These problems were supplied by Charles Trapp and Carmen Giunta.
394 9 Molecular structure

Problems
P9D.1 Show, if overlap is ignored, (a) that if a molecular orbital is expressed as orbital energies and coefficients with both SAB and SAC equal to (i) 0, (ii) 0.2
a linear combination of two atomic orbitals in the form ψ = ψA cos θ + (note that SBC = 0 for orbitals on the same atom).
ψB sin θ, where θ is a parameter that varies between 0 and π, with ψA and ψB
P9D.3 (a) Suppose that a molecular orbital of a heteronuclear diatomic
are orthogonal and normalized to 1, then ψ is also normalized to 1. (b) To
molecule is built from the orbital basis A, B, and C, where B and C are both
what values of θ do the bonding and antibonding orbitals in a homonuclear
on one atom. Set up the secular equations for the values of the coefficients and
diatomic molecule correspond?
the corresponding secular determinant. (b) Now let αA = −7.2 eV,
P9D.2 (a) Suppose that a molecular orbital of a heteronuclear diatomic αB = −10.4 eV, αC = −8.4 eV, βAB = −1.0 eV, βAC = −0.8 eV, and calculate the
molecule is built from the orbital basis A, B, and C, where B and C are both orbital energies and coefficients with both SAB and SAC equal to 0 (note that
on one atom. Set up the secular equations for the values of the coefficients and SBC = 0 for orbitals on the same atom). (c) Explore the consequences of
the corresponding secular determinant. (b) Now let αA = −7.2 eV, increasing the energy separation of the ψB and ψC orbitals. Are you justified in
αB = −10.4 eV, αC = −8.4 eV, βAB = −1.0 eV, βAC = −0.8 eV, and calculate the ignoring orbital ψC at any stage?

TOPIC 9E Molecular orbital theory: polyatomic molecules


Discussion questions
D9E.1 Discuss the scope, consequences, and limitations of the approximations D9E.3 Outline the computational steps used in the self-consistent field
on which the Hückel method is based. approach to electronic structure calculations.
D9E.2 Distinguish between delocalization energy, π-electron binding energy, D9E.4 Explain why the use of Gaussian-type orbitals is generally preferred
and π-bond formation energy. Explain how each concept is employed. over the use of hydrogenic orbitals in basis sets.

Additional exercises
E9E.7 Set up the secular determinants for (i) linear H4, (ii) cyclic H4 within the E9E.11 Set up the secular determinants for (i) azulene (3), (ii) acenaphthylene
Hückel approximation. (4) within the Hückel approximation and using the out-of-plane C2p orbitals
as the basis set.
E9E.8 Predict the electron configurations of (i) the allyl radical, •CH2CHCH2,
(ii) the cyclobutadiene cation C 4 H +4 . Estimate the π-electron binding energy
in each case.
E9E.9 What is the delocalization energy and π-bond formation energy of
(i) the allyl radical, (ii) the cyclobutadiene cation?
3 Azulene 4 Acenaphthalene
E9E.10 Set up the secular determinants for (i) anthracene (1),
(ii) phenanthrene (2) within the Hückel approximation and using the E9E.12 Use mathematical software to estimate the π-electron binding
out-of-plane C2p orbitals as the basis set. energy of (i) anthracene (1), (ii) phenanthrene (2) within the Hückel
approximation.
E9E.13 Use mathematical software to estimate the π-electron binding
energy of (i) azulene (3), (ii) acenaphthylene (4) within the Hückel
approximation.
2+
E9E.14 Write the electronic hamiltonian for LiH .
1 Anthracene 2 Phenanthrene

Problems
P9E.1 Set up and solve the Hückel secular equations for the π electrons of the 2k 
Ek    2  cos k  0, 1, ,N /2 for N even
triangular, planar CO2−
3 ion. Express the energies in terms of the Coulomb N
integrals αO and αC and the resonance integral β. Estimate the delocalization k  0, 1, ,(N  1)/2 for N odd
energy of the ion.
(a) Calculate the energies of the π molecular orbitals of benzene and
P9E.2 For monocyclic conjugated polyenes (such as cyclobutadiene and cyclooctatetraene (5). Comment on the presence or absence of degenerate
benzene) with each of N carbon atoms contributing an electron in a energy levels. (b) Calculate and compare the delocalization energies of
2p orbital, simple Hückel theory gives the following expression for the benzene (using the expression above) and hexatriene. What do you conclude
energies Ek of the resulting π molecular orbitals (all are doubly degenerate from your results? You will need the following expression for the energies
except the lowest and highest values of k): of the π molecular orbitals of a linear polyene with N carbon atoms:
Exercises and problems 395

Ek    2  cos{k/(N  1)}, k  1,2, ,N . (c) Calculate and compare the carbon monoxide, and ethanol. The H3+ ion has also been found in the
delocalization energies of cyclooctatetraene and octatetraene. Are your atmospheres of Jupiter, Saturn, and Uranus. (a) Solve the Hückel secular
conclusions for this pair of molecules the same as for the pair of molecules equations for the energies of the H3 system in terms of the parameters α and
investigated in part (b)? β, draw an energy level diagram for the orbitals, and determine the binding
energies of H3+ , H3, and H3− . (b) Accurate quantum mechanical calculations
by G.D. Carney and R.N. Porter ( J. Chem. Phys. 65, 3547 (1976)) give the
dissociation energy for the process H3  H  H  H  as 849 kJ mol−1. From
this information and data in Table 9C.3, calculate the enthalpy of the reaction
H  (g )  H2 (g )  H3 (g ). (c) From your equations and the information given,
calculate a value for the resonance integral β in H3+ . Then go on to calculate
5 Cyclooctatetraene the binding energies of the other H3 species in (a).

P9E.3 Suppose that a molecular orbital of a heteronuclear diatomic molecule P9E.8 There is some indication that other hydrogen ring compounds and
is built from the orbital basis ψA, ψB, and ψC, where ψB and ψC are both on ions in addition to H3 and D3 species may play a role in interstellar chemistry.
one atom (they can be envisaged as F2s and F2p in HF, for instance). Set up According to J.S. Wright and G.A. DiLabio ( J. Phys. Chem. 96, 10 793 (1992)),
the secular equations for the optimum values of the coefficients and set up the H5− , H6, and H7+ are particularly stable whereas H4 and H5+ are not. Confirm
corresponding secular determinant. these statements by Hückel calculations.

P9E.4 Set up the secular determinants for the homologous series consisting of P9E.9 Use appropriate electronic structure software and basis sets of your or
ethene, butadiene, hexatriene, and octatetraene and diagonalize them by using your instructor’s choosing, perform self-consistent field calculations for the
mathematical software. Use your results to show that the π molecular orbitals ground electronic states of H2 and F2. Determine ground-state energies and
of linear polyenes obey the following rules: equilibrium geometries. Compare computed equilibrium bond lengths to
experimental values.
• The π molecular orbital with lowest energy is delocalized over all carbon
atoms in the chain. P9E.10 Use an appropriate semi-empirical method to compute the equilibrium
bond lengths and standard enthalpies of formation of (a) ethanol, (b)
• The number of nodal planes between C2p orbitals increases with the 1,4-dichlorobenzene. Compare to experimental values and suggest reasons for
energy of the π molecular orbital. any discrepancies.
P9E.5 Set up the secular determinants for cyclobutadiene, benzene, and P9E.11 (a) For a linear conjugated polyene with each of N carbon atoms
cyclooctatetraene and diagonalize them by using mathematical software. Use contributing an electron in a 2p orbital, the energies Ek of the resulting π
your results to show that the π molecular orbitals of monocyclic polyenes with molecular orbitals are given by:
an even number of carbon atoms follow a pattern in which:
k
• The π molecular orbitals of lowest and highest energy are non- Ek    2  cos k  1,2, ,N
degenerate. N 1

• The remaining π molecular orbitals exist as degenerate pairs. Use this expression to make a reasonable empirical estimate of the resonance
integral β for the homologous series consisting of ethene, butadiene, hexatriene,
P9E.6 Electronic excitation of a molecule may weaken or strengthen some and octatetraene given that π←π ultraviolet absorptions from the HOMO to the
bonds because bonding and antibonding characteristics differ between the LUMO occur at 61 500, 46 080, 39 750, and 32 900 cm−1, respectively. (b) Calculate
HOMO and the LUMO. For example, a carbon–carbon bond in a linear the π-electron delocalization energy, Edeloc = Eπ − n(α + β), of octatetraene, where
polyene may have bonding character in the HOMO and antibonding Eπ is the total π-electron binding energy and n is the total number of π-electrons.
character in the LUMO. Therefore, promotion of an electron from the HOMO (c) In the context of this Hückel model, the π molecular orbitals are written as
to the LUMO weakens this carbon–carbon bond in the excited electronic linear combinations of the carbon 2p orbitals. The coefficient of the jth atomic
state, relative to the ground electronic state. Consult Figs. 9E.2 and 9E.4 orbital in the kth molecular orbital is given by:
and discuss in detail any changes in bond order that accompany the π⋆←π
1/ 2
ultraviolet absorptions in butadiene and benzene.  2  jk
ckj    sin N  1 j  1,2, , N

P9E.7 In Exercise E9E.1 you are invited to set up the Hückel secular  N 1
determinant for linear and cyclic H3. The same secular determinant applies Evaluate the coefficients of each of the six 2p orbitals in each of the six π
to the molecular ions H3+ and D3+ . The molecular ion H3+ was discovered as molecular orbitals of hexatriene. Match each set of coefficients (that is, each
long ago as 1912 by J.J. Thomson but the equilateral triangular structure molecular orbital) with a value of the energy calculated with the expression
was confirmed by M.J. Gaillard et al. (Phys. Rev. A17, 1797 (1978)) much given in part (a) of the molecular orbital. Comment on trends that relate
more recently. The molecular ion H3+ is the simplest polyatomic species with the energy of a molecular orbital with its ‘shape’, which can be inferred from
a confirmed existence and plays an important role in chemical reactions the magnitudes and signs of the coefficients in the linear combination that
occurring in interstellar clouds that may lead to the formation of water, describes the molecular orbital.

FOCUS 9 Molecular structure


Integrated activities
I9.1 The languages of valence-bond theory and molecular orbital theory are I9.2 Here a molecular orbital theory treatment of the peptide group (6) is
commonly combined when discussing unsaturated organic compounds. developed, a group that links amino acids in proteins, and establish the
Construct the molecular orbital energy level diagrams of ethene on the basis that features that stabilize its planar conformation. (a) It will be familiar from
the molecule is formed from the appropriately hybridized CH2 or CH fragments. introductory chemistry that valence-bond theory explains the planar
396 9 Molecular structure

conformation by invoking delocalization of the π bond over the oxygen, number of studies indicate that there is a linear correlation between the
carbon, and nitrogen atoms by resonance: LUMO energy and the reduction potential of aromatic hydrocarbons.
(a) The standard potentials at pH = 7 for the one-electron reduction of
– methyl-substituted 1,4-benzoquinones (7) to their respective semiquinone
O O O
radical anions are:
C C + C
C N C N C N ⦵
R2 R3 R5 R6 E /V
H H H
H H H H 0.078
6 Peptide group CH3 H H H 0.023
CH3 H CH3 H −0.067
It follows that the peptide group can be modelled by using molecular orbital
theory by constructing LCAO-MOs from 2p orbitals perpendicular to the plane CH3 CH3 CH3 H −0.165
defined by the O, C, and N atoms. The three combinations have the form: CH3 CH3 CH3 CH3 −0.260
 1  a O  b C  c N  2  d O  e N  3  f  O  g  C  h N
where the coefficients a to h are all positive. Sketch the orbitals ψ1, ψ2, and O
ψ3 and characterize them as bonding, nonbonding, or antibonding. In a
nonbonding molecular orbital, a pair of electrons resides in an orbital confined R6 R2
largely to one atom and not appreciably involved in bond formation. (b) Show
that this treatment is consistent only with a planar conformation of the peptide
link. (c) Draw a diagram showing the relative energies of these molecular
R5 R3
orbitals and identify the occupancy of the orbitals. Hint: Convince yourself that O
there are four electrons to be distributed among the molecular orbitals.
(d) Now consider a nonplanar conformation of the peptide link, in which the 7
O2p and C2p orbitals are perpendicular to the plane defined by the O, C, and
Use the computational method of your or your instructor’s choice (semi-
N atoms, but the N2p orbital lies on that plane. The LCAO-MOs are given by
empirical, ab initio, or density functional theory methods) to calculate ELUMO,
 4  a O  b C  5  e N  6  f  O  g  C the energy of the LUMO of each substituted 1,4-benzoquinone, and plot ELUMO

against E . Do your calculations support a linear relation between ELUMO and
Just as before, sketch these molecular orbitals and characterize them as ⦵
E ? (b) The 1,4-benzoquinone for which R2 = R3 = CH3 and R5 = R6 = OCH3 is a
bonding, nonbonding, or antibonding. Also, draw an energy level diagram and
suitable model of ubiquinone, a component of the respiratory electron transport
identify the occupancy of the orbitals. (e) Why is this arrangement of atomic
chain. Determine ELUMO of this quinone and then use your results from part
orbitals consistent with a nonplanar conformation for the peptide link? (f)
(a) to estimate its standard potential. (c) The 1,4-benzoquinone for which
Does the bonding MO associated with the planar conformation have the same
R2 = R3 = R5 = CH3 and R6 = H is a suitable model of plastoquinone, an electron
energy as the bonding MO associated with the nonplanar conformation? If not,
carrier in photosynthesis. Determine ELUMO of this quinone and then use
which bonding MO is lower in energy? Repeat the analysis for the nonbonding
your results from part (a) to estimate its standard potential. Is plastoquinone
and antibonding molecular orbitals. (g) Use your results from parts (a)–(f) to
expected to be a better or worse oxidizing agent than ubiquinone?
construct arguments that support the planar model for the peptide link.
I9.6 Molecular orbital calculations based on semi-empirical, ab initio,
I9.3 Molecular electronic structure methods may be used to estimate the
and DFT methods describe the spectroscopic properties of conjugated
standard enthalpy of formation of molecules in the gas phase. (a) Use a semi-
molecules better than simple Hückel theory. (a) Use the computational
empirical method of your or your instructor’s choice to calculate the standard
method of your or your instructor’s choice (semi-empirical, ab initio, or
enthalpy of formation of ethene, butadiene, hexatriene, and octatetraene
density functional methods) to calculate the energy separation between
in the gas phase. (b) Consult a database of thermochemical data, and, for
the HOMO and LUMO of ethene, butadiene, hexatriene, and octatetraene.
each molecule in part (a), calculate the difference between the calculated
(b) Plot the HOMO–LUMO energy separations against the experimental
and experimental values of the standard enthalpy of formation. (c) A good
frequencies for π⋆←π ultraviolet absorptions for these molecules (61 500,
thermochemical database will also report the uncertainty in the experimental
46 080, 39 750, and 32 900 cm−1, respectively). Use mathematical software to
value of the standard enthalpy of formation. Compare experimental
find the polynomial equation that best fits the data. (b) Use your polynomial
uncertainties with the relative errors calculated in part (b) and discuss
fit from part (b) to estimate the wavenumber and wavelength of the π⋆←π
the reliability of your chosen semi-empirical method for the estimation of
ultraviolet absorption of decapentaene from the calculated HOMO–LUMO
thermochemical properties of linear polyenes.
energy separation. (c) Discuss why the calibration procedure of part (b) is
I9.4 The standard potential of a redox couple is a measure of the necessary.
thermodynamic tendency of an atom, ion, or molecule to accept an electron
I9.7 The variation principle can be used to formulate the wavefunctions
(Topic 6D). Studies indicate that there is a correlation between the LUMO
of electrons in atoms as well as in molecules. Suppose that the function
energy and the standard potential of aromatic hydrocarbons. Do you expect 2
 trial  N ( )e  ar with N(α) the normalization constant and α an adjustable
the standard potential to increase or decrease as the LUMO energy decreases?
parameter, is used as a trial wavefunction for the 1s orbital of the hydrogen
Explain your answer.
atom. Show that
I9.5 Molecular orbital calculations may be used to predict trends in the
standard potentials of conjugated molecules, such as the quinones and flavins, 32 e2
E( )   1/ 2 3/ 2  1/ 2
that are involved in biological electron transfer reactions. It is commonly 2 2  0
assumed that decreasing the energy of the LUMO enhances the ability of
a molecule to accept an electron into the LUMO, with an accompanying where e is the fundamental charge, and μ is the reduced mass for the atom.
increase in the value of the molecule’s standard potential. Furthermore, a What is the minimum energy associated with this trial wavefunction?
FOCUS 10
Molecular symmetry

In this Focus the concept of ‘shape’ is sharpened into a precise operations (such as rotations and reflections) by matrices. This
definition of ‘symmetry’. As a result, symmetry and its conse- step is important, because once symmetry operations are ex-
quences can be discussed systematically, thereby providing pressed numerically they can be manipulated quantitatively.
a powerful tool for the prediction and analysis of molecular The Topic introduces ‘character tables’ which are key to the ap-
structure and properties. plication of group theory to chemical problems.
10B.1 The elements of group theory; 10B.2 Matrix representations;
10B.3 Character tables

10A Shape and symmetry


This Topic shows how to classify any molecule according to its
symmetry. Two immediate applications of this classification 10C Applications of symmetry
are the identification of whether or not a molecule can have an
electric dipole moment (and so be polar) and whether or not it Group theory provides simple criteria for deciding whether
can be chiral (and so be optically active). certain integrals necessarily vanish. One application is to de-
cide whether the overlap integral between two atomic orbitals
10A.1 Symmetry operations and symmetry elements;
10A.2 The symmetry classification of molecules; is necessarily zero and therefore to decide which atomic or-
10A.3 Some immediate consequences of symmetry bitals can contribute to molecular orbitals. Group theory also
provides a way of constructing linear combinations of atomic
orbitals that have the correct symmetry to overlap with one an-
other and hence form molecular orbitals. By considering the
10B Group theory symmetry properties of integrals, it is also possible to derive the
selection rules that govern spectroscopic transitions.
The systematic treatment of symmetry is an application of 10C.1 Vanishing integrals; 10C.2 Applications to molecular orbital
‘group theory’. This theory represents the outcome of ­symmetry theory; 10C.3 Selection rules

➤ Go to the e-book for videos that feature the derivation and interpretation of equations, and applications of this material.
TOPIC 10A Shape and symmetry

C3 C2
➤ Why do you need to know this material?
Symmetry arguments can be used to make immediate
assessments of the properties of molecules; the initial step
is to identify the symmetry a molecule possesses and then
to classify it accordingly.

➤ What is the key idea?


(a) (b)
Molecules can be classified into groups according to their
symmetry elements. Figure 10A.2 (a) An NH3 molecule has a threefold (C3) axis and
(b) an H2O molecule has a twofold (C2) axis. Both have other
➤ What do you need to know already? symmetry elements too.
This Topic does not draw on others directly, but it will be
useful to be aware of the shapes of a variety of simple
molecules and ions encountered in introductory chemistry
courses.
This Topic puts these intuitive notions on a more formal
foundation. It will be seen that molecules can be grouped to-
gether according to their symmetry, with the tetrahedral spe-
cies CH4 and SO2−
4 in one group and the pyramidal species NH3
and SO2−3 in another. It turns out that molecules in the same
Some objects are ‘more symmetrical’ than others. A sphere is group share certain physical properties, so powerful predic-
more symmetrical than a cube because it looks the same after tions can be made about whole series of molecules once the
it has been rotated through any angle about any axis passing group to which they belong has been identified.
through the centre. A cube looks the same only if it is rotated
through certain angles about specific axes (Fig. 10A.1). For ex-
ample it looks the same after rotation by a multiple of 90° about
an axis passing through the centres of opposite faces (a fourfold
Symmetry operations and
10A.1
axis), or after rotation by a multiple of 120° about an axis pass- symmetry elements
ing through opposite corners (a threefold axis). Similarly, an
NH3 molecule is ‘more symmetrical’ than an H2O molecule be- An action that leaves an object looking the same after it has
cause NH3 looks the same after rotations of 120° or 240° about been carried out is called a symmetry operation. Typical sym-
the axis shown in Fig. 10A.2, whereas H2O looks the same only metry operations include rotations, reflections, and inversions.
after a rotation of 180°. There is a corresponding symmetry element for each symme-
try operation, which is the point, line, or plane with respect
C2 to which the symmetry operation is performed. For instance,
C3
a rotation (a symmetry operation) is carried out around an
axis (the corresponding symmetry element). Molecules can be
classified by identifying all their symmetry elements, and then
C4 grouping together molecules that possess the same set of sym-
metry elements. This procedure, for example, puts the trigonal
planar species BF3 and CO2− 3 into one group and the species
H2O (bent) and ClF3 (T-shaped) into another group.
An n-fold rotation (the operation) about an n-fold ­rotation
Figure 10A.1 Some of the symmetry elements of a cube. The axis, Cn (the corresponding element) is a rotation through
twofold, threefold, and fourfold axes are labelled with the 360°/n. An H2O molecule has one twofold rotation axis, C2. An
conventional symbols. NH3 molecule has one threefold rotation axis, C3, with which is
10A Shape and symmetry 399

associated two symmetry operations, one being a rotation by


120° in a clockwise sense, and the other a rotation by 120° in an
anticlockwise sense. There is only one twofold rotation associ-
ated with a C2 axis because clockwise and anticlockwise 180°
rotations are identical.
A pentagon has a C5 rotation axis, which has a total of four
rotations associated with it. The first two are a clockwise and
an anticlockwise rotation through 72°. The second two are σv
denoted C52 and correspond to two successive C5 rotations—­ σv
clockwise through 144°, and anticlockwise through 144°.
A cube has three C4 rotation axes, four C3 rotation axes, and Figure 10A.3 An H2O molecule has two mirror planes. They are
six C2 rotation axes. However, even this high symmetry is ex- both vertical (i.e. contain the principal axis), so are denoted σ v
ceeded by that of a sphere, which possesses an infinite number and  v .
of rotation axes (along any axis passing through the centre) of
all possible integral values of n.
If a molecule possesses several rotation axes, then the one
with the highest value of n is called the principal axis. The prin-
cipal axis of a benzene molecule is the sixfold rotation axis per-
pendicular to the hexagonal ring (1). If a molecule has more
than one rotation axis with this highest value of n, and it is
wished to designate one of them as the principal axis, then it
is common to choose the axis
σd σd
that passes through the great- σd
est number of atoms or, in the C6
case of a planar molecule (such Figure 10A.4 Dihedral mirror planes (σd) bisect the C2 axes
perpendicular to the principal axis.
as naphthalene, 2, which has
three C2 rotation axes compet-
ing for the title), to choose the ­ enzene molecule has such a horizontal mirror plane, perpen-
b
axis perpendicular to the plane. 1 Benzene, C6H6 dicular to the C6 (­principal) axis.
In an inversion (the operation) through a centre of
­inversion, i (the element), each point in a molecule is imagined
C2 as being moved in a straight line to the centre of inversion and
σv
σv then out the same distance on the other side; that is, the point
(x, y, z) is taken into the point (−x, −y, −z). Neither an H2O
molecule nor an NH3 molecule has a centre of inversion, but
i a sphere and a cube do have one. A benzene molecule has a
C2
centre of inversion, as does a regular octahedron (Fig. 10A.5); a
regular tetrahedron and a CH4 molecule do not.
σh An n-fold improper rotation (the operation) about an
C2 n-fold improper rotation axis, Sn (the symmetry element)
is composed of two successive transformations. The first is a
2 Naphthalene, C10H8

A reflection is the operation corresponding to a mirror Centre of


plane, σ (the element). If the plane contains the principal inversion, i
axis, it is called ‘vertical’ and denoted σv. An H2O molecule
has two vertical mirror planes (Fig. 10A.3) and an NH3 mol-
ecule has three. A vertical mirror plane that bisects the angle
between two C2 axes is called a ‘dihedral plane’ and is denoted
σd (Fig. 10A.4). When the mirror plane is perpendicular to
the principal axis it is called ‘horizontal’ and denoted σh. The Figure 10A.5 A regular octahedron has a centre of inversion (i).
400 10 Molecular symmetry

E10A.2 Locate the symmetry elements of anthracene and locate them in a


S4 S6 drawing of the molecule.
C4 E10A.3 Identify all of the symmetry elements possessed by the square planar
C6 anion PtCl42−. Locate them in a drawing of the molecule.
σh
σh

The symmetry classification


10A.2

(a) (b)
of molecules
Figure 10A.6 (a) A CH4 molecule has a fourfold improper rotation Objects are classified into groups according to the symme-
axis (S4): the molecule is indistinguishable after a 90° rotation try elements they possess. Point groups arise when objects
followed by a reflection across the horizontal plane, but neither are classified according to symmetry elements that cor-
operation alone is a symmetry operation. (b) The staggered form respond to operations leaving at least one common point
of ethane has an S6 axis composed of a 60° rotation followed by a unchanged. The five kinds of symmetry element identified
reflection. so far are of this kind. When crystals are considered (Topic
15A), symmetries arising from translation through space
r­ otation through 360°/n, and the second is a reflection through also need to be taken into account, and the classification ac-
a plane perpendicular to the axis of that rotation. Neither trans- cording to these elements gives rise to the more extensive
formation alone needs to be a symmetry operation. A CH4 space groups.
molecule has three S4 axes, and the staggered conformation of All molecules with the same set of symmetry elements be-
ethane has an S6 axis (Fig. 10A.6). long to the same point group, and the name of the group is
The identity, E, consists of doing determined by this set of symmetry elements. There are two
nothing; the corresponding sym- F systems of notation (Table 10A.1). The Schoenflies system (in
metry element is the entire object. which a name looks like C4v) is more common for the discus-
Because every molecule is indistin- C sion of individual molecules, and the Hermann–Mauguin
guishable from itself if nothing is I Cl system, or International system (in which a name looks like
done to it, every object possesses at 4mm), is used almost exclusively in the discussion of crystal
least the identity element. One rea- symmetry. The identification of the point group to which a
Br
son for including the identity is that molecule belongs (in the Schoenflies system) is simplified by
some molecules have only this sym- 3 CBrClFI referring to the flow diagram in Fig. 10A.7 and to the shapes
metry element (3). shown in Fig. 10A.8.

Brief illustration 10A.1

To identify the symmetry elements of a naphthalene molecule


(2), note that: Table 10A.1 The notations for point groups*

• Like all molecules, it has the identity element, E. Ci 1


• There are three twofold rotation axes, C2: one perpendicu- Cs m
lar to the plane of the molecule, and two others lying in the C1 1 C2 2 C3 3 C4 4 C6 6
plane. C2v 2mm C3v 3m C4v 4mm C6v 6mm
• With the C2 axis perpendicular to the plane of the mol- C2h 2/m C3h 6 C4h 4/m C6h 6/m
ecule chosen as the principal axis, there is a σh plane per- D2 222 D3 32 D4 422 D6 622
pendicular to the principal axis, and two σv planes which D2h mmm D3h 62m D4h 4/mmm D6h 6/mmm
contain the principal axis.
D2d 42m D3d 3m S4 4 S6 3
• There is also a centre of inversion, i, through the midpoint
T 23 Td 43m Th m3
of the molecule, which is mid-way along the C–C bond at
O 432 Oh m3m
the ring junction.
* Schoenflies notation unboxed, Hermann–Mauguin (International system) boxed. In the
Hermann–Mauguin system, a number n denotes the presence of an n-fold axis and m
denotes a mirror plane. A slash (/) indicates that the mirror plane is perpendicular to the
Exercises symmetry axis. It is important to distinguish symmetry elements of the same type but of
different classes, as in 4/mmm, in which there are three classes of mirror plane. A bar over
E10A.1 List the symmetry elements of CH3Cl and locate them in a drawing of a number indicates that the element is combined with an inversion. The only groups listed
the molecule. here are the so-called ‘crystallographic point groups’.
10A Shape and symmetry 401

Molecule

Cp = C5H5
Y N Two or
Linear? more Ru
Cn, n > 2?

Y N C Y
D∞h i? ∞v
N
Y
i?
N 4 Ruthenocene, Ru(Cp)2

N O If the rings are staggered, as they are in an excited state of fer-


Ih Y C5? h
Td
rocene (5), the σh plane is absent. The other mirror planes are
Select Cn with the highest n; still present, but now they bisect the angles between twofold
then, are there nC2 Y
axes and so are described as σd. Tracing the appropriate path in
perpendicular to Cn? Cn? Fig. 10A.7 gives the point group as D5d.
Y
N N
Dnh Y σh?
Y
Cs σ?
N Cp = C5H5
N
Dnd Y nσd?
N D
n
Y N C
Ci i? 1
Fe
Y σ h?
Cnh

N
Cnv Y nσv?
5 Ferrocene, Fe(Cp)2
N (excited state)
S2n Y S2n N C
n

n= 2 3 4 5 6 
Figure 10A.7 A flow diagram for determining the point group of a
molecule. Start at the top and answer the question posed in each
diamond (Y = yes, N = no). The dotted line refers to the path taken Cn
in Brief illustration 10A.2.

Dn

Brief illustration 10A.2


Cnv
To identify the point group to which a ruthenocene molecule Pyramid Cone

(4) belongs, first identify the symmetry elements present and


the use the flow diagram in Fig. 10A.7. Note that: Cnh

• The molecule has a fivefold rotation axis, and five twofold


rotation axes which pass through the Ru atom and are per- Dnh
pendicular to the C5 axis. Plane or bipyramid

• There is a mirror plane, σh, perpendicular to the C5 axis Dnd


and passing through the Ru atom.
• There are five σv planes containing the principal axis: each
passes through one carbon atom in a ring and the mid- S2n
point of the C–C bond on the opposite side. Each one of
these planes contains one of the twofold rotation axes.

Figure 10A.8 A summary of the shapes corresponding to different


The path to trace in Fig. 10A.7 is shown by a dotted line; it ends
point groups. The group to which a molecule belongs can often
at Dnh, and because the molecule has a fivefold axis, it belongs
be identified from this diagram without going through the formal
to the point group D5h.
procedure in Fig. 10A.7.
402 10 Molecular symmetry

10A.2(a) The groups C1, Ci, and Cs


C2
Name Elements A molecule belongs to the group C1 if it
C1 E
has no element other than the identity. It
belongs to Ci if it has the identity and a
Ci E, i
centre of inversion alone, and to Cs if it i
Cs E, σ
has the identity and a mirror plane alone. σh

Brief illustration 10A.3

• The CBrClFI molecule (3) has only the identity element, Figure 10A.9 The presence of a twofold axis and a horizontal
and so belongs to the group C1. mirror plane jointly imply the presence of a centre of inversion in
• Meso-tartaric acid (6) has the identity and a centre of in- the molecule.
version, and so belongs to the group Ci.

Brief illustration 10A.4


COOH
OH • In the H2O2 molecule (8) the two O–H bonds make an an-
H Centre of gle of about 115° to one another when viewed down the
inversion O–O bond direction. The molecule has the symmetry ele-
H OH ments E and C2, and so belongs to the group C2.

COOH C2

6 Meso-tartaric acid, O
HOOCCH(OH)CH(OH)COOH H
• Quinoline (7) has the elements (E,σ), and so belongs to the
group Cs. 8 Hydrogen peroxide, H2O2

• An H2O molecule has the symmetry elements E, C2, and


2σv, so it belongs to the group C2v.
• An NH3 molecule has the elements E, C3, and 3σv, so it
belongs to the group C3v.
N • A heteronuclear diatomic molecule such as HCl belongs
to the group C∞v because rotations around the internuclear
7 Quinoline, C9H7N axis by any angle and reflections in any of the infinite num-
ber of planes that contain this axis are symmetry opera-
tions. Other members of the group C∞v include the linear
10A.2(b) The groups Cn, Cnv, and Cnh OCS molecule and a cone.
• The molecule trans-CHCl=CHCl (9) has the elements E,
A molecule belongs to the group Cn if it possesses an n-fold ro-
C2, and σh, so belongs to the group C2h.
tation axis. Note that symbol Cn is now playing a triple role: as
the label of a symmetry element, a symmetry operation, and σh
Cl σh
a group. If in addition to the identity and a Cn rotation axis a
OH
molecule has n vertical mirror planes σv, then it belongs to the
group Cnv. Molecules that in addition to the identity and an C2 C3
B
n-fold principal axis also have a horizontal mirror plane σh be-
long to the groups Cnh.
The presence of certain symmetry elements may be implied Cl
by the presence of others: thus, in C2h the elements C2 and σh
jointly imply the presence of a centre of 9 trans-CHCl=CHCl
10 B(OH)3
Name Elements
inversion (Fig. 10A.9). Note also that
the tables specify the elements, not the • The molecule B(OH)3, in the planar conformation shown
Cn E, Cn
operations: for instance, there are two in (10), has a C3 axis and a σh plane, and so belongs to the
Cnv E, Cn, nσv
operations associated with a single C3 point group C3h.
Cnh E, Cn, σh
axis (rotations by +120° and −120°).
10A Shape and symmetry 403

10A.2(c) The groups Dn, Dnh, and Dnd C2 C2


Figure 10A.7 shows that a molecule that has an n-fold principal
axis and n twofold rotation axes perpendicular to Cn belongs to
the group Dn. A molecule belongs to Dnh if it also possesses a hor-
izontal mirror plane. The linear mole-
C2 S4
Name Elements cules OCO and HCCH, and a uniform
Dn E, Cn, nC′2 cylinder, all belong to the group D∞h. A
Dnh
molecule belongs to the group Dnd if in 13 Propadiene, C3H4 (D2d)
E, Cn, nC′2, σh
addition to the elements of Dn it pos-
Dnd E, Cn, nC′2, nσd
sesses n dihedral mirror planes σd.

Brief illustration 10A.5


10A.2(d) The groups Sn
• The trigonal planar BF3 molecule (11) has the elements E,
C3, 3C2 (with one C2 axis along each B–F bond), and σh, so Molecules that have not been classified into one of the
belongs to D3h. groups mentioned so far, but which possess one Sn axis, be-
long to the groups Sn. Note that the group S2 is the same
F as Ci, so such a molecule will already have been classified
as Ci. Tetraphenylmethane (14)
belongs to the point group S4; Name Elements
B
molecules belonging to Sn with Sn E, Sn and not
previously classified
n > 4 are rare.

11 Boron trifluoride, BF3

• The C6H6 molecule (1) has the elements E, C6, 3C2, 3C2′ , and
σh together with some others that these elements imply, so
it belongs to D6h. Three of the C2 axes bisect C–C bonds on
opposite sides of the hexagonal ring formed by the carbon
atoms, and the other three pass through vertices on oppo- S4
site sides of the ring. The prime on 3C2′ indicates that these
axes are different from the other three C2 axes.
• All homonuclear diatomic molecules, such as N2, belong 14 Tetraphenylmethane, C(C6H5)4 (S4)
to the group D∞h because all rotations around the internu-
clear axis are symmetry operations, as are end-over-end
rotations by 180°.
• PCl5 (12) is another example of a D3h species. 10A.2(e) The cubic groups
C3 A number of very important molecules possess more than one
Cl
principal axis. Most belong to the cubic groups, and in particu-
lar to the tetrahedral groups T, Td, and Th (Fig. 10A.10a) or to
C2 the octahedral groups O and Oh (Fig. 10A.10b). A few icosahe-
C2 dral (20-faced) molecules belonging to the icosahedral group,
P I (Fig. 10A.10c), are also known.
σh The groups Td and Oh are the groups of the regular
C2
­tetrahedron and the regular octahedron, respectively. If the
object possesses the rotational symmetry of the tetrahedron
12 Phosphorus pentachloride, PCl5 (D3h) or the octahedron, but none of their planes of reflection, then
it belongs to the simpler groups T or O (Fig. 10A.11). The
• Propadiene (an allene, 13), in which the two CH2 groups group Th is based on T but also contains a centre of inversion
lie in perpendicular planes, belongs to the point group D2d. (Fig. 10A.12).
404 10 Molecular symmetry

Name Elements Brief illustration 10A.6


T E, 4C3, 3C2
Td E, 3C2, 4C3, 3S4, 6σd
• The molecules CH4 and SF6 belong, respectively, to the
groups Td and Oh.
Th E, 3C2, 4C3, i, 4S6, 3σh
• Molecules belonging to the icosahedral group I include
O E, 3C4, 4C3, 6C2
some of the boranes and buckminsterfullerene, C60 (15).
Oh E, 3S4, 3C4, 6C2, 4S6, 4C3, 3σh, 6σd, i
I E, 6C5, 10C3, 15C2
Ih E, 6S10, 10S6, 6C5, 10C3, 15C2, 15σ, i

15 Buckminsterfullerene, C60 (I)

• The objects shown in Fig. 10A.11 belong to the groups T


and O, respectively.
(a) (b) (c)

Figure 10A.10 (a) Tetrahedral, (b) octahedral, and (c) icosahedral


shapes drawn to show their relation to a cube: they belong to the 10A.2(f ) The full rotation group
cubic groups Td, Oh, and Ih, respectively. The full rotation group, R3 (the 3
Name Elements
refers to rotation in three dimen-
R3 E, ∞C2, ∞C3,…
sions), consists of an infinite num-
ber of rotation axes with all possible
values of n. A sphere and an atom belong to R3, but no molecule
does. Exploring the consequences of R3 is a very important way
of applying symmetry arguments to atoms, and is an alternative
approach to the theory of orbital angular momentum.

Exercises
E10A.4 Identify the point groups to which the following objects belong:
(i) a sphere, (ii) an isosceles triangle, (iii) an equilateral triangle, (iv) an
unsharpened cylindrical pencil.
(a) (b) E10A.5 List the symmetry elements of the following molecules and name
the point groups to which they belong: (i) NO2, (ii) PF5, (iii) CHCl3,
Figure 10A.11 Shapes corresponding to the point groups (iv) 1,4-difluorobenzene.
(a) T and (b) O. The presence of the decorated slabs reduces
E10A.6 Assign (i) cis-dichloroethene and (ii) trans-dichloroethene to point groups.
the symmetry of the object from Td and Oh, respectively.

10A.3Some immediate consequences


of symmetry
Some statements about the properties of a molecule can be
made as soon as its point group has been identified.

10A.3(a) Polarity
A polar molecule is one with a permanent electric dipole mo-
ment (HCl, O3, and NH3 are examples). A dipole moment is a
Figure 10A.12 The shape of an object belonging to the group Th. property of the molecule, so it follows that the dipole moment
10A Shape and symmetry 405

(which is represented by a vector) must be unaffected by any


symmetry operation of the molecule because, by definition, S2
such an operation leaves the molecule apparently unchanged.
If a molecule possesses a Cn axis (n > 1) then it is not pos-
sible for there to be a dipole moment perpendicular to this axis
i
because such a dipole moment would change its orientation on
rotation about the axis. It is, however, possible for there to be a
dipole parallel to the axis, because it would not be affected by
the rotation. For example, in H2O the dipole lies in the plane
of the molecule, pointing along the bisector of the HOH bond,
which is the direction of the C2 axis. Figure 10A.13 The operations i and S2 are equivalent in the sense
Similarly, if a molecule possesses a mirror plane there can be that they achieve exactly the same outcome when applied to a
no dipole moment perpendicular to this plane, because reflec- point in the object.
tion in the plane would reverse its direction. A molecule that
possesses a centre of symmetry cannot have a dipole moment in (Fig. 10A.13). Furthermore, a mirror plane is the same as S1
any direction because the inversion operation would reverse it. (rotation through 360° followed by reflection). Therefore mol-
These considerations lead to the conclusion that ecules possessing a mirror plane or a centre of inversion effec-
Only molecules belonging to the groups Cn, Cnv, and Cs tively possess an improper rotation axis and so, by the above
may have a permanent electric dipole moment. rule, are achiral.

For Cn and Cnv, the dipole moment must lie along the principal
axis.
Brief illustration 10A.8
Brief illustration 10A.7 • The amino acid alanine (16) does not possess a centre of
inversion nor does it have any mirror planes: it is therefore
• Ozone, O3, which has an angular structure, belongs to the
chiral.
group C2v and is polar.
• Carbon dioxide, CO2, which is linear and belongs to the
group D∞h, is not polar. COOH
• Tetraphenylmethane (14) belongs to the point group S4
and so is not polar. H

NH2
10A.3(b) Chirality CH3

A chiral molecule (from the Greek word for ‘hand’) is a mol- 16 L-Alanine, NH2CH(CH3)COOH
ecule that cannot be superimposed on its mirror image. An
• In contrast, glycine (17) has a mirror plane and so is achiral.
achiral molecule is a molecule that can be superimposed on
its mirror image. Chiral molecules are optically active in the
sense that they rotate the plane of polarized light. A chiral mol-
ecule and its mirror-image partner constitute an enantiomeric
pair (from the Greek word for ‘both’) of isomers and rotate the COOH
H
plane of polarization by equal amounts but opposite directions.
H
A molecule may be chiral, and therefore optically active,
NH2
only if it does not possess an axis of improper rotation, Sn.
An Sn improper rotation axis may be present under a different
name, and be implied by other symmetry elements that are pre- 17 Glycine, NH2CH2COOH
sent. For example, molecules belonging to the groups Cnh pos- • Tetraphenylmethane (14) belongs to the point group S4; it
sess an Sn axis implicitly because they possess both Cn and σh, does not possess a centre of inversion nor does it possess
which are the two components of an improper rotation axis. any mirror planes, but it is still achiral since it possesses an
A centre of inversion, i, is in fact the same as S2 because the improper rotation axis (S4).
two corresponding operations achieve exactly the same result
406 10 Molecular symmetry

Exercises
E10A.7 Which of the following molecules may be polar? (i) pyridine, E10A.8 Can molecules belonging to the point groups D2h or C3h be chiral?
(ii) nitroethane, (iii) gas-phase BeH2 (linear), (iv) B2H6. Explain your answer.

Checklist of concepts
☐ 1. A symmetry operation is an action that leaves an object ☐ 4. To be polar, a molecule must belong to Cn, Cnv, or Cs
looking the same after it has been carried out. (and have no higher symmetry).
☐ 2. A symmetry element is a point, line, or plane with ☐ 5. A molecule will be chiral only if it does not possess an
respect to which a symmetry operation is performed. axis of improper rotation, Sn.
☐ 3. The notation for point groups commonly used for mol-
ecules and solids is summarized in Table 10A.1.

Checklist of symmetry operations and elements


Symmetry operation Symbol Symmetry element
n-Fold rotation Cn n-Fold rotation axis
Reflection σ Mirror plane
Inversion i Centre of inversion
n-Fold improper rotation Sn n-Fold improper rotation axis
Identity E Entire object
TOPIC 10B Group theory

4. The combination R(R′R″), the transformation (R′R″) fol-


➤ Why do you need to know this material? lowed by R, is equivalent to (RR′)R″, the transformation
Group theory expresses qualitative ideas about symmetry R″ followed by (RR′).
mathematically and can be applied systematically to a
wide variety of problems. The theory is also the origin of
the labelling of atomic and molecular orbitals that is used
Example 10B.1 Showing that the symmetry operations of
throughout chemistry.
a molecule form a group
➤ What is the key idea? The point group C2v consists of the elements {E ,C2 , v , v } and
Symmetry operations may be represented by the effect of correspond to the operations {E ,C2 , v , v }. Show that this set
matrices on a basis. of operations is a group in the mathematical sense.

➤ What do you need to know already? Collect your thoughts You need to show that combinations of
the operations match the criteria set out above. The operations
You need to know about the types of symmetry operation
are specified in Topic 10A, and illustrated in Figs. 10A.2 and
and element introduced in Topic 10A. This discussion uses
10A.3 for H2O, which belongs to this group.
matrix algebra, and especially matrix multiplication, as set
out in The chemist’s toolkit 9E.1 and in more detail in Part 1 The solution
of the Resource section. • Criterion 1 is fulfilled because the collection of symmetry
operations includes the identity E.
• Criterion 2 is fulfilled because for each member of this col-
The systematic discussion of symmetry is called group theory. lection of symmetry operations the inverse of the operation
Much of group theory is a summary of common sense about is the operation itself. For example, applying two succes-
the symmetries of objects. However, because group theory is sive twofold rotations brings the object back to its starting
systematic, its rules can be applied in a straightforward, me- position. The effect of these two rotations is to do nothing,
chanical way. In most cases the theory gives a simple, direct which is the effect of the identity operation. It follows that
C2C2 = E. However, the inverse is defined as having the
method for arriving at useful conclusions with the minimum of
property C2C2−1 = E. Comparing these two equations shows
calculation, and this is the aspect that is stressed here.
that C2 = C2−1. A similar argument shows that the inverse of
a reflection and the identity are the operations themselves.
10B.1 The elements of group theory • Criterion 3 is fulfilled, because in each case one operation
followed by another is the same as one of the four symme-
A group in mathematics is a collection of transformations that try operations. For instance, a twofold rotation C2 followed
by the reflection σv is the same as the single reflection  v
satisfy four criteria. If the transformations are written as R, R′, …
(Fig. 10B.1); thus,  vC2   v . A ‘group multiplication table’
(which might be the reflections, rotations, and so on intro-
can be constructed in a similar way for all possible products
duced in Topic 10A), then they form a group if:
of symmetry operations RR′; as required, each product is
1. One of the transformations is the identity (that is, ‘do equivalent to another symmetry operation.
nothing’).
2. For every transformation R, the inverse transformation
R−1 is included in the collection so that the combination R↓ R′→ E C2 σv  v
RR−1 (the transformation R−1 followed by R) is equivalent
E E C2 σv  v
to the identity.
C2 C2 E  v σv
3. The combination RR′ (the transformation R′ followed by
σv σv  v E C2
R) is equivalent to a single member of the collection of
 v  v σv C2 E
transformations.
408 10 Molecular symmetry

C2 σv σv
2 1 C3+ 1
σv σv σv σv

C3+ C3– 2 3

z
x y
3 4 4 C3–
(a) (b)
σv(xz) σv (yz)
Figure 10B.3 (a) The sequence of operations  v1C 3 v when
applied to the point 1 takes it through the sequence 1 → 2 →
Figure 10B.1 A twofold rotation C2 followed by the reflection σv 3 → 4 ( v1 is the operation σv). The single operation C 3− (dotted
gives the same result as the reflection  v . curve) takes point 1 → 4, so C 3+ and C 3− are in the same class. (b) The
sequence of operations (C 3 )1 v C 3 takes point 1 → 4, ((C 3 )1 is the
• Criterion 4 is fulfilled, as it is immaterial how the opera- operation C 3− ). The same transformation can be achieved with the
tions are grouped together. Thus ( v v )C2  C2C2  E and single operation  v (dotted line), so σv and  v are in the same class.
 v ( v C2 )   v v  E , and likewise for all other combinations.
Self-test 10B.1 Confirm that C2h, which has the elements
{E,C2,i,σh} and hence the corresponding operations {E,C2,i,σh}, can be rotated into another by a threefold rotation. The formal
is a group by constructing a group multiplication table. definition of a class is that two operations R and R′ belong to
Answer: Criteria are fulfilled the same class if there is a member S of the group such that

R  S 1 RS Membership of a class  (10B.1)

One potentially confusing point needs to be clarified at the where S−1 is the inverse of S.
outset. The entities that make up a group are its ‘elements’. For Figure 10B.3a shows how eqn 10B.1 can be used to confirm
applications in chemistry, these elements are almost always that C3+ and C3− belong to the same class in the group C3v . The
symmetry operations. However, as explained in Topic 10A, transformation of interest is  v1C3 v and the figure shows how
‘symmetry operations’ are distinct from ‘symmetry elements’, an arbitrary point initially at 1 behaves under this sequence of
the latter being the points, axes, and planes with respect to operations. The operation σv moves the point from 1 to 2, and
which the operations are carried out. A third use of the word then C3+ moves the point to 3. The inverse of a reflection is itself,
‘element’ is to denote the number lying in a particular location  v1   v , so the effect of  v1 is to move the point to 4. From the
in a matrix. Be very careful to distinguish element (of a group), diagram it can be seen that point 4 can be reached by apply-
symmetry element, and matrix element. ing C3− to point 1. It is therefore shown that  v1C3 v  C3 , and
Symmetry operations fall into the same class if they are of hence that C3+ and C3− do indeed belong in the same class.
the same type (for example, rotations) and can be transformed
into one another by a symmetry operation of the group. The
two threefold rotations in C3v belong to the same class because
Brief illustration 10B.1
one can be converted into the other by a reflection (Fig. 10B.2);
the three reflections all belong to the same class because each To show that σv and  v are in the same class in the group
C3v, consider the transformation (C3 )1 vC3 . Because C3− is
the inverse of C3+ , this transformation is the same as C3 vC3 .
The effect of this sequence of operations on an arbitrary point
C3+ C3– 1 is shown in Fig. 10B.3b. The final position, 4, can also be
reached from 1 by applying the operation  v , thus showing that
σv
σv C3 vC3   v and hence that σv and  v are in the same class.

σv

Exercises
Figure 10B.2 Symmetry operations in the same class are related
to one another by the symmetry operations of the group. Thus, E10B.1 Show that all three C2 operations in the group D3h belong to the same
the three mirror planes shown here are related by threefold class.
rotations, and the two rotations shown here are related by E10B.2 In the point group D3h, determine the inverse of each of the following
reflection in σv. operations: (i) E, (ii) σv, (iii) C3, (iv) S3.
10B Group theory 409

10B.2 Matrix representations the row vector (p x p y p z p A pB ). Note that the matrix D appears
to the right of the basis functions on which it acts.
Group theory takes on great power when the notional ideas The same technique can be used to find matrices that repro-
presented so far are expressed in terms of collections of num- duce the other symmetry operations. For instance, C2 has the
bers in the form of matrices. For basic information about how effect ( p x  p y p z  pB  p A )  (p x p y p z p A pB ), and its repre-
to handle matrices see The chemist’s toolkit 9E.1 and a more ex- sentative is
tensive account in Part 1 of the Resource section.
 1 0 0 0 0
 
10B.2(a) Representatives of operations  0 1 0 0 0
D(C2 )   0 0 1 0 0  (10B.2b)
Consider the set of five p orbitals shown on the C2v SO2 mol-  
0 0 0 0 1 
ecule in Fig. 10B.4 and how they are affected by the reflection 0 0
 0 1 0 
operation σv. The corresponding symmetry element is the mir-
ror plane perpendicular to the plane of the molecule and pass-
The effect of  v (reflection in the plane of the molecule, the yz
ing through the S atom (with the labelling used here, σv is the xz
plane as labelled here) is (p x p y p z  p A  pB )  (p x p y p z p A pB ) ;
plane). The effect of this reflection is to leave px and pz unaffected,
the oxygen orbitals remain in the same places, but change sign.
to change the sign of py, and to exchange pA and pB. Its effect can
The representative of this operation is
be written (p x p y p z pB p A )  (p x p y p z p A pB ). This transfor-
mation can be expressed by using matrix multiplication:  1 0 0 0 0
 
D ( v )
0 1 0 0 0
    D( v )   0 0 1 0 0  (10B.2c)
1 0 0 0 0   
  0 0 0 1 0 
 0 1 0 0 0  0 0 0 0 1 

(p x  p y p z p B p A )  (p x p y p z p A p B )  0 0 1 0 0 
  The identity operation has no effect on the basis, so its repre-
0 0 0 0 1 
0 0 0 1 0  sentative is the 5 × 5 unit matrix:
 
  (p x p y p z p A pB )D( v ) (10B.2a) 1 0 0 0 0
 
0 1 0 0 0
The matrix D(σv) is called a representative of the operation D(E )   0 0 1 0 0  (10B.2d)
σv. Representatives take different forms according to the basis,  
the set of orbitals that has been adopted. In this case, the basis is 0 0 0 1 0
0 0 0 0 1 


10B.2(b) The representation of a group


+
The set of matrices that represents all the operations of the
S – – +
pz + p group is called a matrix representation, Γ (uppercase gamma),
+ – B
+ y of the group in the basis that has been chosen. In the current
– –
– + – + px example, there are five members of the basis and the represen-
A –
– tation is five-dimensional in the sense that the matrices are all
– B+
– pB 5 × 5 arrays. The matrices of a representation multiply together
+ A
– + in the same way as the operations they represent. Thus, if for
pA any two operations R and R′, RR′ = R″, then D(R)D(R′) = D(R″)
for a given basis.
Figure 10B.4 The five p orbitals (three on the sulfur and one
on each oxygen) that are used to illustrate the construction of
a matrix representation in a C2v molecule (SO2). The  v plane Brief illustration 10B.2
is the plane of the molecule (the yz plane), and the σv plane is
perpendicular to the plane of the molecule, passing through the S In the group C2v, a twofold rotation followed by a reflection
atom (the xz plane). in a mirror plane is equivalent to a reflection in the second
410 10 Molecular symmetry

mirror plane: specifically,  v C2   v . Multiplying out the repre- These representations will be called Γ(1), Γ(2), and Γ(3), respec-
sentatives specified in eqn 10B.2 gives tively. The remaining two functions (p A pB ) are a basis for a
two-dimensional representation denoted Γ′:
 1 0 0 0 0   1 0 0 0 0
   1 0  0 1 
0 1 0 0 0   0 1 0 0 0 D( E )    D(C2 )   
D( v )D(C2 )   0 0 1 0 0  0 0 1 0 0 0 1  1 0 
   0 1  1 0 
0 0 0 1 0   0 0 0 0 1  D( v )    D( v )   
0
 0 0 0 1   0 0 0 1 0  1 0  0 1 
 1 0 0 0 0  The original five-dimensional representation has been reduced
  to the ‘direct sum’ of three one-dimensional representations
 0 1 0 0 0 
‘spanned’ by each of the p orbitals on S, and a two-dimensional
 0 0 1 0 0   D( v )
  representation spanned by (p A pB ). The reduction is repre-
 0 0 0 0 1  sented symbolically by writing1
 0 0 0 1 0 
 
  (1)  (2)  (3)   Direct sum  (10B.4)
As expected, this multiplication reproduces the same result as
the group multiplication table. The same is true for multiplica- The representations Γ(1), Γ(2), and Γ(3) cannot be reduced any fur-
tion of any two representatives, so the four matrices form a ther, and each one is called an irreducible representation of the
representation of the group. group (an ‘irrep’).
For this basis and in this group the two-dimensional rep-
resentation Γ′ is reducible to a sum of two one-dimensional
representations. This reduction can be demonstrated by form-
The discovery of a matrix representation of the group means ing linear combinations of the basis functions pA and pB, spe-
that a link has been established between symbolic manipula- cifically the linear combinations p1 = pA + pB and p2 = pA − pB
tions of operations and algebraic manipulations of numbers. ­(illustrated in Fig. 10B.5).
This link is the basis of the power of group theory in chemistry. The effect of the operation σv is to exchange pA and pB:
(pB p A ) ← (p A pB ), therefore the effect on the combination pA + pB
is (pB  p A )  (p A  pB ). This is the same as (p1 ) ← (p1 ).
10B.2(c) Irreducible representations Similarly, (pB  p A )  (p A  pB ), corresponding to (p2 )  (p2 ).
Inspection of the representatives found above shows that they It follows from these results, and similar ones for the other op-
are all of block-diagonal form: erations, that the representation in the basis (p1 p2 ) is

 1 0  1 0 
 0 0 0 0 D( E )    D(C2 )   
   0 1 0 1
0  0 0 0
1 0   1 0 
D 0 0  0 0 Block -diagonal form (10B.3) D( v )    D( v )   
   0 1   0 1 
0 0 0  
0 0 0   

– B +
B

A + A –
The block-diagonal form of the representatives implies that
the symmetry operations of C2v never mix px, py, and pz to- –
+

+
gether, nor do they mix these three orbitals with pA and pB, but
pA and pB are mixed together by the operations of the group.
Consequently, the basis can be cut into four parts: three for the
Figure 10B.5 Two symmetry-adapted linear combinations of the
individual p orbitals on S and the fourth for the two oxygen or-
oxygen basis orbitals shown in Fig. 10B.4: on the left p1 = pA + pB,
bitals (p A pB ). The representations in the three one-dimensional
on the right p2 = pA − pB. The two combinations each span a
bases are one-dimensional irreducible representation, and their symmetry
species are different.
For p x : D(E )  1 D(C2 )  1 D( v )  1 D( v )  1
1
The symbol ⨁ is sometimes used to denote a direct sum to distinguish
For p y : D(E )  1 D(C2 )  1 D( v )  1 D( v )  1 it from an ordinary sum, in which case eqn 10B.4 would be written Γ = Γ(1) ⨁
For p z : D(E )  1 D(C2 )  1 D( v )  1 D( v )  1 Γ(2) ⨁ Γ(3) ⨁ Γ′.
10B Group theory 411

The new representatives are all in block-diagonal form, in R E C2 σv  v


 0  χ(R) for Γ (1)
1 −1 1 −1
this case in the form  , and the two combinations are
 0  χ(R) for Γ(1) 1 −1 1 −1
not mixed with each other by any operation of the group. The χ(R) for Γ (2)
1 −1 −1 1
representation Γ′ has therefore been reduced to the sum of
χ(R) for Γ(3) 1 1 1 1
two one-dimensional representations. Thus, p1 spans the one-­ (4)
χ(R) for Γ 1 1 −1 −1
dimensional representation
Sum for Γ: 5 −1 1 −1

D(E )  1 D(C2 )  1 D( v )  1 D( v )  1


At this point, four irreducible representations of the group
which is the same as the representation Γ(1) spanned by px. The C2v have been found. Are these the only irreducible representa-
combination p2 spans tions of the group C2v? There are in fact no more irreducible
representations in this group, a fact that can be deduced from a
D(E )  1 D(C2 )  1 D( v )  1 D( v )  1 surprising theorem of group theory, which states that

which is a new one-dimensional representation and denoted Number of irreducible representations = number of classes
Γ(4). At this stage the original representation has been reduced Numb
ber of irreducible representations  (10B.5)
into five one-dimensional representations as follows, with one
representation, Γ(1) , appearing twice: In C2v there are four classes of operations (the four columns in
the table), so there must be four irreducible representations.
  2(1)  (2)  (3)  ( 4 ) The ones already found are the only ones for this group.
The order of the group, h, is the total number of symmetry
operations. Another powerful result from group theory relates
10B.2(d) Characters the order to the sum of the squares of the dimensions, di, of all
The character, χ (chi), of an operation in a particular ma- the irreducible representations Γ(i)
trix representation is the sum of the diagonal elements of the
representative of that operation. Thus, in the original basis d
irreducible
2
i h
Dimensionality and order (10B.6)
(p x p y p z p A pB ) the characters of the representatives are representations, i

This result applies to all groups other than the pure rotation
R E C2
groups Cn with n > 2. In the group C2v there are four irreducible
representations, all of which are one-dimensional (di = 1) so
D(R) 1 0 0 0 0  1 0 0 0 0
   
0 1 0 0 0  0 1 0 0 0 d 2
i  12  12  12  12  4
0 0 1 0 0 0 0 1 0 0 irreducible
representations, i
   
0 0 0 1 0 0 0 0 0 1 
0 0 0 0 1  0 0 0 1 0  There are indeed four symmetry operations of the group, h = 4,
 
in accord with the theorem.
χ(R) 5 −1

Brief illustration 10B.3


R σv  v
D(R) 1 0 0 0 0  1 0 0 0 0 The operations of the group C3v fall into three classes {E,2C3,3σv},
    so there are three irreducible representations. The order of the
0 1 0 0 0 0 1 0 0 0
0 0 1 0 0 0 0 1 0 0 group is h = 1 + 2 + 3 = 6. Suppose that it is already known that
   
0 0 0 0 1 0 0 0 1 0  two of the irreducible representations are one-dimensional.
0
 0 0 1 0  0
 0 0 0 1  Equation 10B.6 is then used to find the dimension d3 of the
χ(R) 1 −1 remaining irreducible representation: 12  12  d32  6. It follows
that d3 = 2, meaning that the third irreducible representation is
two-dimensional.
The characters of one-dimensional representatives are just
the representatives themselves. For each operation, the sum
of the characters of the reduced representations is the same
as the character of the original representation (allowing for Exercises
the appearance of Γ(1) twice in the reduction Γ = 2Γ(1) + Γ(2) + E10B.3 Use as a basis the 2pz orbitals on each atom in BF3 to find the representative
Γ(3) + Γ(4)): of the operation σh. Take z as perpendicular to the molecular plane.
412 10 Molecular symmetry

E10B.4 Use the matrix representatives of the operations σh and C3 in a basis of Table 10B.2 The C3v character table*
2pz orbitals on each atom in BF3 to find the operation and its representative
resulting from σhC3. Take z as perpendicular to the molecular plane. C3v, 3m E 2C3 3σv h=6
A1 1 1 1 z z2, x2 + y2
A2 1 1 −1
E 2 −1 0 (x, y) (xy, x2 − y2), (yz, zx)
10B.3 Character tables
* More character tables are given in the Resource section.

Tables showing all the characters of the operations of a group


are called character tables and from now on they move to the Character tables, and some of the data contained in them, are
centre of the discussion. The columns of a character table are constructed on the assumption that the axis system is arranged
labelled with the symmetry operations of the group. Although in a particular way. This arrangement is specified in the char-
the notation Γ(i) is used to label general irreducible represen- acter table when there is ambiguity. There is ambiguity in C2v
tations, in chemical applications and for displaying character (and certain other groups), and so a more detailed specification
tables it is more common to distinguish different irreducible of the symmetry operations is then necessary. The principal
representations by the use of the labels A, B, E, and T to denote axis (a unique Cn axis with the greatest value of n), is taken to
the symmetry species of each representation: be the z-direction. If the molecule is planar, it is taken to lie in
the yz-plane (referring to Fig. 10B.6). Then  v is a reflection in
A: one-dimensional representation, character +1 under the the yz-plane and henceforth will be denoted  v ( yz ), and σ v is
principal rotation a reflection in the xz-plane, and henceforth is denoted σ v (xz ).
B: one-dimensional representation, character −1 under the The irreducible representations are mutually orthogonal
principal rotation in the sense that if the set of characters is regarded as form-
E: two-dimensional irreducible representation ing a row vector, the scalar product (as defined in The chemist’s
toolkit 8C.1) of the vectors corresponding to different irreduci-
T: three-dimensional irreducible representation
ble representations is zero. In other words the vectors are mutu-
Subscripts are used to distinguish the irreducible representa- ally perpendicular.2 Formally, this orthogonality is expressed as
tions if there is more than one of the same type: A1 is reserved
for the representation with character 1 for all operations (called 1 0 for i  j

(i ) ( j)
N (C )  (C )  (C )   (10B.7)
the totally symmetric irreducible representation); A2 has 1 for h C 1 for i  j 
the principal rotation but −1 for reflections. There appears to be Orthonormalitty of irreducible representations
no systematic way of attaching subscripts to B symmetry spe-
cies, so care must be used when referring to character tables where the sum is over the classes of the group, N(C) is the num-
from different sources. ber of operations in class C, and h is the number of operations
Table 10B.1 shows the character table for the group C2v, with in the group (its order). The division by h results in the sca-
its four symmetry species (irreducible representations) and its lar product of an irreducible representation with itself being 1,
four columns of symmetry operations. Table 10B.2 shows the which means that the vectors, if divided by h1/2, are normalized
table for the group C3v. The columns are headed E, 2C3, and to 1. Vectors that are both orthogonal and normalized are said
3σv: the numbers multiplying each operation are the num- to be ‘orthonormal’.
ber of members of each class. As inferred in Brief illustration C2
10B.3, there are three symmetry species, with one of them two-
dimensional (E). Operations in the same class have the same
character, which is why all three σv operations can be grouped

together in one column.
z +

y
x
Table 10B.1 The C2v character table* σv (yz)

C2v, 2mm E C2 σ v (xz )  v ( yz ) h = 4 σv(xz)


2 2 2
A1 1 1 1 1 z z,y,x
Figure 10B.6 A px orbital on the central atom of a C2v molecule
A2 1 1 −1 −1 xy
and the symmetry elements of the group.
B1 1 −1 1 −1 x zx
2
B2 1 −1 −1 1 y yz This result is a consequence of the ‘great orthogonality theorem’ of group
theory; see our Molecular quantum mechanics (2011). In this Topic, the charac-
* More character tables are given in the Resource section. ters are taken to be real.
10B Group theory 413

Brief illustration 10B.4

The point group C3v has operations {E,2C3,3σv} which fall into +
three classes; the order h is 6. For the two irreducible represen- +
tations with labels A2 (with characters {1,1,−1}) and E (with – –1 +1
characters {2,−1,0}), eqn 10B.7 is
1
{1  1  2  2  1  (1)  3  (1)  0}  0 ++
6
– +

If the two irreducible representations are both E, the sum in – +
eqn 10B.7 is – –
1
6
{1  2  2  2  (1)  (1)  3  0  0}  1 Figure 10B.7 The two orbitals shown here have different
properties under reflection through the mirror plane: one changes
The sum is also 1 if both irreducible representations are A2:
sign (character −1), the other does not (character +1).
1
{1  1  1  2  1  1  3  (1)  (1)}  1
6 are the sums of the characters for the behaviour of the individ-
ual orbitals in the basis. Thus, if one member of a pair remains
unchanged under a symmetry operation but the other changes
The symmetry species of
10B.3(a) sign (Fig. 10B.7), then the entry is reported as χ = 1 − 1 = 0.
atomic orbitals The symmetry species of s, p, and d orbitals on a central atom
is easily read from the character table. The wavefunction of the pz
The characters of a one-dimensional irreducible representation orbital is of the form zf(r) and so it transforms in the same way as
indicate the behaviour of an orbital under the operation of the the cartesian function z; likewise the px and py orbitals transform
group to which each character corresponds. A character of 1 as the cartesian functions y and z respectively. In the character
indicates that the orbital is unchanged, and a character of −1 table these cartesian functions are indicated on the right-hand
indicates that it changes sign. It follows that the symmetry label side, in the row corresponding to their symmetry species.
of the orbital can be identified by comparing the changes that For example, the character table for C3v indicates that z has the
occur to an orbital under each operation, and then comparing symmetry species A1, so the pz orbital has this same symmetry
the resulting 1 or −1 with the entries in a row of the character species. The cartesian functions x and y are jointly of E symme-
table for the relevant point group. By convention, the orbitals try. In technical terms, it is said that x and y, and hence px and py ,
are labelled with the lower case equivalent of the symmetry jointly span an irreducible representation of symmetry species E.
species label (so an orbital of symmetry species A1 is called an An s orbital on the central atom always spans the totally symmet-
a1 orbital). ric irreducible representation of a group as it is unchanged under
all symmetry operations; in C3v it has symmetry species A1.
The five d orbitals of a shell are represented by the appropri-
Brief illustration 10B.5
ate cartesian functions, for example xy for the dxy orbital. These
Consider an H2O molecule, point group C2v, shown in cartesian functions are also listed on the right of the character
Fig. 10B.6. The effect of C2 on the oxygen 2px orbital is to table. It can be seen at a glance that in C3v dxy and d x2 − y2 on a
cause it to change sign, so the character is −1;  v ( yz ) has the central atom jointly span E.
same effect and so has character −1. In contrast, σv(xz) leaves
the orbital unaffected and so has character 1, and of course the The symmetry species of linear
10B.3(b)
same is true of the identity operation. The characters of the
combinations of orbitals
operations {E ,C2 , v , v ) are therefore {1,−1,1,−1}. Reference
to the C2v character table (Table 10B.1) shows that {1,−1,1,−1} The same technique may be applied to identify the symmetry
are the characters for the symmetry species B1; the orbital is species of linear combinations of orbitals, such as the combina-
therefore labelled b1. A similar procedure gives the characters tion ψ1 = sA + sB + sC of the three H1s orbitals in the C3v mol-
for oxygen 2py as {1,−1,−1,1}, which corresponds to B2: the ecule NH3 (Fig. 10B.8). This combination remains unchanged
orbital is labelled b2, therefore. Both the oxygen 2pz and 2s under a C3 rotation and under any of the three vertical reflec-
are a1. tions of the group, so its characters are

 (E )  1  (C3 )  1  ( v )  1

The characters for irreducible representations of dimension- Comparison with the C3v character table shows that ψ1 is of
ality greater than 1 (for example the E and T symmetry species) symmetry species A1, and therefore has the label a1.
414 10 Molecular symmetry

xz- and yz-planes; choose the x- and y-axes to pass through the
sA corners of the square.

A B

D C

1
sC sB Answer: B1g

Figure 10B.8 The three H1s orbitals used to construct symmetry- 10B.3(c) Character tables and degeneracy
adapted linear combinations in a C3v molecule such as NH3.
In Topic 7D it is pointed out that degeneracy, which is when
different wavefunctions have the same energy, is always related
to symmetry. An energy level is degenerate if the wavefunc-
Example 10B.2 Identifying the symmetry species of tions corresponding to that energy can be transformed into
orbitals each other by a symmetry operation (such as rotating a square
well through 90°). Clearly, group theory should have a role in
Identify the symmetry species of the orbital ψ = ψA − ψB in a the identification of degeneracy.
C2v NO2 molecule, where ψA is a 2px orbital on one O atom and
A geometrically square well belongs to the group C4 (Fig.
ψB is a 2px orbital on the other O atom. As in Fig. 10B.6 take x
10B.10 and Table 10B.3), with the C4 rotations (through 90°)
to be perpendicular to the plane of the molecule.
converting x into y and vice versa.3 As explained in Topic 7D,
Collect your thoughts The negative sign in ψ indicates that the two wavefunctions ψ1,2 = (2/L)sin(πx/L)sin(2πy/L) and
the sign of ψB is opposite to that of ψA. You need to consider ψ2,1 = (2/L)sin(2πx/L)sin(πy/L) both correspond to the energy
how the combination changes under each operation of the 5h2/8mL2, so that level is doubly degenerate. Under the opera-
group, and then write the character as 1, −1, or 0 as specified tions of the group, these two functions transform as follows:
above. Then compare the resulting characters with each row in
the character table for the point group, and hence identify the E : ( 1,2  2,1 )  ( 1,2  2,1 ) C4 : ( 1,2  2,1 )  ( 2,1  1,2 )
symmetry species. C4 : ( 1,2  2,1 )  ( 2,1  1,2 ) C2 : ( 1,2  2,1 )  ( 1,2   2,1 )
The solution The combination is shown in Fig. 10B.9. Under
The corresponding matrix representatives are
C2, ψ changes into itself, implying a character of 1. Under the
reflections σ v (xz ) and  v ( yz ) the wavefunction changes sign
 1 0 0 1
overall, ψ → −ψ, implying a character of −1. The characters D( E )    D(C4 )  


are therefore  0 1  1 0
 0 1   1 0 
 (E )  1  (C2 )  1  ( v (xz ))  1  ( v ( yz ))  1 D(C4 )    D(C2 )   
 1 0  0 1 
These values match the characters of the A2 symmetry species,
so ψ is labelled a2.
C4

Self-test 10B.2 Consider PtCl 2−
4 , in which the Cl ligands form C2
a square planar array and the ion belongs to the point group L
D4h (1). Identify the symmetry species of the combination of Cl
s orbitals ψA − ψB + ψC − ψD. Note that in this group the C2 axes
coincide with the x and y axes, and σv planes coincide with the
y
0
z
y N x
+
x –
O L

O +
Figure 10B.10 A geometrically square well can be treated as
belonging to the group C4 (with elements {E,2C4,C2}).

Figure 10B.9 One symmetry-adapted linear combination of 3


More complicated groups could be used, such as C4v or D4h, but C4 cap-
O2px orbitals in the C2v NO2 molecule. tures the symmetry sufficiently.
10B Group theory 415

Table 10B.3 The C4 character table* is always equal to the dimensionality of the representation, the
maximum degeneracy can be identified by noting the maxi-
C4, 4 E 2C4 C2 h=4
mum value of χ(E) in the relevant character table.
A 1 1 1 z z2, x2 + y2
B 1 −1 1 xy, x2 − y2
Brief illustration 10B.6
E 2 0 −2 (x, y) (yz, zx)

* More character tables are given in the Resource section. • A trigonal planar molecule such as BF3 cannot have triply
degenerate orbitals because its point group is D3h and the
character table for this group (in the Resource section) does
and their characters are not have a T symmetry species.
• A methane molecule belongs to the tetrahedral point
 
 (E )  2  (C )  0  (C )  0  (C2 )  2
4 4
group Td and because that group has irreducible represen-
tations of T symmetry, it can have triply degenerate orbit-
A glance at the character table in Table 10B.3 (noting that the als. The same is true of tetrahedral P4, which, with just four
rotations C4+ and C4− belong to the same class and appear in atoms, is the simplest kind of molecule with triply degen-
the column labelled 2C4) shows that the basis spans the irre- erate orbitals.
ducible representation of symmetry species E. The same is true • A buckminsterfullerene molecule, C60, belongs to the ico-
of all the doubly degenerate energy levels. There are no triply sahedral point group (Ih) and its character table (in the Re-
degenerate (or higher) energy levels in the system. Notice too source section) shows that the maximum dimensionality of
that in the group C4 there are no irreducible representations of its irreducible representations is 5, so it can have five-fold
dimension 3 or higher. These two observations illustrate the degenerate orbitals.
general principle that:
The highest dimensionality of irreducible representation
in a group is the maximum degree of degeneracy in the
group.
Exercises
E10B.5 For the point group C2h, confirm that all the irreducible representations
Thus, if there is an E irreducible representation in a group, 2 are orthonormal according to the property defined in eqn 10B.7. The
is the highest degree of degeneracy; if there is a T irreducible character table will be found in the Resource Section.
representation in a group, then 3 is the highest degree of degen- E10B.6 By inspection of the character table for D3h, state the symmetry species
eracy. Some groups have irreducible representations of higher of the 3p and 3d orbitals located on the central Al atom in AlF3.
dimension, and therefore allow higher degrees of degeneracy. E10B.7 What is the maximum degeneracy of the wavefunctions of a particle
Furthermore, because the character of the identity operation confined to the interior of an octahedral hole in a crystal?

Checklist of concepts
☐ 1. A group is a collection of transformations that satisfy ☐ 5. A matrix representation is the collection of matrix rep-
the four criteria set out at the start of the Topic. resentatives for the operations in the group.
☐ 2. The order of a group is the number of its symmetry ☐ 6. A character table consists of entries showing the char-
operations. acters of all the irreducible representations of a group.
☐ 3. A matrix representative is a matrix that represents the ☐ 7. A symmetry species is a label for an irreducible repre-
effect of an operation on a basis. sentation of a group.
☐ 4. The character is the sum of the diagonal elements of a ☐ 8. The highest dimensionality of irreducible representation
matrix representative of an operation. in a group is the maximum degree of degeneracy in the
group.
416 10 Molecular symmetry

Checklist of equations
Property Equation Comment Equation number
Class membership R′ = S−1RS All elements members of the group 10B.1
Number of irreducible representations Number of irreducible representations = 10B.5
number of classes
Dimensionality and order d 2
i h For groups other than pure rotation 10B.6
irreps i groups with n > 2
1 10B.7
h
(i ) ( j)
Orthonormality of irreducible representations N (C )  (C )  (C ) Sum over classes
C

0 for i  j

1 for i  j
TOPIC 10C Applications of symmetry

➤ Why do you need to know this material?


Group theory is a key tool for constructing molecular orbit- + + –
als and formulating spectroscopic selection rules.
Area of integration
➤ What is the key idea? +

An integral can be non-zero only if the integrand is invari-
ant under the symmetry operations of a molecule.

➤ What do you need to know already? (a) (b)


This Topic develops the material in Topic 10A, where the Figure 10C.1 (a) Only if the integrand is unchanged under each
classification of molecules on the basis of their symmetry symmetry operation of the group (here C4) can its integral over
elements is introduced, and draws heavily on the proper- the region indicated be non-zero. (b) If the integrand changes sign
ties of characters and character tables described in Topic under any operation, its integral is necessarily zero.
10B. You need to be aware that many quantum-mechanical
properties, including transition dipole moments (Topic 8C),
depend on integrals involving products of wavefunctions.
The integral may be non-zero only if the integrand is invariant
under the operation (Fig. 10C.1a).
The same argument applies to each of the symmetry opera-
tions of the area of integration. In addition, it is possible that
the integrand can be expressed as a sum of contributions, one
Group theory shows its power when brought to bear on a vari- of which is invariant under these symmetry operations. In that
ety of problems in chemistry, among them the construction of case, the integral may be non-zero.
molecular orbitals and the formulation of spectroscopic selec- In summary the integral may be non-zero only if the inte-
tion rules. grand, or a contribution to it, is invariant under each symmetry
operation of the group that reflects the shape of the area of in-
tegration or, in three dimensions, the volume of integration. In
group-theoretical terms:
10C.1 Vanishing integrals An integral over a region of space can be non-zero only
if the integrand (or a contribution to it) spans the totally
Any integral, I, of a function f(x) over a symmetric range
symmetric irreducible representation of the point group of
around x = 0 is zero if the function is antisymmetric (odd),
the region.
meaning that f(−x) = −f(x):
The totally symmetric irreducible representation has all charac-
a
I f ( x ) dx  0 if f ( x )   f (x ) ters equal to 1, and is typically the symmetry species denoted A1.
a

For an analogous integral in two dimensions over a symmet-


Brief illustration 10C.1
rical range in x and y the area of integration is a square (Fig.
10C.1). The square has various symmetry operations including To decide whether the integral of the function f = xy may be
a C4 rotation perpendicular to the plane. The integral of the in- non-zero when evaluated over a region the shape of an equi-
tegrand f(x,y) has contributions from regions that are related by lateral triangle centred on the origin (Fig. 10C.2), recognize
this C4 rotation and if f(x,y) changes sign under this operation, that the triangle belongs to the point group C3. Reference to
the contribution of the first region is cancelled by that from the the character table of the group shows that xy is a member of a
symmetry-related region and the integral is zero (Fig. 10C.1b). basis that spans the irreducible representation E. Therefore, its
418 10 Molecular symmetry

Brief illustration 10C.2

Suppose that in the point group C2v f1 has the symmetry spe-
– y + cies A2, and f2 has the symmetry species B1. From the char-
acter table the characters for these species are 1,1,−1,−1 and
x
1,−1,1,−1, respectively. The direct product of these two species
is found by setting up the following table

E C2 σv(xz)  v ( yz )
+ – A2 1 1 −1 −1
B1 1 −1 1 −1

Figure 10C.2 The integral of the function f = xy over an product 1 −1 −1 1


equilateral triangle centred on the origin (shaded) is zero. In
Now recognize that the characters in the final row are those of
this case, the result is obvious by inspection, but group theory
can be used to establish similar results in less obvious cases.
the symmetry species B2. It follows that the symmetry species
of the product f1 f2 is B2. Because the direct product does not
contain the totally symmetric irreducible representation (A1),
the integral of f1 f2 over all space must be zero.

integral must be zero, because the integrand has no component


that spans the totally symmetric irreducible representation (A
The direct product of two irreducible representations Γ(i ) and
in this point group).
Γ is written (i )  ( j ) or sometimes (i )  ( j ). Direct products
( j)

have some simplifying features.

10C.1(a) Integrals of the product of functions • The direct product of the totally symmetric irreducible
representation with any other representation is the latter
Suppose the integral of interest is of a product of two functions, irreducible representation itself: A1 × Γ(i) = Γ(i).
f1 and f2, taken over all space and over all relevant variables
(represented, as is usual in quantum mechanics, by dτ): All the characters of A1 are 1, so multiplication by them leaves
the characters of Γ(i) unchanged. It follows that if one of the
I   f1 f 2 d   (10C.1) functions in eqn 10C.1 transforms as A1, then the integral will
vanish if the other function does not also transform as A1.
For example, f1 and f2 might be atomic orbitals on different
• The direct product of two irreducible representations is A1
atoms, in which case I would be their overlap integral. The
only if the two irreducible representations are identical:
implication of such an integral being zero is that a molecular
Γ(i) × Γ(j) contains A1 only if i = j.
orbital does not result from the overlap of these two orbitals.
It follows from the general point made above that the integral For one-dimensional irreducible representations the characters
may be non-zero only if the integrand itself, the product f1 f2, is are either 1 or −1, and the character 1 is obtained only if the
unchanged by any symmetry operation of the molecular point characters of Γ(i) and Γ(j) are the same (both 1 or both −1). For
group and so spans the totally symmetric irreducible represen- example, in C2v, A1 × A1, A2 × A2, B1 × B1, and B2 × B2, but no
tation (typically the symmetry species with label A1). other combination, all give A1. This requirement is also true for
To decide whether the product f1 f2 does indeed span the to- higher-dimensional representations, as is demonstrated at the
tally symmetric irreducible representation, it is necessary to end of the following section.
form the direct product of the symmetry species spanned by f1 It follows that if f1 and f2 transform as different symmetry
and f2 separately. The procedure is as follows: species, then the product cannot transform as the totally sym-
metric irreducible representation. In this case the integral of
• Write down a table with columns headed by the symmetry
f1 f2 is necessarily zero. If, on the other hand, both functions
operations, R, of the group.
transform as the same symmetry species, then the product
• In the first row write down the characters of the symmetry transforms as the totally symmetric irreducible representation
species spanned by f1; in the second row write down the (and possibly has contributions from other symmetry species
characters of the symmetry species spanned by f2. too). Now the integral is not necessarily zero.
• Multiply the numbers in the two rows together, column by An important point is that group theory is specific about
column. The resulting set of numbers are the characters of when an integral must be zero, but integrals that it allows to be
the representation spanned by f1 f2. non-zero may be zero for reasons unrelated to symmetry. For
10C Applications of symmetry 419

example, in ammonia the 1s orbital on N and the combination 10C.1(b) Decomposition of a representation
(s1+s2+s3), where s1 is a 1s orbital on H1 and so on, both trans-
form as A1. Group theory therefore predicts that the orbitals In some cases, it turns out that the direct product is a sum of ir-
have non-zero overlap, but in practice the 1s orbital on N is so reducible representations (symmetry species), not just a single
contracted that its overlap with the H1s orbitals is negligible at species. For instance, in C3v the characters of the direct product
the N—H distance found in this molecule. E × E are {4,1,0}, which can be decomposed as A1, A2, and E:
Integrals of the form
E 2C3 3σv

I   f1 f 2 f 3 d   A1 1 1 1
(10C.2)
A2 1 1 −1
are also common in quantum mechanics, and it is important to E 2 −1 0
know when they are necessarily zero. For example, they appear sum 4 1 0
in the calculation of transition dipole moments (Topic 8C). As
for integrals over two functions, for I to be non-zero, the prod- This decomposition is written symbolically E × E = A1 + A2 + E.1
uct f1 f2 f3 must span the totally symmetric irreducible representa- In simple cases the decomposition can be done by inspec-
tion or contain a component that spans that representation. To tion. Group theory, however, provides a systematic way of
test whether this is so, the characters of all three irreducible using the characters of the representation to find the irreduc-
representations are multiplied together in the same way as in ible representations of which it is composed. The formal recipe
the rules set out above. for finding the number of times, n(Γ(i ) ), that irreducible repre-
sentation Γ(i ) occurs is:2
Example 10C.1 Deciding if an integral must be zero 1 Decomposition

(i )
n((i ) )  N (C ) (  ) (C ) (C ) of a representatiion  (10C.3a)
Does the integral (d z 2 )x(d xy )d vanish in a C2v molecule, h C
where d z 2 and d xy are the wavefunctions of d orbitals indicated
by the subscripts? Note that the sum is over the classes of operations in the group.
(i )
In this expression h is the order of the group,  (  ) (C ) is the
Collect your thoughts Use the C2v character table to find the
character for operations in class C for the irreducible represen-
characters of the irreducible representations spanned by 3z2 − r2
tation Γ(i ), and χ(C) is the corresponding character of the repre-
(the form of the d z 2 orbital), x, and xy. Then set up a table to
sentation being decomposed. In the character table the number
work out the triple direct product and identify whether the
of operations in each class, N(C), is indicated in the header of
symmetry species it spans includes A1.
the columns.
The solution The C2v character table shows that the function xy, If all that is of interest is the occurrence of the totally sym-
and hence the orbital dxy, transforms as A2, that z2 transforms as metric irreducible representation in the decomposition the cal-
A1, and that x transforms as B1. The table is therefore culation is somewhat simpler. All the characters of the totally
symmetric irreducible representation (symmetry species A1)
E C2 σ v ( xz )  v ( yz )
are 1, so setting (i )  A1 and  ( A1 ) (C )  1 for all C in eqn 10C.3a
A2 1 1 −1 −1 f3 = dxy
gives
B1 1 −1 1 −1 f2 = x
A1 1 1 1 1 f1 = d z 2 1
1 −1 −1 1 product
n(A1 )  N (C) (C)
h C
Occurrence of A
1
 (10C.3b)

The characters in the bottom row are those of B2, not of A1.
Therefore, the integral is necessarily zero.
Brief illustration 10C.3
A quicker solution involves noting that A1 (for f1) has no
effect on the outcome of the triple direct product (by the first In the character table for C3v, the columns are headed E, 2C3,
feature mentioned above), and therefore, by the second feature, and 3σv, indicating that the numbers in each class are 1, 2, and
the symmetry species of the two functions f2 and f3 must be the 3, respectively and h = 1 + 2 + 3 = 6. To decide whether A1
same for their direct product to be A1; but in this example they
are not the same.
Self-test 10C.1 Does the integral (d xz )x(p z )d necessarily 1
As mentioned in Topic 10B, a direct sum is sometimes denoted ⨁. The
vanish in a C2v molecule? analogous symbol for a direct product is ⊗. The symbolic expression is then
written E ⊗ E = A1 ⨁ A2 ⨁ E.
2
Answer: No This result arises from the ‘great orthogonality theorem’: see our Molecular
quantum mechanics (2011). In this Topic, the characters are taken to be real.
420 10 Molecular symmetry

occurs in the representation with characters {4,1,0} in C3v use E10C.3 A set of basis functions is found to span a reducible representation
eqn 10C.3b to give of the group C4v with characters 5,1,1,3,1 (in the order of operations in the
character table in the Resource section). What irreducible representations does
1 it span?
n(A1 )  N (C) (C)
h C
 16 {1   (E )  2   (C3 )  3   ( v )}
 61 {1  4  2  1  3  0}  1
10C.2Applications to molecular
orbital theory
A1 therefore occurs once in the decomposition.
The rules outlined so far can be used to decide which atomic
orbitals may have non-zero overlap in a molecule. Group the-
It is asserted in the preceding section that the direct product ory also provides procedures for constructing linear combina-
of two irreducible representations is A1 only if the two irreduc- tions of atomic orbitals of a specified symmetry.
ible representations are identical. That this is so can now be
shown with the aid of eqn 10C.3b. 10C.2(a) Orbital overlap
The overlap integral, S, between orbitals ψ1 and ψ2 is
How is that done? 10C.1 Confirming the criterion for a
S   2 1d Overlap integral (10C.4)
direct product to contain the totally symmetric irreducible
representation It follows from the discussion of eqn 10C.1 that this integral
Start by considering the characters of the direct product can be non-zero only if the two orbitals span the same symme-
between irreducible representations Γ(i ) and Γ( j ). For irreduc- try species. In other words,
ible representation Γ(i ) the character of an operation in class C
(i ) ( j) Only orbitals of the same symmetry species may have non-
is  (  ) (C ), and likewise  (  ) (C ) for irreducible representa-
zero overlap (S ≠ 0) and hence go on to form bonding and
tion Γ( j ). It follows that the characters in the direct product are
(i ) ( j)
 (C )   (  ) (C ) (  ) (C ). The number of times that the totally antibonding combinations.
symmetric irreducible representation (A1) occurs in this direct- The selection of atomic orbitals with non-zero overlap is the
product representation is given by eqn 10C.3b as central and initial step in the construction of molecular orbitals
1 as LCAOs.

(i ) ( j)
n(A1 )  N (C ) (  ) (C ) (  ) (C )
h C
Irreducible representations are orthonormal in the sense that Example 10C.2 Identifying which orbitals can contribute
(eqn 10B.7)
to bonding
1 0 if i  j

(i ) ( j)
N (C ) (  ) (C ) (  ) (C )   The four H1s orbitals of methane span A1 + T2. With which
h C 1 if i  j of the C2s and C2p atomic orbitals can they overlap? What
It follows that additional overlap would be possible if d orbitals on the C atom
were also considered?
0 if i  j
n(A1 )   Collect your thoughts Refer to the Td character table (in the
1 if i  j Resource section) and look for s, p, and d orbitals spanning A1
In other words, the direct product of two irreducible represen- or T2. As explained in Topic 10B, the symmetry species can be
tations has a component that spans A1 only if the two irreduc- identified by looking for the appropriate cartesian functions
ible representations belong to the same symmetry species. This listed on the right of the table.
result is independent of the dimensionality of the irreducible
representations. The solution A C2s orbital spans A1 in the group Td, so it may
have non-zero overlap with the A1 combination of H1s orbitals.
From the table (x,y,z) jointly span T2, so the three C2p orbitals
together transform as T2; they may have non-zero overlap with
the T2 combination of H1s orbitals.
Exercises The combinations (xy,yz,zx) span T2, therefore the dxy, dyz,
E10C.1 Use symmetry properties to determine whether or not the integral and dzx orbitals do the same and so they may overlap with the
 p x zp z d is necessarily zero in a molecule with symmetry C2v. T2 combination of H1s orbitals. The other two d orbitals span
E10C.2 Show that the function xy has symmetry species B1g in the group D2h. E and so they cannot overlap with the A1 or T2 H1s orbitals.
10C Applications of symmetry 421

It follows that in methane there are a1 orbitals arising from Brief illustration 10C.4
(C2s,H1s)-overlap and t2 orbitals arising from (C2p,H1s)-
overlap. The C3d orbitals might contribute to the latter. The To construct the B1 SALC from the two O2px orbitals in NO2,
lowest energy configuration is probably a12 t 62, with all bonding point group C2v (Fig. 10C.3), choose pA as the basis function
orbitals occupied. and draw up the following table
Self-test 10C.2 Consider the octahedral SF6 molecule, with the
bonding arising from overlap of orbitals on S and a 2p orbital E C2 σv(xz)  v ( yz )
on each fluorine directed towards the central sulfur atom. The Effect on pA pA −pB pB −pA
latter span A1g + Eg + T1u. Which sulfur orbitals have non-zero Characters for B1 1 −1 1 −1
overlap with these F orbitals? Suggest what the ground-state
Product of rows 1 and 2 pA pB pB pA
configuration is likely to be.
Answer: 3s(A1g ),3p(T1u ),(3d x2 − y2 ,3d z 2 )(Eg ); a12g t16u eg4
The sum of the final row, divided by the order of the group
(h = 4), gives  ( B1 )  12 (p A  pB ).

z
Symmetry-adapted linear
10C.2(b) y

combinations x +
Topic 10B introduces the idea of generating a combination of
– B
atomic orbitals designed to transform as a particular symme-
+
try species or irreducible representation. Such a combination
is an example of a symmetry-adapted linear combination A
(SALC), which is a combination of orbitals constructed from
equivalent atoms and having a specified symmetry. SALCs are Figure 10C.3 The two O2px atomic orbitals in NO2 (point group
very useful in constructing molecular orbitals because a given C2v) can be used as a basis for forming SALCs.
SALC has non-zero overlap only with other orbitals of the
same symmetry.
The technique for building SALCs is derived by using the full
power of group theory and involves the use of a projection op- If an attempt is made to generate a SALC with symmetry
(i )
erator, P ( Γ ), an operator that takes one of the basis orbitals and that is not spanned by the basis functions, the result is zero.
generates from it—projects from it—a SALC of the symmetry For example, if in Brief illustration 10C.4 an attempt is made to
species Γ(i ): project an A1 symmetry orbital, all the characters in the second
row of the table will be 1, so when the product of rows 1 and 2 is
1 Projection formed the result is pA − pB + pB − pA = 0.

(i ) (i ) (i ) (i )
P ( )   (  ) (R)R  (  )  P (  ) n operatoor  (10C.5) A difficulty is encountered when aiming to generate an SALC
h R
of symmetry species of dimension higher than 1, because then
(i ) the rules generate sums of SALCs. Consider, for instance, the
Here ψn is one of the basis orbitals and  (  ) is a SALC (there
generation of SALCs from the three H1s atomic orbitals in NH3
might be more than one) that transforms as the symmetry spe-
(point group C3v). The molecule and the orbitals are shown in
cies Γ(i ); the sum is over the operations (not the classes) of the
Fig. 10C.4. The table below shows the effect of applying the pro-
group of order h. To implement this rule, do the following:
jection operator to sA, sB, and sC in turn to give the SALC of
• Construct a table with the columns headed by each sym- symmetry species E.
metry operation R of the group; include a column for each
operation, not just for each class.
Row E C3+ C3− σv  v  v
• Select a basis function and work out the effect that each
operation has on it. Enter the resulting function beneath 1 effect on sA sA sC sB sA sC sB
each operation. 2 characters for E 2 −1 −1 0 0 0

• On the next row enter the characters of the symmetry spe- 3 product of rows 1 and 2 2sA −sC −sB
(i )
cies of interest,  (  ) (R). 4 effect on sB sB sA sC sC sB sA

• Multiply the entries in the previous two rows, operation 5 product of rows 4 and 2 2sB −sA −sC

by operation. 6 effect on sC sC sB sA sB sA sC
7 product of rows 6 and 2 2sC −sB −sA
• Sum the result, and divide it by the order of the group, h.
422 10 Molecular symmetry

sA valence orbitals of the central O atom that can have a non-zero overlap with
+ –
these combinations of F orbitals? How would the situation be different in SF2,
C C where 3d orbitals might be available?
σv 3 3
σv
E10C.5 Consider the C2v molecule NO2. The combination px(A) − px(B) of
the two O atoms (with x perpendicular to the plane) spans A2. Is there any
valence orbital of the central N atom that can have a non-zero overlap with
sC sB that combination of O orbitals? What would be the case in SO2, where 3d
orbitals might be available?
σv

Figure 10C.4 The three H1s atomic orbitals in NH3 (point group
C3v) can be used as a basis for forming SALCs.
10C.3 Selection rules
Application of the projection operator to a different basis func-
tion gives a different SALC in each case (rows 3, 5, and 7). The intensity of a spectral line arising from a transition be-
tween some initial state with wavefunction ψi and a final state
1
6
(2s A − s B − sC ) 1
6
(2s B − s A − sC ) 1
6
(2sC − s A − s B ) with wavefunction ψf depends on the (electric) transition di-
pole moment, μfi (Topic 8C). The q-component, where q is x, y,
These three are not ‘linearly independent’ because any one of or z, of this vector, is defined through
them can be expressed as a combination of the other two. For
example, the sum of the first two is just minus the third: Transition dipole moment
q ,fi  e  f q i d [definition]
 (10C.6)
1
6
(2s A  s B  s C )  16 (2s B  s A  s C )  16 (s A  s B  2sC )
where −e is the charge of the electron. The transition moment
Furthermore, because the symmetry species of the SALCs being has the form of the integral over f1 f2 f3 (eqn 10C.2). Therefore,
projected is two-dimensional, it is expected that there will be once the symmetry species of the wavefunctions and q are
just two SALCs. known, group theory can be used to formulate the selection
It is possible to find two combinations of the SALCs from rules for the transitions.
the table which are linearly independent; they turn out to be
(s A − s B ) and (2sC − s A − s B ). These SALCs are used in the con-
Example 10C.3 Deducing a selection rule
struction of the molecular orbitals. Note that this choice is not
unique, and that other pairs of SALCs formed by cyclic permu- Is py → pz an allowed electric dipole transition in a molecule
tation of the indices A, B, and C are equally valid. with C2v symmetry?
According to the discussion in Topic 9E concerning the con-
struction of molecular orbitals of polyatomic molecules, only Collect your thoughts You need to decide whether the product
orbitals with the same symmetry can overlap to give a molecu- pzqpy, with q = x, y, or z, spans A1. The symmetry species for
lar orbital. In the language introduced here, only SALCs of the py, pz, and q can be read off from the right-hand side of the
character table.
same symmetry species have non-zero overlap and contribute
to a molecular orbital. In NH3, for instance, the molecular or- The solution The py orbital spans B2 and pz spans A1, so the
bitals will have the form required direct product is A1 × Γ(q) × B2, where Γ(q) is the sym-
metry species of x, y, or z. It does not matter in which order
 (a1 )  ca1 s N  ca2 (s A  s B  sC )
the direct products are calculated, so noting that A1 × B2 = B2
 (e x )  ce1 p Nx  ce2 (2sC  s A  s B ) implies that A1 × Γ(q) × B2 = Γ(q) × B2. This direct product can be
equal to A1 only if Γ(q) is B2, which is the symmetry species of y.
 (e y )  ce1 p Ny  ce2 (s A  s B )
Therefore, provided q = y the integral may be non-zero and the
Group theory is silent on the values of the coefficients: they have transition allowed.
to be determined by one of the methods outlined in Topic 9E. Comment. The analysis implies that the electromagnetic radia-
tion involved in the transition has a component of its electric
vector in the y-direction.
Exercises
E10C.4 Consider the C2v molecule OF2; take the molecule to lie in the yz-plane, Self-test 10C.3 Are (a) px → py and (b) px → pz allowed electric
with z directed along the C2 axis; the mirror plane  v is the yz-plane, and σv dipole transitions in a molecule with C2v symmetry?
is the xz-plane. The combination pz(A) + pz(B) of the two F atoms spans A1, Answer: (a) No; (b) yes, with q = x
and the combination pz(A) − pz(B) of the two F atoms spans B2. Are there any
10C Applications of symmetry 423

Exercises
E10C.6 Is the transition A1 → A2 forbidden for electric dipole transitions in a E10C.8 What states of (i) benzene, (ii) naphthalene may be reached by electric
C3v molecule? dipole transitions from their (totally symmetrical) ground states?
E10C.7 The electronic ground state of NO2 is A1 in the group C2v. To what
excited states may it be excited by electric dipole transitions, and what
polarization of light is it necessary to use?

Checklist of concepts
☐ 1. Character tables can be used to decide whether an inte- ☐ 3. Only orbitals of the same symmetry species may have
gral is necessarily zero. non-zero overlap.
☐ 2. For an integral to be non-zero, the integrand must ☐ 4. A symmetry-adapted linear combination (SALC) is a
include a component that transforms as the totally sym- linear combination of atomic orbitals constructed from
metric irreducible representation (A1). equivalent atoms and having a specified symmetry.

Checklist of equations
Property Equation Comment Equation number
1
h
(i )
Decomposition of a representation n((i ) )  N (C ) (  ) (C ) (C ) Real characters* 10C.3a
C

1
h
Presence of A1 in a decomposition n(A1 )  N (C ) (C ) Real characters* 10C.3b
C

Overlap integral S   2 1d Definition 10C.4

1
h
(i ) (i ) (i ) (i )
Projection operator P ( )   ( ) (R)R To generate  (  )  P (  ) n 10C.5
R

Transition dipole moment q ,fi  e  f q i d q-Component, q = x, y, or z 10C.6

* In general, characters may have complex values; throughout this text only real values are encountered.
424 10 Molecular symmetry

FOCUS 10 Molecular symmetry

To test your understanding of this material, work through Selected solutions can be found at the end of this Focus in
the Exercises, Additional exercises, Discussion questions, and the e-book. Solutions to even-numbered questions are available
Problems found throughout this Focus. online only to lecturers.

TOPIC 10A Shape and symmetry


Discussion questions
D10A.1 Explain how a molecule is assigned to a point group. D10A.3 State and explain the symmetry criteria that allow a molecule to be
polar.
D10A.2 List the symmetry operations and the corresponding symmetry
elements that occur in point groups. D10A.4 State the symmetry criterion that allows a molecule to be optically active.

Additional exercises
E10A.9 The BF3 molecule belongs to the point group D3h. List the symmetry
elements of the group and locate them in a drawing of the molecule. H
CO
E10A.10 Identify the group to which the trans-difluoroethene molecule Fe
belongs and locate the symmetry elements in a drawing of the molecule.
CO
E10A.11 Identify the point groups to which the following objects belong: (i) a
sharpened cylindrical pencil, (ii) a box with a rectangular cross-section, (iii) a
coffee mug with a handle, (iv) a three-bladed propeller (assume the sector-like F
blades are flat), (v) a three-bladed propeller (assume the blades are twisted out
of the plane, all by the same amount). 3 4
E10A.12 List the symmetry elements of the following molecules and name the E10A.14 Which of the following molecules may be polar? (i) CF3H, (ii) PCl5,
point groups to which they belong: (i) furan (1), (ii) γ-pyran (2), (iii) trans-difluoroethene, (iv) 1,2,4-trinitrobenzene.
(iii) 1,2,5-trichlorobenzene.
E10A.15 Identify the point group to which each of the possible isomers of
dichloronaphthalene belong.
E10A.16 Identify the point group to which each of the possible isomers of
O O dichloroanthracene belong.

1 Furan 2 -Pyran E10A.17 Can molecules belonging to the point groups Th or Td be chiral?
Explain your answer.
E10A.13 Assign the following molecules to point groups: (i) HF, (ii) IF7
(pentagonal bipyramid), (iii) ClF3 (T-shaped), (iv) Fe2(CO)9 (3), (v) cubane,
C8H8, (vi) tetrafluorocubane, C8H4F4 (4).

Problems
P10A.1 List the symmetry elements of the following molecules and name the each of the following dihedral angles, identify the symmetry elements present
point groups to which they belong: (a) staggered CH3CH3, (b) chair and boat and hence assign the point group: (i) 0°, (ii) 90°, (iii) 45°, (iv) 60°.
cyclohexane, (c) B2H6, (d) [Co(en)3]3+, where en is 1,2-diaminoethane (ignore
P10A.4 Find the point groups of all the possible geometrical isomers for the
its detailed structure), (e) crown-shaped S8. Which of these molecules can be
complex MA2B2C2 in which there is ‘octahedral’ coordination around the
(i) polar, (ii) chiral?
central atom M and where the ligands A, B, and C are treated as structureless
P10A.2 Consider the series of molecules SF6, SF5Cl, SF4Cl2, SF3Cl3. Assign each points. Which of the isomers are chiral?
to the relevant point group and state whether or not the molecule is expected ‡ −
P10A.5 In the square-planar complex anion [trans-Ag(CF3)2(CN)2] , the
to be polar. If isomers are possible for any of these molecules, consider all
Ag–CN groups are collinear. (a) Assume free rotation of the CF3 groups (i.e.
possible structures.
disregarding the AgCF and AgCN angles) and identify the point group of this
P10A.3 (a) Identify the symmetry elements in ethene and in allene, and assign complex ion. (b) Now suppose the CF3 groups cannot rotate freely
each molecule to a point group. (b) Consider the biphenyl molecule, Ph–Ph, (because the ion was in a solid, for example). Structure (5) shows a plane
in which different conformations are possible according to the value of the
dihedral angle between the planes of the two benzene rings: if this angle is 0°, the

molecule is planar, if it is 90°, the two rings are perpendicular to one another. For These problems were provided by Charles Trapp and Carmen Giunta.
Exercises and problems 425


which bisects the NC–Ag–CN axis and is perpendicular to it. Identify the P10A.6 B.A. Bovenzi and G.A. Pearse, Jr (J. Chem. Soc. Dalton Trans.,
point group of the complex if each CF3 group has a CF bond in that plane (so 2763 (1997)) synthesized coordination compounds of the tridentate ligand
the CF3 groups do not point to either CN group preferentially) and the CF3 pyridine-2,6-diamidoxime (C7H9N5O2, 6). Reaction with NiSO4 produced
groups are (i) staggered, (ii) eclipsed. a complex in which two of the essentially planar ligands are bonded at
right angles to a single Ni atom. Identify the point group and the symmetry
operations of the resulting [Ni(C7H9N5O2)2]2+ complex cation.
CF3
HO OH
N N
Ag
N
H2N NH2
CN
CN

CF3 6
5

TOPIC 10B Group theory


Discussion questions
D10B.1 Explain what is meant by a ‘group’. D10B.4 Explain what is meant by the reduction of a representation to a direct
sum of irreducible representations.
D10B.2 Explain what is meant by (a) a representative and (b) a representation
in the context of group theory. D10B.5 Discuss the significance of the letters and subscripts used to denote the
symmetry species of an irreducible representation.
D10B.3 Explain the construction and content of a character table.

Additional exercises
E10B.8 Use as a basis the 2pz orbitals on each atom in BF3 to find the representative E10B.12 By inspection of the character table for D4h, state the symmetry
of the operation C3. Take z as perpendicular to the molecular plane. species of the 4s, 4p, and 3d orbitals located on the central Ni atom in
Ni(CN)2−4 .
E10B.9 Use the matrix representatives of the operations σh and C3 in a basis of
2pz orbitals on each atom in BF3 to find the operation and its representative E10B.13 What is the maximum degeneracy of the wavefunctions of a particle
resulting from C3σh. Take z as perpendicular to the molecular plane. confined to the interior of an icosahedral nanoparticle?
E10B.10 Show that all three σv operations in the group D3h belong to the same class. E10B.14 What is the maximum possible degree of degeneracy of the orbitals in
benzene?
E10B.11 For the point group D3h, confirm that the irreducible representation
E′ is orthogonal (in the sense defined by eqn 10B.7) to the irreducible E10B.15 What is the maximum possible degree of degeneracy of the orbitals in
representations A1′ , A′2, and E″. 1,4-dichlorobenzene?

Problems
P10B.1 The group C2h consists of the elements E, C2, σh, i. Construct the group the group multiplications C2z C2y = C2x ,  xz C2z  C2y and iC2y   xz.
multiplication table. Give an example of a molecule that belongs to the group. Take the molecule as lying in the xy-plane, with x directed along the C–C bond.
P10B.2 The group D2h has a C2 axis perpendicular to the principal axis and a P10B.7 The (one-dimensional) matrices D(C3) = 1 and D(C2) = 1, and
horizontal mirror plane. Show that the group must therefore have a centre of D(C3) = 1 and D(C2) = −1 both represent the group multiplication C3C2 = C6 in
inversion. the group C6v with D(C6) = +1 and −1, respectively. Use the character table to
confirm these remarks. What are the representatives of σv and σd in each case?
P10B.3 Consider the H2O molecule, which belongs to the group C2v. Take the
molecule to lie in the yz-plane, with z directed along the C2 axis; the mirror P10B.8 Construct the multiplication table of the Pauli spin matrices, σ, and the
plane  v is the yz-plane, and σv is the xz-plane. Take as a basis the two H1s 2 × 2 unit matrix:
orbitals and the four valence orbitals of the O atom and set up the 6 × 6 0 1  0 i  1 0  1 0 
matrices that represent the group in this basis. (a) Confirm, by explicit matrix x    y    z    0   
 1 0   i 0   0  1  0 1 
multiplication, that C2 v   v and  v v  C2. (b) Show that the representation
is reducible and spans 3A1 + B1 + 2B2. Do the four matrices form a group under multiplication?
P10B.9 The algebraic forms of the f orbitals are a radial function multiplied by
P10B.4 Find the representatives of the operations of the group Td in a basis of
one of the factors (a) z(5z2 − 3r2), (b) y(5y2 − 3r2), (c) x(5x2 − 3r2), (d) z(x2 − y2),
four H1s orbitals, one at each apex of a regular tetrahedron (as in CH4). You
(e) y(x2 − z2), (f) x(z2 − y2), (g) xyz. Identify the irreducible representations
need give the representative for only one member of each class.
spanned by these orbitals in the point group C2v. (Hint: Because r is the radius,
P10B.5 Find the representatives of the operations of the group D2h in a basis r2 is invariant to any operation.)
of the four H1s orbitals of ethene. Take the molecule as lying in the xy-plane,
P10B.10 Using the same approach as in Section 10B.3(c) find the
with x directed along the C–C bond.
representatives using as a basis two wavefunctions ψ2,3 = (2/L)sin(2πx/L) ×
P10B.6 Find the representatives of the operations of the group D2h in a basis of sin(3πy/L) and ψ3,2 = (2/L)sin(3πx/L)sin(2πy/L) in the point group C4, and
the four H1s orbitals of ethene. Then confirm that the representatives reproduce hence show that these functions span a degenerate irreducible representation.
426 10 Molecular symmetry

TOPIC 10C Applications of symmetry


Discussion questions
D10C.1 Identify and list four applications of character tables. D10C.2 Explain how symmetry arguments are used to construct molecular orbitals.

Additional exercises
E10C.9 Use symmetry properties to determine whether or not the integral character table in the Resource section). What irreducible representations does
 p x zp z d is necessarily zero in a molecule with symmetry D3h. it span?
E10C.10 Is the transition A1g → E2u forbidden for electric dipole transitions in E10C.16 A set of basis functions is found to span a reducible representation of
a D6h molecule? the group D4h with characters 4,0,0,2,0,0,0,4,2,0 (in the order of operations in
the character table in the Resource section). What irreducible representations
E10C.11 Show that the function xyz has symmetry species Au in the group D2h.
does it span?
E10C.12 Consider the C2v molecule OF2; take the molecule to lie in the
E10C.17 A set of basis functions is found to span a reducible representation of
yz-plane, with z directed along the C2 axis; the mirror plane  v is the
the group Oh with characters 6,0,0,2,2,0,0,0,4,2 (in the order of operations in
yz-plane, and σv is the xz-plane. Find the irreducible representations spanned
the character table in the Resource section). What irreducible representations
by the combinations py(A) + py(B) and py(A) − py(B). Are there any valence
does it span?
orbitals of the central O atom that can have a non-zero overlap with these
combinations of F orbitals? E10C.18 What states of (i) anthracene, (ii) coronene (7) may be reached by
electric dipole transitions from their (totally symmetrical) ground states?
E10C.13 Consider BF3 (point group D3h). There are SALCs from the F valence
orbitals which transform as A′′2 and E″. Are there any valence orbitals of the
central B atom that can have a non-zero overlap with these SALCs? How
would your conclusion differ for AlF3, where 3d orbitals might be available?
E10C.14 The ClO2 molecule (which belongs to the group C2v) was trapped in a
solid. Its ground state is known to be B1. Light polarized parallel to the y-axis
(parallel to the OO separation) excited the molecule to an upper state. What is
the symmetry species of that state?
E10C.15 A set of basis functions is found to span a reducible representation
of the group D2 with characters 6,−2,0,0 (in the order of operations in the 7 Coronene

Problems
P10C.1 What irreducible representations do the four H1s orbitals of CH4 span? (8), which belongs to the D4h point group. The ground electronic state is A1g
Are there s and p orbitals of the central C atom that may form molecular and the lowest-lying excited state is Eu. Is a photon-induced transition allowed
orbitals with them? In SiH4, where 3d orbitals might be available, could these from the ground state to the excited state? Explain your answer.
orbitals play a role in forming molecular orbitals by overlapping with the H1s
orbitals?
P10C.2 Suppose that a methane molecule became distorted to (a) C3v N
symmetry by the lengthening of one bond, (b) C2v symmetry, by a kind of
scissors action in which one bond angle opened and another closed slightly. N– N–
Would more d orbitals on the carbon become available for bonding?
2
P10C.3 Does the integral of the function 3x − 1 necessarily vanish when N
integrated over a symmetrical range in (a) a cube, (b) a tetrahedron, (c) a
hexagonal prism, each centred on the origin?

P10C.4 In a spectroscopic study of C60, Negri et al. (J. Phys. Chem. 100, 10849 8
(1996)) assigned peaks in the fluorescence spectrum. The molecule has P10C.7 Consider the ethene molecule (point group D2h), and take it as lying in
icosahedral symmetry (Ih). The ground electronic state is A1g, and the lowest- the xy-plane, with x directed along the C–C bond. By applying the projection
lying excited states are T1g and Gg. (a) Are photon-induced transitions allowed formula to one of the hydrogen 1s orbitals generate SALCs which have
from the ground state to either of these excited states? Explain your answer. (b) symmetry Ag, B2u, B3u, and B1g. What happens when you try to project out a
What if the molecule is distorted slightly so as to remove its centre of inversion? SALC with symmetry B1u?
P10C.5 In the square planar XeF4 molecule, consider the symmetry-adapted
P10C.8 Consider the molecule F2C= CF2 (point group D2h), and take it as lying
linear combination p1 = pA − pB + pC − pD, where pA, pB, pC, and pD are 2pz
in the xy-plane, with x directed along the C–C bond. (a) Consider a basis
atomic orbitals on the fluorine atoms (clockwise labelling of the F atoms).
formed from the four 2pz orbitals from the fluorine atoms: show that the basis
Decide which of the various s, p, and d atomic orbitals on the central Xe atom
spans B1u, B2g, B3g, and Au. (b) By applying the projection formula to one of the
can form molecular orbitals with p1.
2pz orbitals, generate the SALCs with the indicated symmetries. (c) Repeat the
P10C.6 The chlorophylls that participate in photosynthesis and the haem process for a basis formed from four 2px orbitals (the symmetry species will
(heme) groups of cytochromes are derived from the porphine dianion group be different from those for 2pz).
FOCUS 11
Molecular spectroscopy

The origin of spectral lines in molecular spectroscopy is the ab- Topic then focuses on the interpretation of pure rotational and
sorption, emission, or scattering of a photon accompanied by a rotational Raman spectra, in which only the rotational state of
change in the energy of a molecule. The difference from atomic a molecule changes. The observation that not all molecules can
spectroscopy (Topic 8C) is that the energy of a molecule can occupy all rotational states is shown to arise from symmetry
undergo not only electronic transitions but also changes of ro- constraints resulting from the presence of nuclear spin.
tational and vibrational state. Molecular spectra are therefore 11B.1 Rotational energy levels; 11B.2 Microwave spectroscopy;
more complex than atomic spectra. However, they contain in- 11B.3 Rotational Raman spectroscopy; 11B.4 Nuclear statistics and
rotational states
formation relating to more properties, and their analysis leads
to values of bond strengths, lengths, and angles. They also pro-
vide a way of determining a variety of molecular properties,
such as dissociation energies and dipole moments.
11C Vibrational spectroscopy of
diatomic molecules
11A General features of molecular The harmonic oscillator (Topic 7E) is a good starting point for
spectroscopy modelling the vibrations of diatomic molecules, but it is shown
that the description of real molecules requires deviations from
This Topic begins with a discussion of the theory of absorption and harmonic behaviour to be taken into account. The vibrational
emission of radiation, leading to the factors that determine the in- spectra of gaseous samples show features due to the rotational
tensities and widths of spectral lines. The features of the instrumen- transitions that accompany the excitation of vibrations.
tation used to monitor the absorption, emission, and scattering of 11C.1 Vibrational motion; 11C.2 Infrared spectroscopy;
11C.3 Anharmonicity; 11C.4 Vibration–rotation spectra;
radiation spanning a wide range of frequencies are also described. 11C.5 Vibrational Raman spectra
11A.1 The absorption and emission of radiation;
11A.2 Spectral linewidths; 11A.3 Experimental techniques

11D Vibrational spectroscopy of


11B Rotational spectroscopy polyatomic molecules
This Topic shows how expressions for the values of the rota- The vibrational spectra of polyatomic molecules can be dis-
tional energy levels of diatomic and polyatomic molecules are cussed as though they consist of a set of independent harmonic
derived. The most direct procedure, which is used here, is to oscillators. Their spectra can then be understood in much the
identify the expressions for the energy and angular momen- same way as those of diatomic molecules.
tum obtained in classical physics, and then to transform these 11D.1 Normal modes; 11D.2 Infrared absorption spectra;
expressions into their quantum mechanical counterparts. The 11D.3 Vibrational Raman spectra
11E Symmetry analysis of vibrational 11G Decay of excited states
spectra This Topic begins with an account of spontaneous emission
by molecules, including the phenomena of ‘fluorescence’ and
The atomic displacements involved in the vibrations of polya-
‘phosphorescence’. It also explains how non-radiative decay of
tomic molecules can be classified according to the symmetry
excited states can result in the transfer of energy as heat to the
possessed by the molecule. This classification makes it possible
surroundings or result in molecular dissociation. The stimu-
to decide which vibrations can be studied spectroscopically.
lated radiative decay of excited states is the key process respon-
11E.1 Classification of normal modes according to symmetry;
11E.2 Symmetry of vibrational wavefunctions sible for the action of lasers.
11G.1 Fluorescence and phosphorescence; 11G.2 Dissociation and
predissociation; 11G.3 Lasers

11F Electronic spectra


What is an application of this material?
This Topic introduces the key idea that electronic transitions
occur within a stationary nuclear framework. The electronic Molecular spectroscopy is also useful to astrophysicists and
spectra of diatomic molecules are considered first, and it is seen environmental scientists. Impact 16, accessed via the e-book,
that in the gas phase it is possible to observe simultaneous vi- discusses how the identities of molecules found in interstel-
brational and rotational transitions that accompany the elec- lar space can be inferred from their rotational and vibrational
tronic transition. The general features of the electronic spectra spectra. Impact 17, accessed via the e-book, focuses back on
of polyatomic molecules are also described. Earth and shows how the vibrational properties of its atmos-
11F.1 Diatomic molecules; 11F.2 Polyatomic molecules pheric constituents can affect its climate.

➤ Go to the e-book for videos that feature the derivation and interpretation of equations, and applications of this material.
TOPIC 11A General features of
­molecular spectroscopy

of Topic 7A), h  Eu  El , where ν is the frequency of the


➤ Why do you need to know this material? radiation emitted or absorbed. Emission and absorption
To interpret data from the wide range of varieties of molec- spectroscopy give the same information about electronic,
ular spectroscopy you need to understand the experimen- vibrational, or rotational energy level separations, but prac-
tal and theoretical features shared by them all. tical considerations generally determine which technique is
employed.
➤ What is the key idea? In Raman spectroscopy the sample is exposed to monochro-
A transition from a low energy state to one of higher matic (single frequency) radiation and therefore a stream of
energy can be stimulated by absorption of radiation; a photons all of the same energy. When the photons encounter
transition from a higher to a lower state, resulting in emis- the molecules, most are scattered elastically (without change
sion of a photon, may be either spontaneous or stimulated in their energy): this process is called Rayleigh scattering.
by radiation. About 1 in 107 of the photons are scattered inelastically (with
different energy). In Stokes scattering the photons lose energy
➤ What do you need to know already? to the molecules and the emerging radiation has a lower fre-
You need to be familiar with the fact that molecular energy quency. In anti-Stokes scattering, a photon gains energy from
is quantized (Topics 7E and 7F) and be aware of the con- a molecule and the emerging radiation has a higher frequency
cept of selection rules (Topic 8C). (Fig. 11A.1). By analysing the frequencies of the scattered radi-
ation it is possible to gather information about the energy levels
of the molecules. Raman spectroscopy is used to study molecu-
lar vibrations and rotations.

In emission spectroscopy the electromagnetic radiation that


arises from molecules undergoing a transition from a higher
energy state to a lower energy state is detected and its frequency
analysed. In absorption spectroscopy, the net absorption of
radiation passing through a sample is monitored over a range Anti-Stokes
of frequencies. It is necessary to specify the net absorption be-
cause not only can radiation be absorbed but it can also stimu- Rayleigh
Energy

late the emission of radiation, so the net absorption is detected. Incident


radiation
In Raman spectroscopy, the frequencies of radiation scattered Stokes
by molecules is analysed to determine the changes in molecular
states that accompany the scattering process. Throughout this
discussion it is important to be able to express the character-
istics of radiation variously as a frequency, ν , a wavenumber,
   /c, or a wavelength,   c/ , as set out in The chemist’s Figure 11A.1 In Raman spectroscopy, incident photons are
toolkit 7A.1. scattered from a molecule. Most photons are scattered elastically
and so have the same energy as the incident photons (Rayleigh
In each case, the emission, absorption, or scattering of ra-
scattering). Some photons lose energy to the molecule and so
diation can be interpreted in terms of individual photons.
emerge as Stokes radiation; others gain energy and so emerge
When a molecule undergoes a transition between a lower as anti-Stokes radiation. The scattering can be regarded as taking
state with energy El and an upper state with energy Eu a pho- place by an excitation of the molecule from its initial state to a
ton with energy hν is emitted or absorbed. The energy of the series of excited states (represented by the shaded band), and the
photon depends on the difference in energy between the two subsequent return to a final state. Any net energy change is either
states according to the Bohr frequency condition (eqn 7A.10 supplied from, or carried away by, the photon.
430 11 Molecular spectroscopy

11A.1The absorption and emission In this expression Bl,u is the Einstein coefficient of stimulated
emission.
of radiation Einstein went on to suppose that a molecule could lose
energy by spontaneous emission in which the molecule
The separation of rotational energy levels (in small molecules, makes a transition to a lower state without it being driven by
∆E ≈ 0.01 zJ, corresponding to about 0.01 kJ mol−1) is smaller the presence of radiation. The rate of spontaneous emission
than that of vibrational energy levels (∆E ≈ 10 zJ, correspond- is written as
ing to 10 kJ mol−1), which itself is smaller than that of elec-
tronic energy levels (∆E ≈ 0.1–1 aJ, corresponding to about Wuspontaneous
l  Al,u N u Rate of spontaneous emission (11A.1c)
102–103 kJ mol−1). From the Bohr frequency condition in the
where Al,u is the Einstein coefficient of spontaneous emission.
form ν = ∆E/h, the corresponding frequencies of the photons
When both stimulated and spontaneous emission are taken
involved in these different kinds of transitions are about 1010 Hz
into account, the total rate of emission is
for rotation, 1013 Hz for vibration, and in the range 1014–1015 Hz
for electronic transitions. It follows that rotational, vibrational, Wu  l  Bl,u N u  ( )  Al,u N u Total rate of emission (11A.1d)
and electronic transitions result from the absorption or emis-
sion of microwave, infrared, and ultraviolet/visible radiation, When the molecules and radiation are in equilibrium, the
respectively. rates given in eqns 11A.1a and 11A.1d must be equal, and the
populations then have their equilibrium values N leq and N ueq .
Therefore
11A.1(a)Stimulated and spontaneous
radiative processes Bu ,l N leq  ( )  Bl,u N ueq  ( )  Al,u N ueq  (11A.2a)
Albert Einstein identified three processes by which radiation and so
could be either generated or absorbed by matter as a result of
transitions between states. In stimulated absorption a transi- Al,u /Bu ,l
 ( )   (11A.2b)
tion from a lower energy state l to a higher energy state u is N /N ueq  Bl,u /Bu ,l
eq
l
driven by an electromagnetic field oscillating at the frequency
ν corresponding to the energy separation of the two states: However, the ratio of the equilibrium populations must be in
hν = Eu − El. The rate of such transitions is proportional to the accord with the Boltzmann distribution (as specified in Energy:
intensity of the incident radiation at the transition frequency: A first look and Topic 13A):
the more intense the radiation, the greater the number of pho-
N ueq
tons impinging on the molecules and the greater the probabil-  e ( Eu  El )/kT  e  h /kT  (11A.3)
ity that a photon will be absorbed. The rate is also proportional N leq
to the number of molecules in the lower state, Nl, because the and therefore, at equilibrium,
greater the population of that state the more likely it is that a
photon will encounter a molecule in that state. The rate of stim- Al,u /Bu ,l
 ( )  h /kT
 (11A.4)
ulated absorption, Wu←l, can therefore be written e  Bl,u /Bu ,l

Wu  l  Bu,l N l  ( ) Rate of stimulated absorption (11A.1a) Moreover, at equilibrium the radiation density is given by the
Planck distribution of radiation in equilibrium with a black
In this expression, ρ(ν) is the energy spectral density, such that body (eqn 7A.6b, Topic 7A):
ρ(ν)dν is the energy density of radiation in the frequency range
from ν to ν + dν, in the sense that the energy in that range in a 8h 3 /c 3
 ( )  Planck distribution (11A.5)
region of volume V is Vρ(ν)dν. The constant Bu,l is the Einstein eh /kT  l
coefficient of stimulated absorption. By comparing eqns 11A.4 and 11A.5 it follows that
Einstein also supposed that the radiation could induce
the molecule in an upper state to undergo a transition to a 8h 3
Al,u  Bl,u and Bl,u  Bu ,l  (11A.6a)
lower state and thereby generate a photon of frequency ν. c3
This process is called stimulated emission, and its rate de-
pends on the number of molecules in the upper level Nu and Although these relations have been derived on the assumption
the intensity of the radiation at the transition frequency. By that the molecules and radiation are in equilibrium, they are
analogy with eqn 11A.1a, the rate of stimulated emission can properties of the molecules themselves and are independent of
be written as the spectral distribution of the radiation (that is, whether it is
black-body or not) and can be used in eqn 11A.1 for any energy
Wustimulated
l  Bl,u N u  ( ) Rate of stimulated emission (11A.1b) densities.
11A General features of ­molecular spectroscopy 431

The ratio of the rate of spontaneous to stimulated emission


can be found by combining eqns 11A.1b, 11A.1c, and 11A.6a
to give

Wuspontaneous Al,u 8h 3


l
 
Wustimulated
l
Bl,u  ( ) c 3  ( )  (11A.6b)

This relation shows that:


• For a given spectral density, the relative importance of
(a) (b)
spontaneous emission increases as the cube of the transi-
tion frequency and therefore that spontaneous emission is Figure 11A.2 (a) When an electron undergoes a transition
most likely to be of importance at high frequencies. from a 1s orbital to a 2s orbital, there is a spherical migration of
charge. There is no dipole moment associated with this migration
• Spontaneous emission can be ignored at low frequencies,
of charge, so this transition is electric-dipole forbidden. (b) In
in which case the intensities of such transitions can be
contrast, when an electron moves from 1s to 2p, there is a dipole
discussed in terms of stimulated emission and absorption associated with the charge migration; this transition is electric-
alone. dipole allowed.

Brief illustration 11A.1


A gross selection rule specifies the general features that a
On going from infrared to visible radiation, the frequency molecule must have if it is to have a spectrum of a given kind.
increases by a factor of about 100, so the ratio of the rates of For instance, in Topic 11B it is shown that a molecule gives
spontaneous to stimulated emission increases by a factor of 106 a rotational spectrum only if it has a permanent electric di-
for the same spectral density. This strong increase accounts for pole moment. This rule, and others like it for other types of
the observation that whereas electronic transitions are often transition, is explained in the relevant Topic. A detailed study
monitored by emission spectroscopy, vibrational spectroscopy
of the transition moment leads to the specific selection rules,
is an absorption technique and spontaneous (but not stimu-
which express the allowed transitions in terms of the changes
lated) emission is negligible.
in quantum numbers or various symmetry features of the
molecule.
Selection rules and transition
11A.1(b)
moments 11A.1(c) The Beer–Lambert law
A ‘selection rule’ is a statement about whether a transition is It is found empirically that when electromagnetic radiation
forbidden or allowed (Topic 8C). The underlying idea is that, passes through a sample of length L and molar concentration
for the molecule to be able to interact with the electromagnetic [J] of the absorbing species J, the incident and transmitted in-
field and absorb or create a photon of frequency ν, it must pos- tensities, I0 and I, are related by the Beer–Lambert law:
sess, at least transiently, an electric dipole oscillating at that
frequency. This transient dipole is expressed quantum mechan- I = I 0 10−ε [J]L Beer – Lambert law  (11A.8)
ically in terms of the transition dipole moment, µ fi, between
The quantity ε (epsilon) is called the molar absorption coef-
the initial and final states with wavefunctions ψi and ψf:1
ficient (formerly, and still widely, the ‘extinction coefficient’);
Transition dipole moment
it depends on the frequency (or wavenumber and wavelength)
µfi = ∫ψ f µˆψ i dτ [definition]
 (11A.7) of the incident radiation and is greatest where the absorption
is most intense.
where µ̂ is the electric dipole moment operator. The magnitude The dimensions of ε are 1/(concentration × length), and it is
of the transition dipole moment can be regarded as a meas- normally convenient to express it in cubic decimetres per mole
ure of the charge redistribution that accompanies a transition. per centimetre (dm3 mol−1 cm−1); in SI base units it is expressed
Moreover, a transition is active (and generates or absorbs a in metres squared per mole (m2 mol−1). The latter units imply
photon) only if the accompanying charge redistribution is di- that ε may be regarded as a (molar) cross-section for absorp-
polar (Fig. 11A.2). It follows that, to identify the selection rules, tion, and that the greater the cross-sectional area of the mole-
the conditions for which µ fi ≠ 0 must be established. cule for absorption, the greater is its ability to block the passage
of the incident radiation at a given frequency.
1
Equation 11A.7 is derived in A deeper look 11A.1 , available to read in the The Beer–Lambert law is an empirical result. However, its
e-book accompanying this text. form can be derived on the basis of a simple model.
432 11 Molecular spectroscopy

How is that done? 11A.1 Justifying the Beer–Lambert law Therefore


I
You need to imagine the sample as consisting of a stack of ln  [J]L
I0
infinitesimal slices, like sliced bread (Fig. 11A.3). The thickness
of each layer is dx. Now express the natural logarithm as a common logarithm
(to base 10) by using ln x = (ln 10) log x, and a new constant ε
Step 1 Calculate the change in intensity due to passage through defined as ε = κ/(ln 10) to give
one slice
I
The change in intensity, dI, that occurs when electromagnetic log   [J]L
I0
radiation passes through one particular slice is proportional
to the thickness of the slice, the molar concentration of the Take common antilogarithms of each side (that is, form 10x of
absorber J, and (because the absorption is stimulated) the each side) and obtain the Beer–Lambert law (eqn 11A.8).
intensity of the incident radiation at that slice of the sample,
so dI ∝ [J]Idx. The intensity is reduced by absorption, which
means that dI is negative and can therefore be written
The spectral characteristics of a sample are commonly
dI  [J]Idx
r­eported as the transmittance, T, of the sample at a given
where κ (kappa) is the proportionality coefficient. Division of ­frequency:
both sides by I gives
I
dI T= Transmittance [definition] (11A.9a)
 [J]dx I0
I
This expression applies to each successive slice. or its absorbance, A:
Step 2 Evaluate the total change in intensity due to passage I0
through successive slices A = log Absorbance [definition] (11A.9b)
I
To obtain the intensity that emerges from a sample of thickness
L when the intensity incident on one face of the sample is I0, The two quantities are related by A = −log T (note the common
you need the sum of all the successive changes. Assume that logarithm). In terms of the absorbance the Beer–Lambert law
the molar concentration of the absorbing species is uniform becomes
and may be treated as a constant. Because a sum over infinitesi-
A   [J]L (11A.9c)
mally small increments is an integral, it follows that:
The product ε[J]L was known formerly as the optical density of
Integral A.2 Integral A.1
the sample.
I dI L L
∫ I0 I
= −κ ∫ [J]d x = −κ [J] ∫ d x
0 0

[J] a constant Example 11A.1 Determining a molar absorption


coefficient
x x + dx Radiation of wavelength 280 nm passed through 1.0 mm of
Intensity, I – dI a solution that contained an aqueous solution of the amino
acid tryptophan at 0.50 mmol dm−3. The intensity is reduced
Intensity, I
to 54 per cent of its initial value (so T = 0.54). Calculate the
absorbance and the molar absorption coefficient of tryptophan
at 280 nm. What would be the transmittance through a cell of
thickness 2.0 mm?
Collect your thoughts From A = −log T = ε[J]L, it follows that
Length, L ε = A/[J]L. For the transmittance through the thicker cell, you
need to calculate the absorbance by using A = −log T = ε[J]L
and the computed value of ε; the transmittance is T = 10−A.
Figure 11A.3 To establish the Beer–Lambert law, the sample is
supposed to be divided into a large number of thin slices. The The solution The absorbance is A = −log 0.54 = 0.27, and so the
reduction in intensity caused by one slice is proportional to the molar absorption coefficient is
intensity incident on it (after passing through the preceding
 log 0.54
slices), the thickness of the slice, and the concentration of the   5.4  102 dm3 mol 1 mm 1
absorbing species. (5.0  10 mol dm 3 )  (1.0 mm)
4
11A General features of m
­ olecular spectroscopy 433

These units are convenient for the rest of the calculation (but c­ ontribute to a band: a spectroscopic line is not a geometrically
the outcome could be reported as 5.4 × 103 dm3 mol−1 cm−1 if thin line, but has a width.
desired or even as 5.4 × 102 m2 mol−1). The absorbance of a
sample of length 2.0 mm is
Exercises
A  (5.4  102 dm3 mol 1 mm 1 )  (5.0  104 mol dm 3 )
E11A.1 Calculate the ratio A/B for transitions with the following
 (2.0 mm)  0.54 characteristics: (i) 70.8 pm X-rays, (ii) 500 nm visible light, (iii) 3000 cm−1
infrared radiation.
The transmittance is now T = 10−A = 10−0.54 = 0.29.
E11A.2 The molar absorption coefficient of a substance dissolved in hexane is
Self-test 11A.1 The transmittance of an aqueous solution known to be 723 dm3 mol−1 cm−1 at 260 nm. Calculate the percentage reduction
containing the amino acid tyrosine at a molar concentration in intensity when ultraviolet radiation of that wavelength passes through
2.50 mm of a solution of concentration 4.25 mmol dm−3.
of 0.10 mmol dm−3 was measured as 0.14 at 240 nm in a cell of
length 5.0 mm. Calculate the absorbance of the solution and E11A.3 A solution of a certain component of a biological sample when
placed in an absorption cell of path length 1.00 cm transmits 18.1 per
the molar absorption coefficient of tyrosine at that wavelength.
cent of ultraviolet radiation of wavelength 320 nm incident upon it. If the
What would be the transmittance through a cell of length concentration of the component is 0.139 mmol dm−3, what is the molar
1.0 mm? absorption coefficient?
Answer: A = 0.85, 1.7 × 104 dm3 mol−1 cm−1, T = 0.67

11A.2 Spectral linewidths


The maximum value of the molar absorption coeffi-
cient, εmax, is an indication of the intensity of a transition. Several effects contribute to the widths of spectroscopic lines.
However, because absorption bands generally spread over The design of the spectrometer itself affects the linewidth, and
a range of wavenumbers, quoting the absorption coefficient there are other contributions that arise from physical processes
at a single wavenumber might not give a true indication in the sample. Some of the latter can be minimized by alter-
of the intensity of a transition. The integrated absorption ing the conditions whereas others are intrinsic to the molecules
­coefficient, A, is the sum of the absorption coefficients over and cannot be altered.
the entire band (Fig. 11A.4), and corresponds to the area
under the plot of the molar absorption coefficient against 11A.2(a) Doppler broadening
wavenumber:
One important broadening process in gaseous samples is the
Integrated absorption coefficient Doppler effect, in which radiation is shifted in frequency when
A  ( )d  (11A.10)
band [definitio
on] its source is moving towards or away from the observer. When
a molecule emitting electromagnetic radiation of frequency ν0
For bands of similar widths, the integrated absorption co- moves with a speed s relative to an observer, the observer de-
efficients are proportional to the heights of the bands. tects radiation of frequency
Equation 11A.10 also applies to the individual lines that
1/ 2 1/ 2
 1  s /c   1  s /c 
receding    0 approaching    0
 1  s /c   1  s /c 
Doppler shifts  (11A.11a)
Molar absorption coefficient, ε

where c is the speed of light. For nonrelativistic speeds (s << c),


these expressions simplify to

receding  (1  s /c)0 approaching  (1  s /c)0  (11A.11b)


Area =
Atoms and molecules reach high speeds in all directions in
integrated
absorption a gas, and a stationary observer detects the corresponding
coefficient, A Doppler-shifted range of frequencies. Some molecules ap-
proach the observer, some move away; some move quickly,
others slowly. The detected spectral ‘line’ is the absorption or
Wavenumber, 
emission profile arising from all the resulting Doppler shifts.
Figure 11A.4 The integrated absorption coefficient of a transition The challenge is to relate the observed linewidth to the spread
is the area under a plot of the molar absorption coefficient against of speeds in the gas, and in turn to see how that spread depends
the wavenumber of the incident radiation. on the temperature.
434 11 Molecular spectroscopy

How is that done? 11A.2 Deriving an expression for The chemist’s toolkit 11A.1 Exponential and Gaussian
Doppler broadening functions
You need to relate the spread of Doppler shifts to the distribu- An exponential function is a function of the form
tion of molecular kinetic energy as expressed by the Boltzmann
f (x )  ae bx Exponential function
distribution.
This function has the value a at x = 0 and decays toward zero as
Step 1 Establish the relation between the observed frequency and
x → ∞. This decay is faster when b is large than when it is small.
the molecular speed
The function rises rapidly to infinity as x → −∞. See Sketch 1.
It follows from the Boltzmann distribution (see Energy: A first
look) that the probability that an atom or molecule of mass m
1
and speed s in a gas phase sample at a temperature T has kinetic e–x
2
2 10
energy Ek = 12 ms 2 is proportional to e −ms / 2 kT . When s << c, the
9 e–x
Doppler shifts for receding and approaching molecules are

f (x)/a
8
given by the expressions in eqn 11A.11b. It follows that the 7
e –x

shift between the observed frequency and the true frequency 6


is νobs − ν0 ≈ ±ν0s/c. This expression can be rearranged to give

f(x)/a
5 0
0 1 x 2 3
s  c(obs  0 )/0 4
3
Step 2 Evaluate the distribution of frequencies arising from a
2
­distribution of speeds
1 2
e–x
The intensity I of a transition at νobs is proportional to the 0
–5 –4 –3 –2 –1 0 x 1 2 3 4 5
probability of there being a molecule that emits or absorbs at
νobs. Such a molecule would have a speed given by the expres- Sketch 1
sion just derived, s  c(obs  0 )/0, so it follows from the
Boltzmann distribution that
2
The general form of a Gaussian function is
2
( obs  0 )2 / 2 02 kT
I (obs )  e  ms / 2 kT
 e  mc 2
/ 2 2
f (x )  ae ( x b ) Gaussian function
which has the form of a Gaussian function. The width of
the absorption line at half its maximum height, δνobs, can be The graph of this function is a symmetrical bell-shaped curve
inferred directly from the general form of such a function (as centred on x = b; the function has its maximum values of a at its
specified in The chemist’s toolkit 11A.1): centre. The width of the function, measured at half its height,
is δx = 2σ(2 ln 2)1/2; the greater σ, the greater is the width at
2 0 2kT ln2
1/2
(11A.12a) half-height. Sketch 1 also shows a Gaussian function with b = 0.
δ =
obs
c m Doppler broadening

Note that the Doppler linewidth is proportional to the fre-


Doppler broadening increases with temperature (Fig. 11A.5) quency, so Doppler broadening becomes more important as
because the molecules then acquire a wider range of speeds. higher frequencies are observed.
Conversely, reducing the temperature results in narrower lines.
Brief illustration 11A.2

For a molecule such as CO at T = 300 K, and noting that


1 J = 1 kg m2 s−2,
Absorption intensity

T/3
1/ 2
obs 2  2kT ln 2 
  
0 c  mCO 
2
T 
2.998  108 ms 1
1/ 2
3T  2  (1.381  1023 J K 1 )  (300 K )  ln 2 
 
 4.651  1026 kg
 
Frequency 6
 2.34  10
Figure 11A.5 The Gaussian shape of a Doppler-broadened For a transition wavenumber of 2150 cm−1 from the infrared
spectral line reflects the Boltzmann distribution of translational spectrum of CO, corresponding to a frequency of 64.4 THz
kinetic energies in the sample at the temperature of the (1 THz = 1012 Hz), the linewidth is 151 MHz or 5.0 × 10−3 cm−1.
experiment. The line broadens as the temperature is increased.
11A General features of m
­ olecular spectroscopy 435

The linewidth due to Doppler broadening can also be ex- to collisions is sometimes referred to as pressure ­broadening.
pressed in terms of wavelength as This contribution to the linewidth can be minimized by low-
ering the pressure as much as possible, although doing so de-
1/ 2
2  2kT ln 2  creases the intensity of the absorption because there are fewer
obs  0 Doppler broadening (11A.12b)
c  m  molecules to absorb the radiation. In contrast to Doppler
broadening, pressure broadening is independent of the transi-
tion frequency.
11A.2(b) Lifetime broadening
At any instant a molecule exists in a specific state but it does
Brief illustration 11A.4
not remain in that state indefinitely. For example, the molecule
might collide with another and in the process change its state. The linewidth due to pressure broadening in methane gas at
Alternatively, the molecule may fall to a lower level and emit 1 bar and 300 K can be estimated by using the expressions
a photon by the process of spontaneous emission. How long just quoted; the collision cross-section σ is 0.46 nm2. Taking
the state persists depends on the rates of these processes and is mA and mB both to be the mass of a methane molecule, vrel is
characterized by a lifetime, τ. When the Schrödinger equation 888 m s−1. Hence
is analysed for a state that persists for a time τ, it turns out that
the energy is uncertain to an extent δE ≈ ħ/τ. Therefore, spec-  vrel p
z
troscopic transitions that involve this state have a linewidth of kT
the order of (0.46  1018 m2 )  (888 ms 1 )  (1  105 N m 2 )

(1.381  1023 JK 1 )  (300 K )
E 1
 Lifetime broadening (11A.13)  9.9  109 s 1
h 2
It follows that the shorter the lifetime, the broader is the line. The linewidth is therefore z/2π = 1.6 GHz or 0.053 cm−1. In
This process is called lifetime broadening. When the lifetime is Brief illustration 11A.2 the Doppler linewidth for a transition
limited by the process of spontaneous emission rather than ex- in the infrared is estimated as 150 MHz, which is much less
ternal causes such as collisions, the resulting linewidth is called than the pressure broadening estimated for the present set of
the natural linewidth. conditions. As the frequency is raised, the Doppler broadening
increases in proportion, but the pressure broadening remains
unchanged, so Doppler broadening might become dominant.
Brief illustration 11A.3

Excited electronic states of molecules often have short lifetimes


Exercises
due to the high rate of spontaneous emission. A typical life-
time might be 10 ns, which would lead to a natural linewidth E11A.4 What is the Doppler-broadened linewidth of the electronic transition
at 821 nm in atomic hydrogen at 300 K?
of 1/(2π × 10 × 10−9 s) = 16 MHz or 5.3 × 10−4 cm−1. As can be
inferred from Brief illustration 11 A.2, the Doppler linewidth is E11A.5 Estimate the lifetime of a state that gives rise to a line of width
typically much greater than the natural linewidth. (i) 0.20 cm−1, (ii) 2.0 cm−1.
13
E11A.6 A molecule in a liquid undergoes about 1.0 × 10 collisions in each
second. Suppose that (i) every collision is effective in deactivating the
Collisions between molecules are generally efficient at molecule vibrationally and (ii) that one collision in 100 is effective. Calculate
changing their rotational or vibrational energies, so a good es- the width (in cm−1) of vibrational transitions in the molecule.

timate of the resulting collisional lifetime, τcol, is to equate it


to 1/z, where z is the collision frequency (Topic 1B). If it is as-
sumed that each collision results in a change of rotational or
vibrational state, the lifetime of a state can be taken as τcol, and 11A.3 Experimental techniques
hence the resulting broadening is δE/h = 1/2πτcol = z/2π; this
contribution to the linewidth is often referred to as collisional Common to all spectroscopic techniques is a spectrometer,
line broadening. The collision frequency for two molecules an instrument used to detect the characteristics of radia-
with masses mA and mB is given by eqn 1B.12b as tion scattered, emitted, or absorbed by atoms and molecules.
Figure 11A.6 shows the general layout of an absorption spec-
1/ 2
 vrel p  8kT  mA mB trometer. Radiation from an appropriate source is directed to-
z with vrel    and  
kT    mA  mB wards a sample and the radiation transmitted strikes a device
that separates it into different frequencies or wavelengths. The
Note that the collision frequency, and hence the linewidth, is intensity of radiation at each frequency is then analysed by a
proportional to the pressure, which is why the broadening due suitable detector. It is usual to record one spectrum with the
436 11 Molecular spectroscopy

(1) Background Linear Experimental


accelerator stations
(2) Sample
Detector
Source
30 m
Frequency 10 m
analyser

Booster Radiation
synchrotron
Store and digitally subtract
Electron
Figure 11A.6 The layout of a typical absorption spectrometer. beam
Radiation from the source passes through the background material
Figure 11A.7 A simple synchrotron storage ring. The electrons
(1), typically an empty container or a container containing the
injected into the ring from the linear accelerator and booster
solvent only, or a sample (2) held in a container that matches
synchrotron are accelerated to high speed in the main ring. An
the characteristics of that used as background. In each case
electron in a curved path is subject to constant acceleration, and
the radiation is then dispersed according to frequency and the
an accelerated charge radiates electromagnetic energy.
intensity at each frequency is measured with a detector. The
absorption spectrum of the sample is then constructed by digitally
subtracting the background signal from the sample signal. a wide range of frequencies, including infrared radiation and
X-rays. Except in the microwave region, synchrotron radiation
is much more intense than can be obtained by most conven-
sample in place, and one with the sample removed (the ‘back- tional sources.
ground spectrum’): the difference between these two spectra
eliminates any absorption not due to the sample itself. 11A.3(b) Spectral analysis
A common device for the analysis of the frequencies, wave-
11A.3(a) Sources of radiation numbers, or wavelengths in a beam of radiation is a diffraction
Sources of radiation are either monochromatic, those spanning grating, which consists of a glass or ceramic plate into which
a very narrow range of frequencies around a central value, or fine grooves have been cut and covered with a reflective alu-
polychromatic, those spanning a wide range of frequencies. minium coating. For work in the visible region of the spectrum,
In the microwave region frequency synthesizers and various the grooves are cut about 1000 nm apart (a spacing comparable
solid state devices can be used to generate monochromatic to the wavelength of visible light). The grating causes interfer-
radiation that can be tuned over a wide range of frequencies. ence between waves reflected from its surface, and construc-
Certain kinds of lasers and light-emitting diodes are often used tive interference occurs at specific angles that depend on the
to provide monochromatic radiation from the infrared to the wavelength of the radiation being used. Thus, each wavelength
ultraviolet region. Polychromatic black-body radiation from
hot materials (Topic 7A) can be used over the same range.
Examples include mercury arcs inside a quartz envelope (us- To detector
able in the range 35–200 cm−1), Nernst filaments and globars
Incident
(200–4000 cm−1), and quartz–tungsten–halogen lamps (320– beam λ1
Slit
2500 nm). λ2
λ3
A gas discharge lamp is a common source of ultraviolet and
visible radiation. In a xenon discharge lamp, an electrical dis-
charge excites xenon atoms to excited states, which then emit Diffraction grating
ultraviolet radiation. In a deuterium lamp, excited D2 molecules
dissociate into electronically excited D atoms that emit intense
radiation in the range 200–400 nm.
For certain applications, radiation is generated in a synchro- Figure 11A.8 A polychromatic beam is dispersed by a diffraction
tron storage ring, which consists of an electron beam travelling grating into three component wavelengths λ1, λ2, and λ3. In the
in a circular path with circumferences of up to several hundred configuration shown, only radiation with λ2 passes through a
metres. As electrons travelling in a circle are constantly acceler- narrow slit and reaches the detector. Rotation of the diffraction
ated by the forces that constrain them to their path, they gen- grating (as shown by the arrows on the dotted circle) allows λ1 or
erate radiation (Fig. 11A.7). This ‘synchrotron radiation’ spans λ3 to reach the detector.
11A General features of m
­ olecular spectroscopy 437

of light is diffracted into a specific direction (Fig. 11A.8). In a 2


monochromator, a narrow exit slit allows only a narrow range
of wavelengths to reach the detector. Turning the grating on an 1.5
axis perpendicular to the incident and diffracted beams allows

1 + cos 2πp
different wavelengths to be analysed; in this way, the absorption
1
spectrum is built up one narrow wavelength range at a time.
In a polychromator, there is no slit and a broad range of wave-
lengths can be analysed simultaneously by array detectors, such 0.5
as those discussed below.
Currently, almost all spectrometers operating in the infra-
0
red and near-infrared use ‘Fourier transform’ techniques for 0 1 2 3 4
p
spectral detection and analysis. (Fourier transforms are dis-
cussed, but in more detail than needed here, in The chemist’s
Figure 11A.10 An interferogram produced as the path length
toolkit 12C.1.) The heart of a Fourier transform spectrometer difference p is changed in the interferometer shown in Fig. 11A.9.
is a Michelson interferometer, a device for analysing the wave- Only a single wavelength component is present in the signal, so
lengths present in a composite signal. The Michelson interfer- the graph is a plot of 1 cos(2 p ) .
ometer works by splitting the beam after it has passed through
the sample into two and arranging for them to take different s­ pectrometer. Each component gives rise an interference pat-
routes through the instrument before eventually recombining tern proportional to 1  cos(2p ), and the signal recorded by
at the detector (Fig. 11A.9). One beam is reflected from mirror the detector is their sum. Thus, if the intensity of the radiation
M1 and one from mirror M2; by moving M1 it is therefore possi- entering the spectrometer consists of a mixture of wavenumber
ble to introduce a difference in the length of the path traversed νi with intensities I (νi ), the signal measured at the detector is
by the two beams. given by the sum
Consider first the simplest case in which a beam of mono-
chromatic light of wavelength λ is passed into the interferom- I( p)  I (i ){1  cos(2i p)} (11A.14)
i
eter. If the path length difference p is 0, the two beams interfere
constructively; the same is true if p is an integer number of A plot of I( p) against p, which is detected by the system
wavelengths: λ, 2λ, 3λ,…. If p is one half of a wavelength, the and recorded, is called an interferogram. The problem is to
two beams interfere destructively and cancel; the same is true find I(ν ), the variation of the intensity with wavenumber, which
if p is an odd multiple of half-wavelengths: λ/2, 3λ/2, 5λ/2,…. is the spectrum, from the measured I( p). This conversion can be
Therefore, as the mirror M1 is moved the detected signal goes carried out by using a standard mathematical technique, called
through a series of peaks and troughs depending on whether the the Fourier transform, which involves evaluating the integral
two beams interfere constructively or destructively, and the net  Fourier
I ( )  4  {I( p)  12 I(0)}cos(2 p)dp  (11A.15)
signal varies as 1 + cos(2πp/λ), or 1  cos(2p ) (Fig. 11A.10). 0 transform
In a spectroscopic observation a mixture of radiation In practice, the measured values of I( p) are digitized, stored in
of different wavelengths and intensities is passed into the a computer attached to the spectrometer, and then the Fourier
transform is computed numerically.

Movable mirror, M1
Example 11A.2 Relating a spectrum to an interferogram
Beam Compensator
splitter Suppose the light entering the interferometer consists of three
components with the following characteristics:

i /cm 1 150 250 450


Mirror, M2
Detector I (νi ) 1 3 6
where the intensities are relative to the first value listed. Plot the
Figure 11A.9 A Michelson interferometer. The beam-splitting
interferogram associated with this signal. Then calculate and
element divides the incident beam into two beams with a path
plot the Fourier transform of the interferogram.
difference that depends on the location of the movable mirror M1.
The compensator ensures that both beams pass through the same Collect your thoughts For a signal consisting of just these three
thickness of material. component beams, you can use eqn 11A.14 directly. Although
438 11 Molecular spectroscopy

in this case (where I( p) is simply the sum of trigonometric


functions) the Fourier transform I(ν ) can be carried out exact-
ly, in general it is best done numerically by using mathematical
software.

Intensity, I(p)
~
The solution From the data, the interferogram is

I( p)  (1  cos 21 p)  3  (1  cos 2 2 p)  6  (1  cos 23 p)


 10  cos 21 p  3 cos 22 p  6 cos 23 p

This function is plotted in Fig. 11A.11. The result of evaluating


the Fourier transform numerically is shown in Fig. 11A.12. Path length difference, p

Figure 11A.13 The interferogram calculated from the data in


Self-test 11A.2 superimposed on the interferogram obtained in
the Example itself.
Answer: Fig. 11A.13
Intensity, I(p)
~

11A.3(c) Detectors
A detector is a device that converts radiation into an electric
signal for processing and display. Detectors may consist of a
single radiation sensing element or of several small elements
arranged in one- or two-dimensional arrays.
Path length difference, p A microwave detector is typically a crystal diode consisting of
a tungsten tip in contact with a semiconductor. The most com-
Figure 11A.11 The interferogram calculated from data in mon detectors found in commercial infrared spectrometers are
Example 11A.2. sensitive in the mid-infrared region. In a photovoltaic device the
potential difference changes upon exposure to radiation. In a
pyroelectric device the capacitance is sensitive to temperature
6 and hence to the presence of infrared radiation.
A common detector for work in the ultraviolet and visible
5 ranges is a photomultiplier tube (PMT), in which the photoe-
lectric effect (Topic 7A) is used to generate an electrical signal
Intensity, I(ν)

4
~

proportional to the intensity of light that strikes the detector. A


3 common, but less sensitive, alternative to the PMT is a photodi-
ode, a solid-state device that conducts electricity when struck
2
by photons because light-induced electron transfer reactions in
1 the detector material create mobile charge carriers (negatively
charged electrons and positively charged ‘holes’).
00 100 200 300 A charge-coupled device (CCD) is a two-dimensional array of
~ 400 500
ν/cm–1 several million small photodiode detectors. With a CCD, a wide
range of wavelengths that emerge from a polychromator are de-
Figure 11A.12 The Fourier transform of the interferogram tected simultaneously, thus eliminating the need to measure the
shown in Fig. 11A.11. The oscillations arise from the way radiation intensity one narrow wavelength range at a time.
that the signal in Fig. 11A.11 is sampled. As the sampling is
extended to greater path-length differences, the oscillations
disappear, and the peaks become sharper. Interact with the 11A.3(d) Examples of spectrometers
dynamic version of this graph in the e-book. With an appropriate choice of spectrometer, absorption spec-
troscopy can be used to probe electronic, vibrational, and rota-
tional transitions in molecules. It is often necessary to modify
Self-test 11A.2 Explore the effect of varying the wavenumbers the general design of Fig. 11A.6 in order to detect weak signals.
of the three components of the radiation on the shape of the For example, to detect rotational transitions with a microwave
interferogram by changing the value of ν3 to 550 cm−1. spectrometer it is useful to modulate the transmitted intensity
11A General features of ­molecular spectroscopy 439

by varying the energy levels with an oscillating electric field.


In this Stark modulation, an electric field of about 105 V m−1
Source
(1 kV cm−1) and a frequency of between 10 and 100 kHz is ap-
Sample
plied to the sample.
In a typical Raman spectroscopy experiment, a monochro-
Monochromator
matic incident laser beam is scattered from the front face of
Detector or interferometer
the sample and monitored (Fig. 11A.14). Lasers are used as the
source of the incident radiation because the scattered beam is
then more intense. The monochromatic character of laser radi- Figure 11A.14 A common arrangement adopted in Raman
spectroscopy. A laser beam passes through a lens and then
ation makes possible the observation of Stokes and anti-Stokes
through a small hole in a mirror with a curved reflecting surface.
lines with frequencies that differ only slightly from that of the
The focused beam strikes the sample and scattered light is both
incident radiation. Such high resolution is particularly useful deflected and focused by the mirror. The spectrum is analysed by
for observing rotational transitions by Raman spectroscopy. a monochromator or an interferometer.

Checklist of concepts
☐ 1. In emission spectroscopy the electromagnetic radiation ☐ 7. Stimulated emission is a process in which a transition
that arises from molecules undergoing a transition from from a high energy state to one of lower energy is driven
a higher energy state to a lower energy state is detected. by an electromagnetic field oscillating at the transition
☐ 2. In absorption spectroscopy, the net absorption of frequency; its rate is determined in part by the Einstein
radiation passing through a sample is monitored. coefficient of stimulated emission.
☐ 3. In Raman spectroscopy, changes in molecular state are ☐ 8. Spontaneous emission is the transition from a high
explored by examining the energies (frequencies) of the energy state to a lower energy state at a rate independent
photons scattered by molecules. of any radiation also present. The relative importance of
spontaneous emission increases as the cube of the tran-
☐ 4. Photons that are scattered elastically give rise to Rayleigh
sition frequency.
scattering.
☐ 9. A gross selection rule specifies the general features a
☐ 5. In Stokes scattering a photon gives up some of its ener-
molecule must have if it is to have a spectrum of a given
gy to a molecule; in anti-Stokes scattering the photon
kind; a specific selection rule expresses the allowed
gains energy from the molecule.
transitions in terms of the changes in quantum numbers.
☐ 6. Stimulated absorption is a process in which a transition
☐ 10. Collisional line broadening arises from the shortened
from a low energy state to one of higher energy is driven
lifetime due to collisions. The broadening is propor-
by an electromagnetic field oscillating at the transition
tional to the pressure, and is often termed pressure
frequency; its rate is determined in part by the Einstein
broadening.
coefficient of stimulated absorption.

Checklist of equations
Property Equation Comment Equation number
3 3
Ratio of Einstein coefficients of spontaneous and Al,u = (8πhν /c )Bl,u Bu,l = Bl,u 11A.6a
stimulated emission
Transition dipole moment µfi = ∫ψ f µˆψ i dτ Electric dipole transitions 11A.7
Beer–Lambert law I = I010−ε[J]L Uniform sample 11A.8
Absorbance and transmittance A = log(I0/I) = −log T Definition 11A.9b
Integrated absorption coefficient A   band  ( )d Definition 11A.10
Doppler broadening δνobs = (2ν0/c)(2kT ln 2/m)1/2 11A.12a
Lifetime broadening δE/h = 1/2πτ τ is the lifetime of the state 11A.13
TOPIC 11B Rotational spectroscopy

I = 3mAxA2 + 3mDxD2
➤ Why do you need to know this material?
xA
Rotational spectroscopy provides very precise values of
bond lengths, bond angles, and dipole moments of mol- mA
ecules in the gas phase. mB
mC
➤ What is the key idea? mD
The spacing of the lines in rotational spectra is used to
determine the rotational constants of molecules and, xD
through them, values of their bond lengths and angles.

➤ What do you need to know already?


Figure 11B.1 The definition of moment of inertia. In this molecule
You need to be familiar with the classical description of
there are three atoms with mass mA attached to the B atom and
rotational motion (Energy: A first look), the quantization of three atoms with mass mD attached to the C atom. The moment of
angular momentum (Topic 7F), the general principles of inertia about the axis passing through the B and C atoms depends
molecular spectroscopy (Topic 11A), and the Pauli princi- on the perpendicular distance xA from this axis to the A atoms, and
ple (Topic 8B). You also need to be familiar with the con- the perpendicular distance xD to the D atoms.
cepts of dipole moment and polar molecules, as described
in introductory chemistry courses.
perpendicular axes, q = x, y, z. For linear molecules, the moment
of inertia around the internuclear axis is zero (­ because xi = 0 for
all the atoms) and the two remaining moments of inertia, which
are equal, are denoted simply I. Explicit expressions for the mo-
Pure rotational spectra, in which only the rotational state of a ments of inertia of some symmetrical molecules are given in
molecule changes, can be observed only in the gas phase. In Table 11B.1. The principal moments of inertia are also com-
spite of this limitation, rotational spectroscopy can provide a monly recorded as Ia, Ib, and Ic, with Ic ≥ Ib ≥ Ia.
wealth of information about molecules, including precise bond The energy of a body free to rotate about three axes is
lengths, bond angles, and dipole moments.
E  12 I x x2  12 I y  y2  12 I z z2 (11B.3)

The classical angular momentum about the axis q is Jq = Iqωq. It


follows that
11B.1 Rotational energy levels
J x2 J 2y J2 Rotational energy: classical
E   z expressio
on
 (11B.4)
The classical expression for the energy of a body rotating about 2I x 2I y 2I z
an axis q is

Eq  12 I qq2 (11B.1) Table 11B.1 Moments of inertia⋆

where ωq is the angular velocity about the axis q = x, y, z and Iq is 1. Diatomic molecules
the corresponding moment of inertia. The moment of i­ nertia, R mA mB
mA mB I   R2 
I, of a molecule about an axis passing through the centre of m
mass is defined as (Fig. 11B.1)
2. Triatomic linear rotors
Moment of inertia
I  mi xi2 [definition]
 (11B.2)
R R´ I  mA R 2  mC R2
i mA mC
mB (mA R  mC R)2
where mi is the mass of the atom i treated as a point and xi is its 
m
perpendicular distance from the axis of rotation. In general, the
rotational properties of any molecule can be expressed in terms R R I = 2mA R 2
mA mA
of its three principal moments of inertia Iq about three ­mutually mB
11B Rotational spectroscopy 441

3. Symmetric rotors Example 11B.1 Calculating the moment of inertia


I||  2mA f1 ( )R 2
of a molecule
mC I   mA f1 ( )R 2 Calculate the moment of inertia of a 1H2O molecule around
R´ m the axis defined by the bisector of the HOH angle (1). The
mB  A (mB  mC ) f 2 ( )R 2
R m HOH bond angle is 104.5° and the OH bond length is
mC
 {(3mA  mB )R 95.7 pm.
mA θ m
mA
6mA R  13 f 2 ( ) }R
1/ 2

mA

ϕ/2
xH
1
mB I||  2mA f1 ( )R 2
R Collect your thoughts You can compute the moment of inertia
I   mA f1 ( )R 2
by using eqn 11B.2. The mass to use is the actual atomic mass
mA m m
θ mA  A B f 2 ( )R 2 (expressed in terms of the atomic mass constant mu), not the
m
element’s molar mass. The xi are the perpendicular distances
mA
from each atom to the axis of rotation, and you will be able to
calculate these distances by using trigonometry and the bond
angle and bond length.
I||  4mA R 2
The solution From eqn 11B.2,
mC I   2mA R 2  2mC R2
R´ mA I  mi xi2  mH x H2  0  mH x H2  2mH x H2
mA mB mA i
R
In this case, mH is the mass of a 1H atom, so mH = 1.0078mu. If
the bond angle of the molecule is φ and the OH bond length is
mA
R´ R, trigonometry gives x H  R sin 12  . It follows that
mC I  2mH R2 sin2 12 

Substitution of the data gives

I  2  (1.0078  1.6605  1027 kg )  (9.57  10 11 m)2


4. Spherical rotors
 sin2  12  104.5 
I = 83 mA R 2
 1.92  1047 kg m 2
mA

mB
Note that the mass of the O atom makes no contribution to
R
the moment of inertia because the axis passes through this
atom.
mA mA
Self-test 11B.1 Calculate the moment of inertia of a CH35Cl3
mA molecule around a rotational axis along the C−H bond. The
C−Cl bond length is 177 pm and the HCCl angle is 107°;
m(35Cl) = 34.97mu.
I = 4mA R 2
Answer: 4.99 × 10−45 kg m2

mA
mA
mA mB
R 11B.1(a) Spherical rotors
mA Spherical rotors have all three moments of inertia equal, as in
mA CH4 and SF6. If these moments of inertia have the value I, the
classical expression for the energy is
mA
J x2  J 2y  J z2 J2
E   (11B.5)

f1(θ) = 1 − cos θ, f2(θ) = 1 + 2cos θ; in each case, m is the total mass of the molecule. 2I 2I
442 11 Molecular spectroscopy

where J 2 is the square of the magnitude of the angular momen- Because the rotational constant is inversely proportional to I,
tum. The corresponding quantum expression is generated by large molecules have closely spaced rotational energy levels.
making the replacement

J 2  J ( J  1)2 J  0, 1, 2,  Brief illustration 11B.1

where J is the angular momentum quantum number. Therefore, Consider 12C35Cl4. From Table 11B.1 and given the C−Cl
the energy of a spherical rotor is confined to the values bond length (RC−Cl = 177 pm) and the mass of the 35Cl nuclide
(m(35Cl) = 34.97mu), the moment of inertia is
2 Energy levels of
E J  J ( J  1) J  0, 1, 2,  or  (11B.6)
a spherical roto  34.97 mu

2I 35
I  83 m( Cl )R 2
C  C1  83  (5.807  1026 kg )  (1.77  10 10 m)2
The resulting ladder of energy levels is illustrated in Fig. 11B.2.  4.85 1045 kg m 2
The energy is normally expressed in terms of the rotational
From eqn 11B.7 the rotational constant is
constant, B (a wavenumber), of the molecule, where
kg m2 s2

2  Rotational constant 1.05457  10 Js 34
hcB  so B  [definition]
 (11B.7) B 
2I 4 cI 4   (2.998  108 ms 1 )  (4.85 1045 kg m2 )
The expression for the energy is then  5.77 m 1  0.0577 cm 1

 ( J  1) J  0, 1, 2,  Energy levels of It follows from eqn 11B.10 that the separation between the
E J  hcBJ a spherical rotor  (11B.8) J = 0 and J = 1 terms is F (1)  F (0)  2 B  0.1154 cm 1, corre-
sponding to 3.46 GHz.
It is also common to express the rotational constant as a fre-
quency and to denote it B. Then B = ħ/4πI and the energy is
EJ = hBJ(J + 1). The two quantities are related by B = cB .
The energy of a rotational state is normally reported as the
rotational term, F ( J ), a wavenumber, by division of both sides 11B.1(b) Symmetric rotors
of eqn 11B.8 by hc: Symmetric rotors have two equal moments of inertia and a third
that is non-zero. In group theoretical terms (Topic 10A), such
F ( J )  BJ
 ( J 1) Rotational terms of spherical rotor  (11B.9) rotors have an n-fold axis of rotation, with n > 2. The unique
axis of a symmetric rotor (such as CH3Cl, NH3, and C6H6) is its
To express the rotational term as a frequency, use F = cF . The principal axis (or figure axis). If the moment of inertia about
separation of successive terms is the principal axis is larger than the other two, the rotor is clas-
sified as oblate (like a pancake, and C6H6). If the moment of in-
F ( J 1)  F ( J )  B ( J 1)( J  2)  BJ
 ( J 1)  2 B ( J 1) (11B.10) ertia around the principal axis is smaller than the other two, the
rotor is classified as prolate (like a rugby ball or American foot-
ball, and CH3Cl). The two equal moments of inertia (Ix and Iy)
are denoted I⊥ and Iz is denoted I||. Then eqn 11B.4 becomes
16 J

15 J x2  J 2y J z2
E   (11B.11)
Energy

14 2I  2 I||
13
8hcB
~ 12 This expression can be written in terms of J 2  J x2  J 2y  J z2:
11
~
6hcB 10 J 2  J z2 J z2 J2  1 1  2
E      Jz  (11B.12)
~ 9 2I  
2 I|| 2 I   2 I|| 2 I  
4hcB 8 
~ 7 4
2hcB
6 3
2
The quantum expression is generated by replacing J 2 by
5
1
0
J(J + 1)ħ2. The quantum theory of angular momentum
0
(Topic 7F) also restricts the component of angular momentum
Figure 11B.2 The rotational energy levels of a linear or spherical about any axis to the values Kħ, with K = 0, ±1,…, ±J. Then,
rotor. Note that the energy separation between successive levels after making the replacements J 2  J ( J  1)2 and J z2 → K 2 2
increases as J increases. the rotational terms are
11B Rotational spectroscopy 443

 ( J  1)  ( A  B )K 2
F ( J ,K )  BJ Rotational teerms of
a symmetric rotor  (11B.13a) J
J  0,1,2, K  0, 1,,  J

with J

 
A  B   (11B.13b)
4 cI|| 4 cI 
K=0 KJ
Equation 11B.13a matches what is expected for the dependence (a) (b)
of the energy levels on the two distinct moments of inertia of Figure 11B.3 The significance of the quantum number K.
the molecule:1 (a) When K = 0 the molecule has no angular momentum about its
• When K = 0, there is no component of angular momen- principal axis: it is undergoing end-over-end rotation. (b) When |K|
is close to its maximum value, J, most of the molecular rotation is
tum about the principal axis, and the energy levels depend
around the principal axis.
only on I⊥ (Fig. 11B.3a).
• When K = ±J, almost all the angular momentum arises
from rotation around the principal axis, and the energy The energy of a symmetric rotor depends on J and K. Because
levels are determined largely by I|| (Fig. 11B.3b). the states with K and −K have the same energy, each level, ex-
cept those with K = 0, is doubly degenerate. In addition, the
• The sign of K does not affect the energy because opposite
angular momentum of the molecule has a component on an
values of K correspond to opposite senses of rotation, and
external, laboratory-fixed axis. This component is quantized,
the energy does not depend on the sense of rotation.
and its permitted values are MJħ, with MJ = 0, ±1,…, ±J, giving
2J + 1 values in all (Fig. 11B.4). The quantum number MJ does
Example 11B.2 Calculating the rotational energy levels not appear in the expression for the energy, but it is necessary for
of a symmetric rotor a complete specification of the state of the rotor. Consequently,
all 2J + 1 orientations of the rotating molecule have the same
A 14N1H3 molecule is a symmetric rotor with bond length 101.2 pm
energy. It follows that a symmetric rotor level is 2(2J + 1)-fold
and HNH bond angle 106.7°. Calculate its rotational terms.
degenerate for K ≠ 0 and (2J + 1)-fold degenerate for K = 0.
Collect your thoughts Begin by calculating the moments of A spherical rotor can be regarded as a version of a symmetric
inertia by using the expressions given in Table 11B.1. Then use rotor in which I⊥ = I|| and therefore A = B . The quantum num-
eqn 11B.13a to find the rotational terms. The rotational con- ber K still takes any one of 2J + 1 values, but the energy is inde-
stants are found by using eqn 11B.13b. pendent of which value it takes. Therefore, as well as having a
The solution Substitution of mA = 1.0078mu, mB = 14.0031mu, (2J + 1)-fold degeneracy arising from its orientation in space,
R = 101.2 pm, and θ = 106.7° into the second of the sym- the rotor also has a (2J + 1)-fold degeneracy arising from its
metric rotor expressions in Table 11B.1 gives I|| = 4.4128 ×
10−47 kg m2 and I⊥ = 2.8059 × 10−47 kg m2. The expressions in z
MJ
eqn. 11B.13b give A  6.344 cm 1 and B  9.977 cm 1. It fol- J
lows from eqn 11B.13a that
F ( J ,K )/cm 1  9.977 J ( J  1)  3.633K 2
MJ = 0
Multiplication by c converts F ( J ,K ) to a frequency, denoted F(J,K):
F ( J ,K )/GHz  299.1J ( J  1)  108.9K 2
If J = 1, F(1,1) = 16.32 cm−1 (489.3 GHz) for K = ±1, and F(1,0) =
19.95 cm−1 (598.1 GHz) for K = 0.
(a) (b) (c)
Self-test 11B.2 A 12C1H335Cl molecule has a C−Cl bond length
of 178 pm, a C−H bond length of 111 pm, and an HCH angle of Figure 11B.4 The significance of the quantum number MJ.
110.5°. Identify whether the molecule is oblate or prolate, and (a) When MJ is close to its maximum value, J, most of the
calculate its rotational energy terms. molecular rotation is around the laboratory axis (taken as
A  5.0275 cm 1 and B  0.4470 cm 1 ; F ( J ,K )/cm 1  0.447 J ( J  1)  4.58K 2 the z-axis). (b) An intermediate value of MJ. (c) When MJ = 0
Answer: I⊥ = 6.262 × 10−46 kg m2, I|| = 5.568 × 10−47 kg m2; prolate;
the molecule has no angular momentum about the z-axis.
All three diagrams correspond to a state with K = 0; there are
1
The quantum number K is used to signify the component on the princi- corresponding diagrams for different values of K, in which the
pal axis, as distinct from the quantum number MJ, which is used to signify the angular momentum makes a different angle to the principal axis
component on an externally defined axis. of the molecule.
444 11 Molecular spectroscopy

orientation with respect to an arbitrary axis in the molecule.


Centrifugal
The overall degeneracy of a symmetric rotor energy level with force
quantum number J is therefore (2J + 1)2. This degeneracy in-
creases very rapidly: when J = 10, for instance, there are 441
states of the same energy.

11B.1(c) Linear rotors


Figure 11B.5 The effect of rotation on a molecule. The centrifugal
For a linear rotor (such as CO2, HCl, and C2H2), in which the force arising from rotation distorts the molecule, opening out
atoms are regarded as mass points, the rotation occurs only bond angles and stretching bonds slightly. The effect is to increase
about an axis perpendicular to the internuclear axis and there is the moment of inertia of the molecule and hence to decrease its
no rotation around that axis. Therefore the component of angu- rotational constant.
lar momentum around the internuclear axis of a linear rotor is
identically zero, and K ≡ 0 in eqn 11B.13a. The rotational terms The parameter D J is the centrifugal distortion constant. The cen-
of a linear molecule are therefore trifugal distortion constant of a diatomic molecule is related to the
vibrational wavenumber of the bond, ν (which, as seen in Topic
Rotational terms
F ( J )  BJ
 ( J  1) J  0, 1, 2, 
of linear rotor
 (11B.14) 11C, is a measure of its stiffness), through the approximate relation
4 B 3
This expression is the same as eqn 11B.9 but arrived at it in a D J  2 Centrifugal distortion constant (11B.16)
significantly different way: here K ≡ 0, but for a spherical rotor 
A = B and K has a range of values. The angular momentum of a As expected, a bond that is easily stretched, and therefore has a low
linear rotor has 2J + 1 components on an external axis, so its de- vibrational wavenumber, has a high centrifugal distortion constant.
generacy is just 2J + 1 rather than the (2J + 1)2-fold degeneracy
of a spherical rotor.
Brief illustration 11B.3

For 12C16O, B  1.931 cm 1 and   2170 cm 1. It follows that


Brief illustration 11B.2
4  (1.931 cm 1 )3
Equation 11B.10 for the energy separation of successive D J   6.116  106 cm 1
(2170 cm 1 )2
levels of a spherical rotor also applies to linear rotors, so
F (3)  F (2)  6 B . Spectroscopic measurements on 1H35Cl give Because D J << B , centrifugal distortion has a very small
F (3)  F (2)  63.56 cm 1, so it follows that 6 B  63.56 cm 1, effect on the energy levels until J is large. For J = 20,
B  10.59 cm 1 , and therefore D J J 2 ( J  1)2  1.08 cm 1 (corresponding to 32 GHz).

 1.05457  1034 Js
I 
4 cB 4   (2.998  1010 cms 1 )  (10.59 cm 1 ) Exercises
47 2
 2.643  10 kg m E11B.1 Calculate the moment of inertia around the bisector of the OOO angle
and the corresponding rotational constant of an 16O3 molecule (bond angle
117°; OO bond length 128 pm).
11B.1(d) Centrifugal distortion E11B.2 Classify the following rotors: (i) O3, (ii) CH3CH3, (iii) XeO4, (iv) FeCp2
(Cp denotes the cyclopentadienyl group, C5H5).
In the discussion so far molecules have been treated as rigid
E11B.3 Calculate the HC and CN bond lengths in HCN from the rotational
rotors. However, the atoms of rotating molecules are subject to
constants B(1H12C14N) = 44.316 GHz and B(2H12C14N) = 36.208 GHz.
centrifugal forces, which tend to distort the molecular geom- 1 127
E11B.4 Estimate the centrifugal distortion constant for H I, for which
etry and change its moments of inertia (Fig. 11B.5). The effect
B  6.511 cm 1 and   2308 cm 1 . By what factor would the constant change
of centrifugal distortion on a diatomic molecule is to stretch when 2H is substituted for 1H?
the bond and hence to increase the moment of inertia. As a re-
sult, the rotational constant is reduced and the energy levels are
slightly closer together than the rigid-rotor expressions predict.
The effect is usually taken into account by including in the en-
ergy expression a negative term that becomes more important
11B.2 Microwave spectroscopy
as J increases:
Typical values of the rotational constant B for small molecules
F ( J )  BJ
 ( J  1)  D J 2 ( J  1)2 Rotational terms affected are in the region of 0.1–10 cm−1; two examples are 0.356 cm−1
J by centtrifugal distortion (11B.15)
for NF3 and 10.59 cm−1 for HCl. It follows that rotational
11B Rotational spectroscopy 445

Photon
µ µ

Figure 11B.6 To a stationary observer, a rotating polar molecule


looks like an oscillating dipole which will generate an oscillating
electromagnetic wave (or, in the case of absorption, interact
with such a wave). This picture is the classical origin of the gross
selection rule for rotational transitions. Figure 11B.7 When a photon is absorbed by a molecule, the
angular momentum of the combined system is conserved. If the
molecule is rotating in the same sense as the spin of the incoming
t­ransitions can be studied with microwave spectroscopy, a photon, then J increases by 1.
technique that monitors the absorption of radiation in the mi-
crowave region of the spectrum.

The transition ∆J = +1 corresponds to absorption and the tran-


11B.2(a) Selection rules sition ∆J = −1 corresponds to emission.
As usual in spectroscopy, the selection rules can be established
• The allowed change in J arises from the conservation of
by considering the relevant transition dipole moment. The
angular momentum when a photon, a spin-1 particle, is
details of the calculation are shown in A deeper look 11B.1,
emitted or absorbed (Fig. 11B.7).
available to read in the e-book accompanying this text. The
conclusion is that • The allowed change in MJ also arises from the conserva-
tion of angular momentum when a photon is emitted or
The gross selection rule for the observation of a pure absorbed in a specific direction.
rotational transition is that a molecule must have a
permanent electric dipole moment. When the transition moment is evaluated for all possible
relative orientations of the molecule to the line of flight of the
The classical basis of this rule is that a polar molecule appears photon, it is found that the total J + 1 ↔ J transition intensity is
to possess a fluctuating dipole when rotating, but a nonpolar proportional to
molecule does not (Fig. 11B.6). The permanent dipole can be
regarded as a handle with which the molecule stirs the electro- J 1 2
magnetic field into oscillation (and vice versa for absorption). |  J 1,J |2  0  (11B.18)
2J  1

Brief illustration 11B.4 where μ0 is the permanent electric dipole moment of the
molecule. The intensity is proportional to the square of μ0, so
Homonuclear diatomic molecules and nonpolar polyatomic strongly polar molecules give rise to much more intense rota-
molecules such as CO2, CH2=CH2, and C6H6 do not give rise to tional lines than less polar molecules.
microwave spectra. On the other hand, OCS and H2O are polar Rotation of a symmetric rotor about its principal (figure)
and have microwave spectra. Spherical rotors cannot have axis does not lead to any change in the orientation of the dipole;
electric dipole moments unless they become distorted by rota- there is no fluctuating dipole to interact with the radiation, and
tion, so they are rotationally inactive except in special cases. therefore no change in K is possible. For symmetric rotors the
An example of a spherical rotor that does become sufficiently selection rules are therefore:
distorted for it to acquire a dipole moment is SiH4, which has a
dipole moment of about 8.3 μD by virtue of its rotation when Rotational selection rules:
J ≈ 10 (for comparison, HCl has a permanent dipole moment of J  1 M J  0,  1 K  0 symmetric rotors
 (11B.19)
1.1 D; molecular dipole moments and their units are discussed
in Topic 14A). The degeneracy associated with the quantum number MJ (the
orientation of the rotation in space) is partly removed when an
The analysis also shows that, for a linear molecule, the tran- electric field is applied to a polar molecule (Fig. 11B.8). The
sition moment vanishes unless the following conditions are splitting of states by an electric field is called the Stark ­effect.
­fulfilled: The energy shift depends on the permanent electric dipole mo-
ment, μ0, so the observation of the Stark effect in a rotational
Rotational selection rules: spectrum can be used to measure the magnitudes of electric
J  1 M J  0, 1 linear rotors  (11B.17) dipole moments.
446 11 Molecular spectroscopy

Field on
0 M J
±1 J

Energy

...
±2 5
4
±3 3
2
±4 1
Field off 0
±5

1←0
Transmittance

2←1
Frequency

3←2
±6

4←3
5←4
±7

Figure 11B.8 The effect of an electric field on the energy level

6←5

8←7
7←6
etc
with J = 7 of a polar linear rotor. All levels are doubly degenerate
except that with MJ = 0. Figure 11B.9 The rotational energy levels of a linear rotor, the
transitions allowed by the selection rule ΔJ = +1, and a typical
The appearance of microwave
11B.2(b) pure rotational absorption spectrum (displayed here in terms of
spectra the radiation transmitted through the sample). The intensities
reflect the populations of the initial level in each case and the
When the selection rules are applied to the expressions for the strengths of the transition dipole moments.
energy levels of a linear rigid rotor (eqn 11B.14), it follows that
the wavenumbers of the allowed J + 1 ← J absorptions are The form of the spectrum predicted by eqn 11B.20a is shown
in Fig. 11B.9. The most significant feature is that it consists of a
 ( J  1  J )  F ( J  1)  F ( J )  2 B ( J  1) J  0, 1, 2,  series of lines with wavenumbers 2 B,  4 B,
 6 B,…
 and of separa-
Wavenumbers of rotational transitions: linear rotor (11B.20a)  The measurement of the line spacing therefore gives B,
tion 2 B. 
and hence the moment of inertia I⊥ perpendicular to the prin-
When centrifugal distortion is taken into account, the corre- cipal axis of the molecule. Because the masses of the atoms are
sponding expression obtained from eqn 11B.15 is known, it is a simple matter to deduce the bond length of a dia-
 ( J  1  J )  2 B ( J  1)  4 D J ( J  1)3 (11B.20b) tomic molecule. However, in the case of a polyatomic molecule
such as OCS or NH3, a knowledge of one moment of inertia is
However, because the second term is typically very small com- insufficient data from which to infer, for example, the two bond
pared with the first, the appearance of the spectrum closely re- lengths in OCS, or the bond length and bond angle in NH3.
sembles that predicted from eqn 11B.20a. This difficulty can be overcome by measuring the spectra of
isotopologues, isotopically substituted molecules. The spec-
Example 11B.3 Predicting the appearance of a rotational trum from each isotopologue gives a separate moment of in-
spectrum ertia and, if it is assumed that the bond lengths and angles are
unaffected by isotopic substitution, the extra data make it pos-
Predict the form of the rotational spectrum of 14NH3, which is
sible to extract values of the bond lengths and angles. A good
an oblate symmetric rotor with B  9.977 cm 1.
example of this procedure is the study of OCS; the actual cal-
Collect your thoughts The rotational terms are given by culation is worked through in Problem P11B.7. The assump-
eqn 11B.13a. Because ∆J = ±1 and ∆K = 0, the expression for tion that bond lengths are unchanged in isotopologues is only
the wavenumbers of the rotational transitions is identical to an approximation, but it is a good one in most cases. Nuclear

eqn 11B.20a and depends only on B. spin (Topic 12A), which differs from one isotope to another,
The solution The following table can be drawn up for the J + 1 ← J also affects the appearance of high-resolution rotational spectra
transitions. because spin is a source of angular momentum and can couple
with the rotation of the molecule itself and hence affect the ro-
J 0 1 2 3 … tational energy levels.
 /cm 1
19.95 39.91 59.86 79.82 … The intensities of spectral lines increase with increasing J
ν/GHz 598.1 1196 1795 2393 … and pass through a maximum before tailing off as J becomes
−1
large. The most important reason for this behaviour is the exist-
The line spacing is 19.95 cm (598.1 GHz).
ence of a maximum in the population of rotational levels. The
Self-test 11B.3 Repeat the problem for CH35
3 Cl, a prolate sym-
Boltzmann distribution (see Energy: A first look and Topic 13A)
metric rotor for which B  0.444 cm 1. implies that the population of a state decreases exponentially as
Answer: Lines of separation 0.888 cm−1 (26.6 GHz) its energy increases. However, the population of a level is also
proportional to its degeneracy, and in the case of rotational
11B Rotational spectroscopy 447

l­evels this degeneracy increases with J. These two opposite E


trends result in the population of the energy levels (as dis- Distortion
tinct from the individual states) passing through a maximum.
Specifically, the population NJ of a rotational energy level J is
given by the Boltzmann expression
E
 E J /kT
N J  Ng J e
where N is the total number of molecules in the sample and gJ is (a) (b)
the degeneracy of the level J. The value of J corresponding to a
maximum of this expression is found by treating J as a continu- Figure 11B.10 An electric field E applied to a molecule results in
a distortion of its electron density, and the molecule acquires a
ous variable, differentiating with respect to J, and then setting
contribution to its dipole moment (even if it is nonpolar initially).
the result equal to zero. The result for a linear rotor, for which
The polarizability may be different when the field is applied (a)
g J  2 J  1, (see Problem P11B.11) is parallel or (b) perpendicular to the molecular axis (or, in general, in
1/ 2 different directions relative to the molecule); if that is so, then the
 kT  Rotational level with largest
J max     12  (11B.21) molecule has an anisotropic polarizability.
 2hcB  popullation: linear rotor

For a typical molecule (e.g. OCS, with B  0.2 cm 1)


in addition to any permanent dipole moment it might have.
kT /2hcB ≈ 500 at room temperature, so Jmax ≈ 22. However,
An atom is isotropically polarizable: that is, the same distor-
the transition dipole moment depends on the value of J (eqn
tion is induced whatever the direction of the applied field. The
11B.18) and, because the radiation can also cause stimulated
polarizability of a spherical rotor is also isotropic. However,
emission (Topic 11A), the intensity also depends on the popu-
non-spherical rotors have polarizabilities that do depend on
lation difference between the two states involved in the transi-
the direction of the field relative to the molecule, so these mole-
tion. Hence the value of J corresponding to the most intense
cules are anisotropically polarizable (Fig. 11B.10). The electron
line is not quite the same as the value of J for the most highly
distribution in H2, for example, is more distorted when the field
populated level.
is applied parallel to the bond than when it is applied perpen-
dicular to it, and so α|| > α⊥.
All linear molecules, including both heteronuclear and
Exercises homonuclear diatomics, have anisotropic polarizabilities and
E11B.5 Which of the following molecules may show a pure rotational so are rotationally Raman active. This activity is one reason for
microwave absorption spectrum: (i) H2, (ii) HCl, (iii) CH4, (iv) CH3Cl,
the importance of rotational Raman spectroscopy, because the
(v) CH2Cl2?
technique can be used to study many of the molecules that are
E11B.6 Calculate the frequency and wavenumber of the J = 3 ← 2 transition
inaccessible to microwave spectroscopy. Spherical rotors such
in the pure rotational spectrum of 14N16O. The equilibrium bond length is
115 pm. Would the frequency increase or decrease if centrifugal distortion is as CH4 and SF6, however, are rotationally Raman inactive as
considered? well as microwave inactive. This inactivity does not mean that
27
E11B.7 The spacing of lines in the microwave spectrum of Al H is
1 such molecules are never found in rotationally excited states.
−1
12.604 cm ; calculate the moment of inertia and bond length of the molecule. Molecular collisions do not have to obey such restrictive selec-
E11B.8 What is the most highly populated rotational level of Cl2 at (i) 25 °C, tion rules, and hence collisions between molecules can result in
(ii) 100 °C? Take B  0.244 cm 1. the population of any rotational state.
As usual, to establish the selection rules, it is necessary to
consider the transition dipole moment. The full calculation can
be found in A deeper look 11B.1 , available to read in the e-book
11B.3 Rotational Raman spectroscopy accompanying this text, and leads to the conclusion that the
specific rotational Raman selection rules are
Raman scattering (Topic 11A) can also arise as a result of rota-
Linear rotors: J  0,  2
tional transitions.The gross selection rule for rotational Raman Rotatio
onal
Symmetric rotors : J  0, 1,  2 Raman  (11B.23)
transitions is that the molecule must be anisotropically po-
selection rules
larizable. To understand this criterion it is necessary to know K  0
that the distortion of the electron density of a molecule in an
The ∆J = 0 transitions do not lead to a shift in frequency of the
electric field is determined by its polarizability, α (Topic 14A).
scattered photon and therefore contribute to the unshifted ra-
More precisely, if the strength of the field is E, then the mol-
diation (Rayleigh scattering, Topic 11A). A classical argument
ecule acquires an induced dipole moment of magnitude
can be used to give physical insight into the quantum mechani-
   E (11B.22) cal calculation.
448 11 Molecular spectroscopy

How is that done? 11B.1 Δα ≠ 0; hence the polarizability must be anisotropic for there
Justifying the rotational Raman to be Raman lines. This is the gross selection rule for rotational
selection rules Raman spectroscopy.
The incident electric field, E, of a wave of electromagnetic The distortion induced in the molecule by the incident elec-
radiation of frequency ωi induces a molecular dipole moment tric field returns to its initial value after a rotation of 180° (i.e.
given by twice a revolution). This is the classical origin of the specific
selection rule ∆J = ±2.
ind   E (t )   E cos it

If the molecule is rotating at an angular frequency ωR, it To predict the form of the Raman spectrum of a linear rotor
appears to an external observer that the polarizability is also
the selection rule ∆J = ±2 is applied to the rotational energy lev-
time dependent (if it is anisotropic). This dependence can be
els (Fig. 11B.12). For Stokes lines, ∆J = +2 and the scattered ra-
written
diation is at a lower wavenumber than the incident radiation at
   0   cos 2Rt νi , the shift being the difference F ( J  2)  F ( J )

where Δα = α|| − α⊥ and α ranges from α0 + Δα to α0 − Δα as the


 ( J  2  J )  i  {F ( J  2)  F ( J )} Wavenumbers
molecule rotates. The 2ωR appears because the polarizability of Stokes lines:  (11B.24a)
returns to its initial value twice each revolution (Fig. 11B.11).  i  2 B (2 J  3) linear rotor
Combining these expressions gives
For anti-Stokes lines ∆J = −2 and the scattered radiation
μind = (α0 + Δα cos 2ωRt) × (E cos ωit) is at a higher wavenumber, the shift being the difference
= α0E cos ωit + E Δα cos ωit cos 2ωRt F ( J ) − F ( J − 2):
cos x cos y
= 21 {cos(x+ y) + cos(x− y)}  ( J  2  J )  i  {F ( J )  F ( J  2)} Wavenumbers of
anti-Stokes lines: (11B.24b)
= α0E cos ωit + 12 E Δα{cos(ωi + 2ωR)t + cos(ωi − 2ωR)t}    2 B (2 J  1)
i linear rotor 

This calculation shows that the induced dipole has a component The Stokes lines appear to low frequency of the incident radi-
oscillating at the incident frequency (which results in Rayleigh ation and at displacements 6 B,  10 B,
 14 B,…
 from νi for J = 0,
scattering), and that it also has components at ωi ± 2ωR, which
1, 2,…. The anti-Stokes lines appear to high frequency of the
give rise to the shifted Raman lines. These lines appear only if  10 B,
 14 B,…

incident radiation at displacements of 6 B, from νi
for J = 2, 3, 4,…. Note that J = 2 is the lowest state from which

E
J
Energy

...
α⊥ 5
E 0 4
E 3
2
1
0
/2π
3
/2π
1
Rayleigh line

Stokes Anti-Stokes
α|| lines lines
E α||
Signal

α⊥
9←7
8←6

6←4
5←3
4←2
3←1

0←2
←3
2←4
3←5
4←6

6←8
7←9
2←0
7←5

5←7

Frequency

Figure 11B.11 The distortion induced in the electron density of a Figure 11B.12 The rotational energy levels of a linear rotor and
molecule by an applied electric field returns the polarizability to the transitions allowed by the ΔJ = ±2 Raman selection rules. The
its initial value after a rotation of only 180° (i.e. twice a revolution). form of a typical rotational Raman spectrum is also shown. In
This is the origin of the ΔJ = ±2 selection rule in rotational Raman practice the Rayleigh line is much stronger than depicted in the
spectroscopy. figure.
11B Rotational spectroscopy 449

a transition obeying the selection rule ∆J = −2 can take place.


The separation of adjacent lines in both the Stokes and the anti-
Nuclear statistics and
11B.4
 so from the spacing I⊥ can be determined
Stokes regions is 4 B, rotational states
and then used to find the bond length exactly as in the case of
microwave spectroscopy. If eqn 11B.24 is used to analyse the rotational Raman spec-
trum of C16O2, the rotational constant derived from the spacing
of the lines is inconsistent with other measurements of C−O
bond lengths. The results are consistent if it is supposed that
Example 11B.4 Predicting the form of a Raman spectrum the molecule can exist only in states with even values of J, so the
Predict the form of the rotational Raman spectrum of 14N2, for
observed Stokes lines are 2 ← 0, 4 ← 2,…; the lines 3 ← 1,
which B  1.99 cm 1, when it is exposed to 336.732 nm laser 5 ← 3,… are missing.
radiation. The explanation of the missing lines lies in the Pauli princi-
ple (Topic 8B) and the fact that 16O nuclei are spin-0 bosons:
Collect your thoughts The molecule is rotationally Raman just as the Pauli principle excludes certain electronic states,
active because end-over-end rotation modulates its polarizabil- so too does it exclude certain molecular rotational states. The
ity as viewed by a stationary observer. The wavenumbers of the Pauli principle states that, when two identical bosons are ex-
Stokes and anti-Stokes lines are given by eqn 11B.24. changed, the overall wavefunction must remain unchanged.
The solution The incident radiation with wavelength 336.732 nm When a C16O2 molecule rotates through 180°, two identical
16
corresponds to a wavenumber of i  29697.2 cm 1; eqns O nuclei are interchanged, so the overall wavefunction of the
11B.24a and 11B.24b give the following line positions: molecule must remain unchanged. However, inspection of the
form of the rotational wavefunctions (which have the same an-
J 0 1 2 3 gular dependence as the s, p, etc. orbitals of atoms) shows that
Stokes lines they change sign by (−1)J under such a rotation (Fig. 11B.13).
 /cm 1 29 685.3 29 677.3 29 669.3 29 661.4 Therefore, only even values of J are permissible for C16O2, and
hence the Raman spectrum shows only alternate lines.
The selective existence of rotational states that stems from
J 0 1 2 3
the Pauli principle is termed nuclear statistics. Nuclear sta-
Anti-Stokes lines tistics must be taken into account whenever a rotation inter-
 /cm 1 29 709.1 29 717.1 changes equivalent nuclei. However, the consequences are not
always as simple as for C16O2 because there are complicating
There will be a strong central line at 336.732 nm accompanied
features when the nuclei have non-zero spin: it is found that
on either side by lines of increasing and then decreasing inten-
sity (as a result of transition moment and population effects).
there are several different relative nuclear spin orientations
The spread of the entire spectrum is very small, so the incident consistent with even values of J and a different number of spin
light must be highly monochromatic. orientations consistent with odd values of J. For 1H2 and 19F2,
which have two identical spin- 12 nuclei, by using the Pauli prin-
Self-test 11B.4 Repeat the calculation for the rotational Raman ciple it can be shown that there are three times as many ways
spectrum of 35Cl2 (B  0.9752 cm 1 ). of achieving a state with odd J than with even J, and there is
a corresponding 3:1 alternation in intensity in their rotational
anti-Stokes lines at 29 703.1, 29 707.0 cm−1
Answer: Stokes lines at 29 691.3, 29 687.4, 29 683.5, 29 679.6 cm−1,
Raman spectra (Fig. 11B.14).

Exercises +
+ +
+
E11B.9 Which of the following molecules may show a pure rotational Raman –

spectrum: (i) H2, (ii) HCl, (iii) CH4, (iv) CH3Cl? –
E11B.10 The wavenumber of the incident radiation in a Raman spectrometer is J=0 J=1 J=2
20 487 cm−1. What is the wavenumber of the scattered Stokes radiation for the
J = 2 ← 0 transition of 14N2? Take B  1.9987 cm 1. Figure 11B.13 The symmetries of rotational wavefunctions
35
E11B.11 The rotational Raman spectrum of Cl2 shows a series of Stokes lines (shown here, for simplicity as a two-dimensional rotor) under a
separated by 0.9752 cm−1 and a similar series of anti-Stokes lines. Calculate the rotation through 180° depend on the value of J. Wavefunctions
bond length of the molecule. with J even do not change sign; those with J odd do change sign.
450 11 Molecular spectroscopy

with Itotal = 0 (paired spins, ↑↓). The three wavefunctions with


Itotal = 1 are α(A)α(B), α(A)β(B) + α(B)β(A), and β(A)β(B)
with MI = +1, 0, and −1, respectively. Rotation of the molecule
through 180° interchanges the labels A and B, but overall these
three wavefunctions are unchanged. Therefore, to achieve
an overall change of sign, the rotational wavefunction must
change sign, and so only odd values of J are allowed.
The fourth wavefunction, with Itotal = 0 and MI = 0, is
α(A)β(B) − α(B)β(A). When the labels A and B are inter-
changed the nuclear spin wavefunction changes sign:
α(A)β(B) − α(B)β(A) → α(B)β(A) − α(A)β(B) ≡ −{α(A)β(B) −
α(B)β(A)}. Therefore, in this case for the overall wavefunction
Figure 11B.14 The rotational Raman spectrum of a homonuclear to change sign requires that the rotational wavefunction not
diatomic molecule with two identical spin- 21 nuclei shows an change sign. Hence, only even values of J are allowed.
alternation in intensity as a result of nuclear statistics. In practice
the Rayleigh line is much stronger than depicted in the figure. The analysis leads to the conclusion that there are three nu-
clear spin wavefunctions that can be combined with odd values
of J, and one wavefunction that can be combined with even val-
How is that done? 11B.2 Identifying the effect of nuclear ues of J. Therefore, the ratio of the number of ways of achieving
statistics odd J to even J is 3:1. In general, for a homonuclear diatomic
molecule with nuclei of spin I, the numbers of ways of achiev-
Because 1H nuclei have I = 12 , like electrons, they are fermions ing states of odd and even J are in the ratio
and the Pauli principle requires the overall wavefunction to
change sign under particle interchange. However, the rotation Number of ways of achieving odd J
of a 1H2 molecule through 180° has a more complicated effect
Number of ways of achieving even J
than simply relabelling the nuclei (Fig. 11B.15).
There are four nuclear spin wavefunctions: three correspond (I  1)/I for half-integral spin nuclei

to a total nuclear spin Itotal = 1 (parallel spins, ↑↑); and one  I /(I  1) for integral spin nuclei
Nuclear statistics: homonuclearr diatomics (11B.25)
Rotate by 180°
For 1H2, I = 12 and the ratio is 3:1. For 14N2, with I = 1 the ratio
A B B A is 1:2. Additional complications arise when the electronic state
Wavefunction
changes by (–1)J of the molecule is not totally symmetric (as for O2, Topic 11F).
Nuclear statistics have consequences outside spectroscopy.
Interchange Sign change Different relative nuclear spin orientations change into one
spins if antiparallel another only very slowly, so a 1H2 molecule with parallel nu-
clear spins remains distinct from one with paired nuclear spins
Interchange for long periods. The form with parallel nuclear spins is called
fermions
B A ­ortho-hydrogen and the form with paired nuclear spins is
called para-hydrogen. Because ortho-hydrogen cannot exist in
a state with J = 0, it continues to rotate at very low temperatures
Figure 11B.15 The interchange of two identical fermion nuclei and has an effective rotational zero-point energy.
results in the change in sign of the overall wavefunction. The
relabelling can be thought of as occurring in two steps: the first is
a rotation of the molecule; the second is the interchange of unlike Exercise
spins (represented by black and white shading). The wavefunction E11B.12 What is the ratio of weights of populations due to the effects of
changes sign in the second step if the nuclei have antiparallel spins. nuclear statistics for 35Cl2?
11B Rotational spectroscopy 451

Checklist of concepts
☐ 1. A rigid rotor is a body that does not distort under the ☐ 7. A molecule must be anisotropically polarizable for it to
stress of rotation. give rise to rotational Raman scattering.
☐ 2. Rigid rotors are classified as spherical, symmetric, ☐ 8. The specific selection rules for rotational Raman spec-
linear, or asymmetric by noting the number of equal troscopy are: (i) linear rotors, ∆J = 0, ±2; (ii) symmetric
principal moments of inertia (or their symmetry). rotors, ∆J = 0, ±1, ±2; ∆K = 0.
☐ 3. Symmetric rotors are classified as prolate or oblate. ☐ 9. The appearance of rotational spectra is affected by
☐ 4. Centrifugal distortion arises from forces that change nuclear statistics, the selective occupation of rotational
the geometry of a molecule. states that stems from the Pauli principle.
☐ 5. The gross selection rule for a molecule to give a pure
rotational spectrum is that it must be polar.
☐ 6. The specific selection rules for microwave spectroscopy
are ∆J = ±1, ∆MJ = 0, ±1; for symmetric rotors the addi-
tional rule ∆K = 0 also applies.

Checklist of equations
Equation
Property Equation Comment
number
Moment of inertia I  mi xi2 xi is the perpendicular distance of atom i from 11B.2
i the axis of rotation
Rotational terms of a spherical or linear rotor F ( J )  BJ
 ( J  1) J = 0, 1, 2,… 11B.9, 11B.14
B  /4cI
Rotational terms of a symmetric rotor  ( J  1)  ( A  B )K 2
F ( J ,K )  BJ J = 0, 1, 2,… 11B.13a
K = 0, ±1,…, ±J
A  /4 cI|| 11B.13b
B  /4 cI 

Centrifugal distortion F ( J )  BJ
 ( J  1)  D J 2 ( J  1)2
J
Spherical or linear rotor 11B.15
Centrifugal distortion constant D  4 B 3 /  2
J
11B.16
Wavenumbers of rotational transitions  ( J  1  J )  2 B ( J  1) J = 0, 1, 2,…; linear rigid rotors 11B.20a
Rotational state with largest population J  (kT /2hcB )1/ 2  1
max 2
Linear rotors 11B.21
Wavenumbers of (i) Stokes and (ii) anti-Stokes (i)  ( J  2  J )  i  2 B (2 J  3) (i) J = 0, 1, 2,… 11B.24a
lines in the rotational Raman spectrum of (ii)  ( J  2  J )  i  2 B (2 J  1) (ii) J = 2, 3, 4,… 11B.24b
linear rotors
TOPIC 11C Vibrational spectroscopy of
diatomic molecules

Molecular potential energy, V(x)


Parabola Dissociation
➤ Why do you need to know this material?
The observation of vibrational transition frequencies is
used to determine the strengths and rigidities of bonds.
Measurements in the gas phase can also be used to calcu-
late the bond lengths of diatomic molecules.

➤ What is the key idea?


Re
The vibrational spectrum of a diatomic molecule can be
0
interpreted by using the harmonic oscillator model, with Displacement, x
0
modifications that account for bond dissociation and the
coupling of rotational and vibrational motion. Figure 11C.1 A molecular potential energy curve can be
approximated by a parabola near the bottom of the well. The
➤ What do you need to know already? parabolic potential energy results in harmonic oscillations. At high
You need to be familiar with the harmonic oscillator (Topic excitation energies the parabolic approximation is poor (the true
7E) and rigid rotor (Topic 11B) models of molecular motion potential energy is less confining), and is totally wrong near the
dissociation limit.
and the general principles of spectroscopy (Topic 11A).

­derivative of V is zero at the minimum. Therefore, the first sur-


viving term is proportional to the square of the displacement.
For small displacements all the higher terms can be ignored so
One internal mode of motion of a diatomic molecule is its vi- the potential energy can be written
bration, in which the internuclear separation increases and de-
creases periodically. This motion, and the transitions between  d 2V 
V (x )  12  2  x 2 (11C.1b)
the allowed quantum states, can be treated initially as an exam-  dx  0
ple of harmonic motion like that described in Topic 7E.
Therefore, the first approximation to a molecular potential en-
ergy curve is a parabolic potential of the form

11C.1 Vibrational motion V (x )  12 kf x 2 x  R  Re


Parabolic potential
energy
 (11C.2a)

Figure 11C.1 shows a typical potential energy curve of a dia-


where kf is the force constant of the bond, a measure of its
tomic molecule (it is essentially a reproduction of Fig. 7E.1 of
­stiffness:
Topic 7E). The potential energy V(x), where x = R − Re (the
displacement from equilibrium), can be expanded around its  d 2V  Force constant
minimum by using a Taylor series (see Part 1 of the Resource kf   2  [definition]
 (11C.2b)
 dx  0
section):
If the potential energy curve is sharply curved close to its mini-
 dV   2  2
1 d V mum, then kf will be large and the bond stiff. Conversely, if the
V (x )  V (0)    x  2 2 
x   (11C.1a)
 dx  0  dx  0 potential energy curve is wide and shallow, then kf will be small
and the bond easily stretched or compressed (Fig. 11C.2).
The notation (…)0 means that the derivative is evaluated at The Schrödinger equation for the relative motion of two
x = 0. The term V(0) can be set arbitrarily to zero, and the first atoms of masses m1 and m2 with a parabolic potential energy is
11C Vibrational spectroscopy of diatomic molecules 453

The vibrational wavefunctions are the same as those discussed


in Topic 7E for a harmonic oscillator.
The vibrational terms depend on the effectiνe mass of the
Potential energy, V(x)

molecule, not directly on its total mass. This dependence is


physically reasonable, because if atom 1 is very much heavier
Increasing than atom 2, then the effective mass is close to m2, the mass
kf
of the lighter atom. The vibration would then be that of the
light atom relative to an essentially stationary heavy atom. For
a homonuclear diatomic molecule m1 = m2, and the effective
mass is half the total mass: meff = 12 m.
0 Displacement, x

Figure 11C.2 The force constant is a measure of the curvature


Brief illustration 11C.1
of the potential energy close to the equilibrium extension of the
bond. A strongly confining well (one with steep sides, a stiff bond) The force constant of the bond in HCl is 516 N m−1, a reason-
corresponds to high values of kf.
ably typical value for a single bond. The effective mass of 1H35Cl
is 1.63 × 10−27 kg (note that this mass is very close to the mass
2 d 2 1
  kf x 2  E  (11C.3a) of the hydrogen atom, 1.67 × 10−27 kg, implying that the H atom
2meff dx 2 2 is essentially vibrating against a stationary Cl atom). The vibra-
where meff is the effective mass:1 tional frequency is therefore

m1m2 Effective mass kg ms 2


meff  [definition]
 (11C.3b)  516 Nm  l 
1/ 2

m1  m2    5.63  1014 s 1
 1.63  1027 kg 
 
These equations are derived by using the separation of vari-
ables procedure (see A deeper look 8A.1, available to read in the and the corresponding wavenumber is
e-book accompanying this text) to separate the relative motion
of the atoms from the motion of the molecule as a whole.  5.63  1014 s 1
Apart from the appearance of the effective mass, the     2.99  103 cm 1
2c 2  (2.998  1010 cms 1 )
Schrödinger equation in eqn 11C.3a is the same as eqn 7E.2 for
a particle of mass m undergoing harmonic motion. Therefore,
the results from that Topic can be used to write the permitted
vibrational energy levels:
Exercises
1/ 2
 k 
Ev   v  12   ,    f 
E11C.1 Calculate the percentage difference in the fundamental vibrational
v  0,1,2, wavenumbers of 23Na35Cl and 23Na37Cl on the assumption that their force
 meff  constants are the same. The mass of 23Na is 22.9898mu.
Vibrational energy levels [diatomic molecule] (11C.4a) 35
E11C.2 The wavenumber of the fundamental vibrational transition of Cl2 is
564.9 cm−1. Calculate the force constant of the bond.
The vibrational terms of a molecule, the energies of its vibra-
tional states expressed as wavenumbers, are denoted G(  v), with

Ev = hcG(v). Therefore, with ω = 2πν and    /c   /2c:

11C.2 Infrared spectroscopy
1/ 2
1  kf 
G (v)   v  12  ,     The gross and specific selection rules for vibrational transitions
2c  meff 
are established, as usual, by considering the properties of the
Vibrational terms [diatomic molecule] (11C.4b) electric transition dipole moment. The detailed calculation is
shown in A deeper look 11C.1, available to read in the e-book
1
accompanying this text. The conclusion is that
Distinguish effectiνe mass from reduced mass. The former is a measure
of the mass that is moved during a vibration. The latter is the quantity that The gross selection rule for a change in vibrational state
emerges from the separation of relative internal and overall translational mo-
brought about by absorption or emission of radiation is that
tion. For a diatomic molecule the two are the same, but that is not true in gen-
eral for vibrations of polyatomic molecules. Many, however, do not make this the electric dipole moment of the molecule must change
distinction and refer to both quantities as the ‘reduced mass’. when the atoms are displaced relative to one another.
454 11 Molecular spectroscopy

Such vibrations are said to be infrared active. The classical all extensions because it does not allow a bond to dissociate.
basis of this rule is that an oscillating electric dipole generates At high vibrational excitations the separation of the atoms
an electromagnetic wave. Similarly, the oscillating electric field (more precisely, the spread of the vibrational wavefunction)
of an incoming electromagnetic wave induces an oscillating di- allows the molecule to explore regions of the potential energy
pole in the molecule. curve where the parabolic approximation is poor and addi-
Note that the molecule need not have a permanent dipole mo- tional terms in the Taylor expansion of V (eqn 11C.1a) must
ment: the rule requires only a change in dipole moment. Some be retained. The motion then becomes anharmonic, in the
vibrations do not affect the dipole moment of the molecule (for sense that the restoring force is no longer proportional to the
instance, the stretching motion of a homonuclear diatomic mol- displacement. Because the actual curve is less confining than
ecule), so they neither absorb nor generate radiation: such vibra- a parabola, it can be anticipated that the energy levels become
tions are said to be infrared inactive. Weak infrared transitions more closely spaced at high excitations.
can be observed from homonuclear diatomic molecules trapped
within various nanomaterials. For instance, when incorporated
into solid C60, H2 molecules interact through van der Waals forces
11C.3(a) The convergence of energy levels
with the surrounding C60 molecules and acquire dipole moments, One approach to the calculation of the energy levels in the pres-
with the result that they have observable infrared spectra. ence of anharmonicity is to use a function that resembles the
The calculation also shows that the specific selection rule is true potential energy more closely. One example is the Morse
potential energy function:
Specific selection rule
v  1  (11C.5)
[harmonic oscillator] 1/ 2
 m 2 
V (x )  hcD e (1  e  ax )2 a   eff 
Transitions for which Δv = +1 correspond to absorption and 
 2hcDe 
those with Δv = −1 correspond to emission. It follows that the
Morse potentiial energy  (11C.7)
wavenumbers of allowed vibrational transitions, which are de-
noted G v 1 for the transition v + 1 ← v, are At x = 0, V(0) = 0, and at large positive displacements, V(x) ap-
2

proaches hcD e. Therefore hcD e can be identified as the depth of


Vibrational wavenumbers 
G v  1  G (v  1)  G (v)   [harmonic oscillator]
(11C.6) the potential energy well (Fig. 11C.3). Near the well minimum
the variation of V with displacement resembles a parabola (as
2

The wavenumbers of vibrational transitions correspond to radi- can be checked by expanding the exponential and retaining the
ation in the infrared region of the electromagnetic spectrum, so first two terms). The Schrödinger equation can be solved for the
vibrational transitions absorb and generate infrared radiation. Morse potential energy and the permitted levels are
At room temperature kT/hc ≈ 200 cm−1, and because most vi-
G (v)   v  12    v  12  x e
2
brational wavenumbers are significantly greater than 200 cm−1 it
Vibrational terms
follows from the Boltzmann distribution that almost all the mol- v  0,1,2,,vmax  (11C.8)
[Morse potential energy]
ecules are in their vibrational ground states. Hence, the dominant a2

spectral transition is the fundamental transition, 1←0. As a re- xe  
2meff  4 D e
sult, the spectrum is expected to consist of a single absorption line.
If the molecules are formed in a vibrationally excited state,
such as when vibrationally excited HF molecules are formed in
the reaction H2 + F2 → 2 HF⋆, where the star indicates a vibra-
tionally ‘hot’ molecule, the transitions 5→4, 4→3,… may also
appear (in emission). In the harmonic approximation, all these
Potential energy, V(x)

lines lie at the same frequency, and the spectrum is also a single
line. However, the breakdown of the harmonic approximation ~ ~
hcD0 hcDe
causes the transitions to lie at slightly different frequencies, so
several lines are observed.

v=0

11C.3 Anharmonicity 0
0 Displacement, x

The vibrational terms in eqn 11C.4b are only approximate be- Figure 11C.3 The dissociation energy of a molecule, hcD 0 , differs
cause they are based on a parabolic approximation to the ac- from the depth of the potential well, hcD e , on account of the zero-
tual potential energy curve. A parabola cannot be correct at point energy of the vibration of the bond.
11C Vibrational spectroscopy of diatomic molecules 455

a­ nharmonicity is present. Therefore, the selection rule is also


only an approximation. For an anharmonic oscillator, all values
vmax
~ of Δv are allowed, but transitions with Δv > 1 are allowed only
hcDe
4
weakly if the anharmonicity is slight. Typically, the first over-
3 tone is only about one-tenth as intense as the fundamental.
Potential energy

1
Example 11C.1  Estimating an anharmonicity constant
v=0
Estimate the anharmonicity constant xe for 35Cl19F given that
0
–1 0 1 2 3 4 5 the wavenumbers of the fundamental and first overtones are
Displacement, ax found to be 773.8 and 1535.3 cm−1, respectively.
Figure 11C.4 The Morse potential energy curve reproduces Collect your thoughts You can find an expression for the wave-
the general shape of a molecular potential energy curve. The number of the fundamental transition 1←0, by using eqn
corresponding Schrödinger equation can be solved, and the 11C.9b with v = 0, and for the wavenumber of the first overtone
values of the energies obtained. The number of bound levels is 2←0 by using eqn 11C.10 with v = 2. You then need to solve
finite. Interact with the dynamic version of this graph in the e-book. the two equations to give values for ν and x eν , and hence find
xe itself.
The positive dimensionless parameter xe is called the anhar-
The solution From eqn 11C.9b the expression for the wave-
monicity constant. The number of vibrational levels of a Morse
number of the fundamental is   2x e , and eqn 11C.10 gives
oscillator is finite, as shown in Fig. 11C.4. The second term in the expression for the wavenumber of the first overtone as
the expression for G subtracts from the first with increasing ef- 2  6 x e . From the data it follows that 773.8 cm 1    2 x e
fect as v increases, and hence gives rise to the convergence of and 1535.3 cm 1  2  6 x e . The terms in ν alone are elimi-
the levels at high quantum numbers. The dissociation energy nated by noting that (  2 x e )  12 (2  6 x e )  x e to give
hcD 0 is the energy difference between the lowest vibrational
state (v = 0) and the infinitely separated atoms. As can be seen x e  773.8 cm 1  12  1535.3 cm 1  6.15 cm 1
in Fig. 11C.3, the dissociation energy and the well depth are re-
This value for x eν can then be substituted into
lated by D e  D 0  G (0).
773.8 cm 1    2 x e to give
Although the Morse oscillator is quite useful theoretically, in
practice the more general expression   773.8 cm 1  2 x e  773.8 cm 1  2  6.15 cm 1  786.1 cm 1

G (v)   v  12    v  12  x e   v  12  y e  


2 3
(11C.9a) It follows that
1
x e 6.15 cm
where xe, ye,… are empirical dimensionless constants charac- xe    7.82  103
teristic of the molecule, is used to fit the experimental data and  786.1 cm 1
to determine the dissociation energy of the molecule. In this Self-test 11C.1 Predict the wavenumber of the second overtone
case the wavenumbers of transitions with Δv = +1 are for this molecule.

G v  1  G (v  1)  G (v)    2(v  1)x e  


Answer: 2284.5 cm−1
(11C.9b)
2

Equation 11C.9b shows that, because xe > 0, the transitions


move to lower wavenumbers as v increases.
In addition to the strong fundamental transition 1←0, a set 11C.3(b) The Birge–Sponer plot
of weaker absorption lines are also seen and correspond to the When several vibrational transitions are detectable, a graphi-
transitions 2←0, 3←0,…. These transitions are forbidden for a cal technique called a Birge–Sponer plot can be used to de-
harmonic oscillator, but become weakly allowed as a result of termine the dissociation energy of the bond. The basis of the
anharmonicity. The transition 2←0 is known as the first over- plot is that the sum of the successive intervals G v  1 (eqn
tone, 3←0 is the second overtone, and so on. The wavenumber 2
11C.9b) from v = 0 to the dissociation limit is the dissociation
of the overtone transition v←0 is given by wavenumber D 0 :
G (v)  G (0)  v  v(v  1)x e   (11C.10)
D 0  G 1/ 2  G 3/ 2    G v  1  (11C.11)
2
v
The reason for the appearance of overtones is that the selec-
tion rule is derived from the properties of harmonic oscilla- just as the height of a ladder is the sum of the separations of
tor wavefunctions, which are only approximately valid when its rungs (Fig. 11C.5). The construction in Fig. 11C.6 shows
456 11 Molecular spectroscopy

~
D0 2500

2000

Gv+1/2/cm–1
~ ~ 1500
ΔG1/2+ ΔG3/2 +...
...

2 ...
~

~
ΔG3/2 1000
1
~
v ΔG1/2 500
0
0
1 3 v+½ 39
Figure 11C.5 The dissociation wavenumber is the sum of the 2 2 2
separations G v 1 of the vibrational terms up to the dissociation
Figure 11C.7 The Birge–Sponer plot used in Example 11C.2.
2
limit.
The area is obtained simply by counting the squares beneath
the line or using the formula for the area of a triangle
(area  21  base  height ).
~
Area = ΔG1/2

Linear The solution The points are plotted in Fig. 11C.7, and a linear
extrapolation
extrapolation. The area under the curve (use the formula for
the area of a triangle or count the squares) is 216. Each square
ΔGv+1/2

corresponds to 100 cm−1 (refer to the scale of the vertical axis);


~

True
curve hence the dissociation energy, expressed as a wavenumber, is
21 600 cm−1 (corresponding to 258 kJ mol−1).
Self-test 11C.2 The vibrational levels of HgH converge rap-
idly, and successive intervals are 1203.7 (which corresponds to
1 3 5 v+½ the 1←0 transition), 965.6, 632.4, and 172 cm−1. Estimate the
2 2 2
molar dissociation energy.
Figure 11C.6 The area under a plot of G v 1 against vibrational
Answer: 35.6 kJ mol−1
2
quantum number is equal to the dissociation wavenumber of
the molecule. The assumption that the differences approach zero
linearly is the basis of the Birge–Sponer extrapolation.
Exercises
E11C.3 For O2, ∆G values for the transitions v = 1 ← 0, 2 ← 0, and
16

3 ← 0 are, respectively, 1556.22, 3088.28, and 4596.21 cm−1. Calculate ν


that the area under the plot of G v 1 against v + 12 is equal to the and xe. Assume ye to be zero.
sum, and therefore to D 0 . The successive terms decrease linearly
2

E11C.4 The first five vibrational energy levels of HCl are at 1481.86, 4367.50,
when only the xe anharmonicity constant is taken into account. 7149.04, 9826.48, and 12 399.8 cm−1. Calculate the dissociation energy of the
If this is the case, any missing data for high vibrational quan- molecule in reciprocal centimetres and electronvolts.
tum numbers can be estimated by linear extrapolation. Most
actual plots differ from the linear plot as shown in Fig. 11C.6,
so the value of D 0 obtained in this way is usually an overesti-
mate of the true value. 11C.4 Vibration–rotation spectra
Each line of the high resolution vibrational spectrum of a gas-
phase heteronuclear diatomic molecule is found to consist of
Example 11C.2 Using a Birge–Sponer plot a large number of closely spaced components (Fig. 11C.8).
Hence, molecular spectra are often called band spectra. The
The observed vibrational intervals of H2+ lie at the following val- separation between the components is less than 10 cm−1, which
ues for 1←0, 2←1,…, respectively (in cm−1): 2191, 2064, 1941, suggests that the structure is due to rotational transitions ac-
1821, 1705, 1591, 1479, 1368, 1257, 1145, 1033, 918, 800, 677, companying the vibrational transition. A rotational change
548, 411. Determine the dissociation energy of the molecule. should be expected because classically a vibrational transition
Collect your thoughts Plot the separations against v + 12 , extrap- can be thought of as leading to a sudden increase or decrease in
olate linearly to the point cutting the horizontal axis, and then the instantaneous bond length. Just as ice-skaters rotate more
measure the area under the curve. rapidly when they bring their arms in, and more slowly when
11C Vibrational spectroscopy of diatomic molecules 457

8
7

Q-branch
6
5
4

v=1
Absorbance, A

3
R-branch P-branch 2
1
1
H35Cl 0
8
H Cl
1 37 7
6
5
4

v=0
3
2
1
0
3050 3000 2950 2900 2850 2800 2750 2700 P Q R
Wavenumber, /cm–1

Figure 11C.8 A high-resolution vibration–rotation spectrum Frequency


of HCl. The lines appear in pairs because H35Cl and H37Cl both
contribute (their abundance ratio is 3:1). There is no Q branch (see Figure 11C.9 The formation of P, Q, and R branches in a vibration–
below), because ΔJ = 0 is forbidden for this molecule. rotation spectrum. The intensities reflect the populations of the
initial rotational levels and magnitudes of the transition moments.
they throw them out, so the molecular rotation is either accel- If the value of the rotational constant B is independent of the
erated or retarded by a vibrational transition. vibrational state, all the Q branch transitions occur at the same
frequency, leading to a single spectral line. In most cases the
values of B for the v = 0 and v = 1 states do differ slightly, so the Q
11C.4(a) Spectral branches branch transitions occur at slightly different frequencies.

A detailed analysis of the quantum mechanics of simultaneous


vibrational and rotational changes shows that the rotational Q branch consists of all transitions with ΔJ = 0, and its wave-
quantum number J changes by ±1 during the vibrational transi- numbers are the same for all values of J:
tion of a diatomic molecule. If the molecule also possesses an- Q ( J )  S(v  1,J )  S(v,J )   Q branch transitions  (11C.13b)
gular momentum about its axis, as in the case of the electronic
orbital angular momentum of the molecule NO with its con- This branch, when it is allowed (as in NO), appears at the vibra-
figuration …π1, then the selection rules also allow ΔJ = 0. tional transition wavenumber ν . In Fig. 11C.8 there is a gap at
The appearance of the vibration–rotation spectrum of a dia- the expected location of the Q branch because it is forbidden in
tomic molecule can be discussed by using the combined vibra- HCl, which has zero electronic angular momentum around its

tion–rotation terms, S: internuclear axis. The R branch consists of lines with ΔJ = +1:

S(v,J )  G (v)  F ( J ) (11C.12a) R ( J )  S(v  1, J  1)  S(v, J )    2 B ( J  1)


If anharmonicity and centrifugal distortion are ignored, G(  v) J  0, 1, 2,  R branch traansitions (11C.13c)
can be replaced by the expression in eqn 11C.4b, and F ( J ) can be
replaced by the expression in eqn 11B.9, F ( J )  BJ
 ( J 1), to give This branch consists of lines extending to the high-­wavenumber
side of ν at   2 B ,   4 B ,.
S(v,J )  (v  12 )  BJ
 ( J  1) (11C.12b) The separation between the lines in the P and R branches
of a vibrational transition gives the value of B. Therefore, the
In a more detailed treatment, B is allowed to depend on the vi-
brational state and written B v . bond length can be deduced in the same way as from micro-
In the vibrational transition v + 1 ← v, J changes by ±1 and in wave spectra (Topic 11B). However, the latter technique gives
some cases by 0 (when ΔJ = 0 is allowed). The absorptions then more precise bond lengths because microwave frequencies can
fall into three groups called branches of the spectrum. The P be measured with greater precision than infrared frequencies.
branch consists of all transitions with ΔJ = −1:
Brief illustration 11C.2

P ( J )  S(v  1, J  1)  S(v, J )    2 BJ


 The infrared absorption spectrum of 1H81Br contains a band
J  1, 2, 3  P branch transitiions  (11C.13a) arising from v = 0. It follows from eqn 11C.13c and the data in
Table 11C.1 that the wavenumber of the line in the R branch
originating from the rotational state with J = 2 is
This branch consists of lines extending to the low wavenum-
ber side of ν at   2 B ,   4 B , with an intensity distribution R (2)    6 B  2648.98 cm 1  6  (8.465 cm 1 )
reflecting both the populations of the rotational levels and the  2699.77 cm 1
magnitude of the J − 1 ← J transition moment (Fig. 11C.9). The
458 11 Molecular spectroscopy

Table 11C.1 Properties of diatomic molecules⋆ R ( J  1)  P ( J  1)  S(0,J  1)  S(0,J  1). The right-hand side
is evaluated by using the expression for S(v,J ) in eqn 11C.12b
 /cm 1 Re/pm B e /cm −1 kf/(N m−1) D 0 /(104 cm −1 )
1
(with B 0 in place of B)  to give
H2 4400 74 60.86 575 3.61
1
H35Cl 2991 127 10.59 516 3.58  ( J  1)   ( J  1)  4 B  J  1 
R P 0 2 (11C.15a)
1
H127I 2308 161 6.51 314 2.46 Therefore, a plot of the combination difference R ( J  1) 
P ( J 1) against J + 12 should be a straight line of slope 4 B 0 and
35
Cl2 560 199 0.244 323 2.00

More values are given in the Resource section. intercept (with the vertical axis) zero; the value of B 0 can there-
fore be determined from the slope. The presence of centrifugal
distortion results in the intercept deviating from zero, but has
11C.4(b) Combination differences
little effect on the quality of the straight line.
A more detailed analysis of the rotational fine structure shows The two lines νR ( J ) and νP ( J ) have a common lower state,
that the rotational constant decreases as the vibrational quan- and hence their combination difference depends on B1. As
tum number v increases. The origin of this effect is that the before, from Fig. 11C.10 it can be seen that R ( J )  P ( J ) 
average value of 1/R2 decreases because the asymmetry of the S(1,J  1)  S(1,J  1) which is
potential well results in the average bond length increasing
with vibrational energy. A harmonic oscillator also shows this R ( J )  P ( J )  4 B1  J  12   (11C.15b)
effect because although the average value of R is unchanged
with increasing v, the average value of 1/R2 does change (see
Brief illustration 11C.3
Problem P11C.13). Typically, B1 is 1–2 per cent smaller than B 0 .
The result of B1 being smaller than B 0 is that the Q branch (if
The rotational constants of B 0 and B1 can be estimated from
it is present) consists of a series of closely spaced lines; the lines a calculation involving only a few transitions. For 1H35Cl,
of the R branch converge slightly as J increases, and those of the R (0)  P (2)  62.6 cm 1, and it follows from eqn 11C.15a,
P branch diverge. It follows from eqn 11C.12b with B v in place with J = 1, that B 0  62.6/ 4 1  12  cm 1  10.4 cm 1. Similarly,
of B R (1)  P (1)  60.8 cm 1, and it follows from eqn 11C.15b, again
with J = 1 that B1  60.8/ 4 1  12  cm 1  10.1 cm 1. If more
P ( J )    (B1  B 0 )J  (B1  B 0 )J 2 lines are used to make combination difference plots, the values
Q ( J )    (B1  B 0 )J ( J  1)  (11C.14) B 0  10.440 cm 1 and B1  10.136 cm 1 are found. The two rota-
R ( J )    (B1  B 0 )( J  1)  (B1  B 0 )( J  1)2 tional constants differ by about 3 per cent of B 0.

The method of combination differences is used to determine It is common to assume that the rotational constants vary lin-
the two rotational constants individually. The method involves early with the vibrational quantum number v according to
setting up expressions for the difference in the wavenumbers
of transitions to a common state. The resulting expression then B v  B e    v  12  (11C.16)
depends solely on properties of the other states. where B e is the rotational constant for a molecule in which the
As can be seen from Fig. 11C.10, the transitions R ( J 1) atoms are separated by the equilibrium bond length Re.
and P ( J 1) have a common upper state, and so the dif-
ference between these transitions can be anticipated
to depend on B 0 . From the diagram it can be seen that Exercise
1 127
E11C.5 Infrared absorption by H I gives rise to an R branch from v = 0.
J+1 What is the wavenumber of the line originating from the rotational state with
~ J v=1 J = 2? Hint: Use data from Table 11C.1.
B1
J–1
˜ P(J + 1)
˜ R(J – 1)

˜ R(J)
˜ P(J)

11C.5 Vibrational Raman spectra


ν
ν
ν

J+1

J v=0 The gross and specific selection rules for vibrational Raman
~
B0 transitions are established, as usual, by considering the appro-
J–1
priate transition moment. The details are set out in A deeper
Figure 11C.10 The method of combination differences makes use look 11C.1, available to read in the e-book accompanying this
of the fact that certain pairs of transitions share a common state. text. The conclusion is that
11C Vibrational spectroscopy of diatomic molecules 459

The gross selection rule for vibrational Raman transitions The selection rules are ΔJ = 0, ±2 (as in pure rotational Raman
is that the polarizability must change as the molecule spectroscopy), and give rise to the O branch (ΔJ = −2), the Q
vibrates. branch (ΔJ = 0), and the S branch (ΔJ = +2):
The polarizability plays a role in vibrational Raman spectros- O ( J )  i    4 B  J  12  J  2, 3, 4, 
copy because the molecule must be squeezed and stretched
O branch transitions  (11C.17a)
by the incident radiation in order for vibrational excitation to
occur during the inelastic photon–molecule collision. Both
Q ( J )  i   Q branch transitions  (11C.17b)
homonuclear and heteronuclear diatomic molecules swell
and contract during a vibration, the control of the nuclei over
the electrons varies, and hence the molecular polarizability
S ( J )  i    4 B  J  23  J  0, 1, 2, 
changes. Both types of diatomic molecule are therefore vibra- S branch transitions (11C.17c)
tionally Raman active. The analysis also shows that the specific where νi is the wavenumber of the incident radiation. Note that,
selection rule for vibrational Raman transitions in the har- unlike in infrared spectroscopy, a Q branch is obtained for all
monic approximation is Δv = ±1, just as for infrared transitions. linear molecules. The spectrum of CO, for instance, is shown in
The lines to high frequency of the incident radiation, in the Fig. 11C.12: rather than being a single line, as implied by eqn
language introduced in Topic 11A the ‘anti-Stokes lines’, are 11C.17, the Q branch appears as a broad feature. Its breadth
those for which Δv = −1. The lines to low frequency, the ‘Stokes arises from the presence of several overlapping lines arising
lines’, correspond to Δv = +1. The intensities of the anti-Stokes from the difference in rotational constants of the upper and
and Stokes lines are governed largely by the Boltzmann pop- lower vibrational states.
ulations of the vibrational states involved in the transition. It The information available from vibrational Raman spectra
follows that anti-Stokes lines are usually weak because the pop- adds to that from infrared spectroscopy because homonuclear
ulations of the excited vibrational states are very small. diatomics can also be studied. The spectra can be interpreted
In gas-phase spectra, the Stokes and anti-Stokes lines have a in terms of the force constants, dissociation energies, and bond
branch structure arising from the simultaneous rotational tran- lengths, and some of the information obtained is included in
sitions that accompany the vibrational excitation (Fig. 11C.11). Table 11C.1.

8
7
6
5
4
v=1

3
2
1 O Q S
0
8
7
6
5
4
v=0

3
2
1
0
O Q S 2000 2100
/cm–1

Frequency of scattered radiation


Figure 11C.12 The structure of a vibrational line in the vibrational
Raman spectrum (the Stokes lines) of carbon monoxide, showing
Figure 11C.11 The formation of O, Q, and S branches in a the O, Q, and S branches. The horizontal axis represents the
vibration–rotation Raman spectrum of a diatomic molecule (its wavenumber difference between the incident and scattered
Stokes lines). Note that the frequency scale runs in the opposite radiation. For these Stokes lines the wavenumber of the scattered
direction to that in Fig. 11C.9, because the higher energy radiation (as distinct from the difference, which represents the
transitions (on the right) extract more energy from the incident energy deposited in the molecule) increases to the left, as in
beam, resulting in scattered photons with lower frequencies. Fig. 11C.11.
460 11 Molecular spectroscopy

Checklist of concepts
☐ 1. The vibrational energy levels of a diatomic molecule ☐ 6. Anharmonicity gives rise to weaker overtone transi-
modelled as a harmonic oscillator depend on the force tions (v = 2 ← v = 0, v = 3 ← v = 0,…).
constant kf (a measure of the stiffness of the bond) and ☐ 7. A Birge–Sponer plot may be used to determine the dis-
the effective mass of the vibration. sociation energy of a diatomic molecule.
☐ 2. The gross selection rule for infrared spectra is that the ☐ 8. In the gas phase, vibrational transitions have a P, R
electric dipole moment of the molecule must depend on branch structure due to simultaneous rotational transi-
the bond length. tions; some molecules also have a Q branch.
☐ 3. The specific selection rule for infrared spectra (within ☐ 9. For a vibration to be Raman active, the polarizability
the harmonic approximation) is ∆v = ±1. must change as the molecule vibrates.
☐ 4. The Morse potential energy can be used to model ☐ 10. The specific selection rule for vibrational Raman spec-
anharmonic vibration. tra (within the harmonic approximation) is ∆v = ±1.
☐ 5. The strongest infrared transitions are the fundamental ☐ 11. In gas-phase spectra, the Stokes and anti-Stokes lines in
transitions (v = 1 ← v = 0). a Raman spectrum have an O, Q, S branch structure.

Checklist of equations
Property Equation Comment Equation number
Vibrational terms G (v)   v  12  ,   (1/2c)(kf /meff )1/ 2 Diatomic molecules; harmonic 11C.4b
meff  m1m2 /(m1  m2 ) approximation

Infrared spectra (vibrational) G v 1   Diatomic molecules; harmonic 11C.6


2
approximation
Morse potential energy V (x )  hcD e (1  e  ax )2 11C.7
a  (m  2 /2hcD )1/ 2
eff e

G (v)   v  12     v  12  x e , x e   /4 D e
2
Vibrational terms (diatomic molecules) Morse potential energy 11C.8

Infrared spectra (vibrational) G v 1    2(v  1)x e   Anharmonic oscillator 11C.9b
2

G (v)  G (0)  v  v(v  1)x e   Overtones 11C.10

Dissociation wavenumber D 0  G 1/ 2  G 3/ 2    G v 1 Birge–Sponer plot 11C.11


2
v

Vibration–rotation terms (diatomic S(v,J )   v  12    BJ


 ( J  1) Rotation coupled to vibration 11C.12b
molecules)
Infrared spectra (vibration–rotation) P ( J )  S(v  1,J  1)  S(v,J )    2 BJ
 P branch (∆J = −1) 11C.13a
J  1,2,3
Q ( J )  S(v  1,J )  S(v,J )   Q branch (∆J = 0) 11C.13b

R ( J )  S(v  1,J  1)  S(v,J )    2 B ( J  1) R branch (∆J = +1) 11C.13c


J  0,1,2, 
R ( J  1)  P ( J  1)  4 B 0  J  12  Combination differences 11C.15
R ( J )  P ( J )  4 B1  J  12 

Raman spectra (vibration–rotation) O ( J )  i    4 B  J  12  J  2, 3, 4,  O branch (∆J = −2) 11C.17a


Q ( J )  i   Q branch (∆J = 0) 11C.17b

S ( J )  i    4 B  J  23  J  0, 1, 2,  S branch (∆J = +2) 11C.17c


TOPIC 11D Vibrational spectroscopy of
polyatomic molecules

θ ϕ
➤ Why do you need to know this material?
The analysis of vibrational spectra is a widely used analyti- ψ
cal technique that provides information about the identity
and shapes of polyatomic molecules in the gas and con-
densed phases. (b)

➤ What is the key idea?


The vibrational spectrum of a polyatomic molecule can be
interpreted in terms of its normal modes. (a)

Figure 11D.1 (a) The orientation of a linear molecule requires


➤ What do you need to know already?
the specification of two angles. (b) The orientation of a nonlinear
You need to be familiar with the harmonic oscillator (Topic molecule requires the specification of three angles.
7E), the general principles of spectroscopy (Topic 11A),
and the selection rules for vibrational infrared and Raman
spectroscopy (Topic 11C). e­ xample, three coordinates are needed to specify the location of
the centre of mass of the molecule, so three of the 3N displace-
ments correspond to the translational motion of the molecule
as a whole. The remaining 3N − 3 displacements are ‘internal’
modes of the molecule.
There is only one mode of vibration for a diatomic molecule: Two angles are needed to specify the orientation of a linear
the periodic stretching and compression of the bond. In polya- molecule in space: in effect, only the latitude and longitude of the
tomic molecules there are many bond lengths and angles that direction in which the molecular axis is pointing need be speci-
can change, and as a result the vibrational motion of the mol- fied (Fig. 11D.1a). However, three angles are needed for a non-
ecule is very complex. Some order can be brought to this com- linear molecule because the orientation of the molecule around
plexity by introducing the concept of ‘normal modes’. the direction defined by the latitude and longitude also needs to
be specified (Fig. 11D.1b). Therefore, for linear molecules two
of the 3N − 3 internal displacements are rotational, whereas for
nonlinear molecules three of the displacements are rotational.
11D.1 Normal modes That leaves 3N − 5 (linear) or 3N − 6 (nonlinear) non-rotational
internal displacements of the atoms: these are the vibrational
The first step in the analysis of the vibrations of a polyatomic modes. It follows that the number of modes of vibration is:
molecule is to calculate the total number of vibrational modes.
Linear molecule: Nvib = 3N − 5 (11D.1)
Nonlinear molecule: Nvib = 3N − 6 Numbers of
vibrational modes
How is that done? 11D.1 Counting the number of
vibrational modes Brief illustration 11D.1

The total number of coordinates needed to specify the loca- Water, H2O, is a nonlinear triatomic molecule with N = 3,
tions of N atoms is 3N. Each atom may change its location by and so has 3N − 6 = 3 modes of vibration; CO2 is a linear
varying each of its three coordinates (x, y, and z), so the total triatomic molecule, and has 3N − 5 = 4 modes of vibration.
number of displacements available is 3N. These displacements Methylbenzene has 15 atoms and 39 modes of vibration.
can be grouped together in a physically sensible way. For
462 11 Molecular spectroscopy

The greatest simplification of the description of vibrational


motion is obtained by analysing it in terms of ‘normal modes’.
A normal mode is a vibration of the molecule in which the cen-
tre of mass remains fixed, the orientation is unchanged, and the 1 (3652 cm–1) 2 (1595 cm–1) 3 (3756 cm–1)
atoms move synchronously. When a normal mode is excited,
the energy remains in that mode and does not migrate into Figure 11D.3 The three normal modes of H2O. The mode ν2 is
predominantly bending, and occurs at lower wavenumber than
other normal modes of the molecule.
the other two.
A normal mode analysis is possible only if it is assumed that
the potential energy is parabolic (as in a harmonic oscillator, wavenumber and high frequency vibrations. The effective mass
Topic 11C). In reality, the potential energy is not parabolic, the of the mode is a measure of the mass that moves in the vibration
vibrations are anharmonic (Topic 11C), and the normal modes and in general is a combination of the masses of the atoms. For
are not completely independent. Nevertheless, a normal mode example, in the symmetric stretch of CO2, the carbon atom is
analysis remains a good starting point for the description of the stationary, and the effective mass depends on the masses of only
vibrations of polyatomic molecules. the oxygen atoms. In the antisymmetric stretch and in the bends
Figure 11D.2 shows the four normal modes of CO2. Mode ν1 all three atoms move, so the masses of all three atoms contribute
is the symmetric stretch in which the two oxygen atoms move (but to different extents) to the effective mass of each mode.
in and out synchronously but the carbon atom remains sta- The three normal modes of H2O are shown in Fig. 11D.3:
tionary. Mode ν2, the antisymmetric stretch, in which the two note that the predominantly bending mode (ν2) has a lower
oxygen atoms always move in the same direction and opposite frequency (and wavenumber) than the others, which are pre-
to that of the carbon. Finally, there are two bending modes ν3 dominantly stretching modes. It is generally the case that the
in which the oxygen atoms move perpendicular to the inter­ frequencies of bending motions are lower than those of stretch-
nuclear axis in one direction and the carbon atom moves in the ing modes. Only in special cases (such as the CO2 molecule) are
opposite direction: this bending motion can take place in either the normal modes purely stretches or purely bends. In general,
of two perpendicular planes. In all these modes, the position a normal mode is a composite motion involving simultaneous
of the centre of mass and orientation of the molecule are un- stretching of bonds and changes to bond angles. In a given nor-
changed by the vibration. mal mode, heavy atoms generally move less than light atoms.
In the harmonic approximation, each normal mode, q, be- The vibrational state of a polyatomic molecule is specified
haves like an independent harmonic oscillator and has an en- by the vibrational quantum number vq for each of the normal
ergy characterized by the quantum number vq. Expressed as a modes. For example, for H2O with three normal modes, the vi-
wavenumber, these terms are brational state is designated (v1,v2,v3). The vibrational ground
1/ 2 state of an H2O molecule is therefore (0,0,0); the state (0,1,0)
1  k f ,q 
 
G q (v)  vq  12 q , vq  0,1,2, q  
2c  mq


implies that modes 1 and 3 are in their ground states, and mode
2 is in the first excited state.

Vibrational terms of normal modes [harmonic oscillator]  (11D.2)
Exercises
where νq is the wavenumber of mode q; this quantity depends on
E11D.1 How many normal modes of vibration are there for the following
the force constant kf,q for the mode and on the effective mass mq of molecules: (i) H2O, (ii) H2O2, (iii) C2H4?
the mode: stiff bonds and low effective masses correspond to high
E11D.2 Write an expression for the vibrational term for the ground vibrational
state of H2O in terms of the wavenumbers of the normal modes. Neglect
anharmonicities, as in eqn 11D.2.
2 (2349 cm–1)
1 (1388 cm–1)

11D.2 Infrared absorption spectra


The gross selection rule for infrared activity is a straightforward
3 (667 cm–1) 3 (667 cm–1) generalization of the rule for diatomic molecules (Topic 11C):1
The motion corresponding to a normal mode must be
Figure 11D.2 The four normal modes of CO2. The two bending
motions (ν3) have the same vibrational frequency. In this
accompanied by a change of electric dipole moment.
representation, an arrow indicates displacement in one direction,
but the atoms oscillate back and forth about their equilibrium 1
Topic 11E describes a systematic procedure based on group theory for de-
positions. ciding whether a vibrational mode is infrared active.
11D Vibrational spectroscopy of polyatomic molecules 463

Simple inspection of atomic motions is sometimes all that is three normal modes: (1,0,0) ← (0,0,0), (0,1,0) ← (0,0,0), and
needed in order to assess whether a normal mode is infrared (0,0,1) ← (0,0,0).
active. For example, the symmetric stretch of CO2 leaves the Anharmonicity also allows transitions in which more than
dipole moment unchanged (at zero, see Fig. 11D.2), so this one quantum of excitation takes place: such transitions are re-
mode is infrared inactive. The antisymmetric stretch, however, ferred to as overtones. A transition such as (0,0,2) ← (0,0,0)
changes the dipole moment because the molecule becomes in H2O is described as a first overtone, and a transition such
unsymmetrical as it vibrates, so this mode is infrared active. as (0,0,3) ← (0,0,0) is a second overtone of the mode ν3.
Because the dipole moment change is parallel to the principal Combination bands (or combination lines) corresponding to
axis, the transitions arising from this mode are classified as the simultaneous excitation of more than one normal mode in
­parallel bands in the spectrum. Both bending modes are infra- the transition, as in (1,1,0) ← (0,0,0), are also possible in the
red active: they are accompanied by a changing dipole perpen- presence of anharmonicity.
dicular to the principal axis, so transitions involving them lead As for diatomic molecules (Topic 11C), transitions between
to a perpendicular band in the spectrum. vibrational levels can be accompanied by simultaneous changes
in rotational state, so giving rise to band spectra rather than the
single absorption line of a pure vibrational transition. The spec-
Example 11D.1 Using the gross selection rule for infrared tra of linear polyatomic molecules show branches similar to
spectroscopy those of diatomic molecules. For nonlinear molecules, the rota-
State which of the following molecules are infrared active: N2O, tional fine structure is considerably more complex and difficult
OCS, H2O, CH2=CH2. to analyse: even in moderately complex molecules the pres-
ence of several normal modes gives rise to several fundamental
Collect your thoughts Molecules that are infrared active have a transitions, many overtones, and many combination lines, each
normal mode (or modes) in which there is a change in dipole with associated rotational fine structure, and results in infrared
moment during the course of the motion. Therefore, to decide spectra of considerable complexity.
if a molecule is infrared active you need to decide whether These complications are eliminated (or at least concealed)
there is any distortion of the molecule that results in a change
by recording infrared spectra of samples in the condensed
in its electric dipole moment (including changes from zero).
phase (liquids, solutions, or solids). Molecules in liquids do
The solution The linear molecules N2O and OCS both have not rotate freely but collide with each other after only a small
permanent electric dipole moments that change as a result of change of orientation. As a result, the lifetimes of rotational
stretching any of the bonds; in addition, bending perpendicu- states in liquids are very short, which results in a broadening
lar to the internuclear axis results in a dipole moment in that of the associated energies (Topic 11A). Collisions occur at a
direction: both molecules are therefore infrared active. An rate of about 1013 s−1 and, even allowing for only a 10 per cent
H2O molecule also has a permanent dipole moment which success rate in changing the molecule into another rotational
changes either by stretching the bonds or by altering the bond state, a lifetime broadening of more than 1 cm−1 can easily re-
angle: the molecule is infrared active. A CH2=CH2 molecule sult. The rotational structure of the vibrational spectrum is
does not have a permanent dipole moment (it possesses a blurred by this effect, so the infrared spectra of molecules in
centre of inversion) but there are vibrations in which the sym- condensed phases usually consist of bands without any re-
metry is reduced and a dipole moment forms, for example, solved branch structure.
by stretching the two C−H bonds on one carbon atom and
Infrared spectroscopy is commonly used in routine chemi-
simultaneously compressing the two C−H bonds on the other
cal analysis, most usually on samples in solution, prepared as
carbon atom.
a fine dispersion (a ‘mull’), or compressed as solids into a very
Self-test 11D.1 Identify an infrared inactive normal mode of thin layer. The resulting spectra show many absorption bands,
CH2=CH2. even for moderately complex molecules. There is no chance of
stretch synchronously
analysing such complex spectra in terms of the normal modes.
However, they are of great utility for it turns out that certain
Answer: A ‘breathing’ mode in which all the C−H bonds contract and

groups within a molecule (such as a carbonyl group or an −NH2


group) give rise to absorption bands in a particular range of
The specific selection rule in the harmonic approximation is wavenumbers. The spectra of a very large number of molecules
∆vq = ±1. In this approximation the quantum number of only have been recorded and these data have been used to draw up
one active mode can change in the interaction of a molecule charts and tables of the expected range of the wavenumbers of
with a photon. A fundamental transition is a transition from absorptions from different groups. Comparison of the features
the ground state of the molecule to the next higher energy level in the spectrum of an unknown molecule or the product of a
of the specified mode. For example, in H2O there are three such chemical reaction with entries in these data tables is a common
fundamentals corresponding to the excitation of each of the first step towards identifying the molecule.
464 11 Molecular spectroscopy

Exercises
E11D.3 Which of the vibrations of an AB2 molecule are infrared active when it Incident
is (i) angular, (ii) linear? radiation
E11D.4 Consider the vibrational mode that corresponds to the uniform
I||
expansion of the benzene ring. Is it infrared active?
I⊥

Scattered
11D.3 Vibrational Raman spectra radiation

As for diatomic molecules, the normal modes of vibration


of molecules are Raman active if they are accompanied by Figure 11D.4 The definition of the planes used for the
a changing polarizability. A closer analysis of infrared and specification of the depolarization ratio, ρ, in Raman scattering.
Raman activity of normal modes based on considerations of
symmetry leads to the exclusion rule:
If the molecule has a centre of inversion, then no mode I
can be both infrared and Raman active.  Depolarization ratio [definition] (11D.3)
I
(A mode may be inactive in both.) Because it is often possi-
ble to judge intuitively if a mode changes the molecular dipole To measure ρ, the intensity of a Raman line is measured with
moment, this rule can be used to identify modes that are not a polarizing filter (a ‘half-wave plate’) first parallel and then
Raman active. perpendicular to the polarization of the incident beam. If
the emergent light is not polarized, then both intensities are
Brief illustration 11D.2 the same and ρ is close to 1; if the light retains its initial po-
larization, then I⊥= 0 , so ρ = 0. A line is classified as depolar-
The symmetric stretch of CO2 alternately swells and contracts ized if it has ρ close to or greater than 0.75 and as polarized if
the molecule: this motion changes the size and hence the polar- ρ < 0.75. Only totally symmetrical vibrations give rise to po-
izability of the molecule, so the mode is Raman active. Because larized lines in which the incident polarization is largely pre-
CO2 has a centre of inversion the exclusion rule applies, so the served. Vibrations that are not totally symmetrical give rise to
stretching mode cannot be infrared active. The antisymmetric depolarized lines because the incident radiation can give rise to
stretch and the two bends are infrared active, and so cannot be
radiation in the perpendicular direction too.
Raman active.

Exercises
The assignment of Raman lines to particular vibrational modes
E11D.5 Which of the vibrations of an AB2 molecule are Raman active when it
is aided by noting the state of polarization of the scattered light.
is (i) angular, (ii) linear?
The depolarization ratio, ρ, of a line is the ratio of the intensities
E11D.6 Consider the vibrational mode that corresponds to the uniform
of the scattered light with polarization perpendicular and paral-
expansion of the benzene ring. Is it Raman active?
lel, I⊥ and I , to the plane of the incident radiation (Fig. 11D.4):
E11D.7 Does the exclusion rule apply to H2O?

Checklist of concepts
☐ 1. A normal mode is a synchronous displacement of the ☐ 4. The exclusion rule states that, if the molecule has a cen-
atoms in which the centre of mass and orientation of the tre of inversion, then no mode can be both infrared and
molecule remains fixed. In the harmonic approxima- Raman active.
tion, normal modes are mutually independent. ☐ 5. Polarized lines preserve the polarization of the incident
☐ 2. A normal mode is infrared active if it is accompanied radiation in the Raman spectrum and arise from totally
by a change of electric dipole moment; in the harmonic symmetrical vibrations.
approximation the specific selection rule is ∆vq = ±1.
☐ 3. A normal mode is Raman active if it is accompanied by
a change in polarizability; in the harmonic approxima-
tion the specific selection rule is ∆vq = ±1.
11D Vibrational spectroscopy of polyatomic molecules 465

Checklist of equations
Property Equation Comment Equation number
Number of normal modes Nvib = 3N − 5 (linear) Independent if harmonic; N is the number of atoms 11D.1
Nvib = 3N − 6 (nonlinear)
Vibrational terms of normal modes  
G q (vq )  vq  12 q , Harmonic approximation 11D.2

q  (1/2c)(kf ,q /mq )1/ 2

Depolarization ratio   I  /I Depolarized lines: ρ close to or greater than 0.75; 11D.3
polarized lines: ρ < 0.75
TOPIC 11E Symmetry analysis of
­vibrational spectra

3. The resulting representation is decomposed into its com-


➤ Why do you need to know this material? ponent irreducible representations, denoted Γ with char-
The analysis of vibrational spectra is aided by under- acters χ(Γ)(C), by using the relevant character table in
standing the relationship between the symmetry of the conjunction with eqn 10C.3a in the form:
molecule, its normal modes, and the selection rules that 1
govern the transitions.
n()  N (C) () (C) (C)
h C

➤ What is the key idea? where h is the order of the group and N(C) the number of
operations in the class C.
The vibrational modes of a molecule can be classified
according to the symmetry of the molecule. 4. The symmetry species corresponding to x, y, and z (cor-
responding to translations) and those corresponding to
➤ What do you need to know already? the rotations about x, y, and z (denoted Rx, Ry, and Rz) are
You need to be familiar with the vibrational spectra of removed. Their symmetry species are listed in the charac-
polyatomic molecules (Topic 11D) and the treatment of ter tables.
symmetry in Focus 10. 5. The remaining symmetry species correspond to the nor-
mal modes.

Example 11E.1  Identifying the symmetry species of the


The classification of the normal modes of vibration of a normal modes of H2O
­polyatomic molecule according to their symmetry makes it
possible to predict in a very straightforward way which are Identify the symmetry species of the normal modes of H2O,
infra­red or Raman active. which belongs to the point group C2v.
Collect your thoughts You need to identify the axes in the
molecule and then refer to the character table (in the Resource
Classification of normal modes
11E.1
section) for the symmetry operations and their characters. You
need consider only one symmetry operation of each class,
according to symmetry because all members of the same class have the same character.
(In C2v, there is only one member of each class anyway.) Then
Each normal mode can be classified as belonging to one of the follow the five steps outlined in the text. Note from the charac-
symmetry species of the irreducible representations of the mo- ter table the symmetry species of the translations and rotations,
lecular point group. The classification proceeds as follows: which are given in the right-hand column.

1. The basis functions are the three displacement vectors The solution The molecule lies in the yz-plane, with the z-axis
(x, y, and z) on each atom: there are 3N such basis func- bisecting the HOH bond angle. The three displacement vectors
of each atom are shown in Fig. 11E.1. From the character table
tions for a molecule with N atoms.
for C2v, the symmetry operations are E, C2, σv(xz), and  v ( yz ).
2. The character, χ(C), for each class, C, of operations It is seen that:
in the group is found by considering the effect of one
operation of the class and counting 1 for each basis • None of the nine displacement vectors is affected by the
function that is unchanged by the operation, −1 if the operation E, so χ(E) = 9.
basis function changes sign, and 0 if it changes into • The C2 operation moves all the displacement vectors
some other displacement. on the H atoms to other positions, so these count 0;
11E Symmetry analysis of ­vibrational spectra 467

z ­igher-dimensional irreducible representations in C2v mol-


h
C2 σv (yz)
ecules, so vibrational degeneracy never arises. Degeneracy can
arise in molecules with higher symmetry, as illustrated in the
σv(xz) following example.

x
Example 11E.2  Identifying the symmetry species of the
y 2
normal modes of BF3
1 Identify the symmetry species of the normal modes of vibra-
tion of BF3, which is trigonal planar and belongs to the point
Figure 11E.1 The atomic displacements of H2O and the symmetry group D3h.
elements used to calculate the characters.
Collect your thoughts The overall procedure is the same as
in Example 11E.1. However, because the molecule is D3h,
the x and y displacement vectors on the O atom change which has two-dimensional irreducible representations (E′
sign, giving a count of −1 each, whereas the z displace- and E″), you need to be alert for the possibility that there are
ment vector is unaffected, giving a count of +1. Hence doubly degenerate pairs of normal modes. You can treat the
χ(C2) = −1 − 1 + 1 = −1. displacement vectors on the B atom separately from those on
• The operation σv(xz) moves all the displacement vectors the F atoms because no symmetry operation interconverts
on the H atoms to other positions, changes the sign of the these two sets: this separation simplifies the calculations.
y displacement vector on the O atom, and leaves the x and Because the molecule is nonlinear with 4 atoms, there are 6
z vectors unaffected: hence χ(σv) = −1 + 1 + 1 = 1. normal modes.
• The operation  v ( yz ) changes the sign of the x displacement The solution The C3 axis is the principal axis and defines the
vectors on both H atoms and leaves the sign of the y and z z-direction; the molecule lies in the xy-plane. The three C2 axes
displacement vectors unaffected. For the O atom, the x vec- pass along the B−F bonds, and the three σv planes contain the
tor changes sign and the y and z displacement vectors are B−F bonds and are perpendicular to the plane of the molecule.
unaffected: hence   v   1  1  1  1  1  1  1  1  1  3. The σh plane lies in the plane of the molecule, and the S3 axis is
coincident with the C3 axis.
• The characters of the representation are therefore 9, −1, 1, 3.
First, consider the displacement vectors on the B atom.
Because this atom lies on the principal axis, the z displacement
Decomposition by using eqn 10C.3a shows that the reducible
vector must transform as the function z, which from the char-
representation spans the symmetry species 3A1 + A2 + 2B1 + 3B2.
acter table has the symmetry species A′′2 . Similarly, the x and y
The translations have symmetry species B1, B2, and A1 and the
displacement vectors together transform as E′.
rotations have symmetry species B1, B2, and A2; their removal
Next, consider the nine displacement vectors on the F atoms:
leaves 2A1 + B2 as the symmetry species of the normal modes.
As expected, there are three such modes. • The identity operation has no effect, so χ(E) = 9.
Comment. In Fig. 11D.3 (Topic 11D) depicting the normal • The C3 operation moves all these vectors, so χ(C3) = 0.
modes of H2O, ν1 and ν2 have symmetry A1, and ν3 has symme- • A C2 operation about a particular B−F bond has no effect
try B2. This assignment is evident from the fact that the com- on the displacement vector that points along the bond,
bination of displacements for both ν1 and ν2 are unchanged by but the other two vectors change sign; the displacement
any of the operations of the group, so the characters are all 1 vectors on the other F atoms are moved, hence χ(C2) =
as required for A1. In contrast, for ν3 the displacements shown 1 − 1 − 1 = −1.
change sign under C2 and σv giving the characters 1, −1, −1, 1,
which correspond to B2.
• Under σh the z displacement vector on each F changes
sign, but the x and y vectors do not. The character is
Self-test 11E.1 Identify the symmetry species of the normal therefore χ(σh) = 3 × (−1 + 1 + 1) = 3.
modes of methanal, H2C=O, point group C2v (orientate the
• The character for S3 is the same as for C3, χ(S3) = 0.
molecule in the same way as H2O, with the CH2 group in the
yz-plane). • A σv reflection in a plane containing a particular B−F
Answer: 3A1 + B1 + 2B2 bond has no effect on the displacement vector that points
along the bond, nor on the z displacement vector; how-
ever, the other vector changes sign. The displacement
All the normal modes of H2O have symmetry species A vectors on the other atoms are moved, hence χ(σv) =
or B and therefore are non-degenerate. There are no two- or 1 + 1 − 1 = 1.
468 11 Molecular spectroscopy

provided that x is replaced by the normal coordinate, Qq,


which is the combination of displacements that corresponds to
the normal mode. For example, in the case of the symmetric
stretch of CO2 the normal coordinate is zO,1 − zO,2, where zO,i is
the z-displacement of oxygen atom i.
 (A1)
The effect of any symmetry operation on the normal coordi-
nate of a non-degenerate normal mode is either to leave it un-
changed or at most to change its sign. In other words, all the
characters are either 1 or −1. The ground-state wavefunction is
a function of the square of the normal coordinate, so regardless
 (A2 )  (E )  (E ) of whether Qq → +Qq, or Qq → −Qq as a result of any symmetry
operation, the effect on Qq2 is to leave it unaffected. Therefore,
Figure 11E.2 The normal modes of vibration of BF3. all the characters for Qq2 are 1, so the ground-state wavefunction
transforms as the totally symmetric irreducible representation
(typically A1).
The characters of the reducible representation are therefore 9,
The first excited state wavefunction is a product of a part
0, −1, 3, 0, 1; this set can be decomposed into the symmetry
that depends on Qq2 (the exponential term) and a factor propor-
species A1  A2  2E  A1  E for the displacement vectors on
tional to Qq. As has already been seen, Qq2 transforms as the to-
the F atoms. The displacement vectors on the B atom trans-
tally symmetric irreducible representation and Qq has the same
form as A2  E, so the complete set of symmetry species is
A1  A2  3E  2A2  E. symmetry species as the normal mode. The direct product of
The character table shows that z transforms as A′′2 , and x and the totally symmetric irreducible representation with any sym-
y together span E′. The rotation about z, Rz, transforms as A′2 metry species leaves the latter unaffected, hence it follows that
and rotations about x and y together (Rx,Ry) transform as E″. the symmetry of the first excited state wavefunction is the same
Removing these symmetry species from the complete set leaves as that of the normal mode.
A1  2E  A2 as the symmetry species of the vibrational modes.
Figure 11E.2 shows these normal modes. 11E.2(a) Infrared activity of normal modes
Comment. Because the E′ symmetry species is two-­dimensional,
Once the symmetry of a particular normal mode is known it
the corresponding normal mode is doubly degenerate. The
is a simple matter to determine from the appropriate character
above analysis shows that there are two E′ symmetry species
table whether or not the fundamental transition of that mode is
present, which correspond to two different doubly degenerate
allowed and therefore if the mode is infrared active.
normal modes. The total number of normal modes represented
by A1  2E  A2 is therefore 1 + 2 × 2 + 1 = 6.
Self-test 11E.2 Identify the symmetry species of the normal
modes of ammonia NH3, point group C3v. How is that done? 11E.1 Determining the infrared activity
Answer: 2A1 + 2E of a normal mode
You need to note that the fundamental transition of a particular
normal mode is the transition from the ground state, vq = 0, to
Exercise the first excited state, vq = 1. You already know that the state
E11E.1 The molecule CH2Cl2 belongs to the point group C2v. The displacements
with vq = 0 transforms as the totally symmetric irreducible rep-
of the atoms span 5A1 + 2A2 + 4B1 + 4B2. What are the symmetry species of resentation, and the state with vq = 1 has the same symmetry as
the normal modes of vibration? the corresponding normal mode.
Step 1 Formulate the integral used to identify the selection rule
Whether or not the transition between vq = 0 and vq = 1 is
Symmetry of vibrational
11E.2 allowed is assessed by evaluating the transition dipole between
ψ0 and ψ1, µ10 = ∫ψ 1µˆψ 0dτ (Topic 11A); the dipole moment
wavefunctions operator transforms as x, y, or z (Topic 10C). As is shown in
Topic 10C, this integral may be non-zero only if the integrand,
For a one-dimensional harmonic oscillator the ground-state
2 2 that is the product ψ 1µˆψ 0 , spans the totally symmetric irreduc-
wavefunction (with v = 0) is proportional to e  x / 2 , where x ible representation.
is the displacement from the equilibrium position and α is a
constant (Topic 7E). For the first excited state, with v = 1, the Step 2 Identify the symmetry species spanned by the integrand
2 2
wavefunction is proportional to xe  x / 2 . The same wavefunc- You can find the symmetry species of ψ 1µˆψ 0 by taking the
tions apply to a normal mode q of a more complex molecule direct product of the symmetry species spanned by each of
11E Symmetry analysis of ­vibrational spectra 469

ψ1, µ̂ , and ψ0 separately (Topic 10C). Because ψ0 transforms that (x2−y2, xy) together transform as E′. The A1′ normal mode
as the totally symmetric irreducible representation it has no and the two doubly degenerate E′ normal modes are therefore
effect on the symmetry of ψ 1µˆψ 0 , and so you need consider Raman active. The A′′2 mode is not active because no quad-
only the symmetry of the product ψ 1µˆ . As is shown in Topic ratic form has this symmetry species. The E′ modes are both
10C, the only way for this product to span the totally symmet- infrared and Raman active: the exclusion rule does not apply
ric irreducible representation is for ψ 1 and µ̂ to span the same because BF3 does not have a centre of symmetry. The A1′ nor-
symmetry species. In other words, the integral can be non-zero mal mode is the highly symmetrical breathing mode in which
only ifψ 1, and hence the normal mode, has the same symmetry all the B−F bonds stretch together. The corresponding Raman
species as x, y, or z. line is expected to be polarized. The E′ modes are expected to
The result of the analysis can be summarized by the follow- give depolarized lines.
ing rule:
A mode is infrared active only if its symmetry species is
the same as the symmetry species of any of x, y, or z.
11E.2(c) The symmetry basis of the
exclusion rule
Brief illustration 11E.1 The exclusion rule, Topic 11D, can be derived by using a
symmetry argument. If a molecule has a centre of inversion,
The normal modes of BF3 (point group D3h) have symmetry then all the symmetry species of its displacements are either
species A1  2E  A2 (Example 11E.2). From the character g or u according to their behaviour under inversion. If the
table it can be seen that z transforms as A′′2 and (x,y) together character for this operation is positive, indicating that the
transform as E′. The A′′2 normal mode and the two doubly displacement or displacements are unchanged by the opera-
degenerate E′ normal modes are therefore infrared active. The tion, the label is g, whereas if the character is negative, indi-
A′1 mode is not. cating that the sign of the displacement or displacements are
changed, the label is u.
The functions x, y, and z (which occur in the transition di-
11E.2(b) Raman activity of normal modes pole moment) all change sign under inversion, so they must
correspond to symmetry species with a label u. In contrast,
Symmetry arguments also provide a systematic way of deciding the quadratic forms (which govern the Raman activity) are
whether or not the fundamental of a normal mode gives rise to all unchanged by inversion and so have the label g. For ex-
Raman scattering; that is, whether or not the mode is Raman ample, the effect of the inversion on xz is to transform it into
active. The argument is similar to that for assessing infrared (−x)(−z) = xz.
activity except that it is based on the symmetry of the polariz- Any normal mode in a molecule with a centre of inver-
ability operator rather than the dipole moment operator. That sion corresponds to a symmetry species that is either g or u.
operator transforms in the same way as the quadratic forms If the normal mode has the same symmetry species as x, y, or
(x2, xy, and so on, which are listed in the character tables), and z, it is infrared active; such a mode must be u. If the normal
leads to the following rule: mode has the same symmetry species as a quadratic form, it
A mode is Raman active only if its symmetry species is the is Raman active; such a mode must be g. Because a normal
same as the symmetry species of a quadratic form. mode cannot be both g and u, no mode can be both infrared
and Raman active.

Brief illustration 11E.2


Exercises
The normal modes of BF3 (point group D3h) have symmetry E11E.2 Which of the normal modes of CH2Cl2 (found in Exercise E11E.1) are
species A1  2E  A2 (Example 11E.2). From the character infrared active? Which are Raman active?
table it can be seen that x2, y2, and z2 all transform as A1′, and E11E.3 Which of the normal modes of (i) H2O, (ii) H2CO are infrared active?

Checklist of concepts
☐ 1. A normal mode is infrared active if its symmetry species ☐ 2. A normal mode is Raman active if its symmetry species
is the same as the symmetry species of x, y, or z. is the same as the symmetry species of a quadratic form.
TOPIC 11F Electronic spectra

➤ Why do you need to know this material?

Absorbance, A
The study of spectroscopic transitions between different
electronic states of molecules gives access to data on
the electronic structure of molecules, and hence insight
into bonding, as well as vibrational frequencies and bond
(a)
lengths.
500 550 600 650
➤ What is the key idea? Wavelength, λ/nm

Electronic transitions occur within a stationary nuclear

Absorbance, A
framework.

➤ What do you need to know already?


You need to be familiar with the general features of spec-
troscopy (Topic 11A), the quantum mechanical origins of (b)
selection rules (Topics 8C, 11B, and 11C), and vibration–
400 500 600 700
rotation spectra (Topic 11C). It would be helpful to be Wavelength, λ/nm
aware of atomic term symbols (Topic 8C).
Figure 11F.1 Electronic absorption spectra recorded in the visible
region. (a) The spectrum of I2 in the gas phase shows resolved
vibrational structure. (b) The spectrum of chlorophyll recorded
in solution, shows only broad bands with no resolved structure.
Electronic spectra arise from transitions between the electronic (Absorbance, A, is defined in Topic 11A.)
energy levels of molecules. These transitions may also be ac-
companied by simultaneous changes in vibrational energy; for Table 11F.1 Colour, frequency, and energy of light⋆
small molecules in the gas phase the resulting spectral features
can be resolved (Fig. 11F.1a), but in a liquid or solid the indi- Colour λ/nm ν/(1014 Hz) E/(kJ mol−1)
vidual lines usually merge together and result in a broad, al- Infrared >1000 <3.0 <120
most featureless band (Fig. 11F.1b). Red 700 4.3 170
The energies needed to change the electron distributions Yellow 580 5.2 210
of molecules are of the order of several electronvolts (1 eV is Blue 470 6.4 250
equivalent to about 8000 cm−1 or 100 kJ mol−1). Consequently,
Ultraviolet <400 >7.5 >300
the photons emitted or absorbed when such changes occur

lie in the visible and ultraviolet regions of the spectrum More values are given in the Resource section.

(Table 11F.1).

11F.1(a) Term symbols


In a diatomic molecule only the component of the total orbital
11F.1 Diatomic molecules angular momentum along the internuclear axis can be speci-
fied: the quantum number for this component is Λ (uppercase
Topic 8C explains how the states of atoms are described by
lambda). Its value is found by adding together the component
using term symbols. The electronic states of diatomic molecules
of the orbital angular momentum along the internuclear axis,
are also specified by using term symbols, the key difference
λi, for each electron present:
being that the full spherical symmetry of atoms is replaced by
the cylindrical symmetry defined by the axis of the molecule.   1  2   (11F.1)
11F Electronic spectra 471

For an electron in a σ molecular orbital (which is cylindri- gg  g uu  g u  g  u (11F.2)


cally symmetric), λ = 0; for an electron in one of the degener-
for each electron. (These rules are generated by interpreting g
ate pair of π orbitals λ = ±1. In the molecular term symbol
as +1 and u as −1.) The resulting parity label g or u is added as a
the value of Λ is represented by an uppercase Greek letter in
right-subscript to the term symbol. Any molecule in which the
the following way
occupied orbitals are full (in the sense of being occupied by a
|Λ| 0 1 2 … pair of electrons) must have overall parity g because there is an
even number of electrons. The term symbol for such a homo-
Σ Π Δ …
nuclear diatomic molecule is therefore 1Σg (‘singlet sigma g’).
These labels are the analogues of S, P, D,… used for atomic
states with L = 0, 1, 2,…. The total spin, S, of a linear molecule is
Brief illustration 11F.2
specified in the same way as for an atom. As in an atomic term
symbol, the value of 2S + 1 is shown as a left superscript and
Dinitrogen, N2, has the configuration 1 g21 u2  g2  u2u4 3 g2 in
denotes the multiplicity of the term. which all the occupied orbitals are full; the same is true of H2 and
Configurations such σ2 and π4 with all electrons paired have F2: all three therefore have the term symbol 1Σg. The configura-
S = 0. Such configurations do not contribute to the total orbital tion of He2+ is 1σ g21σ u1 . There is one electron outside the doubly
angular momentum, either because both electrons have λ = 0 or occupied bonding orbital, and the parity of the orbital it occu-
because there are equal numbers of electrons with λ = +1 and pies is u. Because S = 12 and Λ = 0, its term symbol is 2Σu (‘dou-
−1. The term symbol for the ground state of H2, configuration blet sigma u’). The ground-state configuration of O2 is 12g.
1σ g2, is therefore 1Σ (read as ‘singlet sigma’), and the same is true Although the πg orbitals may be both singly occupied, both
of the ground state of N2, configuration 1 g2 1 u2  g2  u2 u4 3 g2. electrons are in orbitals with g parity, so the overall parity is
g × g = g. The three terms arising from this configuration are
therefore 1Σg, 3Σg, and 1Δg (see Brief illustration 11F.1).
Brief illustration 11F.1

The ground-state configuration of H2+ is 1σ g1. The single σ elec-


Diatomic molecules (and all linear molecules) possess a mir-
tron has λ = 0, so Λ = 0; for a single electron S = 12 , so 2S + 1 = 2.
ror plane containing the internuclear axis. All σ orbitals (both
The term symbol is therefore 2Σ (read as ‘doublet sigma’). The
bonding and antibonding) are symmetric with respect to reflec-
ground-state configuration of NO is …1π1, where … indicates
tion in this plane. The overall symmetry of a configuration is
completed orbitals that make no contribution to either S or
found by assigning +1 to an electron in a symmetric orbital and
Λ. The single electron can occupy either of the degenerate π
orbitals, so Λ = +1 or −1; for a single electron, S = 12 . The term −1 to an electron in an orbital that changes sign under reflec-
symbol is therefore 2Π (read as ‘doublet pi’). The ground-state tion and then multiplying the numbers for all the electrons. For
configuration of O2 is 12g. If the two electrons occupy the example, for the ground state of H2, in which both electrons are
same π orbital, Λ = (+1) + (+1) = +2 (or Λ = (−1) + (−1) = −2); in σ orbitals, the overall symmetry is (+1) × (+1) = +1. A + sign
in this arrangement the electron spins must be paired, so S = 0. is added as a right-superscript to the term symbol: 1  g (‘singlet
The resulting term is 1Δ (‘singlet delta’); a 3Δ term is not pos- sigma g plus’). Any configuration consisting of electrons solely
sible as it would require two electrons with parallel spins to in σ orbitals necessarily has + overall reflection symmetry; for
occupy one of the π orbitals. If the electrons occupy different example, the ground state of He2+ (Brief illustration 11F.2) is 2  u .
π orbitals, Λ = (+1) + (−1) = 0; in this arrangement the spins The behaviour of the degenerate pair of π molecular orbitals
can be paired or parallel, so S = 0 or S = 1. Two further terms under reflection is more complex: as is shown in Fig. 11F.2, one
therefore arise: 1Σ and 3Σ (‘triplet sigma’); the latter turns out to
be the lowest in energy of all the three terms.
+ –
+
+ –
As explained in Topic 9B, homonuclear diatomic molecules + – +

(but not heteronuclear diatomic molecules) possess a centre of


inversion and their orbitals are labelled g or u according to their –
parity (the behaviour under inversion). Orbitals that are un-
changed upon inversion are g and orbitals that change sign are
Figure 11F.2 A molecular orbital can be classified as symmetric
u. Parity labels also apply to centrosymmetric polyatomic linear (+) or antisymmetric (−) according to whether it changes sign
molecules, such as CO2 and HC≡CH. The overall parity of a con- under reflection in a plane containing the internuclear axis. Shown
figuration of a many-electron homonuclear diatomic molecule is here are the two degenerate π MOs: one is symmetric and one is
found by noting the parity of each occupied orbital and using antisymmetric under reflection in the plane shown.
472 11 Molecular spectroscopy

ϕ Reflection is Ψ+(1,2) = π+(r1)π−(r2) + π+(r2)π−(r1). Reflection changes this


function to π−(r1)π+(r2) + π−(r2)π+(r1) which is +Ψ+(1,2). The
Mirror plane state is symmetric with respect to this reflection, and a super-
–ϕ
script + is added to the term symbol, to give 1  g (‘singlet sigma
g plus’).

Figure 11F.3 In a linear molecule, the molecular orbital depends


on the azimuthal angle φ. Reflection in the mirror plane is
equivalent to reversing the sign of φ. As for atoms, it is sometimes necessary to specify the total
electronic angular momentum, the sum of the orbital and spin
contributions, and hence the different ‘levels’ of a term. In a lin-
of the orbitals changes sign but the other does not. The con-
ear molecule, only the component of the total electronic angu-
sequences of this observation can be explored by considering
lar momentum along the internuclear axis is well defined, and
the mathematical form of the π orbitals and how they depend
is specified by the quantum number Ω (uppercase omega). For
on the angle φ shown in Fig. 11F.3. The orbital πx is propor-
light molecules, where the spin–orbit coupling is weak, Ω is
tional to cos φ and therefore has a nodal plane at φ = π/2 (the
obtained by adding together the components of the orbital an-
yz-plane) with positive and negative lobes on either side of this
gular momentum along the axis (the value of Λ) and the com-
plane; it is unchanged by reflection in the xz-plane. The orbital
ponent of the electron spin on that axis (Fig. 11F.4). The latter is
πy is proportional to sin φ, so the xz-plane at φ = 0 is a nodal
denoted Σ, where Σ = S, S − 1, S − 2,…, −S.1 Then
plane; the orbital changes sign on reflection in the xz-plane.
These two wavefunctions are degenerate, so any linear combi-    (11F.3)
nation of them is also an acceptable wavefunction. For the pre-
The value of |Ω| is then attached to the term symbol as a right-
sent ­discussion the combinations π+ = cos φ + i sin φ = e+iφ and
subscript (just like J is used in atoms) to denote the different
π− = cos φ − i sin φ = e−iφ are convenient for they correspond to
levels. These levels differ in energy, as in atoms, as a result of
λ = +1, and λ = −1, respectively. As shown in Fig. 11F.3, reflection
spin–orbit coupling.
in the xz-plane changes the sign of φ. Because cos(−φ) = cos φ,
the term cos φ is unchanged by this reflection. However, sin φ
changes sign because sin(−φ) = −sin φ. It follows that this reflec-
tion interconverts π+ and π−. Brief illustration 11F.4
Now consider O2, which has the configuration 12g. The tri-
plet state (S = 1), in which the two electrons have parallel spins The ground-state configuration of NO is …1π1, so it is a 2Π
and necessarily occupy different orbitals, is the state of lowest term with Λ = ±1 and S = 12 ; from the latter it follows that
energy. The triplet spin wavefunction is symmetric with re-    12 . There are two levels of the term, one with    12 and
spect to the interchange of the two electrons (it is of the form the other with ± 23 , denoted 2Π1/2 and 2Π3/2, respectively. Each
α(1)α(2), and so on), so it follows from the Pauli principle level is doubly degenerate (corresponding to the opposite signs
of Ω). It turns out that, in NO, 2Π1/2 (‘doublet pi one half ’) lies
(Topic 8B) that the spatial part of the wavefunction must be an-
slightly lower in energy than 2Π3/2.
tisymmetric with respect to interchange. Such a wavefunction,
in which one electron occupies the π+ orbital and the other occu-
pies π−, is Ψ−(1,2) = π+(r1)π−(r2) − π+(r2)π−(r1). Reflection in the
mirror plane, which interconverts π+ and π−, gives π−(r1)π+(r2) − L
π−(r2)π+(r1) = −Ψ−(1,2). That is, the spatial wavefunction of the
triplet state is antisymmetric with respect to reflection in the
S
mirror plane, and so a right-superscript − is attached to the
term symbol, to give 3  g (‘triplet sigma g minus’).
Λ Σ
Ω
Brief illustration 11F.3
Figure 11F.4 The coupling of spin and orbital angular momenta
An alternative, higher energy configuration of O2 has the out- in a linear molecule: only their components along the internuclear
ermost two electrons in separate π orbitals but with their spins axis (Σ and Λ) are well defined.
paired (a 1Σg term). The spin wavefunction for a singlet (which
is proportional to α(1)β(2) − α(2)β(1)) is antisymmetric with
respect to the interchange of the electrons. The spatial func- 1
It is important to distinguish between the upright term symbol Σ and the
tion therefore must be symmetric. A suitable wavefunction sloping quantum number Σ.
11F Electronic spectra 473

11F.1(b) Selection rules


A number of selection rules govern which transitions can be
observed in the electronic spectrum of a molecule. The selec-
tion rules concerned with changes in angular momentum in a
linear molecule are
  0,  1 S  0   0   0,  1
Selection rules for electtronic spectra of linear molecules  (11F.4)

As in atoms (Topic 8C), the origins of these rules are conserva-


tion of angular momentum during a transition and the fact that
Figure 11F.5 A d–d transition is parity-forbidden because it
a photon has a spin of 1.
corresponds to a g–g transition. However, a vibration of the
Two selection rules can be deduced on the basis of symmetry. molecule can destroy the inversion symmetry of the molecule and
the g,u classification no longer applies. The removal of the centre
How is that done? 11F.1 Establishing symmetry-based of inversion gives rise to a vibronically allowed transition.

selection rules
A forbidden g → g transition can become allowed if the centre
As usual when establishing selection rules, you need to con- of inversion is eliminated by an asymmetrical vibration, such
sider properties of the electric-dipole transition moment as the one shown in Fig. 11F.5. When the centre of inversion is
introduced in Topic 8C, µ fi = ∫ψ f µˆψ i dτ , and to note that it lost, g → g and u → u transitions are no longer parity-­forbidden
vanishes unless the integrand is invariant under all symmetry and become weakly allowed. A transition that derives its inten-
operations of the molecule (Topic 10C). sity from an asymmetrical vibration of a molecule is called a
The z-component (the component parallel to the axis of vibronic transition.
the molecules) of the electric dipole moment operator is
responsible for Σ ↔ Σ transitions (the other components of μ
perpendicular to the axis have Π symmetry and cannot make Brief illustration 11F.5
a contribution to this transition). The z-component of μ has
(+) symmetry with respect to reflection in a plane containing Three possible transitions in the electronic spectrum of O2,
the internuclear axis. Therefore, for a (+) ↔ (−) transition, the 3
 g  3  u , 3  g  1  g , 3  g  3  u , can be considered in the
overall symmetry of the integrand is (+) × (+) × (−) = (−), so light of the selection rules in eqn 11F.4 to see which are
the integral must be zero and hence Σ+ ↔ Σ− transitions are not allowed. A table can be drawn up, in which forbidden values
allowed. The integrands for Σ+ ↔ Σ+ and Σ− ↔ Σ− transitions are shown in boxes.
transform as (+) × (+) × (+) = (+) and (−) × (+) × (−) = (+),
respectively. The integrals are therefore not necessarily zero
and so both transitions are allowed.
Change of
The three components of the dipole moment operator trans- ΔS ΔΛ Σ ± ← Σ±
parity
form like x, y, and z, and in a centrosymmetric molecule are
all u. Therefore, for a g → g transition, the overall parity of the
3
 g  3  u 0 0 Σ− ← Σ− g←u Allowed
integrand is g × u × g = u, so the integral must be zero. Likewise, 3 
  g
g
1
+1 −2 Not applicable g←g Forbidden
for a u → u transition, the overall parity is u × u × u = u, 3 
   3 
0 0 
  
g←u Forbidden
g u
so the integral is again zero. Hence, transitions without a
change of parity are forbidden. For a g ↔ u transition the inte-
grand transforms as g × u × u = g, so the transition is allowed.

11F.1(c) Vibrational fine structure


The first part of this analysis can be summarized as follows: An electronic transition may be accompanied by a simultane-
+ + −
For Σ terms, only Σ ↔ Σ and Σ ↔ Σ are allowed. − ous change in the vibrational state of a molecule, giving rise to
vibrational fine structure in the spectrum. In the case of ab-
The second part is in fact the Laporte selection rule for cen- sorption spectra, the transitions are from the ground electronic
trosymmetric molecules (those with a centre of inversion, not state, and typically it is only the ground vibrational level, v″ = 0,
only linear molecules) which states that the only transitions al- of this state that is occupied significantly. In some cases, the
lowed are accompanied by a change of parity. That is, transition from v″ = 0 in the electronic ground state to v′ = 0
For centrosymmetric molecules, only u → g and g → u in the upper electronic state is found to be the strongest, with
transitions are allowed. a sharp decline in intensity as v′ increases. In other cases,
474 11 Molecular spectroscopy

t­ransitions with significant intensity to a range of v′ levels are


seen (as in Fig. 11F.1a).
The Franck–Condon principle accounts for the vibrational Electronically
excited state
fine structure in the electronic spectra of molecules:
Because nuclei are so much more massive than electrons,

Energy
an electronic transition takes place very much faster than
the nuclei can respond. Electronic
ground state
The physical basis of this principle is as follows. As a result
of the electronic transition, electron density is built up rapidly
in new regions of the molecule and removed from others. In
classical terms, the initially stationary nuclei suddenly experi- (a) (b)
ence a new force field, to which they respond by beginning to Internuclear distance, R Internuclear distance, R
vibrate and (in classical terms) swing backwards and forwards Figure 11F.7 (a) If the equilibrium bond lengths of the ground
from their original separation which was maintained dur- and excited electronic states are the same, a vertical transition
ing the rapid electronic excitation. The stationary equilibrium leaves the vibrational state of the molecule unexcited. (b) If the
separation of the nuclei in the initial electronic state therefore equilibrium bond length is greater in the upper electronic state,
becomes a stationary turning point in the final electronic state. the vertical transition ends at a compressed state of the bond and
The transition can be thought of as taking place up the vertical results in vibrational excitation.
line in Fig. 11F.6. This interpretation is the origin of the expres-
sion vertical transition, which denotes an electronic transition
that occurs without change of nuclear geometry and, in classi- during the transition takes the molecule up the vertical line.
cal terms, with the nuclei remaining stationary. The nuclei are not moving initially, and do not start to move
Now consider the two potential energy curves shown in during the transition, so the transition terminates at the
Fig. 11F.7a in which the equilibrium bond lengths are the same turning point of the upper electronic state where the nuclei
and initially the molecule is not vibrating. The vertical transi- are still stationary.
tion takes place from the minimum of the lower curve, the nu- The quantum mechanical version of the Franck–Condon
clei remain at the same separation, and ends at the minimum of principle refines this picture. Instead of saying that the nuclei
the upper curve. stay at the same locations and are stationary during the transi-
Next consider the case shown in Fig. 11F.7b, in which the tion, it replaces that statement by the assertion that the nuclei
equilibrium bond length in the upper state is greater than retain their initial dynamical state. In quantum mechanics, the
that in the ground electronic state, and the molecule is ini- dynamical state is expressed by the wavefunction, so an equiv-
tially not vibrating. Preservation of the nuclear separation alent statement is that the vibrational wavefunction does not
change during the electronic transition. Initially the molecule
is in the lowest vibrational state of its ground electronic state
Turning points
Molecular potential energy, V

(stationary nuclei) with a bell-shaped wavefunction centred on the equilibrium


Electronic bond length (Fig. 11F.8). To find the nuclear state to which
excited the transition takes place, it is necessary to look for the vi-
state brational wavefunction of the upper electronic state that most
closely resembles this initial wavefunction, for that corre-
Electronic sponds to the nuclear dynamical state that is least changed in
ground the transition. That final wavefunction is the one with a large
state
peak close to the position of the initial bell-shaped function.
Nuclei stationary As explained in Topic 7E, provided the vibrational quantum
Internuclear separation, R number is not zero, the biggest peaks of vibrational wavefunc-
tions occur close to the edges of the confining potential, so
Figure 11F.6 According to the Franck–Condon principle, the most the transition can be expected to occur to those vibrational
intense vibronic transition is from the ground vibrational state
states, in accord with the classical description. However, sev-
to the vibrational state lying vertically above it. As a result of the
eral vibrational states have their major peaks in similar posi-
vertical transition, the nuclei suddenly experience a new force
field, to which they respond through their vibrational motion. The
tions, so transitions can be expected to occur to a range of
equilibrium separation of the nuclei in the initial electronic state vibrational states, to give rise to a vibrational progression, a
therefore becomes a turning point in the final electronic state. series of transitions to different vibrational states of the upper
Transitions to other vibrational levels also occur, but with lower electronic state. In a typical progression, the vertical transi-
intensity. tion is the most intense.
11F Electronic spectra 475

where dτe indicates integration over the electronic coordinates,


and dτN integration over the nuclear coordinates. Because the
Electronically two different electronic states are orthogonal (they are eigen-
excited state
Good overlap Poor overlap Good overlap Poor overlap
states of the same hamiltonian but correspond to different
eigenvalues) the integral in the box is zero, which leaves

Energy

S(vf , vi )
   , fi
    
 fi  e    ,f ri  ,i d e  v,f v ,i d N   ,fi S(vf ,vi )

Electronic
i
ground state
The quantity με,fi is the electric-dipole transition moment aris-
ing from the change in the electronic wavefunction: this term
describes the interaction of the electrons with the electromag-
(a) (b) netic field. The factor S(vf,vi), is the overlap integral between
Internuclear distance, R Internuclear distance, R the vibrational level with quantum number vi in the initial
Figure 11F.8 (a) If the equilibrium bond lengths of the ground electronic state of the molecule, and the vibrational level with
and excited electronic states are the same, the wavefunctions quantum number vf in the final electronic state of the molecule.
for v″ = 0 and v′ = 0 are similar and the most probable transition The transition intensity is proportional to the square of the
leaves the molecule vibrationally unexcited. (b) If the equilibrium magnitude of the transition dipole moment, so is propor-
bond length of the upper state is greater than that of the tional to the square of S(vf,vi), which is known as the Franck–
ground state, the wavefunction that most resembles the ground Condon factor
state vibrational wavefunction is that of an excited state. Other
( ∫ψ )
2 2
transitions also occur with lower intensity. S(vf ,vi ) = 
ψ v ,i dτ N
v ,f Franck –Condon factor  (11F.5)

The integral on the right-hand side of eqn 11F.5 is the over-


lap between the two vibrational wavefunctions: the greater
The quantitative version of the Franck–Condon principle this overlap (physically, the greater the resemblance of the
involves considering how the transition dipole moment for a vibrational wavefunctions), the greater is the intensity of the
transition.
given electronic transition varies with the vibrational levels in
the two electronic states.
Example 11F.1   Calculating a Franck–Condon factor
How is that done? 11F.2 Expressing the Franck–Condon
principle quantitatively Consider the transition from one electronic state to another,
their equilibrium bond lengths being Re and Re′; the force con-
Once again, you need to consider the properties of the electric- stants of the two states are assumed to be equal. Calculate the
dipole transition moment. First, note that the electric dipole Franck–Condon factor for the v″ = 0 to v′ = 0 transition (the
moment operator is a sum over all nuclei and electrons in the 0–0 transition) and show that the transition is most intense
molecule: when the bond lengths are equal.
−e ∑ri + e ∑Z N RN
µˆ = Collect your thoughts You need to calculate S(0,0), the
i N
overlap integral of the two ground-state vibrational wave-
where the origin of the vectors r (for locating electrons) and R functions, and then take its square. The difference between
(for locating nuclei) is the centre of charge of the molecule, i harmonic and anharmonic vibrational wavefunctions is
labels the electrons, and N labels the nuclei. Within the Born– negligible for v = 0, so it is safe for you to use the harmonic
Oppenheimer approximation (the separation of electronic and oscillator wavefunctions.
vibrational motion, the Prologue to Focus 9), the overall state of
the molecule consists of an electronic contribution, labelled ε, The solution The (real) wavefunctions are (Topic 7E)
and a vibrational contribution, labelled v. Therefore, the transi- 1/ 2 1/ 2
 1  2 2  1  2 2

tion dipole moment factorizes as follows:  0   1/ 2  e  x  / 2


 0   1/ 2  e  x / 2
     

{
mfi = ∫ψ ⋆ f ψ v⋆,f −e ∑ri + e ∑Z N RN ψ ,i ψ v ,i dτ
µ
i N
} where x = R − Re and x  R  Re, with α = (ћ2/μkf )1/4. It is

convenient to introduce R  Re  Re so that x  x  R . The
overlap integral is
= −e ∑ ∫ψ ⋆,f ri ψ ,i dτ e ∫ψ v⋆,f ψ v ,i dτ N
i  1  ( x 2  x 2 )/ 2 2
S(0,0)    0 0dR 
1/ 2 
0
e dx


+ e ∑ ZN ∫ψ ⋆,fψ ,i dτ e ∫ψ v⋆,f RNψ v ,i dτ N 1  { x 2  ( x  R )2 }/ 2 2


1/ 2 
 e dx
N
476 11 Molecular spectroscopy

The integral can most easily be evaluated by using mathemati- 11F.1(d) Rotational fine structure
cal software (to find the integral by hand use the substitution
x 2  (x  R)2  2  x  12 R   12 R2 . The result is Electronic transitions may be accompanied by simultaneous
2

2
changes in both vibrational and rotational energy. Therefore,
/ 4 2
S(0,0)  e  R when the lines dues to vibrational fine structure are inspected
and hence the Franck–Condon factor is at higher resolution they are found to have rotational fine
 2
/ 2 2
structure and to consist of P, Q, and R branches of the type dis-
S(0,0)2  e ( Re  Re )
cussed in Topic 11C. Because electronic excitation can result
This factor is equal to 1 when Re  Re and decreases as the in much larger changes in bond length than vibrational excita-
equilibrium bond lengths diverge from each other (Fig. 11F.9). tion causes alone, the rotational branches have a more complex
For 79Br2, Re = 228 pm and there is an upper state with structure than in vibration–rotation spectra.
Re  266 pm . With the vibrational wavenumber taken as The rotational constants of the electronic ground and excited
250 cm−1 it follows that α2 = 3.42 × 10−23 m2 and hence S(0,0)2 = states are denoted B and B ′, respectively. The rotational terms of
6.7 × 10−10. That is, the intensity of the 0–0 transition is only the initial and final states are
6.7 × 10−10 times what it would have been if the potential curves
had been directly above each other. F ( J )  BJ
 ( J  1) and F ( J )  B J ( J   1) (11F.6)

Self-test 11F.1 Suppose the normalized vibrational wavefunc- When a transition occurs with ΔJ = −1 the wavenumber of the
tions can be approximated by rectangular functions of width W vibrational contribution to the electronic transition changes
and W′, centred on the equilibrium bond lengths (Fig. 11F.10). from ν to
Find the corresponding Franck–Condon factors when the cen-
  B ( J  1)J  BJ
 ( J  1)    (B   B )J  ( B   B ) J 2
tres are coincident and W′ < W.
This transition is a line in the P branch (just as in Topic 11C).
There are corresponding lines in the Q and R branches
1 with wavenumbers that are calculated similarly. All three
branches are:
P branch (J  1): P ( J )    (B  B )J  (B  B )J 2 (11F.7a)

Q branch (J  0): Q ( J )    (B  B )J ( J  1) (11F.7b)


S(0,0)2

0.5

R branch (J  1):  R ( J )    (B  B )( J  1)  (B  B )( J  1 )2


 (11F.7c)

Spectroscopic data may be analysed by the method of ‘combi-


0 nation differences’ introduced in Topic 11C, as shown in the
0 2 4
(Re – Re´)/21/2α following Example.

Figure 11F.9 The Franck–Condon factor for the arrangement Example 11F.2   Estimating rotational constants from
discussed in Example 11F.1.
electronic spectra
The following rotational transitions were observed in the 0–0
W´ band of the 1Σ+ ← 1Σ+ electronic transition of 63 Cu 2H: R (3) 
23347.69 cm−1,  P (3)  23298.85 cm1, and  P (5)  23275.77 cm1.
Estimate the values of B ′ and B.

Wavefunction, ψ

Collect your thoughts According to the method of combi-


W
nation differences, form the differences R ( J )  P ( J ) and
R ( J  1)  P ( J  1) from eqns 11F.7a and 11F.7c, then use the
resulting expressions to calculate the rotational constants B′ 
and B from the data provided.
Displacement, x
The solution From eqns 11F.7a and 11F.7c it follows that the
Figure 11F.10 The model wavefunctions used in Self-test 11F.1. differences are

νR ( J ) − νP ( J ) = (B ′ + B )( J + 1) + (B ′ − B )( J + 1)2


)J 2 } 4 B ′ ( J + 21 )
−{−(B ′ + B )J + (B ′ − B=
Answer: S2 = W′/W
11F Electronic spectra 477

νR ( J − 1) − νP ( J + 1) = (B ′ + B )J + (B ′ − B )J 2 J max  (B  3B )/2(B   B ). When the bond is shorter in the
−{−(B ′ + B )( J + 1) + (B ′ − B )( J + 1)2 } excited state than in the ground state, B   B and B   B  0.
In this case, the lines of the P branch begin to converge and
= 4 B ( J + 12 )
form a band head when J max  (B   B )/2(B   B ), as shown in
(These equations are analogous to eqn 11C.14.) Then insert Fig. 11F.11b.
the data:
23347.69 23298.85
 Brief illustration 11F.6
For J  3 : R (3)  P (3)  48.84 cm 1  14 B 
23347.69 23275.77
For the transition described in Example 11F.2, B   B,
 so a band
 head is expected in the R branch. The approximate value of J at
For J  4 : R (3)  P (5)  71.92 cm 1  18 B
which this occurs is given by
Therefore, B   3.489 cm 1 and B  3.996 cm 1.
B  3B  (3.996 cm 1 )  3  (3.489 cm 1 )
J max    6.38
Self-test 11F.2 The following rotational transitions were 2(B   B ) 2  {(3.489 cm 1 )  (3.996 cm 1 )}
observed in the 1Σ+ ← 1Σ+ electronic transition of RhN: R (5) 
22 387.06 cm −1, P(5)  22 376.87 cm 1, and P(7)  22 373.95 cm 1. The closest integer is J = 6.
Estimate the values of B ′ and B.
Answer: B  0.4632 cm 1, B   0.5042 cm 1

Exercises
If the bond length in the electronically excited state is E11F.1 What is the full term symbol of the ground electronic state of Li 2 ?
+

greater than that in the ground state, then B   B and there- E11F.2 One of the excited states of the C2 molecule has the valence electron
fore B   B  0. In this case the lines of the R branch converge configuration 1 g2 1 u2 13u 11g . Give the multiplicity and parity of the term.
with increasing J. At sufficiently high values of J, the negative E11F.3 Which of the following transitions are electric-dipole allowed?
term in (J + 1)2 in eqn 11F.7c will dominate the positive term (i) 2Π ↔ 2Π, (ii) 1Σ ↔ 1Σ, (iii) Σ ↔ Δ, (iv) Σ+ ↔ Σ−, (v) Σ+ ↔ Σ+.
in (J + 1) and the lines will start to appear at successively de- 1 1 
E11F.4 The R-branch of the  u   g transition of H2 shows a band head
creasing wavenumbers. That is, the R branch has a band head at the very low value of J = 1. The rotational constant of the ground state is
(Fig. 11F.11a) corresponding to the line of highest frequency 60.80 cm−1. What is the rotational constant of the upper state? Has the bond
length increased or decreased in the transition?
achieved in the branch.
The value of J at which the band head appears can be iden-
tified by finding the maximum wavenumber of the lines
in the R branch; that is, by identifying an integral value of
J close to where dR ( J )/dJ  0. This maximum occurs at
11F.2 Polyatomic molecules
The absorption of a photon can often be traced to the excita-
tion of electrons that belong to a small group of atoms in a
P R P R polyatomic molecule. For example, when a carbonyl group
(C=O) is present, an absorption at about 290 nm is normally
observed. Groups with characteristic optical absorption bands
are called chromophores (from the Greek for ‘colour bringer’),
and their presence often accounts for the colours of substances
(Table 11F.2).

Table 11F.2 Absorption characteristics of some groups and


molecules⋆

Frequency, ν Frequency, ν Group  /cm 1 λmax/nm εmax/(dm3 mol−1 cm−1)


~ ~ ~ ~
(a) B´ < B (b) B´ > B ⋆
C=C (π ← π) 61 000 163 15 000

Figure 11F.11 (a) The formation of a head in the R branch when C=O (π ← n) 35 000–37 000 270–290 10–20
B   B;
 (b) the formation of a head in the P branch when B   B.
 H2O 60 000 167 7000
The dotted curve shows the wavenumbers of the lines in the two ⋆
More values are given in the Resource section; εmax is the molar absorption coefficient (see
branches as they spread away from the centre of the band. Topic 11A). The wavenumbers and wavelengths are the values for maximum absorption.
478 11 Molecular spectroscopy

11F.2(a) d-Metal complexes s­ ymmetry labels are explained in Topic 10B). The t2g orbitals lie
below the eg orbitals in energy; the difference in energy ΔO is
In a free atom, all five d orbitals of a given shell are degenerate. called the ligand field splitting parameter (the o denotes oc-
In a d-metal complex, where the immediate environment of the tahedral symmetry). The ligand field splitting is typically about
atom is no longer spherical, the d orbitals are not all degenerate, 10 per cent of the overall energy of interaction between the li-
and the complex can absorb photons. The result is promotion gands and the central metal ion, which is largely responsible for
of electrons from orbitals of lower energy to orbitals of higher the existence of the complex. The d orbitals also divide into two
energy. sets in a tetrahedral complex, but in this case the two e orbitals
lie below the three t2 orbitals (no g or u label is given because a
tetrahedral complex has no centre of inversion); the separation
3+ of these groups of orbitals is written ΔT.
H2O
eg The values of ΔO and ΔT are such that transitions between the
two sets of orbitals typically occur in the visible region of the
Ti 3
/5 ΔΟ
spectrum. The transitions are responsible for many of the col-
d ΔΟ
ours that are so characteristic of d-metal complexes.
2
/5 ΔΟ
t2g
Brief illustration 11F.7
1 2 [Ti(OH2)6] 3+

The spectrum of [Ti(OH2)6]3+ near 24 000 cm−1 (500 nm) is


shown in Fig. 11F.13, and can be ascribed to the promotion of its
single d electron from a t2g orbital to an eg orbital. The wavenum-
To see the origin of this splitting in an octahedral complex ber of the absorption maximum suggests that ΔO ≈ 24 000 cm−1
such as [Ti(OH2)6]3+ (1), the six ligands can be regarded as for this complex, which corresponds to about 3.0 eV.
point negative charges that repel the d electrons of the central
ion (Fig. 11F.12). As a result, the orbitals fall into two groups,
with d x2 − y2 and d z 2 pointing directly towards the ligand posi-
tions, and dxy, dyz, and dzx pointing between them. An electron
Absorbance, A

occupying an orbital of the former group has a less favourable


potential energy than when it occupies any of the three orbit-
als of the other group, and so the d orbitals split into the two
sets shown in (2): a triply degenerate set comprising the dxy,
dyz, and dzx orbitals and labelled t2g, and a doubly degenerate
set comprising the d x2 − y2 and d z 2 orbitals and labelled eg (these

10 20 30

ν/(10 3
cm–1)
+

– Figure 11F.13 The electronic absorption spectrum of [Ti(OH2)6]3+


+ in aqueous solution.
+
– –

eg dz 2 dx 2–y 2
According to the Laporte rule (Section 11F.1b), d–d transi-
tions are parity-forbidden in octahedral complexes because
– + + they are g → g transitions (more specifically, eg ← t2g transi-

+

tions). However, d–d transitions become weakly allowed as
+

vibronic transitions as a result of coupling to asymmetrical vi-

+
– + brations such as that shown in Fig. 11F.5.
A d-metal complex may also absorb radiation as a result of
t2g d dyz dzx the transfer of an electron from the ligands into the d orbitals of
xy
the central atom, or vice versa. In such charge-transfer transi-
Figure 11F.12 The classification of d orbitals in an octahedral tions the electron moves through a considerable distance, which
environment. The open circles represent the positions of the six means that the transition dipole moment may be large and the
(point-charge) ligands. absorption correspondingly intense. In the p ­ ermanganate ion,
11F Electronic spectra 479

MnO−4 , the charge redistribution that accompanies the migra- + – + +


tion of an electron from the O atoms to the central Mn atom
results in a strong transition in the range 475–575 nm that ac-
counts for the intense purple colour of the ion.
– –
An electronic migration from the ligands to the metal – +
­corresponds to a ligand-to-metal charge-transfer transition π π
(LMCT). The reverse migration, a metal-to-ligand charge-
transfer transition (MLCT), can also occur. An example is the Figure 11F.14 A C=C double bond acts as a chromophore. In the
migration of a d electron onto the antibonding π orbitals of an π⋆ ← π transition illustrated here, an electron is promoted from a
π orbital to the corresponding antibonding orbital.
aromatic ligand. The resulting excited state may have a very
long lifetime if the electron is extensively delocalized over sev-
eral aromatic rings.
In common with other transitions, the intensities of charge-
+ – –
transfer transitions are proportional to the square of the +
transition dipole moment. The transition moment can be
­
thought of as a measure of the distance moved by the electron as +
it migrates between metal and ligand, with a large distance of mi- –
π n
gration corresponding to a large transition dipole moment and
therefore a high intensity of absorption. However, the integrand Figure 11F.15 A carbonyl group (C=O) acts as a chromophore
in the expression for the transition moment is proportional to partly on account of the excitation of a nonbonding O lone-pair
the product of the initial and final wavefunctions; this product electron to an antibonding CO π⋆ orbital; this transition is denoted
is zero unless the two wavefunctions have non-zero values in π⋆ ← n.
the same region of space. Therefore, although large distances of
migration favour high intensities, the diminished overlap of the π⋆ ← n transitions in carbonyls are symmetry forbidden, the
initial and final wavefunctions for large separations of metal and absorptions are weak. By contrast, the π⋆ ← π transition in a
ligands favours low intensities (see Problem P11F.9). carbonyl, which corresponds to excitation of a π electron of the
C=O double bond, is allowed by symmetry and results in rela-
11F.2(b) π⋆ ← π and π⋆ ← n transitions tively strong absorption.

Absorption by a C=C double bond results in the excitation of


a π electron into an antibonding π⋆ orbital (Fig. 11F.14). The Brief illustration 11F.8
chromophore activity is therefore due to a *   transition
The compound CH3CH=CHCHO has a strong absorption in
(a ‘π to π-star transition’). Its energy is about 6.9 eV for an un-
the ultraviolet at 46 950 cm−1 (213 nm) and a weak absorp-
conjugated double bond, which corresponds to an absorption tion at 30 000 cm−1 (330 nm). The former is a π⋆ ← π transi-
at 180 nm (in the ultraviolet). When the double bond is part tion associated with the delocalized π system C=C–C=O.
of a conjugated chain, the energies of the molecular orbitals Delocalization extends the range of the C=O π⋆ ← π transition
lie closer together and the *   transition moves to a longer to lower wavenumbers (longer wavelengths). The latter is an
wavelength; it may even lie in the visible region if the conju- π⋆ ← n transition associated with the carbonyl chromophore.
gated system is long enough.
One of the transitions responsible for absorption in carbonyl
compounds can be traced to the lone pairs of electrons on the
O atom. The Lewis concept of a ‘lone pair’ of electrons is repre- Exercises
sented in molecular orbital theory by a pair of electrons in an or- 3+
E11F.5 The complex ion [Fe(OH2)6] has an electronic absorption spectrum
bital confined largely to one atom and not appreciably involved with a maximum at 700 nm. Estimate a value of ΔO for the complex.
in bond formation. One of these electrons may be excited into
E11F.6 The two compounds 2,3-dimethyl-2-butene and 2,5-dimethyl-2,4-
an empty π⋆ orbital of the carbonyl group (Fig. 11F.15), which hexadiene are to be distinguished by their ultraviolet absorption spectra. The
gives rise to an π⋆ ← n transition (an ‘n to π-star transition’). maximum absorption in one compound occurs at 192 nm and in the other at
Typical absorption energies are about 4.3 eV (290 nm). Because 243 nm. Match the maxima to the compounds and justify the assignment.
480 11 Molecular spectroscopy

Checklist of concepts
☐ 1. The term symbols of linear molecules give the compo- ☐ 6. In gas phase samples, rotational fine structure can be
nents of various kinds of angular momentum around the resolved and under some circumstances band heads are
internuclear axis along with relevant symmetry labels. formed.
☐ 2. The Laporte selection rule states that, for centrosym- ☐ 7. In d-metal complexes, the presence of ligands removes
metric molecules, only u → g and g → u transitions are the degeneracy of d orbitals and vibrationally allowed
allowed. d–d transitions can occur between them.
☐ 3. The Franck–Condon principle asserts that electronic tran- ☐ 8. Charge-transfer transitions typically involve the migra-
sitions occur within an unchanging nuclear framework. tion of electrons between the ligands and the central
☐ 4. Vibrational fine structure is the structure in a spectrum metal atom.
that arises from changes in vibrational energy accompa- ☐ 9. A chromophore is a group with a characteristic optical
nying an electronic transition. absorption band.
☐ 5. Rotational fine structure is the structure in a spectrum
that arises from changes in rotational energy accompa-
nying an electronic transition.

Checklist of equations
Property Equation Comment Equation number
Selection rules (angular momentum) ∆Λ = 0, ±1; ∆S = 0; ∆Σ = 0; ∆Ω = 0, ±1 Linear molecules 11F.4

 
2 2
Franck–Condon factor S(vf ,vi )    v ,i d N

v ,f
11F.5

Rotational structure of electronic spectra (diatomic molecules) P ( J )    (B   B )J  (B   B )J 2


P branch (ΔJ = −1) 11F.7a

Q ( J )    (B   B )J ( J  1) Q branch (ΔJ = 0) 11F.7b

R ( J )    (B   B )( J  1)  (B   B )( J  1)


2
R branch (ΔJ = +1) 11F.7c
TOPIC 11G Decay of excited states

➤ Why do you need to know this material?


Phosphorescence

Emission intensity, I
Information about the electronic structure of a molecule
can be obtained by observing the radiative decay of excit-
ed electronic states back to the ground state. Such decay is
also used in lasers, which are of exceptional technological
importance.
Fluorescence

➤ What is the key idea?


Molecules in excited electronic states discard their excess
energy by emission of electromagnetic radiation, transfer Time, t
as heat to the surroundings, or fragmentation. Figure 11G.1 The empirical (observation-based) distinction
between fluorescence and phosphorescence is that the former is
➤ What do you need to know already? extinguished very quickly after the exciting radiation (indicated
You need to be familiar with electronic transitions in mole- by the tinted area on the left) is removed, whereas the latter
cules (Topic 11F), the difference between spontaneous and continues with relatively slowly diminishing intensity.
stimulated emission of radiation (Topic 11A), and the gen-
eral features of spectroscopy (Topic 11A). You need to be
s­ econds). The difference suggests that fluorescence is a fast con-
aware of the difference between singlet and triplet states
version of absorbed radiation into re-emitted energy, and that
(Topic 8C) and of the Franck–Condon principle (Topic 11F).
phosphorescence involves the storage of energy in a reservoir
from which it slowly leaks.
Figure 11G.2 shows the sequence of steps involved in fluo-
rescence from molecules in solution. The initial stimulated
Radiative decay is a process in which a molecule discards its absorption takes the molecule to an excited electronic state;
excitation energy as a photon (Topic 11A); depending on the if the absorption spectrum were monitored it would look like
nature of the excited state, this process is classified as either the one shown in blue in Fig. 11G.3. The excited molecule is
‘fluorescence’ or ‘phosphorescence’. A more common fate of subjected to collisions with the surrounding molecules. As it
an electronically excited molecule is non-radiative decay, in
which the excess energy is transferred into the vibration, rota-
tion, and translation of the surrounding molecules. This ther- Radiationless
Molecular potential energy, V

decay
mal degradation converts the excitation energy into thermal
motion of the environment (i.e. into ‘heat’). An excited mol-
ecule may also dissociate or take part in a chemical reaction
Emission
(Topic 17G). (fluorescence)
Absorption

Fluorescence and
11G.1
phosphorescence
Internuclear separation, R
In fluorescence, spontaneous emission of radiation occurs Figure 11G.2 The sequence of steps leading to fluorescence
while the sample is being irradiated and ceases as soon as the by molecules in solution. After the initial absorption, the upper
exciting radiation is extinguished (Fig. 11G.1). In phospho- vibrational states undergo radiationless decay by giving up energy
rescence, the spontaneous emission may persist for long peri- to the surrounding molecules. A radiative transition then occurs
ods (even hours, but more commonly seconds or fractions of from the vibrational ground state of the upper electronic state.
482 11 Molecular spectroscopy

Relaxation
Intensity, I

Absorption Fluorescence
(0,0)

Absorption Fluorescence

Figure 11G.4 Solvent–solute interactions can result in a shift in


Wavelength, λ the fluorescence spectrum relative to the absorption spectrum.
On the left absorption occurs with the solvent (depicted by
Figure 11G.3 An absorption spectrum (left) shows vibrational
the ellipsoids) in the arrangement characteristic of the ground
structure characteristic of the upper electronic state. A
electronic state of the molecule (the central blob). However,
fluorescence spectrum (right) shows structure characteristic of
before fluorescence occurs, the solvent molecules relax into a
the lower state; it is also displaced to lower frequencies (but the
new arrangement, and that arrangement is preserved during the
0–0 transitions are coincident) and is often a mirror image of the
subsequent radiative transition, the fluorescence.
absorption spectrum.

gives up energy to them non-radiatively it steps down (typi- v­ ibrational energy has been discarded into the surroundings.
cally within picoseconds) the ladder of vibrational levels to The vivid oranges and greens of fluorescent dyes are an eve-
the lowest vibrational level of the excited electronic state. The ryday manifestation of this effect: they absorb in the ultravio-
surrounding molecules, however, might now be unable to let and blue, and fluoresce in the visible. The mechanism also
accept the larger energy difference needed to lower the mol- suggests that the intensity of the fluorescence ought to depend
ecule to the ground electronic state. The excited electronic on the ability of the solvent molecules to accept the electronic
state might therefore survive long enough to undergo spon- and vibrational quanta. It is indeed found that a solvent com-
taneous emission and emit the remaining excess energy as posed of molecules with widely spaced vibrational levels (such
radiation. The downward electronic transition is vertical, in as water) can in some cases accept the large quantum of elec-
accord with the Franck–Condon principle (Topic 11F), and tronic energy and so extinguish, or ‘quench’, the fluorescence.
the fluorescence spectrum has vibrational structure charac- The rate at which fluorescence is quenched by other molecules
teristic of the lower electronic state (the spectrum shown on also gives valuable kinetic information (Topic 17G).
the right in Fig. 11G.3b). Figure 11G.5 shows the sequence of events leading to phos-
Provided they can be seen, the 0–0 absorption and fluores- phorescence for a molecule with a singlet ground state (de-
cence transitions (where the numbers are the values of vf and vi, noted S0). The first steps are the same as in fluorescence, but the
the vibrational quantum numbers for the final and initial states)
can be expected to be coincident. The absorption spectrum
arises from 0 ← 0, 1 ← 0, 2 ← 0,… transitions, which occur at
progressively higher wavenumbers (shorter wavelengths) and
Molecular potential energy, V

with intensities governed by the Franck–Condon principle. The S1


fluorescence spectrum arises from 0 → 0, 0 → 1,… downward
transitions, which occur with decreasing wavenumbers (longer T1

wavelengths).
The 0–0 absorption and fluorescence peaks are not always Absorption ISC S0
exactly coincident, however, because the solvent may inter-
Phosphorescence
act differently with the solute in the ground and excited elec-
tronic states (for instance, the hydrogen bonding pattern might
differ). Because the solvent molecules do not have time to
rearrange during the transition, the absorption occurs in an en- Internuclear separation, R
vironment characteristic of the solvated ground state; however, Figure 11G.5 The sequence of steps leading to phosphorescence.
the fluorescence occurs in an environment characteristic of the The important step is the intersystem crossing (ISC), the switch
solvated excited state (Fig. 11G.4). from a singlet state (S1) to a triplet state (T1) brought about by
Fluorescence occurs at lower frequencies than the incident spin–orbit coupling. The triplet state acts as a slowly radiating
radiation because the emissive transition occurs after some reservoir because the return to the ground state is spin-forbidden.
11G Decay of excited states 483

presence of a triplet excited state (T1) at an energy close to that S1 T1 S0


of the singlet excited state (S1) plays a decisive role. The singlet 35 IC

Wavenumber,  /(103 cm–1)


and triplet excited states share a common geometry at the point 30 ISC
where their potential energy curves intersect. Hence, if there is
25
a mechanism for unpairing two electron spins (and achieving
the conversion of ↑↓ to ↑↑), the molecule may undergo inter- 20
Ph 471
Fl
system crossing, a non-radiative transition between states of uo os n
15 31 re ph m
different multiplicity, and become a triplet state. As in the dis- 5 sc or
nm e es
10 nc ce
cussion of atomic spectra (Topic 8C), singlet–triplet transitions e nc
6 e
may occur in the presence of spin–orbit coupling. Intersystem
crossing is expected to be important when a molecule contains 0
a moderately heavy atom (such as sulfur), because then the
Figure 11G.6 A Jablonski diagram (here, for naphthalene) is a
spin–orbit coupling is large.
simplified portrayal of the relative positions of the electronic
Once an excited molecule has crossed into a triplet state, it energy levels of a molecule. Vibrational levels of states of a
continues to discard energy into the surroundings. However, given electronic state lie above each other, but the relative
it is now stepping down the triplet’s vibrational ladder and horizontal locations of the columns bear no relation to the
ends in its lowest vibrational level. The triplet state is lower nuclear separations in the states. The ground vibrational states
in energy than the corresponding singlet state (Hund’s rule, of each electronic state are correctly located vertically but the
Topic 8B). The solvent cannot absorb the final, large quan- other vibrational states are shown only schematically. (IC: internal
tum of electronic excitation energy, and the molecule cannot conversion; ISC: intersystem crossing.)
radiate its energy because return to the ground state is spin-
forbidden. The radiative transition, however, is not totally for-
bidden because the spin–orbit coupling that was responsible
Dissociation and
11G.2
for the intersystem crossing also weakens the selection rule. predissociation
The molecules are therefore able to emit weakly, and the emis-
sion may continue long after the original excited state was A chemically important fate for an electronically excited mol-
formed. ecule is dissociation, the breaking of bonds (Fig. 11G.7). The
This mechanism accounts for the observation that the ex- onset of dissociation can be detected in an absorption spectrum
citation energy seems to get trapped in a slowly leaking res- by noting that the vibrational fine structure of a band termi-
ervoir. It also suggests (as is confirmed experimentally) that nates at a certain frequency. Absorption occurs in a continuous
phosphorescence should be most intense from solid samples: band above this dissociation limit because the final state is an
energy transfer is then less efficient and intersystem crossing unquantized translational motion of the fragments. Locating
has time to occur as the singlet excited state steps slowly past the dissociation limit is a valuable way of determining the bond
the intersection point. The mechanism also suggests that the dissociation energy.
phosphorescence efficiency should depend on the presence of a In some cases, the vibrational structure disappears but re-
moderately heavy atom (with significant spin–orbit coupling), sumes at higher frequencies of the incident radiation. This
which is in fact the case.
The various types of non-radiative and radiative transitions
that can occur in molecules are often represented on a sche- Continuum
Molecular potential energy, V

matic Jablonski diagram of the type shown in Fig. 11G.6. Dissociation limit

Brief illustration 11G.1

Fluorescence efficiency decreases, and the phospho-


rescence efficiency increases, in the series of compounds:
naphthalene, 1-chloronaphthalene, 1-bromonaphthalene,
1-­iodonaphthalene. The replacement of an H atom by succes-
sively heavier atoms enhances intersystem crossing from S1
Internuclear separation, R
into T1, thereby decreasing the efficiency of fluorescence. The
rate of the radiative transition from T1 to S0 is also enhanced by Figure 11G.7 When absorption occurs to unbound states of
the presence of heavier atoms, thereby increasing the efficiency the upper electronic state, the molecule dissociates and the
of phosphorescence. absorption spectrum is a continuum. Below the dissociation limit
the electronic spectrum shows a normal vibrational structure.
484 11 Molecular spectroscopy

Continuum Exercise
Molecular potential energy, V

Dissociation limit
E11G.1 An oxygen molecule absorbs ultraviolet radiation in a transition from
Continuum its 3  g ground electronic state to an excited state that is energetically close to a
dissociative 5Πu state. The absorption band has a relatively large experimental
linewidth. Account for this observation.

11G.3 Lasers
Internuclear separation, R An excited state can be driven to discard its excess energy by
Figure 11G.8 When a dissociative state crosses a bound state, as using radiation to induce stimulated emission. The word laser
in the upper part of the illustration, molecules excited to levels is an acronym formed from light amplification by stimulated
near the crossing may dissociate. This predissociation is detected emission of radiation. In stimulated emission (Topic 11A), an
in the spectrum as a loss of vibrational structure that resumes at excited state is stimulated to emit a photon by radiation of the
higher frequencies. same frequency: the more photons that are present, the greater
is the probability of emission.
Laser radiation has a number of striking characteristics
effect provides evidence of predissociation, which can be in-
(Table 11G.1). Each of them (sometimes in combination with
terpreted in terms of the molecular potential energy curves
the others) opens up opportunities for applications in physical
shown in Fig. 11G.8. When a molecule is excited to a high
chemistry. Raman spectroscopy (Topics 11B–D) has flourished
vibrational level of the upper electronic state, its electrons
on account of the high intensity of monochromatic radiation
may undergo a redistribution that results in it undergoing an
available from lasers, and the ultrashort pulses that lasers can
internal ­conversion, a radiationless conversion to another
generate make possible the study of light-initiated reactions on
electronic state of the same multiplicity. An internal conver-
timescales of femtoseconds and even attoseconds.
sion occurs most readily at the point of intersection of the
One requirement of laser action is the existence of a meta-
two molecular potential energy curves, because there the nu-
stable excited state, an excited state with a long enough life-
clear geometries of the two electronic states are the same, as
time for it to undergo stimulated emission. For stimulated
are their energies. The state into which the molecule converts
emission to dominate absorption, it is necessary for there to
may be dissociative, so the states near the intersection have a
be a population inversion in which the population of the ex-
finite lifetime and hence their energies are imprecisely defined
cited state is greater than that of the lower state. Figure 11G.9
(as a result of lifetime broadening, Topic 11A). As a result, the
illustrates one way to achieve population inversion i­ ndirectly
absorption spectrum is blurred. When the incoming photon
brings enough energy to excite the molecule to a vibrational
Table 11G.1 Characteristics of laser radiation and their chemical
level high above the intersection, the internal conversion does
applications
not occur (the nuclei are unlikely to have the same geometry).
Consequently, the levels resume their well-defined, vibrational Characteristic Advantage Application
character with correspondingly well-defined energies, and High power Multiphoton process Spectroscopy
the line structure resumes on the high-frequency side of the Low detector noise Improved sensitivity
blurred region.
High scattering Raman spectroscopy
intensity (Topics 11B–11D)
Brief illustration 11G.2 Monochromatic High resolution Spectroscopy
State selection Photochemical studies
The O2 molecule absorbs ultraviolet radiation in a transition (Topic 17G)
from its 3  g ground electronic state to a 3  u excited state that
Reaction dynamics
is energetically close to a dissociative 3Πu state. In this case, the (Topic 18D)
effect of predissociation is more subtle than the abrupt loss of Collimated beam Long path lengths Improved sensitivity
vibrational–rotational structure in the spectrum; instead, the
Forward-scattering Raman spectroscopy
vibrational structure simply broadens rather than being lost observable (Topics 11B–11D)
completely. As before, the broadening is explained by the short Pulsed Precise timing of Fast reactions (Topics 17G,
lifetimes of the excited vibrational states near the intersection excitation 18C)
of the curves describing the bound and dissociative excited Relaxation (Topic 17C)
electronic states.
Energy transfer (Topic 17G)
11G Decay of excited states 485

Radiationless
I transition
4
F
B

1.06 μm
Energy
Energy Pump
Pump Laser
transition
A
4
I
Thermal
X decay
Relaxation
Figure 11G.10 The transitions involved in a neodymium laser.
Figure 11G.9 The transitions involved in a four-level laser.
Because the laser transition terminates in an excited state (A), the
population inversion between A and B is much easier to achieve electric discharge through xenon or with the radiation from
than when the lower state of the laser transition is the ground another laser.
state, X.
Brief illustration 11G.3

through an intermediate state I. In this process, the molecule The neodymium laser is an example of a four-level solid-state
or atom is excited to I, which then gives up some of its en- laser (Fig. 11G.10). In one form it consists of Nd3+ ions at low
ergy non-radiatively (for example, by passing energy on to concentration in yttrium aluminium garnet (YAG, specifically
vibrations of the surroundings) and decays to a lower state Y3Al5O12), and is then known as an Nd:YAG laser. A neodym-
B. Four levels are involved overall, so this arrangement is ium laser operates at a number of wavelengths in the infrared.
termed a four-level laser. One advantage of this arrangement The most common wavelength of operation is 1064 nm, which
is that the population inversion of the A and B levels is easier corresponds to the electronic transition from the 4F to the 4I
state of the Nd3+ ion.
to achieve than one involving the heavily populated ground
state. The transition from X to I is caused by irradiation with
intense light (either continuously or as a flash) in the process Many of the most important laser systems are solid-state de-
called pumping. In some cases pumping is achieved with an vices; they are discussed in Topic 15G.

Checklist of concepts
☐ 1. Fluorescence is radiative decay between states of the ☐ 7. Predissociation is the observation of the effects of dis-
same multiplicity; it ceases soon after the exciting radia- sociation before the dissociation limit is reached.
tion is removed. ☐ 8. Laser action is the stimulated emission of coherent
☐ 2. Phosphorescence is radiative decay between states of radiation between states between which there is a popu-
different multiplicity; it persists after the exciting radia- lation inversion.
tion is removed. ☐ 9. A metastable excited state is an excited state with a long
☐ 3. Intersystem crossing is the non-radiative conversion to enough lifetime for it to undergo stimulated emission.
an electronic state of different multiplicity. ☐ 10. A population inversion is a condition in which the
☐ 4. A Jablonski diagram is a schematic diagram showing population of an upper state is greater than that of a
the types of non-radiative and radiative transitions that relevant lower state.
can occur in molecules. ☐ 11. Pumping, the stimulation of an absorption with an
☐ 5. An additional fate of an electronically excited species is external source of intense radiation, is a process by
dissociation. which a population inversion is created.
☐ 6. Internal conversion is a non-radiative conversion to an
electronic state of the same multiplicity.
486 11 Molecular spectroscopy

FOCUS 11 Molecular spectroscopy

To test your understanding of this material, work through Note: The masses of nuclides are listed in the Resource
the Exercises, Additional exercises, Discussion questions, and s­ ection.
Problems found throughout this Focus.
Selected solutions can be found at the end of this Focus in
the e-book. Solutions to even-numbered questions are available
online only to lecturers.

TOPIC 11A General features of molecular spectroscopy


Discussion questions
D11A.1 What is the physical origin of a selection rule? D11A.3 Describe the basic experimental arrangements commonly used for
absorption, emission, and Raman spectroscopy.
D11A.2 Describe the physical origins of linewidths in absorption and emission
spectra. Do you expect the same contributions for species in condensed and
gas phases?

Additional exercises
E11A.7 Calculate the ratio A/B for transitions with the following
[dye]/(mol dm−3) 0.0010 0.0050 0.0100 0.0500
characteristics: (i) 500 MHz radiofrequency radiation, (ii) 3.0 cm microwave
radiation. T/(per cent) 72 20 4.0 1.00 × 10−5

E11A.8 The molar absorption coefficient of a substance dissolved in hexane is


E11A.14 A 2.0 mm cell was filled with a solution of benzene in a non-
known to be 227 dm3 mol−1 cm−1 at 290 nm. Calculate the percentage reduction
absorbing solvent. The concentration of the benzene was 0.010 mol dm−3 and
in intensity when ultraviolet radiation of that wavelength passes through
the wavelength of the radiation was 256 nm (where there is a maximum in
2.00 mm of a solution of concentration 2.52 mmol dm−3.
the absorption). Calculate the molar absorption coefficient of benzene at
E11A.9 When ultraviolet radiation of wavelength 400 nm passes through this wavelength given that the transmission was 48 per cent. What will the
2.50 mm of a solution of an absorbing substance at a concentration transmittance be through a 4.0 mm cell at the same wavelength?
0.717 mmol dm−3, the transmission is 61.5 per cent. Calculate the molar
E11A.15 A 5.00 mm cell was filled with a solution of a dye. The concentration
absorption coefficient of the solute at this wavelength. Express your answer in
of the dye was 18.5 mmol dm−3. Calculate the molar absorption coefficient of
square centimetres per mole (cm2 mol−1).
the dye at this wavelength given that the transmission was 29 per cent. What
E11A.10 The molar absorption coefficient of a solute at 540 nm is will the transmittance be through a 2.50 mm cell at the same wavelength?
386 dm3 mol−1 cm−1. When light of that wavelength passes through a 5.00 mm
E11A.16 A swimmer enters a gloomier world (in one sense) on diving to
cell containing a solution of the solute, 38.5 per cent of the light was absorbed.
greater depths. Given that the mean molar absorption coefficient of sea water
What is the molar concentration of the solute?
in the visible region is 6.2 × 10−5 dm3 mol−1 cm−1, calculate the depth at which
E11A.11 The molar absorption coefficient of a solute at 440 nm is a diver will experience (i) half the surface intensity of light, (ii) one tenth the
423 dm3 mol−1 cm−1. When light of that wavelength passes through a 6.50 mm surface intensity.
cell containing a solution of the solute, 48.3 per cent of the light was absorbed.
E11A.17 Given that the maximum molar absorption coefficient of a molecule
What is the molar concentration of the solute?
containing a carbonyl group is 30 dm3 mol−1 cm−1 near 280 nm, calculate
E11A.12 The following data were obtained for the absorption at 450 nm by a the thickness of a sample that will result in (i) half the initial intensity of
dye in carbon tetrachloride when using a 2.0 mm cell. Calculate the molar radiation, (ii) one tenth the initial intensity. Take the absorber concentration
absorption coefficient of the dye at the wavelength employed: to be 10 mmol dm−3.
E11A.18 The absorption associated with a particular transition begins at
[dye]/(mol dm−3) 0.0010 0.0050 0.0100 0.0500 220 nm, peaks sharply at 270 nm, and ends at 300 nm. The maximum value
T/(per cent) 81.4 35.6 12.7 3.0 × 10−3 of the molar absorption coefficient is 2.21 × 104 dm3 mol−1 cm−1. Estimate
the integrated absorption coefficient of the transition assuming a triangular
lineshape.
E11A.13 The following data were obtained for the absorption at 600 nm by
a dye dissolved in methylbenzene using a 2.50 mm cell. Calculate the molar E11A.19 The absorption associated with a certain transition begins at
absorption coefficient of the dye at the wavelength employed: 167 nm, peaks sharply at 200 nm, and ends at 250 nm. The maximum value
Exercises and problems 487

of the molar absorption coefficient is 3.35 × 104 dm3 mol−1 cm−1. Estimate E11A.20 What is the Doppler-broadened linewidth of the vibrational
the integrated absorption coefficient of the transition assuming an inverted transition at 2308 cm−1 in 1H127I at 400 K?
parabolic lineshape (see figure below).
E11A.21 What is the Doppler-shifted wavelength of a red (680 nm) traffic light
approached at 60 km h−1?
εmax ε() = εmax{1 – k( –  max)2} E11A.22 At what speed of approach would a red (680 nm) traffic light appear
Molar absorption coefficient, ε

green (530 nm)?


E11A.23 Estimate the lifetime of a state that gives rise to a line of width
(i) 200 MHz, (ii) 2.45 cm−1.
9
E11A.24 A molecule in a gas undergoes about 1.0 × 10 collisions in each
second. Suppose that (i) every collision is effective in deactivating the
molecule rotationally and (ii) that one collision in 10 is effective. Calculate the
width (in hertz) of rotational transitions in the molecule.

max Wavenumber, 

Problems
P11A.1 The flux of visible photons reaching Earth from the North Star is about
λ/nm 292.0 296.3 300.8 305.4 310.1 315.0 320.0
4 × 103 mm−2 s−1. Of these photons, 30 per cent are absorbed or scattered by
3 −1 −1
the atmosphere and 25 per cent of the surviving photons are scattered by the ε/(dm mol cm ) 1512 865 477 257 135.9 69.5 34.5
surface of the cornea of the eye. A further 9 per cent are absorbed inside the
cornea. The area of the pupil at night is about 40 mm2 and the response time Evaluate the integrated absorption coefficient of ozone over the wavelength range
of the eye is about 0.1 s. Of the photons passing through the pupil, about 290–320 nm. Hint:  ( ) can be fitted to an exponential function quite well.
43 per cent are absorbed in the ocular medium. How many photons from
P11A.6 In many cases it is possible to assume that an absorption band has a
the North Star are focused onto the retina in 0.1 s? For a continuation of this 2

story, see R.W. Rodieck, The first steps in seeing, Sinauer, Sunderland (1998). Gaussian lineshape (one proportional to e − x ) centred on the band maximum.
Assume such a lineshape, and show that A    ( )d   1.0645 max 1/ 2,
P11A.2 The Beer–Lambert law is derived on the basis that the concentration where  1/ 2 is the width at half-height. The absorption spectrum of azoethane
of absorbing species is uniform. Suppose, instead, that the concentration falls (CH3CH2N2) between 24 000 cm−1 and 34 000 cm−1 is shown in the figure
exponentially as [J]  [J]0 e  x /x0 . Develop an expression for the variation of I below. First, estimate A for the band by assuming that it is Gaussian. Then use
with sample length; suppose that L >> x0. mathematical software to fit a polynomial (or a Gaussian) to the absorption
P11A.3 It is common to make measurements of absorbance at two wavelengths
band, and integrate the result analytically.
and use them to find the individual concentrations of two components A and
B in a mixture. Show that the molar concentrations of A and B in a cell of
10
length L are
ε B2 A1 − ε B1 A2 ε A1 A2 − ε A2 A1
=[A] = [B] 8
(ε A1ε B2 − ε A2ε B1 )L (ε A1ε B2 − ε A2ε B1 )L
ε/(dm3 mol–1 cm–1)

where A1 and A2 are absorbances of the mixture at wavelengths λ1 and λ2, and 6
the molar extinction coefficients of A (and B) at these wavelengths are εA1 and
εA2 (and εB1 and εB2).
4
P11A.4 When pyridine is added to a solution of iodine in carbon tetrachloride
the 520 nm band of absorption shifts toward 450 nm. However, the absorbance
of the solution at 490 nm remains constant: this feature is called an isosbestic 2
point. Show that an isosbestic point should occur when two absorbing species
are in equilibrium. Hint: Assume that pyridine and iodine form a 1:1 complex, 0
and that the absorption is due to pyridine and the complex only. 22 26 30 34

Wavenumber, /(103 cm–1)
P11A.5 Ozone, uniquely among other abundant atmospheric constituents,
absorbs ultraviolet radiation in a part of the electromagnetic spectrum
energetic enough to disrupt DNA in biological organisms. This spectral ‡
P11A.7 Wachewsky et al. (J. Phys. Chem. 100, 11559 (1996)) examined the
range, which is denoted UV-B, spans from about 290 nm to 320 nm. The
molar extinction coefficient of ozone over this range is given in the table ultraviolet absorption spectrum of CH3I, a species of interest in connection
below (DeMore et al., Chemical kinetics and photochemical data for use in with stratospheric ozone chemistry. They found the integrated absorption
stratospheric modeling: Eνaluation Number 11, JPL Publication 94–26 (1994)). coefficient to be dependent on temperature and pressure to an extent
inconsistent with internal structural changes in isolated CH3I molecules.
They explained the changes as due to dimerization of a substantial fraction

These problems were supplied by Charles Trapp and Carmen Giunta. of the CH3I, a process which would naturally be pressure and temperature
488 11 Molecular spectroscopy

dependent. (a) Compute the integrated absorption coefficient of CH3I to 61.8 pm. What is the speed of recession and the surface temperature of
over a triangular lineshape in the range 31 250–34 483 cm−1 and a maximal the star?
molar absorption coefficient of 150 dm3 mol−1 cm−1 at the mid-point of the
P11A.10 The Gaussian shape of a Doppler-broadened spectral line reflects
range. (b) Suppose 1.0 per cent of the CH3I units in a sample at 2.4 Torr and
the Maxwell distribution of speeds (see Topic 1B) in the sample at the
373 K exists as dimers. Evaluate the absorbance expected at the mid-point
temperature of the experiment. In a spectrometer that makes use of phase-
of the absorption lineshape in a sample cell of length 12.0 cm. (c) Suppose
sensitiνe detection the output signal is proportional to the first derivative of the
18 per cent of the CH3I units in a sample at 100 Torr and 373 K exists as
signal intensity, dI/dν. Plot the resulting lineshape for various temperatures.
dimers. Calculate the absorbance expected at the wavenumber corresponding
How is the separation of the peaks related to the temperature?
to the mid-point of the lineshape for a sample cell of length 12.0 cm; compute
the molar absorption coefficient that would be inferred from this absorbance P11A.11 The collision frequency z of a molecule of mass m in a gas at a
if dimerization is not considered. pressure p is z = 4σ(kT/πm)1/2p/kT, where σ is the collision cross-section. Find
an expression for the collision-limited lifetime of an excited state assuming
P11A.8 When a star emitting electromagnetic radiation of frequency ν moves
that every collision is effective. Estimate the width of a rotational transition at
with a speed s relative to an observer, the observer detects radiation of 63.56 cm−1 in HCl (σ = 0.30 nm2) at 25 °C and 1.0 atm. To what value must the
frequency νreceding = νf or νapproaching = ν/f, where f = {(1 − s/c)/(1 + s/c)}1/2 and pressure of the gas be reduced in order to ensure that collision broadening is
c is the speed of light. (a) Three Fe I lines of the star HDE 271 182, which less important than Doppler broadening?
belongs to the Large Magellanic Cloud, occur at 438.882 nm, 441.000 nm, and
442.020 nm. The same lines occur at 438.392 nm, 440.510 nm, and 441.510 nm P11A.12 Refer to Fig. 11A.9, which depicts a Michelson interferometer. The
in the spectrum of an Earth-bound iron arc. Decide whether HDE 271 182 is mirror M1 moves in discrete distance increments, so the path difference p
receding from or approaching the Earth and estimate the star’s radial speed is also incremented in discrete steps. Explore the effect of increasing the
with respect to the Earth. (b) What additional information would you need to step size on the shape of the interferogram for a monochromatic beam of
calculate the radial velocity of HDE 271 182 with respect to the Sun? wavenumber ν and intensity I0. That is, draw plots of I(p)/I0 against ν p, each
with a different number of data points spanning the same total distance path
P11A.9 When a star emitting electromagnetic radiation of frequency ν moves
taken by the movable mirror M1.
with a speed s relative to an observer, the observer detects radiation of
frequency νreceding = νf or νapproaching = ν/f, where f = {(1 − s/c)/(1 + s/c)}1/2 and P11A.13 Use mathematical software to elaborate on the results of Example 11A.2
c is the speed of light. A spectral line of 48Ti8+ (of mass 47.95mu) in a distant by: (a) exploring the effect of varying the wavenumbers and intensities of the
star was found to be shifted from 654.2 nm to 706.5 nm and to be broadened three components of the radiation on the shape of the interferogram; and (b)
calculating the Fourier transforms of the functions you generated in part (a).

TOPIC 11B Rotational spectroscopy


Discussion questions
19
D11B.1 Account for the rotational degeneracy of the various types of rigid D11B.6 Does Be F2 exist in ortho and para forms? Hints: (a) Determine the
rotor. Would the loss of rigidity affect your conclusions? geometry of BeF2, then (b) decide whether fluorine nuclei are fermions or
bosons.
D11B.2 Does centrifugal distortion increase or decrease the separation
between adjacent rotational energy levels? D11B.7 Describe the role of nuclear statistics in the occupation of energy levels
in 1H12C≡12C1H, 1H13C≡13C1H, and 2H12C≡12C2H. For nuclear spin data, see
D11B.3 Distinguish between an oblate and a prolate symmetric rotor and give
Table 12A.2.
several examples of each.
D11B.8 Account for the existence of a rotational zero-point energy in
D11B.4 Describe the physical origins of the gross selection rule for microwave
molecular hydrogen.
spectroscopy.
D11B.5 Describe the physical origins of the gross selection rule for rotational
Raman spectroscopy.

Additional exercises
E11B.13 Calculate the moment of inertia around the threefold symmetry axis E11B.17 Calculate the CO and CS bond lengths in OCS from the rotational
and the corresponding rotational constant of a 31P1H3 molecule (HPH bond constants B(16O12C32S) = 6081.5 MHz, B(16O12C34S) = 5932.8 MHz.
angle 93.5°; PH bond length 142 pm). 79 81
E11B.18 Estimate the centrifugal distortion constant for Br Br, for which
E11B.14 Plot the expressions for the two moments of inertia of a pyramidal B  0.0809 cm 1 and   323.2 cm 1. By what factor would the constant change
symmetric top version of an AB4 molecule (Table 11B.1) with equal bond when the 79Br is replaced by 81Br?
lengths but with the angle θ increasing from 90° to the tetrahedral angle.
E11B.19 Which of the following molecules may show a pure rotational
E11B.15 Plot the expressions for the two moments of inertia of a pyramidal microwave absorption spectrum: (i) H2O, (ii) H2O2, (iii) NH3, (iv) N2O?
symmetric top version of an AB4 molecule (Table 11B.1) with θ equal to the
E11B.20 Calculate the frequency and wavenumber of the J = 2 ← 1 transition
 /RAB,
tetrahedral angle but with one A−B bond varying. Hint: Write   RAB
in the pure rotational spectrum of 12C16O. The equilibrium bond length is
and allow ρ to vary from 2 to 1.
112.81 pm. Would the frequency increase or decrease if centrifugal distortion
E11B.16 Classify the following rotors: (i) CH2=CH2, (ii) SO3, (iii) ClF3, (iv) N2O. is considered?
Exercises and problems 489

E11B.21 The wavenumber of the J = 3 ← 2 rotational transition of E11B.26 The wavenumber of the incident radiation in a Raman spectrometer is
1
H35Cl considered as a rigid rotor is 63.56 cm−1; what is the H−Cl bond 20 623 cm−1. What is the wavenumber of the scattered Stokes radiation for the
length? J = 3 ← 1 transition of 16O2? Take B  1.4457 cm 1.
1 81 35
E11B.22 The wavenumber of the J = 1 ← 0 rotational transition of H Br E11B.27 The rotational Raman spectrum of Cl2 shows a series of Stokes lines
considered as a rigid rotor is 16.93 cm−1; what is the H−Br bond length? separated by 0.9752 cm−1 and a similar series of anti-Stokes lines. Calculate the
35 19 bond length of the molecule.
E11B.23 The spacing of lines in the microwave spectrum of Cl F is
−1 19
1.033 cm ; calculate the moment of inertia and bond length of the E11B.28 The rotational Raman spectrum of F2 shows a series of Stokes lines
molecule. separated by 3.5312 cm−1 and a similar series of anti-Stokes lines. Calculate the
bond length of the molecule.
E11B.24 What is the most highly populated rotational level of Br2 at (i) 25 °C,
(ii) 100 °C? Take B  0.0809 cm 1 . E11B.29 What is the ratio of weights of populations due to the effects of
nuclear statistics for 12C32S2? What effect would be observed when 12C is
E11B.25 Which of the following molecules may show a pure rotational Raman
replaced by 13C? For nuclear spin data, see Table 12A.2.
spectrum: (i) CH2Cl2, (ii) CH3CH3, (iii) SF6, (iv) N2O?

Problems
P11B.1 Show that the moment of inertia of a diatomic molecule composed J 0 1 2 3 4
of atoms of masses mA and mB and bond length R is equal to meffR2, where  /cm 1 3.845 033 7.689 919 11.534 510 15.378 662 19.222 223
meff = mAmB/(mA + mB).
Evaluate B and D J for CO.
P11B.2 Confirm the expression given in Table 11B.1 for the moment of inertia

of a linear ABC molecule. Hint: Begin by locating the centre of mass. P11B.9 In a study of the rotational spectrum of the linear FeCO radical,
Tanaka et al. (J. Chem. Phys. 106, 6820 (1997)) report the following J + 1 ← J
P11B.3 The rotational constant of NH3 is 298 GHz. Calculate the separation of
transitions:
the pure rotational spectrum lines as a frequency (in GHz) and a wavenumber
(in cm−1), and show that the value of B is consistent with an N−H bond length J 24 25 26 27 28 29
of 101.4 pm and a bond angle of 106.78°.
1 35
ν/MHz 214 777.7 223 379.0 231 981.2 240 584.4 249 188.5 257 793.5
P11B.4 Rotational absorption lines from H Cl gas were found at the
following wavenumbers (R.L. Hausler and R.A. Oetjen, J. Chem. Phys.
Evaluate the rotational constant of the molecule. Also, estimate the value of J
21, 1340 (1953)): 83.32, 104.13, 124.73, 145.37, 165.89, 186.23, 206.60,
for the most highly populated rotational energy level at 298 K and at 100 K.
226.86 cm−1. Calculate the moment of inertia and the bond length of the
molecule. Predict the positions of the corresponding lines in 2H35Cl. P11B.10 The rotational terms of a symmetric top, allowing for centrifugal
1 2 distortion, are commonly written
P11B.5 Is the bond length in HCl the same as that in HCl? The wavenumbers
1 35
of the J = 1 ← 0 rotational transitions for H Cl and H Cl are 20.8784 and 2 35
 ( J  1)  ( A  B )K 2  D J 2 ( J  1)2  D J ( J  1)K 2  D K 4
F ( J ,K )  BJ J JK K
10.7840 cm−1, respectively. Accurate atomic masses are 1.007 825mu and
(a) Develop an expression for the wavenumbers of the allowed rotational
2.0140mu for 1H and 2H, respectively. The mass of 35Cl is 34.968 85mu. Based
transitions. (b) The following transition frequencies (in gigahertz, GHz) were
on this information alone, can you conclude that the bond lengths are the
observed for CH3F:
same or different in the two molecules?
51.0718 102.1403 102.1421 153.1987 153.2095
P11B.6 Thermodynamic considerations suggest that the copper monohalides
CuX should exist mainly as polymers in the gas phase, and indeed it Evaluate as many constants in the expression for the rotational terms as these
proved difficult to obtain the monomers in sufficient abundance to detect values permit.
spectroscopically. This problem was overcome by flowing the halogen gas over P11B.11 Develop an expression for the value of J corresponding to the most
copper heated to 1100 K (Manson et al., J. Chem. Phys. 63, 2724 (1975)). For
63
highly populated rotational energy level of a diatomic rotor at a temperature
Cu79Br the J = 14←13, 15←14, and 16←15 transitions occurred at 84 421.34, T remembering that the degeneracy of each level is 2J + 1. Evaluate the
90 449.25, and 96 476.72 MHz, respectively. Calculate the rotational constant expression for ICl (for which B  0.1142 cm 1 ) at 25 °C. Repeat the problem
and bond length of 63Cu79Br. The mass of 63Cu is 62.9296mu. for the most highly populated level of a spherical rotor, taking note of the fact
16 12
P11B.7 The microwave spectrum of O CS gave absorption lines (in GHz) as that each level is (2J + 1)2-fold degenerate. Evaluate the expression for CH4
follows: (for which B = 5.24 cm−1) at 25 °C. Hint: To develop the expression, recall that
the first derivative of a function is zero when the function reaches either a
J 1 2 3 4
maximum or minimum value.
32
S 24.325 92 36.488 82 48.651 64 60.814 08
P11B.12 A. Dalgarno, in Chemistry in the interstellar medium, Frontiers of
34
S 23.732 33 47.462 40 Astrophysics, ed. E.H. Avrett, Harvard University Press, Cambridge, MA
(1976), notes that although both CH and CN spectra show up strongly in the
Use the expressions for moments of inertia in Table 11B.1, assuming that the interstellar medium in the constellation Ophiuchus, the CN spectrum has
bond lengths are unchanged by substitution, to calculate the CO and CS bond become the standard for the determination of the temperature of the cosmic
lengths in OCS. microwave background radiation. Demonstrate through a calculation why
P11B.8 Equation 11B.20b may be rearranged into CH would not be as useful for this purpose as CN. The rotational constants B
for CH and CN are 14.190 cm−1 and 1.891 cm−1, respectively.
 ( J  1  J )/{2( J  1)}  B  2D J ( J  1)2
P11B.13 The space immediately surrounding stars, the circumstellar space,
which is the equation of a straight line when the left-hand side is plotted is significantly warmer because stars are very intense black-body emitters
against (J + 1)2. The following wavenumbers of transitions (in cm−1) were with temperatures of several thousand kelvin. Discuss how such factors
observed for 12C16O: as cloud temperature, particle density, and particle velocity may affect the
490 11 Molecular spectroscopy

rotational spectrum of CO in an interstellar cloud. What new features in the P11B.14 Pure rotational Raman spectra of gaseous C6H6 and C6D6
spectrum of CO can be observed in gas ejected from and still near a star with  C H )  0.189 60 cm 1,
yield the following rotational constants: B( 6 6
temperatures of about 1000 K, relative to gas in a cloud with temperature of  C H )  0.156 81 cm 1 . The moments of inertia of the molecules about any
B( 6 6
about 10 K? Explain how these features may be used to distinguish between axis perpendicular to the C6 axis were calculated from these data as I(C6H6) =
circumstellar and interstellar material on the basis of the rotational spectrum 1.4759 × 10−45 kg m2, I(C6D6) = 1.7845 × 10−45 kg m2. Calculate the CC and CH
of CO. bond lengths.

TOPIC 11C Vibrational spectroscopy of diatomic molecules


Discussion questions
D11C.1 Discuss the strengths and limitations of the parabolic and Morse D11C.3 How is the method of combination differences used in rotation–
functions as approximations to the true potential energy curve of a diatomic vibration spectroscopy to determine rotational constants?
molecule.
D11C.4 In what ways may the rotational and vibrational spectra of molecules
D11C.2 Describe the effect of vibrational excitation on the rotational constant change as a result of isotopic substitution?
of a diatomic molecule.

Additional exercises
E11C.6 An object of mass 100 g suspended from the end of a rubber band has 2309.5 cm−1 (H127I). Predict the fundamental vibrational wavenumbers of the
a vibrational frequency of 2.0 Hz. Calculate the force constant of the rubber corresponding deuterium halides.
band.
E11C.12 Calculate the relative numbers of Cl2 molecules (  559.7 cm ) in
1

E11C.7 An object of mass 1.0 g suspended from the end of a spring has a the ground and first excited vibrational states at (i) 298 K, (ii) 500 K.
vibrational frequency of 10.0 Hz. Calculate the force constant of the spring.
E11C.13 Calculate the relative numbers of Br2 molecules (  321 cm ) in the
1

E11C.8 Calculate the percentage difference in the fundamental vibrational second and first excited vibrational states at (i) 298 K, (ii) 800 K.
wavenumbers of 1H35Cl and 2H37Cl on the assumption that their force
E11C.14 For N2, ∆G values for the transitions v = 1 ← 0, 2 ← 0, and 3 ← 0
14
constants are the same.
are, respectively, 2329.91, 4631.20, and 6903.69 cm−1. Calculate ν and xe.
79 81
E11C.9 The wavenumber of the fundamental vibrational transition of Br Br Assume ye to be zero.
is 323.2 cm−1. Calculate the force constant of the bond.
E11C.15 The first five vibrational energy levels of HI are at 1144.83, 3374.90,
E11C.10 The hydrogen halides have the following fundamental vibrational 5525.51, 7596.66, and 9588.35 cm−1. Calculate the dissociation energy of the
wavenumbers: 4141.3 cm−1 (1H19F); 2988.9 cm−1 (1H35Cl); 2649.7 cm−1 (1H81Br); molecule in reciprocal centimetres and electronvolts.
2309.5 cm−1 (H127I). Calculate the force constants of the hydrogen–halogen 1 81
E11C.16 Infrared absorption by H Br gives rise to a P branch from v = 0.
bonds.
What is the wavenumber of the line originating from the rotational state with
E11C.11 The hydrogen halides have the following fundamental vibrational J = 2? Hint: Use data from Table 11C.1.
wavenumbers: 4141.3 cm−1 (1H19F); 2988.9 cm−1 (1H35Cl); 2649.7 cm−1 (1H81Br);

Problems
P11C.1 Use molecular modelling software and the computational method P11C.6 The Morse potential energy (eqn 11C.7) is very useful as a simple
of your choice to construct molecular potential energy curves like the one representation of the actual molecular potential energy. When 85Rb1H
shown in Fig. 11C.1. Consider the hydrogen halides (HF, HCl, HBr, and HI): was studied, it was found that   936.8 cm 1 and x e  14.15 cm 1. Plot the
(a) plot the calculated energy of each molecule against the bond length, and potential energy curve from 50 pm to 800 pm around Re = 236.7 pm. Then
(b) identify the order of force constants of the H−Hal bonds. go on to explore how the rotation of a molecule may weaken its bond
by allowing for the kinetic energy of rotation of a molecule and plotting
P11C.2 Derive an expression for the force constant of an oscillator that can be
 ( J  1) with B  /4 c  R 2. Plot these curves on the same
V   V  hcBJ
modelled by a Morse potential energy (eqn 11C.7).
diagram for J = 40, 80, and 100, and observe how the dissociation energy
P11C.3 Suppose a particle confined to a cavity in a microporous material has is affected by the rotation. Hints: Taking B  3.020 cm 1 at the equilibrium
2 2
a potential energy of the form V (x )  V0 (e  a /x  1) . Sketch V(x). What is the bond length will greatly simplify the calculation. The mass of 85Rb is
value of the force constant corresponding to this potential energy? Would the 84.9118mu.
particle undergo simple harmonic motion? Sketch the likely form of the first ‡
two vibrational wavefunctions. P11C.7 Luo et al. (J. Chem. Phys. 98, 3564 (1993)) reported the observation
23 127
of the He2 complex, a species which had escaped detection for a long time.
P11C.4 The vibrational levels of Na I lie at the wavenumbers 142.81, The fact that the observation required temperatures in the neighbourhood
427.31, 710.31, and 991.81 cm−1. Show that they fit the expression of 1 mK is consistent with computational studies which suggest that hcD e for
 v  12     v  12  xe , and deduce the force constant, zero-point energy,
2
He2 is about 1.51 × 10−23 J, hcD 0  2  1026 J, and Re about 297 pm. (a) Estimate
and dissociation energy of the molecule. the fundamental vibrational wavenumber, force constant, moment of inertia,
1 35
P11C.5 The H Cl molecule is quite well described by the Morse potential and rotational constant based on the harmonic oscillator and rigid-rotor
energy with hcD e = 5.33 eV,   2989.7 cm 1, and x e  52.05 cm 1. Assuming approximations. (b) Such a weakly bound complex is hardly likely to be rigid.
that the potential is unchanged on deuteration, predict the dissociation Estimate the vibrational wavenumber and anharmonicity constant based on
energies (hcD 0 , in electronvolts) of (a) 1H35Cl, (b) 2H35Cl. the Morse potential energy.
Exercises and problems 491

P11C.8 Confirm that a Morse oscillator has a finite number of bound states, oscillator becomes excited to higher quantum states? What would be the effect
the states with Ev < hcD e . Determine the value of vmax for the highest bound of anharmonicity?
state.
P11C.15 The rotational constant for a diatomic molecule in the vibrational
P11C.9 Provided higher order terms are neglected, eqn 11C.9b state with quantum number v typically fits the expression B v  B e    v  12 ,
for the vibrational wavenumbers of an anharmonic oscillator, where B e is the rotational constant corresponding to the equilibrium bond
G v1/ 2    2(v  1)x e  , is the equation of a straight line when the left- length. For the interhalogen molecule IF it is found that B e  0.27971 cm 1
hand side is plotted against v + 1. Use the following data on CO to determine and α = 0.187 m−1 (note the change of units). Calculate B 0 and B1 and use these
the values of ν and x eν for CO: values to calculate the wavenumbers of the transitions originating from J = 3
of the P and R branches. You will need the following additional information:
v 0 1 2 3 4   610.258 cm 1 and x e  3.141 cm 1. Estimate the dissociation energy of the
G v1/ 2 /cm 1
2143.1 2116.1 2088.9 2061.3 2033.5 IF molecule.
  4 B /  ) for the centrifugal distortion constant
P11C.16 Develop eqn 11B.16 (D
3 2
J
−1
P11C.10 The rotational constant for CO is 1.9225 cm and 1.9050 cm in the
−1
D J of a diatomic molecule of effective mass meff. Treat the bond as an elastic spring
ground and first excited vibrational states, respectively. By how much does the with force constant kf and equilibrium length Re that is subjected to a centrifugal
internuclear distance change as a result of this transition? distortion to a new length Rc. Begin the derivation by letting the particles
experience a restoring force of magnitude kf(Rc − Re) that is countered perfectly
P11C.11 The average spacing between the rotational lines of the P and R
by a centrifugal force meff ω2Rc, where ω is the angular velocity of the rotating
branches of 12C21H2 and 12C22H2 is 2.352 cm−1 and 1.696 cm−1, respectively.
molecule. Then introduce quantum mechanical effects by writing the angular
Estimate the CC and CH bond lengths.
momentum as {J(J + 1)}1/2ℏ. Finally, write an expression for the energy of the
P11C.12 Absorptions in the v = 1←0 vibration–rotation spectrum of H Cl
1 35
rotating molecule, compare it with eqn 11B.15, and infer an expression for D J .
−1
were observed at the following wavenumbers (in cm ):
P11C.17 At low resolution, the strongest absorption band in the infrared
2998.05 2981.05 2963.35 2944.99 2925.92 absorption spectrum of 12C16O is centred at 2150 cm−1. Upon closer examination
at higher resolution, this band is observed to be split into two sets of closely
2906.25 2865.14 2843.63 2821.59 2799.00
spaced peaks, one on each side of the centre of the spectrum at 2143.26 cm−1.
Assign the rotational quantum numbers and use the method of The separation between the peaks immediately to the right and left of the centre
combination differences to calculate the rotational constants of the two is 7.655 cm−1. Make the harmonic oscillator and rigid rotor approximations and
vibrational levels. calculate from these data: (a) the vibrational wavenumber of a CO molecule,
(b) its molar zero-point vibrational energy, (c) the force constant of the CO
P11C.13 Suppose that the internuclear distance may be written R = Re + x  and (e) the bond length of CO.
bond, (d) the rotational constant B,
where Re is the equilibrium bond length. Also suppose that the potential well
P11C.18 For C O, R (0)  2147.084 cm , R (1)  2150.858 cm ,
12 16 1 1
is symmetrical and confines the oscillator to small displacements. Deduce
expressions for 1/ R 2 , 1/ R 2 , and 1/R 2  to the lowest non-zero power of P (1)  2139.427 cm 1 and P (2)  2135.548 cm 1 . Estimate the values of
 x 2  /Re2 and confirm that the values are not the same. B 0 and B1.
P11C.14 Continue the development of Problem P11C.13 by using the virial P11C.19 The analysis of combination differences summarized in the text
theorem (Topic 7E) to relate  x 2  to the vibrational quantum number. Does considered the R and P branches. Extend the analysis to the O and S branches
your result imply that the rotational constant increases or decreases as the of a Raman specturm.

TOPIC 11D Vibrational spectroscopy of polyatomic molecules


Discussion questions
D11D.1 Describe the physical origin of the gross selection rule for infrared D11D.3 Can a linear, nonpolar molecule like CO2 have a Raman spectrum?
spectroscopy.
D11D.2 Describe the physical origin of the gross selection rule for vibrational
Raman spectroscopy.

Additional exercises
E11D.8 Which of the following molecules may show infrared absorption E11D.13 Write an expression for the vibrational term for the ground
spectra: (i) H2, (ii) HCl, (iii) CO2, (iv) H2O? vibrational state of SO2 in terms of the wavenumbers of the normal modes.
Neglect anharmonicities, as in eqn 11D.2.
E11D.9 Which of the following molecules may show infrared absorption
spectra: (i) CH3CH3, (ii) CH4, (iii) CH3Cl, (iv) N2? E11D.14 Is the out-of-plane mode of a planar AB3 molecule infrared or Raman active?

E11D.10 How many normal modes of vibration are there for the following E11D.15 Consider the vibrational mode that corresponds to the boat-like
molecules: (i) C6H6, (ii) C6H5CH3, (iii) HC≡C−C≡C−H? bending of a benzene ring. Is it (i) Raman, (ii) infrared active?
E11D.11 How many vibrational modes are there for the molecule E11D.16 Does the exclusion rule apply to C2H4?
NC−(C≡C−C≡C−)10CN detected in an interstellar cloud?
E11D.12 How many vibrational modes are there for the molecule
NC−(C≡C−C≡C−)8CN detected in an interstellar cloud?
492 11 Molecular spectroscopy

Problems
P11D.1 Suppose that the out-of-plane distortion of an AB3 planar molecule P11D.3 The computational methods discussed in Topic 9E can be used to
4
is described by a potential energy V  V0 (1  e bh ), where h is the distance simulate the vibrational spectrum of a molecule, and it is then possible to
by which the central atom A is displaced. Sketch this potential energy as determine the correspondence between a vibrational frequency and the
a function of h (allow h to be both negative and positive). What could be atomic displacements that give rise to a normal mode. (a) Using molecular
said about (a) the force constant, (b) the vibrations? Sketch the form of the modelling software and the computational method of your choice, calculate
ground-state wavefunction. the fundamental vibrational wavenumbers and depict the vibrational normal
+ modes of SO2 in the gas phase graphically. (b) The experimental values of the
P11D.2 Predict the shape of the nitronium ion, NO2 , from its Lewis structure
fundamental vibrational wavenumbers of SO2 in the gas phase are 525 cm−1,
and the VSEPR model. It has one Raman active vibrational mode at 1400 cm−1,
1151 cm−1, and 1336 cm−1. Compare the calculated and experimental values.
two strong IR active modes at 2360 and 540 cm−1, and one weak absorption
Even if agreement is poor, is it possible to establish a correlation between an
in the IR at 3735 cm−1. Are these data consistent with the predicted shape of
experimental value of the vibrational wavenumber with a specific vibrational
the molecule? Assign the vibrational wavenumbers to the modes from which
normal mode?
they arise.

TOPIC 11E Symmetry analysis of vibrational spectra


Discussion question
D11E.1 Suppose that you wish to characterize the normal modes of benzene
in the gas phase. Why is it important to obtain both infrared absorption and
Raman spectra of the molecule?

Additional exercises
E11E.4 A carbon disulfide molecule belongs to the point group D∞h. The nine E11E.6 Which of the normal modes of (i) H2O, (ii) H2CO are Raman
displacements of the three atoms span A1g + 2A1u + 2E1u + E1g. What are the active?
symmetry species of the normal modes of vibration?
E11E.5 The molecule BeH2 is linear and centrosymmetric; its normal modes
have symmetry species A1g + A1u + E1u. Which of these normal modes are
infrared active? Which are Raman active?

Problems
P11E.1 Consider the molecule CH3Cl. (a) To what point group does the have? (ii) Give the symmetry point group of each of the three proposed
molecule belong? (b) How many normal modes of vibration does the conformations of nonlinear H2O2. (iii) Determine which of the proposed
molecule have? (c) What are the symmetry species of the normal modes conformations is inconsistent with the spectroscopic data. Explain your
of vibration of this molecule? (d) Which of the vibrational modes of this reasoning.
molecule are infrared active? (e) Which of the vibrational modes of this
molecule are Raman active?
P11E.2 Suppose that three conformations are proposed for the nonlinear
molecule H2O2 (1, 2, and 3). The infrared absorption spectrum of gaseous
H2O2 has bands at 870, 1370, 2869, and 3417 cm−1. The Raman spectrum
of the same sample has bands at 877, 1408, 1435, and 3407 cm−1. All bands
correspond to fundamental vibrational wavenumbers and you may assume
that: (a) the 870 and 877 cm−1 bands arise from the same normal mode,
and (b) the 3417 and 3407 cm−1 bands arise from the same normal mode. 1 2 3
(i) If H2O2 were linear, how many normal modes of vibration would it

TOPIC 11F Electronic spectra


Discussion questions
3 
D11F.1 Explain the origin of the term symbol  g for the ground state of a D11F.3 How do the band heads in P and R branches arise? Could the Q branch
dioxygen molecule. show a head?
D11F.2 Explain the basis of the Franck–Condon principle and how it leads to D11F.4 Explain how colour can arise from molecules.
the formation of a vibrational progression.
Exercises and problems 493

2+
D11F.5 Suppose that you are a colour chemist and had been asked to intensify D11F.6 Can a complex of the Zn ion have a d–d electronic transition?
the colour of a dye without changing the type of compound, and that the dye Explain your answer.
in question was a conjugated polyene. (a) Would you choose to lengthen or to
shorten the chain? (b) Would the modification to the length shift the apparent
colour of the dye towards the red or the blue?

Additional exercises
2 2 
E11F.7 What is the value of S and the term symbol for the ground state of H2? E11F.20 The P-branch of the    transition of CdH shows a band head
3 at J = 25. The rotational constant of the ground state is 5.437 cm−1. What is
E11F.8 The term symbol for one of the lowest excited states of H2 is Πu. To
the rotational constant of the upper state? Has the bond length increased or
which excited-state configuration does this term symbol correspond?
decreased in the transition?

E11F.9 What are the levels of the term for the ground electronic state of O2 ? 3−
E11F.21 The complex ion [Fe(CN)6] has an electronic absorption spectrum
E11F.10 Another of the excited states of the C2 molecule has the valence electron with a maximum at 305 nm. Estimate a value of Δo for the complex.
configuration 1 g2 1 u2 12u 12g . Give the multiplicity and parity of the term.
E11F.22 Suppose that a charge-transfer transition in a one-dimensional system
E11F.11 Which of the following transitions are electric-dipole allowed? can be modelled as a process in which a rectangular wavefunction that is
(i) 1  g  1 u, (ii) 3  g  3 u, (ii) π⋆ ↔ n. non-zero in the range 0 ≤ x ≤ a makes a transition to another rectangular
wavefunction that is non-zero in the range 12 a ≤ x ≤ b. Evaluate the transition
E11F.12 The ground-state wavefunction of a certain molecule is described by
2 moment  f x i dx. (Assume a < b.) Hint: Don’t forget to normalize each
the vibrational wavefunction  0  N 0 e  ax / 2 . Calculate the Franck–Condon
wavefunction to 1.
factor for a transition to a vibrational state described by the wavefunction
2
 0  N 0 e  a( x x0 ) / 2 . The normalization constants are given by eqn 7E.10. E11F.23 Suppose that a charge-transfer transition in a one-dimensional system
can be modelled as a process in which an electron described by a rectangular
E11F.13 The ground-state wavefunction of a certain molecule is described by
 ax 2 / 2 wavefunction that is non-zero in the range 0 ≤ x ≤ a makes a transition to
the vibrational wavefunction  0  N 0 e . Calculate the Franck–Condon
another rectangular wavefunction that is non-zero in the range ca ≤ x ≤ a
factor for a transition to a vibrational state described by the wavefunction
2 where 0 ≤ c ≤ 1. Evaluate the transition moment  f x i dx and explore its
 1  N1 (x  x 0 )e  a( x x0 ) / 2. The normalization constants are given by eqn 7E.10.
dependence on c. Hint: Don’t forget to normalize each wavefunction to 1.
E11F.14 Suppose that the ground vibrational state of a molecule is modelled by
E11F.24 Suppose that a charge-transfer transition in a one-dimensional system
using the particle-in-a-box wavefunction ψ0 = (2/L)1/2 sin(πx/L) for 0 ≤
can be modelled as a process in which a Gaussian wavefunction centred on
x ≤ L and 0 elsewhere. Calculate the Franck–Condon factor for a transition to
x = 0 and width a makes a transition to another Gaussian wavefunction of
a vibrational state described by the wavefunction ψ′ = (2/L)1/2sin{π(x−L/4)/L}
the same width centred on x = 12 a . Evaluate the transition moment  f x i dx.
for L/4 ≤ x ≤ 5L/4 and 0 elsewhere.
Hint: Don’t forget to normalize each wavefunction to 1.
E11F.15 Suppose that the ground vibrational state of a molecule is modelled by
E11F.25 Suppose that a charge-transfer transition can be modelled in a
using the particle-in-a-box wavefunction ψ0 = (2/L)1/2 sin(πx/L) for 0 ≤ x ≤ L
one-dimensional system as a process in which an electron described by a
and 0 elsewhere. Calculate the Franck–Condon factor for a transition to a
Gaussian wavefunction centred on x = 0 and width a makes a transition to
vibrational state described by the wavefunction ψ′ = (2/L)1/2 sin{π(x − L/2)/L}
another Gaussian wavefunction of width a/2 and centred on x = 0. Evaluate
for L/2 ≤ x ≤ 3L/2 and 0 elsewhere.
the transition moment  f x i dx. Hint: Don’t forget to normalize each
E11F.16 Use eqn 11F.7a to infer the value of J corresponding to the location of wavefunction to 1.
the band head of the P branch of a transition.
E11F.26 3-Buten-2-one (4) has a strong absorption at 213 nm and a weaker
E11F.17 Use eqn 11F.7c to infer the value of J corresponding to the location of absorption at 320 nm. Assign the ultraviolet absorption transitions, giving
the band head of the R branch of a transition. your reasons.
E11F.18 The following parameters describe the electronic ground state and an
excited electronic state of SnO: B  0.3540 cm 1 , B   0.3101 cm 1. Which branch
O
of the transition between them shows a head? At what value of J will it occur?
E11F.19 The following parameters describe the electronic ground state and
an excited electronic state of BeH: B  10.308 cm 1, B   10.470 cm 1. Which 4 3-Buten-2-one
branch of the transition between them shows a head? At what value of J will
it occur?

Problems
P11F.1 Which of the following electronic transitions are allowed in of these two electronic states is 6.175 eV, what is the wavenumber of the
O2 : 3 g  1 g, and 3  g  3  u? lowest energy transition in the band of transitions originating from the v = 0
‡ vibrational state of the electronic ground state to this excited state? Ignore any
P11F.2 J.G. Dojahn et al. (J. Phys. Chem. 100, 9649 (1996)) characterized the
rotational structure or anharmonicity.
potential energy curves of the ground and electronic states of homonuclear
diatomic halogen anions. These anions have a 2  u ground state and 2Πg, P11F.4 A transition of particular importance in O2 gives rise to the
2
Πu, and 2  g excited states. To which of the excited states are electric-dipole Schumann–Runge band in the ultraviolet region. The wavenumbers (in
transitions allowed from the ground state? Explain your conclusion. cm−1) of transitions from the ground state to the vibrational levels of the first
excited state (3  u ) are 50 062.6, 50 725.4, 51 369.0, 51 988.6, 52 579.0, 53 143.4,
P11F.3 The vibrational wavenumber of the oxygen molecule in its electronic
53 679.6, 54 177.0, 54 641.8, 55 078.2, 55 460.0, 55 803.1, 56 107.3, 56 360.3,
ground state is 1580 cm−1, whereas that in the excited state ( u ), to which
56 570.6. What is the dissociation energy of the upper electronic state? (Use a
there is an allowed electronic transition, is 700 cm−1. Given that the separation
Birge–Sponer plot, Topic 11C.) The same excited state is known to dissociate
in energy between the minima in their respective potential energy curves
494 11 Molecular spectroscopy

into one ground-state O atom and one excited-state atom with an energy box, and that the magnitude of the dipole moment can be related to
190 kJ mol−1 above the ground state. (This excited atom is responsible for a the displacement along this length by μ = −ex. Show that the transition
great deal of photochemical mischief in the atmosphere.) Ground-state O2 probability for the transition n = 1 → n = 2 is non-zero, whereas that
dissociates into two ground-state atoms. Use this information to calculate the for n = 1 → n = 3 is zero. Hints: The following relation will be useful:
dissociation energy of ground-state O2 from the Schumann–Runge data. sin x sin y  12 cos(x  y )  12 cos( x  y ). Relevant integrals are given in the
Resource section.
P11F.5 You are now ready to understand more deeply the features of
photoelectron spectra (Topic 9B). The figure below shows the photoelectron P11F.8 1,3,5-Hexatriene (a kind of ‘linear’ benzene) was converted into
spectrum of HBr. Disregarding for now the fine structure, the HBr lines fall benzene itself. On the basis of a free-electron molecular orbital model (in
into two main groups. The least tightly bound electrons (with the lowest which hexatriene is treated as a linear box and benzene as a ring), would you
ionization energies and hence highest kinetic energies when ejected) are those expect the lowest energy absorption to rise or fall in energy as a result of the
in the lone pairs of the Br atom. The next ionization energy lies at 15.2 eV, conversion?
and corresponds to the removal of an electron from the HBr σ bond. (a)
P11F.9 Estimate the magnitude of the transition dipole moment of a charge-
The spectrum shows that ejection of a σ electron is accompanied by a lot of
transfer transition modelled as the migration of an electron from a H1s
vibrational excitation. Use the Franck–Condon principle to account for this
orbital on one atom to another H1s orbital on an atom a distance R away.
observation. (b) Go on to explain why the lack of much vibrational structure
Approximate the transition moment by −eRS where S is the overlap integral
in the other band is consistent with the nonbonding role of the Br4px and
of the two orbitals. Sketch the transition moment as a function of R using the
Br4py lone-pair electrons.
expression for S given in Table 9C.1. Why does the intensity of a charge-
transfer transition fall to zero as R approaches 0 and infinity?
P11F.10 The figure below shows the UV-visible absorption spectra of a
selection of amino acids. Suggest reasons for their different appearances in
terms of the structures of the molecules.
Signal

Trp

Magnification
Absorbance Cystine
Tyr

17 15 13 11 9 Gly
Ionization energy, I/eV

P11F.6 The highest kinetic energy electrons in the photoelectron spectrum of 220 280
Wavelength, λ/nm
H2O using 21.22 eV radiation are at about 9 eV and show a large vibrational
spacing of 0.41 eV. The symmetric stretching mode of the neutral H2O
molecule lies at 3652 cm−1. (a) What conclusions can be drawn from the P11F.11 Propanone (acetone, (CH3)2CO) has a strong absorption at 189 nm
nature of the orbital from which the electron is ejected? (b) In the same and a weaker absorption at 280 nm. Identify the chromophore and assign the
spectrum of H2O, the band near 7.0 eV shows a long vibrational series absorptions to π⋆ ← n or π⋆ ← π transitions.
with spacing 0.125 eV. The bending mode of H2O lies at 1596 cm−1. What
P11F.12 Spin angular momentum is conserved when a molecule dissociates
conclusions can you draw about the characteristics of the orbital occupied by
into atoms. What atom multiplicities are permitted when the ground state of
the photoelectron?
an O2 molecule, (b) an N2 molecule dissociates into atoms?
P11F.7 Assume that the states of the π electrons of a conjugated molecule can
be approximated by the wavefunctions of a particle in a one-dimensional

TOPIC 11G Decay of excited states


Discussion questions
D11G.1 Describe the mechanism of fluorescence. In what respects is a D11G.4 What can be estimated from the wavenumber of the onset of
fluorescence spectrum not the exact mirror image of the corresponding predissociation?
absorption spectrum?
D11G.5 Describe the principles of a four-level laser.
D11G.2 What is the evidence for the usual explanation of the mechanism of
(a) fluorescence, (b) phosphorescence?
D11G.3 Consider an aqueous solution of a chromophore that fluoresces
strongly. Is the addition of iodide ion to the solution likely to increase or
decrease the efficiency of phosphorescence of the chromophore?
Exercises and problems 495

Additional exercises
E11G.2 The line marked A in the figure below is the fluorescence spectrum of E11G.3 A hydrogen molecule absorbs ultraviolet radiation in a transition from
benzophenone in solid solution in ethanol at low temperatures observed when its 1  g ground electronic state to an excited state that is energetically close to a
the sample is illuminated with 360 nm ultraviolet radiation. (a) What can be dissociative 1  u state. The absorption band has a relatively large experimental
said about the vibrational energy levels of the carbonyl group in (i) its ground linewidth. Account for this observation.
electronic state and (ii) its excited electronic state? (b) When naphthalene is
illuminated with 360 nm ultraviolet radiation it does not absorb, but the line
marked B in the figure below is the phosphorescence spectrum of a frozen
solution of a mixture of naphthalene and benzophenone in ethanol. Now a
component of fluorescence from naphthalene can be detected. Account for
this observation.
Emission intensity

15 20 25
/(1000 cm–1)

Problems
P11G.1 The fluorescence spectrum of anthracene vapour shows a series of of half-wavelengths fit into the cavity; all other waves undergo destructive
peaks of increasing intensity with individual maxima at 440 nm, 410 nm, interference with themselves. These wavelengths characterize the resonant
390 nm, and 370 nm followed by a sharp cut-off at shorter wavelengths. The modes of the laser. For a laser cavity of length 1.00 m, calculate (a) the allowed
absorption spectrum rises sharply from zero to a maximum at 360 nm with a frequencies and (b) the frequency difference between successive resonant
trail of peaks of lessening intensity at 345 nm, 330 nm, and 305 nm. Account modes.
for these observations.
P11G.4 Laser radiation is spatially coherent in the sense that the
P11G.2 The Beer–Lambert law states that the absorbance of a sample at a electromagnetic waves are all in step across the cross-section of the beam
wavenumber ν is proportional to the molar concentration [J] of the absorbing emerging from the laser cavity (see Problem P11G.3). The coherence length,
species J and to the length L of the sample (eqn 11A.8). In this problem you lC, is the distance across the beam over which the waves remain coherent, and
are asked to show that the intensity of fluorescence emission from a sample of is related to the range of wavelengths, Δλ, present in the beam by lC = λ2/2Δλ.
J is also proportional to [J] and L. Consider a sample of J that is illuminated When many wavelengths are present, and Δλ is large, the waves get out of
with a beam of intensity I 0 (ν ) at the wavenumber ν . Before fluorescence step in a short distance and the coherence length is small. (a) How does the
can occur, a fraction of I 0 (ν ) must be absorbed and an intensity I(ν ) will coherence length of a typical light bulb (lC = 400 nm) compare with that of a
be transmitted. However, not all the absorbed intensity is re-emitted and He−Ne laser with λ = 633 nm and Δλ = 2.0 pm? (b) What is the condition that
the intensity of fluorescence depends on the fluorescence quantum yield, φ F, would lead to an infinite coherence length?
the efficiency of photon emission. The fluorescence quantum yield ranges
P11G.5 A continuous-waνe laser emits a continuous beam of radiation,
from 0 to 1 and is proportional to the ratio of the integral of the fluorescence
whereas a pulsed laser emits pulses of radiation. The peak power, Ppeak, of a
spectrum over the integrated absorption coefficient. Because of a shift of
pulse is defined as the energy delivered in a pulse divided by its duration. The
magnitude  , fluorescence occurs at a wavenumber νf , with f     . It
average power, Paverage, is the total energy delivered by a large number of pulses
follows that the fluorescence intensity at νf , I f (ν f ), is proportional to φf and to
divided by the duration of the time interval over which that total energy is
the intensity of exciting radiation that is absorbed by J, I abs ( )  I 0 ( )  I ( ).
measured. Suppose that a certain laser can generate radiation in 3.0 ns pulses,
(a) Use the Beer–Lambert law to express I abs (ν ) in terms of I 0 (ν ) , [J], L, and
each of which delivers an energy of 0.10 J, at a pulse repetition frequency of
 ( ) , the molar absorption coefficient of J at ν . (b) Use your result from part
10 Hz. Calculate the peak power and the average power of this laser.
(a) to show that I f ( f )  I 0 ( ) ( )f [J]L.
P11G.6 Light-induced degradation of molecules, also called photobleaching,
P11G.3 A laser medium is confined to a cavity that ensures that only
is a serious problem in applications that require very high intensities. A
certain photons of a particular frequency, direction of travel, and state of
molecule of a fluorescent dye commonly used to label biopolymers can
polarization are generated abundantly. The cavity is essentially a region
withstand about 106 excitations by photons before light-induced reactions
between two mirrors, which reflect the light back and forth. This arrangement
destroy its π system and the molecule no longer fluoresces. For how long will
can be regarded as a version of the particle in a box, with the particle now
a single dye molecule fluoresce while being excited by 1.0 mW of 488 nm
being a photon. As in the treatment of a particle in a box (Topic 7D), the
radiation from a continuous-wave laser? You may assume that the dye has an
only wavelengths that can be sustained satisfy n  12   L, where n is an
absorption spectrum that peaks at 488 nm and that every photon delivered by
integer and L is the length of the cavity. That is, only an integral number
the laser is absorbed by the molecule.
496 11 Molecular spectroscopy

FOCUS 11 Molecular spectroscopy


Integrated activities
I11.1 In the group theoretical language developed in Focus 10, a spherical
rotor is a molecule that belongs to a cubic or icosahedral point group, a
symmetric rotor is a molecule with at least a threefold axis of symmetry,
and an asymmetric rotor is a molecule without a threefold (or higher) axis.
Linear molecules are linear rotors. Classify each of the following molecules as
a spherical, symmetric, linear, or asymmetric rotor and justify your answers
5
with group theoretical arguments: (a) CH4, (b) CH3CN, (c) CO2, (d) CH3OH, 6
(e) benzene, (f) pyridine.
‡ +
I11.2 The H3 ion has been found in the interstellar medium and in
the atmospheres of Jupiter, Saturn, and Uranus. The rotational energy
levels of H3+ , an oblate symmetric rotor, are given by eqn 11B.13a, with
C replacing A,  when centrifugal distortion and other complications are
ignored. Experimental values for vibrational–rotational constants are
 (E)  2521.6 cm 1 , B  43.55 cm 1 , and C  20.71 cm 1. (a) Show that for
a planar molecule (such as H3+ ) I|| = 2I⊥. The rather large discrepancy
with the experimental values is due to factors ignored in eqn 11B.13. 7 8
(b) Calculate an approximate value of the H−H bond length in H3+ .
(c) The value of Re obtained from the best quantum mechanical
calculations by J.B. Anderson (J. Chem. Phys. 96, 3702 (1991)) is
I11.5 The moments of inertia of the linear mercury(II) halides are very
87.32 pm. Use this result to calculate the values of the rotational constants
 (d) Assuming that the geometry and force constants are the large, so the O and S branches of their vibrational Raman spectra show
B and C.
little rotational structure. Nevertheless, the position of greatest intensity in
same in D3+ and H3+ , calculate the spectroscopic constants of D3+ . The
each branch can be identified and these data have been used to measure
molecular ion D3+ was first produced by Shy et al. (Phys. Reν. Lett 45, 535
the rotational constants of the molecules (R.J.H. Clark and D.M. Rippon,
(1980)) who observed the ν2(E′) band in the infrared.
J. Chem. Soc. Faraday Soc. II 69, 1496 (1973)). Show, from a knowledge of
I11.3 Use appropriate electronic structure software to perform calculations the value of J corresponding to the intensity maximum, that the separation
on H2O and CO2 with basis sets of your or your instructor’s choosing. of the peaks of the O and S branches is given by the Placzek–Teller
(a) Compute ground-state energies, equilibrium geometries and  /hc)1/ 2 . The following widths were obtained at the
relation   (32 BkT
vibrational frequencies for each molecule. (b) Compute the magnitude temperatures stated:
of the dipole moment of H2O; the experimental value is 1.854 D.
(c) Compare computed values to experiment and suggest reasons for any Hg35Cl2 Hg79Br2 Hg127I2
discrepancies.
θ/°C 282 292 292
I11.4 The protein haemerythrin is responsible for binding and carrying
2+
O2 in some invertebrates. Each protein molecule has two Fe ions that δ/cm−1 23.8 15.2 11.4
are in very close proximity and work together to bind one molecule of
O2. The Fe2O2 group of oxygenated haemerythrin is coloured and has an Calculate the bond lengths in the three molecules.
electronic absorption band at 500 nm. The Raman spectrum of oxygenated ‡
I11.6 A mixture of carbon dioxide (2.1 per cent) and helium, at 1.00 bar and
haemerythrin obtained with laser excitation at 500 nm has a band at 844 cm−1
298 K in a gas cell of length 10 cm has an infrared absorption band centred at
that has been attributed to the O−O stretching mode of bound 16O2. (a) Proof
2349 cm−1 with absorbances, A(ν ), described by:
that the 844 cm−1 band arises from a bound O2 species may be obtained by
conducting experiments on samples of haemerythrin that have been mixed a1 a4
A( )  
with 18O2, instead of 16O2. Predict the fundamental vibrational wavenumber 1  a2 (  a3 )2 1  a5 (  a6 )2
of the 18O−18O stretching mode in a sample of haemerythrin that has been
treated with 18O2. (b) The fundamental vibrational wavenumbers for the where the coefficients are a1 = 0.932, a2 = 0.005050 cm2, a3 = 2333 cm−1,
O−O stretching modes of O2, O2− (superoxide anion), and O2− 2 (peroxide a4 = 1.504, a5 = 0.01521 cm2, a6 = 2362 cm−1. (a) Draw graphs of A(ν ) and
anion) are 1555, 1107, and 878 cm−1, respectively. Explain this trend in  ( ) . What is the origin of both the band and the band width? What are the
terms of the electronic structures of O2, O2− , and O2− 2 . Hint: Review Topic 9C. allowed and forbidden transitions of this band? (b) Calculate the transition
What are the bond orders of O2, O2− , and O2− 2 ? (c) Based on the data given wavenumbers and absorbances of the band with a simple harmonic
above, which of the following species best describes the Fe2O2 group of oscillator–rigid rotor model and compare the result with the experimental
haemerythrin: Fe2+2O2, Fe2 Fe3 O2 , or Fe3 2O22? Explain your reasoning. (d) spectra. The CO bond length is 116.2 pm. (c) Within what height, h, is
The Raman spectrum of haemerythrin mixed with 16O18O has two bands that basically all the infrared emission from the Earth in this band absorbed by
can be attributed to the O−O stretching mode of bound oxygen. Discuss how atmospheric carbon dioxide? The mole fraction of CO2 in the atmosphere is
this observation may be used to exclude one or more of the four proposed 3.3 × 10−4 and T/K = 288 − 0.0065(h/m) below 10 km. Draw a surface plot of
schemes (5–8) for binding of O2 to the Fe2 site of haemerythrin. the atmospheric transmittance of the band as a function of both height and
wavenumber.
Exercises and problems 497


I11.7 One of the principal methods for obtaining the electronic spectra of I11.10 Aromatic hydrocarbons and I2 form complexes from which charge-
unstable radicals is to study the spectra of comets, which are almost entirely transfer electronic transitions are observed. The hydrocarbon acts as an
due to radicals. Many radical spectra have been detected in comets, including electron donor and I2 as an electron acceptor. The energies hνmax of the charge-
that due to CN. These radicals are produced in comets by the absorption of transfer transitions for a number of hydrocarbon–I2 complexes are given
far-ultraviolet solar radiation by their parent compounds. Subsequently, their below:
fluorescence is excited by sunlight of longer wavelength. The spectra of comet
Hale–Bopp (C/1995 O1) have been the subject of many recent studies. One Hydrocarbon benzene biphenyl naphthalene phenanthrene pyrene anthracene
such study is that of the fluorescence spectrum of CN in the comet at large hνmax/eV 4.184 3.654 3.452 3.288 2.989 2.890
heliocentric distances by R.M. Wagner and D.G. Schleicher (Science 275, 1918
(1997)), in which the authors determine the spatial distribution and rate of
Investigate the hypothesis that there is a correlation between the energy
production of CN in the coma (the cloud constituting the major part of the
of the HOMO of the hydrocarbon (from which the electron comes in the
head of the comet). The (0–0) vibrational band is centred on 387.6 nm and the
charge-transfer transition) and hνmax. Use one of the computational methods
weaker (1–1) band with relative intensity 0.1 is centred on 386.4 nm. The band
discussed in Topic 9E to determine the energy of the HOMO of each
heads for (0–0) and (0–1) are known to be 388.3 and 421.6 nm, respectively.
hydrocarbon in the data set.
From these data, calculate the energy of the excited S1 state relative to the
ground S0 state, the vibrational wavenumbers and the difference in the I11.11 A lot of information about the energy levels and wavefunctions of
vibrational wavenumbers of the two states, and the relative populations of small inorganic molecules can be obtained from their ultraviolet spectra. An
the v = 0 and v = 1 vibrational levels of the S1 state. Also estimate the effective example of a spectrum with considerable vibrational structure, that of gaseous
temperature of the molecule in the excited S1 state. Only eight rotational levels SO2 at 25 °C, is shown in the accompanying figure. Estimate the integrated
of the S1 state are thought to be populated. Is that observation consistent with absorption coefficient for the band centred at 280 nm. What electronic states
the effective temperature of the S1 state? are accessible from the A1 ground state of this C2v molecule by electric-dipole
transitions?
I11.8 Use a group theoretical argument to decide which of the following
transitions are electric-dipole allowed: (a) the π⋆ ← π transition in ethene,
(b) the π⋆ ← n transition in a carbonyl group in a C2v environment. 400

I11.9 Use molecule (9) as a model of the trans conformation of the


chromophore found in rhodopsin. In this model, the methyl group bound
300
ε/(dm3 mol–1 cm–1)
to the nitrogen atom of the protonated Schiff ’s base replaces the protein.
(a) Use molecular modelling software and the computational method of your
instructor’s choice, to calculate the energy separation between the HOMO and
200
LUMO of (9). (b) Repeat the calculation for the 11-cis form of (9). (c) Based
on your results from parts (a) and (b), do you expect the experimental
frequency for the π⋆ ← π visible absorption of the trans form of (9) to be
100
higher or lower than that for the 1-cis form of (9)?

C11 0
200 240 280 320
λ/nm
+
N
H
9
FOCUS 12
Magnetic resonance

The techniques of ‘magnetic resonance’ observe transitions be- 12C Pulse techniques in NMR
tween spin states of nuclei and electrons in molecules. ‘Nuclear
magnetic resonance’ (NMR) spectroscopy observes nuclear The modern implementation of NMR spectroscopy employs
spin transitions and is one of the most widely used spectro- pulses of radiofrequency radiation followed by analysis of the
scopic techniques for the exploration of the structures and dy- resulting signal. This approach opens up many possibilities for
namics of molecules ranging from simple organic species to the development of more sophisticated experiments. The Topic
biopolymers. ‘Electron paramagnetic resonance’ (EPR) spec- includes a discussion of spin relaxation in NMR and how it can
troscopy is a similar technique that observes electron spin tran- be exploited for structural studies.
sitions in species with unpaired electrons. 12C.1 The magnetization vector; 12C.2 Spin relaxation;
12C.3 Spin decoupling; 12C.4 The nuclear Overhauser effect

12A General principles


12D Electron paramagnetic
This Topic gives an account of the principles that govern the resonance
energies and spectroscopic transitions between spin states of
nuclei and electrons in molecules in the presence of a magnetic The detailed form of an EPR spectrum reflects the molecular en-
field. It describes simple experimental arrangements for the de- vironment of the unpaired electron and the nuclei with which
tection of these transitions. it interacts magnetically. From an analysis of the spectrum it is
12A.1 Nuclear magnetic resonance; 12A.2 Electron paramagnetic possible to infer the distribution of electron spin density.
resonance 12D.1 The g-value; 12D.2 Hyperfine structure;
12D.3 The McConnell equation

12B Features of NMR spectra


What is an application of this material?
This Topic describes conventional NMR spectroscopy. It ex-
plains how the properties of a magnetic nucleus are affected by Magnetic resonance is ubiquitous in chemistry, as it is an enor-
its electronic environment and the presence of magnetic nuclei mously powerful analytical and structural technique, especially
in its vicinity. These concepts are used to explain how molecu- in organic chemistry and biochemistry. One of the most strik-
lar structure governs the appearance of NMR spectra both in ing applications of nuclear magnetic resonance is in medicine.
solution and in the solid state. ‘Magnetic resonance imaging’ (MRI) is a portrayal of the distri-
12B.1 The chemical shift; 12B.2 The origin of shielding constants; bution of protons in a solid object (Impact 18, accessed via the
12B.3 The fine structure; 12B.4 The origin of spin−spin coupling; e-book). This technique is particularly useful for diagnosing dis-
12B.5 Exchange processes; 12B.6 Solid-state NMR ease. Impact 19, accessed via the e-book, highlights an application
of electron paramagnetic resonance in materials science and bio-
chemistry: the use of a ‘spin probe’, a radical that interacts with
biopolymers or nanostructures, and has an EPR spectrum that is
sensitive to the local structure and dynamics of its environment.

➤ Go to the e-book for videos that feature the derivation and interpretation of equations, and applications of this material.
TOPIC 12A General principles

an integer (including zero) or a half-integer (Table 12A.1). The


➤ Why do you need to know this material? angular momentum associated with nuclear spin has the same
properties as other kinds of angular momentum (Topic 7F):
Nuclear magnetic resonance spectroscopy and electron
paramagnetic resonance spectroscopy are used widely in • The magnitude of the angular momentum is {I(I + 1)}1/2ħ.
chemistry to identify molecules and to determine their • The component of the angular momentum along a speci-
structures. To understand these techniques, you need to fied axis (the ‘z-axis’) is mIħ where mI = I, I − 1,…, −I.
understand how a magnetic field affects the energies of
• The orientation of the angular momentum, and hence of
the spin states of nuclei and electrons.
the magnetic moment, is determined by the value of mI.
➤ What is the key idea? According to the second property, the angular momentum, and
The spin states of nuclei and electrons have different ener- hence the magnetic moment, of the nucleus may lie in 2I + 1
gies in a magnetic field, and resonant transitions can take different orientations relative to an axis. A 1H nucleus has I = 12
place between them when electromagnetic radiation of so its magnetic moment may adopt either of two orientations
the appropriate frequency is applied.  mI   12 ,  12 . The mI   12 state is commonly denoted α and
the mI   12 state is commonly denoted β. A 14N nucleus has I = 1
➤ What do you need to know already? so there are three orientations (mI = +1, 0, −1). Examples of nu-
You need to be familiar with the quantum mechanical clei with I = 0, and hence no magnetic moment, are 12C and 16O.
concept of spin (Topic 8B), the Boltzmann distribution (see
Energy: A first look and Topic 13A), and the general features
The energies of nuclei in
12A.1(a)
of spectroscopy (Topic 11A).
magnetic fields
The energy of a magnetic moment μ in a magnetic field B is
equal to their scalar product:
Electrons and many nuclei have the property called ‘spin’, an in-
trinsic angular momentum. This spin gives rise to a magnetic E = −μ ⋅ B  (12A.1)
moment and results in them behaving like small bar magnets. More formally, B is the ‘magnetic induction’ and is measured in
The energies of these magnetic moments depend on their ori- tesla, T; 1 T = 1 kg s−2 A−1. The (non-SI) unit gauss, G, is also oc-
entation with respect to an applied magnetic field. casionally used: 1 T = 104 G. The corresponding expression for
Spectroscopic techniques that measure transitions between the hamiltonian is
nuclear and electron spin energy levels rely on the phenomenon
of resonance, the strong coupling of oscillators of the same fre- Hˆ = − μˆ ⋅ B  (12A.2)
quency. In fact, all spectroscopy is a form of resonant coupling
The magnetic moment operator of a nucleus is proportional to
between the electromagnetic field and the molecules, but in mag-
its spin angular momentum operator and is written
netic resonance, at least in its original form, the energy levels are
adjusted to match the electromagnetic field rather than vice versa. μˆ = γ N Iˆ (12A.3a)

Table 12A.1 Nuclear constitution and the nuclear spin quantum


number*
12A.1 Nuclear magnetic resonance
Number of protons Number of neutrons I
The nuclear spin quantum number, I, is a fixed characteristic Even Even 0
property of a nucleus1 and, depending on the nuclide, it is either Odd Odd Integer (1, 2, 3,…)
Even Odd Half-integer  12 , 23 , 25 ,
1
Excited nuclear states, which are states in which the nucleons are arranged Odd Even Half-integer  12 , 23 , 25 ,
differently from the ground state, can have different spin from the ground state.
Only nuclear ground states are considered here. * For nuclear ground states.
12A General principles 501

Table 12A.2 Nuclear spin properties*

Natural Magnetogyric NMR frequency


Nucleus Spin, I g-factor, gI
abundance/% ratio, γN/(107 T−1 s−1) at 1 T, ν/MHz
1
H 99.98 1
2
5.586 26.75 42.576
2
H 0.02 1 0.857 4.11 6.536
13
C 1.11 1
2
1.405 6.73 10.708
11
B 80.4 3
2
1.792 8.58 13.663
14
N 99.64 1 0.404 1.93 3.078

* More values are given in the Resource section.

The constant of proportionality, γN, is the nuclear magne- The corresponding frequency of electromagnetic radiation for
togyric ratio (also called the ‘gyromagnetic ratio’); its value a transition between these states is given by the Bohr frequency
depends on the identity of the nucleus and is determined condition ΔE = hν (Fig. 12A.1). Therefore,
empirically (Table 12A.2). If the magnetic field defines the
 N B0
z-­direction and has magnitude B0, then eqn 12A.2 becomes h   N B0 or    (12A.6)
2
Hˆ = −γ N B 0Iˆz
− μˆ z B 0 = (12A.3b) This relation is called the resonance condition, and n is called
the NMR frequency for that nucleus. Although eqn 12A.6 has
The eigenvalues of the operator Iˆz for the z-component of the been derived for a spin- 12 nucleus, the same expression applies
spin angular momentum are mIħ. The eigenvalues of the ham- for any nucleus with non-zero spin because the allowed transi-
iltonian in eqn 12A.3b, the allowed energy levels of the nucleus tions are between states that differ by ±1 in the value of mI.
in a magnetic field, are therefore It is sometimes useful to compare the quantum mechanical
Energies of a nuclear treatment with the classical picture in which magnetic nuclei
EmI   N B0 mI spin in a magnetic field
 (12A.4a) are pictured as tiny bar magnets. A bar magnet in a magnetic
field undergoes the motion called precession as it twists round
It is common to rewrite this expression in terms of the nuclear the direction of the field and sweeps out the surface of a cone
magneton, μN, (Fig. 12A.2). The rate of precession nL is called the Larmor
e Nuclear magneton ­precession frequency:
N  [definition]
 (12A.4b)
2mp  N B0 Larmor frequency of a nucleus
L   (12A.7)
(where mp is the mass of the proton) and an experimentally de- 2 [definition]
termined dimensionless constant called the nuclear g-factor, gI,
The Larmor precession frequency is the same as the resonance
  Nuclear g- factor frequency given by eqn 12A.6. In other words, the frequency
gI  N  (12A.4c)
N [definition] of radiation that causes resonant transitions between the α and
β states is the same as the Larmor precession frequency. The
Equation 12A.4a then becomes

Energies of a nuclear spin in


EmI   g I  N B0 mI a magnetic field
 (12A.4d) Magnetic Magnetic
field off field on
 mI = –½
The value of the nuclear magneton is μN = 5.051 × 10−27 J T−1.
Typical values of nuclear g-factors range between −6 and +6
(Table 12A.2). Positive values of gI and γN denote a magnetic
moment that lies in the same direction as the spin angular mo- γℏB0
mentum; negative values indicate that the magnetic moment
and spin lie in opposite directions.
When γN > 0, as is the case for the most commonly observed
nuclei 1H and 13C, in a magnetic field the energies of states with
 mI = +½
mI > 0 lie below states with mI < 0. For a spin - 21 nucleus, a nu-
cleus for which I = 12 , the α state lies lower in energy than the β Figure 12A.1 The nuclear spin energy levels of a spin- 21 nucleus
state, and the separation between them is with positive magnetogyric ratio (e.g. 1H or 13C) in a magnetic
field. Resonant absorption of radiation occurs when the energy

E  E1/ 2  E1/ 2  12  N B0   12  N B0   N B0 (12A.5) separation of the levels matches the energy of the photons.
502 12 Magnetic resonance

z a­vailable magnets, all NMR frequencies fall in the radiofre-


quency range (see Brief illustration 12A.1). Therefore, a radi-
ofrequency transmitter and receiver are needed to excite and
detect the transitions taking place between nuclear spin states.
The details of how the transitions are excited and detected are
Figure 12A.2 The classical view of magnetic nuclei pictures discussed in Topic 12C.
them as behaving as tiny bar magnets. In an externally applied The sample being studied is most commonly in the form of
magnetic field the resulting magnetic moment, here represented a solution contained in a glass tube placed within the magnet.
as a vector, precesses round the direction of the field. It is also possible to study solid samples by using more special-
ized techniques. Although the superconducting magnet itself
achievement of resonance absorption can therefore be pictured has to be held close to the temperature of liquid helium (4 K),
as changing the applied magnetic field until the bar magnet the magnet is designed so as to have a room-temperature clear
representing the nuclear magnetic moment precesses at the space into which the sample can be placed.
same frequency as the magnetic component of the electromag- The intensity of an NMR transition depends on a number of
netic field to which it is exposed. factors. They can be identified by considering the populations
of the two spin states.

Brief illustration 12A.1


How is that done? 12A.1 Identifying the contributions to
1
The NMR frequency for H nuclei (I = ) in a 12.0 T magnetic
1
2 the absorption intensity
field can be found using eqn 12A.6, with the relevant value of
γN taken from Table 12A.1: The rate of absorption of electromagnetic radiation is propor-
tional to the population of the lower energy state (Nα in the case
N B
of a spin- 12 nucleus with a positive magnetogyric ratio) and the
    
0

8 1 1
(2.6752  10 T s )  (12.0 T) rate of stimulated emission is proportional to the population of
  5.11  108 s 1  511 MHz the upper state (Nβ). At the low frequencies typical of magnetic
2
resonance, spontaneous emission can be neglected as it is very
This radiation lies in the radiofrequency region of the electro-
slow. Therefore, the net rate of absorption is proportional to the
magnetic spectrum, close to frequencies used for radio com-
difference in populations:
munication.
Rate of absorption  N   N 

Step 1 Write an expression for the intensity of absorption in terms


12A.1(b) The NMR spectrometer of the population difference
The key component of an NMR spectrometer (Fig. 12A.3) is the The intensity of absorption, the rate at which energy is
magnet into which the sample is placed. Most modern spec- absorbed, is proportional to the product of the rate of absorp-
trometers use superconducting magnets capable of produc- tion (the rate at which photons are absorbed) and the energy
ing fields of 12 T or more. Such magnets have the advantages of each photon. The latter is proportional to the frequency ν of
that the field they produce is stable over time and no electri- the incident radiation (through E = hν). At resonance, this fre-
cal power is needed to maintain the field. With the currently quency is proportional to the applied magnetic field (through
   NB0 /2), so it follows that
Intensity of absorption  rate of absorption
Superconducting
 energy of photon  (12A.8a)
magnet  (N   N  )  h NB0 /2

Step 2 Write an expression for the ratio of populations


Probe
Now use the Boltzmann distribution (see the Energy: A first
look and Topic 13A) to write an expression for the ratio of
Radio- Radio-
frequency Preamplifier frequency populations:
receiver transmitter
e−x = 1 − x + …
Figure 12A.3 The layout of a typical NMR spectrometer. The ΔE
Nβ g N B0
sample is held within the probe, which is placed at the centre of = e− g N B0 / kT
≈ 1−
Nα kT
the magnetic field.
12A General principles 503

The expansion of the exponential term (see Part 1 of the


Resource section) is appropriate for E   N B0  kT , a condi- Exercises
tion usually met for nuclear spins. 1
E12A.1 For a H nucleus (a proton), what are the magnitude of the spin
angular momentum and what are its allowed components along the
Step 3 Use that ratio to write an expression for the population z-axis? Express your answer in multiples of ℏ. What angles does the angular
­difference momentum make with the z-axis?
Consider the ratio of the population difference to the total 1
E12A.2 What is the NMR frequency of a H nucleus (a proton) in a magnetic
number of spins, N: (Nα − Nβ)/N and use the expression for the field of 13.5 T? Express your answer in megahertz.
ratio of populations to write this ratio as 1
E12A.3 Calculate the relative population differences (Nα − Nβ)/N for H nuclei
1 N B0 /kT
in fields of (i) 0.30 T, (ii) 1.5 T, and (iii) 10 T at 25 °C.

N   N N  (1  N  /N  ) 1  N  /N 
 
N   N N  (1  N  /N  ) 1  N  /N 

N
 
1 N B0 /kT
Electron paramagnetic
12A.2
1  (1   N B0 /kT )  N B0 /kT resonance
 
1  (1   N B0 /kT ) 2
  The observation of resonant transitions between the energy
1
levels of an electron in a magnetic field is the basis of electron
Therefore paramagnetic resonance (EPR; or electron spin resonance, ESR).
This kind of spectroscopy has several features in common
N  N B0 Population difference with NMR.
N   N   (12A.8b)
2kT [spin- 12 nuclei]

The intensity is now obtained by substituting this expression The energies of electrons in
12A.2(a)
into eqn 12A.8a which gives, after discarding constants that do magnetic fields
not refer to the spins,
The magnetic moment of an electron is proportional to its spin
Nγ N2B02 (12A.8c) angular momentum. Its magnetic moment operator and the
Intensity
T Absorption intensity hamiltonian for its interaction with a magnetic field are

The intensity of absorption is proportional to B02. It follows γ e sˆ and Hˆ =


μˆ = −γ e sˆ ⋅B  (12A.9a)
that the signal can be enhanced significantly by increasing the
strength of the applied magnetic field. The use of high mag-
where ŝ is the spin angular momentum operator and γe is the
netic fields also simplifies the appearance of spectra (a point
magnetogyric ratio of the electron:
explained in Topic 12B) and so allows them to be interpreted
more readily. The intensity is also proportional to γ N2, so nuclei
with large magnetogyric ratios (1H, for instance) give more in- g ee
e   Magnetogyric ratio of electron (12A.9b)
tense signals than those with small magnetogyric ratios (13C, 2me
for instance).
with ge = 2.002 319… as the g-value of the free electron. (Note
that the current convention is to include the g-value in the
Brief illustration 12A.2
definition of the magnetogyric ratio.) Dirac’s relativistic the-
For 1H nuclei γN = 2.675 × 108 T−1 s−1. Therefore, for 1 000 000 of ory, his modification of the Schrödinger equation to make it
these nuclei in a field of 10 T at 20 °C, consistent with Einstein’s special relativity, gives ge = 2; the ad-
ditional 0.002 319… arises from interactions of the electron
N   B0
N       with the electromagnetic fluctuations of the vacuum that sur-
8 1 1 34
1000000  (2.675 10 T s )  (1.055 10 Js)  (10 T) rounds it. The negative sign of γe (arising from the sign of the
N  N 
2  (1.3811023 JK 1 )  (293 K ) charge on the electron) shows that the magnetic moment is
      
k T opposite in direction to the vector representing its angular
35 momentum.
Even in such a strong field there is only a tiny imbalance of The hamiltonian when the magnetic field lies in the
population of about 35 in a million. This small population z-­direction and has magnitude B0 is
difference means that special techniques had to be developed
before NMR became a viable technique.
Hˆ = −γ eB0 sˆz  (12A.10)
504 12 Magnetic resonance

where sˆz is the operator for the z-component of the spin angular Magnetic Magnetic
momentum. It follows that the energies of an electron spin in a field off field on
 ms = +½
magnetic field are

Energies of an electron spin in


Ems   e B0 ms a magnetic fielld
 (12A.11a)
ge µB B 0
with ms   12 . It is common to write this expression in terms of
the Bohr magneton, μB, defined as
e
B  Bohr magneton (12A.11b)
2me  mI = –½

Its value is 9.274 × 10−24 J T−1. This positive quantity is often Figure 12A.4 Electron spin levels in a magnetic field. Note
regarded as the fundamental quantum of magnetic moment. that the β state is lower in energy than the α state (because
Note that the Bohr magneton is about 2000 times bigger than the magnetogyric ratio of an electron is negative). Resonant
the nuclear magneton, so electron magnetic moments are absorption occurs when the frequency of the incident radiation
that much bigger than nuclear magnetic moments. By using matches the frequency corresponding to the energy separation.
eqn 12A.9b the term γeħ in eqn 12A.11a can be expressed as
−geeħ/2me, which in turn can be written −geμB by introducing ­region (see Brief illustration12A.3). The layout of a typical
the definition of the Bohr magneton from eqn 12A.11b. It then EPR spectrometer is shown in Fig. 12A.5. It consists of a fixed-
follows that frequency microwave source (typically a Gunn oscillator, based
on a solid-state device), a cavity into which the sample (held in
Energies of an electron spin in
Ems  g e BB0 ms a magnetic fielld
 (12A.11c) a glass or quartz tube) is inserted, a microwave detector, and an
electromagnet with a variable magnetic field. The sample in an
The energy separation between the ms   12 () and ms   12 () EPR observation must have unpaired electrons, so is either a
states is therefore radical or a d-metal complex. For technical reasons related to
the detection procedure, the spectrum shows the first deriva-
E  E1/ 2  E1/ 2  12 g e BB0    12 g e BB0   g e BB0 (12A.12a) tive of the absorption line (Fig. 12A.6).
As in NMR, the intensities of spectral lines in EPR depend on
with β the lower state. This energy separation comes into reso- the difference in populations between the ground and excited
nance with electromagnetic radiation of frequency n when states. For an electron, the β state lies below the α state in energy
(Fig. 12A.4) and, by a similar argument that led to eqn 12A.8b for nuclei,
Ng e BB0
h  g e BB0 Resonance condition for EPR (12A.12b) N  N   Population difference [electrons] (12A.13)
2kT
where N is the total number of electron spins.
Brief illustration 12A.3

A typical commercial EPR spectrometer uses a magnetic field Microwave Detector


of about 0.33 T. The EPR resonance frequency is source
 B Electromagnet
g e 
 B    0
    Sample
24 1
(2.0023)  (9.274  10 JT )  (0.33 T) cavity
 Phase
6.626  1034 Js
  sensitive
h detector
 9.2  109 s 1  9.2 GHz
This frequency corresponds to a wavelength of 3.2 cm, which
is in the microwave region, and specifically in the ‘X band’ of
frequencies.
Modulation
input
12A.2(b) The EPR spectrometer
Figure 12A.5 The layout of a typical EPR spectrometer. A typical
Most commercial EPR spectrometers use magnetic field magnetic field is 0.3 T, which requires 9 GHz (3 cm) microwaves for
strengths that result in EPR frequencies in the microwave resonance.
12A General principles 505

Brief illustration 12A.4


Absorption, A
When 1000 electron spins experience a magnetic field of 1.0 T
at 20 °C (293 K), the population difference is
Signal

Slope  B
ge
Slope N   B   0
1000  2.0023  (9.274  10 JT 1 )  (1.0 T)
24

N  N  
2  (1.381  1023 JK 1 )  (293 K )
Field, B       
k T
Derivative  2. 3
of absorption, dA/dB
There is an imbalance of populations of only about two elec-
Figure 12A.6 When phase-sensitive detection is used, the signal trons in a thousand. However, the imbalance is much larger for
is the first derivative of the absorption intensity. Note that the electron spins than for nuclear spins (Brief illustration 12A.2)
peak of the absorption corresponds to the point where the because the energy separation between the spin states of elec-
derivative passes through zero. trons is larger than that for nuclear spins even at the lower
magnetic field strengths normally employed for EPR.

Exercises
E12A.4 In which of the following systems is the energy level separation larger? E12A.5 Some commercial EPR spectrometers use 8 mm microwave radiation
(i) A 14N nucleus in a magnetic field that corresponds to an NMR frequency (the ‘Q band’). What magnetic field is needed to satisfy the resonance
for 1H of 600 MHz, (ii) an electron in a field of 0.300 T. condition?

Checklist of concepts
☐ 1. The nuclear spin quantum number, I, of a nucleus is ☐ 4. In NMR the absorption intensity increases with the
either a non-negative integer or half-integer; I can be strength of the applied magnetic field (as B02 ) and is also
zero. proportional to the square of the magnetogyric ratio of
☐ 2. In the presence of a magnetic field a nucleus has 2I + 1 the nucleus.
energy levels characterized by different values of mI. ☐ 5. In the presence of a magnetic field, an electron has two
☐ 3. Nuclear magnetic resonance (NMR) is the observation energy levels corresponding to the α and β spin states.
of the absorption of radiofrequency electromagnetic ☐ 6. Electron paramagnetic resonance (EPR) is the observa-
radiation by nuclei in a magnetic field. tion of the resonant absorption of microwave electro-
magnetic radiation by unpaired electrons in a magnetic
field.

Checklist of equations
Property Equation Comment Equation number
Energies of a nuclear spin in a magnetic field EmI   N B0mI 12A.4a
  g I NB0mI 12A.4d
Nuclear magneton μN = eħ/2mp μN = 5.051 × 10−27 J T−1 12A.4b
Resonance condition (spin - nuclei)
1
2 h   N B0 12A.6

Larmor frequency  L   NB0 /2 12A.7

Magnetogyric ratio (electron) γe = −gee/2me ge = 2.002 319 … 12A.9b


Energies of an electron spin in a magnetic field Ems   e B0ms 12A.11a
 g e BB0ms 12A.11c
Bohr magneton μB = eħ/2me μB = 9.274 × 10−24 J T−1 12A.11b
Resonance condition (electrons) h  g e BB0 12A.12b
TOPIC 12B Features of NMR spectra

i­nduce an electronic current in the molecule, and hence affect


➤ Why do you need to know this material? the strength of the resulting local magnetic field, depends on
To analyse NMR spectra and extract the wealth of informa- the details of the electronic structure near the magnetic nucleus
tion they contain you need to understand how the appear- of interest. Therefore, nuclei in different chemical groups have
ance of a spectrum arises from molecular structure. different shielding constants. As a result, the Larmor frequency
νL of the nucleus (and therefore its resonance frequency)
➤ What is the key idea? changes from  N B0 /2 to
The resonance frequency of a magnetic nucleus is affected  N B1oc  N (B0  B )  N (B0   B0 )  N B0
by its electronic environment and the presence of nearby L     (1   )
 2 2 2 2
magnetic nuclei.
(12B.3)
➤ What do you need to know already? The Larmor frequency is different for nuclei in different envi-
You need to be familiar with the general principles of mag- ronments, even if those nuclei are of the same element.
netic resonance (Topic 12A). The chemical shift of a nucleus is the difference between
its resonance frequency and that of a reference standard. The
standard for 1H and 13C is the resonance in tetramethylsilane,
Si(CH3)4, commonly referred to as TMS. The frequency separa-
tion between the resonance from a particular nucleus and that
Nuclear magnetic moments interact with the local magnetic
from the standard increases with the strength of the applied
field, the field at the location of the nucleus in question. The
magnetic field because the Larmor frequency (eqn 12B.3) is
local field differs from the applied field due to the effects of the
proportional to the applied field.
electrons surrounding the nucleus and the presence of other
Chemical shifts are reported on the δ scale, which is defined as
magnetic nuclei in the molecule. The overall effect is that the
NMR frequency of a given nucleus is sensitive to its molecular     scale
  106  (12B.4a)
environment.  [definition]

where n° is the resonance frequency (Larmor frequency) of the


standard. Because n° is very close to the operating frequency
12B.1 The chemical shift of the spectrometer nspect, which is typically chosen to be in the
middle of the range of Larmor frequencies exhibited by the nu-
cleus being studied, the n° in the denominator of eqn 12B.4a
The applied magnetic field can be thought of as causing a cir-
can safely be replaced by nspect to give
culation of electrons through the molecule. This circulation is
analogous to an electric current and so gives rise to a magnetic   
field. The local magnetic field, Bloc , the total field experienced   106 (12B.4b)
 spect
by the nucleus, is the sum of the applied field B0 and the addi-
tional field δB due to the circulation of the electrons The advantage of the δ scale is that shifts reported with it are
independent of the applied field (because both numerator and
Bloc  B0  B  (12B.1) denominator in eqn 12B.4a are proportional to the applied
field). The resonance frequencies themselves, however, do de-
The additional field is proportional to the applied field, and it is pend on the applied field through1
conventional to write
  spect 
Shielding constant
      6    (12B.5)
B   B0 [definition]
 (12B.2)  10 
1
In much of the literature, chemical shifts are reported in parts per million,
where the dimensionless quantity σ is called the shield- ppm, in recognition of the factor of 106 in the definition; this is unnecessary. If
ing ­constant of the nucleus. The ability of the applied field to you see ‘δ = 10 ppm’, interpret it, and use it in eqn 12B.5, as δ = 10.
12B Features of NMR spectra 507

Brief illustration 12B.1 e­ nvironments of the nuclei in molecules: the higher the atomic
number of the element, the greater the number of electrons
In an NMR spectrometer operating at 500.130 00 MHz around the nucleus and hence the greater the range of the ex-
the resonance from TMS is found to be at a frequency of tent of shielding. By convention, NMR spectra are plotted with
500.127 50 MHz. The chemical shift of a resonance at a fre- δ decreasing from left to right.
quency of 500.128 25 MHz is

   (500.128 25 MHz)  (500.127 50 MHz) Example 12B.1


  106   106 Interpreting a 1H NMR spectrum
 spect 500.130 00 MHz
 1. 5 Figure 12B.2 shows the 1H (proton) NMR spectrum of
1-methoxy-2-propanone, CH3OCH2COCH3. Account for the
On the same spectrometer the frequency separation of two observed chemical shifts.
resonances with chemical shifts δ1 = 1.25 and δ2 = 5.75 is found
using eqn 12B.5 Collect your thoughts You need to consider the effect of any
electron-withdrawing atom: it deshields strongly the protons
 spect 500.130 00  106 Hz to which it is bound and has a diminishing effect on more
 2  1   ( 2  1 )   (5.75  1.25)
10 6
106 distant protons. To identify which of the B and C resonances
 2250 Hz correspond to H atoms 2 and 3 you can take either of two
approaches. One is to look at the large compilations of chemi-
cal shift data available. The second approach is to make use
of the ‘integral’ of a line, the area under the resonance peak,
The relation between δ and σ is obtained by substituting eqn which is proportional to the number of nuclei giving rise to the
12B.3 into eqn 12B.4a: peak. These integrals are commonly shown by step-like curves
superimposed on the spectrum, as is the case in Fig. 12B.2: the
(1 − σ )B0 − (1 − σ °)B0
δ= × 106 integral is proportional to the height of the step.
(1 − σ °)B0
|σ °| << 1
σ°−σ The solution The H atoms labelled 2 and 3 are all attached to
= × 106 (σ ° − σ ) × 106 Relation between
(12B.6) a C atom that is attached to the strongly electron-withdrawing
1− σ°  and 
O atom, whereas the H atoms labelled 1 are further away from
where σ° is the shielding constant of the reference standard. any O atoms. You can expect the deshielding of H atoms 2 and
A decrease in σ (reduction in shielding) therefore leads to an 3 to be greater than that of H atoms 1, and so the chemical
increase in δ. Therefore, nuclei with large chemical shifts are shifts of 2 and 3 will be larger than that of 1. Resonance A at
said to be strongly deshielded. Some typical chemical shifts δ = 2.2 can therefore confidently be assigned to the H atoms
are given in Fig. 12B.1. As can be seen from the illustration, at position 1. It is evident from the spectrum that the integral
the nuclei of different elements have very different ranges of of peak B is greater than that of C (in fact they are in the ratio
chemical shifts. The ranges exhibit the variety of electronic 3:2), immediately identifying peak B as corresponding to the H
atoms at position 3, and peak C to those at position 2.
Self-test 12B.1 The 1H (proton) NMR spectrum of ethanal
RCH3 (acetaldehyde) has lines at δ = 2.20 and δ = 9.80. Which line
–CH2–
R–NH can be assigned to the H atom in the CHO group?
RC–CH3 –CH– 2
–CO–CH3 ArC–CH3 O
ROH B A
–C=CH– O
–CHO ArOH H3C C CH 3
Ar–H C H2
–COOH 3 1
2
(a) 12 10 8 6 4 2 0
δ
R–C–H
R3C–
>C=C<
X –C=C<
–C=C–
R–CHO C–X in ArX 4.0 3.5 3.0 2.5 2.0

R–C=N δ
R2C=O R–COOH 1
R=C=R Figure 12B.2 The H (proton) NMR spectrum of 1-methoxy-
R3C+
2-propanone. The step-like curve indicates the integral of the
300 200 δ 100 0 peak (the area under the peak) with the height of the step being
(b)
proportional to the integral.
Figure 12B.1 The range of typical chemical shifts for (a) 1H Answer: δ = 9.80
resonances and (b) 13C resonances.
508 12 Magnetic resonance

e 2 0
Exercises d  1/r  Lamb formula (12B.9)
1
12me
E12B.1 The H resonance from TMS is found to occur at 500.130 000 MHz.
What is the chemical shift (on the δ scale) of a peak at 500.132 500 MHz?
where μ0 is the magnetic constant (vacuum permeability), r is
E12B.2 What is the frequency separation, in hertz, between two peaks in a
1
the electron–nucleus distance, and the angle brackets 〈…〉 indi-
H NMR spectrum with chemical shifts δ = 9.80 and δ = 2.2 in a spectrometer
cate an expectation value.
operating at 400.130 000 MHz for 1H?

Example 12B.2 Using the Lamb formula


The origin of shielding
12B.2 Calculate the shielding constant for the nucleus in a free H
constants atom.
Collect your thoughts To calculate σd from the Lamb formula,
The calculation of shielding constants (and hence chemical you need to calculate the expectation value of 1/r for a hydro-
shifts) is difficult because it requires detailed knowledge of gen 1s orbital. The radial part of the wavefunction can be found
the distribution of electron density in the ground and excited from Table 8A.1 and the angular part from Table 7F.1.
states and the electronic excitation energies of the molecule.
The solution The normalized wavefunction for a hydrogen
Nevertheless, it is helpful to understand the different contri-
1s orbital is, in spherical polar coordinates (see The chemist’s
butions to chemical shifts so that patterns and trends can be
toolkit 7F.2),
­identified.
A useful approach is to assume that the observed shielding  1 
1/ 2

constant is the sum of three contributions:    3  e  r /a0


 a0 
   (local)   (neighbour )   (solvent) (12B.7) In this coordinate system the volume element is dτ =
The local contribution, σ(local), is essentially the contribution sin θ r2drdθdφ, so the expectation value of 1/r is
of the electrons of the atom that contains the nucleus in ques- 2  2  Integral E.2

tion. The neighbouring group contribution, σ(neighbour),   1 2  
1/r    d  3  d  sin d  re 2r /a0 dr
is the contribution from the groups of atoms that form the rest r a0 0 0 0

of the molecule. The solvent contribution, σ(solvent), is the 1 a2 1


contribution from the solvent molecules.  3  4  0 
a0 4 a0

12B.2(a) The local contribution Therefore,

It is convenient to regard the local contribution to the shielding e 2 0 (1.602  1019 C)2  (4   107 Js2 C 2 m 1 )
d  
constant as the sum of a diamagnetic contribution, σd, and a 12mea0 12  (9.109  1031 kg )  (5.292  1011 m)
paramagnetic contribution, σp: C 2 Js2 C 2 m 1
(1.602  1019 )2  (4   107 )
 
Local contribution to 12  (9.109  1031 )  (5.292  1011 ) kg m
 (1ocal)   d   p the shielding constant
 (12B.8)
 1.775  105
The diamagnetic contribution arises from additional fields that
where 1 J = 1 kg m2 s−2 has been used.
oppose the applied magnetic field and hence shield the nucleus;
σd is therefore positive. The paramagnetic contribution arises Self-test 12B.2 Derive an expression for σd for a hydrogenic
from additional fields that reinforce the applied field and hence atom with nuclear charge Z.
lead to deshielding; σp is therefore negative. Answer: σ d = Ze2μ0/12πmea0
The diamagnetic contribution arises from the ability of the
applied field to generate a circulation of charge in the ground-
state electron distribution. The circulation generates a mag- The diamagnetic contribution is the only contribution in
netic field which opposes the applied field and hence shields closed-shell free atoms and when the electron distribution is
the nucleus. The magnitude of σd depends on the electron den- spherically symmetric. In a molecule the core electrons near
sity close to the nucleus and for atoms it can be calculated from to a particular nucleus are likely to have spherical symmetry,
the Lamb formula:2 even if the valence electron distribution is highly distorted.
Therefore, core electrons contribute only to the diamagnetic
2
For a derivation, see our Molecular quantum mechanics (2011). part of the shielding. The diamagnetic contribution is broadly
12B Features of NMR spectra 509

1 for a diamagnetic group because the induced moment is oppo-


Chemical shift relative to CH4, δ site to the direction of the applied field.
CH3CH2X The induced moment gives rise to a magnetic field which is
2
experienced by neighbouring nuclei. As explained in The chem-
ist’s toolkit 12B.1, a nucleus at distance r and angle θ (defined in 1)
CH3CH2X
from the induced moment experiences a local field that has the
form
3
 induced
Blocal  (1  3 cos 2  ) Local dipolar field (12B.10a)
r3
I Br Cl F
4
2 2.5 3 3.5 4 θ
Electronegativity of halogen, χ r

Figure 12B.3 The variation of chemical shielding with µinduced


electronegativity. The shifts for the methyl protons follow the
simple expectation that increasing the electronegativity of the
1
halogen will increase the chemical shift. However, to emphasize
that chemical shifts are subtle phenomena, notice that the trend This local field is parallel to the applied field, and the angle
for the methylene protons is opposite to that expected. For these
θ is measured from the direction of the applied field. Note that
protons another contribution (the magnetic anisotropy of C–H
the strength of the field is inversely proportional to the cube of
and C–X bonds) is dominant.
the distance r between H and X. If the magnetic susceptibility
is independent of the orientation of the molecule (that is, it is
‘isotropic’) then the induced dipole is also independent of ori-
proportional to the electron density of the atom containing the
entation. It follows that the local field given in eqn 12B.10a av-
nucleus of interest. It follows that the shielding is decreased if
erages to zero because, when averaged over a sphere, 1 − 3 cos2θ
the electron density on the atom is reduced by the influence of
is zero.
an electronegative atom nearby. That reduction in shielding as
However, if the magnetic susceptibility, and hence the in-
the electronegativity of a neighbouring atom increases trans-
duced dipole, varies with the orientation of the molecule with
lates into an increase in the chemical shift δ (Fig. 12B.3).
The local paramagnetic contribution, σp, arises from the
ability of the applied field to force electrons to circulate through
the molecule by making use of orbitals that are unoccupied in The chemist’s toolkit 12B.1   Dipolar magnetic fields
the ground state. It is absent in free atoms and also in linear
molecules (such as ethyne, HC≡CH) when the applied field Standard electromagnetic theory gives the magnetic field at a
lies along the symmetry axis; in this arrangement the electrons point r from a point magnetic dipole μ as
can circulate freely and the applied field is unable to force them
0  3(  r )r 
into other orbitals. Large paramagnetic contributions can be B 
expected for light atoms (because the valence electrons, and 4 r 3  r 2 
hence the induced currents, are close to the nucleus) and in where μ0 is the magnetic constant (previously known as the
molecules with low-lying excited states (because an applied vacuum permeability, a fundamental constant with the value
field can then induce significant currents). In fact, the para- 1.257 × 10−6 T2 J−1 m3). The component of magnetic field in the
magnetic contribution is the dominant local contribution for z-direction is
atoms other than hydrogen.
0  3(  r )z 
Bz   z 
4 r 3  r 2 
12B.2(b) Neighbouring group contributions
The neighbouring group contribution arises from the currents with z = r cos θ, the z-component of the distance vector r. If
induced in nearby groups of atoms. Consider the influence of the magnetic dipole is also parallel to the z-direction and has
magnitude μ, it follows that
the neighbouring group X on the hydrogen atom in a molecule
such as H–X. The applied field generates currents in the electron  r
z   z 
  
distribution of X and gives rise to an induced magnetic moment 0    3 (r cos )(r cos )  0
(an induced magnetic dipole) proportional to the applied field; Bz     (1  3 cos2  )
4 r 3  r2  4 r 3
the constant of proportionality is the magnetic susceptibility, χ  
(chi), of the group X: induced   B0 . The s­ usceptibility is negative
510 12 Magnetic resonance

respect to the magnetic field, the local field may average to a 1


non-zero value. For instance, suppose that the neighbouring
group has axial symmetry (as might be the case for a triple
bond): when the applied field is parallel to the symmetry axis 0

1 – 3 cos2 Θ
the susceptibility is χ‖, and when it is perpendicular the suscep-
tibility is χ⊥. After averaging over all orientations of the mol-
ecule the contribution to the shielding constant of a nucleus at a
distance R has the following form

 1  3 cos 2  
 (neighbour )  (     )  
 R3 
–2
0 ½ 
Θ
Neighbouring group contriibution (12B.10b)
Figure 12B.5 The variation of the function 1 − 3 cos2Θ with the
where Θ (uppercase theta) is the angle between the symmetry
angle Θ.
axis and the vector to the nucleus (2). Equation 12B.10b shows
that the neighbouring group contribution may be positive or
negative according to the relative magnitudes of the two mag-
netic susceptibilities and the direction given by Θ. If 54.7° < Θ <
125.3°, then 1 − 3 cos2 Θ is positive, but it is negative otherwise
(Figs. 12B.4 and 12B.5).
B
Ring
Θ current
r
H Magnetic
χ
field
X χ

A special case of a neighbouring group effect is found in aro-


matic compounds. The strong anisotropy of the magnetic sus-
ceptibility of the benzene ring is ascribed to the ability of the Figure 12B.6 The shielding and deshielding effects of the ring
field to induce a ring current, a circulation of electrons around current induced in the benzene ring by the applied field. Protons
the ring, when the field is applied perpendicular to the molecu- attached to the ring are deshielded but a proton attached to a
lar plane. Protons in the plane are deshielded (Fig. 12B.6), but substituent that projects above the ring is shielded.
any that happen to lie above or below the plane (as members of
substituents of the ring) are shielded.
12B.2(c) The solvent contribution
A solvent can influence the local magnetic field experienced by
a nucleus in a variety of ways. Some of these effects arise from
specific interactions between the solute and the solvent (such as
hydrogen bond formation and other forms of Lewis acid–base
– complex formation). The anisotropy of the magnetic suscepti-
bility of the solvent molecules, especially if they are aromatic,
+ +
can also be the source of a local magnetic field. Moreover, if
– there are steric interactions that result in a loose but specific
interaction between a solute molecule and a solvent molecule,
then protons in the solute molecule may experience shielding
or deshielding effects according to their location relative to the
Figure 12B.4 A depiction of the field arising from a point solvent molecule. An aromatic solvent such as benzene can give
magnetic dipole. The contours represent the strength of field rise to local currents that shield or deshield a proton in a solute
declining with distance (as 1/R3). The field is zero on the straight molecule. The arrangement shown in Fig. 12B.7 leads to shield-
lines. ing of a proton on the solute molecule.
12B Features of NMR spectra 511

state (α, for instance) the local field is increased, whereas when
the spin is in the other state (β in this case), the local field is
B
decreased. Therefore, rather than there being one line in the
spectrum, there are two because there are two possible values
for the local field, corresponding to the second nucleus being
either α or β.
The scalar coupling interaction is represented by a term
(hJ /2 )Iˆ1 ⋅ Iˆ2 in the hamiltonian, where IˆN , with N = 1 or 2, is the
operator for the nuclear spin angular momentum of nucleus N.
That the coupling term is a scalar product simply expresses the
fact that the energy of interaction depends on the relative ori-
Figure 12B.7 An aromatic solvent (such as benzene shown here)
entation of the spins of the two nuclei. The strength of the in-
can give rise to local currents that shield or deshield a proton in
a solute molecule. In the orientation shown the proton on the
teraction is given by the value of the scalar coupling constant,
solute molecule is shielded. J. The presence of ħ2 in (hJ /2 )Iˆ1 ⋅ Iˆ2 cancels the ℏ2 arising from
the eigenvalues of the two angular momenta, leaving the energy
as hJ, so J is a frequency (measured in hertz, Hz). The coupling
Exercise constant can be positive or negative, and it is independent of
the field strength.
E12B.3 The chemical shift of the CH3 protons in ethanal (acetaldehyde) is
δ = 2.20 and that of the CHO proton is 9.80. What is the difference in local If the Larmor frequencies of the two coupled nuclei are sig-
magnetic field between the two regions of the molecule when the applied field nificantly different, they precess at very different rates and the
is (i) 1.5 T, (ii) 15 T? x- and y-components of their magnetic moments are never in
step. Only the z-components remain in alignment whatever the
precession rates, and so the only surviving term in the scalar
product is (hJ / )Iˆ1z Iˆ2 z . The eigenvalues of each IˆNz are mIN ,
2

12B.3 The fine structure so it follows that the eigenvalues (the energies) of the coupling
term are
Figure 12B.8 shows the 1H (proton) NMR spectrum of chloro­
ethane. In this molecule there are two different types of 1H, the EmI mI = hJmI1 mI2 Spin – spin coupling energy (12B.11)
1 2
methylene (CH2) and the methyl (CH3) protons, each with a
characteristic chemical shift. In addition, the spectrum shows
fine structure, the splitting of a resonance line into several
12B.3(a) The appearance of the spectrum
components. The groups of lines are called multiplets. In NMR, letters far apart in the alphabet (typically A and X) are
This fine structure arises from scalar coupling in which the used to indicate nuclei with very different chemical shifts in the
resonance frequency of one nucleus is affected by the spin state sense that the difference in chemical shift corresponds to a fre-
of another nucleus. Qualitatively, the effect of scalar coupling quency that is large compared to J; letters close together (such
arises when the local magnetic field at one nucleus depends on as A and B) are used for nuclei with similar chemical shifts.
the relative orientation of the other spin. If the spin is in one Consider first an AX system, a molecule that contains two
spin- 12 nuclei A and X with very different chemical shifts, so
eqn 12B.11 can be used for the spin–spin coupling energy.
Nucleus A has two spin states with mA   12 corresponding to
CH3CH2Cl
the states denoted αA and βA. The X nucleus also has two spin
states with mX   12 (αX and βX). In the AX system there are
therefore four spin states: αAαX, αAβX, βAαX, and βAβX. The ener-
CH3CH2Cl gies of these states, neglecting any scalar coupling, are therefore

EmA mX   N (1   A )B0 mA   N (1   X )B0 mX


 (12B.12a)
 h A mA  h X mX
3.5 3.0 2.5 δ 2.0 1.5
where nA and nX are the Larmor frequencies of A and X (eqn
Figure 12B.8 The 1H (proton) NMR spectrum of chloroethane. 12B.3). This expression gives the four levels illustrated on the
The arrows indicate the protons giving rise to each multiplet; the left of Fig. 12B.9. When spin–spin coupling is included (by
multiplets arise due to scalar coupling between the protons. using eqn 12B.11) the energy levels are
512 12 Magnetic resonance

EmA mX  h A mA  h X mX  hJmA mX  (12B.12b) E  h A  12 hJ  (12B.13a)

The resulting energy level diagram (for J > 0) is shown on the The spectrum due to A transitions therefore consists of a dou-
right of Fig. 12B.9. The αAαX and βAβX states are both raised by blet of separation J centred on the Larmor frequency of A
1
4
hJ and the αAβX and βAαX states are both lowered by 14 hJ . For (Fig. 12B.11). Similar remarks apply to the transitions in which
J > 0, the effect of the coupling term is to lower the energy of the spin state of X changes while that of A remains fixed. These
the αAβX and βAαX states, and raise the energy of the other two transitions are also shown in Figs. 12B.9 and 12B.10, and the
states. The opposite is the case for J < 0. transition energies are
In a transition, only one nucleus changes its orientation, so
the selection rule is that either mA or mX can change by ±1, but E  h X  12 hJ  (12B.13b)
not both. There are two transitions in which the spin state of
A changes while that of X remains fixed: βAαX ← αAαX and It follows that there is a second doublet with the same separa-
βAβX ← αAβX. They are shown in Fig. 12B.9 and in a slightly dif- tion J as the first, but now centred on the Larmor frequency of
ferent form in Fig. 12B.10. The energies of the transitions are X (as shown in Fig. 12B.11). Overall, the spectrum of an AX
spin system consists of two doublets.
Suppose that there is another X nucleus in the molecule with
the same chemical shift as the first X thus giving an AX2 spin
No spin–spin With spin–spin
coupling coupling system; the two X nuclei are said to be equivalent. The X reso-
βAβX nance is split into a doublet by A, just as for AX (Fig. 12B.12).
βAβX ¼hJ
½hL(A) + ½hL(X)
Energy

βAαX

A resonance

X resonance
½hL(A) – ½hL(X)
¼hJ βAαX

αAβX
–½hL(A) + ½hL(X)
¼hJ αAβX
J J
αAαX

αAαX ¼hJ
–½hL(A) – ½hL(X) A X

Figure 12B.9 The energy levels of an AX spin system. The four Figure 12B.11 The effect of spin–spin coupling on an AX
levels on the left are those in the case of no spin–spin coupling. spectrum. Each resonance is split into two lines, a doublet,
The four levels on the right show how a positive spin–spin separated by J. There is a doublet centred on the Larmor
coupling constant affects the energies. The effect of the coupling frequency (chemical shift) of A, and one centred on the Larmor
on the energy levels has been exaggerated greatly for clarity; in frequency of X.
practice, the change in energy caused by spin–spin coupling is
much smaller than that caused by the applied field.
X resonance
in AX2

βAβX

X resonance
in AX
βAαX

αAβX
J J

αAαX
X X

Figure 12B.10 An alternative depiction of the energy levels and Figure 12B.12 The X resonance of an AX2 spin system is a doublet
transitions shown in Fig. 12B.9. Once again, the effect of spin–spin because of the coupling to the A spin. However, the overall
coupling has been exaggerated. absorption is twice as intense as that of an AX spin system.
12B Features of NMR spectra 513

Successive rows of this triangle are formed by adding together


the two adjacent numbers in the line above.

1
1 1
1 2 1
1 3 3 1
1 4 6 4 1
1 5 10 10 5 1
3

A Example 12B.3 Accounting for the fine structure in a


Figure 12B.13 The origin of the 1:2:1 triplet in the A resonance spectrum
of an AX2 spin system. The resonance of A is split into two by Account for the fine structure in the 1H (proton) NMR spec-
coupling with one X nucleus (as shown in the inset), and then trum of chloroethane shown in Fig. 12B.8.
each of those two lines is split into two by coupling to the second
X nucleus. Because each X nucleus causes the same splitting, Collect your thoughts You need to consider how each group
the two central transitions are coincident and give rise to an of equivalent protons (for instance, the three methyl protons)
absorption line of double the intensity of the outer lines. splits the resonances of the other groups of protons. There is
no splitting within groups of equivalent protons. You can iden-
tify the pattern of intensities within a multiplet by referring to
Pascal’s triangle.

The resonance of A is split into a doublet by one X, and each The solution The three protons of the CH3 group split the
line of the doublet is split again by the same amount by the sec- resonance of the CH2 protons into a 1:3:3:1 quartet with a
ond X (Fig. 12B.13). This splitting results in three lines in the splitting J. Likewise, the two protons of the CH2 group split the
intensity ratio 1:2:1 (because the central frequency can be ob- resonance of the CH3 protons into a 1:2:1 triplet with the same
tained in two ways). splitting J.
Three equivalent X nuclei (an AX3 spin system) split the reso- Self-test 12B.3 What fine structure can be expected in the 1H
nance of A into four lines of intensity ratio 1:3:3:1 (Fig. 12B.14). spectrum, and in the 15N spectrum, of 15 NH +4 ? Nitrogen-15 is
The X resonance remains a doublet as a result of the splitting a spin- 12 nucleus.
caused by A. In general, N equivalent spin- 12 nuclei split the res- the 15N spectrum is a 1:4:6:4:1 quintet
onance of a nearby spin or group of equivalent spins into N + 1 Answer: The 1H spectrum is a 1:1 doublet and
lines with an intensity distribution given by Pascal’s triangle (3).

The magnitudes of coupling


12B.3(b)
constants
The scalar coupling constant of two nuclei separated by N
bonds is denoted NJ, with subscripts to indicate the types of nu-
clei involved. Thus, 1JCH is the coupling constant for a proton
joined directly to a 13C atom, and 2JCH is the coupling constant
when the same two nuclei are separated by two bonds (as in
13
C–C–H). A typical value of 1JCH is in the range 120–250 Hz;
2
JCH is between 10 and 20 Hz. Couplings over both three and
four bonds (3J and 4J) can give detectable effects in a spectrum,
but couplings over larger numbers of bonds can generally be
ignored.
A As remarked in the discussion following eqn 12B.12b, the
Figure 12B.14 The origin of the 1:3:3:1 quartet in the A resonance sign of JXY determines whether a particular energy level is raised
of an AX3 spin system. The third X nucleus splits each of the lines or lowered as a result of the coupling interaction. If J > 0, the
shown in Fig. 12B.13 for an AX2 spin system into a doublet, and levels with antiparallel spins are lowered in energy, whereas if
the intensity distribution reflects the number of transitions that J < 0 the levels with parallel spins are lowered. Experimentally,
have the same energy. it is found that 1JCH is invariably positive, 2JHH is often negative,
514 12 Magnetic resonance

19 10
14 E12B.5 Sketch the form of the F NMR spectrum and the B NMR spectrum
Spin–spin coupling constant, J/Hz
10 −
of BF .4
12 HCCH
E12B.6 Use an approach similar to that shown in Figs. 12B.13 and 12B.14 to
10 predict the multiplet you would expect for coupling to four equivalent spin- 12
nuclei.
8
HNCH
6

4 12B.4 The origin of spin–spin coupling


2
Some insight into the origin of coupling, if not its precise mag-
0 nitude—or always reliably its sign—can be obtained by consid-
0 π/2 π
Angle, ϕ ering the magnetic interactions within molecules. A nucleus
Figure 12B.15 The variation of the spin–spin coupling constant with the z-component of its spin angular momentum specified
with dihedral angle predicted by the Karplus equation for an by the quantum number mI gives rise to a magnetic field with
HCCH group and an HNCH group. z-component Bnuc at a distance R, where, to a good approximation,

 N 0
3 Bnuc   (1  3 cos2  )mI (12B.15)
H ϕ
H
and JHH is often positive. An additional point is 4 R 3
that J varies with the dihedral angle between the
bonds (Fig. 12B.15). Thus, a 3JHH coupling con- The angle θ is defined in (1); this expression is a version of eqn
stant is often found to depend on the dihedral 12B.10a. However, in solution molecules tumble rapidly so it
angle ϕ (4) according to the Karplus equation: is necessary to average Bnuc over all values of θ. As has already
4 been noted, the average of 1 − 3cos2 θ is zero, therefore the di-
rect dipolar interaction between spins cannot account for the
3
J HH  A  B cos   C cos 2 Karplus equation (12B.14) fine structure seen in the spectra of molecules in solution.

with A, B, and C empirical constants with values close to +7 Hz,


−1 Hz, and +5 Hz, respectively, for an HCCH fragment. It fol-
Brief illustration 12B.3
lows that the measurement of 3JHH in a series of related com-
pounds can be used to determine their conformations. The There can be a direct dipolar interaction between nuclei in
coupling constant 1JCH also depends on the hybridization of the solids, where the molecules do not rotate. The z-component of
C atom, as the following values indicate: the magnetic field arising from a 1H nucleus with mI   12 , at
R = 0.30 nm, and at an angle θ = 0 is3
sp sp2 sp3
1
JCH/Hz 250 160 125   N
   0
  (13 cos2  )mI
(2.821  10 JT )  (1.2566  106 T2 J 1 m3 )
26 1 
Bnuc   10 3
 (1)
4   (3.0  10 m)
Brief illustration 12B.2 
R3
4
 1.0  10 T  0.10 mT
The investigation of H–N–C–H couplings in polypeptides can
help reveal their conformation. For 3JHH coupling in such a
group, A = +5.1 Hz, B = −1.4 Hz, and C = +3.2 Hz. For a heli-
cal polymer, ϕ is close to 120°, which gives 3JHH ≈ 4 Hz. For the Spin–spin coupling in molecules in solution can be ex-
sheet-like conformation, ϕ is close to 180°, which gives 3JHH ≈ plained in terms of the polarization mechanism, in which
10 Hz. Experimental measurements of the value of 3JHH should the interaction is transmitted through the bonds. The simplest
therefore make it possible to distinguish between the two pos- case to consider is that of 1JXY, where X and Y are spin- 12 nuclei
sible structures. joined by an electron-pair bond. The coupling mechanism de-
pends on the fact that the energy depends on the relative ori-
entation of the bonding electrons and the nuclear spins. This
electron–nucleus coupling is magnetic in origin, and may be
Exercises
1
E12B.4 Make a sketch, roughly to scale, of the H NMR spectrum expected
for an AX spin system with δA = 1.00, δX = 2.00, and JAX = 10 Hz recorded on 3
As seen in the front matter of the book, μ0 = 1.2566 × 10−6 J s2 C−2 m−1. But
a spectrometer operating at (i) 250 MHz, (ii) 800 MHz. The horizontal scale 1 T = 1 J s C−1m−2, so it is also possible to write μ0 = 1.2566 × 10−6 T2 J−1 m3, with
should be in hertz, taking the resonance from TMS as the origin. these units proving more useful in this calculation.
12B Features of NMR spectra 515

either a dipolar interaction or a Fermi contact interaction. A found mainly at the far end of the bond because electrons tend
pictorial description of the latter is as follows. to stay apart to reduce their mutual repulsion. Because it is en-
First, regard the magnetic moment of the nucleus as arising ergetically favourable for the spin of Y to be antiparallel to an
from the circulation of a current in a tiny loop with a radius sim- electron spin, a Y nucleus with β spin has a lower energy than
ilar to that of the nucleus (Fig. 12B.16). Far from the nucleus the when it has α spin. The opposite is true when X is β, for now the
field generated by this loop is indistinguishable from the field α spin of Y has the lower energy. In other words, the antiparal-
generated by a point magnetic dipole. Close to the loop, how- lel arrangement of nuclear spins lies lower in energy than the
ever, the field differs from that of a point dipole. The magnetic parallel arrangement as a result of their magnetic coupling with
interaction between this non-dipolar field and the electron’s the bond electrons. That is, 1JCH is positive.
magnetic moment is the contact interaction. The contact inter- To account for the value of 2JXY, as for 2JHH in H–C–H, a
action—essentially the failure of the point-dipole approxima- mechanism is needed that can transmit the spin alignments
tion—depends on the very close approach of an electron to the through the central C atom (which may be 12C, with no nuclear
nucleus and hence can occur only if the electron occupies an s spin of its own). In this case (Fig. 12B.18), an X nucleus with
orbital (which is the reason why 1JCH depends on the hybridiza- α spin polarizes the electrons in its bond, and the α electron is
tion ratio). likely to be found closer to the C nucleus. The more favourable
Suppose that it is energetically favourable for an electron arrangement of two electrons on the same atom is with their
spin and a nuclear spin to be antiparallel (as is the case for a spins parallel (Hund’s rule, Topic 8B), so the more favourable
proton and an electron in a hydrogen atom). If the X nucleus arrangement is for the α electron of the neighbouring bond
is α, a β electron of the bonding pair will tend to be found to be close to the C nucleus. Consequently, the β electron of
nearby, because that is an energetically favourable arrangement that bond is more likely to be found close to the Y nucleus, and
(Fig. 12B.17). The second electron in the bond, which must have therefore that nucleus will have a lower energy if it is α. Hence,
α spin if the other is β (by the Pauli principle; Topic 8B), will be according to this mechanism, the lower energy will be obtained
if the Y spin is parallel to that of X. That is, 2JHH is negative.
The coupling of nuclear spin to electron spin by the Fermi
contact interaction is most important for proton spins, but it
is not necessarily the most important mechanism for other
nuclei. These nuclei may also interact by a dipolar mechanism
with the electron magnetic moments and with their orbital
motion, and there is no simple way of specifying whether J
will be positive or negative. The dipolar interaction does not
average to zero as the molecule tumbles if it accounts for the
interaction of both nuclei with their surrounding electrons be-
Figure 12B.16 The origin of the Fermi contact interaction. From cause then 1 − 3 cos2θ appears as its square and therefore with
far away, the magnetic field pattern arising from a ring of current a non-negative value at all orientations, and its average value is
(representing the rotating charge of the nucleus, the pale grey no longer zero.
sphere) is that of a point dipole. However, if an electron can
sample the field close to the region indicated by the sphere, the
field distribution differs significantly from that of a point dipole. Hund
For example, if the electron can penetrate the sphere, then the
spherical average of the field it experiences is not zero. C

Fermi Fermi Fermi Fermi


Pauli
Pauli

Pauli Y
X Y X

Pauli
X Y Figure 12B.18 The polarization mechanism for 2JHH spin–spin
coupling. The spin information is transmitted from one bond to
the next by a version of the mechanism that accounts for the
Figure 12B.17 The polarization mechanism for spin–spin coupling lower energy of electrons with parallel spins in different atomic
(1JCH). The two arrangements have slightly different energies. In orbitals (Hund’s rule of maximum multiplicity). In this case,
this case, J is positive, corresponding to a lower energy when the J < 0, corresponding to a lower energy when the nuclear spins
nuclear spins are antiparallel. are parallel.
516 12 Magnetic resonance

12B.4(a) Equivalent nuclei Step 1 Identify the states and their energies in the absence of ­
spin–spin coupling
A group of identical nuclei are chemically equivalent if they
There are four energy levels; they can be classified according to
are related by a symmetry operation of the molecule and have their total spin angular momentum Itot (the analogue of S for
the same chemical shifts. Chemically equivalent nuclei are several electrons) and its projection on to the z-axis, given by
from atoms that would be regarded as ‘equivalent’ according to the quantum number MI. There are three states with Itot = 1, and
ordinary chemical criteria. Nuclei are magnetically equivalent one further state with Itot = 0:
if, as well as being chemically equivalent, they also have identi-
cal spin–spin interactions with any other magnetic nuclei in the Spins parallel, I tot  1: M I  1 
molecule. MI  0 (1/21/ 2 ){  }
M I  1 
Brief illustration 12B.4 Spins paired, I tot  0 : MI  0 (1/21/ 2 ){  }

The difference between chemical and magnetic equivalence is The effect of a magnetic field on these four states is shown on
illustrated by CH2F2 and H2C=CF2 (recall that 19F is a spin- 12 the left-hand side of Fig. 12B.19: the two states with MI = 0 are
nucleus). In each of these molecules the 1H nuclei (protons) unaffected by the field as they are composed of equal propor-
are chemically equivalent because they are related by sym- tions of α and β spins, and both spins have the same Larmor
metry. The protons in CH2F2 are magnetically equivalent, but frequency.
those in CH2=CF2 are not. One proton in the latter has a cis Step 2 Allow for spin–spin interaction
spin-coupling interaction with a given F nucleus whereas the
The scalar product in the expression E = (hJ/ħ2)I1·I2 can be
other proton has a trans interaction with the same nucleus. In
expressed in terms of the total nuclear spin Itot = I1 + I2 by not-
contrast, in CH2F2 each proton has the same coupling to both
ing that
fluorine nuclei since the bonding pathway between them is
the same. 2
I tot  (I1  I 2 )  (I1  I 2 )  I12  I 22  2 I1  I 2

Rearranging this expression to


Strictly speaking, in a molecule such as CH3CH2Cl the three I1  I 2  12 (I tot
2
 I12  I 22 )
CH3 protons are magnetically inequivalent: each one may have
a different coupling to the CH2 protons on account of the dif- and replacing the square magnitudes by their quantum-
ferent dihedral angles between the protons. However, the three mechanical values gives:
CH3 protons are in practice made magnetically equivalent by
the rapid rotation of the CH3 group, which averages out any I1  I 2  12 {I tot (I tot  1)  I1 (I1  1)  I 2 ( I 2  1)}2
differences. The spectra of molecules with chemically equiva-
lent but magnetically inequivalent sets of spins can become
very complicated. For example, the proton and 19F spectra of  +¼hJ
I tot = 1, MI = +1
H2C=CF2 each consist of 12 lines. Such spectra are not consid- No With
ered further. spin–spin spin–spin
An important feature of chemically equivalent magnetic nu- coupling coupling
clei is that, although they do couple together, the coupling has  +  I tot = 1, MI = 0
+¼hJ
no effect on the appearance of the spectrum. How this comes  – 
about can be illustrated by considering the case of an A2 spin
system. The first step is to establish the energy levels. –3/4hJ
I tot = 0, MI = 0
I tot = 1, MI = –1
 +¼hJ

How is that done? 12B.1 Deriving the energy levels of an Figure 12B.19 The energy levels of an A2 spin system in the
A2 system absence of spin–spin coupling are shown on the left. When spin–
spin coupling is taken into account, the energy levels on the right
Consider an A2 system of two spin- 12 nuclei, and consider first are obtained. Note that the effect of spin–spin coupling is to raise
the energy levels in the absence of spin–spin coupling. When the three states with total nuclear spin Itot = 1 (the triplet) by the
considering spin–spin coupling, be prepared to use the com- same amount (J is positive); in contrast, the one state with Itot = 0
plete expression for the energy (the one proportional to I1·I2), (the singlet) is lowered in energy. The only allowed transitions,
because the Larmor frequencies are the same and the approxi- indicated by arrows, are those for which ΔItot = 0 and ΔMI = ±1.
mate form (I1zI2z) cannot be used as it is applicable only when These two transitions occur at the same resonance frequency as
the Larmor frequencies are very different. they would have in the absence of spin–spin coupling.
12B Features of NMR spectra 517

Then, because I=1 I=2 1


2
, it follows that

E  12 hJ I tot (I tot  1)  23 
(d)  °Δδ << J
For parallel spins, Itot = 1 and E   14 hJ ; for antiparallel spins
Itot = 0 and E   34 hJ , as shown on the right-hand side of
Fig. 12B.19. (c)  °Δδ = 2J

(b) °Δδ = 20J


The calculation shows that the three states with Itot = 1 all
move in energy in the same direction and by the same amount.
The single state with Itot = 0 moves three times as much in the
(a)  °Δδ >> J
opposite direction. In the resonance transition, the relative ori-
entation of the nuclei cannot change, so there are no transitions Figure 12B.20 The NMR spectra of (a) an AX system and (d) a
between states of different Itot. The selection rule ∆MI = ±1 also ‘nearly A2’ system are simple ‘first-order’ spectra (for an actual A2
applies, and arises from the conservation of angular momentum system, Δδ = 0). At intermediate relative values of the chemical
and the unit spin of the photon. As shown in Fig. 12B.19, there shift difference and the spin–spin coupling (b and c), more
are only two allowed transitions and because they have the same complex ‘strongly coupled’ spectra are obtained. Note how the
energy spacing they appear at the same frequency in the spec- inner two lines of the AX spectrum move together, grow in
intensity, and form the single central line of the A2 spectrum.
trum. Hence, the spin–spin coupling interaction does not affect
the appearance of the spectrum of an A2 molecule.
means that the spin system will always be weakly coupled, and
hence described as AX. If the two nuclei are of the same ele-
12B.4(b) Strongly coupled nuclei ment the spin system is described as homonuclear, whereas
The multiplets seen in NMR spectra due to the presence of if they are of different elements the system is described as
spin–spin coupling are relatively simple to analyse provided the ­heteronuclear.
difference in chemical shifts between any two coupled spins is
much greater than the value of the spin–spin coupling constant
Exercises
between them. This limit is often described as weak coupling,
1
and the resulting spectra are described as first-order spectra. E12B.7 Classify the H nuclei in 1-chloro-4-bromobenzene into chemically or
magnetically equivalent groups. Give your reasoning.
When the difference in chemical shifts is comparable to the
19
value of the spin–spin coupling constant, the multiplets take on E12B.8 Classify the F nuclei in PF5 into chemically or magnetically equivalent
groups. Give your reasoning.
a more complex form. Such spin systems are said to be strongly
coupled, and the spectra are described as second-order. In
such spectra the lines shift from where they are expected in the
weak coupling case, their intensities change, and in some cases 12B.5 Exchange processes
additional lines appear. Strongly coupled spectra are more dif-
ficult to analyse in the sense that the relation between the fre- The appearance of an NMR spectrum is changed if magnetic
quencies of the lines in the spectrum and the values of chemical nuclei can jump rapidly between different environments. For
shifts and coupling constants is not as straightforward as in the example, consider the molecule N,N-dimethylmethanamide,
weakly coupled case. HCON(CH3)2, in which the O–C–N fragment is planar, and
Figure 12B.20 shows NMR spectra for two coupled spins as there is restricted rotation about the C–N bond. The lowest en-
a function of the difference in chemical shift between the two ergy conformation is shown in Fig. 12B.21. In this conforma-
spins. In Fig. 12B.20a (an AX species) this difference is large tion the two methyl groups are not equivalent because one is
enough for the weak coupling limit to apply and two doublets cis and the other is trans to the carbonyl group. The two groups
are detected, with all lines having the same intensity. As the therefore have different environments and hence different
shift difference decreases the inner two lines gain intensity at chemical shifts.
the expense of the outer lines, and in the limit that the shift dif- Rotation by 180° about the C–N bond gives the same con-
ference is zero (an A2 species), the outer lines disappear and the formation, but it exchanges the CH3 groups between the two
inner lines converge. environments. When the jumping rate of this process is low,
If the two nuclei belong to different elements (e.g. 1H and the spectrum shows a distinct line for each CH3 environment.
13
C), or different isotopes of the same element (e.g. 1H and 2H), When the rate is fast, the spectrum shows a single line at the
the fact that they have widely different Larmor f­requencies mean of the two chemical shifts. At intermediate rates, the lines
518 12 Magnetic resonance

O
H
C CH3CH2OH
CH3CH2OH
N CH3CH2OH

Figure 12B.21 In this molecule the two methyl groups are in


different environments and so will have different chemical shifts.
Rotation about the C–N bond interchanges the two groups, so 4.0 3.0 δ 2.0 1.0
that a particular methyl group is swapped between environments.
Figure 12B.22 The 1H (proton) NMR spectrum of ethanol. The
arrows indicate the protons giving rise to each multiplet. Due to
start to broaden and eventually coalesce into a single broad
chemical exchange between the OH proton and water molecules
line. Coalescence of the two lines occurs when
present in the solvent, no splittings due to coupling to the OH
21/ 2 proton are seen.
 Condition for coalescence of two NMR lines (12B.16)

where τ is the lifetime of an environment and  is the differ- ­ ecause the protons provided by the water molecules in succes-
b
ence between the Larmor frequencies of the two environments. sive exchanges have random spin orientations.
If the rate constant for the exchange process is fast compared
to the value of the coupling constant J, in the sense 1/τ >> J,
Brief illustration 12B.5
the two lines merge and no splitting is seen. Because the values
The NO group in N,N-dimethylnitrosamine, (CH3)2N–NO of coupling constants are typically just a few hertz, even rather
(5), rotates about the N–N bond and, as a result, the magnetic slow exchange leads to the loss of the splitting. In the case of
environments of the two CH3 groups are interchanged. The OH groups, only by rigorously excluding water from the sol-
two CH3 resonances are separated by 390 Hz in a 600 MHz vent can the exchange rate be made slow enough that splittings
spectrometer. According to eqn 12B.16, due to coupling to OH protons are observed.
21/ 2
  1.2 ms
  (390 s 1 ) Exercise
E12B.9 A proton jumps between two sites with δ = 2.7 and δ = 4.8. What
rate constant for the interconversion process is needed for the two signals to
collapse to a single line in a spectrometer operating at 550 MHz?

5 N,N-Dimethylnitrosamine
12B.6 Solid-state NMR
Such a lifetime corresponds to a (first-order) rate constant of In contrast to the narrow lines seen in the NMR spectra of
1/τ = 870 s−1. It follows that the signal will collapse to a single samples in solution, the spectra from solid samples give broad
line when the rate constant for interconversion exceeds this lines, often to the extent that chemical shifts are not resolved.
value. Nevertheless, there are good reasons for seeking to overcome
these difficulties. They include the possibility that a compound
is unstable in solution or that it is insoluble. Moreover, many
A similar explanation accounts for the loss of fine structure species, such as polymers (both synthetic and naturally occur-
for protons in hydrogen atoms that can exchange with the sol- ring), are intrinsically interesting as solids and might not be
vent. For example, the resonance from the OH group in the open to study by X-ray diffraction: in these cases, solid-state
spectrum of ethanol appears as a single line (Fig. 12B.22). In NMR provides a useful alternative way of probing both struc-
this molecule the hydroxyl protons are able to exchange with ture and dynamics.
the protons in water which, unless special precautions are There are three principal contributions to the linewidths of
taken, is inevitably present as an impurity in the (organic) sol- solids. One is the direct magnetic dipolar interaction between
vent. When this chemical exchange, an exchange of atoms, oc- nuclear spins. As pointed out in the discussion of spin–spin
curs, a molecule ROH with an α-spin proton (written as ROHα) coupling, a nuclear magnetic moment gives rise to a local
rapidly converts to ROHβ and then perhaps to ROHα again ­magnetic field which points in different directions at ­different
12B Features of NMR spectra 519

locations around the nucleus. If the only component of inter-


est is parallel to the direction of the applied magnetic field
(because only this component has a significant effect), then
provided certain subtle effects arising from transformation
54.74°
from the static to the rotating frame are neglected, the classical
expression in The chemist’s toolkit 12B.1 can be used to write the
magnitude of the local magnetic field as
 N 0 mI
Bloc   (1  3 cos2  ) (12B.17a)

Magnetic field
4 R 3
Unlike in solution, in a solid this field is not averaged to zero
by the molecular motion. Many nuclei may contribute to the
total local field experienced by a nucleus of interest, and dif-
ferent nuclei in a sample may experience a wide range of fields. Figure 12B.23 In magic-angle spinning, the sample spins on
Typical dipole fields are of the order of 1 mT, which corresponds an axis at 54.74° (i.e. arccos 1/31/2) to the applied magnetic field.
to splittings and linewidths of the order of 10 kHz for 1H. When Rapid motion at this angle averages dipole–dipole interactions
the angle θ can vary only between 0 and θmax, the average value and chemical shift anisotropies to zero.
of 1 − 3 cos2θ can be shown to be −(cos2 θmax + cos θmax). This
result, in conjunction with eqn 12B.17a, gives the average local
field as The third contribution is the electric quadrupole interaction.
Nuclei with I > 12 have an ‘electric quadrupole moment’, a meas-
 N 0 mI ure of the extent to which the distribution of charge over the
Bloc,av  (cos2  max  cos  max ) (12B.17b)
4 R 3 nucleus is not uniform (for instance, the positive charge may
be concentrated around the equator or at the poles). An elec-
Brief illustration 12B.6 tric quadrupole interacts with an electric field gradient, such as
may arise from a non-spherical distribution of charge around
When θmax = 30° and R = 160 pm, the local field generated by the nucleus. This interaction depends on the orientation of the
a proton is molecule relative to the magnetic field, as in both the dipolar
  N 0
 
2
mI cos max  cos max
 interaction and chemical shift anisotropy.
32 3
(3.546 10 Tm )  ( 12 )  (1.616) Fortunately, there are techniques available for reducing
Bloc,av  the linewidths of solid samples. One technique, magic-angle
4   (1.60  1010 m)3
 spinning (MAS), takes note of the 1 − 3 cos2θ dependence of
R3
the dipole–dipole interaction, the chemical shift anisotropy,
 5.57  104 T  0.557 mT
and the electric quadrupole interaction. The ‘magic angle’ is
the angle at which 1 − 3 cos2θ = 0, and corresponds to 54.74°.
A second source of linewidth is the anisotropy of the chemi- In the technique, the sample is spun at high speed around an
cal shift. Chemical shifts arise from the ability of the applied axis at the magic angle to the applied field (Fig. 12B.23). All the
field to generate electron currents in molecules. In general, this dipolar interactions and the anisotropies average to the value
ability depends on the orientation of the molecule relative to they would have at the magic angle, but at that angle they are
the applied field. In solution, when the molecule is tumbling zero. In principle, MAS therefore removes completely the line-
rapidly, only the average value of the chemical shift is relevant. broadening due to dipole–dipole interactions and chemical
However, the anisotropy is not averaged to zero for stationary shift anisotropy. The difficulty with MAS is that the spinning
molecules in a solid, and molecules in different orientations frequency must not be less than the width of the spectrum,
have resonances at different frequencies. The chemical shift which is of the order of kilohertz. However, gas-driven sample
anisotropy also varies with the angle θ between the applied field spinners that can be rotated at up to 50 kHz are now routinely
and the principal axis of the molecule as 1 − 3 cos2θ. available.

Checklist of concepts
☐ 1. The chemical shift of a nucleus is the difference ☐ 2. The shielding constant is the sum of a local contribu-
between its resonance frequency and that of a reference tion, a neighbouring group contribution, and a solvent
standard. contribution.
520 12 Magnetic resonance

☐ 3. The local contribution is the sum of a diamagnetic con- ☐ 10. If the shift difference between two nuclei is large com-
tribution and a paramagnetic contribution. pared to the coupling constant between the nuclei the
☐ 4. The neighbouring group contribution arises from the spin system is said to be weakly coupled; if the shift
currents induced in nearby groups of atoms. difference is small compared to the coupling, the spin
system is strongly coupled.
☐ 5. The solvent contribution can arise from specific molec-
ular interactions between the solute and the solvent. ☐ 11. Chemically equivalent nuclei have the same chemi-
cal shifts; the same is true of magnetically equivalent
☐ 6. Fine structure is the splitting of resonances into indi-
nuclei, but in addition the coupling constant to any
vidual lines by spin–spin coupling; these splittings give
other nucleus is the same for each of the equivalent
rise to multiplets.
nuclei.
☐ 7. Spin–spin coupling is expressed in terms of the spin–
☐ 12. Coalescence of two NMR lines occurs when nuclei are
spin coupling constant J; coupling leads to the splitting
exchanged rapidly between environments by either con-
of lines in the spectrum.
formational or chemical process.
☐ 8. The coupling constant decreases as the number of bonds
☐ 13. Magic-angle spinning (MAS) is a technique in which
separating two nuclei increases.
the NMR linewidths in a solid sample are reduced by
☐ 9. Spin–spin coupling can be explained in terms of spinning at an angle of 54.74° to the applied magnetic
the polarization mechanism and the Fermi contact field.
­interaction.

Checklist of equations
Property Equation Comment Equation number
δ-Scale of chemical shifts δ = {(ν − ν°)/ν°} × 106 Definition 12B.4a
6
Relation between chemical shift and shielding constant δ ≈ (σ° − σ) × 10 12B.6
Local contribution to the shielding constant σ(local) = σd + σp 12B.8
2
Lamb formula σd = (e μ0/12πme)〈1/r〉 Applies to atoms 12B.9
Neighbouring group contribution to the shielding σ(neighbour) ∝ (χ‖ − χ⊥){(1 − 12B.10b
constant 3 cos2 Θ)/R3}
3
Karplus equation JHH = A + B cos ϕ + C cos 2ϕ A, B, and C are empirical constants 12B.14
1/2
Condition for coalescence of two NMR lines τ = 2 /πδν τ is the lifetime of the exchange 12B.16
process
TOPIC 12C Pulse techniques in NMR

12C.1 The magnetization vector


➤ Why do you need to know this material?
To appreciate the power and scope of modern nuclear To understand the pulse procedure, consider a sample com-
magnetic resonance techniques you need to understand posed of many identical spin- 12 nuclei. According to the vector
how radiofrequency pulses can be used to obtain spectra. model of angular momentum (Topic 7F), a nuclear spin can be
represented by a vector of length {I ( I +1)}1/ 2 with a component
➤ What is the key idea? of length mI along the z-axis. As the three components of the
Sequences of pulses of radiofrequency radiation manipu- angular momentum are complementary variables, the x- and
late nuclear spins, leading to efficient acquisition of NMR y-components cannot be specified if the z-component is known,
spectra and the measurement of relaxation times. so the vector lies anywhere on a cone around the z-axis. For I = 12 ,
the length of the vector is 31/2/2 and when mI   12 it makes an
➤ What do you need to know already? angle of arccos  12 /(31/ 2 /2)  54.7 to the z-axis (Fig. 12C.1);
You need to be familiar with the general principles of when mI   12 the cone makes the same angle to the −z-axis.
magnetic resonance (Topics 12A and 12B), and the vector In the absence of a magnetic field, the sample consists
model of angular momentum (Topic 7F). The development of equal numbers of α and β nuclear spins with their vec-
makes use of the concept of precession at the Larmor fre- tors lying at random, stationary positions on their cones. The
quency (Topic 12A). ­magnetization, M, of the sample, its net nuclear magnetic mo-
ment, is zero (Fig. 12C.2a). There are two changes when a mag-
netic field of magnitude B0 is applied along the z-direction:
• The energies of the two spin states change, the α spins
moving to a lower energy and the β spins to a higher
In modern forms of NMR spectroscopy the nuclear spins are energy (provided γN > 0).
first excited by a short, intense burst of radiofrequency radia-
In the vector model, the two vectors are pictured as precessing
tion (a ‘pulse’), applied at or close to the Larmor frequency. As
at the Larmor frequency (Topic 12A,  L   N B0 /2). At 10 T,
a result of the excitation caused by the pulse, the spins emit
the Larmor frequency for 1H nuclei (commonly referred to as
radiation as they return to equilibrium. This time-dependent
‘protons’) is 427 MHz. As the strength of the field is increased,
signal is recorded and its ‘Fourier transform’ computed (as will
the Larmor frequency increases and the precession becomes
be described) to give the spectrum. The technique is known as
faster.
Fourier-transform NMR (FT-NMR). An analogy for the dif-
ference between conventional spectroscopy and pulsed NMR • The populations of the two spin states (the numbers of α
is the detection of the frequencies at which a bell vibrates. The and β spins) at thermal equilibrium change, with slightly
‘conventional’ option is to connect an audio oscillator to a loud- more α spins than β spins (Topic 12A).
speaker and direct the sound towards the bell. The frequency of
the sound source is then scanned until the bell starts to ring in
resonance. The ‘pulse’ analogy is to strike the bell with a ham- z
mer and then Fourier transform the signal to identify the reso-
nance frequencies of the bell.
One advantage of FT-NMR over conventional NMR is that ½
it improves the sensitivity. However, the real power of the tech- ½√3
nique comes from the possibility of manipulating the nuclear
spins by applying a sequence of several pulses. In this way it is
possible to record spectra in which particular features are em- Figure 12C.1 The vector model of angular momentum for a single
phasized, or from which other properties of the molecule can spin- 21 nucleus with mI   21 . The position of the vector on the
be determined. cone is indeterminate.
522 12 Magnetic resonance

M to a rotating frame, a platform that rotates around the z-axis


at the Larmor frequency: the B1 field is stationary in this frame
(Fig. 12C.3b).
In the laboratory frame, the applied field defines the axis
α
of quantization and the spins are either α or β with respect to
β that axis. In the rotating frame, the applied field has effectively
disappeared and the new axis of quantization is the direc-
tion of the stationary B1 field. The angular momentum states
(a) (b) are still confined to two values with components on that axis,
and will be denoted α′ and β′. The vectors that represent them
Figure 12C.2 The magnetization of a sample of spin- 21 nuclei is
precess around this new axis on cones at a Larmor frequency
the resultant of all their magnetic moments. (a) In the absence of
 L   N B1 /2. This frequency will be termed the ‘B1 Larmor fre-
an externally applied field, there are equal numbers of α and β
spins lying at random angles around the cones: the magnetization
quency’ to distinguish it from the ‘B0 Larmor frequency’ associ-
is zero. (b) In the presence of a field there are slightly more α spins ated with precession about B0.
than β spins. As a result, there is a net magnetization, represented For simplicity, suppose that there are only α spins in the sam-
by the vector M, along the z-axis. There is no magnetization in ple. In the rotating frame these vectors will seem to be bunched
the transverse plane (the xy-plane) because the spins still lie at up at the top of the α′ and β′ cones in the rotating frame
random angles around the cones. (Fig. 12C.4). They precess around B1 and therefore migrate to-
wards the xy-plane. Of course, there are β nuclei present too,
which in the rotating frame are bunched together at the bottom
This imbalance results in a net magnetization in the z-­direction. of the α′ and β′ cones, but precess similarly. At thermal equi-
It can be represented by a vector M lying along the z-axis with a librium there are fewer β spins than α spins, so the net effect
length proportional to the population difference (Fig. 12C.2b). is a magnetization vector initially along the z-axis that rotates
The manipulation of this net magnetization vector is the central around the B1 direction at the B1 Larmor frequency and into the
feature of pulse techniques. xy-plane.
When the radiofrequency field is applied in a pulse of du-
ration Δτ the magnetization rotates through an angle (in ra-
12C.1(a) The effect of the radiofrequency field dians) of     ( N B1 /2)  2; this angle is known as the
The magnetization vector can be rotated away from its equi- flip angle of the pulse. Therefore, to achieve a flip angle φ , the
librium position by applying radiofrequency radiation that duration of the pulse must be    / N B1. A 90° pulse (with
provides a magnetic field B1 lying in the xy-plane and rotating 90° corresponding to   /2) of duration  90  /2 N B1
at the Larmor frequency (as determined by B0, Fig. 12C.3a). rotates the magnetization from the z-axis into the xy-plane
To understand this process, it is best to imagine stepping on (Fig. 12C.5a).

M
M
M

L

 = L
(a) B1 (b) B1

Figure 12C.3 (a) In a pulsed NMR experiment the net Figure 12C.4 When attention switches to the rotating frame, the
magnetization is rotated away from the z-axis by applying vectors representing the spins are in states referring to the axis
radiofrequency radiation with its magnetic component B1 rotating defined by B1, and precess on their cones. A uniform distribution
in the xy-plane at the Larmor frequency. (b) When viewed in a of α spins in the B0 frame is actually a superposition of α′ and β′
frame also rotating about z at the Larmor frequency, B1 appears to states that seem to be bunched together on their cones. As the
be stationary. The magnetization rotates around the B1 field, thus latter precess on the horizontal cones, the magnetization vector
moving the magnetization away from the z-axis and generating rotates into the xy-plane. Vectors representing β spins in the
transverse components. original frame behave similarly.
12C Pulse techniques in NMR 523

90° pulse B0
Detecting
coil

Signal
M
L Time, t
B1
(a) (b)

Figure 12C.5 (a) If the radiofrequency field is applied for the


appropriate time, the magnetization vector is rotated into the
xy-plane; this is termed a 90° pulse. (b) Once the magnetization is Figure 12C.6 A free-induction decay of a sample of spins with a
in the transverse plane it rotates about the B0 field at the Larmor single resonance frequency.
frequency (when observed in a static frame). The magnetization
vector periodically rotates past a small coil, inducing in it an
oscillating current, which is the detected signal.
from the Larmor frequency is small compared to the inverse
Brief illustration 12C.1 of the duration of the 90° pulse. In practice, a spectrum with
several peaks, each with a slightly different Larmor frequency,
The duration of a radiofrequency pulse depends on the strength can be excited by selecting a radiofrequency somewhere in
of the B1 field. If a 90° pulse requires 10 μs, then for protons the centre of the spectrum and then making sure that the 90°
 pulse is sufficiently short. This requirement is met by using an
B1  intense radiofrequency field, so that B1 is still large enough to
2  (2.675  10 T s )  (1.0  105 s)
8 1 1
   achieve rotation through 90°.
N 
 5.9  104 T  0.59 mT
12C.1(b) Time- and frequency-domain signals
Immediately after a 90° pulse the B1 field is removed and the Each line in an NMR spectrum can be thought of as arising
magnetization lies in the xy-plane. Next, imagine stepping out from its own magnetization vector. Once that vector has been
of the rotating frame. The magnetization vector is now rotating rotated into the xy-plane it precesses at the frequency of the
in the xy-plane at the B0 Larmor frequency (Fig. 12C.5b). In an corresponding line. Each vector therefore contributes a decay-
NMR spectrometer a small coil is wrapped around the sample ing-oscillating term to the observed signal and the FID is the
and perpendicular to the B0 field in such a way that the precess- sum of many such contributions. If there is only one line it is
ing magnetization vector periodically ‘cuts’ the coil, thereby possible to determine its frequency simply by inspecting the
inducing in it a small current oscillating at the B0 Larmor fre- FID, but that is rarely possible when the signal is composite.
quency. This oscillating current is detected by a radiofrequency In such cases, Fourier transformation (The chemist’s toolkit
receiver. 12C.1), as mentioned in the introduction, is used to analyse
As time passes the magnetization returns to an equilibrium the signal.
state in which it has no transverse components; as it does so the The input to the Fourier transform is the decaying-­oscillating
oscillating signal induced in the coil decays to zero. This decay ‘time-domain’ function S(t), the FID. The output is the absorp-
is usually assumed to be exponential with a time constant de- tion spectrum, the ‘frequency-domain’ function, I(n), which is
noted T2. The overall form of the signal is therefore a decaying-­ obtained by computing the integral
oscillating free-induction decay (FID) like that shown in 
Fig. 12C.6 and of the form I ( )   S(t )cos(2 t )dt  (12C.2)
0

S(t )  S0 cos(2 L t )e t /T2 Free-induction decay  (12C.1) where I(n) is the intensity at the frequency n. The complete
­frequency-domain function, which is the spectrum, is built up
So far it has been assumed that the radiofrequency radiation by evaluating this integral over a range of frequencies.
is exactly at the B0 Larmor frequency. However, virtually the If the time-domain contains a single decaying-oscillating
same effect is obtained if the separation of the radiofrequency term, as in eqn 12C.1, the Fourier transform (see the extended
524 12 Magnetic resonance

The chemist’s toolkit 12C.1 The Fourier transform


A Fourier transform expresses any waveform as a superposition Sketch 1 shows a fast and slow decay and the corresponding fre-
of harmonic (sine and cosine) waves. If the waveform is the real quency contributions: note that a slow decay has predominantly
function S(t), then the contribution I(n) of the oscillating func- low-frequency contributions and a fast decay has contributions
tion cos(2πnt) is given by the ‘cosine transform’ at higher frequencies.

If an experimental procedure results in the function I(ν) itself,
I ( )   S(t )cos(2 t )dt  (1) then the corresponding signal can be reconstructed by forming
0
the inverse Fourier transform:
There is an analogous transform appropriate for complex func-
tions: see the additional information for this Toolkit in the 2 
 0
S(t )  I ( )cos(2 t )d  (3)
enriched collection of The chemist’s toolkits. If the signal varies
slowly, then the greatest contribution comes from low-frequency Fourier transforms are applicable to spatial functions too. Their
waves; rapidly changing features in the signal are reproduced by interpretation is similar but it is more appropriate to think in
high-frequency contributions. If the signal is a simple exponen- terms of the wavelengths of the contributing waves. Thus, if the
tial decay of the form S(t) = S0e−t/τ, where τ is the time constant of function varies only slowly with distance, then its Fourier trans-
the decay, the contribution of the wave of frequency n is form has mainly long-wavelength contributions. If the features
 S0 vary quickly with distance (as in the electron density in a crys-
I ( )  S0  e t / cos(2 t )dt   (2) tal), then short-wavelength contributions feature.
0 1  (2 )2

1 3
Fourier transform
0.8 2.5
Inverse Fourier transform
2
0.6 I()/S0 τ=3s
τ=3s
S(t) 1.5
0.4
τ=1s 1
τ=1s
0.2
0.5

0 0
0 1 2 3 4 5 6 0 0.5 1 1.5 2 2.5 3
t/s 2π/s–1

Sketch 1

The chemist’s toolkit 12C.1 in the enriched collection of The


chemist’s toolkits) is 1

S0T2
I ( )   (12C.3) 0.8
1  ( L  )2 (2T2 )2
Signal height, I()/S0T2

0.6
The graph of this expression has a so-called ‘Lorentzian’ shape, 1/2 = 1/T2
a symmetrical peak centred at    L and with height S0T2; its
0.4
width at half the peak height is 1/πT2 (Fig. 12C.7). If the FID
consists of a sum of decaying-oscillating functions, Fourier
0.2
transformation gives a spectrum consisting of a series of peaks
at the various frequencies.
0
In practice, the FID is sampled digitally and the integral of 0
eqn 12C.2 is evaluated numerically by a computer in the NMR Frequency offset, – L

spectrometer. Figure 12C.8 shows the time- and frequency-­ Figure 12C.7 A Lorentzian absorption line. The width at half-
domain functions for three different FIDs of increasing height depends on the time constant T2 which characterizes the
­complexity. decay of the time-domain signal.
12C Pulse techniques in NMR 525

process follows an exponential recovery curve, with a time con-


stant called the longitudinal relaxation time, T1. Because lon-
Signal

gitudinal relaxation involves the transfer of energy between the


Time, t spins and the surroundings (the ‘lattice’), the time constant T1 is
also called the spin–lattice relaxation time. If the z-­component
(a)
Frequency,  of magnetization at time t is Mz(t), then the recovery to the
equilibrium magnetization M0 takes the form

Longitudinal relaxation time


M z (t )  M 0  e t /T1  (12C.4)
Signal

[definition]
Time, t
Immediately after a 90° pulse the fact that the magnetiza-
tion vector lies in the xy-plane implies that the α and β spins
(b) Frequency,  are aligned in a particular way on their cones so as to give a
net (and rotating) component of magnetization in the xy-plane.
At thermal equilibrium, however, the spins are at random an-
gles on their cones and there is no transverse component of
Signal

magnetization. The return of the transverse magnetization to


Time, t
its equilibrium value of zero is termed transverse relaxation.
It is usually assumed that this process is an exponential decay
(c) Frequency,  with a time constant called the transverse relaxation time, T2
Figure 12C.8 Free-induction decays (the time domain) and the (or spin–spin relaxation time). If the transverse magnetization
corresponding spectra (the frequency domain) obtained by at time t is Mxy(t), then the decay takes the form
Fourier transformation. (a) An uncoupled A resonance, (b) the A
Transverse relaxation time
resonance of an AX system, (c) the A and X resonances of an A2X3 M xy (t )  e t /T2 [definition]
 (12C.5)
system. Interact with the dynamic version of this graph in the e-book.
This T2 is the same as that describing the decay of the free-in-
duction signal which is proportional to Mxy(t).
Exercise
E12C.1 The duration of a 90° or 180° pulse depends on the strength of the B1 12C.2(a) The mechanism of relaxation
field. If a 180° pulse applied to 1H requires 12.5 μs, what is the strength of the
B1 field? How long would the corresponding 90° pulse require? The return of the z-component of magnetization to its equilib-
rium value involves transitions between α and β spin states so
as to achieve the populations required by the Boltzmann dis-
12C.2 Spin relaxation tribution. These transitions are caused by local magnetic fields
that fluctuate at a frequency close to the resonance frequency
Relaxation is the process by which the magnetization returns of the β ↔ α transition. The local fields can have a variety of
to its equilibrium value, at which point it is entirely along the origins, but commonly arise from nearby magnetic nuclei or
z-axis, with no x- and no y-component (no transverse compo- unpaired electrons. These fields fluctuate due to the tumbling
nents). In terms of the behaviour of individual spins, the ap- motion of molecules in a fluid sample. If molecular tumbling is
proach to equilibrium involves transitions between the two too slow or too fast compared with the resonance frequency, it
spin states in order to establish the thermal equilibrium popu- gives rise to a fluctuating magnetic field with a frequency that
lations of α and β. The attainment of equilibrium also requires is either too low or too high to stimulate a transition between β
the magnetic moments of individual nuclei becoming distrib- and α, so T1 is long. Only if the molecule tumbles at about the
uted at random angles on their two cones. resonance frequency is the fluctuating magnetic field able to in-
As already explained, after a 90° pulse the magnetization duce spin flips effectively, and then T1 is short.
vector lies in the xy-plane. This orientation implies that, from The rate of molecular tumbling increases with temperature
the viewpoint of the laboratory quantization axis, there are now and with reducing viscosity of the solvent, so a dependence of
equal numbers of α and β spins because otherwise there would the relaxation time like that shown in Fig. 12C.9 can be expected.
be a component of the magnetization in the z-direction. At The quantitative treatment of relaxation times depends on set-
thermal equilibrium, however, there is a Boltzmann distribu- ting up models of molecular motion and using, for instance, the
tion of spins, with more α spins than β spins (provided γN > 0) diffusion equation (Topic 16C) adapted for rotational motion.
and a non-zero z-component of magnetization. The return of Transverse relaxation is the result of the individual mag-
the z-component of magnetization to its equilibrium value is netic moments of the spins losing their relative alignment
termed longitudinal relaxation. It is usually assumed that this as they spread out on their cones. One contribution to this
526 12 Magnetic resonance

not perfectly uniform and is different at different locations in


the sample. The inhomogeneity results in a ­inhomogeneous
T1
broadening of the resonance. It is common to express the ex-
tent of inhomogeneous broadening in terms of an effective
Relaxation time

transverse relaxation time, T2 , by using a relation like eqn


12C.5, but writing
T2 1 Effective transverse relaxation time
T2  [definition]
 (12C.6)
Low High  1/ 2
temperature, temperature,
high viscosity low viscosity
where Δν1/2 is the observed width at half-height of the line
Rate of motion (which is assumed to be Lorentzian). In practice an inhomoge-
Figure 12C.9 The variation of the two relaxation times with the neously broadened line is unlikely to be Lorentzian, so the as-
rate at which the molecules tumble in solution. The horizontal axis sumption that the decay is exponential and characterized by a
can be interpreted as representing temperature or viscosity. Note time constant T2 is an approximation. The observed linewidth
that the two relaxation times coincide when the motion is fast. has a contribution from both the inhomogeneous broadening
and the transverse relaxation. The latter is usually referred to
as homogeneous broadening. Which contributions dominate
r­ andomization is any process that involves a transition between depends on the molecular system being studied and the quality
the two spin states. That is, any process that causes longitudinal of the magnet.
relaxation also contributes to transverse relaxation. Another
contribution is the variation in the local magnetic fields expe-
rienced by the nuclei. When the fluctuations in these fields are 12C.2(b) The measurement of T1 and T2
slow, each molecule lingers in its local magnetic environment, The longitudinal relaxation time T1 can be measured by the
and because the precessional rates depend on the strength of inversion recovery technique. The first step is to apply a 180°
the field, the spin orientations randomize quickly around their pulse to the sample by applying the B1 field for twice as long
cones. In other words, slow molecular motion corresponds to as for a 90° pulse. As a result of the pulse, the magnetization
short T2. If the molecules move rapidly from one magnetic en- vector is rotated into the −z-direction (Fig. 12C.10a). The effect
vironment to another, the effects of differences in local mag- of the pulse is to invert the population of the two levels and to
netic field average to zero: individual spins do not precess at result in more β spins than α spins.
very different rates, remain bunched for longer, and transverse Immediately after the 180° pulse no signal can be detected
relaxation does not take place as quickly. This fast motion cor- because the magnetization has no transverse component. The
responds to long T2 (as shown in Fig. 12C.9). Calculations show β spins immediately begin to relax back into α spins, and the
that, when the motion is fast, transverse and longitudinal re- magnetization vector first shrinks towards zero and then in-
laxation have similar time constants. creases in the opposite direction until it reaches its thermal
equilibrium value. Before that has happened, after an inter-
Brief illustration 12C.2 val τ, a 90° pulse is applied. That pulse rotates the remaining
z-component of magnetization into the xy-plane, where it gen-
For a small molecule dissolved in a non-viscous solvent the erates an FID signal. The frequency-domain spectrum is then
value of T2 for 1H can be as long as several seconds. The width obtained by Fourier transformation in the usual way.
(measured at half the peak height) of the corresponding line in
the spectrum is 1/πT2. For T2 = 3.0 s the width is
180° pulse
1 1
 1/ 2    0.11 Hz Relaxation
T2   (3.0 s) M
Signal

90° pulse
In contrast, for a larger molecule such as a protein dissolved in
water T2 is much shorter, and a value of 30 ms is not unusual. Time
The corresponding linewidth is 11 Hz. Interval, τ Interval, τ

(a) (b)
So far, it has been assumed that the applied magnetic field is Figure 12C.10 (a) The result of applying a 180° pulse to
homogeneous (uniform) in the sense of having the same value the magnetization in the rotating frame, and the effect of a
across the sample so that the differences in Larmor frequencies subsequent 90° pulse. (b) The amplitude of the frequency-domain
arise solely from interactions within the sample. In practice, spectrum varies with the interval between the two pulses because
due to the limitations in the design of the magnet, the field is there has been time for longitudinal relaxation to occur.
12C Pulse techniques in NMR 527

The intensity of the resulting spectrum depends on the mag- frequency (the nominal Larmor frequency). Because the rotat-
nitude of the magnetization vector that has been rotated into ing frame is at the Larmor frequency, the ‘fast’ and ‘slow’ vectors
the xy-plane. That magnitude changes exponentially with a rotate in opposite senses when viewed in this frame.
time constant T1 as the interval τ is increased, so the intensity of First, suppose that there is no transverse relaxation but that
the spectrum also changes exponentially with increasing τ. The the field is inhomogeneous. After an evolution period τ, a 180°
longitudinal relaxation time can therefore be measured by fit- pulse is applied along the y-axis of the rotating frame. The pulse
ting an exponential curve to the series of spectra obtained with rotates the magnetization vectors around that axis into mirror-
different values of τ. image positions with respect to the yz-plane. Once there, the
The measurement of T2 (as distinct from T2 ) depends on packets continue to move in the same direction as they did be-
being able to eliminate the effects of inhomogeneous broaden- fore, and so migrate back towards the y-axis. After an interval
ing. The cunning required is at the root of some of the most τ all the packets are again aligned along the axis. The result-
important advances made in NMR since its introduction. ant signal grows in magnitude, reaching a maximum, the ‘spin
A spin echo is the magnetic analogue of an audible echo. The echo’, at the end of the second period τ. The fanning out caused
sequence of events is shown in Fig. 12C.11. The overall mag- by the field inhomogeneity is said to have been ‘refocused’.
netization can be regarded as made up of a number of differ- The important feature of the technique is that the size of the
ent magnetizations, each of which arises from a spin packet of echo is independent of any local fields that remain constant
nuclei with very similar precession frequencies. The spread in during the two τ intervals. If a spin packet is ‘fast’ because it
these frequencies arises from the inhomogeneity of B0 (which happens to be composed of spins in a region of the sample
is responsible for inhomogeneous broadening), so different that experience a higher than average field, then it remains fast
parts of the sample experience different fields. The precession throughout both intervals, and so the angle through which it
frequencies also differ if there is more than one chemical shift rotates is the same in the two intervals. Hence, the size of the
present. echo is independent of inhomogeneities in the magnetic field,
First, a 90° pulse is applied to the sample. The subsequent because these remain constant.
events are best followed in a rotating frame in which B1 is sta- Now consider the consequences of transverse relaxation.
tionary along the x-axis and causes the magnetization to rotate This relaxation arises from fields that vary on a molecular scale,
on to the y-axis of the xy-plane. Immediately after the pulse, the and there is no guarantee that an individual ‘fast’ spin will re-
spin packets begin to fan out because they have different Larmor main ‘fast’ in the refocusing phase: the spins within the packets
frequencies. In Fig. 12C.11 the magnetization vectors of two therefore spread with a time constant T2. Consequently, the ef-
representative packets are shown and are described as ‘fast’ and fects of the relaxation are not refocused, and the size of the echo
‘slow’, indicating their frequency relative to the rotating frame decays with the time constant T2. The intensity of the signal
after a spin echo is measured for a series of values of the delay τ,
and the resulting data analysed to determine T2.
z
90° pulse
M
Exercises
E12C.2 What is the effective transverse relaxation time when the width of a
Lorentzian resonance line is 1.5 Hz?

y y E12C.3 The envelope of a free-induction decay is observed to decrease to half


x x
its initial amplitude in 1.0 s. What is the value of the transverse relaxation
Slow
τ time, T2?
180° pulse
spins
Fast
spins

12C.3 Spin decoupling


Fast Refocused
spins magnetization Carbon-13 has a natural abundance of only 1.1 per cent and
Slow
spins τ is therefore described as a dilute-spin species. The probabil-
ity that any one molecule contains more than one 13C nucleus
is rather low, and so the possibility of observing the effects of
13
Figure 12C.11 The action of the spin echo pulse sequence 90°–τ– C–13C spin–spin coupling can be ignored. In contrast the
180°–τ, viewed in a rotating frame at the Larmor frequency. Note abundance of the isotope 1H is very close to 100 per cent and
that the 90° pulse is applied about the x-axis, but the 180° pulse is is therefore described as an abundant-spin species. All the hy-
applied about the y-axis. ‘Slow’ and ‘Fast’ refer to the speed of the drogen nuclei in a molecule can be assumed to be 1H and the
spin packet relative to the rotating frame frequency. effects of coupling between them is observed.
528 12 Magnetic resonance

Dilute-spin species are observed in NMR spectroscopy and


−ΔN
show coupling to abundant-spin species present in the mole- βAβX
cule. Generally speaking, 13C-NMR spectra are very complex
on account of the spin couplings of each 13C nucleus with the A X
numerous 1H nuclei in the molecule. However, a dramatic sim-

Energy
0 αAβX βAαX 0
plification of the spectrum can be obtained by the use of proton
decoupling. In this technique radiofrequency radiation is ap-
X A
plied at (or close to) the 1H Larmor frequency while the 13C FID
is being observed. This stimulation of the 1H nuclei causes their αAαX
spins state to change rapidly, so averaging the 1H–13C couplings ΔN
to zero. As a result, each 13C nucleus gives a single line rather Figure 12C.12 The energy levels of an AX system and an
than a complex multiplet. Not only is the spectrum simplified, indication of their relative populations. Each filled square above
but the sensitivity is also improved as all the intensity is con- the line represents an excess population above the average, and
centrated into a single line. each empty square below the line represents a lower population
than the average. The labels (±∆N) show the deviations in
population from their average value.
Exercise
13
E12C.4 The C NMR spectrum of ethanoic acid (acetic acid) shows a quartet
centred at δ = 21 with a splitting of 130 Hz. When the same spectrum is
recorded using proton decoupling, the multiplet collapses to a single line.
the purposes of this discussion it is sufficient to consider the
Another quartet, but with a much smaller spacing, is also seen centred deviations of the populations from the average value for all four
at δ = 178; this quartet collapses when decoupling is used. Explain these levels: the αAαX level has a greater population than the average,
observations. the βAβX level has a smaller population, and the other two lev-
els have a population equal to the average. The deviations from
the average are ΔN for αAαX, −ΔN for βAβX, and 0 for the other
12C.4 The nuclear Overhauser effect two levels. These population differences are represented in
Fig. 12C.12. The intensity of a transition reflects the difference
A common source of the local magnetic fields that are responsi- in the population of the two energy levels involved, and for all
ble for relaxation is the dipole–dipole interaction between two four transitions this population difference (lower − upper) is
magnetic nuclei (see The chemist’s toolkit 12B.1). In this interac- ΔN, implying that all four transitions have the same intensity.
tion the magnetic dipole of the first spin generates a magnetic The NOE experiment involves irradiating the two X spin
field that interacts with the magnetic dipole of the second spin. transitions (αAαX ↔ αAβX and βAαX ↔ βAβX) with a radiofre-
The strength of the interaction is proportional to 1/R3, where R quency field, but making sure that the field is sufficiently weak
is the distance between the two spins, and is also proportional that the two A spin transitions are not affected. When applied
to the product of the magnetogyric ratios of the spins. As a re- for a long time, this field saturates the X spin transitions in the
sult, the interaction is characterized as being short-range and sense that the populations of the two energy levels become
significant for nuclei with high magnetogyric ratios. In typical equal. In the case of the αAαX ↔ αAβX transition, the popula-
organic and biological molecules, which have many 1H nuclei, tions of the two levels become 12 ∆N, and for the other X spin
the local fields due to the dipole–dipole interaction are likely to transition  12 N , as shown in Fig. 12C.13a. These changes in
be the dominant source of relaxation. population do not affect the population differences across the
The nuclear Overhauser effect (NOE) makes use of the re- A spin transitions (αAαX ↔ βAαX and αAβX ↔ βAβX) which re-
laxation caused by the dipole–dipole interaction of nuclear main at ΔN, therefore the intensity of the A spin transitions is
spins. In this effect, irradiation of one spin leads to a change in unaffected.
the intensity of the resonance from a second spin provided the Now consider the effect of spin relaxation. One source of
two spins are involved in mutual dipole–dipole relaxation. The relaxation is dipole–dipole interaction between the two spins.
dipole–dipole interaction has only a short range, so the obser- The hamiltonian for this interaction is proportional to the
vation of an NOE is indicative of the closeness of the two nuclei spin operators of the two nuclei and contains terms that can
involved and can be interpreted in terms of the structure of the flip both spins simultaneously and convert αAαX into βAβX.
molecule. This double-flipping process tends to restore the populations
To understand the effect, consider the populations of the four of these two levels to their equilibrium values of ΔN and −ΔN,
levels of a homonuclear AX spin system shown in Fig. 12C.12. respectively, as shown in Fig. 12C.13b. A consequence is that
At thermal equilibrium, the population of the αAαX level is the the population difference across each of the A spin transitions
greatest, and that of the βAβX level is the least; the other two lev- is now 23 ∆N, which is greater than it is at equilibrium. In sum-
els have the same energy and an intermediate population. For mary, the combination of saturating the X spin transitions and
12C Pulse techniques in NMR 529

−½ΔN −ΔN −½ΔN


βAβX βAβX
βAβX Reduced
Saturate Enhanced

A X A X A X

αAβX βAαX
Energy

½ΔN −½ΔN ½ΔN −½ΔN

Relax
αAβX βAαX αAβX βAαX 0 0
Relax
Reduced
Enhanced X A
X A X A ½ΔN
+½ΔN
Saturate αAαX
(a) αAαX (b) αAαX (c)
ΔN

Figure 12C.13 (a) When an X transition is saturated, the populations of the two states are equalized, leading to the populations shown
(using the same symbols as in Fig. 12C.12). (b) Dipole–dipole relaxation can cause transitions between the αα and ββ states, such that
they return to their original populations: the result is that the population difference across the A transitions is increased. (c) Dipole–dipole
relaxation can also cause the populations of the αβ and βα states to return to their equilibrium values: this decreases the population
difference across the A transitions.

dipole–dipole relaxation leads to an enhancement of the inten- Here I°A is the intensity of the signals due to nucleus A before
sity of the A spin transitions. saturation, and IA is the intensity after the X spins have been
The hamiltonian for the dipole–dipole interaction also con- saturated for long enough for the NOE to build up (typically
tains combinations of spin operators that flip the spins in op- several multiples of T1). For a homonuclear system, and if the
posite directions, so taking the system from αAβX to βAαX. This only source of relaxation is due to the dipole–dipole interac-
process drives the populations of these states to their equilib- tion, η lies between −1 (a negative enhancement) for slow
rium values, as shown in Fig. 12C.13c. As before, the popula- tumbling molecules and + 12 (a positive enhancement) for fast
tion differences across the A spin transitions are affected, but tumbling molecules. In practice, other sources of relaxation are
now they are reduced to  12 N , meaning that the A spin transi- invariably present so these limiting values are rarely achieved.
tions are less intense than they would have been in the absence The utility of the NOE in NMR spectroscopy comes about
of dipole–dipole relaxation. because only dipole–dipole relaxation can give rise to the ef-
It should be clear that there are two opposing effects: the re- fect: only this type of relaxation causes both spins to flip si-
laxation-induced transitions between αAαX and βAβX which en- multaneously. Thus, if nucleus X is saturated and a change in
hance the A spin transitions, and the transitions between αAβX the intensity of the transitions from nucleus A is observed,
and βAαX which reduce the intensity of these transitions. Which then there must be a dipole–dipole interaction between the
effect dominates depends on the relative rates of these two re- two nuclei. As that interaction is proportional to 1/R3, where
laxation pathways. As in the discussion of relaxation times in R is the distance between the two spins, and the relaxation
Section 12C.2, the efficiency of the βAβX ↔ αAαX relaxation it causes is proportional to the square of the interaction, the
is high if the dipole field oscillates close to the transition fre- NOE is proportional to 1/R6. Therefore, for there to be signifi-
quency, which in this case is about 2νL; likewise, the efficiency cant dipole–dipole relaxation between two spins they must
of the αAβX ↔ βAαX relaxation is high if the dipole field is sta- be close (for 1H nuclei, not more than 0.5 nm apart), so the
tionary (in this case there is no frequency difference between observation of an NOE is used as qualitative indication of the
the initial and final states). Small molecules tumble rapidly and proximity of nuclei. In principle it is possible to make a quan-
have substantial motion at 2nL. As a result, the βAβX ↔ αAαX titative estimate of the distance from the size of the NOE, but
pathway dominates and results in an increase in the intensity to do so requires the effects of other kinds of relaxation to be
of the A spin transitions. This increase is called a ‘positive NOE taken into account.
enhancement’. On the other hand, large molecules tumble more The value of the NOE enhancement η also depends on the
slowly so there is less motion at 2nL. In this case, the αAβX ↔ values of the magnetogyric ratios of A and X, because these
βAαX pathway dominates and results in a decrease in the inten- properties affect both the populations of the levels and the re-
sity of the A spin transitions. This decrease is called a ‘negative laxation rates. For a heteronuclear spin system the maximal en-
NOE enhancement’. hancement is
The NOE enhancement is usually reported in terms of the X
parameter η (eta), where   (12C.8)
2A
I A  I A where γA and γX are the magnetogyric ratios of nuclei A and X,
 NOE enhancement parameter  (12C.7)
I A respectively.
530 12 Magnetic resonance

Brief illustration 12C.3 It is common to take advantage of this enhancement to im-


prove the sensitivity of 13C NMR spectra. Prior to recording
From eqn 12C.8 and the data in Table 12A.2, the maximum the spectrum, the 1H nuclei are irradiated so that they become
NOE enhancement parameter for a 13C–1H pair is saturated, leading to a build-up of the NOE enhancement on
 the 13C nuclei.
  1H
 
2.675  108 T 1 s 1
  1.99
2  (6.73  107 T 1 s 1 ) Exercise
   
 13C E12C.5 Predict the maximum NOE enhancement (as the value of η) that could
be obtained for 31P as a result of dipole–dipole relaxation with 1H.

Checklist of concepts
☐ 1. The free-induction decay (FID) is the time-domain ☐ 5. The longitudinal relaxation time T1 can be measured by
signal resulting from the precession of transverse mag- the inversion recovery technique.
netization. ☐ 6. The transverse relaxation time T2 can be measured by
☐ 2. Fourier transformation of the FID (the time domain) observing spin echoes.
gives the NMR spectrum (the frequency domain). ☐ 7. In proton decoupling of 13C-NMR spectra, the protons
☐ 3. Longitudinal (or spin–lattice) relaxation is the process are continuously irradiated; the effect is to collapse the
by which the z-component of the magnetization returns splittings due to the 13C–1H couplings.
to its equilibrium value. ☐ 8. The nuclear Overhauser effect is the modification of the
☐ 4. Transverse (or spin–spin) relaxation is the process by intensity of one resonance by the saturation of another:
which the x- and y-components of the magnetization it occurs only if the two spins are involved in mutual
return to their equilibrium values of zero. dipole–dipole relaxation.

Checklist of equations
Property Equation Comment Equation number
 t /T2
Free-induction decay S(t )  S0 cos(2 Lt )e T2 is the transverse relaxation time 12C.1

Width at half-height of an NMR line Δn1/2 = 1/πT2 Assumed Lorentzian


Longitudinal relaxation M z (t )  M 0  e t /T1 T1 is the longitudinal relaxation time 12C.4
 t /T2
Transverse relaxation M xy (t )  e 12C.5

NOE enhancement parameter   (I A  I A )/I A Definition 12C.7


TOPIC 12D Electron paramagnetic
­resonance

molecular framework. This difference is taken into account by


➤ Why do you need to know this material? replacing ge by g and expressing the resonance condition as
Many materials and biological systems contain species
with unpaired electrons and some chemical reactions
h  g BB0 EPR resonance condition (12D.2)

generate intermediates with unpaired electrons. Electron


where g is the g-value of the radical.
paramagnetic resonance is a key spectroscopic tool for
Electron paramagnetic resonance spectra are usually re-
studying them.
corded by keeping the frequency of the microwave radiation
➤ What is the key idea? fixed and then varying the magnetic field so as to bring the
electron into resonance with the microwave frequency. The
The details of an EPR spectrum give information on the
positions of the peaks, and the horizontal scale on spectra, are
distribution of the density of the unpaired electrons.
therefore specified in terms of the magnetic field.
➤ What do you need to know already?
You need to be familiar with the concepts of electron spin Brief illustration 12D.1
(Topic 8B) and the general principles of magnetic reso-
nance (Topic 12A). The discussion refers to spin–orbit cou- The centre of the EPR spectrum of the methyl radical occurs at
pling in atoms (Topic 8C) and the Fermi contact interaction 329.40 mT in a spectrometer operating at 9.2330 GHz (radia-
in molecules (Topic 12B). tion belonging to the X band of the microwave region). Its
g-value is therefore

 h     

(6.62608  10 Js)  (9.2330  109 s 1 )


34

g  2.0027
(9.2740  1024 JT 1 )  (0.329 40 T)
 
B
Electron paramagnetic resonance (EPR), which is also known B0

as electron spin resonance (ESR), is used to study species that


contain unpaired electrons. Both solids and liquids can be
studied, but the study of gas-phase samples is complicated by The g-value is related to the ease with which the applied
the free rotation of the molecules. field can generate currents through the molecular framework
and the strength of the magnetic field these currents gener-
ate. Therefore, the g-value gives some information about elec-
tronic structure and plays a similar role in EPR to that played
12D.1 The g-value by shielding constants in NMR. A g-value smaller than ge im-
plies that in the molecule the electron experiences a magnetic
According to the discussion in Topic 12A, the resonance fre- field smaller than the applied field, whereas a value greater than
quency for a transition between the ms   12 and the ms   12 ge implies that the magnetic field is greater. Both outcomes are
levels of a free electron is possible, depending on the details of the electronic excited
states.
Resonance condition
h  g e BB0  (12D.1) Two factors are responsible for the difference of the g-value
[free electron]
from ge. Electrons migrate through the molecular framework
where ge ≈ 2.0023. If the electron is in a radical the field it expe- by making use of excited electronic states (Fig. 12D.1). This
riences differs from the applied field due to the presence of local circulation gives rise to a local magnetic field that can add to
magnetic fields arising from electronic currents induced in the or subtract from the applied field. The extent to which these
532 12 Magnetic resonance

of the point electric charge of the nucleus. The source of the


hyperfine structure in EPR is the magnetic interaction between
the electron spin and the magnetic dipole moments of the nu-
clei present in the radical that give rise to local magnetic fields.

12D.2(a) The effects of nuclear spin


Figure 12D.1 An applied magnetic field can induce circulation Consider the effect on the EPR spectrum of a single 1H nucleus
of electrons that makes use of excited state orbitals (shown located somewhere in a radical. The proton spin is a source of
schematically here). magnetic field and, depending on the orientation of the nuclear
spin, the field it generates either adds to or subtracts from the
applied field. The total local field is therefore

c­ urrents are induced is inversely proportional to the separation Bloc  B0  amI mI   12  (12D.3)
of energy levels, ΔE, in the radical or complex. Secondly, the
strength of the field experienced by the electron spin as a result where a is the hyperfine coupling constant (or hyperfine split-
of these orbital currents is proportional to the spin–orbit cou- ting constant); from eqn 12D.3 it follows that a has the same
pling constant, ξ (Topic 8C). It follows that the difference of the units as the magnetic field, for example tesla. Half the radicals
g-value from ge is proportional to ξ/ΔE. This proportionality is in a sample have mI   12 , so half resonate when the applied
widely observed. Many organic radicals, for which ΔE is large field satisfies the condition
and ξ (for carbon) is small, have g-values close to 2.0027, not far
removed from ge itself. Inorganic radicals, which commonly are h
h  g B (B0  12 a), or B0   12 a  (12D.4a)
built from heavier atoms and therefore have larger spin–orbit g B
coupling constants, have g-values typically in the range 1.9–2.1.
The g-values of paramagnetic d-metal complexes often differ The other half (which have mI   12 ) resonate when
considerably from ge, varying from 0 to 6, because in them ΔE h
is small on account of the small splitting of d-orbitals brought  
h  g B B0  12 a , or B0 
g B
 12 a  (12D.4b)
about by interactions with ligands (Topic 11F).
The g-value is anisotropic: that is, its magnitude depends on Therefore, instead of a single line, the spectrum shows two lines
the orientation of the radical with respect to the applied field. of half the original intensity separated by a and centred on the
The anisotropy arises from the fact that the extent to which field determined by g (Fig. 12D.2).
an applied field induces currents in the molecule, and there-
fore the magnitude of the local field, depends on the relative
No hyperfine splitting αN
orientation of the molecules and the field. In solution, when
Hyperfine splitting βN
the molecule is tumbling rapidly, only the average value of the due to one proton
g-value is observed. Therefore, the anisotropy of the g-value
is observed only for radicals trapped in solids and crystalline
d-metal ­complexes. h
α
β
h
Exercise
E12D.1 The centre of the EPR spectrum of atomic hydrogen lies at 329.12 mT
in a spectrometer operating at 9.2231 GHz. What is the g-value of the electron
in the atom? βN
αN

Figure 12D.2 The hyperfine interaction between an electron


12D.2 Hyperfine structure and a spin- 21 nucleus results in four energy levels in place of the
original two; αN and βN indicate the spin states of the nucleus. As
a result, the spectrum consists of two lines (of equal intensity)
The most important feature of an EPR spectrum is its ­hyperfine instead of one. The intensity distribution can be summarized by
structure, the splitting of individual resonance lines into com- a simple stick diagram. The diagonal lines show the energies of
ponents. In general in spectroscopy, the term ‘hyperfine struc- the states as the applied field is increased, and resonance occurs
ture’ means the structure of a spectrum that can be traced to when the separation of states matches the fixed energy of the
interactions of the electrons with nuclei other than as a result microwave photon.
12D Electron paramagnetic r­ esonance 533

1.61 mT
0.35 mT

Field strength

1:2:1 1:2:1 1:2:1


Figure 12D.3 The EPR spectrum of the benzene radical anion,
C6H6− , in solution; a is the hyperfine coupling constant. The centre Figure 12D.4 The analysis of the hyperfine structure of a radical
of the spectrum is determined by the g-value of the radical. containing one 14N nucleus (I = 1) and two equivalent protons.

doublet of spacing 0.35 mT by the first proton, and each line


If the radical contains an 14N atom (I = 1), its EPR spectrum of these doublets is split into a doublet with the same 0.35 mT
consists of three lines of equal intensity, because the 14N nucleus splitting by the second proton (Fig. 12D.4). Two of the lines
has three possible spin orientations, and each spin orientation in the centre coincide, so splitting by the two protons gives a
is possessed by one-third of all the radicals in the sample. In 1:2:1 triplet of internal splitting 0.35 mT. Overall the spectrum
general, a spin-I nucleus splits the spectrum into 2I + 1 hyper- consists of three identical 1:2:1 triplets.
fine lines of equal intensity.
Self-test 12D.1 Predict the form of the EPR spectrum of a radi-
When there are several magnetic nuclei present in the radi-
cal containing three equivalent 14N nuclei.
cal, each one contributes to the hyperfine structure. In the case
of equivalent protons (for example, the two CH2 protons in the
radical CH3CH2) some of the hyperfine lines are coincident. If
the radical contains N equivalent protons, then there are N + 1
hyperfine lines with an intensity distribution given by Pascal’s
triangle (1). The spectrum of the benzene radical anion in
Fig. 12D.3, which has seven lines with inten-
1 sity ratio 1:6:15:20:15:6:1, is consistent with
1 1
1 2 1 a radical containing six equivalent protons.
1 3 3 1 More generally, if the radical contains N
1 4 6 4 1 equivalent nuclei with spin quantum number 1 3 6 7 6 3 1
1 I, then there are 2NI + 1 hyperfine lines.
Figure 12D.5 The analysis of the hyperfine structure of a radical
containing three equivalent 14N nuclei.
Answer: Fig. 12D.5
Example 12D.1 Predicting the hyperfine structure of an
EPR spectrum
A radical contains one 14N nucleus (I = 1) with hyperfine con-
stant 1.61 mT and two equivalent protons  I  12  with hyper-
12D.3 The McConnell equation
fine constant 0.35 mT. Predict the form of the EPR spectrum.
The hyperfine structure of an EPR spectrum is a kind of fin-
Collect your thoughts You will need to consider the hyper- gerprint that helps to identify the radicals present in a sample.
fine structure that arises from each type of nucleus or group Moreover, because the magnitude of the splitting depends on
of equivalent nuclei in succession. First, split a line with one the distribution of the unpaired electron in the vicinity of the
nucleus, then split each of the lines again by a second nucleus magnetic nuclei, the spectrum can be used to map the molecu-
(or group of nuclei), and so on. It is best to start with the nucle- lar orbital occupied by the unpaired electron.
us with the largest hyperfine splitting; however, any choice The hyperfine splitting observed in C 6 H6− is 0.375 mT. If it is
could be made, and the order in which nuclei are considered
assumed that the unpaired electron is in an orbital with equal
does not affect the conclusion.
probability at each C atom, this hyperfine splitting can be at-
The solution The 14N nucleus gives three hyperfine lines of tributed to the interaction between a proton and one-sixth of
equal intensity separated by 1.61 mT. Each line is split into a the unpaired electron spin density. If all the electron density
534 12 Magnetic resonance

were located on the neighbouring C atom, a hyperfine coupling Table 12D.1 Hyperfine coupling constants for atoms, a/mT*
of 6 × 0.375 mT = 2.25 mT would be expected. If in another aro-
Nuclide Isotropic coupling Anisotropic coupling
matic radical the hyperfine coupling constant is found to be a, 1
H 50.8 (1s)
then the spin density, ρ, the probability that an unpaired elec-
2
tron is on the neighbouring C atom, can be calculated from the H 7.8 (1s)
14
McConnell equation: N 55.2 (2s) 4.8 (2p)
19
F 1720 (2s) 108.4 (2p)
a  Q McConnell equation (12D.5) * More values are given in the Resource section.

with Q = 2.25 mT. In this equation, ρ is the spin density on a C


atom and a is the hyperfine splitting observed for the H atom to in Topic 12B, an s electron has a ‘Fermi contact interaction’
which it is attached. with the nucleus, a magnetic interaction that occurs when the
point dipole approximation fails. The contact interaction is iso-
tropic (that is, independent of the orientation of the radical),
Brief illustration 12D.2
and consequently is shown even by rapidly tumbling molecules
The hyperfine structure of the EPR spectrum of the naphtha- in fluids (provided the spin density has some s character).
lene radical anion C10H8− (2) can be interpreted as arising from The dipole–dipole interactions of p electrons and the Fermi
interactions with two groups contact interaction of s electrons can be quite large. For exam-
ρ = 0.22 1 8 – of four equivalent protons. ple, a 2p electron in a nitrogen atom experiences an average
ρ = 0.08 2 7 Those at the 1, 4, 5, and 8 field of about 4.8 mT from the 14N nucleus. A 1s electron in a
3 6 positions in the ring have hydrogen atom experiences a field of about 50 mT as a result
4 5 a = 0.490 mT and those in of its Fermi contact interaction with the central proton. More
2 values are listed in Table 12D.1. The magnitudes of the con-
the 2, 3, 6, and 7 positions
have a = 0.183 mT. The spin densities obtained by using the tact interactions in radicals can be interpreted in terms of the s
McConnell equation are, respectively, orbital character of the molecular orbital occupied by the un-
a
paired electron, and the dipole–dipole interaction can be in-
 
0.490 mT 0.183 mT terpreted in terms of the p character. The analysis of hyperfine
  0.218 and    0.0813 structure therefore gives information about the composition
2.25 mT 2.25 mT

   of the orbital, and especially the hybridization of the atomic
Q
orbitals.

The origin of the hyperfine


12D.3(a) Brief illustration 12D.3

interaction From Table 12D.1, the hyperfine interaction between a 2s elec-


An electron in a p orbital centred on a nucleus does not ap- tron and the nucleus of a nitrogen atom is 55.2 mT. The EPR
proach the nucleus very closely, so the electron experiences spectrum of NO2 shows an isotropic hyperfine interaction of
a magnetic field that appears to arise from a point magnetic 5.7 mT. The s character of the molecular orbital occupied by the
dipole. The resulting interaction is called the dipole–dipole unpaired electron is the ratio 5.7/55.2 = 0.10. For a continua-
tion of this story, see Problem P12D.7.
­interaction. The contribution of a magnetic nucleus to the local
field experienced by the unpaired electron is given by an ex-
pression like that in eqn 12B.15 (a dependence proportional
to (1 − 3 cos2θ)/r3). A characteristic of this type of interaction Neither interaction appears to account for the hyperfine
is that it is anisotropic and averages to zero when the radical structure of the C 6 H6− anion and other aromatic radical anions.
is free to tumble. Therefore, hyperfine structure due to the The sample is fluid, and as the radicals are tumbling the hyper-
­dipole–dipole interaction is observed only for radicals trapped fine structure cannot be due to the dipole–dipole interaction.
in solids. Moreover, the protons lie in the nodal plane of the π orbital oc-
There is a second contribution to the hyperfine splitting. An cupied by the unpaired electron, so the structure cannot be due
s electron is spherically distributed around a nucleus and so has to a Fermi contact interaction. The explanation lies in a polari-
zero average dipole–dipole interaction with the nucleus even in zation mechanism similar to the one responsible for spin–spin
a solid sample. However, because an s electron has a non-zero coupling in NMR. The magnetic interaction between a proton
probability of being at the nucleus itself, it is incorrect to treat and the electrons favours one of the electrons being found with
the interaction as one between two point dipoles. As e­ xplained a greater probability nearby (Fig. 12D.6). The electron with
12D Electron paramagnetic ­resonance 535

Hund rule favours parallel electrons on atoms), so the unpaired elec-


Fermi tron can detect the spin of the proton indirectly. Calculation
using this model leads to a hyperfine interaction in agreement
Pauli
C
H with the observed value of 2.25 mT.

Low energy High energy


Exercises
(a) (b) 1
E12D.2 A radical containing two equivalent H nuclei shows a three-line
spectrum with an intensity distribution 1:2:1. The lines occur at 330.2 mT,
Figure 12D.6 The polarization mechanism for the hyperfine
332.5 mT, and 334.8 mT. What is the hyperfine coupling constant for each
interaction in π-electron radicals. The arrangement in (a) is proton? What is the g-value of the radical given that the spectrometer is
lower in energy than that in (b), so there is an effective coupling operating at 9.319 GHz?
between the unpaired electron and the proton.
E12D.3 A radical containing two inequivalent protons with hyperfine coupling
constants 2.0 mT and 2.6 mT gives a spectrum centred on 332.5 mT. At what
­ pposite spin is therefore more likely to be close to the C atom
o fields do the hyperfine lines occur and what are their relative intensities?
at the other end of the bond. The unpaired electron on the C E12D.4 Predict the intensity distribution in the hyperfine lines of the EPR
atom has a lower energy if it is parallel to that electron (Hund’s spectra of the radicals (i) ·C1H3, (ii) ·C2H3.

Checklist of concepts
☐ 1. The EPR resonance condition is expressed in terms of by the magnetic interaction between the electron and
the g-value of the radical. nuclei with spin.
☐ 2. The value of g depends on the ability of the applied field ☐ 4. If a radical contains N equivalent nuclei with spin quan-
to induce local electron currents in the radical and the tum number I, then there are 2NI + 1 hyperfine lines.
magnetic field experienced by the electron as a result of ☐ 5. Hyperfine structure arises from dipole–dipole interac-
these currents. tions, Fermi contact interactions, and the polarization
☐ 3. The hyperfine structure of an EPR spectrum is the mechanism.
splitting of individual resonance lines into components ☐ 6. The spin density on an atom is the probability that an
unpaired electron is on that atom.

Checklist of equations
Property Equation Comment Equation number
EPR resonance condition h  g BB0 No hyperfine interaction 12D.2
h  g B (B0  12 a) Hyperfine interaction between an electron and a proton 12D.4
McConnell equation a = Qρ Q = 2.25 mT 12D.5
536 12 Magnetic resonance

FOCUS 12 Magnetic resonance

To test your understanding of this material, work through Selected solutions can be found at the end of this Focus in
the Exercises, Additional exercises, Discussion questions, and the e-book. Solutions to even-numbered questions are available
Problems found throughout this Focus. online only to lecturers.

TOPIC 12A General principles


Discussion questions
D12A.1 Why do chemists and biochemists require spectrometers that operate D12A.2 Describe the effects of magnetic fields on the energies of nuclei and
at the highest available fields and frequencies to determine the structures of the energies of electrons. Explain the differences.
macromolecules by NMR spectroscopy?
D12A.3 What is the Larmor frequency? What is its significance in magnetic
resonance?

Additional exercises
E12A.6 Given that the nuclear g-factor, gI, is a dimensionless number, what are E12A.13 Calculate the frequency separation (in megahertz) of the nuclear spin
the units of the nuclear magnetogyric ratio γN when it is expressed in tesla and levels of a 14N nucleus in a magnetic field of 14.4 T given that its magnetogyric
hertz? ratio is 1.93 × 10−7 T−1 s−1.
E12A.7 Given that the nuclear g-factor, gI, is a dimensionless number, what are E12A.14 In which of the following systems is the energy level separation larger
the units of the nuclear magnetogyric ratio γN when it is expressed in SI base for a given magnetic field? (i) A 15N nucleus, (ii) a 31P nucleus.
units? 13
E12A.15 Calculate the relative population differences (Nα − Nβ)/N for C
14
E12A.8 For a N nucleus, what is the magnitude of the spin angular nuclei in fields of (i) 0.50 T, (ii) 2.5 T, and (iii) 15.5 T at 25 °C.
momentum and what are its allowed components along the z-axis? Express
E12A.16 By what factor must the applied magnetic field be increased for the
your answer in multiples of ℏ. What angles does the angular momentum make
relative population difference (Nα − Nβ)/N to be increased by a factor of 5 for
with the z-axis?
(i) 1H nuclei, (ii) 13C nuclei?
19
E12A.9 What is the NMR frequency of a F nucleus in a magnetic field of
E12A.17 By what factor must the temperature be changed for the relative
17.1 T? Express your answer in megahertz.
population difference (Nα − Nβ)/N to be increased by a factor of 5 for 1H nuclei
33
E12A.10 The nuclear spin quantum number of S is and its g-factor is 0.4289.
3
2 relative to its value at room temperature? Is changing the temperature of the
Calculate (in joules) the energies of the nuclear spin states in a magnetic field sample a practical way of increasing the sensitivity?
of 6.800 T.
E12A.18 What is the EPR resonance frequency in a magnetic field for which
14
E12A.11 The nuclear spin quantum number of N is 1 and its g-factor is 0.404. the NMR frequency for 1H nuclei (protons) is 500 MHz? Express your answer
Calculate (in joules) the energies of the nuclear spin states in a magnetic field in gigahertz.
of 10.50 T.
E12A.12 Calculate the frequency separation (in megahertz) of the nuclear spin
levels of a 13C nucleus in a magnetic field of 15.4 T given that its magnetogyric
ratio is 6.73 × 10−7 T−1 s−1.

Problems
P12A.1 A scientist investigates the possibility of neutron spin resonance, and nuclei of a particular element, the fraction present as a particular isotope is
has available a commercial NMR spectrometer operating at 300 MHz for 1H affected by the natural abundance of that isotope. Recalculate these results
nuclei. What is the NMR frequency of the neutron in this spectrometer? What taking this dependence into account.
is the relative population difference at room temperature? Which is the lower
P12A.3 The intensity of the NMR signal is given by eqn 12A.8c. The intensity
energy spin state of the neutron?
can be increased further by ‘isotopic labelling’, which involves increasing the

P12A.2 The relative sensitivity of NMR spectroscopy, R, for equal numbers proportion of the atoms present that are of the desired NMR-active isotope.
of different nuclei at constant temperature for a given magnetic field is The degree of labelling is expressed by giving the fractional enrichment. For
R  {I (I  1)} N3 . (a) From the data in Table 12A.2, calculate these sensitivities example, ‘a 10 per cent enrichment in 15N’ would imply that 10 per cent of all
for 2H, 13C, 14N, 15N, and 11B relative to that of 1H. (b) For a given number of the N atoms are 15N. (a) What level of enrichment is needed for the 15N signal
to have the same intensity as that from 13C with its natural abundance?
(b) What is the intensity achievable, relative to natural abundance 13C, by

These problems were supplied by Charles Trapp and Carmen Giunta. 100 per cent enrichment of 17O?
Exercises and problems 537

P12A.4 With special techniques, known collectively as ‘magnetic resonance Similar equations may be written for gradients along the x- and y-directions.
imaging’ (MRI), it is possible to obtain NMR spectra of entire organisms. A The NMR signal at frequency n = n(z) is proportional to the numbers of
key to MRI is the application of a magnetic field that varies linearly across the protons at the position z. Suppose a uniform disk-shaped organ is placed
specimen. If the field varies in the z-direction according to B0 + Gz z, where Gz in such a linear field gradient, and that in this case the NMR signal is
is the field gradient along the z-direction, the 1H nuclei have NMR frequencies proportional to the number of protons in a slice of width δz at each horizontal
given by distance z from the centre of the disk. Sketch the shape of the absorption
intensity for the MRI image of the disk.
N
 L (z )  (B  G z )
2 0 z

TOPIC 12B Features of NMR spectra


Discussion questions
1
D12B.1 The earliest NMR spectrometers measured the spectrum by keeping D12B.3 Explain why the resonance from two equivalent H nuclei does not
the frequency fixed and then scanning the magnetic field to bring peaks exhibit any splitting due to the spin–spin coupling that exists between the
successively into resonance. Peaks that came into resonance at higher nuclei, but that the resonance is split by the coupling to a third (inequivalent)
magnetic fields were described as ‘up field’ and those at lower magnetic fields spin.
as ‘down field’. Discuss what the terms ‘up field’ and ‘down field’ imply about
D12B.4 Explain the difference between magnetically equivalent and chemically
chemical shifts and shielding.
equivalent nuclei, and give two examples of each.
D12B.2 Discuss in detail the origins of the local, neighbouring group, and
D12B.5 Discuss how the Fermi contact interaction and the polarization
solvent contributions to the shielding constant.
mechanism contribute to spin–spin coupling in NMR.

Additional exercises
13 19 11
E12B.10 The C resonance from TMS is found to occur at 125.130 000 MHz. E12B.18 Sketch the form of the F NMR spectrum and the B NMR spectrum

What is the chemical shift (on the δ scale) of a peak at 125.148 750 MHz? 11
of BF . 4
1
E12B.11 In a spectrometer operating at 500.130 000 MHz for H, a resonance
31
E12B.19 Sketch the form of the P NMR spectra of a sample of
31
PF6− .
is found to occur 750 Hz higher in frequency than TMS. What is the chemical 1
E12B.20 Sketch the form of the H NMR spectra of
14
NH and of 15 NH 4+.
+
shift (on the δ scale) of this peak? 4

13 E12B.21 Predict the multiplet you would expect for coupling to two equivalent
E12B.12 In a spectrometer operating at 125.130 000 MHz for C, a resonance
spin-1 nuclei.
is found to occur 1875 Hz lower in frequency than TMS. What is the chemical
shift (on the δ scale) of this peak? E12B.22 Use an approach similar to that shown in Figs. 12B.13 and 12B.14
to predict the multiplet you would expect for coupling to two spin- 12 nuclei
E12B.13 What is the frequency separation, in hertz, between two peaks in the
13 when the coupling to the two nuclei is not the same.
C spectrum with chemical shifts δ = 50.0 and δ = 25.5 in a spectrometer
operating at 100.130 000 MHz for 13C? E12B.23 Predict the multiplet you would expect for coupling of a proton to
1 two inequivalent spin-1 nuclei.
E12B.14 In a spectrometer operating at 400.130 000 MHz for H, on the δ
scale what separation of peaks in the 1H spectrum corresponds to a frequency E12B.24 Use an approach similar to that shown in Figs. 12B.13 and 12B.14 to
difference of 550 Hz? predict the multiplet you would expect for coupling to two equivalent spin- 25
13 nuclei.
E12B.15 In a spectrometer operating at 200.130 000 MHz for C, on the δ scale
what separation of peaks in the 13C spectrum corresponds to a frequency E12B.25 Predict the multiplet you would expect for coupling to three
difference of 25 000 Hz? equivalent spin- 25 nuclei.
1
E12B.16 The chemical shift of the CH3 protons in ethoxyethane (diethyl ether) E12B.26 Classify the H nuclei in 1,2,3-trichlorobenzene into chemically or
is δ = 1.16 and that of the CH2 protons is 3.36. What is the difference in local magnetically equivalent groups. Give your reasoning.
magnetic field between the two regions of the molecule when the applied field 19 −
E12B.27 Classify the F nuclei in SF5 (which is square-pyramidal) into
is (i) 1.9 T, (ii) 16.5 T?
chemically or magnetically equivalent groups. Give your reasoning.
1
E12B.17 Make a sketch, roughly to scale, of the H NMR spectrum expected
E12B.28 A proton jumps between two sites with δ = 4.2 and δ = 5.5. What
for an AX2 spin system with δA = 1.50, δX = 4.50, and JAX = 5 Hz recorded on a
rate constant for the interconversion process is needed for the two signals to
spectrometer operating at 500 MHz. The horizontal scale should be in hertz,
collapse to a single line in a spectrometer operating at 350 MHz?
taking the resonance from TMS as the origin.

Problems
129 + 19
P12B.1 Explain why the Xe NMR spectrum of XeF is a doublet with P12B.2 The F NMR spectrum of IF5 consists of two lines of equal intensity and
J = 7600 Hz but the 19F NMR spectrum appears to be a triplet with a quintet (five lines with intensity ratio 1:4:6:4:1). Suggest a structure for IF5 that
J = 3800 Hz. Hints: 19F has spin- 12 and 100 per cent natural abundance; 129Xe is consistent with this spectrum, explaining how you arrive at your result. Hint:
has spin- 12 and 26 per cent natural abundance. You do not need to consider possible interaction with the I nucleus.
538 12 Magnetic resonance


P12B.3 The Lewis structure of SF4 has four bonded pairs of electrons and P12B.6 It might be unexpected that the Karplus equation, which was first
one lone pair. Propose two structures for SF4 based a trigonal bipyramidal derived for 3JHH coupling constants, should also apply to three-bond coupling
coordination at S, and a further structure based on a square pyramid. For between the nuclei of metallic elements such as tin. T.N. Mitchell and
each structure, describe the expected form of the 19F NMR spectrum, giving B. Kowall (Magn. Reson. Chem. 33, 325 (1995)) have studied the relation
your reasons. Hint: You do not need to consider possible interaction with the between 3JHH and 3JSnSn in compounds of the type Me3SnCH2CHRSnMe3 and
S nucleus. find that (3JSnSn/Hz) = 78.86 × (3JHH/Hz) + 27.84. (a) Does this result support a
Karplus type equation for tin nuclei? Explain your reasoning. (b) Obtain the
P12B.4 Refer to Fig. 12B.15 and use mathematical software or a spreadsheet to
Karplus equation for 3JSnSn and plot it as a function of the dihedral angle.
draw a family of curves showing the variation of 3JHH with ϕ using
(c) Draw the preferred conformation.
A = +7.0 Hz, B = −1.0 Hz, and allowing C to vary slightly from a typical value
of +5.0 Hz. Explore the effect of changing the value of the parameter C on the P12B.7 Show that the coupling constant as expressed by the Karplus equation
shape of the curve. In a similar fashion, explore the effect of the values of A (eqn 12B.14) passes through a minimum when cos ϕ = B/4C.
and B on the shape of the curve.
P12B.8 In a liquid, the dipolar magnetic field averages to zero: show this result

P12B.5 Various versions of the Karplus equation (eqn 12B.14) have been used by evaluating the average of the field given in eqn 12B.15. Hint: The relevant
to correlate data on three-bond proton coupling constants 3JHH in systems of volume element in polar coordinates is sin θ dθdϕ.
the type XYCHCHR3R4. The original version (M. Karplus, J. Am. Chem. Soc. 1
P12B.9 Account for the following observations: (a) The H NMR spectrum
85, 2870 (1963)) is 3JHH = A cos2ϕHH + B. Experimentally it is found that when
of cyclohexane shows a single peak at room temperature, but when the
R3 = R4 = H, 3JHH = 7.3 Hz; when R3 = CH3 and R4 = H, 3JHH = 8.0 Hz; when
temperate is lowered significantly the peak starts to broaden and then
R3 = R4 = CH3, 3JHH = 11.2 Hz. Assuming that only staggered conformations
separates into two. (b) At room temperature, the 19F NMR spectrum of
are important, determine which version of the Karplus equation fits the data
PF5 shows two lines, and even at the lowest experimentally accessible
better. Hint: You will need to consider which conformations to include, and
temperatures the spectrum is substantially unchanged. (c) In the 1H NMR
average the couplings predicted by the Karplus equation over them; assume
spectrum of a casually prepared sample of ethanol a triplet and a quartet are
that X and Y are ‘bulky’ groups.
seen. These multiplets show additional splittings if the sample is prepared with
the careful exclusion of water.

TOPIC 12C Pulse techniques in NMR


Discussion questions
D12C.1 Discuss in detail the effects of a 90° pulse and of a 180° pulse on a increases as the temperature increases, whereas that of a large molecule (like a
system of spin- 12 nuclei in a static magnetic field. polymer) decreases.
13
D12C.2 Suggest a reason why the relaxation times of C nuclei are typically D12C.4 Discuss the origin of the nuclear Overhauser effect and how it can be
1
much longer than those of H nuclei. used to identify nearby protons in a molecule.
D12C.3 Suggest a reason why the spin–lattice relaxation time of a small D12C.5 Distinguish between homogeneous and inhomogeneous broadening.
molecule (like benzene) in a mobile, deuterated hydrocarbon solvent

Additional exercises
13
E12C.6 The duration of a 90° or 180° pulse depends on the strength of the B1 E12C.9 The C NMR spectrum of fluoroethanoic acid shows a multiplet
field. If a 90° pulse applied to 1H requires 5 μs, what is the strength of the B1 centred at δ = 79. When the same spectrum is recorded using proton
field? How long would the corresponding 180° pulse require? decoupling, the multiplet collapses to a doublet with a splitting of 160 Hz.
Another multiplet, but with much smaller splittings, is also seen centred at
E12C.7 What is the effective transverse relaxation time when the width of a
δ = 179; this multiplet collapses to a doublet when decoupling is used. Explain
Lorentzian resonance line is 12 Hz?
these observations.
E12C.8 If the transverse relaxation time, T2, is 50 ms, after what time will the
E12C.10 Predict the maximum NOE enhancement (as the value of η) that
envelope of the free-induction decay decrease to half its initial amplitude?
could be obtained for 19F as a result of dipole–dipole relaxation with 1H.

Problems
P12C.1 An NMR spectroscopist performs a series of experiments in which a P12C.2 In a practical NMR spectrometer the free-induction decay is digitized
pulse of a certain duration is applied, the free-induction decay recorded, and at regular intervals before being stored in computer memory, ready for
then Fourier transformed to give the spectrum. A pulse of duration subsequent processing. Technically, it is difficult to digitize a signal at the
2.5 μs gave a satisfactory spectrum, but when the pulse duration was increased frequencies typical of NMR, so in practice a fixed reference frequency, close to
to 5.0 μs more intense peaks were seen. A further increase to 7.5 μs resulted the Larmor frequency, is subtracted from the NMR frequency. The resulting
in weaker signals, and increasing the duration to 10.0 μs gave no detectable difference frequency, called the offset frequency, is of the order of several
spectrum. (a) By considering the effect of varying the flip angle of the pulse, kilohertz, rather than the hundreds of megahertz typical of NMR resonance
rationalize these observations. Calculate (b) the duration of a 90° pulse and frequencies. This lower frequency can be digitized by currently available
(c) the B1 Larmor frequency  L   NB1 /2. technology. For 1H, if this reference frequency is set at the NMR frequency of
Exercises and problems 539

TMS, then a peak with chemical shift δ will give rise to a contribution to the (a) In the inversion recovery experiment the initial condition (at time zero)
free-induction decay at δ × (nL/106). Use mathematical software to construct is that Mz(0) = −M0, which corresponds to inversion at the magnetization by
the FID curve for a set of three 1H nuclei with resonances of equal intensity at the 180° pulse. Integrate the differential equation (it is separable), impose this
δ = 3.2, 4.1, and 5.0 in a spectrometer operating at 800 MHz. Assume that the initial condition, and hence show that M z ( )  M 0 (1  2e  /T1 ) , where τ is the
reference frequency is set at the NMR frequency of TMS, that T2 = 0.5 s, and delay between the 180° and 90° pulses. (b) Use mathematical software or a
plot the FID out to a maximum time of 1.5 s. Explore the effect of varying the spreadsheet to plot Mz(τ)/M0 as a function of τ, taking T1 = 1.0 s; explore
relative amplitude of the three resonances. the effect of increasing and decreasing T1. (c) Show that a plot of
ln{(M0 − Mz(τ))/2M0} against τ is expected to be a straight line with slope
P12C.3 Refer to Problem 12C.2 for the definition of offset frequency. The
−1/T1. (d) In an experiment the following data were obtained; use them to
FID, F(t), of a signal containing many frequencies, each corresponding to a
determine a value for T1.
different chemical shift, is given by
τ/s 0.000 0.100 0.200 0.300 0.400 0.600 0.800 1.000 1.200
F (t )  S0 j cos(2 j t )e
 t /T2 j

j Mz(τ)/M0 −1.000 −0.637 −0.341 −0.098 0.101 0.398 0.596 0.729 0.819
where, for each resonance j, S0j is the maximum intensity of the signal, νj is the P12C.8 Derive an expression for the time τ in an inversion recovery
offset frequency, and T2j is the spin–spin relaxation time. (a) Use mathematical experiment at which the magnetization passes through zero. In an experiment
software to plot the FID (out to a maximum time of 3 s) for the case it is found that this time for zero magnetization is 0.50 s; evaluate T1. Hint:
S01  1.0  1  50 Hz T21  0.50 s This Problem requires a result from Problem P12C.7.
S02  3.0  1  10 Hz T22  1.0 s P12C.9 The exponential relaxation of the transverse component of the
magnetization Mxy(t) back to its equilibrium value of zero is described by the
(b) Explore how the form of the FID changes as ν1 and T21 are changed.
differential equation
(c) Use mathematical software to calculate and plot the Fourier transforms of
the FID curves you generated in parts (a) and (b). How do spectral linewidths dM xy (t ) M xy (t )
vary with the value of T2? Hint: Most software packages offer a ‘fast Fourier 
dt T2
transform’ routine with which these calculations can be made: refer to the
user manual for details. You should select the cosine Fourier transform. (a) Integrate this differential equation (it is separable) between t = 0 and t = τ
with the initial condition that the transverse magnetization is Mxy(0) at t = 0 to
P12C.4 (a) In many instances it is possible to approximate the NMR lineshape obtain M xy ( )  M xy (0)e  /T2 . (b) Hence, show that a plot of ln{Mxy(τ)/Mxy(0)}
by using a Lorentzian function of the form against τ is expected to be a straight line of slope −1/T2. (c) The following data
S0T2 were obtained in a spin-echo experiment; use the data to evaluate T2.
I Lorentzian ( ) 
1  T22 (  0 )2
τ/ms 10.0 20.0 30.0 50.0 70.0 90.0 110 130
where I(ω) is the intensity as a function of the angular frequency ω = 2πν, ω0 Mxy(τ)/Mxy(0) 0.819 0.670 0.549 0.368 0.247 0.165 0.111 0.074
is the resonance frequency, S0 is a constant, and T2 is the spin–spin relaxation
time. Confirm that for this lineshape the width at half-height is 1/πT2. P12C.10 In the spin echo experiment analysed in Fig. 12C.11, the 180° pulse is
(b) Under certain circumstances, NMR lines are Gaussian functions of the applied about the y-axis, resulting in the magnetization vectors being reflected
frequency, given by in the yz-plane. The experiment works just as well when the 180° pulse is
2 2
applied about the x-axis, in which case the magnetization vectors are reflected
I Gaussian ( )  S0T2 e T2 ( 0 ) in the xz-plane. Analyse the outcome of the spin echo experiment for the case
where the 180° pulse is applied about the x-axis.
Confirm that for the Gaussian lineshape the width at half-height is equal to
2(ln 2)1/2/T2. (c) Compare and contrast the shapes of Lorentzian and Gaussian P12C.11 The z-component of the magnetic field at a distance R from a
lines by plotting two lines with the same values of S0, T2, and ω0. magnetic moment parallel to the z-axis is given by eqn 12B.17a. In a solid,
a proton at a distance R from another can experience such a field and
P12C.5 The shape of a spectral line, I(ω), is related to the free-induction decay
the measurement of the splitting it causes in the spectrum can be used to
signal G(t) by
calculate R. In gypsum, for instance, the splitting in the H2O resonance can be

interpreted in terms of a magnetic field of 0.715 mT generated by one proton
I ( )  a Re  G(t )eit dt and experienced by the other. What is the separation of the hydrogen nuclei
0
in the H2O molecule?
where a is a constant and ‘Re’ means take the real part of what follows.
Calculate the lineshape corresponding to an oscillating, decaying function P12C.12 In a liquid crystal a molecule might not rotate freely in all directions
G(t) = cos ωt e−t/τ. Hint: Write cos ωt as 12 (e  it  eit ). and the dipolar interaction might not average to zero. Suppose a molecule
is trapped so that, although the vector separating two protons may rotate
P12C.6 In the language of Problem P12C.5, show that if G(t) = (a cos ω1t +
freely around the z-axis, the colatitude may vary only between 0 and θ′. Use
b cos ω2t)e−t/τ, then the spectrum consists of two lines with intensities
mathematical software to average the dipolar field over this restricted range of
proportional to a and b and located at ω = ω1 and ω2, respectively.
orientation and confirm that the average vanishes when θ′ = π (corresponding
P12C.7 The exponential relaxation of the z-component of the magnetization to free rotation over a sphere). What is the average value of the local dipolar
Mz(t) back to its equilibrium value M0 is described by the differential field for the H2O molecule in Problem P12C.11 if it is dissolved in a liquid
equation crystal that enables it to rotate up to θ′ = 30°?

dM z (t ) M (t )  M 0
 z
dt T1
540 12 Magnetic resonance

TOPIC 12D Electron paramagnetic resonance


Discussion questions
D12D.1 Describe how the Fermi contact interaction and the polarization D12D.2 Explain how the EPR spectrum of an organic radical can be used to
mechanism contribute to hyperfine interactions in EPR. identify and map the molecular orbital occupied by the unpaired electron.

Additional exercises
E12D.5 The centre of the EPR spectrum of atomic deuterium lies at 330.02 mT E12D.8 Predict the intensity distribution in the hyperfine lines of the EPR
in a spectrometer operating at 9.2482 GHz. What is the g-value of the electron spectra of the radicals (i) ·C1H2C1H3, (ii) ·C2H2C2H3.
in the atom?
E12D.9 The benzene radical anion has g = 2.0025. At what field should you search
E12D.6 A radical containing three equivalent protons shows a four-line for resonance in a spectrometer operating at (i) 9.313 GHz, (ii) 33.80 GHz?
spectrum with an intensity distribution 1:3:3:1. The lines occur at 331.4 mT,
E12D.10 The naphthalene radical anion has g = 2.0024. At what field should you
333.6 mT, 335.8 mT, and 338.0 mT. What is the hyperfine coupling constant for
search for resonance in a spectrometer operating at (i) 9.501 GHz, (ii) 34.77 GHz?
each proton? What is the g-value of the radical given that the spectrometer is
operating at 9.332 GHz? E12D.11 The EPR spectrum of a radical with a single magnetic nucleus is split
into four lines of equal intensity. What is the nuclear spin of the nucleus?
E12D.7 A radical containing three inequivalent protons with hyperfine
coupling constants 2.11 mT, 2.87 mT, and 2.89 mT gives a spectrum centred E12D.12 The EPR spectrum of a radical with two equivalent nuclei of a
on 332.8 mT. At what fields do the hyperfine lines occur and what are their particular kind is split into five lines of intensity ratio 1:2:3:2:1. What is the
relative intensities? spin of the nuclei?

Problems
P12D.1 It is possible to produce very high magnetic fields over small volumes (protons 9, 10). Use the McConnell equation to estimate the spin density at
by special techniques. What would be the resonance frequency of an electron carbons 1, 2, and 9 (use Q = 2.25 mT).
spin in an organic radical in a field of 1.0 kT? How does this frequency NO2
compare to typical molecular rotational, vibrational, and electronic energy- NO2 NO2
level separations? 0.112 0.112
0.011 NO2 0.450 0.272 –
P12D.2 The angular NO2 molecule has a single unpaired electron and can – – 0.112 0.112
be trapped in a solid matrix or prepared inside a nitrite crystal by radiation 0.0172 0.011 0.108 NO2
0.450 NO2
damage of NO2− ions. When the applied field is parallel to the OO direction 0.0172
the centre of the spectrum lies at 333.64 mT in a spectrometer operating 2 3
1
at 9.302 GHz. When the field lies along the bisector of the ONO angle, the
resonance lies at 331.94 mT. What are the g-values in the two orientations? P12D.6 The hyperfine coupling constants observed in the radical anions (1),
(2), and (3) are shown (in millitesla, mT). Use the value for the benzene
P12D.3 (a) The hyperfine coupling constant is proportional to the
radical anion to map the probability of finding the unpaired electron in the π
gyromagnetic ratio of the nucleus in question, γN. Rationalize this
orbital on each C atom.
observation. (b) The hyperfine coupling constant in ·C1H3 is 2.3 mT. Use
the information in Table 12D.1 to predict the splitting between the hyperfine P12D.7 When an electron occupies a 2s orbital on an N atom it has a hyperfine
lines of the spectrum of ·C2H3. What are the overall widths of the multiplet in interaction of 55.2 mT with the nucleus. The spectrum of NO2 shows an
each case? isotropic hyperfine interaction of 5.7 mT. For what proportion of its time is
the unpaired electron of NO2 occupying a 2s orbital? The hyperfine coupling
P12D.4 The 1,4-dinitrobenzene radical anion can be prepared by reduction of
constant for an electron in a 2p orbital of an N atom is 3.4 mT. In NO2 the
1,4-dinitrobenzene. The radical anion has two equivalent N nuclei (I = 1) and
anisotropic part of the hyperfine coupling is 1.3 mT. What proportion of its
four equivalent protons. Predict the form of the EPR spectrum using
time does the unpaired electron spend in the 2p orbital of the N atom in
a(N) = 0.148 mT and a(H) = 0.112 mT.
NO2? What is the total probability that the electron will be found on (a) the N
P12D.5 The hyperfine coupling constants for the anthracene radical anion atoms, (b) the O atoms? What is the hybridization ratio of the N atom? Does
are 0.274 mT (protons 1, 4, 5, 8), 0.151 mT (protons 2, 3, 6, 7), and 0.534 mT the hybridization support the view that NO2 is angular?
Exercises and problems 541

FOCUS 12 Magnetic resonance


Integrated activities
I12.1 Consider the following series of molecules: benzene, methylbenzene,
trifluoromethylbenzene, benzonitrile, and nitrobenzene in which the O
substituents para to the C atom of interest are H, CH3, CF3, CN, and NO2,
respectively. (a) Use the computational method of your or your instructor’s
choice to calculate the net charge at the C atom para to these substituents 5 Phenoxy radical
in this series of organic molecules. (b) It is found empirically that the 13C
chemical shift of the para C atom increases in the order: methylbenzene,
I12.3 Two groups of protons have δ = 4.0 and δ = 5.2 and are interconverted by
benzene, trifluoromethylbenzene, benzonitrile, nitrobenzene. Is there a
a conformational change of a fluxional molecule. In a 60 MHz spectrometer
correlation between the behaviour of the 13C chemical shift and the computed
the spectrum collapsed into a single line at 280 K but at 300 MHz the collapse
net charge on the 13C atom? (c) The 13C chemical shifts of the para C atoms in
did not occur until the temperature had been raised to 300 K. Calculate the
each of the molecules that you examined computationally are as follows:
exchange rate constant at the two temperatures and hence find the activation
Substituent CH3 H CF3 CN NO2 energy of the interconversion (Topic 17D).
δ 128.4 128.5 128.9 129.1 129.4 I12.4 NMR spectroscopy may be used to determine the equilibrium constant
13 for dissociation of a complex between a small molecule, such as an enzyme
Is there a linear correlation between net charge and C chemical shift of the
inhibitor I, and a protein, such as an enzyme E:
para C atom in this series of molecules? (d) If you did find a correlation in
part (c), explain the physical origins of the correlation. EI  E  I K I  [E][I]/[EI]
I12.2 The computational techniques described in Topic 9E have shown that In the limit of slow chemical exchange, the NMR spectrum of a proton in
the amino acid tyrosine participates in a number of biological electron I would consist of two resonances: one at nI for free I and another at nEI for
transfer reactions, including the processes of water oxidation to O2 in plant bound I. When chemical exchange is fast, the NMR spectrum of the same
photosynthesis and of O2 reduction to water in oxidative phosphorylation. proton in I consists of a single peak with a resonance frequency n given by
During the course of these electron transfer reactions, a tyrosine radical forms n = fInI + fEInEI, where fI = [I]/([I] + [EI]) and fEI = [EI]/([I] + [EI]) are,
with spin density delocalized over the side chain of the amino acid. (a) The respectively, the fractions of free I and bound I. For the purposes of analysing
phenoxy radical shown in (5) is a suitable model of the tyrosine radical. Using the data, it is also useful to define the frequency differences δn = n − nI and
molecular modelling software and the computational method of your or your Δn = nEI − nI. Show that when the initial concentration of I, [I]0, is much
instructor’s choice, calculate the spin densities at the O atom and at all of the greater than the initial concentration of E, [E]0, a plot of [I]0 against (δn)−1 is a
C atoms in (5). (b) Predict the form of the EPR spectrum of (5). straight line with slope [E]0Δn and y-intercept −KI.
FOCUS 13
Statistical thermodynamics

Statistical thermodynamics provides the link between the


­microscopic properties of matter and its bulk properties. It pro-
13D The canonical ensemble
vides a means of calculating thermodynamic properties from
Molecules do interact with one another, and statistical thermo-
structural and spectroscopic data and gives insight into the mo-
dynamics would be incomplete without being able to take these
lecular origins of chemical properties.
interactions into account. This Topic shows how that is done in
principle by introducing the ‘canonical ensemble’, and hints at
how this concept can be used.
13A The Boltzmann distribution 13D.1 The concept of ensemble; 13D.2 The mean energy of a system;
13D.3 Independent molecules revisited; 13D.4 The variation of the
The ‘Boltzmann distribution’ is used to predict the populations energy with volume

of states in systems at thermal equilibrium. It is an exceptionally


important result because its predictions are used throughout
chemistry to understand many molecular phenomena. The dis- 13E The internal energy and
tribution also provides insight into the nature of ‘temperature’. the entropy
13A.1 Configurations and weights; 13A.2 The relative populations
of states
This Topic shows how molecular partition functions are used to
calculate and give insight into the two basic thermodynamic func-
tions, the internal energy and the entropy. The latter is based on
13B Molecular partition functions the definition introduced by Boltzmann of the ‘statistical entropy’.
13E.1 The internal energy; 13E.2 The entropy
The Boltzmann distribution introduces a quantity known as
the ‘partition function’ which plays a key role in statistical
thermodynamics. The Topic shows how to interpret the par- 13F Derived functions
tition function and how to calculate it in a number of simple
cases. With expressions relating internal energy and entropy to partition
13B.1 The significance of the partition function; 13B.2 Contributions to functions, it is possible to develop expressions for the derived ther-
the partition function modynamic functions, such as the Helmholtz and Gibbs energies.
The discussion is developed further to show how equilibrium con-
stants can be calculated from structural and spectroscopic data.
13C Molecular energies 13F.1 The derivations; 13F.2 Equilibrium constants

A partition function is the thermodynamic version of a wave-


function, and contains all the thermodynamic information What is an application of this material?
about a system. In this Topic partition functions are used to
calculate the mean values of the energy of the basic modes of There are numerous applications of statistical arguments in bio-
motion of a collection of independent molecules. chemistry. One of the most directly related to partition functions
13C.1 The basic equations; 13C.2 Contributions of the fundamental is explored in Impact 20, accessed via the e-book: the helix–coil
modes of motion equilibrium in a polypeptide and the role of cooperative behaviour.

➤ Go to the e-book for videos that feature the derivation and interpretation of equations, and applications of this material.
TOPIC 13A The Boltzmann distribution

13A.1 Configurations and weights


➤ Why do you need to know this material?
The Boltzmann distribution is the key to understanding a Any individual molecule may exist in states with energies ε0,
great deal of chemistry. All thermodynamic properties can ε1,…. For reasons that will become clear, the lowest available
be interpreted in its terms, as can the temperature depend- state is always taken as the zero of energy (that is, ε0 ≡ 0), and
ence of equilibrium constants and the rates of chemical all other energies are measured relative to that state. To obtain
reactions. There is, perhaps, no more important unifying the actual energy from the energy calculated in this way all that
concept in chemistry. is necessary is to add a term Nε(0), where N is the number of
molecules and ε(0) is the true energy of the ground state. For
➤ What is the key idea? example, when considering the vibrational contribution to the
The most probable distribution of molecules over the avail- energy, ε(0) is the zero-point energy of vibration.
able energy levels subject to certain restraints depends on
a single parameter, the temperature. 13A.1(a) Instantaneous configurations
➤ What do you need to know already? At any instant there will be N0 molecules in the state 0 with en-
ergy ε0, N1 in the state 1 with ε1, and so on, with N 0  N1    N ,
You need to be aware that molecules can exist only in
the total number of molecules in the system. The specification
certain discrete energy levels (Topic 7A) and that in some
of the set of populations N0,N1,… in the form {N0,N1,…} is a
cases more than one state has the same energy.
statement of the instantaneous configuration of the system.
The instantaneous configuration fluctuates with time because
the populations change, perhaps as a result of collisions.
Initially suppose that all the states have exactly the same en-
The problem addressed in this Topic is the calculation of the ergy. The energies of all the configurations are then identical,
populations of the energy levels of a system consisting of a so there is no restriction on how many of the N molecules are
large number of non-interacting molecules. These energy levels in each state. Now picture a large number of different instan-
can be associated with any mode of motion, such as vibration taneous configurations. One, for example, might be {N,0,0,…},
or rotation. Because the molecules are assumed to be non-­ corresponding to every molecule being in state 0. Another
interacting the total energy of the system is the sum of their in- might be {N − 2,2,0,0,…}, in which two molecules are in state 1.
dividual energies. In a real system, a contribution to the total The latter configuration is intrinsically more likely to be found
energy may arise from interactions between molecules, but that than the former because it can be achieved in more ways. The
possibility is discounted at this stage. configuration with all the molecules in the ground state can be
The development is based on the principle of equal a priori achieved in only one way, but there are more ways of achieving
probabilities, the assumption that all possibilities for the distri- the configuration {N − 2,2,0,0,…} as any two of the N molecules
bution of energy are equally probable. ‘A priori’ means loosely can occupy state 1.
in this context ‘as far as one knows’. In other words, there is no The number of ways the configuration {N − 2,2,0,0,…} can be
reason to assume that a vibrational state with a certain energy achieved is calculated in the following way. One molecule from
is any more or less likely to be populated than a rotational state the total N is chosen to migrate to state 1: there are N ways of
with the same energy, provided that the system is at thermal making this choice. For the second molecule that is chosen to
equilibrium. migrate to state 1 there are N − 1 possible choices. Overall, the
One very important conclusion will emerge from the follow- total number of ways of choosing two molecules is N(N − 1).
ing analysis: one distribution of the populations of the levels is However, the final configuration is the same regardless of the
overwhelmingly the most probable and this distribution de- order in which the two molecules are chosen. Therefore, only
pends on a single parameter, the ‘temperature’. The work done half the choices lead to distinguishable configurations, and
here provides a molecular justification for the concept of tem- so the total number of ways of achieving the configuration
perature and some insight into this crucially important quantity. {N − 2,2,0,0,} is 12 N (N − 1).
13A The Boltzmann distribution 545

If, as a result of collisions, the system were to fluctuate Brief illustration 13A.1
­ etween the configurations {N,0,0,…} and {N − 2,2,0,…}, it
b
would almost always be found in the second, more likely con- For the configuration {1,0,3,5,10,1}, which has N = 20, the
figuration, especially if N were large. In other words, a system weight is calculated as (recall that 0! ≡ 1)
free to switch between the two configurations would show 20 !
properties characteristic almost exclusively of the second W   9.31  108
1!0 !3!5!10 !1!
­configuration.
The next step is to develop an expression for the number of
ways that a general configuration {N0,N1,…} can be achieved. It will turn out to be more convenient to deal with the natu-
This number is called the weight of the configuration and ral logarithm of the weight, ln W , rather than with the weight
­denoted W . itself:

ln (x/y) = ln x − ln y

How is that done? 13A.1 Evaluating the weight of a N!


ln W = ln = ln N ! − ln(N0 ! N1 ! N 2 ! )
N0 ! N1 ! N2 !
configuration
Consider the number of ways of distributing N balls into bins ln xy = ln x + ln y

labelled 0, 1, 2 … such that there are N0 balls in bin 0, N1 in bin


= ln N ! − ln N 0 ! − ln N1 ! − ln N 2 ! − = ln N ! − ∑ ln N i !
1, and so on. The first ball can be selected in N different ways, i
the next ball in N − 1 different ways from the balls remaining,
and so on. Therefore, there are N(N − 1) … 1 = N! ways of One reason for introducing lnW is that it is easier to make ap-
selecting the balls. proximations. In particular, the factorials can be simplified by
There are N0! different ways in which the balls could have using Stirling’s approximation1
been chosen to fill bin 0 (Fig. 13A.1). Similarly, there are N1!
ways in which the balls in bin 1 could have been chosen, and ln x !  x ln x  x Stirling’s approximation [ x >> 1] (13A.2)
so on. Therefore, the total number of distinguishable ways of Then the approximate expression for the weight is
distributing the balls so that there are N0 in bin 0, N1 in bin 1,
etc. regardless of the order in which the balls were chosen is lnW  (N ln N  N )  (N i ln N i  N i )
i

 N ln N  N  N i ln N i  N [because N i  N ]
N = 18 i i

  N ln N  N i ln N i (13A.3)
i

13A.1(b) The most probable distribution


The configuration {N − 2,2,0,…} has much greater weight than
{N,0,0,…}, and it should be easy to believe that there may be
other configurations that have a much greater weight than both.
3! 6! 5! 4! In fact for large N there is a configuration for which the weight
is overwhelmingly larger than that of any other configuration.
Figure 13A.1 Eighteen molecules (the vertical bars) shown here
The system will almost always be found in this dominating con-
are distributed into four bins (distinguished by the three vertical
figuration because its weight is so much larger than other pos-
lines) such that there are 3 molecules in the first, 6 in the second,
and so on. There are 18! ways of selecting the balls to achieve
sible configurations. The properties of the system will therefore
this distribution. However, there are 3! equivalent ways of putting be characteristic of the dominating configuration.
three molecules in the first receptacle, and likewise 6! equivalent This dominating configuration can be found by looking for
ways of putting six molecules into the second receptacle, and so the values of Ni that lead to a maximum value of W . Because
on. Hence the number of distinguishable ways of achieving this W is a function of all the Ni, this search is done by varying the
distribution is 18!/3!6!5!4!, or about 515 million. Ni and looking for the values that correspond to dW = 0 just as
in the search for the maximum of any function. In practice it

N! (13A.1)
W = N !N !N ! Weight of a configuration
1
A more precise form of this approximation is ln x !  ln(2)1/ 2 
0 1 2
 x  12  ln x  x .
546 13 Statistical thermodynamics

is more convenient, and equivalent, to search for a maximum • Multiply each constraint by a constant and then add it to
value of lnW . the main variation equation (in this case eqn 13A.4).
Imagine that the configuration changes so that the popula- • Now treat the variables as though they are all independent.
tion of each state i changes from Ni to Ni + dNi. Because lnW
• Evaluate the constants at a later stage of the calculation.
depends on all the Ni when these change by dNi the function
lnW changes to lnW + d lnW , where This method is the key to finding the dominant distribution.

when W is a maximum
How is that done? 13A.2 Identifying the dominant
 lnW
d lnW = ∑ dN i = 0 (13A.4) distribution
i
N i
In preparation for this calculation it is useful to change eqn
The derivative ∂ lnW /∂N i expresses how lnW changes when 13A.3 from
Ni changes: if Ni changes by dNi, then lnW changes by ln W  N ln N  N i ln N i
( lnW /N i )  dN i. The total change in lnW is then the sum of i

all these changes.


to
Up to now it has been assumed that all the states have the
same energy, which means that all the possible configura- ln W  N ln N  N j ln N j
tions have the same total energy. This restriction must now j

be removed so that the states have different energies ε0,


by using j instead of i as the label of the states. Differentiation
ε1,…. as initially supposed. If the system is isolated it has of this expression with respect to Ni gives
a specified, constant, total energy and only configurations
that have this total energy must be retained. It follows that Second
First
term
  ter m
the configuration with the greatest weight must also satisfy (N j ln N j )
 ln W (N ln N )
the condition  
N i N i j N i
Energy constraint
N 
i
i i E [constant total energy]
 (13A.5a)
Step 1 Evaluate the first term in the expression

where E is the total energy of the system. When the Ni change The first term on the right is obtained (by using the product
by dNi the total energy must not change, so rule) as follows:

 dN
i
i i 0 Energy constraint (13A.5b) dfg/dx = fdg/dx + gdf/dx

(N ln N) N  ln N
The second constraint is that, because the total number of mol- = ln N +N
N i  Ni N i
ecules present is also fixed (at N), not all the populations can
be varied independently. Thus, increasing the population of one
state by 1 demands that the population of another state must be Now note that
reduced by 1. Therefore, the search for the maximum value of
W is also subject to the condition N 
 (N1  N 2  )  1
N i N i
Number constraint
N
i
i N [constant total number of moleculess]
 (13A.6a)
because Ni will match one and only one term in the sum what-
It follows that when the Ni change by dNi this sum must not ever the value of i. Next, note that
change, so
d ln y /dx = (1/y)dy/dx
dN
i
i 0 Number constraint  (13A.6b)
1
 ln N 1 N 1
The task is to solve eqn 13A.4, that is to maximize the = =
 Ni N N i N
weight, and at the same time conform to the constraints ex-
pressed in eqns 13A.5b and 13A.6b. The way to take con- Therefore
straints into account was devised by the mathematician
Joseph-Louis Lagrange, and is called the ‘method of unde- (N ln N )
 ln N  1
termined multipliers’: N i
13A The Boltzmann distribution 547

Step 2 Evaluate the second term Step 5 Treat the variables as independent
For the derivative of the second term, first note that The dNi are now treated as independent. Hence, the only way of
satisfying d lnW = 0 is to require that for each i,
dfg/dx = fdg/dx + gdf/dx
Ni
 ( N ln N ) Nj  ln N j  ln     i  0
N
∑ jN i j = ∑ Ni
ln N j + N j
 Ni
j j
and therefore that
d ln y/dx = (1/y)dy/dx Ni
 e  i  (13A.8)
N
 Nj Nj Nj
=∑ ln N j + which is very close to being the Boltzmann distribution.
j
Ni Nj Ni
Step 6 Evaluate the constant α
Now note that the Nj are independent, so the only term that
The final task is to identify the values of the constants α and β.
survives in the differentiation ∂Nj/∂Ni is the one with j = i. The
The first is found by noting that the sum of the populations Ni
derivative of this surviving term is ∂Ni/∂Ni = 1. It follows that
of the states must be equal to the total number of molecules N:
1 if i  j 1 if i  j
0 otherwise 0 otherwise N  N i  Ne  i  Ne e  i

  
   i i i
(N j ln N j )  N j   N j  
j N  j  N  ln N j   N  Because the N cancels on each side of this equality, it follows
that
i  i   i  
 ln N i  1
1
e   (13A.9)
e i
 
Step 3 Bring the two terms together
i
The result of bringing the first and second terms together is This expression for eα is used in eqn 13A.8 to give the
Boltzmann distribution
 lnW
 ln N  1  (ln N i  1)
N i Ni e− β i
(13A.10a)
=
N ∑ e− β i Boltzmann distribution
i
That is,

 lnW N The Boltzmann distribution is commonly written


  ln i
N i N (13A.7) N i e  i
 Boltzmann distribution (13A.10b)
N q
Step 4 Introduce the two constraints by using Lagrange’s method
There are two constraints: a number constraint (eqn 13A.6b) where q is called the partition function:
and an energy constraint (eqn 13A.5b). Therefore introduce
two constants α and −β (the negative sign will be helpful later),
q  e   i
Partition function [definition] (13A.11)
i
leading to
At this stage the partition function is a convenient abbrevia-
Main variation equation Number Energy tion for the sum, but in Topic 13B it is shown that the partition
 constraint constraint
   function is central to the statistical interpretation of thermody-
  lnW 
d lnW     dN i   dN i    i dN i namic properties.
i  N i  i i Equation 13A.10a shows that the most probable populations
 0 at a maximum of a given set of states with particular energies is governed by
the value of a single parameter, here β. The formal deduction of
Substitute ∂ lnW /∂N i using eqn 13A.7 to give the value of β depends on using the Boltzmann distribution to
deduce the perfect gas equation of state (that is done in Topic
 N  13F, and specifically in Example 13F.1). The result is
d lnW     ln i  dN i   dN i    i dN i
i  N  i i 1
  (13A.12)
 Ni  kT
   ln     i  dN i
i  N where T is the thermodynamic temperature and k is
 0 at a maximum Boltzmann’s constant. In other words:
548 13 Statistical thermodynamics

The temperature is the unique parameter that governs the A very important point to note is that the Boltzmann distri-
most probable populations of states of a system at thermal bution gives the relative populations of states, not energy levels.
equilibrium. Several states might have the same energy, and each state has
a relative population given by eqn 13A.13a. When calculating
the relative populations of energy levels rather than states, it
Brief illustration 13A.2
is necessary to take into account this degeneracy. Thus, if the
Suppose that two conformations of a molecule differ in energy level of energy εi is gi-fold degenerate (in the sense that there
by 5.0 kJ mol−1 (corresponding to 8.3 zJ for a single molecule; are gi states with that energy), and the level of energy εj is gj-fold
1 zJ = 10−21 J). There are therefore two states: the ground degenerate, then the relative total populations of the levels are
state, conformation A, with energy 0 and the higher energy given by
state, conformation B, with energy ε = 8.3 zJ. At 20 °C (293 K)
the denominator in eqn 13A.10a is N i g i e  i g   (   ) Boltzmann
   j
 ie i j population  (13A.13b)
N j g je gj
e
21 23 1
  i  ( 8.310 J ) / (1.38110 J K )( 293 K )
 1  e  /kT  1  e  1.12 ratiio of levels
i

The proportion of molecules in conformation B at this tem-


perature is therefore

NB e
 ( 8.31021 J ) / (1.3811023 J K 1 )( 293 K ) Example 13A.1 Calculating the relative populations of
  0.11
N 1.12 rotational levels
or 11 per cent of the molecules. Calculate the relative populations of the J = 1 and J = 0 rota-
tional levels of HCl at 25 °C; for HCl, B  10.591 cm 1 .
Collect your thoughts Although the ground state is non-
degenerate, you need to note that the level with J = 1 is triply
Exercises degenerate (MJ = 0, ±1); the energy of the state with quantum
E13A.1 Calculate the weight of the configuration in which 16 objects are  ( J  1) (Topic 11B). A useful relation is
number J is  J  hcBJ
−1
distributed in the arrangement 0, 1, 2, 3, 8, 0, 0, 0, 0, 2. kT/hc = 207.225 cm at 298.15 K.
E13A.2 What are the relative populations of the states of a two-level system
The solution The energy separation of levels with J = 1 and J = 0
when the temperature is infinite?
is  1   0  2hcB . The ratio of the population of these two levels
is given by eqn 13A.13b

13A.2The relative populations N 1 g 1   ( 1   0 )


 e
of states N0 g 0

Here g0 = 1 and g1 = 3 hence


If only the relative populations of states are of interest the parti-
tion function in eqn 13A.10b need not be evaluated because it N1 
cancels when the ratio is taken:  3e 2hcB 
N0

N i e  i q e  i Boltzmann


  (   ) Insertion of hcB   hcB /kT  (10.591 cm 1 )/(207.225 cm 1 ) 
  e i j n  (13A.13a)
population
Nj q e   j
e
  j
ratio 0.0511… then gives

Note that for a given energy separation the ratio of populations N1


 3e 20.0511  2.708
N1/N0 decreases as β increases (and the temperature decreases). N0
At T = 0 (β = ∞) all the population is in the ground state and
the ratio is zero. Equation 13A.13a is enormously important Comment. Because the J = 1 level is triply degenerate, it has
for understanding a wide range of chemical phenomena and a higher population than the level with J = 0, despite being of
higher energy.
is the form in which the Boltzmann distribution is commonly
employed (for instance, in the discussion of the intensities of Self-test 13A.1 What is the ratio of the populations of the levels
spectral transitions, Topic 11A). It implies that the relative pop- with J = 2 and J = 1 of HCl at the same temperature?
ulation of two states decreases exponentially with their differ- Answer: 1.359
ence in energy.
13A The Boltzmann distribution 549

Exercises
E13A.3 What is the temperature of a two-level system of energy separation E13A.5 Calculate the relative populations of a linear rotor at 298 K in the levels
equivalent to 400 cm−1 when the population of the upper state is one-third that with J = 0 and J = 5, given that B  2.71 cm 1.
of the lower state?
−1
E13A.4 A certain molecule has a non-degenerate excited state lying at 540 cm
above the non-degenerate ground state. At what temperature will 10 per cent
of the molecules be in the upper state?

Checklist of concepts
☐ 1. The principle of equal a priori probabilities assumes ☐ 3. The Boltzmann distribution gives the numbers of mol-
that all possibilities for the distribution of energy are ecules in each state of a system at any temperature.
equally probable in the sense that the distribution is ☐ 4. The relative populations of energy levels, as opposed
blind to the type of motion involved. to states, must take into account the degeneracies of the
☐ 2. The configuration of a system of N molecules is the energy levels.
specification of the set of populations N0, N1,… of the
energy states ε0, ε1,….

Checklist of equations
Property Equation Comment Equation number
  i
Boltzmann distribution N i /N  e /q β = 1/kT 13A.10b

Partition function q   e   i
See Topic 13B 13A.11
i
  ( i  j )
Boltzmann population ratio N i /N j  ( g i /g j )e gi, gj are degeneracies 13A.13b
TOPIC 13B Molecular partition functions

where β = 1/kT. It is called the molecular partition function as


➤ Why do you need to know this material? the sum is over the states of a single molecule. As emphasized
Through the partition function, statistical thermodynam-
in Topic 13A, the sum is over states, not energy levels. If gi states
ics provides the link between thermodynamic data and
have the same energy εi (so the level is gi-fold degenerate), then
molecular properties that have been calculated or derived Molecular partition function
ge
  i
from spectroscopy. Therefore, this material is an essen- q i [alternativee definition]  (13B.1b)
levels i
tial foundation for understanding physical and chemical
properties of bulk matter in terms of the properties of the where the sum is now over energy levels (sets of states with
constituent molecules. the same energy), not individual states. Also as emphasized in
Topic 13A, the lowest available state is taken as the zero of en-
➤ What is the key idea? ergy, so ε0 ≡ 0.
The partition function is calculated by drawing on com- The significance of the partition function is best illustrated by
puted or spectroscopically derived structural information calculating it for a particular set of states. Suppose a molecule
about molecules. has available a set of states with energies 0, ε, 2ε,… (Fig. 13B.1).
The molecular partition function is
➤ What do you need to know already?
You need to know that the Boltzmann distribution express- q  1  e    e 2     1  e    (e   )2  
es the most probable distribution of molecules over the
which is the geometric series 1 + x + x 2 +  with x = e−βε.
available states (Topic 13A). You need to be aware of the
Provided |x| < 1 the sum to infinity of this series is 1/(1 − x). For
expressions for the rotational and vibrational levels of mol-
any finite temperature it is indeed the case that |x| < 1, so the
ecules (Topics 11B–11D) and the energy levels of a particle
series evaluates to
in a box (Topic 7D).
1 Partition function
q  (13B.2a)
1  e   [equally spaced states]

 
Figure 13B.2 shows a plot of this function against the dimen-
The partition function q   e i is introduced in Topic 13A sionless quantity 1/βε = kT/ε, which is proportional to tem-
i
simply as a symbol to denote the sum over states that occurs in perature. At low temperatures, in the sense that kT/ε << 1,
the denominator of the Boltzmann distribution (eqn 13A.10b, the partition function tends to 1. The value of the partition
pi  e  i /q , with pi = Ni/N). But it is far more important than
that might suggest. For instance, it contains all the informa-
tion needed to calculate the bulk properties of a system of in-
dependent (that is, non-interacting) particles. In this respect q
plays a role for bulk matter very similar to that played by the
wavefunction in quantum mechanics for individual molecules:
. . . Energy

q is a kind of thermal wavefunction.


ε

The significance of the


13B.1


partition function ε
0

The molecular partition function is Figure 13B.1 The infinite set of states with equal spacing ε used
in the calculation of the partition function in Brief illustration
Molecular partition function
e
  i
q  (13B.1a) 13B.1. A harmonic oscillator has the same array of non-degenerate
[definition]
states i levels.
13B Molecular partition functions 551

10 the same time, the partition function rises from 1, so its value
gives an indication of the range of states populated at any given
temperature. The name ‘partition function’ reflects the sense
Partition function, q

in which q measures how the total number of molecules is


­distributed—partitioned—over the available states.
5 States that are significantly populated at a particular temper-
ature are said to be thermally accessible. These are the states
that contribute significantly to the sum used to compute the
partition function.
The corresponding expressions for a system in which there
0 are just two states, with energies ε0 = 0 and ε1 = ε (a ‘two-level
0 5 10
Temperature, kT/ε system’), are

Figure 13B.2 The partition function for the system shown in Partition function
q  1  e    (13B.3a)
Fig. 13B.1 (a harmonic oscillator) as a function of temperature. [two -level system]

and
f­ unction starts to rise significantly above 1 when kT/ε ≈ 1, and
e  i e  i
then continues to rise indefinitely. At high temperatures, in the pi    (13B.3b)
q 1  e  
sense that kT/ε >> 1, the partition function increases linearly
with the temperature. The fractional populations of the two states are therefore
The physical significance of a partition function can be dis-
cussed using the expression for its value for an equally spaced 1 e  
p0  p1   (13B.4)
set of states, eqn 13B.2a. To do so, first note that the Boltzmann 1  e   1  e  
distribution gives the fraction, pi = Ni/N, of molecules in the
state with energy εi as pi  e  i /q . For an equally spaced set of Figure 13B.4 shows the variation of the partition function with
states the partition function is given by eqn 13B.2a and the en- temperature and Fig. 13B.5 shows how the fractional popu-
ergies εi are given by εi = iε. Therefore, the fractional popula- lations change. Notice that at very low temperatures, when
tions are kT/ε << 1, the fractional populations tend to p0 = 1 and p1 = 0,
and the partition function tends to q = 1 (one state occupied).
e  i However, the fractional populations tend towards ­ equality
pi   (1  e   )e i   (13B.2b)
q  p0  12 , p1  12  and q = 2 (two states occupied) at very high
­temperatures (when kT/ε >> 1).
Figure 13B.3 shows how pi varies with temperature. At very low A common error is to suppose that at very high temperatures,
temperatures (kT/ε << 1 or βε >> 1), where q is close to 1, only when kT/ε >> 1, all the molecules in the system will be found
the lowest state is significantly populated. As the temperature in the upper energy state. However, as seen from eqn 13B.4,
is raised, the population breaks out of the lowest state, and the in this limit the populations of states become equal. The same
upper states become progressively more highly populated. At

1.4 2
Low High
temperature temperature
Partition function, q

1.2 1.5

1 1
0 0.5 1 0 5 10
βε: 3.0 1.0 0.7 0.3 Temperature, kT/ε Temperature, kT/ε
q : 1.05 1.58 1.99 3.86
Figure 13B.4 The dependence of the partition function of a two-
Figure 13B.3 The populations of the set states shown in Fig. 13B.1 level system on temperature. The two graphs differ in the scale of
at different temperatures, and the corresponding values of the the temperature axis to show the approach to 1 as T → 0 and the
partition function calculated from eqn 13B.2a. Note that β = 1/kT. slow approach to 2 as T → ∞.
552 13 Statistical thermodynamics

1 1
p0 Exercises
E13B.1 Consider a system with an infinite number of doubly degenerate
energy levels, evenly spaced by ε. Derive an expression for the partition
Population, p

p0 function and describe its behaviour as a function of temperature. How does


the behaviour compare with a system of evenly spaced non-degenerate levels?
0.5 0.5
p1 E13B.2 Consider a two-level system in which the ground level is non-degenerate
and has energy zero, and the upper level is doubly degenerate and has energy ε.
At what temperature will the populations of the two levels be equal?
p1

0 0
0 0.5
Temperature, kT/ε
1 0 5
Temperature, kT/ε
10
Contributions to the partition
13B.2

Figure 13B.5 The variation with temperature of the fractional


function
populations of the two states of a two-level system (eqn 13B.4).
The energy of an isolated molecule is the sum of contributions
Note that as the temperature approaches infinity, the populations
of the two states become equal (and the fractional populations from its different modes of motion:
both approach 0.5).
 i   iT   iR   iV   iE (13B.5)

where T denotes translation, R rotation, V vibration, and E


c­ onclusion is also true of many-level systems: if the ­temperature the electronic contribution. The electronic contribution is not
is sufficiently high, all states become equally ­populated. actually a ‘mode of motion’, but it is convenient to include it
Now consider the general case of a system with an infinite here. The separation of terms in eqn 13B.5 is only approximate
number of energy levels. The partition function is defined in ­(except for translation) because the modes are not completely
eqn 13B.1b as independent, but in most cases the separation is acceptable.
Given that the energy is a sum of independent contributions,
ge
  i
q  g 0  g 1e  1  g 2 e   2  
levels i
i
the partition function is a product of contributions:

where, as usual, the energy of the ground level is taken as


q  e    
T
  iR   iV   iE
i
e  i
zero. Now consider the limit as T approaches zero and there- i i ( all states )
fore as the parameter β = 1/kT approaches infinity. Apart
   
T
  iR   iV   iE
from the g0, all the terms in the expression above have the  e  i
i ( translation
nal ) i ( rotational ) i ( vibrational ) i ( electronic )
form e−x and therefore in the limit of low temperature x → ∞,
so these terms all go to zero. The partition function therefore  T  R  V 
   e  i    e  i    e  i 
tends to g0:  i ( translational )   i (rotational )   i ( vibrational ) 
lim q  g 0  E 

T 0    e   i 
 i ( electronic) 
That is, at T = 0, the partition function is equal to the degen-
eracy of the ground level (commonly, but not necessarily, 1). That is,
When T is so high that for each term in the sum βεi = εi/kT ≈ 0,
each term in the sum now contributes gi because e−x = 1 when q = qTqRqVqE Factorization of the partition function (13B.6)
x = 0. It follows that the sum is equal to the sum of the degener- This factorization means that each contribution can be inves-
acies of the levels which is the total number of molecular states. tigated separately. In general, exact analytical expressions for
In general there are an infinite number of states, so partition functions cannot be obtained. However, approximate
lim q   expressions can often be found and prove to be very important
T  for understanding chemical phenomena; they are derived in
In some idealized cases, the molecule may have only a finite the following sections and collected at the end of this Topic.
number of states; then the upper limit of q is equal to the num-
ber of states (as for the two-level system). In summary, 13B.2(a) The translational contribution
The molecular partition function gives an indication of the The translational partition function for a particle of mass m
number of states that are thermally accessible to a molecule free to move in a one-dimensional container of length X can
at the temperature of the system. be evaluated by making use of the fact that the separation of
13B Molecular partition functions 553

energy levels is very small and that large numbers of states are With β = 1/kT this relation has the form
accessible at normal temperatures.
X h (13B.7)
qXT = Λ Λ =
(2πmkT )1/2 Translational partition function

How is that done? 13B.1 Deriving an expression for the


translational partition function The quantity Λ (uppercase lambda) has the dimensions of
length and is called the thermal wavelength (sometimes the
The starting point of the derivation is eqn 7D.6 (En = n2h2/8mL2) ‘thermal de Broglie wavelength’) of the molecule. The thermal
for the energy levels of a particle in a one-dimensional box. wavelength decreases with increasing mass and temperature.
These levels are not degenerate and the quantum number n Equation 13B.7 shows that:
takes values 1, 2, 3 … For a molecule of mass m in a container
of length X it follows that: • The partition function for translational motion increases
2 2
with the length of the box and the mass of the particle:
nh
En = in each case the separation of the energy levels becomes
8mX 2 smaller and more levels become thermally accessible.
Step 1 Write an expression for the sum in eqn 13B.1a • For a given mass and length of the box, the partition func-
The lowest level (n = 1) has energy h2/8mX2, so the energies tion also increases with increasing temperature (decreas-
relative to that level are ing β), because more states become accessible.
 n  (n2  1) ,   h2 /8mX 2 The total energy of a molecule free to move in three di-
The sum to evaluate is therefore mensions is the sum of its translational energies in all three

­directions:
qXT  e (n 1) 
2

 n1n2n3   n(1X )   n(Y2 )   n(3Z ) (13B.8)


n 1

Step 2 Make some approximations and convert the sum to an where n1, n2, and n3 are the quantum numbers for motion in the
integral x-, y-, and z-directions, respectively. Therefore, because ea+b+c =
For a molecule in a container the size of a typical labora- eaebec, the partition function factorizes as follows:
tory vessel, and at any reasonable temperature, there are an   n( X )   n( Y )   n( Z )   n( X )   n( Y )   n( Z )
enormous number of energy levels with energies less than q T  e 1 2 3
 e 1
e 2
e 3

all n all n
or equal to kT. Therefore, many terms contribute to the sum
and the partition function will be very large. Negligible error    ( X )    n( Y )    n( Z ) 
  e n1   e 2   e 3 
is introduced by assuming that the lowest energy level has n
 1   n2   n3 
n = 0 because that simply adds one additional state to the
very large number that contribute to the partition function. That is,
With this change the expression for the partition function
q T = qXT qYT qZT (13B.9)
becomes
 Equation 13B.7 gives the partition function for translational
qXT  e n 
2

motion in the x-direction. The only change for the other two di-
n 0
rections is to replace the length X by the lengths Y or Z. Hence
Many terms contribute to the sum, so it is well approximated the partition function for motion in three dimensions is
by an integral 3/2
 2m  (2mkT )3/ 2
 qT    XYZ  XYZ  (13B.10a)
q  0 e
2
T  n 
X dn h  
2
h3

The integral is now in a standard form. The product of lengths XYZ is the volume, V, of the container, so

Step 3 Evaluate the integral V Translational partition function


qT  [three -dimensional]  (13B.10b)
2
Make the substitution x = n βε, implying n = x/(βε) 2 1/2
and 3
hence dn = dx/(βε)1/2. It therefore follows that with Λ defined in eqn 13B.7. As in the one-dimensional case,
the partition function increases with the mass of the particle (as
Integral G.1: = h 2/8mX 2 m3/2) and the volume of the container (as V); for a given mass
π1/2/2 and volume, the partition function increases with temperature
1/2 1/2 1/2
1 1 π1/2 2πm (as T3/2). At room temperature, qT ≈ 2 × 1028 for an O2 molecule

2
qXT = e− x d x = = X
β 0 β 2 h2β in a vessel of volume 100 cm3. As anticipated above, the n
­ umber
554 13 Statistical thermodynamics

of thermally accessible states is very large indeed, so the ap- this expression. Each level consists of 2J + 1 degenerate states.
proximations made in deriving the expression for the partition Therefore, the partition function of a non-symmetrical (AB)
function are justified. linear rotor is computed using eqn 13B.1b as a sum over levels
 g
 J
J 
Brief illustration 13B.1
q   (2 J  1) e
R  ( J 1)
  hcBJ
 (13B.11)
J
To calculate the translational partition function of an H2 mol-
ecule confined to a 100 cm3 vessel at 25 °C, use m = 2.016mu. The direct method of calculating qR is to substitute the experi-
Then, from Λ = h/(2πmkT)1/2, mental values of the rotational energy levels into this expression
and to sum the series numerically. (The case of symmetrical A2
kg m
 s
2 2
molecules is dealt with later.)
6.626  1034 Js
  {2  (2.016  1.6605  1027 kg )  (1.381  1023 JK 1 )  (298 K)}1/2

kg m 2 s 2
 7.12 1011 m
Example 13B.1 Evaluating the rotational partition
Therefore, function explicitly
Evaluate the rotational partition function of 1H35Cl at 25 °C,
1.00  104 m3
T
q   2.77  10 26
given that B  10.591 cm 1 .
(7.12 1011 m)3
Collect your thoughts You need to evaluate eqn 13B.11 term
About 1026 quantum states are thermally accessible, even at by term, and it is convenient to use kT/hc = 207.225 cm−1 at
room temperature and for this light molecule. Many states 298.15 K. The sum is readily evaluated by using mathematical
are occupied if the thermal wavelength (which in this case software.
is 71.2 pm) is small compared with the linear dimensions of
The solution To show how successive terms contribute, draw
the container.
up the following table by using hcB /kT = 0.05111 (Fig. 13B.6):

J 0 1 2 3 4 … 10
Equation 13B.10b can be interpreted in terms of the aver- (2J + 1)e−0.05111J(J + 1) 1 2.708 3.680 3.791 3.238 … 0.076
age separation, d, of the particles in the container. Because q
is the total number of accessible states, the average number The sum required by eqn 13B.11 (the sum of the numbers in
of translational states per molecule is qT/N. For this quantity the second row of the table) is 19.9, hence qR = 19.9 at this tem-
to be large, the condition V/NΛ3 >> 1 must be met. However, perature. Taking J up to 15 gives qR = 19.902; adding further
V/N is the volume occupied by a single particle, and there- terms results in no further change to the value of qR at this
fore the average separation of the particles is d = (V/N)1/3. precision.
The condition for there being many states available per mol-
ecule is therefore d3/Λ3 >> 1, and therefore d >> Λ. That is, for
eqn 13B.10b to be valid, the average separation of the parti-
cles must be much greater than their thermal wavelength. For 4

1 mol H2 molecules at 1 bar and 298 K, the average separation


is 3 nm, which is significantly larger than their thermal wave-
Contribution to q

3
length (71.2 pm).
The validity of eqn 13B.10b can be expressed in a different
2
way by noting that the approximations that led to it are valid if
many states are occupied, which requires V/Λ3 to be large. That
will be so if Λ is small compared with the linear dimensions of 1
the container. For H2 at 298 K, Λ = 71 pm, which is far smaller
than any conventional container is likely to be (but comparable
0
to pores in zeolites or cavities in clathrates). For O2, a heavier 0 1 2 3 4 5 6 7 8 9 10
molecule, Λ = 18 pm. Quantum number, J

Figure 13B.6 The contributions to the rotational partition


13B.2(b) The rotational contribution function of an HCl molecule at 25 °C. The vertical axis is the value

of (2 J  1)e   hcBJ ( J 1) . Successive terms (which are proportional to the
 ( J  1), with
The energy levels of a linear rotor are  J  hcBJ populations of the levels) pass through a maximum because the
J = 0, 1, 2,… (Topic 11B). The state of lowest energy has zero population of individual states decreases exponentially, but the
energy, so no adjustment need be made to the energies given by degeneracy of the levels increases with J.
13B Molecular partition functions 555

Comment. Notice that about ten J-levels are significantly popu-

K = +1, J = 1,2,3,...
K = +2, J = 2,3,4,...
K = –2, J = 2,3,4,...
K = –1, J = 1,2,3,...
K = 0, J = 0,1,2,...
lated but the number of populated states is larger on account of
K = –J K K = +J J = |K| J = |K|
the (2J + 1)-fold degeneracy of each level. –5 –4 –3 –2 –1 0 +1+2 +3 +4+5

Self-test 13B.1 Evaluate the rotational partition function for J


1
H35Cl at 0 °C. 5
Answer: 18.26 4
3
2
At room temperature, kT/hc ≈ 200 cm−1. The rotational con- 1
stants of many molecules are close to 1 cm−1 (Table 11C.1) and 0
often smaller (though the very light H2 molecule, for which (a) (b)
B  60.9 cm 1, is one important exception). It follows that many Figure 13B.7 The calculation of the rotational partition function
rotational levels are populated at normal temperatures. When includes a contribution, indicated by the circles, for all possible
that is the case, explicit expressions for the rotational partition combinations of J and K. The sum is formed either (a) by allowing J
function can be derived. to take the values 0, 1, 2,… and then for each J allowing K to range
Consider a linear rotor. If kT >> hcB many rotational states from J to −J, or (b) by allowing K to range from −∞ to ∞, and for
are occupied the sum that defines the partition function can be each value of K allowing J to take the values |K|, |K| + 1,…, ∞. The
approximated by an integral: tinted areas show in (a) the sum from K = −2 to +2 for J = 2, and in
(b) the sum with J = 2, 3 … for K = −2.
 
q R  0 (2 J  1)e   hcBJ ( J 1) dJ

This integral can be evaluated without much effort by making with J = 0, 1, 2,…, K = J, J − 1,…,−J, and MJ = J, J − 1,…, −J.
the substitution x   hcBJ ( J  1), so that dx /dJ   hcB (2 J  1) Instead of considering these ranges, the same values can be
and therefore (2 J  1)dJ  dx / hcB . Then covered by allowing K to range from −∞ to ∞, with J confined
to |K|, |K| + 1,…, ∞ for each value of K (Fig. 13B.7).
Integral E.1:
1 Step 1 Write an expression for the sum over energy states
 
1  1
 Because the energy is independent of MJ, and there are 2J + 1
R x
q  
e dx 
 hcB 0  hcB values of MJ for each value of J, each value of J is (2J + 1)-fold
degenerate. It follows that the partition function
which (because β = 1/kT) is
 J J
  E J, K , M J
kT Rotational partition function q  e
qR =  [unsymmetrical linearr molecule] (13B.12a) J  0 K  J M J  J
hcB
can be written equivalently as
Brief illustration 13B.2  J  
  E J, K , M J   E J, K , M J
1 35 −1
q    (2 J  1) e    (2 J  1) e
For H Cl at 298.15 K, use kT/hc = 207.225 cm and J  0 K  J K  J |K |
B  10.591 cm 1 . Then  

e  hc  ( A  B ) K 2
 (2 J  1) e
 ( J 1)
 hc  BJ

1
kT 207.225 cm K  J | K |
qR    19.57
hcB 10.591 cm 1
Step 2 Convert the sums to integrals
The value is in good agreement with the exact value (19.902) As for linear molecules, assume that the temperature is so high
and obtained with much less effort. that numerous states are occupied, in which case the sums may
be approximated by integrals. Then
A similar but more elaborate approach can be used to find   
q   e hc  ( A  B )K


2 
(2 J  1) e  hc  BJ ( J 1)dJdK
an expression for the rotational partition function of nonlinear K

molecules.
Step 3 Evaluate the integrals
How is that done? 13B.2 Deriving an expression for the You should recognize the integral over J as the integral of the
derivative of a function, which is the function itself, so
rotational partition function of a nonlinear molecule
Consider a symmetric rotor (Topic 11B) for which the energy |K| >>1 for most allowed values
levels are
 ( J  1)  hc( A  B )K 2 1 − hcβ B% K ( K +1) 1 − hcβ BK

% % 2
E  hcBJ (2 J +1)e − hcβ BJ ( J +1) d J = % e % e
J, K , M J K hcβ B hcβ B
556 13 Statistical thermodynamics

Use this result for the integral over J in Step 2: the large number of rotational states that are accessible at nor-
Integral G.1 mal temperatures.
  
1  1  It is important not to include too many rotational states in
 hc  ( A  B ) K 2  hc  BK  2

hc  B 
 2

hc  B 
 hc  AK
q e e dK  e dK the sum that defines the partition function. For a homonuclear
1/ 2 diatomic molecule or a symmetrical linear molecule (such as
1    CO2 or HC≡CH), a rotation through 180° results in an indistin-
  
hc  B  hc  A  guishable state of the molecule. Hence, the number of thermally
accessible states is only half the number that can be occupied
now rearrange this expression into
by a heteronuclear diatomic molecule, where rotation through
1   
1/ 2 3/2 1/ 2 180° does result in a distinguishable state. Therefore, for a sym-
 kT    
q  metrical linear molecule,
(hc  )3/ 2  AB
  2     2 
 hc   AB 
kT T Rotational partition function
For an asymmetric rotor with its three moments of inertia, one qR    (13B.13a)
 to give
of the B is replaced by C, 2hcB 2 R [symmetrical linear rotor]

3/2 1/2
The equations for symmetrical and non-symmetrical molecules
π (13B.12b)
q R = kT can be combined into a single expression by introducing the
hc %% %
ABC Rotational partition function
[nonlinear molecule] symmetry number, σ, which is the number of indistinguish-
able orientations of the molecule. Then

A useful way of expressing the temperature above which eqns T Rotational partition function
qR   (13B.13b)
13B.12a and 13B.12b are valid is to introduce the characteristic  R [linear rotor]
rotational temperature,  R  hcB /k . Then ‘high temperature’
For a heteronuclear diatomic molecule σ = 1; for a homonuclear
means T >> θR and under these conditions the rotational parti-
diatomic molecule or a symmetrical linear molecule, σ = 2.
tion function of a linear molecule is
The formal justification of this rule depends on assessing the
1/

R
role of the Pauli principle.
k T
qR   T  R
hcB 
For a nonlinear molecule a characteristic rotational tempera- How is that done? 13B.3 Identifying the origin of the
ture is defined in the same way for each of the rotational con-
 B,  Some typical values of symmetry number
stants:  R  hcX /k , where X is A,  or C.
θ are given in Table 13B.1. The value for 1H2 (87.6 K) is abnor-
R The Pauli principle forbids the occupation of certain states.
mally high, so the approximation must be used carefully for this It is shown in Topic 11B, for example, that 1H2 may occupy
molecule. However, before using eqn 13B.12a for symmetrical rotational states with even J only if its nuclear spins are paired
molecules, such as H2, read on (to eqn 13B.13a). (para-hydrogen), and odd J states only if its nuclear spins are
The general conclusion at this stage is that parallel (ortho-hydrogen). In ortho-H2 there are three nuclear
spin states for each value of J (because there are three ‘paral-
Molecules with large moments of inertia (and hence small lel’ spin states of the two nuclei); in para-H2 there is just one
rotational constants and low characteristic rotational nuclear spin state for each value of J.
temperatures) have large rotational partition functions. To set up the rotational partition function and take into
account the Pauli principle, note that ‘ordinary’ molecular
A large value of qR reflects the closeness in energy (compared
hydrogen is a mixture of one part para-H2 (with only its even-J
with kT) of the rotational levels in large, heavy molecules, and
rotational states occupied) and three parts ortho-H2 (with only
its odd-J rotational states occupied). Therefore, the average
partition function for each molecule is
Table 13B.1 Rotational temperatures of diatomic molecules⋆
 (2 J  1)e  (2 J  1)e
 ( J 1)
  hcBJ  ( J 1)
  hcBJ
q R  14  34
even J odd J
θR/K
1
H2 87.6 The odd-J states are three times more heavily weighted than the
1 35
H Cl 15.2
even-J states (Fig. 13B.8). The illustration shows that approxi-
14
mately the same answer would be obtained for the partition
N2 2.88
35
function (the sum of all the populations) if each J term contrib-
Cl2 0.351
uted half its normal value to the sum. That is, the last equation

More values are given in the Resource section, Table 11C.1. can be approximated as
13B Molecular partition functions 557

Odd J
Even J

ortho-H2
Contributioni to q

Contributioni to q
para-H2

0 1 Rotational quantum number J 0 1 Rotational quantum number J


Figure 13B.8 The values of the individual terms (2 J  1)e
  hcBJ ( J 1)
Figure 13B.9 The values of the individual terms contributing to
contributing to the mean partition function of a 3:1 mixture of the rotational partition function of CO2. Only states with even J
ortho- and para-H2, but with a much smaller rotational constant values are allowed. The full line shows the smoothed, averaged
than in reality (so as to illustrate a procedure that is in fact valid contributions of the levels.
only for heavier homonuclear diatomic molecules). The partition
function is the sum of all these terms. At high temperatures, the Table 13B.2 Symmetry numbers of molecules⋆
sum is approximately equal to the sum of the terms over all values
of J, each with a weight of 21 . This sum is indicated by the curve. σ
1
H2 2
1
H2H 1
NH3 3
q R  12 (2 J  1)e   hcBJ ( J 1)

C6H6 12
J

More values are given in the Resource section, Table 11C.1.
and this approximation is very good when many terms con-
tribute (at high temperatures, T >> θR). At such high tem-
peratures the sum can be approximated by the integral that Brief illustration 13B.3
led to eqn 13B.12a. Therefore, on account of the factor of 12 in
this expression, the rotational partition function for ‘ordinary’ The value σ(H2O) = 2 reflects the fact that a 180° rotation
molecular hydrogen at high temperatures is one-half this value, about the bisector of the H–O–H angle interchanges two indis-
as in eqn 13B.13a. tinguishable atoms. In NH3, there are three indistinguishable
orientations around the axis shown in (1). For CH4, any of
three 120° rotations about any of its four C–H bonds leaves
the molecule in an indistinguishable state (2), so the symmetry
The same type of argument may be used for linear symmet- number is 3 × 4 = 12.
rical molecules in which identical bosons are interchanged
by rotation (such as C16O2). As pointed out in Topic 11B, if
the nuclear spin of the bosons is 0, then only even-J states C3

are admissible. Because only half the rotational states are oc- C3 D

cupied, the rotational partition function is only half the value


of the sum obtained by allowing all values of J to contribute A B
(Fig. 13B.9). A
B
The same care must be exercised for other types of symmetri-
C C
cal molecules, and for a nonlinear molecule eqn 13B.12b is re- 1 2
placed by
3/2 1/ 2
1  kT     Rotational partition funcction For benzene, any of six orientations around the axis per-
qR   (13B.14)
  hc   
 ABC  [nonlinear rotor] pendicular to the plane of the molecule leaves it apparently
unchanged (Fig. 13B.10), as does a rotation of 180° around any
Some typical values of the symmetry numbers are given in of six axes in the plane of the molecule (three of which pass
Table 13B.2. To see how group theory is used to identify the through C atoms diametrically opposite across the ring and the
value of the symmetry number, see Integrated activity I13.1; remaining three pass through the mid-points of C–C bonds on
Brief illustration 13B.3 outlines the approach. opposite sides of the ring).
558 13 Statistical thermodynamics

10

1 2 3 4

Partition function, q V
5 6 7 8 5

9 10 11 12
1
0
0 5 10
Temperature, kT/hc %
Figure 13B.10 The 12 equivalent orientations of a benzene
molecule that can be reached by pure rotations and give rise to a Figure 13B.11 The vibrational partition function of a molecule
symmetry number of 12. The plus signs indicate the underside of in the harmonic approximation. Note that the partition function
the hexagon after that face has been rotated into view. is proportional to the temperature when the temperature is high
(T >> θV ).

In a polyatomic molecule each normal mode (Topic 11D)


has its own partition function (provided the anharmonicities
13B.2(c) The vibrational contribution are so small that the modes are independent). The overall vi-
The vibrational partition function of a molecule is calculated brational partition function is the product of the individual
by substituting the measured vibrational energy levels into the partition functions, and so qV = qV(1)qV(2) …, where qV(K) is
definition of qV, and summing them numerically. However, the partition function for normal mode K and is calculated by
provided it is permissible to assume that the vibrations are direct summation of the observed spectroscopic levels.
harmonic, there is a much simpler way. In that case, the vibra-
tional energy levels form a uniform ladder of separation hcν
Example 13B.2 Calculating a vibrational partition
(Topics 7E and 11C), which is exactly the problem discussed
earlier and leading to eqn 13B.2a. Therefore that result can be function
used by setting   hc , giving The wavenumbers of the three normal modes of H2O are
3656.7 cm−1, 1594.8 cm−1, and 3755.8 cm−1. Evaluate the vibra-
1 Vibrational partition function
qV  [harmonic appro oximation]  (13B.15) tional partition function at 1500 K.
1  e   hc
Collect your thoughts You need to use eqn 13B.15 for each
This function is plotted in Fig. 13B.11 (which is essentially the mode, and then form the product of the three contributions.
same as Fig. 13B.2). Similarly, the population of each state is At 1500 K, kT/hc = 1042.6 cm−1.
given by eqn 13B.2b.
The solution Draw up the following table displaying the contri-
butions of each mode:

Brief illustration 13B.4 Mode 1 2 3


 /cm 1 3656.7 1594.8 3755.8
To calculate the partition function of I2 molecules at 298.15 K
note that their vibrational wavenumber is 214.6 cm−1. Then, hcν /kT 3.507 1.530 3.602
because at 298.15 K, kT/hc = 207.225 cm−1, qV 1.031 1.277 1.028

hc 214.6 cm 1 The overall vibrational partition function is therefore


 hc    1.035
kT 207.225 cm 1
q V  1.031  1.277  1.028  1.353
Then it follows from eqn 13B.15 that
The three normal modes of H2O are at such high wavenumbers
V 1 that even at 1500 K most of the molecules are in their vibra-
q   1.55
1  e 1.035 tional ground state.

From this value it can be inferred that only the ground and first Comment. There may be so many normal modes in a
excited states are significantly populated. large molecule that their overall contribution may be sig-
nificant even though each mode is not appreciably excited.
13B Molecular partition functions 559

For ­example, a nonlinear molecule containing 10 atoms has condition is satisfied eqn 13B.16 is valid and can be written
3N − 6 = 24 normal modes (Topic 11D). If a value of about qV = T/θV (the analogue of the rotational expression).
1.1 is assumed for the vibrational partition function of one
normal mode, the overall vibrational partition function is
about qV ≈ (1.1)24 = 9.8. 13B.2(d) The electronic contribution
Self-test 13B.2 Repeat the calculation for CO2 at the same Electronic energy separations from the lowest energy level are
temperature. The vibrational wavenumbers are 1388 cm , −1 usually very large, so for most cases qE = g0 because only the
667.4 cm−1, and 2349 cm−1, the second being the doubly degen- lowest level, with degeneracy g0, is occupied. Noble gas atoms
erate bending mode which contributes twice to the overall have non-degenerate electronic ground states and so have
vibrational partition function. qE = 1. In contrast, in alkali metal atoms the lowest level is dou-
bly degenerate (corresponding to the two orientations of their
electron spin), so qE = 2.
Answer: 6.79

In many molecules the vibrational wavenumbers are so great Brief illustration 13B.5
that  hc  1. For example, the lowest vibrational wavenumber
of CH4 is 1306 cm−1, so  hc  6.3 at room temperature. Most Some atoms and molecules have degenerate ground states and
C–H stretches normally lie in the range 2850–2960 cm−1, so for low-lying electronically excited degenerate states. An example
them  hc  14. In these cases, e   hc in the denominator of qV is NO, which has a configuration of the form …π1 (Topic
is very close to zero (e.g. e−6.3 = 0.002), and the vibrational par- 11F). The energy of the two degenerate states in which the
orbital and spin momenta are parallel (giving the 2Π3/2 term,
tition function for a single mode is very close to 1 (qV = 1.002
Fig. 13B.12) is slightly greater than that of the two degenerate
when  hc  6.3), implying that only the lowest level is signifi-
states in which they are antiparallel (giving the 2Π1/2 term).
cantly occupied.
The separation, which arises from spin–orbit coupling, is only
Now consider the case of modes with such low vibrational
121 cm−1.
frequencies that  hc  1. When this condition is satisfied, the If the energies of the two levels are denoted E1/2 = 0 and
expression for the partition function in eqn 13B.5 may be ap- E3/2 = ε, then the partition function is
proximated by expanding the exponential (e x  1  x  ) and
retaining just the linear term: ge   i
qE  i  2  2e  
levels i
1 1
qV   This function is plotted in Fig. 13B.13. At T = 0, qE = 2, because
1  e   hc 1  (1   hc  )
only the doubly degenerate ground state is accessible. At high
That is, for low-frequency modes at high temperatures such temperatures, qE approaches 4 because all four states are acces-
that  hc  1 sible. At 25 °C, qE = 3.1.

kT Vibrational partition function


qV  [high-temperature approximation] (13B.16)
hc
S
The temperatures for which eqn 13B.16 is valid can be ex- 2
3/2
pressed in terms of the characteristic vibrational temperature,
L
 V  hc /k (Table 13B.3). The value for H2 (6332 K) is abnor-
mally high because the atoms are so light and the vibrational S
frequency is correspondingly high. In terms of the vibrational
121.1 cm–1

S
temperature, ‘high temperature’ means T >> θV, and when this
L L

S
Table 13B.3 Vibrational temperatures of diatomic molecules⋆ 1/2 L
2

θV/K
1
H2 6332
1 35
H Cl 4304 Figure 13B.12 The doubly degenerate ground electronic level of
14 NO (with the spin and orbital angular momentum around the axis
N2 3393
35
in opposite directions) and the doubly degenerate first excited
Cl2 805
level (with the spin and orbital momenta parallel). The upper level

More values are given in the Resource section, Table 11C.1 is thermally accessible at room temperature.
560 13 Statistical thermodynamics

4
Exercises
E13B.3 Calculate (i) the thermal wavelength, (ii) the translational partition
Partition function, q E

function at 300 K and 3000 K of a molecule of molar mass 150 g mol−1 in a


container of volume 1.00 cm3.
3 E13B.4 The bond length of O2 is 120.75 pm. Use the high-temperature
approximation to calculate the rotational partition function of the molecule
at 300 K.
E13B.5 Estimate the rotational partition function of ethene at 25 °C given that
A  4.828 cm 1 , B  1.0012 cm 1, and C  0.8282 cm 1 . Take the symmetry
number into account.
2
5 10 E13B.6 Calculate the vibrational partition function of CS2 at 500 K
0 Temperature, kT/ε given the wavenumbers of the vibrational modes as 658 cm−1 (symmetric
stretch), 397 cm−1 (bend; doubly degenerate), 1535 cm−1 (asymmetric
Figure 13B.13 The variation with temperature of the electronic stretch).
partition function of an NO molecule. Note that the curve
E13B.7 A certain atom has a fourfold degenerate ground level, a non-
resembles that for a two-level system (Fig. 13B.4), but rises from
degenerate electronically excited level at 2500 cm−1, and a twofold degenerate
2 (the degeneracy of the lower level) and approaches 4 (the total level at 3500 cm−1. Calculate the electronic partition function at 1900 K. What
number of states) at high temperatures. is the relative population of each level at 1900 K?

Checklist of concepts
☐ 1. The molecular partition function is an indication of the ☐ 4. The vibrational partition function of a molecule is
number of thermally accessible states at the temperature found by evaluating the contribution from each normal
of interest. mode treated as a harmonic oscillator.
☐ 2. If the energy of a molecule is given by the sum of con- ☐ 5. Because electronic energy separations from the ground
tributions from different modes, then the molecular state are usually very large, in most cases the electronic
partition function is a product of the partition functions partition function is equal to the degeneracy of the low-
for each of the modes. est level.
☐ 3. The symmetry number takes into account the number of
indistinguishable orientations of a symmetrical molecule.

Checklist of equations
Property Equation Comment Equation number
  i
Molecular partition function q  e Definition, independent molecules 13B.1a
states i

q   g i e   i
Definition, independent molecules 13B.1b
levels i

Uniform ladder q = 1/(1 − e−βε) 13B.2a


−βε
Two-level system q=1+e 13B.3a
1/2
Thermal wavelength Λ = h/(2πmkT) 13B.7
Translation qT = V/Λ3 13B.10b
Rotation q  kT / hcB
R R
T >> θ , linear rotor 13B.13a
   )1/ 2
q R  (1/ )(kT /hc)3/2 (/ABC T >> θR, nonlinear rotor  R  hcX /k , X = A , B , or C 13B.14
Vibration q V  1/(1  e   hc ) Harmonic approximation,  V  hc/k 13B.15
TOPIC 13C Molecular energies

E 1
    N i  i  (13C.1)
➤ Why do you need to know this material? N N i
For statistical thermodynamics to be useful, you need to
In Topic 13A it is shown that at temperature T the popula-
know how to use the partition function to calculate the
tions of the states are given by the Boltzmann distribution, eqn
values of thermodynamic quantities.
13A.10b, N i /N  (1/q )e  i , so
➤ What is the key idea? 1
q
  i
The average energy of a molecule in a collection of inde-
   e i  (13C.2)
i
pendent molecules can be calculated from the molecular
partition function alone. with β = 1/kT. This expression can be manipulated into a form
involving only q. First note that
➤ What do you need to know already?
You need to know how to calculate a molecular partition d  i
 i e  i   e
function from computed or spectroscopic data (Topic 13B) d
and understand its significance as a measure of the n
­ umber
of accessible states. The Topic also draws on expressions It follows that
for the rotational and vibrational energies of molecules q

(Topics 11B–11D). 1
d  i 1 d 1 dq
     e   e  i    (13C.3)
q i d q d i q d

Several points need to be made in relation to eqn 13C.3.


A partition function in statistical thermodynamics is like a Because ε0 ≡ 0, (all energies are measured from the lowest
wavefunction in quantum mechanics. A wavefunction contains available level), 〈ε〉 is the value of the energy relative to the
all the dynamical information about a system; a partition func- actual ground-state energy. If the lowest energy of the mol-
tion contains all the thermodynamic information about a sys- ecule is in fact εgs rather than 0, then the true mean energy is
tem. This Topic shows how one of the simplest thermodynamic εgs + 〈ε〉. For instance, for a harmonic oscillator, εgs is the zero-
properties, the mean energy of the molecules, is calculated point energy, 12 hcν .
from a knowledge of the partition function. The relations de- Secondly, because the partition function might depend on
veloped here are for a system composed of independent, non- variables other than the temperature (e.g. the volume), the
interacting molecules. derivative with respect to β in eqn 13C.3 is actually a partial
derivative with these other variables held constant. The com-
plete expression relating the molecular partition function to the
mean energy of a molecule is therefore
13C.1 The basic equations
1  q 
    gs   Mean molecular energy  (13C.4a)
Consider a collection of N molecules that do not interact with q   V
one another. Any member of the collection can exist in a state i
of energy εi measured from the lowest energy state of the mol- An equivalent form is obtained by noting that dx/x = d ln x:
ecule. The total energy of the collection, E, is
  ln q 
E  N i  i     gs    Mean molecular energy (13C.4b)
i
  V

where Ni is the population of state i. The mean energy of a mol- These two equations confirm that it is only necessary to know
ecule, 〈ε〉, relative to its energy in its ground state, is the total en- the partition function as a function of temperature in order to
ergy of the collection divided by the total number of molecules: calculate the mean energy.
562 13 Statistical thermodynamics

Brief illustration 13C.1 Contributions of the


13C.2
If a molecule has only two available states, one at 0 and the fundamental modes of motion
other at an energy ε, its partition function is
In Topic 13B it is established that each fundamental contribu-
q  1  e   tion to the energy, namely translation (T), rotation (R), vibra-
Therefore, the mean energy of a collection of these molecules tion (V), electronic (E), and electron spin (S) can, to a good
at a temperature T is, from eqn 13C.4a, approximation, be treated separately.

   
1 d(1  e   )  e  
 
 13C.2(a) The translational contribution
1  e   d 1  e   e   1
For a one-dimensional container of length X, the partition
In the final step the numerator and denominator of the frac- function is qXT  X / with   h 1/ 2 /(2m)1/ 2 (Topic 13B, with
tion are multiplied by e   . The expression for 〈ε〉 is plotted ‘constant volume V’ replaced by ‘constant length X’). The par-
in Fig. 13C.1. Notice how the mean energy is zero when tition function can be written as A 1/ 2 where A is a constant
kT   , when only the lower state (at the zero of energy) is independent of temperature. This form is convenient for the
occupied, and starts to rise significantly when kT   . The calculation of the average energy by using eqn 13C.4b:
mean energy tends to 12 ε when kT   ; in this limit the two
states become equally populated.   ln q    ln( A 1/ 2 ) 
 XT        
   X   X
 (ln A  ln  1/ 2 ) 
0.4 0.6   
  X
0.5
    12 ln    1
   
   2 
0.4  X
Energy, ε/ε

That is,
0.2
Mean translational energy
0.2
 XT   12 kT [one dimension]  (13C.5a)

Motion along x, y, and z are independent of each other, so each


one contributes 12 kT to the mean energy. For a molecule free to
0 0 translate in three dimensions the mean energy is therefore
0 0.5 1 0 5 10
Temperature, kT/ε Temperature, kT/ε Mean translational energy
 T   23 kT [three dimensions]  (13C.5b)
Figure 13C.1 The variation with temperature of the mean energy
of a two-level system. The graph on the left shows the slow rise This result is in agreement with the equipartition theorem
away from zero energy at low temperatures; the slope of the
(see Energy: A first look). The classical expression for the en-
graph at T = 0 is 0. The graph on the right shows that as kT/ε
ergy of translation in one dimension is Ek, x = 12 mvx2 which has
becomes large 〈ε〉/ε approaches 0.5. In the limit both states are
equally populated. one quadratic contribution. It follows from the equipartition
theorem that the mean translational energy in one dimension is
1
2
kT , and in three dimensions the mean energy is simply three
times this. The derivation of the expression for the translational
partition function assumes that many translational states are
occupied, so the equipartition theorem is expected to apply.
Exercises
E13C.1 Compute the mean energy at 298 K of a two-level system of energy
separation equivalent to 500 cm−1; the levels are non-degenerate.
13C.2(b) The rotational contribution
E13C.2 Derive an expression for the mean energy of a two-level system in The mean rotational energy of a linear molecule is obtained
which the ground level has energy zero and degeneracy g0, and the upper from the rotational partition function (eqn 13B.11):
level has energy ε and degeneracy g1. Hint: start by writing an expression
q R  (2 J  1)e   hcBJ ( J 1)

for the partition function and then follow the approach used in Brief
illustration 13C.1. J
13C Molecular energies 563

When the temperature is not high (in the sense that it is not and therefore that
true that T   R  hcB /k ) the series must be summed term
1 Mean rotational energy
by term, which for a heteronuclear diatomic molecule or other  R    kT R  (13C.6b)
 [linear molecule, T >>  ]
non-symmetrical linear molecule gives
The high-temperature result, which is valid when many rota-
 
q R  1  3e 2  hcB  5e 6  hcB   tional states are occupied, is also in agreement with the equi-
partition theorem (there are two quadratic contributions).
Hence, because

dqR Brief illustration 13C.2


 hcB (6e 2  hcB  30e 6  hcB  )
 

d
To estimate the mean energy of a nonlinear molecule in the
(qR is independent of V, so the partial derivative has been re- high-temperature limit recognize that its rotational kinetic
placed by a complete derivative) it follows that energy (the only contribution to its rotational energy) is
Ek  12 I aa2  12 I bb2  12 I cc2 . As there are three quadratic con-
hcB (6e 2  hcB  30e 6  hcB  )
 
1 dq
R
tributions, its mean rotational energy is 23 kT . The molar con-
 R       tribution is 23 RT. At 25 °C, this contribution is 3.7 kJ mol−1,
q R d 1  3e 2  hcB  5e 6  hcB  
the same as the translational contribution, giving a total of

Mean rotational energy
(13C.6a) 7.4 kJ mol−1. A monatomic gas has no rotational contribution.
[unsymmetrical linear moleccule]

This ungainly function is plotted in Fig. 13C.2. If the tempera-


ture is much greater than the characteristic rotational tempera- 13C.2(c) The vibrational contribution
ture θR ( R  hcB /k ), qR is given by eqn 13B.13b, q R  T / R .
The vibrational partition function in the harmonic approxima-
The rotational partition function can be expressed in terms of
tion is given in eqn 13B.15, q V  1/(1  e   hc ). Because qV is in-
β and B as follows
dependent of the volume, it follows that
T kT 1
qR  

 d(1/f )/dx = −(1/f 2)df /dx
 R
 hcB  hcB
where σ = 1 for a heteronuclear diatomic molecule. It then fol- d qV d 1 hc%e − βhc
%
= = − (13C.7)
lows that dβ dβ 1− e − β hc
%
(1− e − βhc )2
%

1/

2
and hence
R
1 d q d  1  d 1
 R    R   hcB     1 dq
V
hc e   hc
q d d   hcB  d   V     (1  e   hc
)
q V d (1  e   hc )2
hc e   hc

1.5 1  e   hc
~
Mean energy, εR /hcB

The final result, after multiplying the numerator and denomi-


nator by e  hc is
1

hc Mean vibrational energy


 V    (13C.8)
e hc
1 [harmonic approximmation]
0.5
The zero-point energy, 12 hcν , can be added to the right-hand
side if the mean energy is to be measured from 0 rather than
0 the lowest attainable level (the zero-point level). The vari-
0 1 2
Temperature, T/θR ation of the mean energy with temperature is illustrated in
Figure 13C.2 The variation with temperature of the mean
Fig. 13C.3.
rotational energy of an unsymmetrical linear rotor. At high If the temperature is much greater than the characteristic vi-
temperatures (T >> θR), the energy is proportional to the brational temperature θV ( V  hc /k ) it follows that T  hc /k
temperature, in accord with the equipartition theorem. The which can be expressed as  hc  1.The exponential function
limiting slope is 1, which corresponds to a mean energy of kT. in eqn 13C.8 can be expanded (e x  1  x  ) and, because
564 13 Statistical thermodynamics

10 The addition of the zero-point energy (corresponding to


1
2
 214.6 cm 1) increases this value to 225.4 cm−1.
The mean molar vibrational energy (excluding the zero-
Mean energy, 〈εV〉/hcν
~

point energy) is 1.41 kJ mol−1, which is somewhat less than the


equipartition value of 2.48 kJ mol−1. For N2 molecules, with a
5 very much higher vibrational wavenumber of 2358 cm−1, the
molar vibrational energy is only 0.32 J mol−1.

0 When there are several normal modes that can be treated


0 5 10
Temperature, T/θV
as harmonic, the overall vibrational partition function is the
product of each individual partition function, and the total
Figure 13C.3 The variation with temperature of the mean mean vibrational energy is the sum of the mean energy of
vibrational energy of a molecule in the harmonic approximation. each mode.
At high temperatures (T >> θV ), the energy is proportional to
the temperature, in accord with the equipartition theorem. The
limiting slope is 1, which corresponds to a mean energy of kT. 13C.2(d) The electronic contribution
In most cases of interest, the electronic states of atoms and mol-
x << 1, all but the two leading terms discarded. This approxima- ecules are so widely separated that only the electronic ground
tion leads to state is occupied. All energies are measured from the ground
hc 1 state of each mode, so it follows that
 V     kT
(1   hc  )  1 
 E   0 Mean electronic energy  (13C.10)
Mean vibrational energy [ T >>  ]
V
(13C.9)
In certain cases, there are thermally accessible states at the
This result is in agreement with the value predicted by the clas- temperature of interest. In that case, the partition function and
sical equipartition theorem (two quadratic contributions). Bear hence the mean electronic energy are best calculated by direct
in mind, however, that the condition T >> θV is rarely satisfied. summation over the available states. Care must be taken to take
any degeneracies into account, as illustrated in the following
Brief illustration 13C.3 example.

To calculate the mean vibrational energy of I2 molecules at


298.15 K note that their vibrational wavenumber is 214.6 cm−1
and that k/hc = 0.6950 cm−1 K−1. The characteristic vibrational Example 13C.1
temperature is Calculating the electronic contribution to
the energy
hc 214.6 cm 1
V    309 K The ground electronic level of a certain atom is doubly degen-
k 0.6950 cm 1 K 1 erate and there is an excited level that is fourfold degenerate at
Therefore at 298 K the temperature is not large compared with  /hc    600 cm 1 above the ground level. What is its mean
θV so eqn 13C.9 (the equipartition value) does not apply and electronic energy at 25 °C, expressed as a wavenumber?
eqn 13C.8 must be used instead. Collect your thoughts You need to write the expression for the
partition function at a general temperature T (in terms of β)
hc  and then derive the mean energy by using eqn 13C.3. Doing so
 hc  
kT (k /hc)T involves differentiating the partition function with respect to
214.6 cm 1 β. Finally, substitute the data. Use   hc ,  E   hc  E , and
  1.035 kT/hc = 207.225 cm−1 at 25 °C.
(0.6950 cm 1 K 1 )  (298.15 K )
The solution The partition function is written as a sum over lev-
 V   214.6 cm 1 els by using eqn 13B.1b, noting that g0 = 2 and g1 = 4. Therefore
  hc  1.035  118.1 cm 1
hc e 1 e 1 the partition function is qE = 2 + 4e−βε and the mean energy is
13C Molecular energies 565

  g e B
 e 
4  
g e B B e
  
B

E

1 dq 1 d   1 dq S
1 d
 E     (2  4e )  S     (1  e   g e BB )
q E d 2  4e   d q d
S
1 e   g e B B
d
4 e   
  g e BBe   g e BB
2  4e   12 e   1 
1  e   g e B B
The energy separation ε is expressed as a wavenumber ν using The final expression, after multiplying the numerator and de-
  hc , and likewise the mean energy is expressed as a wave- nominator by e  g e BB , is
number by writing  E    E  /hc
g B
 E   /hc   S   e B
 g e B B
Mean spin energy  (13C.13)
  
E
 1   1 hc /kT e 1
hc 2
e 1 2 e 1
This function is essentially the same as that plotted in Fig. 13C.1.
From the data,
600 cm 1 Brief illustration 13C.4
 E   ( 600 cm 1 ) / ( 207.225 cm 1 )
 59.7 cm 1
1
2
e 1
Suppose a collection of radicals is exposed to a magnetic field
Self-test 13C.1 Repeat the problem for an atom that has a of 2.5 T (T denotes tesla) at 25 °C. With μB = 9.274 × 10−24 J T −1,
threefold degenerate ground level and a sevenfold degenerate
excited level 400 cm−1 above. g e BB  2.0023  (9.274  1024 JT 1 )  (2.5 T)  4.64 1023 J
2.0023  (9.274  1024 JT 1 )  (2.5 T)
 g e BB 
Answer: 101 cm−1
 0.0112
(1.381  1023 JK 1 )  (298 K )

13C.2(e) The spin contribution The mean energy is therefore

An electron spin in a magnetic field B has two possible energy 4.64 1023 J
 S    2.31  1023 J
states that depend on its orientation as expressed by the mag- e0.0112  1
netic quantum number ms, and which are given by This energy is equivalent to 13.9 J mol−1 (note joules, not
Ems  g e BBms Electron spin energies  (13C.11) kilojoules).

where μB is the Bohr magneton and ge = 2.0023. These energies


are discussed in more detail in Topic 12A. The lower state has
ms   12 , so the two energy levels available to the electron lie Exercises
(according to the convention that the lowest state has energy E13C.3 Evaluate the mean vibrational energy of Br2 at 400 K (use
0) at ε−1/2 = 0 and at  1/ 2  g e BB. The spin partition function   323.2 cm 1). Compare your result with the equipartition value and
comment on the difference.
is therefore
E13C.4 A certain atom has a fourfold degenerate ground level, a non-
q S  1  e  g  B e B
Spin partition function  (13C.12) degenerate electronically excited level at 2500 cm−1, and a twofold degenerate
level at 3500 cm−1. Calculate the mean contribution to the electronic energy at
The mean energy of the spin is therefore 1900 K.

Checklist of concepts
☐ 1. The mean molecular energy can be calculated from the ☐ 3. In the high temperature limit, the results obtained for
molecular partition function. the mean molecular energy are in accord with the equi-
☐ 2. Individual contributions to the mean molecular energy partition principle.
from each mode of motion are calculated from the rel-
evant partition functions,
566 13 Statistical thermodynamics

Checklist of equations⋆
Property Equation Comment Equation number
Mean energy 〈ε〉 = εgs − (1/q)(∂q/∂β)V 13C.4a
〈ε〉 = εgs − (∂ ln q/∂β)V Alternative version 13C.4b
Translation    d(kT /2)
T
In d dimensions, d = 1, 2, 3 13C.5
Rotation 〈εR〉 = kT Linear molecule, T >> θR 13C.6b
Vibration  V  hc /(e  hc  1) Harmonic approximation 13C.8
〈εV〉 = kT T >> θV 13C.9
 g e BB
Spin    g e BB /(e
S
1) Electron in a magnetic field 13C.13


β = 1/kT
TOPIC 13D The canonical ensemble

Energy
1 2 3 4 5 6 7 8 9 10
10
➤ Why do you need to know this material?
N
Whereas Topics 13B and 13C deal with independent mol- V
ecules, in practice molecules do interact. Therefore, this T
material is essential for applying statistical thermodynam-
ics to real gases, liquids, and solids and to any system in
which intermolecular interactions cannot be neglected.

➤ What is the key idea? 20


11 12 13 14 15 16 17 18 19
A system composed of interacting molecules is described
in terms of a canonical partition function, from which the Figure 13D.1 A representation of the canonical ensemble, in this
thermodynamic properties of the system may be deduced. case for N = 20. The individual replications of the actual system
all have the same composition and volume. They are in thermal
➤ What do you need to know already? contact with one another and so can exchange energy as heat,
and because they are in thermal equilibrium with one another
The calculations here, which are not carried through in
they all have the same temperature. The total energy of all 20
detail, are essentially the same as in Topic 13A. Calculations
replications is a constant because the ensemble is isolated overall.
of mean energies are also essentially the same as in
Topic 13C and are not repeated in detail.
There are two other important types of ensembles. In the
­ icrocanonical ensemble the condition of constant tempera-
m
ture is replaced by the requirement that all the systems should
have exactly the same energy: each system is individually iso-
The crucial concept needed in the treatment of systems of in- lated. In the grand canonical ensemble the volume and tem-
teracting particles, as in real gases and liquids, is the ‘ensemble’. perature of each system is the same, but they are open, which
Like so many scientific terms, the term has basically its normal means that matter can be imagined as able to pass between
meaning of ‘collection’, but in statistical thermodynamics it has them; the composition of each one may fluctuate, but now the
been sharpened and refined into a precise significance. property known as the chemical potential (μ, Topic 5A) is the
same in each system. In summary:

Ensemble Common properties


13D.1 The concept of ensemble Microcanonical V, E, N
Canonical V, T, N
To set up an ensemble, take a closed system of specified volume,
Grand canonical V, T, μ
composition, and temperature, and think of it as replicated N
times (Fig. 13D.1). All the identical closed systems are regarded
as being in thermal contact with one another, so they can ex- The microcanonical ensemble is the basis of the discussion in
change energy. The total energy of the ensemble is E.  Because Topic 13A; the grand canonical ensemble will not be consid-
the members of the ensemble are all in thermal equilibrium ered explicitly.
with each other, they have the same temperature, T. The vol- The important point about an ensemble is that it is a collec-
ume of each member of the ensemble is the same, so the energy tion of imaginary replications of the system, so the number of
levels available to the molecules are the same in each system, members can be as large as desired; when appropriate, N can be
and each member contains the same number of molecules. This taken as infinite. The number of members of the ensemble in
imaginary collection of replications of the actual system with a a state with energy Ei is denoted N i, and it is possible to speak
common temperature is called the canonical ensemble.1 of the configuration of the ensemble (by analogy with the con-
figuration of the system used in Topic 13A) and its weight, W ,
1
The word ‘canon’ means ‘according to a rule’. the number of ways of achieving the configuration {N 0 ,N 1 ,…}.
568 13 Statistical thermodynamics

Note that N is unrelated to N, the number of molecules in the 13D.1(b)Fluctuations from the most probable
actual system; N is the number of imaginary replications of that distribution
system. In summary:
The canonical distribution in eqn 13D.2 is only apparently an
N is the number of molecules in the system, the same in each exponentially decreasing function of the energy of the system.
member of the ensemble. Just as the Boltzmann distribution gives the occupation of a
T is the common temperature of the members. single state of a molecule, the canonical distribution function
Ei is the total energy of one member of the ensemble (the gives the probability of occurrence of members in a single state
one labelled i). i of energy Ei. There may in fact be numerous states with almost
 identical energies. For example, in a gas the identities of the
N is the number of replicas that have the energy Ei.
i molecules moving slowly or quickly can change without neces-
E is the total energy of the entire ensemble. sarily affecting the total energy. The energy density of states,
N is the total number of replicas (the number of members the number of states in an energy range divided by the width of
of the ensemble). the range (Fig. 13D.2), is a very sharply increasing function of

W is the weight of the configuration {N ,N ,…}. energy. It follows that the probability of a member of an ensem-
0 1
ble having a specified energy (as distinct from being in a speci-
fied state) is given by eqn 13D.2, a sharply decreasing function,
13D.1(a) Dominating configurations multiplied by a sharply increasing function (Fig. 13D.3).
Therefore, the overall distribution is a sharply peaked function.
Just as in Topic 13A, some of the configurations of the ca- That is, most members of the ensemble have an energy very
nonical ensemble are very much more probable than others. close to the mean value.
For instance, it is very unlikely that the whole of the total en-
ergy of the ensemble will accumulate in one system to give
the configuration {N,  0,0,…}. By analogy with the discussion
in Topic 13A, there is a dominating configuration, and ther-
modynamic properties can be calculated by taking the average
over the ensemble using that single, most probable, configura-
Energy

Width of
tion. In the thermodynamic limit of N  , this dominating range
configuration is overwhelmingly the most probable, and domi-
nates its properties. Number of
The quantitative discussion follows the argument in Topic states
13A (leading to the Boltzmann distribution) with the modifica-
tion that N and Ni are replaced by N and N i. The weight W of a
Figure 13D.2 The energy density of states is the number of states
configuration {N 0 ,N 1 ,…} is
in an energy range divided by the width of the range.

N !
W  Weight (13D.1)
N 0 ! N 1 !

Probability Number of
of state states
The configuration of greatest weight, subject to the constraints
Probability
that the total energy of the ensemble is constant at E and that of energy
the total number of members is fixed at N , is given by the
­canonical distribution:

N i e   Ei
 Q  e   Ei Canonical distribution (13D.2)
N Q i
Energy
here the sum is over all members of the ensemble. The quan-
Figure 13D.3 To construct the form of the distribution of
tity Q , which is a function of the temperature, is called the
members of the canonical ensemble in terms of their energies, the
canonical partition function, and β = 1/kT. Like the mo- probability that any one is in a state of given energy (eqn 13D.2) is
lecular partition function, the canonical partition function multiplied by the number of states corresponding to that energy (a
contains all the thermodynamic information about a system steeply rising function). The product is a sharply peaked function at
but allows for the possibility of interactions between the con- the mean energy (here considerably magnified), which shows that
stituent molecules. almost all the members of the ensemble have that energy.
13D The canonical ensemble 569

Brief illustration 13D.1 By the same argument that led to eqn 13C.4a (〈ε〉 = −(1/q)
(∂q/∂β)V, when εgs ≡ 0),
A function that increases rapidly is xn, with n large. A func-
tion that decreases rapidly is e−nx, once again, with n large. The 1  Q    lnQ  Mean energy
 E        of a system 
(13D.5)
product of these two functions, normalized so that the maxima Q   V   V
for different values of n all coincide,
As in the case of the mean molecular energy, the ground-state
f (x )  en x ne nx
energy of the entire system must be added to this expression if
is plotted for three values of n in Fig. 13D.4. Note that the width it is not zero.
of the product does indeed decrease as n increases.

1
Exercise
E
E13D.1 Starting from  E   (1/Q )  Ei e i and the definition of Q in eqn 13D.2,
i

0.8 derive the expression for  E  given in eqn 13D.5. (Hint: use the same approach
10 as in the derivation of eqn 13C.4a.)

0.6
20
f(x)
0.4
50 13D.3Independent molecules
revisited
0.2
When the molecules are in fact independent of each other,
0 Q can be shown to be related to the molecular partition
0 0.5 1 1.5 x 2 2.5 3
­function q.
Figure 13D.4 The product of the two functions discussed in Brief
illustration 13D.1, for three different values of n.
How is that done? 13D.1 Establishing the relation
between Q and q

13D.2 The mean energy of a system There are two cases you need to consider. The first is a system
in which the particles are distinguishable (such as when they
are at fixed locations in a solid). The second is a system in
Just as the molecular partition function q can be used to calcu-
which the particles are indistinguishable (such as when they
late the mean value of a molecular property, so the canonical
are in a gas and able to exchange places).
partition function Q can be used to calculate the mean energy
of an entire system composed of molecules which might or Step 1 Consider a system of independent, distinguishable
might not be interacting with one another. Thus, Q is more gen- ­molecules
eral than q because it does not assume that the molecules are The total energy of a collection of N independent molecules is
independent. Therefore Q can be used to discuss the properties the sum of the energies of the individual molecules. Therefore,
of condensed phases and real gases where molecular interac- the total energy of a state i of the system is w
­ ritten as
tions are important.
 and there are N mem- Ei   i (1)   i (2)     i (N )
The total energy of the ensemble is E,
bers; therefore the mean energy of a member is  E   E /N . The In this expression, εi(1) is the energy of molecule 1 when the
fraction, p i, of members of the ensemble in a state i with energy system is in the state i, εi(2) the energy of molecule 2 when
Ei is given by the analogue of eqn 13A.10b (pi  e  i /q with the system is in the same state i, and so on. The canonical parti-
pi = Ni/N) as tion function is then

e   Ei Q  e  i (1)  i (2)  i ( N )


p i   (13D.3) i
Q
Provided the molecules are distinguishable (in a sense described
Therefore it follows that below), the sum over the states of the system can be reproduced
by letting each molecule enter all its own individual states.
1
 E   p i Ei  E ei
  Ei
 (13D.4) Therefore, instead of summing over the states i of the system,
i Q i sum over all the individual states j of molecule 1, all the states j
570 13 Statistical thermodynamics

of molecule 2, and so on. This rewriting of the original expres-   ln( q N/N !)   (ln q N  ln N !) 
sion leads to  E       
  V   V
q q q  (N ln q ) 
        
          V
Q   e j   e j  e j 
   
 j  j   j    ln q 
      N    N  
  V
and therefore to Q = qN.
Step 2 Consider a system of independent, indistinguishable where the final step uses eqn 13C.4b,    ( ln q / )V . The
­molecules result shows that the mean energy of the gas is N times the
If all the molecules are identical and free to move through mean energy of a single molecule, which is the expected result
space, it is not possible to distinguish them and the relation for non-interacting molecules.
Q = qN is not valid. Suppose that molecule 1 is in some state
a, molecule 2 is in b, and molecule 3 is in c, then one member
of the ensemble has an energy E = εa + εb + εc. This member,
however, is indistinguishable from one formed by putting Exercise
molecule 1 in state b, molecule 2 in state c, and molecule E13D.2 For a system consisting of N distinguishable, independent molecules,
3 in state a, or some other permutation. There are six such N
Q = q . Use the same approach as in Brief illustration 13D.2 to find an
permutations in all, and N! in general. In the case of indis- expression for  E  for such a system. Comment on your answer.
tinguishable molecules, it follows that too many states have
been counted in going from the sum over system states to the
sum over molecular states, so writing Q = qN overestimates the
value of Q . The detailed argument is quite involved, but at all The variation of the energy
13D.4
except very low temperatures it turns out that the correction with volume
factor is 1/N!, so Q = qN/N!.
Step 3 Summarize the results When there are interactions between molecules, the energy of a
It follows that:
collection depends on the average distance between them, and
therefore on the volume that a fixed number occupy. This de-
pendence on volume is particularly important for the discus-
For distinguishable, independent molecules:
sion of real gases (Topic 1C).
Q = qN (13D.6a)
To discuss the dependence of the energy on the volume at
(13D.6b) constant temperature it is necessary to evaluate (∂〈E〉/∂V)T.
For indistinguishable, independent molecules:
Q = q N/N! (In Topics 2D and 3E, this quantity is identified as the ‘internal
pressure’ of a gas and denoted πT.) To proceed, substitute eqn
13D.5 and obtain
For molecules to be indistinguishable, they must be of the
same kind: an Ar atom is never indistinguishable from a Ne   E       lnQ  
atom. Their identity, however, is not the only criterion. Each  V     V      (13D.7)
 T   V T
identical molecule in a crystal lattice, for instance, can be
‘named’ with a set of coordinates. Identical molecules in a lat- If the molecules are non-interacting (independent) the gas will
tice can therefore be treated as distinguishable because their be perfect and Q = qN/N!, with q the partition function for the
sites are distinguishable, and eqn 13D.6a can be used. On translational and any internal (such as rotational) modes. If the
the other hand, identical molecules in a gas are free to move gas is monatomic, only the translational mode is present and
to different locations, and there is no way of keeping track of q = V/Λ3 with Λ = h/(2πmkT)1/2. The canonical partition func-
the identity of a given molecule; therefore eqn 13D.6b must tion would then be VN/Λ3NN!. The presence of interactions is
be used. taken into account by replacing VN/N! by a factor called the
configuration integral, Z, which depends on the intermolecu-
lar potentials (don’t confuse this Z with the compression factor
Brief illustration 13D.2 Z in Topic 1C), and writing

For a gas of N indistinguishable, independent molecules, Z


Q   (13D.8)
Q = qN/N!. The energy of the system is therefore 3N
13D The canonical ensemble 571

It then follows that

Determines b

Potential energy, V
〈E 〉   ln(Z /Λ3 N )
=−
V T
V β
V T

 lnZ   ln(1/Λ3 N )
=− −
V β V β Intermolecular separation
V T V T 0

2f/ x  y =  2f/ yx for the second term Determines a


0

  lnZ   ln(1/Λ3 N ) Figure 13D.5 The intermolecular potential energy of molecules


=− −
V β V
β V in a real gas can be modelled by a central hard sphere that
T T
V determines the van der Waals parameter b surrounded by a
shallow attractive well that determines the parameter a. As
 ln y/ x = (1/y)y/x mentioned in the text, calculations of the canonical partition
function based on this are consistent with the van der Waals
  ln Z  1 Z equation of state (Topic 1C).
=− =− (13D.9)
V  β V
V Z  β V
T T

In the third line, Λ is independent of volume, so its derivative If the potential energy has the form of a central hard sphere
with respect to volume is zero. surrounded by a shallow attractive well (Fig. 13D.5), then de-
For a real gas of atoms (for which the intermolecular inter- tailed calculation, which is too involved to reproduce here (see
actions are isotropic), Z is related to the total potential energy A deeper look 13D.1, available to read in the e-book accompa-
EP of interaction of all the particles, which depends on all their nying this text), leads to
relative locations, by
  E   an2
1   Ep  V   2 Attractive potential (13D.11)
N!
Z e d 1d 2 d N Configuration integral  (13D.10)  T V

where dτi is the volume element for atom i and the integration where n is the amount of molecules present in the volume V
is over all the variables. The physical origin of this term is that and a is a constant that is proportional to the area under the at-
the probability of occurrence of each arrangement of molecules tractive part of the potential. In Example 3E.2 exactly the same
possible in the sample is given by a Boltzmann distribution expression (in the form πT = an2/V2) was derived from the van
in which the exponent is given by the potential energy corre- der Waals equation of state. The conclusion at this point is that
sponding to that arrangement. if there are attractive interactions between molecules in a gas,
then its energy increases as it expands isothermally (because
(∂〈E〉/∂V)T > 0, and the slope of 〈E〉 with respect to V is posi-
Brief illustration 13D.3
tive). The energy rises because, at greater average separations,
Equation 13D.10 is very difficult to manipulate in practice, the molecules spend less time in regions where they interact
even for quite simple intermolecular potentials, except for a favourably.
perfect gas for which EP = 0. In that case, the exponential func-
tion becomes 1 and
Exercise
VN
1 1
 
N

N!  d
Z d 1d 2 d N   E13D.3 Equation 13D.9 gives an expression for ( E  /V )T in terms of
N! N!
the configuration integral Z. For a perfect gas consisting of N molecules,
just as it should be for a perfect gas. Z = V N /N !. Use this expression for Z in eqn 13D.9 to find ( E  /V )T .
Comment on your result.
572 13 Statistical thermodynamics

Checklist of concepts
☐ 1. The canonical ensemble is a collection of imaginary ☐ 3. The mean energy of the members of the ensemble can be
replications of the actual system with a common tem- calculated from the canonical partition function.
perature and number of particles.
☐ 2. The canonical distribution gives the most probable
number of members of the ensemble with a specified
total energy.

Checklist of equations⋆
Property Equation Comment Equation number
  Ei
Canonical partition function Q  e Definition 13D.2
i

Canonical distribution N i /N  e   Ei /Q 13D.2


Mean energy 〈E〉 = −(1/Q )(∂Q /∂β)V = −(∂ ln Q /∂β)V 13D.5

Canonical partition function Q  Z / 3N


13D.8
  EP
Configuration integral Z  (1/N !)  e d 1d 2  d N Isotropic interaction 13D.10
2 2
Variation of mean energy with volume (∂ 〈E〉/∂V)T = an /V Potential energy as specified in Fig. 13D.5 13D.11


β = 1/kT
TOPIC 13E The internal energy
and the entropy

1  q 
      (13E.1)
➤ Why do you need to know this material? q   V
The importance of this discussion is the insight that a
molecular interpretation provides into thermodynamic with β = 1/kT. The total energy of a system composed of N mol-
properties. ecules is therefore N〈ε〉. This total energy is the energy above
the ground state (the calculation of q is based on the conven-
➤ What is the key idea? tion ε0 ≡ 0) and so the internal energy is U(T) = U(0) + N〈ε〉,
The partition function contains all the thermodynamic where U(0) is the internal energy when only the ground state is
information about a system and thus provides a bridge occupied, which is the case at T = 0. It follows that the internal
between spectroscopy and thermodynamics. energy is related to the molecular partition function by

➤ What do you need to know already? N  q 


U (T )  U (0)  N    U (0) 
You need to know how to calculate a molecular partition
q   V
function from structural data (Topic 13B); you should also Internal energy [independeent molecules] (13E.2a)
be familiar with the concepts of internal energy (Topic 2A)
and entropy (Topic 3A). This Topic makes use of the calcula- In many cases, the expression for 〈ε〉 established for each mode
tions of mean molecular energies in Topic 13C. of motion in Topic 13C can be used and it is not necessary to go
back to q itself except for some formal manipulations. For in-
stance, it is established in Topic 13C (eqn 13C.8) that the mean
energy of a harmonic oscillator is  V   hc /(e  hc  1). It fol-
lows that the molar internal energy of a system composed of
oscillators is
Any thermodynamic function can be obtained once the par- N A hc
tition function is known. The two fundamental properties of U mV (T )  U mV (0)  N A  V   U mV (0) 
e  hc  1
thermodynamics are the internal energy, U, and the entropy, S.
Once these two properties have been calculated, it is possible
to calculate the derived functions, such as the Gibbs energy, G, Brief illustration 13E.1
(Topic 13F) and all the chemically interesting properties that
stem from them. The vibrational wavenumber of an I2 molecule is 214.6 cm−1. At
298.15 K, kT/hc = 207.225 cm−1. With these values

hc 214.6 cm 1
 hc    1.035
13E.1 The internal energy kT 207.225 cm 1
hc  (6.626  1034 Js)  (2.998  1010 cms 1 )  (214.6 cm 1 )
The first example of the importance of the molecular partition
 4.261021 J
function, q, is the derivation of an expression for the internal
energy. It follows that the vibrational contribution to the molar internal
energy of I2 is
13E.1(a) The calculation of internal energy (6.022  1023 mol 1 )  (4.261021 J)
U mV (T )  U mV (0) 
It is established in Topic 13C that the mean energy of a mol- e1.035  1
V 1
ecule in a system composed of independent (non-­interacting)  U (0)  1.41 kJmol
m

molecules is related to the molecular partition function by


574 13 Statistical thermodynamics

An alternative form of eqn 13E.2a is V V


By noting that e /T  (e / 2T )2 , this expression can be rearranged
  ln q  into
Internal energy
U (T )  U (0)  N   [independent molecu
ules]
 (13E.2b)
  V 2 2
  V   e / 2T 
V

V
C  Rf (T ), f (T )      V/T 
A very similar expression is used for a system of interacting
 T   e  1
V ,m

molecules (Topic 13D). In that case the mean energy  E , given
by eqn 13D.5 in terms of the canonical partition function, Q, is nal contribution to CV,m
Vibration (13E.3)
identified as the contribution of other than the ground state to
the internal energy to give The graph in Fig. 13E.1 shows how the vibrational heat capacity
depends on temperature. Note that even when the temperature
  lnQ  Internal energy is only slightly above θV the heat capacity is close to its equi-
U (T )  U (0)    [interacting moleculles] (13E.2c)
  V partition value of R. Equation 13E.3 is essentially the same as
the Einstein formula for the heat capacity of a solid (eqn 7A.9a)
with θV the Einstein temperature, θE. The only difference is that
13E.1(b) Heat capacity in a solid the vibrations take place in three dimensions.
The constant-volume heat capacity (Topic 2A) is defined as It is sometimes more convenient to convert the derivative
CV = (∂U/∂T)V. It is therefore possible to find expressions for with respect to T into a derivative with respect to β = 1/kT by
the heat capacity in terms of the partition function by differen- using
tiating an expression for U in terms of the q. The procedure is
illustrated here for a system of harmonic oscillators. d d d 1 d d
  2  k  2  (13E.4)
The mean vibrational energy of a harmonic oscillator is given d T d T d kT d d
by eqn 13C.8, quoted above as  V   hc /(e  hc  1). The calcu-
It follows that
lation of the heat capacity involves taking the derivative with
respect to T, so it is convenient to rewrite this expression for
the mean energy in terms of T and the characteristic vibra- 〈 〉= – (ln q/ β )V
tional temperature  V  hc /k . It follows that hc  k V and
 hc  k V /kT   V /T . With these substitutions the mean en- U 〈 〉  2 ln q
CV = − kβ 2 = − Nkβ 2 = Nkβ 2
ergy is β V
β V
β 2 V

hc k V Heat capacity (13E.5)


 V    hc
  V/T
e 1 e 1
There is a much simpler route to finding CV when the equipar-
The molar internal energy is therefore tition principle can be applied, which is when many energy lev-
els associated with a particular mode of motion are ­populated.
N Ak V V 1 Such a mode is said to be ‘active’. Translational modes are
U mV (T ) = U mV (0) + = U mV (0) + R
e /T −1 −1
V V/T
e

where R = N A k. To evaluate the derivative of U mV (T ) with respect 1


to T note that only the boxed term depends on the temperature
and that by using the rules for differentiating a function of a
Molar heat capacity, CV,m/R

function (see Part 1 of the Resources section)

d 1 1 – V
= (–1) × /T ×e ×
V/T

(e −1) 2 T2 0.5
V
dT e /T −1
V

V/T
V
e
=
T 2 (e −1) 2
V/T

Then the molar heat capacity is 0


0 0.5 1
 U (T ) 
V
V d 1 Temperature, T/θ V
  R
V
C V, m  m

 T V dT e V /T
1 Figure 13E.1 The temperature dependence of the vibrational heat
 V /T 2
 V /T capacity of a molecule in the harmonic approximation calculated
 V
e   e V
 R V   R   V /T by using eqn 13E.3. Note that the heat capacity is within 10 per
2
T (e V /T
 1)2
 T  (e  1)2 cent of its classical value for temperatures greater than θV.
13E The internal energy and the entropy 575

always active at any reasonable temperature. A rotational mode How is that done? 13E.1 Deriving the Boltzmann formula
is active when T >> θR, where θR is the characteristic tempera-
for the entropy
ture for rotation ( R  hcB /k). Similarly, a vibrational mode is
active when T >> θV with  V  hc /k . The starting point for this derivation is the expression for the
Each quadratic term contributes 12 RT to the molar internal internal energy, U(T) = U(0) + N〈ε〉, which, with eqn 13C.1,
energy and, because CV,m = (∂Um/∂T)V, contributes 12 R to the    (1/N )  N i i, can be written
i
molar heat capacity. In gases, all three translational modes are
always active and so contribute 23 R to the molar heat capacity. U (T )  U (0)  N i i
Each active rotational mode contributes 12 R, so if there are ν R  i

The strategy is to use classical thermodynamics to establish


such modes the contribution is 12 ν R  R. For most molecules at
a relation between dS and dU, and then to use this relation
normal temperatures  R   2 for linear molecules, and 3 for
to express dU in terms of the weight of the most probable
nonlinear molecules. Each active vibrational mode contributes
­configuration.
R (two quadratic terms), so if there are ν V  such modes their
contribution is ν V  R. However, in most cases none of the vibra- Step 1 Write an expression for dU(T)
tional modes is active and  V   0. It follows that the total molar A change in U(T) may arise from either a modification of the
heat capacity of a gas is approximately energy levels of a system (when εi changes to εi + dεi) or from a
modification of the populations (when Ni changes to Ni + dNi).
Total heat capacity
CV, m  12 (3   R   2 V  )R [equipartition]
 (13E.6) The most general change is therefore

dU(T) = dU(0) + ∑N i d i + ∑ i dN i
i i

Brief illustration 13E.2 Step 2 Write an expression for dS


Because neither U(0) nor the energy levels change when a sys-
The characteristic temperatures of the vibrations of H2O are
tem is heated at constant volume (Fig. 13E.2), in the absence of
close to 5300 K, 2300 K, and 5400 K; the vibrations are there-
all changes other than heating, only the third (boxed) term on
fore not active at 373 K. The three rotational modes of H2O have
the right survives. The fundamental equation dU = TdS − pdV
characteristic temperatures 40 K, 21 K, and 13 K, so they are all
1 1 (eqn 3E.1) relates the change in internal energy to the changes
active. The translational contribution is 23 R  12.5 JK mol .
−1 −1 in entropy and volume. Under the constant volume conditions
The rotations contribute a further 12.5 JK mol . Therefore,
imposed here it follows that dU = TdS and hence dS = dU/T.
a value close to 25 J K−1 mol−1 is predicted. The experimental
Therefore,
value is 26.1 J K−1 mol−1. The discrepancy is probably due to
deviations from perfect gas behaviour.
dU 1
dS    i dN i  k   i dN i
T T i i

Exercises
E13E.1 Use the equipartition theorem to estimate the constant-volume molar High
heat capacity of (i) I2, (ii) CH4, (iii) C6H6 in the gas phase at 25 °C. temperature

E13E.2 Estimate the values of γ = Cp,m/CV,m for gaseous ammonia and methane.
Do this calculation with and without the vibrational contribution to the
energy. Which is closer to the experimental value of γ = 1.31 for both gases at
25 °C? Hint: Note that Cp,m − CV,m = R for a perfect gas.
Heat Work
2 2 −1
E13E.3 The ground level of Cl is P3/2 and a P1/2 level lies 881 cm above it.
Calculate the electronic contribution to the molar heat capacity at constant
volume of Cl atoms at 500 K.

Low
temperature
13E.2 The entropy Figure 13E.2 (a) When a system is heated, the energy levels are
unchanged but their populations are changed. (b) When work
One of the most celebrated equations in statistical thermody- is done on a system, the energy levels themselves are changed.
namics, the ‘Boltzmann formula’ for the entropy, is obtained by The levels in this case are the one-dimensional particle-in-a-box
establishing the relation between the entropy and the weight of energy levels of Topic 7D: they depend on the size of the container
the most probable configuration. and move apart as its length is decreased.
576 13 Statistical thermodynamics

Step 3 Write an expression for dS in terms of W Step 1 Derive the relation for distinguishable independent
Changes in the most probable configuration (the only one to molecules
consider) are given by eqn 13A.7, (ln W )/N i     i  0. For a system composed of N distinguishable molecules, lnW is
It follows that  i  (ln W )/N i   . This expression for  i is given by eqn 13A.3, ln W  N ln N   N i ln N i; the substitution
i
substituted into the above expression for dS to give N   N i then gives
i

dS = k β∑ i dN i = k ∑ β i dN i  Ni
i i
i

ln W  N ln N  N i ln N i
 lnW
= k∑ N i
+ α dN i i

i  N i ln N  N i ln N i
i i
 lnW Ni
= k∑ Ni
dN i + k ∑ α dN
i  N i (ln N  ln N i )  N i ln
i i i i N

Because the system contains a fixed number of molecules the Use this expression for lnW in eqn 13E.7 to obtain the entropy as
sum of the changes in the populations of the states Σ dN i is zero,
i Ni
so the term in the box is zero. The expression (∂ ln W /∂N i )dN i S  k ln W  k N i ln
simplifies to d lnW , leaving i N

dS = k(d ln W ) The value of Ni/N for the most probable distribution is given by
the Boltzmann distribution, N i /N  e  i /q, and so
This relation strongly suggests that
Ni
(13E.7)
ln  ln e  i  ln q    i  ln q
S = klnW N
Boltzmann formula for the entropy
Therefore,
Ni
This very important expression is the Boltzmann formula for S  k N i ln  k N i (  i  ln q )
i N i
the entropy. In it, W is the weight of the most probable configu- N  
 N

ration of the system (as discussed in Topic 13A).  k  N i i  k N i ln q  k  N i i  k ln q N i
i i i i
 k  N    Nk ln q
13E.2(a) Entropy and the partition function
The statistical entropy, the entropy calculated from the Finally, because N    U (T )  U (0) and β = 1/kT, it follows that
Boltzmann formula, behaves in exactly the same way as the
thermodynamic entropy. Thus, as the temperature is lowered, U (T ) − U (0) (13E.8a)
S= + Nk ln q
the value of W , and hence of S, decreases because fewer config- T The entropy
[independent, distinguishable
urations are consistent with the total energy. In the limit T → 0, molecules]
W = 1, so lnW = 0, because only one configuration (every mol-
ecule in the lowest state) is compatible with U(0). It follows that Step 2 Derive the relation for indistinguishable molecules
S → 0 as T → 0, which is compatible with the Third Law of ther-
To treat a system composed of N indistinguishable molecules,
modynamics, that the entropies of all perfect crystals approach
the weight W is reduced by a factor of N! because the N! per-
the same value as T → 0 (Topic 3C).
mutations of the molecules among the states result in the same
The challenge now is to establish a relation between the state of the system. Therefore, ln W , and therefore the entropy,
Boltzmann formula and the partition function. is reduced by k ln N! from the ‘distinguishable’ value. Because
N is so large, Stirling’s approximation, ln N! = N ln N − N, can
be used to convert eqn 13E.8a into
How is that done? 13E.2 Relating the statistical entropy to
 N
ln ! 
the partition function U (T )  U (0)
S(T )   Nk ln q  k (N ln N  N )
T
It is necessary to separate the calculation into two parts, one U (T )  U (0) q
for distinguishable independent molecules and the other   Nk ln  kN
T N
for indistinguishable independent molecules. Interactions
can be taken into account, in principle at least, by a simple The kN can be combined with the logarithm by writing it as
generalization. kN lne, to give
13E The internal energy and the entropy 577

U (T ) − U (0) qe (13E.8b) 1
S= + Nk ln
T N The entropy
[independent, indistinguishable
molecules] ln 2

Molar entropy, Sm/R


Step 3 Generalize to interacting molecules 0.5
For completeness, the corresponding expression for interacting
molecules, based on the canonical partition function in place
of the molecular partition function, is

0
U (T ) − U (0) (13E.8c) 0 5 10 0 0.5 1
S= + k ln Q Temperature, kT/ε Temperature, kT /ε
T The entropy
[interacting molecules]
Figure 13E.3 The temperature variation of the molar entropy of a
collection of two-level systems expressed as a multiple of R = NAk.
Equation 13E.8a expresses the entropy of a collection of in- When kT >> ε the two states become equally populated and Sm
dependent molecules in terms of the internal energy and the approaches R ln 2.
molecular partition function. However, because the energy of
a molecule is a sum of contributions, such as translational (T),
How is that done? 13E.3 Deriving the expression for the
rotational (R), vibrational (V), and electronic (E), the partition
function factorizes into a product of contributions. As a result, entropy of a monatomic perfect gas
the entropy is also the sum of the individual contributions. In You need to start with eqn 13E.8b for a collection of independ-
a gas, the molecules are free to change places, so they are in- ent, indistinguishable atoms and write N = nNA, where NA is
distinguishable; therefore, for the translational contribution to Avogadro’s constant and n is their amount (in moles). The only
the entropy, use eqn 13E.8b. For the other modes (specifically mode of motion for a gas of atoms is translation and application
R, V, and E), which do not involve the exchange of molecules of the equipartition principle gives the contribution to the molar
and therefore do not require the weight to be reduced by N!, energy as 23 RT. It follows that U (T )  U (0)  23 nRT . The parti-
use eqn 13E.8a. tion function is q = V/Λ3 (eqn 13B.10b), where Λ is the thermal
For a system with two states, with energies 0 and ε, it is wavelength (Topic 13B). Therefore,
shown in Topics 13B and 13C that the partition function and 3 nRT
2 
mean energy are q  1  e   and     /(e   1). It follows that U (T )  U (0) qe U (T )  U (0) qe
U (T )  U (0)  N  /(e   1). The contribution to the molar en- S  Nk ln   nN A k ln
T N T nN A
tropy is evaluated by using eqn 13E.8a with N = NA nR
 Ve
 23 nR  nN A k ln
nN A 3
U m (T )  U m (0)
Sm   N A k ln q  ln e
3/2

T  V e  Vm e5/ 2
 nR  23  ln m 3   nR ln
N A  N A  N A 3
  N A k ln(1  e   )
T (e   1)
   where Vm = V/n is the molar volume of the gas and 23 has been
 R    ln(1  e   ) replaced by ln e3/2. Division of both sides by n gives an expres-
e 1  sion for the molar entropy, the Sackur–Tetrode equation:

This awkward function is plotted in Fig. 13E.3. It should be Vme5/2 (13E.9a)


noted that at high temperatures (when kT >> ε or βε << 1), the Sm = R ln
N A Λ3 Sackur–Tetrode equation
molar entropy approaches R ln 2. [monatomic perfect gas]

where Λ is the thermal wavelength (Λ = h/(2πmkT)1/2). To cal-


13E.2(b) The translational contribution culate the standard molar entropy, note that Vm = RT/p, and
set p = p : ⦵
The expressions derived for the entropy are in line with what is
expected for entropy as a measure of the spread of the popula- R/NA= k
tions of molecules over the available states. This interpretation
5/2
can be illustrated by deriving an expression for the molar en- ⦵ RTe kTe5/2
S m = R ln = R ln (13E.9b)
p⦵ N A Λ3 p Λ3

tropy of a monatomic perfect gas.
578 13 Statistical thermodynamics

These expressions assume that many translational levels are oc- S  nR ln aVf  nR ln aVi Change of entropy
cupied so that the translational partition function is given by eqn V on expansion  (13E.10)
 nR ln f
13B.10b and the equipartition principle applies for the transla- Vi [perfect gas , isothermal]
tional contribution to the energy. Therefore, these expressions for
the entropy do not apply when T is equal to or very close to zero. This expression is the same as that obtained starting from the
thermodynamic definition of entropy (Topic 3B).

Brief illustration 13E.3


13E.2(c) The rotational contribution
The mass of an Ar atom is 39.95mu. At 25 °C, the thermal wave-
The rotational contribution to the molar entropy, SmR, can be cal-
length of Ar is 16.0 pm and kT = 4.12 × 10−21 J. Therefore, the
culated by using eqn 13E.8a once the molecular partition func-
molar entropy of argon at this temperature is
tion is known. For a linear molecule, the high-temperature limit
 (4.12  1021 J)  e5/ 2  of qR is kT /σ hcB (eqn 13B.13b, qR = T/σθR with  R  hcB /k ) and
Sm  R ln  5

2 11 3 the equipartition theorem gives the rotational contribution to


 (10 Nm )  (1 . 60  10 m ) 
the molar internal energy as RT. Therefore, from eqn 13E.8a:
 18.6 R  155 JK 1 mol 1
 RT   / hcB
kT 
On the basis that there are fewer accessible translational states U (T )  U
R m 0)
(
for a lighter atom than for a heavy atom under the same condi- Sm  m
 R ln q R
T
tions (see below), it can be anticipated that the standard molar
entropy of Ne is likely to be smaller than for Ar; its actual value and the contribution at high temperatures is
is 17.6R at 298 K.
kT  Rotational contribution

SmR  R 1  ln  [linear molecule, high  (13E.11a)
  hcB  R
temperature (T >>  )]
The physical interpretation of these equations is as follows:
In terms of the rotational temperature,
• Because the molecular mass appears in the numerator
(because it appears in the denominator of Λ), the molar  T  Rotational contribution
entropy of a perfect gas of heavy molecules is greater than SmR  R 1  ln R  [Linear molecule, high  (13E.11b)
   R
temperature (T >>  )]
that of a perfect gas of light molecules under the same
conditions. This feature can be understood in terms of the This function is plotted in Fig. 13E.4. It is seen that:
energy levels of a particle in a box being closer together for
heavy particles than for light particles, so more states are • The rotational contribution to the entropy increases with tem-
thermally accessible. perature because more rotational states become accessible.
• Because the molar volume appears in the numerator, the • The rotational contribution increases as B decreases (θR
molar entropy increases with the molar volume of the gas, decreases), because then the rotational energy levels
the temperature being unchanged. The reason is similar: large become closer together.
containers have more closely spaced energy levels than small
containers, so once again more states are thermally accessible. 3
• Because the temperature appears in the numerator
(because, like m, it appears in the denominator of Λ), the
Molar entropy, Sm/R

molar entropy increases with increasing temperature, the 2


molar volume being constant. The reason for this behav-
iour is that more energy levels become accessible as the
temperature is raised. Approximate
1
The expression for the entropy can be written in the form
Exact
V e5 / 2 Ve5/ 2 e5 / 2
S  nR ln m 3  nR ln  nR ln aV , a  0
0 1 2 3
NA nN A  3
nN A 3 Temperature, T/θR

where a is a constant that depends on the temperature and the Figure 13E.4 The variation of the rotational contribution to the molar
atomic mass of the gas. This relation for the entropy implies entropy of a linear molecule (σ = 1) using the high-temperature
that when a monatomic perfect gas expands isothermally from approximation (thin line) and the exact expression (thick line; for this
Vi to Vf , its entropy changes by the partition function is evaluated up to J = 20).
13E The internal energy and the entropy 579

It follows that large, heavy molecules have a large rotational 4


contribution to their entropy. As shown in Brief illustration
13E.4, the rotational contribution to the molar entropy of 35Cl2
3

Molar entropy, Sm/R


is 58.6 J K−1 mol−1 whereas that for H2 is only 12.7 J K−1 mol−1.
That is, it is appropriate to regard Cl2 as a more rotationally dis-
ordered gas than H2, in the sense that at a given temperature 2
Cl2 occupies a greater number of rotational states than H2 does.
1

Brief illustration 13E.4


0
0 5 10
The rotational contribution for 35Cl2 at 25 °C, for instance, is Temperature, T /θV
calculated by noting that σ = 2 for this homonuclear diatomic
molecule and taking B  0.2441 cm 1. The rotational tempera- Figure 13E.5 The temperature variation of the molar entropy
ture of the molecule is of a collection of harmonic oscillators expressed as a multiple
of R = NAk. The molar entropy approaches zero as T → 0, and
(6.626  1034 Js)  (2.998  1010 cms 1 )  (0.2441 cm 1 ) increases without limit as the temperature increases.
R 
1.381  1023 JK 1
 0.351 K Once again it is convenient to express this formula in terms of a
Therefore, characteristic temperature, in this case the vibrational tempera-
ture,  V  hc /k :
 298 K  1 1
SmR  R 1  ln   7.05 R  58.6 JK mol   V/T 
 2  (0.351 K )  SmV  R   V/T
V
 ln(1  e  /T )
e 1 
n to the molar entropy 
Vibrational contribution (13E.12b)
R
Equation 13E.11 is valid at high temperatures (T >> θ ). To
track the rotational contribution down to low temperatures it is This function is plotted in Fig. 13E.5. As usual, it is helpful to
necessary to use the full form of the rotational partition func- interpret it, with the graph in mind:
tion (Topic 13B). The resulting graph has the form shown in • Both terms multiplying R become zero as T → 0, so the
Fig. 13E.4, and it is seen that the approximate curve matches entropy is zero at T = 0.
the exact curve very well for T/θR greater than about 1.
• The molar entropy rises as the temperature is increased as
more vibrational states become accessible.
13E.2(d) The vibrational contribution • The molar entropy is higher at a given temperature for
The vibrational contribution to the molar entropy, S , is ob- V molecules with smaller vibrational frequencies. The vibra-
m
tained by combining the expression for the molecular parti- tional energy levels become closer together the smaller the
tion function (eqn 13B.15, q V  1/(1  e   hc )  1/(1  e   ) with vibrational frequency and so more are thermally accessible.
  hc ) with the expression for the mean energy (eqn 13C.8,
〈εV〉 = ε/(eβε − 1)), to obtain Brief illustration 13E.5

N A   V The vibrational wavenumber of I2 is 214.6 cm−1 and so its vibra-


    R
U (T )  U m (0) N A k  tional temperature is 309 K. At 25 °C
1
SmV  m  R ln q V    R ln
T e 1 1  e    (309 K )/(298 K )  ( 309 K ) / ( 298 K ) 
1/k  SmV  R  (309 K )/(298 K )  ln(1  e )  1.01 R
 e  1 
  
 R    ln(1  e   )  8.38 JK 1 mol 1
e 1 

That is,
13E.2(e) Residual entropies
  hc 
SmV  R   hc  ln(1  e   hc ) Entropies may be calculated from spectroscopic data; they may
e 1  also be measured experimentally (Topic 3C). In many cases
Vibrational contributtion to the molar entropy (13E.12a) there is good agreement, but in some the experimental entropy
580 13 Statistical thermodynamics

is less than the calculated value. One possibility is that the ex-
perimental determination failed to take a phase transition into
account and a contribution of the form ∆trsH/Ttrs was incorrectly
omitted from the sum. Another possibility is that some disorder
is present in the solid even at T = 0. The entropy at T = 0 is then
greater than zero and is called the residual entropy.
The origin and magnitude of the residual entropy can be ex-
plained by considering a crystal composed of AB molecules,
where A and B are similar atoms (such as CO, with its very Figure 13E.6 The possible locations of H atoms around a central
small electric dipole moment). There may be so little energy O atom in an ice crystal are shown by the white spheres. Only one
difference between … AB AB AB AB …, … AB BA BA AB …, of the locations on each bond may be occupied by an atom, and
and other arrangements that the molecules adopt the orienta- two H atoms must be close to the O atom and two H atoms must
tions AB and BA at random in the solid. The entropy arising be distant from it.
from residual disorder can be calculated readily by using the
Boltzmann formula, S = k lnW . To do so, suppose that two ori-
entations are equally probable, and that the sample consists of
N molecules. Because the same energy can be achieved in 2N
different ways (because each molecule can take either of two
orientations), the total number of ways of achieving the same
energy is W = 2N . It follows that
= ln 2N Nk ln 2 = nR ln 2
S k= (13E.13a)

and that a residual molar entropy of R ln 2 = 5.8 J K−1 mol−1 is ex-


pected for solids composed of molecules that can adopt either
of two orientations at T = 0. If s orientations are possible, the
residual molar entropy is
Sm (0) = R ln s Residual entropy  (13E.13b)
Figure 13E.7 The six possible arrangements of H atoms in the
−1 −1
For CO, the measured residual entropy is 5 J K mol , which is locations identified in Fig.13E.6. Occupied locations are denoted
close to R ln 2, the value expected for a random structure of the by grey spheres and unoccupied locations by white spheres.
form …CO CO OC CO OC OC….
Similar arguments apply to more complicated cases. Consider ignores the possibility that next-nearest neighbours and those
a sample of ice with N H2O molecules. Each O atom is sur- beyond can influence the local arrangement of bonds.
rounded tetrahedrally by four H atoms, two of which are at-
tached by short σ bonds, the other two being attached by long
hydrogen bonds (Fig. 13E.6). It follows that each of the 2N H Exercises
atoms can be in one of two positions (either close to or far from E13E.4 Calculate the standard molar entropy at 298 K of (i) gaseous helium,
an O atom as shown in Fig. 13E.7), resulting in 22N possible ar- (ii) gaseous xenon.
rangements. However, not all these arrangements are acceptable. E13E.5 At what temperature is the standard molar entropy of helium equal to
Indeed, of the 24 = 16 ways of arranging four H atoms around standard molar entropy of xenon at 298 K?
one O atom, only 6 have two short and two long OH distances E13E.6 Calculate the rotational partition function of H2O at 298 K from its
and hence are acceptable (Fig. 13E.7). Therefore, the number of rotational constants 27.878 cm−1, 14.509 cm−1, and 9.287 cm−1 and use your
permitted arrangements is W  22 N  166    23  . result to calculate the rotational contribution to the molar entropy of gaseous
N N

H2O at 25 °C.
It then follows that the residual entropy is S(0)  k ln  23  
N
2+
E13E.7 The ground state of the Co ion in CoSO4·7H2O may be regarded as
kN ln 23 , and its molar value is Sm (0)  R ln 23  3.4 JK mol 1,
1
4
T9/2. The entropy of the solid at temperatures below 1 K is derived almost
which is in very good agreement with the experimental value entirely from the electron spin. Estimate the molar entropy of the solid at
of 3.4 J K−1 mol−1. The model, ­however, is not exact because it these temperatures.
13E The internal energy and the entropy 581

Checklist of concepts
☐ 1. The internal energy is proportional to the derivative of ☐ 3. The statistical entropy is defined by the Boltzmann for-
the logarithm of the partition function with respect to mula and expressed in terms of the molecular partition
temperature. function.
☐ 2. The total heat capacity of a molecular substance is the ☐ 4. The residual entropy is a non-zero entropy at T = 0 aris-
sum of the contribution of each mode. ing from molecular disorder.

Checklist of equations⋆
Property Equation Comment Equation number
Internal energy U(T) = U(0) − (N/q)(∂ q/∂β)V = U(0) − N(∂ ln q/∂β)V Independent molecules 13E.2a
Heat capacity CV = Nkβ2(∂2 ln q/∂β2)V Independent molecules 13E.5
CV, m  (3  
1
2
R
 2 )R
V
13E.6
Boltzmann formula for the entropy S = k ln W 13E.7
Entropy S = {U(T) − U(0)}/T + Nk ln q Distinguishable molecules 13E.8a
S = {U(T) − U(0)}/T + Nk ln (qe/N) Indistinguishable molecules 13E.8b
Sackur–Tetrode equation Sm = R ln(Vme5/2/NAΛ3) Molar entropy of a monatomic perfect gas 13E.9a
Residual molar entropy Sm(0) = R ln s s is the number of equivalent orientations 13E.13b


β = 1/kT
TOPIC 13F Derived functions

in this Topic make use of the molecular partition function, q


➤ Why do you need to know this material? (Topic 13B). Equations 13F.1a and b can be regarded simply
The power of chemical thermodynamics stems from its as a succinct way of expressing the relations for collections of
deployment of a variety of derived functions, particularly independent molecules by writing Q = qN for distinguishable
the enthalpy and Gibbs energy. It is therefore important to molecules and Q = qN/N! for indistinguishable molecules (as in
relate these functions to structural features through parti- a gas). The advantage of using Q is that the equations to be de-
tion functions. rived are simple and, if necessary, can be used to calculate ther-
modynamic properties when there are interactions between the
➤ What is the key idea? molecules.
The partition function provides a link between spectro-
scopic and structural data and the derived functions of
thermodynamics, particularly the equilibrium constant.
13F.1 The derivations
➤ What do you need to know already?
This Topic develops the discussion of internal energy The Helmholtz energy, A, is defined as A = U − TS. At T = 0,
and entropy (Topic 13E). You need to know the relations S = 0 and the internal energy is U(0), therefore A(0) = U(0).
between those properties and the enthalpy (Topic 2B) and Substitution of the expressions for U(T) and S(T) in eqns 13F.1a
the Helmholtz and Gibbs energies (Topic 3D). The final and b gives
section makes use of the relation between the standard
reaction Gibbs energy and the equilibrium constant (Topic A(T )  U (T )  TS(T )
6A). Although the equations are introduced in terms of the U (0 )
   lnQ   1/kT   lnQ  
canonical partition function (Topic 13D), all the applications  A(0)     T  k     k lnQ 
are in terms of the molecular partition function (Topic 13B).   V    V 
 A(0)  kT lnQ

That is,
Classical thermodynamics makes extensive use of various
­derived functions. Thus, in thermochemistry the focus is on the A(T )  A(0)  kT lnQ Helmholtz energy  (13F.2)
enthalpy and, provided the pressure and temperature are con-
stant, in discussions of spontaneity the focus is on the Gibbs An infinitesimal change in conditions changes the Helmholtz
energy. All these functions are derived from the internal energy energy by dA = −pdV − SdT (Table 3E.1). It follows that on
and the entropy, which in terms of the canonical partition func- imposing constant temperature (dT = 0), the pressure and the
tion, Q , are given by Helmholtz energy are related by p = −(∂A/∂V)T. It then follows
from eqn 13F.2 that
  lnQ  1 Internal
U (T )  U (0)    with   energy  (13F.1a)
  V kT   lnQ 
p  kT   Pressure (13F.3)
U (T )  U (0)  V T
S(T )   k lnQ
T
Entropy  (13F.1b) This relation is entirely general, and may be used for any type
  lnQ 
 k     k lnQ of substance, including perfect gases, real gases, and liquids.
  V Because Q is in general a function of the volume, temperature,
There is no need to worry about this appearance of the ca- and amount of substance, eqn 13F.3 is an equation of state of
nonical partition function (Topic 13D): the only applications the kind discussed in Topics 1A and 3E.
13F Derived functions 583

Example 13F.1 At this stage the expressions for U and p and the definition
Deriving an equation of state
H = U + pV, with H(0) = U(0), can be used to obtain an expres-
Derive an expression for the pressure of a gas of independent sion for the enthalpy, H, of any substance:
particles.
  lnQ    lnQ 
Collect your thoughts You should suspect that the pressure H (T )  H (0)     kTV   Enthalpy  (13F.4)
is that given by the perfect gas law, p = nRT/V. To proceed   V  V T
systematically, substitute the explicit formula for Q for a gas
of independent, indistinguishable molecules. Only the transla- The fact that eqn 13F.4 is rather cumbersome is a sign that the
tional partition function depends on the volume, so there is no enthalpy is not a fundamental property: as shown in Topic 2B,
need to include the partition functions for the internal modes it is more of an accounting convenience. For a gas of independ-
of the molecules. ent, structureless particles U (T )  U (0)  23 nRT and pV = nRT.
Therefore, for such a gas, it follows directly from H = U + pV
The solution For a gas of independent molecules, Q = qN/N! that
with q = V/Λ3:
H (T )  H (0)  25 nRT  (13F.5)

 lnQ  ln( q N /N !) One of the most important thermodynamic functions for


p = kT = kT
V T
V chemistry is the Gibbs energy, G = H − TS. This definition im-
T
plies that G = U + pV − TS and therefore that G = A + pV. Note
(ln q N − lnN !) ( N ln q − lnN !) that G(0) = A(0), both being equal to U(0). The Gibbs energy
= kT = kT
V V T can now be written in terms of the partition function by com-
T

0
bining the expressions for A and p:

ln q  ln N !   lnQ 
= NkT − kT G(T )  G(0)  kT lnQ  kTV  Gibbs energy  (13F.6)
V V 
T T
 V T
d lnf /dx = (1/f ) df/dx This expression takes a simple form for a gas of independent
NkT q molecules because pV in the expression G = A + pV can be re-
= placed by nRT:
q V T

NAk = R G(T )  G(0)  kT lnQ  nRT  (13F.7)


NkT (V /Λ ) 3
NkT nN A kT nRT For a gas of indistinguishable particles Q = qN/N!, and there-
= = = =
V /Λ3 V T
V V V
fore ln Q = N ln q − ln N!. If Stirling’s approximation, ln N! =
N ln N − N, is used the expression for G simplifies as follows
The calculation shows that the equation of state of a gas of
independent particles is indeed the perfect gas law, pV = nRT. G(T )  G(0)  kT lnQ  nRT
Nln N  N
Self-test 13F.1 Derive the equation of state of a gas for which
 G(0)  kT (N ln q  ln N ! )  nRT
Q = qNf/N!, with q = V/Λ3, where f depends on the volume.
 G(0)  kT (N ln q  N ln N  N )  nRT
Answer: p = nRT/V + kT(∂ln f/∂V)T nN A kT nRT

 G(0)  NkT (ln q  ln N  1)  nRT
 G(0)  nRT ln q  nRT ln N  nRT  nRT
q  (13F.8)
If β is left undefined in eqn 13F.1, then the equation of state  G(0)  nRT ln
N
calculated in the same way as in Example 13F.1 is p = N/βV. For
this result to be the perfect gas law, it follows that β = 1/kT. This Note that a statistical interpretation of the Gibbs energy is now
is the formal proof of that relation, as anticipated and used in possible: because q is the number of thermally accessible states
Topics 13A–13E. A similar approach can be used to derive an and N is the number of molecules, the difference G(T) − G(0)
equation of state resembling the van der Waals equation: see is proportional to the logarithm of the average number of ther-
A deeper look 13D.1, available to read in the e-book accompa- mally accessible states available to a molecule. As this average
nying this text. number increases, the Gibbs energy falls further and further
584 13 Statistical thermodynamics

below G(0). The thermodynamic tendency to lower Gibbs en- The solution The chemical reaction is H2 (g )  12 O2 (g )  H2O(g ).
ergy can now be seen to be a tendency to maximize the number Therefore,
of thermally accessible states.
 f G   Gm (H2O,g )  Gm (H2 ,g )  12 Gm (O2 ,g )
⦵ ⦵ ⦵ ⦵
It turns out to be convenient to define the molar partition
function, qm = q/n (with units mol−1 and n = N/NA). The Gibbs
Now write the standard molar Gibbs energies in terms of the
energy is then written in terms of qm as standard molar partition functions of each species J:
qm Gibbs energy
G(T )  G(0)  nRT ln [indistinguishable,  (13F.9a) qm (J)


 Vm R

Gm (J)  E0,m (J)  RT ln qm (J)  qmT (J) q R (J)  q (J)
⦵ ⦵
NA independ dent molecules] NA (J)3

To use this expression, G(0) = U(0) is identified with the Therefore


energy of the system when all the molecules are in their
ground state, E0. To calculate the standard Gibbs energy, the ⦵  q ⦵ (H O)   q ⦵ (H ) 
 f G    E0,m (H2O)  RT ln m 2    E0,m (H2 )  RT ln m 2 
partition function has its standard value, qm. This standard ⦵
 N A   N A 
value is evaluated by setting the molar volume in the trans-  q ⦵ (O ) 
lational contribution equal to the standard molar volume, so  12  E0,m (O2 )  RT ln m 2 
 N A 
qm  (Vm /3 )q R q V with Vm = RT /p . The standard molar
⦵ ⦵ ⦵ ⦵

qm⦵ (H2O)/N A
Gibbs energy is then obtained with these substitutions and   
{Vm /N A (H2O)3 }q R (H2O)


after dividing through by n:  E0,m  RT ln 1/ 2
  
Standard molar {V ⦵ /N (H )3 }q R (H ) {V ⦵ /N (O )3 }q R (O )
qm⦵  m A
2 2 
 mA
2 2 

Gibbs energy
 
Gm (T )  Gm (0)  RT ln
⦵ ⦵
 (13F.9b)  qm⦵ (H2 )/N A   qm⦵(O2 )/N A 
NA [indistiinguishable,
independent molecules] N A1/2 {(H2 )(O2 )1/ 2 /(H2O)}3
 E0,m  RT ln
Vm 1/2 {q R (H2 )q R (O2 )1/ 2 /q R (H2O)}

where Gm (0) = E0,m , the molar ground-state energy of the


­system. Now substitute the data. The molar energy difference (with
bond dissociation energies in kilojoules per mole indicated) is

492
  428
 
 
920  436
 497

Example 13F.2 Calculating a standard Gibbs energy of E0,m  E0,m (H2O)  E0,m (H2 )  12 E0,m (O2 )  236 kJmol 1
formation from partition functions
Calculate the standard Gibbs energy of formation of H2O(g) Next, establish whether the temperature is high enough for the
at 25 °C. approximate expressions for the rotational partition functions
(Topic 13B) to be reliable:
Collect your thoughts Write the chemical equation for the
formation reaction, and then the expression for the standard H2O H2O H 2O H2 O2
Gibbs energy of formation in terms of the standard molar

X/cm , X = A, B, or C 27.877
−1 14.512 9.285 60.864 1.4457
Gibbs energy of each molecule; then express those Gibbs
energies in terms of the molecular molar partition functions. θR/K 40.1 20.9 13.4 87.6 2.1
Ignore molecular vibration as it is unlikely to be excited at
25 °C. The rotational constant for H2 is 60.864 cm−1, that For H2 the characteristic rotational temperature is not much
for O2 is 1.4457 cm−1, and those for H2O are 27.877, 14.512, smaller than 298 K so the rotational partition function must
and 9.285 cm−1. Before using the approximate form of the be computed by summing terms. For all the other molecules θR
rotational partition functions, you need to judge whether the is much smaller than 298 K and the approximate form of the
temperature is high enough: if it is not, use the full expression. partition function quoted above can be used. The data give the
For the values of E0,m, use bond enthalpies from Table 9C.3; following values for Λ and qR:
for precise calculations, bond energies (at T = 0) should be
used instead. You will need the following expressions from (H2 )  71.21 pm (O2 )  17.87 pm (H2O)  23.82 pm
Topic 13B: q R (H2 )  1.88 q R (O2 )  71.67 q R (H2O)  43.14
Linear molecule. Nonlinear molecule,
 2  2 It then follows that
 
3/2 1/ 2
h kT  kT     
2.48 kJmol 1
(J)  qR  q R  12       
(2mJkT )1/ 2 2hcB  hc   ABC   f G  236 kJmol  RT ln 0.0270  227 kJmol 1
⦵ 1
13F Derived functions 585

The value quoted in Table 2C.1 of the Resource section is ∆rG = cGm(C) + dGm(D) − {aGm(A) + bGm(B)}
⦵ ⦵ ⦵ ⦵ ⦵

−228.57 kJ mol−1.
= cGm(C,0) + dGm(D,0) − {aGm(A,0) + bGm(B,0)}
⦵ ⦵ ⦵ ⦵

Self-test 13F.2 Estimate the standard Gibbs energy of forma-


tion of NH3(g) at 25 °C. The rotational constants of NH3 are q C,m

q D,m

q A,m

q B,m

B  10.001 cm 1 and A  6.449 cm 1, and for N2 B  1.9987 cm 1. − RT c ln + d ln − a ln − b ln


NA NA NA NA
Answer: −16 kJ mol−1
Because Gm(J,0) = E0,m(J), the molar ground-state energy of the
species J, the first (boxed) term on the right is
cE0,m (C,0)  dE0,m (D,0)  {aE0,m (A,0)  bE0,m (B,0)}   r E0
Exercises Then, by using a ln x = ln xa and ln x + ln y = ln xy,
−1
E13F.1 A CO2 molecule is linear, with a rotational constant of 0.3902 cm .
( qC⦵,m /N A )c ( qD⦵,m /N A )d
Calculate the rotational contribution to the molar Gibbs energy at 298 K.  rG    r E0  RT ln

( qA⦵,m /N A )a ( qB⦵,m /N A )b
E13F.2 A CO2 molecule is linear, and the wavenumbers of its vibrational modes
are 1388.2 cm−1, 2349.2 cm−1, and 667.4 cm−1, the last being doubly degenerate
Step 2 Write an expression for K
and the others non-degenerate. Calculate the vibrational contribution to the
Recall that ∆rG = −RT ln K and use the above expression for

molar Gibbs energy at 298 K.
∆rG to give

2 2 −1
E13F.3 The ground level of Cl is P3/2 and a P1/2 level lies 881 cm above it.
Calculate the electronic contribution to the molar Gibbs energy of Cl atoms ( qC⦵,m /N A )c ( qD⦵,m /N A )d
at 500 K.  RT ln K   r E0  RT ln
( qA⦵,m /N A )a ( qB⦵,m /N A )b
This equation can now be rearranged into an expression for
ln K and the ∆ r E0 /RT term taken into the logarithm:
13F.2 Equilibrium constants
 r E0 ( q ⦵ /N )c ( q ⦵ /N )d
ln K    ln C⦵,m A a D⦵,m A b
The following discussion considers gas-phase reactions in RT ( qA ,m /N A ) ( qB,m /N A )
which the equilibrium constant is defined in terms of the par- ( qC⦵,m /N A )c ( qD⦵,m /N A )d
 ln e  r E0 /RT  ln
tial pressures of the reactants and products. ( qA⦵,m /N A )a ( qB⦵,m /N A )b
 ( q ⦵ /N )c ( q ⦵ /N )d 
 ln  C⦵,m A a D⦵,m A b e  r E0 /RT 
13F.2(a) The relation between K and the  ( qA ,m /N A ) ( qB,m /N A ) 
partition function
The quantity within the braces on the right is therefore identifi-
The standard molar Gibbs energy of a gas of independent mol- able as the equilibrium constant K
ecules is given by eqn 13F.9a, Gm (T )  Gm (0)  RT ln( qm /N A ),
⦵ ⦵ ⦵

( qC⦵,m /N A )c ( qD⦵,m /N A )d
in terms of the standard molar partition function, qm. The equi- ⦵
K e  r E0 /RT
librium constant K of a reaction is related to the standard reac- ( qA⦵,m /N A )a ( qB⦵,m /N A )b
tion Gibbs energy. The task is to combine these two relations The equilibrium constant  (13F.10a)
and so obtain an expression for the equilibrium constant in
The quantity ∆rE0 is the difference in molar energies of the
terms of the molecular partition functions of the reactants and
ground states of the products and reactants and is calculated
products.
from the bond dissociation energies of the species (Fig. 13F.1).

Continuum

How is that done? 13F.1 Relating the equilibrium constant


to partition functions
~
NAhcD0(reactants)
You need to use the expressions for the standard molar Gibbs ~
NAhcD0(products)
energies, Gm , of each species to find an expression for the

standard reaction Gibbs energy. Then find the equilibrium


ΔrE0
constant K by using eqn 6A.15 (∆rG = −RT ln K).

Step 1 Write an expression for ∆rG


Figure 13F.1 The definition of ΔrE0 for the calculation of


From eqn 13F.9b, the standard molar reaction Gibbs energy for equilibrium constants. The reactants are imagined as dissociating
the reaction aA + bB → cC + dD is into atoms and then forming the products from the atoms.
586 13 Statistical thermodynamics

In terms of the (signed) stoichiometric numbers introduced in Example 13F.3


Topic 2C, eqn 13F.10a can be written Evaluating an equilibrium constant
Evaluate the equilibrium constant for the dissociation Na2(g) →
2 Na(g) at 1000 K. The data for Na2 are: B  0.1547 cm 1,
J
q J,m

(13F.10b)
K= ∏
J
NA
e − Δr E0 /RT
The equilibrium constant   159.2 cm 1, and N AhcD 0  70.4 kJmol 1.
Collect your thoughts Recall that Na has a doublet ground state
(term symbol 2S1/2). You need to use eqn 13F.12 and the expres-
sions for the partition functions assembled in Topic 13B. For
13F.2(b) A dissociation equilibrium such a heavy diatomic molecule it is safe to use the approxi-
Equation 13F.10a can be used to write an expression for the mate expression for its rotational partition function (but check
equilibrium constant for the dissociation of a diatomic mol- that assumption for consistency). Remember that for a homo-
ecule X2: nuclear diatomic molecule, σ = 2.

pX2 The solution The partition functions and other quantities


X 2 (g )  2 X(g ) K = required are as follows:
pX 2 p 

According to eqn 13F.10a (with a = 1, b = 0, c = 2, and d = 0): Λ(Na2) = 8.14 pm Λ(Na) = 11.5 pm
R
q (Na2) = 2246 qV(Na2) = 4.885
( qX,m /N A )2

( qX,m )2⦵

K e  r E0 /RT  e  r E0 /RT  (13F.11a) g(Na) = 2 g(Na2) = 1


qX ,m /N A

2
qX ,m N A

2 hcD /kT = 8.47


0

with
There are many rotational states occupied, so the use of the
 r E0  2 E0,m (X ,0)  E0,m (X 2 ,0)  N A hcD 0 (X  X ) (13F.11b) approximate formula for qR(Na2) is valid. Then, from eqn 13F.12,

where N A hcD 0 (X − X ) is the (molar) dissociation energy of the 


kg m1 s 2

X–X bond. The standard molar partition functions of the atoms 2  (1.38110 JK 1 )  (1000 K )  (8.14 1012 m)3 8.47
2 23
K 12
e
X are 1 (105 Pa
)  (2246)  (4.885)  (11.5 10 m)
6

kg m1 s 2
⦵ ⦵
T Vm = RT/p  2.45
q
qE ⦵
V m g X RT Self-test 13F.3 Evaluate K at 1500 K. Is the answer consistent
q X ,m = g X ×

=
ΛX3 p ΛX3

with the dissociation being endothermic?


Answer: 52; yes
where gX is the degeneracy of the electronic ground state of
X. The diatomic molecule X2 also has rotational and vibra-
tional degrees of freedom, so its standard molar partition
function is


⦵ R V
Vm R V g X2 RT qX2 qX2 Contributions to the equilibrium
13F.2(c)
qX⦵
 g X2 qX qX 
2 ,m
3X2 2 2 p 3X2 ⦵ constant
To appreciate the physical basis of equilibrium constants, con-
where g X2 is the degeneracy of the electronic ground state of X2.
sider a simple R  P gas-phase equilibrium (R for reactants, P
It follows that
for products).
Figure 13F.2 shows two sets of energy levels; one set of states
( g X RT /p ΛX3 )2

%
K= e− N A hcD0 (X − X)/ RT
belongs to R, and the other belongs to P. The populations of the
g X N A RT qXR qXV /p ΛX3

2 2 2 2 states are given by the Boltzmann distribution, and are inde-


R= NAk pendent of whether any given state happens to belong to R or
to P. A single Boltzmann distribution spreads, without distinc-
g X2 kT ΛX3
=
%
e − hcD (X−X)/kT
2 0
(13F.12) tion, over the two sets of states. If the spacings of R and P are
g X p qXR qXV ΛX6

2 2 2 similar (as in Fig. 13F.2), and the ground state of P lies above that
of R, the diagram indicates that R will dominate in the equilib-
All the quantities in this expression can be calculated from rium mixture. However, if P has a high density of states (a large
spectroscopic data. number of states in a given energy range, as in Fig. 13F.3), then,
13F Derived functions 587

N N
N R  N r  e N P  N p  e
P   r   p

r q r p q p
R
The sum over the states of R is its partition function, qR, so
NR = NqR/q. The sum over the states of P is also a partition
function, but the energies are measured from the ground state
of the combined system, which is the ground state of R. If the
ΔrE0 energies of the ground states of R and P are separated by Δε0 (as
in Fig. 13F.3), then the energies of the states of P relative to the
ground state of R,  p , can be written  p   p   0, where ε p are
the energies of the states of P relative to the ground state of P.
Figure 13F.2 The array of R(eactant) and P(roduct) energy levels. Writing  p in this way gives NP as
At equilibrium the populations of the states are given by the
Boltzmann distribution, and are independent of whether any state N
N      N qP   0
q
 e p  e 0 
  ( p   0 )
belongs to R or to P. Here more states of R are populated than are NP  e  e
p q p  q
states of P because the ground state of P lies above that of R. As
ΔrE0 increases, R becomes more and more dominant. N qP
 e r E0 /RT
q
P The switch from ∆ε0/kT to ∆rE0/RT in the last step is the con-
R
version of molecular energies to molar energies. In the final
expression qP is the partition function of P with the energy of
its ground state taken as zero, in the usual way.
Step 2 Write an expression for the equilibrium constant
The ratio of populations is

N P qP  r E0 /RT
 e
ΔrE0 N R qR

The equilibrium constant of the R  P reaction is propor-


Figure 13F.3 It is important to take into account the densities tional to the ratio of the numbers of the two types of molecule.
of states of the molecules. Even though P might lie above R in Therefore,
energy (that is, ΔrE0 is positive), P might have so many states that
its total population dominates in the mixture. qP − Δ E /RT (13F.13)
K=
qR e
r 0

The equilibrium constant

even though its ground-state energy lies above that of R, the spe-
For an R  P equilibrium, the Vm factors in the partition

cies P might still dominate at equilibrium.
It is quite easy to show that the ratio of numbers of R and P mol- functions cancel, so the appearance of q in place of q has no ef- ⦵

ecules at equilibrium is given by a Boltzmann-like expression. fect. In the case of a more general reaction, the conversion from
q to q comes about at the stage of converting the pressures that

occur in K to numbers of molecules.


How is that done? 13F.2 Relating the equilibrium constant
The implications of eqn 13F.13 can be seen most clearly by ex-
to state populations aggerating the molecular features that contribute to it. Suppose
You need to start the derivation by noting that the population that R has only a single accessible level, which implies that qR = 1.
in a state i of the composite (R,P) system is N i  Ne  i /q , Also suppose that P has a large number of evenly, closely spaced
where N is the total number of molecules. levels, with spacing ε (Fig. 13F.4). The partition function for
such an array is calculated in Brief illustration 13B.1, and is
Step 1 Write expressions for the numbers of R and P molecules q = 1/(1−e−βε). Provided the levels are closely spaced compared
The total number of R molecules is the sum of the popula- to kT (ε << kT), the partition function is well-approximated as
tions of the (R,P) system taken over the states belonging to R; q ≈ kT/ε (Topic 13B). In this model system, the equilibrium
these states are labelled r with energies εr. The total number of constant is therefore given by
P molecules is the sum over the states belonging to P; these
states are labelled p with energies  p (the prime is explained kT  r E0 /RT
K e  (13F.14)
in a moment): 
588 13 Statistical thermodynamics

P reflects the predominance of P at equilibrium on account of its


high density of states.
At low temperatures K << 1 and the system consists entirely
R
of R. At high temperatures, meaning RT >> ∆rE0, the exponen-
tial function approaches 1. If ε is such that at this high tempera-
ε
ture the factor kT/ε is large, P will be dominant at equilibrium.
In this endothermic reaction (endothermic because P lies above
ΔE0
R) a rise in temperature favours P, because its states become ac-
cessible. This is the behaviour described, from a macroscopic
perspective, in Topic 6B.
The model also shows why the Gibbs energy, G, and not just
Figure 13F.4 The model used in the text for exploring the effects the enthalpy, determines the position of equilibrium. It shows
of energy separations and densities of states on equilibria. The
that the density of states (and hence the entropy) of each spe-
products P can dominate provided ΔrE0 is not too large and P has
cies as well as their relative energies controls the distribution of
an appreciable density of states.
populations and hence the value of the equilibrium constant.

When ∆rE0 is large compared to RT, the exponential term dom- Exercise
inates and K << 1, which implies that very little P is present at
E13F.4 Calculate the equilibrium constant for the reaction A2(g) ⇌ 2A(g) at
equilibrium. When ∆rE0 is smaller but still positive, K can ex- 500 K using the following data: mass of A = 120 mu; rotational temperature of
ceed 1 if the factor kT/ε is large enough that it can overcome A2 = 3.00 K; D 0  300 cm 1 . Assume that the vibrational partition function = 1,
the exponential term (which is less than 1). The size of K then and that the electronic ground states are all non-degenerate.

Checklist of concepts
☐ 1. The thermodynamic functions A, p, H, and G can be ☐ 4. The physical basis of chemical equilibrium can be
calculated from the canonical partition function. understood in terms of a competition between energy
☐ 2. The equilibrium constant can be written in terms of the separations and densities of states.
partition functions of the reactants and products.
☐ 3. The equilibrium constant for dissociation of a diatomic
molecule in the gas phase may be calculated from spec-
troscopic data.

Checklist of equations
Property Equation Comment Equation number
Helmholtz energy A(T) = A(0) − kT ln Q 13F.2
Pressure p = kT(∂ln Q /∂V)T 13F.3
Enthalpy H(T) = H(0) − (∂ln Q /∂β)V + kTV(∂ln Q /∂V)T 13F.4
Gibbs energy G(T) = G(0) − kT ln Q + kTV(∂ln Q /∂V)T 13F.6
G(T) = G(0) − nRT ln(qm/NA) Perfect gas 13F.9a

Equilibrium constant  J


K  ( qJ⦵,m /N A ) J e  r E0 /RT Perfect gas 13F.10b
Exercises and problems 589

FOCUS 13 Statistical thermodynamics

To test your understanding of this material, work through Assume that all gases are perfect and that data refer to 298 K
the Exercises, Additional exercises, Discussion questions, and unless otherwise stated.
Problems found throughout this Focus.
Selected solutions can be found at the end of this Focus in
the e-book. Solutions to even-numbered questions are available
online only to lecturers.

TOPIC 13A The Boltzmann distribution


Discussion questions
D13A.1 Discuss the relations between ‘population’, ‘configuration’, and ‘weight’. D13A.3 What is temperature?
What is the significance of the most probable configuration?
D13A.4 Summarize the role of the Boltzmann distribution in chemistry.
D13A.2 What is the significance and importance of the principle of equal a
priori probabilities?

Additional exercises
E13A.6 Calculate the weight of the configuration in which 21 objects are E13A.10 What is the temperature of a two-level system of energy separation
distributed in the arrangement 6, 0, 5, 0, 4, 0, 3, 0, 2, 0, 0, 1. equivalent to 300 cm−1 when the population of the upper state is one-half that
of the lower state?
E13A.7 Evaluate 8! by using (i) the exact definition of a factorial, (ii) Stirling’s
approximation (eqn 13A.2), and (iii) the more accurate version of Stirling’s E13A.11 Calculate the relative populations of a spherical rotor at 298 K in the
approximation, x! ≈ (2π)1/2x x+1/2e−x. levels with J = 0 and J = 5, given that B  2.71 cm 1.
E13A.8 Evaluate 10! by using (i) the definition of a factorial; (ii) Stirling’s E13A.12 A certain molecule has a doubly degenerate excited state lying at
approximation (eqn 13A.2), and (iii) the more accurate version of Stirling’s 360 cm−1 above the non-degenerate ground state. At what temperature will
approximation given in Exercise E13A.7. 15 per cent of the molecules be in the upper state?
E13A.9 What are the relative populations of the states of a two-level system as
the temperature approaches zero?

Problems
P13A.1 A sample consisting of five molecules has a total energy 5ε. Each greatest weight, expressing the temperature as a multiple of ε, and compare
molecule is able to occupy states of energy jε, with j = 0, 1, 2,…. (a) Calculate it to the distribution predicted by the Boltzmann expression. Explore what
the weight of the configuration in which the molecules are distributed evenly happens as the value of the total energy is changed.
over the available states with the stated total energy. (b) Draw up a table
with columns headed by the energy of the states and write beneath them all P13A.4 Suppose that two conformations A and B of a molecule differ in
configurations that are consistent with the total energy. Calculate the weights energy by 5.0 kJ mol−1, and a third conformation C lies 0.50 kJ mol−1 above B.
of each configuration and identify the most probable configurations. What proportion of molecules will be in conformation B at 273 K, with each
conformation treated as a single energy level?
P13A.2 A sample of nine molecules is numerically tractable but on the verge of
being thermodynamically significant. A sample consisting of nine molecules P13A.5 A certain atom has a doubly degenerate ground state and an upper
has a total energy 9ε. Each molecule is able to occupy states of energy jε, level of four degenerate states at 450 cm−1 above the ground level. In an
with j = 0, 1, 2,…. Draw up a table of configurations. Before evaluating the atomic beam study of the atoms it was observed that 30 per cent of the atoms
weights of the configurations, guess (by looking for the most ‘exponential’ were in the upper level, and the translational temperature of the beam was
distribution of populations) which of the configurations will turn out to be the 300 K. Are the electronic states of the atoms in thermal equilibrium with
most probable. Go on to calculate the weights and identify the most probable the translational states? In other words, does the distribution of electronic
configuration. states correspond to the same temperature as the distribution of translational
states?
P13A.3 Use mathematical software to evaluate W for N = 20 for at least
ten distributions over a uniform ladder of energy levels with separation ε, P13A.6 Explore the consequences of using the full version of Stirling’s
ensuring that the total energy is constant at 10ε. Identify the configuration of approximation, x! ≈ (2π)1/2xx+1/2e−x, in the development of the expression for
590 13 Statistical thermodynamics

the configuration of greatest weight. Does the more accurate approximation average temperature. Obtain the barometric formula from the Boltzmann
have a significant effect on the form of the Boltzmann distribution? distribution. Recall that the potential energy of a particle at height h above the
‡ surface of the Earth is mgh. Convert the barometric formula from pressure to
P13A.7 The variation of the atmospheric pressure p with altitude h is
number density, N . Compare the relative number densities, N (h)/N (0), for
predicted by the barometric formula to be p = p0e−h/H where p0 is the pressure
O2 and H2O at h = 8.0 km, a typical cruising altitude for commercial aircraft.
at sea level and H = RT/Mg with M the average molar mass of air and T the

TOPIC 13B Molecular partition functions


Discussion questions
D13B.1 Describe the physical significance of the partition function. D13B.3 Why and when is it necessary to include a symmetry number in the
calculation of a partition function?
D13B.2 What is the difference between a ‘state’ and an ‘energy level’? Why is it
important to make this distinction?

Additional exercises
E13B.8 Calculate (i) the thermal wavelength, (ii) the translational partition eqn 13B.12b, which gives the high-temperature limit? Hint: Use mathematical
function of a Ne atom in a cubic box of side 1.00 cm at 300 K and 3000 K. software or a spreadsheet.
E13B.9 Calculate the ratio of the translational partition functions of H2 and He E13B.18 Give the symmetry number for each of the following molecules:
at the same temperature and volume. (i) CO, (ii) O2, (iii) H2S, (iv) SiH4, and (v) CHCl3.
E13B.10 Calculate the ratio of the translational partition functions of Ar and E13B.19 Give the symmetry number for each of the following molecules:
Ne at the same temperature and volume. (i) CO2, (ii) O3, (iii) SO3, (iv) SF6, and (v) Al2Cl6.
E13B.11 The bond length of N2 is 109.75 pm. Use the high-temperature E13B.20 Evaluate the rotational partition function of pyridine, C5H5N, at
approximation to calculate the rotational partition function of the molecule 25 °C given that A  0.2014 cm 1, B  0.1936 cm 1 , C  0.0987 cm 1. Take the
at 300 K. symmetry number into account.
−1
E13B.12 The NOF molecule is an asymmetric rotor with rotational constants E13B.21 The vibrational wavenumber of Br2 is 323.2 cm . Evaluate the
3.1752 cm−1, 0.3951 cm−1, and 0.3505 cm−1. Calculate the rotational partition vibrational partition function explicitly (without approximation) and plot its
function of the molecule at (i) 25 °C, (ii) 100 °C. value as a function of temperature. At what temperature is the value within
5 per cent of the value calculated from eqn 13B.16, which is valid at high
E13B.13 The H2O molecule is an asymmetric rotor with rotational constants
temperatures?
27.877 cm−1, 14.512 cm−1, and 9.285 cm−1. Calculate the rotational partition
−1
function of the molecule at (i) 25 °C, (ii) 100 °C. E13B.22 The vibrational wavenumber of I2 is 214.5 cm . Evaluate the
−1 vibrational partition function explicitly (without approximation) and plot its
E13B.14 The rotational constant of CO is 1.931 cm . Evaluate the rotational
value as a function of temperature. At what temperature is the value within
partition function explicitly (without approximation) and plot its value as a
5 per cent of the value calculated from eqn 13B.16, which is valid at high
function of temperature. At what temperature is the value within 5 per cent of
temperatures?
the value calculated by using eqn 13B.12a, which gives the high-temperature
limit? Hint: Use mathematical software or a spreadsheet. E13B.23 Calculate the vibrational partition function of HCN at 900 K given
−1 the wavenumbers 3311 cm−1 (symmetric stretch), 712 cm−1 (bend; doubly
E13B.15 The rotational constant of HI is 6.511 cm . Evaluate the rotational
degenerate), 2097 cm−1 (asymmetric stretch).
partition function explicitly (without approximation) and plot its value as a
function of temperature. At what temperature is the value within 5 per cent of E13B.24 Calculate the vibrational partition function of CCl4 at 500 K given
the value calculated by using eqn 13B.12a, which gives the high-temperature the wavenumbers 459 cm−1 (symmetric stretch, non-degenerate), 217 cm−1
limit? Hint: Use mathematical software or a spreadsheet. (deformation, doubly degenerate), 776 cm−1 (deformation, triply degenerate),
−1 314 cm−1 (deformation, triply degenerate).
E13B.16 The rotational constant of CH4 is 5.241 cm . Evaluate the rotational
partition function explicitly (without approximation but ignoring the role E13B.25 Calculate the vibrational partition function of CI4 at 500 K given
of nuclear spin) and plot its value as a function of temperature. At what the wavenumbers 178 cm−1 (symmetric stretch, non-degenerate), 90 cm−1
temperature is the value within 5 per cent of the value calculated by using (deformation, doubly degenerate), 555 cm−1 (deformation, triply degenerate),
eqn 13B.12b, which gives the high-temperature limit? Hint: Use mathematical 125 cm−1 (deformation, triply degenerate).
software or a spreadsheet.
E13B.26 A certain atom has a triply degenerate ground level, a non-degenerate
−1
E13B.17 The rotational constant of CCl4 is 0.0572 cm . Evaluate the rotational electronically excited level at 850 cm−1, and a fivefold degenerate level at
partition function explicitly (without approximation but ignoring the role 1100 cm−1. Calculate the partition function of these electronic states at 2000 K.
of nuclear spin) and plot its value as a function of temperature. At what What is the relative population of each level at 2000 K?
temperature is the value within 5 per cent of the value calculated by using


These problems were supplied by Charles Trapp and Carmen Giunta.
Exercises and problems 591

Problems
3 3 3 5
P13B.1 Consider a three-level system with levels 0, ε, and 2ε. Plot the partition P13B.6 The four lowest electronic levels of a Ti atom are F2, F3, F4, and F1, at
−1
function against kT/ε. 0, 170, 387, and 6557 cm , respectively. There are many other electronic states
at higher energies. The boiling point of titanium is 3287 °C. What are the
P13B.2 Plot the temperature dependence of the vibrational contribution to the
relative populations of these levels at the boiling point? Hint: The degeneracies
molecular partition function for several values of the vibrational wavenumber.
of the levels are 2J + 1.
Estimate from your plots the temperature at which the partition function falls

to within 10 per cent of the value expected at the high-temperature limit. P13B.7 J. Sugar and A. Musgrove (J. Phys. Chem. Ref. Data 22, 1213 (1993))
have published tables of energy levels for germanium atoms and cations from
P13B.3 This problem is best done using mathematical software. Equation
Ge+ to Ge31+. The lowest-lying energy levels in neutral Ge are as follows:
13B.15 is the partition function for a harmonic oscillator. Consider a Morse
oscillator (Topic 11C) in which the energy levels are given by 3 3 3 1 1
P0 P1 P2 D2 S0
Ev   v   hc   v  
2
1
2
1
2 hcx e −1
(E/hc)/cm 0 557.1 1410.0 7125.3 16 367.3
Evaluate the partition function for this oscillator, remembering to measure
energies from the lowest level and to note that there is only a finite number of Calculate the electronic partition function at 298 K and 1000 K by direct
bound-state levels. Plot the partition function against kT /hcν for values of xe summation. Hint: The degeneracy of a level J is 2J + 1.
ranging from 0 to 0.1, and—on the same graph—compare your results with P13B.8 The pure rotational microwave spectrum (Topic 11B) of HCl has
that for a harmonic oscillator. absorption lines at the following wavenumbers (in cm−1): 21.19, 42.37,
P13B.4 Explore the conditions under which the ‘integral’ approximation 63.56, 84.75, 105.93, 127.12, 148.31, 169.49, 190.68, 211.87, 233.06, 254.24,
for the translational partition function is not valid by considering the 275.43, 296.62, 317.80, 338.99, 360.18, 381.36, 402.55, 423.74, 444.92, 466.11,
translational partition function of an H atom in a one-dimensional box of side 487.30, 508.48. Calculate the rotational partition function at 25 °C by direct
comparable to that of a typical nanoparticle, 100 nm. Estimate the temperature summation.
at which, according to the integral approximation, q = 10 and evaluate the   5.097 cm and
P13B.9 The rotational constants of CH3Cl are A
1

exact partition function at that temperature. B  0.443 cm 1 . Evaluate the rotational partition function explicitly (without
P13B.5 (a) Calculate the electronic partition function of a tellurium atom at approximation but ignoring the role of nuclear spin) and plot its value as a
(i) 298 K, (ii) 5000 K by direct summation using the following data: function of temperature. At what temperature is the value within 5 per cent
of the value calculated by using eqn 13B.12a, which applies to the high-
Term Degeneracy Wavenumber/cm−1 temperature limit? Hint: Use mathematical software or a spreadsheet.
Ground 5 0 P13B.10 Calculate, by explicit summation, the vibrational partition function
of I2 molecules at (a) 100 K, (b) 298 K given that its vibrational energy levels
1 1 4707
lie at the following wavenumbers above the zero-point energy level: 0, 215.30,
2 3 4751 425.39, 636.27, 845.93 cm−1. What proportion of I2 molecules are in the
3 5 10 559 ground and first two excited levels at the two temperatures?

(b) What proportion of the Te atoms are in the ground term and in the term
labelled 2 at the two temperatures?

TOPIC 13C Molecular energies


Discussion questions
D13C.1 Identify the conditions under which energies predicted from the D13C.2 Describe how the mean energy of a system composed of two levels
equipartition theorem coincide with energies computed by using partition varies with temperature.
functions.

Additional exercises
E13C.5 Compute the mean energy at 400 K of a two-level system of energy of temperature. At what temperature is the equipartition value within 5 per
separation equivalent to 600 cm−1.  CH )  5.241 cm 1.
cent of the accurate value? B( 4

E13C.6 Use mathematical software or a spreadsheet to evaluate, by explicit E13C.9 Use mathematical software or a spreadsheet to evaluate, by explicit
summation, the mean rotational energy of CO and plot its value as a function summation, the mean rotational energy of CCl4 and plot its value as a
of temperature. At what temperature is the equipartition value within 5 per function of temperature. At what temperature is the equipartition value within
 CO)  1.931 cm 1.
cent of the accurate value? B(  CCl )  0.0572 cm 1.
5 per cent of the accurate value? B( 4

E13C.7 Use mathematical software or a spreadsheet to evaluate, by explicit E13C.10 Use mathematical software or a spreadsheet to evaluate the mean
summation, the mean rotational energy of HI and plot its value as a function vibrational energy of Br2 and plot its value as a function of temperature. At
of temperature. At what temperature is the equipartition value within 5 per what temperature is the equipartition value within 5 per cent of the accurate
 HI)  6.511 cm 1.
cent of the accurate value? B( value? Use   323.2 cm 1.
E13C.8 Use mathematical software or a spreadsheet to evaluate, by explicit E13C.11 Use mathematical software or a spreadsheet to evaluate the mean
summation, the mean rotational energy of CH4 and plot its value as a function vibrational energy of I2 and plot its value as a function of temperature. At what
592 13 Statistical thermodynamics

temperature is the equipartition value within 5 per cent of the accurate value? is the equipartition value within 5 per cent of the accurate value? Use the
Use   214.5 cm 1 . wavenumbers 459 cm−1 (symmetric stretch, non-degenerate), 217 cm−1
(deformation, doubly degenerate), 776 cm−1 (deformation, triply degenerate),
E13C.12 Use mathematical software or a spreadsheet to evaluate the mean
314 cm−1 (deformation, triply degenerate).
vibrational energy of CS2 and plot its value as a function of temperature. At
what temperature is the equipartition value within 5 per cent of the accurate E13C.15 Evaluate, by explicit summation, the mean vibrational energy of
value? Use the wavenumbers 658 cm−1 (symmetric stretch), 397 cm−1 (bend; CI4 and plot its value as a function of temperature. At what temperature
doubly degenerate), 1535 cm−1 (asymmetric stretch). is the equipartition value within 5 per cent of the accurate value? Use the
wavenumbers 178 cm−1 (symmetric stretch, non-degenerate), 90 cm−1
E13C.13 Use mathematical software or a spreadsheet to evaluate the mean
(deformation, doubly degenerate), 555 cm−1 (deformation, triply degenerate),
vibrational energy of HCN and plot its value as a function of temperature. At
125 cm−1 (deformation, triply degenerate).
what temperature is the equipartition value within 5 per cent of the accurate
value? Use the wavenumbers 3311 cm−1 (symmetric stretch), 712 cm−1 (bend; E13C.16 Calculate the mean contribution to the electronic energy at 2000 K for
doubly degenerate), 2097 cm−1 (asymmetric stretch). a sample composed of the atoms specified in Exercise E13B.26.
E13C.14 Evaluate, by explicit summation, the mean vibrational energy of
CCl4 and plot its value as a function of temperature. At what temperature

Problems
P13C.1 Evaluate, by explicit summation, the mean rotational energy of CH3Cl Evaluate the partition function and mean energy of an electron as a function
and plot its value as a function of temperature. At what temperature is the of temperature. Hints: Remember to measure energies from the lowest level.
equipartition value within 5 per cent of the accurate value? Use A  5.097 cm 1 Note that 2 /2me R 2 k has dimensions of temperature, so can be used as
and B  0.443 cm 1 . the characteristic temperature θ of the system. It follows that the partition
function can be expressed in terms of Enl  Xnl2 k and the dimensionless
P13C.2 Deduce an expression for the mean energy when each molecule can
parameter T/θ. Let T/θ range from 0 to 25.
exist in states with energies 0, ε, and 2ε.
P13C.7 The NO molecule has a doubly degenerate excited electronic level
P13C.3 How much energy does it take to raise the temperature of
121.1 cm−1 above the doubly degenerate electronic ground term. Calculate and
1.0 mol H2O(g) from 100 °C to 200 °C at constant volume? Consider only
plot the electronic partition function of NO from T = 0 to 1000 K. Evaluate
translational and rotational contributions to the heat capacity.
(a) the populations of the levels and (b) the mean electronic energy at 300 K.
P13C.4 What must the temperature be before the energy estimated from
P13C.8 Consider a system of N molecules with energy levels εj = jε and
the equipartition theorem is within 2 per cent of the energy given by
j = 0, 1, 2,…. (a) Show that if the mean energy per molecule is aε, then the
 V   hc /(e  hc  1)?
temperature is given by
P13C.5 Suppose a collection of species with total spin S = 1 is exposed to a
magnetic field of 2.5 T. Calculate the mean energy of this system at 298 K. Use 1  1
 ln 1 
g = 2.0.   a 

P13C.6 An electron trapped in an infinitely deep spherical well of radius R, Evaluate the temperature for a system in which the mean energy is ε, taking ε
such as may be encountered in the investigation of nanoparticles, has energies equivalent to 50 cm−1. (b) Calculate the molecular partition function q for the
given by the expression Enl = 2 Xnl2 /2me R 2 with Xnl the value obtained by system when its mean energy is aε.
searching for the zeroes of the spherical Bessel functions. The first six values 2 1/2
P13C.9 Deduce an expression for the root-mean-square energy, 〈ε 〉 , in terms
(with a degeneracy of the corresponding energy level equal to 2l + 1) are as
of the partition function and hence an expression for the root-mean-square
follows:
deviation from the mean, Δε = (〈ε2〉 − 〈ε〉2)1/2. Evaluate the resulting expression
 
for a harmonic oscillator. Hint: Use  2   (1/q)  e j  j2 .
n 1 1 1 2 1 2 j

l 0 1 2 0 3 1
Xnl 3.142 4.493 5.763 6.283 6.988 7.725

TOPIC 13D The canonical ensemble


Discussion questions
D13D.1 Why is the concept of a canonical ensemble required? D13D.3 Under what circumstances may identical particles be regarded as
distinguishable?
D13D.2 Explain what is meant by an ensemble and why it is useful in statistical
thermodynamics. D13D.4 What is meant by the ‘thermodynamic limit’?

Additional exercises
E13D.4 Identify the systems for which it is essential to include a factor of E13D.5 Identify the systems for which it is essential to include a factor of 1/N!
1/N! on going from Q to q: (i) a sample of helium gas, (ii) a sample of carbon on going from Q to q: (i) a sample of carbon dioxide gas, (ii) a sample of
monoxide gas, (iii) a solid sample of carbon monoxide, (iv) water vapour. graphite, (iii) a sample of diamond, (iv) ice.
Exercises and problems 593

Problems
P13D.1 For a perfect gas, the canonical partition function, Q , is related to P13D.2 Use statistical thermodynamic arguments to show that for a perfect
the molecular partition function q by Q = q N/N!. In Topic 13F it is established gas, (∂E/∂V)T = 0
that p = kT(∂ln Q /∂V)T. Use the expression for q to derive the perfect gas law
pV = nRT.

TOPIC 13E The internal energy and the entropy


Discussion questions
D13E.1 Describe the molecular features that affect the magnitudes of the D13E.4 Justify the differences between the partition-function expression
constant-volume molar heat capacity of a molecular substance. for the entropy for distinguishable particles and the expression for
indistinguishable particles.
D13E.2 Discuss and illustrate the proposition that 1/T is a more natural
measurement of temperature than T itself. D13E.5 Account for the temperature and volume dependence of the entropy of
a perfect gas in terms of the Boltzmann distribution.
D13E.3 Discuss the relationship between the thermodynamic and statistical
definitions of entropy. D13E.6 Explain the origin of residual entropy.

Additional exercises
E13E.8 Use the equipartition theorem to estimate the constant-volume molar E13E.14 At what temperature is the translational contribution to the standard
heat capacity of (i) O3, (ii) C2H6, (iii) CO2 in the gas phase at 25 °C. molar entropy of CO2(g) equal to that of H2O(g) at 298 K?
E13E.9 Estimate the value of γ = Cp,m/CV,m for carbon dioxide. Do this E13E.15 Calculate the rotational partition function of SO2 at 298 K from its
calculation with and without the vibrational contribution to the energy. rotational constants 2.027 36 cm−1, 0.344 17 cm−1, and 0.293 535 cm−1 and use
Which is closer to the experimental value at 25 °C? Hint: Note that your result to calculate the rotational contribution to the molar entropy of
Cp,m − CV,m = R for a perfect gas. sulfur dioxide at 25 °C.
1
E13E.10 The first electronically excited state of O2 is ∆g (doubly degenerate) E13E.16 Estimate the contribution of the spin to the molar entropy of a solid
and lies 7918.1 cm−1 above the ground state, which is 3  g (triply degenerate). sample of a d-metal complex with S = 25 .
Calculate the electronic contribution to the heat capacity of O2 at 400 K. E13E.17 Predict the vibrational contribution to the standard molar entropy of
E13E.11 Plot the molar heat capacity of a collection of harmonic oscillators methanoic acid (formic acid, HCOOH) vapour at (i) 298 K, (ii) 500 K. The
as a function of T/θV, and predict the vibrational heat capacity of ethyne normal modes occur at wavenumbers 3570, 2943, 1770, 1387, 1229, 1105,
at (i) 298 K, (ii) 500 K. The normal modes (and their degeneracies in 1033, 638, 625 cm−1.
parentheses) occur at wavenumbers 612(2), 729(2), 1974, 3287, and 3374 cm−1. E13E.18 Predict the vibrational contribution to the standard molar
E13E.12 Plot the molar entropy of a collection of harmonic oscillators as entropy of ethyne at (i) 298 K, (ii) 500 K. The normal modes (and their
a function of T/θV, and predict the standard molar entropy of ethyne at degeneracies in parentheses) occur at wavenumbers 612(2), 729(2), 1974,
(i) 298 K, (ii) 500 K. For data, see Exercise E13E.11. 3287, and 3374 cm−1.

E13E.13 Calculate the translational contribution to the standard molar entropy


at 298 K of (i) H2O(g), (ii) CO2(g).

Problems
P13E.1 An NO molecule has a doubly degenerate electronic ground state and a of CH3 about its threefold rotation axis (the axis that passes through the
doubly degenerate excited state at 121.1 cm−1. Calculate and plot the electronic C atom and the centre of the equilateral triangle formed by the H atoms) is
contribution to the molar heat capacity of the molecule up to 500 K. 5.341 × 10−47 kg m2.
P13E.2 Explore whether a magnetic field can influence the heat capacity of a P13E.4 Calculate the temperature dependence of the heat capacity of p-H2
paramagnetic molecule by calculating the electronic contribution to the heat (in which only rotational states with even values of J are populated) at low
capacity of an NO2 molecule in a magnetic field. Estimate the total constant- temperatures on the basis that its rotational levels J = 0 and J = 2 constitute
volume heat capacity using equipartition, and calculate the percentage change a system that resembles a two-level system except for the degeneracy of
in heat capacity brought about by a 5.0 T magnetic field at (a) 50 K, (b) 298 K. the upper level. Use B  60.864 cm 1 and sketch the heat capacity curve.
Hints: Recall that NO2 has one unpaired electron. Assume that the sample is in The experimental heat capacity of p-H2 does in fact show a peak at low
the gas phase at 50 K and 298 K. temperatures.

P13E.3 The energy levels of a CH3 group attached to a larger fragment are P13E.5 In a spectroscopic study of buckminsterfullerene C60, F. Negri et al.
given by the expression for a particle on a ring, provided the group is rotating (J. Phys. Chem. 100, 10849 (1996)) reviewed the wavenumbers of all the
freely. What is the high-temperature contribution to the heat capacity and vibrational modes of the molecule:
entropy of such a freely rotating group at 25 °C? Hint: The moment of inertia
594 13 Statistical thermodynamics

Mode Number Degeneracy Wavenumber/cm−1 diatomic halogen anions. The ground state of F2− is 2  u with a fundamental
vibrational wavenumber of 450.0 cm−1 and equilibrium internuclear distance
Au 1 1 976
of 190.0 pm. The first two excited states are at 1.609 and 1.702 eV above the
T1u 4 3 525, 578, 1180, 1430 ground state. Compute the standard molar entropy of F2− at 298 K.
T2u 5 3 354, 715, 1037, 1190, 1540 ‡
P13E.12 Treat carbon monoxide as a perfect gas and apply equilibrium
Gu 6 4 345, 757, 776, 963, 1315, 1410 statistical thermodynamics to the study of its properties, as specified below,
in the temperature range 100–1000 K at 1 bar. (a) Examine the probability
Hu 7 5 403, 525, 667, 738, 1215, 1342, 1566 distribution of molecules over available rotational and vibrational states.
(b) Explore numerically the differences, if any, between the rotational
molecular partition function as calculated with the discrete energy
How many modes have a vibrational temperature θV below 1000 K? Estimate
distribution with that predicted by the high-temperature limit. (c) Plot against
the molar constant-volume heat capacity of C60 at 1000 K, counting as active
temperature the individual contributions to Um(T) − Um(100 K), CV,m(T),
all modes with θV below this temperature.
and Sm(T) − Sm(100 K) made by the translational, rotational, and vibrational
P13E.6 Plot the function dS/dT for a two-level system, the temperature degrees of freedom. Hints: Let   2169.8 cm 1 and B  1.931 cm 1 ; neglect
coefficient of its entropy, against kT/ε. Is there a temperature at which this anharmonicity and centrifugal distortion.
coefficient passes through a maximum? If you find a maximum, explain its
P13E.13 The energy levels of a Morse oscillator are given in Problem P13B.3.
physical origins.
Set up the expression for the molar entropy of a collection of Morse oscillators
P13E.7 Derive an expression for the molar entropy of an equally spaced three- and plot it as a function of kT /hcν for a series of anharmonicity constants
level system; taking the spacing as ε. ranging from 0 to 0.01. Take into account only the finite number of bound
states. On the same graph plot the entropy of a harmonic oscillator and
P13E.8 Although expressions like 〈ε〉 = −d ln q/dβ are useful for formal
investigate how the two diverge.
manipulations in statistical thermodynamics, and for expressing
thermodynamic functions in neat formulas, they are sometimes more trouble P13E.14 Explore how the entropy of a collection of two-level systems behaves
than they are worth in practical applications. When presented with a table of when the temperature is formally allowed to become negative. You should also
energy levels, it is often much more convenient to evaluate the following sums construct a graph in which the temperature is replaced by the variable
directly (the dots simply identify the different functions): β = 1/kT. Account for the appearance of the graphs physically.
q  e q    j e q   ( j )2 e
  j   j   j
P13E.15 Derive the Sackur–Tetrode equation for a monatomic gas confined to
j j j a two-dimensional surface, and hence derive an expression for the standard
(a) Derive expressions for the internal energy, heat capacity, and entropy in molar entropy of condensation to form a mobile surface film.
terms of these three functions. (b) Apply the technique to the calculation of
P13E.16 The heat capacity ratio of a gas determines the speed of sound in it
the electronic contribution to the constant-volume molar heat capacity of
through the formula cs = (γRT/M)1/2, where γ = Cp,m/CV,m and M is the molar
magnesium vapour at 5000 K using the following data:
mass of the gas. Deduce an expression for the speed of sound in a perfect gas
1 3 3 3 1 3
of (a) diatomic, (b) linear triatomic, (c) nonlinear triatomic molecules at high
Term S P0 P1 P2 P1 S1 temperatures (with translation and rotation active). Estimate the speed of
Degeneracy 1 1 3 5 3 3 sound in air at 25 °C. Hint: Note that Cp,m − CV,m = R for a perfect gas.
 /cm 1
0 21 850 21 870 21 911 35 051 41 197 8
P13E.17 An average human DNA molecule has 5 × 10 binucleotides (rungs on
the DNA ladder) of four different kinds. If each rung were a random choice of
one of these four possibilities, what would be the residual entropy associated
P13E.9 The pure rotational microwave spectrum (Topic 11B) of HCl has
with this typical DNA molecule?
absorption lines at the following wavenumbers (in cm−1): 21.19, 42.37,
63.56, 84.75, 105.93, 127.12, 148.31, 169.49, 190.68, 211.87, 233.06, 254.24, P13E.18 It is possible to write an approximate expression for the partition
275.43, 296.62, 317.80, 338.99, 360.18, 381.36, 402.55, 423.74, 444.92, 466.11, function of a protein molecule by including contributions from only two
487.30, 508.48. (a) Calculate the rotational partition function at 25 °C by states: the native and denatured forms of the polymer. Proceeding with this
direct summation. (b) Calculate the rotational contribution to the molar crude model gives insight into the contribution of denaturation to the heat
entropy over a range of temperature and plot the contribution as a function of capacity of a protein. According to this model, the total energy of a system of
temperature. N protein molecules is E = Nεe−ε/kT/(1 + e−ε/kT), where ε is the energy separation
between the denatured and native forms. (a) Show that the constant-volume
P13E.10 Calculate the standard molar entropy of N2(g) at 298 K from
molar heat capacity is
its rotational constant B  1.9987 cm 1 and its vibrational wavenumber
  2358 cm 1. The thermochemical value is 192.1 J K−1 mol−1. What does this ( / kT )2 e  /kT
CV ,m  f (T )R f (T ) 
suggest about the solid at T = 0? (1  e  /kT )2

P13E.11 J.G. Dojahn et al. (J. Phys. Chem. 100, 9649 (1996)) characterized the (b) Plot the variation of CV,m with temperature. (c) If the function CV,m(T) has
potential energy curves of the ground and electronic states of homonuclear maximum or minimum, calculate the temperature at which it occurs.

TOPIC 13F Derived functions


Discussion questions
D13F.1 Suggest a physical interpretation of the relation between pressure and D13F.3 How does a statistical analysis of the equilibrium constant account for
the partition function. the latter’s temperature dependence?
D13F.2 Suggest a physical interpretation of the relation between equilibrium
constant and the partition functions of the reactants and products in a reaction.
Exercises and problems 595

Additional exercises
E13F.5 An O3 molecule is angular, and its vibrational wavenumbers are hcD e = 1.5422 eV . The ground state of the I atoms is 2P3/2, implying fourfold
−1 −1 −1
1110 cm , 705 cm , and 1042 cm . The rotational constants of the molecule degeneracy.
are 3.553 cm−1, 0.4452 cm−1, and 0.3948 cm−1. Calculate the rotational and
E13F.8 Calculate the equilibrium constant at 298 K for the gas-phase isotopic
vibrational contributions to the molar Gibbs energy at 298 K.
exchange reaction 279 Br 81Br  79 Br 79 Br + 81 Br 81Br . The Br2 molecule has
E13F.6 Use the information in Exercise E13E.10 to calculate the electronic a non-degenerate ground state, with no other electronic states nearby. Base
contribution to the molar Gibbs energy of O2 at 400 K. the calculation on the wavenumber of the vibration of 79Br81Br, which is
323.33 cm−1.
E13F.7 Calculate the equilibrium constant of the reaction I2 (g )  2I(g ) at
1000 K from the following data for I2,   214.36 cm 1, B  0.0373 cm 1,

Problems
P13F.1 Use mathematical software and work in the high-temperature at 1000 K, and calculate the ratio of equilibrium constants K (B )/K , where
limit to calculate and plot the equilibrium constant for the reaction K(B ) is the equilibrium constant when a magnetic field B is present and
CD4 (g ) + HCl(g )  CHD3 (g ) + DCl(g ) as a function of temperature, in the removes the degeneracy of the four states of the 2P3/2 level. Data on the species
range 300 K to 1000 K. Use the following data (numbers in parentheses are are given in Exercise E13F.7. The electronic g-value of the atoms is 43 . Calculate
degeneracies): the field required to change the equilibrium constant by 1 per cent.

P13F.4 R. Viswanathan et al. (J. Phys. Chem. 100, 10784 (1996)) studied
Molecule  /cm
1

B/cm −1 
A/cm −1
thermodynamic properties of several boron–silicon gas-phase species
CHD3 2993 (1), 2142 (1), 1003 (3), 1291 (2), 1036 (2) 3.28 2.63 experimentally and theoretically. These species can occur in the high-
temperature chemical vapour deposition (CVD) of silicon-based
CD4 2109 (1), 1092 (2), 2259 (3), 996 (3) 2.63
semiconductors. Among the computations they reported was computation of
HCl 2991 (1) 10.59 the Gibbs energy of BSi(g) at several temperatures based on a 4   ground state
DCl 2145 (1) 5.445 with equilibrium internuclear distance of 190.5 pm, a fundamental vibrational
wavenumber of 772 cm−1, and a 2Π first excited level 8000 cm−1 above the
⦵ ⦵
P13F.2 The exchange of deuterium between acid and water is an important ground level. Calculate the value of Gm (2000 K) − Gm (0).
type of equilibrium, and you can examine it using spectroscopic data on the ‡
P13F.5 The molecule Cl2O2, which is believed to participate in the seasonal
molecules. Use mathematical software and work in the high-temperature
depletion of ozone over Antarctica, has been studied by several means.
limit, calculate the equilibrium constant for the exchange reaction
M. Birk et al. (J. Chem. Phys. 91, 6588 (1989)) report its rotational constants
H2O(g ) + DCl(g )  HDO(g ) + HCl(g ) at (a) 298 K and (b) 800 K. Use the
(B) as 13 109.4, 2409.8, and 2139.7 MHz. They also report that its rotational
following data:
spectrum indicates a molecule with a symmetry number of 2. J. Jacobs et al.
(J. Amer. Chem. Soc. 116, 1106 (1994)) report its vibrational wavenumbers as
Molecule  /cm 1 
A/cm −1 
B/cm −1 
C/cm −1
753, 542, 310, 127, 646, and 419 cm−1 (all non-degenerate). Calculate the value
H2O 3656.7, 1594.8, 3755.8 27.88 14.51 9.29 of Gm⦵ (200 K ) − Gm⦵ (0) of Cl2O2.

HDO 2726.7, 1402.2, 3707.5 23.38 9.102 6.417 P13F.6 J. Hutter et al. (J. Amer. Chem. Soc. 116, 750 (1994)) examined the
geometric and vibrational structure of several carbon molecules of formula
HCl 2991 10.59 Cn. Given that the ground state of C3, a molecule found in interstellar space
DCl 2145 5.449 and in flames, is an angular singlet-state species with moments of inertia
39.340mu Å2, 39.032mu Å2, and 0.3082mu Å2 (where 1 Å = 10−10 m) and with
P13F.3 Here you are invited to decide whether a magnetic field can influence vibrational wavenumbers 63.4, 1224.5, and 2040 cm−1, calculate the value of
the value of an equilibrium constant. Consider the equilibrium I2 (g )  2 I(g ) Gm⦵ (300.0 K ) − Gm⦵ (0) for C3.

FOCUS 13 Statistical thermodynamics


Integrated activities
I13.1 A formal way of arriving at the value of the symmetry number is to note I13.2 A feature of the rotational molar heat capacity of H2 is that it rises to
that σ is the order (the number of elements) of the rotational subgroup of the above the classical value of R before settling back to approach that value as the
molecule, the point group of the molecule with all but the identity and the temperature is increased from 0. To understand this behaviour, the heat capacity
rotations removed. The rotational subgroup of H2O is {E,C2}, so σ = 2. The can be treated as arising from the sum of transitions between the available
rotational subgroup of NH3 is {E,2C3}, so σ = 3. This recipe makes it easy to levels. (a) Show that the heat capacity of a linear rotor is related to the following
find the symmetry numbers for more complicated molecules. The rotational sum:
subgroup of CH4 is obtained from the T character table as {E,8C3,3C2}, so
1
q2 
σ = 12. For benzene, the rotational subgroup of D6h is {E ,2C6 ,2C3 ,C2 ,3C2′ ,3C2′′,}  ( )  { ( J )  ( J )}2 g ( J ) g ( J )e   { ( J ) ( J )}
so σ = 12. (a) Estimate the rotational partition function of ethene at 25 °C J ,J 

given that A  4.828 cm 1 , B  1.0012 cm 1, and C  0.8282 cm 1 . (b) Evaluate by


the rotational partition function of pyridine, C5H5N, at room temperature
C  12 Nk  2 ( )
( A  0.2014 cm 1 , B  0.1936 cm 1 , C  0.0987 cm 1 ).
596 13 Statistical thermodynamics

where the ε(J) are the rotational energy levels and g(J) their degeneracies. I13.3 Set up a calculation like that in Integrated activity I13.2 to analyse the
Then go on to show graphically that the total contribution to the heat capacity vibrational contribution to the heat capacity in terms of excitations between
of a linear rotor can be regarded as a sum of contributions due to transitions levels and illustrate your results graphically in terms of a diagram like that in
0 → 1, 0 → 2, 1 → 2, 1 → 3, etc. In this way, construct the figure below for the Fig. 13.1.
rotational heat capacities of a linear molecule. (b) Set up a calculation like that
in part (a) to analyse the vibrational contribution to the heat capacity in terms
of excitations between levels and illustrate your results graphically in terms of
a diagram like that in the figure below.

1.2
Molar heat capacity, CV,m/R

1
Total
0.8 0,1

0.6
0,2
0.4
1,2 1,3
0,3
0.2

0
0 1 2 3 4 5
Temperature, T/θ R
FOCUS 14
Molecular interactions

The electric properties of molecules give rise to molecular 14C Liquids


­interactions. In turn, those interactions govern the formation
of condensed phases and the structures and functions of mac- This Topic begins with the basic theory of molecular interac-
romolecules and molecular assemblies. tions in liquids, and then turns to a description of the prop-
Macromolecules are built from covalently linked components. erties of liquid surfaces. It is seen that important effects, such
They are everywhere, inside us and outside us. Some are natural: as ‘surface tension’, ‘capillary action’, the formation of ‘surface
they include polysaccharides (such as cellulose), polypeptides films’, and condensation, can be explained by thermodynamic
(such as protein enzymes), and polynucleotides (such as deoxy- arguments.
ribonucleic acid, DNA). Others are synthetic (such as nylon and 14C.1 Molecular interactions in liquids; 14C.2 The liquid–vapour
polystyrene). Molecules both large and small may also gather to- interface; 14C.3 Surface films; 14C.4 Condensation
gether in a process called ‘self-assembly’ and give rise to aggre-
gates that to some extent behave like macromolecules.

14D Macromolecules
14A The electric properties of
Macromolecules adopt shapes that are governed by molecular
molecules interactions. This Topic considers a range of shapes, but focuses
on the structureless ‘random coil’ and partially structured coils.
Important electric properties of molecules include ‘electric di- It also explores the connection between structure and the me-
pole moments’ and ‘polarizabilities’. These properties reflect the chanical and thermal properties of macromolecules.
degree to which the nuclei of atoms exert control over the elec- 14D.1 Average molar masses; 14D.2 The different levels of structure;
trons in a molecule, either by causing electrons to accumulate 14D.3 Random coils; 14D.4 Mechanical properties;
in particular regions, or by permitting them to respond more or 14D.5 Thermal properties
less strongly to the effects of external electric fields.
14A.1 Electric dipole moments; 14A.2 Polarizabilities;
14A.3 Polarization
14E Self-assembly
14B Interactions between molecules Atoms, small molecules, and macromolecules can form large
aggregates, sometimes by processes involving self-assembly.
This Topic describes the basic theory of several important This Topic explores ‘colloids’, ‘micelles’, and biological mem-
molecular interactions, with a special focus on ‘van der Waals branes, which are assemblies with some of the typical proper-
interactions’ between closed-shell molecules and ‘hydrogen ties of molecules but also with their own characteristic features.
bonding’. Many liquids and solids are bound together by one or It also introduces an important type of molecular interaction,
more of the cohesive interactions explored in this Topic. These the ‘hydrophobic interaction’, which is driven by changes in en-
interactions are also important for the structural organization tropy of the solvent.
of macromolecules. 14E.1 Colloids; 14E.2 Micelles and biological membranes
14B.1 The interactions of dipoles; 14B.2 Hydrogen bonding;
14B.3 The total interaction
What is an application of this material? in proteins and nucleic acids are explored in Impact 21, accessed
via the e-book. In Impact 22, also accessed via the e-book, atten-
Molecular interactions play important roles in biochemistry and tion shifts to the binding of a drug, a small molecule or protein,
biomedicine. Natural macromolecules differ in certain respects to a specific receptor site of a target molecule, such as a larger
from synthetic macromolecules, particularly in their composi- protein or nucleic acid. The chemical result of the formation of
tion and the resulting structure. The different levels of structure this assembly is the inhibition of the progress of disease.

➤ Go to the e-book for videos that feature the derivation and interpretation of equations, and applications of this material.
TOPIC 14A The electric properties
of molecules

Although the SI unit of dipole moment is coulomb metre


➤ Why do you need to know this material? (C m), it is still commonly reported in the non-SI unit debye, D,
The molecular interactions responsible for the formation named after Peter Debye, a pioneer in the study of dipole mo-
of condensed phases and large molecular assemblies arise ments of molecules:
from the electric properties of molecules, so you need to
know how the electronic structures of molecules account 1 D  3.335 64  1030 C m (14A.2)
for these interactions.
The magnitude of the dipole moment formed by a pair of
➤ What is the key idea? charges +e and −e separated by 100 pm is 1.6 × 10−29 C m, which
The nuclei of atoms exert control over the electrons in a corresponds to 4.8 D. The magnitudes of the dipole moments of
molecule and this control affects the distribution of the small molecules are typically about 1 D.
electrons and how it responds to external fields. A polar molecule is a molecule with a permanent electric
dipole moment which arises from the particular distribution
➤ What do you need to know already? of electron density in the molecule. More simply, a perma-
You need to be familiar with the expression for the poten- nent dipole moment can be thought of as arising from partial
tial energy of neighbouring charges (see Energy: A first charges on the atoms which result from differences in elec-
look). You also need to be familiar with molecular orbital tronegativity. Nonpolar molecules acquire an induced dipole
theory, especially the existence of the energy gap between moment in an electric field on account of the distortion the
a HOMO and LUMO (Topic 9E). One calculation draws on field causes in their electronic distributions and nuclear po-
the Boltzmann distribution (Topic 13A). sitions. However, this induced dipole moment is only tempo-
rary, and disappears as soon as the perturbing field is removed.
The dipole moment of a polar molecule is also temporarily
modified by an applied field. Some representative values of μ
The electric properties of molecules are responsible for many of are given in Table 14A.1.
the properties of bulk matter. The small imbalances of charge All heteronuclear diatomic molecules are polar. For a poly­
distributions in molecules and the ability of electron distribu- atomic molecule it is convenient to imagine that each bond
tions to be distorted allow molecules to interact with one an- between atoms of different elements, or between atoms of the
other and to respond to externally applied fields. same element that are inequivalent, has a dipole moment as-
sociated with it. The overall dipole moment of the molecule is
found by combining the dipole moments of all its bonds, taking
14A.1 Electric dipole moments into account both their direction and magnitude. Symmetry
plays an important part in this process, as is illustrated in Brief
An electric dipole consists of two electric charges +Q and −Q illustration 14A.1.
with a vector separation R. A point electric dipole is an electric
dipole in which R is very small compared with the distance to
the observer. The electric dipole moment is a vector μ (1) that
points from the negative charge to the positive charge and has a Table 14A.1 Dipole moments and polarizability volumes*
magnitude given by μ/D α′/(10−30 m3)
Magnitude of the electric dipole moment CCl4 0 10.5
  QR [definition]
 (14A.1)
H2 0 0.819

–Q μ +Q H 2O 1.85 1.48
HCl 1.08 2.63
R HI 0.42 5.45
1 Electric dipole * More values are given in the Resource section.
600 14 Molecular interactions

Brief illustration 14A.1 µ1 μres

Ozone is an angular molecule (2) in μ


Θ
which all the atoms are of the same
δδ+ δ+ μ2
element. However, the central O δ– δ–
atom is different from the outer two 4 Addition of dipole moments
(it is bonded to two atoms, whereas 2 Ozone, O3
the other O atoms are bonded only
When the two contributing dipole moments have the same
to one) so there are dipole moments magnitude (as in the dichlorobenzenes), this equation simpli-
δ– δ+ δ+ δ– fies to
associated with each bond. These
dipole moments make an angle to 1
1+ cos Θ = 2 cos2 2 Θ
each other and so do not cancel 3 Carbon dioxide, CO
2
one another completely: as a result, res {2 12 (1 + cosΘ )}1/2 = 2 1 cos 12 Θ (14A.3b)
ozone has a permanent dipole moment and is therefore
polar. The linear triatomic molecule CO2 is nonpolar because
although there are dipole moments associated with the two CO Brief illustration 14A.2
bonds these are of equal magnitude but point in opposite direc-
tions. They therefore cancel one another (3). Consider ortho (1,2-) and meta (1,3-) disubstituted benzenes,
for which Θortho = 60° and Θmeta = 120°. It follows from eqn
14A.3b that the ratio of the magnitudes of the electric dipole
The dipole moment of a polyatomic molecule can also be moments is:
resolved into contributions from various groups of atoms in
res ,ortho cos 12 ortho cos 21 (60) 31/2/2
the molecule and their relative locations (Fig. 14A.1). Thus,     31/ 2  1.7
1,4-­dichlorobenzene is nonpolar on account of the cancellation res ,meta cos 21 meta cos 12 (120) 1/2
of two equal but opposing C–Cl moments (exactly as in carbon
dioxide). 1,2-Dichlorobenzene, however, has a dipole moment
which is approximately the resultant of two chlorobenzene di- A more reliable approach to the calculation of dipole mo-
pole moments arranged at 60° to each other. This technique of ments is to take into account the locations and magnitudes of
‘vector addition’ can be applied with fair success to other series the partial charges on all the atoms. These partial charges are
of related molecules. included in the output of many molecular structure software
The magnitude of the resultant dipole moment μres of μ1 and packages. To calculate the x-component of the dipole moment,
μ2 that make an angle Θ to each other (4) is approximately (see for instance, it is necessary to know the partial charge on each
The chemist’s toolkit 8C.1) atom and the atom’s x-coordinate relative to a point in the mol-
ecule and form the sum
res  (12  22  2 1 2 cos )1/ 2  (14A.3a)
 x  QJ x J  (14A.4a)
J

Here QJ is the partial charge of atom J, xJ is the x-coordinate


of atom J, and the sum is over all the atoms in the molecule.
Cl Cl
Analogous expressions are used for the y- and z-components.
For an electrically neutral molecule, the origin of the coordi-
Cl nates is arbitrary, so it is best chosen to simplify the calcula-
(a) μobs = 1.57 D (b) μobs = 0, μcalc = 0 tions. In common with all vectors, the magnitude of μ is related
to the three components μx, μy, and μz by

Cl
Cl
Cl
  ( x2   y2  z2 )1/2  (14A.4b)

Cl

Example 14A.1 Calculating a molecular dipole moment


(c) μobs = 2.25 D, μcalc = 2.7 D (d) μobs = 1.48 D, μcalc = 1.6 D

Estimate the magnitude and orientation of the electric dipole


Figure 14A.1 The resultant dipole moments of the
moment of the planar amide group shown in (5) by using the
dichlorobenzene isomers can be obtained approximately by
vectorial addition of two chlorobenzene dipole moments (shown partial charges (as multiples of e) and the locations of the atoms
in (a), with μobs = 1.57 D). shown as (x, y, z) coordinates, with distances in picometres.
14A The electric properties of molecules 601

(182,–87,0) Monopole
+0.18 H
+0.45
(132,0,0) N C (0,0,0)
Dipole Octupole
–0.36
O (–62,107,0)
–0.38 Quadrupole
5

Collect your thoughts You need to use eqn 14A.4a to calculate


Quadrupole Octupole
each of the components of the dipole moment and then eqn
14A.4b to assemble the three components into the magnitude
of the dipole moment. Note that the partial charges are multi-
ples of the fundamental charge, e = 1.602 × 10−19 C. This group Figure 14A.2 Typical charge arrays corresponding to electric
is a fragment of a neutral molecule, so the choice of origin of multipoles. Shaded and unshaded spheres indicate charges with
the coordinates is not arbitrary. Here the origin is taken to be opposite signs, and the magnitude of the charge is proportional
to the size of the sphere.
coincident with the carbon atom.
The solution The expression for μx is
CO2 molecule, 3). As shown in
 x  (0.36e)  (132 pm)  (0.45e)  (0 pm)  (0.18e)  (182 pm) Fig. 14A.2 a quadrupole can arise δ+
 (0.38e)  (62.0 pm) from a linear arrangement of three δ–
 8.8e pm charges or from four charges in a δ+ δ– δ–
 8.8  (1.602  1019 C)  (1012 m)  1.4  1030 C m square, which can be thought of as δ+
two opposed dipoles. An octupole δ–
corresponding to μx = +0.42 D. The expression for μy is: (n = 4) consists of an array of point δ+
charges that sum to zero and which
 y  (0.36e)  (0 pm)  (0.45e)  (0 pm)  (0.18e)  (87 pm) 7 Methane, CH4
has neither a dipole moment nor a
 (0.38e)  (107 pm) quadrupole moment. Figure 14A.2 shows two possible arrange-
 56.3e pm ments that give rise to an octupole: in one two square arrange-
 9.02  1030 C m ments of charges (which give a quadrupole) are stacked on top
of one other; in the other there as a tetrahedral arrangement
It follows that μy = −2.7 D. The amide group is planar, so of charges arrangements around an equal and opposite charge.
μz = 0 and The CH4 molecule, 7, is an example of this latter arrangement.
  {(0.42 D)2  (2.7 D)2 }1/ 2  2.7 D
Exercises
The orientation of the dipole moment is found by arranging an
arrow of length 2.7 units to have x-, -y, and z-components of E14A.1 Which of the following molecules may be polar: CIF3, O3, H2O2?

+0.42, −2.7, and 0 units; the orientation is superimposed on (5). E14A.2 Calculate the resultant of two dipole moments of magnitude 1.5 D and
0.80 D that make an angle of 109.5° to each other.
Self-test 14A.1 Estimate the O
–0.38 (0,118,0) E14A.3 Calculate the magnitude and direction of the dipole moment of the
magnitude of the electric dipole
following arrangement of charges in the xy-plane: 3e at (0, 0), −e at (0.32 nm,
moment of methanal (formalde- +0.02 (0,0,0) 0), and −2e at an angle of 20° from the x-axis and a distance of 0.23 nm from
hyde) by using the information C the origin.
+0.18 +0.18
in (6). (–94,–61,0) H H (94,–61,0)
6
Answer: 3.2 D; experimental: 2.3 D

14A.2 Polarizabilities
Molecules may have higher multipoles, or arrays of point The failure of nuclear charges to control the surrounding elec-
charges (Fig. 14A.2). Specifically, an n-pole is an array of point trons totally means that those electrons can respond to external
charges with an n-pole moment but no lower moment. Thus, a fields. Therefore, an applied electric field can distort a molecule
monopole (n = 1) is a point charge, and the monopole moment as well as align its permanent electric dipole moment. The mag-
is what is normally called the overall charge. A dipole (n = 2), as nitude μ* of the induced dipole moment, μ*, is proportional to
already seen, is an array of charges that has no monopole mo- the electric field strength, E, so
ment (no net charge).
A quadrupole (n = 3) consists of an array of point charges Polarizability
   E [definition]
 (14A.5a)
that has neither a net charge nor a dipole moment (as for a
602 14 Molecular interactions

The constant of proportionality α is the polarizability of the How is that done? 14A.1 Correlating polarizability and
molecule. The greater the polarizability, the larger is the in-
duced dipole moment for a given applied field. In a formal molecular structure
treatment, vector quantities are used to allow for the possibility The argument starts from the quantum mechanical expression
that the induced dipole moment might not lie parallel to the for the molecular polarizability in the z-direction:1
applied field, in which case the scalar α is replaced by α, a 3 × 3 2
matrix.  z , 0n
  2
When the applied field is very strong (as in tightly focused n0 En(0)  E0(0)
laser beams), the magnitude of the induced dipole moment is
not strictly linear in the strength of the field, and where µ z ,0n = ∫ψ n µˆ zψ 0dτ is the z-component of the transition
electric dipole moment, a measure of the extent to which
electric charge is shifted when an electron migrates from the
Hyperpolarizability
    E  12  E 2   [definition]
 (14A.5b) ground state, with wavefunction ψ 0 , to create an excited state,
with wavefunction ψ n. The sum is over the excited states, with
energies En(0) .
The coefficient β is the (first) hyperpolarizability of the
­molecule. Step 1 Introduce approximations
Polarizability has the units (coulomb metre)2 per joule Now approximate the excitation energies by a mean value ΔE
(C2 m2 J−1). That collection of units is awkward, so α is often ex- (an indication of the HOMO–LUMO separation). Also sup-
pressed as a polarizability volume, α′, by using the relation pose that the most important transition dipole moment is
approximately equal to the charge of an electron multiplied by
 Polarizability volume the molecular radius R. Then
  [definition]
 (14A.6)
4  0
2e 2 R2

E
where ε0 is the electric constant (the vacuum permittivity).
The units of 4πε0 are coulomb-squared per joule per metre This expression shows that α increases with the size of the mol-
(C2 J−1 m−1); it follows that α′ has the dimensions of volume ecule and with the ease with which it can be excited electroni-
(hence its name). Polarizability volumes are similar in mag- cally. Therefore, the polarizability increases as the HOMO–
nitude to actual molecular volumes (of the order of 10−30 m3, LUMO separation decreases.
10−3 nm3, 1 Å3). Step 2 Express the excitation energy in terms of the atomic radius
If the excitation energy is approximated by the energy needed
to remove an electron to infinity from a distance R from a sin-
Brief illustration 14A.3 gle positive charge, then ∆E ≈ e2/4πε0R. Insertion of this value
into the expression for α gives
The polarizability volume of H2O is 1.48 × 10−30 m3. It follows
from eqns 14A.5a and 14A.6 that    4  0 E and the magni- 4  0 R
  2e 2 R2   2  4  0 R3
tude of the dipole moment of the molecule (in addition to the e2
permanent dipole moment) induced by an applied electric field To obtain the polarizability volume, divide α by 4πε0, and
of strength 1.0 × 105 V m−1 is ignore the factor of 2 in this approximation. The result is
α′ ≈ R3, which is of the same order of magnitude as the molecu-
μ* = 4π × (8.854 × 10−12 J−1 C2 m−1) × (1.48 × 10−30 m3) lar volume.
× (1.0 × 105 V m−1)
1 V= 1 J C −1
As just shown, polarizability volumes correlate with the
= 1.65 × 10−35 C m = 4.9 × 10−6 D = 4.9 μD HOMO–LUMO separations in atoms and molecules. The elec-
tron distribution can be distorted readily if the LUMO lies close
to the HOMO in energy, so the polarizability is then large. If
the LUMO lies high above the HOMO, an applied field can-
The polarizability volumes of some molecules are given not perturb the electron distribution significantly, and the
in Table 14A.1. It is possible to establish a correlation be-
tween these values and the electronic structure of atoms 1
For a derivation of this equation see our Physical chemistry: Quanta, mat-
and molecules. ter, and change (2014).
14A The electric properties of molecules 603

­ olarizability is low. Molecules with small HOMO–LUMO


p number of molecules with their dipole moments pointing into
gaps are typically large, and have numerous electrons. this patch is proportional to this area multiplied by the Boltzmann
For most molecules, the polarizability is ‘anisotropic’, which factor e−E(θ)/kT. The probability dp that a dipole moment has an
means that its value depends on the orientation of the molecule orientation in the range θ to θ + dθ and at any azimuthal angle φ
relative to the applied field. The polarizability volume of benzene around the direction of the applied field is therefore
when the field is applied perpendicular to the ring is 0.0067 nm3
number pointing at angle  e  E ( )/kT sin d
and it is 0.0123 nm3 when the field is applied in the plane of the dp   
total summed over all angles
e
 E ( ) /kT
ring. The anisotropy of the polarizability determines whether a sin d
0
molecule is rotationally Raman active (Topic 11B).
where 0 ≤ θ ≤ π. The applied electric field is in the z-direction,
so the mean dipole moment is in the same direction and its
Exercises value is
E14A.4 What strength of electric field is required to induce an electric dipole
  z     z dp
moment of magnitude 1.0 μD in a molecule of polarizability volume
2.6 × 10−30 m3?
where µ z is the z-component of a dipole with orientation θ and
E14A.5 The polarizability can be approximated as   2e R /E , where R is
2 2
dp is the probability of that orientation occurring.
the molecular radius and ΔE is the HOMO–LUMO separation. Estimate the
polarizability volume for benzene, for which HOMO–LUMO separation is Step 1 Write an expression for the energy of a dipole in an electric
5.0 eV. Take R as the distance between carbon atoms on opposite sides of the field
ring; the C–C bond length is 139 pm.
The energy E(θ) of a dipole depends on the angle θ it makes
with the electric field E as

E( )   E cos
14A.3 Polarization
Step 2 Set up an expression for the average of the z-component of
The polarization, P, of a bulk sample is the electric dipole mo- the dipole moment
ment density, which is the mean electric dipole moment of the The z-component of a dipole with orientation θ with respect to
molecules, 〈μ〉, multiplied by the number density, N = N /V : the z-axis is  z   cos . Therefore the mean dipole moment is
π
P    N
Polarization
[definition]
 (14A.7)
〈 μ z 〉 = ∫ μ z d p =∫ μ cosθ d p = μ
∫ 0
cosθ e − E (θ )/kT sinθ dθ
π
∫e
− E (θ )/kT
sinθ dθ
A dielectric is a polarizable, non-conducting medium. 0

E(θ) = − μE cos θ

14A.3(a) The mean dipole moment π


∫e
μ E cosθ /kT
cosθ sinθ dθ
=μ 0
The polarization of a fluid dielectric is zero in the absence of π
∫e
μ E cosθ /kT
sinθ dθ
an applied field because the molecules adopt ceaselessly chang- 0

ing random orientations due to thermal motion, so 〈μ〉 = 0. In Step 3 Evaluate the integrals
the presence of a weak electric field, the energy depends on the To simplify the appearance of this expression, write x  E/kT
orientation of the dipole with respect to the field, with lower and obtain
energy orientations being more populated; as a result, the mean

dipole moment is no longer zero. The Boltzmann distribution   e x cos  cos sin d
can be used to find an expression for 〈μ〉.  z   0


0
e x cos  sin d

Then write y = cos θ and dy = −sin θ dθ, and change the limits
How is that done? 14A.2 Deriving an expression for the
of integration to y = −1 (at θ = π) and y = 1 (at θ = 0):
mean dipole moment
Integral E.2
   ex  e x ex  e x 
Imagine a collection of molecules, with dipole moment µ , in an 1
  
  ye xy dy x x2 
electric field applied along the z-direction. The energy of inter-   z   11 
action E(θ ) between the dipole moment and the electric field  ex  e x 

 
1
e xy dy
  
depends on the angle θ between them, but does not depends  x 
Integral E.1
on the azimuthal angle φ .
In spherical polar coordinates, the area of an infinitesimal  ex  e x 1 
   x x  
patch of surface (at constant radius) is r 2 sin d d . The ­ e e x
604 14 Molecular interactions

That is, noting again that x  E/kT , The next contribution to the polarization to be lost as the
frequency increases is the distortion polarization, the polari-
ex + e− x 1 (14A.8a)
〈 μ z 〉 = μ L( x ) L( x ) = − zation that arises from the distortion of the positions of the
ex − e− x x Mean electric
nuclei by the applied field. The molecule is bent and stretched
dipole moment
by the applied field, and the molecular dipole moment changes
accordingly. The time taken for a molecule to bend is approxi-
The function L(x) is called the Langevin function (Fig. 14A.3). mately the inverse of the molecular vibrational frequency, so
Under most circumstances, x is very small. For example, if the distortion polarization disappears when the frequency of
μ = 1 D and T = 300 K, then x exceeds 0.01 only if the field the radiation is increased through the infrared.
strength exceeds 100 kV cm−1, and most measurements are done Polarization disappears in stages: each successive stage oc-
at much lower strengths. When the field is so weak that x << 1 the curs as the frequency of oscillation of the electric field rises
Langevin function is well approximated by expanding it using above the frequency of a particular mode of vibration. At even
a Taylor series about x = 0 and taking the leading term to give higher frequencies, in the visible region, only the electrons are
L(x ) = 13 x . Therefore, the average molecular dipole moment is mobile enough to respond to the rapidly changing direction
of the applied field. The polarization that remains is now due
 2E Mean value of the dipole moment entirely to the distortion of the electron distribution, and the
 z    (14A.8b)
3kT [weak electric field]
surviving contribution to the molecular polarizability is called
As the electric field strength is increased to very high values, the the electronic polarizability. This behaviour can be explored
orientations of molecular dipole moments fluctuate less about by noting that the quantum mechanical expression for the po-
the field direction and the mean dipole moment approaches its larizability of a molecule in the presence of an electric field that
maximum value of 〈μz〉 = μ. is oscillating at a frequency ω in the z-direction is2
2
2 n0  z ,0n
14A.3(b)The frequency dependence  ( )   2
 n  0 n0   2
of the polarization
where the excitation frequency is defined by n0  En(0)  E0(0) .
When the applied field changes direction slowly, the orienta-
Two conclusions can be made:
tion of the permanent dipole moment has time to change—the
whole molecule rotates into a new orientation—and follows • As ω → 0, the equation reduces to the expression for the
the field. However, when the electric field changes direction static polarizability
rapidly, a molecule cannot change orientation fast enough to • As ω becomes very high (and much higher than any exci-
follow and the permanent dipole moment then makes no con- tation frequency of the molecule, so that the ωn02
in the
tribution to the polarization of the sample. The orientation denominator can be ignored), the polarizability becomes
­polarization, the polarization arising from the permanent di-
pole moments, is lost at such high frequencies. Because a mol- 2 2
2  n0
 ( )    z ,0n  0 as   
ecule takes about 1 ps to turn through about 1 radian in a fluid,  n
the loss of the contribution of orientation polarization to the
total polarization occurs when measurements are made at fre- The conclusion applies to each type of excitation, vibrational as
quencies greater than about 1011 Hz (in the microwave region). well as electronic, and accounts for the successive decreases in
polarizability as the frequency is increased.
1
Weak-field
approximation 14A.3(c) Molar polarization
0.8
When two charges Q1 and Q2 are separated by a distance r in a
0.6 medium, the Coulomb potential energy of their interaction is
L(x)

0.4 Q1Q2
V  (14A.9)
4 r
0.2
where ε is the permittivity of the medium, which is reported by
0
0 0.5 1 1.5 2 2.5 3
introducing the relative permittivity εr and writing ε = εrε0. The
x relative permittivity of a substance is measured by comparing
Figure 14A.3 The Langevin function used in the calculation of
the mean electric dipole moment. When x is small the weak-field 2
For a derivation of this equation, see our Physical chemistry: Quanta, mat-
approximation is appropriate. ter, and change (2014).
14A The electric properties of molecules 605

the capacitance of a capacitor with and without the substance θ/°C ρ/(g cm−3) εr
present between the plates of the capacitor (C and C0, respec- 0 0.99 12.5
tively) and using εr = C/C0. The relative permittivity can have 20 0.99 11.4
a very significant effect on the strength of the interactions be-
40 0.99 10.8
tween ions in solution. For instance, water has a relative per-
60 0.99 10.0
mittivity of 78 at 25 °C, so the interionic Coulombic interaction
energy is reduced by nearly two orders of magnitude from its 80 0.99 9.50

vacuum value. 100 0.99 8.90


The relative permittivity of a substance is large if its mole- 120 0.97 8.10
cules are polar or highly polarizable. The quantitative relation 140 0.96 7.60
between the relative permittivity and the electric properties of 160 0.95 7.11
the molecules is obtained by considering the polarization of a 200 0.91 6.21
medium, and is expressed by the Debye equation:

 r  1  Pm
 Debye equation (14A.10) Collect your thoughts The relative permittivity depends on the
r  2 M molar polarization Pm (eqn 14A.10), which in turn depends on
the temperature, polarizability, and the magnitude of the per-
where ρ is the mass density of the sample, M is the molar mass
manent dipole moment (eqn 14A.11). These relations suggest
of the molecules, and Pm is the molar polarization, which is de-
that you should
fined as
• Calculate (εr − 1)/(εr + 2) at each temperature, and then
N  2  Molar polarization multiply by M/ρ to form Pm from eqn 14A.10.
Pm  A      (14A.11)
3 0  3kT  [definition]
• Plot Pm against 1/T.

where α is the polarizability. The term μ2/3kT stems from the This plot will be a straight line because eqn 14A.11 rearranges to
thermal averaging of the electric dipole moment in the pres- 
slope
intercept

ence of the applied field (eqn 14A.8b). N A NA2 1
Pm   
If the permanent electric dipole moment is zero eqn 14A.11 3 0 9 0k T
becomes Pm  N A /3 0. This expression for Pm is used in eqn
The slope of the graph is NAμ2/9ε0k and its intercept at 1/T = 0
14A.10 to give the Clausius–Mossotti equation:
is NAα/3ε0.
 r  1  N A The solution Use the data to draw up the following table, with
 Clausius – Mossotti equation (14A.12)
 r  2 3 M 0 M = 152.23 g mol−1 for camphor.

The Clausius–Mossotti equation is used when there is no


contribution from permanent electric dipole moments to the θ/°C (103 K)/T εr (εr − 1)/(εr + 2) Pm/(cm3 mol−1)
polarization, either because the molecules are nonpolar or be- 0 3.66 12.5 0.793 122
cause the frequency of the applied field is so high that the mol- 20 3.41 11.4 0.776 119
ecules cannot orientate quickly enough to follow the change in 40 3.19 10.8 0.766 118
direction of the field. 60 3.00 10.0 0.750 115
80 2.83 9.50 0.739 114
100 2.68 8.90 0.725 111
Example 14A.2 Determining the dipole moment and
120 2.54 8.10 0.703 110
polarizability
140 2.42 7.60 0.688 109
The relative permittivity of cam- 160 2.31 7.11 0.671 108
phor (8) was measured at a series of 200 2.11 6.21 0.635 106
temperatures with the results given
below. Determine the dipole moment O
and the polarizability volume of the The points are plotted in Fig. 14A.4. The intercept on the vertical
molecule. axis lies at Pm/(cm3 mol−1) = 83.5, so NAα/3ε0 = 83.5 cm3 mol−1 =
8 Camphor 8.35 × 10−5 m3 mol−1. It then follows that
606 14 Molecular interactions

122 until 175 °C, is rotating even in the solid. It is an approximately


spherical molecule.
118 Self-test 14A.2 The relative permittivity of chlorobenzene is
5.71 at 20 °C and 5.62 at 25 °C. Assuming a constant density
Pm/(cm3 mol–1)

(1.11 g cm−3), estimate its polarizability volume and the magni-


114
tude of its dipole moment.
Answer: 1.4 × 10−29 m3, 1.2 D
110

106
2 3 4 The refractive index, nr, of the medium is the ratio of the speed
(103 K)/T of light in a vacuum, c, to its speed c′ in the medium: nr = c/c′.
Figure 14A.4 The plot of Pm/(cm3 mol−1) against (103 K)/T used in According to Maxwell’s theory of electromagnetic radiation, the
Example 14A.2 for the determination of the polarizability and the refractive index at a specified (visible or ultraviolet) wavelength
magnitude of the dipole moment of camphor. is related to the relative permittivity at that frequency by

Relation between refractive


0 nr   r1/2 index and relative permittivvity
 (14A.13)
   intercept
3  (8.854  10 J C m 1 ) 
12 1 2 
  (8 . 35  10 5
m 3
mol 1 ) A beam of light changes direction (‘bends’) when it passes from
6.022  1023 mol 1
   a region of one refractive index to a region with a different re-
NA
fractive index. Therefore, the molar polarization, Pm, and the
39
 3.68  10 C 2 m2 J 1 molecular polarizability, α, can be measured at frequencies
From eqn 14A.6, it follows that α′ = 3.31 × 10−29 m3. The slope typical of visible light (about 1014–1015 Hz) by measuring the
is 10.55, so NAμ2/9ε0k = 1.055 × 104 cm3 mol−1 K = 1.055 × refractive index of the sample and using the Clausius–Mossotti
10−2 m3 mol−1 K, so it follows that equation.
1/ 2
  0
    k  
 9  (8.854  1012 J 1 C 2 m 1 )  (1.381  1023 JK 1 )  Exercises
 
 23 1  E14A.6 The dipole moment of chlorobenzene is 1.57 D and its polarizability
 6 . 022  10 mol 
   volume is 1.23 × 10−23 cm3. Estimate its relative permittivity at 25 °C, when its
 NA  mass density is 1.173 g cm−3.
slope
  E14A.7 At 0 °C, the molar polarization of liquid chlorine trifluoride is
 (1.055  102 m3 mol 1 K )1/ 2 27.18 cm3 mol−1 and its mass density is 1.89 g cm−3. Calculate the relative
permittivity of the liquid.
 4.39  1030 C m  1.32 D
E14A.8 The refractive index of CH2I2 is 1.732 for 656 nm light. Its mass density
Because the Debye equation describes molecules that are free at 20 °C is 3.32 g cm−3. Calculate the polarizability of the molecule at this
to rotate, the data show that camphor, which does not melt wavelength.

Checklist of concepts
☐ 1. An electric dipole consists of two electric charges +Q ☐ 6. Polarizabilities (and polarizability volumes) correlate
and −Q separated by a vector R. with the HOMO–LUMO separations in molecules.
☐ 2. The electric dipole moment μ is a vector that points ☐ 7. The polarization of a medium is the electric dipole
from the negative charge to the positive charge of a moment density.
dipole; its magnitude is μ = QR. ☐ 8. Orientation polarization is the polarization arising
☐ 3. A polar molecule is a molecule with a permanent elec- from the permanent dipole moments.
tric dipole moment. ☐ 9. Distortion polarization is the polarization arising from
☐ 4. Molecules may have higher electric multipoles: an the distortion of the positions of the nuclei by the
n-pole is an array of point charges with an n-pole applied field.
moment but no lower moment. ☐ 10. Electronic polarizability is the polarizability due to the
☐ 5. The polarizability is a measure of the ability of an elec- distortion of the electron distribution.
tric field to induce a dipole moment in a molecule.
14A The electric properties of molecules 607

Checklist of equations
Property Equation Comment Equation number
Magnitude of the electric dipole moment μ = QR Definition 14A.1

Magnitude of the resultant of two dipole moments res  (12  22  2 12 cos  )1/ 2 14A.3a
Magnitude of the induced dipole moment    E Linear approximation; α is the polarizability 14A.5a

    E  12  E 2 Quadratic approximation; β is the 14A.5b


hyperpolarizability
Polarizability volume α′ = α/4πε0 Definition 14A.6
Polarization P    N Definition 14A.7
Debye equation (εr − 1)/(εr + 2) = ρPm/M 14A.10
2
Molar polarization Pm = (NA/3ε0)(α + μ /3kT) 14A.11
Clausius–Mossotti equation (εr − 1)/(εr + 2) = ρNAα/3Mε0 14A.12
TOPIC 14B Interactions between
­molecules

How is that done? 14B.1 Deriving the expression for the


➤ Why do you need to know this material?
interaction between a point charge and a point dipole
Many types of molecular interactions are responsible for
the formation of condensed phases and large molecular You need to consider the interaction between the two charges
assemblies. ±Q1 of a point dipole, which result in a dipole moment of
magnitude μ1 = Q1l, and the point charge Q2 as shown in (1).
➤ What is the key idea? Suppose that the arrangement is in a vacuum, so use ε = ε0.
Attractive interactions result in cohesion but repulsive r
μ1
interactions prevent the complete collapse of matter to
nuclear densities. +Q1 –Q1 Q2
l

➤ What do you need to know already? 1


You need to be familiar with elementary aspects of elec- Step 1 Write an expression for the potential energy due to interac-
trostatics, specifically the Coulomb interaction (see Energy: tion with both charges
A first look), and the relationships between the structure
The sum of the potential energies due to repulsion between like
and electric properties of a molecule, specifically its dipole
charges and attraction between opposite charges is
moment and polarizability (Topic 14A).
x = l/2r

1 Q1Q2 Q1Q2 QQ 1 1
V= − + = 1 2 − +
A van der Waals interaction is an attractive (energy-lowering) 4π 0 r − 12 l r + 12 l 4 π 0r 1− x 1+ x
interaction between closed-shell molecules that depends on
Step 2 Treat the dipole as a point dipole
the separation of the molecules as the inverse sixth power (V ∝
1/r6). This precise criterion is often relaxed to include all non- For a point dipole l << r, therefore x << 1. This expression can
bonding interactions. They occur in various guises and can all be simplified by expanding the terms 1/(1 ± x ) in x by using
be traced to the interaction of partial charges. The underlying 1 1
interaction throughout this discussion is the Coulomb poten-  1  x  x2   1  x  x2  
1 x 1 x
tial energy, V = Q1Q2/4πεr, discussed in Energy: A first look.
and then retaining only the first two terms:
x = l/2r

14B.1 The interactions of dipoles V=


Q1Q2
4 π 0r
−(1 + x) + (1 – x) = −
2xQ1Q2
4 π 0r
QQ l
=− 1 2 2
4 π 0r
A point dipole is a dipole in which the separation between the With μ1 = Q1l this expression becomes
charges l is much smaller than the distance r from which the
dipole is being observed (l << r). μ1Q2 (14B.1)
V=−
4 π 0r 2 Point dipole–point charge interaction
[as in (1)]
14B.1(a) Charge–dipole interactions
The Coulomb potential energy of one charge near another can With μ in coulomb metres, Q2 in coulombs, and r in metres, V
be adapted to find the potential energy of a point charge and a is obtained in joules. In the orientation shown in (1), V is nega-
dipole, and extended to the interaction between two dipoles. tive, representing a net attraction. The expression should be
14B Interactions between ­molecules 609

r
μ1 μ2

+Q1 l –Q1 +Q2 l –Q2

How is that done? 14B.2 Deriving the expression for the


interaction energy of two point dipoles
Figure 14B.1 The energy of interaction between a point charge
and a dipole diminishes with the distance between the charge To calculate the potential energy of interaction of two point
and the dipole as a result of two effects. First, the interaction with dipoles separated by r in a vacuum in the arrangement shown
the two charges that form the dipole diminishes as they move in (2) proceed in exactly the same way as before. In this case the
away (shown here by a their decreasing size). Second, the two total interaction energy is the sum of four pairwise terms. Two
charges appear to merge. The combination of these two effects are attractions between opposite charges, which contribute
results in the energy of interaction approaching zero more rapidly negative terms to the potential energy, and two are repulsions
than as a result of the distance effect alone. between like charges, which contribute positive terms.
Step 1 Write an expression for the potential energy due to interac-
tion of the charges on the two dipoles
multiplied by cos Θ when the point charge lies at an angle Θ to The sum of the four contributions is
the axis of the dipole and ε0 replaced by ε if the medium is not a
vacuum. For future reference, the electric field, E, experienced 1 Q1Q2 QQ QQ QQ
V= − + 1 2 + 1 2 − 1 2
by the charge and arising from the dipole is proportional to 4π 0 r+l r r r−l
dV/dr, and therefore to 1/r3.
x = l/r
As the separation r between the charge and the dipole in-
creases the magnitude of the potential energy decreases as 1/r2. Q1Q2 1 1
=− −2 +
This interaction decreases more rapidly than that between two 4 π 0r 1 + x 1−x
point charges, which goes as 1/r. The reason for this more rapid
decrease is that, from the viewpoint of the point charge, the Step 2 Treat the dipoles as point dipoles
partial charges of the dipole seem to merge and cancel as the As before, provided l << r the two terms in x may be expanded,
distance r increases (Fig. 14B.1). leading to
1/(1 + x) 1/(1 − x)
QQ
V = − 1 2 (1− x + x 2 + − 2 +1 + x + x 2 + )
Brief illustration 14B.1 4 π 0r

Consider a Li+ ion and a water molecule (μ = 1.85 D) separated The terms in boxes sum to zero, so the only surviving term is
by 1.0 nm in a vacuum, with the point charge on the ion and 2x2. It follows that
the dipole of the molecule arranged as in (1). The energy of
interaction is given by eqn 14B.1 as x = l/r
2
QLi H2O
2 x Q1Q2 2l 2Q1Q2
   V=− =−
4 π 0r 4 π 0r 3
(1.602  1019 C)  (1.85  3.336  1030 C m)
V 
4   (8.854  1012 J 1 C 2 m 1 )  (1.0  109 m)2 It follows that because μ1 = Q1l and μ2 = Q2l, the potential
   
0 r energy of interaction in the alignment shown in (2) is
 8.9  1021 J
μ1 μ2 (14B.2)
V=−
This energy corresponds to −5.4 kJ mol . −1
2π 0r 3 Point dipole–point dipole interaction
[as in (2)]

This interaction energy approaches zero more rapidly (as 1/r3)


than for the previous case: now both interacting entities appear
14B.1(b) Dipole–dipole interactions neutral to each other at large separations.
The preceding discussion can be extended to the interaction of Equation 14B.2 applies only to the arrangement in (2).
two dipoles arranged as in (2). More generally, as in the arrangement in (3), the potential
610 14 Molecular interactions

energy of interaction between two point dipoles separated by If one dipole is fixed and the second free to rotate, with
a vector r is the geometry shown in (5), the energy of interaction is found
from eqn 14B.3a by using 1  2  1 2 cos , 1 r = 1r, and
1 3( μ1. r )(r . μ 2 ) (14B.3a)
V= μ .μ − r  2  r 2 cos to give
4 π 0r 3 1 2 r2 Point dipole–point
dipole interaction 1 
[as in (3)] V (1 2 cos  31 2 cos )   1 2 3 cos
4  0 r 3
2 0 r

μ2 l
μ2
+Q2 μ1 μ2 θ
–Q2
r
r r ϕ
Θ 5
μ1
μ1
3 +Q1 l –Q1 The average energy is found by integrating this expression
4 over all possible orientations of the second dipole, which are
specified by the angles θ and φ . The necessary integral is taken
over the surface area element in spherical polar coordinates,
(For the origin of this expression, see A deeper look 14B.1, avail-
sin d d (shown in boxed)
able to read in the e-book accompanying this text.) When the
two dipoles are parallel and arranged as in (4), the potential en- 1 2  2π π

ergy is simply 〈V〉 = −


2 π 0r 3
〈cosθ 〉 = − 1 2 3
2 π 0r ∫ ∫
0 0
cosθ sinθ dθ d
2π 0
1 2 f ( )
V f ( )  1  3 cos 2   2π π
4  0 r 3 = − 1 2 3 ∫ d 12 ∫ sin2θ dθ = 0
2 π 0r 0 0
Point dipole – point dipole interaction
[as in ( 4 )]  (14B.3b) sin θ cos θ = 21 sin2 θ

This conclusion is applicable at any relative location of the two


Brief illustration 14B.2
dipoles.
The average interaction energy of two freely rotating dipoles
Equation 14B.3b can be used to calculate the potential energy of is zero. However, because their mutual potential energy de-
the dipolar interaction between two amide groups. Supposing that pends on their relative orientation, the molecules do not in fact
the groups are separated by 3.0 nm with Θ = 180° (so cos Θ = −1 rotate completely freely, even in a gas. The lower energy orien-
and 1 − 3 cos2 Θ = −2). Take μ1 = μ2 = 2.7 D, corresponding to tations are marginally favoured, so there is a non-zero average
9.0 × 10−30 C m, and find interaction between dipoles. The detailed calculation of the in-
teraction energy between two dipoles is quite complicated, but
 1 2
 1
3 cos2 
the form of the final answer can be constructed quite simply.
30 2
(9.0  10 C m)  (2)
V
4   (8.854  1012 J 1 C 2 m 1 )  (3.0  109 m)3
   
0 r3 How is that done? 14B.3 Deriving the expression for the
2 2
30 2
(9.0  10 )  (2) C m energy of interaction between rotating dipoles

4   (8.854  1012 )  (3.0  109 )3 J 1 C 2 m 1 m3
You can use the simplified model of the interaction for the
 5.4  1023 J arrangement shown in (5), with the second dipole free to
This value corresponds to −33 J mol−1. rotate, but not sampling all orientations equally.
Step 1 Write an expression for the average interaction energy
The average interaction energy of two polar molecules rotating
Equation 14B.3b applies to polar molecules in a fixed, paral- at a fixed separation r is given by
lel orientation in a solid. In a fluid of freely rotating molecules,
12
the interaction between dipoles averages to zero because like V     cos 
2 0r 3
partial charges of two freely rotating molecules are close to-
gether as much as the two opposite partial charges, and the re- where 〈cos θ〉 now includes a weighting factor in the averaging
pulsion of the former is cancelled by the attraction of the latter. that recognizes that not all orientations are equally probable.
14B Interactions between ­molecules 611

z θ If the energy of interaction of the dipoles is much less than kT


sin θ dθ it follows that a << 1. As in Topic 14A in this limit L(a) = a/3
and so the average energy is

dϕ a12 12 22


V     
6 0r 3 122 02kTr 6
y
In a more realistic calculation, where the second dipole is
allowed to roam around the first (at the same distance), a
x
further factor of 12 is introduced, and the final outcome is the
Keesom interaction:
Figure 14B.2 The surface of a sphere showing the area
element sin θ dθ dϕ. C 2 μ12 μ22 (14B.4)
〈V 〉 = − C=
r6 3(4 π 0 )2 kT Keesom interaction

Step 2 Write an expression for the probability of finding a


­particular orientation The important features of eqn 14B.4 are:
The probability that dipole 2 has an orientation which places • The negative sign shows that the average interaction is
it in the area element sin d d on the surface of a sphere attractive.
(Fig. 14B.2) is
• The dependence of the average interaction energy on the
V ( ) /kT inverse sixth power of the separation identifies it as a van
e sin d d 12
dp  2 
V ( )   cos der Waals interaction.
2 0r 3
 e
V ( ) /kT
sin d d
0 0 • The inverse dependence on the temperature reflects how
the greater thermal motion overcomes the mutual orien-
where e−V(θ)/kT is the Boltzmann factor. The (weighted) average tating effects of the dipoles at higher temperatures.
value of cos θ is
• The inverse sixth power arises from the inverse third
2  power of the interaction potential energy weighted by the
 e
V ( ) /kT
cos sin d d
 cos    cos dp  0 0 energy in the Boltzmann term, which is also proportional
2 
 e
V ( ) /kT
sin d d to the inverse third power of the separation.
0 0

Step 3 Evaluate the integrals


Brief illustration 14B.3
With a = μ1μ2/2πkTε0r3 the denominator (after integration over
φ , which gives a factor of 2π, and writing x = cos θ) gives Suppose a water molecule (μ1 = 1.85 D) is at a distance 1.0 nm
 d cos  from an amide group (μ2 = 2.7 D) and that the water molecule


   1 2 can rotate freely. The average energy of their interaction at
2 e a cos 
sin d  2 eax dx  (ea  e  a )
0 1 a 25 °C (298 K) is

Likewise, the numerator is 1 2


    
2  (1.85  3.336  10 C m)  (2.7  3.336  1030 C m)2
30 2
Integral E.2
  V 
3  (1.710  1043 J 1 C 4 m 2 K 1 )  (298 K )  (1.0  109 m)6
 1         
2 e a cos 
cos sin d  2  xe dx ax
( 4  0 )2 k T r
0 1
23
2 a  a 2 a  a  4.0  10 J
 (e  e )  (e  e )
a2 a This interaction energy corresponds (after multiplication by
Avogadro’s constant) to −24 J mol−1 and is much smaller than
It follows that
the energies involved in the making and breaking of chemical
1 ea  e  a bonds.
 cos     a  a  L(a)
a e e

where L(a) is the Langevin function introduced in Topic 14A. Table 14B.1 summarizes the various expressions for the in-
Therefore, teraction of charges and dipoles. It is strightforward to extend
the formulas given there to obtain expressions for the energy
12 of interaction of higher multipoles (electric multipoles are
V    L(a)
2 0r 3 described in Topic 14A). The feature to remember is that the
612 14 Molecular interactions

Table 14B.1 Interaction potential energies

Distance Typical
Interaction
dependence of energy/ Comment
type
potential energy (kJ mol−1)
(a) (b)
Ion–ion 1/r 250 Only between ions
Hydrogen bond 20 Occurs in X − H  Y, Figure 14B.3 (a) A polar molecule (white arrow) can induce a
where X, Y = N, O, or F dipole (dark arrow) in a nonpolar molecule, and (b) the orientation
Ion–dipole 1/r2 15 See Note of the latter follows that of the former, so the interaction does not
Dipole–dipole 1/r 3
2 Between stationary polar average to zero.
molecules
1/r6 0.3 Between rotating polar
molecules
the interaction is attractive regardless of the orientation of the
London 1/r6 2 Between all types of
(dispersion) molecules and ions
two molecules. Moreover, like the dipole–dipole interaction,
the potential energy depends on 1/r6: this distance dependence
Note: The strengths of the interactions between actual ions and solvent molecules are
stems from the 1/r3 dependence of the distorting electric field
more complicated than this point-charge−point-dipole model, and are often much larger.
of molecule 1 (and hence the magnitude of the dipole induced
in molecule 2) and the 1/r3 dependence of the potential energy
i­nteraction energy approaches zero more rapidly the higher of interaction between the permanent and induced dipoles.
the order of the multipole. For the interaction of a stationary
n-pole with a stationary m-pole, the potential energy varies
with distance as Brief illustration 14B.4

1 Energy of interaction between For a molecule with μ = 1.0 D (3.3 × 10−30 C m, such as HCl)
V  (14B.5)
r n  m 1 stationary multipoles separated by 0.30 nm from a molecule of polarizability volume
α′ = 10 × 10−30 m3 (such as benzene, Table 14A.1), the average
The reason for the even steeper decrease with distance is the interaction energy is
same as before: the array of charges appears to blend together
into neutrality more rapidly with distance the higher the num- 12 2 (3.3  1030 C m)2  (10  1030 m3 )
V  
ber of individual charges that contribute to the multipole. Note 4  0r 6
4   (8.854  1012 J 1 C 2 m 1 )  (3.0  1010 m)6
that a given molecule may have a charge distribution that cor-
responds to a combination of several different multipoles, and  1.4  1021 J
in such cases the energy of interaction is the sum of terms, each which, upon multiplication by Avogadro’s constant, corre-
with a distance dependence given by eqn 14B.5. sponds to −0.83 kJ mol−1.

14B.1(c) Dipole–induced dipole interactions


Induced dipole–induced dipole
14B.1(d)
A polar molecule can induce a dipole in a neighbouring polar-
izable molecule (Fig. 14B.3). The induced dipole interacts with
interactions
the permanent dipole of the first molecule, and the two are at- Nonpolar molecules (including closed-shell atoms, such as Ar)
tracted together. The average interaction energy when the sepa- attract one another even though neither has a permanent di-
ration of the centres of the molecules is r is pole moment. The abundant evidence for the existence of in-
teractions between nonpolar molecules is their ability to exist
C 12 2 Potential energy of a as condensed phases, such as liquid hydrogen or argon, and the
V  C polar molecule and a  (14B.6)
r6 4  0 pollarizable molecule fact that benzene is a liquid at normal temperatures.
The interaction between nonpolar molecules arises from the
where  2 is the polarizability volume (Topic 14A) of molecule transient dipoles which all molecules possess as a result of fluc-
2 and μ1 is the magnitude of the permanent dipole moment of tuations in the instantaneous positions of electrons. To appreci-
molecule 1. Note that the C in this expression is different from ate the origin of the interaction, suppose that the electrons in
the C in eqn 14B.4 and other expressions below: the use of the one molecule flicker into an arrangement that gives the mol-
same symbol in C/r6 emphasizes the similarity of form of each ecule an instantaneous dipole moment µ1. This dipole gener-
expression. ates an electric field which polarizes the other molecule, and
The dipole–induced dipole interaction energy is independ- induces in that molecule an instantaneous dipole moment µ2 .
ent of the temperature because thermal motion has no effect on The two dipoles attract each other and the potential energy of
the averaging process: the induced dipole is always such that the pair is lowered.
14B Interactions between ­molecules 613

Exercises
E14B.1 Calculate the molar energy required to reverse the direction of an H2O
molecule located 100 pm from a Li+ ion. Take the magnitude of the dipole
(a) (b) moment of water as 1.85 D.
E14B.2 Use eqn 14B.3b to calculate the molar potential energy of the dipolar
Figure 14B.4 (a) In the dispersion interaction, an instantaneous
interaction between two amide groups in a vacuum and separated by 3.0 nm
dipole on one molecule induces a dipole on another molecule,
with θ = 45°. Take µ1 = µ2 = 2.7 D.
and the two dipoles then interact to lower the energy. (b) The two
instantaneous dipoles are correlated, and although they occur in E14B.3 Calculate the average interaction energy for pairs of molecules in the
gas phase with μ = 1 D when the separation is 0.5 nm at 298 K. Compare this
different orientations at different instants, the interaction does not
energy with the average molar kinetic energy of the molecules.
average to zero.
E14B.4 Calculate the average dipole–induced dipole interaction energy, in
J mol−1, between a water molecule and a benzene molecule separated by 1.0 nm.
Although the first molecule will go on to change the size and
orientation of its instantaneous dipole, the electron distribu-
tion of the second molecule will follow; that is, the two dipoles
are correlated in direction (Fig. 14B.4). Because of this correla- 14B.2 Hydrogen bonding
tion, the attraction between the two instantaneous dipoles does
not average to zero as a result of thermal motion, and gives rise The interactions described so far are universal in the sense that
to an induced dipole–induced dipole interaction. This inter- they are possessed by all molecules independent of their spe-
action is called either the dispersion interaction or the London cific identity. However, there is a type of interaction possessed
interaction (for Fritz London, who first described it). by molecules that have a particular arrangement of atoms. A
The strength of the dispersion interaction depends on the hydrogen bond is an attractive interaction between two species
polarizability of the first molecule because the instantaneous that arises from a link of the form A − HB, where A and B
dipole moment of magnitude µ1 depends on the looseness of are highly electronegative elements and B possesses a lone pair
the control that the nuclear charge exercises over the outer elec- of electrons. Hydrogen bonding is conventionally regarded as
trons. The strength of the interaction also depends on the po- being limited to N, O, and F but if B is an anionic species (such
larizability of the second molecule, because that polarizability as Cl−) it may also participate in hydrogen bonding. There is no
determines how readily a dipole can be induced by the electric strict cut-off for an ability to participate in hydrogen bonding,
field of another molecule. The actual calculation of the disper- but N, O, and F participate most effectively.
sion interaction is quite involved, but a reasonable approxima- The formation of a hydrogen bond can be regarded either
tion to the interaction energy is given by the London formula: as the approach between a partial positive charge of H and a
partial negative charge of B or as a particular example of delo-
C I1 I 2 calized molecular orbital formation in which A, H, and B each
V  C  23 1 2 London formula (14B.7)
r6 I1  I 2 supply one atomic orbital from which three molecular orbitals
where I1 and I2 are the ionization energies of the two molecules. are constructed (Fig. 14B.5). Experimental evidence and theo-
This interaction energy is also proportional to the inverse sixth retical arguments have been presented in favour of both views
power of the separation of the molecules, which identifies it as and the matter has not yet been resolved.
a third contribution to the van der Waals interaction. The dis-
persion interaction generally dominates all the interactions be- A H B
tween molecules other than hydrogen bonds.
+− +
Brief illustration 14B.5
Energy

For two CH4 molecules separated by 0.30 nm, use eqn 14B.7 − +
with α′ = 2.6 × 10−30 m3 and I ≈ 700 kJ mol−1 and obtain
30
3 (2.6  10 m )
3 2
(7.00  105 Jmol 1 )2
V   
2 (0.30  109 m)6 2  (7.00  105 Jmol 1 ) + ++
1
 4.9 kJmol
A very approximate check on this figure is the enthalpy of Figure 14B.5 The molecular orbital interpretation of the
vaporization of methane, which is 8.2 kJ mol−1. However, this formation of an A − HB hydrogen bond. From the three A, H, and
comparison is questionable, partly because the total energy B orbitals, three molecular orbitals can be formed (the relative
of interaction between molecules in a liquid is not due only contributions of the A, H, and B orbitals are represented by the
to pairwise interactions and partly because the long-distance sizes of the spheres). Only the two lower energy orbitals are
assumption breaks down. occupied, and there may therefore be a net lowering of energy
compared with the separate AH and B species.
614 14 Molecular interactions

In the molecular orbital model, the A−H bond is regarded as 300


formed from the overlap of an orbital on A, ψA, and a hydrogen

Potential energy, V/(kJ mol–1)


1s orbital, ψH. The orbital on B, ψB, is occupied by a lone pair. 200
When the two molecules are close together, three molecular or-
bitals of the form ψ = c1ψA + c2ψH + c3ψB are built from these
100
three basis orbitals. One of the molecular orbitals is bonding,
one almost nonbonding, and the third antibonding (Topic 9E).
These three orbitals need to accommodate four electrons (two 0
from the original A−H bond and two from the lone pair of B),
so two enter the bonding orbital and two enter the nonbond-
ing orbital. Because the antibonding orbital remains empty, the –100
–180 –90 0 90 180
net effect—depending on the precise energy of the almost non- Angle, Θ/°
bonding orbital—may be a lowering of energy. Figure 14B.6 The variation of the energy of interaction (according
In practice, the strength of a typical hydrogen bond is found to the electrostatic model) of a hydrogen bond as the angle
to be about 20 kJ mol−1 (there are two hydrogen bonds per mol- between the O−H and :O groups is changed.
ecule in liquid water, and its standard enthalpy of vaporization,
from Table 2C.1, is 44 kJ mol−1). The bonding depends on or-
bital overlap, so it is a contact-like interaction that is turned on The lowest potential energy is at Θ = 0 when the OHO
when AH touches B and is zero as soon as the contact is broken. atoms lie in a straight line; the molar potential energy is then
If hydrogen bonding is present, it dominates the other in- −19 kJ mol−1. Note that the interaction energy is negative (and
termolecular interactions. The properties of liquid and solid the interaction is attractive) only between −12° and +12°, so the
water, for example, are dominated by the hydrogen bonding atoms adopt a nearly linear arrangement.
between H2O molecules. The structural evidence for hydrogen
bonding comes from noting that the internuclear distance be-
tween formally non-bonded atoms is less than expected on the
basis of their van der Waals radii, the radii based on the closest 14B.3 The total interaction
approach of non-bonded atoms, which suggests that a domi-
nating attractive interaction is present. For example, the O−O Consider molecules that are unable to participate in the forma-
distance in O − HO is expected to be 280 pm on the basis of tion of a hydrogen bond. The total favourable (energy-lowering)
van der Waals radii, but is found to be 270 pm in typical com- interaction energy between rotating molecules is then the sum
pounds. Moreover, the HO distance is expected to be 260 pm of the dipole–dipole, dipole–induced dipole, and dispersion in-
but is found to be only 170 pm. teractions. Only the dispersion interaction contributes if both
Hydrogen bonds may be either symmetric or non-­symmetric. molecules are nonpolar. In a fluid phase, all three contributions
In a symmetric hydrogen bond, the H atom lies midway be- to the potential energy vary as the inverse sixth power of the
tween the two other atoms. This arrangement is rare, but occurs separation of the molecules, so for all of them and their sum
in F − HF−, where both bond lengths are 120 pm. More com-
C6
mon is the non-symmetric arrangement, where the A−H bond V   (14B.8)
is shorter than the HB bond. Simple electrostatic arguments, r6
treating A − HB as an array of point charges (partial negative where C6 is a coefficient that depends on the identity of the
charges on A and B, partial positive on H), suggest that the low- molecules.
est energy is achieved when the A − HB fragment is linear, Although attractive interactions between molecules are often
because then the two partial negative charges are farthest apart. expressed as in eqn 14B.8, this equation has only limited valid-
The experimental evidence from structural studies supports a ity. First, only dipolar interactions of various kinds are taken
linear or near-linear arrangement. into account, for they have the longest range and are dominant
if the average separation of the molecules is large. However, in
a complete treatment, quadrupolar and higher-order multipole
Brief illustration 14B.6
interactions should also be considered, particularly if the mol-
A common hydrogen bond is that formed ecules do not have permanent dipole moments. Secondly, the
O
between O−H groups and O atoms, as R
expressions have been derived by assuming that the molecules
in liquid water and ice. The electrostatic can rotate reasonably freely. That is not the case in most solids,
Θ
model can be used to calculate the depend- O H and in rigid media the dipole–dipole interaction is proportional
r
ence of the potential energy of interaction to 1/r3 (as in eqn 14B.3b) because the Boltzmann averaging
6
on the OOH angle, denoted Θ in (6), and procedure is irrelevant when the molecules are trapped into a
the results are plotted in Fig. 14B.6. fixed orientation.
14B Interactions between ­molecules 615

A different kind of limitation is that eqn 14B.8 relates to the hard-sphere potential energy, in which it is assumed that the
interactions of pairs of molecules. There is no reason to suppose potential energy rises abruptly to infinity as soon as the parti-
that the energy of interaction of three (or more) molecules is the cles come within a separation d:
sum of the pairwise interaction energies alone. The total disper-
sion energy of three closed-shell atoms of the same type, for in-  for r  d
V  Hard-sphere potential energy  (14B.10)
stance, is given approximately by the Axilrod–Teller formula:
0 for r  d
C6 C6 C6 C This very simple expression for the potential energy is sur-
V   6  6  Axilrod – Teller  (14B.9a)
rAB rBC rCA (rAB rBC rCA )3
6
formula prisingly useful for assessing a number of properties. Another
widely used approximation is the Mie potential energy:
where
Cn Cm
C   a(3 cos A cos B cosC  1) (14B.9b) V  Mie potential energy  (14B.11)
rn rm
The parameter a is approximately B
with n > m. The first term represents repulsions and the sec-
equal to 34  C6; the angles θ are
rAB ond term attractions. The Lennard-Jones potential energy is a
the internal angles of the triangle rBC special case of the Mie potential energy with n = 12 and m = 6
formed by the three atoms (7). θB
A θA (Fig. 14B.8); it is often written in the form
The term in C′ (which represents θC
rCA
the non-additivity of the pairwise C  r 12  r 6  Lennard- Jones
interactions) is negative for a lin- 7
V  4  0    0    (14B.12)
potential energy
ear arrangement of atoms (so that arrangement is stabilized)  r   r  
and positive for an equilateral triangular cluster (so that ar-
rangement is destabilized). The three-body term contributes The two parameters are ε, the depth of the well, and r0, the sepa-
about 10 per cent of the total interaction energy in liquid argon. ration other than infinity at which V = 0 (Table 14B.2).
When molecules are squeezed together, the nuclear and elec- Although the Lennard-Jones potential energy has been used
tronic repulsions begin to dominate the attractive forces. The in many calculations, there is plenty of evidence to show that
repulsions increase steeply with decreasing separation in a way 1/r12 is a very poor representation of the repulsive potential en-
that can be deduced only by very extensive, complicated mo- ergy, and that an exponential form, e −r /r0 , is greatly superior. An
lecular structure calculations of the kind described in Topic 9E exponential function is more faithful to the exponential decay
(Fig. 14B.7). of atomic wavefunctions at large distances, and hence to the
In many cases, however, progress can be made by using a overlap that is responsible for repulsion. The potential energy
greatly simplified representation of the potential energy, where with an exponential repulsive term and a 1/r6 attractive term is
the details are ignored and the general features expressed by known as an exp-6 potential energy. These expressions for the
a few adjustable parameters. One such approximation is the potential energy can be used to calculate the virial coefficients
of gases, as explained in Topics 1C and 13D, and through them
various properties of real gases. They are also used to model the
structures of condensed fluids.
Potential energy
Potential energy, V(r)

due to repulsion
6
Potential energy
Potential energy due to repulsion,
Potential energy, V(r)/4ϵ

due to attraction 4 1/r12


Total
2
0 Total
21/6r0
0
Separation, r
–2 Potential energy
–ϵ
Figure 14B.7 The general form of an intermolecular potential due to attraction,
energy curve (the graph of the potential energy of two closed- –1/r 6
–4
shell species as the distance between them is changed). The 0 0.5 Separation, r/r 1 1.5
0
attractive (negative) contribution has a long range, but the
repulsive (positive) interaction increases more sharply once the Fig 14B.8 The Lennard-Jones potential energy. Interact with the
molecules come into contact. dynamic version of this graph in the e-book.
616 14 Molecular interactions

Table 14B.2 Lennard-Jones-(12,6) potential energy parameters* The solution Use dxn/dx = nxn−1:

(ε/k)/K r0/pm dF 24   13r013   7r07   6 7 26r06 


 2   14     8    24 r0  8  14 
Ar 111.84 362.3 dr r0   r   r   r r 
F2 104.29 357.1
It follows that dF/dr = 0 when
C6H6 377.46 617.4
Cl2 296.27 448.5 7 26r06
  0, or 7r 6  26r06  0
N2 91.85 391.9 r 8 r 14
O2 113.27 365.4
That is, when
Xe 213.96 426
1/ 6
* More values are given in the Resource section.  26 
r    r0  1.244r0
 7 
With the advent of atomic force microscopy (AFM), in At this separation the force is
which the force between a molecular-sized probe and a surface
24   r 
13
 r0  
7
2.396
is monitored (Topic 19A), it has become possible to measure F 2 
0
   
directly the forces acting between molecules. The force, F, is r0   1.244r0   1.244r0   r0
the negative slope of the potential energy, so for the Lennard-
From Table 14B.2, ε/k = 91.85 K, which gives ε = 1.268 × 10−21 J,
Jones potential energy between individual molecules the force
and r0 = 3.919 × 10−10 m. It follows that
is given by
1 N = 1J m−1
dV 24   r0   r0  
13 7
−21
F   2.396 × (1.268 × 10 J)
2       (14B.13) F=− = − 7.752 × 10−12 N
dr r0   r   r   3.919 × 10−10 m

That is, the magnitude of the force is about 8 pN.


Example 14B.1  Calculating an intermolecular force from Self-test 14B.1 At what separation r does a Lennard-Jones
the Lennard-Jones potential energy potential energy have its minimum value?
Answer: r = 21/6r0
Use the expression for the Lennard-Jones potential energy
to estimate the greatest net attractive force between two N2
molecules.
Collect your thoughts The force is greatest when dF/dr = 0. Exercise
Therefore you need to differentiate eqn 14B.13 with respect E14B.5 The Lennard-Jones parameters for O2 are ε/k = 113.27 K and
to r, set the resulting expression to zero, and then solve for r. r0 = 365.4 pm. Compute the distance at which the potential energy is a
Finally, use the value of r in eqn 14B.13 to calculate the cor- minimum and the value of potential energy at this distance. Comment on the
responding value of F. comparison between this energy with the thermal energy at 298 K.

Checklist of concepts
☐ 1. A van der Waals interaction is an attractive interac- ☐ 3. The van der Waals radius of an atom is based on the
tion between closed-shell molecules; the corresponding closest approach of non-bonded atoms.
potential energy is inversely proportional to the sixth ☐ 4. A hydrogen bond is an interaction of the form A − H B,
power of their separation. where A and B are typically N, O, or F.
☐ 2. The following molecular interactions are important: ☐ 5. The Lennard-Jones potential energy is a model of the
charge–dipole, dipole–dipole, dipole–induced dipole, total intermolecular potential energy, including repulsion.
dispersion (London), and hydrogen bonding.
14B Interactions between ­molecules 617

Checklist of equations
Property Equation Comment Equation number
2
Energy of interaction between a point dipole and a point charge V = −μ1Q2/4πε0r Linear arrangement 14B.1
3
Energy of interaction between two fixed dipoles V = μ1μ2f(Θ)/4πε0r , Parallel dipoles 14B.3b
2
f(Θ) = 1 − 3cos Θ
Energy of interaction between two rotating dipoles V  2 12 22 /3(4  0 )2 kTr 6 14B.4

Energy of interaction between a polar molecule and a polarizable molecule V   12 2 /4  0r 6 14B.6

London formula V   23 1 2 {I1 I 2 /(I1  I 2 )}/r 6 14B.7


12 6
Lennard-Jones potential energy V = 4ε{(r0/r) − (r0/r) } 14B.12
TOPIC 14C Liquids

intermolecular interactions and, just as for a real gas, a model


➤ Why do you need to know this material? based on these interactions can be used to develop an equation
Many substances are liquids under normal conditions of state.
and many chemical reactions take place in liquids, so it is
important to be able to describe and understand the struc- 14C.1(a) The radial distribution function
ture of the liquid phase and the interface with its vapour.
The average relative locations of the particles of a liquid are ex-
➤ What is the key idea? pressed in terms of the radial distribution function (or pair
The properties of liquids reflect the short-range order of distribution function), g(r). This function is defined so that
their molecules in the bulk and the behaviour of their mol- 4 πN g (r )r 2 dr is the number of molecules in a shell of thickness
ecules at the mobile surface. dr at radius r from a given molecule; N = N /V is the overall
number density. For a uniform material, g(r) = 1. When defined
➤ What do you need to know already? in this way the radial distribution function is also the ratio of
You need to be aware of the ways in which molecules inter- the number of molecules in a shell to the number expected in
act with each other (Topic 14B), and to be familiar with the the same shell for a uniform material. If, at a particular dis-
Helmholtz and Gibbs energies and their significance (Topic tance, g(r) > 1 there are more molecules than in a uniform ma-
3D). The Topic makes light use of the Boltzmann distribu- terial, whereas if g(r) < 1, there are fewer.
tion (see Energy: A first look and Topic 13A). In a perfect crystal, g(r) is a periodic array of sharp spikes,
representing the certainty (in the absence of defects and ther-
mal motion) that molecules (or ions) lie at definite locations.
This regularity continues out to the edges of the crystal, so crys-
Molecules attract each other when they are less than a few di- tals are said to have long-range order. When the crystal melts,
ameters apart, but as soon as they come into contact they repel the long-range order is lost and the probability of finding a sec-
each other. The attraction is responsible for the formation of ond molecule at long distances from the first is independent of
condensed phases, including liquids, and the repulsion is re- the distance. However, close to the first molecule the nearest
sponsible for the fact that liquids (and solids) have a definite neighbours might still adopt approximately their original rela-
bulk. In a liquid the kinetic energies of the molecules are com- tive positions and, even if thermal motion drives them away,
parable to their potential energies and, as a result, although the incoming molecules adopt the vacated positions. It is therefore
molecules of a liquid are not free to escape completely from the still possible to detect a shell of nearest neighbours at a distance
bulk at low temperatures, the structure is very mobile. The co- r1, and perhaps beyond them a shell of next-nearest neighbours
hesive forces responsible for the formation of liquids also result at r2. The existence of this short-range order means that the ra-
in the interface between the liquid and another phase having dial distribution function in a liquid can be expected to oscil-
an effect on the thermodynamic and physical properties of the late at short distances, with a peak at r1, a smaller peak at r2, and
liquid. perhaps some more structure beyond that.
The experimentally determined radial distribution func-
tion of the oxygen atoms in liquid water is shown in Fig. 14C.1.
Close analysis of a more elaborate form of the distribution
14C.1Molecular interactions in function shows that any given H2O molecule is surrounded by
liquids other molecules at the corners of a tetrahedron. The form of
g(r) at 100 °C shows that the intermolecular interactions (in
The starting point for the discussion of solids is the well-­ this case, principally hydrogen bonds) are strong enough to
ordered structure of a perfect crystal (Topic 15A). The starting affect the local structure right up to the boiling point. Raman
point for the discussion of gases is the completely disordered spectra indicate that in liquid water most molecules participate
distribution of the molecules of a perfect gas (Topic 1A). in either three or four hydrogen bonds. Infrared spectra show
Liquids lie between these two extremes. The structural and that about 90 per cent of hydrogen bonds are intact at the melt-
thermodynamic properties of liquids depend on the nature of ing point of ice, falling to about 20 per cent at the boiling point.
14C Liquids 619

Radial distribution function, g(r) 2

4 °C

25 °C
100 °C

0
0 200 400 600 800 1000
Radius, r/pm
Figure 14C.2 In a two-dimensional simulation of a liquid that
Figure 14C.1 The radial distribution function of the oxygen atoms uses periodic boundary conditions, when one particle leaves the
in liquid water at three temperatures. Note the expansion as the cell (on the left here) its mirror image enters through the opposite
temperature is raised. (Based on A.H. Narten, M.D. Danford, and face (on the right).
H.A. Levy, Discuss. Faraday Soc. 43, 97 (1967).)

The formal expression for the radial distribution function for Numerical methods typically approach the calculation of the
molecules 1 and 2 in a fluid consisting of N particles is distribution function by considering a box containing about 103
1 particles (the number increases as computers grow more pow-
g (r12 )  2
(N  2)!N Z  e   VN d 3 d 4 d N erful), and then mimicking the rest of the liquid by surround-
ing the box with replications of the original box (Fig. 14C.2).
n (14C.1a)
Radial distribution function
Whenever a particle leaves the box through one of its faces, its
where dτi is the volume element for molecule i, β = 1/kT, VN is image arrives through the opposite face. When calculating the
the N-particle potential energy, and Z is the ‘configuration inte- interactions of a molecule in a box, it interacts with all the mol-
gral’ (this quantity is introduced in Topic 13D): ecules in the box and all the periodic replications of those mol-
ecules (and of itself) in the other boxes.
1   VN Configuration integral In the Monte Carlo method, the particles in the box are first
N!
Z e d 1d 2 d N [definition]
 (14C.1b)
distributed at random and then moved through small random
distances. The change in total potential energy of the N parti-
Equation 14C.1a is nothing more than the Boltzmann distribu-
cles in the box, ΔVN, is calculated, and if the new arrangement
tion for the relative locations of two molecules in a field pro-
has lower potential energy than the original one, it is accepted.
vided by all the molecules in the system. Thus, if there are no
If the potential energy increases, implying that ∆VN is positive,
interactions between molecules (so VN = 0), Z = V N /N ! and
the new arrangement is accepted only if the Boltzmann-type
factor e VN /kT is greater than a random number (chosen to lie
V N−2
somewhere in the range 0 to 1). If this criterion is not met, a
N!
( N − 2)! N 2V N ∫ 3 4
g (r12 ) = dτ dτ … dτ N new arrangement is generated from the original one and tested
in the same way.
N >> 1 The result of applying this selection process is that moving
N ( N −1) N ( N −1) to an arrangement in which the energy is lower is always al-
= = =1 lowed but moving to one of higher energy, although possible,
N 2V 2 N2
becomes increasing unlikely the higher its energy. As a result,
In the absence of interactions the fluid is expected to be uni- the simulation explores a wide range of arrangements, but
form, which is consistent with this value of g(r12). tends to include fewer with high energies. For each accepted ar-
rangement the number of pairs of molecules with a separation
r is counted, and the result is then averaged over the whole col-
14C.1(b) The calculation of g(r) lection of accepted arrangements; by repeating this for different
The integrals in eqn 14C.1 are very difficult to evaluate, so vari- values of r, the form of g(r) is built up.
ous numerical procedures are used to calculate the radial distri- In the molecular dynamics approach, the history of an ini-
bution function. Such calculations involve specifying the form tial arrangement is followed by calculating the trajectories of all
of the intermolecular potential energy, for example by specify- the molecules under the influence of the intermolecular poten-
ing it as a pairwise Lennard-Jones interaction (Topic 14B). tials and the forces they exert. The calculation gives a series of
620 14 Molecular interactions

which it is more revealing to write as


Radial distribution function, g(r)

2 
High U interaction (T )  12 N  V2 (r ){4N g (r )}r 2 dr  (14C.2b)
0
density

This formulation shows that the internal energy is given by


1
4 πN g (r )r 2 dr , which is the number of molecules in a shell of ra-
Low dius r and thickness dr, multiplied by V2(r), and then integrated
density
over r, which gives the total energy of interaction of one mole-
cule with all the others. Multiplication by N then gives the total
0 energy of interaction of all the molecules; the factor 12 is needed
0 1 2 3 4 to avoid counting each interaction twice. Likewise, the equa-
Radius, r/d
tion of state of the dilute fluid including contributions from the
Figure 14C.3 The radial distribution function for a simulation of a pairwise interactions is
liquid using impenetrable hard spheres (such as ball bearings) of
radius d. pV 2N  dV2 (r )
nRT
1
3kT 0
v2 g (r )r 2 dr v2 (r )  r
dr
 (14C.3a)

snapshots of the liquid, and g(r) can be calculated as before. The The quantity v2 (r ) is called the virial (hence the term ‘virial
temperature of the system is inferred by computing the mean equation of state’). To understand the physical significance of
translational kinetic energy of the molecules and using the equi- this expression, it can be rewritten as
partition result (see: Energy, a first look) that 12 mvq2 = 12 kT , for
each coordinate q. nRT 2 2  Pressure in
p  N  v2 (r ) g (r )r 2 dr terms of g( r )
 (14C.3b)
Such numerical calculations for a fluid of hard spheres with- V 3 0

out attractive interactions (a collection of ball-bearings in a


and interpreted as follows:
container) give a radial distribution function that oscillates
for small separations of the molecules (Fig. 14C.3). It appears • The first term on the right is the kinetic pressure, the con-
that one of the factors influencing, and sometimes dominating, tribution to the pressure from the impact of the molecules
the structure of a liquid is the geometrical problem of stacking in free flight, as in a perfect gas.
together reasonably hard spheres. Indeed, the radial distribu- • The second term, as explained below, is essentially the inter-
tion function of a liquid composed of hard spheres shows more nal pressure, πT = (∂U/∂V)T (Topic 2D), representing the
pronounced oscillations at a given temperature than that of any contribution of the intermolecular forces to the pressure.
other model liquid. The attractive part of the potential modifies
this basic structure, and one of the reasons behind the difficulty To see the connection to the internal pressure, the term
of describing actual liquids theoretically is the similar impor- −dV2/dr (in v2 ) should be recognized as the force required to
tance of both the attractive and repulsive (hard core) compo- move two molecules apart, and therefore −r(dV2/dr) is the
nents of the potential energy. work required to separate the molecules through a distance r.
The second term is therefore the average of this work over the
range of pairwise separations in the fluid, with the contribu-
14C.1(c)The thermodynamic properties tions to the average weighted by the probability of finding
of liquids two molecules at separations between r and r + dr, which is
Once g(r) is known, it can be used to calculate the thermo- 4πg(r)r2dr. That is, the integral, when multiplied by the square
dynamic properties of liquids, but for dense fluids the calcu- of the number density, is the change in internal energy of the
lations are very complicated. The calculations are simpler for system as it expands, (∂U/∂V)T , and is therefore equal to the
fluids that are so dilute that they are little more than real gases. internal pressure. A deeper look 14C.1, available to read in
In such a case, if it is assumed that the interaction of each pair the e-book accompanying this text, explains how this interpre-
of molecules is given by an isotropic pairwise potential energy tation can be used to infer the virial equation of state of a real
function, V2(r), the result is a contribution to the internal en- gas and interpret the van der Waals parameters.
ergy given by The pressure given by eqn 14C.3b has nothing to do with
the hydrostatic pressure, the pressure experienced at the foot
2N 2  of a column of incompressible liquid. The mass of a column of
U interaction (T ) 
V 0
V2 (r ) g (r )r 2 dr
liquid of mass density ρ, height h, and cross-sectional area A
Contribution of pairwisee interactions to the internal energy (14C.2a) is ρhA. In a gravitational field the downward force is ρhAgacc,
14C Liquids 621

where gacc is the acceleration of free fall (it is normally denoted Table 14C.1 Surface tensions of liquids at 293 K*
simply g, but in this Topic it is necessary to distinguish it from
γ/(mN m−1)
the radial distribution function). The hydrostatic pressure, the
Benzene 28.88
force divided by the area A on which it is exerted, is therefore
Mercury 472
Hydrostatic pressure Methanol 22.6
p   g acc h [incompressible fluid]
 (14C.4)
Water 72.75
The molecular origin of this pressure is as follows. Bear in * More values are given in the Resource section. Note that 1 mN m−1 = 1 mJ m−2.
mind that the fluid is incompressible, so if a molecule moves,
space must be made available for it. If a molecule moves up
through a molecular diameter from a certain point at a loca- ­surface area, σ, of a sample by an infinitesimal amount dσ is
tion a distance h down in the liquid, the entire column of height proportional to dσ, and is written
h above it must move up by a molecular diameter. The force
Surface tension
required is ρgaccha, where a is the cross-sectional area of that dw   d  (14C.5)
[definition]
molecular column: this force corresponds to a pressure ρgacch.
If a molecule moves to one side through a molecular diameter, The constant of proportionality, γ, is called the surface ­tension;
the incompressible column above the new position must move its dimensions are energy/area and so in SI its units are joules
up to make room for it. Once again, the force required corre- per metre squared (J m−2). However, for reasons that will be-
sponds to a pressure ρgacch. Even if a molecule moves down, a come clear, values of γ are usually reported in newtons per
molecule must move to one side to make room for it, and there- metre (because 1 J = 1 N m, it follows that 1 J m−2 = 1 N m−1);
fore a whole column must move up to allow that movement, Table 14C.1 gives some typical values. The maximum work of
and the force once again corresponds to a pressure ρgacch. This surface formation at constant volume and temperature can be
interpretation shows why the hydrostatic pressure is isotropic identified with the change in the Helmholtz energy:
even though gravity operates downwards. It also shows why the
interior of a solid column does not have an analogous hydro- dA   d  (14C.6)
static pressure: the molecules cannot move past each other, so
The Helmholtz energy decreases (dA < 0) if the surface area
there are no consequent forces involved.
decreases (dσ < 0). A process in which A decreases is spon-
taneous, so it follows that surfaces have a natural tendency to
contract, which is the thermodynamic explanation of the ob-
14C.2 The liquid–vapour interface servations made at the start of this section.

The distinctive feature of the interface between the liquid and Example 14C.1  Using the surface tension
its vapour is that it is mobile and molecules there experience at-
tractive forces that no longer pull equally in all directions. Consider the arrangement shown in Fig. 14C.4 in which a wire
frame of width l is raised out of a liquid to a height h, thereby
generating a rectangular film of liquid within the frame.
14C.2(a) Surface tension Calculate the work needed to draw a frame of width 5.0 cm out
Liquids tend to adopt shapes that minimize their surface area,
because this maximizes the number of molecules that are in
the bulk and hence are surrounded by and interact with neigh- l Force
bours. Droplets of liquids therefore tend to be spherical, be-
Total area
cause a sphere is the shape with the smallest surface-to-volume
= 2hl
ratio. However, the presence of other forces distort this ideal
shape: the combined effect of gravity and adhesion to a surface h
results in small droplets on a surface becoming flattened, and
gravity results in larger volumes of liquid adopting the shape of
the lower part of its container.
Surface effects can be analysed by using the properties of the
Helmholtz and Gibbs energies, A and G (Topic 3D). As shown
there, under appropriate conditions, dA and dG are equal to Figure 14C.4 The model used for calculating the work of forming
the work done on the system, including the work done when a liquid film when a wire frame of width l is raised from a liquid to
the surface area changes. The work needed to change the a height h.
622 14 Molecular interactions

of water at 20 °C (for which γ = 72.75 mJ m−2) through 2.0 cm; Step 1 Evaluate the force due to the external pressure
disregard gravitational potential energy. The force on the surface of a spherical cavity of radius r due to
Collect your thoughts If you assume that the surface tension the external pressure pout is given by area × pressure = 4πr2pout.
does not vary with the area, eqn 14C.5 becomes w = γΔσ for an Step 2 Evaluate the force due to surface tension
increase in surface area Δσ. The increase in surface area is from
The change in surface area when the radius changes from r to
zero to the area of the rectangle, but you need to recognize that
r + dr is
two surfaces are created, one on each side of the frame. Once
you have the appropriate expression, insert the data. d  4 (r  dr )2  4 r 2  8rdr
The solution The area of the rectangle is lh, and hence the (The second-order infinitesimal, (dr)2, has been ignored.) The
increase in surface area of the film is 2lh; the work done is work done when the surface is stretched by this amount is
therefore 2γlh. A frame of width 5.0 cm pulled out of water at therefore
20 °C through 2.0 cm requires
dw   d  8 rdr
2 2 2
w  2  (72.75 mJm )  (5.0  10 m)  (2.0  10 m)  0.15 mJ
As force × distance is work, the force opposing an increase in
the radius by dr is
Comment. The expression 2γlh can be thought of as 2γl × h,
which is force × distance. The term 2γl can be identified as F  8 r
the opposing force on the top of the frame, which has length l.
This interpretation is why γ is called a tension and why its units The total inward force is therefore 4πr2pout + 8πγr.
are often chosen to be newtons per metre (so γl is a force in Step 3 Balance the inward and outward forces
newtons).
If the pressure inside the cavity is pin, the outward force on
Self-test 14C.1 Derive an expression for the work of creating a the surface is 4πr2pin. At equilibrium, the outward and inward
spherical cavity of radius r in a liquid of surface tension γ; eval- forces are balanced:
uate this work for a cavity of radius 1.0 cm in water at 20 °C.
Answer: 4πr2γ, 0.091 mJ 4 r 2 pin  4 r 2 pout  8 r

Division of both sides by 4πr2 results in the Laplace equation:

2γ (14C.7)
14C.2(b) Curved surfaces pin = pout +
r Laplace equation

The minimization of the surface area of a liquid commonly


results in the formation of a curved surface. A bubble is a re-
The Laplace equation shows that the difference in pressure
gion in which vapour (and possibly air too) is trapped by a thin
decreases to zero as the radius of curvature becomes infinite
film; a cavity is a vapour-filled hole in a liquid. What are widely
(when the surface is flat, Fig. 14C.5). Small cavities have small
called ‘bubbles’ in liquids are therefore strictly cavities. True
radii of curvature, so the pressure difference across their sur-
bubbles have two surfaces (one on each side of the film); cavi-
face is quite large.
ties have only one. The treatments of both are similar, but a fac-
tor of 2 is required for bubbles to take into account the doubled
surface area. A droplet is a small volume of liquid surrounded
by its vapour (and possibly also air).
The tendency for a cavity to minimize its surface area re-
sults in the pressure inside the cavity (the concave side of
Pressure inside, pin

pout
the surface) being greater than that outside (the convex side
of the surface). The challenge is to find the relation between pin
these two pressures. Increasing
surface tension, γ
r

How is that done? 14C.1 Relating the pressures inside and


outside a cavity pout
0 Radius, r
A cavity is at equilibrium when the tendency for its surface
area to decrease is balanced by the rise in internal pressure that Figure 14C.5 The dependence of the pressure inside a curved
would result. Equilibrium is achieved when the inward and surface on the radius of the surface, for increasing values of the
outward forces on the surface are equal. surface tension.
14C Liquids 623

Brief illustration 14C.1 p


p – 2γ/r p
The pressure difference across the surface of a spherical droplet
of water of radius 200 nm at 20 °C can be calculated using the
r
Laplace equation:
p – 2γ/r + ρgacch
 water at 20 C h
  p p
2  (72.75  103 N m 1 )
pin  pout 
2.00  107 m
  
r

 7.28  10 N m 2  728 kPa


5
p p p

Figure 14C.6 When a capillary tube is first stood in a liquid, the


liquid climbs up the walls, so curving the surface and creating
14C.2(c) Capillary action a meniscus. The pressure just under the meniscus is less than
When a narrow-bore tube (a ‘capillary tube’; the name comes the atmospheric pressure by 2γ/r. The pressure is equal at equal
from the Latin word for ‘hair’) is dipped in a liquid there is a heights throughout the liquid provided the hydrostatic pressure
tendency for the liquid to rise up the tube. This tendency is (which is equal to ρgacch) cancels the pressure difference arising
from the curvature.
called capillary action and can be understood as a consequence
of surface tension.
Water has a tendency to adhere to the surface of glass, so 78
when a glass capillary tube is first immersed in water a film of
Surface tension, γ/(mN m–1)

solvent spreads along the surface: the further the film spreads, 74
the lower is the energy due to the interaction. As the film
spreads up the inside walls of the capillary tube the surface of 70
the liquid becomes curved creating what is known as a me-
niscus. As has just been discussed, this curvature results in a 66
pressure difference across the surface. The pressure just above
the surface will be the atmospheric pressure, and therefore the 62
pressure just below the surface, the convex side, will be lower
than the atmospheric pressure. 58
0 20 40 60 80 100
If it is assumed that the surface is hemispherical and that the Temperature, θ/°C
tube has radius r, the pressure difference is given by the Laplace
Figure 14C.7 The variation of the surface tension of water with
equation as 2γ/r. The pressure just above the surface is the at-
temperature.
mospheric pressure, p, so the pressure just below the surface is
p − 2γ/r. The atmospheric pressure pushing on the liquid out-
side the tube causes the liquid inside the tube to rise until hy- ­ ecreases with increasing temperature (Fig. 14C.7), which
d
drostatic equilibrium is achieved, which is when there are equal can be understood as arising from the increase in thermal
pressures at equal depths (Fig. 14C.6). motion moving molecules more rapidly between the surface
When the liquid has risen to a height h the column exerts a and the bulk.
hydrostatic pressure given by eqn 14C.4, ρgacch, where ρ is the
mass density of the liquid and gacc is the acceleration of free Brief illustration 14C.2
fall. The total pressure at the base of the column is therefore
(p − 2γ/r) + ρgacch. At the same level outside the tube the pres- If water at 25 °C (with mass density 997.1 kg m−3) rises through
sure is p, but at equilibrium these two pressures must be the 7.36 cm in a capillary of radius 0.20 mm, its surface tension at
same: p = p − 2γ/r + ρgacch. It therefore follows that 2γ/r = ρgacch that temperature is found from eqn 14C.8 as
which rearranges to γ = 12 ρ g acc hr
2 = 12 × (997.1 kg m−3 ) × (9.81 ms −2) × (7.36 ×10−2 m) × (2.0 ×10−4 m)
h  (14C.8)
 g acc r
= 7.2 × 10−3 kg s−2 = 72 mNm −1
This simple expression provides a reasonably accurate way where 1 kg ms 2  1 N is used.
of measuring the surface tension of liquids. Surface tension
624 14 Molecular interactions

+1
γsg

cos θc
γlg γsl 0

θc

–1
0 1 2
wad/γlg
Figure 14C.8 The balance of forces that results in a contact
angle, θc. Figure 14C.9 The variation of contact angle as the ratio wad/γlg
changes.
When the adhesive forces between the liquid and the ma-
terial of the capillary wall are weaker than the cohesive forces
within the liquid (as for mercury in glass), it is energetically fa- that it spreads over the surface. From eqn 14C.10b wetting
vourable for the liquid in the tube to retract from the walls. This occurs when 1 < wad/γlg < 2 (Fig. 14C.9).
retraction curves the surface with the concave, high pressure • When the contact angle is between 90° and 180° (implying
side downwards. To equalize the pressure at the same depth that −1 < cos θc < 0) the liquid does not wet the surface;
throughout the liquid the surface must fall to compensate for this condition corresponds to 0 < wad/γlg < 1.
the increased pressure arising from its curvature. This compen-
sation results in a capillary depression. For mercury in contact with glass, θc = 140°, which corresponds
The angle between the surface and the wall, where the two to wad/γlg = 0.23, indicating a relatively low work of adhesion of
meet, is called the contact angle, θc (Fig. 14C.8). In many cases the mercury to glass on account of the strong cohesive forces
this angle is found to be non-zero, and eqn 14C.8 must then be within the liquid.
modified by multiplying the right-hand side by cos θc. The ori-
gin of the contact angle can be traced to the balance of forces at Exercises
the line of contact between the liquid and the solid (Fig. 14C.8).
E14C.1 Calculate the pressure differential across the surface of a spherical
If the solid–gas, solid–liquid, and liquid–gas surface tensions droplet of water with radius 200 nm at 20 °C.
(essentially the work of forming each interface) are denoted γsg,
E14C.2 The contact angle for water on clean glass is close to zero. Calculate the
γsl, and γlg, respectively, then the vertical forces are in balance if surface tension of water at 20 °C given that at this temperature water climbs to
a height of 4.96 cm in a clean glass capillary tube of internal radius 0.300 mm.
 sg   sl   lg cos c (14C.9a) The mass density of water at 20 °C is 998.2 kg m−3.

and therefore
 sg   sl
cos c 
 lg
 (14C.9b) 14C.3 Surface films
The ‘superficial work’ of adhesion of the liquid to the solid, The compositions of surface layers have been investigated by
wad, is the work of adhesion divided by the area of contact. As the simple but technically elegant procedure of slicing thin
the liquid–solid interface expands it does so at the expense layers off the surfaces of solutions and analysing their com-
of the solid–gas and liquid–gas interfaces, and the net work positions. The physical properties of surface films have also
involved is been investigated. Surface films one molecule thick are called
­monolayers, and when such a monolayer has been transferred
wad   sg   lg   sl  (14C.10a)
to a solid support, it is called a Langmuir–Blodgett film, after
and therefore, from the previous equation, Irving Langmuir and Katharine Blodgett, who developed ex-
perimental techniques for studying them.
wad
cos c  1 Contact angle (14C.10b)
 lg
14C.3(a) Surface pressure
It is now seen that:
The principal apparatus used for the study of surface monolay-
• When the contact angle is between 0 and 90° (implying ers is a surface-film balance (or Langmuir−Blodgett trough,
that 0 < cos θc < 1) the liquid ‘wets’ the surface, meaning Fig. 14C.10). This device consists of a shallow trough and a
14C Liquids 625

Liquid + surfactant 50

Surface pressure, π/(mN m–1)


40
Stearic
acid
30
Liquid Liquid
Isostearic
20
acid Tri-p-cresyl
phosphate
Compression 10
barrier Mica float
0
Figure 14C.10 A schematic diagram of the apparatus used 0 0.2 0.4 0.6 0.8 1
to measure the surface pressure and other characteristics of a Area per molecule/nm2
surface film. The surfactant is spread on the surface of the liquid
in the trough, and then compressed horizontally by moving Figure 14C.11 The variation of surface pressure with the average
the compression barrier towards the mica float. The latter is area occupied by a surfactant molecule. The collapse pressures are
connected to a torsion wire, so the net force on the float can be indicated by the horizontal arrows.
monitored.
O

barrier that can be moved along the surface of the liquid in HO


the trough, thus compressing any monolayer on the surface.
The surface pressure, π, the difference between the surface 1 Stearic acid, C17H35COOH
tension of the pure solvent and the solution (π = γ* − γ), is
measured by using a torsion wire attached to a strip of mica
that rests on the surface and pressing against one edge of the O
monolayer. The parts of the apparatus that are in touch with
liquids are coated in polytetrafluoroethene to eliminate effects HO
arising from the liquid–solid interface. In an actual experi-
ment, a small amount (about 0.01 mg) of the substance under 2 Isostearic acid, C17H35COOH
investigation is dissolved in a volatile solvent and then poured
on to the surface of the water; the compression barrier is then
moved across the surface and the surface pressure exerted on
the mica bar is monitored.
When the surface coverage is low it is found that the surface
pressure is inversely proportional to the total area of the sur-
face. This behaviour is the analogue in two dimensions of a per- O
fect gas (where p ∝ 1/V), and can be interpreted as arising when O P O
the average separation between the molecules at the surface is O
so large that the interactions between them are not important.
As the area is further decreased, the surface pressure eventually 3 Tri-p-cresyl phosphate
starts to increase more rapidly, as is illustrated in Fig. 14C.11.
This behaviour can be thought of as arising from the formation
of a monolayer in which the molecules are in relatively close The second feature to note from Fig. 14C.11 is that the tri-p-
contact; like a liquid, such a layer is almost incompressible. The cresyl phosphate isotherm is much less steep than the stearic
area corresponding to a complete close-packed monolayer is acid isotherms. This difference indicates that the tri-p-cresyl
found by extrapolating the steepest part of the isotherm. phosphate film is more compressible than the stearic acid films,
As can be seen from Fig. 14C.11, even though stearic acid which is consistent with their different molecular structures.
(1) and isostearic acid (2) are chemically very similar (they dif- A third feature of the isotherms is the collapse pressure, the
fer only in the location of a methyl group at the end of a long highest surface pressure at which a monolayer can be main-
hydrocarbon chain), they occupy significantly different areas in tained. When the monolayer is compressed beyond the col-
the monolayer. Neither, though, occupies as much area as the lapse pressure, the monolayer buckles and collapses into a film
tri-p-cresyl phosphate molecule (3), which is like a wide bush several molecules thick. As can be seen from the isotherms in
rather than a lanky tree. Fig. 14C.11, stearic acid has a high collapse pressure, but that of
626 14 Molecular interactions

tri-p-cresyl phosphate is significantly lower, indicating a much At equilibrium the chemical potential of each component is the
weaker film. same in each phase and is written μJ.
Step 2 Integrate the infinitesimal change
14C.3(b) The thermodynamics of surface Following the same argument as in the discussion of partial
layers molar quantities (Topic 5A), this equation can be integrated at
constant temperature, surface tension, and composition to give
A surfactant is a species that accumulates at the interface between
two phases, such as that between hydrophilic and hydrophobic G( )    JnJ ( )
phases, and modifies the surface tension. The relation between J

the concentration of surfactant at the surface and the change in Step 3 Identify the total change in Gibbs energy implied by this
surface tension it brings about can be established by considering expression
a model in which two phases α and β come into contact, therefore An infinitesimal change in each of the quantities on the right of
creating an interface. Within each bulk phase the composition is this expression gives the following total change in G(σ):
constant, but the composition in the interfacial region may be dif-
ferent due to the accumulation of the surfactant. dG( )   d   d  JdnJ ( )  nJ ( )dJ
J J
The total Gibbs energy can be thought of as having a con-
tribution from the two phases, G(α) and G(β), together with a Step 4 Use the fact that G is a state function
contribution G(σ), the surface Gibbs energy, from the interfa- Because Gibbs energy is a state function, the two expressions
cial region for dG(σ) must be the same. Their comparison implies that
 d  nJ ( )dJ  0
Surface Gibbs energy
G  G()  G()  G() [definition]
 (14C.11) J

Division by σ and introduction of the definition of the surface


In a similar way, the total amount of substance J, nJ, can be excess (ΓJ = nJ(σ)/σ) gives the Gibbs isotherm, which relates
thought of as being divided between the amounts in phases α the change in surface tension to the changes in the chemical
and β, nJ(α) and nJ(β), and the amount at the interfacial region, potentials of the substances present in the interface
nJ(σ): nJ = nJ(α) + nJ(β) + nJ(σ). The amount at the interface can
d   J dJ Gibbs isotherm (14C.13)
be expressed in terms of the surface excess, ΓJ: J

nJ ( ) Surface excess Step 5 Relate the change in chemical potentials to the composition
J   (14C.12)
 [definition] If just one species, the surfactant S, accumulates at the surface,
the Gibbs isotherm becomes
where σ is the area of the surface. It is possible to relate the sur-
face tension to the surface excess and therefore to the concen- d    S d S
tration of surfactant at the interface. The chemical potential for species J in a dilute solution can be
written as J  J⦵ RT ln(cJ /c⦵), where cJ is the molar concen-
How is that done? 14C.2 Relating the surface tension to tration and c⦵ is its standard value. It follows that at constant
temperature dS  RTd ln(c /c ⦵). This expression for dμS is used
the surfactant concentration
in the equation above to give
The change in G brought about by changes in T, p, and nJ is
given by eqn 5A.6: d  RT  S d ln(c /c ⦵)
which can be rearranged to
dG  SdT  Vdp  JdnJ
J

where μJ is the chemical potential of substance J. At constant γ (14C.14)


= − RT Γ S
pressure, Vdp = 0 and at constant temperature SdT = 0, so the ln(c / c ⦵ ) T
Dependence of the surface tension
on surfactant concentration
first two terms on the right vanish.
Step 1 Write an expression for the change in Gibbs energy of the When the surfactant accumulates at the interface, its
interface surface excess is positive and eqn 14C.14 implies that
To apply what remains of this relation to the interface, it is nec- ( / ln(c /c ⦵))T  0. That is, accumulation of the surfactant
essary to introduce an additional term γdσ (eqn 14C.5) arising causes the surface tension to decrease. If the variation of γ with
from the work done expanding the interface. The expression concentration is measured, eqn 14C.14 can be used to deter-
for the change in Gibbs energy of the interface, dG(σ), becomes mine the surface excess, and this value can be used to infer the
dG( )   d  JdnJ ( ) area occupied by each surfactant molecule on the surface, as
J illustrated in Example 14C.2.
14C Liquids 627

Example 14C.2 For droplets of water of radius 1 μm and 1 nm the ratios


Determining the surface excess and the
p/p* at 25 °C are about 1.001 and 3, respectively. The second
surface concentration of surfactant molecules
figure, although quite large, is unreliable because at that radius
Measurements of the surface tension of an aqueous solution the droplet is less than about 10 molecules in diameter and the
of 1-aminobutanoic acid as a function of concentration give basis of the calculation is suspect. The first figure shows that
 / ln(c /c ⦵)  40 N m 1 at 20 °C. Calculate the surface the effect is usually small; nevertheless it may have important
excess of 1-aminobutanoic acid and the number of molecules consequences.
per square metre. For instance, consider the formation of a cloud. Warm, moist
Collect your thoughts Equation 14C.14 relates the meas- air rises into the cooler regions higher in the atmosphere. At
ured value of  / ln(c /c ⦵) directly to the surface excess. some altitude the temperature drops to the point that the va-
Multiplication of the surface excess by Avogadro’s constant pour becomes thermodynamically unstable with respect to the
gives the number of molecules per square metre. liquid and it is then expected that the vapour will condense into
a cloud of liquid droplets. The initial step can be imagined as
The solution From eqn 14C.14 it follows that
a swarm of water molecules congregating into a microscopic
1    droplet. The initial droplet is very small and has an enhanced
S    
RT   ln(c /c ⦵ ) T vapour pressure; therefore, instead of growing it evaporates.
This effect stabilizes the vapour because an initial tendency
1 to condense is overcome by a heightened tendency to evapo-
  (4.0  105 N m 1 )
(8.3145 JK 1 mol 1 )  (293 K) rate. The vapour phase is then said to be supersaturated. It is
 1.6  108 mol m 2 thermodynamically unstable with respect to the liquid but not
unstable with respect to the small droplets that need to form
The number of molecules per square metre is NAΓS: before the bulk liquid phase can appear, so the formation of the
N A  S  (6.022  1023 mol 1 )  (1.6  108 mol m 2 ) latter by a simple, direct mechanism is hindered.
Two processes are responsible for overcoming this tendency
 9.6  1015 m 2
of droplets to evaporate and thus for allowing clouds to form.
Self-test 14C.2 Use the result obtained to calculate the area The first is that a sufficiently large number of molecules might
occupied by each molecule of 1-aminobutanoic acid at the congregate into a droplet so big that the enhanced evaporative
surface. effect is unimportant. The chance of one of these spontaneous
Answer: 1.0 × 102 nm2 nucleation centres forming is low, and in rain formation it is
not a dominant mechanism. The more important process de-
pends on the presence of minute dust particles or other kinds of
foreign matter. They nucleate the condensation (that is, provide
14C.4 Condensation centres at which it can occur) by providing surfaces to which
the water molecules can attach.
The concepts from this Topic together with some from Topic Liquids may be superheated above their boiling temperatures
4B can be used to explain aspects of the condensation of a gas and supercooled below their freezing temperatures. In each case
to a liquid. In Topic 4B it is shown that the vapour pressure of a the thermodynamically stable phase is not achieved on account
liquid, p, is increased when additional pressure ΔP is applied to of the kinetic stabilization that occurs in the absence of nuclea-
the liquid: according to eqn 4B.2, p  p  eVm (1) P /RT , where p* is tion centres. For example, superheating occurs because the va-
the vapour pressure when no additional pressure is applied and pour pressure inside a cavity is artificially low, so any cavity that
Vm(l) is the molar volume of the liquid. Because of its curved does form tends to collapse. This instability is encountered when
surface, a droplet experiences an additional pressure, given by an unstirred beaker of water is heated, for its temperature may
the Laplace equation (eqn 14C.7) as 2γ/r, where r is the radius be raised above its boiling point. Violent bumping often ensues
of the surface. When this value is used for ΔP in eqn 4B.2 the as spontaneous nucleation leads to bubbles big enough to sur-
result is the Kelvin equation for the vapour pressure of a liquid vive. To ensure smooth boiling at the true boiling temperature,
when it is dispersed as spherical droplets: nucleation centres, such as small pieces of sharp-edged glass or
bubbles (cavities) of air, should be introduced.
p  p  e2 Vm (1)/rRT Kelvin equation (14C.15)

For a cavity, the pressure of the liquid outside is less than the Exercise
pressure inside, so the sign of the exponent in eqn 14C.15 is
E14C.3 Calculate the vapour pressure of a spherical droplet of water of radius
changed to obtain an expression for the vapour pressure in a 10 nm at 20 °C. The vapour pressure of bulk water at this temperature is
cavity. 2.3 kPa and its mass density is 0.9982 g cm−3.
628 14 Molecular interactions

Checklist of concepts
☐ 1. The radial distribution function, g(r), is the radial den- ☐ 5. The surface pressure is the difference between the sur-
sity of molecules at a distance r from a given molecule. face tension of a pure solvent and a solution.
☐ 2. The radial distribution function may be calculated ☐ 6. The collapse pressure is the highest surface pressure
numerically by using Monte Carlo and molecular that a surface film can sustain.
dynamics techniques. ☐ 7. A surfactant is a species that accumulates at the inter-
☐ 3. Liquids tend to adopt shapes that minimize their surface face between phases and modifies the surface tension
area. and surface pressure.
☐ 4. Capillary action is the tendency of liquids to rise up ☐ 8. Nucleation provides surfaces to which molecules can
(and in some cases, drop down) narrow tubes. attach and thereby induce condensation.

Checklist of equations
Property Equation Comment Equation number
Hydrostatic pressure p = ρgacch Incompressible fluid 14C.4
Laplace equation pin = pout + 2γ/r γ is the surface tension 14C.7
Contact angle cos θc = wad/γlg − 1 14C.10b
Surface Gibbs energy G = G(α) + G(β) + G(σ) Definition 14C.11
Surface excess ΓJ = nJ(σ)/σ Definition 14C.12
Gibbs isotherm d   ΓJ dJ 14C.13
J

Dependence of the surface tension on surfactant concentration (∂γ/∂ln(c/c⦵))T = −RT ΓS 14C.14


Kelvin equation p p e 2 Vm (1) /rRT 14C.15
TOPIC 14D Macromolecules

➤ Why do you need to know this material?


(a)
Macromolecules give rise to special problems that include Monomer
the investigation and description of their molar masses
and shapes. You need to know how to describe the struc-
tural features of macromolecules in order to understand
their physical and chemical properties. (b)

➤ What is the key idea?


The structure of a macromolecule takes on different mean- (c)
ings at the different levels at which the arrangement of the Figure 14D.1 Three varieties of polymer: (a) a simple linear
chain or network of its building blocks is considered. polymer, (b) a cross-linked polymer, and (c) one variety of
copolymer.
➤ What do you need to know already?
The discussion of the shapes of macromolecules depends The number-average molar mass, M n , is obtained by weight-
on an understanding of the nonbonding interactions ing each molar mass by the number of molecules of that mass
between molecules (Topic 14B). You also need to be famil- present in the sample:
iar with the statistical interpretation of entropy (Topic 13E) 1 Number-average
and the concept of internal energy (Topic 2A). Some of the Mn 
N total
N M
i
i i molar mass
[definition]
 (14D.1a)
calculations draw on statistical arguments like those used
in the discussion of the Boltzmann distribution (Topic 13A). where Ni is the number of molecules with molar mass Mi and
Ntotal is the total number of molecules. This type of average is
typically obtained by mass spectroscopic determinations of
molar mass. The weight-average molar mass is the average cal-
culated by weighting the molar masses of the molecules by the
Macromolecules are very large molecules assembled from
mass present in the sample:
smaller molecules biosynthetically in organisms, by chemists
in the laboratory, or in an industrial reactor. Naturally occur- 1 Weight-average
ring macromolecules include polysaccharides such as cellulose,
Mw 
mtotal
m M
i
i i molar mass
[definition]
 (14D.1b)
poly­peptides such as protein enzymes, and polynucleotides
such as deoxyribonucleic acid (DNA). This Topic deals princi- In this expression, mi is the total mass of molecules with molar
pally with synthetic macromolecules. They include polymers, mass Mi and mtotal is the total mass of the sample. This type of
such as nylon and polystyrene, that are manufactured by string- average is typically obtained by measurements that make use
ing together, and in some cases crosslinking, smaller units of the ability of molecules to scatter light and by measurements
known as monomers (Fig. 14D.1). that make use of the distribution of particles in solutions ro-
tated at high speed in an ultracentrifuge.

Example 14D.1 Calculating number- and weight-average


14D.1 Average molar masses molar mass

A monodisperse polymer has a single, definite molar mass. A Evaluate the number-average and the weight-average molar
synthetic polymer, however, is polydisperse, in the sense that ­masses of a sample of poly(vinyl chloride) from the following data:
a sample is a mixture of molecules with various chain lengths
Interval 1 2 3 4 5 6
and molar masses. The various techniques that are used to
Mi/(kg mol−1) 7.5 12.5 17.5 22.5 27.5 32.5
measure molar masses result in different types of mean values
of polydisperse systems. mi/g 9.6 8.7 8.9 5.6 3.1 1.7
630 14 Molecular interactions

Collect your thoughts The relevant equations are eqns 14D.1a long-range order in the solid and therefore higher density and
and 14D.1b. The number average can be expressed in terms melting point. The spread of values is controlled by the choice
of the amounts (in moles) by using Ni = niNA, where ni is the of catalyst and reaction conditions.
amount (in moles) of molecules with molar mass Mi: The masses of macromolecules are often reported in dal-
1 1 1
tons (Da), where 1 Da = mu (with mu = 1.661 × 10−27 kg). Note
Mn 
N total
N M
i
i i  
ntotal N A i
ni N A Mi  ni Mi
ntotal i
that daltons are used to report molecular mass not molar mass.
For example a macromolecule with a reported molecular mass
where ntotal is the total amount of molecules. Calculate the of 100 kDa implies that its mass is 100 × 103 × mu. The molar
amounts in each interval by dividing the mass of the sample mass of the same macromolecule is NA × 100 × 103 × mu =
in each interval by the average molar mass for that interval; 100 kg mol−1. It is common practice to say that ‘the molar mass
ni = mi/Mi. Then calculate the two averages by weighting the is 100 kDa’, but this is incorrect usage: 100 kDa is the molecular
molar mass Mi within each interval by the amount (ni) and mass, not the molar mass.
mass (mi), respectively, of the molecules in each interval.
The solution The amounts in each interval are as follows: Exercises
Interval 1 2 3 4 5 6 E14D.1 Calculate the number-average molar mass and the weight-average
−1 molar mass of a mixture of equal amounts (that is, equal amounts in moles) of
Mi/(kg mol ) 7.5 12.5 17.5 22.5 27.5 32.5
two polymers, one having M = 62 kg mol−1 and the other M = 78 kg mol−1.
ni/mmol 1.3 0.70 0.51 0.25 0.11 0.052
E14D.2 Calculate the dispersity of the polymer described in Example E14D.1.
The total amount is ntotal = 2.92 mmol and the number-average Would you describe the polymer as ‘monodisperse’?
molar mass is
1
M n /(kg mol 1 )  (1.3  7.5  0.70  12.5  0.51  17.5
2.92 14D.2 The different levels of structure
0.25  22.5  0..11  27.5  0.052  32.5)
 13 The concept of the ‘structure’ of a macromolecule takes on dif-
The weight-average molar mass is calculated directly from the ferent meanings at the different levels of the arrangement of
data after noting that the total mass of the sample is 37.6 g: the chain or network of monomers. The primary structure
of a macromolecule is the sequence of small molecular resi-
1
M w /(kg mol 1 )  (9.6  7.5  8.7  12.5  8.9  17.5 dues making up the polymer. The residues may form either a
37.6
chain, as in polyethene, or a more complex network in which
5.6  22.5  3.1  27.5  1.7  32.5) cross-links connect different chains, as in cross-linked poly-
 16 acrylamide. In a synthetic polymer, virtually all the residues are
Comment. Note the different values of the two averages. In this identical and it is sufficient to name the monomer used in the
instance, M w /M n = 1.2 . synthesis. Thus, the repeating unit of polyethene and its deriva-
tives is –CHXCH2–, and the primary structure of the chain is
Self-test 14D.1 The Z-average molar mass, which is obtained specified by denoting it as –(CHXCH2)n–. The concept of pri-
in certain sedimentation experiments, is defined as mary structure ceases to be trivial in the case of synthetic co-
M Z  i N i Mi3 / i N i Mi2. Evaluate its value for the sample in polymers and biological macromolecules, because in general
this example. these substances are chains formed from different molecules.
Answer: 19 kg mol−1
For example, proteins are polypeptides
formed from different amino acids (about O
twenty occur naturally) strung together by
N
The ratio M w /M n is called the (molar-mass) dispersity, Ð the peptide link, –CONH– (1). The deg-
(previously the ‘polydispersity index’, PDI), read ‘d-stroke’ and radation of a biological macromolecule is H
defined as a disruption of its primary structure, when 1 Peptide link
the chain breaks into shorter components.
Mw
Ð= Dispersity [definition]  (14D.2) The term conformation refers to the spatial arrangement
Mn of the different parts of a chain, and one conformation can be
The term ‘monodisperse’ is conventionally applied to synthetic changed into another by rotating one part of a chain around a
polymers for which the dispersity is less than 1.1; commercial bond. The conformation of a macromolecule is relevant at three
polyethene samples might be much more heterogeneous, with levels of structure. The secondary structure of a macromole-
a dispersity close to 30. One feature of a narrow molar-mass cule is the (often local) spatial arrangement of a chain. The sec-
distribution for synthetic polymers is often a higher degree of ondary structure of a molecule of polyethene in some solvents
14D Macromolecules 631

may be a random coil (see below). In the absence of a solvent,


polyethene forms crystals consisting of stacked sheets with a
14D.3 Random coils
hairpin-like bend about every 100 monomer units, presumably The most likely conformation of a chain of identical units not
because for that number of monomers the intermolecular (in capable of forming hydrogen bonds or any other type of spe-
this case intramolecular) potential energy is sufficient to over- cific bond is a random coil. Polyethene is a simple example. The
come thermal disordering. The secondary structure of a pro- simplest model of a random coil is a ‘freely jointed chain’, in
tein is a highly organized arrangement determined largely by which any bond is free to make any angle with respect to the
hydrogen bonds, and taking the form of random coils, helices preceding one (Fig. 14D.4). It is also assumed that the mono-
(Fig. 14D.2a), or sheets in various segments of the molecule. mer units occupy zero volume, so different parts of the chain
The tertiary structure is the overall three-dimensional struc- can occupy the same region of space.
ture of a macromolecule. For instance, the hypothetical protein The model is obviously an oversimplification because a bond
shown in Fig. 14D.2b has helical regions connected by short is actually constrained to a cone of angles around a direction
random-coil sections. The helices interact to form a compact defined by its neighbour (Fig. 14D.5) and real chains are self-
tertiary structure. avoiding in the sense that distant parts of the same chain can-
The quaternary structure of a macromolecule is the man- not fold back and occupy the same space. In a hypothetical
ner in which large molecules are formed by the aggregation of one-dimensional freely jointed chain all the monomer units lie
others. Figure 14D.3 shows how four molecular subunits, each in a straight line, and the angle between neighbours is either 0°
with a specific tertiary structure, aggregate. Quaternary struc- or 180°. The units of a three-dimensional freely jointed chain
ture can be very important in biology. For example, the oxygen- are not restricted to lie in a line or a plane.
transport protein haemoglobin consists of four myoglobin-like
subunits that work cooperatively to take up and release O2.
14D.3(a) Measures of size
The size of a random coil is related to the probability that its
ends are a certain distance apart. That probability can be cal-
culated by considering a one-dimensional freely jointed chain.

Arbitrary
= angle

Arbitrary
angle
(a) (b)

Figure 14D.2 (a) A polymer may adopt a highly organized helical


conformation, an example of a secondary structure. The helix is
represented as a cylinder. (b) Several helical segments connected Figure 14D.4 In a freely jointed chain any bond is free to make
by short random coils pack together, an example of tertiary any angle with respect to the preceding one. Such a chain is like
structure. a three-dimensional random walk, each step being in an arbitrary
direction but of the same length.

Arbitrary
angle
θ θ
θ

Figure 14D.5 A more realistic description of a chain is obtained


Figure 14D.3 Several subunits with specific tertiary structures by fixing the bond angle (for example, at the tetrahedral angle)
pack together, an example of quaternary structure. and allowing free rotation about a bond direction.
632 14 Molecular interactions

1/ 2
How is that done? 14D.1 Calculating the probability  2 
ln P  ln    2 (N  n  1)ln(1   )
1

distribution in a one-dimensional freely jointed chain  N 


 21 (N  n  1)ln(1  )
Your goal is to calculate the probability, P, that the ends of a
long one-dimensional freely jointed chain composed of N units where   n/N . For a compact coil n << N and so   1; the
of length l (and therefore of total length Nl) are a distance nl approximation ln(1   )    12  2 is then appropriate and its
apart. Recall that in such a chain all the monomer units lie in use gives
a straight line, and the angle between neighbours is either 0° 1/ 2
or 180°. Therefore each monomer unit either lies in the same  2 
  2 N
2
ln P  ln  1

direction or the opposite direction to the previous unit.  N 


Step 1 Write expressions for the numbers of bonds pointing to the which rearranges into
left or right 1/2
2 2 (14D.3)
The conformation of a one-dimensional freely jointed chain P= e − n /2 N
πN Probability distribution
can be described by stating the number of bonds pointing to [1D freely jointed chain]
the right (NR) and the number pointing to the left (NL). The
distance between the ends of the chain is (NR − NL)l; it follows
that n = NR − NL. The total number of units is N = NR + NL, This function is plotted in Fig. 14D.6.
therefore, N R  12 (N  n) and N L  12 (N  n).
Step 2 Write an expression for the probability that a polymer has Brief illustration 14D.1
a specified end-to-end separation
The probability, P, that the end-to-end separation of a ran- Suppose that N = 1000 and l = 150 pm, then the probability that
domly selected polymer is nl is the ends of a one-dimensional random coil are nl = 3.00 nm
apart is given by eqn 14D.3 by setting n = (3.00 × 103 pm)/
number of conformations with end-to-end distance nl (150 pm) = 20.0:
P=
total number of possible conformations 1/ 2
 2  20.02 /(21000)
Each of the N monomer units of the polymer may in principle P   e  0.0207
   1000 
lie to the left or the right, so the total number of possible con-
formations is 2N. The total number of ways, W, of forming a meaning that there is a 1 in 48 chance of the ends being
chain of N units with the end-to-end distance nl is the number 3.00 nm apart.
of ways of having NR right-pointing units, the rest being left-
pointing units. Therefore, to calculate W, count the number
of ways of achieving NR right-pointing units given a total of N Equation 14D.3 can be adapted to calculate the probability
units. This is the same problem as selecting NR objects from a that the ends of a long three-dimensional freely jointed chain
collection of N objects (see Topic 13A), and is lie in the range r to r + dr. The probability is written as f(r)dr,
where
N! N! N!
W   1
N R !(N  N R )! N R ! N L !  2 (N  n)! 12 (N  n)!
1

It follows that
0.8
Probability, P/(2/N)1/2

W N!
P N  1
2  2 (N  n)! 12 (N  n)!2N 0.6

Step 3 Consider the case of compact chains 0.4

When the chain is compact in the sense that n << N, it is more


convenient to evaluate ln P: the factorials are then large and it is 0.2

possible to use Stirling’s approximation (Topic 13A). Although


the approximation used there is ln x! = x ln x − x, here it is 0
–4 –2 0 2 4
appropriate to use the more precise form n/N 1/2

ln x !  ln(2)1/ 2   x  12  ln x  x Figure 14D.6 The probability distribution for the separation of the
ends of a one-dimensional freely jointed chain. The separation of
The result, after quite a lot of algebra, is the ends is nl, where l is the length of each monomer unit.
14D Macromolecules 633

Root mean square separation, Rrms/l


 a  2 2
f (r )  4   1/ 2  r 2 e  a r
  Probability distrribution

60
(14D.4)
1/ 2 [3D freely jointed chain]
 3 
a  2 
 2Nl  40

Here and elsewhere the fact that the chain cannot be longer than
Nl is ignored. Although eqn 14D.4 gives a non-zero probability 20
for r > Nl, the values are so small that the errors in pretending
that r can range up to infinity are negligible. For a narrow range
0
of distances δr, the probability density can be treated as a con- 0 1000 2000 3000 4000
stant and the probability calculated from f(r)δr. An alternative Number of monomers, N
interpretation of this expression is to regard each molecule in
Figure 14D.7 The variation of the root-mean-square separation
a sample as ceaselessly writhing from one conformation to an-
of the ends of a three-dimensional random coil, Rrms, with the
other; then f(r)dr is the probability that at any instant the chain number of monomers.
will be found with the separation of its ends between r and r + dr.
l2
N N N
Brief illustration 14D.2
〈 R 2 〉 = 〈 R.R 〉 = ∑ 〈ri .rj 〉 = ∑ 〈r i2 〉 + ∑ 〈ri .rj 〉
i , j =1 i =1 i ≠j
Consider the chain described in Brief illustration 14D.1, with
N = 1000 and l = 150 pm but now in three dimensions. Then When N is large (as assumed throughout), the term in the box
1/ 2 is zero because the individual vectors lie in random directions.
 3 
a  2.58 104 pm 1 The remaining term is Nl2, hence  R 2   Nl 2. It follows that for a
 2  1000  (150 pm)2 
  random coil of any dimensionality
Then the probability density at r = 3.00 nm is given by eqn Root-mean-square
Rrms = N 1/ 2 l  (14D.6)
14D.4 as separation [random coil]
3
 2.58 10 4 pm 1  As the number of monomer units increases, the root-
f (3.00 nm)  4     mean-square separation of the ends of the polymer increases
 1/ 2 
as N1/2 (Fig. 14D.7), and consequently the volume of a three-­
 ( 2.58104 pm 1 )2 ( 3.00103 pm )2
(3.00  103 pm)2  e dimensional coil increases as N3/2. The result must be multiplied
 1.92  104 pm 1 by a factor when the chain is not freely jointed (see below).
Another convenient measure of size is the radius of ­gyration,
The probability that the ends will be found in a narrow range of Rg, which in mechanics is the radius at which a single mass
width δr = 10.0 pm at 3.00 nm (regardless of direction) is therefore point should be located so that it has the same mass and mo-
ment of inertia (and therefore rotational characteristics) as the
f (3.00 nm)r  (1.92  104 pm 1 )  (10.0 pm)  1.92  103
actual molecule. In polymer chemistry, Rg is the mass weighted
or about 1 in 520. mean square distance from the centre of the molecule. Here, we
use the polymer definition. A one-dimensional freely jointed
chain can be used to illustrate the procedure for its calculation.
There are several measures of the geometrical size of a ran-
dom coil. The contour length, Rc, is the length of the polymer
(not only a random coil) measured along its backbone from How is that done? 14D.2 Deriving an expression for the
atom to atom. For a polymer of N monomer units each of radius of gyration
length l, the contour length is
You need to set up an expression for the moment of inertia of a
one-dimensional freely jointed chain of N monomer units each
Rc = Nl Contour length (14D.5) of mass m and then equate it to mtotal Rg2, where mtotal is the total
mass of the polymer molecule, mtotal = Nm.
The root-mean-square separation, Rrms, is the square root of the
mean value of the square of the separation of the ends of the Step 1 Set up the expression for the moment of inertia
coil. Thus, if the vector joining the ends of the coil is R, and each For a one-dimensional freely jointed chain with N identical
monomer is represented by the vector ri, then R  iN1ri and monomers each of mass m, the moment of inertia around the
634 14 Molecular interactions

centre of the chain (which is at the origin of the vector repre- visualize it as a freely jointed chain with N and l as the number
senting the first monomer, because the vectors point in equal and length, respectively, of these rigid units. With the length
numbers to left and right) is l = 45 nm and N = 200 (and using 103 nm = 1 μm),
N N
I  midi2  mdi2 the contour length (eqn 14D.5), Rc  200  45 nm  9.0 m
i 1 i 1
the RMS separation (eqn 14D.6), Rrms  (200)1/ 2  45 nm  0.64 m
where di is the distance of mass mi from the origin. This dis- 1/ 2
 200 
tance is the length of the vector di, the sum of i steps from the the radius of gyration (eqn 14D.7 b), Rg     45 nm  0.26 m
 6 
origin, di  ij 1rj.
Step 2 Evaluate the average distance of a monomer from the origin
The random coil model ignores the role of the solvent: a poor
As in the calculation that led to eqn 14D.6, write
solvent tends to cause the coil to tighten so that solute–­solvent
il 2 contacts are minimized; a good solvent does the opposite.
l2 Therefore, calculations based on this model are better regarded
i i i
as lower bounds to the dimensions for a polymer in a good sol-
〈d 2i 〉 = 〈di ⋅di 〉 = ∑ 〈rj ⋅rk 〉 = ∑ 〈rj2 〉 + ∑ 〈r .r 〉
j ,k=1 j =1 j ≠k
j k
vent and as an upper bound for a polymer in a poor solvent.
The model is most reliable for a polymer in a bulk solid sample,
Again, the boxed term is zero for a freely jointed chain, so where the coil is likely to have its natural dimensions.
di2   il 2 and the average moment of inertia of the coil is

1 + 2 + … N= 21 N(N+ 1) 1
2 N2
14D.3(b) Constrained chains
N N The freely jointed chain model is improved by removing the
〈 I 〉 = m∑ 〈di2 〉 = ml 2 ∑ i = 12 N 2ml 2 = 12 Nmtotall 2 freedom of bond angles to take any value. For long chains, it is
i =1 i =1
convenient to take groups of neighbouring bonds and consider
Step 3 Identify the radius of gyration the direction of their resultant. Although each successive indi-
Finally, set this moment of inertia equal to mtotal Rg2, which vidual bond is constrained to a single cone of angle θ relative
implies that Rg2 = Nl 2 and therefore that to its neighbour, the resultant of several bonds lies in a random
direction. By concentrating on such groups rather than indi-
viduals, it turns out that for long chains the expressions for the
1/ 2
N (14D.7a) root-mean-square separation and the radius of gyration given
Rg = l
2 Radius of gyration above should be multiplied by
[1D freely jointed chain]
1/ 2
 1  cos   (14D.8)
A similar calculation for a three-dimensional freely jointed F  
chain gives  1  cos 

1/ 2 For a tetrahedral arrangement of bonds, for which cos   13


N  Radius of gyration
 (14D.7b) (i.e. θ = 109.5°), F = 21/2. Therefore:
Rg    l [3D freely jointed chain]
6 1/ 2 Dimensions of
N 
The radius of gyration is smaller in this case because the extra Rrms  (2N )1/ 2 l Rg    l a constrained  (14D.9)
dimensions enable the coil to be more compact. 3 tetrah
hedral chain
The radius of gyration may also be calculated for other geom- The jointed chain model is still an approximation, even after
etries. For example, according to the polymer definition, a solid the bond angles have been restricted, because it does not take
uniform sphere of radius R has Rg = (3/5)1/ 2 R, and a long thin into account the impossibility of two or more atoms occupying
uniform rod of length L has Rg = L/121/2 for rotation about an the same place. Such self-avoidance tends to swell the coil, so
axis perpendicular to the long axis. A solid sphere with the same (in the absence of solvent effects) it is better to regard Rrms and
radius and mass as a random coil has a greater radius of gyration Rg as lower bounds to the actual values.
as it is entirely dense throughout.

14D.3(c) Partly rigid coils


Brief illustration 14D.3
An important measure of the flexibility of a chain is the persis-
Consider a polymer that writhes as if it were a three-­dimensional tence length, lp, a measure of the length over which the direc-
random coil. However, suppose that small segments of the tion of the first monomer–monomer direction is sustained. If
macromolecule resist bending, so it is more appropriate to the chain is a rigid rod, then the persistence length is the same
14D Macromolecules 635

as the contour length. For a freely jointed chain, the persistence E14D.4 What is the probability that the ends of a polyethene chain of molar
length is just the length of one monomer unit. Therefore, the mass 65 kg mol−1 are 10 nm apart when the polymer is treated as a one-
dimensional freely jointed chain? Take the mass of the monomer unit to be
persistence length can be regarded as a measure of the stiffness 28.0516 g mol−1 and the C–C bond length to be 154 pm.
of the chain.
E14D.5 The radius of gyration of a long one-dimensional chain molecule is
The mean square distance between the ends of a chain that found to be 7.3 nm. The chain consists of C–C links. Assume that the chain is
has a persistence length greater than the monomer length can randomly coiled and estimate the number of links in the chain. Take the C–C
be expected to be greater than for a freely jointed chain because bond length to be 154 pm.
the partial rigidity of the coil does not let it roll up so tightly. A E14D.6 By what percentage does the radius of gyration of a one-dimensional
detailed calculation shows that polymer chain increase or decrease when the bond angle between units is
limited to 109°? What is the percentage change in volume of the coil?
1/ 2
 2lp  E14D.7 The radius of gyration of a three-dimensional partially rigid polymer
Rrms  N 1/ 2 lF where F    1   (14D.10) of 1000 units each of length 150 pm was measured as 2.1 nm. What is the
 l  persistence length of the polymer?

For a freely jointed chain, lp = l, so Rrms = N1/2l, as already found.


For lp > l, F > 1, so the coil has swollen, as anticipated.
14D.4 Mechanical properties
Example 14D.2  Calculating the root-mean-square
Insight into the consequences of stretching and contracting a
separation of a partly rigid coil polymer can be obtained on the basis of the freely jointed chain
By what percentage does the root-mean-square separation of as a model.
the ends of a polymer chain with N = 1000 increase or decrease
when the persistence length changes from l (the length of one 14D.4(a) Conformational entropy
monomer unit) to 2.5 per cent of the contour length?
A random coil is the least structured conformation of a poly-
Collect your thoughts The contour length is Rc = Nl. When mer chain and therefore corresponds to the state of greatest en-
lp = l, the chain is freely jointed and Rrms,random coil = N1/2l, so tropy. Any stretching of the coil reduces disorder and reduces
eqn 14D.10 can be expressed as Rrms = FRrms,random coil. The frac-
the entropy. Conversely, the formation of a random coil from
tional change in root-mean-square separation is therefore
a more extended form is spontaneous (provided enthalpy con-
Rrms  Rrms,random coil  2lp
1/ 2
 tributions do not interfere). The same model can be used to de-
Rrms
 1    1  1 duce an expression for the change in conformational entropy,
Rrms,random coil Rrms,random coil  l  the statistical entropy arising from the arrangement of bonds,
In the final step you should express this fractional change as a when a one-dimensional chain is stretched or compressed.
percentage.
The solution Because lp = 0.025Rc = 0.025Nl, the fractional
change is How is that done? 14D.3 Deriving an expression for the
1/ 2 conformational entropy of a freely jointed chain
Rrms  Rrms,random coil  2  0.025Nl 
  1  1
Rrms,random coil  l  Consider a freely jointed one-dimensional chain containing
N units of length l that is stretched or compressed through
 (0.050N  1)1/ 2  1 a distance x. You then need to use the Boltzmann formula
With N = 1000, the fractional change is 6.00, so the root-mean- (eqn 13E.7, S = k ln W ) to calculate the conformational entropy
square separation increases by 600 per cent. of the chain, which involves assessing the value of W, the num-
ber of ways of achieving a particular conformation.
Self-test 14D.2 Calculate the fractional change in the volume
of the same three-dimensional coil. Step 1 Calculate W
Answer: 340 Recall the discussion in How is that done? 14D.1 in which the
chain is described as having NR bonds pointing to the right and
NL pointing to the left. To achieve an extension of the chain NR
must increase and, because NR + NL = N, it follows that NL will
Exercises decrease. The extension is expressed in terms of the parameter
λ, defined according to NR − NL = λN. The range of λ is from −1
E14D.3 A one-dimensional polymer chain consists of 700 segments, each
0.90 nm long. If the chain were ideally flexible, what would be the root-mean-
(all bonds to the left) to +1 (all bonds to the right); extending
square separation of the ends of the chain? the chain corresponds to increasing λ.
636 14 Molecular interactions

From NR + NL = N and NR − NL = λN it follows that 0


N R  12 (1   )N and N L  12 (1   )N . The number of ways of

Change of entropy, ΔS/Nk


achieving NR bonds to the right and NL to the left is computed
–0.1
in the same was as before:

N! N!
W   1 –0.2
N R ! N L !  2 (1   )N ! 12 (1   )N !

Step 2 Write an expression for S –0.3


It follows from the expression for W and the Boltzmann for-
mula, S = k lnW, that
–0.4
–0.8 –0.4 0 0.4 0.8
S /k  ln N ! ln  12 (1   )N ! ln  12 (1   )N ! Extension, λ = x/Nl (=x/Rc)

Because the factorials are large (except for large exten- Figure 14D.8 The change in entropy of a one-dimensional freely
sions), use Stirling’s approximation in the form ln x ! ≈ jointed chain as its extension changes; λ = 0 corresponds to the
 x  12  ln x  x  12 ln(2) to obtain conformation of highest entropy and as the chain is extended
(increasing λ) or compressed (decreasing λ) from this point the
S /k   ln(2)1/ 2  (N  1)ln 2 entropy decreases.
  N  12  ln N  12 ln{N 2 N  2 (1   )N (1  ) 1 (1   )N (1  ) 1 }

Step 3 Write an expression for the change in entropy 14D.4(b) Elastomers


The conformation with the greatest W, and hence the high- An elastomer is a flexible polymer that can expand or contract
est entropy, has equal numbers of the right- and left-pointing easily upon application of an external force. Elastomers are
bonds. This corresponds to λ = 0 and so, from the previous ­polymers with numerous crosslinks that pull them back into
expression, the entropy is their original shape when a stress is removed. The weak direc-
S0 /k   ln(2)1/ 2  (N  1)ln 2  12 ln N
tional constraints on silicon–oxygen bonds are responsible for
the high elasticity of silicones. Even a freely jointed chain be-
The extension x of the chain from this highest entropy confor- haves as an elastomer for small extensions. It is a model of a
mation is expressed in terms of λ by noting that x = (NR − NL)l = ‘perfect elastomer’, a polymer in which the internal energy is
λNl, and then introducing the contour length Rc = Nl to give independent of the extension, and the model can be used to de-
x = λRc. The change in entropy when the chain is stretched duce the restoring force associated with stretching or compres-
(or compressed) by the distance λRc is therefore the difference sion of the chain.
between S0 and the entropy Sλ from Step 2. The resulting expres-
sion, after some algebraic manipulation and using N >> 1, is
How is that done? 14D.4 Deriving an expression for the
1+ λ 1− λ (14D.11)
ΔS = − kN ln{(1+ λ ) (1− λ ) } with λ = x / Rc
1
2 restoring force of a perfect elastomer modelled as a ­one-
Conformational entropy change dimensional freely jointed chain
[1D freely jointed chain]
Your goal is to find an expression for the restoring force, F, of
This function is plotted in Fig. 14D.8, and it is seen that mini- a one-dimensional freely jointed chain composed of N units
mum extension corresponds to maximum entropy. each of length l, when the chain is stretched or compressed by
a distance x = νl.

Brief illustration 14D.4 Step 1 Use thermodynamics to relate the restoring force to the
entropy
Suppose that N = 1000 and l = 150 pm, so Rc = 150 nm. The The work done on an elastomer when it is extended reversibly
change in entropy when the one-dimensional freely jointed chain through a distance dx is Fdx. The change in internal energy,
is stretched through 1.5 nm (corresponding to λ = 1/100) is from dU = dwrev + dqrev, with dqrev = TdS, is therefore dU = Fdx +
TdS. It follows that for an isothermal extension
1 1/100 
 1   1 
1 (1/100 )

S   12 k  (1000)  ln  1    1    U   S 
 100   100    x   F  T  x 
 0.050k  T  T

Because R = NAk, the change in molar entropy is ∆Sm = In a perfect elastomer, as in a perfect gas, the internal energy
−0.050R, or −0.42 J K−1 mol−1. is independent of the dimensions (at constant temperature), so
(∂U/∂x)T = 0. The restoring force is therefore
14D Macromolecules 637

 S  where x = λNl is used. This expression shows that, for small


F  T  
 x T displacements, the sample obeys Hooke’s law (Fig. 14D.9): the
restoring force is proportional to the displacement and the
Step 2 Evaluate the force from the change in conformational force constant kf (the constant of proportionality between
­entropy the force and the displacement) is
The conformational entropy (eqn 14D.11) is expressed in terms
of the parameter λ used to express the extension x as x = λNl (or kT
kf =  (14D.12c)
x = λRc). Therefore, replace the derivative with respect to x by Nl 2
the derivative with respect to λ by noting that dx = Nldλ. Then

T  S  T  S  Brief illustration 14D.5


F   
Nl   T Nl   T
Consider a polymer chain with N = 5000 and l = 0.15 nm. If
The replacement of S by the change ΔS is valid because, as the ends of the chain are moved apart by x = 1.5 nm, then
computed in eqn 14D.11, ΔS = Sλ − S0, where S0 is the entropy λ = (1.5 nm)/(5000 × 0.15 nm) = 2.0 × 10−3. Because λ << 1, the
for λ = 0 and hence is independent of the extension. Now use restoring force at 293 K is given by eqn 14D.12b as
eqn 14D.11 to obtain Nm

T Nk d (1.381  10 J K 1 )  (293 K )
23

F   ln{(1   )1  (1   )1  } F  (1.5  109 m)


Nl 2 d 5000  (1.5  1010 m)2
kT d  5.4  1014 N
 {(1   )ln(1   )  (1   )ln(1   )}
2l d
or 54 fN.
kT
 {ln(1   )  ln(1   )}
2l
That is,
Exercises
kT 1+ λ x (14D.12a)
F= ln λ= E14D.8 Calculate the change in molar entropy when the ends of a one-
2l 1− λ Nl Restoring force
[1D freely jointed chain] dimensional polyethene chain of molar mass 65 kg mol−1 are moved apart by
1.0 nm. Take the mass of the monomer unit to be 28.0516 g mol−1 and the C–C
bond length to be 154 pm.
For small displacements (λ << 1, corresponding to x << Nl
E14D.9 Calculate the restoring force when the ends of a one-dimensional
and therefore x << Rc) the logarithms can be expanded by using polyethene chain of molar mass 65 kg mol−1 are moved apart by 1.0 nm at
ln(1 + λ) ≈ λ and ln(1 − λ) ≈ −λ, to give 20 °C. Take the mass of the monomer unit to be 28.0516 g mol−1 and the C–C
bond length to be 154 pm.
 kT kT Restoring force
F  2x [1D freely jointed chain]
 (14D.12b)
l Nl

14D.5 Thermal properties


6

The crystallinity of synthetic polymers can be destroyed by


4
Restoring force, F/(kT/2l)

thermal motion at sufficiently high temperatures. This change


2 in crystallinity may be thought of as a kind of intramolecular
melting from a crystalline solid to a more fluid random coil.
0 Polymer melting also occurs at a specific melting temperature,
Tm, which increases with the strength and number of intermo-
–2
Hooke’s lecular interactions in the material. Thus, polyethene, which
–4 law has chains that interact only weakly in the solid, has Tm = 414 K
and nylon-66 fibres, in which there are strong hydrogen bonds
–6 between chains, have Tm = 530 K. High melting temperatures
–1 –0.5 0 0.5 1
Extension, λ = x/Nl are desirable in most practical applications involving fibres and
Figure 14D.9 The restoring force, F, of a one-dimensional plastics.
freely jointed chain (a model for a perfect elastomer). For small All synthetic polymers undergo a transition from a state of
extensions, F is proportional to the extension, corresponding to high to low chain mobility at the glass transition temperature,
Hooke’s law. Tg. To visualize the glass transition, consider what happens to
638 14 Molecular interactions

an elastomer as its temperature is lowered. There is sufficient


energy available at normal temperatures for limited bond rota-
tion to occur and the flexible chains writhe. At lower tempera-

Specific volume, Vs
tures, the amplitudes of the writhing motion decrease until a
specific temperature, Tg, is reached at which motion is frozen
completely and the sample forms a glass. Glass transition tem-
peratures well below 300 K are desirable in elastomers that are
to be used at normal temperatures.
Both the glass transition temperature and the melting Tg
temperature of a polymer may be measured by calorimetric
methods. Because the motion of the segments of a polymer
Temperature, T
chain increase at the glass transition temperature, Tg may also
be determined from a plot of the specific volume of a poly- Figure 14D.10 The variation of specific volume with temperature
mer (the reciprocal of its mass density) against temperature of a synthetic polymer. The glass transition temperature, Tg, is at
(Fig. 14D.10). the point of intersection of extrapolations of the two linear parts
of the curve.

Checklist of concepts
☐ 1. Macromolecules are very large molecules assembled ☐ 6. The tertiary structure is the overall three-dimensional
from smaller molecules. structure of a macromolecule.
☐ 2. Synthetic polymers are manufactured by stringing ☐ 7. The quaternary structure is the manner in which large
together and in some cases cross-linking smaller units molecules are formed by the aggregation of others.
known as monomers. ☐ 8. In a freely jointed chain any bond in a polymer is free
☐ 3. Macromolecules can be monodisperse, with a single to make any angle with respect to the preceding one.
molar mass, or polydisperse, with a spread of molar ☐ 9. The least structured conformation of a macromolecule
mass. is a random coil, which can be modelled as a freely
☐ 4. The conformation of a macromolecule is the spatial jointed chain.
arrangement of the different parts of a chain. ☐ 10. An elastomer is a flexible polymer that can expand or
☐ 5. The primary structure of a macromolecule is the contract easily upon application of an external force.
sequence of small molecular residues making up the ☐ 11. The disruption of long-range order in a polymer occurs
polymer. at a melting temperature.
☐ 5. The secondary structure is the spatial arrangement of a ☐ 12. Synthetic polymers undergo a transition from a state
chain of residues. of high to low chain mobility at the glass transition
­temperature.
14D Macromolecules 639

Checklist of equations
Property Equation Comment Equation number
1
Number-average molar mass Mn 
N total N M
i
i i Definition 14D.1a

1
Weight-average molar mass Mw 
mtotal m M
i
i i Definition 14D.1b

Dispersity Ð = M w /M n Definition 14D.2


1/ 2  n2 / 2 N
Probability distribution P  (2/N ) e One-dimensional freely 14D.3
jointed chain
2 2
f (r )  4 (a/1/ 2 )3 r 2 e  a r Three-dimensional freely 14D.4
a  (3/2Nl ) 2 1/ 2 jointed chain

Contour length of a random coil Rc = Nl 14D.5


1/2
Root-mean-square separation of a random coil Rrms = N l Unconstrained chain 14D.6
Radius of gyration of a random coil Rg = (N/2)1/2l Unconstrained one- 14D.7a
dimensional chain
Rg = (N/6)1/2l Unconstrained three- 14D.7b
dimensional chain
Root-mean-square separation of a random coil Rrms = (2N)1/2l Constrained tetrahedral chain 14D.9
1  1
Change in conformational entropy on extending a one-dimensional S   kN ln{(1   ) (1   ) }
1
2 14D.11
freely jointed chain
Restoring force of a one-dimensional freely jointed chain l F = (kT/2l) ln{(1 + λ)/(1 − λ)} 14D.12a
F ≈ (kT/Nl2)x x << Rc 14D.12b
TOPIC 14E Self-assembly

• A sol is a dispersion of a solid in a liquid (such as clusters


➤ Why do you need to know this material? of gold atoms in water) or of a solid in a solid (such as
Aggregates of small and large molecules form the basis of ruby glass, which is a gold-in-glass sol, and achieves its
many established and emerging technologies. To see why colour by light scattering).
this is the case, you need to understand their structures • An aerosol is a dispersion of a liquid in a gas (like fog and
and properties. many sprays) or a solid in a gas (such as smoke): the par-
ticles are often large enough to be seen with a microscope.
➤ What is the key idea? • An emulsion is a dispersion of a liquid in a liquid (such as
Colloids and micelles form spontaneously by self-assembly milk).
of molecules or macromolecules and are held together by
• A foam is a dispersion of a gas in a liquid.
molecular interactions.
A further classification of colloids is as lyophilic, or solvent
➤ What do you need to know already? attracting, and lyophobic, solvent repelling. If the solvent is
You need to be familiar with molecular interactions (Topic water, the terms hydrophilic and hydrophobic, respectively,
14B) and interactions between ions (Topic 5E). are used instead. Lyophobic colloids include the metal sols.
Lyophilic colloids generally have some chemical similarity to
the solvent, such as –OH groups able to form hydrogen bonds.
A gel is a semi-rigid mass of a lyophilic sol.
Self-assembly is the spontaneous formation of complex struc- The preparation of aerosols can be as simple as sneezing
tures of molecules or macromolecules that are held together by (which produces an imperfect aerosol). Laboratory and com-
molecular interactions, such as Coulombic, dispersion, hydro- mercial methods make use of several techniques. Material
gen bonding, and hydrophobic interactions. Examples of self- (e.g. quartz) may be ground in the presence of the dispersion
assembly include the formation of liquid crystals and of protein medium. Passing a heavy electric current through a cell may
quaternary structures from two or more polypeptide chains lead to the sputtering (crumbling) of an electrode into colloi-
(Topic 14C). dal particles. Arcing between electrodes immersed in the sup-
port medium also produces a colloid. Chemical precipitation
sometimes results in a colloid. A precipitate (e.g. silver iodide)
already formed may be dispersed by the addition of a ‘peptizing
14E.1 Colloids agent’ (e.g. potassium iodide). Clays may be peptized by alkalis,
the OH− ion being the active agent.
A colloid, or disperse phase, is a dispersion of small particles Emulsions are normally prepared by shaking the two com-
of one material in another that does not settle out under grav- ponents together vigorously, although some kind of emulsify-
ity. In this context, ‘small’ means that one dimension at least ing agent usually has to be added to stabilize the product. This
is smaller than about 500 nm (about the wavelength of visible emulsifying agent may be a soap (the salt of a long-chain car-
light). Many colloids are suspensions of nanoparticles (parti- boxylic acid) or other surfactant (surface active) species, or a
cles of diameter up to about 100 nm). In general, colloidal par- lyophilic sol that forms a protective film around the dispersed
ticles are aggregates of numerous atoms or molecules, but are phase. In milk, which is an emulsion of fats in water, the emul-
commonly but not universally too small to be seen with an or- sifying agent is casein, a protein containing phosphate groups.
dinary optical microscope. It is clear from the formation of cream on the surface of milk
that casein is not completely successful in stabilizing milk: the
dispersed fats coalesce into oily droplets which float to the sur-
14E.1(a) Classification and preparation face. This coagulation may be prevented by ensuring that the
The name given to the colloid depends on the two phases emulsion is dispersed very finely initially: intense agitation
­involved: with ultrasonics brings this dispersion about, the product being
‘homogenized’ milk.
14E Self-assembly 641

One way to form an aerosol is to tear apart a spray of liquid


with a jet of gas. The dispersal is aided if a charge is applied to
the liquid, because then electrostatic repulsions help to blast it
apart into droplets. This procedure may also be used to produce
emulsions by directing the charged liquid phase into another
liquid.
Colloids are often purified by dialysis, the process of squeez-
ing the solution though a membrane. The aim is to remove
much (but not all, for reasons explained later) of the ionic ma-
R
terial that may have accompanied their formation. A mem-
brane (for example, cellulose) is selected that is permeable to
Figure 14E.1 Although the energy of attraction between
solvent and ions, but not to the colloid particles. Dialysis is very
individual molecules is proportional to 1/R6, more molecules
slow, and is normally accelerated by applying an electric field
are within range at large separations (pale region) than at small
and making use of the charges carried by many colloidal parti- separation (dark region), so the total interaction energy declines
cles; the technique is then called electrodialysis. more slowly and is proportional to a lower power of 1/R.

14E.1(b) Structure and stability hydrophilic groups in synthetic detergents) surround the sur-
Colloids are thermodynamically unstable with respect to the face, form hydrogen bonds with water, and give rise to a shell
bulk. This instability can be expressed thermodynamically by of negative charge that repels a possible approach from another
noting that the change in Helmholtz energy, dA, when the similarly charged particle.
surface area of the sample changes by dσ at constant tem-
perature and pressure is dA = γdσ, where γ is the interfacial
surface tension (Topic 14C). It follows that if the surface con-
14E.1(c) The electrical double layer
tracts, dσ < 0, the Helmholtz energy will decrease indicating A major source of kinetic non-lability of colloids is the exist-
that the contraction is a spontaneous process. The survival of ence of an electric charge on the surfaces of the particles. Ions
colloids must therefore be a consequence of the kinetics of of opposite charge tend to cluster near each other, and form an
collapse: colloids are thermodynamically unstable but kineti- ionic atmosphere around the particles, just as for individual
cally non-labile. ions (Topic 5F).
At first sight, even the kinetic argument seems to fail: colloi- There are two regions of charge. First, there is a fairly im-
dal particles attract each other over large distances, so there is a mobile layer of ions that adhere tightly to the surface of the col-
long-range force that tends to condense them into a single blob. loidal particle, and which may include water molecules (if that
The reasoning behind this remark is as follows. The energy of is the support medium). The radius of the sphere that captures
attraction between two individual atoms i and j separated by this rigid layer is called the radius of shear and is the major
a distance Rij, one in each colloidal particle, varies with their factor determining the mobility of the particles. The electric
separation as 1/Rij6 (Topic 14B). The sum of all these pairwise potential at the radius of shear relative to its value in the dis-
interactions, however, decreases only as approximately 1/R2 tant, bulk medium is called the electrokinetic potential, ζ (or
(the precise variation depending on the shape of the particles the zeta potential). Second, the colloidal particle with its layer
and their closeness), where R is the separation of the centres of of immobile ions attracts an oppositely charged atmosphere of
the particles. The change in the power from 6 to 2 stems from mobile ions. The inner shell of charge and the outer ionic at-
the fact that at short distances only a few molecules interact but mosphere constitute the electrical double layer.
at large distances many individual molecules are at about the The theory of the stability of lyophobic dispersions was de-
same distance from one another, and contribute equally to the veloped by B. Derjaguin and L. Landau and independently by
sum (Fig. 14E.1), so the total interaction does not fall off as fast E. Verwey and J.T.G. Overbeek, and is known as the DLVO
as the single molecule–molecule interaction. theory. It assumes that there is a balance between the repulsive
Several factors oppose the long-range dispersion attraction. interaction between the charges of the electrical double layers
For example, there may be a protective film at the surface of on neighbouring particles and the attractive interactions aris-
the colloid particles that stabilizes the interface and cannot be ing from van der Waals interactions between the molecules in
penetrated when two particles touch. Thus, the surface atoms the colloidal particles. The potential energy arising from the
of a platinum sol in water react chemically and are turned into repulsion of double layers on particles of radius a has the form
–Pt(OH)3H3; this layer encases the particle like a shell. A fat can
be emulsified by a soap because the long hydrocarbon tails pen- Aa 2 2  s /rD
Vrepulsion   e
etrate the oil droplet but the carboxylate head groups (or other R
642 14 Molecular interactions

where A is a constant, ζ is the zeta potential, R is the separation from the stabilizing effect of this secondary minimum is called
of centres, s is the separation of the surfaces of the two parti- flocculation. The flocculated material can often be redispersed
cles (s = R − 2a for spherical particles of radius a), and rD is the by agitation because the well is so shallow. Coagulation, the ir-
thickness of the double layer. This expression is valid for small reversible aggregation of distinct particles into large particles,
particles with a thick double layer (rD >> a). When the double occurs when the separation of the particles is so small that they
layer is thin (rD << a), the expression is replaced by enter the primary minimum of the potential energy curve and
van der Waals forces are dominant.
Vrepulsion   12 Ba 2 2 ln(1  e  s /rD ) The ionic strength is increased by the addition of ions, par-
ticularly those of high charge type, so such ions act as floc-
where B is another constant. In each case, the thickness of the culating agents. This increase is the basis of the empirical
double layer can be estimated from an expression like that de- Schulze–Hardy rule, that hydrophobic colloids are flocculated
rived for the thickness of the ionic atmosphere in the Debye– most efficiently by ions of opposite charge type and high charge
Hückel theory (Topic 5F and A deeper look 5F.1, available to number. The Al3+ ions in alum (KAl(SO4)2) are very effective,
read in the e-book accompanying this text) in which there is a and are used to induce the congealing of blood. When river
competition between the assembling influences of the attrac- water containing colloidal clay flows into the sea, the salt water
tion between opposite charges and the disruptive effect of ther- induces flocculation and coagulation, and is a major cause of
mal motion: silting in estuaries.
Metal oxide sols tend to be positively charged whereas sulfur
1/ 2
  RT  Thickness of the electrical and the noble metals tend to be negatively charged. Naturally
rD    
 (14E.3)
double laayer occurring macromolecules also acquire a charge when dis-
 2  F Ib 
2 ⦵

persed in water, and an important feature of proteins and other


where I is the ionic strength of the solution (eqn 5F.28, natural macromolecules is that their overall charge depends on
I  12 i zi2bi /b with b  1 mol kg 1) and ρ its mass density. As
⦵ ⦵
the pH of the medium. For instance, in acidic environments
usual, F is Faraday’s constant and ε is the permittivity, ε = εrε0. protons attach to basic groups, and the net charge of the mac-
The potential energy arising from the attractive interaction has romolecule is positive; in basic media the net charge is negative
the form as a result of proton loss. At the isoelectric point the pH is such
that there is no net charge on the macromolecule.
C
Vattraction    (14E.4)
s
Example 14E.1  Determining the isoelectric point of a
where C is yet another constant. The variation of the total po-
tential energy with separation is shown in Fig. 14E.2. protein
At high ionic strengths, the ionic atmosphere is dense (rD is The velocity with which the protein bovine serum albumin
small compared to a) and the potential shows a secondary min- (BSA) moves through water under the influence of an electric
imum at large separations. Aggregation of the particles arising field was monitored at several values of pH, and the data are
listed below. What is the isoelectric point of the protein?
Increasing rD pH 4.20 4.56 5.20 5.65 6.30 7.00
rD >> a
Velocity/(μm s−1) 0.50 0.18 −0.25 −0.65 −0.90 −1.25
rD << a
Collect your thoughts Plot velocity against pH, then use inter-
Potential energy, V

polation to find the pH at which the velocity is zero, which is


the pH at which the molecule has zero net charge.
The solution The data are plotted in Fig. 14E.3, and a line of
0
best-fit is drawn through the data points. The velocity passes
Coagulation

through zero at pH = 4.8; hence pH = 4.8 is the isoelectric point.


Flocculation
Separation, s Self-test 14E.1 The following data were obtained for another
protein:
Figure 14E.2 The variation of the potential energy of interaction
pH 3.5 4.5 5.0 5.5 6.0
with separation of the surfaces of the two particles and with the
ratio of the particle size a to the thickness of the electrical double Velocity/(μm s−1) 0.10 −0.10 −0.20 −0.30 −0.40
layer, rD. The regions labelled coagulation and flocculation show
Estimate the pH of the isoelectric point.
the dips in the potential energy curves where these processes Answer: 4.0
occur.
14E Self-assembly 643

0.5
4.8
Velocity, v/(μm s–1)

–0.5

–1

–1.5
4 5 pH 6 7 Figure 14E.4 A schematic version of a spherical micelle.
The hydrophilic groups are represented by spheres and the
Figure 14E.3 The plot of the velocity of a moving macromolecule
hydrophobic hydrocarbon chains are represented by the stalks;
against pH allows the isoelectric point to be detected as the pH at
these stalks are mobile.
which the velocity is zero. The data are from Example 14E.1.

The primary role of the electrical double layer is to con-


fer kinetic non-lability. Colliding colloidal particles break
through the double layer and coalesce only if the collision is
sufficiently energetic to disrupt the layers of ions and solvating
molecules, or if thermal motion has stirred away the surface
accumulation of charge. This disruption may occur at high
temperatures, which is one reason why sols precipitate when
they are heated.

Figure 14E.5 When a hydrocarbon molecule is surrounded


Exercise by water, the H2O molecules form a cage. As a result of this
E14E.1 The velocity v with which a protein moves through water under the acquisition of structure, the entropy of the water decreases, so
influence of an electric field varied with values of pH in the range 3.0 < pH < the dispersal of the hydrocarbon into the water is accompanied
7.0 according to the expression v/(μm s−1) = a + b(pH) + c(pH)2 + d(pH)3 with by a local decrease in entropy. However, the aggregation of these
a = 0.50, b = −0.10, c = −3.0 × 10−3, and d = 5.0 × 10−4. Identify the isoelectric
individual caged hydrocarbon molecules into a micelle releases
point of the protein.
many of the caging water molecules back into the bulk and results
in an increase in entropy.

Micelles and biological


14E.2
membranes Amphipathic substances do dissolve slightly in water, and an
understanding of the process gives insight into the formation of
In aqueous solutions surfactant molecules or ions can cluster micelles and biological structures in general.
together as micelles, which are colloid-sized clusters of mole- To understand the dissolution process in more detail,
cules. The hydrophobic tails of the surfactant molecules tend to imagine a hypothetical initial state in which the alcohol is
congregate in the interior of the micelle, and their hydrophilic present in water as individual molecules. Each hydropho-
head groups cluster on the surface (Fig. 14E.4). bic chain is surrounded by a cage of water molecules (Fig.
14E.5). This order reduces the entropy of the water below
its ‘pure’ value. Now consider the final state, in which the
14E.2(a) The hydrophobic interaction hydrophobic chains have clustered together. Although the
Consider a long-chained alcohol, such as pentan-1-ol clustering contributes to the lowering of the entropy of
(CH3CH2CH2CH2CH2OH). The hydrocarbon chain is hydro- the system, fewer (but larger) cages are required, and more
phobic and the –OH group is hydrophilic. A species with both water molecules are free to move. The net effect of the for-
hydrophobic and hydrophilic regions is called amphipathic.1 mation of clusters of hydrophobic chains is therefore a
decrease in the organization of water molecules and there-
fore a net increase in entropy of the system. This increase
1
The amphi- part of the name is from the Greek word for ‘both’, and the in entropy of the solvent (water) means that the associa-
-pathic part is from the same root (meaning ‘feeling’) as sympathetic. tion of hydrophobic groups in an aqueous environment is
644 14 Molecular interactions

s­ pontaneous (provided there are no overwhelming enthalpy 14E.2(b) Micelle formation


effects). This spontaneous clustering of hydrophobic groups
in the presence of water gives the appearance of it being the Micelles form only above the critical micelle concentration
outcome of an actual intermolecular force and is called the (CMC) and above the Krafft temperature. The CMC is de-
hydrophobic interaction. tected by noting a pronounced change in physical properties of
Some insight into the processes involved can be obtained the solution, particularly the molar conductivity (Fig. 14E.6).
from studies of the thermodynamics of dissolving (as distinct However, some properties do not show an abrupt change at the
from micelle formation). The entropy of dissolution of largely CMC; rather, there is a transition region corresponding to a
hydrophobic molecules in water is positive (ΔdissS > 0) as the
⦵ range of concentrations around the CMC where physical prop-
molecules disperse and the structure of the water changes to erties vary smoothly but nonlinearly with the concentration.
accommodate them. The process is commonly endothermic The hydrocarbon interior of a micelle is like a droplet of oil.
(ΔdissH > 0), but the Gibbs energy of dissolution (ΔdissG ) is
⦵ ⦵ Nuclear magnetic resonance shows that the hydrocarbon tails are
typically negative, as is illustrated by the following data (at mobile, but slightly more restricted than in the bulk. Micelles are
298 K): important in industry and biology on account of their solubiliz-
ing function: matter can be transported by water after it has been
ΔdissG⦵/ ΔdissH⦵/ ΔdissS⦵/ dissolved in the hydrocarbon interior of a micelle. For this rea-
(kJ mol−1) (kJ mol−1) (J K−1 mol−1) son, micellar systems are used as detergents, for organic synthesis,
CH3CH2CH2CH2OH −10 +8 +61 froth flotation for the treatment of ores, and petroleum recovery.
CH3CH2CH2CH2CH2OH −13 +8 +70 The self-assembly of a micelle has the characteristics of a
cooperative process in which the addition of a surfactant mol-
In other words, the tendency to dissolve (at least to a small ex- ecule to a cluster that is forming becomes more probable the
tent) is entropy-driven, with contributions from the dispersion larger the size of the aggregate, so after a slow start there is a
of the solute molecules and the restructuring of the water. Once cascade of formation of micelles. If it is supposed that the dom-
dissolved, further reorganization of the water occurs to drive inant micelle MN consists of N monomers M, then the domi-
the formation of micelles. The experimental values are consist- nant equilibrium to consider is
ent with a general rule that each additional −CH2− group con-
tributes a further −3 kJ mol−1 to the standard Gibbs energy of [M N ]/c⦵
NM  M N K=  (14E.6a)
dissolution. ([M]/c⦵)N
A further aspect of this discussion is that it is possible to where it has been assumed, probably dangerously on account
establish a scale of hydrophobicities. The hydrophobicity of a of the large sizes of monomers, that the solution is ideal and
small molecular group R is reported by defining the hydropho- that activities can be replaced by molar concentrations divided
bicity constant, π, as by the standard concentration. The total concentration of sur-
s(RX ) factant, [M]total, is [M] + N[MN] because each micelle consists of
Hydrophobicity constant
  log [definition]
 (14E.5) N monomer molecules; with this substitution for [M] the equi-
s(HX )
librium constant is written
where s(RX) is the ratio of the molar solubility of the hydro-
[M N ](c⦵)N 1
phobic compound RX in the largely hydrocarbon solvent K  (14E.6b)
­octan-1-ol to that in water, and s(HX) is the analogous ratio for ([M]total  N[M N ])N
the compound HX. A positive value of π indicates that RX is
more hydrophobic than RH.
It is found that the π values of most compounds do not de- Molar
pend on the identity of X (which might be OH, NH2, and so conductivity
Surface
on). However, measurements suggest that π increases by the
Physical property

tension
same amount each time a CH2 group is added:

–R −CH3 −CH2CH3 −(CH2)2CH3 −(CH2)3CH3 −(CH2)4CH3


π 0.5 1.0 1.5 2.0 2.5
Osmotic
pressure
CMC

It follows that acyclic saturated hydrocarbons become more hy-


drophobic as the carbon chain length increases. This trend can
be rationalized by noting that ΔdissG becomes more negative

Concentration of surfactant
as the number of carbon atoms in the chain increases, with the Figure 14E.6 The typical variation of some physical properties of
data on butan-1-ol and pentan-1-ol (see above) suggesting that an aqueous solution of sodium dodecyl sulfate close to the critical
the principal effect is due to the entropy. micelle concentration (CMC).
14E Self-assembly 645

0.4 Table 14E.1 Micelle shape and the surfactant parameter


Fraction present as micelles

Ns Micelle shape
0.3 <0.33 Spherical
0.33–0.50 Cylindrical rods
N=3
0.2 0.50–1.00 Vesicles
1.00 Planar bilayers
N = 30 (magnified × 10)
0.1 >1.00 Reverse micelles and other shapes

0 ­ olecules on the micellar surface and interacting favourably


m
0 2 4 6 8 10
with solvent and ions in solution. Hydrophobic interactions
Total concentration of surfactant molecules, [M]total/c⦵
stabilize the aggregation of the hydrophobic surfactant tails
Figure 14E.7 The dependence of the fraction of surfactant in the micellar core. Under certain experimental conditions,
molecules present as micelles on the total concentration of a liposome may form (Fig. 14E.8). This structure has an outer
surfactant molecules and assuming K = 1. layer of surfactant molecules with their head groups on the
outside, and underneath this is another layer of surfactant mol-
ecules with their head groups on the inside. Liposomes may be
Brief illustration 14E.1 used to carry nonpolar drug molecules in blood.
Increasing the ionic strength of the aqueous solution re-
Equation 14E.6b can be solved numerically for the variation of
duces repulsions between surface head groups, and cylindrical
the fraction of molecules present as micelles with the number
micelles can form. These cylinders may stack together in rea-
of molecules present in a micelle and some results for K = 1 are
sonably close-packed (hexagonal) arrays, forming lyotropic
shown in Fig. 14E.7. For large N, there is a reasonably sharp
mesomorphs and, more colloquially, ‘liquid crystalline phases’.
transition in the fractions of surfactant molecules that are
Reverse micelles form in nonpolar solvents, with small polar
present in micelles. This behaviour accounts for the existence
of a CMC. surfactant head groups in a micellar core and more voluminous
hydrophobic surfactant tails extending into the organic bulk
phase. These spherical aggregates can solubilize water in or-
ganic solvents by creating a pool of trapped water molecules in
Non-ionic surfactant molecules may cluster together in
the micellar core. As aggregates arrange at high surfactant con-
clumps of 1000 or more, but ionic species tend to be disrupted
centrations to yield long-range positional order, many other
by the electrostatic repulsions between head groups and are
types of structures are possible including cubic and hexagonal
normally limited to groups of less than about 100. However, the
shapes.
disruptive effect depends more on the effective size of the head
As already noted, micelle formation is driven by hydropho-
group than the charge. For example, ionic surfactants such
bic interactions. The enthalpy of formation reflects contribu-
as sodium dodecyl sulfate (SDS) and cetyltrimethylammo-
tions of interactions between chains within the micelles and
nium bromide (CTAB) form rods at moderate concentrations
between the polar head groups and the surrounding medium.
whereas sugar surfactants form small, approximately spheri-
Consequently, enthalpies of micelle formation display no read-
cal micelles. The micelle population commonly spans a wide
ily discernible pattern and may be positive (endothermic)
range of particle sizes (i.e. it is polydisperse), and the shapes of
or negative (exothermic). Many non-ionic micelles form en-
the individual micelles vary with shape of the constituent sur-
dothermically, with ΔH of the order of 10 kJ per mole of sur-
factant molecules, surfactant concentration, and temperature.
factant molecules. That such micelles do form above the CMC
A useful predictor of the shape of the micelle is the surfactant
­parameter, Ns, defined as
V
Ns = Surfactant parameter [definition] (14E.7)
Al
where V is the volume of the hydrophobic surfactant tail, A is
the area of the hydrophilic surfactant head group, and l is the
maximum length of the surfactant tail. Table 14E.1 summa-
rizes the dependence of aggregate structure on the surfactant
parameter.
In aqueous solutions spherical micelles form, as shown Figure 14E.8 The cross-sectional structure of a spherical
in Fig. 14E.4, with the polar head groups of the surfactant liposome.
646 14 Molecular interactions

indicates that the entropy change accompanying their forma-


tion must then be positive, and measurements suggest a value
of about +140 J K−1 mol−1 at room temperature.

14E.2(c) Bilayers, vesicles, and membranes


Some micelles at concentrations well above the CMC form
extended parallel sheets two molecules thick, called planar
­bilayers. The individual molecules lie perpendicular to the (a) (b)
sheets, with hydrophilic groups on the outside in aqueous so-
Figure 14E.9 A depiction of the variation with temperature
lution and on the inside in nonpolar media. When segments of the flexibility of hydrocarbon chains in a lipid bilayer. (a) At
of planar bilayers fold back on themselves, unilamellar vesicles physiological temperature, the bilayer exists as a liquid crystal, in
may form where the spherical hydrophobic bilayer shell sepa- which some order exists but the chains writhe. (b) At a specific
rates an inner aqueous compartment from the external aque- (lower) temperature, the chains are largely frozen and the bilayer
ous environment. exists as a gel.
Bilayers show a close resemblance to biological membranes,
and are often a useful model on which to base investigations
of biological structures. However, actual membranes are The membrane then exists as a gel. Biological membranes exist
highly sophisticated structures. The basic structural element as liquid crystals at physiological temperatures.
of a membrane is a phospholipid, such as phosphatidyl cho- Phase transitions in membranes are often observed as ‘melt-
line (1), which contains long hydrocarbon chains (typically ing’ from gel to liquid crystal by calorimetric methods. The
in the range C14–C24) and a variety of polar groups, such as data show relations between the structure of the lipid and the
CH2 CH2 N(CH3 )3. The hydrophobic chains stack together to melting temperature. Interspersed among the phospholipids of
form an extensive layer about 5 nm across. The lipid molecules biological membranes are sterols, such as cholesterol (2), which
form layers instead of micelles because the hydrocarbon chains is largely hydrophobic but does contain a hydrophilic –OH
are too bulky to allow packing into nearly spherical clusters. group. Sterols, which are present in different proportions in dif-
ferent types of cells, prevent the hydrophobic chains of lipids
O from ‘freezing’ into a gel and, by disrupting the packing of the
(CH)7 (CH2)7 CH3 chains, spread the melting point of the membrane over a range
cis
O of temperatures.
O– O
+(H C) N
3 3 O P O O
O (CH2)14 CH3

1 Phosphatidyl choline

The bilayer is a highly mobile structure. Not only are the


hydrocarbon chains ceaselessly twisting and turning in the HO
region between the polar groups, but the phospholipid mol-
ecules migrate over the surface. It is better to think of the 2 Cholesterol
membrane as a viscous fluid rather than a permanent struc-
ture, with a viscosity about 100 times that of water. Typically, Brief illustration 14E.2
a phospholipid molecule in a membrane migrates through
about 1 μm in about 1 min. To predict trends in melting temperatures you need to assess
All lipid bilayers undergo a transition from a state of high to the strengths of the interactions between molecules. Longer
low chain mobility at a temperature that depends on the struc- chains can be expected to be held together more strongly by
ture of the lipid. To visualize the transition, consider what hap- hydrophobic interactions than shorter chains, so you should
pens to a membrane as its temperature is lowered (Fig. 14E.9). expect the melting temperature to increase with the length
of the hydrophobic chain of the lipid. On the other hand, any
There is sufficient energy available at normal temperatures for
structural elements that prevent alignment of the hydropho-
limited bond rotation to occur and the flexible chains writhe.
bic chains in the gel phase lead to low melting temperatures.
However, the membrane is still highly organized in the sense
Indeed, lipids containing unsaturated chains (those containing
that the bilayer structure does not come apart and the system
some C=C bonds) form membranes with lower melting tem-
is best described as a liquid crystal. At lower temperatures,
peratures than those formed from lipids with fully saturated
the amplitudes of the writhing motion decrease until a spe- chains (those consisting of C–C bonds only).
cific temperature is reached at which motion is largely frozen.
14E Self-assembly 647

Checklist of concepts
☐ 1. A disperse system is a dispersion of small particles of ☐ 8. Flocculation is the reversible aggregation of colloidal
one material in another. particles.
☐ 2. Colloids are classified as lyophilic and lyophobic. ☐ 9. Coagulation is the irreversible aggregation of colloidal
☐ 3. A surfactant is a species that accumulates at the inter- particles.
face of two phases or substances. ☐ 10. The Schulze–Hardy rule states that hydrophobic col-
☐ 4. Many colloidal particles are thermodynamically unsta- loids are flocculated most efficiently by ions of opposite
ble but kinetically non-labile. charge type and high charge number.
☐ 5. The radius of shear is the radius of the sphere that ☐ 11. An amphipathic species has both hydrophobic and
captures the rigid layer of charge attached to a colloid hydrophilic regions.
particle. ☐ 12. The hydrophobic interaction results in the clustering of
☐ 6. The electrokinetic potential is the electric potential at nonpolar solutes in water.
the radius of shear relative to its value in the distant, ☐ 13. A micelle is a colloid-sized cluster of molecules that
bulk medium. forms at and above the critical micelle concentration
☐ 7. The inner shell of charge and the outer ionic atmosphere and above the Krafft temperature.
jointly constitute the electrical double layer. ☐ 14. Unilamellar vesicles are micelles that exist as extended
parallel sheets.

Checklist of equations
Property Equation Comment Equation number

Thickness of the electrical double layer rD = (εRT/2ρF2Ib )1/2 Debye–Hückel theory 14E.3
Hydrophobicity constant π = log{s(RX)/s(HX)} Definition 14E.5
Surfactant parameter Ns = V/Al Definition 14E.7
648 14 Molecular interactions

FOCUS 14 Molecular interactions

To test your understanding of this material, work through Selected solutions can be found at the end of this Focus in
the Exercises, Additional exercises, Discussion questions, and the e-book. Solutions to even-numbered questions are available
Problems found throughout this Focus. online only to lecturers.

TOPIC 14A The electric properties of molecules


Discussion questions
D14A.1 Explain how the permanent dipole moment and the polarizability of a D14A.3 Describe the experimental procedures available for determining the
molecule arise. electric dipole moment of a molecule.
D14A.2 Explain why the polarizability of a molecule decreases at high frequencies.

Additional exercises
3 −1
E14A.9 Which of the following molecules may be polar: SO3, XeF4, SF4? E14A.15 At 0 °C, the molar polarization of a liquid is 32.16 cm mol and its
mass density is 1.92 g cm−3. Calculate the relative permittivity of the liquid.
E14A.10 Calculate the resultant of two dipole moments of magnitude 2.5 D
Take M = 85.0 g mol−1.
and 0.50 D that make an angle of 120° to each other.
E14A.16 The refractive index of a compound is 1.622 for 643 nm light. Its mass
E14A.11 Calculate the magnitude and direction of the dipole moment of the
density at 20 °C is 2.99 g cm−3. Calculate the polarizability of the molecule at
following arrangement of charges in the xy-plane: 4e at (0, 0), −2e at (162 pm,
this wavelength. Take M = 65.5 g mol−1.
0), and −2e at an angle of 30° from the x-axis and a distance of 143 pm from
the origin. E14A.17 The polarizability volume of H2O at optical frequencies is
1.5 × 10−24 cm3. Estimate the refractive index of water. The experimental
E14A.12 What strength of electric field is required to induce an electric dipole
value is 1.33.
moment of magnitude 2.5 μD in a molecule of polarizability volume
1.05 × 10−29 m3 (like CCl4)? −1
E14A.18 The polarizability volume of a liquid of molar mass 72.3 g mol and
mass density 865 kg m−3 at optical frequencies is 2.2 × 10−30 m3. Estimate the
E14A.13 The molar polarization of fluorobenzene vapour varies linearly with
refractive index of the liquid.
T −1, and is 70.62 cm3 mol−1 at 351.0 K and 62.47 cm3 mol−1 at 423.2 K. Calculate
−30
the polarizability and dipole moment of the molecule. E14A.19 The dipole moment of bromobenzene is 5.17 × 10 C m and its
polarizability volume is approximately 1.5 × 10−29 m3. Estimate its relative
E14A.14 The molar polarization of the vapour of a compound was found to
permittivity at 25 °C, when its mass density is 1491 kg m−3.
vary linearly with T −1, and is 75.74 cm3 mol−1 at 320.0 K and 71.43 cm3 mol−1 at
421.7 K. Calculate the polarizability and dipole moment of the molecule.

Problems
P14A.1 The electric dipole moment of methylbenzene (toluene) is 0.4 D. H
O
Estimate the dipole moments of the three isomers of dimethylbenzene (the
xylenes). About which answer can you be sure?
P14A.2 Plot the magnitude of the electric dipole moment of hydrogen
peroxide as the H–O–O–H (azimuthal) angle ϕ changes from 0 to 2π. Use the
2
dimensions and partial charges shown in (1).

P14A.4 D.D. Nelson et al. (Science, 238, 1670 (1987)) examined several
+0.45e weakly bound gas-phase complexes of ammonia in search of examples in
H
ϕ which the H atoms in NH3 formed hydrogen bonds, but found none. For
97 pm –0.83e
example, they found that the complex of NH3 and CO2 has the carbon atom
nearest the nitrogen (299 pm away): the CO2 molecule is at right angles to
O 149 pm
the C–N ‘bond’, and the H atoms of NH3 are pointing away from the CO2.
–0.83e The magnitude of the permanent dipole moment of this complex is reported
1 +0.45e as 1.77 D. If the N and C atoms are the centres of the negative and positive
charge distributions, respectively, what is the magnitude of those partial
P14A.3 Ethanoic (acetic) acid vapour contains a proportion of planar, charges (as multiples of e)?
hydrogen-bonded dimers (2). The apparent dipole moment of molecules in
pure gaseous ethanoic acid has a magnitude that increases with increasing

temperature. Suggest an interpretation of this observation. These problems were supplied by Charles Trapp and Carmen Giunta.
Exercises and problems 649

−30
P14A.5 The polarizability volume of NH3 is 2.22 × 10 m3; calculate the T/K 292.2 309.0 333.0 387.0 413.0 446.0
contribution to the dipole moment of the molecule induced by an applied
3 −1
electric field of strength 15.0 kV m−1. Pm/(cm mol ) 57.57 55.01 51.22 44.99 42.51 39.59

P14A.6 The magnitude of the electric field at a distance r from a point charge The refractive index of ammonia at 273 K and 100 kPa is 1.000 379 (for yellow
Q is equal to Q/4πε0r2. How close to a water molecule (of polarizability sodium light). Calculate the molar polarization of the gas at this temperature.
volume 1.48 × 10−30 m3) must a proton approach before the dipole moment it Combine the value calculated with the static molar polarization at 292.2 K and
induces has a magnitude equal to that of the permanent dipole moment of the deduce from this information alone the molecular dipole moment.
molecule (1.85 D)? P14A.10 Values of the molar polarization of gaseous water at 100 kPa as
P14A.7 The relative permittivity of trichloromethane (chloroform) was determined from capacitance measurements are given below as a function of
measured over a range of temperatures with the following results: temperature.

θ/°C −80 −70 −60 −40 −20 0 20 T/K 384.3 420.1 444.7 484.1 521.0
3 −1
εr 3.1 3.1 7.0 6.5 6.0 5.5 5.0 Pm/(cm mol ) 57.4 53.5 50.1 46.8 43.1
−3
ρ/(g cm ) 1.65 1.64 1.64 1.61 1.57 1.53 1.50 Calculate the dipole moment of H2O and its polarizability volume.

The freezing point of trichloromethane is −64 °C. Account for these P14A.11 From data in Table 14A.1 calculate the molar polarization, relative
results and calculate the dipole moment and polarizability volume of the permittivity, and refractive index of methanol at 20 °C. Its mass density at that
molecule. temperature is 0.7914 g cm−3.
P14A.8 The relative permittivities of methanol (with a melting point of −95 °C) P14A.12 Show that, in a gas (for which the refractive index is close to 1), the
corrected for density variation are given below. What molecular information refractive index depends on the pressure as nr = 1 + constant × p, and find the
can be deduced from these values? Take ρ = 0.791 g cm−3. constant of proportionality. Go on to show how to deduce the polarizability
volume of a molecule from measurements of the refractive index of a gaseous
θ/°C −185 −170 −150 −140 −110 −80 −50 −20 0 20 sample.
εr 3.2 3.6 4.0 5.1 67 57 49 43 38 34 P14A.13 Ethanoic (acetic) acid vapour contains a proportion of planar,
P14A.9 In his classic book Polar molecules, Debye reports some early
hydrogen-bonded dimers. The relative permittivity of pure liquid ethanoic
measurements of the polarizability of ammonia. From the selection acid is 7.14 at 290 K and increases with increasing temperature. Suggest an
below, determine the dipole moment and the polarizability volume of the interpretation of the latter observation. What effect should isothermal dilution
molecule. have on the relative permittivity of solutions of ethanoic acid in benzene?

TOPIC 14B Interactions between molecules


Discussion questions
D14B.1 Identify the terms in and of the following expressions and specify D14B.5 Some polymers have unusual properties. For example, Kevlar (3) is
the conditions under which they are valid: (a) V = −Q2μ1/4πε0r2, strong enough to be the material of choice for bulletproof vests and is stable
(b) V = −Q2μ1cos θ/4πε0r2, and (c) V = μ2μ1(1 − 3 cos2θ)/4πε0r3. at temperatures up to 600 K. What molecular interactions contribute to the
formation and thermal stability of this polymer?
D14B.2 Draw examples of the arrangements of electrical charges that
correspond to monopoles, dipoles, quadrupoles, and octupoles. Suggest a
reason for the different distance dependencies of their electric fields.
NH O
D14B.3 Account for the theoretical conclusion that many attractive H N
interactions between molecules vary with their separation as 1/r6.
H
O O H
D14B.4 Describe the formation of a hydrogen bond in terms of (a) electrostatic
n
interactions and (b) molecular orbitals. How would you identify the better
model? 3 Kevlar

Additional exercises
E14B.6 Calculate the molar energy required to reverse the direction of an HCl E14B.9 Calculate the potential energy of the interaction between two linear
molecule located 300 pm from a Mg2+ ion. Take the magnitude of the dipole quadrupoles when they are parallel and separated by a distance r.
moment of HCl as 1.08 D.
E14B.10 Calculate the average interaction energy for pairs of molecules in the
E14B.7 Use eqn 14B.3b to calculate the molar potential energy of the dipolar gas phase with μ = 2.5 D when the separation is 1.0 nm at 273 K. Compare this
interaction between an amide group (μ = 2.7 D) and a water molecule energy with the average molar kinetic energy of the molecules.
(μ = 1.85 D) separated by 3.0 nm with θ = 45° in a medium with relative
E14B.11 Calculate the average dipole–induced dipole interaction energy, in
permittivity of 3.5.
joules per mole (J mol−1), between a water molecule and a CCl4 molecule
E14B.8 Calculate the potential energy of the interaction between two linear separated by 1.0 nm.
quadrupoles when they are collinear and their centres are separated by a
distance r.
650 14 Molecular interactions

E14B.12 Estimate the energy of the dispersion interaction (use the London E14B.13 Estimate the energy of the dispersion interaction (use the London
formula) for two He atoms separated by 1.0 nm. Relevant data can be found in formula) for two Ar atoms separated by 1.0 nm. Relevant data can be found in
the Resource section. the Resource section.

Problems
P14B.1 In general, atoms in molecules have partial charges arising from the P14B.6 Consider the London interaction between the phenyl groups of
spatial variation in electron density in the ground state. If these charges were two phenylalanine (Phe, 4) residues. (a) Estimate the potential energy of
separated by a medium, they would attract or repel each other in accord interaction between two such rings (treated as benzene molecules) separated
with Coulomb’s law: V = Q1Q2/4πεr where Q1 and Q2 are the partial charges, by 0.4 nm. For the ionization energy, use I = 5.0 eV. (b) Given that force is the
r is their separation, and ε is the permittivity of the medium lying between negative slope of the potential, calculate the distance dependence of the force
the charges. Different values of the permittivity of the medium take into acting between two non-bonded groups of atoms, such as the phenyl groups
account the possibility that other parts of the molecule, or other molecules, lie of Phe, in a polypeptide chain that can have a London dispersion interaction
between the charges. (a) Calculate the energy of interaction between a partial with each other. What is the separation at which the force between the phenyl
charge of −0.36 (that is, Q1 = −0.36e) on the N atom of an amide group and groups (treated as benzene molecules) of two Phe residues is zero? Hint:
the partial charge of +0.45 (Q2 = +0.45e) on the carbonyl C atom at a distance Calculate the slope by considering the potential energy at r and r + δr, with δr
of 3.0 nm, on the assumption that the medium between them is a vacuum. << r, and evaluating {V(r + δr) − V(r)}/δr. At the end of the calculation, let δr
(b) Repeat the calculation for bulk water as the medium. become vanishingly small.
P14B.2 In general, atoms in molecules have partial charges arising from the
spatial variation in electron density in the ground state. If these charges were O
separated by a medium, they would attract or repel each other in accord
with Coulomb’s law: V = Q1Q2/4πεr where Q1 and Q2 are the partial charges,
H2N OH
r is their separation, and ε is the permittivity of the medium lying between
the charges. Different values of the permittivity of the medium take into
account the possibility that other parts of the molecule, or other molecules, 4 Phenylalanine
lie between the charges. Use the electrostatic model to calculate the energy (in
kJ mol−1) required to break an O H hydrogen bond in a vacuum (εr = 1) and P14B.7 Given that F = −dV/dr, calculate the distance dependence of the force
in water (εr ≈ 80.0). Let the O H distance be 170 pm, and partial charges on acting between two non-bonded groups of atoms in a polymer chain that have
the H and O atoms be +0.42e and −0.84e, respectively. a London dispersion interaction with each other.

P14B.3 An H2O molecule (μ = 1.85 D) is aligned by an external electric field P14B.8 Consider the arrangement shown in 5 for a system consisting of
of strength 1.0 kV m−1 (so that the dipole moment vector is parallel to the an O–H group and an O atom, and then use the electrostatic model of the
direction of the electric field) and an Ar atom (α′ = 1.66 × 10−30 m3) is brought hydrogen bond to calculate the dependence of the molar potential energy of
up slowly from one side. At what separation is it energetically favourable for interaction on the angle θ.
the H2O molecule to rotate by 90° and the dipole moment point towards the
O –0.83e
approaching Ar atom?
200 pm
P14B.4 Suppose an H2O molecule (μ = 1.85 D) approaches an anion. What is
θ H
the favourable orientation of the molecule? Calculate the magnitude of the –0.83e
+0.45e
electric field (in volts per metre) experienced by the anion when the water
95.7 pm
dipole is (a) 1.0 nm, (b) 0.3 nm, (c) 30 nm from the ion.
5
P14B.5 Phenylalanine (Phe, 4) is a naturally occurring amino acid. What is
the energy of interaction between its phenyl group and the electric dipole P14B.9 Suppose you distrusted the Lennard-Jones (12,6) potential for
moment of a neighbouring peptide group? Take the distance between the assessing a particular polypeptide conformation, and replaced the repulsive
groups as 0.4 nm and treat the phenyl group as a benzene molecule. The term by an exponential function of the form ae −br /r0. (a) Sketch the form of the
dipole moment of the peptide group is μ = 2.7 D and the polarizability volume potential energy and locate the distance at which it is a minimum. Hint: Use
of benzene is α′ = 1.04 × 10−29 m3. mathematical software.
P14B.10 The cohesive energy density, U , is defined as U/V, where U is the mean
O potential energy of attraction within the sample and V its volume. Show that
U  12 N 2  V (R)d where N is the number density of the molecules and V(R)
is their attractive potential energy and where the integration ranges from d to
H2N OH infinity and over all angles. Go on to show that the cohesive energy density of
a uniform distribution of molecules that interact by a van der Waals attraction
4 Phenylalanine of the form −C6/R6 is equal to (2/3)(N A2 /d 3 M 2 ) 2C6, where ρ is the mass
density of the solid sample and M is the molar mass of the molecules.

TOPIC 14C Liquids


Discussion questions
D14C.1 Explain the Monte Carlo and molecular dynamics methods for the D14C.2 Describe the process of condensation.
calculation of the radial distribution function in liquids.
Exercises and problems 651

Additional exercises
E14C.4 Calculate the vapour pressure of a spherical droplet of water of radius E14C.6 Calculate the pressure differential of ethanol across the surface of a
20.0 nm at 35.0 °C. The vapour pressure of bulk water at that temperature is spherical droplet of radius 220 nm at 20 °C. The surface tension of ethanol at
5.623 kPa and its mass density is 994.0 kg m−3. that temperature is 22.39 mN m−1.
E14C.5 The contact angle for water on clean glass is close to zero. Calculate the E14C.7 Calculate the surface tension of a liquid at 25 °C given that at that
surface tension of water at 30 °C given that at that temperature water climbs temperature, the liquid climbs to a height of 10.00 cm in a clean glass capillary
to a height of 9.11 cm in a clean glass capillary tube of internal diameter tube of internal radius 0.300 mm. The mass density of the liquid at 25 °C is
0.320 mm. The mass density of water at 30 °C is 0.9956 g cm−3. 0.9500 g cm−3. Assume that the contact angle is zero.

Problems
P14C.1 A simple pair distribution function has the form P14C.2 The surface tensions of a series of aqueous solutions of a surfactant A
were measured at 20 °C, with the following results:
 4r 
g (r )  1  cos   4  e (r /r0 1)
 r0  [A]/(mol dm−3) 0 0.10 0.20 0.30 0.40 0.50

for r ≥ r0 and g(r) = 0 for r < r0. Here the parameter r0 is the separation γ/(mN m−1) 72.8 70.2 67.7 65.1 62.8 59.8
at which the Lennard-Jones potential energy function (eqn 14B.12)
Calculate the surface excess concentration.
V = 4ε{(r0/r)12 − (r0/r)6} is equal to zero. (a) Plot the function g(r). Does it
resemble the form shown in Fig. 14C.1? (b) Plot the virial n2(r) = r(dV/dr).

TOPIC 14D Macromolecules


Discussion questions
D14D.1 Distinguish between number-average, weight-average, and Z-average D14D.4 Define the terms in the following expressions and specify the
molar masses. Which experiments give information about each one? conditions for their validity: (a) Rc = Nl, (b) Rrms = N1/2l, (c) Rrms = (2N)1/2l,
(d) Rrms = N1/2lF, (e) Rg = (N/2)1/2l, (f) Rg = (N/6)1/2l, (g) Rg = (N/3)1/2l.
D14D.2 Distinguish between the four levels of structure of a macromolecule:
primary, secondary, tertiary, and quaternary. D14D.5 Distinguish between the melting temperature and the glass transition
temperature of a polymer.
D14D.3 What are the consequences of there being partial rigidity in an
otherwise random coil?

Additional exercises
E14D.10 Calculate the number-average molar mass and the weight-average E14D.17 What is the probability that the ends of a polyethene chain of molar
molar mass of a mixture of two polymers, one having M = 62 kg mol−1 and the mass 75 kg mol−1 are between 14.0 nm and 14.1 nm apart when the polymer is
other M = 78 kg mol−1, with their amounts in moles in the ratio 3:2. treated as a three-dimensional freely jointed chain?
E14D.11 A one-dimensional polymer chain consists of 1200 segments, each E14D.18 By what percentage does the root-mean-square separation of the ends
1.125 nm long. If the chain were ideally flexible, what would be the root- of a one-dimensional polymer chain increase (+) or decrease (−) when the
mean-square separation of the ends of the chain? bond angle between units is limited to 120°? What is the percentage change in
volume of the coil?
E14D.12 Calculate the contour length (the length of the extended chain) and
the root-mean-square separation (the end-to-end distance) of polyethene E14D.19 By what percentage does the root-mean-square separation of the ends
with molar mass 280 kg mol−1, modelled as a one-dimensional chain. of a one-dimensional polymer chain consisting of 1000 monomers increase
(+) or decrease (−) when the persistence length changes from l (the bond
E14D.13 Calculate the contour length (the length of the extended chain) and
length) to 5.0 per cent of the contour length? What is the percentage change
the root-mean-square separation (the end-to-end distance) for polypropene
in volume of the coil?
of molar mass 174 kg mol−1, modelled as a one-dimensional chain.
E14D.20 By what percentage does the root-mean-square separation of the ends
E14D.14 The radius of gyration of a long one-dimensional chain molecule is
of a one-dimensional polymer chain consisting of 1000 monomers increase
found to be 18.9 nm. The chain consists of links of length 450 pm. Assume that
(+) or decrease (−) when the persistence length changes from l (the bond
the chain is randomly coiled and estimate the number of links in the chain.
length) to 2.5 per cent of the contour length? What is the percentage change
E14D.15 What is the probability that the ends of a polyethene chain of molar in volume of the coil?
mass 85 kg mol−1 are 15 nm apart when the polymer is treated as a one-
E14D.21 The radius of gyration of a three-dimensional partially rigid polymer
dimensional freely jointed chain?
of 1500 units each of length 164 pm was measured as 3.0 nm. What is the
E14D.16 What is the probability that the ends of a polyethene chain of molar persistence length of the polymer?
mass 65 kg mol−1 are between 10.0 nm and 10.1 nm apart when the polymer is
treated as a three-dimensional freely jointed chain?
652 14 Molecular interactions

E14D.22 Calculate the restoring force when the ends of a one-dimensional E14D.23 Calculate the change in molar entropy when the ends of a one-
polyethene chain of molar mass 85 kg mol−1 are moved apart by 2.0 nm at dimensional polyethene chain of molar mass 85 kg mol−1 are moved apart by
25 °C. 2.0 nm.

Problems
P14D.1 Evaluate the mechanical radius of gyration, Rg , of (a) a solid sphere of
A M/(g mol−1) vs/(cm3 g−1) Rg/nm
radius a, (b) a long straight rod of radius a and length l. Show that in the case
of a solid sphere of specific volume vs, Rg/nm ≈ 0.056 902 × {(vs/cm3 g−1)(M/g Serum albumin 66 × 103 0.752 2.98
mol−1)}1/3. Evaluate Rg for a species with M = 100 kg mol−1, vs = 0.750 cm3 g−1, Bushy stunt virus 10.6 × 10 6
0.741 12.0
and, in the case of the rod, of radius 0.50 nm.
DNA 4 × 106 0.556 117.0
P14D.2 Use eqn 14D.4 to deduce expressions for (a) the root-mean-square
separation of the ends of the chain, (b) the mean separation of the ends, and P14D.8 Develop an expression for the fundamental vibrational frequency
(c) their most probable separation. Evaluate these three quantities for a fully of a one-dimensional random coil that has been slightly stretched and then
flexible chain with N = 4000 and l = 154 pm. released. Evaluate this frequency for a sample of polyethene of molar mass
2 2 65 kg mol−1 at 20 °C. Account physically for the dependence of frequency on
P14D.3 Deduce the relation ri   Nl for the mean square distance of a
temperature and molar mass.
monomer from the origin in a freely jointed chain of N units each of length l.
Hint: Use the distribution in eqn 14D.4. P14D.9 On the assumption that the tension, t, required to keep a sample at a
constant length is proportional to the temperature (t = aT, the analogue of
P14D.4 Derive expressions for the moments of inertia and hence the
p ∝ T), show that the tension can be ascribed to the dependence of the
(mechanical) radii of gyration of (a) a uniform thin disk, (b) a long uniform
entropy on the length of the sample. Account for this result in terms of the
rod, (c) a uniform sphere.
molecular nature of the sample.
P14D.5 Construct a two-dimensional random walk by using a random number
P14D.10 The following table lists the glass transition temperatures, Tg, of
generating routine with mathematical software or spreadsheet. Construct a
several polymers. Discuss the reasons why the structure of the monomer unit
walk of 50 and 100 steps. If there are many people working on the problem,
has an effect on the value of Tg.
investigate the mean and most probable separations in the plots by direct
measurement. Do they vary as N1/2?
Polymer Poly- Polyethene Poly(vinyl Polystyrene
P14D.6 Show that it is possible to define the radius of gyration Rg as the (oxymethylene) chloride)
average root-mean-square distance of the atoms or groups (all assumed to
Structure –OCH2)n– –(CH2CH2)n– –(CH2–CHCl)n– –(CH2–CH(C6H5))n–
be of the same mass), that is, that Rg2  (1/N )  j R 2j , where Rj is the distance of
atom j from the centre of mass. Tg/K 198 253 354 381

P14D.7 Use the information below and the expression for Rg of a solid sphere
quoted in the text, to classify the species below as globular or rod-like.

TOPIC 14E Self-assembly


Discussion questions
D14E.1 Distinguish between a sol, an emulsion, and a foam. Provide examples D14E.4 What effect is the inclusion of cholesterol likely to have on the
of each. transition temperatures of a lipid bilayer?
D14E.2 Account for the hydrophobic interaction and discuss its D14E.5 Why do bacterial and plant cells grown at low temperatures synthesize
manifestations. more phospholipids with unsaturated chains than do cells grown at higher
temperatures?
D14E.3 It is observed that the critical micelle concentration of sodium dodecyl
sulfate in aqueous solution decreases as the concentration of added sodium
chloride increases. Explain this effect.

Additional exercise
E14E.2 The velocity v with which a protein moves through water under the with a = 0.80, b = −4.0 × 10−3, and c = −5.0 × 10−2. Identify the isoelectric point
influence of an electric field varied with values of pH in the range of the protein.
3.0 < pH < 5.0 according to the expression v/(μm s−1) = a + b(pH) + c(pH)2

Problems
P14E.1 The binding of nonpolar groups of amino acids to hydrophobic sites respectively, 0.5, 1.0, 1.5, 2.0, and 2.5. Use these data to predict the π value
in the interior of proteins is governed largely by hydrophobic interactions. for (CH2)6CH3. (b) The equilibrium constants KI for the dissociation of
(a) Consider a family of hydrocarbons R–H. The hydrophobicity constants, inhibitors (6) from the enzyme chymotrypsin were measured for different
π, for R = CH3, CH2CH3, (CH2)2CH3, (CH2)3CH3, and (CH2)4CH3 are, substituents R:
Exercises and problems 653

R CH3CO CN NO2 CH3 Cl


R NH
π −0.20 −0.025 0.33 0.5 0.9
CHO
log KI −1.73 −1.90 −2.43 −2.55 −3.40
6
Plot log KI against π. Does the plot suggest a linear relationship? If so, what P14E.2 Use mathematical software to reproduce the features in Fig. 14E.7.
are the slope and intercept to the log KI axis of the line that best fits the data? P14E.3 Equation 14E.6b is surprisingly tricky to solve, but it is possible
(c) Predict the value of KI for the case R = H. to make good progress with simple cases. With N = 2 and K = 1, find an
expression for [M2].

FOCUS 14 Molecular interactions


Integrated activities
I14.1 Show that the mean interaction energy of N atoms of diameter d O
interacting with a potential energy of the form −C6/R6 is given by
U = −2N2C6/3Vd3, where V is the volume in which the molecules are confined
CH3
N
and all effects of clustering are ignored. Hence, find a connection between the H
van der Waals parameter a and C6, from n2a/V2 = (∂U/∂V)T .

I14.2 F. Luo et al. (J. Chem. Phys., 98, 3564 (1993)) reported experimental 9 trans-N-methylacetamide
observation of the He2 complex, a species that had escaped detection for I14.5 Before attempting this problem, read Impact 22 on the website for this
a long time. The fact that the observation required temperatures in the text. Derivatives of the compound TIBO (10) inhibit the enzyme reverse
neighbourhood of 1 mK is consistent with computational studies which transcriptase, which catalyses the conversion of retroviral RNA to DNA. A
suggest that hcD e , for He2 is about 1.51 × 10−23 J, hcD 0 about 2 × 10−26 J, and quantitative structure–activity relationship (QSAR) analysis of the activity A
R about 297 pm. (a) Determine the Lennard-Jones parameters r0, and ε and of a number of TIBO derivatives suggests the following equation:
plot the Lennard-Jones potential for He–He interactions. (b) Plot the Morse
log A  b0  b1S  b2W
potential given that a = 5.79 × 1010 m−1.
where S is a parameter related to the drug’s solubility in water and W is a
I14.3 Before attempting this problem, read Impact 21, available via the
parameter related to the width of the first atom in a substituent X shown in
e-book. Molecular orbital calculations may be used to predict structures of
10. (a) Use the following data to determine the values of b0, b1, and b2. Hint:
intermolecular complexes. Hydrogen bonds between purine and pyrimidine
The QSAR equation relates one dependent variable, log A, to two independent
bases are responsible for the double helix structure of DNA. Consider methyl–
variables, S and W. To fit the data, you must use the mathematical procedure
adenine (7, with R = CH3) and methyl–thymine (8, with R = CH3) as models
of multiple regression, which can be performed with mathematical software or
of two bases that can form hydrogen bonds in DNA. (a) Use molecular
a spreadsheet.
modelling software and the computational method of your or your instructor’s
choice to calculate the atomic charges of all atoms in methyl–adenine and X H Cl SCH3 OCH3 CN CHO Br CH3 CCH
methyl–thymine. (b) Based on your tabulation of atomic charges, identify the
atoms in methyl–adenine and methyl–thymine that are likely to participate log A 7.36 8.37 8.3 7.47 7.25 6.73 8.52 7.87 7.53
in hydrogen bonds. (c) Draw all possible adenine–thymine pairs that can be S 3.53 4.24 4.09 3.45 2.96 2.89 4.39 4.03 3.80
linked by hydrogen bonds, keeping in mind that linear arrangements of the
W 1.00 1.80 1.70 1.35 1.60 1.60 1.95 1.60 1.60
A − H  B fragments are preferred in DNA. For this step, you may want to
use your molecular modelling software to align the molecules properly.
(d) Consult Impact 21 and determine which of the pairs that you drew in part (b) What should be the value of W for a drug with S = 4.84 and log A = 7.60?
(c) occur naturally in DNA molecules. (e) Repeat parts (a)–(d) for cytosine
O
and guanine, which also form base pairs in DNA.
HN
NH2 O N

N CH3
N HN N
N X C
N O
R R 10 TIBO
7 R = CH3, Methyl adenine 8 R = CH3, Methyl thymine I14.6 Consider the thermodynamic description of stretching rubber. The
observables are the tension, t, and length, l (the analogues of p and V for
I14.4 Molecular orbital calculations may be used to predict the dipole gases). Because dw = tdl, the basic equation is dU = TdS + tdl. If G =
moments of molecules. (a) Using molecular modelling software and the U − TS − tl, find expressions for dG and dA, and deduce the Maxwell relations
computational method of your or your instructor’s choice, calculate the dipole
moment of the peptide link, modelled as a trans-N-methylacetamide (9). Plot  S   t   S   l 
 l     T   t    T 
the energy of interaction between these dipoles against the angle θ for  T  l  T  t
r = 3.0 nm in the arrangement shown in structure 4 of Topic 14B. Go on to deduce the equation of state for rubber,
(b) Compare the maximum value of the dipole–dipole interaction energy
 U   t 
from part (a) to 20 kJ mol−1, a typical value for the energy of a hydrogen  l   t T  T 
bonding interaction in biological systems.  T  l
654 14 Molecular interactions

I14.7 Before attempting this problem, read Impact 21 avaiable via the O
e-book. Commercial software (more specifically ‘molecular mechanics’ H
or ‘conformational search’ software) automates the calculations that lead N
N
to Ramachandran plots. In this problem the model for the protein is the H
dipeptide (11) in which the terminal methyl groups replace the rest of the O R H
polypeptide chain. (a) Draw three initial conformers of the dipeptide with
R = H: one with ϕ = +75°, ψ = −65°, a second with ϕ = ψ = +180°, and a 11
third with ϕ = +65°, ψ = +35°. Use software of your or your instructor’s I14.8 The effective radius, a, of a random coil is related to its radius of
choice to optimize the geometry of each conformer and find the final ϕ and gyration, Rg, by a = γRg, with γ = 0.85. Deduce an expression for the osmotic
ψ angles in each case. Did all the initial conformers converge to the same virial coefficient, B (Topic 5B), in terms of the number of chain units for (a) a
final conformation? If not, what do these final conformers represent? (b) Use freely jointed chain, (b) a chain with tetrahedral bond angles. Evaluate B for
the approach in part (a) to investigate the case R = CH3, with the same three l = 154 pm and N = 4000. Estimate B for a randomly coiled polyethylene chain
initial conformers as starting points for the calculations. Rationalize any of arbitrary molar mass, M, and evaluate it for M = 56 kg mol−1. Hint: Use
similarities and differences between the final conformers of the dipeptides B = 12 N A vP , where vP is the excluded volume due to a single molecule.
with R = H and R = CH3.
FOCUS 15
Solids

This Focus explores the structures and physical properties of


solids. The solid state includes most of the materials that make
15C Bonding in solids
modern technology possible. It includes the wide varieties of
The constituents of solids are held together by a variety of in-
steel that are used in architecture and engineering, the semi-
teractions, which impart characteristic properties. This Topic
conductors and metallic conductors that are used in infor-
explores the interactions and prepares the ground for a discus-
mation technology and power distribution, the ceramics that
sion of the resulting properties.
increasingly are replacing metals, and the synthetic and natural
15C.1 Metals; 15C.2 Ionic solids; 15C.3 Covalent and molecular solids
polymers discussed in Focus 14 that are used in the textile in-
dustry and in the fabrication of many of the common objects of
the modern world.

15D The mechanical properties


of solids
15A Crystal structure
The characteristic mechanical properties of a solid include
The characteristic feature of a crystal is the regular arrangement various aspects of its rigidity. These properties are reported in
of its constituents. This Topic shows how that regularity is de- terms of several parameters that can be related to the structure
scribed in terms of the symmetry of the arrangement, and then of the solid.
explains how the arrangements are described quantitatively.
15A.1 Periodic crystal lattices; 15A.2 The identification of lattice planes

15E The electrical properties of solids


15B Diffraction techniques One very important property of a solid is its ability to transport
an electric current. This Topic explores how solids are classified
Diffraction techniques enable the structures of solids to be de- according to their electrical conductivity and how the different
termined in great detail. This Topic considers the basic prin- classes can be rationalized by using the ‘band theory’ of elec-
ciples of ‘X-ray diffraction’, and describes how the diffraction tronic structure. It goes on to show how the introduction of low
pattern can be interpreted in terms of the distribution of elec- concentrations of impurities can have a profound effect on the
tron density. The diffraction of electrons and neutrons adds to properties of semiconductors, and how this effect is exploited
the information that can be obtained about the structures of in- in making the semiconductor devices that are ubiquitous in
dividual molecules and atoms in solids. modern electronics.
15B.1 X-ray crystallography; 15B.2 Neutron and electron diffraction 15E.1 Metallic conductors; 15E.2 Insulators and semiconductors;
15E.3 Superconductors
15F The magnetic properties of solids respond nonlinearly, leading to useful phenomena such as
‘frequency doubling’.
15G.1 Excitons; 15G.2 Metals and semiconductors;
The magnetic properties of solids are reported in terms of
15G.3 Nonlinear optical phenomena
their ‘susceptibility’. If the magnetic centres are independent,
this property can be traced to individual atoms or molecules.
If the centres interact, properties such as ferromagnetism
emerge.
15F.1 Magnetic susceptibility; 15F.2 Permanent and induced magnetic
What is an application of this material?
moments; 15F.3 Magnetic properties of superconductors
The deployment of X-ray diffraction techniques for the deter-
mination of the location of all the atoms in biological macro-
molecules has revolutionized the study of biochemistry and
15G The optical properties of solids molecular biology. Impact 23, accessed via the e-book, dem-
onstrates the power of the techniques by exploring one of the
Spectroscopy is a key tool for exploring the electronic structure most seminal X-ray images of all: the characteristic pattern ob-
of solids. As well as exploring the electronic band structure of tained from strands of DNA and used in the construction of the
a solid material, spectroscopic observations give insight into ­double-helix model of DNA. Nanometre-sized assemblies that
phenomena that arise from the interactions present in such conduct electricity are currently of great technological interest,
materials. When subject to very intense radiation some ­solids and Impact 24, accessed via the e-book, describes their synthesis.

➤ Go to the e-book for videos that feature the derivation and interpretation of equations, and applications of this material.
Topic 15A Crystal structure

➤ Why do you need to know this material?


Crystalline solids are important in many technologies, and
to be able to account for their mechanical, electrical, opti-
cal, and magnetic properties you need to understand their Lattice point
microscopic structures.
Structural motif
➤ What is the key idea?
The regular arrangement of the atoms in periodic crystals
can be described in terms of unit cells.
Figure 15A.1 Each lattice point specifies the location of a
➤ What do you need to know already? structural motif (e.g. a molecule or a group of molecules). The
space lattice is the entire array of lattice points; the crystal
Light use is made of some of the language used to describe
structure is the collection of structural motifs arranged according
symmetry (Topic 10A).
to the lattice.

The internal structure of a crystal is a regular array of its atoms,


ions, or molecules. The features of this regular array, such as the
details of the stacking pattern and its characteristic dimensions,
are a crucial aspect of the link between the structure and prop-
erties of the solid.

15A.1 Periodic crystal lattices


A periodic crystal is built up from regularly repeating ‘struc- Figure 15A.2 A unit cell is a parallel-sided (but not necessarily
tural motifs’, which may be atoms, molecules, or groups of rectangular) figure from which the entire space lattice can be
constructed by using only translations (not reflections, rotations,
atoms, molecules, or ions. A space lattice is the pattern formed
or inversions). In the two-dimensional case shown here, each
by lattice points representing the locations of these motifs
lattice point is shared by four neighbouring cells.
(Fig. 15A.1). More formally, a space lattice is a three-dimen-
sional, infinite array of points, each of which is surrounded in
an identical way by its neighbours. In effect, a space lattice is straight lines, and such unit cells are described as primitive
an abstract scaffolding for the crystal structure. In some cases (Fig. 15A.3). If each of the four points of a two-dimensional
there may be a structural motif centred on each lattice point, unit cell in Fig. 15A.2 is regarded as shared with its four neigh-
but that is not necessary: all that is necessary is that an iden- bours, then the cell has only one lattice point overall; this is a
tical structural motif is associated with each lattice point. The requirement for a unit cell to be described as primitive. The
solids known as quasicrystals are ‘aperiodic’, in the sense that same definition applies in three dimensions, where each of the
the space lattice, though still filling space, does not have trans- eight points of a primitive unit cell is shared by eight neigh-
lational symmetry. This Topic deals only with periodic crystals. bours, giving one lattice point overall. It is often more con-
A unit cell is an imaginary parallelepiped (parallel-sided venient to draw larger non-primitive unit cells that also have
figure) from which the entire space lattice can be constructed lattice points at their centres or on pairs of opposite faces. An
by purely translational displacements (Fig. 15A.2). The cell is infinite number of different unit cells can describe the same
commonly formed by joining neighbouring lattice points by lattice, but the one with sides that have the shortest lengths and
658 15 Solids

• A monoclinic unit cell has one twofold rotation axis


(Fig. 15A.6).
• A triclinic unit cell has no rotational symmetry, and typi-
cally all three sides and angles are different (Fig. 15A.7).
Table 15A.1 lists the essential symmetries, the elements that must
be present for the unit cell to belong to a particular crystal system.
In three dimensions there are only 14 distinct space lat-
tices. The unit cells of these Bravais lattices are illustrated in
Fig. 15A.8. It is conventional to portray the lattices by primitive
unit cells in some cases and by non-primitive unit cells in oth-
Figure 15A.3 A primitive unit cell, an example of which is shown
ers. The following notation is used:
by the shaded volume, has lattice points only at its vertices. If each
of the eight points is regarded as shared with its eight neighbours, • A primitive unit cell (P) has lattice points only at the corners.
the unit cell has only one lattice point overall.
• A body-centred unit cell (I) also has a lattice point at its
centre.
a
b c β
α
γ a a
c b c
b γ β C2
α

Figure 15A.4 The notation for the sides and angles of a unit cell.
Note that the angle α lies in the plane (b,c). Figure 15A.6 A unit cell belonging to the monoclinic system has
a twofold rotation axis (denoted C2 and shown in more detail in
the insert).
that are most nearly perpendicular to one another is normally
chosen. The lengths of the sides of a unit cell are denoted a, b,
and c, and the angles between them are denoted α, β, and γ
(Fig. 15A.4).
Unit cells are classified into seven crystal systems by noting
the rotational symmetry elements they possess. For example:
• A cubic unit cell has four threefold rotation axes pointing
to the corners of a tetrahedron, and passing through the
centre of the cube (Fig. 15A.5).

C3 Figure 15A.7 A triclinic unit cell has no axes of rotational symmetry.


C3
C3 C3 Table 15A.1 The seven crystal systems⋆

System Essential symmetries


Triclinic None
Monoclinic One C2 axis
Orthorhombic Three perpendicular C2 axes
Rhombohedral One C3 axis
Tetragonal One C4 axis
Hexagonal One C6 axis

Figure 15A.5 A unit cell belonging to the cubic system has four Cubic Four C3 axes in a tetrahedral arrangement
threefold rotation axes, denoted C3, arranged tetrahedrally. The ⋆
Cn denotes an n-fold rotation axis, in which identical structures are obtained after rota-
insert shows the threefold symmetry. tion by 360°/n about that axis.
15A Crystal structure 659

a a C2

Cubic P Cubic I Cubic F C2

a b
a a
β
c c
Figure 15A.9 The two-dimensional lattice used in Brief
illustration 15A.1; a (non-primitive) unit cell is indicated by the
Tetragonal P Tetragonal I Monoclinic P Monoclinic C shaded area. The lattice points at the corners of the unit cell are
related by rotations about the twofold rotation axes of symmetry
a b
shown; the lattice point at the centre is unaffected by these
operations.
c

Orthorhombic P Orthorhombic C Orthorhombic I Orthorhombic F


a a a
a
b a
Exercise
γ 120° a ≠90° E15A.1 The orthorhombic unit cell of NiSO4 has the dimensions a = 634 pm,
c α β c
b = 784 pm, and c = 516 pm, and the mass density of the solid is estimated as
3.9 g cm−3. Identify the number of formula units in a unit cell and calculate a
more precise value of the mass density.
Triclinic Hexagonal Trigonal R

Figure 15A.8 The 14 Bravais lattices. The points are lattice points,
and are not necessarily occupied by atoms. P denotes a primitive
unit cell (R is used for a trigonal lattice), I a body-centred unit
The identification of lattice
15A.2
cell, F a face-centred unit cell, and C (or A or B) a cell with lattice planes
points on two opposite faces. Trigonal lattices may belong to the
rhombohedral or hexagonal systems (Table 15A.1). The interpretation of the diffraction techniques that are used
to measure the size of unit cells and the arrangement of mol-
ecules within them makes use of the orientation and sepa-
• A face-centred unit cell (F) has lattice points at its corners
ration of planes that pass through the crystal (Topic 15B).
and also at the centres of its six faces.
Two-dimensional lattices are easier to visualize than three-­
• A side-centred unit cell (A, B, or C) has lattice points at dimensional lattices, so in this discussion the concepts involved
its corners and at the centres of two opposite faces. in identifying lattice planes are introduced for two dimensions
For simple structures, it is often convenient to choose an atom be- initially, and then the results are extended by analogy to three
longing to the structural motif, or the centre of a molecule, as the dimensions. Note that lattice planes do not necessarily pass
location of a lattice point or the vertex of a unit cell, but that is not through lattice points.
a necessary requirement. Symmetry-related lattice points within
the unit cell of a Bravais lattice have identical surroundings.
15A.2(a) The Miller indices
Consider a two-dimensional rectangular lattice formed from a
Brief illustration 15A.1 unit cell of sides a and b (Fig. 15A.10). Each panel in the illus-
tration shows a set of evenly spaced planes that can be identified
The two-dimensional lattice shown in Fig. 15A.9 consists
of a rectangular array of lattice points, with one additional by considering, for each set, the plane lying closest to the origin
point at the centre of each rectangle; a (non-primitive) unit (but not passing through it) and then quoting the distances at
cell is indicated by the shading. This cell has twofold rotation which this plane intersects the a and b axes. These distances are:
axes passing through the mid-points of opposite sides of the (a) (1a,1b), (b)  12 a, 13 b , (c) (−1a,1b), and (d) (∞a,1b), with ∞
rectangle. Rotations about these axes interchange the lattice indicating that the plane is parallel to an axis and intersects it
points at the corners of the rectangle, but the lattice point at (notionally) at infinity. If it is agreed to quote distances along
the centre is not affected. It therefore follows that the lattice the axes as multiples of the corresponding dimensions of the
points at the corners are equivalent, but the lattice point at the unit cell, then these intersections can be expressed more simply
centre is distinct. as (1,1),  12 , 13 , (−1,1), and (∞,1), respectively. If the lattice in
660 15 Solids

a b

b
{110} {111}
a
(a) (b) a

c
{010} {111}

Figure 15A.11 Some representative planes in three dimensions


(c) (d)
and their Miller indices; the origin is indicated by the open circle.
Figure 15A.10 Some of the sets of equally spaced planes that can Note that the index 0 indicates that a plane is parallel to the
be drawn in a rectangular space lattice; the origin is indicated by corresponding axis, and that the indexing may also be used for
the open lattice point. The Miller indices {hkl} of each set of planes unit cells with non-orthogonal axes.
are: (a) {110}, (b) {230}, (c) { 110} , and (d) {010}.
Figure 15A.11 shows a three-dimensional representation
of a selection of planes, including one in a lattice with non-­
orthogonal axes.
Fig. 15A.10 is the top view of a three-dimensional orthorhom-
bic lattice, all four sets of planes intersect the c axis at infinity.
Therefore, in the three-dimensional case the labels are (1,1,∞), 15A.2(b) The separation of neighbouring
 12 , 13 , , (−1,1,∞), and (∞,1,∞). planes
The inconvenience of fractions and infinity in these labels The Miller indices are very useful for expressing the separation
can be avoided by specifying a plane by using its Miller indices, of neighbouring planes.
(hkl), where h, k, and l are the reciprocals of the intersection
distances along the a, b, and c axes, respectively. For example, How is that done? 15A.1 Deriving an expression for the
the plane  12 , 13 ,  has Miller indices (230). As will be seen, this
notation brings with it additional advantages. The Miller nota- separation of planes
tion has the following features: Consider the {hk0} planes of a square lattice built from a
unit cell with sides of length a (Fig. 15A.12). The separation
• Negative indices are written with a bar over the number,
between the lattice planes is equal to the perpendicular dis-
as in ( 110).
tance dhk0 from the (hk0) plane to the origin.
• The notation (hkl) denotes an individual plane. A set of Expressions for the sine and cosine of the angle φ are found
parallel planes with identical spacing is denoted {hkl}. by considering the sides of the two right-angle triangles shown
For example, in the figure:

dhk 0 hdhk 0 dhk 0 kdhk 0


sin    cos   
(a/h) a (a/k) a
Intersect axes at (a,b,∞c)  12 a, 13 b,c  (−a,b,∞c) (∞a,b,∞c)
Remove cell dimensions (1,1,∞)  12 , 13 ,  (−1,1,∞) (∞,1,∞) a
Take reciprocals (1,1,0) (2,3,0) (−1,1,0) (0,1,0)
Express as indices (110) (230) ( 110) (010) (hk0)

Sets of parallel planes {110} {230} {110} {010}


a
a/k
A helpful feature to remember is that the smaller the abso- dhk0
lute value of h in {hkl}, the more nearly parallel the set of planes ϕ
ϕ
is to the a axis (the {h00} planes are an exception). The same is
true of k and the b axis, and l and the c axis. When h = 0, the
a/h
planes intersect the a axis at infinity, so the {0kl} planes are par-
allel to the a axis. Similarly, the {h0l} planes are parallel to the b Figure 15A.12 The construction used to find the spacing of the
axis and the {hk0} planes are parallel to the c axis. (hk0) plane in a square unit cell.
15A Crystal structure 661

The length of the hypotenuse of the lower triangle is a/h Collect your thoughts For the first part, all you need to do is
because a Miller index h indicates that the plane intersects the a substitute the given values into eqn 15A.1b. You could do the
axis at a distance a/h from the origin. Likewise, the hypotenuse same for part (b), but note that the Miller indices for the sec-
of the upper triangle is a/k. Then, because sin2φ + cos2φ = 1, it ond set of planes are just twice those of the first part. By refer-
follows that ring to eqn 15A.1b you can see that multiplying the values of
h, k, and l by n gives the following expression for the spacing of
2 2
 hdhk 0   kdhk 0  the {nh nk nl} planes
    1
 a   a  1/dhkl 2
  
2
which can be rearranged by dividing both sides by dhk0 into 1 2 2
(nh) (nk) (nl )2
2h
2
k 2 l 2  n2
2
 2  2  2 n  2  2  2  2
dnh a b c  a b c  dhkl
1 h2  k 2 a ,nk ,nl

2
 or dhk 0 
d hk 0 a2 (h2  k 2 )1/ 2 which implies that
By extension to three dimensions, the separation of the {hkl} dhkl
dnh ,nk ,nl =
planes, dhkl, of a cubic lattice is given by n
The solution Substituting the indices into eqn 15A.1b gives
1 h2 + k 2 + l 2
2 =
dhkl a2 (15A.1a) 1 l2 22 32
a Separation of planes 2
    22.0nm 2
dhkl = 2 2 2 1/2 d123 (0.82 nm)2 (0.94 nm)2 (0.75 nm)2
(h + k + l ) [cubic lattice]

Hence, d123 = 0.21 nm. It then follows immediately that d246 is


one-half this value, or 0.11 nm.
The corresponding expression for a general orthorhombic lat-
tice (one in which the axes are mutually perpendicular, but not Self-test 15A.1 Calculate the separation of (a) the {133} planes
equal in length) is the generalization of this expression: and (b) the {399} planes in the same lattice.

1 h2 k 2 l 2
Answer: 0.19 nm, 0.063 nm
Separation of planes
2
 2 2 2 [orthorhombic latttice]
 (15A.1b)
dhkl a b c

Example 15A.1 Exercises


Using the Miller indices
E15A.2 State the Miller indices of the planes that intersect the crystallographic
Calculate the separation of (a) the {123} planes and (b) the axes at the distances (2a, 3b, 2c) and (2a, 2b, ∞c).
{246} planes of an orthorhombic unit cell with a = 0.82 nm, E15A.3 The unit cells of SbCl3 are orthorhombic with dimensions a = 812 pm,
b = 0.94 nm, and c = 0.75 nm. b = 947 pm, and c = 637 pm. Calculate the spacing, d, of the {321} planes.

Checklist of concepts
☐ 1. A periodic crystal is built up from regularly repeating ☐ 5. Unit cells are classified into seven crystal systems
structural motifs. according to their rotational symmetries: unit cells are
☐ 2. A space lattice is the pattern formed by points (the classified as cubic, monoclinic, or triclinic according to
lattice points) representing the locations of structural the essential symmetries they possess.
motifs (atoms, molecules, or groups of atoms, mol- ☐ 6. The Bravais lattices are the 14 distinct space lattices in
ecules, or ions). three dimensions (Fig. 15A.8).
☐ 3. A unit cell is an imaginary parallel-sided figure from ☐ 7. The unit cells of the Bravais lattices are classed as primi-
which the entire space lattice can be constructed by tive (P), body-centred (I), face-centred (F), and side-
purely translational displacements. centred (A, B, or C).
☐ 4. A primitive unit cell has lattice points only at its vertices ☐ 8. A lattice plane is specified by a set of Miller indices
and only one lattice point overall; non-primitive unit (hkl); sets of planes are denoted {hkl}.
cells also have lattice points at their centres or on pairs
of opposite faces.
662 15 Solids

Checklist of equations

Property Equation Comment Equation number


Separation of planes in a cubic lattice 2
1/dhkl  (h2  k 2  l 2 )/a 2 h, k, and l are Miller indices 15A.1a
2
Separation of planes in an orthorhombic lattice 1/dhkl  h2 /a 2  k 2 /b2  l 2 /c 2 15A.1b
TOPIC 15B Diffraction techniques

for structural studies of solid materials. The actual process of


➤ Why do you need to know this material? going from the observed diffraction pattern to a structure is
To account for the properties of solids it is necessary to rather involved, but such is the degree of integration of com-
understand their detailed structures and how they are puters into the experimental apparatus that the technique is
determined by a variety of diffraction techniques. almost fully automated, even for large molecules and com-
plex solids. The analysis is aided by molecular modelling tech-
➤ What is the key idea? niques, which can guide the investigation towards a plausible
The regular arrangement of the atoms in periodic crystals structure.
can be determined by techniques based on diffraction. X-rays are electromagnetic radiation with wavelengths of the
order of 10−10 m. They are typically generated by bombarding a
➤ What do you need to know already? metal with high-energy electrons (Fig. 15B.1). The electrons de-
You need to be familiar with the description of crystal celerate as they plunge into the metal and generate radiation with a
structures and the use of Miller indices to identify lattice continuous range of wavelengths called Bremsstrahlung (Bremse
planes (Topic 15A). You also need to be familiar with the is German for deceleration, Strahlung for ray). Superimposed on
wave description of electromagnetic radiation (The chem- the continuum are a few high-intensity, sharp peaks (Fig. 15B.2).
ist’s toolkit 7A.1 and The chemist’s toolkit 7A.2), and the basic These peaks arise from collisions of the incoming electrons with
properties of the Fourier transform (The chemist’s toolkit the electrons in the inner shells of the atoms. The collision expels
12C.1). Light use is made of the de Broglie relation (Topic an electron from an inner shell, and an electron of higher energy
7A) and the equipartition theorem (Energy: A first look). drops into the vacancy, emitting the excess energy as an X-ray
photon (Fig. 15B.3). If the electron falls into a K shell (a shell with
n = 1), the X-rays are classified as ‘K-radiation’, and similarly for
transitions into the L (n = 2) and M (n = 3) shells. Strong, distinct
lines are labelled Kα, Kβ, and so on. Synchrotrons (Topic 11A) gen-
Diffraction techniques can be used to determine the details of erate high-intensity X-ray radiation which is increasingly used in
the arrangement of ions, atoms, and molecules in a crystalline diffraction experiments on account of the resulting greater inten-
solid to high precision. Such techniques are now so well de- sity in the diffraction pattern, and hence higher sensitivity.
veloped that both the collection of the diffraction data and its An early method of observing diffraction consisted of pass-
interpretation in terms of a structure are automated to a high ing a beam containing X-rays with a range of wavelengths into
degree. a single crystal, and recording the diffraction pattern pho-
tographically. The idea behind this approach is that a crystal
might not be suitably orientated to cause diffraction for a sin-
gle wavelength but, whatever its orientation, diffraction would
15B.1 X-ray crystallography be achieved for at least one of the wavelengths present in the

As explained in The chemist’s toolkit 7A.2, a characteristic prop-


erty of waves is that when they are present in the same region of X-rays
space they interfere with one another. A greater displacement
is obtained where peaks or troughs of the waves coincide and
a smaller displacement where peaks coincide with troughs. Cooling water Electron
Diffraction is the interference caused by an object in the path of beam
waves; it occurs when the dimensions of the diffracting object
Metal
are comparable to the wavelength of the radiation. target
Beryllium X-rays
window
15B.1(a) X-ray diffraction
Figure 15B.1 X-rays are generated by directing an electron beam
X-rays diffract when passed through a crystal because their on to a cooled metal target. Beryllium is transparent to X-rays (on
wavelengths are comparable to the separation of lattice planes. account of the small number of electrons in each atom) and is
Consequently, X-ray diffraction is a very powerful technique used for the windows.
664 15 Solids


Calcite


Intensity

Bremsstrahlung
Aragonite

Wavelength, λ 20 30 40 50 60 70 80
Glancing angle, 2θ/°
Figure 15B.2 The X-ray emission from a metal consists of a
Figure 15B.4 X-ray powder diffraction patterns of two
broad, featureless Bremsstrahlung background, with sharp peaks
polymorphs of CaCO3, calcite and aragonite. The patterns are
superimposed on it. The label K indicates that the radiation comes
distinctive and can be used to identify the polymorph present in
from a transition in which an electron falls into a vacancy in the K
an unknown sample.
shell of the atom.

Ejected X-ray
electron Ω
Ionization
χ
Electron
ϕ
beam
Sample
Energy

X-ray
L beam 2θ

K To
detector

Figure 15B.3 The processes that contribute to the generation


of X-rays. An incoming electron collides with an electron (in the Figure 15B.5 A four-circle diffractometer. The settings of the
K shell), and ejects it. Another electron (from the L shell in this orientations (φ, χ, θ, and Ω) of the components are controlled by
illustration) falls into the vacancy and emits its excess energy as an computer; each (hkl) reflection is monitored in turn, and their
X-ray photon. intensities are recorded.

beam. There is currently a resurgence of interest in this ap- The method developed by the Braggs (William and his son
proach because synchrotron radiation spans a range of X-ray Lawrence) is the foundation of almost all modern work in
wavelengths. X-ray crystallography. They used a single crystal and a mono-
A more common technique uses monochromatic radiation chromatic beam of X-rays, and rotated the crystal until a reflec-
and a powdered sample, which consists of many tiny crystal- tion was detected. There are many different sets of planes in a
lites, oriented at random. At least some of the crystallites will crystal, so there are many angles at which a reflection occurs.
be appropriately orientated to give diffraction. In modern The complete set of data consists of the list of angles at which
‘powder diffractometers’ the intensities of the reflections are reflections are observed together with their intensities.
monitored electronically as the detector is rotated around the Single-crystal diffraction patterns are measured by using a
sample in a plane containing the incident ray. Powder diffrac- ‘four-circle diffractometer’ (Fig. 15B.5). Once the dimensions
tion techniques are used to identify the composition of a sam- and symmetry of the unit cell of the crystal in question have
ple of a solid substance by comparison of the positions of the been identified, the angular setting of the detector on the four
diffraction lines and their intensities with previously recorded circles is adjusted so that the precise position and intensity of
patterns of known structures (Fig. 15B.4); large databases of each peak in the diffraction pattern can be measured. Modern
such information are available. This approach can be used to instruments use area detectors and image plates, which sam-
determine the composition of mixed phases, and hence to con- ple whole regions of diffraction patterns simultaneously, rather
struct a phase diagram. The technique is also used for the initial than peak by peak, thus increasing the speed with which data is
determination of the dimensions and symmetries of unit cells. collected.
15B Diffraction techniques 665

15B.1(b) Bragg’s law   2d sin Bragg’s law [alternative form] (15B.1b)

An early approach to the analysis of diffraction patterns pro- and to regard the nth-order reflection as arising from the {nh
duced by crystals was to regard a lattice plane as a semi-trans- nk nl} planes. As established in Example 15A.1 in Topic 15A,
parent mirror and to model a crystal as a stack of reflecting the spacing of the {nh nk nl} planes is dhkl/n, where dhkl is the
lattice planes of separation d (Fig. 15B.6). The model makes it spacing of the {hkl} planes. The primary use of Bragg’s law is in
easy to calculate the angle the crystal must make to the incom- the determination of the spacing between the layers in the lat-
ing beam of X-rays for constructive interference to occur. It tice because d may readily be calculated from a measured value
has also given rise to the name reflection to denote an intense of the angle θ.
beam arising from constructive interference.
Consider the reflection of two parallel rays of the same wave-
length and phase by two adjacent planes of a lattice, as shown Brief illustration 15B.1
in Fig. 15B.6. One ray strikes point D on the upper plane and
the phase of the parallel ray at point A is the same as that of the A first-order reflection from the {111} planes of a cubic crystal
first ray at point D. The parallel ray must travel an additional was observed at a glancing angle, 2θ, of 22.4° when X-rays
of wavelength 154 pm were used. According to eqn 15B.1b,
distance AB before striking the plane immediately below. The
the {111} planes responsible for the diffraction have separation
reflected rays also differ in path length by the distance BC. As
d111 = λ/(2 sin θ), therefore
is evident from the inset in Fig. 15B.6, both the lengths AB and
BC are d sin θ; the total path length difference of the two rays  154 pm
is then d111    396 pm
2 sin 2 sin11.2
AB  BC  2d sin
The separation of the {111} planes of a cubic lattice of side a is
where 2θ is the glancing angle (2θ rather than θ, because the given by eqn 15A.1a in Topic 15A as d111 = a/31/2. It therefore
beam is deflected through 2θ from its initial direction). For follows that a =31/2d111 = 687 pm.
many glancing angles the path-length difference is not an in-
teger number of wavelengths, and the waves interfere largely
destructively. However, when the path-length difference is an
integer number of wavelengths (AB + BC = nλ), the reflected Some types of unit cell give characteristic patterns of lines.
waves are in phase and interfere constructively. It follows that a In a cubic lattice with dimension a, the spacing of the {hkl}
reflection should be observed when θ satisfies Bragg’s law: planes, dhkl, is given by eqn 15A.1a (dhkl = a/(h2 + k2 + l2)1/2); the
angles at which the {hkl} planes give first-order reflections are
n  2d sin Bragg’s law  (15B.1a)
given by
Reflections with n = 2, 3,… are called second order, third order,
and so on; they correspond to path-length differences of 2, 3,… dhkl = a/(h 2 + k 2 + l 2)1/2
wavelengths. In modern work it is normal to absorb the n into λ λ
d, to write Bragg’s law as sinθ = = (h 2 + k 2 + l 2 )1/2
2dhkl 2a

Not all integral values of h2 + k2 + l2 are obtained when integer


values of the indices are substituted:
θ θ

d D {hkl} {100} {110} {111} {200} {210}


A θ C
h2 + k2 + l2 1 2 3 4 5
d
{hkl} {211} {220} {300} {221} {310}
θ h2 + k2 + l2 6 8 9 9 10

B
Notice that h2 + k2 + l2 = 7 (and 15,…) does not appear. As a
Figure 15B.6 The conventional derivation of Bragg’s law treats result, in the pattern of diffraction lines there is a larger gap
each lattice plane as a plane reflecting the incident radiation. between the {211} and {220} reflections than between nearby
The path lengths for reflection from adjacent planes differ by lines, and likewise between {321} (for which h2 + k2 + l2 = 14)
AB + BC, which depends on the angle θ. Constructive interference and {400} (for which h2 + k2 + l2 = 16). The absence of these lines
(a ‘reflection’) occurs when AB + BC is equal to an integer number leads to a characteristic pattern which helps in identifying the
of wavelengths. type of unit cell.
666 15 Solids

40 corresponds to sin θ = 0. That is, the scattering factor has its


maximum value in the forward direction. In this limit eqn
Br–
15B.2 simplifies to
Scattering factor, f(θ)

30
 1
Fe2+
 sin kr 2
20 f (0)  lim 4   (r ) r dr
Ca2+ kr  0 0 kr
volume
 element
 
10
Cl–  4   (r )r dr    (r ) 4 r 2dr
2

O 0 0

C
H The factor 4πr2dr is the volume of a spherical shell of radius r
0
0 0.2 0.4 0.6 0.8 1 1.2 and thickness dr. Multiplication of this volume by the electron
(sin θ)/λ density at r gives ρ(r)4πr2dr which is the number of electrons in
Figure 15B.7 The variation of the scattering factor of atoms and
this shell. Integration over the full range of r then gives the total
ions with atomic number and angle. The scattering factor in the number of electrons in the atom. Hence f (0) = Ne. For example,
forward direction (at θ = 0, and hence at (sin θ)/λ = 0) is equal to the scattering factors of Na+, K+, and Cl− in the forward direc-
the number of electrons present in the species. tion are 8, 18, and 18, respectively.

15B.1(c) Scattering factors 15B.1(d) The electron density


X-ray scattering is caused by the oscillations an incoming elec- The structure factor, Fhkl, is the net amplitude of a given {hkl}
tromagnetic wave generates in the electrons of atoms. Heavy, reflection that takes into account the positions and types of all
electron-rich atoms give rise to stronger scattering than light the atoms in the unit cell. It can be expressed in terms of the
atoms. This dependence on the number of electrons is ex- locations and scattering factors of the atoms.
pressed in terms of the scattering factor, f, of the element. If the
scattering factor is large, then the atoms scatter X-rays strongly.
It turns out that, in a spherically symmetrical atom, the scatter- How is that done? 15B.2 Relating the structure factor to
ing factor for the atom is related to the electron density, ρ(r), the location of the atoms and their scattering factors
and the glancing angle, 2θ, by
Suppose that a unit cell contains several atoms with scattering
 sin kr 2 4 factors fj and coordinates (xja,yjb,zjc), where xj is the coordinate
f ( )  4   (r ) r dr k sin of the atom j in the a direction, expressed as a fraction of the
0 kr 
length a, and likewise for the other coordinates.
Scattering factor  (15B.2)
Step 1 Consider the (h00) reflection with h = 1
The scattering factor is greatest in the forward direction (θ = 0, The reflection shown in Fig. 15B.8 corresponds to two waves
Fig. 15B.7), and it can be shown that in that direction it is equal from adjacent A planes; the phase difference of the waves is 2π.
to the total number of electrons in the atom, Ne. If there is a B atom at a fraction x of the distance between the
two A planes, then it gives rise to a wave with a phase differ-
ence 2πx relative to an A reflection. To understand this point,
How is that done? 15B.1 Evaluating the scattering factor note that, if x = 0, there is no phase difference; if x = 12 the phase
difference is π; if x = 1, the atom in the B plane (a ‘B atom’) lies
in the forward direction
where the atom in the lower A plane (an ‘A atom’) is and the
The first step is to examine the function (sin kr)/kr in the limit phase difference is 2π.
that kr → 0. Use sin x  x  16 x 3   and write
Step 2 Consider the (h00) reflection with h = 2
sin kr 3
kr  (kr )  
1
In this case, there is a 2 × 2π phase difference between the
lim
kr  0 kr
 lim
kr  0
6

kr kr  0

 lim 1  16 (kr )2    1  waves from the two A layers, and if B were to lie at x = 12 it
would give rise to a wave that differed in phase by 2π from
The point kr = 0 is a maximum which you can see by inspect- the wave from the lower A layer. Thus, for a general fractional
ing the first two terms in the expansion, 1 − 16 (kr )2. As kr moves position x, the phase difference for a (200) reflection is 2 × 2πx.
away from zero the value decreases, so kr = 0 is a maximum.
In fact it turns out to be the global maximum, which is most Step 3 Generalize these conclusions
easily verified simply by plotting the function. It therefore fol- For a general (h00) reflection, the phase difference is h × 2πx. For
lows that the scattering factor is a maximum for kr = 0, which three dimensions, this result generalizes to φhkl = 2π(hx + ky + lz).
15B Diffraction techniques 667

Phase Phase Collect your thoughts You need to evaluate the sum in eqn
difference = 2πx difference = 2 × 2πx 15B.3. The sum is over all atoms in the unit cell, so you need
to know the location of each atom, expressed as a fraction of
the unit cell parameters. Several of the atomic coordinates are
A A already marked in the figure. Note that:
xa xa
a a
• The atoms at the corners of the cube are shared between
B B eight adjacent unit cells, so in the calculation of the struc-
ture factor these atoms are given a weight of 18 and their
A A scattering factors taken to be 18 f .
Phase Phase • The atoms on the faces are shared between two cells
(a) difference = 2π (b) difference = 2 × 2π and so have a weight of 12 ; those on the edges are shared
Figure 15B.8 Diffraction from a crystal containing two kinds of between four cells, and so have a weight of 14 .
atoms. (a) For a (100) reflection from the A planes, there is a phase Then follow these steps:
difference of 2π between waves reflected by neighbouring planes.
(b) For a (200) reflection, the phase difference is 4π. The reflection • Write f + for the Na+ scattering factor and f −
for the Cl−
from a B plane at a fractional distance xa from an A plane has a scattering factor.
phase that is x times these phase differences. • For simplicity, ignore the fact that scattering occurs in
non-forward directions and suppose that all the Na+ have
Step 4 Formulate the total amplitude of the scattered waves the same scattering factor, and likewise for the Cl− ions.
If the amplitude of the waves scattered from A is fA at the detec- • The best way to proceed is to draw up a table showing the
tor, that of the waves scattered from B with phase difference φhkl weights, positions, and phases.
is f Beiφhkl . The total amplitude at the detector is therefore
• The phase factors eiφ are evaluated by noting that h, k,
hkl

Fhkl  f A  f Beihkl and l are integers. A useful identity is that einπ is +1 for
even n, and −1 for odd n, more succinctly expressed as
When there are several atoms present, each with scattering fac- ein  (1)n .
tor fj and phase φhkl(j) = 2π(hxj + kyj + lzj), the total amplitude
of the (hkl) reflection, the structure factor, is The solution The table for the Na+ ions is

Fhkl = ∑ f j e
iϕ hkl ( j )
(15B.3) Atom Weight x y z ϕhkl
j Structure factor
1 1
8
0 0 0 0
2 1
8
1 0 0 2πh
3 1
8
0 1 0 2πk
Example 15B.1  Calculating a structure factor 4 1
0 0 1 2πl
8

5 1
1 1 0 2π(h + k)
Calculate the structure factor for the unit cell of NaCl depicted 8

in Fig. 15B.9. 6 1
8
1 0 1 2π(h + l)
7 1
8
0 1 1 2π(k + l)
8 1
8
1 1 1 2π(h + k + l)
Na+
9 1
2
1
2
1
2
0 2  12 h  12 k 
(1,0,1)
(1,1,1) 10 1
2
1
2
0 1
2 2  12 h  12 l 
11 1
2
0 1
2
1
2 2  12 k  12 l 
Cl

12 1
2
1 1
2
1
2 2  h  12 k  12 l 
13 1
2
1
2
1 1
2 2  12 h  k  12 l 
14 1
2
1
2
1
2
1 2  12 h  12 k  l 
(0,0,0)
(½,0,0)
The phase factors for the first eight Na+ ions in the table are all
(½,½,0) +1, and as they each have a weight of 18 , the total contribution
(1,0,0) (1,1,0) to the structure factor is f +. The remaining six ions all have
weight 12 , and their contribution to the structure factor is
Figure 15B.9 The location of the atoms for the structure factor
calculation in Example 15B.1. The small spheres are Na+, the
1
2
[e i 2  ( h / 2  k / 2 )  e i 2  ( h / 2  l / 2 )  e i 2  ( k / 2  l / 2 )  e i 2  ( h  k / 2  l / 2 )
large spheres are Cl−.  ei 2 (h / 2  k  l / 2)  ei 2 (h / 2  k / 2 1) ]
668 15 Solids

The terms ei2πh , ei2πk , and ei2πl are all +1, so the last three terms Comment. If f + = f −, which is the case for identical atoms, the
can be simplified hkl all-odd reflections have zero intensity; such a structure
would be a cubic P lattice with lattice parameter a/2.
1
2
[e i 2  ( h / 2  k / 2 )  e i 2  ( h / 2  l / 2 )  e i 2  ( k / 2  l / 2 )  e i 2  ( k / 2  l / 2 ) Self-test 15B.1 Which reflections cannot be observed for a
 ei 2 ( h / 2  l / 2 )  ei 2 ( h / 2  k / 2 ) ] cubic I lattice?
Answer: for h + k + l odd, Fhkl = 0
A further simplification is to use ein  (1)n :

1
2
[(1)h  k  (1)h  l  (1)k  l  (1)k  l  (1)h  l  (1)h  k ]
 (1)h  k  (1)h  l  (1)k  l The intensity of a reflection is proportional to the square
modulus of the amplitude of the wave, which is in turn pro-
The overall contribution of the Na+ ions to the structure factor portional to the structure factor, Fhkl. If the structure factor is
is therefore Fhkl  f A  f B eihkl , the intensity, Ihkl, is
f [1  (1)h  k  (1)h  l  (1)k  l ]
* Fhkl = ( f A + fBe
I hkl  Fhkl
− iϕ hkl
)( fA + fBeiϕ ) hkl

The table for the Cl− ions is


iϕ hkl
= fA2 + fB2 + fA fB(e + e − iϕ )
hkl

eiϕ + e−iϕ = 2cos ϕ


Atom Weight x y z ϕhkl
1 1
4
1
2
0 0 πh = f A2 + f B2 + 2 fA fB cos ϕhkl
2 1
4
0 1
2
0 πk
3 1
4
1 1
2
0 2  h  12 k  The cosine term either adds to or subtracts from f A2 + f B2 de-
4 1
4
1
2
1 0 2  h  k 
1
2 pending on the value of φhkl, which in turn depends on h, k, and
5 1
4
0 0 1
2
πl l and x, y, and z. Hence, there is a variation in the intensities of
6 1
4
1 0 1
2 2  h  12 l  the reflections with different hkl. The A and B reflections in-
7 1
0 1 1
2  k  12 l  terfere destructively when the phase difference is π, and in this
4 2

8 1
1 1 1
2  h  k  12 l 
case the total intensity is zero if the atoms have the same scatter-
4 2
ing power. For example, if the unit cell is cubic I with a B atom
9 1 1 1 1
2  12 h  12 k  12 l 
2 2 2
at x= y= z = 12 , then the A,B phase difference is (h + k + l)π.
10 1 1
0 1 2  12 h  l 
4 2
Therefore, all reflections for odd values of h + k + l vanish if A
11 1
4
0 1
2
1 2  12 k  l  and B are identical atoms because the waves from A and B are
12 1
4
1 1
2
1 2  h  12 k  l  displaced in phase by π.
13 1
4
1
2
1 1 2  12 h  k  l  For a cubic P lattice diffraction is possible for all {hkl}, there-
fore the diffraction pattern for a cubic I lattice can be con-
structed from that for cubic P by striking out all reflections
A similar procedure to that used for the Na+ ions gives the fol- with odd values of h + k + l. Similarly, for a cubic F lattice the
lowing contribution of the Cl− ions to the structure factor missing lines are ones with two out of h, k, and l odd, and the
remaining one even, or two even and one odd. Recognition of
f [(1)h  k  l  (1)h  (1)k  (1)l ] these systematic absences in a powder spectrum can be used to
The structure factor is therefore assign the lattice type (Fig. 15B.10).
The intensity of the {hkl} reflection is proportional to |Fhkl|2,
Fhkl  f [1  (1)h  k  (1)h  l  (1)k  l ] so in principle the structure factors can be determined experi-
 f [(1)h  k  l  (1)h  (1)k  (1)l ] mentally by taking the square root of the corresponding inten-
sities (but see below). Then, once all the structure factors Fhkl
Now note that: are known the electron density distribution, ρ(r), in the unit
cell can be calculated by using
• if h, k, and l are all even, Fhkl = f +{1 + 1 + 1 + 1} +
f −{1 + 1 + 1 + 1} = 4( f + + f −) 1
• if h, k, and l are all odd, Fhkl = 4( f − f ) + −
 (r )  Fhkl e2i(hx  ky lz )
V hkl
Fourier synthesis  (15B.4)

• if one index is odd and two are even, or vice versa, Fhkl = 0 where V is the volume of the unit cell. Equation 15B.4 is called
The hkl all-odd reflections are therefore less intense than the a Fourier synthesis of the electron density. Fourier transforms
hkl all even, and some of the reflections are absent. occur throughout chemistry in a variety of guises, and are
15B Diffraction techniques 669

(221)(300)

(410)(322)

(502)(430)
(411) (330)
(a)

(200)

(400)
(220)

(222)
(320)
(321)

(331)
(420)
(421)
(332)
(422)
(100)

(210)

(310)
(211)

(311)
(110)

(111)
(b)

Electron density, ρ(x)


Cubic P

Cubic I
0

Cubic F 0 0.5 1
x

Figure 15B.10 The powder diffraction patterns and the systematic Figure 15B.11 The plot of the electron density calculated in
absences of three versions of a cubic cell as a function of angle: Example 15B.2 (thick) and Self-test 15B.2 (thin). Interact with the
cubic F (fcc; h, k, l all even or all odd are present), cubic I (bcc; dynamic version of this graph in the e-book.
h + k + l = odd are absent), cubic P. Comparison of the observed
pattern with patterns like these enables the unit cell to be
identified. The locations of the lines give the cell dimensions. Only 15 values of Fh are given, so ρ(x) will be approximate: the
result (computed using mathematical software) is plotted in
­ escribed in more detail in The chemist’s toolkit 12C.1. The essence
d Fig. 15B.11 (thick line). There are three clear maxima in this
of the procedure in this case is to express the varying electron den- function, which can be identified as the positions of three atoms.
sity in a unit cell as a superposition of sine and cosine waves.
Comment. The more terms that are included (meaning the
more reflections that are measured), the more accurate is the
Example 15B.2 density plot. Terms corresponding to high values of h (which
Calculating an electron density by Fourier correspond to short-wavelength cosine terms in the sum)
synthesis account for the finer details of the electron density; low values
Consider the {h00} planes of a crystal extending indefinitely in of h account for the broad features.
the x direction. In an X-ray analysis the structure factors were Self-test 15B.2 Use mathematical software to experiment with
found as follows: the result of altering the structure factors in the table: con-
h 0 1 2 3 4 5 6 7 8 9 sider the effect of both changing the signs and amplitudes. For
example, use the same values of Fh as above, but change all the
Fh 16 −10 2 −1 7 −10 8 −3 2 −3
signs for h ≥ 6.
h 10 11 12 13 14 15 Answer: Fig. 15B.11 (thin line)
Fh 6 −5 3 −2 2 −3

It was also found that F−h = Fh. Construct a plot of the electron
density projected on to the x-axis of the unit cell.
15B.1(e) The determination of structure
Collect your thoughts You need to substitute these values into
eqn 15B.4, but because the problem is one-dimensional, the
The structure factors used in eqn 15B.4 to compute the electron
sum is over only the index h and only the terms e 2ihx need be density are in general complex quantities that can be written
considered. |Fhkl|eiα, where |Fhkl| is the amplitude and α the phase. However,
the observed intensity Ihkl is proportional to the square modu-
The solution Because F−h = Fh, the sum, rather than running lus of the structure factor, |Fhkl|2, so from the experiment no in-
from h = −∞ to +∞, can be written as running from 1 to +∞: formation is available about the phase, which may lie anywhere
= Fh from 0 to 2π. This ambiguity is called the phase problem; its
} consequences are illustrated by comparing the two plots in
V ρ( x ) = ∑F h
−2 πihx
= F0 + ∑(F e h
−2πihx
+ F− h e 2 πihx )
Fig. 15B.11 in which the phases of the structure factors have
h=− h=1

been changed but the amplitudes kept the same. Some way
= F0 + ∑F (e h
−2 πihx
+ e 2 πihx ) must be found to assign phases to the structure factors, because
h=1
unless these are known ρ cannot be evaluated using eqn 15B.4.
e ix + e−i x = 2cos x
The phase problem is less severe for centrosymmetric unit cells,
= F0 + 2∑Fh cos πhx for then the structure factors are real. It still remains a problem
h=1 though, to decide whether Fhkl is positive or negative.
670 15 Solids

The phase problem can be overcome to some extent by a of each peak, relative to the origin, corresponds to the separa-
variety of methods. One procedure that is widely used for in- tion and relative orientation of a pair of atoms in the cell. Note
organic materials with a reasonably small number of atoms in that the Patterson map is centrosymmetric and has a strong
a unit cell, and for organic molecules with a small number of feature at the origin.
heavy atoms, is the Patterson synthesis. Instead of the structure
factors Fhkl, the values of |Fhkl|2, which can be obtained without
ambiguity from the intensities, are used in an expression that Heavy atoms dominate the scattering because their scatter-
resembles eqn 15B.4: ing factors are large, of the order of their atomic numbers, and
1 their locations may be deduced quite readily. The sign of Fhkl
V
P (r )  | Fhkl |2 e 2 i (hx ky lz ) Patterson synthesis (15B.5) can now be calculated from the known locations of the heavy
hkl
atoms in the unit cell, and to a high probability the phase calcu-
where the r are the vector separations between the atoms in the
lated for them will be the same as the phase for the entire unit
unit cell; that is, the distances and directions between atoms.
cell. To see why, consider a centrosymmetric cell for which each
Whereas the electron density function ρ(r) is the probability
term in the structure factor is either positive or negative. The
density of the positions of atoms, the function P(r) is a map of
structure factor has the form
the probability density of the separations between atoms; P(r)
is often called the Patterson map. In such a map, a peak at a F  () f heavy  () f light  () f light   (15B.6)
position specified by a vector r from the origin arises from pairs
of atoms that are separated by r. Thus, if atom A is at the coor- where fheavy is the scattering factor of the heavy atom and flight
dinates (xA,yA,zA) and atom B is at (xB,yB,zB), then there will be the scattering factors of the light atoms. The flight are all much
a peak at (xA − xB, yA − yB, zA − zB) in the Patterson map. There smaller than fheavy, and their phases are more or less random if
will also be a peak at the negative of these coordinates, because the atoms are distributed throughout the unit cell. Therefore,
there is a separation vector from B to A as well as a separation the net effect of the flight is to change F only slightly from fheavy,
vector from A to B. The height of the peak in the map is propor- and with reasonable confidence F will have the same sign as
tional to the product of the atomic numbers of the two atoms, that calculated from the location of the heavy atoms. This phase
ZAZB. The Patterson map also shows a strong feature at its ori- can then be combined with the observed |F| (from the reflec-
gin arising from the separation between each atom and itself, tion intensity) to perform a Fourier synthesis of the full elec-
which is necessarily zero. tron density in the unit cell, and hence to locate the light atoms
as well as the heavy atoms.
Modern structural analyses make extensive use of direct
methods. Direct methods are based on the possibility of treat-
Brief illustration 15B.2
ing the atoms in a unit cell as being virtually randomly dis-
For the electron density shown in Fig. 15B.12a, the corre- tributed (from the radiation’s point of view), and then to use
sponding Patterson map is shown in Fig. 15B.12b. The location statistical techniques to compute the probabilities that the
phases have a particular value. It is possible to deduce rela-
tions between some structure factors and sums (and sums of
squares) of others, which have the effect of constraining the
phases to particular values (with high probability, so long as the
R1
R4
structure factors are large). For example, the Sayre probability
R4
R3 relation has the form
R2 R1
R2 sign of Fh  h,k  k ,l  l  is probably equal to (sign of Fhkl )
R3 –R1 –R2  (sign of Fhk l  ) Sayre probability relation  (15B.7)
–R3
–R4 For example, if F122 and F232 are both large and negative, then it
is highly likely that F354, provided it is large, will be positive.
In the final stages of the determination of a crystal structure,
(a) (b) the parameters describing the structure (atom positions, for in-
stance) are adjusted systematically to give the best fit between
Figure 15B.12 The Patterson map corresponding to the electron
density in (a) is shown in (b). The distance and orientation of each
the observed intensities and those calculated from the model
spot from the origin gives the orientation and separation of one of the structure deduced from the diffraction pattern. This pro-
atom–atom separation in (a); in addition, there is a large spot at cess is called structure refinement. Not only does the proce-
the origin. Some of the typical distances and their contribution to dure give accurate positions for all the atoms in the unit cell,
(b) are shown as R1, etc. but it also gives an estimate of the errors in those positions and
15B Diffraction techniques 671

in the bond lengths and angles derived from them. The proce- h

dure also provides information on the vibrational amplitudes (mkT )1/ 2
of the atoms.
Therefore, at 373 K,

Exercises
6.626 ×10−34 Js
E15B.1 What are the values of the angle θ of the three diffraction lines with λ=
{(1.675 ×10−27 kg) × (1.381×10−23 JK −1 ) × (373 K)}1/2
the smallest θ expected from a cubic I unit cell with lattice parameter 291 pm
when the X-ray wavelength is 72 pm? Hint: Are all reflections possible for
1 J = 1kg m 2s −2
such a unit cell?
6.626 ×10−34 kgm 2 s −1
E15B.2 What is the value of the scattering factor in the forward direction for Br ?

=
(1.675 ×10−27 ×1.381×10−23 × 373)1/2 (kg 2 m 2 s −2 )1/2
E15B.3 The coordinates, in units of a, of the atoms in a cubic P unit cell are
(0,0,0), (0,1,0), (0,0,1), (0,1,1), (1,0,0), (1,1,0), (1,0,1), and (1,1,1). Calculate = 2.26 ×10−10 m = 226 pm
the structure factor Fhkl when all the atoms are identical. Where possible,
simplify your expression by using einπ = (−1)n, as in Example 15B.1.
Self-test 15B.3 Calculate the temperature needed for the aver-
age wavelength of the neutrons to be 100 pm.

Neutron and electron


Answer: 1900 K
15B.2
diffraction
Neutrons and electrons can also give rise to diffraction on ac- Neutron diffraction differs from X-ray diffraction in two
count of their wave nature; their wavelength is given by the de main respects. First, the scattering of neutrons is a nuclear
Broglie relation (Topic 7A, λ = h/p). Neutrons generated in a phenomenon. Neutrons pass through the extranuclear elec-
nuclear reactor and then slowed to thermal velocities have trons of atoms and interact with the nuclei through the ‘strong
wavelengths similar to those of X-rays and may also be used force’ that is responsible for binding nucleons together. As a
for diffraction studies. For instance, a neutron generated in result, the intensity with which neutrons are scattered is in-
a reactor and slowed to thermal velocities by repeated colli- dependent of the number of electrons and neighbouring el-
sions with a moderator (such as graphite) until it is travelling ements in the periodic table might scatter neutrons with
at about 4 km s−1 has a wavelength of about 100 pm. In practice, markedly different intensities. Neutron diffraction can be used
a range of wavelengths occurs in a neutron beam, but a mono- to distinguish atoms of elements such as Ni and Co that are
chromatic beam can be selected by diffraction from a crystal, present in the same compound and to study order–disorder
such as a single crystal of germanium. phase transitions in FeCo. A second difference is that neutrons
possess a magnetic moment due to their spin. This magnetic
moment can couple to the magnetic fields of atoms or ions in
Example 15B.3 Calculating the typical wavelength of a crystal (if the ions have unpaired electrons) and modify the
thermal neutrons diffraction pattern. One consequence is that neutron diffrac-
tion is well suited to the investigation of magnetically ordered
Calculate the typical wavelength of neutrons after reach-
lattices in which neighbouring atoms may be of the same el-
ing thermal equilibrium with their surroundings at 373 K.
For simplicity, assume that the particles are travelling in one
ement but have different orientations of their electronic spin
dimension. (Fig. 15B.13).
Electrons accelerated through a potential difference of 40 kV
Collect your thoughts In order to use the de Broglie rela- have wavelengths of about 6 pm, and so are also suitable for dif-
tion, you need to know the momentum of the neutrons, and fraction studies of molecules. Consider the scattering of elec-
therefore their velocity. The velocity can be computed from trons from pairs of atoms with centres separated by a distance
the kinetic energy, which you can assume has its equipartition Rij and orientated at a definite angle θ to an incident beam of
value for translation in one dimension, Ek = 12 kT (see Energy: A electrons. When the molecule consists of a number of atoms,
first look). The mass of a neutron is 1.675 × 10−27 kg.
the scattering intensity can be calculated by summing over the
The solution From the equipartition principle, the mean trans- contribution from all pairs. The total intensity I(θ) is given by
lational kinetic energy of a neutron at a temperature T travel- the Wierl equation:
ling in the x-direction is Ek = 12 kT . The kinetic energy is also
equal to p2/2m, where p is the momentum of the neutron and sin sRij 4 1
m is its mass. Hence, p = (mkT)1/2. It follows from the de Broglie I ( )   fi f j s sin 2  Wierl equation (15B.8)
relation, λ = h/p, that the wavelength of the neutron is i ,j sRij 
672 15 Solids

where λ is the wavelength of the electrons in the beam, and


f is the electron scattering factor, a measure of the elec-
tron ­s cattering power of the atom. The main application of
electron diffraction techniques is to the study of surfaces
(Topic 19A).

Exercises
E15B.4 What speed should neutrons have if they are to have a wavelength of
Figure 15B.13 If the spins of atoms at lattice points are orderly, 65 pm?
as in this material, where the spins of one set of atoms are aligned
E15B.5 Calculate the wavelength of neutrons that have reached thermal
antiparallel to those of the other set, neutron diffraction detects
equilibrium by collision with a moderator at 350 K.
two interpenetrating simple cubic lattices on account of the
magnetic interaction of the neutron with the atoms, but X-ray
diffraction would see only a single bcc lattice.

Checklist of concepts
☐ 1. A reflection refers to an intense beam emerging in a ☐ 8. The phase problem arises because it is possible to
particular direction and arising from constructive inter- measure only the intensity of the reflections and not
ference. their phases; as a result Fourier synthesis cannot be
☐ 2. The glancing angle, 2θ, is the angle through which a used in a straightforward way to determine the electron
beam is deflected. density.
☐ 3. Bragg’s law relates the angle of a diffracted beam to the ☐ 9. A Patterson map is a map of the interatomic vectors.
spacing of a given set of lattice planes. ☐ 10. Direct methods use statistical techniques to determine
☐ 4. The scattering factor is a measure of the ability of an the likely phases of the reflections.
atom to scatter electromagnetic radiation. ☐ 11. Structure refinement is the adjustment of structural
☐ 5. The structure factor is the overall amplitude of a wave parameters to give the best fit between the observed
diffracted by the {hkl} planes and atoms distributed intensities and those calculated from the model of the
through the unit cell. structure deduced from the diffraction pattern.
☐ 6. The electron density and the diffraction pattern are ☐ 12. The Wierl equation relates the intensity of electron
related by a Fourier transform. scattering to the distances between pairs of atoms in the
sample.
☐ 7. Fourier synthesis is the construction of the electron
density distribution from structure factors.

Checklist of equations
Property Equation Comment Equation number
Bragg’s law λ = 2d sin θ d is the lattice spacing, 2θ the glancing angle 15B.1b

Scattering factor f  4  0 { (r )sin kr /kr }r dr, k = (4π/λ) sin θ
2
Spherically symmetrical atom 15B.2
ihkl ( j )
Structure factor Fhkl   f j e , φhkl(j) = 2π(hxj + kyj + lzj) Definition 15B.3
j

Fourier synthesis  (r )  (1/V )  Fhkl e 2 i (hx ky lz ) V is the volume of the unit cell 15B.4
hkl

Patterson synthesis P (r )  (1/V )  | Fhkl |2 e 2 i (hx ky lz ) 15B.5


hkl

Wierl equation I ( )   fi f j (sin sRij /sRij ), s  (4 / )sin 12  15B.8


i ,j
TOPIC 15C Bonding in solids

➤ Why do you need to know this material?


To understand the properties and structures of solid mate-
rials you need to know about the type of bonding that
holds together the atoms, ions, and molecules.

➤ What is the key idea?


A
Four characteristic types of bonding result in metals, ionic
solids, covalent solids, and molecular solids.
Figure 15C.1 A layer of close-packed spheres used to build a
➤ What do you need to know already? three-dimensional close-packed structure.
You need to be familiar with molecular interactions (Topic
14B) and the general features of crystal structures (Topic are structures that are identical in two dimensions (the close-
15A). For the discussion of metallic bonding you should be packed layers) but differ in the third dimension.
aware of the principles of Hückel molecular orbital theory In all polytypes, the spheres of the second close-packed layer
(Topic 9E). The discussion of ionic bonding makes use of lie in the depressions of the first layer (Fig. 15C.2). The third
the concept of enthalpy (Topic 2B). layer may be added in either of two ways. In one, the spheres
are placed directly above the first layer to give an ABA pattern
of layers (Fig. 15C.3a). Alternatively, the spheres may be placed
over the depressions in the first layer that are not occupied by
Solids may be classified into four broad types, namely metals, the second layer (these depressions are visible in Fig. 15C.2),
ionic solids, covalent (or network) solids, and molecular sol- so giving an ABC pattern (Fig. 15C.3b). Two polytypes are
ids. Each type is characterized by the nature of the bonding be- formed if the two stacking patterns are repeated in the vertical
tween the constituents. ­direction:
• Hexagonally close-packed (hcp): the ABA pattern is
repeated, to give the sequence of layers ABABAB.…
• Cubic close-packed (ccp): the ABC pattern is repeated, to
15C.1 Metals give the sequence ABCABC.…

In a metal the electrons are delocalized over arrays of identi- The origins of these names can be seen by referring to
cal cations and bind the whole together into a rigid but duc- Fig. 15C.4. The ccp structure gives rise to a face-centred unit
tile and malleable structure. The crystalline forms of metallic
elements can be discussed in terms of a model in which their
atoms are treated as identical hard spheres. Most metallic ele-
ments crystallize in one of three simple forms, two of which can
be explained in terms of the hard spheres packing together in
the closest possible arrangement.

15C.1(a) Close packing B


A
Figure 15C.1 shows a close-packed layer of identical spheres,
one with maximum utilization of space. A close-packed three- Figure 15C.2 To achieve the greatest packing density, the second
dimensional structure is obtained by stacking such layers layer of close-packed spheres must sit in the depressions of the
on top of one another. However, this stacking can be done in first layer. The two layers are the AB component of the close-
different ways and results in close-packed polytypes, which packed structure.
674 15 Solids

Table 15C.1 The crystal structures of some elements⋆

Structure Element
hcp‡ Be, Cd, Co, He, Mg, Sc, Ti, Zn
fcc‡ (ccp, cubic F) Ag, Al, Ar, Au, Ca, Cu, Kr, Ne, Ni, Pb, Pd, Pt, Rh, Rn,
Sr, Xe
C bcc (cubic I) Ba, Cr, Cs, Fe, K, Li, Mn, Mo, Na, Rb, Ta, V, W
A
cubic P Po
B B
A A ⋆
The notation used to describe unit cells is introduced in Topic 15A. The structures are
for the elements at 298 K and 1 bar; for the elements that are normally gases, the structures
(a) (b) are for their solid form close to the freezing temperature and 1 bar.

Close-packed structures.
Figure 15C.3 (a) The third layer of close-packed spheres might
occupy the depressions lying directly above the spheres in the first
layer, resulting in an ABA structure, which corresponds to hexagonal ccp and hcp structures. Another measure of their compactness
close-packing. (b) Alternatively, the third layer might lie in the
is the packing fraction, the fraction of space occupied by the
depressions that are not above the spheres in the first layer, resulting
spheres, which is 0.740 (see Example 15C.1) for these close-
in an ABC structure, which corresponds to cubic close-packing.
packed structures. That is, in a close-packed solid of identical
hard spheres, only 26.0 per cent of the volume is empty space.
The fact that many metals are close-packed accounts for their
high mass densities.
A B
C

B
Example 15C.1 Calculating a packing fraction

A A Calculate the packing fraction of a ccp structure formed from


hard spheres.
(a) (b)
Collect your thoughts You need to calculate the volume of
Figure 15C.4 Fragments of the structures shown in Fig. 15C.3 the unit cell and the volume of the spheres that are wholly or
revealing the (a) hexagonal (b) cubic symmetry. The labels of the partly contained within the cell, and then take the ratio of the
spheres are the same as for the layers in Fig. 15C.3. two volumes. The key step is to establish a relation between
the radius of the spheres, R, and the cell dimension, a. Figure
cell, so may also be denoted cubic F (or fcc, for face-centred 15C.5 shows that the sphere in the middle of a face just touches
cubic). The hcp structure has a hexagonal unit cell, with a struc- the two spheres at opposite corners of the face. The length of
tural motif consisting of a sphere at one corner of the cell and the face diagonal is therefore 4R. To calculate the volume of the
another at the centre of the cell (1). It is also possible to have spheres in the cell, you need to account for the fraction of each
random sequences of layers; however, the hcp and ccp poly- sphere that is contained within the cell. Spheres at the corners
types are the most important. Table 15C.1 lists the structures contribute 18 of their volume to the cell; those on the faces con-
adopted by a selection of elements. tribute 12 to the cell.

4R
a
a
a = 81/2R

4R
a = 81/2R

The compactness of close-packed structures is indicated by Figure 15C.5 In a ccp unit cell (a fcc unit cell) a sphere located
their coordination number, the number of spheres immedi- on the face of the cube just touches the spheres at opposite
ately surrounding any selected sphere, which is 12 for both the corners of the face.
15C Bonding in solids 675

The solution As can be seen from Fig. 15C.5, the length of


(a) N=1
the face diagonal is 4R. From Pythagoras’ theorem it follows
that a2 + a2 = (4R)2, so 2a2 = 16R2 and therefore a = 81/2R. (b) N=2
The volume of the unit cell is a3, which is therefore 83/2R3.
There are eight spheres at the corners, each contributing 18 (c) N=3
of their volume to the cell (giving a net contribution of 1
sphere), and six spheres on the faces, each contributing 12 (d) N=4
(giving a net contribution of 3 spheres). The total volume
occupied by the spheres is equivalent to 4 complete spheres.
Because the volume of each sphere is 34 πR3 , the total occu- (e) N=∞
pied volume is 163 πR3. The fraction of space occupied is
therefore
Figure 15C.6 The formation of a band of N molecular orbitals by
83/2 = 16 2 successive addition of N atomic orbitals in a line. When N becomes
16
πR 3 π infinite the band covers a finite range of energy and the orbitals
3 = = 0.740
3
3/2
8 R 3 2 within it are very closely spaced, but still discrete.

An hcp structure has the same coordination number, so its


packing fraction is the same. of electrical properties of solids discussed in Topic 15E, and is
described here.
Self-test 15C.1 Calculate the packing fraction of the cubic I As a starting point, consider a one-dimensional solid, which
(body-centred cubic, bcc) structure in which one sphere is at consists of a single, infinitely long line of atoms. Suppose that
the centre of a cube formed by eight others. The spheres touch each atom has one s orbital available for forming molecular
along the body diagonal of the cube. orbitals. The LCAO-MOs of the solid are constructed by add-
Answer: 31/2π/8 = 0.680 ing N atoms in succession to a line, and then inferring the elec-
tronic structure by using the building-up principle. One atom
contributes one s orbital at a certain energy (Fig. 15C.6). When
a second atom is brought up it overlaps the first and forms a
As shown in Table 15C.1, a number of common metals bonding and an antibonding orbital. The third atom overlaps
adopt structures that are less than close-packed. The depar- its nearest neighbour (and only slightly the next-nearest), and
ture from close packing suggests that factors, such as specific from these three atomic orbitals, three molecular orbitals are
covalent bonding between neighbouring atoms, are begin- formed: one is fully bonding, one fully antibonding, and the
ning to influence the structure and impose a particular geo- intermediate orbital is nonbonding between neighbours. The
metrical arrangement. One such arrangement results in a fourth atom leads to the formation of a fourth molecular or-
cubic I (bcc, for body-centred cubic) structure, with one bital. At this stage, it can be seen that the effect of bringing up
sphere at the centre of a cube formed by eight others. The co- successive atoms is to spread the range of energies covered by
ordination number of a bcc structure is only 8, but there are the molecular orbitals and to fill in the range of energies with
six more atoms not much further away than the eight near- more and more orbitals (one more for each atom). When N
est neighbours. The packing fraction of 0.680 (Self-test 15C.1) atoms have been added to the line, there are N molecular orbit-
is not much smaller than the value for a close-packed struc- als covering a finite range of energies: this set of orbitals is said
ture (0.740), and shows that about two-thirds of the available to form a band.
space is occupied by the atoms. The energies of the molecular orbitals that form the band
can be found by using the Hückel approximations described in
15C.1(b) Electronic structure of metals Topic 9E. The energies Ek of the molecular orbitals are then

The central aspect of solids that determines their electrical k


properties (Topic 15E) is the distribution of their electrons. Ek    2  cos k  1,2,,N
N 1
There are two models of this distribution. In one, the nearly Energy levels [linear array of s orbitals] (15C.1)
free-electron approximation, the valence electrons are as-
sumed to be trapped in a box with a periodic potential, with where α is the Coulomb integral and β is the (s,s) resonance
low potential energy corresponding to the locations of cations. integral. It is not hard to show that this expression implies that
In the tight-binding approximation, the valence electrons are when N is infinitely large, the separation between neighbouring
assumed to occupy molecular orbitals delocalized throughout levels, Ek+1 − Ek, is infinitely small, but the band still has finite
the solid. The latter model is more in accord with the discussion width overall, with EN − E1 → −4β.
676 15 Solids

Highest level of p band Fully antibonding


How is that done? 15C.1 Evaluating the separation p Band
+ −+−+−+−
between neighbouring levels and the width of a band
p
This calculation requires looking at a specific limiting case of
eqn 15C.1. Lowest level of p band Fully bonding
− + + − − + +
Step 1 Write an expression for the energy difference between two
Band
neighbouring levels Fully antibonding
gap Highest level of s band
From eqn 15C.1 it follows that the energy separation between + − + − +
neighbouring energy levels k and k +1 is s

+ + + + +
(k +1)π kπ s Band
Ek+1 − Ek = α + β cos − α + 2β cos
N +1 N +1 Lowest level of s band Fully bonding

(k +1)π kπ Figure 15C.7 The overlap of s orbitals gives rise to an s band and
= 2β cos − cos
N +1 N +1 the overlap of p orbitals gives rise to a p band. In this case, the s
and p orbitals of the atoms are so widely spaced in energy that
Step 2 Find the limit as N → ∞ there is a band gap; it is also possible that the separation will be
First use the trigonometric identity cos(A + B) = cos A cos B − less, resulting in the bands overlapping.
sin A sin B to express the boxed term as

→1 as N→ → 0 as N→ The band can be thought of as consisting of N different molecu-


(k +1)π kπ π kπ π
lar orbitals, the lowest-energy orbital (k = 1) being fully bonding,
cos = cos cos − sin sin and the highest-energy orbital (k = N) being fully antibonding
N +1 N +1 N +1 N +1 N +1
between adjacent atoms (Fig. 15C.7). The molecular orbitals of
intermediate energy have k − 1 nodes distributed along the chain
As N → ∞, /(N  1)  0 and therefore cos{/(N  1)}  1
of atoms. Similar bands form in three-­dimensional solids (see
and sin{/(N  1)}  0. In this limit the boxed term tends to
Problem 15C.5).
cos{k/(N  1}) and therefore the energy difference becomes

 k k  Brief illustration 15C.1


Ek 1  Ek  2   cos  cos 0
 N 1 N  1 
To illustrate the dependence of Ek+1 − Ek on N, note that
It follows that when N is infinitely large, the difference between
 2 
neighbouring energy levels is infinitely small. for N  3: E2  E1  2   cos  cos   1.414 
 4 4
Step 3 Write an expression for the width of the band for N → ∞  2 
for N  30: E2  E1  2   cos  cos   0.0307 
The width of the band is simply EN − E1. Each of the energies  31 31 
can be approximated as follows for the case that N → ∞.  2  
for N  300: E2  E1  2   cos  cos  0.000 327 
  301 301 
E1    2  cos
N 1
The energy difference decreases with increasing N, as expected.
As N → ∞, the term π/(N + 1) tends to zero so the cosine tends
to 1; therefore in this limit The band formed from overlap of s orbitals is called the s
E1    2 
band. If the atoms have p orbitals available, the same procedure
leads to a p band (as shown in the upper half of Fig. 15C.7).
When k has its maximum value of N, If the atomic p orbitals lie higher in energy than the s orbit-
als, then the p band lies higher than the s band, and there may
N be a band gap, a range of energies to which no orbital corre-
EN    2  cos
N 1 sponds. However, it is also possible for the bands to touch, with
As N → ∞, the 1 in the denominator can be ignored, so the the highest orbital of the s band coincident with the lowest level
cosine term becomes cos π = −1. Therefore, in this limit EN = of the p band, or even overlap (as is the case for the 3s and 3p
α − 2β, and so the band width is EN − E1 → −4β. Recall that β is bands in magnesium).
negative, so that the band width, −4β, is positive. Now consider the electronic structure of a solid formed from
N atoms each able to contribute one electron (for example, the
15C Bonding in solids 677

Cl–
Unoccupied levels
Cs+
Energy

Fermi level

Occupied levels

Figure 15C.8 When N electrons occupy a band of N orbitals, only Figure 15C.9 The caesium chloride structure consists of two
half of the orbitals are occupied (at T = 0) because two electrons interpenetrating simple cubic arrays of ions, one of cations and
will occupy each orbital. The highest occupied level is known as the other of anions, so that each cube of ions of one kind has a
the Fermi level. counter-ion at its centre.

alkali metals). The N atomic orbitals give rise to a band consist- number of nearest neighbours of opposite charge; the structure
ing of N molecular orbitals. Each of these orbitals can accom- itself is characterized as having (N+,N−) coordination, where N+
modate two spin-paired electrons, so at T = 0 only the lowest is the coordination number of the cation and N− that of the
1
2
N molecular orbitals are occupied (Fig. 15C.8). The HOMO anion.
is called the Fermi level. Only the small number of electrons Even if, by chance, the ions have the same size, the re-
close to the Fermi level can undergo thermal excitation, so only quirement that the unit cells are electrically neutral make
these electrons contribute to the heat capacity of the metal. It it impossible to achieve 12-coordinate close-packed ionic
is for this reason that Dulong and Petit’s law for heat capacities structures. As a result, ionic solids are generally less dense
(Topic 7A) gives reasonable agreement with experiment at nor- than metals. The best packing that can be achieved is the
mal temperatures by considering only the atoms in a sample, (8,8)-coordinate caesium chloride structure in which each
not the atoms plus the ‘free’ electrons. The presence of an in- cation is surrounded by eight anions and each anion is sur-
completely filled band is responsible for electrical conductivity, rounded by eight cations (Fig. 15C.9). In this structure, an
as explained in Topic 15E. ion of one charge occupies the centre of a cubic unit cell with
eight counter-ions at its corners. The cell is electrically neu-
tral because the ions at the corners of the cell are shared be-
Exercises tween eight cells and so contribute one eighth of their charge
E15C.1 Calculate the packing fraction for close-packed cylinders; you need to each. The structure shown in Fig. 15C.9 is adopted by CsCl
only consider one layer. Hint: Start by identifying a suitable unit cell.
itself and also by CaS.
E15C.2 Calculate the packing fractions of (i) a primitive cubic unit cell, (ii) a When the radii of the ions differ more than they do in CsCl,
bcc unit cell, (iii) an fcc unit cell, where each cell is composed of identical hard
even eight-coordinate packing cannot be achieved. One com-
spheres. Hint: Start by identifying the unit cell and working out which atoms
are in contact. mon structure adopted is the (6,6)-coordinate rock salt struc-
ture typified by rock salt itself, NaCl (Fig. 15C.10). In this
structure, each cation is surrounded by six anions and each
anion is surrounded by six cations. The rock salt structure can
15C.2 Ionic solids be pictured as consisting of two interpenetrating slightly ex-
panded cubic F (fcc) arrays, one composed of cations and the
An ionic solid consists of cations and anions held together by
electrostatic interactions. Two key questions arise in consider-
ing such solids: the relative locations adopted by the ions and Cl–
the energetics of the resulting structure.
Na+

15C.2(a) Structure
When crystals of compounds of monatomic ions (such as NaCl
and MgO) are modelled by stacks of hard spheres it is neces-
sary to allow for the possibility that the ions have different radii Figure 15C.10 The rock salt (NaCl) structure consists of two
(typically with the cations smaller than the anions) and differ- mutually interpenetrating slightly expanded face-centred cubic
ent electrical charges. The coordination number of an ion is the arrays of ions. The assembly shown here is a unit cell.
678 15 Solids

S2– Other scales are also available (such as one based on F− for dis-
cussing halides), and it is essential not to mix values from differ-
Zn2+ ent scales. Ionic radii are so arbitrary that predictions based on
them must be viewed cautiously.

Brief illustration 15C.2

From the values of ionic radii in the Resource section, the radius
ratio for MgO is
Figure 15C.11 The structure of the sphalerite form of ZnS
showing the location of the Zn ions in half the tetrahedral holes radius of Mg 2

formed by the fcc array of S ions. 72 pm
  0.51
other of anions. This structure is adopted by NaCl itself and 140 pm

  
also by several other MX compounds, including KBr, AgCl, radius of O2

MgO, and ScN. which is consistent with the observed rock salt structure of
The switch from the caesium chloride structure to the rock MgO crystals.
salt structure is related to the value of the radius ratio, γ:
rsmaller
 Radius ratio [definition] (15C.2) 15C.2(b) Energetics
rlarger
The lattice energy of a solid is the change in potential energy
where rsmaller is the radius of the smaller ions in the crystal and when the ions go from being packed together in a solid to being
rlarger that of the larger ions. The radius-ratio rule, which is de- widely separated as a gas. All lattice energies are positive; a high
rived by considering the geometrical problem of packing the lattice energy indicates that the ions interact strongly with one
maximum number of hard spheres of one radius around a hard another to give a tightly bonded solid. The lattice enthalpy,
sphere of a different radius, can be summarized as follows: ∆HL, is the change in standard molar enthalpy for the process
MX(s) → M+(g) + X−(g) and its equivalent for other charge
Radius ratio Structural type types and stoichiometries. At T = 0 the lattice enthalpy is equal
1/2
γ<2 − 1 = 0.414 sphalerite (Fig. 15C.11) to the lattice energy; at normal temperatures they differ by only
0.414 < γ < 31/2 − 1 = 0.732 rock salt (Fig. 15C.10) a few kilojoules per mole, an amount so small compared to
γ > 0.732 caesium chloride (Fig. 15C.9) typical lattice energies that the difference is normally neglected.
Each ion in a solid experiences favourable (energy-lowering)
The deviation of a structure from that expected on the basis of electrostatic interactions from all the other oppositely charged
this rule is often taken to be an indication of a shift from ionic ions and unfavourable (energy-raising) interactions from all
towards covalent bonding. A major source of unreliability, the other like-charged ions. The total Coulomb potential en-
though, is the arbitrariness of ionic radii (as explained in a mo- ergy is the sum of all the electrostatic contributions. Each cat-
ment) and their variation with coordination number. ion is surrounded by anions, and so there is a large negative
Experimental measurements give the distances between the contribution to the potential energy from the interaction of op-
centres of two ions, and a decision has to be made about how to posite charges. Beyond those nearest neighbours, there are cati-
apportion this difference between the two ions. One approach is ons that contribute a positive term to the total potential energy
simply to assign a value to the radius of one ion and then use this of the central cation. There is also a negative contribution from
value to infer the radii of other ions. A scale based on the value the anions beyond those cations, a positive contribution from
140 pm for the radius of the O2− ion is widely used (Table 15C.2). the cations beyond them, and so on, to the edge of the solid.
These favourable and unfavourable interactions become pro-
Table 15C.2 Ionic radii, r/pm⋆ gressively weaker as the distance from the central ion increases,
but the net outcome of all these contributions is dominated by
Na+ 102(6‡), 116(8) the interaction between nearest neighbours and is therefore a
K+ 138(6), 151(8) lowering of the potential energy.
F− 128(2), 131(4) First, consider a simple one-dimensional model of an ionic
Cl −
181 (close packing) solid that consists of a long line of uniformly spaced alternating

cations and anions; the distance between neighbouring centres
This scale is based on a value 140 pm for the radius of the O2− ion. More values are given
in the Resource section.
is d, the sum of the ionic radii (Fig. 15C.12). If the charge num-

Coordination number. bers of the ions have the same absolute value (+1 and −1, or +2
15C Bonding in solids 679

+z –z +z –z +z –z Table 15C.3 Madelung constants

d Structural type A
Caesium chloride 1.763
Figure 15C.12 A one-dimensional model of an ionic solid
Fluorite 2.519
consisting of a line of alternating cations and anions.
Rock salt 1.748
2
and −2, for instance), then z1 = +z, z2 = −z, and z1z2 = −z . The Rutile 2.408
potential energy of the central ion is calculated by summing all Sphalerite (zinc blende) 1.638
the terms, with negative terms representing favourable inter- Wurtzite 1.641
actions with oppositely charged ions and positive terms repre-
senting unfavourable interactions with like-charged ions. For
the interaction with ions extending in a line to the right of the
central ion, the contribution of the Coulomb interaction to the
lattice energy is Overlap contribution

Potential energy, Ep
1  ze 2 2
ze ze 2 2
ze2 2
 2 2
Ep       Lattice parameter, d
4  0  d 2d 3d 4d  0
  ln 2 
2 2
ze
  1  12  13  14  
Total

4  0 d
Coulombic contribution
z 2e2
  ln 2
4  0 d

To complete the calculation Ep is multiplied by 2 to obtain the Figure 15C.13 The contributions to the total potential energy of
total energy arising from interactions on both sides of the ion, an ionic crystal.
and then by Avogadro’s constant, NA, to obtain an expression
for the Coulomb contribution to the (molar) lattice energy. The
outcome is where d is the distance between the atoms, and C′ and d * are
2
z N Ae 2 constants. It turns out that the value of C′ is not needed (it can-
Ep  2 ln 2  cels in expressions that make use of this formula; see below);
4  0 d
d * is commonly taken to be 34.5 pm. The total potential energy
with d = rcation + ranion. This energy is negative, corresponding to is the sum of Ep and Ep*, and passes through a minimum when
a net favourable interaction. The calculation can be extended to d(Ep  Ep* )/dd  0 (Fig. 15C.13).
three-dimensional arrays of ions with different charge numbers A short calculation leads to the Born–Mayer equation for
zA and zB: the minimum total potential energy (see Problem P15C.9):
| zA zB | N Ae2
Ep   A   (15C.3) N | z z | e2  d*  Born–Mayer
4 0 d Ep,min =
− A A B 1 −  A  (15C.5)
4 πε 0d  d  equation
The factor A is a positive numerical constant called the Madelung
constant; its value depends on how the ions are arranged in the Provided zero-point contributions to the energy are ignored,
crystal. For a rock salt structure, A = 1.748; Table 15C.3 lists the negative of this potential energy can be identified with the
Madelung constants for other common structures. lattice energy. The important features of this equation are:
The Coulomb interaction is not the only contribution to the
lattice energy. When atomic orbitals overlap to form bonding • Because Ep,min ∝ |zAzB|, the potential energy decreases
and antibonding molecular orbitals and both kinds of orbitals (becomes more negative) with increasing charge number
are full, there is an increase in energy because the antibond- of the ions.
ing orbital is raised in energy more than the bonding orbital is • Because the electrostatic (and dominant) contribution to
lowered (Topic 9D). This positive contribution to the potential Ep,min is proportional to 1/d, the potential energy decreases
energy depends on the overlap of the atomic orbitals, and, be- (becomes more negative) with decreasing ionic radius.
cause orbitals decay exponentially with distance, at large dis-
The second conclusion follows from the fact that the smaller
tances from the nucleus it is often modelled by writing
the ionic radii, the smaller is the value of d. High lattice ener-

Ep  N A C e  d /d  (15C.4) gies are expected when the ions are highly charged (so |zAzB| is
large) and small (so d is small).
680 15 Solids

Brief illustration 15C.3 cycle to be useful, experimental values for the enthalpy
changes for all the steps need to be available (for example,
To estimate Ep,min for MgO, which has a rock salt structure from tabulated data), apart, of course, from the step involv-
(A = 1.748), use the following values: d = r(Mg2+) + r(O2−) = 72 + ing the formation of the lattice from the ions. For the cycle in
140 pm = 212 pm. Note that Fig. 15C.14 the enthalpy changes are (for convenience, starting
at the elements):
23 1 19 2
N Ae 2 (6.022 14  10 mol )  (1.602 176  10 C)

4  0 4   (8.854 19  1012 J 1 C 2 m 1 )
∆H/(kJ mol−1)
4 1
 1.38935  10 Jm mol 1. Sublimation of K(s) +89 [dissociation enthalpy of
K(s)]
Then 2. Dissociation of 12 Cl 2 (g ) +122 [ 12 × dissociation enthalpy
of Cl2(g)]
|z z |
O Mg 2  2
N A e2/ 4  0 3. Ionization of K(g) +418 [ionization enthalpy of
 
4 4 K(g)]
Ep, min   (1 . 38935  10 J m mol 1 )
2.12  1010 m 4. Electron attachment to −349 [electron-gain enthalpy of
   Cl(g) Cl(g)]
d

1
 d /
d  5. Formation of solid from −∆HL/(kJ mol−1) [value to be determined]
A gaseous ions
 34.5 pm  
1    1.748 6. Decomposition of +437 [negative of enthalpy of
 212 pm 
 compound to its formation of KCl(s)]
elements in their
 3.84  103 kJmol 1 reference states

The cycle is closed, so the sum of these enthalpy changes is


It is not possible to measure the lattice enthalpy directly, but equal to zero, and the lattice enthalpy can be inferred from the
values can be obtained by combining experimental values of resulting equation.
other enthalpy changes by using a Born–Haber cycle. Such a
The solution The sum of contributions around the cycle is
cycle is a closed path of transformations starting and ending at
the same point, one step of which is the formation of the solid 89  122  418  349  H L /(kJmol 1 )  437  0
compound from a gas of widely separated ions.
It follows that ∆HL = +717 kJ mol−1.
Self-test 15C.2 Calculate the lattice enthalpy of CaO from the
Example 15C.2 Using the Born–Haber cycle following data:

Calculate the lattice enthalpy of KCl.


∆H/(kJ mol−1)
Collect your thoughts You need to construct a suitable Born–
Sublimation of Ca(s) +178
Haber cycle, such as the one shown in Fig. 15C.14. For the 2+
Ionization of Ca(g) to Ca (g) +1735
K (g) + e (g) + Cl(g)
+ – Dissociation of O2 (g )
1
2 +249
Electron attachment to O(g) −141
–349 4 Electron attachment to O (g) −
+844
3 +418
K+(g) + Cl–(g) Formation of CaO(s) from Ca(s) and 12 O2 (g ) −635
K(g) + Cl(g)
2 +122 Answer: +3500 kJ mol−1
K(g) + ½Cl2(g)
1 +89 –ΔHL
K(s) + ½Cl2(g) 5

6 +437 Some lattice enthalpies obtained by the Born–Haber cycle


KCl(s) are listed in Table 15C.4. As can be seen from the data, the
trends in values are in general accord with the predictions
Figure 15C.14 The Born–Haber cycle for KCl at 298 K. Enthalpy of the Born–Mayer equation. Agreement is typically taken
changes are in kilojoules per mole. to imply that the ionic model of bonding is valid for the
15C Bonding in solids 681

Table 15C.4 Lattice enthalpies at 298 K, ∆HL/(kJ mol−1)⋆ Brief illustration 15C.4
NaF 926
Diamond and graphite are two allotropes of carbon. In dia-
NaBr 752
mond each sp3-hybridized carbon is bonded tetrahedrally to
MgO 3850 its four neighbours (Fig. 15C.15). The network of strong C−C
MgS 3406 bonds is repeated throughout the crystal and, as a result, dia-

More values are given in the Resource section. mond is very hard (in fact, the hardest known substance). In
graphite, σ bonds between sp2-hybridized carbon atoms form
hexagonal rings which, when repeated throughout a plane,
s­ ubstance; disagreement implies that there is a covalent contri- give rise to ‘graphene’ sheets (Fig. 15C.16). The sheets can slide
bution to the bonding. It is important, though, to be cautious, against each other when impurities are present; as a result,
because numerical agreement might be coincidental and, as impure graphite is used widely as a lubricant.
noted above, the values for ionic radii are subject to significant
uncertainty.

Exercises
E15C.3 Determine the radius of the smallest cation that can have (i) sixfold
and (ii) eightfold coordination with the Cl− ion (radius 181 pm).
E15C.4 Does titanium expand or contract as it transforms from hcp to bcc?
The atomic radius of titanium is 145.8 pm in hcp but 142.5 pm in bcc. Hint:
Consider the change in packing fraction.
E15C.5 Calculate the lattice enthalpy of MgO from the following data:

Figure 15C.15 A fragment of the structure of diamond. Each C


ΔH/(kJ mol−1) atom is tetrahedrally bonded to four neighbours. This framework-
Sublimation of Mg(s) +15 like structure results in a rigid crystal.
Ionization of Mg(g) to Mg2+(g) +2188
Dissociation of O2(g) +249
Electron attachment to O(g) −141

Electron attachment to O (g) +844
Formation of MgO(s) from Mg(s) and −602
2 O2 (g )
1
in their reference states.

15C.3 Covalent and molecular solids


X-ray diffraction studies of solids reveal a huge amount of in-
formation, including interatomic distances, bond angles, ste-
reochemistry, and vibrational parameters. This section can do (a) (b)
no more than hint at the diversity of types of solids found when Figure 15C.16 Graphite consists of flat planes of hexagons of
molecules pack together or atoms link together in extended carbon atoms lying above one another. (a) The arrangement of
networks. carbon atoms in a ‘graphene’ sheet; (b) the relative arrangement of
In covalent solids (or covalent network solids), covalent neighbouring sheets. The planes can slide over one another easily
bonds in a definite spatial orientation link the atoms together when impurities are present.
into a network that extends through the crystal; effectively the
crystal is a giant molecule. The demands of directional bond-
ing, which have only a small effect on the structures of many
metals, now override the geometrical problem of packing Molecular solids, which are the subject of the over-
spheres together, resulting in a wide variety of often quite elab- whelming majority of modern structural determinations,
orate structures. are held together by van der Waals interactions between
682 15 Solids

the individual molecular components (Topic 14B). The ob-


served crystal structure is nature’s solution to the problem
of condensing objects of various shapes into an aggregate
of minimum energy (actually, for T > 0, of minimum Gibbs
energy). The prediction of the structure is difficult, but soft-
ware specifically designed to explore interaction energies
can now make reasonably reliable predictions. The problem
is made more complicated by the role of hydrogen bonds,
which in some cases dominate the crystal structure, as in ice
(Fig. 15C.17), but in others (for example, in solid phenol)
distort a structure that is determined largely by the van der
Figure 15C.17 A fragment of the crystal structure of ice (ice-I). Waals interactions.
Each O atom (shown as coloured spheres) is at the centre of a
tetrahedron of four O atoms at a distance of 276 pm. The central
O atom is attached by two short O−H bonds to two H atoms
(shown as white spheres) and by two long hydrogen bonds to
the H atoms of two of the neighbouring molecules. For each O−O
separation both of the alternative H atom locations are shown.
Overall, the structure consists of planes of hexagonal puckered
rings of H2O molecules (like the chair form of cyclohexane).

Checklist of concepts
☐ 1. A close-packed layer is a layer of spheres arranged so ☐ 8. A band gap is a range of energies to which no orbital
there is maximum utilization of space. corresponds.
☐ 2. A hexagonally close-packed structure is one where the ☐ 9. The Fermi level is the highest occupied molecular
sequence of close-packed layers is ABABAB …. orbital at T = 0.
☐ 3. A cubic close-packed structure is one where the ☐ 10. The coordination number of an ionic lattice is denoted
sequence of close-packed layers is ABCABC …. (N+,N−), where N+ is the number of nearest neighbour
☐ 4. The coordination number is the number of spheres anions around a cation and N− the number of nearest
immediately surrounding any selected sphere. neighbour cations around an anion.
☐ 5. In the nearly free-electron approximation the valence ☐ 11. The lattice energy of a solid is the change in potential
electrons are assumed to be trapped in a box with a peri- energy when the ions go from being packed together in
odic potential energy, with low energy corresponding to a solid to being widely separated as a gas.
the locations of cations. ☐ 12. A Born–Haber cycle is a closed path of transformations
☐ 6. In the tight-binding approximation the valence elec- starting and ending at the same point, one step of which
trons are assumed to occupy molecular orbitals delocal- is the formation of the solid compound from a gas of
ized throughout the solid. widely separated ions.
☐ 7. In metals atomic orbitals overlap to form a band, which ☐ 13. A molecular solid is a solid consisting of discrete mol-
is a set of molecular orbitals that are closely spaced and ecules held together by van der Waals interactions, and
cover a finite range of energy; electrons occupy the possibly hydrogen bonds.
orbitals within the band.

Checklist of equations
Property Equation Comment Equation number
Energy levels of a linear array of orbitals Ek = α + 2β cos{kπ/(N+1)}, k = 1,2,…, N Hückel approximation 15C.1
Band width EN − E1 → −4β as N → ∞ Hückel approximation
Radius ratio γ = rsmaller/rlarger For criteria, see Section 15C.2 15C.2
2 ⋆
Born–Mayer equation Ep,min = −{NA|zAzB|e /4πε0d}(1−d /d)A 15C.5
TOPIC 15D The mechanical properties
of solids

➤ Why do you need to know this material?


An understanding of the mechanical properties of
solid materials is crucial to the development of modern
(a) (b)
materials.

➤ What is the key idea?


The mechanical properties of solids are expressed in terms
of various ‘moduli’ that are related to the intermolecular
(c)
potential energy of the constituents.
Figure 15D.1 Types of stress applied to a body. (a) Uniaxial stress,
➤ What do you need to know already?
(b) pure shear stress, (c) hydrostatic pressure.
You need to be familiar with the Lennard-Jones potential
energy (Topic 14B).

Yield point

Plastic
deformation
The fundamental concepts needed for the discussion of the me-
Stress

chanical properties of solids are stress and strain. The stress on


an object is the applied force divided by the area to which the
force is applied. For instance, if a mass m hangs from a wire Elastic deformation
of radius r, and therefore cross-section πr2, the mass exerts a
gravitational force mg on the wire. The uniaxial stress along the
length of the wire would be reported as mg/πr2. The strain is Strain
the resulting fractional distortion of the object. The study of the
relation between stress and strain is called rheology from the Figure 15D.2 At small strains, a body obeys Hooke’s law (stress
proportional to strain) and is elastic (recovers its shape when the
Greek word for ‘flow’.
stress is removed). At high strains, the body is no longer elastic,
Stress may be applied in different ways (Fig. 15D.1):
may become plastic, and finally yield.
• Uniaxial stress is a stress, in this case a compression or
extension, applied in one direction.
threshold, the strain becomes plastic in the sense that recovery
• Hydrostatic stress is a stress applied simultaneously in all does not occur when the stress is removed. Plastic deformation
directions, as in a body immersed in a fluid. occurs when bond breaking takes place and, in pure metals,
• Pure shear is a stress that tends to push opposite faces of typically takes place through the agency of dislocations. Brittle
the sample in opposite directions. materials, such as ionic solids, exhibit sudden fracture as the
stress focused by cracks causes them to spread catastrophically.
A sample subjected to a low stress typically undergoes ­elastic
The response of a solid to an applied stress is commonly
deformation in the sense that it recovers its original shape
summarized by a number of coefficients of proportionality
when the stress is removed. For low stresses, the strain is lin-
known as moduli:
early proportional to the stress, and the stress–strain relation is
a Hooke’s law of force (Fig. 15D.2). The response becomes non- uniaxial stress
Young’s modulus: E =  (15D.1a)
linear at high stresses but may remain elastic. Above a certain uniaxial strain
684 15 Solids

Uniaxial The Young’s modulus of iron at room temperature is 215 GPa.


(normal) Therefore
Transverse strain
strain Shear strain 1.24 1010 kg m 1 s 2
uniaxial strain   0.0581
2.15  1011 kg m 1 s 2
 
Pa

which corresponds to elongation of the wire by 5.81 per cent.


(a) (b)

Figure 15D.3 (a) Uniaxial stress and the resulting uniaxial and
transverse strain; Poisson’s ratio indicates the extent to which a
body changes shape when subjected to a uniaxial stress. (b) Shear If neighbouring molecules interact by a Lennard-Jones po-
stress and the resulting strain. tential energy (Topic 14B), then the bulk modulus and the
compressibility of the solid are related to the Lennard-Jones pa-
rameter ε (the depth of the potential well) by
pressure
Bulk modulus: K =  (15D.1b)
fractional change in volume 8N A  1 V
K T   m  (15D.4)
shear stress Vm K 8N A 
Shear modulus: G =  (15D.1c)
shear strain
For the derivation of these relations, see A deeper look 15D.1,
‘Uniaxial strain’ refers to stretching and compression of the
available to read in the e-book accompanying this text. The
material in one direction, as shown in Fig. 15D.3a, and ‘shear
bulk modulus is large and the compressibility low (the solid
strain’ refers to the distortion arising from a shear stress, as de-
stiff) if the potential well is deep and the solid is dense (its
picted in Fig. 15D.3b. The fractional change in volume is δV/V,
molar volume small).
where δV is the change in volume of a sample of volume V;
The differing rheological characteristics of metals can be
similarly, the uniaxial strain and the shear strain are (dimen-
traced to the presence of slip planes, which are planes of
sionless) fractional changes in dimensions. The bulk modulus
atoms that, when under stress, may slip or slide relative to
is the inverse of the isothermal compressibility, κT, discussed in
one another. The slip planes of a ccp structure are the close-
Topic 2D (eqn 2D.7, κT = −(∂V/∂p)T/V).
packed planes, and careful inspection of a unit cell shows
A third ratio, called Poisson’s ratio, indicates how the sample
that there are eight sets of slip planes in different directions.
changes its shape:
As a result, metals with ccp structures, like copper, are mal-
transverse strain leable, meaning they can easily be bent, flattened, or ham-
P  on]
Poisson’s ratio [definitio (15D.2)
normal strain mered into shape. In contrast, a hexagonal close-packed
structure has only one set of slip planes so that metals with
Transverse and normal strains are illustrated in Fig. 15D.3a:
hexagonal close packing, such as zinc or cadmium, tend to
they are the mutually perpendicular uniaxial distortions aris-
be more brittle.
ing from the ‘normal’ uniaxial stress. The three moduli intro-
duced in eqn 15D.1 are interrelated in the following way (see
Problem P15D.1):

E E
Exercises
Relations between
G K moduli
 (15D.3) E15D.1 The bulk modulus of polystyrene is 3.43 GPa. Calculate the hydrostatic
2(1   P ) 3(1  2 P )
pressure (stress) needed for a sample of this material to change volume by
1.0 per cent.
E15D.2 The Young’s modulus of polystyrene is 4.42 GPa. A polystyrene rod of
Brief illustration 15D.1
diameter 2.0 mm is subject to a force of 500 N along its length. Calculate the
stress and hence the percentage increase in the length of the rod when the
The uniaxial stress when a mass of m = 10.0 kg is suspended stress is applied.
from an iron wire of radius r = 0.050 mm is
E15D.3 Poisson’s ratio for polyethene is 0.45. What change in volume takes
(10.0 kg )  (9.81 ms 2 ) place when a cube of polyethene of volume 1.0 cm3 is subjected to a uniaxial
mg
uniaxial stress   stress that produces a strain of 1.0 per cent?
r 2 (5.0  105 m)2
 1.24 1010 kg m 1 s 2
15D The mechanical properties of solids 685

Checklist of concepts
☐ 1. Uniaxial stress is a simple stress (compression or exten- ☐ 5. The response of a solid to an applied stress is summa-
sion) applied to a sample in one direction. rized by the Young’s modulus, the bulk modulus, the
☐ 2. Hydrostatic stress is a stress applied simultaneously in shear modulus, and Poisson’s ratio.
all directions, as in a body immersed in a fluid. ☐ 6. The differing rheological characteristics of metals can be
☐ 3. Pure shear is a stress that tends to push opposite faces of traced to the presence of slip planes.
the sample in opposite directions.
☐ 4. A sample subjected to a small stress typically undergoes
elastic deformation; as the stress increases the sample
becomes plastic.

Checklist of equations
Property Equation Comment Equation number
Young’s modulus E = uniaxial stress/uniaxial strain Definition 15D.1a
Bulk modulus K = pressure/fractional change in volume Definition 15D.1b
Shear modulus G = shear stress/shear strain Definition 15D.1c
Poisson’s ratio νP = transverse strain/normal strain Definition 15D.2
TOPIC 15E The electrical properties
of solids

• A metallic conductor is a substance with an electrical


➤ Why do you need to know this material? conductivity that decreases as the temperature is raised.
The electrical properties of solids underlie numerous tech- • A semiconductor is a substance with an electrical conduc-
nological applications on which the infrastructure of the tivity that increases as the temperature is raised.
modern world depends. • A superconductor is a solid that, below a critical tempera-
ture, conducts electricity without resistance.
➤ What is the key idea?
Electrons in solids occupy bands that determine the elec- A semiconductor generally has a lower conductivity than that
trical conductivities of various types of solid. typical of metallic conductors, but the magnitude of the con-
ductivity is not the criterion for distinguishing between them.
➤ What do you need to know already? It is conventional to classify semiconductors with very low
You need to be familiar with the formation of bands in electrical conductivities, such as most synthetic polymers, as
solids (Topic 15C). insulators. The term is one of convenience rather than one of
fundamental significance.

The electrical conductivity of common materials arises from 15E.1 Metallic conductors
the motion of electrons, but some ionic solids display ionic
conductivity in which whole ions migrate through the lattice. To understand the origins of the electric conductivity in
Three types of solid are distinguished by the temperature de- ­conductors and semiconductors, it is necessary to explore the
pendence of their electrical conductivity (Fig. 15E.1): consequences of the formation of bands (Topic 15C). The start-
ing point is Fig. 15C.8, which is repeated here for convenience
as Fig. 15E.2. It shows the electronic structure of a solid formed
from a line of N atoms, each of which contributes one electron
108
Metallic conductor (such as the alkali metals). At T = 0, only the lowest 12 N molec-
ular orbitals are occupied, up to the Fermi level. The levels are
Superconductor
very closely spaced, so there are unoccupied molecular orbitals
Conductivity/(S cm–1)

104
just above the Fermi level. A solid with a partially filled band is
1
Semiconductor

10–4

Unoccupied levels
Energy

10–8
0 10 100 1000 Fermi level
Temperature, T/K
Occupied levels
Figure 15E.1 The variation of the electrical conductivity of a
substance with temperature is the basis of its classification as
a metallic conductor, a semiconductor, or a superconductor.
Conductivity is expressed in siemens per metre (S m−1 or, as here, Figure 15E.2 (A reproduction of Fig. 15C.8.) When N electrons
S cm−1), where 1 S = 1 Ω−1 (the resistance is expressed in ohms, Ω); occupy a band of N orbitals at T = 0, it is only half full. The highest
note the log scale. occupied level is the Fermi level.
15E The electrical properties of solids 687

Conduction
Energy
Fermi level band

Band

Energy
Thermal
gap, Eg excitation

(a) (b) (c)


Valence
Figure 15E.3 Each part of this illustration separates out the band
waves travelling in opposite directions. (a) The two sets of waves
have the same energy, are equally occupied, and there is no net (a) T = 0 (b) T > 0
motion. (b) When a potential difference is applied (positive on the
right) the two sets no longer have the same energy. There are now Figure 15E.4 (a) Typical band structure for a semiconductor:
more electrons moving to the right than to the left and therefore at T = 0 the valence band is full and the conduction band is
there is a net current. (c) If the band is full, the two sets remain empty. (b) At higher temperatures electrons populate the levels
equally populated and there is no net flow even when a potential of the conduction band leading to electrical conductivity which
difference is applied. increases with temperature.

at a higher energy, separated from the top of the lower band by


expected to be a metallic conductor, an observation which can an energy Eg, known as the band gap. At T = 0 this material is
be understood in the following way. an insulator (formally, a low conductivity semiconductor) be-
The key point is that each molecular orbital in a band can cause there are no partially filled bands. If the temperature is
be regarded as the superposition of two waves travelling in high enough electrons are excited out of the lower band into
opposite directions (in the same sense that cos x ∝ eix + e−ix). the upper. There are now incomplete bands and conduction can
Figure 15E.3a is an adaptation of Fig. 15E.2 in which the waves occur.
are separated into two components according to their direction The question that now arises is how the populations of the
of travel. In the absence of any applied field, electrons occupy two bands, and therefore the conductivity of the semiconduct-
both components equally and there is no net motion through ing material, depend on the temperature. The discussion starts
the solid. This absence of net motion is true whether the band by introducing the density of states, ρ(E), defined such that the
is full or incomplete. When a potential difference is applied the number of states between E and E + dE is ρ(E)dE. Note that
energies of the components differ, as it is energetically favour- the ‘state’ of an electron includes its spin, so each spatial orbital
able for electrons to travel towards regions of positive poten- counts as two states. To obtain the number of electrons dN(E)
tial. Now the two components are no longer equally occupied that occupy states between E and E + dE, ρ(E)dE is multiplied
(Fig. 15E.3b) and provided the band is not full there are more by the probability f(E) of occupation of the state with energy E.
electrons travelling in one direction than the other and electri- That is,
cal conduction occurs. If the band is full, however, the popu-
Number of
lations of the two components remain equal (Fig. 15E.3c) and states
Probability of
occupation of
there is no net motion in either direction. The material does not between a state with
E and E  dE
conduct: it is an insulator. 
    energy E

The conductivity of a metallic conductor decreases as the dN (E )   (E )dE  f (E )  (15E.1)


temperature is raised. This decrease is due to collisions between
the moving electrons and the atoms. The higher the temper- The function f(E) is the Fermi–Dirac distribution, a version of
ature, the more vigorous is the thermal motion of the atoms, the Boltzmann distribution that takes into account the Pauli ex-
so collisions between the moving electrons and an atom are clusion principle, that each orbital can be occupied by no more
more likely. That is, the electrons are scattered out of their paths than two electrons:
through the solid, and are less efficient at transporting charge.
1
f (E) = Fermi–Dirac distribution  (15E.2a)
e( E − μ )/kT + 1
In this expression μ is a temperature-dependent parameter
15E.2 Insulators and semiconductors known as the ‘chemical potential’ (it has a subtle relation to the
familiar chemical potential of thermodynamics); provided T > 0,
Now consider a solid which has an arrangement of bands as μ is the energy of the state for which f = 12 . At T = 0, only states
shown in Fig. 15E.4. At T = 0 the lower band is full, and the up to a certain energy, known as the Fermi energy, EF, are
Fermi level lies at the top of the band. A second empty band lies ­occupied. Provided the temperature is not so high that many
688 15 Solids

electrons are excited to states above the Fermi energy, the the conductivity increases as more electrons are promoted
chemical potential can be identified with EF, in which case the across the band gap, so the material is a semiconductor.
Fermi–Dirac distribution becomes The lower band, which is full at T = 0, is called the valence
1 band and the upper band, which is empty at T = 0 and to which
f (E) = Fermi–Dirac distribution  (15E.2b) electrons are thermally excited, is called the conduction band.
e( E − EF )/kT + 1
When electrons leave the valence band they can be thought
This expression implies that f (EF ) = 12 . For energies well above of as creating positively charged ‘holes’ in that band (i.e. the
EF, the exponential term is so large that the 1 in the denomina- absence of an electron), and the electrical conductivity arises
tor can be neglected, and then from the movement of these holes and the promoted electrons.
Fermi–Dirac distribution Figure 15E.4 depicts an intrinsic semiconductor, in which
f (E ) ≈ e −( E − EF )/kT  (15E.2c)
[approximate form for E > EF ] semiconduction is a property of the band structure of the pure
material. Examples of intrinsic semiconductors include silicon
The function now resembles a Boltzmann distribution, decay-
and germanium. A compound semiconductor is an intrinsic
ing exponentially with increasing energy; the higher the tem-
semiconductor formed from a combination of different ele-
perature, the longer is the exponential tail.
ments, such as GaN, CdS, and many d-metal oxides.
There is a distinction between the Fermi energy and the
An extrinsic semiconductor is one in which charge carriers
Fermi level:
(electrons or holes) are present as a result of the replacement of
• The Fermi level is the uppermost occupied level at T = 0. some atoms (to the extent of about 1 in 109) by dopant atoms, the
• The Fermi energy is the energy level at which f (E ) = 12 at atoms of another element. If, for example, pure silicon (a Group
any temperature. 14 element) is doped with atoms of indium (a Group 13 element)
an electron can be transferred from a Si atom to a neighbouring
The Fermi energy coincides with the Fermi level as T → 0. In atom, thereby creating a hole in the valence band and increas-
Figure 15E.5 shows the form of f(E) at different temperatures. ing the conductivity. The resulting semiconductor is described
At T = 0 the probability distribution is a step function, equal to as p-type, the p indicating that the positive holes are responsible
1 for E < EF, and 0 at higher energies, as in Fig. 15E.2. At higher for conduction. Figure 15E.6a shows the band structure of such
temperatures the probability of occupation of levels above EF a semiconductor. The dopant atoms result in a set of empty lev-
increases at the expense of those below EF, with the greatest els, called acceptor levels, which lie just above the top of the va-
changes occurring in the energies close to EF. As the tempera- lence band. Electrons from the valance band are transferred into
ture is raised, electrons are promoted from the lower band to the these levels, thereby generating holes in the band.
upper. This promotion is represented by the tail of the Fermi– If the dopant atoms are from a Group 15 element (e.g. phos-
Dirac distribution extending across the band gap and is signifi- phorus), an electron can be transferred from a P atom into the
cant only when kT is comparable to or greater than the band gap. otherwise empty conduction band, thereby increasing the con-
The material, an insulator at T = 0, is now a conductor, because ductivity. This type of doping results in an n-type ­semiconductor,
both bands are partially filled. As the temperature is increased,

1
T= 0
Probability of occupation, f

0.8 Acceptor
Energy

Donor
levels
levels
0.6

1/10
0.4
1/3
0.2
1 (a) (b)
3
10
0 Figure 15E.6 (a) A dopant with fewer electrons than its host
–6 –4 –2 0 2 4 6
(E – EF)/ EF contributes levels that accept electrons from the valence band.
The resulting holes in the band give rise to electrical conductivity;
Figure 15E.5 The Fermi–Dirac distribution, which gives the the doped semiconductor is classified as p-type. (b) A dopant
probability of occupation of a state with energy E and at a with more electrons than its host contributes occupied levels that
temperature T. At higher energies the probability decays can supply electrons to the conduction band, thus giving rise
exponentially towards zero. The curves are labelled with the value to electrical conductivity; the substance is classed as an n-type
of EF/kT. semiconductor.
15E The electrical properties of solids 689

where n refers to the negative charge of the carriers. The band recombination stimulates lattice vibrations. This is the case for
structure is shown in Fig. 15E.6b. The dopant atoms create a silicon semiconductors, and is one reason why computers need
set of occupied levels, called donor levels, just below the bot- efficient cooling systems.
tom of the conduction band, and electrons from these levels are Another electronic device, a ‘transistor’, consists of a p-type
transferred into the conduction band. In practical cases the level semiconductor sandwiched between two n-type semiconduc-
of doping is such that the charge carriers created by the dopant tors, and as such has two p–n junctions. Under the correct condi-
atoms are greatly in excess of those arising from thermal exci- tions it is possible to control the current flowing between the two
tation across the band gap: electrical conductivity is therefore n-type semiconductors by varying the current flowing into the
dominated by the type and extent of the doping. p-type semiconductor. Most significantly, the change in the current
Doped semiconductors are of great technological impor- between the n-type semiconductors can be larger than the change
tance because they are the materials from which the active com- in the current in the p-type semiconductor; in other words the de-
ponents of electronic circuits are made. The simplest example of vice can act as an amplifier. It is the exploitation of this property
an electronic device constructed from doped semiconductors that has led to the development of modern solid-state electronics.
is the ‘p–n diode’ which consists of a p-type semiconductor in
contact with an n-type semiconductor, thereby creating a p–n
junction. A p–n junction conducts electricity only in one di-
Exercises
rection. To understand this property consider first the arrange- E15E.1 Calculate f(E), the probability predicted by the Fermi–Dirac
distribution, for the case where the energy E is the thermal energy, kT, above
ment shown in Fig. 15E.7a where the p-type semiconductor is the Fermi energy: E = EF + kT.
attached to the negative electrode and the n-type is attached to
E15E.2 Is arsenic-doped germanium a p-type or n-type semiconductor?
the positive electrode; this arrangement is known as ‘reverse
bias’. The positively charged holes in the p-type semiconduc-
tor are attracted to the negative electrode, and the negatively
charged electrons in the n-type semiconductor are attracted
to the positive electrode. As a consequence, charge does not 15E.3 Superconductors
flow across the junction so the device does not conduct. Now
consider what happens when the electrodes are connected the The resistance to flow of electrical current of a normal metallic
other way round, as shown in Fig. 15E.7b, an arrangement conductor decreases smoothly with decreasing temperature but
known as ‘forward bias’. Electrons in the n-type semiconduc- never vanishes. However, a superconductor conducts electric-
tor move towards the positive electrode, and holes move in the ity without resistance once the temperature is below the critical
opposite direction: as a result charge flows across the junction. temperature, Tc. Following the discovery in 1911 that mercury
The p–n junction therefore conducts only under forward bias. is a superconductor below 4.2 K, the normal boiling point of
As electrons and holes move across a p–n junction under liquid helium, physicists and chemists made slow but steady
forward bias, they recombine and release energy. However, as progress in the discovery of superconductors with higher val-
long as the forward bias persists, the flow of charge from the ues of Tc. Metals, such as tungsten, mercury, and lead, have Tc
electrodes to the junction replenishes them with electrons and values below about 10 K. Intermetallic compounds, such as
holes. In some solids, the energy of electron–hole recombi- Nb3X (X = Sn, Al, or Ge), and alloys, such as Nb/Ti and Nb/Zr,
nation is released as heat and the device becomes warm. The have intermediate Tc values ranging between 10 K and 23 K. In
reason lies in the fact that the return of the electron to a hole in- 1986, high-temperature superconductors (HTSCs) were dis-
volves a change in the electron’s linear momentum, which the covered. Several ceramics, inorganic powders that have been
atoms of the lattice must absorb, and therefore electron–hole fused and hardened by heating to a high temperature, contain-
ing oxocuprate motifs, CumOn, are now known with Tc values
n p n p well above 77 K, the boiling point of the inexpensive refrigerant
liquid nitrogen. For example, HgBa2Ca2Cu3O8 has Tc = 153 K.
Electron Hole The elements that exhibit superconductivity cluster in cer-
tain parts of the periodic table. The metals iron, cobalt, nickel,
+ – – + copper, silver, and gold do not display superconductivity; nor
do the alkali metals. One of the most widely studied oxocu-
prate superconductors YBa2Cu3O7 (informally known as ‘123’
on ­account of the proportions of the metal atoms in the com-
pound) has the structure shown in Fig. 15E.8. The square-­
(a) (b)
pyramidal CuO5 units arranged as two-dimensional layers and
Figure 15E.7 A p–n junction under (a) reverse bias, the square planar CuO4 units arranged in sheets are common
(b) forward bias. structural features of oxocuprate HTSCs.
690 15 Solids

Cu O

Ba

e–
Y

(a) (b)
Figure 15E.9 The formation of a Cooper pair. One electron
Figure 15E.8 Structure of the YBa2Cu3O7 superconductor. (a) Metal distorts the crystal lattice and the second electron has a lower
atom positions. (b) The polyhedra show the positions of oxygen energy if it goes to that region. These electron–lattice interactions
atoms and indicate that the Cu ions are either in square-planar or effectively bind the two electrons into a pair.
square-pyramidal coordination environments.

The mechanism of superconduction is well-understood for e­ lectron as it travels through the solid because the distortion
low-temperature materials, and is based on the properties of a caused by one electron can attract back the other electron
Cooper pair, a pair of electrons that exists on account of the should it be scattered out of its path in a collision. Because the
indirect electron–electron interactions mediated by the nuclei Cooper pair is stable against scattering, it can carry charge freely
of the atoms in the lattice. Thus, if one electron is in a particu- through the solid, and hence give rise to superconduction.
lar region of a solid, the nuclei there move towards it to give The Cooper pairs responsible for low-temperature supercon-
a distorted local structure (Fig. 15E.9). That local distortion is ductivity are likely to be important in HTSCs, but the mecha-
rich in positive charge, so it is favourable for a second electron nism for pairing is hotly debated. There is evidence implicating
to join the first. Hence, there is a virtual attraction between the the arrangement of CuO5 layers and CuO4 sheets in the mecha-
two electrons and they move together as a pair. The local dis- nism. It is believed that movement of electrons along the linked
tortion is disrupted by thermal motion of the ions in the solid, CuO4 units accounts for superconductivity, whereas the linked
so the virtual attraction occurs only at very low temperatures. CuO5 units act as ‘charge reservoirs’ that maintain an appropri-
A Cooper pair undergoes less scattering than an individual ate number of electrons in the superconducting layers.

Checklist of concepts
☐ 1. Electronic conductors are classified as metallic conduc- ☐ 5. In a semiconductor at T = 0 there is a full valence band
tors or semiconductors according to the temperature and, at higher energy, an empty conduction band.
dependence of their conductivities; an insulator is a ☐ 6. In an intrinsic semiconductor electrical conductivity is
semiconductor with very low conductivity. due to electrons thermally promoted from the valence
☐ 2. Superconductors conduct electricity without resistance band to the conduction band.
below a critical temperature Tc. ☐ 7. In an extrinsic semiconductor electrical conductivity is
☐ 3. The Fermi–Dirac distribution gives the probability due to electrons or holes generated by the inclusion of
that a state with a particular energy is occupied by an dopant atoms.
electron. ☐ 8. Semiconductors are classified as p-type or n-type
☐ 4. The Fermi energy is the energy of the level for which the according to whether conduction is due to holes in the
probability of occupation is 12 . valence band or electrons in the conduction band.

Checklist of equations
Property Equation Comment Equation number
( E  EF ) /kT
Fermi–Dirac distribution f (E )  1/{e  1} EF is the Fermi energy 15E.2b
TOPIC 15F The magnetic properties
of solids

where χ is the dimensionless volume magnetic susceptibility.


➤ Why do you need to know this material? A closely related quantity is the molar magnetic susceptibil-
The magnetic properties of solids give an indication of the ity, χm:
electronic structures of individual molecules and many  m  Vm Molar magnetic susceptibility [definition] (15F.2)
modern information storage devices make use of the
additional properties that arise when the spins on different where Vm is the molar volume of the substance.
centres interact. The magnetization can be thought of as contributing to the
density of lines of force in the material (Fig. 15F.1). Materials
➤ What is the key idea? for which χ > 0 are called paramagnetic; they tend to move into
The principal magnetic properties of solids arise from the a magnetic field and the density of lines of force within them,
spins of unpaired electrons and their interactions. formally the ‘magnetic flux density’, is greater than in a vacuum.
Those for which χ < 0 are called diamagnetic and tend to move
➤ What do you need to know already? out of a magnetic field; the density of lines of force within them
You need to be aware of the properties of electron angu- is lower than in a vacuum. A paramagnetic material consists
lar momentum (Topic 8B) and the relation of magnetic of ions or molecules with unpaired electrons, such as radicals
moments to angular momenta (Topic 8C). and many d-metal complexes; a diamagnetic substance (which
is far more common) is one with no unpaired electrons.
The magnetic susceptibility is traditionally measured with a
‘Gouy balance’. This instrument consists of a sensitive balance
from which the sample, contained in a narrow tube, hangs be-
The magnetic properties of metallic solids and semiconductors
tween the poles of a magnet. If the sample is paramagnetic, it
depend strongly on the band structures of the material. In this
is drawn into the field and its apparent weight is greater when
section, attention is confined largely to the much simpler mag-
the field is turned on. A diamagnetic sample tends to be ex-
netic properties of collections of individual molecules or ions,
pelled from the field and appears to weigh less when the field is
such as d-metal complexes. Much of the discussion therefore
turned on. The balance is normally calibrated against a sample
applies to liquid- and gas-phase samples, as well as to solids.
of known susceptibility.
The modern way of measuring magnetic susceptibility
makes use of a ‘superconducting quantum interference de-
vice’ (a SQUID, Fig. 15F.2). A SQUID makes use of the quan-
15F.1 Magnetic susceptibility tization of magnetic flux and the property of current loops in
Some molecules possess permanent magnetic dipole mo-
ments. In the absence of an external magnetic field, the ori-
entation of the dipole is random and the material has no
net magnetic moment. However, certain orientations are fa-
voured when a magnetic field is applied. The magnetization,
M , the net dipole-moment density, is the resulting average
molecular magnetic dipole moment multiplied by the num-
ber density of molecules in the sample. The magnetization in- (a) (b) (c)

duced by a magnetic field of strength H is proportional to H , Figure 15F.1 (a) In a vacuum, the strength of a magnetic field
and is written can be represented by the density of lines of force; (b) in a
diamagnetic material, the density is reduced; (c) in a paramagnetic
M  H Volume magnetic susceptibility [definition] (15F.1) material, the density is increased.
692 15 Solids

of the electron spins fluctuate. Some o­ rientations have lower


SQUID Magnetic energy than others, and the magnetization depends on the
field
­randomizing influence of thermal motion. Thermal averag-
ing of the permanent magnetic moments in the presence
Superconducting of an applied magnetic field results in a contribution to the
wire Current Sample magnetic susceptibility that is proportional to m2/3kT. It fol-
lows that the spin contribution to the molar magnetic sus-
ceptibility is

N A g e2 0 B2 S(S  1) Molar magnetic susceptibility


m  [spin contribution]
 (15F.4a)
3kT
Figure 15F.2 The arrangement used to measure magnetic
susceptibility with a SQUID. The sample is moved upwards in where μ0 is the magnetic constant (formerly, and still widely,
small increments and the potential difference across the SQUID is the ‘vacuum permeability’). This susceptibility is positive, so
measured. the spin magnetic moments contribute to the paramagnetic
susceptibilities of materials. Equation 15F.4a is commonly writ-
ten as the Curie law:
Table 15F.1 Magnetic susceptibilities at 298 K⋆
C N g 2   2 S(S  1)
χ/10−6 χm/(10−10 m3 mol−1) m  , C A e 0 B Curie law  (15F.4b)
T 3k
H2O(l) −9.02 −1.63
The spin contribution to the susceptibility decreases with in-
NaCl(s) −16 −3.8
creasing temperature because the thermal motion randomizes
Cu(s) −9.7 −0.69
the spin orientations.
CuSO4·5H2O(s) +167 +183

More values are given in the Resource section.
Brief illustration 15F.1
s­uperconductors that, as part of the circuit, include a weakly
conducting link through which electrons must tunnel. The cur- Consider a complex salt with three unpaired electrons per
complex cation and molar volume 61.7 cm3 mol−1; its molar
rent that flows in the loop in a magnetic field depends on the
magnetic susceptibility can be calculated using eqn 15F.4b.
value of the magnetic flux, and a SQUID can be exploited as a
First, note that
very sensitive magnetometer. Table 15F.1 lists some experimen-
tal values of the magnetic susceptibility. N A g e2 0 B2
 6.3001  106 m3 K mol 1
3k
Then with S = 23 eqn 15F.4b gives
Permanent and induced
15F.2   1
3 3

magnetic moments  m  (6.3001  106 m3 K mol 1 )  2 2


298 K
 7.92 108 m3 mol 1
The permanent magnetic moment of a molecule arises from
any unpaired electron spins in the molecule. The magnitude, m, and from eqn 15F.2
of the magnetic moment of an electron is proportional to the 8 3 1
magnitude of the spin angular momentum, {s(s +1)}1/ 2 :  m 7.92 10 m mol
   1.28  103
Vm 6.17  10 5 m 3 mol 1
e Magnetic
m  g e {s(s  1)}1/ 2 B B  moment  (15F.3)
2me [magnitude]
At low temperatures, some paramagnetic solids make a phase
where ge = 2.0023 and μB, the Bohr magneton, has the value transition to a state in which large domains of spins align with
9.274 × 10−24 J T−1. If there are several electron spins in each parallel orientations. This cooperative alignment gives rise
molecule, they combine to give a total spin S, and then s(s + 1) to a very strong magnetization and is called ­ferromagnetism
is replaced by S(S + 1). (Fig. 15F.3). In other cases, exchange interactions lead to al-
The magnetization, and consequently the magnetic suscep- ternating spin orientations: the spins are locked into a low-­
tibility, depends on the temperature because the o ­ rientations magnetization arrangement to give an antiferromagnetic phase.
15F The magnetic properties of solids 693

(a) Magnetic properties of


15F.3
superconductors
(b)

Superconductors have unique magnetic properties. Supercon­


(c)
ductors classed as Type I show abrupt loss of superconductivity
when an applied magnetic field exceeds a critical value H c char-
Figure 15F.3 (a) In a paramagnetic material, the electron spins are acteristic of the material. An empirical relation between the
aligned at random in the absence of an applied magnetic field. value of H c , the temperature T, and the critical temperature Tc is
(b) In a ferromagnetic material, the electron spins are locked into a
parallel alignment over large domains. (c) In an antiferromagnetic
 T2 
material, the electron spins are locked into an antiparallel H c (T )  H c (0)  1  2  Dependence H c on T  (15F.5)
arrangement. The latter two arrangements survive even in the  Tc 
absence of an applied field.
provided T ≤ Tc. Note that the critical field falls as T rises from
0 towards Tc. Therefore, to maintain superconductivity in the
The ferromagnetic phase has a non-zero magnetization in the
presence of a magnetic field, it is best to keep T well below Tc
absence of an applied field, but the antiferromagnetic phase has
and to select a material with a high H c (0).
zero magnetization because the spin magnetic moments cancel.
The ferromagnetic transition occurs at the Curie temperature,
and the antiferromagnetic transition occurs at the Néel tempera-
Brief illustration 15F.2
ture. Which type of cooperative behaviour occurs depends on
the details of the band structure of the solid. Lead has Tc = 7.19 K and H c (0)  63.9 kA m 1. At T = 6.0 K
Magnetic moments can also be induced in molecules. To the magnetic field that would quench its superconductivity
see how this effect arises, it is necessary to note that the circu- would be
lation of electronic currents induced by an applied field gives
rise to a magnetic field which usually opposes the applied field  0
.304 
and makes the substance diamagnetic. In these cases, the in- 1
 (6.0 K )2 
H c (6.0 K )  (63.9 kA m )  1   19 kAm 1
duced electronic currents occur within the molecular orbit-  (7.19 K )2 
 
als that are occupied in its ground state. There are a few cases
in which molecules are paramagnetic despite having no un-
The lead remains superconducting at 6.0 K for this and weaker
paired electrons. In these materials the induced electron cur- applied field strengths. If the temperature is lowered to 5.0 K
rents flow in the opposite direction because they can make the corresponding calculation gives H c (5.0 K )  33 kA m 1, and
use of unoccupied orbitals that lie close to the HOMO in en- superconductivity survives at higher field strengths. In each
ergy (a similar effect is the paramagnetic contribution to the case, superconductivity would survive to higher field strengths
chemical shift, Topic 12B). This orbital paramagnetism is dis- for a material with a higher H c (0) .
tinguished from spin paramagnetism by being independent of
temperature; it is called temperature-independent paramag-
netism (TIP).
These remarks can be summarized as follows. All molecules A magnetic field for which H > H c does not penetrate into a
have a diamagnetic component to their susceptibility, but this Type I superconductor at temperatures below Tc. This complete
contribution is dominated by spin paramagnetism if the mol- exclusion of a magnetic field from a material is known as the
ecules have unpaired electrons. In a few cases (where there are Meissner effect, which can be demonstrated by the levitation
low-lying excited states) TIP is strong enough to make the mol- of a superconductor above a magnet. Type II superconductors,
ecules paramagnetic even though all their electrons are paired. which include the HTSCs, show a gradual loss of diamagnetism
with increasing magnetic field.

Exercises
E15F.1 The magnitude of the magnetic moment of CrCl3 is 3.81μB. How many Exercise
unpaired electrons does the Cr possess? 1
E15F.3 Niobium, Nb, has Tc = 9.5 K and H c (0)  158 kA m . Calculate the
E15F.2 Estimate the spin-only molar susceptibility of CuSO4·5H2O at 25 °C. highest magnetic field at which superconductivity can be maintained at 6 K.
694 15 Solids

Checklist of concepts
☐ 1. The magnetization of a material is the average molecu- ☐ 6. Ferromagnetism is the cooperative alignment of elec-
lar magnetic dipole moment multiplied by the number tron spins in a material and gives rise to strong perma-
density of the molecules. nent magnetization.
☐ 2. The magnetic susceptibility expresses the relation ☐ 7. Antiferromagnetism results from alternating spin ori-
between the magnetization and the applied magnetic entations in a material and leads to weak magnetization.
field strength. ☐ 8. Temperature-independent paramagnetism arises from
☐ 3. Diamagnetic materials have negative magnetic suscep- induced electron currents.
tibilities and tend to move out of a magnetic field. ☐ 9. The Meissner effect is the exclusion of a magnetic field
☐ 4. Paramagnetic materials have positive magnetic suscep- from a Type I superconductor.
tibilities and tend to move into a magnetic field.
☐ 5. The Curie law describes the temperature dependence of
the magnetic susceptibility.

Checklist of equations
Property Equation Comment Equation number
Magnetization M  H Definition 15F.1
Molar magnetic susceptibility χm = χVm Definition 15F.2
Magnetic moment m = ge{s(s + 1)}1/2μB B  e/2me 15F.3
Curie law  m  C /T , C  N A g e2 0 B2 S(S  1)/3k Paramagnetism 15F.4b
Dependence of H c on T H c (T )  H c (0)(1  T 2 /Tc2 ) Empirical 15F.5
TOPIC 15G The optical properties
of solids
➤ Why do you need to know this material?
The optical properties of solids are of ever-increasing
importance in modern technology, not only for the gen-
eration of light but for the propagation and manipulation
of information.

➤ What is the key idea? Figure 15G.1 The electron–hole pair shown on the left can
The optical properties of molecules in solids differ from migrate through a solid lattice as the excitation hops from
molecule to molecule. The mobile excitation is called an exciton.
those of isolated molecules as a result of the interaction of
their transition dipoles.
on one molecule could not move to another. This interaction
➤ What do you need to know already? affects the energy levels of the system. The strength of the inter-
action also governs the rate at which an exciton moves through
You need to be familiar with the concept of transition
the crystal: a strong interaction results in fast migration and a
dipoles (Topics 8C and 11A) and of the band theory of
vanishingly small interaction leaves the exciton localized on its
solids (Topic 15C).
original molecule. The specific mechanism of interaction that
leads to exciton migration is the interaction between the tran-
sition dipoles of the excitation (Topic 11A). Thus, an electric
dipole transition in a molecule is accompanied by a shift of
Topic 11A explains the factors that affect the energy and inten- charge, and this transient dipole exerts a force on an adjacent
sity of light absorbed by isolated atoms and molecules in the molecule. The latter responds by shifting its charge. This pro-
gas phase and in solution. Significant differences arise when the cess continues and the excitation migrates through the crystal.
molecules are neighbours in a solid. The energy shift arising from the interaction between tran-
sition dipoles can be explained as follows. The potential en-
ergy, Ep, of interaction between two parallel electric dipole
moments μ1 and μ2 separated by a distance r is V = μ1μ2(1 −
15G.1 Excitons 3 cos2θ)/4πε0r3, where the angle θ is defined in (1). A head-to-tail
alignment corresponds to θ = 0, and a parallel alignment cor-
Consider an electronic excitation of a molecule (or an ion) in a responds to θ = 90°. From the expression for V it follows that
crystal. If the excitation corresponds to the removal of an elec- V < 0 (a favourable, energy-lowering interaction) for 0 ≤ θ < 54.7°,
tron from one orbital of a molecule and its elevation to an or- V = 0 when θ = 54.7° (at this angle 1 − 3 cos2θ = 0), and V > 0 (an
bital of higher energy, then the excited state of the molecule can unfavourable, energy-raising interaction) for 54.7° < θ ≤ 90°.
be envisaged as the coexistence of an electron and a hole. This
electron–hole pair, which behaves as a particle-like exciton, μ1
migrates from molecule to molecule in the crystal (Fig. 15G.1).
A migrating excitation of this kind is called a Frenkel exciton,
r
and is commonly found in molecular solids. The electron and
μ2 θ
hole can also be on different molecules, but in each other’s vi-
cinity. A migrating excitation of this kind, which is now spread
over several molecules (more usually ions), is called a Wannier 1
exciton. Exciton formation causes spectral lines to shift, split,
and change intensity. In a head-to-tail arrangement, there is a favourable interac-
The migration of a Frenkel exciton (the only type considered tion between the region of partial positive charge in one mol-
here) implies that there is an interaction between the molecules ecule and the region of partial negative charge in the other
that constitute the crystal: if this were not the case the ­excitation molecule. In contrast, in a parallel arrangement, the molecular
696 15 Solids

that are in-phase between neighbouring unit cells. Within


each unit cell the transition dipoles may be arrayed in the two
different ways shown in the illustration. The two orientations
correspond to different interaction energies, with interaction
(a) being unfavourable in one and favourable in the other, so the
two transitions appear in the spectrum as two bands of differ-
ent frequencies. The magnitude of the Davydov splitting is de-
termined by the energy of interaction between the transition
dipoles within the unit cell.

(b)

Figure 15G.2 (a) The alignment of transition dipoles (the arrows) 15G.2 Metals and semiconductors
shown here is energetically unfavourable, and the exciton
absorption is shifted to higher energy (higher frequency). (b) The Figure 15C.8 shows the band structure in an idealized metallic
alignment shown here is energetically favourable for a transition conductor at T = 0. The absorption of a photon can excite elec-
in this orientation, and the exciton band occurs at lower frequency trons from the occupied levels to the unoccupied levels. There
than in the isolated molecules. is a near continuum of unoccupied energy levels above the
Fermi level, so absorption occurs over a wide range of frequen-
cies. In metals, the bands are sufficiently wide that radiation is
interaction is unfavourable because of the close approach of re-
absorbed from the radiofrequency to the ultraviolet region of
gions of partial charge with the same sign. It follows that an all-
the electromagnetic spectrum but not to very high-frequency
parallel arrangement of the transition dipoles (Fig. 15G.2a) is
electromagnetic radiation, such as X-rays and γ-rays, so met-
energetically unfavourable, so the absorption occurs at a higher
als are transparent at these frequencies. This range of absorbed
frequency than in the isolated molecule. Conversely, a head-to-
frequencies includes the entire visible spectrum, so it might be
tail alignment of transition dipoles (Fig. 15G.2b) is energeti-
expected that all metals should be black. However, metals are in
cally favourable, and the transition occurs at a lower frequency
fact lustrous (that is, they reflect light) and some are coloured
than in the isolated molecules.
(that is, they absorb light of certain wavelengths), so the model
If there are N molecules per unit cell, there are N exciton
clearly needs some improvement.
bands in the spectrum (if all of them are allowed). The splitting
between the bands is the Davydov splitting. To understand the
origin of the splitting, consider the case N = 2 with the mol- 15G.2(a) Light absorption
ecules arranged as in Fig. 15G.3 and suppose that the transi-
To explain the lustrous appearance of a smooth metal sur-
tion dipoles are along the length of the molecules. The radiation
face, it is important to realize that the absorbed energy can be
stimulates the collective excitation of the transition dipoles
­reemitted very efficiently as light, with only a small fraction
of the energy being released into the bulk as heat. Because the
atoms near the surface of the material absorb most of the ra-
diation, emission also occurs primarily from the surface. In
essence, if the sample is excited with visible light, then elec-
trons near the surface are driven into oscillation at the same
(a) frequency, and visible light is emitted from the surface, so ac-
counting for the lustre of the material.
The perceived colour of a metal depends on the frequency
range of reflected light. That in turn depends on the frequency
(b) (a)
range of light that can be absorbed and, by extension, on the
band structure. Silver reflects light with nearly equal efficiency
(b)
across the visible spectrum because its band structure has
Davydov splitting many unoccupied energy levels that can be populated by ab-
sorption of, and depopulated by emission of, visible light. On
Figure 15G.3 When the transition moments within a unit cell
lie in different relative directions, as depicted in (a) and (b), the the other hand, copper has its characteristic colour because it
energies of the transitions are shifted and give rise to the two has relatively fewer unoccupied energy levels that can be ex-
bands labelled (a) and (b) in the spectrum. The separation of the cited with violet, blue, and green light. The material reflects at
bands is the Davydov splitting. all wavelengths, but more light is emitted at lower frequencies
15G The optical properties of solids 697

kind are widely used in electronic displays. The wavelength of


Conduction
band
emitted light depends on the band gap of the semiconductor.
Gallium arsenide itself emits infrared light, but its band gap is

Energy
Band
gap, Eg widened by incorporating phosphorus, and a material of com-
Valence
position approximately GaAs0.6P0.4 emits light in the red region
band of the spectrum.
A light-emitting diode is not a laser (Topic 11G) because
stimulated emission is not involved. In diode lasers, light
Figure 15G.4 In some materials, the band gap Eg is very large
and electron promotion can occur only by excitation with emission due to electron–hole recombination is employed
electromagnetic radiation. as the basis of laser action, and the population inversion can
be sustained by sweeping away the electrons that fall into the
holes of the p-type semiconductor. High-power diode lasers
are also used to pump other lasers. One example is the pump-
(corresponding to yellow, orange, and red). Similar arguments
ing of Nd:YAG lasers (Topic 11G) by Ga0.91Al0.09As/Ga0.7Al0.3As
account for the colours of other metals, such as the yellow of
diode lasers. Diode lasers are used widely in laser pointers,
gold. It is interesting to note that the colour of gold can be ac-
laser printers, medicine (instead of a scalpel in some surgical
counted for only by including relativistic effects in the calcula-
procedures), and telecommunications (where information en-
tion of its band structure.
coded in a modulated laser beam travels through fibre optic
Now consider semiconductors. If the band gap Eg is compa-
cables).
rable to kT the promotion of electrons from the conduction to
the valence band of a semiconductor can be the result of ther-
mal excitation. In some materials, the band gap is very large Exercise
and electron promotion can occur only by excitation with elec- E15G.1 The promotion of an electron from the valence band into the
tromagnetic radiation. However, as is seen from Fig. 15G.4, conduction band in pure TiO2 by light absorption requires a wavelength
there is a frequency νmin = Eg/h below which light absorption of less than 350 nm. Calculate the energy gap in electronvolts between the
cannot occur. Above this frequency threshold, a wide range of valence and conduction bands.
frequencies can be absorbed by the material, as in a metal.

Brief illustration 15G.1 15G.3 Nonlinear optical phenomena


The energy of the band gap in the semiconductor cadmium Nonlinear optical phenomena arise from changes in the op-
sulfide (CdS) is 2.4 eV (equivalent to 3.8 × 10−19 J). It follows tical properties of a material in the presence of intense elec-
that the minimum electronic absorption frequency is tromagnetic radiation. In frequency doubling (or ‘second
harmonic generation’), an intense laser beam is converted
3.8  1019 J to radiation with twice (and in general a multiple) of its ini-
 min   5.8  1014 s 1
6.626  1034 Js tial frequency as it passes through a suitable material. It fol-
lows that frequency doubling and tripling of an Nd:YAG
This frequency, of 580 THz, corresponds to a wavelength of laser, which emits radiation at 1064 nm (Topic 11G), produce
520 nm (green light). Lower frequencies, corresponding to yel- green light at 532 nm and ultraviolet radiation at 355 nm, re-
low, orange, and red, are not absorbed and consequently CdS spectively. Common materials that can be used for frequency
appears yellow-orange. doubling in laser systems include crystals of potassium dihy-
drogenphosphate (KH2PO4), lithium niobate (LiNbO3), and
β-barium borate (β-BaB2O4).
15G.2(b)Light-emitting diodes and Frequency doubling can be explained by examining how a
substance responds nonlinearly to incident radiation of fre-
diode lasers quency ω = 2πν. Radiation of a particular frequency arises
The unique electrical properties of p–n junctions between sem- from oscillations of an electric dipole at that frequency and
iconductors (which are described in Topic 15E) can be put to the incident electric field E of the radiation induces an elec-
good use in optical devices. In some materials, most notably tric dipole moment of magnitude μ, in the substance. At low
gallium arsenide, GaAs, energy from electron–hole recombi- light intensity, most materials respond linearly, in the sense that
nation is released not as heat but is carried away by photons    E, where α is the polarizability (Topic 14A). At high light
as electrons move across the junction driven by the appropri- intensity, the hyperpolarizability β of the material becomes im-
ate potential difference. Practical light-emitting diodes of this portant (Topic 14A) and the induced dipole becomes
698 15 Solids

Induced dipole  E 2   E02 cos 2 t  12  E02 (1  cos 2t ) (15G.2)


   E  12  E 2   moment in terms  (15G.1)
of the hyperpolaarizability
Hence, the nonlinear term contributes an induced electric di-
The nonlinear term β E can be expanded as follows if it is sup-
2 pole that includes a component that oscillates at the frequency
posed that the incident electric field is E0 cos ωt: 2ω and that can act as a source of radiation of that frequency.

Checklist of concepts
☐ 1. An exciton is an electron–hole pair caused by optical ☐ 3. Nonlinear optical phenomena arise from changes in
excitation in a solid; Frenkel excitons are localized on the optical properties of a material in the presence of
a single molecule, whereas Wannier excitons are spread intense electromagnetic radiation; they can give rise to
over several molecules. frequency doubling.
☐ 2. If the unit cell contains N molecules, there are N ­exciton
bands in the spectrum separated by the Davydov
­splitting.
Exercises and problems 699

FOCUS 15 Solids

To test your understanding of this material, work through Selected solutions can be found at the end of this Focus in
the Exercises, Additional exercises, Discussion questions, and the e-book. Solutions to even-numbered questions are available
Problems found throughout this Focus. online only to lecturers.

TOPIC 15A Crystal structure


Discussion questions
D15A.1 Describe the relationship between space lattice and unit cell. D15A.3 Draw unit cells representative of the three possible cubic lattices. State
how many lattice points are in each of your cells and identify whether or not
D15A.2 Explain how planes in a lattice are labelled.
the cells you have drawn are primitive.

Additional exercises
−1
E15A.4 An orthorhombic unit cell of a compound of molar mass 135.01 g mol E15A.6 Calculate the separations of the planes {112}, {110}, and {224} in a
has the dimensions a = 589 pm, b = 822 pm, and c = 798 pm. The mass density crystal in which the cubic unit cell has side 562 pm.
of the solid is estimated as 2.9 g cm−3. Identify the number of formula units in a
E15A.7 Calculate the separations of the planes {123}, {222}, and {246} in a
unit cell and calculate a more precise value of the mass density.
crystal in which the cubic unit cell has side 712 pm.
E15A.5 State the Miller indices of the planes that intersect the crystallographic
E15A.8 An orthorhombic unit cell has dimensions a = 769 pm, b = 891 pm,
axes at the distances (−a, 2b, −c) and (a, 4b, −4c).
and c = 690 pm. Calculate the spacing, d, of the {312} planes.

Problems
P15A.1 Although the crystallization of large biological molecules may not be pyridine-2,6-diamidoxime (1, C7H9N5O2). The compound they isolated
as readily accomplished as that of small molecules, their crystal lattices are no from the reaction of the ligand with CuSO4(aq) did not contain a
different. The protein tobacco seed globulin forms face-centred cubic crystals [Cu(C7H9N5O2)2]2+ complex cation as expected. Instead, X-ray diffraction
with unit cell dimension of 12.3 nm and a mass density of 1.287 g cm−3. analysis revealed a linear polymer of formula [{Cu(C7H9N5O2)(SO4)}·2H2O]n,
Determine its molar mass (assume there is one molecule associated with each which features bridging sulfate groups. The unit cell was primitive monoclinic
lattice point). with a = 1.0427 nm, b = 0.8876 nm, c = 1.3777 nm, and β = 93.254°. The mass
density of the crystals is 2.024 g cm−3. How many monomer units are there in
P15A.2 Show that the volume of a monoclinic unit cell is V = abc sin β.
the unit cell?
P15A.3 Derive an expression for the volume of a hexagonal unit cell.

P15A.4 Show that the volume of a triclinic unit cell of sides a, b, and c and
angles α, β, and γ is O O
V  abc(1  cos   cos   cos   2 cos  cos  cos  )
2 2 2 1/ 2
HO N OH
N N
Use this expression to derive expressions for monoclinic and orthorhombic H H
unit cells. For the derivation, it may be helpful to use the result from vector
analysis that V = a·b×c and to calculate V2 initially. The compound Rb3TlF6
has a tetragonal unit cell with dimensions a = 651 pm and c = 934 pm. 1 Pyridine-2,6-diamidoxime
Calculate the volume of the unit cell.
P15A.5 The volume of a monoclinic unit cell is abc sin β. Naphthalene has
a monoclinic unit cell with two molecules in each cell and sides in the ratio ‡
P15A.8 D. Sellmann et al. (Inorg. Chem. 36, 1397 (1997)) describe
1.377:1:1.436. The angle β is 122.82° and the mass density of the solid is the synthesis and reactivity of the ruthenium nitrido compound
1.152 g cm−3. Calculate the dimensions of the cell. [N(C4H9)4][Ru(N)(S2C6H4)2]. The ruthenium complex anion has the two
1,2-benzenedithiolate ligands (2) at the base of a rectangular pyramid and
P15A.6 Fully crystalline polyethene has its chains aligned in an
the nitrido ligand at the apex. Compute the mass density of the compound
orthorhombic unit cell of dimensions 740 pm × 493 pm × 253 pm. There
given that it crystallizes with an orthorhombic unit cell with a = 3.6881 nm,
are two repeating CH2CH2 units in each unit cell. Calculate the theoretical
b = 0.9402 nm, and c = 1.7652 nm and eight formula units in each cell. The
mass density of fully crystalline polyethene. The actual mass density ranges
replacement of the ruthenium with osmium results in a compound with the
from 0.92 to 0.95 g cm−3.

P15A.7 B.A. Bovenzi and G.A. Pearse, Jr (J. Chem. Soc. Dalton Trans. 2793

(1997)) synthesized coordination compounds of the tridentate ligand These problems were supplied by Charles Trapp and Carmen Giunta.
700 15 Solids

same crystal structure and a unit cell with a volume less than 1 per cent larger. P15A.9 Show that the separation of the {hkl} planes in an orthorhombic crystal
Estimate the mass density of the osmium analogue. with sides a, b, and c is given by eqn 15A.1b.

S–

S–

2 1,2-Benzenedithiolate ion

TOPIC 15B Diffraction techniques


Discussion questions
D15B.1 What is meant by a systematic absence? How do they arise and how D15B.3 Describe the consequences of the phase problem in determining
can they be helpful in identifying the type of unit cell? structure factors and how the problem is overcome.
D15B.2 Discuss what is meant by ‘scattering factor’. How is it related to the
number of electrons in the atoms scattering X-rays?

Additional exercises
E15B.6 The angle of a Bragg reflection from a set of crystal planes separated by E15B.16 Calculate the structure factors for a body-centred cubic (cubic I)
99.3 pm is 20.85°. Calculate the wavelength of the X-rays. structure in which the scattering factor of the central ion is twice that of the
ions at the corners of the cube.
E15B.7 The angle of a Bragg reflection from a set of crystal planes separated by
128.2 pm is 19.76°. Calculate the wavelength of the X-rays. E15B.17 In an X-ray investigation, the following structure factors were
determined (with F−h00 = Fh00):
E15B.8 What are the values of the angle θ of the three diffraction lines with
the smallest θ expected from a cubic F unit cell with lattice parameter 407 pm h 0 1 2 3 4 5 6 7 8 9
when the X-ray wavelength is 129 pm?
Fh00 10 −10 8 −8 6 −6 4 −4 2 −2
E15B.9 Potassium nitrate crystals have orthorhombic unit cells of dimensions
Construct the electron density along the corresponding direction.
a = 542 pm, b = 917 pm, and c = 645 pm. Calculate the values of θ for the
(100), (010), and (111) reflections using radiation of wavelength 154 pm. E15B.18 In an X-ray investigation, the following structure factors were
determined (with F−h00 = Fh00):
E15B.10 Calcium carbonate crystals in the form of aragonite have
orthorhombic unit cells of dimensions a = 574.1 pm, b = 796.8 pm, and h 0 1 2 3 4 5 6 7 8 9
c = 495.9 pm. Calculate the values of θ for the (100), (010), and (111)
Fh00 10 10 4 4 6 6 8 8 10 10
reflections using radiation of wavelength 83.42 pm.
Construct the electron density along the corresponding direction.
E15B.11 Radiation from an X-ray source consists of two components of
wavelengths 154.433 pm and 154.051 pm. Calculate the difference in glancing E15B.19 Construct the Patterson map from the information in Exercise
angles (2θ) of the diffraction lines arising from the two components in a E15B.17.
diffraction pattern from planes of separation 77.8 pm.
E15B.20 Construct the Patterson map from the information in Exercise
E15B.12 Consider a source that emits X-radiation at a range of wavelengths, E15B.18.
with two components of wavelengths 93.222 and 95.123 pm. Calculate the
E15B.21 In a Patterson map, the spots correspond to the lengths and directions
separation of the glancing angles (2θ) arising from the two components in a
of the vectors joining the atoms in a unit cell. Sketch the pattern that would be
diffraction pattern from planes of separation 82.3 pm.
obtained for a planar, triangular isolated BF3 molecule.
E15B.13 What is the value of the scattering factor in the forward direction
E15B.22 In a Patterson map, the spots correspond to the lengths and directions
for Mg2+?
of the vectors joining the atoms in a unit cell. Sketch the pattern that would be
E15B.14 The coordinates, in units of a, of the atoms in a cubic I unit cell are (0,0,0), obtained from the carbon atoms in an isolated benzene molecule.
(0,1,0), (0,0,1), (0,1,1), (1,0,0), (1,1,0), (1,0,1), (1,1,1), and  12 , 12 , 12 . Calculate the
E15B.23 What speed should electrons have if they are to have a wavelength of
structure factor Fhkl when all the atoms are identical. Where possible, simplify your
105 pm?
expression by using einπ = (−1)n, as in Example 15B.1.
E15B.24 Calculate the wavelength of electrons that have reached thermal
E15B.15 Calculate the structure factors for an orthorhombic C structure in which
equilibrium by collision with a moderator at 380 K.
the scattering factors of the two ions on the faces are twice that of the ions at the
corners of the cube. Assume that a = b = c, that is the unit cell is a cube.

Problems
P15B.1 In the early days of X-ray crystallography there was an urgent need to angle from a mechanically ruled grating. Another method was to estimate the
know the wavelengths of X-rays. One technique was to measure the diffraction separation of lattice planes from the measured density of a crystal. The mass
Exercises and problems 701

density of NaCl is 2.17 g cm−3 and the (100) reflection using radiation of a for 0 ≤ r ≤ R and ρ(r) = 0 for r > R, with R a parameter that represents the
certain wavelength occurred at 6.0°. Calculate the wavelength of the X-rays. radius of the atom. Explore how f varies with Z and R.
P15B.2 The element polonium crystallizes in a cubic system. Bragg reflections, P15B.6 The coordinates of the four I atoms in the unit cell of KIO4 are (0,0,0),
with X-rays of wavelength 154 pm, occur at sin θ = 0.225, 0.316, and 0.388  0, 12 , 12 ,  12 , 12 , 12 ,  12 ,0, 34  . By calculating the phase of the I reflection in the
from the {100}, {110}, and {111} sets of planes. The separation between the structure factor, show that the I atoms contribute no net intensity to the (114)
sixth and seventh lines observed in the diffraction pattern is larger than reflection.
between the fifth and sixth lines. Is the unit cell cubic P, I, or F? Calculate the
P15B.7 The coordinates, as multiples of a, of the A atoms, with scattering
unit cell dimension.
factor fA, in a cubic lattice are (0,0,0), (0,1,0), (0,0,1), (0,1,1), (1,0,0),
P15B.3 Elemental silver reflects X-rays of wavelength 154.18 pm at angles of (1,1,0), (1,0,1), and (1,1,1). There is also a B atom, with scattering factor fB,
at  2 , 2 , 2 . Calculate the structure factors Fhkl and predict the form of the
1 1 1
19.076°, 22.171°, and 32.256°. However, there are no other reflections at angles
of less than 33°. Assuming a cubic unit cell, determine its type and dimension. diffraction pattern when (a) fA = f, fB = 0, (b) f B = 12 f A, and (c) fA = fB = f.
Calculate the mass density of silver. Hint: Calculate the expected reflections
P15B.8 Here we explore electron diffraction patterns. (a) Predict from the
from different types of cubic unit cell and compare those with the data given.
Wierl equation, eqn 15B.8, the positions of the first maximum and first
P15B.4 In their book X-rays and crystal structures (which begins ‘It is now minimum in the neutron and electron diffraction patterns of a Br2 molecule
two years since Dr. Laue conceived the idea …’) the Braggs give a number of obtained with neutrons of wavelength 78 pm wavelength and electrons of
simple examples of X-ray analysis. For instance, they report that the reflection wavelength 4.0 pm. (b) Use the Wierl equation to predict the appearance of
from {100} planes in KCl occurs at 5.38°, but for NaCl it occurs at 6.00° for the electron diffraction pattern of CCl4 with an (as yet) undetermined C–Cl
X-rays of the same wavelength. If the side of the NaCl unit cell is 564 pm, bond length but of known tetrahedral symmetry; assume the electron energy
what is the side of the KCl unit cell? The mass densities of KCl and NaCl are to be 10 keV. Take fCl = 17f and fC = 6f and note that R(Cl,Cl) = (8/3)1/2R(C,Cl).
1.99 g cm−3 and 2.17 g cm−3 respectively. Do these values support the X-ray Plot I/f 2 against positions of the maxima occurred at 3.17°, 5.37°, and 7.90°
analysis? and minima occurred at 1.77°, 4.10°, 6.67°, and 9.17°. What is the C–Cl bond
length in CCl4?
P15B.5 Use mathematical software to draw a graph of the scattering factor f
against (sin θ)/λ for an atom of atomic number Z for which ρ(r) = 3Z/4πR3

TOPIC 15C Bonding in solids


Discussion questions
D15C.1 In what respects is the hard-sphere model of metallic solids D15C.2 Describe the caesium-chloride and rock-salt structures in terms of the
deficient? occupation of holes in expanded close-packed lattices.

Additional exercises
E15C.6 Calculate the packing fraction for equilateral triangular rods stacked as E15C.8 Determine the radius of the smallest anion that can have (i) sixfold and
shown in (3). (ii) eightfold coordination with the Rb+ ion (radius 149 pm).
E15C.9 Does iron expand or contract as it transforms from hcp to bcc? The
atomic radius of iron is 126 pm in hcp but 122 pm in bcc.
E15C.10 Calculate the lattice enthalpy of MgBr2 from the following data:

ΔH/(kJ mol−1)
Sublimation of Mg(s) +148
Ionization of Mg(g) to Mg2+(g) +2187
Vaporization of Br2(l) +31
Dissociation of Br2(g) +193
3
Electron attachment to Br(g) −331
E15C.7 Calculate the packing fraction for an orthorhombic C cell in which all
Formation of MgBr2(s) from Mg(s) −524
three sides are the same (assume that the spheres touch along one of the face
and Br2(l) in their reference states.
diagonals which includes an atom on the face).

Problems
P15C.1 Calculate the atomic packing factor for diamond (refer to Fig. 15C.15); P15C.2 Rods of elliptical cross-section with semi-minor and semi-major
assume that the atoms touch along the body diagonal. axes a and b are close-packed as shown in (4). What is the packing
702 15 Solids

fraction? Draw a graph of the packing fraction against the eccentricity ε P15C.6 The energy levels of N atoms in the tight-binding Hückel
of the ellipse. For an ellipse with semi-minor axis a and semi-major axis b, approximation are the roots of a tridiagonal determinant (eqn 15C.1):
ε = (1 − b2/a2)1/2.
k
Ek    2  cos k  1,2, ,N
N 1

a If the atoms are arranged in a ring, the solutions are the roots of a ‘cyclic’
b determinant:

2k 
Ek    2  cos k  0,  1,  2, ,  12 N
N
(for N even). Discuss the consequences, if any, of joining the ends of an
initially straight length of material.
P15C.7 Verify that the lowest value of the radius ratio for (a) sixfold
4 coordination is 0.414, and (b) for eightfold coordination is 0.732.
P15C.3 (a) Calculate the mass density of diamond assuming that it is a close-
P15C.8 (a) Use the Born–Mayer equation for the lattice enthalpy and a Born–
packed structure of hard spheres with radii equal to half the carbon–carbon Haber cycle to estimate the enthalpy of formation of CaCl(s). The sublimation
bond length of 154.45 pm. (b) The diamond lattice is in fact based on a enthalpy of Ca(s) is 176 kJ mol−1 and it can be assumed that the ionic radius
face-centred cubic lattice but with two atoms per lattice point, such that the of Ca+ is close to that of K+; other necessary data are to be found in Example
structure consists of two interpenetrating fcc lattices, one with its origin 15C.2 or in the tables in the Resource section. (b) Show that an explanation
at (0,0,0) and the other with its origin at (1/4,1/4,1/4). The experimentally for the nonexistence of CaCl(s) can be found in the reaction enthalpy for the
determined mass density is 3.516 g cm−3: can you explain the difference disproportionation reaction 2 CaCl(s) → Ca(s) + CaCl2(s).
between this value and that in (a)?
P15C.9 Derive the Born–Mayer equation (eqn 15C.5) by calculating the
P15C.4 When energy levels in a band form a continuum, the density of states
energy at which d(Ep  Ep )/dd  0 , with Ep and Ep given by eqns 15C.3 and
ρ(E), the number of levels in an energy range divided by the width of the 15C.4, respectively.
range, may be written as ρ(E) = dk/dE, where dk is the change in the quantum
number k and dE is the energy change. (a) Use eqn 15C.1 to show that P15C.10 Suppose that ions are arranged in a (somewhat artificial) two-
dimensional lattice like the fragment shown in the figure below. Calculate the
(N  1)/2  Madelung constant for the central ion in this array.
 (E)   1/ 2
  E   2  
1    
  2   

where k, N, α, and β have the meanings described in Topic 15C. (b) Use this
expression to show that ρ(E) becomes infinite as E approaches α ± 2β. That is, etc.
show that the density of states increases towards the edges of the bands in a
one-dimensional metallic conductor.
P15C.5 In three dimensions the variation of density of states is like that shown
in the figure below. Account for the fact that in a three-dimensional solid the
greatest density of states is near the centre of the band and the lowest density
is at the edges.

+
p Band –
Energy

s Band

Density of states, 

TOPIC 15D The mechanical properties of solids


Discussion question
D15D.1 Distinguish between the behaviour of a solid which undergoes elastic
deformation when a stress is applied to one which undergoes plastic deformation.
Exercises and problems 703

Additional exercises
3
E15D.4 A sample of polystyrene of volume 1.0 cm is subjected to a hydrostatic E15D.6 Poisson’s ratio for lead is 0.41. What change in volume takes place
pressure (stress) of 1000 bar. Calculate the volume of the sample after the when a cube of lead of volume 1.0 dm3 is subjected to a uniaxial stress that
pressure has been applied. The bulk modulus of polystyrene is 3.43 GPa. produces a strain of 2.0 per cent?
E15D.5 Calculate the force which needs to be applied to a polystyrene rod
of diameter 1.0 mm to increase its length from 10.00 cm to 10.05 cm. The
Young’s modulus of polystyrene is 4.42 GPa.

Problems
P15D.1 For an isotropic substance, the moduli and Poisson’s ratio may be P15D.2 The bulk modulus for liquid water at 298 K is 3.4 GPa. If it is assumed
expressed in terms of two parameters λ and μ called the Lamé constants: that the molecules are interacting with a Lennard-Jones potential energy,
estimate the well depth ε (give your answer in kJ mol−1).
 (3  2  ) 3  2  
E K G   p 
 3 2(   )
Use the Lamé constants to confirm the relations between G, K, and E given in
eqn 15D.3.

TOPIC 15E The electrical properties of solids


Discussion question
D15E.1 Describe the characteristics of the Fermi–Dirac distribution; contrast
it with the Boltzmann distribution.

Additional exercises
E15E.3 Calculate f(E), the probability predicted by the Fermi–Dirac E15E.5 Suppose that the Fermi energy is 2.00 eV. At 298 K, calculate the energy
distribution, for the case where the energy E is the thermal energy, kT, below above the Fermi energy at which the probability has fallen to 0.10; express
the Fermi energy: E = EF − kT. Comment on the value you obtain in relation your answer in eV.
to that from Exercise E15E.1.
E15E.6 Is gallium-doped germanium a p-type or n-type semiconductor?
E15E.4 A typical value for the Fermi energy is 1.00 eV. At 298 K, calculate the
energy above the Fermi energy at which the probability has fallen to 0.25;
express your answer in eV.

Problems
P15E.1 Refer to eqn 15E.2b and express f(E) as a function of the variable P15E.4 In an intrinsic semiconductor, the band gap is so small that the
(E − EF)/EF and EF/kT. Then, using mathematical software, display the set of Fermi–Dirac distribution results in some electrons populating the conduction
curves shown in Fig. 15E.5 as a single surface. band. It follows from the exponential form of the Fermi–Dirac distribution
that the conductance G, the inverse of the resistance (with units of siemens,
P15E.2 In this and the following problem you are invited to explore further
1 S = 1 Ω−1), of an intrinsic semiconductor should have an Arrhenius-like
some of the properties of the Fermi–Dirac distribution, f(E), eqn 15E.2a.  E / 2 kT
temperature dependence, shown in practice to have the form G  G0 e g ,
(a) Show that at T = 0, f(E) = 1 for E < μ, and f(E) = 0 for E > μ. (b) For
where Eg is the band gap. The conductance of a sample of germanium varied
a three-dimensional solid of volume V, it turns out that in eqn 15E.1
with temperature as indicated below. Estimate the value of Eg.
ρ(E) = CE1/2, with C = 4πV(2me/h2)3/2. If the number of electrons in the solid of
volume V is N, then this number must be equal to the result of integrating eqn
  T/K 312 354 420
15E.1 over the full range of energy: N  0 dN (E )  0  ( E ) f ( E )dE. Evaluate
the integral at T = 0. (Hint: It can be split into two integrals, one G/S 0.0847 0.429 2.86
between E = 0 and μ, and one between E = μ and ∞.) (c) Equate the
expression obtained by evaluating the integral with N, and hence show P15E.5 A sample of an n-type semiconductor is found to be an insulator at
that   (3N /8)2 / 3 (h2 /2me ), where N = N /V , the number density of very low temperatures. As the temperature is raised, there comes a point
electrons in the solid. (d) Evaluate μ for sodium, which has mass density at which the conductivity increases markedly, but after this point the
0.97 g cm−3; assume that each atom contributes one electron. conductivity remains pretty much constant as the temperature is raised
further. At much higher temperatures, the conductivity starts to increase
P15E.3 By inspection of eqn 15E.2a and the expression for dN(E) in eqn 15E.1
steadily, with no sign of it reaching a plateau. Explain these observations.
(and without attempting to evaluate integrals explicitly), show that in order

for N to remain constant as the temperature is raised, the chemical potential P15E.6 P.G. Radaelli et al. (Science 265, 380 (1994)) reported the synthesis and
must decrease from its value at T = 0. structure of a material that becomes superconducting at temperatures below
704 15 Solids

45 K. The compound is based on a layered compound Hg2Ba2YCu2O8−δ, which replacing Y by Ca, accompanied by a change in unit cell volume by less than
has a tetragonal unit cell with a = 0.38606 nm and c = 2.8915 nm; each unit cell 1 per cent. Estimate the Ca content x in superconducting Hg2Ba2Y1−xCaxCu2O7.55
contains two formula units. The compound is made superconducting by partially given that the mass density of the compound is 7.651 g cm−3.

TOPIC 15F The magnetic properties of solids


Discussion question
D15F.1 Compare and contrast the polarization (Topic 14A) with the
magnetization.

Additional exercises
2+ −8 3 −1
E15F.4 The magnitude of the magnetic moment of Mn in its complexes is E15F.8 Data on a single crystal of NiSO4·7H2O give χm = 6.00 × 10 m mol at
typically 5.3μB. How many unpaired electrons does the ion possess? 298 K. Identify the effective number of unpaired electrons in this compound
and compare your result with the theoretical value.
E15F.5 Calculate the molar susceptibility of benzene given that its volume
susceptibility is −7.2 × 10−7 and its mass density is 0.879 g cm−3 at 25 °C. E15F.9 Estimate the spin-only molar susceptibility of MnSO4·4H2O at 298 K.

E15F.6 Calculate the molar susceptibility of cyclohexane given that its volume E15F.10 To what temperature must Nb be cooled for it to remain
susceptibility is −7.9 × 10−7 and its mass density is 811 kg m−3 at 25 °C. superconducting in a magnetic field of 150 kA m−1? The necessary data are
3 −1 given in E15F.3.
E15F.7 Data on a single crystal of MnF2 give χm = 0.1463 cm mol at 294.53 K.
Identify the effective number of unpaired electrons in this compound and
compare your result with the theoretical value.

Problems

P15F.1 J.J. Dannenberg et al. (J. Phys. Chem. 100, 9631 (1996)) carried out P15F.2 An NO molecule has thermally accessible electronically excited states.
theoretical studies of organic molecules consisting of chains of unsaturated It also has an unpaired electron, and so may be expected to be paramagnetic.
four-membered rings. The calculations suggest that such compounds have However, its ground state is not paramagnetic because the magnetic moment
large numbers of unpaired spins, and that they should therefore have unusual of the orbital motion of the unpaired electron almost exactly cancels the
magnetic properties. For example, the lowest-energy state of the compound spin magnetic moment. The first excited state (at 121 cm−1) is paramagnetic
shown as (5) is computed to have S = 3, but the energies of S = 2 and S = 4 because the orbital magnetic moment adds to, rather than cancels, the spin
structures are each predicted to be 50 kJ mol−1 higher in energy. Compute the magnetic moment. The upper state has a magnetic moment of magnitude
molar magnetic susceptibility of these three low-lying levels at 298 K. Estimate 2μB. Because the upper state is thermally accessible, the paramagnetic
the molar susceptibility at 298 K if each level is present in proportion to its susceptibility of NO shows a pronounced temperature dependence even near
Boltzmann factor (effectively assuming that the degeneracy is the same for all room temperature. Calculate the molar paramagnetic susceptibility of NO and
three of these levels). plot it as a function of temperature.

TOPIC 15G The optical properties of solids


Discussion questions
D15G.1 Explain the origin of Davydov splitting in the exciton bands of a D15G.2 Explain how the nonlinear response of a material to an electric field
crystal. may give rise to frequency doubling. Why is frequency doubling typically seen
only when using an intense beam from a laser as the light source?

Additional exercise
E15G.2 The band gap in silicon is 1.12 eV. Calculate the maximum wavelength
of electromagnetic radiation that results in promotion of electrons from the
valence to the conduction band.
Exercises and problems 705

Problems
P15G.1 This and the following problem explore quantitatively the spectra S   b (1) b (2)d ; do the same for the second eigenvector, which corresponds
of molecular solids. First consider a dimer formed from two identical to Ψ− = ψb(1) − ψb(2). (c) The monomer transition dipole moment is
monomers. For the first monomer, the normalized ground state wavefunction μmon = ∫ψ b (1)μˆψ a (1)dτ =∫ψ b (2)μˆψ a (2)dτ . For the dimer the transition
is ψa(1) and the normalized excited state wavefunction is ψb(1); for the second moment is μdim = ∫Ψ ± μˆΨ 0 dτ , where Ψ0 is the wavefunction of the dimer
monomer the wavefunctions are ψa(2) and ψb(2)—the label in parenthesis ground state. Because it is assumed that there is no interaction between the
identifies to which monomer the wavefunction refers, but otherwise ψa and ground-state wavefunctions of the dimer, Ψ0 can be written as (1/21/2)(ψa(1) +
ψb are the same for each monomer. In each monomer there is a transition ψa(2)). Find expressions for μdim , for the two excited state wavefunctions, Ψ±.
between ψa and ψb with transition dipole moment μmon and wavenumber In solving this problem it is helpful to realize that it is closely analogous to the
νmon . For convenience the energy of the ground state is taken as zero, so the overlap of two atomic orbitals to give molecular orbitals (Topic 9E).
energy of the excited state, expressed as a wavenumber, is νmon . It is assumed
P15G.2 Continues from the previous problem. (a) Consider a dimer formed of
that dimerization does not affect the ground state wavefunctions, but the
monomers which have μmon = 4.00 D, mon  25000 cm 1, and r = 0.50 nm. Plot
excited state wavefunctions become mixed so the excited state of the dimer
a graph to show how the energies (expressed as a wavenumber) of the excited
has wavefunctions Ψ± = c1,±Ψb(1) + c2,±Ψb(2); the mixing of the two monomer
states,  vary with the angle θ. (b) Now expand the treatment given above to a
wavefunctions gives two dimer wavefunctions, denoted   , the coefficients
chain of N monomers with μmon = 4.00 D, mon  25000 cm 1, and r = 0.50 nm.
ci,± are to be determined. In the basis ψb(1), ψb(2) the hamiltonian matrix has
For simplicity, assume that θ = 0 and that only nearest neighbours interact
the form
with interaction energy V (expressed here as a wavenumber). For example the
 νmon β  hamiltonian matrix for the case N = 4 is
Ĥ =  
 β ν 
 mon   νmon V 0 0 
 
 V νmon V 0 
The diagonal elements are the energies (as a wavenumber) of the excited Hˆ = 
state of the monomer. The off-diagonal elements correspond to the energy of  0 V νmon V 
interaction between the transition dipoles. Using the arrangement illustrated  0 0 V νmon 

in (1) of Topic 15G, this interaction energy (as a wavenumber) is:
This matrix is analogous to the one that characterizes a band in a solid
mon
2
  (1  3 cos 2  ) (15C.1b) and so the eigenvalues (which in this case are the wavenumber of
4  0hcr 3 the transitions) can be written down by analogy with eqn 15C.1. Calculate the
The eigenvectors of the hamiltonian matrix are the wavefunctions for the wavenumber of the lowest energy transition for N = 5, 10, and 15, and then
c
excited state of the dimer, and these can be written (c12,,±± ). The eigenvalues are generalize your result for large N. (c) How does the transition dipole moment
the energies corresponding to these wavefunctions, and because it has been of the lowest energy transition vary with the size of the chain?
assumed that the ground states of the dimer are the same as for the monomer, P15G.3 Show that if a substance responds nonlinearly to two sources of
these energies will correspond to the transitions in the dimer. radiation, one of frequency ω1 and the other of frequency ω2, then it may
1 1
(a) Show that (1 ) and (−1 ) are eigenvectors of the hamiltonian matrix give rise to radiation of the sum and difference of the two frequencies. This
and that the corresponding eigenvalues are   mon   . (b) The first nonlinear optical phenomenon is known as frequency mixing and is used
eigenvector corresponds to writing the wavefunction as Ψ+ = c1,+ψb(1) + to expand the wavelength range of lasers in laboratory applications, such as
c2,+ψb(2) = ψb(1) + ψb(2). Normalize the wavefunction, assuming that spectroscopy and photochemistry.

FOCUS 15 Solids
Integrated activities
I15.1 Calculate the thermal expansion coefficient, α = (∂V/∂T)p/V, of diamond f as a function of (sin θ)/λ. Hint: Interpret 4πρ(r)r2 as the radial distribution
given that the (111) reflection shifts from 22.0403° to 21.9664° on heating a function P(r); use mathematical software to evaluate the necessary integrals.
crystal from 100 K to 300 K and 154.0562 pm X-rays are used. (c) Explore how the scattering factor changes when the actual 1s wavefunction
of a hydrogenic atom is replaced by a Gaussian function. Use mathematical
I15.2 Calculate the scattering factor for a hydrogenic atom of atomic number Z
software to evaluate the necessary integrals.
in which the single electron occupies (a) the 1s orbital, (b) the 2s orbital. Plot
FOCUS 16
Molecules in motion

This Focus is concerned with understanding how matter and resistance of electrolyte solutions and to analyse it in terms of
other physical properties (such as energy and momentum) the response of the ions to an applied electric field.
are transported from one place to another in both gases and 16B.1 Experimental results; 16B.2 The mobilities of ions
­liquids.

16C Diffusion
16A Transport properties of a
perfect gas The diffusion of solutes and various physical properties is an
important process in liquids. It can be discussed by introduc-
The transport of matter and physical properties can be de- ing the concept of a general ‘thermodynamic force’ that can be
scribed by a set of closely related empirical equations. For a regarded as being responsible for the motion of molecules. This
gas, it is possible to understand the form of these equations by apparent force can be used to construct the important ‘diffusion
building a model based on the kinetic theory of gases discussed equation’, which describes how solutes spread out in space with
in Topic 1B. With this approach, the rate of diffusion, the rate of increasing time. An alternative model of diffusion as a random
thermal conduction, viscosity, and effusion can all be related to walk gives further insight.
quantities arising from the kinetic theory. 16C.1 The thermodynamic view; 16C.2 The diffusion equation;
16C.3 The statistical view
16A.1 The phenomenological equations; 16A.2 The transport
parameters

What is an application of this material?


16B Motion in liquids
A great deal of chemistry, chemical engineering, and biology
Molecular motion in liquids is different from that in gases on depends on the ability of molecules and ions to migrate through
account of the presence of significant intermolecular interac- media of various kinds. Impact 25, accessed via the e-book, ex-
tions and the much higher density typical of a liquid. One way plains how conductivity measurements are used to analyse the
to monitor motion in such systems is to explore the electrical motion of ions through biological membranes.

➤ Go to the e-book for videos that feature the derivation and interpretation of equations, and applications of this material.
TOPIC 16A Transport properties
of a perfect gas

derivation and physical interpretation of this expression, see


➤ Why do you need to know this material? Topic 1B. Another important result from kinetic theory is the
mean speed of molecules of molar mass M at a temperature T,
Many physical processes take place by the transfer of
which is given by eqn 1B.9 and is repeated here:
a property from one region to another, and gas-phase
chemical reactions depend on the rate at which molecules 1/ 2
 8RT  Mean speed
collide. The material presented here also includes general vmean    [kinetic theory] (16A.1b)
aspects of transport in fluid systems of any kind, and which  M 
are applicable to reactions taking place in solution.

➤ What is the key idea? The phenomenological


16A.1
A molecule carries properties through space in steps of
about the distance of its mean free path.
equations
➤ What do you need to know already? A ‘phenomenological equation’ is an equation that summa-
rizes empirical observations on phenomena without, initially at
This Topic builds on and extends the kinetic theory of gases
least, being based on an understanding of the molecular pro-
(Topic 1B). You need to be familiar with the concepts of the
cesses responsible for the property. Such equations are encoun-
mean speed of molecules and the mean free path and their
tered commonly in the study of fluids.
dependence on pressure and temperature.
The rate of migration of a property is measured by its
flux, J, the quantity of that property passing through a two-­
dimensional region perpendicular to the flow in a given time
interval divided by the area of the region and the duration of
A transport property is a process by which matter or an the interval. If matter is flowing (as in diffusion), the matter
­attribute of matter, such as momentum, is carried through a flux is reported as so many molecules per square metre per
medium from one location to another. The rate of transport is second (number or amount m−2 s−1). If the property migrating
commonly expressed in terms of an equation that is an empiri- is energy (as in thermal conduction), then the energy flux is
cal summary of experimental observations. These equations expressed in joules per square metre per second (J m−2 s−1), and
apply to all kinds of properties and media, and can be adapted so on. The total quantity of a property transferred through a
to the discussion of transport properties of gases. In such cases, given region of area A in a given time interval Δt is |J|AΔt. The
the kinetic theory of gases provides simple expressions that flux J may be positive or negative: the significance of its sign is
show how the rates of transport of these properties depend on discussed below.
the pressure and the temperature. The most important con- Experimental observations on transport properties show
cept from the kinetic theory developed in Topic 1B, and used that the flux of a property is usually proportional to the first
throughout this Topic, is the mean free path, λ, the average dis- derivative of a related quantity. For example, the flux of matter
tance a molecule travels between collisions. According to eqn diffusing parallel to the x-axis of a container is found to be pro-
1B.14, at a temperature T and a pressure p, portional to the gradient of the concentration along the same
direction:
kT Mean free path
  (16A.1a) dN
p [kinetic theory] J (matter ) ∝ Fick’s first law of diffusion (16A.2)
dx
The parameter σ is the collision cross-section of a molecule, where N is the number density of particles, with units num-
a measure of the target area it presents in a collision. For the ber per metre cubed (m−3). The proportionality of the flux
16A Transport properties of a perfect gas 709

of matter to the concentration gradient is sometimes called Table 16A.1 Transport properties of gases at 1 atm⋆
Fick’s first law of diffusion: the law implies that diffusion is
κ/(mW K−1 m−1) η/μP‡
faster when the concentration varies steeply with position than
when the concentration is nearly uniform. There is no net flux 273 K 273 K 293 K

if the concentration is uniform (dN /dx = 0). Similarly, the rate Ar 16.3 210 223
of thermal conduction (the flux of the energy associated with CO2 14.5 136 147
thermal motion) is found to be proportional to the tempera- He 144.2 187 196
ture gradient: N2 24.0 166 176

dT Flux of energy

More values are given in the Resource section.
J (energy of thermal motion) ∝ as heat
 (16A.3)
dx

1 μP = 10−7 kg m−1 s−1.

A positive value of J signifies a flux towards positive x; a


negative value of J signifies a flux towards negative x. Because Brief illustration 16A.1
matter flows down a concentration gradient, from high con-
centration to low concentration, J is positive if dN /dx is nega- Suppose that two metal plates are placed perpendicular to the
tive (Fig. 16A.1). Therefore, the coefficient of proportionality in x-axis, with the first at x = 0 and the second at x = +1.0 cm, and
that the temperature of the second plate is 10 K higher than
eqn 16A.2 must be negative, and it is written as −D:
that of the first plate. The temperature gradient is
dN Fick’s first law in terms of
J (matter )  D the diffusion coefficient
 (16A.4) dT 10 K
dx   1.0  103 K m 1
dx 1.0  102 m
The constant D is the called the diffusion coefficient; its SI
If the plates are separated by air, for which κ = 24.1 mW K−1 m−1,
units are metre squared per second (m2 s−1). Energy migrates
the energy flux is
down a temperature gradient, and the same reasoning leads to
dT
dT J    (24.1 mW K 1 m 1 )  (1.0  103 Km 1 )
J (energy of thermal motion)   dx
dx  24 Wm 2
Flux of energy as heat in
n terms of the
coefficient of thermal conductivity  (16A.5)
The flux is negative because energy is transferred from the
hotter plate at x = +1.0 cm to the cooler plate at x = 0, which is
where κ (kappa) is the coefficient of thermal ­conductivity. a flow in the −x direction. The energy transferred through an
The units of κ are joules per kelvin per metre per second area of 1.0 cm2 between the two plates in 1 h (3600 s) is
(J K−1 m−1 s−1) or, because 1 J s−1 = 1 W, watts per kelvin per
metre (W K−1 m−1). Some experimental values are given in Transfer  | J | At
Table 16A.1.  (24 Wm 2 )  (1.0  104 m2 )  (3600 s)  8.6 J

Viscosity arises from the flux of linear momentum. To see


the connection, consider a fluid in a state of Newtonian flow, in
which a series of layers move past one another and, in this case,
N in the z-direction (Fig. 16A.2). The layer next to the wall of the
dN vessel is stationary, and the velocity of successive layers varies
<0
dx linearly with distance, x, from the wall. Molecules ceaselessly
x move between the layers and bring with them the z-component
of linear momentum they possessed in their original layer.
A layer is retarded by molecules arriving from a more slowly
moving layer because such molecules have a lower momentum
J>0
in the z-direction. A layer is accelerated by molecules arriving
Figure 16A.1 The flux of particles down a concentration gradient. from a more rapidly moving layer. The net retarding effect is
In this case, the concentration gradient is negative (concentration interpreted as the viscosity of the fluid.
decreases as the distance x increases) and the resulting flux of The retarding effect depends on the transfer of the z-­
matter is positive (toward longer distances x). component of linear momentum into the layer of interest, so
710 16 Molecules in motion

region may be an imaginary window, a part of a wall, or a hole


in a wall). Specifically, the collision flux is the number of colli-
Brings Brings sions in a time interval divided by the area of the region and the
Wall

z low high duration of the time interval. Its dependence on pressure and
momentum momentum
temperature can be derived from the kinetic theory.

How is that done? 16A.1 Deriving an expression for the


collision flux
x
Consider a wall of area A perpendicular to the x-axis (Fig. 16A.3).
Figure 16A.2 The viscosity of a fluid arises from the transport In the following calculation, note that for a perfect gas the equa-
of linear momentum. In this illustration the fluid is undergoing tion of state pV = nRT can be used to relate the number density,
Newtonian (laminar) flow in the z-direction, and particles bring =
N , to the pressure by N N= /V= nN A /V nN = A p/nRT p/kT .
their initial momentum when they enter a new layer. In the final equality R = NAk is used, where k is Boltzmann’s
constant.
the viscosity depends on the flux of this z-component in the Step 1 Identify the number of molecules that will strike an area
x-direction. The flux of the z-component of momentum is pro- If a molecule has vx > 0 (that is, it is travelling in the direction
portional to dvz/dx, where vz is the velocity in the z-direction; of positive x), then it will strike the wall within an interval Δt if
the flux can therefore be written it lies within a distance vxΔt of the wall. Therefore, all molecules
in the volume AvxΔt, and with a positive x-component of
dv z velocity, will strike the wall in the interval Δt. The total number
J (z -component of momentum)  
dx of collisions in this interval is therefore NAvx ∆t , where N is the
Momentum flux in terms off
 (16A.6) number density of molecules.
the coefficient of viscosity
Step 2 Take into account the range of velocities
The constant of proportionality, η, is the coefficient of viscosity The velocity vx has a range of values described by the probabil-
(or simply ‘the viscosity’). Its units are kilograms per metre per ity distribution f(vx) given in eqn 1B.3:
second (kg m−1 s−1). Viscosities are often reported in the non-SI
1/ 2
unit poise (P), with 1 P = 10−1 kg m−1 s−1. Some experimental val-  m   mvx2 / 2 kT
f (v x )    e
ues are given in Table 16A.1.  2kT 
Although it is not strictly a transport property, closely related
where f(vx)dvx is the probability of finding a molecule with a
to diffusion is effusion, the escape of matter through a small
component of velocity between vx and vx + dvx. The total num-
hole. The essential empirical observations on effusion are sum-
ber of collisions is found by summing NAvx ∆t over all positive
marized by Graham’s law of effusion, which states that the rate
values of vx (because only molecules with a positive component
of effusion is inversely proportional to the square root of the
of velocity are moving towards the area of interest) with each
molar mass.
value of vx being weighted by the probability of it occurring:


Exercise Number of collisions  NAt  vx f (vx )dvx


0

E16A.1 In a double-glazed window, the panes of glass are separated by 1.0 cm


and the space is filled with a gas with thermal conductivity 24 mW K−1 m−1.
|vxΔt|
What is the rate of transfer of heat by conduction from the warm room (28 °C)
to the cold exterior (−15 °C) through a window of area 1.0 m2? You may assume
that one pane of glass is at the same temperature as the inside and the other as Area, A
the outside. What power of heater is required to make good the loss of heat?

Will

16A.2 The transport parameters Won’t


Volume = |vxΔt|A
x
The kinetic theory of gases (Topic 1B) can be used to derive ex-
pressions for the transport parameters of a perfect gas. All the Figure 16A.3 A molecule will reach the wall on the right within an
expressions depend on knowing the collision flux, ZW, which interval Δt if it is within a distance vxΔt of the wall and travelling to
is the rate at which molecules strike a region in the gas (this the right.
16A Transport properties of a perfect gas 711

The collision flux is the number of collisions divided by A and Brief illustration 16A.2
Δt, so

The collision flux of O2 molecules, with m = M/NA and
Z W  N  vx f (vx )dvx M = 32.00 g mol−1, at 25 °C and 1.00 bar is
0

kg m1 s 2
Step 3 Evaluate the integral 
From 1.00  105 Pa
ZW  1/ 2
Integral G.2  32.00  103 kgmol 1 
1/ 2  1/ 2 2  23 1
 (1.381  1023 JK 1 )  (298 K )
  m    kT   6.022  10 mol 
 
2
vx f (vx )dv x    v x e  mvx / 2 kT dvx   
0
 2kT  0
 2m  27
 2.70  10 m s 2 1

it follows that This flux corresponds to 0.45 mol cm−2 s−1.

N = p/kT
1/2 1/2
kT p kT
ZW = N
2πm
=
kT 2πm 16A.2(a) The diffusion coefficient
The first application of the result in eqn 16A.7b is to use it to
and therefore find an expression for the net flux of molecules arising from a
p concentration gradient.
(16A.7a)
ZW =
(2πmkT )1/2 Collision flux in terms of pressure
[perfect gas]

How is that done? 16A.2 Deriving an equation for the net


Step 4 Develop an alternative expression in terms of the mean
speed flux of matter
The mean speed is given by eqn 16A.1b as Consider the arrangement depicted in Fig. 16A.4. The mol-
ecules passing through the area A at x = 0 have travelled an
R = NAk , M = NAm average of about one mean free path, λ, since their last collision.

8 RT
1/2
8kT
1/2 Step 1 Set up expressions for the flux in each direction
vmean = =
πM πm If the number density at x is N (x ), then the number density at
x = λ can be estimated by using a Maclaurin expansion of the
It follows that form f (x )  f (0)  (df /dx )0 x  , truncated after the second
term (see Part 1 of the Resource section):
1/ 2
 kT 
 2m   14 vmean
   dN 
N (  )  N (0)    
 dx 0
and therefore Z W  N (kT /2m)1/ 2 can be expressed as

(16A.7b)
Z W = 14 Nvmean
Number density, N

Collision flux N (–λ)


[perfect gas] N (0)
N (+λ)
x Area, A
According to eqn 16A.7a, the collision flux increases with
pressure simply because increasing the pressure increases the
number density and hence the number of collisions. The flux
decreases with increasing mass of the molecules because heavy
molecules move more slowly than light molecules. Caution,
however, is needed with the interpretation of the role of tem- +λ
perature: it is wrong to conclude that because T1/2 appears in the 0
–λ
denominator that the collision flux decreases with increasing
temperature. If the system has constant volume, the pressure Figure 16A.4 The calculation of the rate of diffusion of a gas
increases with temperature (p ∝ T ), so the collision flux is in considers the net flux of molecules through a plane of area A as
fact proportional to T/T1/2 = T1/2. The flux increases with tem- a result of molecules arriving from an average distance λ away in
perature because the molecules are moving faster. each direction.
712 16 Molecules in motion

Similarly, the number density at x = −λ is

 dN 
N ( )  N (0)    
 dx 0
Short flight
The average number of impacts on the imaginary region of area (survives)
A during an interval Δt is ZWAΔt, where ZW is the collision flux. Long flight
Therefore, the matter flux from left to right, J(L→R), arising (collides in flight)
from the supply of molecules on the left, is this number of col-
lisions divided by the time interval and the area:
Figure 16A.5 One issue ignored in the simple treatment is that
some molecules might make a long flight to the plane even
ZW = 14 N vmean
though they are only a short perpendicular distance away from
Z W AΔt it. A molecule taking the longer flight has a higher chance of
J (L → R) = = Z W = 14 N (− )vmean
AΔt colliding during its journey.

The number density is that at x = −λ, where the molecules


originate before striking the area. There is also a flux of mol- Brief illustration 16A.3
ecules from right to left. The molecules making this journey
have originated from x = +λ where the number density is In Brief illustration 1B.3 of Topic 1B it is established that the
N ( ). Therefore, mean free path of N2 molecules in a gas at 1.0 atm and 25 °C is
91 nm; in Example 1B.1 of the same Topic it is calculated that
J(L  R )  14 N (  )vmean under the same conditions the mean speed of N2 molecules is
475 m s−1. Therefore, the diffusion coefficient for N2 molecules
Step 2 Evaluate the net flux under these conditions is
The net flux from left to right is
D  13  (91  109 m)  (475 ms 1 )  1.4  105 m2 s 1
J x  J (L  R )  J (L  R ) The experimental value is 1.5 × 10−5 m2 s−1.
 14 vmean {N ( )  N (  )}
  dN     dN  
 14 vmean N (0)       N (0)    dx    There are three points to note about eqn 16A.9:
  d x 0    0  
That is, • The mean free path, λ, decreases as the pressure is
increased (eqn 16A.1a), so D decreases with increasing
dN (16A.8) pressure and, as a result, the gas molecules diffuse more
J x = − 12 vmean λ
dx 0
slowly.
• The mean speed, vmean, increases with the temperature
(eqn 16A.1b), so D increases with temperature. As a result,
This equation shows that the flux is proportional to the gra-
molecules in a hot gas diffuse more quickly than those
dient of the concentration, in agreement with the empirical ob-
when the gas is cool (for a given concentration gradient).
servation expressed by Fick’s law, eqn 16A.2.
At this stage it looks as though a value of the diffusion coef- • Because the mean free path increases when the collision
ficient can be picked out by comparing eqns 16A.8 and 16A.4, cross-section σ of the molecules decreases, the diffusion
so obtaining D  12  vmean. It must be remembered, however, coefficient is greater for small molecules than for large
that the calculation is quite crude, and is little more than an as- molecules.
sessment of the order of magnitude of D. One aspect that has
not been taken into account is illustrated in Fig. 16A.5, which 16A.2(b) Thermal conductivity
shows that although a molecule may have begun its journey
very close to the window, it could have a long flight before it According to the equipartition theorem (Energy: A first look),
gets there. If the path is long, the molecule is likely to collide each molecule carries an average energy    kT, where ν de-
before reaching the window, so it ought not to be counted as pends on the number of quadratic contributions to the energy
passing through the window. Taking this effect into account re- of the molecule. For atoms, which have three translational de-
sults in the appearance of a factor of 23 representing the lower grees of freedom,   23 . When one molecule passes through the
flux. The modification results in imaginary window, it transports that average energy. An argu-
ment similar to that used for diffusion can be used to discuss
D  13  vmean Diffusion coefficient  (16A.9) the transport of energy through this window.
16A Transport properties of a perfect gas 713

How is that done? 16A.3 Deriving an expression for the   13 vmean [J]CV ,m Thermal conductivity  (16A.10b)

thermal conductivity Yet another form is found by starting with eqn 16A.10a, rec-
Assume that the number density is uniform but that the tem- ognizing that N = p/kT and then using the expression for D in
perature, and hence the average energy of the molecules, is not. eqn 16A.9:
Molecules arrive from the left after travelling a mean free path
N  p/kT D  31  vmean
from their last collision in a hotter region, and therefore arrive
with a higher energy. Molecules also arrive from the right after p  pD
travelling a mean free path from a cooler region, and hence   13  kvmean  N  13  kvmean 
kT T
arrive with lower energy.
Thermal conductivity (16A.10c)
Step 1 Write expressions for the forward, reverse, and net energy
flux
If the average energy of the molecules at x is ε(x), the two Brief illustration 16A.4
opposing energy fluxes are
In Brief illustration 16A.3 the value D = 1.4 × 10−5 m2 s−1 was
ZW
  ZW
  calculated for N2 molecules at 25 °C and 1.0 atm. The thermal
J (L  R )  4 N vmean  ( )
1
J (L  R )  4 N vmean  ( )
1 conductivity can be calculated by using eqn 16A.10c and not-
ing that for N2 molecules   25 (three translational modes and
and the net flux is two rotational modes; the vibrational mode is inactive at this
temperature).
J m 3
J x  J (L  R) − J (L  R) 
5
 (1.01  10 Pa )  (1.4  105 m 2 s 1 )
5

 vmean N { (−l ) − (l )}


1
4
 2

298 K
 (x)  (0)  x(d /d x)0, x =   1.2  10 JK m 1 s 1
2 1

d d or 12 mW K−1 m−1. The experimental value is 26 mW K−1 m−1.


 14 vmean N (0) − l − (0)  l
dx 0
dx 0

d
 − 12 vmeanl N
dx 0 To interpret eqn 16A.10, note that:
• The mean free path is inversely proportional to the pressure
Step 2 Express the energy gradient as a temperature gradient
(  kT / p), but the number density N is proportional to
A gradient of energy dε/dx, with    kT , can be expressed as a the pressure (N = p/kT ). It follows that the product λN ,
gradient of temperature: d /dx   k(dT /dx ); therefore which appears in eqn 16A.10a, is independent of pressure,
and so therefore is the thermal conductivity.
 d   dT 
J x   12 vmean N     12  kvmean N   • The thermal conductivity is greater for gases with a high
 dx  0  dx  0
heat capacity (eqn 16A.10b) because a given temperature
This equation shows that the energy flux is proportional to the gradient then corresponds to a steeper energy gradient.
temperature gradient, which is the desired result. As before, The physical reason for the pressure independence of the ther-
the constant is multiplied by 23 to take long flight paths into
mal conductivity is the result of the cancellation of two effects.
account, and comparison of this equation with eqn 16A.5
One is that the conductivity can be expected to be large when
shows that
many molecules are available to transport the energy. The other
is that the presence of so many molecules limits their mean free
  13  kvmean λN
(16A.10a) path and they cannot carry the energy over a great distance.
Thermal conductivity These two effects cancel.
The thermal conductivity is indeed found experimentally
The number density can be written in terms of the to be independent of the pressure, except when the pressure is
molar concentration [J] of the particles J that carry energy: very low, and then κ ∝ p. At very low pressures it is possible for
=N nN = A /V [J]N A. Next, the quantity ν kN A can be identi- λ to exceed the dimensions of the apparatus, and the distance
fied as the molar constant-volume heat capacity of the gas: the over which the energy is transported is then determined by the
molar energy is N Aν kT , so it follows from CV,m = (∂Um/∂T)V that size of the container and not by collisions with the other mol-
CV ,m  N A k . With these substitutions, eqn 16A.10a becomes ecules present. The flux is still proportional to the number of
714 16 Molecules in motion

Step 2 Identify the coefficient of viscosity


z Fast
layer The flux is proportional to the velocity gradient, in line with
the phenomenological equation. Comparison of this expres-
Slow sion with eqn 16A.6, and multiplication by 23 in the normal
layer way, leads to

(16A.11a)
η = 13 vmean λmN
Viscosity
x
λ Two alternative forms of this expression (after using mNA = M
0 and eqn 16A.9, D  13  vmean) are
−λ
  MD[J] (16A.11b)
Figure 16A.6 The calculation of the viscosity of a gas examines
the net x-component of momentum brought to a plane from pMD
  (16A.11c)
faster and slower layers a mean free path away in each direction. RT
where [J] is the molar concentration of the gas molecules J and
carriers, but the length of the journey no longer depends on λ, M is their molar mass.
so κ ∝ [J], which implies that κ ∝ p.

16A.2(c) Viscosity Brief illustration 16A.5

If the momentum in the z-direction depends on x as mvz(x), From Brief illustration 16A.3 the value of D for N2 molecules
then molecules moving from the right in Fig. 16A.6 (from a (with M = 28.02 g mol−1) at 25 °C and 1.0 atm is 1.4 × 10−5 m2 s−1.
fast layer to a slower one) transport a momentum mvz(+λ) to Equation 16A.11c gives
their new layer at x = 0; those travelling from the left transport Jm3
mvz(−λ) to it. This picture can be used to build an expression 
(1.01  10 Pa )  (28.02  103 kg mol 1 )  (1.4  105 m 2 s 1 )
5
for the coefficient of viscosity. 
(8.3145 JK 1 mol 1 )  (298 K )
 1.6  105 kg m 1 s 1
How is that done? 16A.4 Deriving an expression for the
viscosity or 160 μP. The experimental value is 177 μP.

The strategy is the same as in the previous derivations, but now


the property being transported is the linear momentum of the
layers. The physical interpretation of eqn 16A.11a is as follows:
Step 1 Set up expressions for the flux of momentum in each • As has already been noted for thermal conductivity,
direction and the net flux the product λN is independent of p. Therefore, like the
Molecules arriving from the right bring a momentum thermal conductivity, the viscosity is independent of the
pressure.
 dv  • Because vmean ∝ T1/2, η ∝ T1/2 at constant volume (and
mvz ( )  mvz (0)  m  z 
 dx  0 η ∝ T3/2 at constant pressure). That is, the viscosity of a gas
increases with temperature.
Those arriving from the left bring a momentum
The physical reason for the pressure independence of the vis-
 dv  cosity is the same as for the thermal conductivity: more mol-
mvz ( )  mvz (0)  m  z 
 dx  0 ecules are available to transport the momentum, but they carry
it less far on account of the decrease in mean free path. The
The net flux of z-momentum in the x-direction is therefore
increase of viscosity with temperature is explained when it is

recalled that at high temperatures the molecules travel more
 dv     dv  
J x  14 vmeanN  mvz (0)  m  z    mvz (0)  m  z   quickly, so the flux of momentum is greater. In contrast, as
  dx  0    dx  0  discussed in Topic 16B, the viscosity of a liquid decreases with
 dv  increase in temperature because intermolecular interactions
  12 vmean mN  z 
 dx  0 must be overcome.
16A Transport properties of a perfect gas 715

16A.2(d) Effusion rate in terms of mass, multiply the number of atoms that escape
by the mass of each atom.
The mean speed of molecules is inversely proportional to M1/2.
Therefore, the rate at which they strike the area of the hole The solution The mass loss Δm in an interval Δt is equal to the
through which they are effusing is also inversely proportional number of molecules that strike the area of the hole in this
interval multiplied by the mass of each molecule:
to M1/2, in accord with Graham’s law. However, by using the ex-
pression for the collision flux, a more detailed expression for
the rate of effusion can be obtained and used to interpret effu- eqn
 16 A.12

pA0 N A
sion data in a more sophisticated way. m  rate of effusion  t  m   t  m
When a gas at a pressure p and temperature T is separated (2MRT )1/ 2
from a vacuum by a small hole, the rate of escape of its mol-
ecules is equal to the rate at which they strike the hole of area where A0 is the area of the hole and M is the molar mass.
Rearrangement of this equation and using m = M/NA gives an
A0. That rate is the product of the collision flux and the area of
expression for p:
the hole:
1/ 2
 2RT  m
p 
m = M/NA, k = R/NA  M  A0 t

pA0 pA0 N A
Rat of ffusion = Z W A0 = = Substitution of the data and M = 132.9 g mol−1 gives
(2πmkT )1/2 (2πMRT )1/2
Rate of effusion (16A.12) 1/ 2
 2  (8.3145 JK 1 mol 1 )  (773 K) 
p 
 0.1329 kg mol 1
 
This rate is inversely proportional to M1/2, in accord with 385  106 kg
Graham’s law. Do not conclude, however, that because the ex-   1.1  104 Pa, or 11 kPa
  (0.25  103 m)2  (100 s)
pression includes a factor of T −1/2 the rate of effusion decreases  
A0
as the temperature increases. As p ∝ T, the rate is in fact pro-
portional to T1/2 and increases with temperature.
Self-test 16A.1 How long would it take for 200 mg of Cs atoms
Equation 16A.12 is the basis of the Knudsen method for
to effuse out of the oven under the same conditions?
the determination of the vapour pressures of liquids and sol-
ids, particularly of substances with very low vapour pressures Answer: 52 s
which cannot be measured directly. Thus, if the vapour pres-
sure of a sample is p, and it is enclosed in a cavity with a small
hole, then the rate of loss of mass from the container is propor-
tional to p.
Exercises
E16A.2 A solid surface with dimensions 2.5 mm × 3.0 mm is exposed to argon
gas at 90 Pa and 500 K. How many collisions do the Ar atoms make with this
surface in 15 s?
Example 16A.1  Calculating the vapour pressure from a
E16A.3 Calculate the diffusion constant of argon at 20 °C and (i) 1.00 Pa,
mass loss (ii) 100 kPa, (iii) 10.0 MPa; take σ = 0.36 nm2. If a pressure gradient of
Caesium (m.p. 29 °C, b.p. 686 °C) was introduced into a con- 1.0 bar m−1 is established in a pipe, what is the flow of gas due to diffusion at
each pressure?
tainer and heated to 500 °C; the container is pierced by a hole
−1 −1
of diameter 0.50 mm. It is found that in 100 s the mass of the E16A.4 Calculate the thermal conductivity of argon (CV,m = 12.5 J K mol ,
2
σ = 0.36 nm ) at 298 K.
container decreased by 385 mg. Calculate the vapour pressure
of liquid caesium at 500 °C. E16A.5 Calculate the viscosity of air at (i) 273 K, (ii) 298 K, (iii) 1000 K. Take
σ as 0.40 nm2 and M as 29.0 g mol−1.
Collect your thoughts The pressure of vapour is constant inside
E16A.6 An effusion cell has a circular hole of diameter 2.50 mm. If the molar
the container despite the effusion of atoms because the hot mass of the solid in the cell is 260 g mol−1 and its vapour pressure is 0.835 Pa
liquid metal replenishes the vapour. The rate of effusion is at 400 K, by how much will the mass of the solid decrease in a period of
therefore constant, and given by eqn 16A.12. To express the 2.00 h?
716 16 Molecules in motion

Checklist of concepts
☐ 1. Flux is the quantity of a property passing through a ☐ 5. Viscosity is the migration of linear momentum down a
given area in a given time interval divided by the area velocity gradient; the flux of momentum is proportional
and by the duration of the interval. to the velocity gradient.
☐ 2. Diffusion is the migration of matter down a concentra- ☐ 6. Effusion is the emergence of a gas from a container
tion gradient. through a small hole.
☐ 3. Fick’s first law of diffusion states that the flux of matter ☐ 7. Graham’s law of effusion states that the rate of effusion
is proportional to the concentration gradient. is inversely proportional to the square root of the molar
☐ 4. Thermal conduction is the migration of energy down a mass.
temperature gradient; the flux of energy is proportional
to the temperature gradient.

Checklist of equations
Property Equation Comment Equation number
Fick’s first law of diffusion J  DdN /dx 16A.4
Flux of energy of thermal motion J   dT /dx 16A.5
Flux of momentum along z J   dvz /dx 16A.6

Diffusion coefficient of a perfect gas D  13  vmean KMT ⋆


16A.9

Coefficient of thermal conductivity of a perfect gas   vmean [J]CV, m


1
3 KMT and equipartition 16A.10b
Coefficient of viscosity of a perfect gas   vmean mN
1
3 KMT 16A.11a
Rate of effusion Rate ∝ 1/M1/2 Graham’s law 16A.12


KMT indicates that the equation is based on the kinetic theory of gases.
TOPIC 16B Motion in liquids

Table 16B.1 Viscosities of liquids at 298 K⋆


➤ Why do you need to know this material?
η/(10−3 kg m−1 s−1)
Many chemical reactions take place in liquids, so for a full Benzene 0.601
understanding of them it is important to know how solute
Mercury 1.55
molecules and ions move through such environments.
Pentane 0.224
➤ What is the key idea? Water ‡
0.891

Ions reach a terminal velocity when the electrical force on More values are given in the Resource section.

Note that 1 cP = 10−3 kg m−1 s−1; the viscosity of water corresponds to 0.891 cP.
them is balanced by the drag due to the viscosity of the
solvent.

➤ What do you need to know already? dv z


J (z -component of momentum)    (16B.1)
The discussion of viscosity starts with the definition of the dx
coefficient of viscosity introduced in Topic 16A. Some cal-
culations make use of the information about electrostatics The units of the coefficient of viscosity are kilograms per metre
set out in The chemist’s toolkit 16B.1. per second (kg m−1 s−1), but they may also be reported in the
equivalent units of pascal seconds (Pa s). The non-SI units
poise (P) and centipoise (cP) are still widely encountered:
1 P = 10−1 Pa s and so 1 cP = 1 mPa s. Table 16B.1 lists some val-
The motion of ions and molecules is an important aspect of ues of η for liquids.
the properties of liquids and of the reactions taking place in Unlike in a gas, for a molecule to move in a liquid it must
them. It can be studied experimentally by a variety of methods. acquire at least a minimum energy (an ‘activation energy’, Ea, in
For example, bulk measurements of viscosity and its tempera- the language of Topic 17D) to escape from its neighbours. From
ture dependence can be used to build models of the motion. the Boltzmann distribution it follows that the probability that a
Another important technique is inelastic neutron scattering, molecule has at least an energy Ea is proportional to e − Ea /RT , so
in which the energy neutrons collect or discard as they pass the mobility of the molecules in the liquid should follow this
through a sample is interpreted in terms of the motion of its type of temperature dependence. As the temperature increases,
molecules. the molecules become more mobile and so the viscosity is re-
duced. The expected temperature dependence of the viscosity,
which decreases as the mobility of the molecules increases, is
therefore of the form
16B.1 Experimental results
Temperature dependence
  0 e Ea /RT  (16B.2)
There are two ‘classical’ methods of investigating molecular of viscosity (liquid)
motion in liquids. One is the measurement of viscosity and its
temperature dependence. Another involves inferring details (Note the positive sign of the exponent.) The activation energy
about molecular motion by dragging ions through a solvent typical of viscosity is comparable to the mean potential energy
under the influence of an electric field. of intermolecular interactions. Equation 16B.2 implies that the
viscosity should decrease sharply with increasing temperature.
Such a variation is found experimentally, at least over reason-
16B.1(a) Liquid viscosity ably small temperature ranges (Fig. 16B.1). Intermolecular in-
The coefficient of viscosity, η (eta), is introduced in Topic teractions govern the magnitude of Ea, but the calculation of its
16A as the constant of proportionality in the relation be- value from an assumed form of the potential energy due to the
tween the flux of linear momentum and the velocity gradient interactions between the molecules is an immensely difficult
in a fluid: and largely unsolved problem.
718 16 Molecules in motion

2 electrical property of the solution is its resistance, R, which is


expressed in ohms, Ω (1 Ω = 1 C−1 V s). It is often more con-
Viscosity, η/(10–3 kg m–1 s–1)

1.6 venient to work in terms of the conductance, G, which is the


inverse of the resistance: G = 1/R, and therefore expressed in
1.2 Ω−1. The reciprocal ohm used to be called the mho, but its SI
designation is now the siemens, S, and 1 S = 1 Ω−1 = 1 C V−1 s−1.
0.8 Electric current is expressed in amperes, A, with 1 A = 1 C s−1,
so a more physically revealing relation is 1 S = 1 A V−1.
0.4 The conductance of a sample is proportional to its cross-
sectional area, A, and inversely proportional to its length, l.
0
0 20 40 60 80 100
Therefore
Temperature, θ/°C
A Conductivity
Figure 16B.1 The temperature dependence of the viscosity of G  [definition]
 (16B.3)
l
water. As the temperature is increased, more molecules are able to
escape from the potential wells provided by their neighbours, and where the constant of proportionality κ (kappa) is the electri-
so the liquid becomes less viscous. cal conductivity of the sample. With the conductance in sie-
mens and the dimensions in metres, it follows that the units of
Brief illustration 16B.1
κ are siemens per metre (S m−1). The conductivity is a property
The viscosity of water at 25 °C and 50 °C is 0.891 mPa s and
of the material, whereas the conductance depends both on the
0.547 mPa s, respectively. It follows from eqn 16B.2 that the material and its dimensions. For a solution of an electrolyte,
activation energy for molecular migration is the solution of the conductivity depends on the concentration of ions in the
sample, which suggests that it is sensible to introduce the molar
 (T2 ) ( Ea /R )(1/T2 1/T1 ) ­conductivity, Λm, defined as
e
 (T1 )
 Molar conductivity
m   (16B.4)
which, after taking logarithms of both sides, is c [definition]

R ln{ (T2 )/ (T1 )} where c is the molar concentration of the added electrolyte.
Ea 
1/T2  1/T1 The units of molar conductivity are siemens metre-squared per
(8.3145 JK 1 mol 1 )ln{(0.547 mPa s)/(0.891 mPa s)} mole (S m2 mol−1), and typical values are about 10 mS m2 mol−1
 (where 1 mS = 10−3 S).
1/(323 K )  1/(298 K )
Experimentally, it is found that the value of the molar con-
 1.56  10 Jmol 1
4
ductivity varies with the concentration. One reason for this
variation is that the number of ions in the solution might not
or 15.6 kJ mol−1. This value is comparable to the strength of a
hydrogen bond.
be proportional to the nominal concentration of the electro-
lyte. For instance, the concentration of ions in a solution of a
weak electrolyte depends on the degree of dissociation, which
One problem with the interpretation of viscosity measure- is a complicated function of the total amount of solute: dou-
ments is that the change in density of the liquid as it is heated bling the total concentration of the solute does not double the
makes a pronounced contribution to the temperature variation number of ions. Secondly, even for fully dissociated strong
of the viscosity. Thus, the temperature dependence of viscosity electrolytes, because ions interact strongly with one another,
at constant volume, when the density is constant, is much less the conductivity of a solution is not exactly proportional to the
than that at constant pressure. At low temperatures and very number of ions present.
high pressures (on the order of thousands of atmospheres), the In an extensive series of measurements during the nine-
viscosity of water decreases as the pressure is increased. This teenth century, Friedrich Kohlrausch established what is now
behaviour is consistent with the need to rupture hydrogen called Kohlrausch’s law, that at low concentrations the molar
bonds for migration to occur. conductivity of a strong electrolyte depends on the square root
of the concentration:
16B.1(b) Electrolyte solutions m  m  Kc1/2 Kohlrausch’s law  (16B.5)

When a potential difference is applied between two electrodes The quantity Λm is the limiting molar conductivity, the molar
immersed in a solution containing ions there is a flow of cur- conductivity in the limit of zero concentration when the ions
rent due to the migration of ions through the solution. The key are so far apart that they move independently of each other.
16B Motion in liquids 719

Kohlrausch also established that this limiting molar conduc- Self-test 16B.1 The conductivity of KClO4(aq) at 25 °C is
tivity is the sum of contributions from the individual ions pre- 13.780 mS m−1 when c = 1.000 mmol dm−3 and 67.045 mS m−1
sent. If the limiting molar conductivity of the cations is denoted when c = 5.000 mmol dm−3. Determine the values of the limit-
λ+ and that of the anions λ−, then his law of the independent ing molar conductivity Λ and the constant K for this system.
m

migration of ions states that Answer: K  9.491 mS m 2 mol 1 (mol dm 3 )1/ 2 , m  14.080 mS m 2 mol 1

m        
Law of the independent
migration of ions
 (16B.6)

where   and   are the numbers of cations and anions pro- Exercises
vided by each formula unit of electrolyte. For example, for HCl,
E16B.1 The viscosity of water at 20 °C is 1.002 cP and 0.7975 cP at 30 °C. What
NaCl, and CuSO4,       1, but for MgCl2    1,    2. is the energy of activation associated with viscosity?
E16B.2 The limiting molar conductivities of NaI, NaNO3, and AgNO3 are
12.69 mS m2 mol−1, 12.16 mS m2 mol−1, and 13.34 mS m2 mol−1, respectively (all
Example 16B.1  Determining the limiting molar at 25 °C). What is the limiting molar conductivity of AgI at this temperature?
Hint: Each limiting molar conductivity can be expressed as a sum of two ionic
conductivity conductivities.

The conductivity of KCl(aq) at 25 °C is 14.688 mS m−1


when c = 1.0000 mmol dm−3, and 71.740 mS m−1 when c =
5.0000 mmol dm−3. Determine the values of the limiting molar 16B.2 The mobilities of ions
conductivity Λm and the constant K in Kohlrausch’s law.
Collect your thoughts You need to use eqn 16B.4 to determine The reason why different ions have different molar conduc-
the molar conductivities at the two concentrations. Then, by tivities in solution can be understood by analysing the motion
using eqn 16B.5, you can express the difference between these of an ion subject to an electric field and at the same time sur-
two values as m (c2 )  m (c1 )  K (c11/ 2  c12/ 2 ). From this relation rounded by a viscous medium.
you can identify K and then go on to find Λm by using one of
the values of the molar conductivity in eqn 16B.5, rearranged
into m  m  Kc1/ 2.
16B.2(a) The drift speed
An ion in a vacuum is accelerated by an electric field, but in a
The solution The molar conductivity of KCl(aq) when c =
−3 −3 viscous liquid the motion of the ion is impeded by the need for
1.0000 mmol dm (which is the same as 1.0000 mol m ) is
it to push its way through the tightly packed solvent molecules.
14.688 mSm 1 The latter effect is called viscous drag. As the ion accelerates
m   14.688 mSm2 mol 1
1.0000 molm 3 under the influence of the field, the viscous drag increases and
the ion quickly reaches a steady terminal speed, called the drift
Similarly, when c = 5.0000 mol dm−3 the molar conductivity is
speed, which can be found by balancing the two forces.
14.348 mS m2 mol−1. It then follows that

m (c2 )  m (c1 ) How is that done? 16B.1 Deriving an expression for the
K 
c11/ 2  c12/ 2
drift speed
(14.348  14.688) mSm2 mol 1
 The starting point for this calculation is the result from electro-
(0.001000001/ 2  0.00500001/ 2 ) (mol dm 3 )1/ 2
2 1 3 1/ 2
statics that when the potential difference between two planar
 8.698 mS m mol (mol dm ) electrodes a distance l apart is , the ions in the solution between
them experience a uniform electric field of magnitude E   /l.
(For reasons that will become clear immediately below, it is
best to keep this awkward but convenient array of units rather Here, and throughout this section, the sign of the charge number
is disregarded so as to avoid notational complications.
than converting them to the equivalent 10−3/2 S m7/2 mol−3/2.) The
limiting molar conductivity is then found by using the data for Step 1 Find the force on the ion due to the field
c = 1.0000 mmol dm−3: In an electric field E, an ion of charge ze experiences a force
mSm2 mol 1 of magnitude zeE (see The chemist’s toolkit 16B.1). Therefore,
m  14.688 mSm2 mol 1  8.698
(mol dm 3 )1/ 2 ze
Felectric  zeE 
3
 (11.0000  10 mol dm ) 3 1/ 2 2
 14.963 mSm mol 1
l

Comment. Although the value of K has been given to five Step 2 Find the force on the ion due to viscous drag
significant figures in conformity with the data, that degree of As the ion moves through the solvent it experiences a fric-
precision is probably over-optimistic in practice. tional retarding force proportional to its speed. For a spherical
720 16 Molecules in motion

particle of radius a travelling at a speed s, this force is given by the drift speed is 7.3 μm s−1. That speed might seem slow, but
Stokes’ law, which Stokes derived by considering the hydrody- not when expressed on a molecular scale, because it cor-
namics of the passage of a sphere through a continuous fluid: responds to an ion passing about 104 solvent molecules per
second.
Fviscous  fs f  6a Stokes’ law  (16B.7)

where η is the coefficient of viscosity. In this calculation it is


assumed that Stokes’ law applies on a molecular scale; experi- Equation 16B.9 implies that the mobility of an ion decreases
mental evidence suggests that it often gives at least the right with increasing solution viscosity and ion size. Experiments
order of magnitude for the viscous force. confirm these predictions for bulky ions (such as R4N+ and
Step 3 Find the drift speed by balancing the two forces RCO2− ) but not for small ions. For example, the mobilities of the
alkali metal ions in water increase from Li+ to Rb+ (Table 16B.2)
The two forces act in opposite directions and the ions quickly
even though the ionic radii increase. The paradox is resolved
reach a terminal speed, the drift speed, s, when they are in bal-
when it is realized that the radius a in the Stokes formula is the
ance. This balance occurs when fs = zeE, and therefore

zeE (16B.8a) The chemist’s toolkit 16B.1 Electrostatics


s=
f Drift speed
A charge Q 1 (units: coulomb, C) gives rise to a Coulomb poten-
Equation 16B.8a shows that the drift speed is proportional tial φ (units: volt, V). The potential energy (units: joule, J, with
to the electric field strength. The constant of proportionality is 1 J = 1 V C) of a second charge Q in that potential is
called the mobility of the ion, u: EP  Q 
s = uE Mobility [definition] (16B.8b) In one dimension, the electric field strength (units: volt per
−1 metre, V m−1), E, is the negative of the gradient of the electric
With the electric field strength in volts per metre (V m )
potential φ :
and the drift speed in metres per second (m s−1) the slightly
awkward units of u are metres-squared per volt per second d
E  Electric field strength
(m2 V−1 s−1; note that m2 V−1 s−1 × V m−1 = m s−1); a selection of dx
values of mobilities is given in Table 16B.2. Comparison of the In three dimensions the electric field is a vector, and
last two equations shows that E = −∇φ
The electric field between two plane parallel plates separated by
f = 6πηa a distance l, and between which there is a potential difference
ze ze  , is uniform and given by
u= = (16B.9)
f 6πηa 
E 
l
where the Stokes’ law value for the frictional coefficient f has A charge Q experiences a force that depends on the electric
been used. field strength at its location according to:
Felectric = Q E
Note that because the electric field strength is the gradient of
Brief illustration 16B.2 the potential there is only a force if the potential varies with
distance.
An order-of-magnitude estimate of the mobility can be found
using eqn 16B.9 with z = 1 and a = 130 pm, which is typical of
the radius of a hydrated ion; the viscosity of water at 25 °C is
0.9 cP, or 0.9 mPa s. Then Table 16B.2 Ionic mobilities in water at 298 K⋆

JV 1 u/(10−8 m2 V−1 s−1) u/(10−8 m2 V−1 s−1)



19
1.6  10 C H +
36.23 OH −
20.64
u
6  (0.9  10 3 Pa 12
 s)  (130  10 m) Li+ 4.01 F− 5.70
Jm3 +
5.19 −
7.91
Na Cl
8 2 1 1
 7.3  10 m V s K+ 7.62 Br− 8.09
+
Rb 7.92 SO 2− 8.29
This value means that when there is a potential difference of 1.0 V 4

across a solution of length 1.0 cm (so E   /l  100 Vm 1), ⋆


More values are given in the Resource section.
16B Motion in liquids 721

+ Area, A

sΔt
Figure 16B.2 A highly schematic diagram showing the effective
motion of a proton in water. Figure 16B.3 In the calculation of the current, all the ions, moving
at speed s, within a distance sΔt (i.e. those in the volume sAΔt) will
pass through the area A.
hydrodynamic radius (or ‘Stokes radius’) of the ion, its effective
radius in the solution taking into account all the H2O molecules
it carries in its hydration shell. Small ions give rise to stronger
Step 1 Calculate the number of ions passing through an
electric fields than large ones (the electric field at the surface of
imaginary window
a sphere of radius r is proportional to z/r2, so the smaller the ra-
dius the stronger the field). Consequently, small ions are more By referring to Fig. 16B.3 you can see that the number of ions
extensively solvated than big ions and a small ion may have a of one kind, moving at speed s, that pass through an imaginary
window of area A during an interval Δt is equal to the number
large hydrodynamic radius because it drags many solvent mol-
within the distance sΔt, and therefore to the number in the vol-
ecules through the solution as it migrates. The hydrating H2O
ume sΔtA. It follows that the number of ions passing through
molecules are often very labile, however, and NMR and isotope
the window in that period is NstA  stA cN A .
studies have shown that the exchange between the coordina-
tion sphere of the ion and the bulk solvent is very rapid for ions Step 2 Calculate the charge passing through the window, and
of low charge but slow for ions of high charge. hence the current
The proton, although it is very small, has a very high mobil- Each ion carries a charge ze, so the charge passing through the
ity (Table 16B.2). Proton and 17O-NMR show that the charac- window is zestA cN A which, by introducing Faraday’s con-
teristic lifetime of protons hopping from one molecule to the stant F = eNA, can be written zstA cF . The electrical current, I,
next is about 1.5 ps, which is comparable to the time that inelas- is the rate of passage of charge, which in this case is the charge
tic neutron scattering shows it takes a water molecule to turn divided by the time interval Δt: so I  zstA cF / t  zsA cF .
through about 1 radian (1 to 2 ps). According to the Grotthuss Step 3 Set up expressions for the conductance, the conductivity,
mechanism, there is an effective motion of a proton that in- and the molar conductivity
volves the rearrangement of bonds in a group of water mole-
The conductance is given by G = I/ , where  is the potential
cules (Fig. 16B.2). However, the actual mechanism is still highly
difference across the solution. It follows that
contentious. The mobility of protons in liquid ammonia is also
anomalous and presumably occurs by an analogous mechanism. I zsA cF
G 
 
16B.2(b) Mobility and conductivity The conductivity is
The limiting molar conductivity of an ion is a measurable quan- Gl zsA cFl zs cFl
tity, and the mobility of an ion can, in principle, be calculated on   
A  A 
the basis of a model of its motion through the solvent. It should
be possible to find a relation between these two quantities. The limiting molar ionic conductivity is

 zs cFl zsFl


  
How is that done? 16B.2 Establishing the relation between  c  c 
ionic mobility and limiting molar conductivity
Step 4 Introduce the ionic mobility
To keep expressions notationally simple, ignore the signs of At this point you can identify  /l as the electric field strength
quantities in what follows and focus on their magnitudes. E, and s/E as the mobility, u:
Consider a solution of an electrolyte at a molar concentration c.
Let each formula unit give rise to   cations of charge z+e and Δϕ
=E s = uE
  anions of charge z−e. The molar concentration of each type l

of ion is therefore ν c (with     or     ), and the number zsFl zsF Ion molar conductivity
λ= = E = zuF (16B.10)
density of each type is N   cN A . Δϕ in terms of mobility
722 16 Molecules in motion

Equation 16B.10 applies to the cations and to the anions. For Substitution of these expressions for s and F into eqn 16B.12
an electrolyte where there are   and   cations and anions, gives, on cancelling the E, the Einstein relation:
respectively, from each formula unit, it follows from eqn 16B.6
zDF
that m        . Therefore u= Einstein relation (16B.13)
RT
(16B.11a)
om  (z u  z u)F The mobility is high when the diffusion coefficient (which is
Limiting molar conductivity
in terms of mobilities inversely proportional to the viscosity) is high, indicating that
the solute molecules are very mobile. Don’t be misled by the
For a symmetrical z:z electrolyte (e.g. CuSO4 with z+ = z− = 2, presence of temperature in the denominator into thinking that
and       1), this equation simplifies to the ion mobility decreases with increasing temperature: the dif-
fusion coefficient increases more rapidly with temperature than
m  z (u  u )F  (16B.11b) T itself, so u increases with increasing temperature.

Brief illustration 16B.3 Brief illustration 16B.4


−8 2 −1 −1
In Brief illustration 16B.2 the mobility of a typical ion is esti- From Table 16B.2, the mobility of SO2− 4 is 8.29 ×10 m V s .
mated as 7.3 × 10−8 m2 V −1 s−1. For z = 1, this value can be used It follows from eqn 16B.13 in the form D = uRT/zF that the
to estimate a typical limiting molar conductivity of the ion diffusion coefficient for the ion in water at 25 °C is
from eqn 16B.10 as
(8.29  108 m2 V 1 s 1 )  (8.3145 JK 1 mol 1 )  (298 K )
  zuF D 1
2  (9.649  104 C
 mol )
8 2 1 1 4 1
 1  (7.3  10 m V s )  (9.649  10 C mol ) JV 1
9 2 1
3 2
 7.0  10 m V s C mol 1 1 1
 1.06  10 m s

From 1 V−1 C s−1 = 1 S, the value can be expressed as


7.0 mS m2 mol−1. The experimental value for K+(aq) is The Einstein relation can be developed to provide a link be-
7.6 mS m2 mol−1.
tween the limiting molar conductivity of an electrolyte and the
diffusion coefficients of its ions. First, by using eqns 16B.10 and
16B.13 the limiting molar conductivity of an ion can be written
16B.2(c) The Einstein relations
u = zDF/RT
The relation between drift speed and the electric field strength
in eqn 16B.8a (s = zeE/f ) is a special case of a more general re- z 2 DF 2
λ = zuF = (16B.14)
lation derived in Topic 16C (eqn 16C.5): RT

DF Then, by using m         (eqn 16B.6), the limiting


s= Drift speed in terms of diffusion coefficient  (16B.12)
RT molar conductivity is
where F is the force (per mole of ions) driving the ions through F 2
Λm (ν + z +2 D+ + ν − z −2 D− )
Nernst–Einsten
the viscous medium and D is the diffusion coefficient for the = equation
 (16B.15)
RT
species (Table 16B.3). For an ion in solution the drift speed is
s = uE (eqn 16B.8b), and the force on each ion in an electric which is the Nernst–Einstein equation. One application of this
field of strength E is ezE . It follows that the force per mole of equation is to the determination of ionic diffusion coefficients
ions is NA ezE which, by using NAe = F, can be written zFE. from conductivity measurements; another is the prediction of
conductivities based on models of ionic diffusion.

Table 16B.3 Diffusion coefficients at 298 K, D/(10−9 m2 s−1)⋆


Exercises
Molecules in liquids Ions in water + + +
E16B.3 At 25 °C the molar ionic conductivities of Li , Na , and K are
I2 in hexane 4.05 K+ 1.96 Br− 2.08 2 −1 2 −1 2
3.87 mS m mol , 5.01 mS m mol , and 7.35 mS m mol , respectively. −1

in benzene 2.13 H+ 9.31 Cl− 2.03 What are their mobilities?


+ −8 2 −1 −1
Glycine in water 1.055 Na+ 1.33 I− 2.05 E16B.4 The mobility of a Rb ion in aqueous solution is 7.92 × 10 m s V at
25 °C. The potential difference between two electrodes, separated by 7.00 mm
H2O in water 2.26 OH− 5.03
and placed in the solution, is 25.0 V. What is the drift speed of the Rb+ ion?
Sucrose in water 0.5216 −
E16B.5 The mobility of a NO3 ion in aqueous solution at 25 °C is

More values are given in the Resource section. 7.40 × 10−8 m2 s−1 V−1. Calculate its diffusion coefficient in water at 25 °C.
16B Motion in liquids 723

Checklist of concepts
☐ 1. The viscosity of a liquid decreases with increasing tem- ☐ 4. An ion reaches a drift speed when the acceleration due
perature. to the electrical force is balanced by the viscous drag.
☐ 2. Kohlrausch’s law states that, at low concentrations, the ☐ 5. The hydrodynamic radius of an ion may be greater than
molar conductivities of strong electrolytes vary as the its ionic radius.
square root of the concentration. ☐ 6. The high mobility of a proton in water is explained by
☐ 3. The law of the independent migration of ions states the Grotthuss mechanism.
that the molar conductivity, in the limit of zero ☐ 7. The mobility of an ion can be related to its limiting
concentration, is the sum of contributions from indi- molar conductivity and, via the Einstein relation, to its
vidual ions. diffusion coefficient.

Checklist of equations
Property Equation Comment Equation number
Viscosity of a liquid   0 e Ea /RT Over a narrow temperature range 16B.2
Conductivity κ = Gl/A, G = 1/R Definition 16B.3
Molar conductivity Λm = κ/c Definition 16B.4

Kohlrausch’s law m  m  Kc1/ 2 Empirical observation 16B.5

Law of independent migration of ions m        Limiting law 16B.6


Stokes’ law Fviscous = fs, f = 6πηa 16B.7
Drift speed s = uE Defines u 16B.8b
Ion mobility u  ze /6 a Assumes Stokes’ law 16B.9
Conductivity and mobility λ = zuF 16B.10

Molar conductivity and mobility m  (z  u   z u  )F 16B.11a


Drift speed s = DF /RT F is a general (molar) force 16B.12
Einstein relation u = zDF/RT 16B.13
Nernst–Einstein equation m  ( z D   z D )(F /RT )
2
 
2
 
2
16B.15
TOPIC 16C Diffusion

16C.1 The thermodynamic view


➤ Why do you need to know this material?
The diffusion of chemical species through space controls At constant temperature and pressure, the maximum non-
the rates of many chemical reactions in chemical reactors, expansion work that can be done by a spontaneous process is
living cells, and the atmosphere. equal to the change in the Gibbs energy (Topic 3D). In this case
the spontaneous process is the spreading of a solute, and the
➤ What is the key idea? work it could achieve per mole of solute molecules can be iden-
Molecules and ions tend to spread into a uniform distribution. tified with the change in the chemical potential of the solute:
dwm = dμ.
➤ What do you need to know already? Imagine that the chemical potentials are μ(x) at x and
This Topic draws on arguments relating to the calculation μ(x + dx) at x + dx, and that the solute moves between these two
of flux (Topic 16A) and the notion of drift speed introduced locations. The difference in chemical potential is then
in Topic 16B. It also uses the concept of chemical potential
to discuss the direction of spontaneous change (Topic 5A).   
d   ( x  dx )   ( x )    dx
The final section uses a statistical argument like that used  x T ,p
to discuss the properties of a random coil in Topic 14D.
The molar work associated with migration through dx is equal
to this change in the chemical potential

  
dw m    dx
That solutes in gases, liquids, and solids have a tendency to spread  x T ,p
can be discussed from three points of view. The thermodynamic The work done in moving a distance dx against an opposing
viewpoint makes use of the Second Law of ­thermodynamics force F (in this context, a molar quantity) is dwm  Fdx . By
and the tendency for entropy to increase. If the temperature and comparing the two expressions for dwm it is seen that the slope
pressure are constant, then equivalently there is a tendency for of the chemical potential with respect to position can be inter-
the Gibbs energy to decrease. The second approach is to set up a preted as an effective force per mole of molecules. This thermo-
differential equation for the change in concentration in a region dynamic force is written as
by considering the flux of material through its boundaries. The
third approach is based on a model in which diffusion is imag-    Thermodynamic force

F    [definition]
(16C.2)
ined as taking place in a series of random small steps.  x T ,p
Several derivations in this Topic use Fick’s first law of dif-
There is not a real force pushing the molecules down the slope of
fusion, which is discussed in Topic 16A and repeated here for
the chemical potential: the apparent force represents the spon-
convenience:
taneous tendency of the molecules to disperse as a consequence
dN Fick’s first law  (16C.1a)
of the Second Law and the tendency towards greater entropy.
J (number )  D [number] In a solution in which the activity of the solute is a, the chem-
dx
ical potential is μ = μ + RT ln a. The thermodynamic force can

where N is the number density and D is the diffusion coefficient. therefore be written in terms of the gradient of the logarithm of
In a number of cases it is more convenient to discuss the flux in the activity:
terms of the amount of molecules and the molar concentration, c.

Division by Avogadro’s constant turns eqn 16C.1a into ( / x)T,p  ( RT ln a)/ x

dc  ln a
J (amount)  D
Fick’s first law
 (16C.1b) F    RT (16C.3a)
[amount] x x
dx T, p T, p
16C Diffusion 725

If the solution is ideal, a may be replaced by c/c , where c is the ⦵

Concentration, c
molar concentration and c is its standard value (1 mol dm−3):

d ln y/dx = (1/y)(dy/dx) Constant


positive

Property
gradient
ln(c / c )

RT c
F = − RT =−
c
(16C.3b)
x T, p
x T, p

Thermodynamic force, F

Example 16C.1 Calculating the thermodynamic force


Suppose that the concentration of a solute varies linearly along Position, x
x according to c = c0 + αx, where c0 is the concentration at x = 0.
Figure 16C.1 The thermodynamic force is proportional to
Find an expression for the thermodynamic force at position x,
−∂ln c/∂x = −(1/c)∂c/∂ x. The force thus drives the molecules
and evaluate this force at x = 0 and x = 1.0 cm for the case where from a region with higher concentration to one with lower
c0 = 1.0 mol dm−3 and α = 10 mol dm−3 m−1. Take T = 298 K. concentration, and becomes smaller in magnitude as the
Collect your thoughts You will need to use eqn 16C.3b to find concentration increases.
the force, so begin by evaluating ∂c/∂x.
The solution The gradient of the molar concentration is

 c   (c0   x )  The thermodynamic force acts in many respects like a real


 x    x   physical force. In particular it is responsible for accelerating sol-
 T ,p  T ,p
ute molecules until the viscous drag they experience balances
Then, from eqn 16C.3b, the thermodynamic force is the apparent driving force and they settle down to a steady ‘drift
speed’ through the medium. By considering the balance of ap-
( c/ x)T,p= α; c = c0 + α x parent driving force and the retarding viscous force it is possi-
ble to derive Fick’s first law of diffusion and relate the diffusion
RT c α RT
F =− c =− coefficient to the properties of the medium.
x T ,p
c0 + α x

At x = 0, How is that done? 16C.1 Deriving Fick’s first law of


diffusion
(10 moldm 3 m 1 )  (8.3145 JK 1 mol 1 )  (298 K)
F  The flux due to a concentration gradient is the amount (in
1.0 mol dm 3
moles) of molecules passing through a region of area A in an
N
interval Δt divided by the area and the interval. This flux can be
 2.5  10 Jm 1 mol 1
4
related to the drift speed, s, by using an approach like that used
in Topic 16A for diffusion in a gas.
or −25 kN mol−1. A similar calculation at x = 1.0 cm = 1.0 × 10−2 m
Step 1 Find an expression for the flux due to molecules moving
gives the force as −23 kN mol−1.
at the drift speed
Comment. The negative sign indicates that the force is towards In a time interval Δt all the particles within a distance sΔt can
the left (towards negative x), because the concentration increas- pass through the window, which means that all the particles in
es towards the right (as c0 + αx) and there is therefore a tendency a volume sΔtA pass through the window (there is no reverse
for the solute to migrate to the left under the influence of that flux because, unlike in a gas, in this model all the solute mol-
apparent force. The magnitude of the thermodynamic force ecules are moving down the concentration gradient). Hence,
decreases as x increases because the gradient of ln(c/c0), which is the amount (in moles) of solute molecules that pass through
α/(c0 + αx), becomes smaller on going to the right (Fig. 16C.1). the window is sΔtAc. The flux J is this number divided by the
area A and by the time interval Δt:
Self-test 16C.1 Suppose that the concentration of a solute
decreases exponentially to the right as c(x) = c0e−x/l. Derive an J (amount) = sc
expression for the thermodynamic force at any position. Step 2 Find an expression for the drift speed
Answer: F = RT /l The apparent molar driving force acting on the solute mol-
ecules is eqn 16C.3b, F  (RT /c)dc /dx . The molecules also
726 16 Molecules in motion

experience a viscous drag which is assumed to be proportional the diffusion coefficient is 5.0 × 10−9 m2 s−1. The corresponding
to the speed: expressed as a molar quantity this force is writ- thermodynamic force is therefore
ten NAfs, where f is a constant, the ‘frictional constant’, which
depends on the medium. The molecules move at the drift speed (1.0  106 ms 1 )  (8.3145 JK 1 mol 1 )  (298 K)
F
when these two forces are in balance. It therefore follows that, 5.0  109 m2 s 1
with R/NA = k, N
 5.0  105 Jm 1 mol 1
RT dc RT dc kT dc
N A fs   , hence s   
c dx N A fc dx fc dx or about 500 kN mol−1. This thermodynamic force is many
times that of gravity, which explains why solutions do not
The negative sign indicates that the molecules are moving
sediment.
opposite to the direction of increasing concentration.
Step 3 Combine the two expressions
Now substitute the expression for the drift speed into that for
the flux to give 16C.2 The diffusion equation
kT dc kT dc
J (amount)  sc   c Diffusion results in the modification of the distribution of con-
fc dx f dx
centration of the solute (or of a physical property) as inhomo-
This expression has the same form as Fick’s first law, eqn geneities disappear. The discussion is expressed in terms of the
16C.1b, that the flux is proportional to the concentration gradi- diffusion of molecules, but similar arguments apply to the dif-
ent. In addition, the diffusion constant D can be identified as fusion of other entities, such as ions, and of various physical
kT/f, which is the Stokes–Einstein relation:
properties, such as temperature.

kT (16C.4a)
D=
f Stokes–Einstein relation 16C.2(a) Simple diffusion
The diffusion equation, one of the most important equations
The constant f that appears in the Stokes–Einstein relation for discussing the properties of fluids, is an equation that ex-
can be inferred from Stokes’ law for the viscous drag (Topic presses the rate of change of concentration of a species in terms
16B). According to this law, the magnitude of the viscous force of the inhomogeneity of its concentration. It is also called ‘Fick’s
is 6πηas for a spherical particle of radius a. It therefore follows second law of diffusion’, but that name is now rarely used. The
that f = 6πηa, which leads to the Stokes–Einstein equation diffusion equation can be derived on the basis of Fick’s first law.
kT
D= Stokes –Einstein equation  (16C.4b)
6πηa
How is that done? 16C.2 Deriving the diffusion equation
This equation is an explicit relation between the diffusion coef-
ficient and the viscosity for a species of hydrodynamic radius a. The diffusion equation is developed by considering the net
It confirms that D is inversely proportional to η. flux of particles entering a thin slab of cross-sectional area
The drift speed can be related to the diffusion constant and A that extends from x to x + l (Fig. 16C.2) and therefore has
the thermodynamic force by equating two expressions for the volume Al.
flux, J = sc and J = −Ddc/dx, to obtain s = −(D/c)dc/dx. The
concentration gradient can be expressed in terms of the ther-
modynamic force, that is, eqn 16C.3b rearranged in the form Volume, Al Flux JR
Area, A
dc /dx  cF /RT to give Flux JL
DF
s=  (16C.5)
RT
This relation, in the form F = sRT/D, provides a way to esti-
mate the thermodynamic force from measurements of the drift
speed and the diffusion coefficient. x+l
x

Brief illustration 16C.1 Figure 16C.2 The net flux in a thin slab is the difference between
the flux entering from the region of high concentration (on the
Laser measurements show that a particular molecule has a left) and the flux leaving to the region of low concentration (on
drift speed of 1.0 μm s−1 in water at 25 °C, at which temperature the right).
16C Diffusion 727

Step 1 Find an expression for the net rate of change of the concen-
tration in the slab due to particles entering from each side Spreads
If the flux of molecules from the left is JL, then the rate at which
the particles enter the slab is JLA. The rate of increase of the molar

Concentration, c
concentration inside the slab due to the flux in from the left is Negative
curvature
 c  J L A J L
 t   Al  l
 L Fills
Molecules also flow out of the right face of the slab. If this flux
is JR, then by a similar argument Positive
curvature
 c  JR
 t    l
 R Position, x

Note the minus sign: this flux reduces the concentration. The Figure 16C.3 The diffusion equation implies that, over time,
net rate of change of concentration in the slab is peaks in a distribution (regions of negative curvature) spread and
troughs (regions of positive curvature) fill in.
c  c   c  J J
   L R
t  t L  t R l

Step 2 Relate the fluxes to the concentration gradients


The diffusion equation shows that the rate of change of con-
From Fick’s first law (eqn 16C.1b), each flux can be expressed
centration in a region is proportional to the curvature (more
in terms of the diffusion coefficient and the concentration gra-
precisely, to the second derivative) of the concentration with
dient at each face:
respect to distance in that region. If the concentration changes
J = −D( c/ x) sharply from point to point (if the distribution is highly wrin-
kled), then the concentration changes rapidly with time.
c c c c Specifically:
JL − JR = − D +D =D −
x L
x R
x R
x L
• Where the curvature is positive (a dip, Fig. 16C.3), the
where (∂c/∂x)L is the concentration gradient at the left face change in concentration with time is positive; the dip
of the slab, and similarly (∂c/∂x)R is that at the right face. The tends to fill.
concentration gradients at the two faces can be expressed in
• Where the curvature is negative (a heap), the change in
terms of the gradient (the first derivative of the concentration)
concentration with time is negative; the heap tends to
at the centre of the slab, (∂c/∂x)0, and the first derivative of that
spread.
gradient (which is the second derivative of the concentration),
∂2c/∂x2. The distances of the faces from the centre are 12 l in each • If the curvature is zero, then the concentration is constant
direction, so it follows that in time.

 c   c   c  l  2c   c  l  2c   2c The diffusion equation can be regarded as a mathematical for-


 x    x    x   2 x 2    x   2 x 2   l x 2 mulation of the intuitive notion that there is a natural ten

You might also like